Está en la página 1de 561

MANUAL DE TRABAJO DEL CURSO ENARM CMN SIGLO XXI

Pharmed Solutions Institute


Dr. Angel Mauricio Soriano Perez
MANUAL DE TRABAJO DEL CURSO ENARM CMN SIGLO XXI

INTRODUCCION

Este manual de trabajo del curso para aprobar el ENARM es el resultado de más de 13 años de estudio de los contenidos del ENARM
desde el año 2000 a la fecha, la modalidad, contenido y características pedagógicas han cambiado de forma importante desde entonces
hasta ahora, con la versión moderna del ENARM podemos mencionar que es la mejor.

Los cursos de preparación para el ENARM que se imparte en diversas instituciones públicas y privadas han tenido que adaptarse a estos
cambios, sin embargo ni todos los cursos ni los diferentes libros dedicados a la preparación de los médicos aspirantes ha tomado con
toda seriedad y responsabilidad modificando y adaptándose a los cambios cruciales para así ofrecer una verdadera alternativa al
médico general que desea prepararse para aprobar el ENARM.

Nuestro curso, los medios que son empleados y la metodología han sido ampliamente analizados para ser un curso integral que ofrezca
un medio apropiado para incrementar las probabilidades de ser un seleccionado nacional.

Agradecemos a todos los médicos especialista que participaron en la realización del presente manual de trabajo así como a los
participantes de los diversos cursos.

Dirección General.
Dr. Ángel Mauricio Soriano Pérez

Ponente Titular Medicina Interna.


Dra. Luz Elena Castro Vargas .

Ponente Titular Pediatría.


Dra. Gabriela Arenas Ornelas.

Ponente Titular Gineco-Obstetricia.


Dra. Rosario León Gayosso.

Ponente Titular Urgencias.


Dr. Juan Manuel Lara Hernández.

Ponente Titular Cirugia.


Dr. Ängel Santino Figueroa.

Ponente Titular Inglés.


Dr. Juan Carlos Díaz Torre.

“No hay hombre más digno de estimación que el médico que, habiendo estudiado la naturaleza desde su juventud, conoce las
propiedades del cuerpo humano, las enfermedades que le ataca, los remedios que pueden beneficiarle y que ejerce su arte”.

Voltaire

CURSO ENARM CMN SIGLO XXI TEL: 36246001 Pharmed Solutions Institute PÁGINA 2
MANUAL DE TRABAJO DEL CURSO ENARM CMN SIGLO XXI

INSTRUCCIONES

Este manual de trabajo esta formada por las areas de medicina interna, urgencias, ginecología y obstetricia, pediatría y cirugía asi como
una sección de compresión de ingles.

El manual de trabajo presenta un resumen minimo por cada patología previo a los casos clínico, las patologías fueron seleccionadas
mediante un análisis de salida de los sustentantes del ENARM cada año.

Comprende 450 casos clínicas distribuidas por especialidad proporcionalmente y semejante al ENARM, 405 en español y 45 casos
clínicos en ingles.

Cada caso clínico tiene de 1, 2 ó 3 preguntas con 4 posibles respuestas, todas las preguntas se realizaron buscando la mejor respuesta
donde comprenden las siguientes areas de exploración: diagnostico, tratamiento, patogenia, complicaciones, salud publica, ciencias
básicas y rehabilitación.

El orden de los temas se realizará como esta en el índice, posterior a la sesión clínica, se resolvera los casos clínicos del tema, en
conjunto al profesor. Posteriormente resolverá en el auditorio o en su domicilio, las 6 opciones más de exploración denominado
“explicaciones y preguntas complementarias”.

1.- Sesión clínica.


2.- Casos clínicos.
3.- Explicaciones y preguntas complementarias.
4.- Videosesiones para despejar dudas.
5.- Simuladores mensuales.
6.- Elaboración de casos clínicos por los participantes.

CURSO ENARM CMN SIGLO XXI TEL: 36246001 Pharmed Solutions Institute PÁGINA 3
MANUAL DE TRABAJO DEL CURSO ENARM CMN SIGLO XXI

MEDICINA INTERNA

1) HIPERTENSION ARTERIAL PRIMARIA Y SECUNDARIA


2) INSUFICIENCIA CARDIACA AGUDA Y CRONICA
3) CARDIOPATIAS CONGENITICAS Y VALVULOPATIAS
4) MIOCARDITIS, ENDOCARDITIS Y PERICARDITIS
5) ASMA, BRONQUITIS, NEUMONIAS, NEUMOPATIAS.
6) HIPERTENSION PULMONAR Y COR PULMONAR, EPOC
7) CARCER PULMONAR Y BRONCOPULMONAR.
8) TRASTORNOS HIPOTALAMICOS E HIPOFISIARIOS.
9) TRASTORNOS TIROIDEOS Y PARATIROIDEOS.
10) TRASTORNOS SUPRARRENALES Y GONADALES.
11) SOBREPESO, OBESIDAD Y DISLIPIDEMIAS.
12) DIABETES MELLITUS Y COMPLICACIONES.
13) PANCREATITIS AGUDA Y CRONICA.
14) ESTEATOSIS, HIGADO GRASO, CIRROCIS HEPATICA Y HEPATOCARCINOMA
15) LEUCEMIAS, LINFOMAS, LINFOMA NO HODGKIN, LINFOMA HODGKIN
16) FIEBRE REUMATICA, ARTRITIS REUMATOIDE, OSTEARTROSIS, OSTEOCONDRITIS Y GOTA.
17) LUPUS ERITEMATOSO SISTEMICO, VASCULITIS, ARTERITIS DE TAKAYASU, SX DE SJOGREN
18) TUBERCULOSIS, PALUDISMO Y DENGUE, ENFERMEDAD DE CHAGAS, LEPRA,
19) TRIPANOSOMIASIS, LEISHMANIASIS, ESQUISTOSOMIASIS, FILARIASIS, ONCOCERCOSIS.
20) RICKETTSIAS, LEGIONELLA, PSEUDOMONA AERUGINOSA, LISTERIA MONOCYTOGENES, CRIPTOCOCOCIS
21) LEPTOSPIROSIS, ACTINOMICOSIS, NORCARDIA. CMV Y VEB.
22) STAFILOCOCCUS AUREUS, EPIDERMIDIS, ESTREPTOCOCCUS PNEUMONIAE Y HAEMOPHILUS.
23) BORTEDELLA PERTUSIS, MORAXELLA CATARRHALIS, BRUCELLA, FRANCISELLA TULARENSIS, RABIA
24) VIH Y SIDA, ETS, NEISSERIA GONORROEAE, SIFILIS, HERPES, CHLAMYDIA.
25) CANCER E INFECCIONES DEL SISTEMA NERVIOSO CENTRAL.
26) CEFALEA, EPILEPSIAS Y ENCEFALOPATIAS
27) DEMENCIA VASCULAR, ALZHEIMER Y PICK, HUNTINGTON Y PARKINSON
28) ESCLEROSIS MULTIPLE, ESCLEROSIS LATERAL AMNIOTROFICA Y MIASTENIA GRAVIS.
29) DEPRESION, ANSIEDAD, ESQUIZOFRENIA Y TRASTORNOS DE LA ALIMENTACION.

URGENCIAS
1) MUERTE SUBITA, RCP.
2) ESTADO CHOQUE
3) URGENCIAS, EMERGENCIA HIPERTENSIVA. HIPERTENSION MALIGNA.
4) INSUFICIENCIA CARDIACA AGUDA Y CRONICA AGUDIZADA.
5) TAPONAMIENTO CARDIACO, DISECCION DE LA AORTA, ANEURISMA AORTICO.
6) ANGINA ESTABLE, ANGINA INESTABLE, ANGINA DE PRNIZTMETAL.
7) SICA I, SICA II
8) TRASTORNOS DEL RITMO (ARRITMIAS CARDIACAS)
9) FALLA ORGANICA MULTIPLE, COAGULACION INTRAVASCULAR DISEMINADA.
10) ANAFILAXIA Y ALERGIAS.
11) EPOC AGUDIZADO, TROMBOSIS VENOSA PROFUNDA, TROMBOEMBOLIA PULMONAR.
12) SINDROME DE INSUFICIENCIA RESPIRATORIA AGUDA, EDEMA AGUDO PULMONAR.
13) NEUMOTORAX, NEUMOMEDIASTINO, HEMOTORAX, CONTUSION CARDIACA.
14) TRAUMA TORACICO CERRADO Y ABIERTO.
15) TRAUMA ABDOMINAL ABIERTO Y CERRADO.
16) INSUFICIENCIA RENAL AGUDA, GLOMERULOPATIAS AGUDAS.
17) SINDROME NEFRITICO Y NEFROTICO.
18) TRASTORNOS ACIDO-BASE AGUDOS.
19) TRASTORNOS ELECTROLITICOS AGUDOS.
20) HIPOGLUCEMIA, HIPERGLUCEMIA, ESTADO HIPEROSMOLAR, CETOACIDOSIS DIABETICA.
21) TRAUMATISMO CRANEOENCEFLICO, ISQUEMIA CEREBRAL TRANSITORIA, ACCIDENTE VASCULAR CEREBRAL.
22) CRISIS CONVULSIVAS, ESTATUS EPILEPTICO, COMA Y MUERTE CEREBRAL.
23) TRASTORNO BIPOLAR, DELIRIUM, PSICOSIS, SUICIDIO.

GINECOLOGIA Y OBSTETRICIA
1) PLANIFICACION FAMILIAR, ANTICONCEPCION, INFERTILIDAD, ESTERILIDAD.
2) AMENORREA PRIMARIA Y SECUNDARIA, ENDOMETRIOSIS.
3) CERVICOVAGINITIS, ENFERMEDAD PELVICA INFLAMATORIA, ENFERMEDADES DE TRANSMISION SEXUAL.
4) INFECCION POR VIRUS DEL PAPILOMA HUMANO. DISPLASIAS, CANCER CERVICOUTERINO Y OVARICO.
5) SANGRADO UTERINO ANORMAL Y DISFUNCIONAL. SINDROME DE OVARIO POLIQUISTICO.
6) MASTOPATIA FIBROQUISTICA, CANCER DE MAMA, MENOPAUSIA Y CLIMATERIO

CURSO ENARM CMN SIGLO XXI TEL: 36246001 Pharmed Solutions Institute PÁGINA 4
MANUAL DE TRABAJO DEL CURSO ENARM CMN SIGLO XXI

OBSTETRICIA
1) CONTROL PRENATAL Y DEL EMBARAZO.
2) EMBARAZO ECTOPICO. AMENAZA DE ABORTO, ABORTO, INCOMPETENCIA CERVICAL.
3) HIPEREMESIS GRAVIDICA, ENFERMEDAD TROFOBLASTICA.
4) HIPERTENSION ARTERIAL Y ENFERMEDAD HIPERTENSIVA DEL EMBARAZO.
5) PREECLAMPSIA Y ECLAMPSIA.
6) DIABETES MELLITUS, INTOLERANCIA A LA GLUCOSA, DIABETES GESTACIONAL.
7) POLIHIDRAMNIOS, OLIGOHIDRAMNIOS, PLACENTA PREVIA, DESPRENDIMIENTO DE PLACENTA.
8) ANEMIAS, IVU, PARTO PRETERMINO, PARTO NORMAL Y CESAREA.
9) COMPLICACIONES DEL PUERPERIO, HEMORRAGIA OBSTETRICA, PLACENTA ACCRETA.
10) SINDROME DE TENSION PREMENSTRUAL, PSICOSIS POSPARTO Y DEPRESION POSTPARTO.

PEDIATRIA
1) SX DE ADAPTACION PULMONAR, SX DE DIFICULTAD RESPIRATORIA, TAQUIPNEA TRANSITORIA DEL RN
2) ASPIRACION DE MECONIO, ENFERMEDAD POR MEMBRANA HIALINA, NEUMONIA NEONATAL.
3) SEPSIS NEONATAL, ENTEROCOLITIS NECOTRIZANTE, MENINGITIS NEONATAL.
4) HIPOGLUCEMIA, ICTERICIA, KERNICTERUS.
5) ATRESIA ESOFAGICA, FISTULA TRAQUEOESOFAGICA, HERNIA HIATAL, ESTENOSIS PILORICA.
6) CARDIOPATIAS CONGENITAS Y ADQUIRIDAS
7) KWASHIORKOR, MARASMO, AVITAMINOSIS,
8) OBESIDAD, VACUNACION, CONTROL DEL NIÑO SANO.
9) DERMATITIS, HIPERQUERATOSIS HIPEDERMOLITICA.
10) IMPETIGO ERISIPELA, PIODERMA, TB CUTANEA, MAL DEL PINTO.
11) HERPES SIMPLE, ZOSTER, VERRUGAS VIRALES, MOLUSCO CONTAGIOSO
12) FOLICULITIS, PEDICULOSIS, ESCABIASIS.
13) TIÑAS, PITIRIASIS VERSICOLOR, CANDIDIASIS, MICETOMA.
14) URTICARIA, PRURIGOS, ERITEMA PALMAR, ERITEMA NODOSO.
15) DERMATITIS DE CONTACTO, DERMATITIS DEL PAÑAL.
16) PSORIASIS, DERMATITIS SEBORREICA, ACNE
17) PURPURA DE SCHÖNLEIN-HENOCH, ANEMIAS,
18) LEUCEMIAS, RETINOBLASTOMA, TUMORES DEL SNC.
19) PAROTIDITIS, HEPATITIS, INFLUENZA, MENINGITIS
20) OTITIS, SINUSITIS, FARINGITIS, AMIGDALITIS
21) EPIGLOTITIS, LARINGOTRAQUEITIS, BRONQUITIS.
22) BRONQUIOLITIS, BRONCONEUMONIA, NEUMONIAS
23) ALERGIAS, ASMA, FIBROSIS QUISTICA.
24) URETRITIS, CISTITIS, PIELONEFRITIS.
25) GLOMERULONEFRITIS, SINDROME NEFRITICO, SINDROME NEFROTICO.
26) GASTROENTERITIS, PARASITOSIS.
27) DESHIDRATACION, LIQUIDOS Y ELECTROLITOS
28) QUEMADURAS, ACCIDENTES
29) PICADURAS, MORDEDURAS
30) INTOXICACIONES, ENVENENAMIENTOS
31) TRAUMATISMO CRANEOENCEFALICO, CEFALEA Y EPILEPSIAS
32) ENURESIS, ENCOPRESIS, TDAH, TRASTORNOS DEL DESARROLLO.

CIRUGIA
1) ESOFAGITIS, ACALASIA, REFLUJO ESOFAGICO Y ERGE.
2) DISPLASIAS DE ESOFAGO, ESOFAGO DE BARRETT, CANCER DE ESOFAGO.
3) VARICES ESOFAGICAS, RUPTURA ESOFAGICA Y HERNIA HIATAL.
4) GASTRITIS, ULCERA GASTRICA Y ULCERA DUODENAL, CANCER GASTRICO.
5) COLANGITIS, COLECISTITIS, COLEDOCOLITIASIS
6) CANCER DE PANCREAS, COLANGIOCARCINOMA,
7) ESTEATOSIS Y CIRROCIS HEPATICA, CARCINOMA HEPATOCELULAR.
8) PANCREATITIS AGUDA Y CRONICA.
9) ISQUEMIA MESENTERICA, INFARTO MESENTERICO, DIVERTICULITIS.
10) APENDICITIS, POLIPOS, COLITIS, CROHN, CUCI.
11) HEMORROIDES, FISTULA ANAL, CANCER DE COLON
12) HERNIAS DIAFRAGMATICAS, HIATAL, DE PARED E INGUINAL,
13) ORQUIEPIDIDIMITIS, TORSION TESTICULAR, CANCER TESTICULAR.
14) HIPERTROFIA PROSTATICA BENIGNA, CANCER DE PROSTATA
15) TCE, TRAUMA FACIAL Y TRAUMA RAQUIMEDULAR.
16) TRAUMA TORACICO, TRAUMA ABDOMINAL Y PELVICO
17) PIE DIABETICO Y COMPLICACIONES QUIRURGICAS DE LA DM.

CURSO ENARM CMN SIGLO XXI TEL: 36246001 Pharmed Solutions Institute PÁGINA 5
MANUAL DE TRABAJO DEL CURSO ENARM CMN SIGLO XXI
18) RETINOPATIA DIABETICA E HIPERTENSIVA.
19) URGENCIAS OFTALMOLOGICAS, PERFORACION, GLAUCOMA. (CIRUGIA)
20) TRAUMA OTICO RUPTURA, PERFORACION, TRAUMA FACIAL.

CURSO ENARM CMN SIGLO XXI TEL: 36246001 Pharmed Solutions Institute PÁGINA 6
MANUAL DE TRABAJO DEL CURSO ENARM CMN SIGLO XXI
MEDICINA INTERNA

1) HIPERTENSION ARTERIAL PRIMARIA Y SECUNDARIA


2) INSUFICIENCIA CARDIACA AGUDA Y CRONICA
3) CARDIOPATIAS CONGENITICAS Y VALVULOPATIAS
4) MIOCARDITIS, ENDOCARDITIS Y PERICARDITIS
5) ASMA, BRONQUITIS, NEUMONIAS, NEUMOPATIAS.
6) HIPERTENSION PULMONAR Y COR PULMONAR, EPOC.
7) CARCER PULMONAR Y BRONCOPULMONAR.
8) TRASTORNOS HIPOTALAMICOS E HIPOFISIARIOS.
9) TRASTORNOS TIROIDEOS Y PARATIROIDEOS.
10) TRASTORNOS SUPRARRENALES Y GONADALES.
11) SOBREPESO, OBESIDAD Y DISLIPIDEMIAS.
12) DIABETES MELLITUS Y COMPLICACIONES.
13) PANCREATITIS AGUDA Y CRONICA.
14) ESTEATOSIS, HIGADO GRASO, CIRROCIS HEPATICA Y HEPATOCARCINOMA
15) LEUCEMIAS, LINFOMAS, LINFOMA NO HODGKIN, LINFOMA HODGKIN
16) FIEBRE REUMATICA, ARTRITIS REUMATOIDE, OSTEARTROSIS, OSTEOCONDRITIS Y GOTA.
17) LUPUS ERITEMATOSO SISTEMICO, VASCULITIS, ARTERITIS DE TAKAYASU, SX DE SJOGREN
18) TUBERCULOSIS, PALUDISMO Y DENGUE, ENFERMEDAD DE CHAGAS, LEPRA,
19) TRIPANOSOMIASIS, LEISHMANIASIS, ESQUISTOSOMIASIS, FILARIASIS, ONCOCERCOSIS.
20) RICKETTSIAS, LEGIONELLA, PSEUDOMONA AERUGINOSA, LISTERIA MONOCYTOGENES, CRIPTOCOCOCIS,
21) LEPTOSPIROSIS, ACTINOMICOCIS, NORCARDIA. CMV Y VEB.
22) STAFILOCOCCUS AUREUS, EPIDERMIDIS Y ESTREPTOCOCCUS PNEUMONIAE. HAEMOPHILUS.
23) BORTEDELLA PERTUSIS, MORAXELLA CATARRHALIS, BRUCELLA, FRANCISELLA TULARENSIS, RABIA,
24) VIH Y SIDA, ETS, NEISERIA GONORRAE, SIFILIS, HERPES, CHLAMYDIA.
25) CANCER E INFECCIONES DEL SISTEMA NERVIOSO CENTRAL.
26) CEFALEA, EPILEPSIAS Y ENCEFALOPATIAS
27) DEMENCIA VASCULAR, ALZHEIMER Y PICK, HUNTINGTON Y PARKINSON
28) ESCLEROSIS MULTIPLE, ESCLEROSIS LATERAL AMNIOTROFICA Y MIASTENIA GRAVIS.
29) DEPRESION, ANSIEDAD, ESQUIZOFRENIA Y TRASTORNOS DE LA ALIMENTACION.

CURSO ENARM CMN SIGLO XXI TEL: 36246001 Pharmed Solutions Institute PÁGINA 7
MANUAL DE TRABAJO DEL CURSO ENARM CMN SIGLO XXI
HIPERTENSIÓN ARTERIAL PRIMARIA Y SECUNDARIA (TEMA)

DEFINICION: Es el incremento sostenido de la tensión arterial diastólica > 90 mmHg, La OMS la define como un trastorno cuya
patogenia se desconoce en muchos casos, que en última instancia causa aumento de la presión diastólica y sistólica, así como
alteraciones del lecho vascular y alteraciones funcionales de los tejidos afectados. EPIDEMIOLOGIA: Prevalencia mundial: 1 billón de
individuos, 7.1 millones de muertes al año, OMS: 62 % de la enfermedad cerebrovascular y 49 % de la enfermedad isquémica cardiaca.
Prevalencia incrementa conforme avanza la edad, 50% de personas entre 60 y 69 años y 3/4 de los mayores de 70 años están
afectados. La prevalencia de hipertensión sistólica se incrementa con la edad. A partir de los 50 años forma más común. PAD factor de
riesgo CV más potente hasta los 50 años. A partir de entonces, la PAS >140mmHg es de importancia mayor como factor de riesgo de
(ECV). En los ensayos clínicos, la terapia antihipertensiva se ha asociado con reducciones en incidencias de ictus de un 35-40 %, de
infarto de miocardio de un 20-25 %, y de insuficiencia cardiaca en más de un 50 %. Por cada incremento de 20 mmHg en PAS ó 10 en
PAD, hay el doble de mortalidad por enfermedad isquémica e ictus. Estudio Framingham: PA de 130 a 139/85 a 89 mmHg: 2 veces de
incremento RR de enfermedad cardiovascular comparados con los niveles por debajo de 120/80 mmHg. El riesgo de ECV comienza a
partir de 115/75 mmHg se dobla con cada incremento de 20/10 mmHg. ETIOLOGIA: HTA primaria es más frecuente, la secundaria
causada por: estenosis de arteria renal, insuficiencia renal, enfermedad de las glándulas suprarrenales, feocromocitomas, hiperplasia
cortical, síndrome de cushing, hiperaldosteronismo, otras causas son la preeclampsia, coartación de la aorta, enfermeades cerabrales.
Factores de Riesgo Mayores, Hipertensión, Edad (>55 en varones, 65 en mujeres), Diabetes mellitus, LDL elevado, Colest total alto ó
bajo HDL, TFG < 60 ml/min, Historia familiar de ECV prematura (H <55 ó M < 65), Microalbuminuria, Obesidad (IMC > 30 Kg/m2)
Inactividad Física, Tabaco. FISIOPATOGENIA: Patologia multifactorial producto de la interacción de factores prevenibles y no
prevenibles, además de hiperactividad del sistema nervioso simpatico, alteración de catecolaminas y quimio y barorreceptores.
Alteraciones del sistema renina-angiotensina-aldosterona, altaracion de resistencias vascular, del volumen intravascular, constricion
arteriolar, e hipopotasemia y defecto natriuretico por capacidad disminuida de reducir la carga de sodio e hídrica. Aumento de sodio y
calcio intracelular, alteración del transporte Na/K. CUADRO CLINICO: Buscar intensionadamente, medida apropiada de PA, Verificación
en el brazo contralateral, Examen del fondo de ojo, IMC, Auscultación carotídea, abdominal y soplos femorales, Palpación de la
glándula tiroidea, Examen cardiaco y pulmonar completo, Exploración abdominal: riñones, detección de masas y pulsaciones aórticas
anormales, palpación de pulsos y edemas en extremidades inferiores, Valoración neurológica, buscar síntomas de órgano blanco,
cefalea, acufenos, fosfenos, apresion torácica, palpitaciones, fatiga, adinamia, cansancio, edema, alteraciones visuales, alteraciones
sensitivas. DIAGNOSTICO: Se recomienda realizar toma de TA a pacientes mayores de 35 años con o sin síntomas sugestivos de HTA. La
clasificación de la presión arterial según la OMS: Normal menor de 140/90 mmHg. Limítrofe: 140-160 y 90-95 respectivamente, HTA
(leve) 140-180 y 90-105; HTA moderada y severa > 180 y > 105 mmHg, o la JNC7: optima <120/80, Normal <130/85, Normal alta
<139/89, HTA estadio 150/99, HTA estadio 2 <179/109, HTA estadio 3 >180/110. TRATAMIENTO: PAS y PAD < 140/90 mmHg. Descenso
en complicaciones cardiovasculares. Con DM o IR el objetivo es de PA es < 130/80 mmHg. IMC 25, NaCl < 6 grs, ejercicio, no alcohol,
tabaco, iniciar hidroclorotiazida genera natriuresis, depleciona el espacio intravascular, inhiben reabsorción de Na+ y Cl- en el túbulo
distal. IECA´s. Bloquean de forma competitiva la enzima que convierte angiotensina I en angiotensina II. Disminución importante de
angiotensina II y aldosterona, con estímulo secundario de la síntesis de renina. Antagonistas de los receptores AT1: Ejercen un bloqueo
competitivo uniéndose al receptor AT1 y evitando las acciones derivadas del estímulo de éste. Simpaticoliticos: alfametildopa estimula
los receptores a2adrenérgicos en el SNC, disminuyendo el tono simpático periférico. Alfabloqueantes: prazocin actúa sobre receptores
a-1 postsinápticos. Metoprolol: Reduce gasto cardíaco a través de la frecuencia y del inotropismo cardíaco, bloqueo de la liberación de
renina hasta 60% a nivel renal, calcioantagonistas: verapamilo, diltiazem y nifedipino. PRONOSTICO: Las causas de muerte entre los
hipertensos se distribuye de la siguiente manera: 50% fallecerán por enfermedad cardiovascular, principalmente por cardiopatía
isquémica. 33% por enfermedad cerebrovascular. 10 a 15% por insuficiencia renal. 2 a 7% por otras causas. COMPLICACIONES: Daño
órgano diana: hipertrofia ventricular izquierda, angina, infarto de miocardio, revascularización coronaria, insuficiencia cardiaca, ictus ó
ataque isquémico transitorio, demencia, enfermedad renal crónica, enfermedad arterial periférica, retinopatía. REHABILITACION:
Disminuir la remodelación vascular (engrosamiento de la intima, media y adventicia) y la repercusión visceral (remodelado miocárdico,
nefropatía hipertensiva, retinopatía hipertensiva).

CASO CLINICO
Se trata de femenino de 52 años de edad la cual acude a consulta por presentar desde hace 6 meses cefalea global de intensidad leve a
moderada, que se incrementa con los esfuerzos y bajo estrés laboral principalmente, agrega que ha tenido episodios de mareo no
relacionado a el movimiento, asi como ruidos e intolerancia a la luz, las ultimas semanas ha notado que le aprietan los zapatos, y se
cansa con facilidad al subir las escaleras en su trabajo. Cuenta con antecedentes familiares de importancia padre finado por IAM súbito
a la edad de 40 años, madre actualmente viva con diabetes mellitus tipo 2, dislipidemia e hipertensión arterial bajo tratamiento. APP
tabaquismo positivo desde la edad de 20 años a razón de 5 cigarrillos diarios, alcoholismo social, vida sedentaria y obesidad grado I.
AGO desarrollo eclampsia hace 30 años y tomo anticonceptivos por 15 años, A la EF IMC 32, TA 145/100mmHg. FC 93 lpm, FR 23,
Paciente con palidez de tegumentos, manchas algodonosas y hemorragias en llama en fondo de ojo, discreta ingurgitación yugular al
decúbito dorsal, ruidos respiratorios disminuidos, ruidos cardiacos normales, edema maleolar ++, laboratorios Hb 12, Hto 49, glucosa
116, creatinina 1.6, acido urico 9, colesterol 256, triglicéridos 380, EGO: proteinuria +, bacterias ++, Estudios de gabinete: tele de torax
con discreta hipertrofia ventricular izquierda, ECG con QRS amplios de V4, V5 y V6.

PREGUNTA
¿Cuál de los siguientes criterios es el más útil para para establecer el diagnostico?

RESPUESTA
a.- Cifras tensionales altas en repetidas ocaciones.
b.- Sospecha de alteraciones de órgano blanco.
c.- Factores de riesgo presentes.

CURSO ENARM CMN SIGLO XXI TEL: 36246001 Pharmed Solutions Institute PÁGINA 8
MANUAL DE TRABAJO DEL CURSO ENARM CMN SIGLO XXI
d.- Alteraciones de estudios paraclinicos.

PREGUNTA
¿En que categoria se encuentra actualmente el paciente?

RESPUESTA
a.- Normal superior.
b.- Etapa 2.
c.- Etapa 1.
d.- Etapa 3.

PREGUNTA
¿Cuál es la patología diferencial más probable que puede sospecharse con los datos clínicos y laboratorios actuales?

RESPUESTA
a.- Patología renal.
b.- Uso de estrógenos.
c.- Hipertensión vascular renal.
d.- Hiperaldosteronismo primario.

PREGUNTA
¿En relación al estado del paciente, cuál de los auxiliares diagnosticos esta indicado para continuar con el manejo integral del
padecimiento?

RESPUESTA
a.- Realizar ecocardiograma.
b.- Realizar urografía excretora.
c.- Repetir BH, QS y EGO.
d.- Solicitar electrolitos sericos.

PREGUNTA
¿Considerando los signos y síntomas que presenta el paciente, que intervención no farmacológica proporciona un mayor impacto para
la reducción de cifras tensionales?

RESPUESTA
a.- Dieta hiposodica.
b.- Dieta hipocalórica.
c.- Dieta hipoproteica.
d.- Dieta normokalemica.

PREGUNTA
¿Cuál de los siguientes farmacos presenta mayor utilidad para el paciente?

RESPUESTA
a.- IECA´s.
b.- Calcio antagonistas.
c.- Diureticos.
d.- ARA II.

PREGUNTA
¿Cuál de los siguientes esquemas farmacológicos presenta mayor utilidad para el paciente?

RESPUESTA
a.- Metoprolol, hidroclorotiazida y nifedipina.
b.- Captopril, hidroclorotiazida y metoprolol.
c.- Lisonapril, amlodipino ó nifedipino.
d.- Losartan, hidroclorotiazida.

PREGUNTA
¿Cuáles son las condiciones no es causa de envio a segundo nivel a los pacientes con hipertensión arterial de difícil control?

RESPUESTA
a.- Falta de recursos para continuar con el manejo integral del paciente.
b.- Datos sugestivos de daño a órgano blanco.
c.- Requerir más de 3 farmacos.
d.- Presencia de crisis hipertensivas.

CURSO ENARM CMN SIGLO XXI TEL: 36246001 Pharmed Solutions Institute PÁGINA 9
MANUAL DE TRABAJO DEL CURSO ENARM CMN SIGLO XXI
PREGUNTA
¿Cuál órgano blanco presenta mayor evidencia de daño en el paciente actual?

RESPUESTA
a.- Retina.
b.- Riñon.
c.- Corazón.
d.- Arterias.

PREGUNTA
¿Cuáles son los cambios bioquímicos más probables por los datos clínicos en el caso descrito?

RESPUESTA
a.- Aumento de angiotensina II.
b.- Aumento de endotelinas.
c.- Aumento de noradrenalina.
d.- Disminución de tromboxano A2

PREGUNTA
¿Cuáles de los cambios funcionales no es dependiente del sistema cardiovascular?

RESPUESTA
a.- Hiperactividad simpatica.
b.- Aumento del volumen sanguineo.
c.- Disminucion FNA.
d.- Expanción del volumen extracelular.

PREGUNTA
¿Cuál de los elementos, es menos adecuado para garantizar el adecuado control, vigilancia y seguimiento en este paciente?

RESPUESTA
a.- Toma de presión arterial al menos 2 veces al mes.
b.- Envio oportuno a segundo nivel al diagnostico inicial.
c.- Evaluación oftalmológica para manejo de retinopatía.
d.- Solicitud de ecocardiograma.

PREGUNTA
¿Cuál de los resultados de los índices o clasificaciones siguientes es más firme para la referencia del paciente antes descrito a segundo
nivel de atención?

RESPUESTA
a.- Clasificación de keith, Wagener y Barker.
b.- Indice de KDOQI.
c.- Índice de Sokolow-Lyon.
d.- ATP III.

PREGUNTA
¿Cuáles son las complicaciones mas frecuentes en el paciente con hipertensión arterial?

RESPUESTA
a.- Infarto agudo al miocardio.
b.- Accidente vascular cerebral.
c.- Insuficiencia cardiaca congestiva.
d.- Insuficiencia renal crónica.

PREGUNTA
¿Cuál de los siguientes cambios histológicos renales no se presentan en la hipertensión arterial?

RESPUESTA
a.- Inadecuado desarrollo del sistema capilar.
b.- Esclerosis glomerular.
c.- Necrosis fibrinoide glomerular.
d.- Fibrosis intersticial.

PREGUNTA
¿Cuáles son los cambios anatomicos menos frecuentes que se presentan de la retina en la hipertensión arterial?

CURSO ENARM CMN SIGLO XXI TEL: 36246001 Pharmed Solutions Institute PÁGINA 10
MANUAL DE TRABAJO DEL CURSO ENARM CMN SIGLO XXI
RESPUESTA
a.- Papiledema del nervio óptico.
b.- Atrofia.
c.- Hemorragias.
d.- Ceguera.

PREGUNTA
¿Cuál es el efecto farmacodinamico más útil para la nefroproteccion en el tratamiento de la hipertensión arterial?

RESPUESTA
a.- Diurético con ahorro de K.
b.- Inhibidores de la enzima de conversión de la AGT
c.- Antagonistas de receptores de AGT II.
d.- Bloqueadores de canales de calcio.

PREGUNTA
¿Cuál es la estrategia más útil para la rehabilitación de las alteraciones cardiovasculares?

RESPUESTA.
a.- Control clínico y de los factores de riesgo.
b.- Entrenamiento físico programado.
c.- Tratamiento psicológico.
d.- Estudio de familiares de primer grado.

PREGUNTA
¿Cuál es la estrategia más útil para la rehabilitación de las alteraciones nefrológicas del caso actual?

RESPUESTA
a.- Mantener los niveles de tensión arterial estricta.
b.- Monitorización de KDOQI semestral.
c.- Reducción de peso, lípidos y tabaco.
d.- Disminuir la actividad del sistema nervioso simpatico.

PREGUNTA
¿Cuál es la estrategia más útil para rehabilitacion de las alteraciones oftamologicas del caso actual?

RESPUESTA
a.- Mantener los niveles de tensión arterial estricta.
b.- Monitorización de KDOQI semestral.
c.- Reducción de peso, lípidos y tabaco.
d.- Disminuir la actividad del sistema nervioso simpatico.

PREGUNTA
¿Cuál es la acción más importante para prevenir los factores de riesgo de hipertensión arterial en el primer nivel de atención?

RESPUESTA
a.- Mantener IMC menor a 25/Kg/m2.
b.- Dieta hiposodica, hipocalórica y normokalemica.
c.- Evitar tabaquismo, alcoholismo y otras toxicomanias.
d.- Estilo de vida no sedentario, ejercicio aerobico y manejo del estrés.

PREGUNTA
¿Cuáles son las acciones para limitar los factores de riesgo de hipertensión arterial en el primer nivel de atención para este caso
específico?

RESPUESTA
a.- Establecer la meta de tensión arterial 130/90 mmHg.
b.- Uso de IECA para nefroprotección.
c.- Uso de hipoglucemiante, bezafibrato y pravastatina.
d.- Establecer meta de 10 % de peso por reducir?

CASO CLINICO
Se trata de masculino de 78 años de edad el cual se acude a consulta por dolor articular en rodilla izquierda, durante la exploración
usted identifica que su cifras tensionales son las siguientes, TA 80/170mmHg, repite la toma de presión y se mantiene semejante, el
paciente niega sintomatología asociado a las cifra tensionales. El paciente cuenta con antecedente de un episodio de isquemia cerebral
transitoria hace 6 meses.

CURSO ENARM CMN SIGLO XXI TEL: 36246001 Pharmed Solutions Institute PÁGINA 11
MANUAL DE TRABAJO DEL CURSO ENARM CMN SIGLO XXI
PREGUNTA
Cual es conducta a seguir para el manejo de este caso?

RESPUESTA
a.- Reducción de peso y sal.
b.- Hidralacina 25 mg cada 24 hrs.
c.- Captopril e hidralacina.
d.- Enalapril y lisonipril.

PREGUNTA
Cual es el objetivo sobre el índice de masa corporal para el tratamiento del paciente?

RESPUESTA
a.- Mantener el índice corporal entre 20 y 25 Kg / m2.
b.- Mantener la tensión arterial sistólica al menos en 140 mmHg.
c.- Mantener la tensión arterial diastólica menor a 90 mmHg?.
d.- Disminuir el riesgo de IAM o EVC.

PREGUNTA
Cual es el tratamiento de elección farmacológico considerando el antecedente del paciente?

RESPUESTA
a.- Enalapril 10 a 20 mg.
b.- Losartan 50 mg
c.- Hidroclorotiacida 12.5 a 25mg.
d.- Amlodipino.

PREGUNTA
El paciente se encuentra estable actualmente con cifras tensionales de 120/80 mmHg, cual de las siguientes medidas de referencia
debe realizar para envio a tercer nivel?

RESPUESTA
a.- Realizar QS, BH, EGO.
b.- Laboratorios de rutina más Rx de torax y electrocardiograma.
c.- Buscar primeramente daño a órgano blanco.
d.- No es necesario ya que fue un hallazgo reciente.

PREGUNTA
El paciente fue evaluado por cardiólogo y volvió para continuo de control, dentro de las medidas enviadas por el mismo fue monitorizar
la tensión arterial supina, sentado y de pie. Cual es la razón de esta medida.

RESPUESTA
a.- Es una medida que garantiza las cifras.
b.- Buscar ortostatismo asintomático.
c.- Verificar el efecto del tratamiento.
d.- No tiene un efecto demostrable.

CASO CLINICO
Acude a consulta paciente femenino de 51 años de edad originaria de Campeche, acude a su control mensual, cuenta con antecedentes
de Hipertensión arterial y diabete mellitus desde hace 10 y 5 años respectivamente, sus signos vitales fueron TA 155/95 mmHg, FC 58
lpm, FR 27 rpm, laboratorios glucosa en ayunas de 210, hemoglobina glucosilada 6, urea 17, creatinin 1,1 mg/dl, refiere haber
presentado mareo ocacional, cefalea global acompañado de sensación de sueño, frio con mareo ocacional y disnea de leve a
moderados esfuerzos, se observa edema de miembros inferiores. Su tratamiento es captopril, metoprolol y glibenclamida.

PRENGUNTA
Se realiza ECG, considerando las manifestaciones que se han presentado, que conducta terapéutica considera más apropiada realizar
de forma mediata?

a.- Incrementar los farmacos hasta meta de <120/80 mmHg.


b.- Retirar metoprolol continuar con captopril y glibenclamida.
c.- Mantener glibenclamida, retirar metoprolol e iniciar losartan.
d.- Agregar hidroclorotiazida al manejo actual.

PREGUNTA
Cuales son objetivos mas importantes con el nuevo manejo debido a los antecedentes?

CURSO ENARM CMN SIGLO XXI TEL: 36246001 Pharmed Solutions Institute PÁGINA 12
MANUAL DE TRABAJO DEL CURSO ENARM CMN SIGLO XXI
RESPUESTA
a.- Disminuir el riesgo de IAM.
b.- Disminuir el riesgo a EVC.
c.- Disminuir el bloque b-adrenergico por riesgo a hipoglucemia.
d.- Disminuir el riesgo de progresión de nefropatía.

PREGUNTA
Dos años después el paciente continua en tratamiento, actualmente ha surgido edema de miembro inferiores, fatiga de medianos
esfuerzos, además de dos eventos de isquemia cerebral transitoria menor a 2 horas, cual es la conducta a seguir?

RESPUESTA
a.- Indica estudios para identificar alguna complicación a órgano blanco.
b.- Modifica la dosis para mantener un mejor manejo.
c.- Indica mayores medidas generales de control de la tensión arterial y diabetes mellitus.
d.- Considera suficientes criterios para enviar a segundo nivel.

CASO CLINICO
Se trata de masculino de 22 años de edad el cual es ingresado al servicio de urgencia debido a la presencia de cefalea intensa,
palpitación, sensación de muerte inminente, diaforesis, refiere que ya le había ocurrido esto anteriormente pero no tan intenso, su
tensión arterial fue de 140/100 mmHg. FC 115, FR 32, se observa agitado con dolor precordial resto de exploración sin datos por
agregar, laboratorios de rutina sin datos patológicos.

PREGUNTA
Considerando la presencia del cuadro clínico cual es la conducta a seguir mas adecuada.

RESPUESTA
a.- Evaluar metanefrinas /24 hrs.
b.- Medicina nuclar con MIBG.
c.- Prueba de supresión con clonidina.
d.- Prueba de supresión con glucagon.

PREGUNTA
Posterior a la realización del estudio anterior, se observo una imagen isointensa y continuaron los síntomas, cual es la conducta aseguir.

RESPUESTA
a.- Realizar búsqueda en cuello, torax y abdomen.
b.- Identificar CAS/CARS
c.- Supresion con Fluhidrocortisona.
d.- Realizar urografía escretora.

INSUFICIENCIA CARDIACA AGUDA Y CRONICA (TEMA)


DEFINICION: La Insuficiencia Cardíaca (IC) puede ser definida como un síndrome clínico complejo que proviene de anormalidades
cardíacas estructurales y/o funcionales, adquiridas o hereditarias, que conllevan al empeoramiento de la capacidad de llenado y
eyección ventricular. EPIDEMIOLOGIA: Es la tercera causa más frecuente de ingresos hospitalarios y la primera causa de ingresos por
enfermedad cardiovascular. 15 millones de pacientes con IC. Disfunción ventricular asintomática 4% Prevalencia de la IC: 2-3% y
aumenta drásticamente a los 75 años de edad. 10-20% en el grupo de pacientes de 70-80, La IC-FEC es más común en pacientes de
edad avanzada, mujeres, hipertensos y diabéticos. La IC es la causa del 5% de los ingresos hospitalarios urgentes, 10% de las camas
hospitalarias, 2% de gastos sanitarios nacionales. ETIOLOGIA: Daño o perdida del musculo cardiaco, isquemia miocárdica, hipertensión
arterial sistémica, miocardiopatías, valvulopatías y arritmias. En muchos de los casos, sin embargo, la etiología permanece sin
conocerse. La enfermedad coronaria causa la IC en cerca del 70-80% de los pacientes. Enfermedad valvular es origen del 10% de los
casos. Miocardiopatías 10%. FISIOPATOGENIA: Los mecanismos neurohumorales pueden causar efectos indeseables como:
vasoconstricción prolongada, aumento de poscarga, hipertrofia ventricular, dilatación ventricular, excesiva retención de Na y H2O,
Arritmias. PNA: respuesta a dilatación auricular, hormona contrarreguladora, vasodilatación, incremento en excreción de Na y H2O,
niveles reflejan severidad de la IC. Activacion del SN simpatico: vasoconstricción, incremento en poscarga, incremento en el consumo
miocárdico de o2, hipertrofia ventricular, activa SRAA: incremento en precarga (incremento en trabajo ventricular). CUADRO CLINICO:
Síndrome clínico en el que los pacientes presentan las siguientes características: Síntomas típicos de IC (disnea en reposo o con el
ejercicio, fatiga, edema) + signos típicos de IC (taquicardia, taquipnea, estertores pulmonares y edema periférico, hepatomegalia) +
Evidencia de una anomalia estructural o funcional del corazón en reposo (cardiomegalia, 3er ruidos, soplos cardiacos, anomalias en ekg,
elevación de PNA). Criterios de framingham: mayores (disnea paroxistica nocturna u ortopnea, ingurgitacion yugular, estertores,
cardiomegalia, edema agudo de pulmon, galope por tercer tono, PCV > 16mmHg, tiempo de circulacion >25sg, reflujo hepatoyugular.
Criterios menores, edema maleolar, tos nocturna, disnea de esfuerzo, hepatomegalia, derrame pleural, reduccion de la capacidad vital,
taquicardia >120 lpm. DIAGNOSTICO: La ACC/AHA, Estadio A: con alto riesgo de IC. Anomalía estructural o funcional no identificada; sin
signos ni síntomas, Estadio B: enfermedad cardiaca estructural desarrollada claramente en relacion con la IC pero sin signos ni
sintomas. Estadio C: IC sintomatica asociada a enfermedad estructural subyacente. Estadio D: enfermedad cardiaca estructural
avanzada y sintomas acusados de IC en reposo a pesar de tratamiento medico maximo. La NYHA: Clase I sin limitaciones de la actividad
fisica, el ejercicio fisico normal no causa fatiga, palpitaciones o disnea, Clase II ligera limitacion de la actividad fisica sin sintomas en

CURSO ENARM CMN SIGLO XXI TEL: 36246001 Pharmed Solutions Institute PÁGINA 13
MANUAL DE TRABAJO DEL CURSO ENARM CMN SIGLO XXI
reposo; la actividad fisica normal causa fatiga, palpitaciones o disnea. Clase III Acusada limitacion de la actividad fisica, sin sintomas en
reposo, cualquier actividad fisica provoca la aparicion de los sintomas, Clase IV incapacidad de realizar actividad fisica, los sintomas de
la IC estan presentes en reposo y aumentan con cualquier actividad fisica. DIAGNOSTICO DIFERENCIAL: Estados que semejan la
sintomatología de IC: anemia, tiortoxicosis, embarazo, septicemia, insuficiencia hepática, cortocircuitos A-V, enfermedad de paget,
beriberi. TRATAMIENTO: Autocontrol, adherencia al tratamiento: solo 20-60% cumple Tx farmacológico y no farmacológico,
reconocimiento de síntomas, control del peso, dieta: restricción de Na, restricción de líquidos. No alcohol: inotrópico negativo,
hipertensa, arritmias. IECA a todos los pacientes con IC sintomática y una FEVI ≤ 40%. En pacientes hospitalizados, se iniciará el
tratamiento con IECA antes del alta. Se indicará el uso de un BB en todos los pacientes con IC sintomática y una FEVI ≤ 40%. ARA II con
una FEVI ≤ 35% e IC grave y sintomática. Sin hiperpotasemia ó disfunción renal significativa. Hidralazina + isosorbide en sintomáticos
con FEVI ≤ 40%, (Tx en intolerancia a IECA y ARA). Digoxina en IC y FA (controla la FC, en ritmo sinusal con IC sintomática y FEVI ≤ 40.
Diurético recomendado en IC y signos o síntomas clínicos de congestión. Alivio de los síntomas y signos de congestión venosa pulmonar
y sistémica. PRONOSTICO: Del número total de pacientes, el 50% fallece a los 4 años y el 40% de los pacientes ingresados por IC fallece
o reingresa durante el primer año. COMPLICACIONES: Insuficiencia cardiaca conjestiva, arritmias, muerte súbita, edema agudo
pulmonar, etc. REHABILITACION: Cambio de estilo de vida, evitar la evolucacion de la enfermedad, reducir riesgos, ejercicio
programado.

CASO CLINICO
Se trata de paciente femenino de 73 años de edad la cual cuenta con 20 años de hipertensión arterial y diabetes mellitus, con
tratamiento médico pero mal apego farmacológico y no farmacológico, actualmente se encuentra con el siguiente esquema
hidroclorotiazida 100mg/dia, furosemida 20 mg/dia, enalapril 60 mg/dia, amlodipino 10 mg/día, acido acetilsalicilico 100 mg/dia,
glipizida 20 mg dividido en dos dosis, actualmente acude por mareo ocasional con caída de su altura en dos ocasiones, se ha
desorientado dos veces desconociendo a la personas. A la exploración física se observa paciente orientada, adecuado estado de alerta,
con leve desihratacion con facies pletóricas, se observa ingurgitación yugular de 5 cm, campos pulmonares con estertores de
predominio en las bases, se escucha desdoblamiendo del segundo ruido cardiaco, leve hepatomegalia, miembros inferiores con edema
+++, los cuales se encuentran calientes. Signos vitales: TA 105/80mmHg, FC 109 lpm, FR 31rpm. Se realiza ECG que se muestra y tele de
torax. Se calculo una FE >45%,

PREGUNTA
Cuáles son las manifestaciones clínicas más especificas para establecer el diagnostico de la enfermedad?

RESPUESTA
a.- Ortopnea, disnea y tos noctura.
b.- Edema, fatiga y congestion torácica.
c.- Electrocardiograma y radiografia de torax.
d.- Ingurgitacion yugular, hepatomegalia y fracción de eyección.

PREGUNTA
Que de Framingham para el diágnosticos de IC tiene este paciente?

RESPUESTA
a.- 4 mayores y 2 menores.
b.- 4 mayores y 3 menores.
c.- 3 mayores y 2 menores.
d.- 3 mayores y 3 menores.

PREGUNTA
Cuál es el factor de riesgo que puede conducir a un estado descompensado en este caso?

RESPUESTA
a.- Incumplimiento del tratamiento.
b.- SAOS.
c.- Arritmia, anemia ó hemorragia.
d.- Antiarritmico de clasa Ia y Ic.

PREGUNTA
Cuáles son las manifestaciones clínicas mas útiles para realizar un diagnostico diferencial de la IC sistólica vs IC diastólica?

RESPUESTA
a.- No son distinguibles.
b.- Disnea y edema.
c.- Taquicardia y taquipnea.
d.- Congestión hepática.

PREGUNTA
Cuáles son los auxiliares diagnosticos mas útiles que no han sido realizados en el paciente para identificar las complicaciones de la
enfermedad?

CURSO ENARM CMN SIGLO XXI TEL: 36246001 Pharmed Solutions Institute PÁGINA 14
MANUAL DE TRABAJO DEL CURSO ENARM CMN SIGLO XXI
RESPUESTA
a.- Puebas de función hepática.
b.- Electrolitos sericos.
c.- Pruebas de estrés.
d.- Ecocardiograma.

PREGUNTA
Cuál es la medida farmacológica que intervendrá predominantemente sobre la respuesta neurohumoral en la retención de sodio y
liquidos?

RESPUESTA
a.- Sistema renina-angiotensina-aldosterona.
b.- Sistema arginina-vasopresina.
c.- Liberación de péptido natriuretico auricular.
d.- Inhibe el sistema de transporte Na + Cl -.

PREGUNTA
Cual de los siguientes factores no es determinante del consumo de oxigeno miocardico?

RESPUESTA
a.- Estrés parietal.
b.- Contractilidad.
c.- Frecuencia cardiaca.
d.- Fracción de eyección.

PREGUNTA
Antes de definir el tratamiento farmacológico, cual de las causas reversibles menos probable se encuentra en el caso?

RESPUESTA
a.- Depresión miocárdica inducida por alcohol.
b.- Antiinflamatorio no esteroideo.
c.- Estado de poscarga elevado.
d.- Uso de bloqueador de canales de calcio.

PREGUNTA
Cual de las siguientes medidas no es la mas recomendable para el actual caso clínico?

RESPUESTA
a.- Dieta con 5 a 6 g. de sal al día.
b.- Dieta sin sodio absoluta.
d.- Programa de gradual de ejercicio.
c.- Dieta normokalemica.

PREGUNTA
Cosiderando el estado actual de velocidad de filtrado glomerular del paciente, cual de los siguientes diureticos proporcionan un
equilibrio adecuado de natriuresis y caliuresis?

RESPUESTA
a.- Espironolactona.
b.- Hidroclorotiazida.
c.- Furosemida.
d.- Clortalidona.

PREGUNTA
Considerando el esquema farmacológico actual del paciente, que medida farmacológica es la mas adecuada para promover un menor
índice de morbilidad y mortalidad?

RESPUESTA
a.- Incrementar la dosis de enalapril.
b.- Añadir losartan.
c.- Iniciar esquema con digoxina.
d.- Iniciar sildenafil.

PREGUNTA
Cuál de los cambios neurohumorales que se presentan en la insuficiencia cardiaca es menos probable en este caso?

CURSO ENARM CMN SIGLO XXI TEL: 36246001 Pharmed Solutions Institute PÁGINA 15
MANUAL DE TRABAJO DEL CURSO ENARM CMN SIGLO XXI
RESPUESTA
a.- Aumento de la contractibilidad miocárdica.
b.- Aumento de la frecuencia cardiaca.
c.- Aumento del tono venoso.
d.- Disminución del volumen sanguíneo central efectivo.

PREGUNTA
Cuál de los cambios funcionales que se presentan en el paciente con insuficiencia cardiaca es menos probable en este caso?

RESPUESTA
a.- Volumen sistólico reducido.
b.- Poscarga excesiva.
c.- Depresión de la contractibilidad.
d.- Disminución del volumen telediastolico.

CASO CLINICO
Se trata de paciente masculino de 63 años de edad la cual acude a consulta de revisión, en primer nivel de atencion, cuenta con
antecedentes de dislipidemia, hipertensión arterial y diabetes mellitus, actualmente tratada con atorvastatina, hidroclorotiazida,
losartan y glibenclamida, a dosis minima toxica, hace 2 años presento IAM, el cual se trato oportunamente, acude debido a la presencia
de edema de miembros inferiores, fatiga, disnea de medianos esfuerzos, ortopnea y tos, a la exploración física se aprecia reflujo
hepatoyugular, soplo carotideo, estertores finos bibasales, FC 91 lpm, FR 26, TA 150/80 mmHg.

PREGUNTA
Se solicita estudios de laboratorio y gabinete, los cuales están pendientes de resultado, considerando los criterios de framingham para
el diagnostico de la insuficiencia cardiaca, cuantos mayores y menores respectivamente presenta?

RESPUESTA
a.- 2 mayores y 3 menores.
b.- 2 mayores y 2 menores.
c.- 3 mayores y 3 menores.
d.- 3 mayores y 2 menores.

PREGUNTA
Cual es la clase funcional y el ICT que en este momento presenta el paciente?

RESPUESTA
a.- Clase funcional I e ICT 0.62.
b.- Clase funcional III e ICT 0.59.
c.- Clase funcional IV e ICT 0.68.
d.- Clase funcional II e ICT 0.70

CASO CLINICO
Masculino de 43 años de edad el cual reingresa a urgencias debido a dificultad respiratoria, disnea de minimos esfuerzos, tos
incluyendo en la noche, además de dolor toraccico cuando se presenta la disnea, este dolor disminuye muy lentamente con el reposo,
el paciente cuenta con el antecedente de IAM egresando 15 dias antes de este evento. Se observa reflujo hepatoyugular con
hepatomagalia, se ausculta tercer ruido, Se observo es siguiente ECG. Sus contantes vitales son: IMC 31, FC 119 lpm, FR 31, TA 140/70
mmHg,

PREGUNTA
Cuales son los criterios de framingham presenta el paciente en este momento?

RESPUESTA
a.- 3 menores y 4 mayores.
b.- 4 menores y 4 mayores.
c.- 3 menores y 3 mayores.
d.- 4 menores y 2 mayores.

PREGUNTA
Cual es condición que se presenta tomando la clasificación de killip y Kimball.

RESPUESTA
a.- Clase funcional I.
b.- Clase funcional II.
c.- Clase funcional III.
d.- Clase funcional IV.

CURSO ENARM CMN SIGLO XXI TEL: 36246001 Pharmed Solutions Institute PÁGINA 16
MANUAL DE TRABAJO DEL CURSO ENARM CMN SIGLO XXI
PREGUNTA
Considerando el estado actual del paciente cual es el pronostico?
a.- Tasa de mortalidad del 6%.
b.- Tasa de mortalidad del 17%.
c.- Tasa de mortalidad del 38%.
d.- Tasa de mortalidad del 81%.

CASO CLINICO
Ingresa paciente de 39 años de edad el cual ingresa a urgencias 4 horas antes, por presencia súbita de dolor retrosternal acompañado
de presión toracicca “sensación de mucho peso sobre su pecho” que se presento en la madrugada, nausea, dificultad para respirar,
refiere que la noche anterior comio abundante y consumio alcohol hasta la embriaguez, al ingreso se observa confuso, diaforético con
palidez y cianosis distal, ingurgitación yugular, se asculta estertores cripitantes bibasales, tercer ruido, TA 90/60 mmHg, FC 132 lpm, FR
36 rpm, Temp. 36.3 grados. Gasto urinario de < 20 ml/h, se observaron los siguientes estudios.

PREGUNTA
Cual es la clase funcional de Kllip que presenta el caso?

RESPUESTA
a.- Clase funcional Killip I.
b.- Clase funcional Killip II.
c.- Clase funcional Killip III.
d.- Clase funcional Killip IV.

PREGUNTA
Cual es el pronostico para la vida en este caso considerando la clase funcional?

a.- Tasa de mortalidad del 6%.


b.- Tasa de mortalidad del 17%.
c.- Tasa de mortalidad del 38%.
d.- Tasa de mortalidad del 81%

PREGUNTA
Se calculo los siguientes resultados (LC) de 2 L/m2/min. Considerando los elementos clínicos y el (LC), en la curva de starling cual es el
diagnostico?

RESPUESTA
a.- Insuficiencia cardiaca.
b.- Edema agudo pulmonar.
c.- Choque cardiogenico.
d.- Choque distributivo.

PREGUNTA
Se realizó ecocardiograma transesofagico de urgencias, donde se reporta FE < 30%, cual es % de mortalidad a 6 meses del paciente?

RESPUESTA
a.- del 15 al 20 %
b.- del 20 al 25 %
c.- del 25 al 30 %
d.- 30 al 35 %

CASO CLINICO
Se trata de masculino de 62 años de edad diabético e hipertensión controlado con glibenclamida y enalapril a dosis minimas toxicas
respectivamente, fue fumador hasta los 45 años. Hace 5 años presento infarto al miocardio tratado con trombolitico con buena
respuesta, acude a consulta debido cansacio, fatiga y dolor toracicco opresivo que sede con el reposo, ocasionalmente el dolor
toraccico es ardoroso o urente, además de palpitaciones ocacionales, disnea de medianos esfuerzos, ortopnea así como fatiga, a la EF
presenta ingurgitación yugular, frote carotideo, area cardiopulmonar con estertores crepitantes finos, palidez de tegumento, cianosis
distal, miembros con edema de miembros inferiores, TA 140/70 mmHg, FC 93 lpm, FR 26 rpm, IMC 29.5.

PREGUNTA
Se realizarón los siguientes estudios de rutina hemoglobina glucosilada 8, creatinina 0.5, acido urico 7.0 mg/dl, colesterol HDL 27 mg/dl,
triglicéridos 210 mg/dl, considerando el estado actual de caso clínico, cual es la conducta a seguir?

RESPUESTA
a.- Solicitar BNP ó NT-proBNF.
b.- Solicitar ecocardiograma transesofagico.
c.- Solicitar ECG en reposo y holter.

CURSO ENARM CMN SIGLO XXI TEL: 36246001 Pharmed Solutions Institute PÁGINA 17
MANUAL DE TRABAJO DEL CURSO ENARM CMN SIGLO XXI
d.- Solicitar prueba de estrés.

PREGUNTA
Cual es la conducta terapéutica a seguir para disminuir el riesgo cardiometabolico?

RESPUESTA
a.- Iniciar hidroclorotiazida
b.- Iniciar verapamilo.
c.- Iniciar betabloqueador.
d.- Iniciar atorvastatina

PREGUNTA
Se realizo ecocardiograma que reporta una FE > 45 %, el electrocardiograma reporta extrasístoles ventriculares, cual es la conducta
farmacológica mas adecuada para disminuir el remodelado ventricular y mejorar la actividad eléctrica cardiaca?

RESPUESTA
a.- Iniciar betabloqueador.
b.- Iniciar verapamilo.
c.- Iniciar losartan.
d.- Iniciar diltiazem.

PREGUNTA
El paciente se mantiene en buenas condiciones sin embargo 12 meses después acude a cita de control, se realizo electrocardiograma de
control, donde se apresia bloqueo de primer grado, el paciente agrega que la monitorización ambulatoria de la presión fue adecuada,
sin embargo la opresión toracicca se ha presentado de forma expontanea y con mayor duración, asi como la disnea se ha presentado
mas frecuente, cual es el fármaco mas adecuado?

RESPUESTA
a.- Isosorbide 10 mg.
b.- Verapamilo 80 mg.
c.- Diltiazem 60 mg.
d.- Nitroglicerina 18 mg.

CASO CLINICO
Se trata de paciente femenino de 67 años de edad, que acude a consulta por cansancio, fatiga, dificultad para respirar, dolor toraccico
que disminuye con reposo, disnea de moderados esfuerzos, edema de miembros inferiores hasta 2/3 de la pierna, a la exploración física
se observo palidez de piel y mucosas, llenado capilar lento, estertores crepitantes leves, presencia de galope y trill. La paciente es
hipertensa desde hace 20 años con moderado apego a tratamiento con captopril 50 mg/dia, hidroclorotiazida 25 mg/12 hrs. Ha
presentado 2 crisis hipertensivas, asi como un evento de insuficiencia mesentérica, sus laboratorios EGO proteínuria, glucosuria, uratos,
densidad urinaria disminuida, colesterol 289 mg/dl, triglicéridos 720 mg/dl, creatinina 6.2 mg/dl, BUM 29, kalemia de 5.3, los signos
vitales fueron TA 160/105 mmHg, FC 96, FR 28, IMC 32.

PREGUNTA
Se realizo un ecocardiograma donde se observa una FE de < 40 %, durante su ingreso la paciente presento volumen urinario de 20
ml/hr. Considerando las características actuales cual es la clase funcional que presenta el caso?

RESPUESTA
a.- Clase funcional I.
b.- Clase funcional II.
c.- Clase funcional III.
d.- Clase funcional IV.

PREGUNTA
Considerando las características actuales cual es el K/DOQI que presenta el paciente, cual es el método mas adecuado para determinar
el estadio de la IRC?

RESPUESTA
a.- Depuracion de creatinina de 24 hrs.
b.- Biopsia renal.
c.- Ultrasonografia renal.
d.- Urografia escretora.

PREGUNTA
Cual es la conducta farmacológica mas adecuada considerando el K/DOQI y la clase funcional?

RESPUESTA

CURSO ENARM CMN SIGLO XXI TEL: 36246001 Pharmed Solutions Institute PÁGINA 18
MANUAL DE TRABAJO DEL CURSO ENARM CMN SIGLO XXI
a.- Verapamilo.
b.- Diltiazem.
c.- Telmisartan.
d.- Isosorbide.

PREGUNTA
Cual es la conducta inmediata a seguir para mejorar la precarga disminuyendo los niveles de potasio?

RESPUESTA
a.- Furosemide.
b.- Dialisis peritoneal.
c.- Hemodialisis.
d.- Soluciones metabolizadas.

PREGUNTA
Cual de las siguientes patologias es la principal causa de IRC?

RESPUESTA
a.- Diabetes mellitus.
b.- Hipertensión arterial.
c.- Glomerulonefritis.
d.- Enfermedad renal poliquística.

PREGUNTA
Cuales factores de riesgo aceleran el deterioro del funcionamiento renal en el caso actual?

RESPUESTA
a.- Dislipidemia.
b.- Edad avanzada.
c.- Diabetes mellitus.
d.- Anemia.

CASO CLINICO
Se trata de masculino de 59 años de edad, ingresa a urgencias debido a dificultad para respirar, cansancio, fatiga, tos productiva desde
hace 15 dias intensificandoce la noche previa al ingreso actual, cuenta con antecedentes de EPOC diagnosticado hace 10 años, posterior
a tabaquismo (una cajetilla diaria), con tratamiento de bromuro de iprapropio, salbutamol y betametasona, además fue diagnosticado
como hipertenso hace 2 años, inicio con cambios del estilo de vida y dieta con restricción de sodio, hace un año se considero incluir
captopril que fue cambiado 3 meses después por incremento de tos, desde entoces toma losartan, hidralazina. A la EF se observa facies
pletórica con hiperemia conjuntival, mucosas orales cianóticas moderadamente, se ausculta un soplo carotideo derecho, ingurgitación
yugular grado II, estertores subcrepitantes bibasales, con resonancia tipanica a la percusión, los ruidos cardiacos levemente
disminuidos, rítmico, abdomen con hepatomegalia 5 cm debajo del borde costal.

PREGUNTA
Considerando el estado actual del paciente cual es la conducta mas apropiada a seguir, para establecer la función cardiopulmonar?

RESPUESTA
a.- Espirometria.
b.- Ecocardiograma.
c.- Valor de natriuretico.
d.- Radiografia de torax.

PREGUNTA
Luego de realizar las pruebas anteriores se observo una FE menor a 40 %, la radiografia de torax mostro un indice cardiotoraccico
patológico, exudados algodonosos bilaterales, trama brocovascular incrementada, gases con PCO 49, PO2 85, potasio 6.3, Factor
natriuretico disminuido, a la expxloracion física se auscultan estertores crepitantes, sibilancias moderadas, a la percusión
hiperresonancia, se alcanza apresiar galope, cual es la conducta mas apropiada a seguir para disminuir la precarga, considerando todos
los factores?

RESPUESTA
a.- Retirar diurético ahorrador de potasio por diurético de asa.
b.- Iniciar con calcioantagonista dihidropiridinico de acción rápida.
c.- Incrementar la capacitancia vascular periférica con nitritos.
d.- Restriccion hídrica, dieta libre de sodio, diurético de asa.

CURSO ENARM CMN SIGLO XXI TEL: 36246001 Pharmed Solutions Institute PÁGINA 19
MANUAL DE TRABAJO DEL CURSO ENARM CMN SIGLO XXI
CASO CLINICO
Paciente varón, de 41 años de edad, diagnosticado previamente de SAF y con antecedentes personales de infarto lacunar
protuberancial izquierdo hacía 4 años, hipertensión arterial e hipercolesterolemia. Acudió a urgencias por dolor retrosternal al esfuerzo
rápidamente progresivo, con algún episodio en reposo, durante las dos semanas previas. El ECG demostraba amputación de los
vectores iniciales en las precordiales derechas y supradesnivel del segmento ST, de hasta 2 mm, en las derivaciones V1 a V4. En la
auscultación cardíaca se detectaba un soplo holosistólico rudo de intensidad II-III/VI en el foco aórtico y diastólico II/IV en el borde
esternal izquierdo bajo. CPK máxima de 71 U/l, troponina T de 0,17 ng/ml, 58.000 plaquetas, TTPA 44 s y en el estudio inmunológico, un
anticoagulante lúpico positivo débil y unos anticuerpos anticardiolipina (AAC) IgG moderadamente elevados (68 unidades fosfolipídicas
GPL). En la radiografía de tórax se observaba cardiomegalia global con campos pulmonares limpios. El ecocardiograma transtorácico
objetivó una válvula aórtica engrosada, con apertura estenótica y gradiente sistólico máximo instantáneo estimado de 59 mmHg y
regurgitación.

PREGUNTA
Cual es la conducta a seguir mas adecuada para identificar el compromiso fisiológico coronario?

RESPUESTA
a.- Ecocardiograma de estrés con dobutamina.
b.- Prueba de esfuerzo convencional
c.- Coronariografia percutenea.
d.- IRM-f con FDG cardiaca.

PREGUNTA
El paciente presento durante las siguientes semanas se present ortopnea, disnea de medianos esfuerzo, disminución de la uresis, cabe
destacar de hipokalemia, la gasometría reporto pH 7.21, PCO 41, Na 121, Cl 115, HCO3 42, se diagnostica acidosis respiratoria
secundaria, considerando la patologia de base, cual es la causa menos probable,

RESPUESTA
a.- Depresion respiratoria central secundaria.
b.- Sindrome de Pickwickian.
c.- Edema pulmonar crónico.
d.- Compensación respiratoria por alcalosis metabolica.

PREGUNTA
24 horas después de estancia intrahospitalaria, el paciente presenta esputo salmonelado, disminución del gasto urinario < 25 ml/hora,
confusión con agitación psicomotriz, diaforético, cianosis central, ingurgitación yugular III, estertores crepitantes, TA 70/50 mmHg, FC
139 lpm, FR 50 rpm. Cuál es la conducta a seguir para mantener la fracción de eyección adecuada sin incrementar la precarga?

RESPUESTA
a.- Dobutamina 20 µg/kg/min.
b.- Dopamina > 20 µg/kg/min.
c.- Epinefrina > 10 µg/min.
d.- Digoxina 10 ug/Kg/min.

PREGUNTA
Considerando la clasificación funcional clásica de la insuficiencia cardíaca (NYHA), cuál es la clase del paciente previa a la desición
anterior?

RESPUESTA
a.- Grado I
b.- Grado IIA
c.- Grado IIIA
d.- Grado IVA

PREGUNTA
Considerando los lineamientos del American College of Cardiology y el American Heart Association, en que estadio se encuentra?

RESPUESTA
a.- Estadio A
b.- Estadio B
c.- Estadio C
d.- Estadio D

CASO CLINICO
Se ingresa a urgencias a paciente femenino de 48 años de edad, originaria de Veracruz, empleada de agrícola, casada desde los 17 años,
gesta 5, para 3, aborto 1, cesarea 1, histerectomía a los 38 por miomatosis uterina, no fuma ni toma bebidas alcoholicas,
aparentemente sana, no toma medicamentos, anteriormente presento caries dentales pero fueron tratadas hace 7 meses sin embargo

CURSO ENARM CMN SIGLO XXI TEL: 36246001 Pharmed Solutions Institute PÁGINA 20
MANUAL DE TRABAJO DEL CURSO ENARM CMN SIGLO XXI
hasta hace 3 meses que comenzó padecimiento actual caracterizado por fatiga, disnea de medianos esfuerzos, sensación de
ahogamiento de predominio nocturno, es ingresada debido a que presenta tos con esputo fino color rosa, disnea de leves esfuerzo, a la
exploración física se observa paciente con estado grave, edematizada con cianosis central y periférica, ingurgitación yugular, soplo
mediodiastolico III/IV el cual predomina en posición erecta, edema de miembros pélvicos con lesiones isquémicas en dedos 1 y 2 de pie
derecho, tratado y contaminado con remedios locales, no se apercibe olor ni pus, los signos vitales son los siguientes TA 125/85 mmHg,
FC 102, FR 26, Temperatura 37.5 C. La paciente recibe diagnostico de insuficiencia cardiaca congestiva.

PREGUNTA
Cuál es la causa más probable para la insuficiencia cardiaca congestiva de este caso.

RESPUESTA
a.- Endocarditis bacteriana con insuficiencia valvular.
b.- Estenosis de la valvula mitral.
c.- Mixioma auricular.
d.- Melanoma metastasico.

CASO CLINICO
Se estabiliza a la paciente y se realizan estudios de gabinete donde se identifica una masa tumoral intracavitaria de 6 centímetros,
dentro de los estudios se realizaron TAC, IRM y cateterismo, 4 horas después la paciente presenta dificultad respiratoria intensa,
presencia de tos abundante con esputo asalmonelado, por lo que se intuba para soporte ventilatorio, se observa cianosis periférica con
pérdida de pulsos distales de miembros pélvicos, ausencia de ruidos abdominales seguido de parada cardiaca en dos ocasiones,
declarando su muerte 2 horas más tarde.

PREGUNTA
Cuál es la fue la intervención más probable que culmino con la embolia masiva en esta paciente.

RESPUESTA
a.- No se anticoagulo profilácticamente.
b.- El cateterismo cardiaco innecesario.
c.- El desprendimiento del mixioma.
d.- El paciente ya tenía signos de embolia.

CASO CLINICO
Hombre de 20 años de edad sin antecedentes mórbidos conocidos. Presentó cuadro de 3 días de evolución caracterizado por
odinofagia, tos seca, dolor precordial y dorsal asociado a disnea de reposo. Consultó en centro de atención donde se le solicita
radiografía de tórax, destacando gran cardiomegalia y signos de congestión pulmonar. Se contaba con examen físico y radiografía de
tórax de 3 años atrás sin ninguna alteración.

PREGUNTA
Fue derivado a centro de atención terciaria al cuarto día de evolución del cuadro. Hemodinamia siempre estable. Se realizo
electrocardiograma (ECG). Cual es la conducta farmacológica inicial mas apropiada para el caso?

RESPUESTA
a.- Amiodarona de 100 a 400 mg cada 24 horas.
b.- Diltiazem de 90 mg a 120 mg cada 12 horas.
c.- Verapamilo de 240 a 360 mg cada 24 horas.
d.- Esmolol de 50 a 100 µg/kg/min.

PREGUNTA
Se realizo un electrocardiograma previo a la decisión farmacológica, considerando el ECG el cual se observa anormal, cual de las
siguientes alteraciones del ritmo es más probable encontrar considerando la evolucion?

RESPUESTA
a.- Taquicardia ventricular intermitente.
b.- Taquicardia auricular intermitente rapida
c.- Taquicardia sinusal.
d.- Taquicardia auricular.

PREGUNTA
Ingresó hipotenso (84/56 mmHg), taquicárdico (116 latidos por minuto, irregular), afebril, frecuencia respiratoria de 18 por minuto y
saturando 92% con oxígeno ambiental, bien perfundido. La faringe se encontraba algo enrojecida, yugulares ingurgitadas. Sin
congestión pulmonar y examen cardíaco demostró tercer ruido y ritmo de galope. Extremidades sin edema. De los exámenes de
laboratorio destacaba discreta leucocitosis, PCR baja, enzimas cardíacas y troponinas normales, cual es la conducta mas adecuada a
seguir?

CURSO ENARM CMN SIGLO XXI TEL: 36246001 Pharmed Solutions Institute PÁGINA 21
MANUAL DE TRABAJO DEL CURSO ENARM CMN SIGLO XXI
RESPUESTA
a.- Prueba de esfuerzo con dobutamina.
b.- Ecocardiograma.
c.- Biopsia endocardica.
d.- Holter ambulatorio.

PREGUNTA
Entre otros estudios se realizó ecocardiograma, considerando el cuadro clínico actual cual de las siguientes aseveraciones es menos
probable esperar?

RESPUESTA
a.- Ventrículo izquierdo (VI) severamente dilatado.
b.- Disfunción sistólica global severa. FE > 40 %.
c.- Hipokinesia difusa, disfunción diastólica tipo III de IV.
d.- Disfunción sistólica ventricular derecha e (HTP).

CASO CLINICO
Una paciente de 41 años se encontraba en espera para ablación de vía accesoria (VA) en el contexto de un síndrome de Wolff-
Parkinson-White (WPW). La paciente, estando todavía asintomática, había sido diagnosticada de WPW 9 años antes en una revisión
rutinaria. Tres años antes del actual ingreso empezó a referir episodios cortos (< 5 min) de palpitaciones de inicio y fin súbitos, que
nunca fueron registradas por electrocardiograma (ECG). En los últimos 9 meses empezó a sufrir clínica compatible con insuficiencia
cardiaca en forma de disnea de esfuerzo clase funcional II de la New York Heart Association. La exploración física era normal, pero en la
ergometría solo alcanzó 6 MET.

PREGUNTA
Cuales son los datos que se registraron en la imagen 1A antes de la ablación?

RESPUESTA
a.- Ritmo sinusal con BRD.
b.- Ritmo sinusal con preexitacion con BRI.
c.- Ritmo sinusal con preexcitación por una VA anteroseptal derecha con (BRI).
d.- Ritmo sinusal con preexitacion por una VA anterioseptal derecha con BRD.

PREGUNTA
Considerando la evolución clínica y el ECG 1B, el paciente paso de una FE del 48 % al 63 %, cual es la tasa de mortalidad en este
momento para el pacientes?

RESPUESTA
a.- Tasa de mortalidad del 6%.
b.- Tasa de mortalidad del 17%.
c.- Tasa de mortalidad del 38%.
d.- Tasa de mortalidad del 81%.

CARDIOPATIAS CONGENITICAS (ADULTO)


DEFINICION: La incidencia de las formas moderadas a severas de las cardiopatías congénitas es de 6 por 1000 nacidos vivos, los
pacientes sin manejo farmacológico y/o quirúrgico temprano no llegan a la vida adulta, ya que con el tratamiento adecuado el
pronostico es de 85% para llegar a la vida adulta, la mayor parte de los adultos con cardiopatías congénitas no requieren tratamiento
llegando a la vida adulta. Los defectos mas frecuentes son 1.- defecto septal ventricular estrecho, 2.- defecto artrial septal secundario,
3.- estenosis pulmonar leve a moderada, 4.- valva aortica bicuspidea, 5.- prolapso de valvula mitral. DIAGNOSTICO: Los adultos con
cardiopatías congénitas presentan tres formas básicas: con una historia de cirugía previa paliativa o reparativa durante la infancia, 2.-
con conocimiento de la cardiopatía congénita sin intervención, 3.- presentación en la vida adulta sin antecedentes de la infancia del
padecimiento. La excepción se presenta en los pacientes con persistencia de conducto arterioso leve. Cianosis en el adulto debe buscar
la causa: Eritrocitosis, deficiencia de hierro, sangrado, o cardiovascular. Cuando se presentan las siguientes alteraciones: Alteracion de
la función ventricular (es la mas frecuente que presente síntomas), arritmias (las atriales es la mas frecuente, en las ventriculares son
secundarias a fibrosis, dilatación ventricular, y son causas de muerte súbita). Enfermedad por conducción, (intriseco o posoperatorio de
enfermedad de noso SA con disfunción) se puede observar fracción baja de eyección, disincronia, puede causar bloqueos. DEFECTO
ATRIAL SEPTAL: 75 % es defecto secundario a ostium encontrado en el adulto, se observa murmullo y alteraciones inespecíficas de RX y
ECG, presencia de segundo ruido, sobrecarga ventricular derecha crónica, hipertensión pulmonar, arritmias atriales, raramente
embolizacion paradoxica). Tratamiento: Especifico por alteración. PREVENCION: Endocarditis (es mas frecuente cuando hay
antecedentes de cardiopatía congénita), Embarazo (cuando hay antecedentes de lesiones estenoticas severas pulmonar, hipertensión
pulmonar, shunts derecha a izquierda, regurgitación, vigilancia estrecha por riesgo de endocarditis durante el embarazo y sobrecarga a
las 20 a 22 SDG indicado ecocardiografia), Anticoncepcion (Los Estrogenos incrementan mas los riesgos para tromboembolico, usar
contracepción con progestágenos); Ejercicio (identificar taquicardia inducida, evaluar la competencia cronotropica, se contraindica el
ejercicio intenso isométrico). DEFECTO SEPTAL VENTRAL: Existen 4 tipos, la perimembranosa es la mas común, la muscular es común
en la infancia con cierre expontaneo, puede presentar murmullo holosistolico inversamente relacionado al tamaño del defecto, pueden
desarrollar el complejo Eisenmenger al no tratarse. TRATAMIENTO: al desarrollar hipertensión pulmonar puede estar indicado

CURSO ENARM CMN SIGLO XXI TEL: 36246001 Pharmed Solutions Institute PÁGINA 22
MANUAL DE TRABAJO DEL CURSO ENARM CMN SIGLO XXI
sindanafil o análogos de prostanglandinas. DEFECTO SEPTAL ATRIOVENTRICULAR: Se presenta lesiones en los septos atriales y
ventriculares clasificados en parcial o complejo, se acompaña comúnmente de subestenosis aortica, puede coexistir regurgitación
mitral. El ECG puede mostrar desviación a la izquierda del eje, y bloqueo incompleto, lesión frecuente en síndrome de down, requiere
reparación antes de los 6 meses de edad para prevenir la hipertensión pulmonar y consecuentemente el complejo Eisenmenger.
TETRALOGIA DE FALLOT: Es la mas común de las cardiopatías congénitas cianogenas representa del 7 al 10 %, difícilmente paciente sin
antecedentes solo en forma minimas, pueden presentar alteraciones de las intervenciones realizadas durante la vida adulta. Las
arritmias son las manifestaciones mas frecuentes en la vida adulta, junto a la disfunción ventricular derecha, Tratamiento con
marcapaso y defibrilador interno. ESTENOSIS PULMONAR AISLADA: Presentan estenosis infundivular subpulmonar pudiendo sobrevivir
a la vida adulta, presente en sx de noonan, Tratamiento con valvuloplastia con balón. OBSTRUCCION DEL TRACTO DE SALIDA DEL
VENTRICULO IZQUIERDO: Obstruccion valvular bicuspidea, subvalvular o supravalvular, puede presentarse afecciones en multiples
niveles. COARTACION DE LA AORTA: Presente en el adulto con menor posibilidad de reparación, se presenta predominantemente
distal al inicio de la subclavia izquierda, es silenciosa hasta que se presenta un evento súbito como hipertensivo. TRANSPOSICION DE
GRANDES ARTERIAS: La sobrevida a la vida adulta es casi imposible, se realiza un swich de vasos en la infancia, 50 % de los casos
desarrollan trasntornos sinusales, arritmias, regurgitación valvular, falla ventricular derecha.

CASO CLINICO
Posteriormente a la corrección de la tetralogía de Fallot realizada con 19 años de edad, con ampliación del tracto de salida del
ventrículo derecho, valvotomía pulmonar y cierre de la comunicación interventricular, deliberadamente quedaron como defectos
residuales, la estenosis pulmonar al nivel valvular por un pequeño anillo pulmonar y la comunicación interatrial. Pero incluso así,
permaneció con una buena evolución por 28 años cuando surgió la primera complicación por embolia cerebral paradójica con un
discreto accidente cerebrovascular, sin consecuencias sensoriales o motoras. Además, hace un año se suscitó una complicación
arrítmica por fibrilación atrial con frecuencia cardíaca elevada, que fue revertida con amiodarona y choque eléctrico. Recientemente,
una nueva fibrilación atrial, pero ahora con una frecuencia ventricular baja (entre 50 a 60 lpm), y insaturación arterial (70%),
posteriormente a la deambulación en pocos minutos, motivó la investigación diagnóstica y una conducta más precisa. Al examen físico,
el paciente se presentó en un buen estado general, eupneico, enrojecido, con pulsos normales, pesando 114 kg, altura de 170 cm, PA
de 110/80 mmHg y FC de 60 lpm en ritmo de fibrilación atrial. La aorta se palpaba discretamente en la fúrcula. En el precordio, no había
impulsos o deformidades y el ictus cordis no se palpaba. Los ruidos eran hipofonéticos y se auscultaba un soplo sistólico, +/++ de
intensidad, de eyección, rudo, en el 1º, 2º y 3º espacios intercostales izquierdos en el borde esternal y en la fúrcula. El hígado no se
palpaba.

PREGUNTA
Cual de las siguientes condiciones es la mas probable causa del estado actual del paciente?

RESPUESTA
a.- Estenosis pulmonar valvular moderada e hipertrofia del ventrículo derecho
b.- Shunt de sangre de la derecha hacia la izquierda por comunicación interatrial
c.- Defectos residuales posteriores a la corrección de la tetralogía de Fallot
d.- Insaturación arterial grave a pequeños esfuerzos.

CASO CLINICO
Mujer de 69 años, intervenida a los 48 años por CIA tipo ostium primum con disnea de medianos esfuerzos. Se cerró quirúrgicamente el
defecto con parche de teflón y se corrigió, mediante sutura, la hendidura de la valva anterior mitral. Precisó implante de marcapasos
VVI por bloqueo auriculoventricular completo. Permaneció asintomática hasta 22 años después, cuando presentó dolor torácico
inespecífico y una prueba de esfuerzo interrumpida por la disnea. Ecocardiográficamente presentaba un ventrículo izquierdo
hipertrófico con función conservada e hipertensión pulmonar severa. La válvula mitral estaba fibrosada, con insuficiencia moderada e
insuficiencia aórtica ligera. Presentaba un gradiente sistólico fijo en el tracto de salida del ventrículo izquierdo (TSVI) de 84 mmHg,
ocasionado por una membrana, que parecía partir de la valva anterior mitral. En el cateterismo cardíaco se diagnosticó una estenosis
severa del tronco de la coronaria izquierda.

PREGUNTA
La ESA tras cirugía correctora de CIA de tipo ostium primum es infrecuente, considerando las datos previos referidos cual de las
siguientes afirmaciones es menos probable?

RESPUESTA
a.- Afecta al 2-15% de los casos
b.- Suele diagnosticarse 6-8 años tras corrección del defecto en la infancia.
c.- Falta de seguimiento postoperatorio en adultos por evolución más lenta.
d.- Su incidencia es de 60-70% y permanece silente durante pocos años.

CASO CLINICO
Una mujer de 43 años de edad. Su historial médico incluye la reparación de una transición atrioventricular (AV), comunicación
interventricular (D-AV) a la edad de 14 años, que consiste en un ostium primum AVSD y musculoso comunicación interventricular
restrictiva (VSD), asociado con un velo anterior mitral hendida. Después de su cirugía, ella permaneció asintomática hasta que tenía 34
años, cuando experimentó la fatiga progresiva. Una ecocardiografía transtorácica mostró shunt residual, tricúspide moderada e
insuficiencia aórtica, insuficiencia mitral moderada y estenosis subaórtica con un tracto de salida del ventrículo izquierdo fijo (TSVI)

CURSO ENARM CMN SIGLO XXI TEL: 36246001 Pharmed Solutions Institute PÁGINA 23
MANUAL DE TRABAJO DEL CURSO ENARM CMN SIGLO XXI
gradiente pico de 52 mm Hg. La estenosis subaórtica se agravó en los últimos años, alcanzando un máximo gradiente sistólico de 100
mm Hg. Permaneció activa, pero experimentó episodios diaforéticas más frecuentes con actividad leve.

PREGUNTA
¿Cual de las siguientes alteraciones favorece más la condición actual de ESA?

RESPUESTA
a.- Crestas subaórtica fibromuscular.
b.- Anormales válvula AV izquierda.
c.- Cuerdas tendinosas de los archivos adjuntos.
d.- Inserción anómala de los músculos papilares del ventrículo izquierdo y generalizada hipoplasia del LVOT.

CASO CLINICO
Masculino de 20 años con soplo cardíaco, cianosis y cansancio observados desde el nacimiento, con nítida acentuación en los últimos
años. Sin control médico, hematocrito de 83% y hemoglobina de 28 g. Hemaféresis repetidas disminuyeron el hematocrito para 73%.
Ninguna medicación específica había sido administrada desde el nacimiento. Examen físico: Regular estado general, eupneico, cianosis
acentuada, pulsos normales. Peso: 42,6 Kg; Altura: 160 cm; PA: 105/65 mmHg; FC: 82 lpm; FR: 20 rpm. Saturación O2: 75%. La aorta era
discretamente palpada en la fúrcula. En el precordio había pulsos sistólicos discretos, frémito sistólico nítido en todo el borde esternal
izquierdo. Ictus cordis no era palpado. Los ruidos cardíacos eran muy hiperfonéticos y se auscultaba soplo sistólico acentuado en todo
el borde esternal izquierdo, con nítida irradiación hacia el área aórtica. El hígado no era palpado y los pulmones eran limpios.

PREGUNTA
Considerando el cuadro clínico, antecedetnes y frecuencia de las cardiopatías congenitcas, cual es la condición mas probable del caso?

RESPUESTA
a.- Estenosis pulmonar.
b.- Doble vía de salida de ventrículo derecho.
c.- Doble vía de entrada de ventrículo único.
d.- Estenosis pumonar aislada.

CASO CLINICO
Varón de 21 años con atresia pulmonar y comunicación interventricular intervenido de anastomosis aortopulmonar de Waterston-
Cooley (WC) en el primer mes de vida. A los 2 años se le realizó fístula de Blalock-Taussig (BT) izquierdo modificado y a los 8, ampliación
de arterias pulmonares con parche de Dacron, implantándose además un conducto no valvulado entre el ventrículo derecho y las
arterias pulmonares. A los 12 años se cerró la comunicación interventricular mediante un parche en teja y se reconstruyó el conducto
con técnica de Danielson. Dos años más tarde, el cateterismo cardíaco demostró estenosis severa entre el tronco pulmonar y ambas
ramas, con presiones sistémicas en el ventrículo derecho. Se realizó angioplastia con balón de ambas ramas, que fue ineficaz. Nueve
años después el paciente se encontraba en grado funcional III de la NYHA y la ecocardiografía puso de manifiesto presión
suprasistémica en el ventrículo derecho, comunicación interventricular residual pequeña con cortocircuito bidireccional e insuficiencia
tricuspídea grado III/IV.

PREGUNTA
Cual de las siguientes causas es menos frecuente en la estenosis e hipoplasias localizadas de las arterias pulmonares secundarias a
cirugía paliativa?

RESPUESTA
a.- Constriccion progresiva.
b.- Ineficacia de angioplastia con balón.
c.- Distorsión de tejido fibroso en su interior.
d.- Crecimiento de tejido fibroso en su interior.

CASO CLINICO
Mujer de 28 años con atresia pulmonar y comunicación interventricular intervenida quirúrgicamente a los 4 años de edad,
practicándosele fístula de BT izquierdo clásica. A los 6 años se le realizó anastomosis de WC por estenosis de la fístula. A los 14 años
presentaba cianosis progresiva severa y disnea de esfuerzo. El cateterismo cardíaco demostró estenosis severa de la fístula de BT e
hipoplasia de las arterias pulmonares, descartándose para cirugía correctora. A los 26 años, la paciente presentaba cianosis severa,
acropaquias y disnea de mínimos esfuerzos, decidiéndose la realización de una nueva fístula paliativa, que no pudo completarse por la
aparición de sangrado muy abundante durante la disección quirúrgica. Mediante angiografía se demostró obstrucción completa de la
fístula de BT en su tercio distal y estenosis severa de la anastomosis de WC.

PREGUNTA
El tratamiento quirúrgico de esta complicación es técnicamente difícil y la dilatación con balón habitualmente ineficaz por que razo?

RESPUESTA
a.- Retroceso elástico de la arteria.
b.- Fibrosis persistente suprapulmonar.

CURSO ENARM CMN SIGLO XXI TEL: 36246001 Pharmed Solutions Institute PÁGINA 24
MANUAL DE TRABAJO DEL CURSO ENARM CMN SIGLO XXI
c.- Falta de angenten inmunomoduladores.
d.- Persistencia de las resistencias intracavitarias.

CASO CLINICO
Varón de 19 años diagnosticado de tetralogía de Fallot severa a los 3 años de edad, practicándosele fístula de BT izquierdo seis meses
después. A los 5 años se rechazó la cirugía correctora por presentar hipoplasia severa de tronco y arterias pulmonares. Desde entonces
presentaba cianosis progresiva severa, disnea de esfuerzo, acuclillamiento y síncopes. A los 9 años se realizó ampliación con parche del
tracto de salida del ventrículo derecho y del origen de ambas ramas pulmonares, así como cierre de la comunicación interventricular
con parche fenestrado, quedando con presión sistémica en el ventrículo derecho, gradiente transpulmonar severo y cortocircuito
bidireccional ventricular.

PREGUNTA
Cual de las siguientes aseveraciones es correcta, al manejo de stens en esta patologia?.

RESPUESTA
a.- La dilatación mediante stent no es un medoto seguro ni eficaz.
b.- La combinación de stent y valvuloplastia con balón tiene mayor eficacia y eficiencia.
c.- La dilatación mediante stent es un método seguro y eficaz de tratamiento.
d.- En las estenosis de las arterias pulmonares en adultos con cardiopatías congénitas operadas en la infancia no es útil los Stens.

VALVULOPATIAS (TEMA)
CIENCIAS BASICAS: Estenosis valvular o dificultad para que la válvula pueda abrirse. Insuficiencia valvular o imposibilidad de que la
válvula al cerrarse ocluya completamente el orificio valvular, reflujo anormal de sangre. Focos de auscultación: foco mitral o apexiano
(5to. espacio intercostal izq. con línea medio clavicular); foco aórtico (2do. espacio intercostal con paraesternal der.); foco tricuspídeo
(porción inferior del esternón); foco pulmonar (2do. espacio intercostal paraesternal der.). Pueden generarse a consecuencia de fiebre
reumática (infección de origen estreptocócico más mecanismo inmunológico afectando endocardio valvular), endocarditis bacteriana
(da lugar a formación de vegetaciones en las válvulas mutilándolas), malformaciones congénitas, degeneración mixomatosa (prolapso
de los velos valvulares en insuficiencia), disfunción y/o rotura de las cuerdas tendinosas (IAM). ESTENOSIS MITRAL: Valvulopatía más
frecuente. Su principal causa es la fiebre reumática. Más a mujeres. PATOGENIA: Disminución del área del orificio valvular mitral
(normal 5 cm), con lo cual hay dificultad para el relleno del VI, durante la diástole, esto va generando hipertrofia de la pared auricular,
porque se requiere mayor presión para mantener el flujo hacia el VI, que puede desencadenar una FA, puede haber estasis de sangre
en la aurícula, generar un trombo y este embolismo sistémico. Área inferior a 2cm, genera síntomas al ejercicio, área <1cm genera
elevación de la presión auricular, aumento de presión venosa y capilar (hipertensión pulmonar;) por fallo retrogrado, congestión
pulmonar que genera disnea de reposo, mas EAP. DIAGNOSTICO: Cuadro clínico: Disnea de esfuerzo (síntoma principal), ortopnea,
disnea paroxística nocturna, edema agudo del pulmón (EAP), episodios de infección respiratoria de repetición, astenia por disminución
de gasto cardíaco, datos de IC der. (Ingurgitación yugular, hepatomegalia, ascitis) en fases avanzadas, fascies mitral por rubicundez de
las regiones malares, pulso parvo y latido de punta débil (por caída de vol. sistólico), estertores pulmonares húmedos (si hay EAP).
Auscultación: Refuerzo del 1er ruido, cierre brusco y enérgico de la válvula mitral, soplo diastólico con sonoridad de retumbo (se
acentúa después del ejercicio), y reforzamiento presistólico inmediatamente antes del 1er ruido. ECG: Onda p mitral, si no existe FA. Rx:
datos de congestión venocapilar pulmonar, con dilatación de ramas pulmonares y AI. y ecocardiograma, forma no invasiva mas útil para
dx. TRATAMIENTO: Dirigido a aliviar síntomas de congestión pulmonar con diuréticos, la FA (beta-bloqueadores, bloqueadores de
canales de calcio) y anticoagulación. La cirugía es lo definitivo; comisurotomía o cambio valvular. COMPLICACIONES: EAP, hemoptisis,
arritmias (FA); embolismo sistémico (fractura de trombo intraauricular, causando EVC, isquemia mesentérica y de extremidades
inferiores), afonía (por compresión de la AI sobre el nervio recurrente). INSUFICIENCIA MITRAL: Causa más común cardiopatía
reumática. Pacientes jóvenes. PATOGENIA: La válvula permite reflujo anormal de sangre desde VI a la AI, durante la sístole ventricular,
esto genera sobrecarga en aurícula izq. que lleva a congestión y edema pulmonar. Durante la diástole hay sobrecarga de volumen en VI,
generándose una dilatación compensatoria que genera IC izq. DIAGNÓSTICO: Cuadro clínico: astenia, fatigabilidad, disnea, ortopnea,
disnea paroxística nocturna y/o EAP, ICC en fases avanzadas, latido de punta desplazado a la izq. y abajo, amplio poco sostenido, el
ventrículo grande regurgita, puede palparse trill sistólico apical. Auscultación: 1er ruido apagado (por no cierre completo), soplo
holosistólico (regurgitante, en chorro de vapor), que se propaga hacia la axila, pulso arterial normoamplio y celer. ECG: onda p mitral y
crecimiento de VI con R altas. Rx de tórax: dilatación del VI y AI, con desplazamiento del bronquio principal izq. TRATAMIENTO: Reducir
la pre y poscarga, para disminuir síntomas por congestión pulmonar; el edema pulmonar con diuréticos y nitratos; reducción de la
poscarga con IECAS. Qx definitiva, plastia o cambio valvular. ESTENOSIS AÓRTICA: Principal causa degenerativa. Adultos 40-60 años.
Hombres, foma mas común es la congénita (valva uni o bivalva). PATOGENIA: Dificultad para vaciar el VI, lo que genera aumento de la
poscarga e hipertrofia concéntrica del VI (nos puede llevar a isquemia, arritmias y muerte súbita), que conduce a disminución de la
distensibilidad ventricular, aumentándose la presión telediastolica del VI, mas dilatación del VI, lleva a insuficiencia mitral, que lleva a IC
retrógrada. DIAGNOSTICO: Cuadro clínico: asintomática por mucho tiempo. Triada clásica: angina de pecho (30-40%), insuficiencia
cardiaca (20%) y sincope (10-15%, tras esfuerzo intenso), astenia progresiva primer síntoma, disnea de esfuerzo, de reposo,
palpitaciones, visión borrosa. Auscultación: Latido de punta desplazado hacia la izq. y abajo por hipertrofia de VI, pulso parvo y tardo,
TAS disminuida con escasa modificación de la TAD. chasquido clic de eyección después del primer ruido, soplo mesositolico de
morfología romboidal, por disminución del flujo aórtico de la válvula estenosada, cuando la presión intraortica alcanza un determinado
nivel. ECG: signos de crecimiento ventricular izq. con grandes ondas R en V5 y V6 y S profunda en V1 y V2, ondas T negativas por la
sobrecarga de presión y trastornos de la conducción. Rx de tórax: silueta cardiaca normal, porque la hipertrofia concéntrica no la
modifica, en fases finales hay cardiomegalia. Ecodopler: nos permite valorar el área valvular (área valvular en cm2 >1.5 leve; 1.0-1.5
moderada; < 1.0 severa)y el gradiente transvalvular. TRATAMIENTO: De elección es quirúrgico (prótesis mecánica). INSUFICIENCIA
AÓRTICA: Causas fiebre reumática, sífilis y endocarditis bacteriana. PATOGENIA: Los velos valvulares aórticos, no tienen un cierre

CURSO ENARM CMN SIGLO XXI TEL: 36246001 Pharmed Solutions Institute PÁGINA 25
MANUAL DE TRABAJO DEL CURSO ENARM CMN SIGLO XXI
hermético, lo que genera reflujo diastólico esto aumenta la precarga y genera hipertrofia del VI, que puede llevar a IC, por otro lado el
aumento en el consumo de O2 y la disminución de la presión diastólica, lleva a isquemia, frecuente en el Sx. de Marfan, disección
aórtica entre otras. DIAGNOSTICO: Cuadro clínico: manifestaciones de IC o isquemia hasta 4 o 5 década de la vida, disnea en reposo,
ortopnea, disnea paroxística nocturna, angina, palpitaciones, pulso capilar (signo de Quincke)c se presiona sobre la uña se observa,
como el borde de la zona pálida avanza y retrocede, durante la sístole y diástole. Danza carotidea (signo de Corrigan), latido carotideo
visible y muy amplio, ampliación de la TA diferencial por elevación de la TA sistólica y disminución de la diastólica, que incluso puede
ser 0. Auscultación: 1er y 2do ruidos apagados, soplo pandiastólico decreciente (por la progresiva caída de la presión en la raíz de la
aorta durante la regurgitación), de tono alto, suave y aspirativo, irradiándose a lo largo del borde esternal. ECG: Signos de crecimiento
del VI, sobrecarga diastólica (ondas T picudas y supradesnivel del segmento ST. Rx. cardiomegalia con signos de crecimiento del VI y
dilatación de la raíz aortica, ecodopler permite medir grado de ingurgitación. TRATAMIENTO: Enfocado a corregir síntomas de IC,
oxigeno, diuréticos, inotrópicos, IECAS. Qx definitivo, se recomienda en pacientes con diámetros ventriculares >50mm. ESTENOSIS
TRICÚSPIDEA: Condiciona sobrecarga de presión en sentido retrógrado sobre la aurícula y congestión venosa con clínica de IC der.
Auscultación en foco tricúspide soplo diastólico, con refuerzo presistólico y chasquido de apertura de la tricúspide. INSUFICIENCIA
TRICÚSPIDEA: Clínica de IC der. por regurgitación sistólica, hacia la aurícula. Auscultación: soplo sistólico, similar al de la insuficiencia
mitral, pero se ausculta mejor en el foco tricúspide., se incrementa durante la inspiración, por aumentar esta condición el retorno
venoso. ESTENOSIS PULMONAR: IC der., sobrecarga de presión del ventrículo der.. Auscultación: Foco pulmonar muestra un clic de
eyección y un soplo sistólico de morfología romboidal. INSUFICIENCIA PULMONAR: IC der., regurgitación y sobrecarga del VD.
Auscultación: soplo diastólico decreciente en foco pulmonar, equiparable en su génesis al de la insuficiencia aortica. CLAVES: La
profilaxis con antibióticos es recomendada, si hay procedimientos invasivos, por alto riesgo de endocarditis infecciosa.

CASO CLINICO
Paciente de 66 años de edad, con obesidad, hipertensión arterial y dislipemia con diagnóstico de estenosis aórtica grave y cardiopatía
coronaria al realizarse ecocardiografías transtorácica y transesofágica; se concluyó estenosis aórtica grave (válvula aórtica con
gradientes máximo de 68, medio de 46 y área de 0,95 cm2) con función sistólica conservada e hipertrofia concéntrica del ventrículo
izquierdo (septum interventricular de 15 mm y pared posterior de 14 mm). El cateterismo cardiaco demostró enfermedad coronaria
(lesión oclusiva del 80% en el tercio proximal de la descendente anterior y del 75% en el tercio medio de la circunfleja).

PREGUNTA
Considerando los factores del paciente cual es la causa mas probable?

RESPUESTA
a.- Calcificacion valvular.
b.- Degenerativa.
c.- Cardiopatia reumática.
d.- Fibrosis autoinmune.

PREGUNTA
Cual es el pronóstico del paciente conociendo la mortalidad anual de esta patologia?

RESPUESTA
a.- 3 %.
b.- 6 %.
c.- 9 %.
d.- 12 %.

PREGUNTA
Cual de las siguientes etiologias es menos frecuente en este caso?

RESPUESTA
a.- Sindrome de marfan.
b.- Hipercolesterolemia.
c.- Cardiopatia reumática.
d.- Lupus eritematoso.

CASO CLINICO
Un varón de 87 años con estenosis aórtica sintomática fue remitido para implante de válvula aórtica percutánea. El ecocardiograma
preoperatorio mostraba estenosis valvular aórtica severa (área calculada, 0,6 cm2), fracción de eyección ventricular izquierda normal y
anatomía cardiaca adecuada para el implante percutáneo. La coronariografía previa no mostró lesiones significativas. El paciente fue
rechazado para cirugía por su elevada comorbilidad (EuroSCORE, 23%).

PREGUNTA
Considerando la clasificación del padecimiento este paciente esta considerado como?

RESPUESTA
a.- Leve.
b.- Moderada.

CURSO ENARM CMN SIGLO XXI TEL: 36246001 Pharmed Solutions Institute PÁGINA 26
MANUAL DE TRABAJO DEL CURSO ENARM CMN SIGLO XXI
c.- Grave.
d.- Critica.

PREGUNTA
Cual de los siguientes síntomas no esta relacionado con el incremento de la presión venocapilar pulmonar?

RESPUESTA
a.- Disnea en reposo.
b.- Ortopnea
c.- Disnea paroxística nocturna.
d.- Opresion toracicca.

CASO CLINICO
Paciente de femenino de 65 años de edad, se detectó hipertensión arterial (160/100 mmHg), fibrilación auricular de inicio
indeterminado. Se inició tratamiento con atenolol 50 mg/día, enalapril 20 mg/día. Desde el último año disnea progresiva con esfuerzos
menores de los habituales y palpitaciones frecuentes espontáneos. En la auscultación cardíaca se apreció arritmia completa por
fibrilación auricular, un soplo holosistólico 4/6 de alta frecuencia en el foco mitral irradiado a la axila y otro sistólico, de menor
intensidad, en el borde esternal izquierdo bajo, que aumentaba con la inspiración. En la radiografía de tórax se apreció una
cardiomegalia global grado III/IV con campos pulmonares normales. El ECG demostró una fibrilación auricular controlada, con signos de
crecimiento ventricular izquierdo y alteraciones secundarias de la repolarización.

PREGUNTA
Cuál es el manejo más adecuado para el manejo de la insuficiencia cardiaca.

RESPUESTA
a.- Amiodarona.
b.- Hidroclorotiazida.
c.- Digoxina.
d.- Nifedipina.

PREGUNTA
Cuál es el manejo más adecuado para el manejo de los tratornos del ritmo.

RESPUESTA
a.- Amiodarona.
b.- Hidroclorotiazida.
c.- Digoxina.
d.- Nifedipina.

PREGUNTA
Cual es la etiología mas probable considerando los factores de riesgo que presenta la paciente?

RESPUESTA
a.- Fiebre reumática.
b.- Sindrome de lutenbacher.
c.- Enfermedad de fabry.
d.- Enfermedad de whipple.

PREGUNTA
Considerando la condición de la paciente cual de las siguientes causas es la más frecuente de muerte?

RESPUESTA
a.- Embolia sistémica.
b.- Insuficiencia cardiaca.
c.- Embolia pulmonar.
d.- Infección

CASO CLINICO
Mujer de 83 años, con prótesis mitral mecánica. Fibrilación auricular crónica. Ecocardiograma transesofágico: área valvular de 0,5cm2,
hipertensión pulmonar de 75mmHg, fracción de eyección del 65%, anillo aórtico de 18 mm y prótesis mitral normofuncionante. La
paciente evolucionó sin complicaciones. En la revisión de 1 y 3 meses se encuentra en clase funcional I.
PREGUNTA
Cual es el agente etiológico mas frecuente de esta patología.

RESPUESTA
a.- Estreptococcos

CURSO ENARM CMN SIGLO XXI TEL: 36246001 Pharmed Solutions Institute PÁGINA 27
MANUAL DE TRABAJO DEL CURSO ENARM CMN SIGLO XXI
b.- Estafilococcos.
c.- Treponema.
d.- Cocobacilos

CASO CLINICO
Mujer de 74 años de edad hipertensa, dislipémica, diabética tipo 2, con nefropatía diabética e insuficiencia renal. Fue sometida a una
comisurotomía mitral, sustitución valvular mitral con prótesis. Sufre fibrilación auricular crónica. Deterioro de clase funcional, tras
descartar disfunción de la prótesis mitral, se la remitió para valoración de TAVI. Rechazada para cirugía (EuroSCORE, 25%).
Coronariografía: sin estenosis angiográficas. En el ecocardiograma transesofágico se observó lo siguiente: presión pulmonar sistólica de
80mmHg, anillo aórtico de 22mm y área valvular de 0,6cm2. Distancia mitroaórtica, 7 mm.

PREGUNTA
Cual es el factor de mayor influencia en el pronóstico a mediano plazo.

RESPUESTA
a.- Diabetes mellitus.
b.- Dislipidemia.
c.- Hipertensión
d.- Cardiopatia coronaria.

CASO CLINICO
Paciente masculino, 20 años, previamente sano, presenta episodio de disnea durante el sueño, se quejaba de cansancio a los esfuerzos
medios con evolución de alrededor de 15 días, siendo caracterizado como clase funcional II de la NYHA (New York Heart Association). Al
examen físico, presentaba presión arterial 110/70 mmHg, ritmo cardíaco regular en dos tiempos, soplo sistólico en foco mitral. El ECG
evidenciaba bloqueo de rama derecha.

PREGUNTA
Cual es la terapeutica mas apropiada para el trastorno del ritmo.

RESPUESTA
a.- Marcapasos.
b.- Amiodarona.
c.- Amlodipido.
d.- Digoxina.

CASO CLINICO
Posteriormente a la corrección de la tetralogía de Fallot realizada a los 19 años de edad de la paciente, con ampliación del tracto de
salida del ventrículo derecho, valvotomía pulmonar y cierre de la comunicación interventricular, quedaron como defectos residuales,
estenosis pulmonar al nivel valvular por un pequeño anillo pulmonar y comunicación interatrial. Permaneció con una buena evolución
por 28 años cuando presenta embolia cerebral por fibrilación atrial.

PREGUNTA
Considerando el cuadro clínico cual de los siguientes fármacos es el mas apropiado para mantener controlada la agregación
plaquetaria.

RESPUESTA
a.- Acetilsalicilico.
b.- Clopidrogel.
c.- Acecumarol.
d.- Enoxoparina.

CASO CLINICO
Mujer de 59 años con historia de implantación de marcapasos definitivo endocavitario por bloqueo auriculoventricular completo.
Presentó trombosis de vena cava, que originó un síndrome de vena cava superior y requirió de la colocación de un stent
autoexpandible. Presentó estenosis de la vena subclavia izquierda y tromboembolia pulmonar. Es egresado luego de 30 dias de estancia
hospitalaria.
PREGUNTA
Cual de los siguientes factores etiológicos es mas frecuente en esta patología.

RESPUESTA
a.- Insuficiencia venosa profunda.
b.- Arterio esclerosis de grandes vasos.
c.- Insuficiencia tricuspidea.
d.- Anticoagulacion insuficiente.

CURSO ENARM CMN SIGLO XXI TEL: 36246001 Pharmed Solutions Institute PÁGINA 28
MANUAL DE TRABAJO DEL CURSO ENARM CMN SIGLO XXI
PERICARDITIS
DEFINICION: El pericardio consiste en una capa visceral y una lamina parietal que entre si forma un espacio potencial, la cavidad
pericardica contiene 50 ml de plasma infiltrado, sus fnciones limitan la distención cardiaca, facilita el acople e interacion cardica,
mantienen el volmen de presión cardiaca, la ergometría del ventrículo izquierdo, además de que lubrica, minimiza la friccion, regula la
inercia, las fuerzas hidrostática y gravitacionales del coraccion y es una barrera mecánica contra la infecccion, presenta acciones
inmunológicas, vasomotoras, fibrinoliticas, modula la estructura del miocito, su función, asi como su expresión génica, es un vehiculo
para fármacos y terapia génica. No es escencial para la vida, sin embargo puede comprometerlo al presentar una patologia.
PERICARDITIS AGUDA: Es un estado agudo firbinoso o sudativo por pericarditis, es un Sx caracterizado por dolor en pecho, friccion
pericardica, cambios ECG, manifestando grado, severidad y localización, puede modificar el ST o alteraciones de la repolarizacion como
en el IAM, aun cuando estos son difusos, el RX puede ser normal o relevar la silueta cardiaca por exudado aumentado, sugiriendo
miocarditis o taponade por ejemplo, los marcadores imflamatorios puede incrementarse, en caso de isoenzimas cardiacas es mas
frecuente en epicarditis, en epicarditis idiopáticas pueden elevarse semejando IAM, la hospitalización de estos casos son para
diferenciar los casos, responden frecuentemente a AINES, añadiendo profilaxis con H2 bloqueadores, la adicion de colchicina es útil en
casos de recurrencias, el dolor permanece 1 a 2 dias, el frote y el ECG giualmente, en caso viral se prolonga a mas de 4 dias o en caso de
drogadicción el dolor es mas severo, requiririendo corticoides sistémicos. DERRAME PERICARDICO: Es el acumulo de transudado,
exudado o hemosudado en el saco pericaidico frecuente en las complicaciones de enfermedades del pericardio, en pericarditis aguda,
el hidropericardio se presenta en pericarditis crónica frecuente en retenion de agua y sodio, o procesos inflamatorios crónicos, la
presencias hemática o sero hemática es mas frecuente en infecciones o inflamaciones, en pericarditis quilosa  lesiones u obstruccion
del conducto torácico, por coresterol, hipotiroidismo, artritis reumática, o tuberculososis. El ECG es el procedimiento de elección, la
TAC y el IRM son utiles para identificar la inoculación típica o atípica, la etiología del derrame pericardico deberá ser identificado por
citología, análisis inmunológico o por biopsia guiada. La pericardiocentesis es diagnostica y terapéutica, ya que evita la falla del corazón
hemodinamicamente hablando, por pulso paradójico, disfunción atrial y ventricular diastólica, generando pulso paradoxico,
disminusion de la presión arterial, hipotensión arterial, la fusión cardiaca decrese hasta verse negativa a negativa, hasta llegar al
prolapso y falla cardiaca por agotacion de los mecanismos compensatorios. TAPONADE CARDIACO: Definido como agudo o crónico, se
observa como una alteración dinámica continua, con presiones de 10 a 20 mmHg como máximas, en los casos leves es asintomática, en
la moderada el disconfort y la disnea predominan, las ondas yugulares predominan, se presenta bajo voltaje en el ECG, durante la
inspiración disminuye, se debe realizar pericardocentesis cuando los síntomas los amerite, a los 50 ml ya iniciaran, sin embargo se
recomienda cutltivo, citología. PERICARDITIS CONSTRICTIVA: Es una condición calcificante que limita la función diastólica de los
ventrículos, idiopático principalmente, trauma cardiaco, quirúrgico o tuberculoso otras infecciones, neoplasias, radioterapia, falla renal
y enfermedades del tejido conectivo como menor causa. La condición crónica congestivo a semeja enfermedad miocárdica y
enfermedad hepática crónica. El paciente refiere fatiga, disnea, aumento de peso, disconfort abdominal y nausea. Puede llegar a ascitis,
hepatoesplenomegalia, edema confundiendoce con cirrosis, signo de Kussmaul, Knock, tercer ruido, pulso paradójico. QRS de bajo
voltaje, cambios en P y T, fluter atrial es común, la TAC e IRM define entre pericarditis restrictiva y cardiomiopatía contrictiva, pueden
necesitar pericardiectomia con una mortalidad del 30 al 40 %,

CASO CLINICO
Varón de 48 años, fumador. Había participado recientemente en la recogida de restos de aves enfermas sacrificadas en granjas.
Consultó en urgencias por dolor retroesternal que aumentaba con la inspiración profunda y la tos empeoraba con el decúbito. En la
exploración física destacaba el hallazgo de febrícula (37,5 °C) y roce pericárdico. En el electrocardiograma y en la radiografía de tórax no
se observaron en aquel momento hallazgos patológicos. Con el diagnóstico de pericarditis aguda el paciente fue remitido a su domicilio
bajo tratamiento con ácido acetilsalicílico a dosis antiinflamatorias. Tras una mejoría inicial, consultó diez días más tarde por fiebre de
39 °C, tos con escasa expectoración mucosa y dolor torácico de características pericardíticas, de tres días de evolución.

PREGUNTA
Cual de las siguientes manifestaciones es menos probable encontrar?

RESPUESTA
a.- Roce pericárdico.
b.- Alteración difusa de la repolarización con aplanamiento de las ondas T.
c.- Radiografía de tórax con cardiomegalia e infiltrado pulmonar en la língula.
d.- Determinación seriada de creatincinasa y de su fracción MB elevadas.

CASO CLINICO
Mujer de 60 años con DM diagnosticada 3 años, HTA de 10 años de evolución. Un año antes tenía una creatinina de 0,8 mg/dl; FGE
(MDRD) >60 ml/min/1,73 m; sistemático y sedimento de orina sin alteraciones. Refiere en las 6 semanas antes del ingreso astenia
progresiva, con posterior aparición de fiebre y tos seca. No presentaba disnea, dolor torácico ni alteración en la diuresis. En la
exploración tenía una presión arterial (PA) de 190/80 mmHg, estaba eupneica, con presión venosa yugular elevada, auscultación
cardíaca rítmica con roce pericárdico, normoventilación en ambos hemotórax y ausencia de edemas. En la analítica, presentaba una
hemoglobina de 7,7 mg/dl, urea de 217 mg/dl, creatinina 4,5 mg/dl, potasio 3,6 mEq/l, saturación basal de O2 del 98%; en orina, tenía
proteinuria de +++ y 60 hematíes/campo. En el ECG no había alteraciones. En la radiografía de tórax se observó una gran cardiomegalia,
se solicitó una ecocardiografía de urgencia, en la que se evidenció un derrame pericárdico moderado-grave.

PREGUNTA
Cual es la conducta mas apropiada a seguir?

CURSO ENARM CMN SIGLO XXI TEL: 36246001 Pharmed Solutions Institute PÁGINA 29
MANUAL DE TRABAJO DEL CURSO ENARM CMN SIGLO XXI
RESPUESTA
a.- Pericardiocentesis.
b.- Pericardiectomia.
c.- Diuretico, esteroide.
d.- Esteroide, ciclofosfamida.

CASO CLINICO
Mujer de 69 años de edad que sufre un trauma torácico cerrado por compresión antero posterior al quedar atrapada por las puertas de
un autobús y 30 días después comienza a presentar falta de aire a los esfuerzos, aumentando progresivamente hasta desencadenarse a
los pequeños esfuerzos. Ingresa en el hospital con diagnóstico de cardiopatía isquémica. Se realiza ecocardiograma y se comprueba
gran colección líquida pericárdica.

PREGUNTA
Cual de las siguientes manifestaciones es mas frecuente encontrar para establecer el diagnostico actual?

RESPUESTA
a.- Triada de Beck.
b.- Signo de Kussmaul.
c.- Disminución del voltaje de QRS.
d.- Elevación de CPK-MB y Troponinas.

PREGUNTA
Considerando la fisiopatogenia del presente caso, que tipo de choque es mas frecuente que presente?

RESPUESTA
a.- Choque Distributivo.
b.- Choque Obstructivo.
c.- Choque Cardiogenito.
d.- Choque hipovolemico.

CASO CLINICO
Varón de 55 años con recambio valvular mitral (1998) y síndrome pospericardiotomía, que cursó con fiebre y cansancio a mínimos
esfuerzos. Posteriormente quedó asintomático. En abril de 2004 presentó derrame pericárdico severo y taponamiento cardiaco.
Requirió pericardiocentesis. Tras ésta, quedó inicialmente asintomático. Reingresó en julio de 2004 y refierió que tras el alta clínica se
sintió bien, pero que en pocas semanas comenzó a sentir cansancio a grandes y luego a moderados esfuerzos. En el momento de su
reingreso la presión arterial (PA) era 110/70 mmHg, sin pulso paradójico, pero presentaba ingurgitación yugular. La auscultación
cardiaca era rítmica, con 70 lat/min y ruidos protésicos normales. Presentaba una discreta hepatomegalia y edemas con fóvea en los
miembros inferiores.

PREGUNTA
Cual de las siguientes afirmaciones no es correcta para el PEC?

RESPUESTA
a.- Es una infrecuente forma de síndrome pericárdico.
b.- Donde la constricción cardiaca ocurre en presencia de derrame pericárdico significativo.
c.- Los pacientes presentan taponamiento cardiaco, sin sintomáticos pospericardiocentesis.
d.- Ésta se resuelve sólo tras pericardiectomía.

CASO CLINICO
Mujer de 32 años con disnea a los pequeños esfuerzos, edema de miembros inferiores y astenia hace tres meses, se le prescribió
digoxina y furosemida, con una ligera mejora. Rechazaba comorbilidades cardiovasculares, pero con una historia de artralgia
inespecífica en la adolescencia y tres abortos entre 20 y 30 años de edad. El paciente estaba demacrado con auscultación pulmonar
normal y las extremidades bien perfundidas. En la evaluación cardíaca, se hallaba con FC = 98 lpm, PA = 100 / 70 mmHg con un ritmo
cardíaco regular y un ruido diastólico mitral de +++/4+, definido como un ruido protodiastólico ("knock" pericárdico).

PREGUNTA
Cuál es la conducta a seguir para establecer el diagnostico.

RESPUESTA
a.- ECG.
b.- TAC
c.- Rx de torax.
d.- IRM.

PREGUNTA
Cual de las siguientes manifestaciones no es propio de la pericarditis?

CURSO ENARM CMN SIGLO XXI TEL: 36246001 Pharmed Solutions Institute PÁGINA 30
MANUAL DE TRABAJO DEL CURSO ENARM CMN SIGLO XXI

RESPUESTA
a.- Localización retrosternal súbito.
b.- Naturaleza pleurítica.
c.- Se exacerba con la inspiración.
d.- Disminuye al inclinarse.

PREGUNTA
Cuales no son cambios que se presentan en ECG del paciente?

RESPUESTA
a.- Elevación del segmento ST de forma cóncava.
b.- Hay desarrollo de ondas Q.
c.- El voltaje de la onda R se mantiene.
d.- Depresión del segmento PR.

PREGUNTA
Considerando los antecedentes del caso clínico cual es la etiología mas frecuente?

RESPUESTA
a.- Idiopatico.
b.- Traumatico.
c.- Bacteriano.
d.- Viral.

PREGUNTA
Cual de no es una secuela de la patologia del paciente?

RESPUESTA
a.- Taponade cardiaco.
b.- Pericarditis recurrente.
c.- Prericarditis restricitva.
d.- Pericarditis constrictiva.

CARDIOMIOPATIA Y MIOCARDITIS (TEMA)


Es un espectro de patologías con diversos mecanismo patogénicos, con un final semejante a un síndrome congestivo de falla cardiaca,
el daño o padecimiento del miocardio puede ser primario o secundario, las causas de miocardiopatias mas frecuentes son: miocarditis
infecciosa viral (coxsaxkievirus, echovirus, HIC, Epstein barr, influenza, CMV. Bacteriano (Corynebacterium diphtheriae, streptococcus
pyogenes, staphylococcus aureus, haemophilus pneumoniae, salmonella spp, neisseria gonorrhoeae, leptospirosis, lyme, syphilis,
brucelosis; Fungico (Candida spp, arpergillus spp, histoplasmosis, blastomicosis, cryptoccosis, coccidioidomycosis; Parasitario
(Toxoplasmosis, schistosomiasis, trchinosis). Dilatada (desconocida); Infiltrativa (amiloidosis, sarcoidosis, hemocromatosis).
MIOCARDITIS: Las causas mas frecuentes son procesos infecciosos virales del 1 al 9 % son coxsackie virus B, las manifestaciones son
variadas, puede pasar asintomáticas hasta un profundo choque cardiogenico, el antecedente de afeccion viral de 7 a 10 dias y ataque al
estado generalizado el 60 % de los casos, 35% presentan dolor torácico, otra presentación son los bloqueos cardiacos completos o
taquicardia ventricular. Las manifestaciones clínicas incluyen fiebre, quicardia, signos de insuficiencia cardiaca, disminución de ruidos
cardiacos, galope, murmullo por regurgitación mitral, frote pericardico, laboratorios son leucocitosis, eosinofilia, incremento de IgM,
IgG, CPK en aproximadamente 10 %, el ECG taquicardia, alteración de ST y cambios de la onda T, prolongación QT, el ECOCG puede
manifestar alteraciones de la función ventricular, se clasifica en fulminante, agudo, crónico activo, crónico persistente. Sobrevida a 5
años de 50 %. El tratamiento es de soporte, diuréticos, ECA´s, bloqueadores antagonistas de aldosterona, la digoxina puede
incrementar la expresión inflamatoria, AINE´s no son utiles, son utiles; esteroides con azatioprine o esteroide con ciclosporina.
ENFERMEDAD DE CHAGAS La tripanosomiasis o enfermedad de chagas es mas común en america central y del sur, es causada por
tripanosoma cruzi, la lesión cardiaca es mediada por inumunocomplejos. Se disemina via hematogena a varios órganos y sistemas con
una intensa reacción inflamatoria, fiebre, sudoración, mialgias y miocarditis. El 5 % son fatales, del 20 al 30 % permanecen
asintomáticos en fase latente, los casos crónicos presentan fibrosis de miofibrillas causando cardiomegalia, falla cardiaca, bloqueos y
arritmias. El tratamiento es sintomático, marcapasos y agente antiparasitarios- ENFERMEDAD DE LYME: Es causada por la infección con
la espiroqueta borrelia burgdorferi introducido por picadura, los síntomas iniciales del padecimiento es bloqueo cardiaco completo,
puede verse disfunción ventricular izquierda, la miopsia muestra datos de miocarditis activa. MIOCARDITIS REUMATICA: Se puede
observar durante la fiebre reumática aguda seguido de faringitis streptococcica del grupo A, El diagnostico clínico se hace con los
criterios de JONES. Mayores; carditis, poliartritis, corea, eritema marginatm, nódulos subcutáneos, y evidencia de infección previa. Los
criterios Menores son fiebre artralgias, fiebre reumática, elevada sedimentación eritrocitaria, proteína C-reactiva y prolongación del
intervalo PR. El diagnostico es con dos mayores y un mayor y dos menores, del 5 al 10 % desarrollan insuficiencia cardiaca. Se presenta
carditis, lesion valvular aortica predominantemente. ECG prolongación de PR y cambios inespecíficos del ST-T. Tx aspirina, corticoides y
1.2 millones U de penicilina G benzatina. MIOCARDITIS NO INFECCIOSAS: La miocarditis hipersensitivas y son causadas por reacción a
medicamentos, caracterizada por eosinofilia, infiltración al miocardio de eosinofilos, células gigantes multinucleadas, y leucocitos, los
fármacos mas frecuentes son: metildopa, penicilina, tetraciclina y antituberculosos, pueden causar emadamente raras que generan
miocariditis, progresiva sin respuesta a tratamiento, mas frecuente en adulto jove, asociado a enfermedades autoinmunes 20 %, se

CURSO ENARM CMN SIGLO XXI TEL: 36246001 Pharmed Solutions Institute PÁGINA 31
MANUAL DE TRABAJO DEL CURSO ENARM CMN SIGLO XXI
observa histiocitos y linfocitos y eosinofilia, es fatal solo ttx transplante. Cardiomiopatía periparto, 1 en 3,000 partos, multiples factores,
obesidad, gestacion multiple, preeclapmsia, hipertensión crónica, px con 30 años aproximadamente. La falla cardiaca es variable inicia
en el tercer trimestre, ECG con hipertrofia ventricular izquierda, pronostico reservado. Miocardiopatias por enfermedades
neuromusculares, distrofias neuromusculares hereditarias, asociadas a cardiomiopatías Becker, Duchenne, Steinert, distrofia miotonica,
Friedreich, Barth. CARDIOMIOPATIAS POR ENFERMEDADES ENDOCRINAS: El exceso o disminución de hormonas tiroideas, en la
tirotoxicosis se compromete eventualmente la función ventricular izquierda con dilatación pudiendo llegar a falla cardiaca, El
feocromocitoma es otra causa con hipertensión, sudoración, palpitaciones e hipotensión ortostatica, puede presentarse miocarditis
inducida por catecolaminas, puede llegar a fallar cardiaca o arritmias ventriculares malignas. La acromegalia presenta cardiomiopatía en
el 10 al 20 % de los casos por el exceso de hormona de crecimiento que genera hipertrofia de miocitos que se fibrosan, con disfunción
diastólica y sistólica finalmente, también degeneración del nodo AV con bloqueo completo. MIOCARDIOPATIAS POR TOXINAS: El abuso
crónico de ALCOHOL es un riesgo mayor para desarrollar cardiomiopatía congestiva, 45 % de todas las dilatadas, puede generar
insuficiencia cardica, hipertensión y arritmias, el daño es directo, acetilaldehido y metabolitos, asi como deficiencia nutrimentales,
estimulación simpatica y cofactores toxicos. La edad promedio de 30 a 55 años, con historia de 10 años de consumo intenso, la
fibrilación atrial es la forma mas frecuente de preentacion luego la muerte súbita. COCAINA produce isquemia miocárdica, infarto,
espasmo coronario, arritmias cardiacas, muerte súbita, miocarditis y cardiomiopatía dilatada. Mal pronostico, tratamiento sintomático
los betabloqueadores puede disminuir espasmos. QUIMIOTERAPICOS: doxirrubicina, ciclofosfamida, generan citotoxicidad, la radiación
de mediastino, el trastuzumab es un anticuerpo monoclonal se puede presentar miotoxicidad. La CLOZAPINA antipsicotico de uso
crónico. Fenotiacinas, cloroquina, litio, cobalto, hidrocarbonos, interferon alfa IL-2. CARDIOMIOPATIAS ASOCIADAS CON DEFICIENCIAS
NUTRIMENTALES: Deficiencia de TIAMINA, resulta en beri beri con síntomas caracteristicos de falla cardiaca, pronostico fatal sin
tratamiento, Deficiencia de VITAMINA D, la deficiencia o el exceso aumenta el riesgo, falta de absorsion de SELENIO, la disminución de
L-CARNITINA. CARDIOMIOPATIAS TAKO-TSUBO: La miocardiopatía de Takotsubo es un síndrome recientemente descrito atribuido al
exceso de catecolaminas, probablemente relacionado con una hiperactividad simpática inducida por un factor estresante de cualquier
tipo. Son numerosas las enfermedades neurológicas que se han descrito relacionadas con este síndrome: hemorragia subaracnoidea,
ictus, esclerosis múltiple, síndrome de Guillain-Barré, crisis miasténicas y crisis epilépticas. Del mismo modo, son numerosos los casos
de síndrome de Takotsubo asociados a enfermedades psiquiátricas, dependencia a opioides, alcoholismo, trastorno maníaco-depresivo,
trastorno depresivo, esquizofrenia. También se han relacionado diversos factores emocionales (malas noticias, fiestas sorpresa,
discusiones, divorcio, muertes inesperadas...) y físicos (ejercicio, neumotórax, hipoglucemia, ataque de asma, cirugías...). El síndrome
de Takotsubo, descrito por primera vez en Japón en 1991, se caracteriza por acinesia o discinesia de la porción apical y media del
ventrículo izquierdo que no corresponde con un único territorio vascular en ausencia de enfermedad coronaria, demostrada mediante
cateterismo cardíaco. El electrocardiograma puede mostrar elevación del segmento ST o inversión de la onda T. Como criterios de
exclusión, entre otros, se encuentran antecedentes recientes de hemorragia intracraneal, feocromocitoma, miocarditis, miocardiopatía
hipertrófica, hipertiroidismo, pancreatitis y envenenamiento. Como desencadenantes se han relacionado factores de estrés tanto de
tipo emocional como físico. CARDIOMIOPATIA DILATADA: Es una causa frecuente de insuficiencia cardíaca y es el diagnóstico más
frecuente en pacientes sometidos a trasplante cardíaco. Desde el punto de vista clínico, la MD se caracteriza por dilatación y disfunción
contráctil del ventrículo izquierdo o de ambos ventrículos. La dilatación ventricular es generalmente severa y se acompaña siempre de
hipertrofia. La MD puede ser idiopática, genética/familiar, viral y/o inmune, alcohólica/tóxica, o asociada a otras cardiopatías en las
cuales el grado de disfunción miocárdica no se explicaría por una determinada sobrecarga hemodinámica o severidad de daño
isquémico. Probablemente, el síndrome clínico de la MD representa un final común al que se llega a través de múltiples mecanismos
citotóxicos, metabólicos, inmunológicos, infecciosos y familiares. Anatomía patológica: Los estudios post mortem demuestran
habitualmente dilatación de las cuatro cámaras cardíacas, en especial los ventrículos, que se acompaña a veces de aumento de grosor
de la pared. Las válvulas cardíacas son intrínsecamente normales y es frecuente la presencia de trombos intracavitarios. Es típica la
presencia de miocitos hipertróficos y muertos, sustituidos por fibrosis con variable afectación del sistema de conducción. Los
componentes de la matriz extracelular están aumentados de forma no selectiva. Aunque la ausencia de células inflamatorias se utiliza
como criterio para diferenciar la MD de la miocarditis, en algunos casos de MD se detectan células T inflamatorias y células endoteliales
activadas, sugiriendo la posible contribución de un proceso inflamatorio crónico en la patogenia de esta enfermedad. La etiología y los
mecanismos patogénicos son desconocidos en alrededor de la mitad de los casos de MD. Para explicar el daño miocárdico crónico y
progresivo se han propuesto tres principales etiopatogenias: a) infección viral crónica del miocardio que produce daño celular; b)
alteración de los mecanismos inmunes que conduce probablemente a una enfermedad autoinmune, y c) factores genéticos que serían
directa o indirectamente responsables de la enfermedad. DIAGNÓSTICO: El estudio del paciente con MD debe enfocarse no sólo al
establecimiento del diagnóstico sindrómico, sino hacia la identificación, por los métodos de diagnóstico habituales, de posibles causas
tratables o reversibles de la enfermedad. La historia clínica debe incluir preguntas relativas al posible consumo de alcohol y cocaína,
medicamentos, hábitos nutricionales, estancias en zonas endémicas para infecciones, relación con animales, embarazos recientes,
transfusiones sanguíneas, historia familiar de MD, somnolencia diurna y exposición profesional a tóxicos. Asimismo, la idea de que la
MD idiopática es con frecuencia un problema genético hereditario debe ser tenida en cuenta en la práctica clínica, estudiando
sistemáticamente a los familiares de primer grado del paciente. En la mayoría de los pacientes la MD se manifiesta clínicamente entre
los 20 y 60 años de edad, aunque la enfermedad puede afectar también a niños y ancianos. Los síntomas más frecuentes son los de
insuficiencia cardíaca (disnea de esfuerzo progresiva, ortopnea, disnea paroxística nocturna y edemas periféricos). Otras formas de
presentación son la detección accidental de cardiomegalia asintomática y los síntomas relacionados con arritmias, alteraciones de
conducción, complicaciones tromboembólicas o muerte súbita. La exploración física suele revelar diferentes grados de cardiomegalia y
signos de insuficiencia cardíaca. La presencia de un galope presistólico (cuarto ruido) puede preceder a la aparición de insuficiencia
cardíaca. El ritmo de galope ventricular (tercer ruido) es la regla en los casos con descompensación de la IC. Es frecuente la presencia de
soplos sistólicos de insuficiencia mitral o, menos frecuentemente, tricuspídea. Las dos causas más frecuentes de muerte en pacientes
con MD e insuficiencia cardíaca son la muerte súbita y el fallo de bomba progresivo. MIOCARDIOPATÍA HIPERTRÓFICA: La
miocardiopatía hipertrófica (MH) es una enfermedad con una importante heterogeneidad en cuanto a su base genética,
manifestaciones clínicas y pronóstico 93-96. Se caracteriza, fundamentalmente, por la presencia de una hipertrofia ventricular de causa
desconocida (generalmente de predominio septal) y por una excelente función sistólica 93-96. Los gradientes intraventriculares

CURSO ENARM CMN SIGLO XXI TEL: 36246001 Pharmed Solutions Institute PÁGINA 32
MANUAL DE TRABAJO DEL CURSO ENARM CMN SIGLO XXI
dinámicos constituyen un rasgo primordial de esta entidad, pero sabemos que sólo se detectan en una parte de los pacientes y buena
parte de las manifestaciones son consecuencia de las alteraciones diastólicas. El diagnóstico clínico de los pacientes con MH sigue
basándose en la demostración de una hipertrofia ventricular en ausencia de factores cardíacos o sistémicos que la justifiquen. Los
síntomas más frecuentes -disnea, angina, palpitaciones-, se manifiestan en más de la mitad de los pacientes, pero son comunes en
otras patologías cardiovasculares. Sin embargo, algunos datos clínicos como, por ejemplo, disnea en presencia de una buena función
sistólica o angina con coronarias angiográficamente normales pueden orientar hacia el diagnóstico. Mucho más específica, sin
embargo, sería la aparición de síncope o presíncope en individuos jóvenes previamente asintomáticos. Un episodio de muerte súbita
recuperada en un niño, joven, o adulto, sin duda obliga a descartar esta patología. Tras la anamnesis, la exploración física dirigida
puede ser diagnóstica cuando se detecta una semiología característica de gradiente intraventricular dinámico, pero en caso contrario,
es poco reveladora. Con frecuencia, un electrocardiograma manifiestamente patológico (ondas Q, hipertrofia ventricular, patrones de
preexcitación o severas alteraciones de la repolarización), muchas veces en un individuo asintomático, es la primera clave diagnóstica.

CASO CLINICO
Masculino de 38 años de edad, sin antecedentes personales ni familiares de interés, sin hábitos tóxicos, ni tratamiento habitual. Refería
un cuadro catarral de 7 días de evolución tratado con amoxicilina-clavulánico. Un día antes del ingreso presentó disnea progresiva hasta
la ortopnea, presentaba una tensión arterial indetectable y una expectoración espumosa, abundante y hemoptóica, junto a signos de
insuficiencia respiratoria severa (taquipnea, cianosis, crepitantes pulmonares generalizados y desaturación del 70%). El ECG mostraba
una taquicardia sinusal a 180 lpm, con QRS ancho, disociación auriculoventricular y morfología de bloqueo de rama izquierda, criterios
de taquicardia ventricular (TV) y, la radiografía torácica, unos infiltrados algodonosos bilaterales. Analíticamente, salvo poliglobulia,
acidosis metabólica e intensa hipoxemia, no presentaba otros hallazgos de interés incluyendo los enzimas cardiacos.

PREGUNTA
Cual es la forma mas frecuente que debute esta patologia MCG?

RESPUESTA
a.- Taquicardia ventricular.
b.- Insuficiencia cardiaca.
c.- Infarto al miocardio.
d.- Bloqueo auriculoventricular completo el 5%.

CASO CLINICO
Paciente mujer de 18 años de edad, natural y procedente de Lima. Alergia a AINES. Acudió por presentar en forma brusca, malestar
general, nauseas, vómitos y deposiciones líquidas sin moco, ni sangre (4 horas de inicio). En el examen de ingreso se constató que sus
funciones vitales estaban dentro de límites normales. Se le diagnosticó gastroenterocolitis aguda y deshidratación leve-moderada; en
ese sentido, y de acuerdo con los diagnósticos planteados, en el manejo inicial se administró hidratación endovenosa con cloruro de
sodio al 9%, pargeverina clorhidrato 10mg y dimenhidrinato 50mg. Una hora después de la admisión, la paciente desarrolló un cuadro
de hipotensión (presión arterial: 70/50 mmHg) y palidez marcada, recibió tratamiento de soporte, pero ante el deterioro clínico
paulatino la paciente fue conducida a la unidad de cuidados intensivos.

PPREGUNTA
Cual es la imprension diagnostica inicial del caso?

RESPUESTA
a.- Estado de choque séptico.
b.- Estado de choque anafiláctico.
c.- Estado de choque cardiogenico.
d.- Estado de choque hipovolemico.

CASO CLINICO
Un varón de 42 años fue hospitalizado por fiebre y síntomas de dolor torácico. Antecedente de (VIH), diagnosticado 15 años antes del
ingreso. El recuento de células T CD4+ al ingreso era de 874 cél./μl y la carga viral era indetectable (< 50 copias/ml). En tratamiento con
tenofovir + didanosina + atazanavir + ritonavir, con una carga viral indetectable en los últimos 6 años. El cuadro se inició con fiebre,
diarrea, mal estado general y mialgia de 1 semana de duración. En los 2 días previos al ingreso sufrió un dolor torácico punzante intenso
y progresivo, en reposo, con irradiación a la espalda, que se agravaba con el decúbito y la inspiración profunda. La intensidad del dolor
torácico disminuyó tras la administración de analgésicos intravenosos, pero no con nitroglicerina sublingual. En la exploración física, las
constantes vitales eran normales, excepto la temperatura corporal, que era de 38 °C. No presentaba signos de congestión sistémica o
pulmonar ni había signos de bajo gasto ni dificultad respiratoria. Los ruidos cardiacos eran normales. Los datos de laboratorio revelaron
leucocitosis con linfocitosis, proteína C reactiva en 19 mg/ml y un pico de troponina I cardiaca que alcanzó 30 ng/ ml. La radiografía de
tórax fue anodina. El ECG mostró un ritmo sinusal normal, con una elevación del segmento ST de 1 mm en las derivaciones DI, aVL y V4-
V6.

PREGUNTA
Cual es el agente mas frecuente de esta patologia?

RESPUESTA
a.- Parvovirus B19.

CURSO ENARM CMN SIGLO XXI TEL: 36246001 Pharmed Solutions Institute PÁGINA 33
MANUAL DE TRABAJO DEL CURSO ENARM CMN SIGLO XXI
b.- H1N1.
c.- VIH.
d.- Cocxackie B6.

CASO CLINICO
Mujer de 43 años con historia de epilepsia focal del lóbulo temporal izquierdo criptogénica farmacorresistente, trastorno límite de la
personalidad, etilismo crónico, infección por el virus de la hepatitis C. Fue ingresada en Urgencias por presentar una crisis focal motora
secundariamente generalizada. Posteriormente, durante las primeras 24 horas de observación, sufrió cuatro crisis tonicoclónicas
generalizadas más, la frecuencia cardíaca durante la crisis descendió hasta 35 latidos por minuto. Mostró también disnea y se objetivó
en la auscultación cardiopulmonar crepitantes bibasales, bradicardia extrema.

PREGUNTA
Considerando las manifestaciones clínicas cual de las siguientes condiciones es la mas probable que presenta el paciente?

RESPUESTA
a.- Choque Distributivo.
b.- Choque Cardiogenico.
c.- Choque Hipovolemico.
d.- Choque Oculto.

CASO CLINICO
Mujer de 50 años, procedente de zona rural. Presentó un cuadro clínico de 15 días de evolución consistente en fiebre, deposiciones
diarréicas y, posteriormente, disnea, el cual la obligó a consultar en varias ocasiones. La sintomatología, especialmente la disnea,
empeoró, por lo cual consultó nuevamente, encontrándose en el examen físico signos claros de insuficiencia cardiaca. En la radiografía
de tórax se 7evidenció cardiomegalia y derrame pleural, y el ecocardiograma reportó taponamiento cardiaco.

PREGUNTA
Cual es su conducta a seguir mas adecuada?

RESPUESTA
a.- Pericardiocentesis.
b.- Pericardiotomia.
c.- Diuretico, corticosteroide, b-bloqueador.
d.- Antiparasitario, pericardiocentesis.

CASO CLINICO
Hombre de 26 años, procedente de zona rural. Consultó por cuadro de cinco días de fiebre, escalofríos, dolores musculares, cefalea,
artralgias, astenia y adinamia. Se realizó estudio para hemoparásitos, encontrándose ocasionales tripomastigotes. El ecocardiograma
mostró leve derrame pericárdico y leve dilatación ventricular izquierda. Además, se encontró franca leucopenia (3.200 leucocitos/µl) y
plaquetopenia (69.000 plaquetas/µl).

PREGUNTA
Cual es la conducta farmacológica especifica para el caso?.

RESPUESTA
a.- Metronidazol.
b.- Albendazol.
c.- Benzonidazol.
d.- Prazicuantel.

CASO CLINICO
Varón de 36 años, labrador, sin antecedentes de interés salvo picaduras frecuentes de garrapatas. Ingresa en nuestro hospital por
episodio sincopal y bloqueo AV avanzado. En las horas siguientes al ingreso se continúa apreciando un bloqueo AV completo, con
pausas de hasta 6 segundos de duración, implantándose un marcapasos provisional. Un mes previo al ingreso refiere un eritema en
región glútea (cuyas características no se pudieron precisar), fatiga y mareos. Dos semanas más tarde, comienza con artralgias en codos
y rodillas, astenia y disnea. La exploración física fue anodina salvo por febrícula vespertina. La radiografía de tórax y la analítica básica
fueron normales.

PREGUNTA
Considerando el agente causal mas probable del caso por sus antecedentes, cual es la pauta antibiótica mas adecuada?

RESPUESTA
a.- Ceftriaxona.
b.- Doxiciclina.
c.- Ciprofloxacina.
d.- Gentamicina.

CURSO ENARM CMN SIGLO XXI TEL: 36246001 Pharmed Solutions Institute PÁGINA 34
MANUAL DE TRABAJO DEL CURSO ENARM CMN SIGLO XXI
CASO CLINICO
Una mujer de 41 años con antecedente de HTA ligera no tratada acude a urgencias de su hospital presentando dolor torácico de
características anginosas y elevación del segmento ST en cara lateral. Desde 3 años antes sufría un trastorno de ansiedad que había
requerido ingreso en una ocasión. En las últimas semanas la paciente sufría un gran estrés laboral. Presión arterial de 110/70 mmHg;
tras nitroglicerina sublingual, la clínica cedió y se normalizó el ECG. A las 12 h vuelve a tener dolor con elevación del segmento ST,
cateterismo cardiaco normal, pero la ventriculografía muestra una disfunción medioventricular severa, con hipercontractilidad de los
segmentos basales y apicales. Se inició tratamiento con BB e IECA, manteniendo la anticoagulación y la aspirina. La paciente
permaneció asintomática, con creatincinasa normal y troponina I de 1,4. A los 5 días del ingreso se realizó una RM cardiaca que todavía
mostró una hipocinesia medioventricular, sin realce tardío tras la administración de gadolinio. Al ingreso en nuestro hospital se habían
solicitado catecolaminas en sangre y en orina para una mejor caracterización del cuadro. Éstas mostraron una elevación supranormal
(noradrenalina y adrenalina en sangre > 5.000 y 190 pg/ml; noradrenalina y metanefrina en orina de 24 h, 582 y 5.386 μ g/24 h).

PREGUNTA
Cual es la conducta mas apropiada a seguir?

RESPUESTA
a.- Resonancia magnetica abdominal.
b.- Tomografia axial computada.
c.- USG abdominal.
d.- Tomografia helicoidal.

CASO CLINICO
Mujer de 30 años sin antecedentes de cardiopatía que ingresa en nuestro centro debido a un cuadro de insuficiencia cardiaca
congestiva un mes y medio después de un parto sin complicaciones. Se realizó un ecocardiograma en el que se objetivó dilatación
moderada del ventrículo izquierdo (diámetro telediastólico, 64 mm; diámetro telesistólico, 46 mm) junto con hipocinesia generalizada y
fracción de eyección del ventrículo izquierdo del 32%; la aurícula izquierda estaba ligeramente dilatada y las cavidades derechas, en el
límite superior de la normalidad; se apreciaron regurgitaciones mitral y tricuspídea moderadas-severas sobre válvulas estructuralmente
normales, con estimación de presión arterial pulmonar sistólica de 46 mmHg. Tras la estabilización, se realizó una cardiorresonancia
magnética, en la que se confirmó que el ventrículo izquierdo estaba moderadamente dilatado, con fracción de eyección del 37%, así
como hipocinesia generalizada del ventrículo derecho con función sistólica general severamente deprimida (fracción de eyección, 25%);
en el estudio de retención miocárdica tardía de contraste con gadolinio, se observó un depósito mesocárdico lineal a nivel septal
extenso. En una coronariografía no se objetivaron lesiones coronarias. Con el diagnóstico de miocardiopatía periparto.

PREGUNTA
Cuales son los siguiente criterios no es útil para establecer el diagnostico de cardiomiopatía periparto?

RESPUESTA
a.- Desarrollo de insuficiencia cardiaca en el último mes de embarazo.
b.- Desarrollo en los 2 meses siguientes al parto.
c.- Ausencia de causa identificable para el desarrollo de insuficiencia cardiaca.
D.- Ausencia de enfermedad cardiaca primaria en el último mes de embarazo.

ENDOCARDITIS
La endocarditis infecciosa es una enfermedad causada por un agente microbiano que afecta la capa endotelial de estructuras
intracardiacas que invaliablemente es fatal sin tratamiento. La infección más frecuente reside en una o mas válvulas que envuelve el
endocardio mural, miocardio y pericardio. Los dispositivos intracardiacos o endovascular es una fuente de infección. La mortalidad es
del 25 % dentro de los 6 meses. EPIDEMIOLOGIA: La causa inicial de EI era una complicación de fiebre reumática o de origen dental,
actualmente la degeneración de válvulas cardiacas o dispositivos se han incrementado. Los pacientes con DM, VIH, IRC esta en mayor
riesgo de EI con riesgo nosocomial. PATOGENESIS: La EI es una bacteremia persistente y continua endocardica o endovascular. El
agente patógeno lesiona estructuras cardiacas y presenta capacidad de adhesión, y evitar la actividad inmunológica del huésped,
generandoce vegetaciones en válvulas. PATOGENIA: 80 % son estreptococos y estafilococos, en el mundo el Staphylococcus aureus es
el más común. ENDOCARDITIS DE VALVULAS NATIVAS: El estreptococo viridans o el estreptococo alfa-hemolitico es el agente mas
común adquirido en la comunidad. Es un agente nativo en la orofaringe que fácilmente entra a la circulación via daño dental o gingival.
El estreptococo viridan (S. bovis, S. mutans, S. mitor) nativos en sistema gastrointestinal o debido a patologia del sistema GI. El grupo D
de estrptococos en particular el enterococos spp. Causan hasta el 18 % de los casos de EI. E. fecalis se presenta hasta el 80 %, su ingreso
a circulación es por manipulación con sonda Foley, colonoscopia, etc. El estreptococo del grupo A, es una causa rara el S. pneumonie
causa 10 % de EI es agudo, fulminante, asociado a daño severo valvular con extensión perivalvular, complicaciones embolicas,
pericarditis, meningitis con mortalidad del 25 al 50 %. El grupo B (S. agalactiae) se presenta en neonatos, parturientas, en pie diabético,
carcinoma, falla hepática, alcoholismo, uso de drogas inyectables. Estafilococo aureus causa del 80 a 90 % de IE, es la causa aguda mas
frecuente, la nasofaringe es el sitio mas común de conolizacion, los factores de riesgo son diálisis, diabetes, quemaduras, VIH, usuarios

CURSO ENARM CMN SIGLO XXI TEL: 36246001 Pharmed Solutions Institute PÁGINA 35
MANUAL DE TRABAJO DEL CURSO ENARM CMN SIGLO XXI
de drogas, en condiciones dermatológicas crónicas. La IE fulminante es causada por El S. aureus con falla carrdiaca, afeccion
perivalvular, trastornos de conducción, infección metastasica con mortalidad del 25 al 30 %, ocacionalmente es debido a estafilococo
coagulaza positivo nativo de la piel. El S. epidermidis es una causa importante en dispositivos ol prótesis valvulares. La IE debido a
bacilos gran negativos, es infrecuente, en patologia hepática o prótesis cardiaca, es difícil cultivarlo, Coxiella burneti agente causante de
la fiebre Q, la Bartonella es otra causa. La IE la causa fungica presenta alta mortalidad 20 % en pacientes inmunocomprometidos, con
dispositivos, candida y aspergelius son los mas comunes, generan vegetaciones tratamiento con anfotericina. Puede presentarse por
causas autoinmune. En caso de IE se presenta entre los 2 meses y el primer año del tratamiento quirúrgico, las causas nosocomiales son
estafilococo coagulasa negativo (s. epidemidis).

CASO CLINICO
Mujer de 46 años, afectada de nefropatía lúpica IV, en programa de diaslisis peritoneal. Desde entonces mantiene marcadores
positivos. Tuvo un brote cutáneo-articular, por lo que recibía con micofenolato sódico a dosis de 180 mg y prednisona a dosis de 5 mg
diarios. Ingresa por disnea y malestar general progresivo, de 15 días de evolución. Dolor torácico en el hemitórax izquierdo que
aumenta con la inspiración profunda y que mejora relativamente en anteversión. No refiere síndrome febril, ni otra clínica
acompañante. EF soplo diastólico en el foco aórtico, irradiado a las carótidas, con roce pericárdico importante, sin signos de fallo
cardíaco. El resto de la exploración física fue anodina. En la analítica destacan: leucocitos * 21,3 K/µl (4,4-11,3), cayados 3%, neutrófilos
* 92,0% (50-70), linfocitos * 3,0% (25-40), proteína C reactiva (PCR) * 17,73 mg/dl (0,1-0,5), procalcitonina * 4,84 ng/ml (<0,5).
Anticuerpos antinucleares (IFI) 320-640 u arb (0-80), anticuerpos anti-ADN (EIA) 1,3/ml (<10 U/ml), anticuerpos anti-ADN (IFI) <80 u arb
(0-80), anticuerpos cardiolipina (IgG) 3,5 U GPL/ml, IgM 2,5 U GPL/ml (negativos), C3 118 mg/dl (79-152), C4 24,4 mg/dl (16-38), urea
127, creatinina 8,46 mg/dl, hemoglobina (Hb) 9,6 g/dl, hematocrito 29%. Tiempo de cefalina 27,9/30 segundos (29-31 s),
anticoagulante lúpico positivo.

PREGUNTA
Cual es la conducta antibiótica empirica mas adecuada?

RESPUESTA
a.- Vancomicina, ceftriaxona y dicloxacilina.
b.- Amilacina, doxiciclina y ceftaxidima.
c.- Vancomicina, ceftazidima y gentamicina
d.- Gentamicina, vancomicina y imipenem.

CASO CLINICO
Varón de 38 años de ocupación pintor, con signos de disnea, ortopnea, edemas en miembros inferiores, además de registros febriles
aislados de 2 meses de evolución. Relató también la presencia de un soplo cardíaco desde hacía 3 años sin estudio, enfermedad
periodontal, caries dentales y otros antecedentes sin relevancia para el caso. Negaba contacto con animales de cría o consumo de
productos sin pasteurización. Dos semanas antes de la consulta presentó tos con expectoración purulenta, que cedió
espontáneamente. Dos horas antes de la consulta presentó dolor precordial, opresivo, intensidad 8/10, con irradiación a brazo
izquierdo. El ecocardiograma transtorácico, la presencia de dilatación de la aurícula izquierda, dos imágenes ecodensas sobre cara
ventricular de la válvula aórtica bicúspide, compatibles con vegetaciones de 28,4 mm x 9 mm y de 24 mm x 6 mm cada una,
insuficiencia valvular aórtica grave, ventrículo izquierdo dilatado, hiperdinámico, con signos de sobrecarga de volumen, función sistólica
conservada y leve derrame pericárdico, todos signos compatibles con endocarditis infecciosa.

PREGUNTA
Cual de los criterios es el indicado para endocarditis bacteriana?

RESPUESTA
a.- Criterios de Jones.
b.- Criterios de duke.
c.- Criterios de Baltazar.
d.- Criterios de Hermosillo.

PREGUNTA
Se aislo Brucella canis en los cultivos, cual es la conducta terapéutica mas adecuada?

RESPUESTA
a.- Doxiciclina, rifampicina y trimetoprim + sulfametoxazol.
b.- Vancomicina, rifampicina y doxiciclina.
c.- Ceftriaxona, doxiciclina y trimetoprim + sulfametoxazol.
d.- Imipenem, doxiciclina y rifampicina.

CASO CLINICO
Varón de 29 años de edad con cefalea, confusión y fiebre de hasta 40,3 °C, de 3 días de evolución. Es fumador de tabaco, cannabis,
heroína y cocaína, con hepatopatía crónica por virus de la hepatitis C. A la EF resaltaban somnolencia y desorientación. Laboratorios
con elevación de las transaminasas y leucocitosis con neutrofilia. Punción lumbar mostró un líquido cefalorraquídeo inflamatorio
agudo. El diagnóstico inicial fue de encefalitis herpética, posteriomente se aislo Staphylococcus aureus en dos hemocultivos, el

CURSO ENARM CMN SIGLO XXI TEL: 36246001 Pharmed Solutions Institute PÁGINA 36
MANUAL DE TRABAJO DEL CURSO ENARM CMN SIGLO XXI
ecocardiograma transesofágico evidenció una endocarditis mitral: había una vegetación móvil de 3 x 3 mm de diámetro, anclada en la
base de la superficie auricular del velo mitral posterior, que estaba perforado y ocasionaba regurgitación mitral ligera.

PREGUNTA
Cuál es el factor más probable que cause la endocarditis en este paciente.

RESPUESTA
a.- La bacteria aislada.
b.- La adicción a inyectables.
c.- La hepatitis C.
d.- La vegetación mitral.

CASO CLINICO
Mujer de 45 años de edad, fumadora activa, HTA mal controlada, crioglobulinemia mixta en tratamiento con corticoides e insuficiencia
renal crónica por nefroangiosclerosis en hemodiálisis. La paciente ingresa ante la sospecha de meningitis, tras presentar un cuadro de
fiebre de 5 días de evolución y disminución del nivel de conciencia. En la exploración física con GSC 8 (M4, O2, V2), rigidez de nuca,
soplo sistólico piante en foco mitral; hipoventilación basal derecha y signos de infección. En la radiografía de tórax se observa una
imagen de condensación alveolar bilateral. TAC craneal en la que se objetiva una lesión hipodensa occipital izquierda, sin dilatación del
sistema ventricular. Se realiza punción lumbar cuyos resultados son compatibles con meningitis bacteriana aguda, al mismo tiempo se
obtienen hemocultivos y broncoaspirado. Se inició tratamiento antibiótico de manera empírica. Posteriormente, se aisló en todas las
muestras cultivadas S. aureus metilicín sensible (SAMS).

PREGUNTA
Cuál es la causa más probable que genera los cambios en la TAC.

RESPUESTA
a.- Proceso infeccioso.
b.- Hipertension arterial.
c.- Incremento de la actividad plaquetaria.
d.- Uso crónico de corticoides.

CASO CLINICO
Se trata de un paciente de sexo masculino, de 51 años que ingresó a urgencias por 10 días de evolución de fiebre de 40ºC, asociada a
diaforesis y deposiciones líquidas sin moco ni sangre. Colecistectomía por laparoscopia 41 días antes, que requirió hospitalización por
31 días para el tratamiento de una infección del sitio operatorio con compromiso de organo/espacio, tratada con meropenem. No se
había realizado monitorización hemodinámica invasiva y no tenía antecedentes de uso de drogas ilícitas intravenosas. En el examen
físico se encontraba alerta, orientado, con FC de 72 latidos por minuto, FR de 16 respiraciones por minuto, TA de 95/57 mm Hg,
temperatura de 38,5°C y deshidratación grado I. Se auscultó un soplo holosistólico en el foco aórtico, grado II/VI. En el abdomen había
dolor a la palpación en epigastrio, sin signos de irritación peritoneal. En los exámenes de laboratorio con anemia normocítica,
normocrómica, elevación de la proteína C reactiva (PCR) y leve aumento de las transaminasas, y fosfatasa alcalina y amilasa en rango
normal.

PREGUNTA
Cual de siguientes factores de riesgo asociados para la patologia actual es menos frecuente?

RESPUESTA
a.- Cardiopatía congénita cianosante.
b.- Comunicación interventricular.
c.- Conducto arterioso permeable.
d.- Presencia de válvulas protésicas.

PREGUNTA
Entre los siguientes signos, cual es el menos frecuente en la patologia actual?

a.- Embolismo.
b.- Esplegnomegalia.
c.- Aneurisma micoticos.
d.- Lesiones en retina.

ASMA
CIENCIAS BASICAS: Definición: Obstrucción reversible al flujo aéreo producida por inflamación bronquial, así como por respuesta a
diversos estímulos externos o mediadores endógenos. La mayoría de los asmáticos tienen enfermedad durante la infancia. El asma
ocupacional puede resultar de gran variedad de productos químicos, incluyendo disocianato de tolueno y anhídrido trimelítico, y esto
puede ser de aparición en el adulto. Diferentes disparadores pueden empeorar la sintomatología como alérgenos inhalados, en especial
a individuos sensibles, infecciones virales, bloqueadores B- adrenérgicos pueden empeorar los síntomas de asma y por lo general se
deben evitarse, el ejercicio entre otros como la contaminación, exposiciones ocupacionales, cambios bruscos de temperatura, ERGE y

CURSO ENARM CMN SIGLO XXI TEL: 36246001 Pharmed Solutions Institute PÁGINA 37
MANUAL DE TRABAJO DEL CURSO ENARM CMN SIGLO XXI
estrés. SALUD PUBLICA: Prevalencia en México 10% de la población; 80% son niños menores de 8 años, del total de casos; 12% tienen
entre 9-17 años y 8% adultos. La mayoría de los asmáticos son atópicos y a menudo tienen asma y rinitis alérgica. La mayoría de los
asmáticos tienen enfermedad durante la infancia. PATOGENIA: En el asma la respuesta del árbol bronquial consiste en disminución del
calibre de su luz originada por 3 fenómenos; 1) constricción de la musculatura bronquial (broncoconstricción). 2) edema de la mucosa e
infiltración celular (inflamación). 3) incremento de la secreción bronquial (moco). Los mecanismos anatomopatologicos más
representativos son hipertrofia de musculo liso bronquial, transformación de fibroblastos a miofibroblastos y depósito de colágena
subepitelial (fenómeno de remodelación), todo esto lleva a largo plazo a disminución de la función pulmonar. DIAGNOSTICO: Cuadro
clínico: Síntomas respiratorios comunes son sibilancias, disnea y tos en accesos, opresión torácica, con predominio nocturno y/o
matutino. Exploración física: Depende gravedad podemos observar taquipnea, uso de músculos accesorios, cianosis, voz entrecortada,
diaforético, taquícardico, rudeza respiratoria. La evidencia de rinitis alérgica, sinusitis, reacción en la piel podrían asesorarnos. Pruebas
de función pulmonar: La prueba ideal es la espirometría (medimos resistencia al flujo aéreo), nos reportara un patrón obstructivo con
reducción de FEV1 (volumen espiratorio forzado en el primer segundo) y CVF (capacidad vital funcional), si hay síntomas de asma y la
espirometría es normal se pueden realizar pruebas de metacolina, histamina o provocación con ejercicio. La tasa de flujo espiratorio
máximo (PEF) puede ser utilizado, para seguir el control del asma objetivamente en casa. Rx. de tórax: Usualmente normal, en
exacerbaciones agudas podemos identificar neumotórax, sobredistención por atrapamiento aéreo, horizontalización de arcos costales,
abatimiento de hemidiafragmas, infiltrados pulmonares eosinofilicos. CLASIFICACION Y TRATAMIENTO: Asma extrínseca, pacientes en
o que puede demostrarse una reacción Ag-Ac como desencadenante del proceso, mediadad por IgE, generalmente atópica u
ocupacional. Asma intrínseca no se encuentra Ag específico, comienza en vida adulta, se puede asociar a pólipos nasales, sinusitis,
idiosincrasia, AS o AINES.

GRADO CARACTERISTICAS TX. RESCATE TX. MANTENIMIENTO


INTERMITENTE Síntomas menos de 1 vez por semana Los agonistas β2 adrenérgicos, relajan el Baja dosis de antiinflamatorios
Exacerbaciones de corta duración musculo liso (salbutamol) de corta acción,
Síntomas nocturnos no más de 2 veces al mes cuantas veces sea necesario
FEV1 o PEF >80% del valor predicho
Variabilidad en el PEF o FEV <20%
PERSISTENTE Síntomas más de 1 vez por semana pero menos de 1 vez al día Los agonistas β2 de corta acción, cuantas Agonista B, de larga acción (salmeterol,
LEVE Exacerbaciones que pueden afectar la actividad y el sueño veces sea necesario formoterol) + antiinflamatorios en baja dosis
Síntomas nocturnos más de 2 veces por mes
FEV1 o PEF >80% del valor predicho
Variabilidad en el PEF o FEV1 <20 a 30%
MODERADO Síntomas diarios Los agonistas B de corta acción, cuantas Esteroide inhalado dosis baja (u otro
PERSISTENTE Exacerbaciones que afectan la actividad y el sueño veces sea necesario antiinflamatorio a dosis alta) +
Síntomas nocturnos más de 1 vez a la semana broncodilatador de larga acción = reducen
Uso diario de inhaladores con b agonistas de acción corta las exacerbaciones y proporciona excelente
FEV1 o PEF 60 a 80% del valor predicho tx. a largo plazo
Variabilidad en el PEF o FEV1 >30%
INTENSA Síntomas diarios Los agonistas B de corta acción, cuantas Esteroide inhalado dosis media (u otro
PERSISTENTE Exacerbaciones frecuentes veces sea necesario. El uso excesivo antiinflamatorio a dosis alta),
Síntomas frecuentes de asma nocturna indica un control inadecuado broncodilatador de acción larga
FEV1 o PEF <60% valor predicho
Variabilidad en el PEF o FEV1 >30%

Los efectos secundarios de los B2 agonistas son temblores musculares y palpitaciones. Los agentes antiinflamatorios principalmente
usados son los esteroides inhalados (inhiben la infiltración celular y disminuyen el edema, efecto de 3 a 6 hrs). Son la primera línea en
pacientes que no se encuentran en crisis, son los mas efectivos en el control del tx. del asma, el efecto local más importante es la
candidosis oral, y ronquera los más usados, budesonina, fluticasona y beclometasona, reducen síntomas durante el ejercicio, y
síntomas nocturnos. Los antileucotrienos (montelukast, zafirlukast), beneficiosos solo en algunos puntos. La adrenalina produce
disminución de la resistencia muscular sobre el árbol bronquial. Otros brocodilatadores incluyen los anticolinérgicos (que se dan
principalmente en EPOC) y la teofilina es un inhibidor de la fosfodiesterasa que aumenta el adenosin monofosfato ciclico en los
neutrófilos, que tienen efecto brocodilatador y antiinflamatorio, pero debido a su toxicidad asociada, no se usa comunmente.
CRISIS ASMATICAS: Caracterizado por aumento del trabajo respiratorio, tos, opresión torácica y sibilancias; usar salbutamol o
terbutalina, micronebulizados, durante 1 h continua para romper el broncoespasmo. Esteroides endovenosos, metilprednisolona o
prednisona para romper cascada de inflamación.

CASO CLINICO
Mujer de 40 años, fumadora con una exposición tabáquica de 20 paquetes-año. Trabajaba en una empresa metalúrgica desde hacía 3
años. Desde hacía un año presentaba clínica de disnea, tos y sibilancias de predominio nocturno, que mejoraba los fines de semana y
períodos vacacionales. Requirió un ingreso por un episodio de broncospasmo y al reincorporarse a su lugar habitual de trabajo se
reiniciaron dichos síntomas. Con títulos de IgE sérica total de 59 U/ml. El estudio de la función pulmonar fue normal, mostrando una
capacidad vital forzada de 4,17 l (114%), volumen espiratorio forzado en el primer segundo FEV1% del 79% y prueba broncodilatadora
negativa. Se efectuó una prueba de metacolina, que fue negativa, con PC20 superior a 16 mg/ml (concentración de metacolina que
causó un descenso del FEV1 basal del 20%).

PREGUNTA
Considerando el estado actual que grado de asma presenta el caso?

RESPUESTA
a.- Intermitente.
b.- Leve persistente.

CURSO ENARM CMN SIGLO XXI TEL: 36246001 Pharmed Solutions Institute PÁGINA 38
MANUAL DE TRABAJO DEL CURSO ENARM CMN SIGLO XXI
c.- Moderado persistente.
d.- Intenso persistente.

PREGUNTA
Considerando la gravedad del cuadro, cual es el tratamiento mas adecuado para este caso?

RESPUESTA
a.- Esteroide inhalado dosis media (u otro antiinflamatorio a dosis alta), broncodilatador de acción larga.
b.- Esteroide inhalado dosis baja (u otro antiinflamatorio a dosis alta) + broncodilatador de larga acción.
c.- Agonista B, de larga acción (salmeterol, formoterol) + antiinflamatorios en baja dosis.
d.- Baja dosis de antiinflamatorios.

CASO CLINICO
Paciente de 63 años de edad. Diabetes mellitus tipo 2 diagnosticada hace un mes, en tratamiento. Exfumador. Sobrepeso (IMC:27).
Asma bronquial de diez años de evolución, en tratamiento combinado con un anticolinérgico de larga acción (bromuro de tiotropio), un
antagonista leucotriénico (Montelukast), corticoides orales e inhalados. Hernia hiatal. Estenosis aórtica leve degenerativa. Hiperplasia
benigna de próstata. Acude al Servicio de Urgencias por clínica de quince días de evolución con fiebre de predominio vespertino,
malestar general, sudoración profusa, astenia, pérdida de peso, vómitos, dolor epigástrico y somnolencia creciente además disuria,
polaquiuria, tenesmo o urgencia miccional. Destaca un episodio de odontalgia diez días antes en relación con absceso peridental,
resuelto con antibioterapia oral (amoxicilina-clavulánico). EF: TA de 89/57 mm Hg, temperatura 39,3ºC. El paciente se mostraba
consciente, orientado globalmente, con deshidratación cutáneo-mucosa y palidez cutánea, aunque con mucosas normocoloreadas.
Destaca un pequeño hematoma en la cara interna del brazo izquierdo. Auscultación cardíaca era rítmica, FC (120 1pm), FR (24 rpm),
hipoventilando en ambas bases pulmonares con hepatoesplenomegalia. Ligeros edemas en tercio distal de ambas piernas.

PREGUNTA
Considerando el estado actual que grado de asma presenta el caso?

RESPUESTA
a.- Intermitente.
b.- Leve persistente.
c.- Moderado persistente.
d.- Intenso persistente.

PREGUNTA
Considerando la gravedad del cuadro, cual es el tratamiento mas adecuado para este caso?

RESPUESTA
a.- Esteroide inhalado dosis media (u otro antiinflamatorio a dosis alta), broncodilatador de acción larga.
b.- Esteroide inhalado dosis baja (u otro antiinflamatorio a dosis alta) + broncodilatador de larga acción.
c.- Agonista B, de larga acción (salmeterol, formoterol) + antiinflamatorios en baja dosis.
d.- Baja dosis de antiinflamatorios.

PREGUNTA
En caso de requerir tratamiento de rescate, cual es la opción mas adecuada para el caso?

RESPUESTA
a.- Los agonistas B de corta acción, cuantas veces sea necesario.
b.- Los agonistas B de corta acción, cuantas veces sea necesario. El uso excesivo indica un control inadecuado.
c.- Los agonistas β2 de corta acción, y de larga acción cuantas veces sea necesario.
d.- Los agonistas β2 adrenérgicos, relajan el musculo liso (salbutamol) de corta acción, cuantas veces sea necesario.

CASO CLINICO
Varón de 40 años, diagnosticado de asma bronquial desde niño. Desde hace un año hace frecuentes visitas al servicio de urgencias del
hospital por disnea. Ha sido tratado con esteroides inhalados y orales, adrenérgicos beta-2 de corta y larga duración, y antibióticos. En
su historia actual cabe destacar que desde unos días antes de su ingreso en el hospital presentaba tos, disnea y fiebre. A la EF, disneico,
tórax hiperinsuflado y tiraje con disminución de ruidos respiratorios y presencia de lesiones cutáneas eritematosas y vesiculosas en pie
izquierdo, hombros y tórax. Las radiografías de tórax presentaron una condensación en el lóbulo superior derecho en dos focos y otra
en el lóbulo superior izquierdo menos densa. Leucocitos 15.000/µl (eosinófilos 31%, segmentados 44%). Según la espirometría, sus
valores de flujos espiratorios máximos en el primer segundo oscilaron entre 800 y 2.650 ml, es decir, entre el 23 y el 71% del valor
predicho. El servicio de dermatología informó una consulta sobre las lesiones cutáneas como probablemente secundarias a alergia
inespecífica. Se trató con esteroides orales a dosis medias decrecientes, mejorando de su clínica respiratoria y desapareciendo las
lesiones cutáneas y las alteraciones radiológicas, elaborándose el diagnóstico de probable SCS.

PREGUNTA
Considerando el estado actual que grado de asma presenta el caso?

CURSO ENARM CMN SIGLO XXI TEL: 36246001 Pharmed Solutions Institute PÁGINA 39
MANUAL DE TRABAJO DEL CURSO ENARM CMN SIGLO XXI
RESPUESTA
a.- Intermitente.
b.- Leve persistente.
c.- Moderado persistente.
d.- Intenso persistente.

PREGUNTA
Considerando la gravedad del cuadro, cual es el tratamiento mas adecuado para este caso?

RESPUESTA
a.- Esteroide inhalado dosis media (u otro antiinflamatorio a dosis alta), broncodilatador de acción larga.
b.- Esteroide inhalado dosis baja (u otro antiinflamatorio a dosis alta) + broncodilatador de larga acción.
c.- Agonista B, de larga acción (salmeterol, formoterol) + antiinflamatorios en baja dosis.
d.- Baja dosis de antiinflamatorios.

PREGUNTA
En caso de requerir tratamiento de rescate, cual es la opción mas adecuada para el caso?

RESPUESTA
a.- Los agonistas B de corta acción, cuantas veces sea necesario.
b.- Los agonistas B de corta acción, cuantas veces sea necesario. El uso excesivo indica un control inadecuado.
c.- Los agonistas β2 de corta acción, y de larga acción cuantas veces sea necesario.
d.- Los agonistas β2 adrenérgicos, relajan el musculo liso (salbutamol) de corta acción, cuantas veces sea necesario.

PREGUNTA
El paciente presenta datos sugerentes del síndrome de Churg-Strauss (SCS), cual de las siguientes aseveraciones no es correcta para el
diagnostico de SCS?

RESPUESTA
a.- Síndrome hipereosinofílico de origen desconocido.
b.- Caracterizado por una vasculitis necrosante con granulomas extravasculares.
c.- Infiltrados eosinófilos y de polimorfonucleares en la pared de vasos de mediano y pequeño calibre.
d.- Es muy poco frecuente, afecta sobre todo a menores de 30 años.

PREGUNTA
Clásicamente el SCS se presenta en 3 fases clínicas, cual no es una de ellas?

RESPUES
a.- Prodrómica (afección de vías aéreas altas y presencia de asma).
b.- Eosinofílica (eosinofilia periférica e infiltrados pulmonares).
c.- Vasculítica (afección sistémica).
d.- Necrotica (afeccion necrótica sistémica terminal).

PREGUNTA
Cual de los criterios para el diagnóstico establecidos por el American College of Rheumatology (ACR) para SCS no son ciertos?:

RESPUESTA
a.- Asma.
b.- Eosinofilia periférica mayor del 10%,
c.- Radiculopatias y Polineuritis.
d.- Infiltrados pulmonares transitorios.

NEUMONIA ADQUIRIDA EN LA COMUNIDAD (NAC)


CIENCIAS BASICAS: Definición: Proceso infeccioso agudo del parénquima pulmonar adquirido fuera del entorno hospitalario, se
relaciona con síntomas y signos pleuropulmonares que acompañan casi siempre a infiltrados recientes observados en una
radiografía de tórax. Factores de riesgo: Edad > 65 años, EPOC, tabaquismo, alcoholismo, Enf. cardiovascular, DM, hepatopatía e Insuf.
Renal, aspiración, exposición ambiental a aves, IVRS previas, Enf. Estructural pulmonar, Obstrucción de la vía aérea. Etiología:
Streptococcus pneumoniae 50-65% (>65 años, alcoholicos, EPOC); Haemophilus influenzae 8-10% (EPOC, fumadores); Staphylococcus
aureus 3-5% (abseso pulmonar), otros Klebsiella, Escherichia, Moraxella catharralis, Mycoplasma pneumoniae, legionella pneumophila,
pesudomona auregunosa, anaerobios (aspiración, obstrucción de vía aérea).
SALUD PUBLICA: Es una causa de frecuente de morbi-mortalidad en la población general. Incidencia de 2-10 casos/1000
habitantes/año. 20-35% requieren ingreso hospitalario. Mortalidad del 5% de hospitalizados y de un 35% ingresados a la UCI. La
mortalidad oscila entre un máximo del 61% para las NAC debidas a Pseudomonas y un 35% para las producidas por enterobacterias,
Staphylococcus aureus y las de etiología mixta. Las tasas de mortalidad por NAC la sitúan en el quinto lugar como causa más frecuente
de muerte en los países industrializados, tras las enfermedades cardiovasculares, neoplásicas y cerebrovasculares y la enfermedad
pulmonar obstructiva crónica (EPOC). 60-70% de todos los casos de NAC son originados por Streptococcus pneumoniae. En los últimos

CURSO ENARM CMN SIGLO XXI TEL: 36246001 Pharmed Solutions Institute PÁGINA 40
MANUAL DE TRABAJO DEL CURSO ENARM CMN SIGLO XXI
años se ha informado la aparición de Staphylococcus aureus resistente a la meticilina como causa de infecciones extrahospitalarias
graves. PATOGENIA: Vías de entrada: Aspiración de organismos que colonizan la orofaringe (mecanismo más común, sobre todo en
ancianos, alcoholicos, residentes de silos, DM; mas propensos a infecciones por gram -, provenientes del estómago, comida
contaminada), inhalación de aerosoles infecciosos, diseminación hematógena (por el embolismo séptico a partir de focos de infección
distantes, adictos a drogas parenterales, endocarditis bacteriana, colonización de catéteres intravenosos, S. aureus. patógeno más
frecuente), inoculación directa (es muy rara, el microorganismo traspasa las barreras). Hay factores que inhiben la actividad mucocoiliar
como: El humo del tabaco, el aire frío, fármacos (anestésicos), óxido de azufre, óxido de nitrógeno, fibrosis quística. DIAGNOSTICO:
Cuadro clínico: Tos con o sin esputo, escalofríos, fatiga, disnea y dolor torácico pleurítico, hemoptisis, mialgias, en caso de legionella;
puede existir sintomatología gastrointestinal. Exploración física: taquipnea, matidez a la percusión estertores y/o crepitancias, frémitos
y egofonía. Rx de tórax; neumonía típica (causada usualmente por Streptococcus pneumoniae, mas en niños y ancianos): consolidación
lobar y en neumonía atípica (influenzae, mycoplasma, legionella, no detectables en tinción de gram -, de ahí su nombre, más en
jóvenes) infiltrados difusos: de no observar nada repetir Rx a las 24-48 hrs. Los estudios especiales dirigidos a un agente como
hemocultivo, , cultivos de esputo, tinción gram, Ag urinario para legionellla y Streptococcus solo cuando: neumonía extensa que ingresa
a UCI, falta de respuesta al tx., infiltrados cavitarios, leucopenia, alcoholismo severo.
ESCALAS DE GRAVEDAD: Útiles para no sobreestimar la intensidad de los cuadros neumónicos y menor tasa de hospitalización.
Pneumonia Severity Index
ESCALA DE RIESGO Y MORTALIDAD DE PNEUMONIA SEVERITY INDEX
Características del paciente Puntaje
Puntaje total Riesgo Clase Mortalidad Lugar de Tx.
Características demográficas S/factores de riesgo Bajo I 0.1 Domicilio
Sexo masculino Edad en años <70 Bajo II 0.6 Domicilio
Sexo femenino Edad en años – 10 71-90 Bajo III 2.8 Intrahospitalario (24h)
Asilo +10 91-130 Moderado IV 8.2 Intrahospitalario
Comorbilidades >130 Severo V 29.2 UCI
Enf. Neoplásica +30
Enf. Hepática +20 CRITERIOS DE ADMISION A UCI; CRITERIOS DE NEUMONIA GRAVE
Criterios menores
Enf. Cardiaca +10 FR >30rpm o necesidad de apoyo ventilatorio
Enf. cerebrovascular +10 Indice de PaO2/FiO2 <250
Insuficiencia renal +10 Infiltrados multilobulares
Exploración física Confusión o desorientación
Alteración del estado mental +20 Uremia (BUN >20 mg/dl)
Leucopenia <4,000 Cualquier
FR > 30rpm +20 Trombocitopenia < 100,000
P.sistólica <90mmHg +20 mayor o 3
Hipotermia <35
Temp. <35°C o >40°C +15 Hipotensión menores
FC >125 lpm +10 Hipoglucemia en no diabéticos
Alcoholismo
Estudios de laboratorio y Rx
Hiponatremia
pH arterial <7.35 +30 Acidosis metabólica
BUN > 10.7 mmol/l +20 Cirrosis
Na < 130 mEq/l +20 Asplenia
Glucosa >250 mg/dl +10 Criterios mayores
Hto. <30% +10 Ventilación mecánica invasiva
Choque séptico que requiere vasopresores
Derrame pleural +10

TRATAMIENTO: En la mayoría de los casos el tx. es empírico ,pero debe ser activo frente a los patógenos más frecuentemente
implicados (S. pneumoniae). La duración del tratamiento minimo 5 días después de que el paciente permanezca por mas de 48-72 hrs
asintomáticos. 7-10 dias en caso de Streptococcus; 10-14 para Mycoplasma. Grupo I; pacientes sin historia de, enfermedad
cardiopulmonar y sin factores de riesgo; 1ra elección macrolidos, 2da elección doxicilina. Grupo II; pacientes con enfermedad
cardiopulmonar y con factores de riesgo; 1ra elección fluroquinolonas (moxifloxacino, levofloxacino) 2da. Elección B-lactamico
(cefotaxima, ceftriaxona, ampi/sulbactam). Grupo III; pacientes no admitidos a UCI que tienen lo siguiente: A: con enf.cardiopulmonar y
otros factores modificables y B: sin enf.cardiopulmonar y sin otros factores modificables; 1ra. Elección fluroquinolona, 2da. elección b-
lactamico+ macrolido. Grado IV: pacientes que ameritan ingreso a UCI. A: sin riesgo para pseudomona 1ra elección cefotaxima,
ceftriaxona+ azitromicna o fluoroquinolona. B: con riesgo para pseudomona; 1ra. Elección b- lactamico, antineumococo y
antipseudomona (piperacilina/tazobactam, cefepime, imipenem)+ ciprofloxacino o levofloxacino. 2da elección piperacilina/tazobactam,
cefepime + aminoglucosidos y azitromicina . Antibiotico recomendado para neumonía atípica (legionella, mycoplasma) es claritromicna.
PREVENCION: La primera medida es abandonar el habito tabáquico. Vacuna de neumococo se recomienda en >65 años, o en sujetos de
2-4 años con factores de riesgo; revacunación cada 5 años. Vacuna inactiva contra influenza; >65 años, 6-49 años con factores de
riesgo.; revacunación anual.

CASO CLINICO
Hombre de 68 años de edad, con historia de consumo de licor cada fin de semana hasta la embriaguez, cuatro días de tos con
expectoración purulenta, disnea de moderados esfuerzos, fiebre de 38,5°C, dolor tipo pleurítico en el hemitórax derecho y
automedicación con una dosis de dexametasona de 4 mg intramuscular dos días antes de la admisión. Ingresó febril, taquicárdico,
taquipneico, hipo-xémico y en la radiografía de tórax presentaba signos de derrame pleural y consolidación en la base derecha. La
evaluación de laboratorio muestra leucocitosis de 14.500 por mm3, neutrofilia de 96%, hiperglucemia de 638 mg/dl, hiperazoemia
(nitrógeno ureico en sangre de 27,45 mg/dl). El paciente fue hospitalizado con diagnóstico de neumonía grave extrahospitalaria IIIA,
según las guías nacionales de manejo de esta enfermedad.

PREGUNTA
¿Cuál es el agente etiológico más probable en este paciente?

RESPUESTA
a.- Mycoplasma pneumoniae
b.- Legionella pneumoniae
c.- Pseudomona

CURSO ENARM CMN SIGLO XXI TEL: 36246001 Pharmed Solutions Institute PÁGINA 41
MANUAL DE TRABAJO DEL CURSO ENARM CMN SIGLO XXI
d.- Strepococcus pneumoniae

PREGUNTA
¿Qué criterios de neumonía grave tiene este paciente, para ser tratado intrahospitalariamente?

RESPUESTA
a.- leucocitosis, derrame pleural, tx con dexametasona
b.- Hiperazoemia, hiperglucemia, taquipnea
c.- Hiperazoemia, leucocitosis, consolidación en base derecha
d.- Hiperglucemia, leucocitosis, tx. con dexametasona

CASO CLINICO
Hombre de 54 años de edad, con historia de consumo de licor cada fin de semana hasta la embriaguez, cuatro días de tos con
expectoración purulenta, disnea de moderados esfuerzos, fiebre de 38,5°C, dolor tipo pleurítico en el hemitórax derecho y
automedicación con una dosis de dexametasona de 4 mg intramuscular dos días antes de la admisión. Ingresó febril, taquicárdico,
taquipneico, hipoxémico y en la radiografía de tórax presentaba signos de derrame pleural y consolidación en la base derecha.
Leucocitosis de 14.500 por mm3, neutrofilia de 96%, hiperglucemia de 638 mg/dl, hiperazoemia (nitrógeno ureico en sangre de 27,45
mg/dl). El paciente progresó rápidamente a falla respiratoria hipoxémica, con choque séptico y, posteriormente, disfunción orgánica
múltiple.

PREGUNTA
Considerando el cuadro clínico, a que grupo pertenece para considerar tratamiento antibiótico?

RESPUESTA
a.- Grupo I
b.- Grupo II.
c.- Grupo III
d.- Grupo IV.

PREGUNTA
Considerando el Pneumonia Severity Index, que puntaje presenta el caso?

RESPUESTA
a.- <70
b.- 71-90
c.- 91-130
d.- >130

PREGUNTA
Considerando la escala de riesgo y mortalidad de pneumonia severity index que riesgo de motralidad presenta el caso?
RESPUESTA
a.- 0.6
b.- 2.8
c.- 8.2
d.- 29.2

NEUMONIA NOSOCOMIAL:
CIENCIAS BASICAS: Se desarrolla después de una estancia intrahospitalaria de 48hrs, algunos requerirán intubarse y pueden desarrollar
neumonía asociada al ventilador. SALUD PUBLICA: La neumonía es la 2da. Infección nosocomial más frecuente, con una mortalidad
asociada de 27-50%. Mas en niños, adultos >65años o con comorbilidades graves asociadas, o sometidas a cirugía toracoabdominal.
Debidas generalmente a la aspiración de secreciones orofaríngeas o de tracto respiratorio superior, contaminación de equipo de
terapia respiratoria. Las infecciones generalmente son polimicrobianas, con alto contenido de bacilos gram- ; pseudomona aeruginosa
(más frecuente) escherichia coli, klebsiella pneumoniae y Acinetobacter (55-85%), originan las neumonías nosocomiales, los gram + ,
son responsables del 20-30%; rara vez por hongos y virus (inmunodeprimidos). Recientemente ha emergido infecciones por
Staphylococcus aureus, particularmente resistente a meticilina, más frecuente en DM, y pacientes de UCI. DIAGNÓSTICO: La sospecha
dx. surge cuando el paciente presenta infiltrados radiográficos nuevos y/o progresivos, en conjunto con dos o más hallazgos clínicos
sugerentes de infección (fiebre de recién aparición >38°C, leucocitosis o leucopenia, esputo purulento). Rx de tórax; desde infiltrados,
consolidación, derrames, cavitaciones. Para dx. etológico: cultivos, hemocultivos, identificación de legionella requiere
inmunofluorescencia. Si hay derrame pleural de gran tamaño y/o datos de sepsis, realizar toracocentesis dx. para descartar empiema o
derrame paraneumonico. TRATAMIENTO: Inicio empírico; 1. Esquema de espectro limitado (menos de 5 días hospitalizado y sin
factores de riesgo para bacterias multiresistente) Ceftriaxona, levofloxacino, ampi/sulbactam, ertapenem. 2. Esquema de espectro
amplio (>5 dias hospitalizado, factores de riesgo para multiresistentes). Esquema 1: a.- cefalosporina o carbapenem antiseudomonica
(cefepime, ceftazidima) (imipenem, meropenem) o un b lactamico + un inhibidor de b- lactamasas (piperacilina/tazobactam). b.-
fluroquinolona antiseudomonica (cipro o levofloxacino). Esquema 2: a.- aminoglucocido. b.- linezolid o vancomicina. La mejoría clínica
se observa dentro de 48-72 hrs, posteriores al inicio del antibiótico, por esta razón la terapia antibiotica, no debera de ser modificada

CURSO ENARM CMN SIGLO XXI TEL: 36246001 Pharmed Solutions Institute PÁGINA 42
MANUAL DE TRABAJO DEL CURSO ENARM CMN SIGLO XXI
hasta después de 72 hrs. L a duración deberá individualizarse, para la neumonía por bacterias gram - usualmente es de 14-21 días.
PREVENCION: Control estricto de infecciones, desinfección de manos con soluciones etílicas, vigilancia microbiológica, monitoreo y
retiro temprano de dispositivos invasivos y evitar prácticas erróneas en prescripción de antibióticos, métodos de ventilación no
invasivos.

CASOS CLINICOS
Varón de 34 años, trabajador en la hostelería, fumador y bebedor importante (con dependencia alcohólica). Fue traído a urgencias por
un traumatismo craneoencefálico con hematoma subdural. El paciente ingresó en la unidad de cuidados intensivos (UCI) intubado y
conectado a ventilación mecánica, y recibió tratamiento antibiótico empírico con amoxicilina-ácido clavulánico. La cifra de leucocitos en
sangre era de 12,05 × 103/l. En el primer día del ingreso se realizó un broncoaspirado (BAS). En el cultivo creció una flora mixta
respiratoria. A los 8 días del ingreso presentó fiebre y secreciones purulentas espesas, y en la radiografía de tórax se objetivó una
neumonía en el lóbulo inferior izquierdo complicada con atelectasias bibasales. La concentración de leucocitos en sangre era en ese
momento de 4,10 × 103/l.

PREGUNTA
Cual es el agente etiológico mas probable para este caso?

RESPUESTA
a.- Pseudomona aeruginosa.
b.- Escherichia coli.
c.- klebsiella pneumoniae.
d.- Acinetobacter.

PREGUNTA
Cual es el tratamiento de primera intensión en este caso?

RESPUESTA
a.- Ceftriaxona, levofloxacino.
b.- Cefepime, ceftazidima.
c.- Imipenem, meropenem.
d.- Piperacilina/tazobactam

NEUMOCONIOSIS
CIENCIAS BASICAS: Definición: Enfermedad producida por acumulación de polvos minerales secundaria a la inhalación crónica de los
mismos. Los materiales mas implicados son: asbesto, sílice, carbón, berilio, hierro, el daño tisular que provocan depende de; tamaño,
tiempo e intensidad de exposición, estado inmunológico, relación inhalación/ aclaramiento ciliar pulmonar. ASBESTOSIS: Hay 3 tipos de
asbesto: amosita, asbesto azul (cocidolite) y asbesto blanco (crisolito, el 90%del consumo de asbesto es bajo esta forma, es mas soluble
y tiende a fragmentarse). Las formas en las que se inhalan las fibras de asbesto es en anfibole y serpentinas. PATOGENIA: Exposición
directa, cuando se trabaja con materiales o en lugares donde hay asbestos. Exposición indirecta, cuando se vive o trabajo cercano a un
lugar de concentración de asbesto. Lo podemos encontrar en materiales para pulir piedra preciosa, afiladores de piedra, resistencias de
algunos electrodomésticos y algunos plásticos. La enfermedad se puede presentar como: placas pleurales benignas 40%, enf., maligna
20% (mesotelioma maligno), asbestosis como tl 40%. Una vez inhalada las partículas, llegan a los espacios alveolares y son fagocitados
por lo macrófagos, si son menor a 3 µm, si son mayores son fagocitadas incompletamente ( cuerpos de asbesto)e, ya fagocitadas son
trasportadas a los ganglios y después a las pleuras, donde se depositan para formar las placas pleurales, los cuerpos de asbesto se
acumulan de manera progresiva en espacios aéreos e intersticio, las células afectadas liberan citocinas, fibronectina y colágena,
causando así migración de otras células de defensa y favoreciendo proliferación de fibroblastos, provocando zonas de regeneración
parenquimatosa, provocando la asbestosis, es decir la fibrosis pulmonar afecta mas zonas inferiores y regiones subpleurales.
DIAGNÓSTICO: Relación clara entre exposición de asbesto y fibrosis pulmonar. El consumo de tabaco aumenta el riesgo de asbestosis.
Cuadro clínico: lo primero es disnea, la cual se va haciendo progresiva, después tos seca pertinaz, pueden auscultarse estertores
crepitantes basales, a veces hipocratismo. Rx.: opacidades irregulares que al principio de la enfermedad son de localización periférica y
basal, hiperclaridad alrededor de silueta cardiaca. TAC: se puede ver fibrosis, como vidrio despulido, hasta lesiones fibroticas con
engrosamiento de los septos interlobares, engrosamiento de pleura viscerales. El patrón funcional observado en pacientes con
asbestosis es restrictivo. Se puede realizar lavado bronquioalveolar. SILICOSIS: Provocada por inhalación de cristales de sílice, en la
mayoría en sus formas de cuarzo, las actividades de riesgo son la construcción, la fundición, demolición o reparación de estructuras de
concreto, taladrar piedras. PATOGENIA: Hay silicosis aguda, donde se observa proteinosis y la crónica la mas frecuente, donde se
observan nódulos con centro fibrotico, rodeado de parénquima con cúmulos de sílice, que ha sido fagocitado por macrófagos. En la
forma complicada llamada fibrosis masiva progresiva, se observan nódulos confluentes, que se localizan principalmente en lóbulos
superiores, que se relacionan con una pobre calidad de vida. La silicosis se relaciona con una mayor susceptibilidad a Tb, artritis
reumatoide, esclerodermia, lupus y enf., renal progresiva. DIAGNÓSTICO: Cuadro clínico: síntoma principal disnea, la cual es progresiva,
raros otros síntomas. Rx: pequeñas opacidades redondeadas, con distribución casi siempre bilateral, de localización en lóbulos
superiores, puede haber crecimiento de adenopatías hiliares, las cuales pueden calcificarse en una forma característica conocida como
"cascara de huevo". Funcionalmente la silicosis se comporta con limitación al flujo aéreo y con reducción de la capacidad de difusión.
ANTRACOSIS: Provocada por inhalación crónica de carbón masiva, la forma mas común es el antracite. Existen 2 formas principales: la
mácula o nódulo antracotico y fibrosis pulmonar masiva. PATOGENIA: En la antracosis hay mayor susceptibilidad a presentar artritis
reumatoide, y cuando se presentan ambas condiciones, se conoce como Sx., de Caplan, en el cual debe haber un nódulo reumatoideo,
que en algunas condiciones puede cavitarse debido a la necrosis central o bien calcificarse. DIAGNÓSTICO: La macula o mancha de

CURSO ENARM CMN SIGLO XXI TEL: 36246001 Pharmed Solutions Institute PÁGINA 43
MANUAL DE TRABAJO DEL CURSO ENARM CMN SIGLO XXI
carbón suele ser una lesión de 5cm que puede observarse principalmente en regiones apicales, en ocasiones rodeadas de parénquima
que muestra enfisema centroacinar. Existen tantas neumoconiosis como diversidad de polvos minerales y son una causa importante de
morbimortalidad a nivel mundial.

CASOS CLINICOS
70 años, minero de minas carbón jubilado, fumador de 30 cigarrillos/día hasta hace 10 años, bebedor de ¼ l de vino al día y 2 cervezas.
HTA en tratamiento con diuréticos y dieta hiposalina. Hipercolesterolemia en tratamiento con dieta. Desde hace 25 años presenta tos y
expectoración crónica de predominio matutino. Desde hace 15 años se asocia disnea de esfuerzo progresiva con aumento de la disnea
en relación con las infecciones respiratorias. Presenta con frecuencia expectoración de color negro. Paciente que acude al servicio de
urgencias por un cuadro de aumento de su tos habitual, aumento de su expectoración habitual, siendo actualmente mucopurulenta y
aumento de la disnea que se hizo de reposo. Desde hace una semana nota que se le hinchan los tobillos. Paciente consciente,
orientado. Cianótico. Taquipneico (30 respiraciones por minuto). Hepatomegalia blanda y no dolorosa de 2 cm. Ingurgitación yugular
+++. Edemas maleolares ++++. Tonos cardiacos rítmicos a 130 por minuto. Roncus y sibilantes diseminados por ambos campos
pulmonares.TA: 160/100. Hb 16, Hto 49, leucocitos 13500 con aumento de neutrófilos en la formula leucocitaria. P02 52 PC02 40, PH
7.35. RX de torax: Signos de atrapamientoretroesternal y retrocardiaco, engrosamientos peribronquiales. 3 imágenes de aumento de
densidad nodulares de 3-7 cmde diámetro en Lóbulo superior derecho y 2 en Lóbulo superior izquierdo.

PREGUNTA
Se realizaron pruebas funcionales respiratorias con los siguientes resultados FVC 60%, FEV1 30%, cual es la FEV1-FVC?

RESPUESTA
a.- 40
b.- 50
c.- 60
d.- 30

HIPERTENSION PULMONAR PRIMARIA (HAP)


CIENCIAS BASICAS: Definición: Entidad que afecta a la circulación CAUSAS DE HIPERTENSION PULMONAR
pulmonar a nivel de las pequeñas arterias y arteriolar, caracterizada por Hipertensión arterial pulmonar: Hipertensión pulmonar primaria (idiopatica
una proliferación fibromuscular y remodelación vascular endotelial, que o familiar). Enfermedades vasculares de la colágena (esclerodermia, artritis,
lupus). Cortocircuitos congénitos sistémico-pulmonares (CIV, CIA, PCA).
da lugar a estenosis de la luz vascular. Hemodinámicamente se define Hipertensión portal. Infección por VIH. Fármacos anorexigenicos
como una presión arterial pulmonar media > 25mmHg en reposo o de 30 (fenfluramina, fentermina). Otros: Enf. De depósito de glucógeno, de
mmHg durante el ejercicio, con la adición de una presión capilar Gaucher, hemoglobinopatías,
Hipertensión venosa pulmonar: EPOC. Enfermedad pulmonar intersticial
pulmonar < 15mmHg, con RVP elevada. SALUD PUBLICA: Incidencia difusa. Apnea del sueño. Enfermedad con hipotensión alveolar crónica
anual es de 1-2 casos por 1000 000, con edad media de diagnóstico de Tromboembolia pulmonar: Embolismo pulmonar agudo. Embolismo
36 años (4ta-5ta décadas de la vida). La HAP primaria (idiopática) 30- pulmonar crónico.
40%; predomina en la mujer. Los casos de HAP familiar representan el
10%. La HAP asociada a fenómeno de Eisenmenger (cortocircuito der-
izq), representa 30-35%. La hipertensión es la causa más común de cor pulmonale. PATOGENIA: Se han demostrado 3 factores
trascendentales que incrementan las resistencias vasculares pulmonares: a) vasoconstricción sostenida, b) proliferación vascular
pulmonar, c) trombosis in situ, condicionando así una arteriolopatía obstructiva, dificultando flujo sanguíneo, lo que eleva la presión en
las arterias pulmonares, generando demasiada tensión en el VD, hipertrofiándolo y llevándolo a insuficiencia. La disfunción endotelial
juega un rol pivote y trascendental, sobre todo por el imbalance entre las sustancias vasodilatadoras (óxido nítrico, prostaciclina, PG- 1-
2, péptido intestinal vasoactivo) y vasoconstrictoras (Tx A-2, endotelina-1, serotonina, factor de crecimiento derivado de cel.
endoteliales). En HAP familiar, se encuentran alteraciones en los receptores que codifican para los factores transformadores de
crecimiento tipo beta ((BMPR-II), que regulan la angiogénesis y apoptosis celular. DIAGNÓSTICO: Cuadro clínico: El síntoma más
frecuente y más temprano es la disnea progresiva 60%, acompañada de fatiga, presíncope o síncope, edema de miembros inferiores,
cianosis %20, angina, así como dolor precordial secundario a isquemia del VD. Puede haber ingurgitación yugular, por contractilidad
aumentada de la de la AD, por falla ventricular derecha crónica. Auscultación: segundo ruido, en su componente pulmonar desdoblado
o acentuado, datos de regurgitación tricúspidea, así como la presencia de galope sobre el precordio correspondiente al VD. El
diagnostico de certezas solo puede establecerse, según la definición mediante el cateterismo derecho. ECG, tele de tórax,
ecocardiografía transtoracica (valorar afección valvular o miocárdica como pb. causa, así como cortocircuitos izq-der). Pruebas de
función respiratoria para determinar neumopatías. El gamagrama pulmonar ventilatorio-perfusorio es estudio clave para detectar HAP
tromboembolica crónica, confirmación dx. con angiografía pulmonar. TRATAMIENTO: Si está limitada la actividad física usar diuréticos,
para edema periférico, O2 suplementario si esta reducida la PO2 y anticoagulación crónica con warfarina (objetivo INR= 2.0-3.0). Como
los vasoconstrictores juegan un papel importante, el tx. con vasodilatadores se basa en tales procesos, pero no se puede predecir quien
responderá sin resistencia a los vasodilatadores orales como a los antagonistas de canales de calcio, ya que estos mejoran la sobrevida.
Mediante una prueba se puede predecir que pacientes tendrán respuesta sostenida a dichos agentes, se define una prueba positiva,
cuando al administrar vasodilatadores endovenosos de acción corta como adenosina o epoprostenol, existe una disminución de
10mmHg o mayor en la presión arterial pulmonar media o como un descenso en más de 25% de las resistencias vasculares pulmonares
sin presentar variación en la presión arterial sistémica, solo responden 7-10% a dicho reto, habrá que monitorizar hipotensión o
empeoramiento de la insf. Cardiaca der. Otros fármacos son los prostanoides, son sustancias tanto vasodilatadoras como
antiproliferativas, usados por varias vías, epoprostenol endovenoso, iloprost inhalado, treprostinil subcutáneo, el mecanismo de acción
radica en estimular la producción de AMP cíclico como segundo mensajero induciendo vasodilatación. En pacientes con hipertensión
arterial pulmonar existe un déficit relativo de óxido nítrico, por lo que los inhibidores de fosfodiesterasas tipo 5, al prolongar la vida
media de los segundos mensajeros GMPc, prolongan el efecto vasodilatador y antiproliferativo del óxido nítrico, como es el caso del

CURSO ENARM CMN SIGLO XXI TEL: 36246001 Pharmed Solutions Institute PÁGINA 44
MANUAL DE TRABAJO DEL CURSO ENARM CMN SIGLO XXI
sildenafil, el cual es de 1ra línea en el tx. por vía oral de la hipertensión arterial pulmonar. Si persiste la Insf. Cardiaca derecha, debe
considerarse el trasplante pulmonar bilateral. PRONÓSTICO: Deletéreo y muy pobre. Sobrevida de 2.5 años a partir del dx. sin
tratamiento. Supervivencia a 5 años de 75-85% en pacientes con tx. Adecuado y adyuvante con combinación o sinergismo de fármacos
con diferentes mecanismos de acción. Las variables más importantes para establecer pronóstico son presión media de la arteria
pulmonar, presión de AD y GC.

CASO CLINICO
Paciente varón de 58 años de edad, ex fumador (con una dosis acumulada de 20 paquetes-año) y ex enolismo moderado, y con
antecedentes personales de hipertensión arterial sistémica, linfangiectasias duodenales, epistaxis de repetición y cirrosis hepática
enólica ChildA (MELD13) con un único episodio de hemorragia digestiva alta por sangrado de varices esofágicas. Tiene 2 hijas, una de
las cuales refería episodios de epistaxis de repetición. Ingresó por clínica de infección respiratoria e insuficiencia respiratoria aguda con
una saturación de O2 de 77% (FIO20,21). En la radiografía de tórax se observó un infiltrado en la base derecha que se interpretó como
una probable neumonía adquirida en la comunidad y se inició tratamiento antibiótico con fluoroquinolonas (levofloxacino).

PREGUNTA
Ante la persistencia de hipoxemia severa refractaria cual es la conducta mas adecuada?

RESPUESTA
a.- Cambiar esquema antibiótico.
b.- Realizar TAC.
c.- Realizar ecocardiograma doppler.
d.- Realizar gamagrafia.

CASO CLINICO
Mujer de 40 años, con xeroftalmia de 5 años de evolución a la que se habían añadido en los últimos 6 meses xerostomía y artralgias, sin
otras manifestaciones extraglandulares. Las pruebas complementarias nos permitieron hacer el diagnóstico de SS: ANA positivo, anti-
SSa positivo, anti-SSb positivo, test de Shirmer anormal, datos de inflamación crónica en biopsia de glándula salival menor y
gammagrafía de glándulas salivales compatible. Se realizó ecocardiograma de rutina donde apareció por primera vez datos de HTP
ligera (32 mm Hg) que se confirmó con cateterismo derecho. La paciente presentó disnea de esfuerzo. En ecocardiografía se estimó una
HTP de 46 mm Hg. Mediante TC torácica se descartó la existencia de fibrosis pulmonar y eventos tromboembólicos. Actualmente
persisten artralgias que ceden con analgésicos habituales y su disnea está estable.

PREGUNTA
Cual de las siguientes afirmaciones es la incorrecta, respecto a la HTP del caso?

RESPUESTA
a.- En asociación con enfermedades del tejido conectivo CREST.
b.- En asociación con LES
c.- Enfermedad mixta del tejido conectivo.
d.- La asociación con SS es común.

CASO CLINICO
Un hombre de 33 años fue admitido, con disnea reciente y progresiva, edema y pérdida de peso de aproximadamente 10 kg. Dos
hermanas a los 11 y 14 años de edad. Al EF, parecía "débil" (subnutrido) (peso 60 Kg; altura 1,82 m), presentaba hepatomegalia leve y
edema de miembros inferiores. Un sonido alto de cierre de la válvula pulmonar era oído en el precordio. El examen de los pulmones
reveló ronquidos leves y estertores finos en ambos lados, que desaparecieron completamente después de la administración de
diuréticos. Los datos de la cateterización cardíaca derecha fueron compatibles con hipertensión arterial pulmonar (HAP). Óxido nítrico
inhalado y sildenafila fueron eficaces en la reducción de la resistencia vascular pulmonar. Una acentuada elevación en la presión de
enmentoencuñamiento pulmonar fue registrada durante la inhalación del óxido nítrico.

PREGUNTA
Cual es la conducta adecuada para establecer un diagnostico definitivo?

RESPUESTA
a.- Gamagrama pulmonar.
b.- Ecodoppler pulmonar.
c.- Biopsia pulmonar abierta.
d.- Tomografia axial computada.

CASO CLINICO
Mujer de 20 años, de profesión camarera, que consultó por cuadro de disnea, edema en el miembro superior derecho y síncope. Tenía
antecedentes de ser fumadora activa de 15 cigarrillos/día y padecer de rinoconjuntivitis y asma bronquial. La paciente comenzó un mes
previo con disnea a moderados esfuerzos. Cuando consultó en urgencias, presentaba disnea de reposo, debilidad, astenia y refería
haber tenido un síncope. A la exploración presentaba PA de 120/80mmHg, FC de 110 lat/min y SatO2 del 95% sin O2 suplementario. Se
apreciaba aumento del perímetro del miembro superior derecho, doloroso a la palpación y sin signos de empastamiento. La analítica

CURSO ENARM CMN SIGLO XXI TEL: 36246001 Pharmed Solutions Institute PÁGINA 45
MANUAL DE TRABAJO DEL CURSO ENARM CMN SIGLO XXI
presentaba perfiles bioquímico, hepático, renal, hemograma y coagulación dentro de la normalidad, excepto los dímeros D (2.500
ng/ml).

PREGUNTA
Cual es la conducta a seguir mas adecuada, para confirmar el diagnostico para el caso actual?

RESPUESTA
a.- Realizar rx de torax.
b.- Realizar TAC de torax.
c.- Realizar Angio-TAC de torax.
d.- Realizar IRM de torax.

COR PULMONALE
Enfermedad cardíaca pulmonar, y se define como una dilatación y/o hipertrofia del ventrículo derecho a consecuencia de problemas en
la vasculatura o el parénquima pulmonar y puede conducir a insuficiencia cardíaca derecha. El término se usa para describir cambios en
la estructura y función del ventrículo derecho como resultado de un desorden respiratorio que produzca hipertensión pulmonar. La
hipertrofia del ventrículo derecho es el cambio principal en el cor pulmonale crónico y dilatación del ventrículo en los casos agudos,
ambos consecuencia de un incremento de presión en el ventrículo derecho del corazón. Sin tratamiento, el cor pulmonale puede causar
insuficiencia cardíaca derecha y muerte. ETIOLOGÍA: Para que una alteración del corazón derecho sea clasificado como cor pulmonale,
el origen de la anomalía debe encontrarse bien en el sistema de circulación pulmonar o bien en el parénquima pulmonar,
produciéndose en cualquier caso una hipertensión arterial pulmonar (HTAP). Las causas principales de HTAP son: enfermedades que
producen la oclusión de la red vascular pulmonar: tromboembolismo pulmonar recurrente, hipertensión pulmonar primaria,
enfemedad veno-oclusiva, enfermedad del colágeno vascular o enfermedades pulmonares inducidas por drogas; enfermedades que
producen vasoconstricción pulmonar hipóxica crónica, como ocurre en la bronquitis crónica, la enfermedad pulmonar obstructiva
crónica (EPOC), la fibrosis quística, la hipoventilación crónica (que tiene lugar en la obesidad, las enfermedades neuromusculares como
la distrofia muscular de Duchenne o la disfunción de la pared torácica), o en las personas que viven en altitud. Enfermedades que
producen alteraciones del parénquima pulmonar, como ocurre en la bronquitis crónica, la EPOC, la bronquiectasia, la fibrosis quística,
las neumoconiosis, la sarcoidosis o la fibrosis pulmonar idiopática. El compromiso del ventrículo derecho causado por un defecto
sistémico, diferente al sistema respiratorio no es considerado cor pulmonale, debe ser de origen pulmonar subyacente. La prevalencia
del cor pulmonale es difícil de definir, porque no todos los casos de enfermedad pulmonar desarrollan cor pulmonale, la capacidad de
diagnosticar la HTAP mediante el examen físico de rutina es baja y los tests de laboratorio son relativamente poco sensibles. Sin
embargo, avances recientes en imaginería con ecografía Doppler y biomarcadores hace más fácil el diagnóstico del cor pulmonale.
Según la agudeza y la severidad del estímulo que genera la disfunción cardíaca, se puede distinguir: Cor pulmonale agudo: ocurre tras
un estímulo repentino y severo, que produce dilatación y fallo del ventrículo derecho, pero no hipertrofia; puede ser producido por:
Embolismo pulmonar masivo; Maligno: carcinomatosis miliar, linfangitis carcinomatosa. Empeoramiento del cor pulmonale crónico. Cor
pulmonale crónico: se produce a causa de un aumento progresivo y lento de la HTAP, que conduce a la hipertrofia y dilatación del
ventrículo derecho; puede generarse a causa de: Enfermedad obstructiva crónica (EPOC), como bronquitis crónica y enfisema, en la que
la alteración de la estructura alveolar y la consecuente insuficiencia respiratoria representa el inicio del cor pulmonale. Pérdida de
tejido pulmonar por razones traumáticas o quirúrgicas. Pneumocistitis terminal. Escoliosis, especialmente con desviación severa de la
columna lo cual interfiere con el desarrollo normal de los pulmones. Dicha restricción respiratoria causa hipoxemia, hipercapnia con
subsecuente vasoconstricción pulmonar e hipertrofia del ventrículo derecho. Sarcoidosis por fibrosis del parenquima pulmonar en una
pequeña porción de los pacientes (apróx. 5%). FISIOPATOLOGÍA: Como se indica en el apartado "Etiología", existen muchas patologías
diferentes que pueden producir cor pulmonale, pero todas ellas tienen en común la generación de hipertensión arterial pulmonar
(HTAP), suficiente para generar la dilatación del ventrículo derecho, con o sin hipertrofia asociada. Anatómicamente, el ventrículo
derecho tiene una pared delgada, con una compliancia elevada, por lo que está mejor preparado para adaptarse a variaciones de
volumen que a variaciones de presión. Por ello, cuando se produce un aumento de la presión pulmonar (HTAP) y un aumento de la
resistencia vascular pulmonar, el ventrículo derecho no puede desarrollar la fuerza suficiente como para superar dicho aumento, por lo
cual se dilata (en los casos agudos) o se hipertrofia (en los casos crónicos), pudiendo llegar a fallar. La severidad de la HTAP y el fallo del
ventrículo derecho están influidos por múltiples factores que pueden producirse de manera intermitente, como hipoxia secundaria a
alteraciones en el intercambio gaseoso, hipercapnia y acidosis, además de cambios en la sobrecarga de volumen del ventrículo derecho
que se producen con el ejercicio, con el aumento del ritmo cardíaco, la policitemia o la retención de sales debido a una disminución del
gasto cardíaco. MANIFESTACIONES CLÍNICAS: Síntomas: Generalmente están asociados al problema pulmonar de fondo. El síntoma
más común es disnea (dificultad para respirar), debido a que hay un aumento del trabajo necesario para respirar, a consecuencia de los
cambios en la elasticidad del pulmón (como ocurre en la fibrosis) o a modificaciones en la mecánica respiratoria (como sucede en la
EPOC), y ambos pueden verse agravados por un incremento de la hipoxemia. La hipoxemia pulmonar puede generarse por una
disminución de la permeabilidad de los capilares, por desequilibrios ventilación-perfusión o por la presencia de un shunt cardíaco o
pulmonar. La presencia de ortopnea o disnea paroxística nocturna son raramente síntomas aislados de fallo cardíaco derecho. Pero si
se presentan, indican que hay un aumento del trabajo respiratorio en el decubito. También puede presentarse tos o síncope en
pacientes con cor pulmonale con HTAP severa, debido a la incapacidad del corazón derecho de bombear sangre a través de los
pulmones hacia el corazón izquierdo. También puede presentarse dolor abdominal y ascitis, así como edema de las extremidades
inferiores. Signos: Muchos de los signos que se encuentran en los pacientes con cor pulmonale también se encuentran en pacientes con
una fracción de eyección reducida: taquipnea, presión venosa yugular elevada, hepatomegalia y edema en las extremidades inferiores.
Además, el ventrículo derecho puede palparse a lo largo del borde izquierdo del esternón o en el epigastrio. En el cor pulmonale, la
aparición de cianosis es un evento tardío, secundario a una reducción del gasto cardíaco con vasoconstricción sistémica y desequilibrio
ventilación-perfusión en el pulmón. EDEMA DE PULMÓN AGUDO: La causa más común del edema pulmonar agudo es el aumento de la
presión capilar (desequilibrio de las fuerzas de Starling) en la microvasculatura pulmonar, causando fuga plasmática hacia el espacio

CURSO ENARM CMN SIGLO XXI TEL: 36246001 Pharmed Solutions Institute PÁGINA 46
MANUAL DE TRABAJO DEL CURSO ENARM CMN SIGLO XXI
intersticial. Cualquier evento que curse con hipertensión venosa pulmonar ocasionará una congestión venosa y un aumentado volumen
de sobrecarga al ventrículo derecho. La incapacidad del ventrículo derecho de expandir para adaptarse a ese incrementado volumen
aumenta aún más el volumen venoso, y por ende retrógrada, la presión capilar. La fuga de plasma por una mayor presión hidrostática
en el capilar pulmonar que en el espacio intersticial acumula líquido en el parénquima pulmonar. La suma de estas anormalidades en el
pulmón y su circulación vascular es la complicación más importante producida por el edema pulmonar en la instalación del cor
pulmonale. COMPLICACIONES: El cor pulmonale retrasa el flujo de la sangre desde la circulación venosa hacia la arterial. En la
hiperemia, la sangre se acumula en el sistema venoso, incluyendo la vena hepática. La congestión prolongada de sangre en la región
centro-lobulillar del hígado conlleva a hipoxia y cambios grasos de los hepatocitos periféricos, produciendo agrandamiento voluminoso
del hígado, llegando a tener hasta el doble del volumen de sangre. El aspecto cianótico combinado con islotes grasos amarillentos le da
la apariencia al hígado de nuez moscada. Primero el hígado se congestiona, aumenta de tamaño, se vuelve cianótico y los sinusoides, en
especial en la zona centrolobulillar, se repletan de sangre. Comienza también una infiltración grasosa que termina por darle el tinte
amarillento final. De persistir la deficiencia de la circulación pulmonar, se instala en las trabéculas del hígado una atrofia que suele ser
cianótica. Con el tiempo se agrega una acumulación fibrótica (cicatrización) que tiende a reducir al hígado de tamaño y a endurecerlo.
Un proceso patológico similar ocurre en los riñones y el bazo: congestión sanguínea, atrofia cianótica y endurecimiento. En los
pulmones, la hiperemia produce ingurgitación capilar, edema en el intersticio pulmonar y, como consecuencia, fibrosis entre los
alvéolos y la matriz capilar, traduciéndose en trastornos de la relación perfusión y ventilación pulmonar. El pulmón adopta una
consistencia dura y (macroscópicamente) de color "rojo morena". El cor pulmonale puede también llevar a insuficiencia cardíaca
congestiva (ICC), empeorando la respiración por razón del edema pulmonar, hinchazón de las piernas debido a edema en la periferia y
hepatomegalia congestiva y dolorosa. La ICC es un indicador negativo en el pronóstico del cor pulmonale. DIAGNÓSTICO: La causa
principal de fallo cardíaco derecho no es la disfunción de la vasculatura o el parénquima pulmonar, sino el fallo cardíaco izquierdo. Por
ello es importante evaluar la posible disfunción del ventrículo izquierdo en un paciente con síntomas de fallo cardiaco derecho. Varios
exámenes son de utilidad para el diagnóstico del cor pulmonale: Electrocardiograma (ECG): los pacientes con HTAP severa muestran
una onda P pulmonar, una desviación del eje derecho e hipertrofia del ventrículo derecho; Ecocardiograma: útil para determinar el
grosor del ventrículo derecho y las dimensiones de las cámaras cardíacas, así como la anatomía de las válvulas; Ecografía Doppler, para
verificar la presión arterial pulmonar; Radiografía de tórax, que puede mostrar un agrandamiento de la arteria pulmonar principal, los
vasos del hilio pulmonar y la arteria pulmonar descendente derecha; TAC del tórax, útil en el diagnóstico de tromboembolia pulmonar
aguda; además, es el método más fiable para diagnosticar enfisema y enfermedad intersticial pulmonar; Pruebas de función pulmonar;
Gases sanguíneos; Cateterización de Swan-Ganz: el cateterismo del ventrículo derecho permite confirmar el diagnóstico de HTAP y
excluir la presión elevada del ventrículo izquierdo como causa del fallo del ventrículo derecho. TRATAMIENTO: La eliminación de la
causa es la intervención más importante. En el embolismo pulmonar, se apela a trombolisis (disolución enzimática del coágulo
sanguíneo), en particular si hay trastornos del ventrículo derecho. En la EPOC, la terapia con oxígeno a larga duración puede mejorar el
cor pulmonale. Los principios generales de tratamiento del cor pulmonale incluyen la reducción del trabajo respiratorio, mediante
ventilación mecánica no invasiva, broncodilatadores y esteroides, además de tratar las infecciones subyacentes. Una oxigenación
adecuada (saturación de oxígeno 90-92%) también permite reducir la resistencia vascular pulmonar y reducir la demanda sobre el
ventrículo derecho. Los pacientes anémicos necesitan una transfusión y si el hematocrito excede 65% debe efectuarse una flebotomía
para reducir la HTAP. El edema requiere diuréticos (reduciendo el sobreuso del corazón), a veces nitratos para mejorar el flujo
sanguíneo, inhibidores de la fosfodiesterasa como sildenafil y tadalafil, y ocasionalmente fármacos ionotropos para mejorar la
contractilidad del corazón. Otros medicamentos ayudan a mejorar la hipertensión pulmonar, tales como la prostaciclina o sus análogos.
Asimismo se han prescrito anticoagulantes y bloqueadores de los canales de calcio para las etapas iniciales del trastorno.

CASO CLINICO
Mujer de 48 años, exfumadora de 10 paquetes/año, diagnosticada de síndrome de CREST e hipertensión pulmonar secundaria en
seguimiento en consulta de Neumología desde hace seis años. En la actualidad, se encuentra en grado funcional III de la NYHA y está en
tratamiento con acenocumarol, sildenafilo, treprostinil subcutáneo y oxigenoterapia domiciliaria. Desde hace dos meses, presenta
disnea de mínimos esfuerzos (grado funcional IV de la NYHA) y edemas en miembros inferiores, por lo que se añadió furosemida y
espironolactona al esquema terapéutico. Hace una semana, comenzó con disnea de reposo y aumento de los edemas, por lo que se
decidió ingreso hospitalario. A la exploración estaba afebril, taquicárdica y taquipneica. A la auscultación cardiaca presentaba un soplo
sistólico en foco pulmonar con refuerzo del segundo tono y en la auscultación pulmonar, crepitantes finos bibasales. En la exploración
del abdomen se palpaba ascitis. En miembros inferiores presentaba edemas con fóvea hasta rodillas. Se realizó analítica sin alteraciones
significativas y radiografía de tórax con aumento del índice cardiotorácico e infiltrado intersticial bilateral. En el ecocardiograma se
apreciaba dilatación severa de cavidades derechas con regurgitación tricuspídea severa y PAPs de 59mmHg.

PREGUNTA
Cual de las siguientes aseveraciones no es correcta en el manejo de esta patologia?

RESPUESTA
a.- El tratamiento de la hipertensión pulmonar se basa en tres pilares fundamentales: medidas generales, anticoagulación y tratamiento
vasodilatador.
b.- Entre las medidas generales están evitar el ejercicio físico que provoque síntomas, las grandes altitudes, las situaciones de estrés
agudo y el embarazo.
c.- La anticoagulación hay que iniciarla siempre que no exista contraindicación por el aumento del riesgo de trombosis derivado del
estado de hipercoagulabilidad de la hipertensión pulmonar.
d.- En todos los casos deberá realizar vacunación antigripal, antineumococica y mantener profilaxias antibiótica.

CURSO ENARM CMN SIGLO XXI TEL: 36246001 Pharmed Solutions Institute PÁGINA 47
MANUAL DE TRABAJO DEL CURSO ENARM CMN SIGLO XXI
CASO CLINICO
Un hombre, albañil de 50 años de edad y agricultor fue admitido por falta de aire y tos, de 4 años de duración, y edema bilateral
inferior durante 2 meses. En el EF dificultad respiratoria, una frecuencia respiratoria de 50 rpm, pulso de 120 latidos por minuto,
cianosis, hipocratismo digital, pedal edema con fóvea bilateral, a la izquierda paraesternal tirón, p2 voz alta, grado 3 pansistólico
murmullo, máxima abajo a la izquierda borde esternal, aumento de la presión venosa yugular, hepatomegalia, ascitis, así como las
conclusiones del pecho de parches sonidos respiratorios bronquiales y crepitantes.

PREGUNTA
Cual de las siguientes acciones no es adecuada para el caso?

RESPUESTA
a.- Oxígeno intermitente 4 l / min.
b.- Furosemida 80 mg cada 12 horas y espironolactona 50 mg.
c.- Ceftriaxona intravenosa y enoxaparina 40 mg subcutánea.
d.- Prednisolona 80 mg al día.

ENFERMEDAD PULMONAR OBSTRUCTIVA CRONICA (EPOC)


CIENCIAS BASICAS: Definición: Enfermedad inflamatoria, prevenible y tratable con efectos extrapulmonares significativos que pueden
contribuir a la gravedad de los individuos. Su componente pulmonar se caracteriza por limitación al flujo aéreo, que no es totalmente
reversible y es usualmente progresiva. El tabaquismo es el mayor factor de riesgo ambiental para EPOC, otros serian ciertas
exposiciones ocupacionales (minería de carbón, oro, fábricas de algodón, etc), en algunas ciudades la combustión de biomasa con mala
ventilación usada para cocinar, también incrementan el riesgo de EPOC. PATOGENIA: El agente irritante (humo de cigarro, biomasa),
desencadena inflamación de las vías aéreas y del parénquima pulmonar, activando macrófagos alveolares, PMN, linfocitos T, con la
consiguiente liberación de mediadores de inflamación (leucotrienos, TNF a, IL-8) y estos estimulan la liberación de proteasas,
destruyendo bronquios y parénquima = disminución de la elasticidad, atrapamiento aéreo, y obstrucción al flujo aéreo. La bronquitis
crónica se caracteriza por hiperplasia de las glándulas submucosas de los bronquios con calibre mayor a 2mm con infiltrado
inflamatorio. El enfisema pulmonar se clasifica en centroacinar cuando se afectan las partes centrales del acino (bronquiolo
respiratorio) y están preservados los alveolos distales y se observa en los ápices; el panacinar los acinos están afectados uniformemente
desde el bronquiolo respiratorio hasta el alvéolo y es más frecuente en las bases. DIAGNOSTICO: Síntomas cardinales: disnea
(progresiva, persistente, y empeora con el ejercicio), tos crónica (>8sem.), intermitente y presenté durante todo el día, expectoración
crónica, sibilancias, sensación de opresión torácica, pérdida de peso, ansiedad. Estos síntomas serán más perceptibles en los períodos
de agudización, que pueden deberse a infecciones respiratorias virales o bacterianas e incrementar la severidad del EPOC. Existe una
escasa respuesta a esteroides. Para dx. es indispensable la espirometría, que demuestre obstrucción, que no es del todo reversible,
posterior a aplicar broncodilatadores. Se concluye obstrucción tras relación FEV 1/CVF menor a 70%, el FEV1 sirve para clasificar enf. de
acuerdo a intensidad. La paO2 se mantiene cerca de lo normal hasta que el FEV1 cae <50% del valor predicho; la hipercapnia y la
hipertensión pulmonar son mas comunes por debajo de 25%. La Rx. de tórax: poco sensibles, lo mas frecuente rectificación de arcos
intercostales, abatimiento de diafragmas, hiperclaridad de campos pulmonares, corazón en gota. Medir oximetría de pulso (normal
95% a nivel del mar y 92% en Ciudad de México). Gasometría para identificar insuficiencia respiratoria.

CLASIFICACION:
CLASIFICACION DE GRAVEDAD FUNCIONAL DE EPOC TRATAMIENTO
I LEVE FEV1 >80% Agonista β de corta acción o anticolinérgico de corta
acción
II MODERADO FEV1 50-80% Agonista β de corta acción o anticolinérgico de corta
acción
III SEVERO FEV1 30-50% Combinar lo anterior: Agonista β de larga acción y/o
anticolinérgico y/ esteroides orales
IV MUY SEVERO FEV1 <30% o <50% con Insf. respiratoria y/o cor pulmonale Lo anterior mas esteroides sistemicos
TRATAMIENTO: Suspender la exposición al agente nocivo y oxígeno suplementario si hay hipoxemia. Los medicamentos sólo mejorara
síntomas, calidad de vida y disminución en número de exacerbaciones. Broncodilatadores hay anticolinérgicos de acción larga
(tiotropio), acción corta (bromuro de ipratropio); agonistas b2 de acción prolongada (salmeterol), de acción corta (albuterol, salbutamol
y terbutalina) los cuales se usan como rescate en todos los cuadros; las xantinas (aminofilina, teofilina), las cuales estimulan el mov.
ciliar, broncodilatan, por sus efectos adversos solo se usan en quien no responde a los anteriores. Los esteroides inhalados (
budesonina, fluticasona, triamcinolona), se usan por periodos prolongados, indicados cuando disminuyen el FEV1 en 12% o mas y en
grado III y IV con más de una exacerbación al año, no recomendados por múltiples complicaciones como osteoporosis, ganancia de
peso, cataratas, DM. Los esteroides sistémicos solo en periodos cortos, durante exacerbaciones. El tx. Quirúrgico con reducción de
volumen (resecando 20-30% de cada ápice pulmonar) o trasplante pulmonar. Antibióticos fuertemente recomendados, cuando
incrementa volumen de esputo o cambia el color del mismo. PREVENCION: Suspender agente nocivo, dejar de fumar disminuye el
deterioro de la función pulmonar y prolonga la vida. PRONÓSTICO: Los pacientes con EPOC, tienen menor sobrevida; a 3 años de dx. la
mortalidad es de 20%, a 6 es de 30% y a 8 años es de 40%. Los factores que más influyen en la mortalidad son el FEV1, la edad y el IMC.

CASO CLINICO
Paciente de 80 años, exfumador desde hace 30 años con una dosis acumulada de 70 paquetes/año, diagnosticado de enfermedad
pulmonar obstructiva crónica (EPOC) con una obstrucción grave al flujo aéreo (volumen espiratorio forzado en el primer segundo
[FEV1] del 40% posbroncodilatador), en tratamiento con oxígeno crónico domiciliario a 2l/min durante las 24h (PaO2 en fase estable de
64mmHg con oxígeno y 46mmHg sin él) desde hace 10 años. El paciente refiere dolor torácico tipo quemazón que le dificulta la

CURSO ENARM CMN SIGLO XXI TEL: 36246001 Pharmed Solutions Institute PÁGINA 48
MANUAL DE TRABAJO DEL CURSO ENARM CMN SIGLO XXI
respiración, el dolor es diario y continuo, aunque mejora al tumbarse y se hace más intenso a última hora de la tarde; también refería
sensación de hormigueo en ambos pies y en el muslo derecho, con anestesia del dorso interno del pie izquierdo.

PREGUNTA
Cual es su conducta a seguir mas adecuada para el caso?

RESPUESTA
a.- AINE´s más antihistamínico.
b.- Antiviral tópico y oral.
c.- Gabapentina y capsaisina.
d.- Complejo B1, B6, B12.

CASO CLINICO
Varón de 57 años, con antecedentes de ex tabaquismo, ex enolismo, bronquiectasias y EPOC grave de 17 años de evolución, con
múltiples ingresos por exacerbaciones. Ingresó por aumento de su disnea habitual, febrícula y tos con expectoración purulenta de 10
días de evolución. Como tratamiento de base precisaba oxígeno domiciliario, broncodilatadores, esteroides inhalados, teofilina,
antibioterapia ocasional y esteroides orales de forma discontinua, que no recibía desde hacía 2 meses. En la exploración estaba
consciente y orientado, sin cianosis, la frecuencia cardíaca era de 100 lat/min y la respiratoria de 25 respiraciones/min. En la
auscultación pulmonar se apreciaban roncus y sibilancias diseminadas, sin otros datos destacables. En las pruebas complementarias
destacaban: 10.700 leucocitos con desviación izquierda y velocidad de sedimentación globular de 77 mm/h; gasometría arterial
(fracción inspiratoria de oxígeno: 0,24) con pH de 7,35, presión arterial de oxígeno de 85 mmHg, presión arterial de anhídrido carbónico
de 51 mmHg y HCO3 de 28.

PREGUNTA
Cual de los siguientes datos radiológicos es menos probable observar al momento del ingreso?

RESPUESTA
a.- Insuflación pulmonar, lesiones residuales en ambos ápex pulmonares.
b.- Aumento de la trama broncovascular.
c.- Signos de hipertensión pulmonar de tipo precapilar y discreto patrón alveolar en la língula.
d.- Infiltrados alveolares parcheados bilaterales que afectaban al lóbulo inferior izquierdo.

PREGUNTA
El examen microbiológico y cultivo procedente del catéter telescopado y broncoaspirado mostró bacilos grampositivos ramificados,
débilmente teñidos con la técnica de Ziehl-Neelsen, que se identificaron como N. asteroides, Cual es tratamiento mas adecuado?

RESPUESTA
a.- Amoxicilina y ácido clavulánico.
b.- Cefuroxima y gentamicina,
c.- Vancomicina y trimetoprim-sulfametoxazol.
d.- Cefotaxima e imipenem.

CASO CLINIO
Varón de 78 años, exfumador desde hacía 18 años de 40 paquetes/año, diagnosticado de EPOC (estadio IV de la clasificación GOLD) con
buen control en consultas externas; síndrome de apneas-hipopneas del sueño tratado con presión positiva continua en la vía aérea;
insuficiencia respiratoria crónica con oxigenoterapia crónica domiciliaria, y artritis reumatoide en tratamiento con leflunomida. En los 3
meses previos había ingresado en 3 ocasiones en el hospital por exacerbación de la EPOC, con aislamiento de Escherichia coli en
esputo, que se trató según antibiograma con cefditorén por vía oral (400mg/12h) durante 21 días. Ingresó por un cuadro de 5 días
consistente en aumento de la disnea habitual hasta hacerse de reposo, expectoración purulenta y febrícula. La exploración física reveló
roncus a la auscultación pulmonar. En la analítica destacaba un valor de proteína C reactiva de 2,48mg/dl. La radiografía de tórax (figura
1) mostró signos de hipertensión pulmonar precapilar y aumento de la trama broncovascular bilateral. La exploración funcional
respiratoria era: volumen espiratorio forzado en el primer segundo (FEV1) de 670ml (28%), capacidad vital forzada (FVC) de 1.200ml
(35%) y FEV1/FVC del 56%, con test broncodilatador negativo.

PREGUNTA
Según la clasificación de gravedad funcional del EPOC, en cual se encuentra el caso.

RESPUESTA
a.- Leve
b.- Moderado
c.- Severo
d.- Muy Servero

CASO CLINICO
Acude al servicio de urgencias paciente de 45 años de edad del sexo masculino, refiere que desde hace 3 semanas ha presentado tos,
con secresiones que inicialmente fueron transparentes y actualmente son verdosas amarillentas, agrega que se siente cansado y no ha

CURSO ENARM CMN SIGLO XXI TEL: 36246001 Pharmed Solutions Institute PÁGINA 49
MANUAL DE TRABAJO DEL CURSO ENARM CMN SIGLO XXI
podido trabajar adecuadamente, esta tos la ha presentado desde hace mas de 5 años y ha presentado al menos un cuadro semejante a
este una vez al año, sus antecedentes patológicos: originario de puebla en comunidad rural que emigro a la ciudad de mexico a los 15
años, es trabajador de una maderería desde entonces y tabaquismo positivo desde hace 20 años a razón de una cajetilla diaria. A la
exploración física se observa leve cianosis peribucal, levemente pletórico, campos pulmonares con estertores crepitantes, sibilancias. El
area cardiaca se apresia disminución de ruidos cardiacos.

PREGUNTA
Se realizo radiografia de torax, cual de las siguientes características observadas no se observan en la imagen?

RESPUESTA
a.- Rectificación de los arcos costales.
b.- Abatimiento del diafragma.
c.- Ensanchamiento de los espacios intercostales.
d.- Hiperclaridad de campos pulmonares.

PREGUNTA
Se realizo espirometria tres meses después del cuadro actual, encontrandoce FEV1 > 55 % y FEV1/CVF 0.8, tomando encuenta los datos
cual es el grado que presenta el paciente, de acuerdo con la Iniciativa Global para la Enfermedad Pulmonar Obstructiva Cronica?

RESPUESTA
a.- Leve.
b.- Moderado.
c.- Severo.
d.- Muy severo.

PREGUNTA
Considerando el diagnotico y clasificación actual del paciente cuales son las medidas preventivas y de rehabilitación mas adecuados
para influir sobre el pronostico del paciente?

RESPUESTA
a.- Vacunacion antineumococo y antiinfluenza.
b.- Rehabilitacion pulmonar.
c.- Resección de apices y transplante pulmonar.
d.- Retiro de factores de riesgo.

CANCER DE PULMON
CIENCIAS BASICAS: Factores de riesgo: El principal tabaquismo, mostrando una relación entre consumo y número de cigarrillos
consumidos por día, así como por el inició y tiempo de consumo dandonos el indice tabaquico que calcula asi: N. de cigarrillos fumados
al dia x años de consumo entre 20. Indice tabaquico >20 paquetes/ año; se considera con incremento en factor de riesgo para cáncer
pulmonar. Otros son asbesto, arsenico, niquel. SALUD PUBLICA: Primer lugar de mortalidad en todos los tipos de cáncer en el mundo
occidental. En México la tasa de mortalidad para cáncer pulmonar es de 125.2 por cada 100,000 en hombres y 48.8 por cada 100,000
en mujeres. Mas comun en hombres de 50-70 años. PATOGENIA: El humo de tabaco contiene alrededor de 4800 compuestos; 60 son
cancerígenos, ya que reaccionan con el DNA humano, formando complejos con las bases nitrogenadas, así en la replicación de DNA, se
introducen errores en la copia, dando lugar a mutaciones. El mas estudiado es el benzopireno, que hace una tramsversion de guanina
por tiamina y las nitrosaminas que median reacciones de alquilación de DNA. Se han detectado mutaciones en el gen p53 (cromosoma
17), el cual es un gen supresor de tumores, que al no funcionar adecuadamente permite la sobrevida de elementos geneticamente
dañados, que conducen a la transformación tumoral. También hay alteraciónes en el gen ras y myc. CLASIFICACIÓN: Se dividen en
benignas (2-5%) y malignas (95%). Dentro de los benignos, el hamartoma es el mas frecuente, en general asintomático, los sintomas
dependerán de localización, intratraqueales; tos, estridor, disnea incluso hemoptisis. Endobronquiales; obstrucción bronquial,
neumonía, neumonitis, hiperinsuflacion. Intraparenquimatosos; gnral. asintomaticos sin importar tamaño. El hamartoma es una masa
formafa de cartilago, tej. conectivo, grasa, musculo liso y epitelio respiratorio, en Rx. presentan calcificaciones en palomita de maiz,
caracteristico. Malignas: Se dividen en 2: Cáncer broncogenico de celulas pequeñas (20%; subtipos: carcinoma de cel. pequeñas, mixto
de cel. pequeñas y grandes, combinado de cel. pequeñas) y de celulas no pequeñas (80%; subtipos: adenocarcinoma 60%, escamoso o
epidermoide 30%, carcinoma indiferenciado de cel. grandes <10%). TUMOR DE PANCOAST: Es una forma de presentación del
carcinoma de pulmón, localizado en el surco pulmonar superior, cerca del vértice pulmonar, evolución lenta, tendencia a infiltrar los
linfáticos subpleurales y por contiguidad raices de 8vo. cervical, 2do y 3er dorsal, cadena simpática y estructuras óseas. Clinica: Dolor
em hombro irradiado a lo largo del brazo, patestesias (por infiltración del plexo braquial), irradiación a hombro y codo, debilidad y
atrofia de musc. intrínsecos, Sx. Horner (ptosis, miosos, anhidrosis). Rx. opacidad o engrosamiento unilateral en vértice pulmonar,
destruccion de 1ra y 2da. costilla. CA. BRONCOGENICO DE CEL. PEQUEÑAS: Muy agresivo, localización central, crecimiento rápido,
supervivencia de 2-4 meses del dx. sin tratamiento. Tiende a la diseminación. Sensible a quimio y radioterapia.tratamiento CA.
BRONCOGENICO DE CEL. NO PEQUEÑAS: Son agregados heterogéneos. Adenocarcinoma pulmonar: 60%, tiende a ser periférico, se
subdivide en acinares, sólidos, papilares y bronquioloalveolares, invade la pleura hasta en 50%, mas comun en pacientes no fumadores
y en mujeres. Escamoso o epidermoide: 30%, deriva del epitelio superficial de los bronquis, suele ser central y con tendencia a la
exfoliacion, crecimiento lento, buen pronostico. Carcinoma de cel. grandes: el menos frecuente y de peor pronóstico, células con
citoplasma y nucleo grandes.CUADRO CLÍNICO: Inespecificos; 5-15% asintomáticos. Fatiga, anorexia, perdida d peso. Asociados al
tumor; tos seca (lo mas común), puede haber expectoración, hemoptisis, dolor torácico, disnea frecuente. Manifestaciones por

CURSO ENARM CMN SIGLO XXI TEL: 36246001 Pharmed Solutions Institute PÁGINA 50
MANUAL DE TRABAJO DEL CURSO ENARM CMN SIGLO XXI
compresión; Sx. de Horner, paralisis del hemidiafragma por invasión del N. frenico, tumor de Pancoast, Sx. de vena cava superior (por
obstrucción vascular, ingurgitación yugular, edema en esclavina), paralisis laringea. Metástasis; mas comun a huesos ( fracturas óseas),
hígado, glándulas suprarrenales, ganglios retroperitoneales.DIAGNÓSTICO: Precoz; mayores de 45 años, fumadores, citologia de
esputo, radiografía de tórax, TAC. Pacientes con sintomas y signos; establecer dx. histológico, biopsia, broncoscopia. TRATAMIENTO:
Carcinoma de cel. no pequeñas; cirugía y radioterapia, potencialmente curativo en estadios precoces. Carcinoma de cel. pequeñas, es
quimioterapia asociada a radioterapia. CANCER PULMONAR METASTÁSICO: El pulmon es un sitio muy frecuente para la implantación
de metástasis de tumores pulmonares y extrapulmonares, hasta 30-40% de los canceres producirán metástasis a pulmón. Los tumores
que mas afectan son: cáncer de mama, de prostata, gastrointestinales, riñon. Ya sea por via hematógena, via linfática.

CASO CLINICO
Una mujer de 64 años fue ingresada en el hospital por una disnea creciente y dolor torácico. La paciente tenía antecedentes de
carcinoma pulmonar no microcítico y estaba recibiendo tratamiento. Anteriormente había fumado 4-5 cigarrillos al día durante un
periodo de hasta 10 años. A la exploración, se trataba de una mujer con sensación de enfermedad y emaciación, que estaba
clínicamente anémica y presentaba signos de un derrame pleural izquierdo masivo en la TC torácica. La frecuencia del pulso era de
85/min, la presión arterial de 120/85, y no había signos de insuficiencia cardiaca. La radiografía de tórax confirmó el derrame unilateral
izquierdo. La hemoglobina era de 9,1g/100ml, con un frotis de sangre normocrómico y normocítico. Los niveles séricos de ferritina,
vitamina B12 y folato eran normales. La velocidad de sedimentación globular fue de 90mm en la primera hora.

PREGUNTA
Considerando el caso clínico cual es el agente causal mas probable?

RESPUESTA
a.- Bordetela perturssi.
b.- Streptococcus epidermidis.
c.- Staphylococcus aureus.
c.- Enteroccocus fecalis.

CASO CLINICO
Paciente de 69 años, exfumador, diabético y dislipémico con el diagnóstico de adenocarcinoma pulmonar obtenido mediante
fibrobroncoscopia que mostró una tumoración en el orificio del bronquio segmentario anterior del lóbulo superior derecho. La
tomografía por emisión de positrones (PET-TC) informaba de una masa hipercaptante (SUVmáx. de 15g/ml) paramediastínica en el LSD
compatible con tumoración maligna, una adenopatía paratraqueal inferior derecha con centro graso sin captación de FDG y una lesión
paratraqueal superior derecha de 8mm discretamente hipermetabólica que se interpretó como adenopatía. Ante estos hallazgos se
estadificó como T3NxM0. Al tratarse de un paciente candidato a tratamiento quirúrgico con intencionalidad radical, se consideró
imprescindible la obtención de muestras para descartar la afectación N2.
PREGUNTA
Cual de las siguientes met´s es menos frecuente en este caso?

RESPUESTA
a.- Huesos
b.- Hígado,
c.- Tiroides.
d.- Glándulas suprarrenales.

CASO CLINICO
Varón de 26 años, sin hábitos tóxicos ni antecedentes patológicos de interés, que acudió a urgencias por disnea progresiva de una
semana de evolución y expectoración hemoptoica en las últimas 48 h, acompañadas de malestar general, distermia, dolor
centrotorácico, dolor en pelvis derecha y columna lumbar. Hemoglobina de 9 g/dl, hematócrito del 27% y leucocitosis de 13.500
células/μ l con neutrofilia. En la bioquímica general se observaban patrón de colestasis e hipoalbuminemia. La gasometría mostraba
insuficiencia respiratoria (pH: 7,45; presión arterial de anhídrido carbónico: 37 mmHg; presión arterial de oxígeno: 54 mmHg; HCO3: 27
mmol/l. En el estudio de coagulación se objetivó una actividad de protrombina del 50%, actividad de tromboplastina tisular de 17 s y
fibrinógeno de 300 mg/dl. En la radiografía de tórax se observaban un derrame pleural derecho que ocupaba un tercio del hemitórax y
patrón intersticial bilateral, más evidente en los lóbulos superior derecho y medio.

PREGUNTA
Considerando el cuadro clínico cual es la conducta inmediata mas adecuada?

RESPUESTA
a.- Toracocentesis.
b.- Toracostomia diagnostica.
c.- Colocacion de sello.
d.- Drenaje endotoracico.

PREGUNTA
Considerando el cuadro clínico, cual es la clase mas probable?

CURSO ENARM CMN SIGLO XXI TEL: 36246001 Pharmed Solutions Institute PÁGINA 51
MANUAL DE TRABAJO DEL CURSO ENARM CMN SIGLO XXI
RESPUESTA
a.- Cáncer broncogenico de celulas pequeñas.
b.- Carcinoma de células grandes.
c.- Adenocarcinoma escamoso.
d.- Carcinoma indiferenciado de celulas grandes.

CASO CLINICO
Masculino de 51 años de edad que inicia desde hace 3 días con tos productiva, ataque al estado generalizado, fiebre, dificultad
respiratoria, en Rx de torax se aprecia datos compatibles con neumonía lobar inferior derecha y un nódulo redondo de 1,5 cm lóbulo
superior izquierdo, recibe tratamiento presentando mejoría egresando en una semana, se realiza TAC ambulatoria, donde se aprecia
nódulo de 1.5 por 1.8 ubicado en la misma posición, no se aprecian calcificaciones con los bordes levemente festoneados, en
mediastino no se observan adenopatías ni datos patológicos, no se aprecia exudados pleurales.

PREGUNTA
Cuál es el estudio más apropiado para continuar con el diagnostico en este caso

RESPUESTA
a.- MRI
b.- PET con FDG
c.- Broncoscopia.
d.- TAC de control a 6 meses.

CASO CLINICO
Masculino de 71 años de edad con dolor en la espalda desde hace dos meses de inicio súbito sin factor precipitante, agrega que se
incrementa el dolor cuando se acuesta y disminuye durante el día y con el movimiento. El paciente es originario del Estado de México,
fue trabajador de la construcción con alcoholismo y tabaquismo positivo por más de 30 años, los cuales dejo hace 15 años, no es
diabético ni hipertenso, al examen físico se observa con aspecto general adecuado, alerta y orientado, índice de masa corporal de 25,
se realiza laboratorios de rutina donde solo se realiza el hallazgo de fosfatasa alcalina elevada y en la radiografía de columna se observa
lesión lítica única en la vertebra L3.

PREGUNTA
El cuadro clínico nos indica una probable tumoración maligna, cual es la más probable.

RESPUESTA
a.- Cáncer pancreático.
b.- Cáncer gástrico.
c.- Cáncer tiroideo.
d.- Cáncer pulmonar.

TRASTORNOS HIPOTALAMICOS E HIPOFISIARIOS. El hipotálamo libera factores liberadores de hormonas hipofisiaria, asi como
dopamina que inhibe la prolactina evitando su liberación, además de la síntesis de arginina, vasopresina y oxitocina, El síndrome
hipotalámico mas frecuente es la secresion anormarl de GnRH consecuentemente reducción de gonadotrofinas e hipogonadismo. Las
causas mas comunes, esta el estrés, perdida de peso o ejercicio exesivo, el tratamiento envuelve el manejo de la causa cuando es
posible, la administracion pulsatile GnRH es usualmente afectivo. SINDROME HIPOTALAMICO: Poco frecuente, ocurre en tumores,
posterior a neurocirugía por craneofaringeoma, o por infiltración como en la histiocitosis de células de Langerhans. La típica
presentación es hiperfagia, con cambios de pero, pérdida de sensación, somnolencia, cambios de conducta. Se debe regular los liquidos
diarios y el balance. Los efectos metabolicos del daño hipotalámico por masa son frecuentemente alteración del apetito (hiperfagia y
obesidad y anorexia), sed, adipsia, beber compulsivo, Temperatura (hipertermia, hipotermia), somnolencia y coma.
HIPOPITUITARISMO: Es una patologia que se caracteriza por la disminución parcial o completa de las hormonas hipofisiarias anterior o
posterior o como producto de la interferencia del funcionamiento del hipotálamo sobre la actividad hipofisiaria. Las CAUSAS mas
frecuentes son los tumores hipofisiarios, tumores parahipofisiarios (craniofaringiomas, meningiomas, secundarios “pecho, pulmon”,
cordomas, gliomas) por radioterapia (hipofisiaria, craneal y nasofaríngea) por infarto hipofisiario (síndrome de sheehan), Infiltrado
hipofisiario (sarcoidosis, linfociticas, hemocromatosis, histiocitosis de células de langerhans, enfermedad de Erdheim-Chester),
síndrome de silla turca vacia, por causa infecciosa, traumatica, síndrome de Kallman.

CASO CLINICO
Varón de 34 años de edad, sin antecedentes, inició a los 31 años con dolor típico, en región supraorbitaria derecha, intenso, pulsátil, de
30-60 minutos de duración, asociado a inyección conjuntival ipsilateral, congestión nasal y en ocasiones rinorrea. Presentaba 2-3
ataques por día en períodos que duraban de 7-10 días y se diagnosticó con Cefalea en Racimos CR episódica. El examen neurológico no
mostró datos patológicos y la imagen de resonancia magnética (RM) inicial no mostró anormalidades. Se inició el tratamiento con
verapamilo 240 mg/día y se administraba oxígeno y sumatriptán durante los ataques desde el inicio del cuadro, con respuesta
adecuada. Tres años después se añadió disminución de la agudeza visual (AV) y defecto en el campo visual del ojo derecho. La
evaluación oftalmológica reportó AV 20/100 y escotoma central en el ojo derecho y AV 20/20 sin alteración del campo visual en el ojo
izquierdo.

PREGUNTA

CURSO ENARM CMN SIGLO XXI TEL: 36246001 Pharmed Solutions Institute PÁGINA 52
MANUAL DE TRABAJO DEL CURSO ENARM CMN SIGLO XXI
Cual es su conducta a seguir mas adecuada para el caso?

RESPUESTA
a.- Realizar RX con énfasis en silla turca.
b.- Evaluación de fondo de ojo con dilatación.
c.- Resonancia magnetica contrastada.
d.- Perfil hormonal hipofisiario.

CASO CLINICO
Mujer de 61 años con diabetes mellitus tipo 2, hipercolesterolemia, histerectomía más doble anexectomía y síndrome de apnea
obstructiva del sueño (SAOS) severo. Estaba diagnosticada desde hacía 3 años de miocardiopatía dilatada con coronarias
angiográficamente normales y disfunción ventricular severa. Seguía tratamiento con betabloqueador 25 mg, digoxina en dosis bajas,
ARA 2, espironolactona y estatinas. La exploración física mostraba una presión arterial de 120/60 mmHg y sobrepeso; la auscultación
pulmonar fue normal y la cardiaca era rítmica, con un soplo sistólico eyectivo I/VI. La analítica mostró una glucosa de 148 mg/dl, y el
resto de la bioquímica, la coagulación y el hemograma eran normales. El electrocardiograma (ECG) estaba en ritmo sinusal a 40 lat/min
con bloqueo de rama izquierda, y había cardiomegalia en la radiografía de tórax.

PREGUNTA
Cual es su conducta mas adecuada a seguir?

RESPUESTA
a.- Perfil hormonal hipofisiario.
b.- Ecocardiograma transesofagico.
c.- Resonanacia magnetica selar.
d.- Coronariografia.

CASO CLINICO
Mujer de 32 años, con ciclos menstruales regulares, asintomática y sin tratamientos crónicos, a la que se le practicó una RNM craneal
en febrero de tras un traumatismo craneoencefálico. Se objetivó un adenoma hipofisario de 13 x 10 mm, intraselar, lateralizado hacia la
derecha. El campo visual y la exploración oftalmológica eran normales, y las determinaciones de hormonas hipofisarias también. Tras
tratamiento con cabergolina (0,5 mg dos veces por semana) durante 3 meses. Ante la necesidad de un seguimiento indefinido,
previsiblemente largo por la edad de la paciente, se acordó con ella la exéresis quirúrgica del macroadenoma no funcionante, que se
efectuó de forma total y selectiva, por vía transesfenoidal sublabial.

PREGUNTA
Cual es el porcentaje de reducción de los incidentalomas?

RESPUESTA
a.- 60 %
b.- 30 %
c.- 20 %
d.- 10 %

CASO CLINICO
Varón de 66 años, con depresión no tratada farmacológicamente, y con hiperplasia benigna de próstata que precisó biopsia en 3
ocasiones por cifras elevadas de antígeno prostático (PSA). El paciente refería astenia intensa, sofocos, pérdida de fuerza muscular y
disfunción eréctil completa en el último año. La campimetría era normal. El estudio hormonal hipofisario mostró una testosterona total
de 0,19 ng/mL (valor normal (VN).: 2,9-8,0), una testosterona libre de 1,2 pg/mL (VN: 9-27), LH: 1,5 U/L (VN: 7-24), prolactina: 35 ng/mL
(VN: 4,0-18,5), subunidad alfa libre de las hormonas glucoproteicas y resto de determinaciones normales.

PREGUNTA
Cual de los siguientes estudios es el mas indicado para establecer una etiología?

RESPUESTA
a.- USG de Suprarrenales.
b.- USG de Testiculos.
c.- TAC de Hipofisis.
d.- IRM de Hipotalamo.

CASO CLINICO
Niño de cinco años con un cuadro de pubertad precoz de origen central, que durante el estudio etiológico fue diagnosticado de una
lesión ocupante de espacio de localización supraselar El paciente fue intervenido y posteriormente ingresado en la unidad de cuidados
intensivos pediátricos (UCIP) para el control postoperatorio. Se administró tratamiento antiinflamatorio con dexametasona y hormonal
sustitutivo con hidrocortisona. Transcurridas ocho horas desde la intervención el paciente presentó poliuria (diuresis de 1.080ml en 4
horas, 8ml/kg/h) y el control analítico reveló: sodio plasmático (Nap): 140mmol/l; osmolaridad plasmática (Osmp): 287 mosm/kg, con
sodio urinario (Nau): <3mmol/l y una osmolaridad urinaria (Osmu): 108 mosm/kg.

CURSO ENARM CMN SIGLO XXI TEL: 36246001 Pharmed Solutions Institute PÁGINA 53
MANUAL DE TRABAJO DEL CURSO ENARM CMN SIGLO XXI

PREGUNTA
Considerando el cuadro clínico, cual de los siguientes diagnosticos presuntivo es el mas frecuente?

RESPUESTA
a.- DI transitoria.
b.- DI permanente.
c.- DI patrón bifásico (DI-SIADH).
d.- DI trifásico (DI-SIADH-DI).

CASO CLINICO
Hombre de 63 años que refiere un cuadro de dos semanas de evolución caracterizado por fiebre y deposiciones diarreicas que ceden de
forma espontánea; posteriormente, comienza con parestesias en los dedos de las manos y de los pies, alteraciones de la movilidad e
inestabilidad de la marcha, por lo que acude al servicio de urgencias donde la inestabilidad de la marcha se hace más manifiesta y le
impide caminar, y comienza a presentar paresias de las cuatro extremidades y de la musculatura facial. Como antecedentes personales
destaca que es hipertenso sin tratamiento ni seguimiento y bebedor de 20 gramos de alcohol diarios. En la exploración neurológica
destacan: diplejía facial con habla hipofónica, reflejos osteotendinosos rotulianos y aquíleos abolidos e hiporreflexia bicipital y tricipital
sin pérdida de fuerza, abolición de la sensibilidad vibratoria y disminución bilateral de la sensibilidad táctil y algésica con marcha
atáxica. En las pruebas complementarias presenta: leucocitos 10.100 con fórmula normal, hemoglobina 16 g/l, creatinina 0,6 mg/dl,
ácido úrico 2,8 mg/dl, sodio 120 mmol/l, potasio 4,3 mmol/l, cloro 87 mmol/l, bicarbonato 16 mEq/l, colesterol total 232 mg/dl, glucosa
110 mg/dl, y osmolaridad sérica 259 mOsm/kg. Sodio urinario 144 mmol/l y osmolaridad en orina 719 mOsm/kg.

PREGUNTA
Cual de los siguientes patologías presentan mas frecuentemente SIADH?

RESPUESTA
a.- Traumatismo cráneo encefálico.
b.- Tratamiento quirúrgico hipofisiario.
c.- Cáncer de pulmón de células pequeñas.
d.- Síndrome paraneoplásico.

CASO CLINICO
Varón de 61 años, con hipercolesterolemia e hipertensión arterial de muchos años de evolución, controlada con medicación. Se realizo
operación de menisco de rodilla, bajo anestesia raquídea con 10 mg de bupivacaina 0,5%. Durante la operación la presión arterial
sistolica decreció desde 120 a 80 mm Hg, permaneciendo en ese nivel durante tres horas y, el pulso, en 30-45 s/m. En el inmediato
postoperatorio el enfermo sufrió una agudización grave de la cefalea, junto con un cuadro confusional leve y fi ebre de 39º. La cefalea
era generalizada, opresiva y no relacionada con la posición ortostática. Cuarenta y ocho horas más tarde el paciente presenta pérdida
de visión bilateral, agitación y mayor confusión mental. La determinación de Na plasmático fue de 129 mmol/L.

PREGUNTA
Cual es la complicación mas grave tras la corrección rápida del Na plasmático?

RESPUESTA
a.- Mielonolisis pontina.
b.- Degeneración pontino cerebelosa.
c.- Mielonolisis cortico basal.
d.- Hemorragia periacueductal.

CASO CLINICO.
Masculino de 73 años con una pérdida de peso de 5 kg, fatiga, pérdida de apetito, desinteres por ambiente, falta de intensionalidad y
tos desde hace 2 meses, con expectoración, Informó fumar un paquete de cigarrillos por día durante los últimos 50 años. EF 162 cm de
altura, 53 kg de peso, temperatura era de 35,2 º C, el pulso 74/min y la presión arterial 102/60 mmHg. Se auscultaron sibilancias y
roncus audibles en la zona basal izquierda. Se diagnostico cáncer pulmonar, se agrego posteriormente poliuria y polidipsia.

PREGUNTA
Cual es su impresión diagnostica en este momento?

RESPUESTA
a.- Diabetes insípida.
b.- Sindrome de addison.
c.- Crisis hipotiroidea.
d.- Panhipopituitarismo.

CURSO ENARM CMN SIGLO XXI TEL: 36246001 Pharmed Solutions Institute PÁGINA 54
MANUAL DE TRABAJO DEL CURSO ENARM CMN SIGLO XXI
CASO CLINICO
Se trata de femenino de 18 años con diagnostico de retrazo psicomotor con deterioro del desarrollo del lenguaje a la EF se clasifica con
Tanner 1 con una altura de 144,8 cm. BH y QS sin datos importantes, se detectaron estradiol y progesterona baja. FSH y LH con minima
respuesta a la estimulación con GnRH, resto de laboratorio sin datos por agregar, campimetría normal.

PREGUNTA
Cual la conducta a seguir mas adecuada para el caso?

RESPUESTA
a.- RX de cráneo.
b.- TAC de cráneo.
c.- IRM de cráneo.
d.- USG pélvico.

CASO CLINICO
Mujer de 35 años, con antecedentes de asma bronquial y consumidora de fin de semana de cocaína inhalada, hábito no referido al
ingreso, que acude al Servicio de Urgencias por un cuadro de cefalea intensa de predominio occipital, con nauseas, vómitos, fotofobia y
fiebre de hasta 38 °C. Se le practica una tomografía axial computarizada cerebral, que no muestra hallazgos patológicos, y una punción
lumbar, que evidencia líquido cefalorraquídeo (LCR) con pleocitosis de predominio polimorfonuclear e hipoglucorraquia. En la
exploración neurológica, no se evidenciaron hallazgos excepto rigidez de nuca con signos meníngeos positivos.

PREGUNTA
Cual de la siguientes aseveraciones es incierta, en el padecimiento actual?

REPUESTA
a.- La clínica puede ser indistinguible de la de un tumor hipofisario o una infección del sistema nervioso central.
b.- Habitualmente, se presentan con insuficiencia pituitaria, manifestada como hiperprolactinemia ó panhipopituitarismo.
c.- Las alteraciones visuales están presentes en el 50-75%, la heminopsia uni o bitemporal es el patrón hallado con más frecuencia.
d.- La fiebre y la leucocitosis están presentes hasta en el 46% de los pacientes.

CASO CLINICO
Se trata de femenino de 27 años de edad la cual acude a consulta refiriendo visión borrosa en ambos ojos que le ha causado algunos
accidentes como tropezar o golpearse debido a que no alcanza a ver los objetos, refiere además que desde hace 2 años
aproximadamente ha tenido cefalea que anteriormente se controlaba con la administración de AINES, en el examen del campo visual
con confrontación digital se revela reducción de la visión en la periferia izquierda del ojo izquierdo y en la periferia derecha del ojo
derecho, a la exploración general observa que la paciente presenta obesidad grado I, acné e hirsutismo, la paciente agrega presentar
alteraciones menstruales.

PREGUNTA
Considerando los hallazgos en los campos visuales, donde es más probable que se ubique la lesión:

RESPUESTA
a.- Espacio suprasillar.
b.- Nervios ópticos.
c.- Tracto posquiasmaticos.
d.- Cuerpos geniculados.

CASO CLINICO
Acude a consulta paciente masculino de 31 años de edad el cual refiere que desde hace 6 meses presenta cefalea continua que no sede
a AINES indicados por su médico familiar, agrega que tuvo un accidente automovilístico hace un mes, niega nausea o vomito, a la
exploración física se observa con leve crecimiento de areas acras del cuerpo principalmente en dedos y maxilar inferior, sus estudios de
laboratorio muestran dislipidemia e incremento de glucosa, finalmente refiere disfunción eréctil.

PREGUNTA
Cuál es la hormona responsable que más probable se encuentre alterada.

RESPUESTA
a.- ACTH
b.- PRL
c.- GH
d.- TSH.

CURSO ENARM CMN SIGLO XXI TEL: 36246001 Pharmed Solutions Institute PÁGINA 55
MANUAL DE TRABAJO DEL CURSO ENARM CMN SIGLO XXI
TRASTORNOS TIROIDEOS
CIENCIAS BASICAS: Tiroides mayor glándula endocrina del cuerpo pesa 20grs. La producción de tiroxina (T4) y triyodotironina (T3), se
controla a través de un bucle de retroalimentación endocrina clásica, se forman con la unión de la tiroglobulina mas iodo, mediante una
peroxidacion. Parte de T3 es secretada por tiroides, pero más es producida por desyodación de T4 en los tejidos periféricos. Tanto la T4 y
T3 están ligadas a proteínas transportadoras (globulina de unión a la tiroxina 80% (TBG), transtiretina, y albúmina) en la circulación.
Aumento de los niveles de T4 total y T3 con niveles libres normales se observan en los estados de aumento de proteínas transportadoras
(embarazo, los estrógenos, cirrosis, hepatitis y trastornos hereditarios). Por el contrario, la disminución de los niveles totales de T4 y T3,
con niveles libres normales se observan en la enfermedad sistémica grave, enfermedad hepática crónica, y nefrosis.
HIPOTIROIDISMO (Hipofunción de glándula tiroides)
H. PRIMARIO 1.- Déficit de yodo: Principal causa a nivel mundial. 2.- Autoinmunitario: Tiroiditis de Hashimoto: 1ra causa donde no existe déficit
131
(tiroides) de Iodo. Tiroiditis atrófica. 3.- Iatrogena: Tiroidectomía total o subtotal, radiación de cuello por tx. de linfoma o cáncer, uso de I .
4.- Medicamentos: Amiodarona, litio, antitiroideos, ac. p-aminosalicilico, aminoglutecimida. 5.- Congénito: Disgenesia de tiroides
(80-90%), errores en hormonas tiroideas (10-15%), ectopia o hipoplasia de tiroides. 6.- Trastornos que infiltran tiroides:
Amiloidosis, sarcoidosis, hemocromatosis, esclerodermia.
H. TRANSITORIO 1.- Tiroiditis silenciosa (puerperio). 2.- Tiroiditis subaguda
H. SECUNDARIO 1.- Hipopituitarismo: Tumores de hipófisis, Qx, radiación hipofisiaria, trastorno infiltrativo, traumatismos, Sx. de Shehann (necrosis
(hipófisis) hipofisiaria post-parto, por colapso circulatorio e isquemia, secundario a hemorragia obstétrica).. 2.- Déficit o inactividad aislada de
la TSH. 3.- Tratamiento con bexaroteno. 4.- Enf. Hipofisiaria: Tumores, traumatismos, trastornos infiltrativos, idiopático.
CUADRO CLINICO Debilidad, cansancio, intolerancia al frio, retardo mental, poco comunicativo, aumento de peso (almacenan glucosa como lípidos),
disminución de apetito, bradicardia, piel seca y áspera, caída de pelo, extremidades frías, macroglosia (por depósito de
glucosaminoglicanos), menorragia, disfonía, estreñimiento, disminución de reabsorción tubular, derrame pericárdico.
DIAGNOSTICO Si TSH elevada; T4L y T4 normales = H. subclínico o leve Buscar Ac-TPO; positivo-tx. con T4. Negativo-vigilancia anual
Si TSH elevada; T4L y T4 disminuidas = H. clínico  Buscar Ac TPO; positivo-autoinmunitario (presentes en 90-95%). Negativo-
iatrogeno, otro. Si TSH N o elevada; T4L Yt4 disminuidas = Orienta hacia causa hipofisiaria (H. secundario). TSH elevada marcador
sensible de hipotiroidismo 1ro.
TRATAMIENTO Primera elección levotiroxina (T4). Comienza su efecto hasta 4 sem. después de su inicio. Dosis de 100-150µg/24h VO, si hay
cardiopatía 12.5-25µg, incrementando cada 6-8 semanas. Objetivo TSH normal; llegar al eutiroidismo.
COMA Forma más severa del hipotiroidismo, urgencia médica, alto índice de mortalidad. Característico alteraciones de las funciones
MIXEMATOSO mentales. Hipotensión, hipotermia, bradicardia, convulsiones, hiponatremia, hipoglucemia. Factores de riesgo: sepsis, qx.,
anestesia, IAM, trauma, EVC
Se presenta en 8% mujeres y 2% en hombres. TX.: levotiroxina 400µg IV impregnación; 100µg IV c/24h para control. Hidrocortisona
50 mgs
CONGENITO Prevalencia 1 de cada 4000, transitorio o permanente dependiendo si madre tiene AC frente a TSH-R. Cuadro clínico: hipotonía,
intolerancia a VO, macroglosia, fontanela posterior amplia, ictericia prolongada. DX.: Tamiz neonatal, para evitar daño neurológico
permanente. TX.: Administración de T4, dosis 10.15µg/kg/día
AUTOINMUNITARIO Prevalencia en >60 años. Incidencia 4 por cada 100 mujeres y 1 en cada 1000 hombres. Característica clínica presencia de Bocio,
irregular y firme, frecuente asociado a DM, vitíligo, LES, artritis reumatoide

HIPERTIROIDISMO (Hiperfunción de glándula tiroides)


PRIMARIO 1.- Enf. Graves-Basedow. 2.- Bocio multinodular toxico. 3.- Adenoma toxico (estos 3 los más frecuentes). 4.- Metástasis de cáncer
de tiroides. 5.- Mutación activada del receptor de TSH. 6.- Estruma ovárico. 7.- Fenómeno de Jod-Basedow (exceso de I)
SECUNDARIO 1.- Tumores que secretan gonadotrofina corionica con TSH baja. 2.- Tirotoxicosis en el embarazo con TSH baja. 3.- Tiroiditis
subaguda. 4.- Tiroiditis silenciosa
TIROTOXICOSIS sin Estado hipermetabolico donde existe un exceso de hormonas T3 y T4 libres. 1.- Fármacos: amiodarona. 2.- radiación a cuello por
hipertiroidismo linfomas o cáncer. 3.- Infarto de un adenoma. 4.- Tirotoxicosis facticia (ingesta excesiva de T4)
CUADRO CLINICO Hiperactividad, nerviosismo, irritabilidad, disforia, palpitaciones, taquicardia sinusal (lo más frecuente y que puede llevar a
fibrilación auricular), fatiga, debilidad muscular, pérdida de peso, diarrea, intolerancia al calor, piel caliente, transpiración
exagerada, poliuria, perdida de libido, temblor, miopatía proximal, retracción palpebral “mirada de asombro”, ginecomastia
DIAGNOSTICO TSH sérica disminuida es un marcador sensible de hipertiroidismo causado por la enfermedad de Graves, los nódulos tiroideos
autónomos, tiroiditis, y el tratamiento con levotiroxina exógena. Nivel T4 elevada suficiente para tirotoxicosis.
TRATAMIENTO Antitiroideos (tionamidas): Carbimazol, metimazol 10-20mg c/8-12h inicio; 2.5-10mg para control. Propiltiouracilo (PTU) (se puede
usar en embarazo) 100-200mg c/6-8h inicio; 50-100mgpara control, efectos secundarios rash, urticaria, fiebre, agranulocitosis (más
importante). Iodo radiactivo; destrucción de glándula tiroides indicado en recidivas, contraindicación absoluta en embarazo y
lactancia. Tiroidectomía; cuando falla tx. médico y se rehúsan al Iodo, útil en jóvenes con bocios voluminosos, complicaciones;
hemorragia, edema laríngeo, hipoparatiroidismo, lesión de N. laríngeo recurrente. Función tiroidea debe ser checada cada 3-4 sem.
después de iniciar tx., con ajustes para mantener T 4 en niveles normales. Propanolol 20-40mg VO, para disminuir taquicardia y
síntomas adrenérgicos.
Tormenta tiroidea: PTU 500mg impregnación VO, rectal o por sonda nasogástrica, para continuar PTU 200-300mg c/6h, propanolol
y dexametasona.
GRAVES-BASEDOW Hasta 60-80% el más común. Principalmente mujeres. Edad entre 20-50 años. Para su desarrollo se necesita la actividad
inmunológica de las inmunoglobulinas estimulantes de tiroides (TSI). Se puede desarrollar oftalmopatia por la alteración de l os
músculos extraoculares por infiltración de linfocitos T activados, que inducen la liberación de IL-1, IFNγ y TNFα, que a su vez activan
a los fibroblastos y aumentando síntesis de glicosaminoglicanos, produciendo tumefacción de los músculos. Cuando se comprime
el N. óptico se produce edema papilar, defectos en campos periféricos y ceguera. Puede haber dermopatía; que es una placa
indurada violácea o rosada “piel de naranja”.
CRISIS TIROTOXICA O Es rara, suele ocurrir después de una enf. Estresante en pacientes no tratados, se caracteriza por taquicardia, fiebre, delirium,
TORMENTA TIROIDEA vomito diarrea, deshidratación, arritmias, coma. Exacerbación de hipertiroidismo.

AMIODARONA: Hay dos formas principales de tirotoxicosis inducida por amiodarona (AIT). Tipo 1 AIT se asocia con alteración tiroidea
subyacente (Enf. preclínica de Graves, bocio nodular). La síntesis de hormona tiroidea se convierte en excesiva como resultado de
incremento de la exposición a yodo. TIA tipo 2 se produce sin anormalidades intrínsecas de tiroides y es el resultado de la tiroiditis
destructiva. El fármaco debe interrumpirse, si es posible, con la administración de fármacos antitiroideos altas dosis, en el tipo 1
perclorato potásico y los glucocorticoides en el tipo 2.

CURSO ENARM CMN SIGLO XXI TEL: 36246001 Pharmed Solutions Institute PÁGINA 56
MANUAL DE TRABAJO DEL CURSO ENARM CMN SIGLO XXI
BOCIO MULTINODULAR TOXICO: Bocio voluminoso, por lo general en edad avanzada y se puede presentar con fibrilación auricular o
palpitaciones, taquicardia, nerviosismo, pérdida de peso. La exposición reciente a yodo, a partir de los medios de contraste o de otras
fuentes, puede precipitar o exacerbar la tirotoxicosis; esto puede evitarse mediante la administración previa de un fármaco
antitiroideo. Dx: Niveles de TSH bajos, T4 N o mínimamente elevado. Gammagrafía de la tiroides muestra una captación heterogénea
con zonas múltiple de aumento y disminución de la captación. TUMORES TIROIDEOS: Benignos=adenomas, malignos =carcinomas.
Incidencia mundial de 0.5-10 casos por 100 000, más en mujeres, entre 40-50 años, peor pronóstico en extremos de la vida. Factores de
riesgo: radiación a cabeza y cuello en la infancia, nódulo tiroideo de 2-4 cm, signos de fijación a estructuras vecinas, invasión a ganglios
linfáticos, parálisis de cuerdas vocales, antecedente de neoplasia endocrina múltiple tipo 2 (NEM2). Dentro de los carcinomas hay
foliculares, papilares y anaplasicos de tiroides (raro, muy maligno y fatal). Carcinoma papilar tiroideo subtipo más frecuente (70-90%) y
menos agresivo, es multicentrico, el antecedente principal es radiación a cuello en la infancia, generalmente asintomático en ocasiones
disfagia y voz ronca, sospecharlo en nódulos solitarios palpables.

CASO CLINICO
Acude a la consulta externa paciente femenino de 25 años de edad, acude debido a que no produce suficiente leche para lactar tiene 8
dias de puerperio, niega otra sintomatologia, se le explica la técnica para lactación, sin embargo a la exploración se observa incremento
simetrico de glandula tiroides, no se palpan nódulos ni zonas dolorosas, no se identificas cambios de voz, la paciente es originaria de
Chiapas.

PREGUNTA
Cual la conducta a seguir mas adecuada para el caso.

RESPUESTA
a.- Realizar valoración de THS y T4.
b.- Indicar suplementacion de yodo.
c.- Indicar alimentación rica en yodo, cita 6 meses.
d.- Solicitar perfil de anticuerpos antitiroideos.

PREGUNTA
6 meses después se envio a endocrinología debido a que presento incremento de peso, se mantuvo las mismas características de
volumen y consistencia de la glandula tiroidea, fue realizado un perfil tiroide con los siguientes resultados T4 total baja, TSH normal e
indico 100 mcg/dia, considerando la evolución, cual de los siguientes diagnostico diferenciales se debe causas no tiroideas.

RESPUESTA
a.- Sindrome de T4 baja.
b.- Bocio multinodular no toxico.
c.- Bocio multinodular toxico.
d.- Nodulo solitario hiperfuncionante.

CASO CLINICO
Paciente diagnosticada de hipotiroidismo subclínico y tratada adecuadamente con tiroxina oral. Unos meses después desarrolla
exoftalmos bilateral y simultáneamente presenta anticuerpos séricos anti-receptor de TSH positivos. Se suspende entonces el
tratamiento con tiroxina, y se comprueba que el hipotiroidismo ha progresado hasta hacerse primario. Se establece el diagnóstico de
enfermedad de Graves hipotiroidea.

PREGUNTA
Cual es la complicación mas frecuente a largo plazo para el paciente debido al tratamiento que se debe instaurar.

RESPUESTA
a.- Osteoporosis.
b.- Arteriosclerosis.
c.- Edema pulmonar.
d.- Dislipidemia.

CASO CLINICO
Acude a consulta de revisión anual paciente femenino de 31 años, la cual cuenta con el diagnostico de bocio simple, refiere que ha
presentado auscencia de periodo menstrual en 4 ocaciones en el años, además de presentar infecciones de vías aéreas altas en 2
ocaciones, ha presentado cansancio, debilidad generalizada, incremento de peso. A la exploración física se observa bradipsiquia y
bradilalica, tiene un aspecto seco y aspero. Los signos vitales fueron TA 110/70 mmHg.

PREGUNTA
Se ingreso a la paciente debido al malestar generalizado e hipotermia, cual es la conducta inmediata a seguir para establecer un
diagnostico presuntivo.

RESPUESTA
a.- BH, QS, EGO.
b.- Solicita TSH, T4.

CURSO ENARM CMN SIGLO XXI TEL: 36246001 Pharmed Solutions Institute PÁGINA 57
MANUAL DE TRABAJO DEL CURSO ENARM CMN SIGLO XXI
c.- Solicitar Anticuerpos peroxidasa tiroideos.
d.- Biopsia de tiroides.

PREGUNTA
Se obtuvieron los siguientes resultados hematocrito 32 %, hemoglobina 10.1, Leucocitos 5,900, glucosa 219 central, colesterol 310.
Considerando los resultados cual es la conducta a seguir más adecuada.

RESPUESTA
a.- Iniciar levotiroxina 150 mcg/ dia.
b.- Solicitar perfil tiroideo con anticuerpos.
c.- Estabilizacion y posterior envio a segundo nivel.
d.- Administración de prednisona 20 mg y levotiroxina 100 mcg/dia.

PREGUNTA
2 meses después regresa la paciente para control por el primer nivel de atención, en su nota de contrarreferencia se encuentra el
diagnostico de tiroiditis de Hashimoto, considerando la evolución del paciente, cual de las siguientes comorbilidades es la menos
probable.

RESPUESTA
a.- Diabetes mellitus.
b.- Enfermedad de Addison.
c.- Anemia perniciosa.
d.- Infiltracion leucocitaria hipofisiaria.

CASO CLINICO
Se trata de una mujer de 89 años con Hipertensión Arterial, Cardiopatía Hipertensiva, Fibrilación Auricular, Insuficiencia Mitral y Aórtica
moderadas, Enfermedad Cerebral de pequeño vaso y Síndrome Hipoventilación – Obesidad y Colecistectomía. Precisa varios ingresos
hospitalarios por cuadros de FA rápida, Ictus lacunar, Insuficiencia Cardiaca e intoxicación por Digoxina. Se observa deterioro progresivo
de la Función Renal. La Ecografía realizada muestra atrofia renal izquierda (diámetro de 76 mm con desestructuración córtico-medular)
y riñón derecho de 118 mm con moderada atrofia cortical. En este momento se diagnostica de Enfermedad Renal Crónica grado 4. Se
detecta hipotiroidismo primario debido a bocio multinodular por tiroiditis autoinmune.

PREGUNTA
Considerando la comorbilidad cual es la patología que complica el uso de levotiroxina.

RESPUESTA
a.- Fibrilacion auricular.
b.- Hipertensión arterial.
c.- Deterioro de la función renal.
d.- Insuficiencia cardiaca.

CASO CLINICO
Paciente de 44 años de edad con síndrome de Down que acude a urgencias por cuadro de disnea progresiva de 2 meses de evolución,
hasta hacerse de reposo, y aumento del perímetro abdominal. Analíticamente se objetiva: TSH, 18.890 μ U /ml y T4L: < 0,15 ng/dl. El
ecocadiograma muestra derrame pericárdico severo con signos de compromiso hemodinámico y colapso de cavidades derechas
confirmado posteriormente con tomografía computarizada toracoabdominal con contraste. En la UCI se realiza pericardiocentesis y
evacuación de 2.400 ml de líquido seroso; los estudios bioquímicos, serológicos, micro-biológicos y de inmunidad son negativos. Se
comienza tratamiento sustitutivo con levotiroxina con buena evolución posterior

PREGUNTA
Cual el pronostico mas probable de este caso.

RESPUESTA
a.- Cardiopatia dilatada.
b.- Bloqueo auricular.
c.- Diestres respiratorio agudo.
d.- Hipotiroidismo subclinico.

CASO CLINICO
Niña de 12 años de edad, que desde hace 4-5 meses inicia con nerviosismo, taquicardia, intolerancia al calor, sudoración profusa e
impulsividad para comer, sin pérdida de peso ni insomnio. Asimismo, refiere abultamiento en la parte anterior del cuello, sin dolor ni
otros signos inflamatorios asociados. Su madre tiene hipotiroidismo en tratamiento con L-tiroxina desde hace 7 años. En la exploración
física destaca una protrusión ocular bilateral y un bocio grado II, de consistencia elástica, sin dolor ni otros signos inflamatorios. El
estudio de hormonas tiroideas mostró hipertiroidismo con anticuerpos antitiroideos elevados. La ecografía tiroidea objetivó ambos
lóbulos tiroideos aumentados de tamaño (derecho 18 x 18,8 x 53mm, izquierdo 15,9 x 18 x 51mm), heterogéneos, con múltiples zonas
hipoecogénicas de bordes imprecisos que confluyen y una marcada hipervascularidad con Doppler.

CURSO ENARM CMN SIGLO XXI TEL: 36246001 Pharmed Solutions Institute PÁGINA 58
MANUAL DE TRABAJO DEL CURSO ENARM CMN SIGLO XXI

PREGUNTA
Cual es la medida terapéutica mas apropiada a largo plazo.

RESPUESTA
a.- Tiroidectomia parcial.
b.- Iodo radioactivo.
c.- Propiltiouracilo.
d.- Metimazol.

CASO CLINICO
Mujer de 66 años de edad, que ingresa por proteinuria, edema generalizado e hipertensión de reciente comienzo, refractaria a
tratamiento. En la analítica destacaban: urea 78 mg/dl, creatinina 1,1 mg/dl; aclaramiento de creatinina de 48 ml/min; proteínas totales
4,6 g/dl; albúmina 1,2 g/dl. Perfil proteico compatible con síndrome nefrótico. Colesterol: 484 mg/dl. Triglicéridos: 180 mg/dl. LDL: 386
mg/dl. Autoinmunidad y complemento dentro de la normalidad. Inmunocomplejos circulantes negativos. TSH: 10,17 mU/l, T4: 0,78
mg/dl. Anticuerpos antimicrosomales: 84 U/ml. Anticuerpos antitiroglobulina: 4 U/ml. Anticuerpos antirreceptor TSH: 1,6 U/l.
Proteinuria hasta 10 g/24 hrs. Se realizó biopsia renal eco-dirigida con resultado anatomopatológico de glomerulonefritis membranosa.
Ante el diagnóstico de glomerulonefritis membranosa probablemente secundaria a tiroiditis de Hashimoto, se comenzó tratamiento
mediante levotiroxina, estatina y doble bloqueo del sistema renina-angiotensina-aldosterona.

PREGUNTA
Cual es la condición que genera el edema en este caso.

RESPUESTA
a.- Proteinuria.
b.- Presion hidrostática.
c.- Glomerulonefritis.
d.- Bloque ECA.

CASO CLINICO
Femenino de 28 años de edad que acude a urgencias debido a que presenta irritabilidad, ideas delirantes de daño, taquicardia,
diaforesis, taquipnea, hipertermia, opresión torácica, vomito y diarrea así como hipotensión arterial, además presenta dolor abdominal
difuso, peristaltismo presente, la paciente presenta piel caliente, delgada, pelo quebradizo y con caída de este, al llegar a urgencias se
observa en delirium, los familiares acompañantes refieren que la paciente tiene antecedentes de enfermedad endocrinológica y se
encontraba en espera de resección de nódulos tiroides.

PREGUNTA
Usted considera por el cuadro clínico y el antecedente referido que se trata de una crisis tirotoxica, usted debe decidir cuál es el
tratamiento más adecuado de primera instancia:

RESPUESTA
a.- Emplear rodio radioactivo inmediatamente.
b.- Usted indica dexametazona.
c.- Emplea metoprolol y metimazol.
d.- Decide indicar Propiltiouracilo.

GLANDULAS PARATIROIDES
CIENCIAS BASICAS: Las glándulas producen hormona paratiroidea (PTH), la cual controla las concentraciones de calcio y fosforo en la
sangre, incrementa la resorción ósea, aumenta la recaptación de calcio y disminución de fosfato, incrementa la absorción
gastrointestinal de calcio, e induce la conversión renal de la vitamina D hasta su forma activa. HIPERPARATIROIDISMO PRIMARIO
(HPP): Trastorno endocrino más frecuente (después de DM), caracterizado por secreción excesiva de hormona paratiroidea, que
condiciona a hipercalcemia crónica, predomina en mujeres que en hombres (4:1), entre 4ta y 5ta década de la vida, origen
desconocido, se ha descrito asociación con irradiación ionizante de cuello, las causas incluyen adenoma 85%, hiperplasia primaria
difusa o nodular 10-55%, y por último el carcinoma <1%. El adenoma paratoiroideo: frecuentemente asociado a patrón familiar,
esporádico por inversión del cromosoma 11 y mutación del gen MEN-1, suelen ser masas únicas de color rojizo–marrón con capsula
bien definida, principalmente de cel. prIncipales y solo afectan una glándula. Hiperplasia primaria: Suele afectar las 4 glándulas, se
manifiesta por aumento de las células prIncipales, los hallazgos clásicos de hiperplasia y la presencia de tejido adiposo es característico:
Carcinoma de paratiroides: infrecuente más común en la 6ta. Década de la vida, bulto en cuello, grande (12g) dificultad al hablar o
tragar, astenia y es muy común este asociado a hipercalcemia severa. PATOGENIA: Existen 3 mecanismos: cambio del punto de
inflexión del sensor de calcio, mayor proporción de cel. Activas reflejando la mayor actividad secretora, y un aumento del número total
de cel. Parenquimatosas. DIAGNOSTICO: Presentación de 2 formas principales; la que cursa con afección renal y la que manifiesta
principalmente una participación ósea. Cuadro clínico: cefalea frecuente, depresión, HTA, incapacidad para concentrarse, hipercalcemia
(22%), normocalcemico (8%). Ósea; dolor, deformidad quistes y susceptibilidad a las fracturas, lo clásico es la osteítis fibrosa quística.
Renal; nefrolitiasis (38%), nefrocalcinosis, disminución del filtrado glomerular y diversas anomalías tubulares, acidosis tubular. Otras
manifestaciones derivan de la hipercalcemia crónica; debilidad muscular proximal, fatiga fácil y atrofia, depresión, cambios de la
personalidad y psicosis, estreñimiento, pancreatitis, osteoartritis degenerativa y avulsión de tendones. Laboratorio: El HPP, es la causa

CURSO ENARM CMN SIGLO XXI TEL: 36246001 Pharmed Solutions Institute PÁGINA 59
MANUAL DE TRABAJO DEL CURSO ENARM CMN SIGLO XXI
mas frecuente de elevación de calcio sérico ambulatorio, hay que recordar la asociación de HPP a tumores, la determinación de PTH en
plasma es el método dx., definitivo para discriminar un HPP de otras causas de hipercalcemia. Las únicas hipercalcemias de origen no
paratiroideo, donde la PTH esta elevada son los tratamientos con litio o diuréticos tiazidicos. El aumento de calcio filtrado a través del
glomérulo conduce a hipercalciuria y aunque la PTH produce un aumento de la reabsorción tubular de calcio, el resultado de ambos
factores contradictorios condiciona a la aparición de calcio en orina, que justifica la mayor frecuencia de litiasis. Rx de torax (masa
mediastinica), densitometría ósea, TAC sesibilidad de 65%. TRATAMIENTO: Indicaciones de tx. médico inicial; calcemia inferior a
11.5mg/ml, ausencia de síntomas en relación directa con la enfermedad o contraindicación para cirugía. Aumentar ingesta hídrica,
reducir calcio de la dieta y movilización inmediata, suprimir fármacos como diuréticos tiazidicos y carbonato de litio. Se pueden usar
calcitonina y bifosfonatos orales (para inhibir los efectos de la PTH), para disminuir la absorción intestinal de calcio y en nefrolitiasis, es
útil la celulosa fosfato sódica. El tx., de HPP es exclusivamente quirúrgico, sobre todo en sus fases sintomáticas con hipercalcemia o
complicaciones viscerales. Son indicaciones absolutas de cirugía; calcemia >11mg7dl, hipofosfatemia grave, disminucion de la función
renal, urolitiasis metabólicamente activa, síntomas objetivos de hipercalcemia , hipercalciuria >400mg/dl, calcificación de tejidos
blandos, evidencia radiográfica de osteítis fibroquistica, ulcera péptica o pancreatitis recidivante. HIPOPARATIROIDISMO: El
hipoparatiroidismo es un trastorno causado por la hipofunción de las glándulas paratiroides, caracterizada por una muy baja
concentración de hormona paratiroidea (PTH), de calcio y un aumento en la concentración de fósforo sanguíneo. La PTH es la hormona
encargada de mantener los niveles normales de calcio en el suero sanguíneo, favoreciendo su absorción en el tracto digestivo por
medio de la activación de la vitamina D3 en el riñón con su subsiguiente migración hacia en epitelio intestinal y la resorción ósea
aumentando indirectamente la actividad de los osteoclastos en el hueso, aparte del incremento de la captación por parte de los túbulos
distales de las nefronas que reabsorben este mineral. CAUSAS: Esta patología puede ser producida por una variada gama de factores,
entre los que encontramos: La extirpación quirúrgica accidental o intencional durante la tiroidectomía, siendo esta la causa más
frecuente. Dentro de las causas no quirúrgicas los procesos autoinmunes son los más importantes, encontrando dentro de ellos el
síndrome poliglandular autoinmune La Ausencia o disfunción congénita de la glándula paratiroides la cual se ha relacionado con
microdelecciones dentro del cromosoma 22q11 (síndrome de DiGeorge). Las deficiencias de magnesio. La Hemocromatosis, causante
de un cuadro de disfunción de los órganos endocrinos debido al acumulo de hierro en el interior de las células. Idiopáticos. PATOLOGÍA:
Siendo el calcio uno de los iones más importantes en el mantenimiento de la homeostasia normal del cuerpo, además de estar con
implicado en la contracción del músculo esquelético estriado, liso y cardiaco, cualquier variación en sus concentración desencadena un
alto número de desordenes funcionales todo esto producido en el caso del hipoparatiroidismo por la disminución del potencial de
acción, lo que lleva al aumento de la excitabilidad celular, favoreciendo en el caso del músculo la aparición de tetania y el SNC de
convulsiones. Además de esto el calcio está relacionado con los procesos de secreción glandular favoreciendo la unión de las vesículas
de secreción a la pared celular promoviendo así su exocitosis, la disminución de este mineral afecta significativamente este suceso lo
que se ve dilucidado en la reducción de la sudoración que presentan los paciente que tienen este proceso patológico. CLÍNICA: La
aparición de síntomas varían entre una persona y la otra, en especial en los niños, sin embargo, los más frecuentes pueden incluir:
Disnea, Dolor cólico agudo, Hormigueos, Contracciones de distintos grupos musculares. Son muy característicos las localizadas en
algunos músculos del brazo y mano. Esta posición incluso puede ser provocada estimulando los músculos del brazo mediante una
compresión con el manguito de un esfigmomanómetro, lo que se conoce como el signo de Trousseau. También aparecen
frecuentemente cataratas, trastornos psíquicos, y piel seca y otros trastornos tróficos de la piel. Los trastornos mentales más
frecuentes de la hipocalcemia aguda causada por el hipoparatiroidismo es el delirium y la psicosis. En la hipocalcemia crónica son más
frecuentes los síntomas de ansiedad, irritabilidad, labilidad emocional, depresión, psicosis, alteración de la memoria y de la
concentración, deterioro cognitivo y a veces retraso mental en función de la edad de comienzo. El compromiso intelectual aparece en
el 50% de los casos de hipoparatiroidismo. Variantes clínicas: Pseudohipoparatiroidismo: En este cuadro, además de los datos descritos,
se añaden trastornos constitucionales como talla corta, cara ancha, y metacarpianos y metatarsianos cortos. Pseudo-
pseudohipoparatiroidismo: En el que sólo aparecen los trastornos constitucionales descritos, sin alteración del calcio. Tratamiento: El
tratamiento consistiría en la administración de la hormona paratiroidea (PTH) pero principalmente de la administración de calcio y
vitamina D en caso de que se trate de la verdadera forma del hipoparatiroidismo la cual presenta bajos niveles de calcio sérico.

CASO CLINICO
Mujer de 77 años que consulta por disnea progresiva de medianos esfuerzos en el último año, ortopnea e hinchazón de los miembros
inferiores. Entre sus antecedentes no había factores de riesgo cardiovascular, no tenía hábitos tóxicos ni recibía ninguna medicación. El
examen físico reveló una presión arterial de 160/100 mmHg, leve cianosis perioral e ingurgitación yugular hasta ángulo mandibular. No
tenía bocio ni adenopatías periféricas. Los tonos cardíacos eran rítmicos, sin soplos, y en la auscultación pulmonar había crepitantes
bibasales. Tenía leves edemas maleolares bilaterales. La exploración neurológica era normal, con negatividad de los signos de
Trousseau y Chvostek. En el hemograma destacaba macrocitosis y en la bioquímica, un calcio corregido con la albúmina de 4,2 mg/dl,
fósforo 8 mg/dl, magnesio 1,6 mg/dl y la función renal normal. La CPK fue de 739 con CPK-Mb normal, GOT 305 y LDH de 1.038.

PREGUNTA
Cual de las siguientes manifestaciones esperaría mas frecuente encontrar en el caso?

RESPUESTA
a.- QT estrecho.
b.- QT alargado.
c.- T picudas.
d.- T aplanada.

CASO CLINICO
Varón de 21 años, con cefalea y vómitos de 24h de evolución. En la EF sólo se encontró un discreto edema y eritema amigdalar.
Leucocitosis con desviación izquierda (14.700leucocitos/μl con 85% neutrófilos). A las 24–48h del ingreso, el cuadro clínico empeoró

CURSO ENARM CMN SIGLO XXI TEL: 36246001 Pharmed Solutions Institute PÁGINA 60
MANUAL DE TRABAJO DEL CURSO ENARM CMN SIGLO XXI
con la aparición de obnubilación y somnolencia. En la nueva analítica, destacaba una calcemia de 15,8mg/dl. Se administró
fluidoterapia, corticoides y aporte adicional de potasio y magnesio, evolucionando favorablemente, con recuperación del estado de
consciencia. El electrocardiograma se normalizó y la sintomatología digestiva desapareció en 24h. PTH 237pg/ml (9–65), calciuria
290mg/24h (5–35), magnesio sérico 1,63mg/dl (1,69–2,29), creatinina 0,46mg/dl (0,53–0,79).

PREGUNTA
Cual es el diagnostico mas probable?

RESPUESTA
a.- Adenoma de paratiroides.
b.- Hiperplasia de paratiroides.
c.- Carcinoma de paratiroides.
d.- Sindrome paraneoplasico.

PREGUNTA
En caso de la presencia de síndrome paraneoplasico, cual es la fuente mas frecuente?

RESPUESTA
a.- Riñon
b.- Pulmon
c.- Cabeza
d.- Cuello

CASO CLINICO
Mujer de 38 años con IRC secundaria a nefropatía tubulointersiticial crónica. A su ingreso destacaban niveles séricos de calcio de 12
mg/dL, Fósforo 6,6 mg/dL, FA 2.490 UI/L, PTHi 1.450 pg/ml. Destacaba la presencia de una tumoración cervical de 3 x 2 x 1 cm situada
en la zona inferior del lóbulo derecho de la glándula tiroides con 4 ganglios linfáticos palpables. Radiológicamente se observaba
resorción subperióstica a nivel de las falanges proximales de ambas manos y una estructura granular a nivel del cráneo. Se ingresó por
un cuadro de astenia, debilidad muscular y dolores óseos generalizados.

PREGUNTA
Cual es la conducta a seguir mas adecuada para el caso clínico?

RESPUESTA
a.- Hidroxido de aluminio.
b.- Quelantes cálcicos.
c.- Vitamina D.
D. Paratiroidectomía.

TRASTORNOS DE GLANDULAS SUPRARRENALES


CIENCIAS BASICAS: Las suprarrenales están compuestas de medula y corteza está dividida en glomerular (produce aldosterona-regula
Na y vol. Extracelular por medio de la retención de H2O, Na y la excreción renal de K), fascicular (produce cortisol-aumenta niveles de
glucosa, metabolismo de proteínas y lípidos, efecto inmunosupresor, antiinflamatorio, actúa en SNC) y reticular (produce cortisol y
esteroides androgénicos: dihidroepiandrostendiona y androstendiona). En médula las células cromafines secretan adrenalina y
adrenalina, cuando la medula produce catecolaminas en exceso conduce a feocromocitoma. HIPERFUNCION DE GLÁNDULAS
SUPRARRENALES: SINDROME DE CUSHING: La causa más común es la iatrogénica, debido a la administración de glucocorticoides. El
Cushing endógeno resulta del exceso de producción de cortisol (y otras hormonas esteroideas). La mayor causa de hiperplasia
suprarrenal bilateral secundaria es la hipersecreción de Hormona adrenocorticotropica (ACTH) por la pituitaria (Enf. de Cushing) o de
fuentes ectópicas tales como carcinoma de cel. Pequeñas de pulmón, carcinoma medular de tiroides o tumores de timo, páncreas,
ovario. Los adenomas y carcinomas de glándulas suprarrenales representan aproximadamente el 25% de los casos de síndrome de
Cushing. CUADRO CLÍNICO: Más común; obesidad central, HTA, osteoporosis, psicosis, acné, amenorrea y DM, pero inespecíficas, más
específicos incluyen hematomas, estrías púrpura, miopatía proximal, la deposición de grasa en la cara y las zonas interescapular (facies
de luna y joroba de búfalo), y virilización, piel delgada y frágil. La hipopotasemia y alcalosis metabólica son prominentes, sobre todo con
la producción ectópica de ACTH. DIAGNÓSTICO: Se requiere de demostración del incremento de cortisol y supresión anormal de
cortisol en respuesta a dexametasona. Para el cribado, medir cortisol libre urinario en 24 hrs. TRATAMIENTO: Para adenoma o
carcinoma suprarrenal requiere escisión quirúrgica, dar glucocorticoides antes y después de la operación, para regular el estrés.
Metástasis y carcinomas irresecables se tratan con mitotano con incrementos graduales de 6g por día dividido c/8-6h. En ocasiones la
citoreducción de carcinoma de pulmón, o resección de otros tumores, podemos remitir el Sx. de Cushing ectópico. Si las fuentes de
ACTH no pueden ser resecadas, hacer adrenalectomía bilateral total administrar ketoconazol (600-1200mg/dl), metirapona (2-3g/d),
mitotano (2-3mg/d), podrían aliviar las manifestaciones por exceso de cortisol. HIPERALDOSTERONISMO: Es causada por
hipersecreción de aldosterona adrenal. PRIMARIO hipersecreción autónoma de aldosterona (suprarrenal) con supresión de los niveles
de renina, puede ser debido a un adenoma suprarrenal 35% (productor de aldosterona=Sx. de Conn) o hiperplasia suprarrenal bilateral
60%, patogenia: aldosterona produce retención renal de sodio y pérdida de potasio. Esto se traduce en la expansión de contenido de
sodio del cuerpo, lo que lleva a la supresión de la síntesis de renina renal. La acción directa de la aldosterona en la nefrona distal
provoca la retención de sodio y pérdida de hidrógeno y los iones de potasio, lo que resulta en una alcalosis hipocalemia, la aldosterona
tiene efectos fisiopatológicos en un rango de otros tejidos, causando fibrosis cardiaca, disfunción vascular endotelial y nefroesclerosis.

CURSO ENARM CMN SIGLO XXI TEL: 36246001 Pharmed Solutions Institute PÁGINA 61
MANUAL DE TRABAJO DEL CURSO ENARM CMN SIGLO XXI
El SECUNDARIO ocurre secundario a elevación de los niveles de renina circulante, esto es típico en falla cardiaca, cirrosis, síndrome
nefrótico, pero podría ser debido a estenosis de la arteria renal o tratamiento diurético, muy raro reninoma. CUADRO CLINICO:
Primario; tienen dolores de cabeza e hipertensión diastólica. El edema es característicamente ausente, a menos que la insuficiencia
cardíaca congestiva o la enfermedad renal está presente. Las pérdidas de potasio urinario, puede causar debilidad muscular y fatiga,
aunque los niveles de potasio pueden ser normales en aldosteronismo primario suave. También se puede producir alcalosis metabólica
e hipernatremia. DIAGNOSTICO: Se sugiere por la hipertensión y es asociado con hipocalemia persistente, en un paciente sin edema,
que no este recibiendo diuréticos ahorradores de K. TRATAMIENTO: La cirugía puede ser curativa en adenoma adrenal, pero no efectiva
en hiperplasia suprarrenal, que se maneja con restricción de Na y espironolactona. El secundario; es tratado con restricción de sal y
corrección de la causa subyacente. HIPOFUNCIÓN DE GLÁNDULAS SUPRARRENALES: La insuficiencia suprarrenal primaria es debida a
falla de la glándula suprarrenal, mientras la insuficiencia suprarrenal secundaria es debida a falla de la producción o liberación de ACTH.
ENFERMEDAD DE ADDISON: Ocurre cuando más >90% del tejido suprarrenal está destruido quirúrgicamente, por enf. granulomatosa
(Tb, histoplasmosis, coccidioidomicosis, criptococosis), vía autoinmune, metástasis bilateral, hemorragia bilateral, CMV,HIV,
Amiloidosis, sarcoidosis. CUADRO CLINICO: Fatiga, debilidad, anorexia, nausea y vómito, pérdida de peso, dolor abdominal,
pigmentación cutánea y mucosa, deseo de sal, hipotensión y ocasionalmente hipoglicemia, si hay depleción del fluido extracelular se
acentúa la hipotensión. DIAGNOSTICO: La mejor prueba de detección es la respuesta del cortisol 60 min después de 250 µg de ACTH IV
o IM. Los niveles de cortisol deben exceder de 18 mg / dl 30 a 60 minutos después de ACTH. Si la respuesta es anormal, la deficiencia
primaria y secundaria se puede distinguir mediante la medición de aldosterona en sangre. TRATAMIENTO: Hidrocortisona 20-30mg/d,
dividido 2/3 en la mañana y 1/3 en la tarde es el pilar de la sustitución de glucocorticoides. La sustitución de mineralocorticoides es
necesaria en la insuficiencia suprarrenal primaria. Durante la crisis suprarrenal se usan dosis altas de hidrocortisona (10mg/h continuas
IV o 100mg bolo IV tres veces al día). HIPOALDOSTERONISMO: Deficiencia aislada de aldosterona. Con producción normal de cortisol,
se produce con hiporeninismo, como un defecto biosintético hereditario, después de la extirpación de los adenomas secretores de
aldosterona, y durante el tratamiento prolongado con heparina. Hipoaldosteronismo hiporreninémico es más frecuente en adultos con
insuficiencia renal leve y la diabetes mellitus en relación con la hiperpotasemia desproporcionada. Fludrocortisona oral, restablece el
equilibrio electrolítico si la ingesta de sal es adecuada. En IRC y falla cardiaca está aprobado el uso de furosemide. MASAS
SUPRARRENALES INCIDENTALES (INCIDENTALOMA): Las masas suprarrenales son común encontrarlas en TAC o RMI. La mayoría (70-
80%), son no funcionantes y la probabilidad de un carcinoma suprarrenal es baja (<0.01%). El primer paso es determinar el estado
funcional y medir metanefrinas libres en plasma para detección de feocromocitoma. Si hay un tumor maligno conocido, hay 30-50%de
posibilidades que el incidentaloma sea una metástasis. Evaluación hormonal adicional debe incluir la prueba durante la noche
dexametasona 1mg supresión de todas los puntos, la renina plasmática, actividad / aldosterona ratio en pacientes hipertensos, DHEAS
en mujeres con signos de exceso de andrógenos y estradiol en hombres con feminización. FEOCROMOCITOMA: Son tumores
adrenomedulares que secretan catecolaminas, son raros, forman <0.1% de las causas de hipertensión, sin embargo es muy importante
su diagnóstico debido a: el desarrollo de una crisis hipertensiva fatal, la reversión de todas las manifestaciones después de remover el
tumor quirúrgicamente, la falta de eficacia a largo plazo de tratamiento médico, la apreciable incidencia de malignidad.
Manifestaciones; hipertensión resistente a tx. convencional, sudoración e intolerancia al calor (80%), palidez o enrojecimiento,
sentimiento de aprensión, pirexia, dolor de cabeza palpitante y constante (65%), parestesias, disturbios visuales, palpitaciones (65%),
dolor de pecho, hipotensión postural. La cirugía es curativa en la mayoría de los pacientes. USOS CLINICOS DE GLUCOCORTICOIDES:
Son usados en una variedad de enfermedades tales como asma, artritis reumatoide y psoriasis. Es casi segura la aparición de
complicaciones (ganancia de peso, HTA, fascies cushinoide, DM, osteoporosis, miopatía, incremento de presión intraocular, necrosis
ósea isquémica, infecciones e hipercolesterolemia), por ello debe valorarse riesgo-beneficio. Estos efectos secundarios pueden
minimizarse mediante la elección cuidadosa de preparaciones de esteroides, alternando días o interrumpiendo terapias; el uso de
esteroides tópicos, inhalados, intranasales o dérmicos siempre que sea posible. Altas dosis de estos pueden ser requeridas durante
periodos de estrés.

CASOS CLINICOS
Paciente de 49 años de edad de sexo masculino con hipertensión severa, cambios de humor, inestabilidad, alternando entre la
agresividad y la depresión. Se observó debilidad muscular progresiva, oscurecimiento de la piel y aumento de la grasa abdominal. Sus
síntomas y pruebas de laboratorio confirmaron un síndrome de Cushing dependiente de ACTH.

PREGUNTA
Cual es la conducta a seguir para determinar el origen?

RESPUESTA
a.- Radiografia de torax.
b.- Tomografia computarizada abdomen.
c.- Resonancia magnetica selar.
d.- USG suprarrenal.

CASO CLINICO
Femenino de 49 años de edad, hipertensa con debilidad generalizada que implica sobre todo sus piernas. Con potasio sérico 1.8mEq/L y
creatina fosfoquinasa 1753 U / L. En ese momento, se había observado aldosterona (336.73pg/ml ). La TAC abdominal mostró tumores
suprarrenales bilaterales, homogéneos, hipodensas, derecha 2,29 cm y 0,92 cm de diámetro. Recibió adrenalectomía laparoscópica
derecha y un tumor de 2x2 cm fue extirpado.

PREGUNTA
Cual de las siguientes alteraciones ECG es mas probable observar en este caso?

CURSO ENARM CMN SIGLO XXI TEL: 36246001 Pharmed Solutions Institute PÁGINA 62
MANUAL DE TRABAJO DEL CURSO ENARM CMN SIGLO XXI
RESPUESTA
a.- QT corto.
b.- T invertida.
c.- U normal.
d.- P aplanada.

PREGUNTA
Cual de las siguientes alteraciones acido-base es mas probable observar en esta patologia?

RESPUESTA
a.- Alcalosis hipocalemica.
b.- Alcalosis hipocalcemica.
c.- Acidosis hipocalemica.
d.- Acidosis hipocalcemica.

CASO CLINICO
Un hombre de 53 años de edad con antecedente de cáncer de colon tratado y dos masas en glándulas suprarrenales de 1 y 2 cm
respectivamente, el paciente presenta hipertensión sin control adecuado tratado con amlodipino y telmisartan. Ingreso por cefalea leve
persistente, con TA 160/90 mmHg, con incremento de los ROT´s, no se observa edema. Se observo potasio serico de 2.6.

PREGUNTA
Cual es el diagnostico mas probable del presente caso?

RESPUESTA
a.- Aldosteronismo secundario
b.- Aldosteronismo primario.
c.- Pseudoaldosteronismo.
d.- Pseudo-pseudoalteronismo

CASO CLINICO
Mujer de 29 años de edad, con hipertensión no controlada e hipopotasemia espontánea desde hace 4 años. En la investigación de las
causas secundarias de la hipertensión, la relación de actividad de la renina-aldosterona se elevó en dos ocasiones separadas,
diagnosticando como hiperaldosteronismo primario.

PREGUNTA
Cual de las siguientes pruebas confirman el diagnostico?

RESPUESTA
a.- Infusion de solución glucosada.
b.- Infusión de solución salina.
c.- Infusion de solución mixta.
d.- Infusion de soluciones hipertónicas.

CASO CLINICO
Una mujer de 68 años de edad con nivel de calcio sérico de (2,95 mM, el valor normal: 2,25 a 2,75 mM) con un bajo nivel de potasio
(1,7 mM, el valor normal: 3.5 a 5.3 mM), un nivel de aldosterona plasmática elevada (0.244 nm, el valor normal: 0,014 a 0,083 Nm), y
un bajo niveles de renina plasmática normal (0.021 pM, el valor normal: desde 0,008 hasta 0,3 pM). TC adrenal mostró un nódulo
redondo de 1 cm en la glándula suprarrenal izquierda (1,1 x 1,0 cm), sugestivo de adenoma suprarrenal.

PREGUNTA
Cual es el diagnostico mas probable de este caso?

RESPUESTA
a.- Hiperaldosteronismo primario.
b.- Hiperaldosteronismo secundario.
c.- Pseudoaldosteronismo.
d.- Hiperaldosteronismo terciario.

PREGUNTA
La paciente se negó a un procedimiento quirúrgico, cual es una medida terapéutica mas adecuada?

RESPUESTA
a.- Indicar amlodipino.
b.- Indicar espironolactona.
c.- Indicar ECA.
d.- Indicar furosemida.

CURSO ENARM CMN SIGLO XXI TEL: 36246001 Pharmed Solutions Institute PÁGINA 63
MANUAL DE TRABAJO DEL CURSO ENARM CMN SIGLO XXI
CASO CLINICO
Paciente de sexo masculino, de 63 años de edad, con antecedentes de tabaquismo activo (40 paquetes/año), hipertensión arterial
crónica y diabetes mellitus tipo 2 en tratamiento con hipoglicemiantes orales, cardiopatía coronaria y enfermedad arterial oclusiva
crónica de extremidades inferiores. Consultó por cuadro de un año de evolución de astenia, adinamia, baja de peso, hiperpigmentación
de piel y mucosas, episodios de lipotimia e hipoglicemia sintomática a pesar de disminución de la terapia hipoglicemiante y
antihipertensiva.

PREGUNTA
Cuál es la conducta a seguir para establecer el diagnostico.

RESPUESTA
a.- Cuantificacion de cortisol.
b.- Cuantificacion de ACTH.
c.- Cuantificacion de CRF.
d.- Cuantificacion de DEAS.

CASO CLINICO
Hombre de 53 años de edad, que consulto por náuseas, vómito y diarrea, en el examen fisico se detecto hipotensión sostenida con
presiones sistólicas en el rango de 60 a 70 mmHg. Por lo cual se inicio hidratación intravenosa, sin embargo, el paciente persistió con
una presión arterial de 70/42 mmHg. Los exámenes paraclínicos documentaron hipoglucemia, hiponatremia e hiperkalemia, por lo cual
se hace la impresión diagnóstica de Enfermedad de Addison.

PREGUNTA
Cual es la conducta a seguir mas apropiada en el caso?

RESPUESTA
a.- Soluciones meatabolicas y corticoides.
b.- Soluciones normotonica y corticoides.
c.- Soluciones fisiológicas, catecolaminas y glucocorticoides.
d.- Soluciones metabolicas, catecolaminas y glucocorticoides.

CASO CLINICO
Paciente masculino de 62 años con antecedentes de ser fumador, que dos meses atrás había comenzado con pérdida del apetito y
fiebre por las tardes de 38 ºC, Mucosas húmedas hipercoloreadas, tonos cardiacos rítmicos. FC 79 x´. TA 160/100. Ultrasonido
abdominal imagen compleja ecolúcida a predominio de cara anterior y porción superior del riñón derecho de 76 mm. El tórax ofrece
datos de interés como la presencia de una radioopacidad no homogénea de contornos irregulares, mas acentuad a la base pulmonar
derecha, fina banda radio opaca, que parte desde la región periférica hacia el hilio. 5 días después del ingreso, el paciente comienza a
presentar cifras tensionales que en algunas ocasiones eran muy altas y otras bajas; en esa oportunidad se aprecia la presencia de
lesiones dermatológicas papulares rojizas, comienza con lesiones purpuro-hemorrágicas y se auscultan crepitantes bibasales. Tensión
arterial (TA) 160/100 Frecuencia cardiaca (FC): 110 latidos/minuto.

PREGUNTA
Cual es el procentaje diagnostico de feocromocitomas de forma directa?

RESPUESTA
a.- 5 %
b.- 10 %
c.- 15 %
d.- 20 %

CURSO ENARM CMN SIGLO XXI TEL: 36246001 Pharmed Solutions Institute PÁGINA 64
MANUAL DE TRABAJO DEL CURSO ENARM CMN SIGLO XXI
SOBREPESO Y OBESIDAD
CIENCIAS BASICAS: La obesidad, incluyendo al sobrepeso como un estado premórbido, es una enfermedad crónica caracterizada por
el almacenamiento en exceso de tejido adiposo en el organismo, acompañada de alteraciones metabólicas, que predisponen a la
presentación de trastornos que deterioran el estado de salud, asociada en la mayoría de los casos a patología endócrina, cardiovascular
y ortopédica principalmente y relacionada a factores biológicos, socioculturales y psicológicos. SALUD PÚBLICA: Obesidad
padecimiento costoso (25% mayor que una persona con peso normal). México ocupa 1er lugar en sobrepeso y obesidad (30% de la
población), 2do EU, en México 31% de la población infantil sufre de sobrepeso y obesidad.
CLASIFICACIÓN:
2
IMC (Kg/m ) CATEGORIA OMS CATEGORIA SEEDO Riesgo de enfermedad EVALUACIÓN: En la evaluación se debe incluir el IMC (índice de
<18.5 Infrapeso Peso insuficiente
masa corporal), medición de la circunferencia de la cintura. Para
18.5-24.9
25.26.9
Normopeso
Sobrepeso
Normopeso
Sobrepeso grado I IMC= peso en Kg/ talla2 mts (kg/m2), esta proporciona una
27-29.9 Sobrepeso grado II (preobesidad) Incrementado medida precisa del contenido de grasa corporal, en adultos sin
30-34.9 Obesidad moderada Obesidad tipo I Alto
35-39.9 Obesidad severa Obesidad tipo II Muy alto importar género. El exceso de grasa abdominal, es un factor de
40-49.9 Obesidad morbida Obesidad tipo III (mórbida) Extremadamente alto
>50 Superobesidad Obesidad IV (extrema) Extremadamente alto
riesgo por si solo, la medición de la circunferencia abdominal (a
nivel de crestas iliacas, cinta ajustada sin comprimir piel, con una
espiración normal), es de particular importancia en pacientes categorizados como "normales" o con "sobrepeso", pero no es necesario
en pacientes con IMC >35kg/m2. Los hombres que presentan circunferencia abdominal >102 cm y mujeres con >88 cm, tendrán un
mayor riesgo de desarrollar diabetes, dislipidemia, hipertensión y enf. cardiovasculares, enf. de vesicula biliar; estos pacientes deben
ser colocados en una categorías de riesgo superior, que aquella que les corresponde por IMC. La incidencia de cáncer endometrial, de
mama, próstata, colorectal, esteatohepatitis, osteoartritis y gota en hombres y mujeres esta incrementado por la obesidad.
PATOGENIA: Puede ser resultado de aumento del aporte calórico o disminución gasto energético o la combinación de ambos. La
susceptibilidad a la obesidad es de naturaleza poligénica, 30-50% de la variabilidad de reservas de grasa puede ser determinada
genéticamente, este es el principal factor. Las causas secundarias de obesidad incluyen: enf. Hipotalámica, hipotiroidismo, Sx. de
Cushing e hipogonadismo. El aumento de peso también es inducido por fármacos, común en aquellos que usan antidiabéticos,
glucocorticoides, agentes psicotrópicos, estabilizadores del humor (litio), antidepresivos, antiepilépticos. Existen factores que al
combinarse con el sobrepeso y la obesidad, colocan al paciente en un altísimo riesgo de morir prematuramente, entre ellos: infartos,
angina de pecho, cirugía de arterias coronarias, ateroesclerosis, DM tipo II (esta coloca al paciente en riesgo absoluto muy alto). Tres o
mas de los siguientes factores de riesgo definitivamente añaden un riesgo absoluto: HTA, tabaquismo, elevación del colesterol LDL,
glucosa en ayuno alterada, antecedente de enf. cardiovascular prematura y edad (hombres >45 y mujeres >55). TRATAMIENTO: En
>25kg/m2 se les sugieren cambios en estilo de vida (dieta, comidas pequeñas, frecuentes, desayuno, ejercicio. La pérdida de peso se
recomienda en pacientes con IMC >30, en quienes tienen IMC 25-29.9 con factores de riesgo o aquellos que se encuentren en la
categoría "alto riesgo" con base en la circunferencia abdominal. El primer objetivo es una reducción de 10% de la masa corporal en un
periodo de 6 meses (a una velocidad de 0.5-1kg por sem), mediante una reducción calórica de 500-1000kcal/día, posteriormente
enfocarse en mantenimiento de peso con la combinación de dieta, actividad física y cambios en la conducta. Esta fase de
mantenimiento se define como una ganancia de peso no mayor a 3kg durante 2años y una reducción sostenida de la circunferencia
abdominal de por lo menos 4 cm. Se usan cambios en el estilo de vida y adicionar farmacoterapia: sibutramina (inhibidor de la
recaptura de norepinefrina y serotonina), produce pérdida de peso de 5-9% en 12 meses, aunque produce alteraciones en la
frecuencia cardiaca y presión arterial, COFEPRIS pidió su retiro e 2010; orlistat (inhibidor de la lipasa intestinal= inhibe la absorción de
grasas), produce pérdida de peso de 9-10%, en 12 meses con cambios en el estilo de vida; metformina tiende a la disminución del peso
corporal. Si logramos la primer meta, nos enfocamos en la tercera fase que es pérdida de peso adicional. En pacientes que no es posible
perder peso, la meta es prevenir mayor ganancia de peso. El tratamiento quirúrgico: Estará indicado según la NOM exclusivamente en
los individuos adultos con obesidad severa e índice de masa corporal >40, o >35 asociado a comorbilidad importante y cuyo origen en
ambos casos no sea puramente de tipo endócrino. Deberá existir el antecedente de tratamiento médico integral reciente, por más de
18 meses sin éxito; salvo ocasiones cuyo riesgo de muerte, justifique el no haber tenido tratamiento previo. Deberá ser resultado de la
decisión de un equipo de salud multidisciplinario. Las cirugías para bajar de peso son A) Restrictivas (limita la cantidad de comida que el
estómago puede contener y frenar el vaciamiento gástrico), pérdida significativa y sostenida por mas de 5 años; aquí encontramos, la
colocación laparoscópica de la banda gástrica ajustable, gastroplastia de banda vertical, , gastrectomía vertical en banda (qx. De
emergencia). B) Cirugías que limitan la ingesta de alimentos y alteran la digestión. La mas famosa es el puente gástrico tipo Y de Roux.
PRONÓSTICO: Personas con obesidad mórbida viven entre 8-10 años menos que quienes tienen un peso normal. Por cada 15kg arriba
del peso ideal se incrementa el riesgo de muerte temprana en 30%. Incrementan riesgo de morbilidad respecto a HTA, dislipidemia, DM
tipo II, EVC, apnea del sueño, enf. arterial coronaría.

CASO CLINICO
Varón obeso de 59 años de edad con hemiplejía izquierda. A los 54 años de edad presento disnea intensa e hipertensión arterial
descontrolada. Recibió el diagnóstico de hipertensión arterial a los 44 años de edad. El examen físico reveló peso de 163,8kg, altura de
1,74 m, índice de masa corpórea 54,1 kg/m2, pulso de 84 lpm, presión arterial de 200/110 mmHg. El shock de punta del corazón fue
palpado en el 6º espacio intercostal, hacia fuera de la línea hemiclavicular izquierda, y la ausculta no reveló ruidos accesorios. Había
soplo sistólico en área mitral y borde esternal izquierdo. El abdomen estaba voluminoso sin visceromegalias. Había edema discreto de
miembros inferiores y los pulsos en estos miembros estaban disminuidos.

PREGUNTA
Cual es la conducta a seguir mas apropiadas.

RESPUESTA
a.- Cirugia bariatrica.
b.- Colocación de banda gástrica.

CURSO ENARM CMN SIGLO XXI TEL: 36246001 Pharmed Solutions Institute PÁGINA 65
MANUAL DE TRABAJO DEL CURSO ENARM CMN SIGLO XXI
c.- Orlistat mas sibutramina.
d.- Dieta estricta, orlistad y metformida.

CASO CLINICO
Mujer de 39 años con hipertensión arterial, en tratamiento con candesartán, y obesidad grado III que comenzó a tratar con sibutramina
12 días antes. Desde el inicio de la toma de medicación refería cifras más elevadas de presión arterial y palpitaciones. El día del ingreso
presentó en reposo dolor intenso retroesternal irradiado a la extremidad superior izquierda y sudoración de unos 15 min de duración,
que cedió tras nitroglicerina sublingual, y llegó a urgencias asintomática. Los máximos séricos de creatincinasa y troponina T fueron 388
UI/l (normal hasta 140) y 0,23 ng/ml (normal hasta 0,035), respectivamente, con curva enzimática típica de infarto agudo de miocardio.
El electrocardiograma realizado sin dolor torácico fue normal durante todo el ingreso. El ecocardiograma no mostró alteraciones de la
contractilidad. La coronariografía mostró coronarias normales

PREGUNTA
Se programará para cirugía bariatrica más liposucción, considerando la comorbilidad, cual es la complicación aguda mas frecuente.

RESPUESTA
a.- Embolia grasa.
b.- Embolia pulmonar.
c.- Insuficiencia cardiaca.
d.- Sindrome de absorción.

CASO CLINICO
Se trata de una mujer de 50 años de edad con obesidad mórbida (peso 105kg, talla 155cm e IMC de 43) y con antecedentes de
trombosis venosa profunda de repetición. Ingresó en Dermatología por lesiones cutáneas ampollosas generalizadas con biopsia cutánea
compatible con eritema polimorfo que se atribuyó al tratamiento anticoagulante. Tras valoración por el servicio de alergología, se
aconsejó su retirada y su no reintroducción. Se indicó tratamiento con heparina de bajo peso molecular, desarrollando importantes
hematomas dolorosos en el sitio de la administración, motivo por el que se suspendió.

PREGUNTA
Cual es la medida terapéutica que presenta menos complicaciones por las características del caso?.

RESPUESTA
a.- Cirugia bariatrica.
b.- Colocación de banda gástrica.
c.- Dieta y ejercicio.
d.- Orlistat.

CASO CLINICO
Paciente varón de 60 años, con síndrome de Alport, microhematuria y proteinuria de 4gr/día, creatinina de 2 mg/dl, urea 123mg/dl,
perdida moderada de la audición, hipertensión arterial tratada con enalapril 20 mg, obesidad con IMC=36,26 (P 120Kg, h 1,81mts),
glucemias en ayunas alteradas (menor a 126mg/dl)Hb glicosilada normal, dislipidemia e hiperuricemia en tratamiento. Se indica dieta
bajas en proteínas, hipocalórica y se agrega al tratamiento losartan en dosis de 75 mg/ día. Se realiza cirugía de by pass gástrico
presentado en su evolución disminución de 35 kg en total llegando a IMC de 25,6. Control nefrológico: creatinina 1,56mg/dl,
proteinuria 0,3 g/dia, Urea 65mg/dl, normotension con enalapril 5 mg/d, se mantiene dosis mínima de hipolipemiantes, normouricemia
y normoglucemias.

PREGUNTA
Cual es la complicación más importante que presenta a largo plazo el paciente?.

RESPUESTA
a.- Insuficiencia renal.
b.- Hipotiriodismo.
c.- Sindrome de mala absorción
d.- Anemia perniciosa.

CURSO ENARM CMN SIGLO XXI TEL: 36246001 Pharmed Solutions Institute PÁGINA 66
MANUAL DE TRABAJO DEL CURSO ENARM CMN SIGLO XXI
DISLIPIDEMIAS
CIENCIAS BASICAS: Definición: son un grupo de trastornos caracterizados por la presencia de concentraciones anómalas de lípidos (LDL,
HDL, triglicéridos) en sangre. Son importantes por su contribución en la génesis de ateroesclerosis, por lo que se relacionan
directamente con la enf. cardiovascular. SALUD PUBLICA: Prevalencia en adultos mexicanos es de 30% para hipercolesterolemia, 40%
hipertrigliceridemia y 50% para hipoalfalipoproteinemia, siendo la combinación de esta última mas hipertrigliceridemia, lo más
frecuente en el paciente diabético. CLASIFICACION: Primarias: Secundarias a causa de obesidad, diabetes mellitus, consumo de alcohol,
anticonceptivos orales, glucocorticoides, falla renal, daño hepático e hipotiroidismo, o empeorar las subyacentes.
CARACTERISTICAS DE LAS PRINCIPALES DISLIPIDEMIAS PRIMARIAS
HIPERCOLESTEROLEMIA AISLADA HIPERTRIGLICERIDEMIA AISLADA COMBINADA
Hipercolesterolemia LDL colesterol elevados; colesterol Hipertrigliceridemia VLDL elevadas, TG 250-750, leve incremento Disbetalipoproteinemia
familiar total de 275-500mg/dl familiar de colesterol <250mg/dl TG 250-500; VLDL elvado
Autosómica dominante Autosómica dominante. Obesidad, Alteraciones en la Apo E
Puede ser a consecuencia de mutación hiperglicemia e hiperinsulinemia son Autosómico recesivo, puede
para el receptor LDL caracteristicos haber Xantomas tuberosos o
Xantomas tendinosos en edad adulta y Asintomático, xantomas eruptivos, puede palmares
xantelasmas, enfermedad vascular estar relacionado con incremento del riesgo Complicaciones: cardiopatía
Complicaciones: cardiopatía isquémica de enfermedad vascular. Complicaciones: isquémica y sx. metabólico
Pancreatitis, sx. metabólico
Hipercolesterolemia LDL elevada, colesterol total Deficiencia de TG = >750
poligénica <350mg/dl, defectos genéticos y lipoprotein lipasa Asintomática recesiva, que a su vez
factores a,mbientales, dieta, edad, familiar deteriora el metabolismo de los
ejercicio. quilomicrones, la acumulación de estos uede
Asintomática, desarrollan enfermedad ser asociada a pancreatitis, dolor abdominal,
vascular, no xantomas hapatoesplenomegalia
DIAGNÓSTICO: Cuadro clínico: Mas asintomáticas, datos que pueden orientar hacia el origen, xantomas tuberosos (nódulos, en codos y
rodillas; disbetalipoproteinemia, hipercolesterolemia familiar), xantomas tendinosos (en tendón calcáneo y extensores de manos; LDL
>300mg/dl), xantomas eruptivos (borde eritematoso y centro blanquecino, confluentes; TG >1000mg/dl), el xantoma estría palmaris o
depósito de lípidos en pliegues de las manos (disbetalipoproteinemia), el xantelasma es una pápula o placa amarillenta en párpados de
forma bilateral. Escrutinios a partir de 20 años y repetirse cada 5 años, dx., se basa en lípidos séricos, con ayuno de 8-10hrs.
NIVELES NORMALES DE LIPIDOS
LDL <100 optimo; 100-129 casi Colesterol total <200 normal; 200- HDL <40 bajo TG <150 normal; 150-199 limitrofe
optimo; 130-159 limitrofe alto; 239 limitrofe alto; >240 alto (hipoalfalipoproteinemia); 40-59 alto; 200-499 alto; >500 muy alto
160-189 alto; >190 muy alto normal; >60 alto
TRATAMIENTO: Determinar número de factores de riesgo cardiovascular (hombre >45, mujer >55 años, historia en familiares de primer
grado de cardiopatía isquémica a edad temprana hombre <55, mujer <65 años, tabaquismo, HTA, HDL <40). El orden en que se deben
perseguir las metas es: primero colesterol LDL, luego triglicéridos y al último colesterol HDL. Es importante señalar que cuando hay TG
>500mg/dl, la meta principal son estos, debido al riesgo de desarrollar pancreatitis. El tx., no farmacológico l( dieta, ejercicio, reducción
de peso), logra disminuir hasta 10% los niveles de colesterol, cuando no se logra control iniciar tx. Farmacológico; en general las
estatinas son el fármaco de elección para la hipercolesterolemia, mientras que los fibratos para hipertrigliceridemia.
Hipoalfalipoproteinemia usar niacina y fibratos. hipercolesterolemia aislada, estatinas, colestiramina, colestipol, niacina. Estatinas;
Inhibidores de la hidroximetil glutaril coenzima A reductasa (prava, atorva, rosuva, sinvastatinas), tienen efecto hipolipemiante,
antiinflamatorio, mejoran función endotelial y disminuye la agregación plaquetaria, sus efectos adversos mas importantes, miositis,
rabdomiolisis (cuando se usan con fibratos se prefiere gemfibrozil, para disminuir este riesgo), hepatotoxicidad, elevación de CPK por
arriba de 10 veces el limite superior normal es una indicación de suspensión de estatinas. Colestiramina y colestipol; quelantes de ac.
biliar, disminuye LDL y aumenta HDL, no usar en hipertrigliceridemia. Fibratos; reducen síntesis de VLDL y LPL, por el hígado
(gemfibrozil, fenofibrato, clofibrato), efectos adversos nausea, exantema, miopatia. Niacina; disminuye síntesis hepática de lipoproteina
con Apo B es el mejor fármaco para elevar HDL, efectos adversos bochorno, náusea, prediabetes, hiperuricemia.

CASO CLINICO
Varon de 78 años de edad con IRC secundaria a hipertension arterial y diabetes mellitus (creatinina serica basal 1,5 mg/dl), exfumador
con hipercolesterolemia. Se diagnostico de cardiopatia isquemica con enfermedad grave de un vaso, llevandose a cabo una angioplastia
y stent. Un mes después de este procedimiento, el paciente acudio a Urgencias por hematuria franca, detectando una elevacion de la
creatinina serica a 4,5 mg/dl, motivo por el que ingreso. En la exploracion fisica se detecto una arritmia en la auscultacion cardiaca y
una cianosis en primer, tercer y quinto dedo del pie derecho. El EGO mostraba sangre +++ y la cuantificacion de proteinas de 24 horas
fue de 1,2 g/dia egresa con colesterol 350, triglicéridos 780, HDL 28.

PREGUNTA
Cuál es la conducta terapéutica a seguir al egreso más adecuada.

RESPUESTA
a.- Bezafibrato y atorvastatina.
b.- Pravastatina, bezafibrato, metformida.
c.- Dieta, ejercicio, bezafibrato y pravastatina.
d.- Dieta, rehabilitación, bezafibrato y pravastatina.

PREGUNTA
El paciente regresa a urgencias 4 semanas después, somnoliento, urémico, con CPK de 1870, hematura macroscópica, mialgias
generalizadas, considerando el cuadro, cual es su diagnostico actual?

RESPUESTA

CURSO ENARM CMN SIGLO XXI TEL: 36246001 Pharmed Solutions Institute PÁGINA 67
MANUAL DE TRABAJO DEL CURSO ENARM CMN SIGLO XXI
a.- Insuficiencia Renal Aguda.
b.- Sindrome Hepatorrenal.
c.- Rabdomiolisis.
d.- Glomerulonefritis.

CASO CLINICO
Paciente de 69 años de edad con antecedentes de HTA de carácter moderada desde hace 8 años aproximadamente, con enalapril 10
mg e hidroclorotiazida 12,5 mg una vez por día. Consulta por cefalea a predominio frontal, la cual se manifiesta desde hace varios días
acompañado de sensación de inestabilidad, mareo que no sabe referir exactamente pero si manifiesta sentirse muy extraño, con
alteraciones de la sensación de la cara como hormigueo y falta de fuerza. Refiere presentar en una ocasión perdida de la visión de ojo
izquierdo durante menos de 1 minuto.

PREGUNTA
Cual es su conducta a seguir en este momento para el caso actual?

RESPUESTA
a.- IRM cráneo contrastada.
b.- Perfil lipidico.
c.- Analitica de electrolitos sericos.
d.- Iniciar antilipemiantes y oxigenadores.

CASO CLINICO
Femenino de 58 años, antecedentes de diabetes tipo II, con diagnóstico de dislipidemia. No presenta enfermedad vascular previa. En
tratamiento con metformina 850 mg/día, AAS 100 mg/día y atorvastatina 10 mg/día, benzafibrato 200mg/dia, acude por dolor
muscular en miembros pelvicos. Padre y tío diabético, hermano con dislipemia. EF: PA: 135/90, IMC: 29, Perímetro cintura: 100 cm, Glu:
198, Col: 410, Tg: 560, HDL: 28, HbAic: 8.7, Io: 143/4.3, Cr: 2.1, CPK: 320, Microalb: 30.

PREGUNTA
Cual de los siguientes elementos son más importantes para considerar rabdomiolisis?

RESPUESTA
a.- Estatinas y fibratos.
b.- Dolor muscular.
c.- Elevacion de CPK.
d.- Valores de creatinina.

PREGUNTA
Cual es el objetivo principal en las dislipidemias?

RESPUESTA
a.- LDL
b.- Trigliceridos
c.- HDL
d.- Glucosa

PREGUTA
Considerando la respuesta anterior cual es la complicación mas importante para establecer el objetivo?

RESPUESTA
a.- Pancreatitis.
b.- Estado hiperosmolar.
c.- Infarto al miocardio.
d.- Enfermedad vascular.

CASO CLINICO
Pte de 51 años, diabético tipo II con glibenclamida 5mg/d + metformina 1700 mg/d y enalapril 20 mg/d. FRP: Sedentarismo y
Tabaquismo (15 cig/d), Antecedentes: Madre y hermano hipertensos, EF: PA: 150/90 mmhg, Peso: 56 kg, Talla: 1.75 mts, Perimétro
Cint.: 103 cm, Rx Tx: Cardiomegalia c/aorta elongada y calcificación botón aórtico. Hto: 44, Urea 31. Cr: 0.8. Glu: 118, HbA1c: 7.4; Col:
207, LDL: 433, TG: 408, HDL: 22; Ac.Urico: 7.8. GOT: 52; GPT: 38; Microalb: 40 mg/24hs.

PREGUNTA
Cual es el objetivo principal en la dislipidemia del caso?

RESPUESTA
a.- LDL
b.- Trigliceridos

CURSO ENARM CMN SIGLO XXI TEL: 36246001 Pharmed Solutions Institute PÁGINA 68
MANUAL DE TRABAJO DEL CURSO ENARM CMN SIGLO XXI
c.- HDL
d.- Glucosa

PREGUNTA
Cual es medicamento de elección para incrementar HDL en el presente caso?

RESPUESTA
a.- Niacina.
b.- Nicotinamina.
c.- Riboflavina.
d.- Hidroxicobalamina.

DIABETES MELLITUS TIPO II


CIENCIAS BASICAS: Definición: Enfermedad sistémica cronico-degenerativa, de caracter heterogéneo con grados variables de
predisposición hereditaria y con participación de diversos factores ambientales y que se caracteriza por hiperglucemia crónica debido a
la deficiencia en la produccion o accion de la insulina lo que afecta al metabolismo intermedio de los hidratos de carbonola, proteínas y
grasas. (def. NOM). Factores de riesgo: Edad, obesidad si esta es central genera mas resistencia a la insulina, insulina(la acantosis
nigricans en pliegues es un dato de inicio de resistencia a la insulina), sexo (M60%; H40%),sedentarismo, dieta hipercalorica. SALUD
PUBLICA: 90-95% DM tipo II, 5-10% DM tipo I. Mortalidad 70 de cada 100,000. 7 de cada 10 diabéticos, muere antes de cumplir la edad
promedio. Es la 7ma causa de muerte, principal causa de insuficiencia renal, causa de ceguera en personas de 20-74 años. PATOGENIA:
Convinacion entre resistencia a la union a la insulina y una inadecuada respuesta secretora, que iniciañmente las celulas b del páncreas
incrementa. Círculo vicioso de hiperglucemia (pospandrial) hiperinsulinismo regulacion a la baja de receptores de insulina
(periféricos). DIAGNÓSTICO: Clinico; poliuria, polidipsia, polifagia, perdida de peso, visión borrosa, susceptibilidad algunas infecciones.
Criterios caso confirmado de diabetes; 1. hemoglobina glicosilada >6.5%. 2. Glucemia plasmatica en ayuno (8hrs) >126mg/dl (normal
60-100mg/dl). 3. Glucemia plasmatica casual >200 mg/dl, en un paiente con sintomas clasicos de hiperglucemia (normal 100-200
mg/dl). 4. Glucemia >200mg/dl a las 2 hrs después de una carga oral de 75mg de glucosa (PTOG). Prediabetes: Persona que tiene
antecedente padre, madre o ambos con estado metabólico entre lo normal y la diabetes glucosa anormal en ayuno 100-125 mg/dl,
PTOG; >140 y <200 mg/dl. (normal <140mg/dl). COMPLICACIONES: En descompensación; estado hiperosmolar, cetoacidosis,
hipoglucemia. A largo plazo; nefropatía = IRC, retinopatia=amaurosis, neuropatía periférica = ulceras, malformacion de Charcot,
neuropatía autonómica = vejiga neurogenica, disautonomia cardiovascular, disfunción sexual, diarrea, estreñimiento. TRATAMIENTO:
Nutricional; disminucion de carbohidratos, incremento en fibra, disminucion de lipidos. Aumentar la actividad física al menos 150
min./semana como caminar o trotar. Perder 7% de peso corporal (2-4 meses)1. Modificaciones al estilo de vida + metformina (usar
pioglitazona por intolerancia), especial en pacietes con IMC > 35, >60 años o con diabetes gestacional previa. Pacientes sin sobrepeso
se puede iniciar con sulfonilureas, no control. 2. metformina + sulfonilurea (1ra eleccion). Metformina + tiazolidendionas (2 elección) no
control. Metformina + sulfonilurea + insulina o metformina + tiazolidendionas + sulfonilureas. (tiazolidendionas contraindicadas en
pacientes con insuficiencia cardiaca o juntas con insulina). sintomas marcados o glucemias o hb glicosilafas muy elevafas iniciar con
insulina con o sin agentes adicionales. PREVENCION: En diabeticos control glicemico con hb glicosilada 2 por años, si hay descontrol
hacer cada 3 meses, es lo que tiene de memoria, valores <7% reduce retinopatia, nefropatia. Pacientes asintomáticos adultos a
cualquier edad, con sobrepeso u obesidad (IMC>25), con uno o mas factores de riesgo, hacer prueba cada 3 meses si salen normales.
Asintomático con glicosa en ayuno >100 y <126; repetir glucosa en ayuno, se repite resultado = prediabetes, sale >126 hacer PTGO.
Prediabetes; seguimiento anual. Paciente con cifras normales sin factores de riesgo cada 3 años. Deteccion de albuminuria una vez por
año y al momento del dx., toda cifra superiora 300mg/dl = albuminuria clinica o macroalbuminuria. VAloracion por oftalmologia una vez
por año. PRONÓSTICO: Las variables que nos pueden hacer predecir el peor pronostico estan relacionadas con el grado de
sedentarismo, coexistencia de IC, descenso del aclaramiento de creatinina (nefropatia principal), y de hemoglobina. REHABILITACION:
MIcro y macroalbuminuria, iniciar trtatamiento con IECAS o ARA II, para nefroproteccion ( sin olvidar medir niveles de K+). Reducir
ingesta de proteinas de 0.8-1 g/kg/día. CLAVES: Biguanidas (Metformina) 1ra. Eleccion en obesos, complicacion mas grave acidosis
láctica. Sulfonilireas (glibenclamida) se eliminan por riñón si hay IRC, se acumula; con una depuración renal <50, ya no dar
hipoglucemiantes por riesgo de hipoglucemia. Insulina glargina, dura 24hrs, se mantienemantiene basal todo el dia.mantiene Insulina
NPH, dura hasta 12hrs, dosis 0.15 UI/kg, 2/3 matutino y 1/3 vespertino. Efecto somogy; hiperglucemia reactiva matutina, secundaria a
una hipoglucemia nocturna.Insulina lispro ultrarapida, ideal para tx. de picos de insulina.

CASO CLINICO
Mujer de 78 años, independiente en actividades de la vida diaria, con escolaridad completa, tiene antecedentes de hipertensión
arterial, hipotiroidismo, hipoacusia y diabetes mellitus tipo 2 (DM 2) manejada con glibenclamida 10 mg y metaformina 1.700 mg al día,
con lo cual lograba glicemias capilares adecuadas. Fue llevada por familiares al Servicio de Urgencia, para evaluación por compromiso
cualitativo de conciencia, no fluctuante, de más de 24 hrs de evolución, sin signos de focalización y con recuperación espontánea
completa. Su familia señaló la presencia cada vez más frecuente de episodios similares en los últimos 2 meses, indicando además,
alteraciones de la memoria reciente, de la conducta y alteraciones autolimitadas de la marcha, sin trastorno del control de esfínteres,
con repercusión funcional por pérdida parcial de su autovalencia.

PREGUNTA
Cuál es el factor más probable que genere la hipoglucemia en este paciente.

RESPUESTA
a.- La glucosuria.
b.- El hipoglucemiante.

CURSO ENARM CMN SIGLO XXI TEL: 36246001 Pharmed Solutions Institute PÁGINA 69
MANUAL DE TRABAJO DEL CURSO ENARM CMN SIGLO XXI
c.- El antipertensivo.
d.- Depuración renal

CASO CLINICO
Mujer de 70 años de edad, hipertensa, diabética de reciente diagnostico durante un examen de rutina y cursando con bacteriuria
asintomatica, procuró atención médica por tos seca, disnea, inapetencia y desánimo. La paciente sabía que era portadora de
hipertensión arterial desde la edad de 37 años, de diabetes mellitus desde los 56 años y de hipertrigliceridemia. A los 56 años de edad,
se quejó de palpitaciones taquicárdicas con duración de 20 minutos, sin síncope. Hacía uso de 75 mg de captopril, 50 mg de
clortalidona, 600 mg de quinidina y 0,25 mg de digoxina. El examen físico reveló peso de 54 kg, altura de 1,49 m, presión arterial de
170/110 mmHg. En el examen físico, fue identificado soplo sistólico +/4+ en área mitral. Lo restante del examen físico no reveló
alteraciones. El ECG mostró ritmo sinusal, sobrecarga ventricular izquierda, infradesnivel de ST en V5 y V6 y presencia de onda U.

PREGUNTA
Cual de los siguientes fármacos tiene mayor evidencia de efecto cardioprotector en esta condición del paciente?

RESPUESTA
a.- IECA´s
b.- ARA II.
c.- Digoxina.
d.- Clortalidona.

PREGUNTA
Considerando las condiciones del caso clínico cual es la indicación mas adecuada para envio a olftalmologia?

RESPUESTA
a.- Disminucion de la función renal.
b.- Incremento de creatinina.
c.- Aumento de microalbuminauria.
d.- Presencia de fosfenos.

CASO CLINICO
Se trata de masculino de 46 años de edad el cual acude a consulta debido a que ha presentado un proceso infecciosos urinario ya
tratado, refiere mejoría completa posterior a una semana de tratamiento antibiótico, a la exploración identifica un perímetro
abdominal de 102 cms debido a ello realiza una glucosa periférica donde se obtiene 130 mg/dl.

PREGUNTA
Cual es la conducta a seguir para sustentar el diagnostico.

RESPUESTA
a.- Curva de tolerancia a la glucosa.
b.- Glucosa en ayuno.
c.- Determinar hemoglobina glucosilada.
d.- Glucosa pospandrial.

CASO CLINICO
Varón de 47 años con DM diagnosticada hace 9 años, en tratamiento actual con esquema insulínico intensivo (una dosis nocturna de
insulina glargina e insulina lispro previo a las comidas). Entre sus antecedentes destacaban gran tabaquismo, dislipidemia mixta en
tratamiento irregular con ciprofibrato 100 mg/día y microalbuminuria de 50 mg/12 h, sin hipertensión arterial en tratamiento con
enalapril 5 mg/día. Consultó en un centro de salud por sensación febril no cuantificada, calofríos, odinofagia, náuseas y vómitos
alimentarios de 2 días de evolución. Se constató deshidratación moderada, temperatura axilar de 37,8°C y glicemia capilar mayor de
400 mg/dl. Fue internado, recibiendo solución fisiológica e insulina en esquema de minidosis. Entre los exámenes de laboratorio
obtenidos al ingreso destacaban glicemia 549 mg/dl, creatinina 1,67 mg/dl, pH 6,91, BE -27, HC03 4,6 (en sangre arterial), cetonemia
+++, natremia 141,7 mEq/L, kalemia 5,92 mEq/L, cloremia 98 mEq/L, amilasemia 48 U/L, hematocrito 40,5%, leucocitos 9100 mm3, VHS
38 mm/h y urocultivo negativo.

PREGUNTA
Que cambios electrocardiográficos es probable esperar.

RESPUESTA
a.- QRS corto.
b.- P-R prolongado.
c.- T picudas.
d.- Ondas U.

CURSO ENARM CMN SIGLO XXI TEL: 36246001 Pharmed Solutions Institute PÁGINA 70
MANUAL DE TRABAJO DEL CURSO ENARM CMN SIGLO XXI
PREGUNTA
Se envio urocultivo y hemocultivo donde se observo crecimiento de e. coli cual es el manejo antimicrobiano mas adecuado
considerando el KDOQI actual?

RESPUESTA
a.- Ceftriaxona y vancomicina.
b.- Ampicilina y amikacina.
c.- Imipenem y gentamicina.
d.- Imipenem, vacomicina y gatifloxacina.

CASO CLINICO
Hombre de 62 años con antecedentes de hipertensión arterial, infarto agudo de miocardio y DM tipo 2. Ingresado por pielonefritis con
IR aguda (MDRD 21 ml/min), interpretándose en el contexto de la propia infección, agravada por el consumo de antiinflamatorios no
esteroideos (AINE) y persistiendo al ser dado de alta. Un mes después presenta lo siguiente (MDRD 13,83 ml/min), proteinuria de 5
g/24 h y microhematuria persistente.

PREGUNTA
Cual es su conducta diagnostica mas útil en este momento?

RESPUESTA
a.- Urocultivo.
b.- Citologia para malignidad.
c.- Fondo de ojo.
d.- Ecografia abdominal.

PREGUNTA
Considerando el MDRD actual en que grado de IR se encuentra?

RESPUESTA
a.- IR grado 4.
b.- IR grado 3.
c.- IR grado 2.
d.- IR grado 1.

CASO CLINICO
Varón de 58 años, con hipertensión arterial en tratamiento farmacológico con regular control y diabetes mellitus tipo 2 en tratamiento
con insulina con buen control metabólico. Con (IRC) y microalbuminuria. Ingresa la tensión arterial era 139/87 mmHg, peso de 81 kg,
talla de 180 cm, urea de 61 mg/dl, creatinina 1,7 mg/dl, aclaramiento de creatinina 44,37 ml/min, microalbuminuria 11,75 ug/min y
hemoglobina de 5,5; albúmina 4,2 g/dl, proteínas totales 6,7 g/dL, colesterol total: 150 mg/dl, triglicéridos: 168 mg/dL.

PREGUNTA
En que estadio de IRC se encuentra?

RESPUESTA
a.- Estadio 1.
b.- Estadio 2.
c.- Estadio 3.
d.- Estadio 4.

CASO CLINICO
Varón de 69 años, con historia de HTA, cardiopatía isquémica, hernia hiatal, gastritis crónica y diabetes mellitus tipo II, con pobre
control metabólico habitual, tratada con hipoglicemiantes orales, alcoholismo crónico, acude por debilidad muscular proximal en las
cuatro extremidades (dificultad para realizar tareas como abducir los brazos, subir escaleras, levantarse desde una silla baja, caminar
distancias moderadas, etc.), de inicio insidioso, simétrico y lentamente progresivo, acompañada de marcada inestabilidad para la
marcha, pérdida de la musculatura distal en las extremidades y ‘saltos musculares’, así como una acentuación de las parestesias
habituales (pinchazos, adormecimientos, etc.) fundamentalmente en horarios nocturnos. Paralelamente, se hallaba aquejada por
disminución bilateral y rápidamente progresiva de la agudeza visual, astenia, poliuria, polidipsia y disminución de peso. Paciente vigil,
bien orientado, con deterioro cognitivo, con lenguaje conservado; marcha ligeramente atáxica; cuadriparesia fláccida simétrica de
predominio proximal, arreflexia osteotendinosa generalizada, hipotrofia muscular distal más acentuada en las manos y antebrazos,
acompañada de fasciculaciones; signo de Romberg positivo.

PREGUNTA
Cual de los siguientes trastornos es mas probable que presenta actualmente el paciente?

RESPUESTA
a.- Demencia por alcoholismo.

CURSO ENARM CMN SIGLO XXI TEL: 36246001 Pharmed Solutions Institute PÁGINA 71
MANUAL DE TRABAJO DEL CURSO ENARM CMN SIGLO XXI
b.- Degeneracion cerebelosa.
c.- Polineuropatia diabética.
d.- Degeneracion pontoolivocerebeloso.

CURSO ENARM CMN SIGLO XXI TEL: 36246001 Pharmed Solutions Institute PÁGINA 72
MANUAL DE TRABAJO DEL CURSO ENARM CMN SIGLO XXI
PANCREATITIS AGUDA Y CRONICA
CIENCIAS BASICAS:
SALUD PUBLICA: En Europa incidencia de 5.4 casos por 100,000 hab, en EU es de 7.9 por 100,000 hab. La mayoría se recupera pero 1-
5% desarrolla un episodio de pancreatitis aguda, y 20% fallece a pesar de tx. Pancreatitis crónica más frecuente entre 50 y70 años, con
una prevalencia de 0.04-5%.
PANCREATITIS AGUDA: El espectro patológico de la pancreatitis
CAUSAS DE PANCREATITIS AGUDA
aguda varía desde pancreatitis intersticial, que suele ser un
Causas comunes: Cálculos biliares (microlitiasis) 30-60%, Alcohol 15-30%,
trastorno leve y autolimitada, a la pancreatitis necrotizante, en el Hipertrigliceridemia, CEPRE, trauma, posoperatorio, drogas (aziatropina,
que el grado de necrosis pancreática se correlaciona con la ciclosporina, tacrolimus, 6-mercaptopurina, sulfonamidas, estrógenos, tetraciclinas,
severidad del ataque y sus manifestaciones sistémicas. ac. valproico, furosemida), disfunción del esfínter de Oddi.
Causas menos comunes: Causas vasculares y vasculitis, desordenes del tejido
PATOGENIA: 3 fases: primera; activación de enzimas pancreáticas conectivo, purpura trombocitopenica trombotica, cáncer de páncreas,
(tripsinogeno, quimiotripsinogeno, fosfolipasas y proteasas), lo huipercalcemia, divertículo periampular, páncreas divisum, pancreatitis herditaria,
cual produce daño tisular, aumento de permeabilidad de fibrosis quística, falla renal.
Causas raras: Infecciones (parotiditis, caxaquievirus, CMV, echovirus), autoinmune
conductos pancreáticos, que permiten salida de enzimas al
intersticio generando autodigestión tisular. La tripsina una vez
activada por la enterocinasa y la catepsina B, produce daño directo, activa otras enzimas y estimula producción de citosinas, las cuales
dañan a distancia. Segunda; químioatracción, activación y secuestro de neutrófilos (puede llevar a necrosis y hemorragia). Tercera;
enzimas proteolíticas y citosinas en órganos distantes, que se manifiestan como sx. de respuesta inflamatoria sistémica. En litiasis
vesicular, es debida al reflujo de material biliar dentro del conducto pancreático o a la obstrucción del flujo de salida, que ocasiona
incremento de la presión intraductal y cambio en la permeabilidad. DIAGNÓSTICO: Cuadro clínico; paciente ansioso e inquieto, dolor
abdominal súbito en epigastrio, región periumbilical (se irradia a espalda y tórax, dolor más intenso en decúbito supino, mejora en
posición de gatillo, sensibilidad abdominal y rigidez), fiebre, náusea, vómito, distensión abdominal, ataque al edo. general. Taquicardia,
taquipnea, hipotensión, disminución de ruidos intestinales, nódulos eritematosos en piel debido a necrosis grasa subcutánea, signo de
Cullen (hematoma periumbilical), signo de Turner (hematoma en flancos), ictericia en casos secundarios a coledocolitiasis. Laboratorio:
amilasa elevada mas de 3 veces lo normal (mas en casos secundarias a litiasis vesicular), lipasa elevada es la de mayor sensibilidad y
especificidad, leucocitosis, hto >44%, hiperglucemia, hipocalcemia (25%), hiperbilirrubinemia, hipertrigliceridemia (15-20%) AST y DHL
elevadas, nivel de proteína C >150mg/dl, habla de necrosis y enfermedad grave. Estudios de imagen, asa centinela es un hallazgo
radiológico sugestivo de pancreatitis aguda (no especifico), ultrasonido; para visualizar páncreas, gas intestinal, litos, pesudoquistes,
edema o crecimiento de páncreas. TAC gold standard para dx., podemos ver zonas de necrosis, la cual tiene una elevada tasa de
infección hasta de 50% y mortalidad asociada hasta de 30%, podemos utilizar criterios de Balthazar. Determinación de gravedad: Con
criterios de Ranson. TRATAMIENTO: La piedra angular es el ayuno (sonda nasoenteral o parenteral), iniciar VO de 3-5 días de iniciado el
tx., soluciones IV abundantes (mas cristaloides), analgésicos, O2, corrección de alteraciones hidroelectrolíticas. CEPRE en pacientes con
pancreatitis secundaria a litos biliares y colangitis. Antibióticos se recomienda en casos con evidencia de necrosis pancreática mayor de
30%, y alta sospecha de infección. El fármaco recomendado es imipenem 500mg tres veces al día por 7 días. Cirugía indicada en casos
de necrosis infectada. COMPLICACIONES: necrosis pancreática (40-60%), pseudoquistes (15%), abscesos pancreáticos, ascitis
pancreática, insuficiencia renal y/o respiratoria, sangrado gastrointestinal, choque séptico.
PANCREATITIS CRÓNICA: Daño permanente en la estructura, como consecuencia de un proceso inflamatoria crónica, superpuesto
sobre un páncreas previamente lesionado, que lleva a fibrosis y perdida de la función. Se puede dividir en calcificante obstructiva (más
común, relacionada con alcohol, hereditaria, hiperlipidemia) y obstructiva (tumores, estenosis o el páncreas divisum). PATOGENIA: Lo
mas común el consumo crónico de alcohol, en niños la principal causa fibrosis quística. Menos relacionada con enf. autoinmunitaria,
hipertrigliceridemia, pancreatitis tropical, obstrucción del conducto e idiopáticas.
DIAGNÓSTICO: Cuadro clínico: Dolor síntoma cardinal. Triada presencia de
calcificaciones pancreáticas, diabetes mellitus y esteatorrea. Mala digestión y dolor
abdominal muy variable, algo característico es la relación que guarda con los
alimentos, por lo que el paciente disminuye los mismos y pierde peso. Para el
desarrollo de esteatorrea es necesaria la pérdida de 90% o más de tejido exocrino, lo
cual disminuye la producción de amilasa, lipasa y proteasas con la consecuente mala
digestión de los nutrientes. No existe un incremento en los niveles de enzimas
pancreáticas, las bilirrubinas en suero y la fosfatasa alcalina pueden estar elevadas.
TAC, calcificaciones
pancreáticas en 30-60%.
CEPRE, provee
información acerca de los
conductos pancreáticos.
COMPLICACIONES: Síntomas por Malabsorción de vitamina B12 en 40%, de
alcohólicos y fibrosis quística, tolerancia a la glucosa alterado, retinopatía
no diabética debido a deficiencia de vitamina A y zinc, necrosis grasa
subcutánea y dolor óseo. Se incrementa el riego de cáncer de páncreas.
TRATAMIENTO: Los ataques intermitentes se tratan como la pancreatitis
aguda. Alcohol, comidas grandes y grasosas deben ser evitadas, narcóticos
en dolor severo, mantener una adecuada hidratación y deberían de ser
hospitalizados, si hay síntomas leves de manera ambulatoria. Cirugía podría
controlar el dolor si hay una estenosis ductal. Pancreatectomia subtotal
podría controlar el dolor pero el costo es insuficiencia exocrina y diabetes.

CURSO ENARM CMN SIGLO XXI TEL: 36246001 Pharmed Solutions Institute PÁGINA 73
MANUAL DE TRABAJO DEL CURSO ENARM CMN SIGLO XXI
CASO CLINICO
Mujer de 34 años sin antecedentes; ingresó en el hospital por pancreatitis aguda biliar leve. La evolución inicial fue favorable, con
comienzo de tolerancia oral al tercer día. Al quinto día presentó dolor abdominal espigástrico y en el hipocondrio izquierdo de inicio
brusco, asociado a palidez, sudoración, taquicardia a 120 lpm y PA de 105/52 mmHg. A la exploración existían signos de irritación
peritoneal. Analíticamente destacaba hemoglobina (Hb) 8,4g/dl, 4 puntos menos que al ingreso. En la TC abdominal se objetivó la
existencia edema pancreático y un gran hematoma subcapsular esplénico de 16 x 13cm, con líquido libre peritoneal denso de alta
cuantía. No se consiguió la estabilización hemodinámica con reposición de volemia y se decidió realizar una laparotomía exploradora
urgente. Tras acceder a la cavidad abdominal a través de laparotomía media, se puso de manifiesto hemoperitoneo masivo (2.500ml),
un gran hematoma periesplénico y un bazo decapsulado en su cara anterior. Se realizó una esplenectomía y una colecistectomía.

PREGUNTA
Cual de los siguientes factores predictivos tiene mayor relevancia para el pronostico inmediato.

RESPUESTA
a.- Hemoglobina.
b.- Leucocitos.
c.- Glucemia.
d.- TAC.

CASO CLINICO
Paciente de 74 años con antecedentes de hipertensión arterial, diabetes mellitus no insulinodependiente y enfermedad de Parkinson.
Acudió a las urgencias hospitalarias por dolor abdominal epigástrico de 3 días de evolución, mareos y síncope. A la exploración
presentaba una PA de 80/40 mmHg y una frecuencia cardíaca de 102 lpm. El abdomen presentaba dolor meso-epigástrico y sensación
de masa. Analítica: amilasemia de 854 U/l, 13.500 leucocitos/μl y Hb de 8g/dl. En la TC se confirmó pancreatitis aguda en la cabeza
pancreática, con desestructuración del parénquima y signos sugestivos de necrosis a ese nivel, complicada con un hematoma
retroperitoneal de 20 x 7 cm, con signos de sangrado arterial activo que situaba en una rama cercana al tronco celiaco.

PREGUNTA
Cual es la complicación crónica mas frecuente.

RESPUESTA
a.- Insuficiencia pancreática.
b.- Anemia crónica.
c.- Diabetes mellitus.
d.- Pancreatitis crónica.

CASO CLINICO
Mujer de 78 años de edad, diabética e hipertensa con dolor abdominal de 24 horas de evolución. Con inestabilidad hemodinámica,
insuficiencia respiratoria grave e irritación peritoneal y equimosis periumblical. Se realiza TAC abdominal compatible con pancreatitis
enfisematosa, preservándose solo la cola pancreática y observando en su lugar aire extraluminal que rodea los vasos regionales, así
como de forma difusa intraabdominal. Múltiples infartos hepáticos. Se realiza cirugía urgente objetivándose líquido libre peritoneal
serohemorrágico con crepitación de tejidos blandos y esteatonecrosis difusa. Se reinterviene donde se observa páncreas
desestructurado con aspecto necrohemorrágico. La paciente presenta mala evolución y fallece a las 48 horas del ingreso.

PREGUNTA
Cual es la complicación aguda mas frecuentes con descenlace fatal.

RESPUESTA
a.- Sepsis abdominal.
b.- Falla organica multiple.
c.- Coagulacion intravascular diseminada.
d.- Tromboembolia pulmonar.

CASO CLINICO
Masculino de 89 años de edad, refiere dolor abdominal con náuseas y vómitos de 2 días de evolución. En la exploración destaca un
abdomen doloroso de forma generalizada con predominio en hemiabdomen derecho. En la analítica se detectan 20.500 leucocitos
(78% neutrófilos, 17 bandas), hematocrito 42%, actividad de protrombina 80%, fibrinógeno 600 mg/dl, bilirrubina 3,3 mg/dl, amilasa
229 U/l, lipasa 310 U/l, GPT 141 mg/dl; sedimento de orina y radiografía de tórax sin alteraciones significativas. Se mantiene en
observación progresando el cuadro clínico requiriendo cirugía y estancia en terapia intensiva, se resuelve el cuadro agudo y es egresado
un mes después de su ingreso.

PREGUNTA
Cual es la sintomatología crónica más probable del caso.

RESPUESTA
a.- Esteatorrea.

CURSO ENARM CMN SIGLO XXI TEL: 36246001 Pharmed Solutions Institute PÁGINA 74
MANUAL DE TRABAJO DEL CURSO ENARM CMN SIGLO XXI
b.- Mala absorción.
c.- Anemia perniciosa.
d.- Hiperglucemia.

ESTEATOSIS HEPATICA (EH):


CIENCIAS BASICAS: Definición: es la acumulación de lípidos histológicamente visible (grasa en hígado), en el citoplasma de los
hepatocitos. Es la alteración más frecuente a nivel hepático. Definición de
ETIOPATOGENIA
esteatosis no alcohólica: entidad anatomoclínica caracterizada por esteatosis
hepática, diferentes grados de inflamación y fibrosis, hallazgos semejantes a ESTRÉS OXIDATIVO
PEROXIDACION LIPIDICA
hepatopatía alcohólica, ausencia de ingesta significativa de alcohol. Se puede
clasificar de acuerdo a su etiología en esteatosis alcohólica y esteatosis no
ESTIMULOS TNF-alfa CELULAS ANTIGENOS
alcohólica (Etiologia: síndrome metabólico, obesidad, diabetes tipo II e QUIMIOTACTICOS IL-6, IL-8 ESTRELLADAS PROTEICOS
hiperlipidemia, rápida pérdida de peso en los obesos, nutrición parenteral
total, síndrome de intestino corto, gastroplastia, hipolipoproteinemias, INFLAMACION FIBROSIS H. MALLORY

tirosinemi, enf. de Wilson, tratamiento con drogas como amiodarona,


estrógenos, tetraciclinas, coticoides, tamoxifeno, nifedipino, lipodistrofia, hígado graso del embarazo). SALUD PUBLICA: La esteatosis
hepática no alcohólica es un padecimiento que afecta al 20-30% de la población general en varios países; la prevalencia aumenta en los
sujetos con obesidad mórbida 75-92%, mientras que en la población pediátrica es de aproximadamente 13-14%. La forma progresiva de
la esteatosis hepática no alcohólica, la esteatohepatitis, se estima que aproximadamente en 3-5% de los casos puede progresar a
cirrosis. En México la prevalencia de esteatosis hepática no se conoce con precisión; sin embargo, si podemos extrapolar datos de
obesidad obtenidos de la encuesta nacional de salud y nutrición (ENSANUT) de 2006; en ella se informa que el 30% de la población
reúne criterios de obesidad y de ellos, más de las dos terceras partes pueden tener esteatosis en el hígado; por otra parte, la
prevalencia promedio de DMT 2 es de 7%, y el 92% de estos pacientes tienen síndrome metabólico y resistencia a la insulina.
CLASIFICACION: Según el porcentaje de hepatocitos afectados, se clasifica en: 1) Leve; menos de 25% de los hepatocitos afectados, 2)
Moderada; de 25-50%, 3) Severa; más de 50%. PATOGENIA: La EH, resulta de un desequilibrio entre la síntesis hepatocitica de
triglicéridos a partir de ácidos grasos y de la secreción desde el hepatocito bajo forma de lipoproteínas. La retención de triglicéridos en
los hepatocitos es requisito indispensable para el desarrollo de esteatosis hepática. El metabolismo de los triglicéridos en el tejido
adiposo da lugar a la liberación de ácidos grasos libres a la circulación y éstos son captados por los hepatocitos. El acumulo de lípidos en
el hígado resulta de una pérdida del balance entre la captación, la síntesis, la salida y la oxidación de los ácidos grasos libres. Estudios
recientes demuestran que el flujo de ácidos grasos libres provenientes del tejido adiposo y que llegan al hígado representa la mayor
fuente de la grasa intrahepática, 62% – 82% de los triacilgliceroles del hígado y que la contribución de lipogénesis “de novo” que en
sujetos normales es menor del 5%, en pacientes con esteatohepatitis aumenta hasta ser del 26%. La insulina, inhibe el metabolismo de
los triglicéridos en el tejido adiposo, aumenta la síntesis intrahepática de ácidos grasos libres y de triglicéridos e inhibe la beta
oxidación de los ácidos grasos libres en los hepatocitos. Los estados de hiperinsulinismo como la DMT2 y la obesidad disminuyen la
sensibilidad tisular a la insulina, esta resistencia a la acción de la insulina es el factor más reproducible en el desarrollo de esteatosis
hepática. Muchos casos son atribuidos al alcohol, su presencia se asocia con un espectro de enfermedades tales como: ingesta de
drogas, tóxicos, obesidad, dislipidemias, diabetes, caquexia y nutrición parenteral. En la esteatosis hepática al igual que en otros
padecimientos metabólicos se altera la liberación de adipocinas. Estas proteínas sintetizadas específicamente en el adipocito,
comparten funciones que regulan energía y procesos de inmunidad. Cuando se altera el lenguaje y comunicación entre el hígado y el
tejido adiposo da como resultado una serie de anormalidades metabólicas e inflamatorias. ESTEATOHEPATITIS: La esteatohepatitis es
una forma más agresiva de esteatosis hepática en la cuál la infiltración grasa del hígado en un porcentaje variable de pacientes (que
puede llegar hasta el 20–30%), se acompaña de intensa actividad necro-inflamatoria y puede progresar a fibrosis y cirrosis hepática y
eventualmente a insuficiencia hepática y hepatocarcinoma. De los factores que participan en la progresión de hígado graso a
esteatohepatitis se reconoce que tanto el síndrome metabólico (Resistencia insulínica / hiperinsulinismo / DM tipo II,Obesidad,
Dislipemia (TG >180 o HDL-col <40), HTA)como la resistencia a la insulina son factores predictivos independientes en esa progresión.
DIAGNOSTICO: Clínico: puede haber dolor en hipocondrio derecho y presentar una hepatomegalia lisa, firme, a veces dolorosa.
Laboratorio: nada especifico, puede haber aumento en las transaminasas y GGT pueden dar sospecha. Imagenología: La ecografía es el
método mas accesible para el diagnóstico de EH ( se observa, aumento de la ecogenicidad hepática “hígado brillante”, acompañado de
una atenuación del haz sonoro en los sectores profundos del hígado). En función de los hallazgos clasificamos a los pacientes en tres
grados: Grado 0: No presenta esteatosis hepática. Grado I: Presenta esteatosis hepática, pudiendo distinguir la grasa periportal. Grado
II: Presenta esteatosis hepática, con borramiento de la grasa periportal. Supone un mayor grado de acúmulo graso. TAC Y RM,
diagnóstico certero, pero altos costos y disponibilidad. Laparoscópico: observación del típico color amarillento de la superficie hepática.
El examen histológico mediante punción biópsica hepática, permite el diagnostico de EH, constituyéndose en el patrón de oro en el
diagnóstico de esta enfermedad. TRATAMIENTO: En el tratamiento propiamente de la esteatosis y esteatohepatitis hay un grupo de
acciones que se aceptan universalmente y que podemos englobarlas como cambios en el estilo de vida, para ello tenemos que
considerar entre otras cosas el consumo de calorías y el tipo de alimentos, la actividad física que ese paciente desarrolla, su situación
socioeconómica, etc. Por experiencia, los médicos sabemos bien lo difícil que es lograr cambios en el estilo de vida de los pacientes. A la
fecha, no hay un tratamiento farmacológico que haya demostrado ser de utilidad precisa, algunas modalidades descritas incluyen:
medicamentos utilizados para bajar de peso, medicamentos que intentan bloquear la resistencia a la insulina, agentes hipolipemiantes,
antioxidantes, citoprotectores, además de la cirugía bariátrica. PRONOSTICO: La esteatosis hepática simple parece tener un pronóstico
relativamente benigno mientras que aquellos con datos de esteatohepatitis o fibrosis en la biopsia del hígado tienen un peor
pronóstico.

CASO CLINICO
Se trata de paciente masculino de 41 años de edad con antecedentes de obesidad grado II, dislipidemia, bajo tratamiento con
bezafibrato, pravastatina y orlistat además refiere ingesta crónica de alcohol, fue diagnosticado con esteatosis hace 6 meses y hernia

CURSO ENARM CMN SIGLO XXI TEL: 36246001 Pharmed Solutions Institute PÁGINA 75
MANUAL DE TRABAJO DEL CURSO ENARM CMN SIGLO XXI
hiatal con mal apego a tratamiento, acude a consulta debido a dolor en cuadrante superior derecho, tos con leve expectoración, con
disminución de ruidos respiratorios en bases de predominio derecho, se observa leve tinte ictérico central. Signos vitales TA 130/100
mmHg, FR 27, FC 98, Temperatura 37.6 grados. Se realiza tele de torax donde se observa elevación del diafragma de predominio
derecho.

PREGUNTA
Cuál es la conducta terapéutica a seguir más adecuada.

RESPUESTA
a.- Iniciar antibiótico de amplio espectro.
b.- Realizar USG de via biliar.
c.- Indica bloqueadores de H.
d.- Realiza panendoscopia.

CIRROSIS HEPATICA:
CIENCIAS BASICAS: Cirrosis es una definición histopatológica, y tienen una variedad de causas, existe lesión hepatocelular, fibrosis
hepática, formación de nódulos de regeneración, dando como resultado decremento en la función hepática. Recordar: Factores de
coagulación dependientes de la vitamina K son: II, VII, IX y X. SALUD PÚBLICA: Causas de cirrosis: alcoholismo (60-70%),
esteatohepatitis no alcohólica (10-15%), hepatitis viral crónica, VHB,VHC (10%), hepatitis autoinmune, cirrosis biliar, cirrosis biliar
primaria (5%), colangitis esclerosante, colangiopatía autoinmune, cirrosis cardiaca (Sx. Budd-Chiari= ICD y pericarditis constrictiva),
hemocromatosis, enfermedad de Wilson, deficiencia de antitripsina α 1, fibrosis quística. PATOGENIA: El principal evento en la génesis
es el aumento en la producción y depósito de tejido de colágeno (I,III, el colágeno tipo I es más raro,). Al ser estimuladas las células
hepáticas estrelladas expresan más RNA mensajero para sintetizar colágeno, esto produce la fibrosis hepática, y está en combinación
con distorsión vascular genera hipertensión portal, la cirrosis se considera irreversible en estadios avanzados; es probable que la
diferencia entre la fibrosis reversible e irreversible sea el tipo de colágeno depositado (tipo I no es degradable, tipo III, sí lo es).
Clasificación histopatológica: micronodular, típica de cirrosis alcohólica, se caracteriza por un tamaño uniforme de nódulos, <3mm de
diámetro. La variedad macronodular >3mm, se presenta secundaria a hepatitis viral o autoinmune
DIAGNOSTICO: Cuadro clínico: Puede estar ausente, anorexia, fatiga,
trastornos del sueño, nausea, vómito, diarrea, dolor vago en cuadrante
superior der. (70% hígado palpable y firme), debilidad, fiebre, ictericia,
amenorrea, impotencia, infertilidad, angiomas en arañas, eritema
palmar, hipertrofia de glándulas paratiroides y lagrimales, contractura
de Dupuytren (fibrosis de la fascia palmar), líneas blancas en la uñas,
ginecomastia, atrofia testicular, hepatoesplenomegalia, ascitis,
sangrado gastrointestinal, encefalopatía hepática. Laboratorio: Anemia
(microcitica, debida a pérdida de sangre, macrocitica, debida a
deficiencia de folato; hemolítica llamada Sx. de Zieve), pancitopenia
(hiperesplenismo), PT prolongado, hiponatremia, alcalosis hipocalemica,
intolerancia a la glucosa, hipoalbuminemia. hiperbilirrubinemia,
aumento de AST, ALT, FA y globulinas, el Us abdominal muestra
hepatomegalia dependiente de lóbulo izq. O caudado, descarta ascitis y
hepatocarcinoma, permite evaluar, vena porta, esplénica y hepática.
Diagnóstico definitivo a menudo depende de la biopsia de hígado
(percutánea, transyugular o abierta). El Child-Pugh es usado para predecir la severidad de la cirrosis y el riesgo de complicaciones.
COMPLICACIONES: 1.-Hipertensión portal: Se define como el aumento sostenido de la presión hidrostática en el sistema venoso portal,
que genera un gradiente de presión entre la vena porta y la vena cava inferior >5mmHg, lo cual favorece el desarrollo de circulación
colateral y derivación del flujo portal hacia la circulación sistémica, es causada por aumento en la resistencia intrahepatica e
incremento en el flujo sanguíneo esplácnico secundario a vasodilatación, sus 3 principales complicaciones son: varices gastroesofágicas
con hemorragia, ascitis e hiperesplenismo. La vena porta está formada por vena gástrica isq., esplénica y mesentérica superior. 2.-
Varices gastroesofágicas con hemorragia: Son vasos colaterales generalmente con un gradiente de presión >12mmHg, para la
prevención primaria del sangrados se utilizan bloqueadores beta no selectivos (propanolol). 3.-Ascitis: acumulación de líquido en
cavidad peritoneal, se considera la complicación más frecuente de la cirrosis, se asocia a una mortalidad de 50% a los 2 años, dx., por
exploración física y USG abdominal, se debe realizar paracentesis para determinar la albumina, proteínas, glucosa, LDH, conteo celular,
tinción de Gramm y cultivo. El tx., consiste en restricción de la ingesta de Na, diuréticos (espironolactona, furosemida). 4.-Peritonitis
bacteriana espontanea (PBE): ocurre en 10-20% de pacientes con cirrosis y ascitis, factores de riesgo, proteínas totales menores a 1g/dl,
antecedente de PBE y sangrado gastrointestinal, se manifiesta con fiebre dolor abdominal y encefalopatía, dx., con conteo celular de
liq. de ascitis debe tener >250 neutrófilos/mm3 en el cultivo lo mas frecuente es E. coli, Klensiella (70%), Enterococos y Streptococcus
(30%). 5.- Encefalopatía hepática. 6.- Otros com: síndrome hepatorrenal tipo I,II , síndrome hepatopulmonar, hipertensión
portopulmonar, malnutrición, coagulopatía, fibrinólisis, trombocitopenia, osteopenia, osteoporosis,, anemia, hemolisis
TRATAMIENTO: La cirrosis hepática compensada no requiere de tratamiento, especifico, solo dieta restringida en proteínas (1-1.2
g/Kg), evitar consumo de alcohol y endoscopia al momento del dx., y periódica, cuando no hay varices esofágicas o son muy pequeñas,
realizar cada 2 años; con varices grado II y III iniciar bloqueadores β, y cuando hay sangrado endoscopia cada 6-12 meses. El único
tratamiento que modifica claramente el pronostico es el trasplante hepático. Trasplante de hígado: Indicaciones: Niños; atresia biliar,
hepatitis neonatal, fibrosis hepática congénita, enfermedad de Angille´s, enfermedad de Byler´s, desordenes inherentes del
metabolismo, enfermedad de Wilson´s, enfermedades de depósito lisosomal, Crigler-Najar tipo I, hipercolesterolemia familiar. Adultos;
cirrosis biliar primaria y secundaria, colangitis esclerosante primaria, hepatitis autoinmune, enfermedad de Caroli´s, cirrosis

CURSO ENARM CMN SIGLO XXI TEL: 36246001 Pharmed Solutions Institute PÁGINA 76
MANUAL DE TRABAJO DEL CURSO ENARM CMN SIGLO XXI
criptogenica, trombosis venosa hepática, hepatitis crónica con cirrosis, hepatitis fulminante, cirrosis alcohólica, malignidad
hepatocelular primaria, esteatohepatitis no alcohólica Contraindicaciones: Infección extrahepatobiliar no controlada, sepsis activa sin
tratar, anomalías congénitas incorregibles que limitan la vida, abuso de sustancias o de alcohol, enfermedad cardiopulmonar avanzada,
malignidad extrahepatobiliar, colangiocarcinoma, SIDA. CIRROSIS ALCOHOLICA: Se debe documentar abuso de alcohol en la historia
clínica, asintomática, clínica >10 años de abuso de alchol. En este tipo de cirrosis la atrofia testicular debida a alteraciones hormonales o
efecto toxico del alcohol. Pueden presentar anemia hemolítica por efecto de la hipercolesterolemia, en la membrana eritrocitica, la cual
genera acantocitos. Es característica la elevación de AST sobre ALT, lo produce relación AST/ALT >2. La biopsia hepática reporta
necrosis, cuerpos de Mllory e infiltración por neutrófilos. ESTEATOHEPATITIS NO ALCOHOLICA: Es la causa mas frecuente de cirrosis
criptogénica. Enfermedad metabólica adquirida que se origina por el depósito de triglicéridos en los hepatocitos, asociada a inflamación
y fibrosis, ciertos factores predisponen como, sexo femenino, obesidad, DM y dislipidemia. Los pacientes suelen presentar dolor en
hipocondrio derecho y hepatomegalia, Laboratorio: hipertransaminemia con predominio de ALT, diagnostico mediante biopsia
hepática. CIRROSIS BILIAR PRIMARIA (CBP): Enfermedad progresiva, con inflamación y destrucción de conductos biliares
intrahepáticos, lo cual produce colestasis crónica y cirrosis; predomina en mujeres (95%), entre 30-65 años de edad, probablemente
debida a un trastorno autoinmune además se asocia con frecuencia a enfermedades autoinmunes. Asintomáticos, manifestación inicial
con frecuencia es prurito, predominio nocturno y se asocia a piel seca, se puede ver ictericia, hiperpigmentación, xantelasma, y
xantomas, hepatoesplenomegalia, malabsorción intestinal. Laboratorio: anemia normocitica normocromica, eosinofilia, elevación de
FA, GGT, AST, ALT, bilirrubinas normales al principio, después elevadas, hipergamaglobulinemia, anticuerpos antimitocondriales (AMA),
presentes hasta en 95%, tienen sensibilidad de 95% y especificidad de 98%. Anticuerpos antinucleares (ANA) en 70% de los casos.
Tratamiento alivi0 del prurito (colestiramina, colestipol, fenobarbital), correcion de las consecuencias de malabsorción intestinal. El tx.,
especifico de la CBP se basa en administración de esteroides y ac. ursodexosicolico (13-15mg/Kg/24h), y en el trasplante hepático.
COLANGITIS ESCLEROSANTE PRIMARIA (CEP): Inflamación y fibrosis de los conductos biliares intra y extrahepáticos, los cuales se
estenosan y obliteran, ocasionando cirrosis hepática. La CEP suele asociarse a colitis ulcerativa crónica inespecífica (40-80%), LES y
arttris reumatoide. Predomina en hombres. Presentan astenia progresiva, prurito e ictericia. El hallazgo característico en la biopsia es la
fibrosis concéntrica periductal (en cascara de cebolla). No hay tratamiento específico, se debe considerar el trasplante hepático, ya que
la supevivencia después del dx., es de 12 años.

CASO CLINICO
Hombre de 65 años, con antecedentes de cirrosis hepática por hepatopatía crónica VHC (Child-Pugh 5). Habia sufrido un episodio de
hemorragia digestiva alta por sangrado de varices esofágicas, resuelto con ligadura. Mientras pasea y de forma aguda presenta dolor
intenso a nivel hipogástrico. A la llegada del equipo de asistencia médica extrahospitalario el paciente ya está inconsciente (GCS 7), con
signos de mala perfusión periférica, intensa palidez y situación de shock. No hay evidencia de hemorragia digestiva alta. Se procede a
intubación y reposición de volemia.

PREGUNTA
Cual de los siguiente sitios de sangrado en la cirrosis hepática es la menos frecuente y muy grave?

RESPUESTA
a.- Hemo-retroperitoneo.
b.- Hemo-peritoneo.
c.- Varices esofagogastricas.
d.- Varices hemorroidales.

PREGUNTA
Considerando la respuesta anterior cuales signos y síntomas son indicativos del sitio de sangrado?

RESPUESTA
a.- Dolor abdominal intenso de inicio brusco y distención súbita.
b.- Distension abdominal súbita y signos de choque.
c.- Manifestaciones de hipovolemia y dolor abdominal.
d.- Signos de choque y dolor abdominal intenso.

CASO CLINICO
Mujer de 70 años con cirrosis hepática por el virus de la hepatitis C, en estadio B de Child, con trombosis portal, episodios de
hemorragia digestiva alta, descompensación hidrópica y un último ingreso por ascitis e hidrotórax secundario, que se resolvió con
tratamiento diurético. Acudió a urgencias por un cuadro de ascitis, edemas y aumento de la disnea de 15 días de evolución. En la
auscultación pulmonar tenía disminución del murmullo vesicular en los dos tercios inferiores del hemitórax izquierdo. En el hemograma
destacaba únicamente la trombopenia (76.000 células/µl) y en la bioquímica, la elevación de la creatinina (1,9 mg/dl), bilirrubina (3,1
U/l), gammaglutamiltranspeptidasa (71 U/l) y fosfatasa alcalina (268 U/l), así como el descenso de la albúmina (2,3 U/l). En la
radiografía de tórax se objetivaba un derrame pleural que ocupaba los dos tercios inferiores del hemitórax izquierdo y sin focos de
compensacion.

PREGUNTA
Considerando el cuadro clínico, cual de las siguientes complicaciones es la mas probable que presenta este caso?

CURSO ENARM CMN SIGLO XXI TEL: 36246001 Pharmed Solutions Institute PÁGINA 77
MANUAL DE TRABAJO DEL CURSO ENARM CMN SIGLO XXI
RESPUESTA
a.- Peritonitis.
b.- Neumonia.
c.- Sindrome hepato-renal.
d.- Derrame pericardico.

CASO CLINICO
Paciente de sexo femenino de 61 años de edad, con antecedentes de diabetes mellitus tipo 2, hipertensión arterial y cirrosis hepática
Child B de etiología no precisada, Consultó por cuadro de una semana de evolución caracterizado por fiebre de predominio nocturno,
cuantificada hasta en 39°C y precedida de un día de diarrea no disentérica y autolimitada. Se evaluó en Servicio de Urgencia, se
realizaron exámenes dentro de los cuales destacan: hemograma sin leucocitosis ni desviación a izquierda, pruebas de coagulación,
electrolitos plasmáticos y función renal normales. A los tres días presenta espectoracion la cual se cultiva con resultado de L.
monocytogenes.

PREGUNTA
Cual es el tratamiento de primera elección ya que el paciente es alérgico a la penicilinas, además de la patologia de base?

RESPUESTA
a.- Cotrimoxazol.
b.- Eritromicina.
c.- Cloranfenicol.
d.- Tetraciclinas.

CASO CLINICO
Paciente de 38 años ex-adicto a drogas por vía parenteral sufre de forma espontánea sin traumatismo previo dolor abdominal de
localización dorsal con irradiación a ambos hipocondrios, hipotensión arterial y anemización importante (5g/dl de hemoglobina), se
realiza TC abdominal ante la sospecha de rotura de anerurisma aórtico, y se evidencia la existencia de rotura de tumoración hepática
(hepatocarcinoma) en segmento VIII sobre un hígado cirrótico y hemoperitoneo masivo. El paciente es intervenido quirúrgicamente de
forma inmediata realizándose alcoholización de la lesión hepática y sutura hemostática.

PREGUNTA
Cuál es el agente etiológico más probable.

RESPUESTA
a.- Virus tipo A
b.- Virus tipo B
c.- Virus tipo C
d.- Virus tipo D

CASO CLINICO
Hombre de 65 años con DHC Child A y HCC único de 2 cm en lóbulo hepático derecho. Sin ascitis al momento del diagnóstico pero con
signos de hipertensión portal (HTP) en la resonancia magnética (RM) y adecuada permeabilidad de la vena porta. Fue definido etapa 1
según la clasificación de Okuda.

PREGUNTA
Cuál es la conducta a seguir mas adecuada.

RESPUESTA
a.- Radioterapia.
b.- Quimioterapia.
c.- Quimioembolizacion.
d.- Transplante hepático.

PREGUNTA
Respecto a las MET´s en hígado, cual de las siguientes observaciones son menos probable?

RESPUESTA
a.- MT´s hepática desde Ca. Mamario.
b.- MT´s Hepática desde CA. de Vesícula.
c.- Lesión solitaria de MT.
d.- Gran metástasis hiperecoicas creando efecto de masa.

CASO CLINICO
Masculino de 74 años de edad, refiere cambios evacuatorios hace 2 meses, liquidas, amarillentas, cuatro por días, sin moco ni sangre, a
la semana se anexa al cuadro fiebre recurrente nocturna, no cuantificada precedida de escalofríos la cual atenúa con medio físicos. A
los 15 días se suma alcuadro dolor abdominal de fuerte intensidad tipo cólico, Antecedente de esplenectomía comoresolución a anemia

CURSO ENARM CMN SIGLO XXI TEL: 36246001 Pharmed Solutions Institute PÁGINA 78
MANUAL DE TRABAJO DEL CURSO ENARM CMN SIGLO XXI
hemolítica a los 46 años. Padre fallece a los 73 años por CA. Gástrico. Hermanos: 12 Vivos, 3 fallecen por CA. Pulmón, CA. Óseo. y CA.
Gástrico. Hijo fallecido por CA. Testicular. Alcohol desde los 22 años hasta los 46 años los fines de semana tipo cerveza llegando a la
embriaguez. Tabáquico: Desde los 14 años hasta los 46 años 10 paquetes por año. Refiere disminución de 20 Kg aproximadamente en 2
meses. Cuello: Adenopatía cervical de 2 cm aproximadamente. Pulmón: Crepitantes bilaterales difusos. Abdomen: Hepatometria
12/17/18 cm, bordes lisos irregulares, Hemograma: WBC 9.7, HGB 10.3, PLT 386. NEUT: 73%, LINF: 21,7%. BILIRRUBINA TOTAL 3,50
mg/dl. BILIRRUBINA DIRECTA: 2,12 mg/dl. FOSFATASA ALCALINA 200 UL. UROANALISIS: Bacterias moderadas, leucocitos 4-8 x cpo.
Trazas proteínas, Pig. Biliares +++, Bilirrubina +++. TGO: 198,3 U/L. TGP: 30,7 U/L. ALBUMINA 2,5; PROTEINA C REACTIVA 36.

PREGUNTA
Considerando el cuadro, la evolución y antecedentes, cual es la condición secundaria que presenta el paciente al padecimiento de
base?

RESPUESTA
a.- Cirrosis
b.- Hepatitis Viral
c.- Adenoma hepático
d.- Hiperplasia adenomatosa

PREGUNTA
Considerando la presencia de los adenomas hepáticos, cual de las siguientes afirmaciones cual es menos frecuente?

RESPUESTA
a.- Un adenoma hepatocelular es un tumor frecuente y nocanceroso del hígado.
b.- Los adenomas hepatocelulares inciden principalmenteen mujeres en edad fértil.
c.- En general, no presenta síntomas, de modo que muchos de los casos no se llegan a detectar.
d.- En casos moderados, una denoma puede hacerse canceroso.

PREGUNTA
Relacionado al carcinoma hepatocelular, cual de las siguientes afirmaciones es mas frecuente?

RESPUESTA
a.- Carcinoma hepatocelular en paciente con cirrosis.
b.- Carcinoma hepatocelular multifocal en paciente sin cirrosis.
c.- Un paciente con Sindrome de Bud-Chiari cronico con una lesion nodular sospechosa.
d.- Mismo paciente con el uso de contraste, ayuda a delimitar mejor la lesion.

CASO CLINICO
Mujer de 26 años de edad. Acudió por un cuadro de una semana de evolución de dolor en hipocondrio derecho que se acompaña de
astenia, náuseas, coluria, acolia, y febrícula. En la exploración física sólo destacaba la presencia de ictericia cutaneomucosa y un
reborde hepático doloroso a la palpación a 4 cm de la arcada costal.

PREGUNTA
Cuál es factor etiológico más probable.

RESPUESTA
a.- Viral.
b.- Alcohol.
c.- Oncologico.
d.- Autoinmune.

CASO CLINICO
Mujer de 77 años diabética, hipertensa y con hepatopatía crónica por el virus de la hepatitis C que consultó en Urgencias por cefalea de
dos días de evolución con posterior aparición de confusión, desorientación, agitacion y fiebre de 39,2°C. Tos productiva, con campos
pulmonares con ruidos crepitantes, FR 12, FC 92, TA 140/100 mmHg.

PREGUNTA
Cuál es la conducta a seguir mas adecuada.

RESPUESTA
a.- Evaluar enzimas hepáticas.
b.- Evaluar funcionamiento renal.
c.- Realizar medidas para encefalopatía.
d.- Iniciar antibióticos de amplio espectro.

CURSO ENARM CMN SIGLO XXI TEL: 36246001 Pharmed Solutions Institute PÁGINA 79
MANUAL DE TRABAJO DEL CURSO ENARM CMN SIGLO XXI
CASO CLINICO
Paciente de 40 años de edad, sexo masculino, con antecedente de depresión en tratamiento con psicoterapia. Consultó con cuadro de
un mes de compromiso del estado general. Una semana antes de consultar presentó dolor en hipocondrio derecho, intermitente, que
no cedía con la ingesta de analgésicos. Tres días antes de consultar presentó fiebre no cuantificada. FC de 110 lx', FR de 29 rpm, temp
de 38,5°C, murmullo disminuido en la base pulmonar derecha y dolor a la palpación en hipocondrio derecho. Leucocitos 12.000
células/mm3, con neutrófilos de 78% y baciliformes de 1%; hemoglobina 12,6 g/dl; velocidad de eritrosedimentación 85 mm/h;
proteína C reactiva (PCR) 239 mg/dl; bilirrubina total 0,34 mg/ di; bilirrubina directa 0,11 mg/dl; fosfatasa alcalina 151 U/L; tiempo de
protrombina 71,9%; amilasa 44 U/L; lipasa 116 U/L. Se realizó ecotomografía abdominal que mostró una lesión de márgenes irregulares
predominantemente hipoecogénica, con zonas hiperecoicas en su interior, de 14 cm x 7 cm, ubicada en el lóbulo hepático derecho,
antígeno carcinoembrionario (ACE), 0,6 ng/ml; alfa feto proteína (AFP), 0,87 ng/ml, CA-19,9, 0,15 U/ml; anticuerpos IgM para hepatitis
A, negativos; antíge-no de superficie de virus de hepatitis B, negativo; anticuerpos anti-hepatitis C, negativos; anticuerpos IgG e IgE para
hidatidosis, negativos; anticuerpos IgG para amebiasis, negativos.

PREGUNTA
Considerando el cuadro clínico y los estudios de laboratorio y gabinete, cual es su impresión diagnostica inicial?

RESPUESTA
a.- Absceso Hepatico.
b.- Quiste Hepatico.
c.- Sarcoma Hepatico.
d.- Hepatocarcinoma.

PREGUNTA
El paciente continuo con fiebre motivo por lo que se instalo tratamiento antibiótico, cual es la conducta diagnostica mas adecuada en
este momento?

RESPUESTA
a.- Realizar Biopsia Percutanea.
b.- Realizar Laparatomia Exploratoria.
c.- Realizar Gamagrafia Hepatica.
d.- Realizar Arteriografia Hepatica.

PREGUNTA
Considerando la respuesta previa se determino un HCTS, cual de las siguientes aseveraciones es mas probable para su desarrollo?

RESPUESTA
a.- La patogénesis de la transformación sarcomatoide del carcinoma hepático no ha sido clarificada. Sin embargo, la evidencia sugiere
que las células fusadas del hepatocarcinoma representan una diferenciación sarcomatosa de las células epiteliales más que una
combinación de sarcoma y hepatocarcinoma.
b.- El mecanismo de degeneración, necrosis y regeneración de las células de carcinoma debidas a fármacos antineoplásicos o
quimioembolización transarterial, han sido postulados como posibles inductores en otros casos.
c.- Los trabajos previos muestran combinaciones con componentes condrosarcomatosos, rabdo-miosarcomatosos y con presencia de
células gigantes tipo osteoclasto.
d.-Los hepatocarcinomas en general, se desarrollan en hígados previamente dañados con cirrosis post virales y de tipo nutricio-
alcohólico. En el caso de los hepatocarcinomas con componente sarcomatoide.

HEPATITIS:
CIENCIAS BASICAS: Definición: Infección sistémica que afecta principalmente el hígado (inflamación). Causada por los virus
hepatotroficos (A,B,C,D,E) que es la causa más común y por otros virus (VEB, CMV, coxackievirus, etc), alcohol, drogas, hipotensión e
isquemia y enfermedades de la vía biliar. Hepatitis aguda: Enfermedad inflamatoria de hígado, de menos de 6 meses de evolución, por
lo general produce anorexia, ictericia, coluria, y alteraciones en pruebas de función hepática. Hepatitis crónica: inflamación persistente
de hígado, de más de 6 meses de evolución, puede desarrollarse de manera favorable o condicionar cirrosis hepática. Hepatitis
fulminante: necrosis masiva del hígado, se manifiesta por encefalopatía hepática aguda, coagulopatía, insuficiencia renal y coma,
aparece en una enfermedad de hígado con menos de 8 semanas de evolución. SALUD PUBLICA: 85% de los pacientes con hepatitis A,
tienen recuperación clínica y bioquímica aprox. a los 3 meses. Cerca de 5% de la población mundial está infectada con VHB, en EU, es
responsable de 5-10% de los casos de hepatopatía crónica y cirrosis. El riesgo de transmisión por punción con aguja es VHB se transmite
en 30% de las exposiciones, VHC en 3% y VIH en 0.3%. La prevalencia mundial es de 0.5-2%, en México es de 1.4% La evolución natural
de la infección por VHC es la hepatitis crónica activa (hasta 74%). El riesgo de presentar carcinoma hepatocelular en pacientes con
cirrosis por VHC es de 1-4% por año. DIAGNOSTICO: Cuadro clínico: Manifestaciones comunes: Malestar, nausea, vómito, diarrea,
fatiga, febrícula seguida de orina oscura, ictericia y hepatomegalia dolorosa puede ser subclínica y detectarse por niveles elevados de
AST y ALT. Manifestaciones poco comunes: vasculitis, artritis, neuritis óptica, anemia aplasica, aplasia de serie roja, y mielitis transversa.
HEPATITIS VIRALES AGUDAS Y CRONICAS: HEPATITIS A: El VHA, es picornavirus RNA no cubierto. Principal vía de transmisión fecal-oral (
alimentos contaminados, guarderías), población de riesgo son niños y adolescente, la infección no suele ser grave (autolimitada),
seguido del proceso se producen anticuerpos contra el virus de la hepatitis A, lo cual confiere inmunidad. Incubación promedio de 30
días, la hepatitis se produce por daño secundario de la respuesta del sistema inmune del huésped mediada por daño citotóxico por
linfocitos T CD8 y células NK. Puede presentarse desde asintomática hasta insuficiencia hepática aguda. DIAGNOSTICO: IgM anti-VHA

CURSO ENARM CMN SIGLO XXI TEL: 36246001 Pharmed Solutions Institute PÁGINA 80
MANUAL DE TRABAJO DEL CURSO ENARM CMN SIGLO XXI
en una muestra de suero de convalecientes aguda o temprana, en la exposición pasada se caracteriza por anticuerpos anti-IgG positivos
con anti-IgM negativos (las cuales se negativizan a los 6 meses), elevación de ALT, AST, bilirrubinas y FA. TRATAMIENTO: Medidas de
soporte, cuando hay complicaciones como falla hepática fulminante, valorar terapia intensiva y trasplante hepático. PREVENCION:
Inmunización pasiva posexposición, inmunoglobulina humana 0.2 ml/Kg IM, protección por 6 meses, indicado en pacientes que
planeen viajar a zona endémica, o contacto íntimo con alguien infectado con VHA. Inmunización activa: Vacuna contra VHA (1ml IM y
refuerzo a los 6 y 12, 0.5ml para niños) indicado en pacientes con enf. hepática crónica, varones homosexuales, drogadictos, personal
de salud que trabaje con VHA. HEPATITIS B: El VHB, es hepadnavirus DNA, está compuesto por la polimerasa con actividad de
transcriptasa reversa, una proteincinasa rodeada por el antígeno del núcleo (HBcAg) y una envoltura que contienen el antígeno de
superficie glucoproteico (HBsAg). El HVB puede causar acción citopatica directa y a través de respuesta inmune. Factores de riesgo son;
contacto sexual, uso de drogas intravenosas (20%), transfusiones, contaminación con suero o leche materna, principal vía perinatal.
Periodo de incubación de 40-150 días, puede presentarse desde asintomática hasta hepatitis fulminante (<1%) Hepatitis aguda:
malestar general, nausea, vomito, anorexia, dolor en hipocondrio der., ictericia que dura de 1-3 meses (70% anictericos). Hepatitis
crónica: Ag de superficie positivo por mas de 6 meses, concentraciones séricas de DNA >10 copias/ml, aumento de aminotransferasas
persistentes, biopsia hepática que muestre inflamación. DIAGNOSTICO: HBsAg en suero infección aguda o crónica (si persiste por mas
de 6 meses). IgM anti-HBc (indica infección aguda o reciente). La prueba más sensible para la detección de HVB DNA en suero (actividad
viral replicativa en suero). Paciente con AntiHBs es una persona sana vacunada. Las transaminasas pueden aumentar hasta 1000 o
2000 y si permanecen por más de 6 meses se vuelve crónica. TRATAMIENTO: En aguda es de sostén, en la crónica, suprimir la
replicación del virus con el fin de evitar progresión a cirrosis y cáncer (realizar tamizaje de alfafetoproteina y ultrasonido hepático cada
6 meses). El interferón alfa-2b, 5-10 millones de U, diarias, 3 veces a la semana SC, por 16 sem. Actualmente se usa alfa-2b pegilado, se
aplica una vez por semana a una dosis de 180mg SC por 48 sem. La lamivudina (3TC), inhibe la transcriptasa reversa, disminuyendo la
carga viral, otros adefovir y entecavir. PREVENCION: Vacunación administrar HBsAg, para estimular la producción de antiHBs, en
individuos no infectados, efectividad >95%; vacunación universal en todos los recién nacidos, así como en trabajadores de la salud,
paciente s en hemodiálisis, familiares y parejas sexuales de pacientes con HBsAg. Vacunación via IM a los 0, 1, 6 meses. HEPATITIS C: El
VHC, es RNA monocatenario. Periodo de incubación de 2 meses. Factores de riesgo, uso de drogas IV, transfusión sanguínea antes de
1990, la transmisión vertical es infrecuente, está mas relacionada con una coinfección con VIH-1, en la madre. La transmisión sexual
tiende a ser menos frecuente que otros virus (VIH), por la menor carga viral en líquidos y tejidos genitales. Las células blanco del VHC
son los hepatocitos y lis linfocitos B. La sintomatología aparece aprox. De 6-8 sem de la exposición, puede pasar asintomático, ictérica,
malestar general, nausea. La infección crónica se caracteriza por presentar periodos prolongados sin manifestaciones, la aguda
progresara a crónica, con diversos grados de hepatitis, fibrosis y cirrosis; entre los factores que promueven la progresión clínica se
observan el consumo de alcohol, coinfección con VIH-1 o VHB, género masculino, y edad mayor al momento de infección. Otras
manifestaciones están asociadas a enf. autoinmuines (tiroideas, DM). DIAGNOSTICO: La prueba de rutina es la serológica mediante
ensayo inmunoenzimático, que detecta anticuerpos a partir de 4-10 sem posteriores a la infección. El inmunoblot se usa como prueba
confirmatoria. Los estudios moleculares se basa en detección del RNA viral mediante PCR, pueden ser cuantitativos (limite menor de
detección 100 copias/ml) o cualitativos, la carga viral nos ayuda con la evolución y respuesta a tx. Los anticuerpos anti-VHC se pueden
detectar en >97% de las personas a los 6 meses posteriores a la exposición. La biopsia es el estándar de oro para determinar la actividad
de VHC, es el único predictor pronóstico de la progresión. TRATAMIENTO: Indicado en: carga viral positiva, biopsia hepática con fibrosis
> grado 1 o enfermedad hepática compensada (CHILD A o B). Contraindicaciones: cirrosis descompensada, embarazo, enf. psiquiátrica,
consumo activo de drogas, enf. pulmonar o cardiaca grave, DM descontrolada, enf. autoinmunes. Fármacos: interferón alfa en
monoterapia o combinado con ribavirina; la combinación confiere 55% de respuesta viral sostenida. HEPATITIS D: El VHD, es RNA
monocatenario, requiere la presencia del VHB para su supervivencia y replicación, la forma más común de contagio es por compartir
agujas en personas que usan drogas IV, también transmisión sexual y perinatal. La coinfección aguda es la infección por exposición
simultánea a VHB y VHD. La superinfección es la exposición del VHD, en un individuo ya infectado con VHB. El diagnóstico con IgM e IgG
antiVHD o por la detección de RNA de VHD en suero. HEPATITIS AIUTOINMUNE: Sospechar ante una hepatitis que presente aumento
de globulinas plasmáticas y autoanticuerpos específicos; es una enfermedad necroinflamatoria crónica del hígado de causa
desconocida, parece haber predisposición genética (HLA-B8, DR3 y DR5). La hepatitis autoinmune se divide en 3 de acuerdo con los
anticuerpos en suero: Tipo I: Clásica o lupoide, más común en mujeres jóvenes, 70% menores de 40 años, los anticuerpos que se
distinguen son los antinucleares (ANA), antimusculo liso (AML), antiactina y antineutrofilos (pANCA) e hipergammaglobulinemia. Tipo II:
Más común en Europa, en especial en niños, aquí encontramos annticuerpos anticromosomas de hígado y riñón tipo 1 (anti-LKM1),
asociada a enf. inmunológicas, tiende a progresar rápido a cirrosis. Tipo III: 90% mujeres entre 20-40 años, se observa anticuerpo anti-
SLA/LP, amplia relación con tiroiditis autoinmune. La hepatitis autoinmune es de progresión lenta, como hepatitis aguda además,
múltiples telangiectasias, estrías, cutáneas, acné, hirsutismo, hepatomegalia, artralgias y amenorrea. Para tx se usan glucocorticoides
de síntesis (prednisona, metilprednisolona), también aziatropina en biterapia con cortoicoesteroides (éxito de 80-90%). HEPATITIS
ALCOHOLICA: Se presenta con niveles de aminotransferasas <300-500 con AST: ALT >2.1 y GGT elevada, y deficiencia concomitante de
vitamina B12, los gravemente afectados presentan al inicio fiebre, hepatomegalia, leucocitosis, ictericia y coagulopatías, asi como
manifestaciones de hipertensión portal, es común que desarrollen cirrosis. El pronóstico se ve en particular afectado por la presencia
de elevadas concentraciones de bilirrubina (>12mg/dl), ascitis e insuficiencia renal. Tx.: indicado en pacientes con encefalopatía,
administración de prednisona, pentoxifilina por un mes disminuye mortalidad e incidencia de sx. hepatorrenal. TOXICIDAD POR
PARACETAMOL: Se presenta con dosis mayores de 10 gr, puede presentarse en pacientes con desnutrición o alcoholismo con 2-6 gr,
parece proveer un reservorio de grupos sulfidrilo y esto obliga a los metabolitos tóxicos o estimulación de la síntesis de glutatión
hepático si la falla hepática se presenta en las primeras 4 hrs, el tx.: consiste en la administración de carbón activado o colestiramina y
lavado gástrico. La N-acetilcisteina debe ser iniciado dentro de las primeras 8 hrs de la ingestión, pero puede ser efectivo incluso en las
24-36hrs después de la sobredosis, se prefiere vía oral con una dosis carga de 140mg/Kg y después 70mg/Kg cada 4 hrs por 17 dosis.
Otros medicamentos que producen hepatitis son: amiodarona. Azoles, isoniacida, metildopa, fenitoina, rifampicina, sulfas. Hay que
observar al paciente de 72-96hrs para evaluar daño hepático.

CURSO ENARM CMN SIGLO XXI TEL: 36246001 Pharmed Solutions Institute PÁGINA 81
MANUAL DE TRABAJO DEL CURSO ENARM CMN SIGLO XXI
CASO CLINICO
Varón de 33 años no fumador con el único antecedente personal de hipercolesterolemia a tratamiento con dieta. Realiza ejercicio físico
de forma regular y su trabajo es sedentario. Refiere malestar general acompañado de cefalea, astenia y fiebre vespertina de hasta 39,2
ºC, acolia y coluria. En la EF no se observa ictericia, se auscultan niveles hidroaereos aumentados, Refiere haber comido ostras vivas
una semana antes, las PFH se encuentran en parámetros elevados, mas un patrón colestasico, se realiza serología resultado positivo a
virus a hepatitis A (IgM positiva e IgG negativa).

PREGUNTA
Cual es la complicación mas frecuente observar en esta patologia?

RESPUESTA
a.- Hepatitis recidivante.
b.- Hepatitis colestásica.
c.- Hepatitis aguda fulminante.
d.- Síndrome post-hepático.

PREGUNTA
Considerando la fisiopatogenia del caso, en cuanto tiempo se puede eliminar el virus por las heces?

RESPUESTA
a.- Una semana antes de la clínica.
b.- Durante la presentación de la clínica.
c.- Una semana después de la clínica.
d.- dos semanas después de la clínica.

PREGUNTA
Habitualmente la acolia y la coluria se presenta cuantos días antes de la ictericia?

RESPUESTA
a.- 1 a 5 dias.
b.- 6 a 10 dias.
c.- 11 a 15 dias.
d.- 16 a 20 dias.

CASO CLINICO
Hombre de 27 años sin antecedentes mórbidos, consultó por compromiso del estado general y fiebre de 5 días, con posterior aparición
de ictericia. Se hospitalizó con bilirrubina total de 13 mg/dl, aminotransferasa oxalacética de 1.977 mU/ml y pirúvica de 5.975 mU/ml,
fosfatasas alcalinas de 157 U/L, y tiempo de protrombina 15%. Del panel viral resultó positivo un HBsAg. Evolucionó con compromiso de
conciencia hasta Glasgow 8, con tomografía computada (TC) de cerebro normal. Fue recibido en la Unidad de Cuidados Intensivos con
Glasgow 5 sospechándose broncoaspiración. Se conectó a ventilación mecánica, se iniciaron antibióticos y N-acetil cisteína. La TC de
tórax reveló neumonía, y la de cerebro edema cerebral leve. La ecotomografía abdominal muestró hígado normal y ascitis leve. Dentro
del estudio etiológico presenta serología para virus hepatitis C y A y estudio de autoinmunidad (anticuerpos y recuento de
inmunoglobulinas) negativos. Se repitió el estudio del VHB destacando aparición de anticuerpos contra el HBsAg (antiHBs) y
negativización del HBsAg, con anticore positivos, IgM y total. Se realizó ELISA para VIH, que resultó positivo, y recuento de CD4 de 634
células/ul. La carga viral (CV) del VHB fue de 140 copias/ml, y la del VIH 16.900 copias RNA/ml. 12 dias después fallece el paciente.

PREGUNTA
Cual de las siguientes observaciones sobre el presente caso clínico explica mas adecuadamente la complicacion del paciente?

RESPUESTA
a.- Falla Hepatica Fulminante por manejo inoportunamente diagnosticada y tratada.
d.- La falla hepatica por VHB con HIV ocurre en aproximadamente 20 %.
c.- En la falla hepática, mientras mayor es la respuesta del hospedero mayor es su probabilidad.
d.- En estados de inmunodepresión como es el tener VIH avanzado, el desarrollo de Falla Hepatica sea infrecuente.

PREGUNTA
Considerando la evolución del caso, cual es el periodo necesario para considerar falla hepática fulminante.

RESPUESTA
a.- Dentro de las 72 horas.
b.- Dentro de la primera semana.
c.- Dentro de las primeras 8 semanas.
d.- Dentro de las primeras 12 semanas.

PREGUNTA
De las siguientes observaciones relacionadas a esta patologia cual es la menos probable?

CURSO ENARM CMN SIGLO XXI TEL: 36246001 Pharmed Solutions Institute PÁGINA 82
MANUAL DE TRABAJO DEL CURSO ENARM CMN SIGLO XXI
RESPUESTA
a.- Mexico se encuentra dentro de los países de baja seroprevalencia.
b.- Se estima que hay almenos de 110,000 portadores crónicos.
c.- El VIH tiene una mayor virulencia que el VHB.
d.- La infección crónica por el VHB después de una exposición aguda es del 5 %.

PREGUNTA
La infección crónica por el VHB, tiene cuatro fases, de acuerdo a la presencia o ausencia del HBeAg, carga viral, nivel de ALT y hallazgos
histológicos, cual de las siguiente no es correcta?

RESPUESTA
a.- Inmunotolerancia.
b.- Portador activo.
c.- Inmunoeliminacion.
d.- Hepatitis crónica HBeAg negativa.

CASO CLINICO
Masculino de 70 años con hepatitis crónica por el VHC genotipo 1 de cronología incierta, infección pasada por el VHB y bebedor de
unos 60 gramos de alcohol al dia. Se observo elevación de GGT y FA, en la ecografía/TAC abdominal convencional en fase venosa se
puso de manifiesto una tumoración de 7 cm de características solidas, en lóbulo hepático izquierdo, sobre el hígado de morfología
normal, la biopsia revelo que se trataba de un CHC, sin presencia clínica ni paraclinica de cirrosis previa y el valor de la alfafetoproteina
era de 6.9, se realizo tratamiento QX.

PREGUNTA
Considerando el caso clínico cual es la tasa de sobrevida al año del presente caso?

RESPUESTA
a.- del 15 %.
b.- del 45 %.
c.- del 75 %
d.- del 95%.

PREGUNTA
Relacionado a la patologia descrita en el caso previo, cual de las siguientes aseveraciones no es cierta?

RESPUESTA
a.- La hepatitis C es una hepatopatía crónica que cursa silenciosa y tiene una prevalencia de 1 a 1.9 en México.
b.- La muerte por infección crónica de VHC están esencialmente relacionadas con la descompensación hepática, enfermedad hepática
terminal y el carcinoma hepatocelular.
c.- Las pruebas serológicas actuales permiten distinguir entre infección aguda, crónica o resuelta, por lo cual se puede llevar un buen
monitoreo.
d.- La infección por el VHC se ha situado como la principal causa de hepatopatía crónica, cerca del 27 % de los casos de cirrosis y el 25 %
de los casos de hepatocarcinomas.

PREGUNTA
Cual de las siguientes aseveraciones no es correcta en el diagnostico de infección por VHC?

RESPUESTA
a.- Durante la fase aguda de la infección la dereccion de acido nucleico puede generar inadecua interpretación de resultados.
b.- Con una prueba anti-VHC positiva se debe realizar prueba PCR-RNA-VHC.
c.- Se debe solicitar prueba de RNA del VHC en los pacientes que se esta considerando la administración de tratamiento antiviral.
d.- Se debe solicitar una prueba anti-VHC en pacientes con VIH para determinar comorbilidad.

CASO CLINICO
Mujer de 62 años que refiere, desde hace 13 años, múltiples episodios de visión borrosa en ambos ojos con recuperación posterior
completa de la agudeza visual tratada con esteoides, TA 100/70mmHg, Glucosa 84 mg/dl, con antecedente de una hermana con el
mismo cuadro, presión intraocular normal, inicia cuadro actual con dispepsia, vomito, prurito, hiporexia, astenia, GOT 110, GPT 90, GGT
110, hipergammaglobulinemia (IgG 1620), ANA 1/320, AMA negativos, Antigeno de superficie VHB, Anticuerpo anti HBs, Anticuerpo
anti-HBc y Anticuerpo anti-VHC negativos, USG hepático normal.

PREGUNTA
Considerando las manifestaciones clínicas asi como los estudios de laboratorio y gabinete, cual es la conducta mas adecuada?

RESPUESTA
a.- Repetir estudios serológicos más sensibles y específicos.
b.- Realizar biopsia hepática percutánea.

CURSO ENARM CMN SIGLO XXI TEL: 36246001 Pharmed Solutions Institute PÁGINA 83
MANUAL DE TRABAJO DEL CURSO ENARM CMN SIGLO XXI
c.- Mantener en observacion y repertir estudios en 6 meses.
d.- Manejo conservador y corticoides.

PREGUNTA
Considerando la respuesta anterior, los antecedentes familiares y personales cual es el diagnostico mas probable?

RESPUESTA
a.- Hepatitis B.
b.- Hepatitis A.
c.- Hepatitis autoinmune.
d.- Hepatitis Ideopatica.

CASO CLINICO
Masculino de 64 años con historia de multiples ingresos por alteraciones del estado de conciencia debido a insuficiencia hepática y
atrofia cortical, acude 14 dias después su ultimo internamiento por alteración del estado de conciencia, por fiebre, desorientación y
dolor abdominal, urea 103, creatinina 2,6. LDH 699, CPK 864, GOT 57, GPT 46. La gasometría revela una acidosis respiratoria. En la
radiografia se observa torax enfisematoso e importante cardiomegalia sin signos neumónicos ni deramme pleural.

PREGUNTA
Se diagnostico como neumonía, considerando el caso, cual de los siguientes agentes es mas frecuente?

RESPUESTA
a.- Streptococcus neumonae.
b.- Stafilococcus aureus.
c.- Listeria monocitogenes.
d.- Campylobacter jejuni.

CASO CLINICO
Masculino de 54 años con DM, artrosis lumbar y hepatopatía alcoholica, acude por dolor abdominal generalizado, fiebre, perdida de 20
kg en 6 meses. Leucocitosis a expensas de neutrofilos, acompañado de trobocitosis, glucosa 263, urea 185, creatinina 1,2. LDH 204 y
PCR 358. La radiografia de torax no muestra imágenes de condensación.

PREGUNTA
Se diagnostico como neumonía, considerando el caso, cual de los siguientes agentes es menos frecuente?

RESPUESTA
a.- Streptococcus neumonae.
b.- Stafilococcus aureus.
c.- Listeria monocitogenes.
d.- Campylobacter fetus.

CASO CLINICO
Masculino de 18 años de edad que consulta de rutina, los laboratorios de rutina reportaron elevación de GOT de 56 y el GPT de 78 las
cuales fueron incrementando lentamente desde hace 16 meses. GGT estaban discretamente aumentada y el resto de parámetros
hepáticos eran normales. No había serología positiva a virus de la hepatitis B y C. Los autoanticuerpos para descartar hepatopatía
autoinmune eran negativos. No hay antecedentes de de hemotransfusion ni quirúrgicos. Hace ejercicio y como único antecedente
refiere tomar finasterida 1 mg diario para evitar la alopecia desde hace 2 años.

PREGUNTA
Cual es la conducta mas apropiada a seguir?

RESPUESTA
a.- Retirar finasterida.
b.- Realizar USG hepático.
c.- Realizar biopsia hepática.
d.- Repetir serología.

CASO CLINICO
Varón de 60 años con antecedentes de hipertensión arterial, adenoma de próstata e hiperuricemia ocasional. Había sido diagnosticado
de miocardiopatía dilatada idiopática hacía 5 años y seguía tratamiento con digoxina, ibopamina, quinapril, espironolactona,
hidralacina, dicumarina, furosemida y nitratos transdérmicos, con lo que se mantenía en CF-II. Presentaba además fibrilación auricular
paroxística por lo que esporádicamente había recibido AMD por vía oral (200 mg/día). No tenía antecedentes de enolismo. Nunca se
detectaron alteraciones de la función hepática. Ingresó por un cuadro de taquicardia ventricular sostenida a 190 por minuto con
morfología de bloqueo de rama derecha. La arritmia cursó con regular tolerancia clínica y sin constatarse en ningún momento signos de
compromiso hemodinámico (hipotensión, oliguria, etc.). Se había restablecido el ritmo sinusal por medio de un bolo de 100 mg de
lidocaína y se había instaurado tratamiento con AMD intravenosa a dosis de 1.200 mg/día durante los 7 días previos. A su llegada a la

CURSO ENARM CMN SIGLO XXI TEL: 36246001 Pharmed Solutions Institute PÁGINA 84
MANUAL DE TRABAJO DEL CURSO ENARM CMN SIGLO XXI
unidad coronaria el paciente presentaba estabilidad hemodinámica con presión arterial de 120/80 mmHg y frecuencia cardíaca
irregular a 90 por minuto con pulsos normales. Se apreciaban a la auscultación cardíaca un tercer sonidos y un débil soplo holosistólico
en foco mitral. No había edemas, ingurgitación yugular o alteraciones en los ruidos respiratorios. Destacaba especialmente una ictericia
franca en piel y conjuntivas, y una evidente depresión del nivel de conciencia con flapping-tremor. No se apreciaron hepatomegalia,
ascitis ni ningún estigma de hepatopatía crónica. En el electrocardiograma se observaba fibrilación auricular a 90 por minuto con
bloqueo de rama izquierda. La radiografía de tórax presentaba cardiomegalia y signos de hipertensión venocapilar pulmonar. En la
analítica al ingreso se observaba como único dato destacable una alanina-aminotransferasa (ALT) de 52,4 µkat/l (normal: 0,46-0,77).

PREGUNTA
Cual es la conducta mas apropiada a seguir?

RESPUESTA
a.- Retirar amiodarona.
b.- Realizar USG hepático.
c.- Realizar biopsia hepática.
d.- Repetir serología

LINFOMAS
CIENCIAS BASICAS: Todas las neoplasias linfoides son malignas y derivan de una célula que ha sufrido mutación maligna son
monoclonales, también alteran la inmunidad. 80-85%, tiene su origen en células B, los derivados de NK o histiociticas son raros. Se
clasifican en Linfoma de Hodgking (EH) y linfomas no Hodgkin (LNH). SALUD PUBLICA: LH: Constituye el 10% de todos los linfomas. Más
frecuente de 25 años y >60 años. LNH constituyen el 3% de todas las neoplasias y son más frecuentes que Los LH (60% de los linfomas
en adultos, edad promedio 64 años) El subtipo histológico más común es el difuso B de cel. Grande (LDGBC)s con incidencia en
individuos infectados por el VIH de más de 100 veces la incidencia en la población en geral (25%), seguido del linfoma folicular.
LINFOMA DE HODGKIN (LH): Origen en cel. B activados del centro germinal. El de celularidad mixta es el subtipo histológico más
frecuente en LH extranodales, mientras que el subtipo esclerosis nodular es predominante en los de afectación nodal. PATOGENIA: Se
sospecha del VEB como agente de transformación maligna, pacientes con mononucleosis infecciosa, tienen mayor riesgo, se reconoce
el genoma de VEB en 70%, también común en casos de inmunodeficiencia. Características; extensión ordenada, localizado, rara vez
afecta ganglios mesentéricos, rara extensión extraganglionar, muy común dolor en ganglios linfáticos, después del consumo de alcohol
(<10%), en la variante esclerosis nodular DIAGNÓSTICO: Típico escenario paciente joven, masculino, linfadenopatías periféricas no
dolorosas (70%), especialmente en región cervical (60-80%) o masa mediastinal. Cuadro clínico: Síntomas B: pérdida de peso
inexplicable de más de 10%, en los últimos 6 meses; fiebre inexplicable >38°C, de duración mayor a 2 semanas; sudoración nocturna
profusa. Prurito en 10-15% de los casos, no es síntoma B. Fiebre Pel- Ebstein; fiebre cíclica que dura de 3-5 días, seguida de un periodo
equivalente sin fiebre. Biopsia excisional ganglionar cel de Reed Sternberg 2 o más núcleos con nucléolo prominente.
Inmunohistoquimica: positivos para CD15 y 30, nos ayuda a diferenciar entre LNH. PCR en ganglio detecta VEB en 60-80%. Tac de tórax,
abdomen y pelvis, PET. ESTADIFICACION: La clasificación mas usada es la Ann Arbor- Costwold.

ESTADIO I Afecta una sola región de ganglios linfáticos I


Afecta a un solo órgano extralinfático IE
ESTADIO II Afecta 2 o mas áreas ganglionares en el mismo lado del diafragma II
Afecta a un solo órgano extralinfático y sus ganglios regionales al IIE
mismo lado del diafragma
ESTADIO III Afecta áreas ganglionares a ambos lados del diafragma
Puede acompañarse de afección esplénica IIIS
Órganos extralinfático localizados IIIE
ESTADIO IV Afecta de forma difusa a uno o más órganos extralinfático (hígado,
pulmón, medula ósea) con o sin involucro ganglionar
Agregar A cuando no se presentan síntomas y B si la persona refiere síntomas B
TRATAMIENTO: Radioterapia en estadios tempranos sin factores desfavorables. Quimioterapia; tratamiento de elección para casos
localizados (IA y IIA), 2 ciclos de quimioterapia con ABVD (adriamicina, vinplastina, bleomicina, y decarbazina)+ radioterapia en caso de
urgencias avanzadas ( sx. de vena cava superior, obstrucción de vía aérea). Estadios III y IV quimio combinada. PRONOSTICO: Factores
de mal pronóstico: Hb <11mg/dl, albúmina <4g/dl, masculino, edad >45 años, adenopatía mediastinal, estadio IV, leucocitosis >15,000,
linfocitosis <500, histología del tipo celularidad mixta, presencia de síntomas B. Después de 3 años sin enfermedad sobrevida >53%.
Tasa de curación 65%, para pacientes con trastornos avanzados
LINFOMA NO HODGKING (LNH): Tumores sólidos malignos de los tej. Linfoides, suelen provenir de cel. B (80%) Diseminación menos
previsible, generalmente diseminada al realizar el dx., suele afectar ganglios mesentéricos, frecuentemente afecta tej. extraganglionar.
Entre los factores de riesgo están ; inmunodefoiciencias (LNH sistémico puede afectar el miocardio, particularmente en pacientes
inmunocomprometidos), tratamientos con radioterapia o quimioterapia, enfermedades autoinmunes y enfermedades previas. Los
linfomas no hodgkiniano son la causa más frecuente de linfoma de cabeza y cuello. Se clasifican en bajo grado (transformación en
agresivos, no curables, edad avanzada, sobrevida larga a 8 años) y agresivos: potencialmente curables, sobrevida corta si no remiten;
cuadro clínico, síntomas B crecientes. Dentro de los de bajo grado; linfoma folicular, sus células neoplásicas se parecen a cel. B
normales, adenopatías indoloras generalizadas, incurables. LNH de bajo grado Linfoma de la zona marginal (MALToma= tumores de los
tej. linfoides asociados a mucosas) se origina, en bazo, ganglios y tej. extraganglionares, afectados por procesos inflamatorios
autoinmunitarios. LNH agresivo en general, debilidad, fatiga, pérdida de peso, 50% pérdida de peso, hepato y esplenomegalia,
hiperviscosidad (que genera mareos, sordera, cefalea, formación de complejos y crioglobulinemia) linfoma difuso de cel. B grandes;
afecta una sola región ganglionar o extraganglionar, primera manifestación tubo digestivo, piel, hueso cerebro. LNH agresivo Linfoma
de Burkit; tumor de cel. B relativamente maduras, translocación del gen c- myc en cromosoma 8, en tej. extraganglionar, el LB africano

CURSO ENARM CMN SIGLO XXI TEL: 36246001 Pharmed Solutions Institute PÁGINA 85
MANUAL DE TRABAJO DEL CURSO ENARM CMN SIGLO XXI
endémico; masa en mandíbula, vísceras abdominales (riñones, ovarios), agresivo pero responde a quimioterapia fácil. DIAGNOSTICO:
Sospecha clínica, biopsia de nódulo o órgano involucrado. Estos se diagnostican por resultados de laboratorios anormales; citopenias,
hipercalciemia, hiperuricemia, proteinemia o LDH elevada) TRATAMIENTO: LNH; linfoma folicular R-CHOP (rituximab, ciclofosfamida,
hidrixidaunomicina, vincristina y prednisona). LNH agresivos; R-CHOP más de 50% de curación. En todos los linfomas en casos
refractarios trasplante de medula ósea. PRONÓSTICO:. LNH agresivos; tasa de remisión de 60-80% , para estadios II al IV. PREVENCION:
Hombre almacenar espermas, mujeres no embarazarse en 2 años posteriores.

CASO CLINICO
Una paciente de 27 años, del sexo femenino, se presentó en la Sala de Emergencias con palpitaciones de inicio súbito y disconfort
torácico. La presión arterial (PA) era de 68 x 40 mmHg y la frecuencia cardíaca (FC) era de 184 lpm. Al examen físico, la paciente
presentaba aumento de volumen de la mama izquierda y masa abdominal difusa. El electrocardiograma (ECG) de 12 derivaciones
realizado en la admisión reveló taquicardia de complejo ensanchado con estándar de bloqueo de ramo izquierdo y concordancia
negativa en los electrodos precordiales. Con base en la presentación inicial, el diagnóstico de taquicardia ventricular monomórfica fue
establecido y la paciente fue sometida a cardioversión eléctrica para el ritmo sinusal normal. El ECG fue repetido y mostró ritmo sinusal
normal, sin alteraciones del segmento ST (Figura 1B). La paciente recibió una infusión intravenosa de amiodarona y entonces fue
admitida en la Unidad de Terapia Intensiva (UTI). La investigación fue negativa para isquemia miocárdica o embolia pulmonar. Una
ecocardiografía transtorácica (ETT) reveló funciones ventriculares izquierda y derecha normales, derrame pericárdico posterior difuso y
engrosamiento del miocardio basal-septal, que no estaba presente en examen anterior realizado un año antes para investigación de
disnea durante actividad física.

PREGUNTA
Cuál de las siguientes condiciones patológicas previas es más probable para el desarrollo del LNH (LDGCB)?

RESPUESTA
a.- Carcinoma metastasico.
b.- Carcinoma pulmonar.
c.- Antecedente de VIH.
d.- Lupus eritematoso sistémico discoide.

PREGUNTA
¿En pacientes con LNH, cual es la presentación mas común de compromiso cardíaco?

RESPUESTA
a.-Dolor torácico
B.-Insuficiencia cardíaca congestiva
C.-Derrame pericárdico
D.-Bloqueo atrioventricular

PREGUNTA
¿Cuál es el manejo inicial de la taquicardia ventricular en este caso?

RESPUESTA
a.- Cardioversión eléctrica
b.- Verapamilo
c.- Amiodarona
d.- Desfibrilación

CASO CLINICO
Varón de 39 años con antecedentes de amigdalectomía en infancia, sin hábitos tóxicos referidos y abuela materna con leucemia
linfática crónica. El paciente es visto en consulta por insuficiencia respiratoria nasal y ronquido asociado a fatiga diurna, más acentuada
en el último año, sin otra clínica asociada. Es trabajador a turnos y conductor profesional por lo que se solicita interconsulta en la
Unidad de Trastornos Respiratorios del Sueño, con diagnóstico de trastorno de sueño leve. En la exploración se visualiza, por rinoscopia
anterior, una desviación septal derecha con hipertrofia turbinal compensatoria. En nasofibroscopia, se identifica una masa en cavum,
compatible con hipertrofia adenoidea. La rinomanometría demuestra mayor resistencia aérea en fosa nasal derecha, sin gran mejoría
tras vasoconstrictor, siendo el resto de la exploración ORL normal. Dado el agravamiento de la clínica en el último año y el tamaño de la
masa en cavum, se realiza biopsia de la misma, informada como hiperplasia folicular linfoide, y RM, objetivándose una masa de 2,3cm
de diámetro compatible con hipertrofia de tejido adenoideo. Teniendo en cuenta la clínica referida y la exploración realizada, el
paciente es intervenido de septoplastia con turbinectomía y adenoidectomía. El informe AP es de linfoma de Hodgkin (LH) clásico rico
en linfocitos, sin detectar presencia de virus Epstein Barr (VEB).

PREGUNTA
¿Cuál es la manifestación clásica de Linfoma Hodgking?

RESPÚESTA
a.- Paciente joven, masculino, linfadenopatías periféricas.
b.- Adulto mayor, masculino, afección extraganglionar

CURSO ENARM CMN SIGLO XXI TEL: 36246001 Pharmed Solutions Institute PÁGINA 86
MANUAL DE TRABAJO DEL CURSO ENARM CMN SIGLO XXI
c.- Paciente joven, femenino, linfadenopatías periféricas
d.- Paciente joven, masculino, afección extraganglionar

PREGUNTA
¿Qué utilidad tiene hacer estudio inmunohistoquimico a este paciente?

RESPUESTA
a.- Decidir tratamiento
b.- Diferenciar LNH bajo grado de un LNH agresivo
c.- Establecer un diagnóstico diferencial con el linfoma no hodgkiniano
d.- Saber si es un tumor en cabeza y cuello

PREGUNTA
El paciente descrito presenta un LH, sin síntomas B, localizado en adenoides, sin afectación de otras regiones ni adenopatías asociadas.
¿ En qué estadio de la clasificación de Ann Arbor- Costwold se encuentra?

a.- Estadio IV
b.- Estadio III E-A
c.- Estadio IE-A
d.- Estadio IE-B

CASO CLINICO
Una paciente de 27 años, del sexo femenino, se presentó en la Sala de Emergencias con palpitaciones de inicio súbito y disconfort
torácico. La presión arterial (PA) era de 68 / 40 mmHg y la frecuencia cardíaca (FC) era de 184 lpm. Al examen físico, la paciente
presentaba aumento de volumen de la mama izquierda y masa abdominal difusa. El ECG reveló taquicardia de complejo ensanchado
con estándar de bloqueo de ramo izquierdo, fue sometida a cardioversión eléctrica para el ritmo sinusal normal. El ECG fue repetido y
mostró ritmo sinusal normal, sin alteraciones del segmento ST. La paciente recibió amiodarona. Una ecocardiografía transtorácica
reveló derrame pericárdico posterior difuso y engrosamiento del miocardio basal-septal. VHI positiva y SIDA con CD4 de 39
células/mm3. El diagnóstico de linfoma difuso de grandes células B (LDGCB), un linfoma no Hodgkin fue el diagnostico definitivo.

PREGUNTA
Cuál es el medio diagnostico mas adecuado.

RESPUESTA
a.- Biopsia.
b.- IRM-f
c.- Frotis periférico.
d.- BH.

CASO CLINICO
Se trata de masculino de 21 años de edad el cual acude a consulta externa por cuadro faríngeo, se observa odinofagia, rinorrea hialina
con descarga retrofaringea, amígdalas hiperemicas e hipertróficas, adenopatía cervical y axilar, al interrogatorio que presenta
frecuentemente cuadros infecciosos, pero en la actualidad refiere que desde hace 5 meses ha presentado fiebre intermitente, ataque al
estado generalizado, fatiga, adinamia, hiporexia, con pérdida de peso del 10 % aproximadamente, además de prurito generalizado.

PREGUNTA
Cuál es la condición más probable en este caso.

RESPUESTA
a.- Considerando la edad es más frecuente la mononucleosis infecciosa.
b.- Lo más probable es Leucemia Linfoblastica Aguda.
c.- Lo más frecuente es Leucemia Linfocitica Aguda.
d.- Considerando el cuadro Linfoma de Hodgkin.

CASO CLINICO
Varón de 69 años de edad con antecedentes personales de LMA M1 (mieloblástica sin diferenciación), en remisión completa (RC) desde
hace 11 meses tras tratamiento quimioterápico según protocolo PETHEMA-LAM 99. Ingresa por cuadro respiratorio catarral, con
importante congestión nasal, sin respuesta a antibiótico. Presenta, a su vez, pápulas eritematosas en espalda, adenopatías
supraclaviculares y submaxilares. Presenta ademas ojo rojo de dos días de evolución, sin otra sintomatología ocular. En la exploración
presentaba una agudeza visual con corrección de 20/20 en OD y 18/20 en OI. Los reflejos, motilidad y tonometría oculares se
encontraban dentro de la normalidad. En la biomicroscopía de polo anterior se observó en OD una lesión carnosa sobreelevada de
color salmón en el tarso superior e inferior. En el OI se encontró una lesión similar en conjuntiva tarsal inferior. En el fondo de ojo no se
apreciaron alteraciones.

PREGUNTA
Cual es la conducta mas apropiada a seguir?

CURSO ENARM CMN SIGLO XXI TEL: 36246001 Pharmed Solutions Institute PÁGINA 87
MANUAL DE TRABAJO DEL CURSO ENARM CMN SIGLO XXI
RESPUESTA
a.- Biopsia de lesión.
b.- Busquedad de marcadores.
c.- Administracion de esteroides tópicos.
d.- Punsion medular.

PREGUNTA
Sobre la patologia anterior descrita, cual de las siguientes afirmaciones es menos frecuente?

RESPUESTA
a.- La afectación ocular por la leucemia es bien conocida,
b.- Los más frecuente es la hemorragia retiniana.
c.- Es debida a la leucocitosis.
d.- La retina es, a su vez, la estructura ocular más frecuentemente afectada de forma directa por células leucémicas, hasta en un 30%.

CASO CLINICO
Paciente de sexo femenino de 32 años de edad que consulta por presentar equimosis, hematomas generalizados, gingivorragia e
hipertrofia gingival, astenia, adinamia y síndrome febril. Tras un laboratorio, se asume a la paciente como Leucemia Aguda. Se realiza
punción-aspiración de médula ósea, cuyo análisis citogenético es compatible con una Leucemia Mieloide Aguda de subtipo M3. Se inicia
el tratamiento quimioterápico con Mitoxantrona y Tretinoina, farmacoprofilaxis con Dexametasona y Omeprazol. A los 20 días de
iniciado el dicho tratamiento, la paciente desarrolla Síndrome de Diferenciación.

PREGUNTA
Cual de los siguientes mecanismos fisiopatológicos no intervienen en el síndrome de diferenciación presente en el caso?

RESPUESTA
a.- Respuesta inflamatoria sistémica.
b.- Daño endotelial con síndrome de fuga capilar.
c.- Obstrucción de la microcirculación que produce infiltración tisular.
d.- Disminución de la presión oncotica sobre la hidrostática.

CASO CLINICO
Varón de 14 años, acude por esplenomegalia. Historia pre-hospitalaria de palidez de piel y mucosas, de moderada intensidad, de un
mes de evolución, fiebre de 38ºC sin escalofríos ni sudoración, dolor en el pecho y vómito de contenido alimentario con estrías de
sangre, 24hs antes del ingreso. Al examen físico presenta petequias dispersas en ambas piernas y pápulas en miembro inferior derecho.
El laboratorio informa Leucocitos: 25.200/uL con 68% de blastos, Hemoglobina: 6,1g/dL, Hematocrito: 17%, Plaquetas: 31.000/uL.

PREGUNTA
Cual es la conducta diagnostica mas adecuada?

RESPUESTA
a.- Aspirado de medula osea.
b.- Gamagrama oseo.
c.- IRM.
d.- Marcadore tumorales.

FIEBRE REUMATICA (FRA):


CIENCIAS BASICAS: Enfermedad inflamatoria ocurre como secuela alejada de una infección por estreptococo beta hemolítico grupo A,
habitualmente faringoamigdalitis o escarlatina. Su nombre proviene del daño articular, pero los daños más importantes se producen en
corazón. SALUD PÚBLICA: La FRA aparece de 0.5-5% de pacientes que hacen faringoamigdalitis estreptocócica. Aparece especialmente
entre 5-15 años. Mortalidad <1% por carditis grave. Aparición favorecida por factores climáticos (húmedo, frio) y socioeconómico
(hacinamiento, pobreza). PATOGENIA: Existe una relación entre faringoamigdalitis y estreptococo grupo A y la fiebre reumática que se
sustentan en: 1. Relación epidemiológica sustentada entre estas infecciones y la aparición de casos de FRA. 2. Comprobación
inmunológica de infección estreptocócica (antiestreptolisinas en suero). 3. Episodios primarios y secundarios pueden prevenirse
mediante antibiótico precoz para la infección. Debido a que las toxinas del estreptococo producen inmunidad cruzada.
DIAGNOSTICO: Clínico; Cuadro febril insidioso, con aparición de artritis (70-80%), es una poliartritis migratoria de grandes
articulaciones. Carditis (40-60%), generalmente dentro de las primeras semanas del episodio, si no hay carditis (infiltrado inflamatorio
que afecta miocardio, endocardio y pericardio) en los primeros meses el pronóstico es muy bueno; puede manifestarse por soplos de
insuficiencia mitral (75-80%) o aortica (30%), frote pericardico, galope, cardiomegalia, congestión pulmonar, a veces son poso

CURSO ENARM CMN SIGLO XXI TEL: 36246001 Pharmed Solutions Institute PÁGINA 88
MANUAL DE TRABAJO DEL CURSO ENARM CMN SIGLO XXI
sintomáticas. Corea de Syndenham (10-20%) que son movimientos desordenados, descoordinados, inesperados e involuntarios.
Eritema marginado y nódulos subcutáneos de Meynet (<2-3%). Puede haber compromiso del estado gnarl., artralgias. Laboratorio:
Antiestreptolisinas (ASO) elevadas (desde los primeros días y duran hasta 6-8 semanas). VSG Y Proteína C reactiva elevadas. Se puede
hacer cultivo faríngeo para estreptococo. Histológico: Los cuerpos de Aschoff son una confluencia de monocitos y macrófagos rodeados
por fibrosis se consideran típicos de FRA. En ECG puede haber bloque AV simple, fenómeno no diagnostico de carditis. La ecografía
cardiaca es útil para el diagnostico y el tratamiento de la carditis (valoración de daño de válvulas, y de insuficiencia cardiaca). CRITERIOS
DIAGNSOTICOS de Jones: Criterios mayores; poliartritis, carditis, corea, eritema marginado, nódulos subcutáneos. Criterios menores;
fiebre, artralgias, FRA previa, VSG y PCR elevadas, prolongación de PR. El diagnostico se hace con 2 criterios mayores ó 1 mayor y 2
menores, junto con alguna evidencia de infección estreptocócica, tal como elevación de las ASO o antecedente de escarlatina, cultivo
faríngeo positivo o prueba rápida de antígeno estreptocócico positivo. TRATAMIENTO: Se realiza en 2 partes del episodio actual y
prevención de la recurrencia. Reposo en caso, de artritis o carditis hasta recuperación completa. El medicamento de primera elección
para la artritis es la aspirina, segunda elección naproxen. En la carditis se pueden utilizar corticoides. Prevención primaria: Tratamiento
adecuado a las faringoamigdalitis estreptocócicas (su diagnostico se basa marcadamente en cuadros febriles y exudativos), con
penicilina benzatinica 1,200,000U dosis única o eritromicina VO por 10dias (en caso de alergia a penicilinas). Prevención secundaria:
Uso prolongado de penicilina benzatinica 1,200,000U cada 4 semanas o sulfadiazina 1gr/dia (en caso de alergia a penicilinas). La
duración de profilaxis depende de la gravedad y edad del paciente. El periodo mínimo recomendado en paciente con antecedente de
FRA, sin carditis es al menos de 5 años, pero directrices reciente refieren 10 años o hasta 21 años de edad. PRONOSTICO: Los episodios
de FRA tienden a remitir espontáneamente en 75%, antes de 6 semanas y un 90% antes de 12 semanas. El pronóstico a largo plazo
dependerá de la gravedad de la carditis, aparición de nuevos brotes y magnitud de daño valvular.

CASO CLINICO
Femenino de 30 años, antecedente IAM, estenosis mitral con remplazo biologico, tabaquismo, 4 gestas, con dificultad para hablar y
movimientos involuntarios en dedos, manos, boca y lengua hacía 18 meses, disnea de moderado esfuerzo. Actualmente con fracción de
eyección de 44% y prótesis biológica mitral sin alteraciones estructurales con reflujo mínimo. Inició cuadro de cefalea acompañada de
parestesia en miembro superior izquierdo y movimientos involuntarios en manos, boca, lengua y dedos. Enseguida, presentó
convulsiones tónico-clónicas generalizadas y episodios de alucinación visual, delirium e intensa labilidad emocional. Al examen físico, se
encontraba en regular estado general, taquicárdica (frecuencia cardíaca=130 latidos por minuto), presión arterial de 120x70 mmHg,
presencia de ruidos rítmicos con hiperfonesis de B1 y frote pericárdico audible en mesosístole y proto y telediástole y estertores finos
crepitantes en ambas bases pulmonares.

PREGUNTA
Que clase funcional de la NYHA se encuentra el paciente?

RESPUESTA
a.- Clase I
b.- Clase II
c.- Clase III
d.- Clase IV

PREGUNTA
Cual es la complicación neurológica que se presento en caso?

RESPUESTA
a.- Acidente cerebro vascular.
b.- Corea de huntintong.
c.- EVC y corea.
d.- Corea de Syndeham.

PREGUNTA
Cual es la causa de muerte aguda más frecuente en estos casos?

RESPUESTA
a.- IAM.
b.- Carditis.
c.- Insuficiencia cardiaca.
d.- Edema agudo pulmonar.

PREGUNTA
Cual de las siguientes medidas farmacológicas es la mas apropiada para el manejo de síntomas neurológicos?

RESPUESTA
a.- Fenitoina.
b.- Haloperidol.
c.- Biperideno.
d.- Clonacepam.

CURSO ENARM CMN SIGLO XXI TEL: 36246001 Pharmed Solutions Institute PÁGINA 89
MANUAL DE TRABAJO DEL CURSO ENARM CMN SIGLO XXI
PREGUNTA
Cual es la causa de la carditis en la fiebre reumática?

PREGUNTA
a.- Proteina M Streptococcica + N-Metil glucosamina con reacción cruzada con la miosina.
b.- Proteina M Streptococcica + N-Acetil-D glucosamina con reacción cruzada con la miosina.
c.- Proteina P Streptococcica + N-Acetil-D glucosamina con reacción cruzada con la miosina.
d.- Proteina P Streptococcica + N-Metil glucosamina con reacción cruzada con la miosina.

PREGUNTA
Considerando los Criterios de Jones cual es la respuesta correcta en este caso?

RESPUESTA
a.- 2 Criterios Mayores y 2 menores
b.- 1 Criterio Mayor y 2 menores.
c.- 2 Criterios Mayores y 1 menor.
d.- 3 Criterios Mayores y 2 menores.

CASO CLINICO
Femenino de 18 años que acude a urgencias por poliartralgias en rodillas y codos de 72 h de evolución, asociadas a fiebre. Antecedente
de escarlatina hace 3 semanas, tratada con penicilina V 7 días y 3 episodios anuales de faringoamigdalitis estreptocócica en los últimos
2 años. A la exploración, presenta impotencia funcional de rodilla derecha, dificultad para la extensión, posición en flexo, edema y signo
de la tecla positivo. Dolor a la palpación en rodilla contralateral y ambos codos, con menor inflamación e impotencia funcional. Soplo
sistólico II/VI vibratorio multifocal. Con leucocitos 19.910/μl, neutrófilos 12.040/μl; PCR 28,7mg/l y VSG 55mm.

PREGUNTA
Cuál es la conducta a seguir en esta paciente.

RESPUESTA
a.- Succinato de eritromicina, vía oral, por diez días.
b.- Amoxicilina, vía oral, por diez días 1500 mg una vez al día por diez días.
c.- Cefalexina, 500 mg cada ocho horas, vía oral durante diez días.
d.- Clindamicina vía hrs VO 600 mg al día c/6 dosis durante 10 días.

CASO CLINICO
Mujer de 31 años, portadora de cardiopatía reumática, hasta entonces asintomática, refirió disnea de esfuerzo progresiva y
palpitaciones aun usando regularmente penicilina benzatina de tres en tres semanas. A los 13 años presentó artritis migratoria de
grandes articulaciones tras estreptococcia de la orofaringe, y a los 19 años, en el 14º día de puerperio, se quejó de recurrencia de la
artritis, detectándose soplo cardiaco. El examen físico reveló taquipnea, con discreto edema de miembros inferiores. Se constataron los
siguientes datos: PA = 130/70 mmHg; frecuencia cardiaca de 120 rpm; ritmo cardíaco regular a tres tiempos con intenso soplo
holosistólico en área mitral (AM) y soplo sistólico rudo en el área tricúspide (AT); y hepatomegalia dolorosa discreta. Ecocardiograma
doppler fue compatible con insuficiencia mitral importante e insuficiencia tricúspide moderada. Hemograma detectó anemia
hipocrómica (hemoglobina = 11,1 g/dl), leucocitosis discreta (13.000/mm3) y hemosedimentación acelerada (37 mm/1ª hora). Los
niveles de antiestreptolisina O (AEO) fueron normales.

PREGUNTA
Cuál es el tratamiento farmacología para limitar el daño.

RESPUESTA
a.- Salicilatos dosis 30-60 mg/kg/día.
b.- Paracetamol 20 mg/Kg/dia.
c.- Naproxeno 750 mg/dia.
d.- Prednisona 2mg/kg/dia.

CURSO ENARM CMN SIGLO XXI TEL: 36246001 Pharmed Solutions Institute PÁGINA 90
MANUAL DE TRABAJO DEL CURSO ENARM CMN SIGLO XXI
ARTRITIS REUMATOIDE (AR):
CIENCIAS BASICAS: AR es la mas común de las artritis inflamatorias. Es una enfermedad multisistémica crónica, de etiología
desconocida, caracterizada por inflamación persistente (sinovitis), y consecuentemente lesión y destrucción de las estructuras
periarticulares y articulares. Participan factores genéticos y ambientales los cuales juegan un rol en el desarrollo de la enfermedad.
SALUD PUBLICA: Se presenta 0.5-1% de la población, en relación mujer:hombre 3:1. Asociada a HLA-DRA-4. Más frecuente en cuarta y
quinta década de la vida. PATOGENIA: En la AR los T CD4 son los principales para desencadenar engrosamiento sinovial e invasión de
articulaciones y cartílagos. En AR el órgano blanco es la membrana
ANTIGENO Respuesta inmunitaria-reacción sinovial.
DESCONOCIDO + inflamatoria con activación de DIAGNOSTICO: Clínico; Poliartritis crónica simétrica, rigidez matutina
PREDISPOSICION cel. Plasmáticas y linfocitos T (>1h), inicia de forma insidiosa, afecta principalmente manos
GENETICA
(metacarpofalalgicas 90%; carpo 80%), rodilla (60%), pies
(metarasofalangicas 50%), articulación atlo-axoidea (40%; se manifiesta
Infiltración sinovial por linfocitos como cervicalgía). Síntomas generales: astenia, adinamia, hiporexia,
Manifestaciones generales T CD4> T CD8 y monocitos afectación musculo-esquelética variable. Exploración física: desviación
por citocinas secretadas cubital, nódulos de Bouchar (interfalangicas proximales), sinovitis,
por los macrófagos
dedos en cuello de cisne o botonero, hallux valgus (miembros
Formación de tejido de inferiores), hipotrofia en músculos interóseos en manos (por vasculitis),
granulación por activación de disminución de fuerza prensil, derrame bilateral de rodillas de
Destrucción articular y fibroblastos (pannus) con predominio izq. (complicación frecuente de AR, el quiste de Barker en
ósea por citocinas hiperplasia de células móviles
hueco poplíteo asociado a ruptura e invasión muscular).
formadas en el pannus.
Manifestaciones extraarticulares: piel; nódulos reumatoides (25-50%).
Hematológicos; anemia normocitica normocrómica (35-30%), trombocitosis (>50,000), trombocitopenia, linfadenopatía. Hepático;
transaminemia. Pulmón (30%), dolor pleurítico (25%), nódulos pulmonares, arteritis pulmonar, enfermedad pulmonar intersticial. Ojos;
queratoconjuntivitis Sicca (35%), ojo seco, epiescleritis, uveítis, queratitis ulcerativa. Corazón; pericarditis, enf. Ateroesclerótica
avanzada. Neurológico; neuropatía periférica, síndrome del túnel del carpo. Renal; nefropatía Laboratorio: Factor reumatoide
(FR=presente hasta 65%): Auto-anticuerpos dirigidos contra la porción de Fc de la IgG tiene sensibilidad de 70%, especificidad 80%;
también presente en otros procesos infecciosos crónicos e inmunitarios (Tb, endocarditis, neoplasias, lupus esclerodermia). Nos indica
artritis seronegativa o seropositiva esta última se relaciona con forma más agresiva de la enfermedad. Autoanticuerpos anti-péptido
cíclico citrulinado (Anti-Pcc =presente hasta 95%), sensibilidad de 70% y especificidad 90-95%, al igual que FR se relaciona con formas
más agresivas de la enfermedad. VSG PCR elevadas. Anti-DNA negativo (especifico para (LES). Radiológico (PA de mano): Inflamación de
articulaciones metacarpofalángicas, desviación cubital, pulgar en Z, dedos en cuello de cisne, erosiones óseas y osteopenia
yuxtaarticular, luxaciones metacarpofalángicas proximales. CRITERIOS DIAGNOSTICOS: 1. Rigidez matutina al menos de 1 hora de la
mejoría máxima. 2. Artritis de 3 o más articulaciones. 3. Artritis de las manos. 4. Artritis simétrica. 5. Nódulos reumatoideos (sobre
prominencias óseas, superficies extensoras o regiones yuxtaarticulares). 6. Factor reumatoide positivo. 7. Cambios radiológicos. El
diagnostico se hace con 4 de 7 criterios. TRATAMIENTO: AR activa leve: hidroxicloroquina (HCQ) o sulfazalazina (SSZ). AR moderada a
severa: Considerar dosis bajas de prednisona 5-10mgs e iniciar metotrexate (MTX) 10mg, se puede incrementar hasta 25mg/sem. Si hay
una respuesta inadecuada se puede agregar otro FARME (SSZ, HCQ, d-penicilamina)los cuales previenen el daño erosivo , terapia
bilógica (abatacep, etarnecep), anti TNF. Inmunosupresores (aziatropina, leflunomide, ciclosporina). Cirugía puede ser considerada en
una deformidad funcional severa. La prednisona al igual que los AINEs se puede dar como medidas de control, pero no modifican el
curso de la enfermedad. El metotrexate es la piedra angular del tratamiento de la AR (por ser un inductor de la remisión). SINDROMES
ESPECIFICOS EN PACIENTES CON AR: Síndrome de Sjogren secundario= AR + Sicca (xerostomía-xeroftalmia). Síndrome de Felty= AR +
Esplenomegalia + leucopenia (neutropenia). Síndrome de Caplan= AR + Nódulos reumatoideos pulmonares + Neumoconiosis.
Amiloidosis en AR= AR de larga evolución y grave + proteinuria o insuficiencia renal.

CASO CLINICO
Se trata de un varón de 63 años con antecedentes de AR de más de 15 años de evolución bajo tratamiento. Entre otros antecedentes
padece hipertensión arterial, dislipidemia y cardiopatía isquémica. Ingresa por síndrome nefrótico observándose a la exploración física:
100,7 kg de peso, regular estado general, eupneico en reposo, saturación del 96%; hábito cushingoide; tensión arterial de 130/70mmHg
y temperatura de 37°C; auscultación cardiopulmonar normal; abdomen globuloso, blando y depresible; edemas con fóvea hasta la raíz
de los muslos, abdomen y manos; artritis bilateral de las articulaciones metacarpofalángicas y limitación a la flexo-extensión en ambos
carpos; deformidad de ambos tobillos y pies, más el derecho. Entre las exploraciones complementarias destaca anemia normocítica
normocrómica; velocidad de sedimentación globular (VSG) de 130 mm/1.ª hora; hiperfibrinogenemia reactiva; urea de 112 mg/dl;
creatinina de 3,8 mg/dl; aclaramiento de creatinina de 45,4 ml/min; ácido úrico de 8,2 mg/dl, proteínas totales de 4,5 g/dl; proteína C
reactiva de 10,9 mg/dl; proteinograma con aumento de fracciones alfa 1, alfa 2 y betaglobulinas y disminución de las gammaglobulinas;
albúmina de 1,5 g/dl y en el sistemático de orina se detecta proteinuria, microhematuria y abundantes cilindros granulosos-hialinos y
más de 11 g de proteínas en orina de 24 h.

PREGUNTA
Considerando el manejo habitual del padecimiento de base AR, cual de las siguientes observaciones es mas probable que se encuentre
relacionado con el síndrome nefrotico?

RESPUESTA
a.- Uso crónico de AINES.
b.- Uso de inmunomoduladores.
c.- Presencia de depósitos inmunologicos.
d.- Presencia de amiloidosis secundaria a AR.

CURSO ENARM CMN SIGLO XXI TEL: 36246001 Pharmed Solutions Institute PÁGINA 91
MANUAL DE TRABAJO DEL CURSO ENARM CMN SIGLO XXI
PREGUNTA
Considerando la pregunta anterior, cual de los siguientes estudios es de mayor utilidad en este caso?

RESPUESTA
a.- Tomografia computada abdominal.
b.- Biopsia renal.
c.- Radiografia de torax.
d.- Ecocardiografia.

PREGUNTA
Cual de las siguientes observaciones referentes al diagnostico de AR es de mayor utilidad?

RESPUESTA
a.- Los anticuerpos anti-CCP tiene un cociente de probabilidad para el diagnostico de AR superior al de Factor Reumatoide y es
pronostico de la enfermedad.
b.- La evaluación radiográfica por método de Sharp/van der Heijde permite evaluar daño estructural para (erosion y pinzamiento
articular).
c.- Inflamación articular con anticuerpos anti-CCP y factor reumatoide elevadoes indicativo de enfermedad agresiva con mal pronostico.
d.- Se deberá realizar IRM precoz para detectar erosiones, sinovitis o derrame articular.

CASO CLINICO
Mujer de 68 años de edad con antecedentes de AR de 20 años de evolución, en tratamiento con AINE, glucocorticoides a dosis bajas (5
mg al día de prednisona) y metotrexate; prótesis bilateral de caderas y rodilla derecha; hipertensión arterial, síndrome mielodisplásico
en tratamiento con eritropoyetina e insuficiencia renal crónica leve. Ingresa por cuadro de diarreas de 8 semanas de evolución, pérdida
de peso de 20 kg y edema en los miembros inferiores. En la exploración física se aprecia: 45 kg de peso, regular estado general, tensión
arterial de 110/60 mmHg y una temperatura de 36°C; auscultación cardiopulmonar normal; abdomen blando y depresible; edemas con
fóvea hasta raíz de muslos y en manos. Artritis de metacarpofalángicas bilaterales y limitación a la flexo-extensión en ambos carpos. En
la analítica destaca anemia normocítica normocrómica, VSG de 110 mm/1.ª h, urea de 46 mg/dl, creatinina de 1,3 mg/dl, sistemático de
orina normal, con proteínas en orina de 24 h negativa, proteinograma con aumento de fracciones alfa 1, alfa 2, beta y gammaglobulinas
y albúmina disminuida.

PREGUNTA
Considerando el cuadro clínico, cual de los siguientes estudios es el mas adecuado para una aproximación dignostica?

RESPUESTA
a.- Tomografia abdominal.
b.- Endoscopia abdominal.
c.- Panendoscopia.
d.- Biopsia por colonoscopia.

PREGUNTA
Considerando el manejo habitual del padecimiento de base AR, cual de las siguientes observaciones es mas probable que se encuentre
relacionado con el síndrome diarreico?

RESPUESTA
a.- Uso crónico de AINES.
b.- Uso de inmunomoduladores.
c.- Presencia de depósitos inmunologicos.
d.- Amiloidosis secundaria a AR.

CASO CLINICO
Paciente varón de 60 años con artritis reumatoide de 6 años de evolución en tratamiento con sulfasalazina 1.500mg/día y deflazacort
6mg/día, con buen control clínico y analítico de su enfermedad. Se trata de un paciente fumador activo de 20 cigarrillos/día y con
cardiopatía isquémica crónica. En una revisión habitual refiere la aparición de una tumoración dolorosa en el hueco poplíteo derecho,
cuyo dolor se incrementa con la bipedestación prolongada. En la exploración física se objetivó una tumoración en el hueco poplíteo.

PREGUNTA
Considerando las complicaciones mas frecuentes en la AR cual de las siguientes es la mas probable?

RESPUESTA
a.- Quiste de Baker.
b.- Aneurisma popitleo.
c.- Lipoma popitleo.
d.- Varice popitlea.

CURSO ENARM CMN SIGLO XXI TEL: 36246001 Pharmed Solutions Institute PÁGINA 92
MANUAL DE TRABAJO DEL CURSO ENARM CMN SIGLO XXI
PREGUNTA
Cual de las siguientes aseveraciones relacionadas al diagnostico de AR no es adecuado?

RESPUESTA
a.- BH, QS, Transaminasas, VSG, EGO, actividad, funcionalidad articular y daño estructural.
b.- Compromiso en 3 articulaciones metacarpofalangicas o metatarsofalangicas y rigidez matinal de más de 30 minutos.
c.- La elevación de la VSG y la PCR tiene estrecha relación con la actividad inflamatoria de la AR.
d.- La presencia de Factor Reumatoide confiere el diagnostico de artritis reumatoide.

PREGUNTA
Relacionado a al tratamiento cual de las conducta siguientes es la mas apropiada en el presente caso?

RESPUESTA
a.- Los AINES no debe reemplazar a los FARME.
b.- Considerando el uso de corticoides debe usarse calcio y vitamida D.
c.- El MTX es el fármaco de elección para AR en riesgo de enfermedad persistente.
d.- En caso de falla con MTX se recomienda LEF ó SSZ ó anti-TNF.

OSTEOARTRITIS (OA):
CIENCIAS BASICAS: Enfermedad degenerativa crónica y forma más común de enfermedad de las articulaciones, caracterizada por falla
común y progresiva de todas las estructuras de la articulación, cartílago, revestimiento, ligamentos y hueso. Acompañadas por el
aumento de espesor y esclerosis de la placa ósea subcondral. SALUD PUBLICA: Mas de 70% de personas >70 años, tienen evidencia
radiográfica de OA. Es considerada la causa de dolor e incapacidad más frecuente en ancianos. La incidencia de osteoartrosis de rodilla
se ha calculado en 240 casos por 100, 000 habitantes y es una causa de morbilidad y alto costo para la comunidad.. Su prevalencia es
igual para ambos géneros entre los 45 y 55 años, pero después de los 55 años es más frecuente en mujeres. Enfermedad articular mas
frecuente en mujeres. PATOGENIA: Deterioro de la articulación debido con frecuencia a tensión mecánica, alteraciones biomecánicas y
celulares que producen un desequilibrio entre la degradación y la síntesis de los condrocitos, la matriz extracelular y del hueso
subcondral, lo que hace que el hueso que está bajo el cartílago falle, tiende afectar las articulaciones que se usan con frecuencia
(manos, columna, cadera, rodillas). Los 2 principales componentes del cartílago son: colágeno tipo 2 (provee fuerza tensil) y un
proteoglucano. El OA es caracterizado por depleción gradual del proteoglucano, desplegando de la matriz el colágeno y perdiendo
colágeno tipo 2, lo que nos lleva a incremento de la vulnerabilidad.
CLASIFICACION DE LA OSTEOARTRITIS
Primaria: localizada; Cadera Dentro de la fisiopatología ocurren cambios morfológicos:
Rodilla fragmentación del cartílago, formación de fisuras y ulceración de la
Apófisis espinosas y cuerpos vertebrales
superficie articular, eburneación, la posterior proliferación celular en
(cervical,lumbar)
Mano (interfalangicas distales) acúmulos. Cambios bioquímicos: mayor cantidad de agua, sintesisi de
Pie (primer dedo) fibras de colágeno pequeñas y frágiles, menor concentración de
Generalizada: Manos proteoglucanos y sulfato de queratán con aumento de la
Manos y rodillas
Secundaria: Displasica: condrodisplasias, displasias epifisiarias, concentración de sulfato de condritin que desencadena la producción
Desplazamiento congénito articular de proteoglucanos de menor tamaño y de menor capacidad para
Postraumática: aguda, repetitiva, posoperatoria
agregarse al ácido hialurónico, que predispone a fragilidad. Cambios
Insuficiencia estructural: osteonecrosis, osteocondritis
Posinflamatoria: artrits reumatoide y séptica biomecánicos: perdida de la rigidez y elasticidad del cartílago, Factores
Endocrina y metabolica: acromegalia, hemocromatosis de riesgo: edad avanzada, tener familiares con OA, obesidad, lesiones
Tejido conectivo: Hipermovilidad (Ehlers- Danlos) en articulación o uso repetitivo, defectos congénitos, densidad ósea,
Formas generalizadas: Hiperparatiroidismo, acromegalia
ocupación, enfermedades concomitantes.

DIAGNOSTICO: Clínico; Dolor (empeora con la actividad, mejora con el reposo), rigidez matutina breve (<30min), hinchazón
prominente, crujidos, rechinado de toda la articulación, llevando a un menor funcionamiento de la misma, puede haber aumento de
temperatura local y de volumen articular, si hay derrame, deformidad (angulacion en varo o valgo), subluxación en casos avanzados,
nódulos de Heberden (interfalangicas distales), Bouchar (interfalangicas proximales). Laboratorio: Generalmente todo normal. VSG y
PCR normales. Si se hace estudio de liquido sinovial, ligeramente turbio, no contienen cristales y con leucocitos <2,000/microlitro y
<25% de neutrofilos. Radiología: Disminución de espacio articular, esclerosis subcondral, osteofitos marginales y quistes subcondrales.
COMPLICACIONES: Derrame articular, herniación capsular, bloqueo articular, tendinitis, estenosis espinal espondilolistesis.
TRATAMIENTO: Ningún tratamiento especifico. Objetivo reducir dolor y mejorar funcionamiento: 1. Medidas físicas: pérdida de peso y
ejercicio con lo que se disminuye dolor, mejora fuerza muscular y disminuye rigidez de la articulación, útiles usar ortesis o bastones.
Evitar sobrecarga y trauma de articulaciones. Calor o frio local. Reposo articular 2. Medico: Tópico; capsaicina, lidocaína, diclofenaco.
Vía oral AINES, paracetamol (primera elección), se puede considerar el tramadol (200-300mgs dividido en 3 a 4 dosis) si el AINE no es
efectivo. Los corticoides orales no están indicados en OA. En 2010 se aprobó duloxetina para dolor musculo-esquelético crónico (largo
plazo). La diacerina es una quinona, que posee propiedades antioxidantes, inhibiendo a IL-1 y metaloproteinasas, pero aun no es
contundente su eficacia y seguridad. El Ac. Hialurónico intraarticular; mejoría parcial y se usa para retraso de reemplazo de rodilla. Los
glucocorticoides intraarticulares son recomendados en los brotes agudos inflamatorios en las distintas guías de OA, de rodilla, no debe
administrase una inyección cada 3 o 4 meses. Cirugía: Artroscopía, artroplastia u osteotomía por angulación (disminuye el dolor, en
pacientes no candidatos a artroplastia), en casos muy severos. Glucosamina/condroitina, aunque ampliamente vendidos, no están
aprobados por FDA, su eficacia no esta establecida.

CURSO ENARM CMN SIGLO XXI TEL: 36246001 Pharmed Solutions Institute PÁGINA 93
MANUAL DE TRABAJO DEL CURSO ENARM CMN SIGLO XXI
CASO CLINICO
Varón de 66 años con antecedentes personales de EPOC severo con hiperreactividad bronquial y bullas enfisematosas en ambos
vértices pulmonares, en tratamiento con oxigeno domiciliario (15h al día) junto a broncodilatadores, diuréticos y corticoides
(prednisona 10mg) a días alternos. El paciente había tenido varios ingresos hospitalarios por reagudizaciones de EPOC. Refirie dolor en
la región lumbar central y en ambas caderas con la deambulación. Se realizaron exploraciones complementarias que incluyeron una
radiografía de pelvis y un TAC lumbosacro que mostraron espóndilolisis con espondilolistesis de L5 y artrosis de cadera
respectivamente, procesos a los que se atribuyeron los síntomas. En la radiografía de pelvis inicial se observa estrechamiento del
espacio articular y esclerosis subcondral. Se encuentra en silla de ruedas por la incapacidad para deambular. Leucocitos 9,5×103/mm3
(neutrófilos 48.4%), Hb 12mg/dl, Hto 36, plaquetas 355×103/mm3, VSG 32mm/h, glucosa 86mg/dl, urea 44mg/dl, creatinina 0,88mg/dl,
colesterol 190mg/dl, triglicéridos 116mg/dl, fosforo 4,2mg/dl, calcio total 9,5mg/dl, actividad de protrombina 106%, INR 0,97, ratio
tromboplastina 1,10, proteína C 1,22, proteína S 0,73, antitrombina III 0,99U/l. Los anticuerpos antinucleares, anticuerpos
antifosfolípido y el factor reumatoide fueron negativos, así como la serología para virus de hepatitis B, C y VIH.

PREGUNTA
Considerando la entidad nosológica descrita, asi como los reportes de laboratorio, cual de los siguientes diagnosticos diferenciales es el
mas frecuente?

RESPUESTA
a.- Infeccioso.
b.- Metabólico.
c.- Inflamatorio crónico.
d.- Neuropatías.

CASO CLINICO
Mujer de 74 años de edad que acude por aumento de dolor en la rodilla izquierda. Le administraron paracetamol más ibuprofeno,
refiere incapaz para limpiar la casa debido a que el dolor empeore. Ella usa un masaje tópico al acostarse y ha intentado usar unas
compresas frías, con beneficios limitados. Es hipertensa bajo tratamiento, En el examen, su presión arterial es de 140/ 92 mm Hg, pulso
es de 80 latidos por minuto. Ella es de 162 cm de alto y pesa 80 kg (índice de masa corporal de 30,5 kg/m2). Ella califica su dolor como 8
de 10 cuando está activo, y 5 de 10 en reposo. Camina con un bastón y cojea. Sus resultados del examen rodilla izquierda no se han
modificado. Previo análisis de rayos- x mostraron cambios degenerativos severos. Resultados de creatinina de 90 mmol / L
Aclaramiento de creatinina estimado ( CrCl ) es entre 40 y 60 ml / min.

PREGUNTA
Cual de los siguientes criterior del presente caso, lo hace candidato a artroplastia?

RESPUESTA
a.- Artrosis bicorpartamental.
b.- Artrosis patelofemora aislada.
c.- Artrosis media o lateral.
d.- Necrosis avascular.

PREGUNTA
Considerando el estudio de liquido sinovial para el caso, que resultados es menos probable observar?

RESPUESTA
a.- Celular completa.
b.- Tincion Gram.
c.- Cultivo de liquido.
d.- Cristales de oxalato.

OSTEOCONDRITIS:
CIENCIAS BASICAS: Definición clínica: Conjunto de trastornos óseos de origen idiopático, caracterizados por trastornos en la osificación
endocondrial que afecta tanto la condrogenesis como la ostéogenesis (se presenta en centros óseo en pleno crecimiento). Definición
radiológica: Trastornos óseos idiopáticos en relación a su etiología que comparten las siguientes características; 1. Predilección por el
esqueleto inmaduro. 2. Afección de epífisis, apófisis o huesos epifisoides. 3. Cuadro radiológico caracterizado por fragmentación,
colapso, esclerosis y posteriormente reosificación y restitución de contorno óseo. Se da en niños clínicamente sanos y en periodo de
crecimiento rápido (3-15años) y expuestos a traumatismos; sus localizaciones más frecuentes son las articulaciones que soportan carga.
PATOGENIA: Idiopática. Son procesos cíclicos que pasan por diversas etapas en su evolución y que pueden dejar secuelas por
deformaciones producidas en el estadio de fragilidad de la osteonecrosis. Se describen 4 etapas de la necrosis: 1. La zona vecina a la
necrosis sufre un proceso de descalcificación por hiperemia reactiva. La zona necrótica, en cambio no se descalcifica por falta de vasos
que drenen sus componentes minerales. Resulta así una mayor densidad del fragmento necrótico respecto al sano. 2. Más tarde, la
zona necrótica es invadida gradualmente por tejido conjuntivo y vasos sanguíneos, que van a retirar el tejido necrótico por reabsorción.
3. Esas zonas aparecen rarefactas en medio de la necrosis, fenómenos de sustitución progresiva. 4. Cuando el proceso de reabsorción y
reconstitución está avanzado, todo el fragmento esta descalcificado para ir luego a la recalcificación y restauración ósea.
DIAGNOSTICO: Clínico; dolor al apoyo, hipersensibilidad a la presión a nivel del hueso afectado. El inicio de la sintomatología puede ser
agudo o gradual y puede o no existir anamnesis de traumatismo. Radiológico: No siempre guarda relación con la intensidad del cuadro

CURSO ENARM CMN SIGLO XXI TEL: 36246001 Pharmed Solutions Institute PÁGINA 94
MANUAL DE TRABAJO DEL CURSO ENARM CMN SIGLO XXI
clínico. Hay descalcificación precoz con necrosis subsiguiente y finalmente los signos de reparación. En las fases precoces se observan
zonas de densidad disminuida, que aumentan en intensidad y extensión; las epífisis se fisuran, se ensanchan presentando un contorno
borroso con aspecto deshilachado. Pueden observarse zonas de hueso necrótico denso. LEGG CALVE PERTHES: Necrosis avascular
idiopática de la cabeza femoral en niños. La teoría más aceptada de su origen es por insuficiente aporte sanguíneo al núcleo epifisiario
de la cabeza femoral, debido a oclusión vascular sea de una rama importante de la circunfleja o de una rama terminal. Se presenta
entre 3-10 años de edad, mas en sexo masculino, con mayor frecuencia unilateral. Tiene una evolución radiológica característica: A)
Estadio incipiente o de sinovitis: que dura de 1-3 semanas. B) Estadio necrótico, aséptico o avascular: dura varios meses a 1 año. C)
Estadio regenerativo o de fragmentación: dura de 1-3 años. D) Estadio residual, que corresponde a la etapa de crecimiento y de
reparación definitiva. El objetivo del tratamiento es la contención de la cabeza femoral dentro del acetábulo, para que ella se rehaga en
forma concéntrica. OSGOOD-SCHLATTER: Osteocondritis de la tuberosidad de la tibia, apófisis tibial anterior. Tiene como factor
desencadenante un traumatismo. Ocurre entre los 17 y 20 años, más frecuente en masculinos y en deportistas sometidos a
microtraumatismos repetidos. Hay dolor a la presión, hinchazón en tuberosidad anterior de anterior de la tibia en extensión. KOHLER:
Osteocondrosis del escafoides tarsiano: Más frecuente en hombres (5:1), incidencia entre 5 y 10 años. Dolor dorsal en mesopie de
meses a años. RX: aumento perchado de densidad con fragmentación, en estadios avanzados; aplanamiento, fragmentación, colapso y
esclerosis. THIEMANN: Necrosis aséptica de las epífisis basales de las falanges. Igual en ambos sexos. Caracterizada por hinchazón
progresiva y no dolorosa a nivel de la articulación interfalangica (proximal de manos y pies). Rx: irregularidad de epífisis de las falanges
que aparecen escleróticas y fragmentadas, ocasionalmente el espacio articular se ve disminuido, base de la falange ensanchada.
PANNER: Osteocondrosis del epicondilo lateral de húmero (capitelum). Rx: fisuración, aumento de densidad, disminución del tamaño
del capitelum, fragmentación, resorción. KIEMBOCK: Osteocondritis del semilunar. Es rara por debajo de los 15 años, hay antecedente
de un traumatismo, se da mas del lado derecho y en los trabajadores manuales, las personas jóvenes acusan dolor progresivo en
muñeca con rigidez de la misma e hipersensibilidad local a nivel de semilunar. FREIBERG: Osteocondrosis de la cabeza de los
metatarsianos. Más frecuente en mujeres (5:1), típico en segunda década de la vida, 10% bilateral. Es rara la afectación de más de un
dedo por pie. Presencia en orden decreciente dedo 2>3>4, muy raro en 1 y 5. SCHEUERMANN: Cifosis torácica rígida por acuñamiento
de los cuerpos vertebrales que ocurre en la infancia tardía. Rx: Cifosis torácica >45°, irregularidad de platillos vertebrales,
estrechamiento del espacio intervertebral.

CASO CLINICO
Femenino de 69 años de edad con cuadro clínico de 10 meses de evolución de inicio súbito; caracterizado por dolor tipo punzada en
rodilla izquierda que aparece luego de mantenerla en reposo prolongado, sin predominio de horario, asociado a crepitación, rigidez e
inflamación de la rodilla; intensidad 5/10 en la escala visual análoga, la cual cede a medicación con acetaminofén. Fue valorada por
medicina general quien indico 4 ampollas de betametasona; sulfato de glucosamina, diclofenaco, colestiramina; con mejoría de
síntomas. La radiografía ántero-posterior de rodilla izquierda muestra una lesión radio lúcida subcondral, rodeada por un halo de
esclerosis. Al examen físico, presenta crepitación bilateral de rodilla de predominio izquierdo con inflamación moderada de rodilla
izquierda. Sinovitis moderada en rodilla izquierda.

PREGUNTA
Considerando el cuadro clinico y las imágenes cual es su impresión dianostica?

RESPUESTA
a.- Osteocondritis Aseptica.
b.- Osteocondritis Disecante de Rodilla.
c.- Osteonecrosis.
d.- Osteocondritis osificante.

PREGUNTA
Considerando la respuesta previa, cual la localizacion mas frecuente?

RESPUESTA
a.- Lateral del cóndilo femoral medial.
b.- Rotula.
c.- Cóndilo femoral externo.
d.- Meniscos discoideos.

CURSO ENARM CMN SIGLO XXI TEL: 36246001 Pharmed Solutions Institute PÁGINA 95
MANUAL DE TRABAJO DEL CURSO ENARM CMN SIGLO XXI

GOTA:
CIENCIAS BASICAS: La gota se caracteriza por hiperuricemia y la presencia de episodios inflamatorios habitualmente monoarticulares y
recurrentes con lapsos asintomáticos, que son intensos y autolimitados. La inflamación está directamente relacionada con la presencia
de cristales de urato monosódico (UMS) intraarticular de forma constante. Hiperuricemia: Nivel plasmático de acido úrico >7.0mg/dl en
hombres o más de 6.0mg/dl en mujeres, surge de causas en la disminución de su eliminación, de aumento de la producción o de
ambos. El acido úrico es el producto final de la degradación de los nucleótidos de purina, su producción está relacionada con la
concentración intracelular de 5-fosforibosil-1-pirofosfato (PRPP), es el mayor determinante de la velocidad de biosíntesis del acido
úrico. El acido úrico es excretado principalmente por el riñón a través de mecanismos de filtración glomerular, secreción tubular y
reabsorción.
SALUD PÚBLICA: Gota causa común de artritis en hombres mayores de 40 años y mujeres posmenopáusicas. La prevalencia de
hiperuricemia en población adulta es de alrededor de 15%.
PATOGENIA: La enfermedad comienza con la formación y la CAUSAS DE HIPERURICEMIA
Por disminución en la Hiperuricemia primaria (idiopática)
subsecuente acumulación de acido úrico secundario a un eliminación Hiperuricemia secundaria:
aumento en su producción y/o disminución en su eliminación Función renal disminuida
(fase asintomática). Posteriormente factores Inhibición de la secreción de uratos (cetoacidosis, acidosis
láctica).
desencadenantes como alcohol, diuréticos, obesidad, Por aumento en su Hiperuricemia primaria:
trauma, pH acido, ambientales y otros ocasionan producción Hiperactividad de la fosforibosil fosfato sintetasa
acumulación de cristales de urato monosódico en las Deficiencia de hipoxantina-guanina fosforibosiltransferasa
Hiperuricemia secundaria:
articulaciones (artritis), riñones (nefropatía) y vías urinarias. Ingesta excesiva de purinas
El depósito de cristales produce liberación de citocinas y Recambio de nucleótidos elevado: psoriasis
activación, así como reclutamiento de células inflamatorias Incremento de la degradación de ATP (ejercicio)
Mecanismo Deficiencia de glucosa-6-fosfatasa
(PMN) y no inflamatorias (endotelio y sinoviocitos) y combinado Hipoperfusión tisular
quimiotaxis. No se conoce a detalle el proceso completo de Alcoholismo
la formación de cristales pero algunos de los factores son: el
acido úrico tiene menor solubilidad a menor temperatura, por lo cual pueden formarse cristales con mayor facilidad en lugares distales
como dedos de los pies y de las orejas. La solubilidad de los UMS es menor cuando el pH es ácido, microtraumas repetidos, alteración
de la matriz extracelular. DIAGNSOTICO: Clínico; Artritis aguda, la más frecuente y temprana manifestación de gota, inicialmente afecta
una articulación (suele localizarse en la primera metatarsofalangica = podagra), pero puede ser poliarticular en episodios tardíos. La
gota aguda inicia por la noche con dolor severo, hinchazón, calor. El ataque generalmente desaparece espontáneamente en 3-10 días.
Aunque algunos pacientes suelen tener solo un ataque, otros episodios recurrentes, con intervalos de variación sin síntomas. La gota
aguda puede ser precipitada por exceso de purinas en la dieta, trauma cirugía, ingestión excesiva de alcohol, terapia hipouricemica y
enfermedades serias como infarto al miocardio. Artritis crónica; una proporción de pacientes con gota pueden tener una sinovitis
unilateral crónica, esta suele ser rara. Puede también presentarse con tofos (agregados de UMS, rodeados por células de reacción
inflamatoria, nódulos, subcutáneos, de aspecto amarillento y duros) periarticulares. Tofos extraarticulares: Algunos están en la cápsula
del olecranon, hélix y antihélix del pabellón auricular, superficie cubital o antebrazo, tendón de Aquiles, generalmente se desarrollan
entre 5-10 años después del inicio de ataques de gota. Tenosinovitis. Nefropatía por uratos: por depósito de UMS en el intersticio y
pirámides, puede causar IRC. Nefropatía aguda por acido úrico: causa reversible de falla renal aguda debida a precipitación de los
uratos en los túbulos, los pacientes que reciben tratamiento citotóxico para neoplasia están en riesgo. Nefrolitiasis por acido úrico:
responsable de 10% de las litiasis renales. CRITERIOS PARA LA CLASIFICACION de artritis gotosa aguda: A) La presencia de cristales de
UMS característicos en el liquido sinovial (gold estándar para dx) o bien B) Un tofo cuyo contenido de cristales de UMS este
comprobado por medios químicos o microscopia de luz polarizada o bien C) La presencia de 6 de los siguientes 12 fenómenos: 1. Mas
de un ataque de artritis aguda. 2. Desarrollo máximo de brote inflamatorio en el curso del día. 3. Ataque monoarticular. 4. Franco
enrojecimiento del area articular. 5. Localización primera articulación metatarsofalángica. 6. Ataque unilateral que afecte la primera
articulación metatarsofalngica. 7. Ataque unilateral en una articulación del tarso. 8. Tofo: sospecha o confirmación. 9. Hiperuricemia.
10. Infamación asimétrica en estudio radiográfico. 11. Quistes subcutáneos sin erosiones óseas. 12. Cultivo negativo del liquido articular
obtenido durante un ataque agudo. Los ataques de gota agudos se relacionan con cambios en el nivel plasmático de AU (tanto aumento
como disminución). La identificación de UMS en liquido sinovial es patognomónico, se ven al microscopio como agujas y
birrefringentes. Radiológico: En ataque agudos pueden mostrar tumefacción de tejidos blandos. Los tofos son radiotransparentes,
aunque en ocasiones pueden tener calcificaciones en su interior, los cambios característicos son edema difuso, prominencias nodulares,
erosiones intraarticulares y extraarticulares con imagen de “techo colgante” y calcificaciones. TRATAMIENTO: Dentro de la medidas
generales dieta baja en alimentos ricos en purinas como: carnes, vísceras, piel, alimentos ahumados, rostizados, los dos factores más
importantes que favorecen la hiperuricemia son: ingesta de alcohol y ayuno prolongado. Artritis gotosa aguda: Analgesia, AINES
(indometacina, diclofenaco, naproxen) primera elección. Colchicina efectiva dentro de las primeras 24hrs del ataque, contraindicada
cuando hay insuficiencia renal o citopenias. Dosis de 1-6 mg/día. Glucocorticoides intraarticulares; se debe descartar primero artritis
séptica. Glucocorticoides sistémicos; uso breve, considerado en pacientes con ataque de gota poliarticular. Agentes hipouricemicos:
Indicados para iniciar a bajar el acido úrico, incluyendo recurrencias de artritis gotosa aguda, artritis poliarticular, gota tofosa, litiasis
renal. No deben iniciarse en el ataque agudo, pueden precipitar una llamarada y se debe considerar su uso concomitante con
colchicina. Alopurinol: disminuye la síntesis de acido úrico por inhibición de la xantino-oxidasa, reducir dosis en insuficiencia renal.
Dosis de 100-300mgs, pueden darse hasta 800mgs. Uricosúricos (probenecid, sulfinpirazona): Incrementan la excreción de acido úrico,
por inhibición de la reabsorción tubular (inefectivo en insuficiencia renal), no debería usarse en >60 años de edad. Iniciarse una vez que
haya pasado el ataque agudo.

CASO CLINICO
Varón de 71 años con antecedentes personales de hipertensión arterial sistémica en tratamiento con enalapril, 20 mg diarios, e
hiperuricemia, con episodios de monoartritis en el pie izquierdo, Ingresa por dolor y tumefacción en el segundo dedo de la mano

CURSO ENARM CMN SIGLO XXI TEL: 36246001 Pharmed Solutions Institute PÁGINA 96
MANUAL DE TRABAJO DEL CURSO ENARM CMN SIGLO XXI
derecha y en la cuarta articulación interfalángica proximal de la mano izquierda, sin fiebre ni otros datos clínicos. La exploración física
mostró nódulos de Heberden en ambas manos, con signos de artritis en la segunda interfalángica distal de la mano derecha, con gran
tumefacción distal y aumento de partes blandas, con un tofo acompañante, así como en la cuarta interfalángica proximal de la mano
izquierda. En las pruebas de laboratorio destacó una uricemia de 7,20 mg/dl, con hemograma, perfil bioquímico y reactantes de fase
aguda dentro de la normalidad (VSG: 14 mm/h, PCR: 0,49 mg/dl). En las radiografías de las manos se apreciaron cambios degenerativos
en las articulaciones metacarpofalángicas, interfalángicas y trapeciometacarpianas. La interfalángica distal del segundo dedo presentó
aumento de partes blandas, con disminución del espacio articular y erosiones en sacabocados. La ecocardiografía y los hemocultivos
seriados fueron negativos.

PREGUNTA
Cual de los siguientes resultados del aspirado de liquido sinovial en este caso es mas frecuente encontrar?

RESPUESTA
a.- Cristales de urato monosódico.
b.- Cristales de pirofosfato cálcico.
c.- Ambos tipos de cristales.
d.- Cristales de pirofosfato.

LUPUS ERITEMATOSO SISTEMICO (LES)


CIENCIAS BASICAS: Prototipo de enf. autoinmune, multiorganica, multisistémica, se asocia con la producción de autoanticuerpos contra
componentes del núcleo, mediada por inmunocomplejos (hipersensibilidad III). SALUD PUBLICA: Más en mujeres de 20-40años.
Relación H:M es de 9:1. Lupus eritematoso sistémico 70%, lupus cutáneo l5%, Sx de sobreexposicion/Enf. mixta de tejido conectivo
10%, lupus eritematoso inducido por drogas 5%. PATOGENIA: Enfermedad de etiología desconocida. Participan para el desarrollo
factores genéticos como: HLA-DR2, DR3, DR4 (75%), deficiencia de complemento C1, C2, C4 (a mayor deficiencia mayor riesgo para
nefritis lupica), mayor incidencia en familiares de 1er grado. Factores
ambientales: Rayos UVB (apoptosis de queratinocitos), medicamentos Clasificación de nefritis lupica según la OMS
(hidralacina, procainamida, fenitoina, isoniazida), infecciones (VEB), TIPO I Nefropatía lupica mesangial mínima
mimetismo molecular. Factores hormonales: estrógenos son permisivos TIPO II Nefropatía lupica mesangial
de autoinmunidad, puerperio y anticonceptivos exacerban él LES. Factores TIPO III Nefropatía lupica focal
inmunológicos: falla en el control del sist. Inmunológico, producción TIPO IV Nefropatía lupica difusa
excesiva de autoanticuerpos. Se producen autoanticuerpos, se forman TIPO V Nefropatía lupica membranosa
complejos inmunes, disminuye la apoptosis de cel. T autoreactivas, hay TIPO VI Nefropatía lupica esclerosante (terminal)
actividad aumentada de linfocitos T cooperadores (CD4) y disminución de
linfocitos T supresores (CD8); todo esto junto con los factores nos lleva a LES. DIAGNÓSTICO: Cuadro clínico: Hay períodos de remisión
y exacerbación, síntomas inespecíficos en 95%; fiebre, astenia, adinamia. Manifestaciones musculo-esqueléticas son las más frecuentes
(90-95%); artralgias (70-100%), artritis no erosiva (60%), pulgar en z, mialgias, hiperlaxitud, osteonecrosis avascular (dolor en cadera y/o
hombro). Mucocutaneas frecuentes (80-90%); lo mas común presencia de lupus discoide (15-30%: cara, hombro, cuello, retroauricular),
rash en alas de mariposa (50%), eritema malar tiene la característica de inestabilizarse e incrementarse con la exposición al sol,
fotosensibilidad (70%), ulceras orales (40%), vasculitis cutánea y livedo reticularis. Hematológicas frecuentes (80-90%); anemia (70%),
leuco, linfo y trombocitopenia, linfadenopatía y esplenomegalia. Neurológicas (50-60%); neuralgia, meningitis aséptica, disfunción
cognitiva (50%), sx. neural orgánico (35%), convulsiones y psicosis (20%), neuropatía periférica, EVC, cefalea. Cardiopulmonares;
pleuritis (50%), derrame pleural (30%), neumonitis lupica ( en 10%; de las complicaciones más graves, mortalidad 60-70%), pericarditis
(30%), endocarditis de Libman-Sack (insf. aórtica y mitral: es aséptica). Renales frecuentes (50%); principal factor que condiciona la
sobrevida del paciente (Cr >1.4, hipotensión y sx. nefrótico), glomerulonefritis membranosa y mesangial, nefritis lupica factor de mal
pronóstico y es la principal causa de muerte en lupus hasta 50%. Laboratorio: Perfil metabólico (tienen aterogenesis acelerada y lípidos
alterados), EGO (proteinuria), reactantes de fase aguda (PCR y VSG), anticuerpos antinucleares (ANA; considerados gold standar para
LEG, especificidad media), C3 y C4, anti-DNA ds más específico para lupus ( especificidad excelente), anti- histona ( relacionado con
lupus inducido por drogas), anti sm ( especificidad excelente), anti-Ro (SSA, se relaciona con lupus neonatal, pedir en embarazadas),
anti-La (SSB, si sale positivo protege contra daño renal). Anti-Ro y anti-La; son específicos pero son más sensibles para Sx. de Sjogren.

CRITERIOS DIAGNOSTICOS:
Cutáneos
1. Rash malar: eritema malar fijo, plano o palpable
2. Rash discoide: parches eritematosos palpables, con ampliación queratosica y taponamiento folicular, pueden existir cicatrices atroficas.
3. Fotosensibilidad: reacción inusual a la luz solar, observado por medico
4. Ulceras orales: o nasofaringeas, usualmente indoloras, observadas por medico
Sistémicos
1. Artritis: no erosiva que involucre 2 o más articulaciones periféricas, caracterizadas por hipersensibilidad, inflamación o derrame.
2. Serositis: pleuritis o pericarditis
3. Trastorno renal: proteinuria persistente (>0.5 g/d o más de +++), o cilindros celulares de cualquier tipo.
4. Trastorno neurológico: convulsiones o psicosis en ausencia de otras causas.
Laboratorio:
1. Trastorno hematológico: Anemia hemolítica o leucopenia (>4000 en 2 ocasiones), linfopenia (>1500 en 2 ocasiones) o trombocitopenia (>100,000 en
ausencia de drogas causantes)
2. Trastorno inmunológico: Anti-DNA o anti- sm, o anticuerpos antifosfolipido
3. Anticuerpos antinucleares: en ausencia de drogas conocidas, que puedan ser asociadas a lupus inducido por drogas.
SE DICE QUE UNA PERSONA TIENE LEG, SI PRESENTA 4 DE LOS 11 CRITERIOS DE FORMA SIMULTÁNEA O SERIADA EN EL CURSO DE LA ENFERMEDAD,
DURANTE O EN CUALQUIER PERIODO DE LA MISMA.

CURSO ENARM CMN SIGLO XXI TEL: 36246001 Pharmed Solutions Institute PÁGINA 97
MANUAL DE TRABAJO DEL CURSO ENARM CMN SIGLO XXI

FARMACOS QUE INDUCEN LUPUS: Constituye el 10%; Hidralacina, procainamida, isoniacida, clorpromacina, d-penicilamina, quinidina,
interferon alfa, fenitoina, anti-TNF,predominan manifestaciones constitucionales y pleuropericarditis, no afección renal, ni neurológica,
90% remite a los 3 meses de suspender el fármaco. Asociado a HLA-DR4, anti- DNA negativos, ANA positivos, anti-histona positivo e
hipocomplementemia. LUPUS NEONATAL: Se produce en menos de 5% de los RN de madres portadoras de autoanticuerpos anti-Ro y
anti-La, hasta en 25% de madre sanas con anti- Ro circulante, lesiones cutáneas en ateas fotoexpuestas a partir de los 2 meses de vida,
bloqueo A-V completo (complicación más grave del lupus neonatal) y alteraciones hematológicas. TRATAMIENTO: Basado en el tipo y
la severidad. AINES (Ibuprofeno 400-800mg c/6-8hrs)Corticoesteroides (prednisona 1mg/Kg, en caso de daño orgánico agudo;
metilprednisolona 1g/d x3dias ), hidroxicloroquina (antimalaricos) y aspirina (estos autorizados por la FDA). No exponerse al sol, dietas
sin alfalfa. Manifestaciones leves: AINES, esteroides a dosis bajas (>20mg/kg/día), hidroxicloroquina (400mg/día. efecto secundario
toxicidad retiniana). Lesiones cutáneas, fotoprotección (FPS>100), corticoides tópicos. Manifestaciones graves (neurolupus,
glomerulonefritis, neumonitis, trombocitopenia, anemia hemolítica) pulsos de metilprednisolona, prednisona, inmunosupresores como
aziatropina, metotrexate (son ahorradores de corticoides). 1ra elección en nefritis lupica ciclofosfamida (10_15mg/kg/día). Aziatropina;
nefritis lupica y daño hematológico. Metotrexatearticulares (piedra angular en artritis reumatoide), manifestaciones articulares 7.5-
15mg/sem. Si no responden a la terapia inicial lo más usado en terapia biológica es rituximab (anti-CD-20).). Anticoagulación puede
estar indicada en complicaciones tromboticas.

CASO CLINICO
Mujer de 17 años que padecía de LES. En el momento del diagnóstico debutó con una nefritis lúpica manifestada por un síndrome
nefrótico, con proteínas en orina de 24 horas de 3,3 g, colesterolemia de 280 mg% y albuminemia de 1,8 g/dL, por lo cual recibió
tratamiento con metil-prednisolona succinato en bolos durante tres días (tres dosis), y se mantuvo con prednisona 1 mg/kg de
peso/día, observándose mejoría del cuadro. Igualmente la paciente presentó una cefalea intensa que calmaba parcialmente con
analgésicos y sin signos de irritación meníngea, que fue interpretada como un síntoma asociado al lupus. Continuó el tratamiento con
prednisona hasta 15 días después de su egreso, pero disminuyó a una dosis de 35 mg/día. Un mes después, la paciente acudió
nuevamente para control. Los exámenes demostraron deterioro de la función renal, asociado a un síndrome nefrótico impuro, con
hipertensión arterial (160/110). Se aumentó la dosis de prednisona a 1 mg/kg de peso/día y se agregó captopril 50 mg cada 8 horas. La
paciente refería persistencia de la cefalea frontoparietal, opresiva, de fuerte intensidad, que ya no mejoraba con analgésicos y que era
exacerbada por los movimientos de la cabeza, sin otros signos y síntomas asociados. A los pocos días del ingreso presentó fiebre de 40
°C, náuseas, vómitos y fotofobia.

PREGUNTA
Cual es la conducta a seguir mas adecuada?

RESPUESTA
a.- Agregar diurético, neuroproteccion y antibiótico.
b.- Realizar punsion lumbar para citoquimico, citológico y cultivo.
c.- Realizar IRM de cráneo.
d.- Aumentar corticoide y FARME.

PREGUNTA
Entre 30 y 50% de la morbilidad y mortalidad de los pacientes con lupus eritematoso sistémico (LES) son atribuidas a las infecciones,
cual de las siguientes es la menos frecuente?

RESPUESTA
a.- SNC.
b.- Piel.
c.- Pulmones.
d.- Sistema genitourinario.

CASO CLINICO
Mujer de 67 años de edad diagnosticada de hipertensión arterial. Ingresó por sensación nauseosa, infección urinaria y anemia. Análisis:
Htº 27,8%; Hb 9,5 g/dl. VSG 63 mm. Urea 182 mg/dl; creatinina 4,7 mg/dl; calcio 9 mg/dl; fósforo 4,2 mg/dl, proteínas totales 9 g/dl.
Aclaramiento de creatinina (fórmula de Cockroft-Gault): 17,73 ml/min. Inmunoproteínas y complemento normal. Cadenas Kappa-s 774
mg/dl, cadenas Lambda 392 mg/dl. En el proteinograma se observó un pico de amplia base en región Gamma, con aumento de IgG
(193%) y cadenas ligeras Kappa (191%) y Lambda (180%). Índice K/L= 1,97. Cadenas ligeras en orina: cadenas Kappa 13,7 mg/dl (0-0,7);
cadenas Lambda 6,880 (0-0,39). Se realizó aspirado y biopsia de médula ósea, que resultó normal. THS: 4,85 µUI/ml, T4 libre 1,03 ng/fl;
Ac anti-TPO 22,5 UI/ml; Ac anti-Tiroglobulina 115,3 UI/ml. PTH: 110 pg/ml. Orina (tira reactiva): proteínas 25 mg/l; sedimento:
abundantes leucocitos. Marcadores tumorales: normales. Serología viral: negativa. Autoanticuerpos: ANA +, anti-ADN positivos.
PREGUNTA
Considerando las características del caso clínico, cual es el abordaje terapéutico mas adecuado?

RESPUESTA
a.- Metotrexate.
b.- Ciclosporina.
c.- Prednisona.
d.- Aziatropina.

CURSO ENARM CMN SIGLO XXI TEL: 36246001 Pharmed Solutions Institute PÁGINA 98
MANUAL DE TRABAJO DEL CURSO ENARM CMN SIGLO XXI
PREGUNTA
El lupus eritematoso sistémico (LES) es una enfermedad inflamatoria con afectación sistémica, cual es el porcentaje de pacientes que
presentan signos de nefropatía?.

RESPUESTA
a.- Al menos 60%.
b.- Al menos 50 %.
c.- Al menos 40 %.
d.- Al menos 30%

CASO CLINICO
Mujer de 47 años de edad con Nefropatía lúpica. AHP a los 17 años presenta Púrpura Trombopénica que lleva a una esplenectomía, a
los 19 años de edad ingresa tras presentar un brote de artralgias, fiebre, edemas y lesiones eritematosas en cara, siendo diagnosticado
de LES. En ese momento destaca la presencia de síndrome nefrótico con alteraciones en el sedimento, una creatinina sérica de 1.9
mg/dl y en el estudio cardiológico se objetiva prolapso de la válvula mitral. Se practica biopsia renal. El cilindro contiene 12 glomérulos,
grandes, lobulados, con proliferación mesangial difusa de distribución irregular, núcleos en carirorrexis, trombos intracapilares
ocasionales y paredes capilares engrosadas en “asa de alambre”. Las pequeñas arterias, no tienen alteraciones. La técnica de
inmunofluorescencia directa demuestra la presencia de depósitos subendoteliales y mesangiales de IgG, C3, C1q, IgM e IgA.
Diagnóstico: Nefritis lúpica Clase IV. Se decide tratamiento con corticoides y azatriopina, normalizándose la función renal y
negativizándose la actividad inmunológica, persistiendo proteinuria de 2 gr. /24 h. A los 21 años se detectan cifras elevadas de tensión
arterial. En su primer embarazo, presenta aborto espontáneo tras 2 meses de amenorrea. A los 22 años se retira la inmunosupresión en
el curso de su segundo embarazo que llega a término, presentando hipertensión arterial en el octavo mes. Inmediatamente tras el
parto en mayo de presenta empeoramiento de su situación con hipertensión arterial y proteinuria de 3 gr. /24 h, reiniciándose el
tratamiento con esteroides y azatriopina, quedando la paciente asintomática, sin proteinuria, con función renal normal, estudio
inmunológico negativo pero persistiendo la hipertensión arterial. A los 25 años de edad comienza con crisis epilépticas detectándose en
TAC cerebral lesiones isquémicas múltiples y un mes después, segundo brote lúpico con fiebre, afectación articular, cutánea,
reaparición de proteinuria con función renal normal y actividad inmunológica. A los 36 años de edad, es intervenida de aneurisma
fusiforme de la arteria carótida derecha. Un año después presenta un accidente cerebro vascular con hemiplejia izquierda y
recuperación total, se observaron dilataciones en carótida y aorta abdominal en estudios posteriores.

PREGUNTA
Considerando la comorbilidad del LES cual de las siguientes entidades clínicas es mas probable considerando la evolución del caso?

RESPUESTA
a.- Sindrome infeccioso.
b.- Arteriosclerosis.
c.- Sindrome antifosfolipidico.
d.- Necrosis avascular ósea.

CASO CLINICO
Un varón de 70 años, con antecedentes de diabetes mellitus, ingresó por febre de 38,5ºC de una semana de evolución. A la exploración
destacaba presión arterial 106/75 mmHg, frecuencia cardíaca de 95 lpm y edema importante hasta ambas rodillas. En el tobillo derecho
y en el talón izquierdo presentaba úlceras cutáneas con un exudado purulento del cual se tomaron muestras. En la bioquímica, al
ingreso, destacaba leucocitosis 19.800 cél/mm3, AST 68 U/L, creatinina 260 μmol/L, urea 16,4 mmol/L; el resto fue normal. El ECG era
anodino. La radiografía de tórax evidenció una cardiomegalia moderada. El Eco-Doppler cardíaco mostró un derrame pericárdico difuso
de aproximadamente 1 cm, con imágenes compatibles con depósito de fibrina y signos sugestivos de compromiso hemodinámico. En
las 12 horas siguientes presentó taquicardia y oliguria progresivas, con hipotensión arterial mal tolerada.

PREGUNTA
Cuál es la conducta a seguir más adecuada.

RESPUESTA
a.- Pericardiocentesis.
b.- Pericardientomia urgente.
c.- Manejo conservador.
d.- Corrección del estado hemodinamico.

CASO CLINICO
Paciente con 50 años de edad, con LES hacía cuatro años. Durante su evolución presentó como intercurrencia un síndrome
antifosfolipídico y nefritis lúpica que requirió tratamiento con anticoagulantes orales, azatioprina 100 mg/día, hidroxicloroquina 200
mg/día y prednisona 20 mg/día en dosis decreciente. Es ingresado a urgencias por familiares debido a que presenta desorientación,
dificultad para movilizar el brazo y pierna del mismo lado y cegura súbita.

PREGUNTA
Cuál es la complicación mas probable en este caso?

CURSO ENARM CMN SIGLO XXI TEL: 36246001 Pharmed Solutions Institute PÁGINA 99
MANUAL DE TRABAJO DEL CURSO ENARM CMN SIGLO XXI
RESPUESTA
a.- Vasculitis cerebral.
b.- Isquemia cerebral transitoria.
c.- Embolia cerebral.
d.- Neurolupus.

CASO CLINICO
Se trata de paciente femenino de 32 años de edad la cual se encuentra diagnosticada con lupus eritematoso sistémico con 4 años de
evolución con tratamiento irregular, ingresa a admisión continua debido a que desde hace 48 horas inicia súbitamente con estado
confusional agudo previamente con letargia, a la exploración física se observa ictérica, su estado no permite responder preguntas, sus
signos vitales se encuentra taquicardia, hipotensión, palidez y disnea, no se auscultan alteraciones cardiorespiratorios, sin embargo en
abdomen se detecta esplegnomegalia, se realizan exámenes de rutina con hemoglobina de 6 g/dl, leucos de 6,200 y plaquetas de
300,000, bilirrubina 4,5 g/dl con reticulocitos de 19 % así como microesferocitosis, resto sin datos por agregar:

PREGUNTA
Cual es la complicación actual que presenta esta paciente que es potencialmente mortal aguda?

RESPUESTA
a.- Sindrome hemolítico-uremico.
b.- Purpura trombocitopenica trombotica.
c.- Anemia hemolítica autoinmunitaria.
d.- Anemia Hemolitica microangiopatica.

VASCULITIS:
CIENCIAS BASICAS: Grupo heterogéneo de enfermedades raras, que pueden ocurrir de forma independiente o como complicaciones de
una enfermedad previamente establecida. Involucran la vasculatura de cualquier órgano, la presentación depende del calibre del vaso
afectado.
CLASIFICACION:
ARTERITIS DE CELULAS GIGANTES (ACR): La más común de las vasculitis
VASOS DOMINANTES VASCULITIS
Grandes vasos Arteritis de células gigantes (ACR)
sistémicas. También llamada enf. de Horton, afecta a las ramas craneales
Arteritis de Takayasu del arco aórtico, especialmente a la arteria temporal. Más común en
Medianos vasos Poliarteritis nodosa (PAN) mayores de 50 años. Más frecuente en mujeres en relación 2:1. CLINICO:
Enf. De Kawasaki (AT)
Fiebre de bajo grado, fatiga, pérdida de peso, mialgias, anorexia, cefalea
Pequeños vasos y Granulomatosis de Weneger (GW)
arterias medianas Sx. de Churg-Strauss (SCS) (60%). Afección de la arteria oftálmica (20%), neuritis óptica, diplopía,
ASOCIADAS A ANCAS Poliangitis microscópica (PA) amaurosis fugaz y ceguera (dar tx. médico precoz urgente; esteroides en
Pequeños vasos Purpura de Henoch-Scholein (PHS) dosis altas). Afección de la A. facial: claudicación mandibular. CRITERIOS
(leucocitoclástica) Arteritis leucocitoclástica cutánea
DIAGNOSTICOS: 1. Edad >50 años 2. Cefalea de reciente inicio 3.
Hipersensibilidad de la arteria temporal (piel cabelluda) o disminución de la pulsación 4. VSG >50mm/h 5. Biopsia granulomas y
vasculitis. Con 3 de 5 criterios se hace el diagnostico. DIAGNOSTICO: VSG (98%), PCR elevadas, anemia y alteración de pruebas de la
función hepática (70%), biopsia de la arteria afectada (afección en parches, no es continua); infiltrado mononuclear, con presencia de
granulomas y células gigantes. TRATAMIENTO: Prednisona 1mg/kg/día, durante las primeras semanas con descenso gradual. ARTERITIS
DE TAKAYASU (AT): Llamada enf. “sin pulso” o síndrome del arco aórtico. Más común en mujeres en edad reproductiva. Proceso
inflamatorio crónico granulomatoso, que afecta la aorta y sus ramas, provocando generalmente síntomas isquémicos. CLINICA: Se
divide en fases: fase I; periodo inflamatorio con fiebre, artralgias y pérdida de peso, trill aórtico o subclavio. Fase II; hipersensibilidad y
dolor en vasos sanguíneos, disminución asimétrica en pulsos en extremidades, claudicación, hipertensión renovascular, sincope
neurogénico. Fase III; periodo fibrotico. La artería más afectada hasta 93% es la subclavia. CRITERIOS DIAGNOSTICOS: 1. Edad <40años
de edad 2. Claudicación de extremidades 3. Disminución del pulso de la arteria braquial 4. Diferencia de presión arterial sistólica
>100mmHg en los brazos 5. “Trill” sobre arterias subclavia o aorta. 6. Arteriografía anormal. Con 3 de 6 criterios se hace el diagnostico.
CLASIFICACION: de acuerdo al sitio afectado: I: Ramas del arco aórtico. IIa Aorta ascendente, arco aórtico y sus ramas. IIb Aorta
descendente, arco aórtico y sus ramas , A. torácica descendente. III. A. torácica descendente, A abdominal y/o arterias renales. IV. A.
abdominal y/o A. renales. V. Combinación de los tipos IIb y IV. DIAGNOSTICO: VSG Y PCR elevadas. Arteriografía=Gold estándar;
oclusión, estenosis, irregularidad y aneurisma. TRATAMIENTO: Esteroides, MTX, Aziatropina y antiplaquetarios, en enfermedad
refractaria micofenolato y ciclofosfamida. PANARTERITIS NODOSA CLASICA (PAN): Inflamación necrotizante de arteria de pequeño y
mediano calibre (respeta capilares y vénulas). Disminución de la luz vascular. Se asocia a VHB 30% y a VHC 5% CLINICA: Insuficiencia
renal, HTA (70%), purpura papable (50%), mononeuritis múltiple (50%), dolor abdominal (40%), ICC, IAM, pericarditis (30%), raras
afecciones pulmonares. CRITERIOS DIAGNOSTICOS: 1. Pérdida de peso >4Kg 2. Livedo reticularis 3. Dolor e hipersensibilidad testicular
(orquitis) 4. Mialgias, debilidad, dolor de extremidades (50-60%). 5. Mononeuropatía o polineuropatía 6. Presión diastólica >90mmHg 7.
BUN elevado (>40mg/dl) o Cr mayor de 1.5 mg/dl 8. Virus de hepatitis B 9. Arteriografía anormal (aneurisma, oclusión, estenosis de
arterias viscerales) 10. Biopsia; vasculitis de mediano y pequeños vasos. Con 3 de 10 criterios se hace el diagnostico. DIAGNOSTICO:
Angiografía, biopsia; infiltrado de PMN y necrosis fibrinoide (característico ausencia de granulomas y eosinofilos).TRATAMIENTO:
Glucocorticoides, ciclofosfamida, antiviral si está asociada a VHB O VHC. GRANULOMATOSIS DE WENEGER (GW): Enfermedad
inflamatoria granulomatosa necrotizante con vasculitis sistémica. Predomina en adultos jóvenes. Predomina en capilares y vénulas.
CLINICO: Vías aéreas superiores (95%); Sinusitis, otitis media y destrucción de tabique nasal. Pulmón (90%); Infiltrados pulmonares
cavitados bilaterales y no migratorios. Nódulos hemorrágicos y hemoptisis. Glomerulonefritis (75%); Focal y segmentaria rápidamente
progresiva. Ocular (50%); epiescleritis, uveítis. Neuropatías periféricas o craneales. CRITERIOS DIAGNOSTICOS: 1. Inflamación oral o
nasal; ulceras orales, descarga nasal purulenta o hemorrágica 2. Radiografía de tórax con presencia de nódulos, infiltrados fijos o

CURSO ENARM CMN SIGLO XXI TEL: 36246001 Pharmed Solutions Institute PÁGINA 100
MANUAL DE TRABAJO DEL CURSO ENARM CMN SIGLO XXI
cavitaciones 3. Hematuria microscópica o restos de eritrocitos 4. Biopsia con inflamación granulomatosa. Con 2 de 4 criterios se hace el
diagnostico. DIAGNOSTICO: c-ANCA (Anticuerpos antiproteinasa-3) positivos 90%. TAC o Rx de tórax; nódulos, infiltrados, cavitaciones.
Elevación de BUN, Cr, proteinuria, sedimento urinario activo. Biopsia; inflamación granulomatosa necrotizante de arteriolas, capilares y
vénulas. TRATAMIENTO: Ciclofosfamida + prednisona, MTX o aziatropina. TMP/SFX puede prevenir las recaídas de infecciones de vías
aéreas superiores. POLIANGITIS MICROSCOPICA (PA): Vasculitis necrotizante de pequeños vasos, puede haber o no granulomas.
CLINICO: Síntomas constitucionales; pérdida de peso, fiebre (50%), fatiga, mialgias. Renal; hematuria, glomerulonefritis rápidamente
progresiva (90%). Pulmonar; tos y/o hemoptisis, hemorragia pulmonar (30%). Neurológico; mononeuritis multiplex (40%).
DIAGNOSTICO: p-ANCA (anti-mieloperoxidasa) positivos 70%. Biopsia; inflación pauci-inmune necrotizante de arteriolas capilares y
vénulas, con o sin granulomas o infiltrados eosinofilicos. TRATAMIENTO: Ciclofosfamida + aziatropina, MTX o aziatropina (ahorradores
de esteroides). SINDROME DE CHURG-STRAUSS (SCS): Inflamación granulomatosa eosinofilica que involucra pulmón. A cualquier edad
más entre 30-40 años. CLINICO: Asma de difícil control (95%), enfermedad eosinofilica infiltrativa, vasculitis de pequeños vasos con
granulomas. Neuropatía; mononeuritis multiplex (60-70%), parestesias en guante o calcetín, arteritis coronaria y miocarditis.
Glomerulonefritis. CRITERIOS DIAGNOSTICO: 1. Asma 2. Eosinofilia (>10%) 3. Mono o polineuropatía 4. Infiltrados pulmonares
migratorios transitorios 5. Anormalidades paranasales 6. Eosinofilia extravascular en la biopsia. Con 4 de 6 criterios se hace el
diagnostico. DIAGNOSTICO: Dato cardinal para diagnostico: eosinofilia periférica 80%. ANCA (c-ANCA o p-ANCA) positivo en 50%.
Biopsia: microgranulomas, necrosis fibrinoide y trombosis de péquelas arterias y venas con infiltrados eosinofilicos. Rx. de tórax: puede
haber infiltrados pulmonares. TRATAMIENTO: Ciclofosfamida + prednisona, MTX o aziatropina. PURPURA DE HENOCH-SCHONLEIN
(PHS): La más común de las vasculitis (leucocitoclástica) en niños (<16años). Suele aparecer después de una infección de tracto
respiratorio o ingesta de fármacos, es mediada por IgA. Producida por depósito de inmunocomplejos. CLINICO: Purpura palpable en
superficies extensoras y glúteos, poliartralgias, dolor abdominal tipo colico (edema de pared, invaginación intestinal poco frecuente).
Nefritis; hematuria microscópica y proteinuria. Fiebre. CRITERIOS DIAGNOSTICOS: 1. Purpura palpable 2. Edad menor de 20 años al
momento de la aparición 3. Angina intestinal (dolor abdominal) 4. Biopsia; granulocitos en la pared de arteria o venas. Con 2 de 4
criterios se hace el diagnostico. DIAGNOSTICO: Conteo plaquetario, elevación de IgA sérica. Biopsia de piel: vasculitis leucocitoclástica
con IgA y depósitos de C3 en la pared de los vasos. Biopsia renal; deposito mesangial de IgA. TRATAMIENTO: De sostén. Esteroides
(1mg/kg) y fármacos modificadores de enfermedad severa. CRIOGLOBULINEMIA: Vasculitis rara con depósitos inmunes de
crioglobulinas (proteínas que se precipitan con el frío), afecta piel y glomérulo, se asocia con infección por VHC (90%). CLINICA:
Discinecia, confusión, cefalea, y EVC isquémico, causado por hiperviscocidad en tipo I. Estasis vascular; livedo reticularis, necrosis
digital, en tipo I. Triada de manifestaciones en tipo II Y III; purpura, artralgias, mialgias. CLASIFICACION: Tipo I: Constituida por una sola
inmunogloblina IgM. Macroglobulinemia de Waldestron o mieloma. Tipo II: IgM monoclonal tipo K, que presenta actividad tipo factor
reumatoide e IgG policlonal. Sx. proliferativos. Tipo III: Constituida por 2 inmunoglobulinas (IgM-IgG) de carácter policlonal: Forman
complejos Ag:Ac circulantes: procesos que cursan con inmunocomplejos, infecciones crónicas, cirrosis. DIAGNOSTICO: Serología viral,
hipocomplementemia, crioglobulinas. TRATAMIENTO: Terapia antiviral, esteroides, ciclofosfamida, plasmaferesis. SINDROME DE
BEHCET: Vasculitis multisistemica, caracterizada por presencia de ulceras genitales y orales recurrentes dolorosas. Se asocia con HLA-
B51. CLINICO: Ulceras orales dolorosas (>3 episodios anuales), de base necrótica amarillenta, se curan en 1 o 2 sem sin dejar cicatriz.
Ulceras genitales (80%), si dejan cicatriz. Foliculitis (80%), eritema nodoso (50%), erupciones acneiformes. Ocular; manifestación más
grave, uveítis posterior que puede condicionar pérdida de visión (20%, tx. Agresivo con ciclosporina o anti-TNF). Puede haber meningitis
aséptica, meningoencefalitis. DIAGNOSTICO: Fenómeno de patergia (patognomónico), se inocula sol. Inyectable lesión de >2cm es
positiva. Biopsias de ulceras. Fondoscopia. CRITERIOS DIAGNOSTICOS: 1. Ulceras aftosas orales recurrentes 2. Ulceras genitales
recurrentes 3. Lesiones oftálmicas: uveítis (con hipopión), escleritis, vasculitis retiniana, neuritis óptica. 4. Lesiones cutáneas; pápulas
foliculitis, eritema nodoso 5. Patergia positiva. El diagnostico se hace con el N.1 y 2 más del resto. TRATAMIENTO: Aziatropina, para
ulceras (colchicina, dapsona, talidomida), corticoides, para uveítis la ciclosporina. ENFERMEDAD DE KAWASAKI: Sindrome
linfomucocutáneo. Más común en menores de 5 años (80%), ligero predominio en sexo masculino. Etiología desconocida. CLINICA:
Cuadro exantemático febril, que no responde a antibióticos y congestión conjuntival o no. Alteraciones labiales; eritema, fisuras,
sangrado con formación de costras y lengua en frambuesa. Adenopatías cervicales dolorosas y exantema en tronco y extremidades.
Formación de aneurismas (25%), vasculitis coronarias (forma más mortal), carditis (80%). DIAGNOSTICO: Clínico, factores reactantes de
fase aguda, angiografía coronaria, no se asocia a ningún autoanticuerpo. CRITERIOS DIAGNOSTICOS: Fiebre persistente por al menos 5
dias, mas 2 de los siguientes: 1. Cambios en extremidades inferiores 2. Exantema polimorfo 3. Inyección conjuntival bilateral 4. Cambios
en labios y cavidad oral 5. Linfadenopatía cervical. TRATAMIENTO: Gammaglobulina IV (2g/Kg dosis única o 400 mg/kg, durante 4 dias.
ASA de 100-1500mg/día. Controversial uso de esteroides

CASO CLINICO
Mujer de 60 años, con antecedentes de hipertensión arterial, que bruscamente presentó un cuadro de diarrea matutina, acompañada
durante 30 minutos de cefalea frontal invalidante, que cedió con analgesia convencional y repitió varias mañanas sucesivas. Quince días
después, durante tres horas y coincidiendo con la cefalea, presentó una hipoestesia derecha global, acompañada de parestesias y
debilidad en la extremidad inferior derecha. TAC craneal y resonancia magnética nuclear (RMN) vertebromedular que no mostraron
enfermedad, ecocardiográfico fue normal. A las 72 horas, presentó otro episodio de debilidad, también de inicio brusco, con una
duración de 10 horas y que afectó al hemicuerpo izquierdo. Una nueva TAC craneal tampoco mostró enfermedad. El estudio del líquido
cefalorraquídeo incluyendo bioquímica, inmunoglobulinas, anticuerpos antineuronales y bandas oligoclonales resultó normal, salvo una
discreta linfocitosis, así como negativos fueron los cultivos, las serologías y diversas PCR como virus herpes, BK y JC. Se descartó la
enfermedad de Whipple.

PREGUNTA
Cuál es el estudio más adecuado para establecer el diagnostico de vasculitis?

RESPUESTA
a.- Angio-IRM.

CURSO ENARM CMN SIGLO XXI TEL: 36246001 Pharmed Solutions Institute PÁGINA 101
MANUAL DE TRABAJO DEL CURSO ENARM CMN SIGLO XXI
b.- Angiografia.
c.- Biopsia cerebral.
d.- ANA y ANCA.

CASO CLINICO
Paciente de sexo femenino, de 21 años de edad, diagnosticada de AT. Consultó por dolor articular y claudicación del miembro superior
izquierdo de 4 meses de evolución. Al examen físico presentaba ausencia de pulso radial izquierdo y signos de insuficiencia aórtica. Se
realizó una angiografía digital (AD) que mostró hallazgos típicos de AT. Por falta de respuesta al tratamiento con prednisona, se indicó
una angioplastia de arteria carótida primitiva izquierda y tratamiento con ácido acetilsalicílico (AAS) y metotrexato, logrando mejoría
clínica. Pero iniciando con diarrea y dolor abdominal. Se solicitó angiografía por resonancia magnética abdominal que no evidenció
alteraciones; la inmunoglobulina A (IgA) anti-endomisio resultó positiva y biopsia de duodeno.

PREGUNTA
Que resultados histopatológicos espera encontrar.

RESPUESTA
a.- Atrofia difusa de vellosidades.
b.- Infiltrado inflamatorio en la lamina propia.
c.- Datos de isquemia generalizado.
d.- Infiltrado inflamatorio crónico.

CASO CLINICO
Paciente femenina de 22 años de edad con historia de 2 meses de evolución con sinusitis, otitis media bilateral complicada con
mastoiditis, conjuntivitis bilateral, artralgia en hombros, codos, rodillas y tobillos, tos con expectoración sanguinolenta, debilidad
generalizada y cansancio fácil, asociado a fiebre no cuantificada sin predominio de horario y a pérdida de peso aproximadamente de
10% de su peso habitual. Hace más o menos tres semanas presenta cuadro de odinofagia, lumbalgia, disuria, coluria y refiere un
episodio de epistaxis aislada. SV: Temperatura: 38,2 °C, FC: 110 latidos/minuto, FR: 22 ciclos/minuto. Tensión Arterial: 120/90 mmHg.
Radiografía de tórax: se identifica infiltrado de tipo intersticial de predominio derecho parahiliar, en menor grado, este tipo de
infiltrado se visualiza hacia la base pulmonar izquierda. No hay signos de lesiones nodulares ni cavitaciones. TAC: infiltrados nodulares
en ambos campos pulmonares, cavitación en lóbulo superior derecho, tenue patrón de vidrio esmerilado posterobasal derecho y
crecimientos ganglionares precarinales, parahiliares y subcarinales.

PREGUNTA
Considerando el diagnostico del caso, cual de las siguientes aseveraciones sobre su tratamiento no es adecuada?

RESPUESTA
a.- Pulsos de ciclofosfamida 1 g intravenoso (I.V.) mensuales por 6 meses y luego pulsos de ciclofosfamida 1 g I.V. cada 3 meses por un
año.
b.- Pulsos de Metilprednisolona 1 g I.V. cada día por 7 días, luego se administra 1 mg/kg/día por 6 semanas y posterior reducción
gradual.
c.- La combinación de ciclofosfamida y metilprednisolona se utiliza para inducir la remisión del cuadro.
d.- Leuprolide 11,5 mg I.M. cada mensual, para evitar amenorrea como efecto secundario de la ciclofosfamida.

PREGUNTA
Según los criterios diagnósticos de GW según el American College of Rheumatology (ACR), son: Sedimento urinario anormal (cilindros
hemáticos o más de 5 eritrocitos por campo de alto poder). Radiografía de tórax con hallazgos anormales (nódulos, cavidades o
infiltrados). Ulceras orales o secreción nasal; y biopsia con inflamación granulomatosa. Cuantos criterios se requieren para confirmar el
diagnostico?

RESPUESTA
a.- 1.
b.- 2.
c.- 3.
d.- 4

CASO CLINICO
Paciente femenina, de 44 años de edad, con antecedentes patológicos personales de asma bronquial de inicio tardío (35 años), quien
llevaba tratamiento esteroideo regular en los períodos de crisis, y que comenzó 4 años después con entumecimiento y debilidad en
ambos miembros inferiores, por lo que se interpretó como un síndrome polineuropático. A los 3 meses comenzó con ligera mejoría del
cuadro, fue ingresada en varias ocasiones con la misma impresión diagnóstica pero agrego gran debilidad en ambos miembros
inferiores. Al interrogatorio se constató: asma bronquial de inicio tardío precedida de rinitis, fenómeno de Raynaud, diarreas
frecuentes, con antecedentes de gastritis. Al examen físico: cuadriparesia con predominio en miembros inferiores, con cambios tróficos
dístales, dedos en martillo con aumento del arco plantar, disminución del tono muscular, así como abolición de los reflejos
osteotendinosos. Eosinofilia marcada (valores de 4). El estudio de rayos X de tórax mostró infiltrado inflamatorio. El estudio de
conducción nerviosa (ECN) evidenció la presencia de un síndrome polineuropático. Se comenzó tratamiento con corticosteroides,
metilprednisolona y se continuó con prednisona y vitaminoterapia, con lo cual la paciente mostró una mejoría significativa. El ciclo de

CURSO ENARM CMN SIGLO XXI TEL: 36246001 Pharmed Solutions Institute PÁGINA 102
MANUAL DE TRABAJO DEL CURSO ENARM CMN SIGLO XXI
tratamiento se repitió cada 2 meses. Al cabo de 6 meses la paciente se encuentra deambulando sin apoyo y con discreta afección del
ECN.

PREGUNTA
Se diagnostico SCS, considerando la patologia cual de las siguientes manifestaciones no es frecuente observar?

RESPUESTA
a.- Vasculitis de los pequeños vasos.
b.- Granulomas vasculares y/o extravasculares.
c.- Eosinofilia periférica que ocurre en los pacientes asmáticos sin historia de rinitis alérgica o sinusitis.

CASO CLINICO
Paciente femenina, de 45 años de edad, con antecedentes de asma bronquial que comenzó a los 27 años. Por un cuadro clínico, 3
meses antes, caracterizado por lesiones en piel, diagnosticado una vasculitis. Comenzó con tratamiento esteroideo con lo que mejoró
su cuadro de forma relativa. Posteriormente la paciente comenzó a presentar entumecimiento en el miembro superior derecho y
posteriormente en la extremidad inferior contralateral, con ligera impotencia funcional. Después padeció de diarreas, edemas en la
cara y en miembros inferiores y cefalea de característica vascular. Se encontró hipoestesia distal, superficial y profunda, cambios
tróficos, con disminución de reflejos osteotendinosos, además se encontró el fenómeno de Raynaud, hipereosinofilia y aumento del
valor de creatinina (210 ml/L). El ecocardiograma mostró evidencias de pericarditis de pequeña cuantía y el rayos X de tórax evidenció
infiltrado inflamatorio. Se encontró evidencia de mononeuropatía multiplex. La TAC de cráneo evidenció la presencia de 3 pequeñas
lesiones hipodensas con características vasculares. Se comenzó tratamiento con methilprednisolona, no teniendo una respuesta
adecuada, solo con leve mejoría sintomática. La paciente se mantiene actualmente bajo tratamiento inmunosupresor, azatioprina
100/diarios.

PREGUNTA
Los criterios para la clasificación del SCS 6 criterios: el asma, la eosinofilia mayor que 10 % en recuento de leucocitos diferenciales, la
mononeuropatía o polineuropatía, infiltrados pulmonares, anormalidad del seno paranasal y la biopsia de vasos sanguíneos con
eosinofilia intra/extravascular. Cuantos criterios son necesarios para establecer el diagnostico?

RESPUESTA
a.- 2.
b.- 3.
c.- 4.
d.- 5.

CASO CLINICO
Paciente varón de 62 años que acude a consulta externa para una exploración rutinaria. En la exploración oftalmológica la agudeza
visual (AV) fue de 1 en ambos ojos (AO). Con pupilas isocóricas y normorreactivas, sin defectos aferentes, tensión por aplanación de 16
mmHg en AO. En el estudio biomicroscópico anterior destacaba una esclerosis cristaliniana moderada. En la retina del ojo izquierdo (OI)
había dos manchas algodonosas asintomáticas, una de 250 micras y otra puntiforme a lo largo de la arcada temporal inferior. Un
estudio físico completo no detectó patología. Los estudios de laboratorio fueron normales salvo una proteína C reactiva de 4,4 mg/dl,
una velocidad de sedimentación glomerular (VSG) de 56 mm/1.ª hora.

PREGUNTA
Cual es la conducta a seguir mas adecuada para establecer el diagnostico?

RESPUESTA
a.- Biopsia de la arteria temporal.
b.- Biopsia de la arteria carótida.
c.- Biopsia de la occipital.
d.- Biopsia de la oftálmica.

CASO CLINICO
Mujer de 83 años que acude a consulta por disminución bilateral de la agudeza visual. Entre sus antecedentes destacaba cirugía
bilateral de cataratas hace 5 años. La AV era de 0,4 en el ojo derecho (OD) y 0,3 en OI. La tensión por aplanación fue de 18 mmHg en OD
y de 16 mmHg en OI, con pupilas isocóricas y normorreactivas. En la exploración fundoscópica destacaba una esclerosis coroidea y una
mancha algodonosa de 20 micras de diámetro en el OD. La exploración física demostró una dificultad importantísima en la movilidad
con claudicación de las extremidades inferiores, astenia, un leve soplo carotídeo, cefalea en casco durante 8 meses y unas arterias
temporales endurecidas. Durante el estudio (en 4 días) la paciente desarrolló una neuritis óptica anterior en el OD y con velocidad de
sedimentación glomerular de 94 mm/1.ª hora, proteína C de 5,85 mg/dl, 400 Mil/mm3 plaquetas, anemia hipoproliferativa leve y
elevación de la fosfatasa alcalina (134 UI/L).

PREGUNTA
Considerando las sintomatologia oftalmológica cuales la causa mas frecuente?

RESPUESTA

CURSO ENARM CMN SIGLO XXI TEL: 36246001 Pharmed Solutions Institute PÁGINA 103
MANUAL DE TRABAJO DEL CURSO ENARM CMN SIGLO XXI
a.- Diabetes mellitus.
b.- Hipertensión arterial.
c.- Colagenopatías.
d.- Patología tumoral.

CASO CLINICO
Mujer de 27 años que consultó por un cuadro de un mes de evolución de dolor en brazo izquierdo acompañado de parestesias y
paresias homoloterales que aumentaban con la actividad y cedían con el reposo. Antecedentes: 12 años antes: Fiebre Reumática
tratada con penicilina benzatínica durante 7 años, 4 años antes: Embarazo ectópico en trompa de Falopio con ooforectomía izquierda 3
años antes: Episodio de dolor lumbar de comienzo súbito durante una actividad física tratado con corticoides que abandonó por cuenta
propia 2 años antes: Diagnóstico de anemia, tratada con hierro intramuscular durante un año 1 año antes: Disminución de la agudeza
visual y fotofobia durante la actividad física que persiste hasta la actualidad. Episodio de taquicardia y disnea de comienzo súbito. Se
diagnosticó taquicardia sinusal y comenzó tratamiento con atenolol 8 meses antes: Cefalea de inicio súbito, intensa, holocraneana
acompañada de hipertensión arterial en tratamiento con enalapril. Examen Físico: Signos vitales: PA 140/60 mmHg en brazo derecho,
inaudible en brazo izquierdo; FC 96 lpm; FR 18 cpm; Tº 36.1º C. Cabeza y Cuello: pulsos carotídeos asimétricos con disminución franca
del lado izquierdo, frémito carotídeo derecho, soplo carotídeo bilateral. Cardiovascular: soplo sistólico 5/6 en focos aórtico y pulmonar
que irradia a cuello y aumenta con la inspiración. Pulsos radial, femoral, tibial posterior y pedio asimétricos, con disminución en
hemicuerpo izquierdo. Abdomen: hígado palpable a 1cm del reborde costal, Miembros: disminución de la fuerza en hemicuerpo
izquierdo. Tono, trofismo, temperatura conservada. Hematocrito 36%, Hemoglobina 12 g/dL, Glóbulos blancos 9300/mm3, Plaquetas
284000/mm3, Glicemia 112 mg/dL, Urea 37 mg/dL, Creatinina 0.6 mg/dL, Na 134 mEq/L, K 3.3 mEq/L, plasma no ictérico, ASAT 17 UI/L,
ALAT 37 UI/L, Proteínas totales 8.8 g/dL, Albúmina 4.7 g/dL, Tiempo de protrombina 13.3 segundos, KPTT 33 segundos, Velocidad de
eritrosedimentación 70 mm/1º hora.

PREGUNTA
Ante la presencia de un cuadro clínico con manifestaciones sistémicas, anemia, velocidad de eritrosedimentación (VES) elevada y
alteración de la aorta y sus ramas, debe pensarse en aortitis, de las siguientes patologías cual es la menos frecuente?

RESPUESTA
a.- Espondolitis anquilosante.
b.- Arteritis de células gigantes.
c.- Enfermedad de Behcét.
d.- Sindrome de Marfan.

PREGUNTA
Considerando su diagnostico, cual es la indicación mas frecuente para revascularizar?

RESPUESTA
a.- Hipertensión arterial renovascular.
b.- Claudicación severa de los miembros.
c.- Isquemia mesentérica
d.- Infarto agudo de miocardio

CASO CLINICO
Un varón de 22 años, consumidor ocasional de cocaína y fumador de 20 cigarrillos/día, fue remitido a nuestro centro por dolor torácico.
La exploración física y la analítica de control fueron normales. El electrocardiograma mostró bloqueo de rama derecha con ondas q en
V1, V2 y V3. La radiografía de tórax mostró una imagen calcificada, redondeada superpuesta a la silueta cardiaca. En el ecocardiograma
se objetivó un ventrículo izquierdo dilatado y adelgazado, con hipocinesia general más marcada en el septo y la cara anterior, con
disfunción sistólica severa (fracción de eyección del 30%). Se realizó una coronariografía que mostró oclusión completa de la arteria
descendente anterior distal a un aneurisma gigante calcificado y oclusión completa de la coronaria derecha.

PREGUNTA
Cual es el porcentaje de afeccion a las arterias coronarias en esta patologia?

RESPUESTA
a.- 25 %.
b.- 35 %.
c.- 45 %
d.- 55 %.

PREGUNTA
El diagnóstico clásico de la EK se basa en la presencia de los siguientes síntomas, cual no es frecuente?

RESPUESTA
a.- Fiebre.
b.- Alteraciones cutáneas en las extremidades.
c.- Exantema polimórfico.

CURSO ENARM CMN SIGLO XXI TEL: 36246001 Pharmed Solutions Institute PÁGINA 104
MANUAL DE TRABAJO DEL CURSO ENARM CMN SIGLO XXI
d.- Conjuntivitis bilateral exudativa.

CASO CLINICO
Femenino de 14 años de edad, acude a consulta por presentar sangramiento en las encías desde hace varios meses, refiere la madre
que el estomatólogo le indicó el cepillado correcto y buchadas de manzanilla 3 veces al día. Examen físico: Se observa presencia de
caries, encías edematosas, al sondeo bolsas de 4mm en el sector anteroinferior (brecha edente de 3.1-3.2), perdidos por movilidad, no
restituidos por prótesis, sarro supragingival y manchas de origen medicamentoso. Se aprecia pigmentación melánica en la encía
adherida, hay xerostomia, lengua lisa y brillante e higiene deficiente.

PREGUNTA
Cual es la conducta a seguir en relación al caso?

RESPUESTA
a.- Yodopovidona oral.
b.- Pilocarpina oral.
c.- Clorhexidina oral.
d.- Prednisona oral.

CASO CLINICO
Mujer hipertensa de 74 años. Unas semanas antes comienza con astenia, fatigabilidad y disnea progresiva, dolor torácico de
características inespecíficas, hipotensión sintomática y síncope; no refería fiebre. En el momento del ingreso presentaba una presión
arterial sistólica (PAS) de 90 mmHg, un electrocardiograma con taquicardia sinusal, bloqueo de rama derecha más hemibloqueo
anterior izquierdo, con PR normal y cardiomegalia-congestión en la radiografía de tórax. Analítica básica: creatinina 1,4 mg/dl, Na 121
mEq/l, K 5,6 mEq/l, aclaramiento de creatinina 95 ml/min, microalbuminuria 9,4 mg/dl, troponina I 2,4 ng/ml, cLDL 78 mg/dl, GOT-GTP
239 U/l, GGT 136 U/l, FA 154 U/l, LDH 904 U/l. Leucocitos 5,9 3 103/ μ l; hemoglobina 11 g/dl; plaquetas 233 3 103/ μ l; actividad de
protrombina 87%, VSG 133 mm en la primera hora y PCR 13,2 mg/dl. Hemocultivos y serologías varias negativas. La ecocardiografía en
el momento del ingreso muestra un ventrículo izquierdo no dilatado, sin alteraciones segmentarias de la contractilidad y disfunción
sistólica severa, biventricular, con fracción de eyección del ventrículo izquierdo (FEVI) del 15%, sin otros hallazgos.

PREGUNTA
Considerando los hallazgos clínicos mas relevantes cual es el fármaco de elección.

RESPUESTA
a.- Corticosteroides.
b.- Azatioprina
c.- Metotrexato
d.- Ciclofosfamida.

TUBERCULOSIS:
CIENCIAS BASICAS: Es una enfermedad infecciosa que suele afectar a los pulmones y es causada por el complejo Mycobacterium
tuberculosis (M. tuberculosis, M. bovis, M. africanum). La infección por M. tuberculosis suele ser asintomática en personas sanas, dado
que su sistema inmunitario actúa formando una barrera alrededor de la bacteria. SALUD PÚBLICA: Considerada una emergencia
mundial por la OMS, es prioridad casos complicados en los extremos de la edad, multifármacorresistencia y vínculo con el sida y la
diabetes, e inmunocomprometidos (neoplasias, quimioterapias, trasplantes). OMS informa que 1/3 de la población mundial está
infectada por Mycobacterium tuberculosis. Cada año: >10 millones de casos nuevos y 3.5 millones de defunciones por TB. En África y
este de Europa: incidencia es de >300 casos/100,000 hab. por año. La TB que responde al tratamiento estándar tiene una tasa de
curación >95%, en multiresistencia es de 50-80%. Menos de 30% de los inmunocompetentes logran la curación y >del 50% muere
dentro de los primeros 5 años. En multiresistencia la tasa de mortalidad es de 90%. PATOGENIA: La transmisión de bacilos tuberculosos
se produce básicamente por vía aérea (también ingestión de leche de vaca infectada, contacto con personas enfermeas baciliferas o
animales bovinos enfermos). Las personas infectantes eliminan bacilos a partir de aerosoles (tos, expectoración) y la infecciosidad
depende del número de bacilos eliminados y la susceptibilidad del huésped. Las partículas aerosolizadas que contienen bacilos, son
suficientemente pequeñas para eludir la 1ª barrera defensiva (aparato muco-ciliar), para alcanzar los alveolos pulmonares, donde
comienza la multiplicación de los bacilos, son fagocitados por lo macrófagos alveolares, se liberan multiples citocinas proinflamatorias,
TNF (ocasiona fiebre y caquexia), IL-1 y IL-6, además liberan proteasas, urocinasa, implicadas en la destrucción tisular. Secundariamente
los bacilos son transportados por los propios macrófagos a los ganglios regionales donde se produce la respuesta inmunitaria mediada
fundamentalmente por los linfocitos T (inmunidad celular). Período de incubación que oscila entre 6 a 8 semanas. Puede ocurrir que
antes del desarrollo de la respuesta inmunitaria celular se produzca una diseminación vía linfo-hematógena que dé lugar a siembra de
bacilos en diversos tejidos: zonas apicales de pulmón, vértebras, epífisis de huesos largos etc. que condicionen la evolución ulterior a
enfermedad progresiva tras períodos largos de latencia. En la mayoría de los casos de infección tuberculosa, hay una destrucción
rápida de bacilos y no se produce enfermedad, el único indicio residual es la positividad de la PPD. El granuloma se forma por la
interacción del M. tuberculosis, con la respuesta inmune del huésped y liberación tisular de proteasas; al inicio se hallan neutrofilos,
mas tarde necrosis caseosa (típica de granulomas producidos por micobacterias) y finalmente calcificaciones. El bacilo no siempre es
eliminado y permanece inactivo, causando una infección latente. INFECCION PRIMARIA: Afecta pulmón, se adquiere al inhalar bacilo,
se desarrolla después de una exposición inicial. Se necesitan de 50-200 microorganismos para producir la infección. De las personas
expuestas al bacilo 30% se infecta y 5% desarrolla enfermedad. Los bacilos inhalados causan alveolitis, adenopatías y linfagitis, lo que se
conoce como complejo primario de Ghon. Clínica; hemoptisis, dolor pleurítico, disnea, fiebre, diaforesis nocturna y pérdida de peso.

CURSO ENARM CMN SIGLO XXI TEL: 36246001 Pharmed Solutions Institute PÁGINA 105
MANUAL DE TRABAJO DEL CURSO ENARM CMN SIGLO XXI
Complicación: TB endobronquial. REACTIVACION: Es progresiva se desarrolla de un foco previo de TB, el cual puede producir TB
pulmonar o extrapulmonar. Más común en inmunocomprometidos (IRC, DM, esteroides, desnutrición, deficiencia de vitamina D.
Clínica; tos, pérdida de peso, fatiga, fiebre y diaforesis nocturna. Complicaciones agudas; hemoptisis y neumotórax. Cavitaciones en
(20-45%), manifestación radiológica más común consolidación Los aspergillomas se desarrollan dentro de la cavitación (20%). Hasta en
5% de pacientes reactivación la manifestación principal es el tuberculoma (centro: material caseoso y periferia histiocitos epiteliales y
cel. Gigantes multinucleadas). DEFINICIONES (Estándares para la atención de la TB en México): Caso de tuberculosis: Persona en quien
se establece el diagnostico de tuberculosis pulmonar o extrapulmonar y se clasifica por bacteriología o estudio histopatológico en caso
confirmado o no confirmado. Caso de tuberculosis confirmado: Toda persona con cuadro clínico compatible con tuberculosis pulmonar
o extrapulmonar que cumpla además cualesquiera de los siguientes criterios: 1. Aislamiento de Mycobacterium tuberculosis por
cultivo. 2. Resultado positivo en la baciloscopia. 3. Deteccion de genes de micobacterias por métodos de biología molecular (PCR o
ampliación de RNA). Caso de tuberculosis no confirmado: Toda persona con cuadro clínico compatible con tuberculosis pulmonar o
extrapulmonar sin confirmación por baciloscopía, cultivo o estudios de biología molecular pero presenta uno o mas criterios como: 1.
Cuantificación de adenosin desaminasa (ADA): los niveles de ADA parab TB peritoneal y pleural, están >70U/ml; para SNC >7U/l. 2.
Antecedente epidemiológico de convivencia con un caso bacilifero. 3. Reactor a la PPD, con o sin antecedente de BCG. 4. Respuesta
favorable al tratamiento antituberculoso. 5. Estudio histopatológico compatible con TB. Y alguno de los siguientes estudios de gabinete:
A. TB pulmonar; las radiografías de tórax PA y lateral pueden mostrar imagen de síndrome del lóbulo medio, derrame pleural,
ensanchamiento mediastinal o patrón miliar. B. TB ganglionar; ultrasonido con imágenes de material calcificado y liquido. C. TB del SNC;
la TAC de cráneo puede mostrar datos de aracnoiditis e hidrocefalia; la radiografía de cráneo, datos de hipertensión endocraneana. D.
TB osea o enfermedad de Pott: las rx., de columna vertebral AP y lateral muestran destrucción de las vertebras dorsolumbares y
rotoxifoescoliosis. E. TB geniturinaria: urografía excretora muestra imágenes tortuosas debido a rigidez de uréteres. F. TB abdominal;
USG o TAC muestran ascitis e imágenes compatibles con tabicaciones. La laparoscopia muestra lesiones granulomatosas ascitis y
fibrina. DIAGNOSTICO: Búsqueda entre contactos de un caso de TB y en grupos o poblaciones de alto riesgo. La baciloscopía se debe
realizar en A) En cualquier muestra clínica excepto orina. B) En todos los casos probables, entre los contactos de un caso, en grupos o
poblaciones de alto riesgo, pacientes dados de alta que acudan con tos productiva, en 3 muestras sucesivas de esputo. C)
Independientemente de la edad, en en quienes clínica y radiológicamente, se sospecha TB, si la primera serie de 3 hubiera sido
negativa. D) En el control del tratamiento antituberculoso, con una muestra cada mes, la ultima al terminar el tratamiento E) En caso de
sospecha de TB extrapulmonar, F) Sospecha de TB en niños. G) Pacientes en tx., estrictamente supervisado, en quienes al cuarto mes,
persiste baciloscopia positiva. H) Para confirmar fracaso de tratamiento. Prueba de la tuberculina (PPD), con derivado de proteína
puirificado, tiene un papellimitado en el diagnostico de TB activa. Reactor al PPD, persona que a las 72hrs presenta induración
intradérmica de 10mm o mas en el sitio de la
aplicación. En menores de 5 años con o sin BCG,
recién nacidos, niños desnutridos y personas
inmunodeprimidas, la induración de 5mm o mas.
Clínico; TB pulmonar activa son tos, a veces con
esputo que puede ser sanguinolento, dolor
torácico, debilidad, pérdida de peso, fiebre y
sudoración nocturna. RADIOLOGICO:
Linfadenopatía; alteración más común niños (90-
95%), adultos (40%), más frecuentes unilaterales,
paratraqueal derecho e hiliar y su frecuencia
disminuye con edad. La combinación de ganglios
hiliares calcificados y focos de Ghon, se conoce
como Complejo de Ranke (sugiere TB previa,
histoplasmosis). En infección primaria: se puede
observar focos de consolidación generalmente en
lóbulo medio o inferior, derrame pleural (60%),
más unilateral, se presenta 3-7 meses después de
la exposición inicial. En reactivación: afección del
segmento apical y posterior de lóbulos superiores
(85%), la cavitación secundaria a necrosis caseosa,
se observa como nivel hidroaereo; se pueden ver
atelectasias lobares. TAC: Los ganglios muestran
hipodensidad central (necrosis caseosa) y
reforzamiento periférica, que representa el anillo
vascular de tejido granulomatosos inflamatorio que
indica enfermedad activa. En pacientes de alto riesgo, se pueden formar cavitaciones, diseminación hematogena y tuberculosis miliar.
TUBERCULOSIS MILIAR: Se da por una infección diseminada por vía hematogena. Afecta de 1-7%, frecuente en ancianos, lactantes e
inmunocomprometidos. Suele manifestase con fiebre, con frecuencia existen granulomas pulmonares, afeccion del SNC, en raros casos
sx. de dificultad respiratoria aguda con mortalidad de 90%. En casos crónicos caquexia, lesiones dérmicas maculares o papulares y
tuberculos coroides (2-3mm), estos últimos patognomónicos de tuberculosis miliar. RX: Múltiples imágenes nodulares <5mm
distribuidos en ambos pulmones difusas bilaterales, engrosamiento de septos interlobulillares. TB Y VIH: La inmunodepresión favorece
manifestaciones atípicas y extrapulmonares o inusuales de TB, al iniciar el tratamiento antirretroviral hay mayor riesgo de desarrollar
TB. Antes de indicar tratamiento, se debe realizar prueba de tuberculina. Las reactivaciones en estos grupos son 20 veces mayor. La TB
aumenta la replicación viral y la progresión de la enfermedad VIH. La TB se presenta independiente de la cuenta de CD4. La TB miliar y
del SNC son más frecuentes con cuentas de CD4 <200 cel/mm3.comun. Todos los pacientes VIH positivos, sin datos clínicos de TB,
deben recibir quimioprofilaxis con isoniacida.

CURSO ENARM CMN SIGLO XXI TEL: 36246001 Pharmed Solutions Institute PÁGINA 106
MANUAL DE TRABAJO DEL CURSO ENARM CMN SIGLO XXI
TRATAMIENTO: Debe de ser estrictamente CUADRO 2. Tratamiento primario acortado
supervisado. Quimioprofilaxis, primario acortado y FASE INTENSIVA Diario, de lunes a sábado, hasta completar 60 dosis o 2 meses.
Administración en 1 toma
retratamiento. Se considera TB multiresistente, FARMACOS: Dosis:
cuando no es susceptible a isoniacida, ni rifampicina Rifampicina (R) 600mgs
administradas simultáneamente. Quimioprofilaxis: Por Isoniacida (H) 30mgs
6 meses a los contactos menores a 5 años, con o sin Pirazinamida (Z) 1,500-2000 mg
Etambutol (E) 1,200mgs
antecedente de BCG. Por 6 meses a contactos de 5-14 FASE DE SOSTEN Intermitente, 3 veces por semana, lunes, miércoles y viernes.
años de edad, no vacunados BCG, en quienes se haya Hasta completar 45 dosis o 4 meses. Administrar en una toma
descartado TB. Por 12 meses a contactos de 15 años o Fármacos: Dosis:
más con inmunosupresión (previo descarte de TB Isoniacida (H) 800mgs
Rifampicina (R) 600mgs
pulmonar o extrapulmonar). El fármaco a usar es la
isoniacida a dosis de 10mg/kg, sin exceder de 300mgs, en una toma diaria VO, estrictamente supervisada. El tratamiento acortado
estrictamente supervisado (TAES), se instituye a todo caso nuevo y al que lo reanuda posterior al primer abandono. TAES por 25
semanas, hasta completar 105 dosis. En 2 etapas: fase intensiva; 60 dosis, de lunes a sábado con HRZE (ver cuadro 1, medicamentos 1ra
línea). Fase de sosten; 45 dosis ( ver cuadro 2, intermitente 3 veces a la semana con HR), con fármacos en combinación fija y etambutol
separado
PREVENCION: Vacunación; BCG, de bacilos mycobacterium bovis vivos atenuados (bacilo de Calmette y Guerin) inmunidad activa
contra la TB, dosis 0.1ml (contiene como mínimo 200 000UFC), intradérmica, brazo der., se aplica a todos los recién nacidos y hasta los
14 años de edad cuando se considera necesario, disminuye incidencia de TB en SNC. Contraindicada, en recién nacidos con <2Kg,
personas inmunodeprimidas (excepto en infección por VIH en estado asintomático), fiebre >38.5
NOTAS: La TB ganglionar es la forma más de TB extrapulmonar. La TB vertebral (torácica) o Enf. de Pott es la manifestación más
frecuente de TB ósea (50%), mas en varones.

CASO CLINIC
Se trata de paciente masculino de 46 años de edad, originario de la región selvática de Chiapas, campesino, analfabeta el cual acude a
consulta debido a que se encuentra fatigado, lo cual le impide trabajar, agrega perdida de peso y fiebre frecuente, además refiere que
tienes tos continua con esputo ocacionalmente con trazas de sangre, a la exploración física, no se observa cicatriz de BCG, refiere
consumo de mezcal diario y tabaquismo 3 a 5 cigarrillos diarios, el índice de masa corporal actual es de 20.5, signos vitales dentro de
parámetros normales, a la auscultación torácica se apresian ruidos respiratorios anforicos posteriores de predominio superior derecho
con estertores aislados en la misma región, no se observa cianosis ni hipocratismo, resto sin datos por agregar.

PREGUNTA
Cual es la conducta a seguir mas adecuada.

RESPUESTA
a.- Internamiento en aislamiento ambiental hasta 3 expectoraciones negativas.
b.- Internamiento sin aislamiento.
c.- Realizar biopsia e interconsulta a oncología.
d.- Realizar prueba PPD.

TUBERCULOSIS
Se ingresa a paciente masculino de 45 años de edad el cual cuenta con antecedente de ser portador de VIH positivo con cifras de CD4
de 85, actualmente suspendió el tratamiento antirretroviral, el motivo del ingreso es debido a dolor en la región lumbar intenso el cual
mostro datos de ostemielitis, el paciente cuenta con el antecedente de presencia de tos productiva con esputo amarillento y
sanguinolento, perdida de peso importante, por lo que previamente se realizao baciloscopia siendo positiva para tuberculosis, apoyada
por los parámetro radiográficos.

PREGUNTA
Cual es el tratamiento inicial mas adecuado.

RESPUESTA
a.- Isoniazida, rifampicina, etambutol y pirazinamida.
b.- Isoniazida, rifampicina, etambutol y pirazinamida mas terapia antirretroviral.
c.- Isoniazida, rifampicina, etambutol, pirazinamida y estreptomicina.
d.- Isoniazida, rifampicina y etambutol.

CASO CLINICO
Paciente de 56 años, sexo femenino, con antecedente de dermatomiositis, en tratamiento con hidroxicloroquina 200 mg, metotrexato
10 mg y prednisona 30 mg diarios. Fue hospitalizada por cuadro de 10 días de evolución, con fiebre, compromiso del estado general y
pérdida de peso, encontrándose crepitaciones bilaterales al examen pulmonar. La TAC de tórax mostró un infiltrado nodular bilateral,
extenso, con cavitaciones biapicales. La baciloscopia obtenida por lavado bronquio-alveolar (LBA) resultó positiva y la reacción de
polimerasa en cadena (RPC) positiva para Mycobacterium tuberculosis. Además úlceras en los pulgares aparecidas cuatro semanas
previas al ingreso. Al examen físico se observó una úlcera de bordes bien delimitados con fondo sucio y bordes necróticos, no dolorosa,
ubicada en el pulpejo del dedo pulgar izquierdo. S. aureus y Candidaparapsilosis. La tinción de Ziehl Neelsen en tejido fue positiva.

PREGUNTA

CURSO ENARM CMN SIGLO XXI TEL: 36246001 Pharmed Solutions Institute PÁGINA 107
MANUAL DE TRABAJO DEL CURSO ENARM CMN SIGLO XXI
Cual es el origen mas probable que cause las ulceras en pulgares.

RESPUESTA
a.- Farmacologica.
b.- Bacteriana.
c.- Autoinmune.
d.- Mixta.

CASO CLINICO
Se trata de paciente femenino de 70 años de edad la cual acude a consulta debido a que presenta desde hace varios meses, tos no
productiva, fatiga, pérdida de peso, la paciente cuenta con antecedente de tuberculosis hace mas de 20 años la cual fue tratada, se
logra obtener esputo para cultivo asi como sangre para cultivo, los reportes de laboratorio informan presencia de bacilos
acidorresistentes que indican complejo Mycobacterium avium, la radiografia de torax reporta bronquiectasias y nódulos pequeños
esparcidos a lo largo del parénquima pulmonar. La paciente no cuenta con otros antecedentes de importancia para el cuadro clínico.

PREGUNTA
Cuál es el esquema de tratamiento más adecuado.

RESPUESTA
a.- Claritromicina y etambutol.
b.- Claritromicina y rifampicina.
c.- Levofloxacino y rifampicina.
d.- Prizinamida, isonizida, rifampicina y etambutol.

PALUDISMO
CIENCIAS BÁSICAS: Enfermedad parasitaria febril aguda causada por protozoarios del genero plasmodium que se trasmite a los
humanos por la picadura de las hembras infectadas del los mosquitos del genero anopheles. Las especies del plasmodium causantes
son ovale, malaria, vivax y falciparum; estas dos últimas la de mayor distribución en el mundo, el más mortal P. falciparum. Los
mosquitos se crían en agua dulce de poca profundidad y tienen preferencia por los humanos sobre los animales, su transmisión
depende de condiciones climáticas, como el régimen de lluvias, la temperatura y humedad. Enfermedad endémica en poblaciones con
clima tropical. SALUD PUBLICA: OMS; Aparecen aproximadamente de 300-500 millones de casos nuevos cada año, de los cuales mas
de un millón resultan fatales. El 90% de las muertes atribuibles al paludismo ocurre en África en niños menores de 5 años. En México el
principal agente etimológico es P. vivax, algunos casos aislados por falciparum, en estados de la frontera sur. La transmisión en México
se concentra en costas del Pacifico, Chiapas, Oaxaca, Chihuahua, Baja California y Yucatán. La tasa de mortalidad por paludismo se ha
reducido >25% desde el año 2000 a nivel mundial. VIH/SIDA + paludismo=altos índices de morbimortalidad en zonas de alta
prevalencia. PATOGENIA: Inicia con el vector (hembra anopheles) infectada, transfusiones, uso compartido de agujas hipodérmicas
contaminadas. El ciclo de vida de plasmodium se divide en asexual (2 etapas) que se realiza en el humano y sexual en el mosquito. Ciclo
asexual, etapa hepática: tras la picadura del mosquito este inocula el esporozoito existente en su saliva en la sangre del huésped, a
través del torrente sanguíneo llegan a los hepatocitos, aquí se multiplican posteriormente se rompe el hepatocito, apareciendo un
nuevo estadio el merozoito, vuelven al torrente sanguíneo donde penetran al eritrocito. Inicia etapa eritrocitica: comienza a
alimentarse de la hemoglobina, apareciendo entonces el trofozoito, nuevamente por esquizogenesis se multiplica en el interior de los
eritrocitos, a los cuales rompe liberando nuevos merozoitos y así continua el ciclo, pero algunos se convierten en gametocitos
masculinos y femeninos. Ciclo sexual: se da en la hembra anopheles, la cual adquiere el plasmodium mediante la picadura al humano
infectado con gametocitos en sangre. Estos gametocitos forman un cigoto en el intestino medio del insecto, el cigoto madura hasta
formarse un ovocineto, que migra a la glándula salival del insecto. Al destruirse los eritrocitos hay liberación de sustancias del parasito
y de hematíes a la circulación sanguínea. Los eritrocitos parasitados su membrana se vuelve rígida y se forman protuberancias que
favorecen su adherencia al endotelio vascular en capilares de órganos vitales, fenómeno conocido como "citoadherencia", sobre todo
en cerebro, la adherencia produce enlentecimiento del flujo sanguíneo, aumento local de citocinas, glicolisis anaerobia y acidosis
láctica. DIAGNOSTICO: Clínica: Los síntomas aparecen a los 7 días (10-15 días), de la picadura del mosquito, lo primero es fiebre,
acompañada de cefalea, escalofríos, debilidad, insomnio, artralgias, mialgias y vomito. Toda persona sospechosa de paludismo con
cuadro escalofríos (dura de 15-30 min, pulso débil y rápido, piel algo cianótica), fiebre (inicio brusco puede llegar hasta 41.5 grados,
puede delirios, convulsiones, taquicardia, hipotensión) y sudoración, que resida o provenga de áreas endémicas se le deberán realizar
examen microscópico de gota gruesa de sangre y a los contactos de los casos. La enfermedad tiende hacia la cronicidad, en donde
existen periodos de recaídas y latencia por aumento de la parasitemia. Todos los niños de zonas endémicas con enfermedad grave
suelen presentar anemia grave, esplenomegalia, sufrimiento respiratorio relacionado con la acidosis metabólica o paludismo cerebral.
En adulto es común afección multiorganica. Laboratorio: Realizar frotis de sangre, finas y de gota gruesa, identificar parásitos y
cuantificar los, el porcentaje de eritrocitos con parásitos dará la medida de la gravedad del la enfermedad. Técnicas
inmunocromatograficas, detectan el Ag del parasito en sangre. Se puede usar PCR para detención de estos. Pacientes que sufren
complicaciones o enfermedad severa sobre todo por P. falciparum, pueden presentar hiperparasitemia >100 000 parásitos/microl de
sangre con un >5-10% de eritrocitos parasitados, hemoglobina debajo de 7g/dl, ictericia. COMPLICACIONES: Malaria cerebral,
insuficiencia renal, fiebre biliosa hemoglobinurica, anemia severa, edema pulmonar, daño hepático, hemorragia, coagulopatia.
TRATAMIENTO: Actualmente se dispone de la combinación de artesunato (4mg/kg) y mefloquina en una sola tableta para falciparum,
no complicado. Para P. vivax, el tratamiento de elección para cura radical es primaquina (0.25-0.5 mg/kg), es esquizonticida mas
cloroquina (25mg/kg), son tabletas de 150mgs, por 3 días. La OMS considera que el fármaco de elección para P. vivax es cloroquina, las
combinaciones de artesunato, se recomiendan en casos de P. vivax resistente a cloroquina. La quinina, es un ezquizonticida hemático
muy eficaz, gametocida eficaz contra P. vivax y P. malarie, inactiva frente a P. falciparum. Artemisina por via parenteral acción rápida

CURSO ENARM CMN SIGLO XXI TEL: 36246001 Pharmed Solutions Institute PÁGINA 108
MANUAL DE TRABAJO DEL CURSO ENARM CMN SIGLO XXI
empleado para P. falciparum resistente a cloroquina, sus derivados son artesunato y artemer ambos en presentación via oral y
parenteral. La ONU recomienda no emplear artemisina en monoterpia, porque acelera la resistencia, debe tratar e agregarse siempre
algún otro antipalúdico. Si no se trata en las primeras 24 hrs el paludismo por P. falciparum, puede agravarse llevando a la muerte. En
caso de P. vivax y ovale, pueden producirse recidivas clínicas semanas o meses después de la infección inicial, aunque el paciente haya
abandonado la zona palúdica. Estos nuevos episodios se deben a formas hepáticas "durmientes" del parasito (inexistentes en
falciparum, malariae) y para lograra curación completa es obligatorio tratamiento dirigido a esas formas hepáticas. PREVENCIÓN: La
lucha antivectorial es el medio principal de reducir la transmisión del paludismo. Algunas medidas son: mosquitos tratados con
insecticidas (peritroides), fumigación de interiores con insecticidas, vigilancia continua y eliminación del mismo.

CASO CLINICO
Masculino de 35 años de edad residente del estado de mexico, inicia padecimiento 9 dias posteriores a su regreso de la ribera del rio
Niger en Malí donde permaneció 30 dias, cursando con un cuadro clínico caracterizado por fiebre de 40 C, de predominio vespertino y
nocturno terciada que se controlaba con paracetamol 500 mg cada 8 hrs. Al cuadro clínico se agrego estreñimiento, disfagia, tos no
productiva, cefalea 8/10 holocraneana y perdida ponderal de 4 kg en una semana. EF. Temp.39.2 C FC 127, FR 24, TA 122/72 mmHg,
somnoliento, orientado en tres esferas, mucosas secas, ruidos cardiacos aumentados en frecuencia sin agregados, campos pulmonares
bien ventilados, abdomen distendido, doloroso a la palpación media y profunda en ambos hipocondrios, timpánico a la percusión.

PREGUNTA
Cual es la conducta a seguir en el caso?

RESPUESTA
a.- Artesunato/mefloquina.
b.- Cloroquina.
c.- Cloroquina/mefloquina.
d.- Mefloquina.

CASO CLINICO
Ingresa paciente masculino de 48 años de edad, ingeniero que acaba de regresar de convención en medio oriente, presenta fiebre,
diaforesis, ataque al estado generalizado, fatiga y adinamia, así como obnubilación, a la exploración se observa hepato y
esplegnomegalia leve a moderada, es ingresado para realizar estudios encontrando paludismo por plasmodium falciparum, parasitemia
de 7 %, hematocrito de 21 %, bilirrubina de 7.8 mg/100 ml, creatinina 2.6 mg/100 ml. Se ingresa posteriormente a cuidados intensivos,
se realiza asistencia ventilatoria y se indica neuroproteccion debido a la gravedad del cuadro.

PREGUNTA
Cual es el mejor esquema de tratamiento para este caso:

RESPUESTA
a.- Quinina intravenosa.
b.- Quinidina Intravenosa.
c.- Artesunato Intravenoso.
d.- Cloroquina.

DENGUE:
CIENCIAS BASICAS: Enfermedad viral de carácter endemico-epidemico. Producido por un arbovirus (flaviviridae). Transmitido por Aedes
aegypti (vector), compuesto de ARN. El cual tiene 4serotipos (DE 1,2,3,4), el 2 es el serio tipo más peligroso. Cada serio tipo proporciona
una inmunidad específica para toda la vida, así como inmunidad cruzada a corto plazo. Dos clases de dengue el clásico y el hemorrágico.
SALUD PÚBLICA: México ocupa el segundo lugar en casos y defunciones de América, después de Brasil, mayormente entre 10-19 Años,
ligeramente más en sexo femenino.OMS; Entre 50-100 millones por año en el mundo, 500mil hospitalizados. 50mil muertes en más de
100 países y 2 mil millones de personas en riesgo. Mayor incidencia en épocas de lluvia. En México hay condiciones que propician la
presencia de mosquitos, temperatura lluvias constantes, condiciones inadecuadas de saneamiento e higiene. PATOGENIA: EL virus
ingresa al organismo por la picadura del artrópodo se replica en los macrófagos y monocitos, lo que produce supresión de la medula
ósea. El periodo de incubación es de 5-7 días. Se desarrolla una respuesta inmune que acaba en el periodo de viremía e induce
producción duradera. Existen 3 etapas: 1. Febril; se da entre el 3-6to días, es variable, presencia de virus en sangre (viremía). En esta
etapa no es posible reconocer si el paciente va a evolucionar a la curación espontánea o si es el comienzo de un dengue grave. Al
disminuir la fiebre, el dolor abdominal se hace intenso y mantenido, se puede observar derrame plural o ascitis. 2. Crítico; momento de
mayor frecuencia de instalación de choque. Hay extravasación de plasma (vasodilatación), su manifestación más grave el choque,
grandes hemorragias digestivas, alteraciones hepáticas y quizá de otros órganos, ascitis o derrame pleural derecho o bilateral,
aumento de hematocrito (la máxima elevación de este coincide con el choque) y disminución de plaquetas. 3. Recuperación: evidente
mejoría del paciente a veces hay sobrecarga de liquido, así como coinfeccion bacteriana. Esta etapa es la de mayor riesgo de aparición
de complicaciones, hay que vigilar y controlar al paciente 48hrs posteriores al cese de la fiebre, los signos de alarma son: dolor
abdominal intenso y continuo, vómitos frecuentes, somnolencia o irritabilidad, derrame seroso (en peritoneo, pleura o pericardio),
sangrado de mucosa, hepatomegalia (>2cm), aumento rápido de hematocrito y disminución rápida de plaquetas. DIAGNOSTICO:
Dengue clásico: Inicio repentino, fiebre (de menos de 7 días) bradicardia, pulso lento, ex antena máculas punto forme, rinitis, tos, ardor
faríngeo, adenopatías, fatiga, cambios en el sentido del gusto, anorexia, cefaleas, mialgias, artralgias, dolor óseo y retroorbitario,
congestión conjuntival, edema palpebral, petequias, equimosis, miocarditis. Dengue hemorrágico: Mas grave, incremento en
permeabilidad vascular. El choque depende de múltiples factores como la presencia de anticuerpos no neutralizantes. Dengue grave;

CURSO ENARM CMN SIGLO XXI TEL: 36246001 Pharmed Solutions Institute PÁGINA 109
MANUAL DE TRABAJO DEL CURSO ENARM CMN SIGLO XXI
shock hipovolemico por fuga de plasma, diestres respiratorio por acumulación de líquidos, sangrado grave, daño orgánico importante.
BH; leucopenia y trombocitopenia, a veces transaminasas elevadas. ELISA, para determinación de anticuerpo IgM e IgG antidengue, se
puede tomar prueba pareada una en fase aguda y otra en convalecencia. PCR o ELISA se usan para detección de antígenos virales.
Anticuerpos IgM específicos para virus de dengue: Si es negativo se descarta el diagnostico de dengue; si es positivo es un caso
probable de dengue, se requiere una segunda muestra para confirmar mediante prueba de neutralización; negativo=se descarta caso
de dengue, constante=caso de dengue anterior, negativo=se confirma dengue. CASO PROBABLE DE FIEBRE HEMORRÁGICA POR
DENGUE: Toda persona que además de un cuadro de probable fiebre por dengue desarrolle fiebre persistente y uno o más de los
siguientes: A) Datos de fuga de plasma (ascitis, derrame pleural, edema, hipoalbuminemia). B) Datos de fragilidad capilar (petequias,
equimosis, hematomas). C) Hemorragias (gingivorragia, hematemesis, metrorragia). D) Tombocitopenia <100 000 plaquetas/ml3 o
hemoconcentración. CLASIFICACIÓN: Del dengue hemorrágico: Grado I; fiebre y síntomas constitucionales no específicos, prueba del
torniquete positiva (única manifestación hemorrágica). Grado II; grado I más petequias, epistaxis, hematemesis, melena. Grado III;
insuficiencia circulatoria (taquicardia, hipotensión, disminución de la presión diferencial de pulso). Grado IV; choque, pulso y tensión
arterial no detectable. TRATAMIENTO: No especifico, solo de sostén. El dengue sin signos de alarma, ni comorbilidades, manejo
ambulatorio (reposo, aislamiento de mosquitos, líquidos, paracetamol, no aspirina, AINES, ni corticoides, evaluar mejoría). Dengue con
comorbilidades, estricto ambulatorio o internación en sala general (hidratación vía oral o IV con cristaloides a 2-3 ml/kg, mas medidas
anteriores, buscar signos de alarma hasta 48hrs de cese de fiebre, hemograma diario). Dengue con signos de alarma, sin criterios para
dengue grave, (obtener hematocrito antes de expandir al paciente, fisiológica o Ringer 10ml/kg/hr, en una hora, repetir hto después de
cada carga y evaluar signos de alarma, mejoría reducir goteo, si empeora, aumenta hto y caen plaquetas, manejarlo como dengue
grave). Pacientes con signos de alarma y criterios para dengue grave (tomar hto, iniciar SF o Ringer lactato a 20ml/kg en 15-30 min,
evaluar si hay mejoría disminuir a 10ml/kg, si mejora ir disminuyendo líquidos, de no mejorar continuar con coloides, no mejoría
considerar uso de drogas vasoactivas, si paciente mejora y hto baja, indica sangrado y requiere transfundir, tx. De hemorragia según
criterio) Soluciones intravenosas (para reponer pérdidas producidas por extravasación de plasma, a veces se agrava por perdidas del
exterior; sudoración, vomito, diarrea) y requerimientos transfucionales. PREVENCIÓN: No se dispone de vacunas, ni antivirales
específicos. La única forma para controlarlo, es mediante el control del mosquito del dengue y evitar su picadura. Más de 50% de
criaderos de mosquitos se sitúa, en tiraderos de llantas y panteones.

CASO CLINICO
Femenino de 18 años de edad originaria de Tepic, Nayarit camarera, ingreso por hipotensión e hipotermia, refiere que 48 hrs previas al
ingreso inicio con cuadro de cansancio, fiabre, dolor osteomuscular generalizado, dolor retro-ocular y mal estado general, Hg 14.6 Htc
44.7, plaquetas de 51,000/mm y leucocitos 15,200/mm se enviaron muestras para serología, dos horas después la Hb 13.7, Htc 43%,
plaquetas de 31,000 a las 5 horas la tensión arterial era de 80/50, FC 100, FR 24, FC 100, temperatura de 35 C.

RESPUESTA
a.- Cristaloides y aminas.
b.- Solucion coloides, aminas vasoactivas.
c.- Aminas vasoactivas, hemoderivados.
d.- Cristaloides, aminas y hemoderivados.

CASO CLINICO
Masculino de 38 años de edad el cual inicio 5 dias previo a su ingreso con debilidad, hiporexia, fiebre, nausea y vomito, el paciente
presentaba dolor torácico, TA 40/30 mmHg, FC 120, FC 24, Temp 35. La piel marmórea, mucosas secas, Rx de toras con datos de
derrame pleural, torax hipoventilados, dolor epigástrico, Hb 12.8, Htc 51 %, plaquetas 79,000, leucocitos 10,500.

PREGUNTA
Que criterios de gravedad ó de dengue grave presenta?

RESPUESTA
a.- 1.
b.- 2.
c.- 3.
d.- 4.

CASO CLINICO
Se trata de una mujer de raza negra, de 21 años de edad, que presentó un cuadro febril agudo de 39-40 ºC, seguido en los días
sucesivos de una erupción cutánea pruriginosa generalizada. Tras buscar asistencia en un centro pediátrico de su región, se diagnosticó
clínica y serológicamente de dengue clásico y se trató sintomáticamente con paracetamol. Una semana después, una vez que los
síntomas de la fase aguda se encontraban en una fase casi completa de regresión, comenzó a sentir dificultad para articular el lenguaje
y para deambular, disfagia, debilidad muscular progresiva en los cuatro miembros y disminución del nivel de conciencia.

PREGUNTA
Cual es la conducta diagnostica mas apropiada para identificar la causa de las complicaciones.

RESPUESTA
a.- IRM.
b.- TAC.
c.- Pruebas de funcionamiento hepático.

CURSO ENARM CMN SIGLO XXI TEL: 36246001 Pharmed Solutions Institute PÁGINA 110
MANUAL DE TRABAJO DEL CURSO ENARM CMN SIGLO XXI
d.- Biometia hemática.

CASO CLINICO
Se trata de una mujer de 50 años fumadora ocasional. Entre sus antecedentes personales de interés, destacaba la realización de un
viaje turístico de nueve días de duración a la Rivera Maya, quince días antes del inicio de los síntomas. Durante el viaje la paciente,
sufrió numerosas picaduras de mosquitos. En las 48 horas previas a la consulta, la paciente se autotrató con ácido acetilsalicílico (AAS)
como antipirético y como tratamiento sintomático de la cefalea. En la exploración física la paciente estaba consciente, orientada y
febricular (37,6ºC), TA=110/70. Existía rubor facial e inyección conjuntival bilateral así como la presencia de un exantema hemorrágico,
simétrico, máculo-papular en tronco y extremidades, especialmente en las inferiores, donde era confluente, y que no se modificaba a la
vitropresión

PREGUNTA
Cual es la conducta mas adecuada para establecer el diagnostico.

RESPUESTA
a.- Demostración o prueba de NS1+.
b.- Elevación cuatro veces o más de títulos de IgG o IgM contra algún antígeno de virus de dengue en muestras pareadas de suero.
c.- Presencia de antígeno circulante en sangre del virus de dengue determinada mediante ELISA, o de antígeno en hígado por
inmunohistoquímica (inmunoperoxidasa) o inmunofluorescencia.
d.- Detección de secuencias genómicas de virus de dengue mediante prueba de PCR.

ENFERMEDAD DE CHAGAS:
CIENCIAS BÁSICAS: Chagas o trypanosomosis americana, es una infección sistémica causada por el protozoario flagelado Trypanozoma
cruzi. Su distribución en las zonas más marginadas del continente americano y su curso crónico, degenerativo y muerte repentina,
hacen de esta enfermedad una gran carga para la economía y la salud. Es una zoonosis en la que participan un gran número de
reservoríos vertebrados (perros, ratas, gatos, zarigueyas) y transmisores triatominos (chinche). SALUD PUBLICA: Elevada prevalencia e
incurabilidad. Se estima que en la región de las Américas, se presenta en 21 países, afecta a unos 7-8 millones de personas, y se
encuentra en riesgo de adquirir la infección aprox. 7-8 millones de personas, con 56,000 nuevos casos anuales y 12,000 muertes por
año. En México es endémica en el estado de Veracruz. PATOGENIA: La infección se transmite principalmente por la "chinche besucona"
otros modos de transmisión son; transfucional, congénito, trasplante de órganos y oral (alimentos contaminados con heces del
artrópodo). La chinche durante la picadura defeca en la piel del hospedero, después de ingerir los parásitos (en su forma de
tripomastigoto útil para dx.) en la sangre de un ser humano, estos se transforman en tripomastigotes metaciclicos que son la forma
infectante, estos son excretados en las heces del parasito, que al ponerse en contacto con la conjuntiva o laceración en la piel, con el
rascado se facilita la entrada del patógeno al torrente circulatorio. Las primeras células invadidas por los tripomastigotes son los
histiocitos, donde se transforman en amastigotos (intracelulares, replicativo) y desarrollan varios ciclos de fisión binaria. Casi cualquier
célula puede ser invadida, a excepción de las neuronas. La infección persiste en el cuerpo humano de por vida. DIAGNOSTICO: Espectro
clínico en 3 fases: Fase aguda (70% se encuentran en esta fase); la incubacion de 14 dias y dura de 2-4 meses, asintomática, se
caracteriza por alta parasitemía e invasión tisular multiparenquimatosa, puede presentarse el llamado "chagoma de inoculación",
nódulo subcutáneo con adenitis regional en el sitio de la picadura, cuando el sitio de entrada es la conjuntiva se conoce como signo de
Romaña (edema bipalpebral unilateral, conjuntivitis y linfadenitis preauricular), esta fase se manifiesta con fiebre, linfadenopatías,
hepatoesplenomegalia y mal estado general, se pueden presentar complicaciones como miocarditis aguda, meningoencefalitis, 5% de
niños fallece en esta etapa. Fase indeterminada; duración variable sin parasitemia detectable, se han reportado anomalías anatómicas y
funcionales, serología positiva. Fase crónica; hay compromiso visceral irreversible: cardiomiopatía chagasica (míocardiopatía dilatada,
arritmias severas, bloqueos), o de tubo digestivo, con la mayor frecuencia en intestino o esófago. Las manifestaciones incluyen disnea
de grandes a pequeños esfuerzos, palpitaciones, edema en miembros inferiores, dolor torácico, cuadros sincopales. La destrucción de
células ganglionares de la submucosa de tracto digestivo, dan lugar aperistalsia, retención de residuos y dilatación de órganos
(megaesofago, megacolon), puede haber disfagia, pirosis, dispepsia. Laboratorio: en etapa aguda búsqueda de Trypanozoma cruzi en
sangre, por examen directo y tinción de extendidos de sangre. En etapa crónica, la parasitemia es transitoria, por ello diagnostico
mediante el hallazgo de anticuerpos circulantes de T. cruzi, con hemaglutinación directa, ELISA, inmundo fluorescencia, además
contemplar microscopia de gota gruesa o extendida, PCR, xenodiagnostico indirecto y hemocultivo. Estudios para función cardiaca;
ECG, ecocardiografia, Holter, cateterismo, biopsia. TRATAMIENTO: Benzonidazol; 5-7.5 mg/kg/día en fase aguda, casos congénitos y
pacientes en etapa crónica. Nirfurtimox; 8-10mg/kg/dia 60 a 90 dosis, en fase aguda y crónica. Dichos medicamentos, sin embargo no
logran erradicar el parasito, ni previenen la progsion de la enfermedad una vez que esta se ha establecido. En la míocardiopatía, se
utilizan digitalicos, antiarritmicos, diuréticos, marcapasos, trasplante cardiaco. PREVENCIÓN: La mejor solución a largo plazo, consiste
en la eliminación de los transmisores mediante el mejoramiento de viviendas, cuya construcción a base de adobe permite la aparición
de grietas que albergan a los insectos, empleo de insecticidas (piretroides), medidas de educación en salud. Tamizare obligatorio en
bancos de sangre y a mujeres embarazadas en regiones endémicas.

CASO CLINICO
Masculino de 42 años de edad, agricultor. Motivo de consulta por presentar hace mas menos 15dias “dolor de barriga” con sensación
de plenitud abdominal, estreñimiento, dolor a la deglusion, edema de miembros pélvicos, casi todos los dias “faltade aire” se siente
cansado con dolor en el pecho que aparece a pequeños esfuerzos aunque no es de mucha intensidad y desaparece con el reposo. Por lo
cual se decide su ingreso paramejor estudio. Refiere que hace mucho tiempo lo pico un insecto y tuvo inflamación en un ojo pero no
sabe con exactitud que insecto fue ni la fecha. Condiciones de la vivienda: pesima. Casa con piso de tierra agua de pozo y letrina a
distancia. EF: disnea, cardiovascular, dolor precordial simple ligero a moderado que disminuye con el reposo, edema en miembros
inferiores, astenia, dolor en toda la región abdominal, colicos, disfagia, constipación, disuria, cefalea, anorexia.

CURSO ENARM CMN SIGLO XXI TEL: 36246001 Pharmed Solutions Institute PÁGINA 111
MANUAL DE TRABAJO DEL CURSO ENARM CMN SIGLO XXI

PREGUNTA
Cual es el pronostico del caso?

RESPUESTA
a.- Malo para la función y malo para la vida
b.- Malo para la función y bueno para la vida.
c.- Bueno para la función y bueno para la vida.
d.- Bueno para la función y malo para la vida.

LEPRA (MYCOBACTERIUM LEPRAE):


CIENCIAS BASICAS: Enfermedad granulomatosa multisistémica crónica causada por Mycobacterium leprae parasito intracelular
obligado, no móvil, no espolurado, que afecta hígado, piel, mucosas de las vías respiratorias altas, testículos (disfunción sexual, atrofia),
ojos y nervios periféricos. SALUD PÚBLICA: Actualmente existen zonas de alta incidencia en África, Asia y América Latina. En México 21
estados presentan incidencia, los de mayor son Sinaloa, Michoacán, Nuevo León, Guerrero y Nayarit. La razón hombre: mujer 2:1, más
común en >25 años. PATOGENIA: El modo de transmisión e invasión sigue siendo controvertido. Es de notar que pacientes con lepra-
lepromatosa la mayor diseminación del bacilo al ambiente se realiza a través de la descarga nasal, el estornudo y la tos; por ello se
plantea que la principal ruta de infección es la via aérea (microgotas), la diseminación a través de la piel es menos importante.
Incubación de 2-4 años o más. Para adquirir la infección se requiere la convivencia estrecha y prolongada con un enfermo bacilifero y
con inmunidad celular disminuida identificada. DIAGNOSTICO: Clínica; En piel maculas, pápulas, nódulos, placas o infiltraciones,
afectando especialmente macrófagos. La hipopigmentación o eritema de la piel con déficit sensorial (hiperestesia, hipoestesia o
anestesia), es uno de los signos clínicos más importantes en el diagnostico, se observan en la palma de la mano o planta del pie, y los
origina una infección de la bacteria en las fibras nerviosas (engrosamiento de troncos nerviosos periféricos), alteraciones motoras. El
daño al sistema nervioso es una característica, a pesar de no tener habilidad locomotora, puede moverse en el endotelio a través del
tejido conectivo y alcanzar las células de Schwan. Las pruebas inmunológicas son muy limitadas, no existe un examen confirmatorio,
baciloscopia positiva con resultado completo, detección de anticuerpos anti-PGL-I, donde se presentan grandes cantidades de IgM.
ELISA, pero tienen sensibilida y especificidad baja. El diagnostico histopatológico es obligatorio para el pronóstico y de esta manera
favorecer el tratamiento, se pueden obsrevar infiltrados en la dermis, hipodermis y órganos internos tales como células de Virchow,
que son macrófagos con muchos bacilos y gotas de lípidos en su citoplasma, con apariencia espumosa. En secreciones nasales y
cutáneas el hallazgo del bacilo aislado en forma de globia puede ser detectado cpon coloración de Zieehl Neelsen. CLASIFICACION: Para
fines de control sanitario los casos se clasifican en: Multibacilares (MB); los lepromatosos, dimorfos y con infiltración difusa .
Paucibacilares (PB); los tuberculoides e indeterminados. Clasificación clínica:
Características 5 LEPROMATOSA 3 BODERLINE 4 BODERLINE 2 BODERLINE 1 TUBERCULOIDE
(progresa a necrosis y ulceración) LEPROMATOSA BODERLINE (dimorfa) TUBERCULOIDE
Tipo de lesión Maculas, pápulas, nódulos, Maculas, pápulas, Placas y lesiones en forma Placas de infiltrados Placas de infiltrados
infiltración difusa nódulos, infiltración de cúpula y sacabocados
Numero de Numerosas Muchas Muchas Única, usualmente con <5 lesiones
lesiones lesiones satélites o >5
lesiones
Distribución de las Simétricas (afecta más labios, Con tendencia a la Evidentemente No difusas y asimétricas Asimétricas
lesiones encías, dorso de lengua, paladar simetría Asimétricas
duro)
Definición de las Imprecisas, difícil de definir la Imprecisas delimitadas Imprecisas delimitadas por Bien definidas, bordes bien Bien definidas bordes
lesiones enfermedad y la afección de la piel por los bordes externos los bordes externos mal delimitados, eritematosas o bien delimitados
definido cobrizas
Sensibilidad No afectada Disminuida Disminuida Ausente Ausente
Bacilos en las Muchas globias Muchos Muchos +/- Negativa
lesiones en la piel
TRATAMIENTO: Los esfuerzos globales para controlar la lepra mediante la poliquimioterapia han permitido disminuir significativamente
el número de pacientes infectados. Sin embargo la detección de casos nuevos y las estrategias de control no han logrado la eficacia
esperada. Casos multibacilares usar dosis mensual supervisada de rifampicina, clofazimina, dapsona y dosis diaria autoadmnistrada de
dapsona y clofazimina. Para casos paucibacilares usar en dosis mensual supervisada (como mínimo 6 dosis mensuales) rifampicina y
dapsona y dosis diaria autoadministrada (mínimo 162 dosis) de dapsona. Usar hasta desaparición de lesiones.

CASO CLINICO
Masculino de 54 años de edad, constructor, nacido en Tamaulipas y radicado en Veracruz. Presenta dermatosis diseminada
caracterizada por nódulos de diferentes tamaños desde 3 x 3 hasta 8 x 8 mm, algunos de coloración de la piel y otros eritematosos en
tronco y extremidades superiores e inferiores de 8 meses de evolución.

PREGUNTA
Que tipo es?

RESPUESTA
a.- Lepromatosa.
b.- Borderline lepromatosa.
c.- Borderline tuberculoide.
d.- Tuberculoide.

CURSO ENARM CMN SIGLO XXI TEL: 36246001 Pharmed Solutions Institute PÁGINA 112
MANUAL DE TRABAJO DEL CURSO ENARM CMN SIGLO XXI
CASO CLINICO
Masculino de 59 años de edad originario de boca de rio, acude por presencia de dermatosis diseminada con nódulos de 5 a 8 mm en
cara, tronco, miembros torácicos y pélvicos de un año de evolución, con manchas residuales hipercrómicas, con eritema mínimo en
ambas extremidades pélvicas. Se detectó alopecia en tercio distal de ambas cejas y madarosis bilateral. La investigación seriada de
bacilos alcohol ácido resistentes en linfa y moco nasal fueron positivas. Se agregaban descamación plantar y onicodistrofia de dedos
gordos de ambos pies. El paciente cursaba con reacción leprosa tipo II además de calosfríos, artralgias maleolares y epistaxis de
repetición. La exploración física detectó alteraciones de sensibilidad y motoras en dedos de ambas manos e insuficiencia venosa en
ambos miembros pélvicos.

PREGUNTA
Que tipo es?

RESPUESTA
a.- Lepromatosa.
b.- Borderline lepromatosa.
c.- Borderline tuberculoide.
d.- Tuberculoide.

TRIPANOSOMIASIS AFRICANA (TRYPANOSOMA BRUCEI):


CIENCIAS BASICAS: También llamada enfermedad del sueño, es una parasitosis transmitida por un vector. ocurre en África es
transmitida a los seres humanos por la picadura de la mosca tsetse, que ha contraído la infección de personas o animales que albergan
los parasitos patógenos para el ser humano. Las personas mas expuestas personas que se dedican a la agricultura, la pesca, la ganadería
o la caza. Trypanosoma brucei gambiense causa 95% de los casos de esta enfermedad y es una infección crónica, generalmente
asintomática, los síntomas aparecen cuando la enfermedad ya esta muy avanzada, en la etapa de afección al SNC. Trypanosoma brucei
rhodesiense causa 5% y causa una infección aguda, también afecta SNC. SALUD PÚBLICA: en los últimos 10 años, mas del 70% de los
casos notificados ocurrieron en la República Democrática del Congo, en 2010 esta reporto mas de 500 casos nuevos por año.
PATOGENIA: Transmisión principal por picadura de mosca, también atraves de la placenta, con agujas contaminadas. En la primer
etapa, los tripanosomas se multiplican en los tejidos subcutáneos, la sangre ya la linfa; se conoce como etapa hemolinfatica y se
caracteriza por episodios de fiebre, cefalea, dolores articulares y prurito. eN la segunda etapa, los parasitos atraviesan la barrera
hematoencefalica e infectan el SNC; es la etapa de afección nerviosa, aquí se presentan las manifestaciones mas evidentes como :
cambios en el comportamiento, confusión, trastornos sensoriales y falta de coordinación. Los trastornos en el ciclo del sueño que le dan
el nombre a la enfermedad, son una característica importante, si no se da tratamiento es mortal. DIAGNOSTICO: Tamizaje de una
posible infección (pruebas serologicas y exploración Clinica), diagnostico de la presencia del parasito, determinación de la etapa en la
que se encuentra la afección (punción lumbar y estudio del LCR). TRATAMIENTO: Depende de la etapa, cuanto mas pronto se inicie,
mejores las probabilidades de curación. En la primer etapa: pentamidina y suramina. Tratamiento en la segunda etapa deben ser
medicamentos que atraviesen la barrera hematoencefalica como: melarsoprol, eflornitina, nifurtimox.

CASO CLINICO
Femenino que ingresa por astenia, somnolencia de varios días de evolución, sin determinar tiempo, acompañada de fiebre de 38.5º C,
las mucosas están hipocoloreadas, sin otros síntomas y signos acompañantes que ligeras adenopatías cervicales bilaterales, una punta
de bazo y discreta rigidez de nuca. Al día siguiente de su ingreso se constata fiebre de 39 y 40º C, está apática, asténica, deja de comer y
de hablar pasa a un estado estuporoso y en horas de la madrugada tiene convulsiones tónico-clónicos generalizadas, se observan
temblores que se repetían igual que las convulsiones, a pesar de las medidas antitérmicas y anticonvulsinantes; se muestra
quejumbrosa en oportunidades, posteriormente se comporta muy intranquila y con gran agitación psicomotora; cae en una fase más
profunda del coma, no responde sino a estímulos profundos, tiene rigidez de descerebración. Se constata Kerning positivo y marcada
rigidez nucal.

PREGUNTA
De acuerdo a la etapa cual es el tratamiento de elección?

RESPUESTA
a.- Nifurtimox.
d.- Pentamidina.
c.- Suramina.
d.- Prazicuantel.

ONCOCERCOSIS (ONCHOCERCA VOLVULUS):


CIENCIAS BÁSICAS: Parasitosis del hombre causada por onchocerca volvulus, que afecta piel, ojos llegando a producir ceguera,
transmitida por insectos hematofagos del genero Simulium. SALUD PUBLICA: En regiones subtropicales del África occidental y en
reducidas áreas del Yemen y América Central. En México se encuentran 3 focos endémicos en Oaxaca y norte de Chiapas. PATOGENIA:
La O. volvulus es una filaria, la hembra vivípara, libera embriones móviles y activos (microfilarias, atraviesan los nódulos y llegan a los
tejidos dermicos), en vez de huevos, algunas son ingeridas por una hembra de simulido, la mayoría muere y es eliminada por
fagocitosis. Los adultos viven en nódulos fibrosos u oncocercomas subcutáneos, aunque algunos se han localizado adheridos al
periostio, oquedades óseas. En México, los nódulos se ubican fundamentalmente en la cabeza y tronco, mientras que en África es más
común en cintura pélvica. La oncocercosis cutánea resulta de mecanismos originados por el desplazamiento de las microfilarias y de las
secuelas resultantes de reacciones inflamatorias acumuladas. DIAGNOSTICO: Clínica; las primeras manifestaciones son cutáneas

CURSO ENARM CMN SIGLO XXI TEL: 36246001 Pharmed Solutions Institute PÁGINA 113
MANUAL DE TRABAJO DEL CURSO ENARM CMN SIGLO XXI
irritación, prurito, edema e hipertermia localizadas, la piel se engrosa y hay erupciones papulares y ligeros cambios en la pigmentación.
El prurito se intensifica y el rascado causa excoriaciones que se infectan secundariamente, zonas de hiperpigmentacion y
despigmentacion puede presentarse liquenificacion (epidermis engrosada, formas nodulares y descamación). La migración continua y
prolongada de las microfilarias, junto con la respuesta inflamatoria origina la perdida de elasticidad cutánea y explica la fascies leonina.
Al engrosamiento de la piel mas perdida de elasticidad se debe la paquidermitis. En ojo podemos encontrar las microfilarias en humor
vítreo y acuoso, afectan regiones anterior y posterior de ambos ojos, en cornea se presenta queratitis punteada (corta evolución)
después queratitis esclerosante que opacas permanentemente la cornea, la visión periférica se ve reducida ya la función visual limitada.
También se produce uveitis, atrofia del nervio óptico y alteraciones en retina, que causan ceguera irreversible. Epidemiología: individuo
con residencia permanente o antecedentes de visita en área endémica. Parásitologico; confirma impresión clinica y se realiza mediante
la observación de microfilarias que emergen de biopsias cutáneas o directamente por la observación de microfilarias en la cámara
anterior y posterior del ojo con lampara de hendidura. TRATAMIENTO: Para eliminar a los adultos de O. volvulus, la extirpación de los
nódulos subcutáneos palpables, continua siendo el procedimiento de elección. Actualmente se utiliza la ivermectina, eficaz contra las
microfilarias, en piel y ojo y el efecto perdura 8 meses, en una sola dosis (0.15mg/kg de peso). En algunos países se ha utilizado un
esquema mixto, con doxiciclina.

CASO CLINICO
Se trata de una paciente de 24 años de edad, que acudió por presentar dolor y fotofobia en OD. En la exploración, se apreció la
existencia de una uveítis anterior, por lo que se comenzó un tratamiento con midriáticos y antiinflamatorios. A los pocos días acude a
urgencias de Oftalmología, donde se ve una disminución del cuadro uveítico anterior, pero, se descubre la existencia de una vitritis con
un foco de coriorretinitis parapapilar inferior, que afecta a la papila, con exudados algodonosos y hemorragias, así como perivasculitis
zonal, observada también en la AFG; el fondo de ojo del Ol era normal. Ante este cuadro, la paciente ingresa con la sospecha de
coriorretinitis por citomegalovirus por lo que se inicia tratamiento sistémico con ganciclovir IV. Sin antecedentes patológicos de interés,
había recibido tratamiento para una infección urinaria hacía un mes y tenía un antecedente de anexitis, ya curada. En el estudio
analítico inicial aparecía una eosinofilia mayor de 1.000/mm3.

PREGUNTA
Cual es la conducta a seguir mas adecuada?.

RESPUESTA
a.- Ascultacion con lámpara de hendidura.
b.- Frotis sanguíneo.
c.- Verificar los valores de eosinofilia.
d.- Iniciar ivermectina.

CASO CLINICO
Paciente masculino de 62 años, natural y procedente de oaxaca, de oficio agricultor, quien consultó por presentar tumoraciones
dérmicas en miembros superiores e inferiores, prurito generalizado, cefalea, artralgias y disminución de la agudeza visual, con
presencia de nódulo subcutáneo, Bh eosinofilia importante.

PREGUNTA
Cual de los siguientes métodos diagnosticos es mas adecuado para establecer el diagnostico?

RESPUESTA
a.- Verificar por lámpara de hendidura.
b.- Biopsia del nodulo.
c.- Frotis de sangre.
d.- Eosinofilia persistente.

LEISHMANIASIS (LEISHMANIA ssp):


CIENCIAS BASICAS: Enfermedad tropical, casada por el protozoo leishmania, intracelular obligado del humano (localizado en
macrófagos y células dendríticas), produce lesiones a nivel cutáneo, mucocutaneo y visceral. Los insectos transmisores son dípteros del
genero Lutzomyia el cual adquiere la infección del humano y reservorios como roedores, canidos y primates. También se puede adquirir
por contacto con material de una lesión, trasplante de órganos, transfusión y a través de placenta. Enfermedad emergente oportunista,
asociada a SIDA. SALUD PÚBLICA: Se estima que 12 millones de personas se encuentran infectadas, con unos 1-2 millones de casos
nuevos al año. La falta de sistemas de vigilancia y el subdiagnostico, hacen difícil estimar la real incidencia y el índice de letalidad. Más
de 90% de casos de leishmaniasis visceral se presentan en Bangladesh, Sudán, Sudán del Sur, Etiopia y Brasil. En México en Chiapas,
Oaxaca, Tabasco, Veracruz, Puebla y Guerrero. PATOGENIA: Periodo de incubación de 1-12 semanas. El promastigote metacíclico
extracelular (forma infectante), es introducido en la piel del hospedero a través del piquete de la mosca hembra. Los parásitos son
fagocitados en la piel por macrófagos y células de Langerhans y activan el complemento. Aunque muchos promastigotes son destruidos
por los leucocitos PMN, unos pocos se transforman en amatigotes que es intracelular (forma replicativa), se multiplica por división
binaria, en los fagolisosomas dentro de fagocitos mononucleares de los hospederos. Las células infectadas se rompen y los amastigotes
se diseminan a los tejidos, induciendo a producción de citocinas proinflamatorias. La leishmaniasis cutánea difusa se presenta como
consecuencia de factores inmunes del hospedero, asociado a ciertas especies del parasito. La forma visceral (kala-azar)es la más
severa, causada por L. donovani. DIAGNOSTICO: Clínica; Se consideran 2 cuadros clínicos cutáneos: la leishmaniosis cutánea localizada
(LCL), generalmente circunscrita al sitio de inoculación, gracias a la respuesta inmune celular protectora, en México conocida como
“ulcera de los chicleros” (redondeada de borde elevado y bien definido, indurado, cubierta por una costra amarillenta, cuan do esta se

CURSO ENARM CMN SIGLO XXI TEL: 36246001 Pharmed Solutions Institute PÁGINA 114
MANUAL DE TRABAJO DEL CURSO ENARM CMN SIGLO XXI
desprende, fondo de tejido de granulación limpio), con frecuencia afectando el pabellón auricular y la leishmaniasis cutánea
diseminada (LCD), caracterizada por pobre respuesta inmune y diseminación no controlada en piel, caracterizada por lesiones
nodulares con gran numero de parásitos, diseminadas en toda la piel excepto cuero cabelludo, región inguinal y axilar, genitales,
plantas y palmas. Fiebre, palidez, anorexia, pérdida de peso, tos, vomito, diarrea hepatoesplenomegalia, linfadenopatías, sangrado
gingival, epistaxis, equimosis, anemia; posteriormente taquicardia, ictericia, distención abdominal, ascitis, edema, sangrados
importantes, hiperpigmentacion, lesiones verrucosas no ulceradas y alopecia. Laboratorio: trombocitopenia, anemia normocitica
normocrómica, leucopenia. Hipoalbuminemia, hipergammaglobulinemia. Biopsia de bazo e hígado (amastigotes). Raspado, biopsia,
improntas y extendido de lesiones y ganglios linfáticos. La intradermorreacción de Montenegro, prueba de hipersensibilidad celular a
antígenos de Leishmania, es un método indirecto. TRATAMIENTO: En casos graves la mortalidad sin tratamiento es de 100%. La
primera línea esta constituida por los antomoniales pentavalentes: antimoniato de meglumina (glucantime) y estibogluconato de sodio
(pentostam). Otros fármacos sistémicos utilizados son: anfotericina B, paromomicina, sitamaquina.

CASO CLINICO
Mujer de 70 años que consulta por la aparición de una pápula en la frente, algo pruriginosa, que sigue creciendo y a la que se van
añadiendo lesiones similares en su periferia, pese al tratamiento empírico con corticoides, sin sospecha diagnóstica. La lesión se
considero como placa eritematopapulosa, la biopsia de la lesión demuestra una dermatitis granulomatosa no necrotizante. La reacción
en cadena de la polimerasa (PCR) diagnostica una leishmaniasis cutánea (LC).

PREGUNTA
Cual es el tratamiento mas adecuado para el caso?

RESPUESTA
a.- Antimoniato de meglumina.
b.- Anfotericina B.
c.- Paromomicina.
d.- Sitamaquina.

ESQUISTOSOMIASIS (SCHISTOSOMA):
CIENCIAS BASICAS: Enfermedad parasitaria crónica causada por duelas sanguíneas (trematodos) del genero Schistosoma. Corren el
riesgo de contraer la infección las personas que realizan actividades agrícolas, domesticas o recreativas que las exponen a aguas
infestadas. La falta de higiene y las actividades lúdicas hacen que los niños sean especialmente vulnerables a la infección. SALUD
PÚBLICA: Es prevalente en las regiones tropicales y subtropicales, en especial en comunidades sin acceso a agua potable. El número de
personas tratadas aumento de 12,4 millones en 2006 a 33,5 millones en 2010. Al menos 230 millones de personas necesitan
tratamiento cada año. PATOGENIA: Las personas se infectan cuando las formas larvarias del parasito, liberadas por caracoles de agua
dulce (son el reservorio, donde maduran los huevos), penetran en la piel durante el contacto con aguas infestadas. En el interior del
organismo, las larvas se convierten en esquistosomas adultos, que viven en los vasos sanguíneos, emigran a las venas abdominales
donde se instalaran definitivamente y se reproducirán es aqui donde las hembras ponen sus huevos. Algunos de esos huevos salen del
organismo con las heces o la orina y continúan el ciclo vital del parasito. Otros quedan atrapados en los tejidos corporales, donde
causan una reacción inmunitaria y un daño progresivo de los órganos. Hay dos formas principales: Esquistosomiasis intestinal
(Schistosoma mansoni, japonicum, mekongi) y esquistosomiasis urogenital (Schistosoma haematobium). DIAGNOSTICO: Clínico; Los
síntomas son causados por la reacción del organismo al huevo y no por el gusano en si mismo. En la forma intestinal, dolor abdominal,
diarrea y sangre en las heces, en casos avanzados, hepatoesplenomegalia, ascitis e hipertensión portal. El signo clásico de la forma
urogenital es la hematuria, puede haber disuria, poliuria, en casos avanzados son frecuentes la fibrosis de la vejiga y los uréteres, así
como las lesiones renales. El cáncer de vejiga es otra posible complicación tardía. En las mujeres puede haber lesiones genitales,
hemorragias vaginales, dispareunía y nódulos vulvares. En el hombre trastornos de las vesículas seminales y próstata, en ambos puede
llevar a la infertilidad. En niños puede causar anemia, retraso de crecimiento y problemas de aprendizaje. Laboratorio: Detección de
huevos del parasito en muestras de heces (frotis en glicerina con azul de metileno) u orina, en la forma urogenital puede haber
microhematuria. TRATAMIENTO: El prazicuantel es el único tratamiento disponible contra todas las formas de esquistosomiasis.

CASO CLINICOS
Niño de ocho años procedente de guerrero, que acude para estudio de hematuria macroscópica al final de la micción y proteinuria de
más de un año de evolución. Refiere baños anteriores frecuentes en río. No tiene antecedentes personales ni familiares a destacar. La
exploración física fue normal. Los análisis de laboratorio muestran un hemograma con anemia microcítica, con Hb 10,9 g/dL, Hto 32,8%,
VCM 74,9 fL y eosinofilia con 767 eosinófilos/µL; VSG 40 mm/h. La bioquímica muestra urea y creatinina e iones normales. Estudio
inmunológico con niveles de inmunoglobulinas y complemento dentro de la normalidad. El sistemático de orina muestra hematuria
microscópica y presencia de cristales de oxalato cálcico. La ecografía abdominal fue informada como hidronefrosis grado II de pelvis
renal izquierda. La investigación de Schistosoma sp. en orina fue positiva visualizándose huevos de S. haematobium. El coprocultivo fue
negativo. Las serologías para Plasmodium falciparum, Entamoeba histolytica, Schistosoma sp., y virus de las hepatitis A, B y C fueron
negativas. El tratamiento se realizó mediante dos dosis de praziquantel a 20 mg/kg/dosis separadas por 12 horas. El paciente no acudió
a posteriores revisiones.

PREGUNTA
Cual de las siguientes manifestaciones es mas caracteristicos en esta patologia?

RESPUESTA
a.- Presencia de huevesillos en heces.

CURSO ENARM CMN SIGLO XXI TEL: 36246001 Pharmed Solutions Institute PÁGINA 115
MANUAL DE TRABAJO DEL CURSO ENARM CMN SIGLO XXI
b.- Positivo en frotis periférico.
c.- Hepato y esplenomegalia.
d.- Hematuria y alteración renal.

FILARIASIS (WUCHERERIA BANCROFTI):


CIENCIAS BASICAS: La filariasis linfática, es causada por la infección por nematodos de la familia Filarioidea. Hay tres tipos de estos
gusanos filiformes: Wuchereria bancrofti, que es responsable del 90% de los casos; Brugia malayi, que causa la mayoría de los casos
restantes; B. timori, que también causa la enfermedad, conocida generalmente como elefantiasis, es una enfermedad tropical
desatendida. La infección humana se produce por la transmisión de unos parásitos denominados filarias a través de los mosquitos.
SALUD PÚBLICA: En el mundo hay más de 1300 millones de personas de 72 países en riesgo de sufrir esta enfermedad. En la actualidad
hay más de 120 millones de personas infectadas, y unos 40 millones están desfigurados e incapacitados por la enfermedad.
Aproximadamente un 65% de los infectados viven en la Región de Asia Sudoriental, el 30% en la Región de África, y los demás en otras
zonas tropicales. La filariasis linfática afecta a más de 25 millones de hombres con enfermedad genital y a más de 15 millones de
personas con linfedema. Como la prevalencia de la enfermedad y la intensidad de la infección están relacionadas con la pobreza.
PATOGENIA: Cuando un mosquito (cullex, anopheles, aedes), que contiene larvas en estado infectivo pica a una persona deposita los
parásitos en la piel. Desde ahí las larvas invaden el organismo, migrando hacia los vasos linfáticos donde se desarrollan y transforman
en gusanos adultos en el sistema linfático. Los gusanos adultos se alojan en el sistema linfático y alteran el sistema inmunitario. Estos
gusanos viven entre 6 y 8 años, y a lo largo de su vida producen millones de pequeñas larvas (microfilarias) que circulan en la sangre.
DIAGNOSTICO: La infección suele adquirirse en la infancia, pero las manifestaciones dolorosas y muy desfigurantes de la enfermedad
aparecen más tarde. Mientras que los episodios agudos de la enfermedad causan discapacidad transitoria, la filariasis linfática produce
discapacidad permanente. La filariasis linfática adopta formas asintomáticas, agudas y crónicas. La mayoría de las infecciones son
asintomáticas y no presentan signos externos. A pesar de ello dañan el sistema linfático, los riñones y el sistema inmunitario. El
linfedema crónico (tumefacción de los tejidos), o elefantiasis (engrosamiento de la piel), se acompaña a menudo de episodios agudos
de inflamación local de la piel y de los ganglios y los vasos linfáticos. Algunos de esos episodios son causados por la respuesta
inmunitaria del organismo contra el parásito. Sin embargo, la mayoría se debe a infecciones bacterianas cutáneas porque las defensas
normales se han deteriorado debido al daño linfático. Es frecuente la afectación de las mamas y de los órganos genitales.
TRATAMIENTO Y PROFILAXIS: El régimen recomendado es administración conjunta de dos medicamentos en dosis únicas: albendazol
(400 mg) más ivermectina (150-200 mcg/kg) en zonas donde la oncocercosis (ceguera de los ríos) también es endémica, o citrato de
dietilcarbamazina (6 mg/kg) en zonas donde la oncocercosis no es endémica. Estos medicamentos eliminan las microfilarias del
torrente sanguíneo y matan a la mayoría de los gusanos adultos. Para interrumpir totalmente la transmisión de la infección es necesario
que la farmacoterapia colectiva dure entre 4 y 6 años. Se recomienda que los pacientes con discapacidad crónica (elefantiasis,
linfedema o hidrocele) mantengan una higiene rigurosa y tomen las precauciones necesarias para prevenir las infecciones secundarias y
la agravación de la enfermedad. Para eleiminacion la OMS recomienda farmacoterapia colectiva.

CASO CLINICO
Mujer de 33 años originaria de chiapas, ama de casa y agricultora, quien refiere enfermedad insidiosa de 8 años de evolución,
caracterizado por la aparición de lesiones papulares que pustulizan, localizadas a predominio de tronco y extremidades, acompañadas
de prurito y en ocasiones dolor pungitivo. Antecedentes: padeció paludismo y dengue, Al examen físico, adelgazada, con leve palidez de
piel y mucosas, múltiples máculas hipercrómicas en todo el cuerpo. Se palpan pápulas sin flogosis en axilas, espalda y muslo izquierdo.
A la vitropresión elimina secreción blanquecina con parásito filiforme de 3 cm. de longitud con cabeza y cola. No adenopatías. No
edemas. No visceromegalia. El resto no contributorio. Su hemograma mostraba Hemoglobina 8.5gr% Leucocitos 6,100 A0, S58, E24, B0,
M6, L12.

PREGUNTA
Respecto al tratamiento cual de los siguientes esquemas es el mas adecuado?

RESPUESTA
a.- Levamisol 150 mg diarios durante 3 meses. Mas Piperacina 400 mg diarios 45 dias.
b.- Albendazol 800 mg diarios por 45 días en dos cursos.
c.- Albendazol (400 mg) más ivermectina (150-200 mcg/kg).
d.- Citrato de dietilcarbamazina (6 mg/kg).

CRIPTOCOCOSIS (CRYPTOCOCCUS NEOFORMANS):


CIENCIAS BÁSICAS: Micosis sistémica aguda, subaguda o crónica. Causada por el hongo levaduriforme Cryptococcus neoformans. La
forma pulmonar es generalmente transitoria y leve, la inclusión del sistema nervioso central con meningitis es la forma más familiar de
la micosis. SALUD PÚBLICA: Esporádico. La meningoencefalitis tiene mortalidad de 100%, en no tratados y 40% en los tratados.
PATOGENIA: La infección se adquiere al inhalar el hongo (presente en heces de palomas y sus nidos). El SIDA es el principal factor
predisponente, también uso de esteroides, neoplasias; la DM, cirrosis y leucopenia aumentan el riesgo de padecer la enfermedad. Se
transmite en la forma de basidiosporas, presentes en el ambiente por tiempos cortos y son infecciosas para el humano y otros
animales, después de su diseminación pueden sintetizar material pólisacarido y convertirse en levaduras encapsuladas. Al entrar por
vías respiratorias altas, el hongo coloniza el árbol bronquial, por ello inicia como una enfermedad pulmonar, que se disemina a piel,
huesos, vísceras, abdominales y SNC. DIAGNOSTICO: Clínica; asintomática, a veces dolor torácico, tos, fiebre, infiltrados intersticiales o
lobeares y derrame pleural; las lesiones pulmonares se caracterizan por inflamación granulatiza, la diseminación hematogena permite
que infecte el SNC (sustancia gris cortical y ganglios basales). La meningoencefalitis, los datos clínicos están relacionados con el
desarrollo de lesiones granulomatosas en las meninges, se presenta con nausea, cefalea, irritabilidad, confusión, demencia, visión
borrosa, fiebre, rigidez de cuello y parálisis de nervios craneales. En el 10% aparecen lesiones cutáneas (papilas, úlceras, abscesos).

CURSO ENARM CMN SIGLO XXI TEL: 36246001 Pharmed Solutions Institute PÁGINA 116
MANUAL DE TRABAJO DEL CURSO ENARM CMN SIGLO XXI
Laboratorio: cultivo en Sabouraud, forma colonias de color crema pálido. Antígeno manan se detecta en 33% de los casos. Biopsia para
diagnostico de criptococosis pulmonar. LCR vemos levadura encapsulada, hipoglucorraquia, pleocitocis linfocitica y aumento de
proteínas. TRATAMIENTO: Anfotericina B a 0.7mg/kg/día y 5-fluorocitocina a 100mg/kg/día durante 6-10 semanas.

CASO CLINICO
Paciente femenina de 35 años con antecedentes de presentar desde hacía dos meses cefalea, irritabilidad, pérdida de la memoria,
cambios de personalidad, alteraciones del sensorio, dificultad para caminar por disminución de la fuerza muscular en el miembro
inferior izquierdo y ataxia de la marcha. EF: presenta alteraciones del sensorio con somnolencia, hemiparesia izquierda ligera a
predominio crural, ataxia de la marcha y rigidez nucal y lesiones vesiculosas herpéticas en la región peribucal. El fondo de ojo mostraba
borramiento de los bordes temporales de ambas papilas. Estando ingresada se queja de vértigo y presenta varios vómitos. Se le realizan
los siguientes exámenes de laboratorio: Hemoglobina: 122 g/l, Hematocrito: leucocitos: 6.1x109 g/l, neutrófilos: 59, linfocitos: 40,
monocitos: 1; eritrosedimentación: 120 min; glicemia: 4.8 mmol/l; creatinina: 212 mmol/l, TGP: 11 unid, urocultivo: normal, sin
crecimiento; tomografía axial computarizada (TAC) de cráneo simple: sin alteraciones evidentes; Líquido cefalorraquídeo (LCR)
citoquímico claro y transparente, células: 11, pandy (-), proteínas: 25, glucosa 1.9 mmol/l.

PREGUNTA
Cual es la conducta diagnostica mas adecuada para el presente caso?

RESPUESTA
a.- Serologia e inmunológica.
b.- Elisa y west-blot.
c.- Resonancia magnetica con contraste.
d.- Cultivo y prueba de tinta china en LCR.

PREGUNTA
El paciente es positivo para VIH, al termino del tratamiento habitual cual de las siguientes alternativas para continuar la profilaxis
secundaria es mas adecua?

RESPUESTA
a.- Fluconazol 200 mg / dia
b.- Ketoconazol 400 mg / dia.
c.- Itraconazol 100 mg / dia
d.- Clortrimazol 300 mg / dia.

CASO CLINICO
Se trata de paciente femenino de 38 años de edad la cual acude al servicio de urgencias debido a que presenta fiebre, cefalea y dolor en
la región de la nuca, como antecedentes de importancia la paciente trabaja en un aviario, no es diabética ni hipertensa, aparentemente
sana antes de este episodio, a la exploración física se observa conciente, irritable, orientada, facies algicas, se realiza TAC de cabeza el
cual no revela datos de importancia, se realiza punción lumbar reportándose presión de apertura de 20 cmH2O, leucocitos 15 células
(90 % monocitos), proteínas de 0.6 g/L, glucosa de 50mg/dl y tinción positiva con tinta de la India.

PREGUNTA
Cual es el tratamiento mas adecuado para el caso.

RESPUESTA
a.- Anfotericina.
b.- Fluconazol.
c.- Cexfriaxona.
d.- Vancomicina.

CITOMEGALOVIRUS (CMV):
CIENCIAS BASICAS: CMV es un herpes virus, que hace a las células infectadas 2-4 veces el tamaño de las células circundantes. Como
resultado de la primoinfección se establece una infección latente o persistente. El virus permanece principalmente en células
endoteliales y leucocitos durante toda la vida del individuo a menos que la reactivación se desencadene por la inmunidad celular
deprimida (por ejemplo, en receptores de trasplante o infectados por el VIH. Enfermedad de inclusión citomegálica (células gigantes,
con inclusiones que semejan un “ojo de lechuza”) SALUD PÚBLICA: Enfermedad por CMV se encuentra en todo el mundo. En los
Estados Unidos, aprox. 1 % de los recién nacidos están infectados. La seroprevalencia en países desarrollados va de 30-70% y en países
en vías de desarrollo alcanza hasta 90% en la edad adulta. Los homosexuales masculinos tienen mayor seropositividad 94%
PATOGENIA: El virus se puede transmitir por la leche materna, saliva, contacto sexual (virus en semen y secreciones cervicales), vía
transplacentaria, transfusión sanguínea, transfusión de órganos. La transmisión requiere el contacto repetido o prolongado. La
infección se establece en las células del hospedero, en la línea mieloide, que se transforma en el reservorio para infecciones
posteriores. DIAGNOSTICO: Clínico; La infección congénita por CMV; se produce en aproximadamente el 5 % de los fetos infectados en
el contexto de la infección por CMV materna primaria durante el embarazo. Presentan petequias, hepatoesplenomegalia e ictericia.
Otros hallazgos; microcefalia con o sin calcificaciones cerebrales, retraso del crecimiento intrauterino, prematuridad y coriorretinitis.
Los hallazgos de laboratorio incluyen pruebas anormales de la función hepática (PFH), trombocitopenia, hemólisis, y el aumento de los
niveles de proteínas del fluido cerebroespinal (CSF). La tasa de mortalidad es del 20-30% de los lactantes con enfermedad grave, los

CURSO ENARM CMN SIGLO XXI TEL: 36246001 Pharmed Solutions Institute PÁGINA 117
MANUAL DE TRABAJO DEL CURSO ENARM CMN SIGLO XXI
sobrevivientes tienen dificultades intelectuales o auditivas. Infección perinatal, se adquiere por la lactancia o contacto con secreciones
maternas infectadas. La mayoría de pacientes son asintomáticos, pero las infecciones oportunistas neumonitis intersticial y otras
pueden ocurrir, particularmente en los bebés prematuros. Mononucleosis; es el síndrome más común del CMV en huéspedes
inmunocompetentes. El período de incubación varía de 20 a 60 días. Los síntomas duran 2-6 semanas e incluyen fiebre, fatiga profunda
y malestar general, mialgias, dolor de cabeza, y esplenomegalia, faringitis y adenopatías cervicales son raras. Los hallazgos de
laboratorio incluyen linfocitosis relativa con > 10 % de linfocitos atípicos. El aumento de los niveles séricos de transaminasas y fosfatasa
alcalina así como anormalidades inmunológicas (por ejemplo, la presencia de crioglobulinas o aglutininas frías) pueden ser evidentes. La
recuperación es completa, pero astenia postviral puede persistir durante meses. La excreción de CMV en la orina, las secreciones
genitales, y/o la saliva pueden continuar durante meses o años. CMV es el patógeno viral más común e importante que complica el
trasplante de órganos, la infección por CMV es un factor de riesgo de pérdida del injerto y muerte. El riesgo de infección es mayor 1-4
meses después del trasplante, pero la retinitis puede ocurrir más tarde. El órgano trasplantado es un riesgo especial, por ejemplo,
neumonitis por CMV suele seguir un trasplante de pulmón. Alrededor del 15-20% de los receptores de trasplante de médula ósea va a
desarrollar neumonía por CMV, 5-13 semanas después del trasplante, con una tasa de letalidad del 84-88 %. El riesgo de enfermedad
grave se puede reducir por la profilaxis o terapia antiviral preventiva. El CMV es un patógeno importante en pacientes con infección por
el VIH cuyo recuento de células CD4 + han caído por debajo de 50 a 100 / l. En esta configuración, el CMV causa retinitis, colitis y
enfermedad diseminada. Pacientes inmunodeprimidos con infección por CMV desarrollan fiebre, malestar general, anorexia, fatiga,
sudores nocturnos y artralgias o mialgias. Taquipnea, hipoxia, y tos no productiva. La participación del tracto gastrointestinal puede ser
localizada o extensa, con úlceras que pueden sangrar o perforar. La hepatitis es común. SNC, con mayor frecuencia afectan a pacientes
infectados por el VIH, retinitis grave, encefalitis, polirradiculomielopatía. Retinitis por CMV puede dar lugar a ceguera. Las lesiones
comienzan como pequeñas áreas blancas de necrosis retiniana granular, con el desarrollo posterior de las hemorragias y edema
retiniano. Infección grave se asocia con viremía persistente y compromiso multiorgánico. Amplia necrosis suprarrenal se ve a menudo
en la autopsia. El cultivo viral (diagnostico de certeza) y los anticuerpos monoclonales se utilizan para detectar el antígeno temprano
inmediato de CMV. La detección de antígenos de CMV en los leucocitos de sangre periférica o de ADN de CMV en sangre. Los
anticuerpos no pueden ser detectados por un máximo de 4 semanas de la infección primaria, y los títulos pueden permanecer elevados
durante años. IgM puede ser útil en el diagnóstico de infección aguda. Cuando sea posible, los donantes seronegativos deben ser
utilizados para los receptores de trasplante seronegativos. TRATAMIENTO: Ganciclovir (o valganciclovir, el profármaco oral de
ganciclovir ) se usa como tratamiento y como profiláctico, produce tasas de respuesta del 70-90 % entre los infectados por el VIH,
pacientes con retinitis por CMV o colitis. La terapia de inducción con ganciclovir (5-12mg/kg/día IV 2-7 semanas) o valganciclovir (900
mg/24hrs VO) se da durante 14-21 días. En los receptores de trasplante, se ha demostrado reducción de la presencia de CMV, cuando
se administra tratamiento profiláctico a base de vanganciclovir cuando el paciente es susceptible. Actualmente se puede utilizar el
fármaco durante los primeros 100 días postrasplante. La dosis de inducción es de 900mg c/12 hrs y después 900mg c/24 hrs. Otros
fármacos activos contra CMV son: El foscarnet es activa contra la infección por CMV, pero se reserva para casos de insuficiencia a
ganciclovir o intolerancia a causa de sus efectos tóxicos, que incluyen la disfunción renal, hipomagnesemia , hipopotasemia ,
hipocalcemia , y parestesia. Este medicamento debe ser dado a través de una bomba de infusión, y su administración debe ser
monitoreado de cerca. Un régimen de inducción de 60 mg / kg cada 8 horas o 90 mg / kg cada 12 horas durante 2 semanas es seguido
por los regímenes de mantenimiento de 90-120 mg / kg al día. Cidofovir (en casos de retinitis) tiene una intracelular larga vida media.
Los regímenes de inducción de 5 mg / kg por semana durante 2 semanas son seguidos por los regímenes de mantenimiento de 3-5 mg /
kg cada 2 semanas. Cidofovir produce nefrotoxicidad grave por lesión de las células del túbulo proximal. El uso de la hidratación salina y
probenecid reduce este efecto adverso.

CASO CLINICO
Hombre de 53 años, sometido a trasplante renal hace un mes, que acude a urgencias por cuadro febril con temperatura de 38,5 ºC de
24 horas de evolución con escalofríos, dolor abdominal leve difuso, astenia, anorexia y disminución subjetiva del volumen de diuresis. El
cuadro progresó con un aumento del dolor abdominal, aparecieron nuevos picos febriles junto con cuadro diarreico, con melena,
asociado a deterioro neurológico, hepatoesplenomegalia, así como alteración función hepática, anemia y trombocitopenia. Analítica:
GOT/GPT 135/156 U/l; LDH 558 U/l; sodio 130 meq/l, fibrinógeno 133 mg/dl, hemoglobina 9,2 g/dl, y hematocrito del 26,8% y
plaquetas de 48.000 µl con leucocitos normales (5.500 µl con fórmula normal), aumento muy importante de triglicéridos (738 mg/dl),
deterioro progresivo de función renal (creatinina entorno a 4-5 mg/dl). Se solicitaron nuevas pruebas en las que destacaron: antígeno
precoz CMV positivo con PCR de CMV mayor a 100.000 copias/ml.

PREGUNTA
Considerando la complicación, cual es la medida profiláctica mas apropiada?

RESPUESTA
a.- Ganciclovir 200 mg por 100 días.
b.- Aciclovir 200 mg por 75 dias.
c.- Cidofovir 500 por 100 dias.
d.- Oseltanmivir 100 mg por 74 dias.

CASO CLINICO
Femenino de 42 años. Manifiesta 8 meses con herpes recurrente en boca. Ingresa por padecimiento de 2 meses con tos seca en accesos
y disnea rápidamente progresiva. Además dolor torácico bilateral, fiebre hasta 39°C y pérdida de peso de 14kg. La recurrencia de la
lesión herpética le ocasionaba disfagia y odinofagia. Al examen físico presentaba placas blanquecinas en orofaringe; la exploración del
tórax, con disminución del ruido respiratorio y estertores finos. Al aire ambiente la saturación de oxígeno era del 86%. El reporte de la
gasometría arterial con oxígeno suplementario al 70% fue: pH 7,30, pCO2 40,5mmHg, pO2 132mmHg, HCO3 19,5mmol/l, exceso de
base −5,8mmol/l, saturación de oxígeno al 97,9%. Índice de oxigenación (IO) de 188. Laboratorio: linfopenia de 600células/mm3, Hb

CURSO ENARM CMN SIGLO XXI TEL: 36246001 Pharmed Solutions Institute PÁGINA 118
MANUAL DE TRABAJO DEL CURSO ENARM CMN SIGLO XXI
11,8gr/dl, deshidrogenasa láctica de 971UI/l y albúmina 3,3gr/dl. La Rx de tórax presentaba opacidades bilaterales en parche con vidrio
deslustrado y neumomediastino, por lo cual, en el diagnóstico diferencial se incluyó inmunosupresión asociada a VIH y neumonía por P.
jirovecii (PJP). El análisis para VIH por ELISA fue POSITIVO, se confirmó por Western Blot. Se le realizó broncoscopia con biopsia
transbronquial y lavado broncoalveolar (LBA). El estudio histopatológico se reporta en la figura 1. Recibió tratamiento con
Trimetoprim/Sulfametoxasol y Prednisona en dosis de reducción por 21 días. Se somete a LBA cuyo estudio de patología confirma la
presencia de hemorragia alveolar reciente y activa (fig. 2B). Además, por rt-PCR se documenta infección por CMV

PREGUNTA
Considerando la complicación, cual es la medida terapeutica mas apropiada?

RESPUESTA
a.- Ganciclovir.
b.- Aciclovir.
c.- Cidofovir.
d.- Gamaglobulina.

MONONUCLEOSIS INFECCIOSA (VIRUS DE EPSTEIN-BARR):


CIENCIAS BÁSICAS: Enfermedad causada por el virus de Epstein-Barr (VEB), pertenece a la familia de los gammaherpesvirus (DNA
bicatenario), también llamada enfermedad del beso. El VEB se descubrió al observar en el microscopio biopsias de linfoma de Burkitt,
un hallazgo fortuito lo asocio a mononucleosis infecciosa. Se trasmite por secreciones de oro faringe y por contacto sexual. SALUD
PUBLICA: Distribución mundial, con mayor frecuencia en mujeres adolescentes y adultos jóvenes. La incidencia a nivel mundial se
aproxima al 100%. La enfermedad recurrente es una fuente de contagio y puede causar diseminación asintomática. PATOGENIA: El VEB
infecta las células epiteliales de la cavidad oral donde se multiplica en los linfocitos, causa lisis celular, pasa a la saliva y se disemina en
el tejido linfático, es mitogeno para las células B y permanece en ellas en forma latente. La inmunidad celular permanente participa en
el control de la infección. DIAGNOSTICO: Clínica; en niños generalmente es subclínica, a veces, faringitis, otitis, adenopatías cervicales.
Periodo de incubación de 10-14 dias, en adultos jóvenes fiebre, adenopatías en 2 cadenas ganglionares, exantema maculopapular,
faringitis, fatiga, anorexia, petequias en paladar y frecuentemente hepatoesplenomegalia. Serología, buscar aglutininas heterofilicas
(titulo >40) o identificación de cuerpos de inclusión intracelular. En infección aguda se observa IgM e IgG contra el antígeno de la casi
de viral (anti-VCA), además del anticuerpo contra el antígeno temprano (anti-EA). COMPLICACIONES: Puede originar trastornos
neurológicos como: meningoencefalitis, parálisis de Bell, síndrome de Guillan-Barre, mielitis transversa, neuritis periférica, anemia
hemolítica, CID. Se le a asociado carcinoma nasofaríngeo, linfoma de Burkitt y leucoplasia oral vellosa. TRATAMIENTO: No especifico,
solo sintomático, reposo para evitar rotura espléndida. Corticoides, si hay obstrucción de vía aérea.

CASO CLINICO
Varón de 19 años de edad, estudiante, tres semanas antes del ingreso había comenzado con fiebre, adenopatías laterocervicales
bilaterales, rash cutáneo y dolor faríngeo. Había sido seguido de forma ambulatoria, pero se decidió su ingreso ante la intensa
afectación del estado general. Presentaba leucocitosis (24.500cel/μl) con predominio de linfocitos (71%) que en el frotis tenían aspecto
atípico en un alto porcentaje, trombopenia ligera (106.000plaquetas/μl), alargamiento discreto de los tiempos de coagulación (INR 1,7)
y datos de colostasis (bilirrubina directa 3,1mg/dl). Tenía un título alto (>1/256) de IgM frente al antígeno de la cápside del VEB. El
estudio serológico de CMV, VHA, VHB y VIH era negativo. Una TAC de tórax y abdomen puso de manifiesto la presencia de
esplenomegalia, hepatomegalia, ascitis y derrame pleural bilateral. Se instauró tratamiento con metilprednisolona (1mg/kg/día) que se
mantuvo durante 30 días. Tras su inicio remitió la fiebre y se produjo una lenta, pero progresiva mejoría en un periodo de 35 días. El
paciente fue dado de alta a los 10 días, pero en el seguimiento posterior refería debilidad y dificultad para la concentración durante tres
meses más. Los parámetros hepáticos se normalizaron a los 6 meses del inicio de los síntomas.

PREGUNTA
La manifestación más frecuente de este padecimiento es una mononucleosis severa o fatal, cual de los siguientes cuadros mas
frecuente observar como diagnostico diferencial?

RESPUESTA
a.- Síndrome hemofagocítico asociado a virus.
b.- Trastornos linfoproliferativos.
c.- Disgammaglobulinemias (hipogammaglobulinemia).
d.- Anemia aplástica.

RABIA:
CIENCIAS BÁSICAS: Zoonosis causada por Rabdovirus, se trasmite al hombre principalmente por la saliva de animales infectados (y en
periodo de transmisión) a partir de una mordedura, rasguño o solución de continuidad en la piel o mucosas, se presenta como una
encéfalomielitis de curso agudo. PATOGENIA: La velocidad con se manifiesta la rabia, depende de la cepa del virus, de la concentración
de receptores para el virus en las células nerviosas del músculo esquelético, de la magnitud del inoculo, de la internación en el sitio de
entrada y de la proximidad de la lesión al SNC. Cuando la vía de entrada es transcutanea o epidérmica, el virus se queda un tiempo en
el sitio de inoculación. En ese lapso puede ocurrir una primera replicación en las células nerviosas de las placas musculares más
cercanas a la herida y con ello aumenta la carga viral, después el agente infeccioso se desplaza de 8-mm por día, avanzando por
continuidad al infectar nuevas neuronas del SNP. Por endocitosis o fusión de membranas. El camino que sigue el virus de SNP al SNC se
denomina diseminación centrípeta. Una vez en el SNC la infección es irreversible, conduciendo invariablemente a la muerte del
individuo. La multiplicación del virus en el encéfalo inicia en el sistema límbico y luego al resto del cerebro, produciendo edema,

CURSO ENARM CMN SIGLO XXI TEL: 36246001 Pharmed Solutions Institute PÁGINA 119
MANUAL DE TRABAJO DEL CURSO ENARM CMN SIGLO XXI
congestión vascular, infiltración directa de linfocitos e hiperemia de las leptomeninges, después de haber completado su invasión al
cerebro comienza la diseminación centrifuga, en la cual el virus regresa a los órganos con alta intervención nerviosa, cuando llega a
glándulas salivales, se difunde por el nervio trigémino, lo que inicia la eliminación del virus a través de la saliva. En perros la eliminación
de virus por saliva es de 3-10 días antes de que se manifiesten los primeros signos clínicos, esto es importante ya que el animal esta
asintomático, pero ya es infectivo. CURSO CLÍNICO: Periodo de incubación: Va desde la exposición hasta inicio de signos clínicos,
depende de la cantidad de virus inoculados y sitio anatómico (cabeza 30-48 días, mano 40-59 días, pierna 38-72 días). Las primeras
manifestaciones se presentan de 1-3 meses después de la agresión. Prodromo y primeros síntomas: Dura de 1-20 días, síntomas
neurológicos como ansiedad y agitación que son precedidos por malestar general, parestesias en el sitio de la lesión, fiebre, anorexia,
vomito, dolor de cabeza, letargo y fiebre. Periodo neurológico agudo: Signos de daño a SNC, se puede presentar como rabia paralítica o
furiosa. La paralítica es más común en ganado bovino y roedores, la furiosa en mamíferos carnívoros que son reservorios de la
enfermedad. Los signos y síntomas para ambas son: fiebre, rigidez de la nuca, parestesias, fascículacion muscular, convulsiones
generalizadas y focalizadas, hiperventilación e hipersalivacion, la presencia de hidrofobia que es patognomónico de rabia, dura de 1-5
min y se espacia por periodos de lucidez, es producto de un reflejo exagerado para proteger el tracto respiratorio, más que como
espasmo de faringe y laringe. En esta fase evoluciona hasta la afección del SNC, que incluye hiperactividad, desorientación, delirio,
alucinaciones, convulsiones, rigidez de nuca y progresan hasta que se presenta parálisis irregular de músculos respiratorios lo que
conduce a estado de coma. Durante el periodo de transición entre la fase neurológica y el coma se ostentan lapsos de lucidez y locura,
respiración rápida e irregular, ya posteriormente parálisis generalizada particularmente en la zonas inervadas por los nervios craneales
y en el músculo cardiaco, se paralizan los músculos respiratorios y sobreviene la muerte. DIAGNOSTICO: Se pueden hacer muestras
postmortem (biopsia 10-20g de cerebro, 1-3 g de hipocampo o 5-10g de medula espinal) en animales y seres humanos y antemortem
(biopsia de cuero cabelludo, impronta de cornea, saliva suero y liquido cefalorraquiedeo), en seres humanos, tomar muestras de 7-14
días de iniciados los signos neurológicos. La técnica de rutina para diagnostico de rabia es inmunofluorescencia directa, otras técnicas
auxiliares son determinación en LCR, ELISA. TRATAMIENTO: Observar al perro o gato 10 días posteriores a la agresión. La atención
inmediata de la lesión es; lavar con agua y jabón de forma abundante, desinfectar con agua oxigenada o tintura de yodo, si requiere
sutura aplicar primero inmunoglobulina antirrábica humana y se aproximan los bordes, valorar aplicación de antibióticos y toxoide
tetánico en heridas contaminadas o punzantes, secar con gasa y cubrir. Si animal agresor no se localiza iniciar tratamiento antirrábico:
A) Exposición leve; una dosis de vacuna en los días 0, 3, 7, 14, 28, en la región deltoidea por vía IM. B) Exposición grave; aplicar
inmunoglobulina antirrábica humana lo más cerca al día de la agresión (20 UI/kg), la mitad alrededor de la herida y el resto IM. También
debemos aplicar la vacuna antirrábica humana, si el animal sigue sano durante 5 dias posteriores se suspende la vacuna, de lo cobtrario
prodeguir. PREVENCIÓN: El control en las poblaciones y la variación parenteral y oral de reservorios silvestres y domésticos es ahora el
mejor tipo de prevención.

CASO CLINICO
Niña de 12 años ingresa al Hospital General O´Horán de la Ciudad de Mérida, referida de la localidad de Tekax, Yucatán, por una historia
de dolor en miembro torácico derecho por aparente traumatismo que no había cedido al tratamiento médico, sin más antecedentes de
importancia. La afección continuó con limitación funcional de la muñeca y disminución de la fuerza de dicha extremidad superior, la
cual fue progresiva, hasta acompañarse de dolor torácico con limitación de la respiración, fiebre, odinofagia, presentando aerofobia e
hidrofobia y cambios en la conducta que se hicieron mas evidentes, por lo que se sospechó de una encefalitis por virus rábico. La niña
continúo con deterioro general que hizo necesaria la asistencia ventilatoria, con evolución tórpida y datos clínicos de muerte cerebral
declarada 12 dias después.

PREGUNTA
Cual es la conducta a seguir?

RESPUESTA
a.- Profilaxia con inmunoglobulina a contactos.
b.- Vacunacion antirrábica canina en cerco sanitario.
c.- Cerco epidemologico con vacunación antirrábica a contactos.
d- Busqueda de perros sospechos para rabia.

PREGUNTA
Cual de los siguiente vectores no puede transmitir el virus?

RESPUESTA
a.- Canes.
b.- Felinos.
c.- Murcielagos.
d.- Roedores.

CASO CLINICO
Se trata de masculino de 25 años de edad dedicado al campo, refiere que al estar trabajando fue mordido por perro desconocido, el
cual no pudo capturar, se conoce de casos de rabia en la zona en animales no domesticados, acude a consulta una hora después del
hecho.

PREGUNTA
Cuál es la conducta más apropiada a seguir en este caso.

CURSO ENARM CMN SIGLO XXI TEL: 36246001 Pharmed Solutions Institute PÁGINA 120
MANUAL DE TRABAJO DEL CURSO ENARM CMN SIGLO XXI
RESPUESTA
a.- Lavar la herida y vigilancia.
b.- Lavar la herida y administración de toxoide.
c.- Lavar la herida, administración de toxoide y aplicar globulina inmunitaria humana.
d.- Lavar la herida, administrar toxoide, aplicar globulina inmunitaria humana y vacuna de células diploides humanas.

ENFERMEDAD DEL LEGIONARIO (LEGIONELLA PNEUMOPHILA):


CIENCIAS BÁSICAS: Llamada así porque afecto a los asistentes de una reunión de la Legión Americana, celebrada en un hotel de
Filadelfia (1976). Legionella bacteria gran negativa, de forma bacilar crece en aguas (entre 20- 50) naturales, lagos, ríos, arroyos, lodos.
Supervivencia en aire es corta SALUD PÚBLICA: Legionella es causa frecuente de neumonía comunitaria y nosocomial, con una
mortalidad que oscila entre el 3-50%. CLASIFICACIÓN: Fiebre de pontiac; El 95%, la forma más leve y la más frecuente se parece a una
"gripe", no produce neumonía es autolimitada, con buena evolución. Enfermedad del Legionario; De 1-5%, es la forma más grave,
provoca neumonía y afecta al estado general. La mayoría de casos se dan en forma esporádica. PATOGENIA: Factores de riesgo: uso
previo de corticoides, e inmunosupresores, neoplasias, trasplante, diálisis, sexo masculino, mayores de 50 años, tabaquismo, EPOC, vía
de contagio parece ser a través de aerosoles de gotas de agua contaminada que son aspirados o inhalados. Estas gotas preceden de
reservorios de agua infectados por legionella (aparatos de refrigeración, condensados de agua, duchas, sistemas de canalización y
distribución de agua que sirve de reservorio en hospitales, edificios). Sospechar en un contexto epidemiológico adecuados datos
clínicos, un habito tabáquico importante, la ausencia de supera a antibióticos betalactamicos y la observación de escasos
microorganismos en un esputo llenos de neutrofilos son característicos. Solicitar antígeno de legionella en orina. DIAGNOSTICO:
Clínico: Inicio brusco con fiebre alta de predominio matutino, con afección general, tos no productiva, esputos, escalofríos, cefalea,
presencia de manifestaciones extrapulmonares (diarreas o confusión mental), hiponatremía y elevación de la creatincinasa. La
neumonía por legionella se incluye dentro de las neumonías de presentación graves. TRATAMIENTO: De elección clásico es la
eritromicina (efectos secundarios; flebitis, alteraciones gastrointestinales, ototoxicidad), por ello se puede utilizar claritromicina (500mg
c/12hrs 10-14 días). También útil azitromicina (500mg/día). La experiencia con fluoroquinolonas como levofloxacino mas en caos
graves o neumonía intrahospitalaria (500mg/día en los primeros días c/12 hrs e intravenoso) y ofloxacino son positivas.

CASO CLINICO
Varón de 46 años, fumador de 40 paquetes al año y cirugía de colesteatoma con parálisis facial periférica derecha residual. Una semana
antes de la clínica refiere estancia en un hotel del Levante. Acude al médico de guardia del centro de salud por fiebre de más de 39º C,
sin otra sintomatología acompañante y con exploración física normal. Se le pauta tratamiento sintomático con paracetamol. Al día
siguiente acude a su médico de cabecera por persistencia de la fiebre, no existiendo variaciones en la exploración. Se decide continuar
con el tratamiento prescrito el día anterior y observación domiciliaria. A los cinco días tras persistir la fiebre, y aparecer disnea y tos con
expectoración hemoptoica, el paciente es remitido al Servicio de Urgencias hospitalario, donde se le practican diversas pruebas
complementarias: hemograma (16.430 leucocitos con 93% neutrófilos), bioquímica (Na 134, PCR 340), GAB (pH 7.46, PO2 60, PCO2 30),
Rx de tórax (infiltrado alveolar bilateral con derrame pleural derecho).

PREGUNTA
Considerando el agente infeccioso presuntivo, cual es la conducta diagnostica mas apropiada?

RESPUESTA
a.- Realizar Hemocultivo.
b.- Baciloscopia de Esputo.
c.- Antigeno en Orina.
d.- Inmunoglobulinas.

CASO CLINICO
Se trata de paciente masculino de 57 años de edad el cual acude al servicio de urgencias debido a tos intensa no productiva y fiebre, el
paciente cuenta con los siguientes antecedentes, originario de guanajuato, empleado de la construcción, alcoholismo y tabaquismo
positivo desde hace 40 años, actualmente bajo tratamiento por EPOC y cirrosis hepática, se realizan estudio de laboratorio y gabinete
de rutina donde se observa un reporte con disminución de la respuesta inmunológica esperada, la radiografia de torax presenta un
patrón característico, sin embargo el cuadro respiratorio bajo se ha intensificado por lo que se traslada a terapia intensiva.

PREGUNTA
Cual es tratamiento antibiótico más apropiado para este caso.

RESPUESTA
a.- Azitromicina.
b.- Levofloxacina.
c.- Trimetoprim/sulfametoxazol.
d.- Penicilina.

RICKETTSIASIS (R. RICKETTSIA; R. CORONII):


CIENCIAS BÁSICAS: Son bacterias estrictamente intracelulares gramm negativas, que se transmiten a través de vectores artrópodos
(garrapatas, pulgas y piojos) a diferentes mamíferos que actúan como reservorios. SALUD PÚBLICA: Sus características epidemiológicas,
su alta infectividad y elevada mortalidad de algunas, pueden hacer que se conviertan en armas biológicas ideales. CLASIFICACIÓN:
Existen 2 tipos de vasculitis que en función de ellas podemos diferenciar en 2 grupos: grupo de las fiebres manchadas; afectan

CURSO ENARM CMN SIGLO XXI TEL: 36246001 Pharmed Solutions Institute PÁGINA 121
MANUAL DE TRABAJO DEL CURSO ENARM CMN SIGLO XXI
endotelio, capa intima y media, entre ellas Fiebre manchada de Montañas Rocosas (en América, transmitida por garrapatas causada
por R. rickettsia), Fiebre Botónosa mediterránea (causada por R. coronii, transmitida por garrapata de perro). El grupo de las fiebres
tíficas; afectan endotelio y capa intima entre ellas Tifus o exantema epidémico (causado por R. prowazekii, transmitido por piojos)y
tifus murino (causado por R. typhi, transmitido por pulgas). PATOGENIA: La rickettsia penetra en el organismo, tras la picadura, o
atraves de una solución de continuidad contaminada con heces de los artrópodos (ingresan a través del piquete a la hora des rascado),
o bien a través de la conjuntiva o vías respiratorias, rápidamente alcanza torrente sanguíneo, allí se une a la célula endotelial y se
producen cambios en la conformación de la membrana que facilitan la fagocitosis de la bacteria, esta rompe la membrana fagosomica y
escapa libre a citosol aquí se produce la multiplicación (intracelular)a través de un mecanismo de expulsión activo, se produce
inoculación en células contiguas al área infectada, también hay diseminación linfática. Y este ciclo, ocasiona la aparición de múltiples
focos de vasculitis y un estado procoagulante endovascular erupción, hemorragias, trombosis y a veces gangrena). La destrucción
celular y liberación de rickettsias explican la fiebre, las lesiones capilares son la base del colapso vascular y alteración del estado de
conciencia. El aumento de permeabilidad capilar, produce extravasacion de líquidos, y proteínas, dando edema, y agravamiento del
estado de choque. DIAGNOSTICO: Clínico; fase de pródromo; malestar general, cefalea y febrícula. Fase de estado; fiebre elevada
sostenida de 2-3 semanas, cefalea intensa, vértigos ataque al estado general con gran postración, piel seca, estado de conciencia
alterado con alternancia de estupor y delirio, tos seca, nausea, vomito. Fase de exantema; máculas eritematosas en axilas y flancos que
se extienden al tronco y finalmente a extremidades. En la eruptiva, es común que exista oliguría, taquicardia desproporcionada a la
temperatura, polipnea. Complicaciones; gangrena por obstrucción. Serologico: IgM e IgG aparecen después de 1 semana a 10 días, y
duran de 3-4 meses. Existencia de antígenos con prueba de Weil-Félix. En México prueba de Ruiz-Castañeda. TRATAMIENTO: primera
elección tetera inclinas, clorhidrato de tetraciclina 40mg/kg dividida c/6 o 8 o 12 hrs VO o parenteral, doxiciclina 2-4mg/kg/día (dosis
máxima 200mg/ dia en dos dosis), mantenerse 3 días después de remisión térmica. Alternativa cloranfenicol, el tratamiento debe
continuar hasta 24 hrs después de haber desaparecido la fiebre. El estado de choque, hiponatremía, edema y coagulopatia de
consumo; infusiones de albúmina, heparina (la sangre total y plasma contraindicados por CID). Los medicamentos solo detienen el
desarrollo, de las rickettsias sin destruirlas, no son raras las recaídas.

CASO CLINICO
Paciente masculino de 8 años de edad, que inició 5 días previos con presencia de fiebre intensa de hasta 40ºC de difícil control, dolor
faríngeo y tos productiva, astenia y adinamia, llevado con médico 4 días previos quien recomienda trimetoprim/sulfametoxazol a dosis
de 7mg/kg/día (8 dosis), ambroxol y clorfenamina con paracetamol para tratamiento decuadro respiratorio. El paciente no muestra
mejoría clínicamente, e incluso aparecen múltiples zonas de erupción en lapiel con eritema y petequias, pruriginosas, principalmente
encara y que se han ido extendiendo a tronco y extremidades.Con patrón tipo centrípeto. Al no encontrar mejoría es traído a revisión
médica a la consulta externa de con diagnóstico probable de enfermedad de Kawasaki. El cuadro clínico inició posterior a una visita de
establo y en contacto con ganado bovino y ovino, se desconoce sí tenían vectores ya que el paciente luce descuidado pues no vive con
los padres. Tiene hábitos de higiene regulares. Baño cada tercer día pero en vacaciones hasta cada semana. No muestra cartilla,
desconoce cuáles vacunas le faltan. Tuvo varicela a los 4 años. El paciente se observó hipotrófico, consciente y cooperador, con buena
coloración de piel y lapresencia de una erupción maculo papular eritematosa, descamativa, petequial o micromacular en parpados, así
como otras regiones de cara, pecho, dorso, abdomen y extremidades, respetando palmas y plantas de pies. Kerning y
Brudsinskinegativos y sin otros datos de meningismo; cuello simétricocon un ganglio palpable en región media, móvil no doloroso
menor a 1cm. Hemorragia en flama conjuntival bilateral, sin secreción conjuntival ni prurito, mucosa nasal conrinorrea hialina, mucosa
oral bien hidratada y faringe hiperémica, sin exudados. Tórax sin dificultad respiratoria, con estertores gruesos bilaterales transmitidos,
sin problemas a la ventilación. Precordio rítmico sin soplos y pulsos periféricos normales. Abdomen blando, levemente distendido, con
peristalsis normal, sin hepatomegalia ni esplenomegalia.

PREGUNTA
Cual es el agente etiológico mas probable?

RESPUESTA
a.- Rickettsia prowazekii
b.- Rickettsia typhi
c.- Rickettsia rickettsii
d.- Rickettsia coronii

CASO CLINICO
Se trata de masculino de 19 años de edad ingresado al servicio de urgencias presentando fiebre, cefalea intensa, dolos abdominal
difuso pero ligero, pero con mialgias intensas, al examen físico se observa con 38,5 c, FC 112, TA 120/70 su estado general es adecuado
en crecimiento y desarrollo con diaforesis, se mantiene alerta y orientado, se realizan estudios de laboratorio y gabinete de rutina
donde se observa únicamente plaquetas con 84,000, se realiza puncion lumbar donde se observa auscencia de eritrocitos, proteínas y
glucosa normales, solo se observa 5 monocitos. Como antecedente de importancia refiere regresar de un campamento y realizar
escalado de montañas, actualmente continua en el servicio de medicina interna.

PREGUNTA
Considerando los hallazgos asi como los antecedentes usted integra un diagnostico, y debe iniciar tratamiento, cual es el más
adecuado:

RESPUESTA
a.- Rifampicina.
b.- Doxiciclina.

CURSO ENARM CMN SIGLO XXI TEL: 36246001 Pharmed Solutions Institute PÁGINA 122
MANUAL DE TRABAJO DEL CURSO ENARM CMN SIGLO XXI
c.- Vancomicina.
d.- Ceftriaxona.

LISTERIOSIS (LISTERIA MONOCYTOGENES):


CIENCIAS BÁSICAS: Bacteria Gramm positiva anaerobia, facultativa intracelular, puede causar infecciones invasoras muy graves.
Principales reservorios son el suelo, forrajes el agua, los silos y tracto gastrointestinal en aves, peces y mamíferos incluyendo el
hombre. Suele considerarsele un patógeno oportunista. SALUD PUBLICA: Distribución universal, relativamente resistente a la
refrigeración, la sequedad y el calor extremo. Afecta a inmunodeprimidos, pacientes con enfermedades crónicas debilitantes mayores
de 60 años, cirróticos, embarazadas en (en la Ciudad de México 1 por 4000) y neonatos (1 por 1500). Continúa causando una elevada
morbilidad y una preocupante mortalidad. PATOGENIA: la infección se adquiere mediante el consumo de Alimentos contaminados,
carnes, pescado y vegetales, crudos y lácteos no pasteurizados y productos refrigerados, los recién nacidos la adquieren a través de la
placenta o del canal de parto infectado. Puede sobrevivir en el entorno gástrico, colonizar el intestino y cruzar la barrera
hematoencefalica y maternofetal, con sus proteínas de superficie internalinas, se favorece su invasión celular por un mecanismo tipo
zipper, en que la bacteria se hunde progresivamente en la superficie celular, después estas se lizan y escapan, la proteína
polímerizadora de activa, le permite azucares en el citosol celular durante su replicación intracelular. DIAGNOSTICO: Clínica; La más
habitual es la listeriosis gastrointestinal; importante puede causar brotes de toxiinfeccion alimentaria, puede ser asintomática o sufren
deposiciones acuosas, nauseas, vómitos, cefalea, artromialgias y fiebre, síntomas que se limitan a 2 días, salvo que estén
inmunocomprometidos. Listeriosis invasora; enfermedad gestacional y neonatal, frecuentemente fiebre sin foco aparente, la gravedad
de Listeriosis materna radica en el tercer trimestre, ya que puede haber aborto, muerte fetal intrautero, prematuro dad, sepsis, muerte
neonatal. Es fundamental hacer hemocultivos a toda mujer embarazada con fiebre, aunque semiología parezca banal. Los neonatos
pueden sufrir sepsis, microabscesos y granulomas diseminados (granulomatosis infantoseptica), infección respiratoria o meningitis, que
conlleva una mortalidad y morbilidad importantes como la hidrocefalia persistente en el caso de meningitis. Infección del SNC; pueden
tener meningitis, meningoencefalitis o cerebritis. Endocarditis es una rara y grave complicación de la bacteriemia (forma clínica mas
frecuente en inmunodeprimidos, sin foco identificable, ocurre con fiebre, deteriroro rapido y a menudo fulminante) por listeria. El
diagnostico definitivo es aislamiento en sangre, LCR, liquido articular, placenta. TRATAMIENTO: La mayoría de los antibióticos
incluyendo penicilinas son bacteriostáticos contra la listeria, los aminoglucosidos, glucopeptidos y cotrimoxaxol son bactericidas.
Neonatos y formas graves; cotrimoxaxol 20mg/kg/día, en 4 dosis o gentamicina 5-7mg/kg/día en dosis única por 3 días. Infección del
SNC ampicilina 400mg/kg/ día en 6 dosis, asociada a gentamicina (si no hay daño renal) o ampicilina y cotrimoxaxol (mas eficacia,
menos nefrotoxicidad). En embarazadas ampicilina mejor opción.

CASO CLINICO
Femenino de 33 años con fiebre de una semana de evoluacion, de tipo intermitente, mialgias, artralgia y escalofríos, acude a consulta a
primer nivel donde no se integra proceso infeccioso, la paciente se encuentra embarazada con 21 SDG por FUM, a la exploración se
observa con mucosas orales deshidratadas, durante su ingreso presenta nausea, cefalea y diarrea, al tercer dia se observa con
somnolencia con inversión del ciclo vigilia sueño, posteriormente refiere sensación de salida de liquido transvaginal, al tacto no se
observa elementos ovulo placentarios.

PREGUNTA
Cual es la conducta terapéutica mas adecuada sobre el agente casusal del caso?

RESPUESTA
a.- Ampicilina.
b.- Gentamicina.
c.- Cotrimoxaxol.
d.- Ceftriaxona.

PSEUDOMONA AERUGINOSA (PAE):


CIENCIAS BÁSICAS: Es una bacteria bacilo Gramm negativo, aerobio estricto, dispersa en el ambiente, emergente relevancia como
patógeno oportunista causante de infecciones en pacientes hospitalizados (equipos de ventilación mecánica, soluciones de limpieza,
instrumental, medicamentos), críticos y/o inmunodeprimidos (quemados, ventilados, fibrosis quística, EPOC, pos quirúrgicos).m La PAE
causa infecciones en la mayoría de las partes del cuerpo, se adapta rápidamente al tracto respiratorio y es el lugar más frecuente de
infección por PAE (principal causante de la neumonía asociada a la ventilación mecánica NAR). SALUD PUBLICA: Diseminada
prácticamente en toda la geografía mundial, es cosmopolita. Tasa de mortalidad atribuible a pseudomoma es de 34%. PATOGENIA:
Factores de riesgo; inmunodeprimidos. La fuente de origen puede ser hematogena o como puerta de entrada en un sitio de punción o a
través de vías centrales o catéteres de diálisis que pueda tener el paciente al parecer la lesión inicial provocada por la P. aeruginosa al
epitelio respiratorio y otras mucosas esta mediada por pili o fimbrias y por un exapolisacorido mucoide conocido como alginato. Existen
receptores de estas adhesinas en las células epiteliales. El microorganismo produce diversas enzimas extra celulares como la proteasa
alcalina, elastasa, fosfolipasa, citotoxina y exoenzimas A y S, la alteración de los tejidos del huésped por estos productos bacterianos,
crea las condiciones necesarias para la proliferación e invasión bacteriana y la consiguiente destrucción del tejido. Infecciones óseas
(fracturas, cirugías traumatológicas) y articulares (osteomielitis vertebral), por bacteriemia, inoculación directa dentro del hueso y por
diseminación contigua desde otro sitio de infección. Infección en SNC (meningitis, infecciones subdurales o extradurales) secundarias a
cirugía y a trauma de cráneo o bacteriemias. Infección urinaria secundaria a cuerpo extraño (calculo, atenta, sonda) o a estenosis. Una
situación importante es la infección en tejidos quemados. DIAGNOSTICO: El cuadro clínico es el de un paciente séptico, el único punto
diferente con otros tipos de sepsis por gramm negativos es la presencia de lesiones cutáneas llamadas ectima gangrenoso, que se ve
mas frecuente en pacientes neutropenicos. Lesión maculopapular, pequeña, roja, dolorosa, mal circunscrita, que comienza de color
rosa, se oscurece hasta volverse púrpura y finalmente negra y necrótica. Debe sospecharse neumonía nosocomial en un paciente con

CURSO ENARM CMN SIGLO XXI TEL: 36246001 Pharmed Solutions Institute PÁGINA 123
MANUAL DE TRABAJO DEL CURSO ENARM CMN SIGLO XXI
uno infiltrado radiológico nuevo o progresivo asociado a datos clínicos sugestivos de infección; fiebre, esputo purulento o leucocitosis,
tras diagnostico sindromatico, intentantar confirmación microbiología. . La característica más importante en laboratorio es la
producción de pigmento llamado piocianina, que le da un color azulado. Obtención de material respiratorio (BAL, aspirado traqueal,
minibar) TRATAMIENTO: Las pseudomonas son productoras de enzimas MBL (metalo-b-lactamasas) capaces de resistir prácticamente a
todo tipo de antibiótico, por ello su actividad es devastadora. Basándose en esquema para neumonías nosocomiales A) inicio reciente y
sin factores de riesgo, cualquier gravedad: ceftriaxona, fluroquinolonas, ampicilina- sulbactam o ertapenem B) inicio tardío o factores
de riesgo, cualquier gravedad: beta-lactamico antipseudomonico (carbenicilina, ticarcilina, mezlocilina, piperacilina) mas
fluoroquinilona o aminoglucosidos mas linezonid o vancomicina. Antibióticos mas utilizados céfepime (1-2 g cada 8 hrs IV), ceftazidime (
2g c/8hrs IV), imipenem (500mgs c/6 o 1g c/8hrs IV), meropenem (1g c/8hrs), gentamicina (7mg/kg/día IV), amikacina (20 mg/kg/día
IV), levofloxacino (750mg/día IV), colistin 100mg c/8hrs, piperacilina/tazobactam (4.5g c/6hrs IV). No hay un esquema especifico lo que
realmente cambia la mortalidad es el tratamiento empírico inicial adecuado.

CASO CLINICO
Un varón de 27 años, diabético, fue internado por fiebre, compromiso de conciencia y rigidez de nuca aparecidos tras tres días de
cursar con una infección respiratoria aguda. Se le diagnosticó una meningoencefalitis aséptica (LCR de aspecto claro, con presión de 12
cm H2O; leucocitos: 200/mm3; tinción de Gram directa y cultivo bacteriano: negativos) y recibió ceftriaxona más vancomicina y
corticoterapia (dexametasona, 10 mg iv cada 6-horas durante 4 días). Inicialmente ingresó en una UCI y estuvo en ventilación mecánica,
siendo desconectado a los 4 días. Desarrolló fiebre y tos productiva tras una semana de evolución. Una Rx de tórax detectó un infiltrado
pulmonar bilateral. Tres días más tarde, una segunda Rx reveló la presencia de múltiples abscesos pulmonares con niveles hidro-aéreos.

PREGUNTA
Cual es la conducta diagnostica mas adecuada?

RESPUESTA
a.- TAC de torax.
b.- Toracostomia.
c.- Cultivo de esputo.
d.- Hemocultivo.

PREGUNTA
Se obtiene cultivo positivo para P. aeruginosa, cual es la conducta terapéutica asociada mas apropia?

RESPUESTA
a.- Linezolid.
b.- Meropenem.
c.- Amikacina.
d.- Ciprofloxacina.

LEPTOSPIROSIS (LEPTOSPIRA INTERROGANS):


CIENCIAS BÁSICAS: Se considera la zoonosis más frecuente, la leptospira interrogans, es la única espiroqueta que infecta al hombre,
gramm negativo, aerobio y móvil. Frecuente en campesinos, granjeros, taladores, cazadores, trabajadores de cultivo de arroz y
militares, actualmente también en niños (natación cabotaje, ciclismo, caza deportiva, animales domésticos infectados). PATOGENIA:
Las leptospiras en agua o tierra contaminada por orina de animales infectados, penetran a la piel a través de cortaduras o abrasiones,
membranas, mucosas y conjuntivas, se disemina a través de la sangre a todos los órganos y sistemas incluyendo el SNC. Los fenómenos
hemorrágicos son secundarios a vasculitis severa con daño endotelial. Los riñones presentan nefritis intersticiales, necrosis tubular y
permeabilidad capilar anormal que acompaña a hipovolemia y puede llegar a IRC. La ictericia es secundaria al necrosis centrolobulillar y
proliferación de células de Kupffer, con disfunción hepatocelular. Miocarditis y uveítis ocurren por invasión tisular. DIAGNOSTICO:
Clínica; leptospirosis anicterica (85-90%), incubación de 7-12 días, inicia fiebre, cefalea, mialgias calosfríos severos y mal estado general,
linfadenopatias hepatoesplenomegalia. Durante este periodo pueden aislaras las leptospiras en sangre o LCR. Entran en perdidos
asintomático de 1-3 días e inician ahora la llamada fase inmune o desarrollo de anticuerpos IgM específicos, puede reaparecer la fiebre,
cefalea y vómitos y se puede desarrollar meningitis aséptica, existe aumento de células en el liquido cefalorraquideo, primero
predominando los PMN y después los mononucleraes, las proteínas están elevadas, glucosa normal. Puede o no haber irritación
meninges, esta fase dura de 3-40 días, y ya no se pueden aislar las leptospiras de sangre y/LCR, pero si en orina. Leptospirosis ictericia
(sx. de Weill); forma grave y de mayor mortalidad, los pacientes pueden desarrollar ictericia pero no daño renal, lo usual es que haya
ictericia y elevación de urea y creatinina, así como elevación de transaminasas, proteinuria, hematuria, cilindruria, puede haber anemia,
trombocitopenia, leucocitosis, hemorragia, Asi como neumonías hemorrágica. Laboratorio observación de leptospiras es el estándar de
oro para el diagnostico, la búsqueda en orina es la más confiable. Las pruebas en suero para detección de anticuerpos de la fase aguda
(1-2 semanas) y su elevación en 4 tantos durante la fase convaleciente (2semanas después), incluyen la prueba de aglutinación al
microscopio, confirman diagnostico serológico. Una sola determinación de títulos de 1:800 o mayores sugieren fuertemente la
enfermedad. Analizar LCR en pacientes con meningitis. La radiografía de tórax puede mostrar cardiomegalia, edema agudo pulmonar
asociado a miocarditis o infiltrados en parches asociados a hemorragia alveolar por capilaritis. TRATAMIENTO: La penicilina G sódica
(20-24 millones U/día c/4-6hrs), doxiciclina (100mg c/12hrs IV) o eritromicina (500mg c/6hrs IV)¡ Son eficaces para detener la infección
y el daño orgánico. La doxiciclina se usa para prevención en dosis semanal de 200mg.

CURSO ENARM CMN SIGLO XXI TEL: 36246001 Pharmed Solutions Institute PÁGINA 124
MANUAL DE TRABAJO DEL CURSO ENARM CMN SIGLO XXI
CASO CLINICO
Femenino de 29 años la cual labora como asistente de veterinario, refiere que hace 10 dias inicio con fiebre, escalofríos, cefalea,
nauseas, dolor muscular, se automedica con fármacos sintomáticos, mejorando su estado general sin embargo regresan los síntomas y
agregandoce ictericia, a la exploración clínica se presenta 38.9 C temperatura, FC 1010, TA 140/90 mmHg, la saturación por oximetro 92
%, a la percusión se presenta dolor hepático, se persibe leve crecimiento, sin esplenomegalia, los estudios de laboratorio BUN de 64,
creatinina 3.6, bilirrubina de 64, AST 86, ALT 103, fosfatasa alcalina de 390, leucocitos 11 000, 13% de bandas y 80 %
polimorfonucleares, hematocrito 33% y plaquetas de 145, puncion lumbar revela pleocitosis, TAC de torax muestra inflitrados en flama
difusos compatibles con hemorragia pulmonar.

PREGUNTA
Cual es diagnostico más probable?

RESPUESTA
a.- Neumonitis intersticial aguda.
b.- Leucemia mieloide aguda.
c.- Fiebre por Streptobacillus moniliformis.
d.- Infeccion por Leptospira interrogans.

ACTINOMICOSIS (ACTINOMYCES ISRAELII):


CIENCIAS BASICAS: Es causada por bacterias anaeróbicas o microaerófilos, gramm positivos principalmente del género Actinomyces
(por ejemplo, A. israelii), que está caracterizada por formación de absceso, fibrosis tisular, las regiones más afectadas son la
cervicofacial, torácica y pélvico-abdominal. El germen tiene una baja virulencia y produce enfermedad solo cuando la barrera mucosa
normal se altera, produciendo formación de abscesos múltiples, fistulización o una masa, requiriendo la presencia de otras bacterias,
que destruyan la vascularidad tisular convirtiendo el anaeróbica (generalmente infecciones polimicrobianas). Actinomicosis se asocia
con mala higiene dental, absceso dentario, uso de dispositivos anticonceptivos intrauterinos (DIU), y la inmunosupresión. SALUD
PÚBLICA: Su incidencia está disminuyendo, probablemente como resultado de una mejor higiene dental y un inicio más temprano del
tratamiento antibiótico. PATOGENIA: Los agentes de actinomicosis son miembros de la flora oral normal. La enfermedad se presenta
sólo después de la interrupción de la barrera de la mucosa. Infección local se extiende en forma contigua en una manera lenta y
progresiva, haciendo caso omiso de los planos del tejido. En crecimiento in vivo produce grumos llamados granos o gránulos de azufre.
Las paredes fibróticas de la masa a menudo se describen como "madera”. DIAGNOSTICO: La enfermedad oral - cervicofacial: La
infección comienza como una inflamación de los tejidos blandos, abscesos, o masa, a menudo en el ángulo de la mandíbula con
extensión contigua al cerebro, espina dorsal cervical, o en el tórax. El dolor, la fiebre, y leucocitosis son variables. Enfermedad torácica:
El parénquima pulmonar y / o en el espacio pleural suele participar. Produce dolor en el pecho, fiebre y pérdida de peso. Radiografía de
tórax muestra una lesión de masa o la neumonía. Enfermedad cavitaria o adenopatía hiliar pueden ocurrir, y > 50 % de los puntos
tienen engrosamiento pleural, derrame o empiema. Las lesiones cruzan fisuras o pleura y pueden implicar el mediastino, el hueso
contiguo, o la pared torácica. Enfermedad abdominal: El diagnóstico es difícil y no se puede hacer hasta meses después del evento
inicial (por ejemplo, diverticulitis, cirugía intestinal). La enfermedad generalmente se presenta como un absceso, masa o lesión fijado al
tejido subyacente y, a menudo se confunde con el cáncer. Fístulas en la pared abdominal, en la región perianal, u otros órganos pueden
desarrollar e imitar la enfermedad inflamatoria intestinal. Participación del tracto urogenital puede presentarse como pielonefritis o
absceso perirrenal. Actinomicosis pélvica se asocia a menudo con DIU. La presentación es indolente y puede seguir a la extracción del
dispositivo se presenta fiebre, pérdida de peso, dolor abdominal y sangrado vaginal anormal. Endometritis progresa a masas pélvicas o
abscesos tuboovárico. Actinomicosis pueden afectar el tejido músculo-esquelético, tejido blando, o el SNC y puede difundir vía
hematógena, más comúnmente a los pulmones y el hígado. Aspiraciones, biopsias, o la escisión quirúrgica puede ser necesaria para
obtener el material para el diagnóstico. La identificación microscópica de los gránulos de azufre en pus o tejidos establece el
diagnóstico. Necrosis central de las lesiones con los neutrófilos y los gránulos de azufre es prácticamente de diagnóstico de la
enfermedad. Por lo general requieren de 5-7 días, pero puede tardar 2-4 semanas para convertirse en positiva, incluso una sola dosis de
antibiótico puede afectar el rendimiento de los cultivos. TRATAMIENTO: Requiere un tratamiento prolongado. Tratamiento IV durante
2-6 semanas (por lo general con la penicilinas- amoxicilina, penicilina G, ampicilina) seguido de tratamiento oral durante 6-12 meses
(por ejemplo, con penicilina o ampicilina) se sugiere para la infección grave y enfermedad voluminosa. En particular la enfermedad de la
región oral y cervicofacial, se puede curar con un tratamiento más corto. En alérgicos eritromicina o azitromicina. Si el tratamiento se
prolonga más allá del punto de la resolución de la enfermedad medible (cuantificada mediante TAC o RM), se minimiza la recaída.
Agentes alternativos adecuados incluyen las tetraciclinas (por ejemplo, minociclina, 200 mg/día administrado por vía IV o PO cada 12
horas) o clindamicina (2,7 g /d administrado por vía IV cada 8 horas).

CASO CLINICO
Mujer de 53 años, con antecedentes de amigdalectomía en la infancia y sin manipulaciones dentales recientes. Consultó por sensación
de cuerpo extraño faríngeo derecho de 1 año de evolución, con expulsión por boca de material blanquecino al menos en tres ocasiones.
El otorrinolaringólogo extrajo un cúmulo de detritus del lecho amigdalar derecho, y apareció en la zona adyacente una pared de
aspecto inflamatorio. El análisis anatomopatológico de la muestra informó de masa (1,5×1,5×1cm). Durante los 6 meses posteriores de
seguimiento presentó tres episodios de expulsión con la tos, de masas de aproximadamente 1cm de iguales características. A la
exploración física destacaba la visualización repetida de un cúmulo de detritus blanco en una cavidad en el polo superior del lecho
amigdalar derecho, cavidad atribuible a una cicatrización anómala de la amigdalectomía previa. La higiene bucal era buena. La analítica
básica y la radiografía de tórax eran normales. Serología para el virus de la inmunodeficiencia humana, negativa. La tomografía
computarizada cervical identificó una prominencia inespecífica a nivel amigdalar derecho, sin objetivar trayectos en partes blandas ni
adenopatías

CURSO ENARM CMN SIGLO XXI TEL: 36246001 Pharmed Solutions Institute PÁGINA 125
MANUAL DE TRABAJO DEL CURSO ENARM CMN SIGLO XXI
PREGUNTA
Se realiza cultivo y es positivo para actinomyces israelii, cual es la conducta mas adecuada para minimizar recaidas?

RESPUESTA
a.- Amoxicilina.
b.- Clindamicina.
c.- Penicilina.
d.- Tetraciclina.

PREGUNTA
Cual de los siguientes factores de riesgo es mas frecuente para la presencia actinomicosis craneofacial?

a.- Otitis media.


b.- Sinusitis cronica.
c.- Absceso dentario.
d.- Amigdalas hipertróficas.

PREGUNTA
En el caso de la actinomicosis abdominopelviana, cual de los siguientes factores es el mas frecuente?

RESPUESTA
a.- Cuerpo extraño.
b.- Aborto séptico.
c.- EPI.
d.- LUI.

NOCARDIASIS (N. ASTERIODES, N. BRASILIENSIS):


CIENCIAS BASICAS: Nocardias son actinomicetos aeróbicos saprófitos comunes en el suelo. Varias especies están asociadas con la
enfermedad humana. N. asteroides es la especie más comúnmente asociada con la enfermedad invasiva. N. brasiliensis se asocia más
con lesiones cutáneas localizadas. SALUD PÚBLICA: En los Estados Unidos, 1.100 casos de infección por nocardia se producen cada año,
de los cuales 85 % son pulmonares o sistémicos. El riesgo de enfermedad es mayor de lo habitual entre las personas con deficiencia de
inmunidad mediada por células, por ejemplo, que asocia con el linfoma, el trasplante, la terapia con glucocorticoides, o SIDA. Las tasas
de mortalidad son altas entre los pacientes con nocardiosis del cerebro. PATOGENIA: La neumonía y la enfermedad diseminada siguen
a la inhalación de micelios. Nocardiosis causa abscesos con infiltración de neutrófilos y necrosis. DIAGNOSTICO: Clínico; Enfermedad
pulmonar suele ser subaguda, presentando en días o semanas. Enfermedad extrapulmonar se documenta en > 50 % de los casos, y un
poco de afectación pulmonar es evidente en el 80 % de los pacientes con enfermedad extrapulmonar. Una tos productiva importante
de pequeñas cantidades de esputo espeso purulenta, fiebre, anorexia, pérdida de peso y malestar general son comunes, disnea,
hemoptisis y dolor pleurítico son menos comunes. La radiografía de tórax; suele mostrar infiltrados nodulares únicos o múltiples de
diferentes tamaños que tienden a cavitación. Empiema se observó en un tercio de los casos. La infección puede diseminarse a los
tejidos adyacentes, como el pericardio o el mediastino. Enfermedad extrapulmonar normalmente se manifiesta como abscesos
subagudos en el cerebro, la piel, los riñones, los huesos y / o musculares. Algunos abscesos forman fístulas y pequeñas cantidades de
descarga de pus, pero no las de los pulmones o el cerebro. Los abscesos cerebrales son generalmente supratentorial, a menudo
multiloculada, pueden ser únicos o múltiples y tienden a enterrarse en los ventrículos o extender en el espacio subaracnoideo.
Enfermedad después de la inoculación transcutánea Celulitis: La celulitis subaguda puede presentar 1-3 semanas después de una lesión
en la piel (a menudo contaminada con tierra). Síndrome linfocutáneo: Una lesión piodérmica se desarrolla en el sitio de inoculación,
con ulceración central y secreción purulenta. Esta lesión suele drenada por nódulos SC a lo largo de los vasos linfáticos. Esta forma se
asemeja a la esporotricosis. Actinomicetoma: Son formas nodulares de hinchazón en el sitio de traumatismo local, por lo general en los
pies o las manos. Forma de fístulas y la descarga de drenaje seroso o purulento que puede contener gránulos que consisten en masas
de micelio. Las lesiones, que se extienden poco a poco a lo largo de los planos faciales involucrar a la piel, tejido subcutáneo y hueso,
pueden provocar graves deformidades. Infecciones oculares: queratitis por lo general sigue a un trauma ocular. La endoftalmitis puede
ocurrir después de la cirugía del ojo o en la enfermedad diseminada. Exámenes en los que se identifica a la bacteria; broncoscopia,
cultivo de esputo o pus (en agar enriquecido con extracto de levadura y carbón activado), se pueden ver filamentos gramm positivos,
biopsia de piel, cerebro, pulmón (también aspiración pulmonar percutánea, lavado broncoalveolar). TRATAMIENTO: Las sulfonamidas
son los fármacos de elección, Trimetropim (10-20mg/kg) y sulfametoxaxol (50-100mg/kg). Para enfermedad grave, los niveles séricos
de sulfonamidas deben ser monitoreadas y mantenidas a 100-150 mg /mL. Una vez que la enfermedad está controlada, la dosis de
trimetoprim-sulfametoxazol puede ser reducida por 50%. Las pruebas de sensibilidad pueden guiar tratamientos alternativos.En
enfermedad del SNC; minociclina (200-400mg). Sindrome de celulitis linfocutanea; linezolid (1200mg). Actinomicetoma amikacina (10-
15mg/kg), cefotaxima (6g), ceftriaxona (1-2g), imipenem (2g). La terapia para la nocardiosis debe continuar mientras el paciente
permanezca inmunodeprimido. Los abscesos cerebrales que son grandes o no responde a los antibióticos deben ser aspiradas. Las
recaídas son comunes. El paciente debe ser seguido por al menos 6 meses después de la terapia se haya completado.

CASO CLINICO
Mujer de 42 años, diestra, portadora de poliglobulia diagnosticada 15 años antes y catalogada como primaria luego de exhaustivos
estudios. Tratada con ácido acetil salicílico en las ocasiones que se le realizó flebotomías, tuvo mala tolerancia a las mismas. Un mes
previo fue sometida a un procedimiento de extracción e implante dentario, presentando 10 a 15 días después un cuadro de inicio y
progresión insidiosos, en el curso de 15 días, y caracterizado por cefalea, vómitos, fiebre, a lo que se agregaron posteriormente

CURSO ENARM CMN SIGLO XXI TEL: 36246001 Pharmed Solutions Institute PÁGINA 126
MANUAL DE TRABAJO DEL CURSO ENARM CMN SIGLO XXI
alteraciones de la agudeza visual y campimétricas y en las últimas cuarenta y ocho horas, depresión de la vigilia. En el examen físico, la
paciente se presentaba confusa, por momentos excitada, con guiño a la amenaza abolido a izquierda, paresia de VI par izquierdo,
parálisis facial central izquierda, paresia leve de miembros izquierdos, con hiporreflexia y signo de Babinski. En la paraclínica de
laboratorio el hematocrito fue de 6,79 %, hemoglobina 20,9 mg/dL, VCM 90,1, HBCM 30,7%, leucocitos 12,800/mm3, PMN 80%,
linfocitos 17%, eosinófilos 1%, basófilos 2%, plaquetas 150/mm3, VES 1 mm 1ª hora
PREGUNTA
Cual es la conducta diagnostica mas adecuada a seguir?

RESPUESTA
a.- Hemocultivo.
b.- IRM cráneo.
c.- TAC cráneo.
d.- Serologia

PREGUNTA
La RM con gadolinio evidencia proceso expansivo cortico-subcortical con anillo grueso irregular de realce con centro necrótico. La
orientación diagnóstica imagenológica RM orientó a un absceso cerebral, se realizo drenaje y cultivo, que resulto positvo a Norcardia
asteroides, cual es la conducta terapéutica mas adecuada?

RESPUESTA
a.- Linezolid (1200mg).
b.- Minociclina (200-400mg).
c.- Amikacina (10-15mg/kg)
d.- TMP/SMX (3.200/640mg)

INFECCIONES ESTREPTOCOCICAS:
CIENCIAS BÁSICAS: Son un genero de los más importantes como patógeno para el ser humano, son bacterias esférica, eróticos
facultativos, no forman esporas, catalana negativos e inmóviles. El Streptococcus pyogenes, es el agente etiológico mas frecuente de
faringoamigdalitis aguda, pudiendo ocasionar secuelas no supurativas, tales como la fiebre reumática y la glomerulonefritis post-
estreptocócicas. Otro el Streptococcus pneumoniae. De acuerdo con el tipo de hemolisis de eritrocitos que producen, en beta
(hemolisis completa), alfa (hemolisis parcial) y gamma (sin hemolisis). La primera la presentan prácticamente todos los S. pyogenes y la
segunda los S. pneumoniae y viridans. SALUD PUBLICA: S. pyogenes incidencia mayor en raza blanca, en adolescencia y sexo
masculino. La faringoamigdalitis más frecuentes en zonas templadas y frías y en invierno, mientras el impétigo y la piodermitis lo son en
los climas tropicales y en meses calurosos. PATOGENIA: La fuente de infección son las secreciones de nariz, garganta y piel de
enfermos, la transmisión se realiza persona a persona, mediante secreciones respiratorias al toser o estornudar o por transferencia
manual (piel con traumatismo previo). Incubación de 2-4 días, la fase aguda constituye el periodo de mayor contagiosidad. Ingresa a
vías respiratorias superiores y se disemina, tiene especial afinidad por el sistema linfático y el lugar de implantación es el tejido linfoide
de faringe. Se extiende por los linfáticos o por continuidad a otras áreas cercanas, causando complicaciones tales como adenitis
cervical, absceso periamigdalino o retrofaringeo, sinusitis, otitis. Para el desarrollo de escarlatina, la infección faríngea es por una cepa
productora de toxina eritrogenica, la cual es antigénica y estimula producción de anticuerpos protectores (por eso solo da un cuadro).
ESTREPTOCOCOS BETA HEMOLITICO DEL GRUPO A o Streptococcus pyogenes: Tiene una estructura celular compleja, su proteína M
parece ser su principal factor de virulencia, las cepas ricas en ellas son resistentes a la fagocitosis. Elabora productos extracelulares; la
toxina eritrogenica es la responsable del exantema en la escarlatina, su producción es inducida por un bacteriofago. La estreptomicina
O y la S ambas tóxicas para los eritrocitos y otras células, incluso miocardio. La elevación de niveles serios de anti estreptomicina O. Es
un buen indicador de infección estreptocócica reciente. Por evidencia seroepidemiologica, se considera que ciertos serotipos,
causantes de faringoamigdalitis, están más relacionados con fiebre reumática, los de piel no causan fiebre reumática pero ambos son
nefritogenicos (glomerulonefritis). DIAGNOSTICO: Clínica; faringoamigdalitis, forma tóxica con fiebre alta, nausea y vómitos, comienzo
agudo, odinofagia, cefalea, malestar, faringe hiperemica, exudado blanco-amarillento en parches, adenopatías anterior dolorosa.
Escarlatina; fiebre elevada, vomito, dolor faríngeo y cefalea, 12-48 hrs después aparece exantema y el enantema (lengua=fresa roja,
paladar, amígdalas y faringe) característico. El rash se inicia en el tronco y se generaliza con rapidez, se presenta con una erupción con
fondo eritematosa y a la vez puntico ene, con sección táctil de aspereza (papel de lija), respetando la cara. Siempre se presenta la
descamación en mayor o menor grado. Impétigo; vesícula superficial con poco eritema e indolora, progresiva pústula con costra gruesa,
con secreción meliserica, mas en extremidades y comisuras nasales y bucales. Erisipela; poco frecuente, celulitis caracterizada por una
lesión eritematosa, con engrosamiento de la piel, dolorosa y con aumento de temperatura, borde bien definido. Otras infecciones son;
piodermitis, ectima, celulitis o fscitis necrozante. El diagnostico de faringoamigdalitis se fundamenta en la triada amígdalas con
exudado, odinofagia y adenopatias cervical anterior dolorosa, en un paciente que NO presenta tos, o síntomas nasales. El dx., de
certeza aislamiento de S. pyogenes en cultivo de exudado faríngeo, aglutinación en látex y coaglutinacion, detectan antígeno. En la
escarlatina el rash, y se puede hacer cultivo y antiestreptolisinas. TRATAMIENTO: Debido a que S. pyogenes sigue siendo sensible a
penicilina, esta es el tx., de elección. Niños menores de 4 años; penicilina procainica 400,000UI c/24 hrs 2-3 días + p. Benzatinica
600,000UI, en una sola dosis, después de la procainica o penicilina benzatinica 600,000UI dosis única. Niños mayores de 4 años y
adultos; penicilina procainica 800, 000UI c/24hrs 2-3 días + P. Benzatinica 1,200,000UI dosis única después de la procainica o penicilina
benzatinica 1,200,000UI dosis única. El medicamento sustituto en alergia a penicilinas es eritromicina oral 30mg/kg/día, en dos o tres
dosis diarias durante 10 días (dosis máxima 2 gramos). En caso de miosotis y fascitis necrozante, se recomienda penicilina, clindamicina
y amikacina , hasta corroborar agente, si es pyogenes continuar solo penicilina y clindamicina.

CURSO ENARM CMN SIGLO XXI TEL: 36246001 Pharmed Solutions Institute PÁGINA 127
MANUAL DE TRABAJO DEL CURSO ENARM CMN SIGLO XXI
CASO CLINICO
Varón con 43 años de edad, diabético sin control, que 24 horas antes presentó una contusión accidental contra el respaldo de la cama
en la zona periorbitaria derecha, por lo que acudió a consultorio, donde se le realizó una curación y prescribió tratamiento con anti-
inflamatorios (diclofenaco sódico). En las horas siguientes cursó con fiebre, compromiso de conciencia y vómitos. Al examen físico se
describió un paciente de aspecto tóxico, con los siguientes signos vitales: frecuencia cardíaca: 130/min, frecuencia respiratoria: 49/min,
presión arterial: 77/55 mm Hg, T° rectal: 37,8° C, escala de Glasgow neurológico: 14. Destacaba un gran edema periorbitario bilateral
con eritema y calor en la zona palpebral y en la mejilla derecha. Se hospitalizó con el diagnóstico de celulitis preseptal, iniciándose
tratamiento con dicloxacilina endovenosa.

PREGUNTA
Cual de los siguientes agentes infecciosos es mas probable aisla en este caso?

RESPUESTA
a.- Estreptococos del grupo A
b.- Estreptococos del grupo B
c.- Estreptococos del grupo C
d.- Estreptococos del grupo D

CASO CLINICO
Varón de 62 años, sin antecedentes clínicos de interés, que ingresó por presentar en las 48-72 h previas un cuadro de fiebre elevada,
sudoración profusa, escalofríos, mialgias y posteriormente cefalea frontooccipital intensa, postración y hematuria macroscópica. No
presentaba síntomas gastrointestinales, infecciosos, traumatismos ni intervenciones quirúrgicas previas. En la exploración física
destacaba rigidez de nuca y afectación del estado general. No se encontraron signos de afectación cardiovascular ni digestiva. En los
estudios analíticos destacan la presencia de leucocitosis con desviación izquierda, inicio de patrón analítico de coagulación intravascular
diseminada (CID) y leucocituria con nitritos positivos en orina. Se realizó punción lumbar, obteniéndose líquido claro a presión normal
cuyo examen mostró: leucocitos 1.200 cél./ml, proteínas 8 g/l, glucosa 9 mg/dl, Gram: se identificaron cocos grampositivos en cuyo
cultivo se aisló S. bovis biotipo II

PREGUNTA
Cual de los siguientes grupos agentes infecciosos es mas probable aisla en este caso?

RESPUESTA
a.- Estreptococos del grupo A
b.- Estreptococos del grupo B
c.- Estreptococos del grupo C
d.- Estreptococos del grupo D

CASO CLINICO
Mujer de 59 años con cuadro de 72 horas de evolución con astenia, anorexia y deterioro del nivel de conciencia que se acompañaba en
las últimas 24 horas de disnea. Se observo con insuficiencia respiratoria parcial y una fibrilación auricular con respuesta ventricular
rápida e inestabilidad hemodinámica secundaria motivo por el que se decidió el traslado a la Unidad de Cuidados Intensivos (UCI).
Ingresa a UCI obnubilada, hipotensa (tensión arterial: 90/60 mmHg), en fibrilación auricular a 130 lpm, taquipnéica a 30 rpm y con
saturación arterial de oxigeno de 90% según pulsioximetría respirando con mascarilla al 50% de fracción inspirada de O2. En la
exploración física destacó únicamente la existencia de ingurgitación yugular bilateral e hipoventilación en la base pulmonar izquierda. A
nivel analítico llamó la atención la existencia de leucocitosis con 15.300 leucocitos, neutrofilia y una importante desviación izquierda.
Electrocardiograma con fibrilación auricular rápida sin alteraciones de la repolarización. Radiografía de tórax con cardiomegalia y
borramiento de ambos hemidiafragmas. Se canalizó un catéter venoso central y un catéter PICCO, objetivándose un patrón
hemodinámico compatible con sepsis (gasto cardiaco elevado con resistencias vasculares sistémicas disminuidas) y unos valores de
presión venosa central elevados.

PREGUNTA
Cual de los siguientes grupos agentes infecciosos es mas probable aisla en este caso?

RESPUESTA
a.- Estreptococos del grupo A
b.- Estreptococos del grupo B
c.- Estreptococos del grupo C
d.- Estreptococos no A no B.

CASO CLINICO
Paciente de 35 años, sexo femenino, no fumadora, fue sometida a tiroidectomía total por neoplasia folicular de tiroides y fue dada de
alta en buenas condiciones generales a las 48 h. Consultó 5 dias despues por malestar general, odinofagia, cefalea, sensación febril,
escalofríos, dolor cervical, tos productiva y expectoración mucopurulenta, siendo admitida a sala de cuidados generales. En el examen
físico destacaba paciente en regulares condiciones generales, lúcida, orientada, FC: 90 lat/min, PA: 106/60 mmHg, FR: 24 resp/min, T:
37,8°C. Dolor a la palpación en la zona de herida operatoria de la región cervical anterior, faringe sana, examen cardiopulmonar y
abdominal normales. Se planteó el diagnóstico de infección de herida operatoria y se inició tratamiento con ceftriaxona 1 g/día EV. La

CURSO ENARM CMN SIGLO XXI TEL: 36246001 Pharmed Solutions Institute PÁGINA 128
MANUAL DE TRABAJO DEL CURSO ENARM CMN SIGLO XXI
paciente evolucionó febril y con dificultad respiratoria progresiva, requiriendo aporte de oxígeno al 35% para corregir el trastorno del
intercambio gaseoso (Pa02/Fi02: 259). En el hemograma destacaba leucopenia (2.700/mm3) con desviación a izquierda (13%
baciliformes), PCR: 5,4 mg/dL, GSA: pH: 7,46, Pa02: 54,4 mmHg, Sa02: 89%, PaC02: 27,5 mmHg, B. act: 19,6 mEq/L. La radiografía de
tórax mostró opacidades parenqui-matosas en lóbulo superior derecho, lóbulo medio, língula y lóbulo inferior izquierdo.

PREGUNTA
Cual de los siguientes grupos agentes infecciosos es mas probable aisla en este caso?

RESPUESTA
a.- Estreptococos del grupo A
b.- Estreptococos del grupo B
c.- Estreptococos del grupo C
d.- Estreptococos no A no B.

CURSO ENARM CMN SIGLO XXI TEL: 36246001 Pharmed Solutions Institute PÁGINA 129
MANUAL DE TRABAJO DEL CURSO ENARM CMN SIGLO XXI
INFECCIONES POR ESTAFILOCOCOS: CIENCIAS BASICAS: Los estafilococos son cocos gram-positivos que forman racimos de uvas en la
tinción de Gramm, son catalasa positivo (a diferencia de los estreptococos), no móviles, aeróbicos y anaerobias facultativas.
Staphylococcus aureus, se distingue de otros estafilococos por su producción de coagulasa, es la especie más virulentas, causando la
enfermedad a través de ambos mecanismos no mediadas por toxinas y mediada por toxina. Los estafilococos coagulasa negativos
(ScoN) son menos virulentas que S. aureus, pero son causas importantes y comunes de las infecciones de dispositivos de prótesis. S.
epidermidis con mayor frecuencia causa la enfermedad, este organismo es un componente normal de la piel, orofaríngea, y la flora
vaginal. S. saprophyticus es una causa de las infecciones urinarias. Otras dos especies de ScoN, S. lugdunensis y S. schleiferi, son más
virulentas y causan infecciones graves tales como válvulas nativas endocarditis y osteomielitis. SALUD PÚBLICA: S. aureus es
responsable 25 a 35 % de los casos de endocarditis bacteriana. Las
tasas de mortalidad oscilan entre 20 a 40% a pesar de la DEFINICIONES CLINICAS PARA SINDROME DE CHOQUE TOXICO
disponibilidad de antibióticos eficaces. PATOGENIA: Una vez que se Temperatura > 38.9°
Exantema macular difuso
presenta la infección, la invasión local y sistémica ocurre por vía
Descamación 1-2 semanas posteriores al inicio de la enfermedad
hematógena y liberación de toxinas. Localmente los organismos (+en palmas y plantas)
pueden invadir o necrosar el tejido y ocasionar una potente Hipotensión: Presión sistólica <90mmHg. Caída ortostática de la
respuesta inflamatoria (como resultado de abrasiones, quemaduras, presión diastólica > 15mmHg de acostado a sentado, sincope
herida, inyecciones o a través de catéteres), mediada ortostático o mareo.
principalmente por PMN (respuesta primaria). La formación de Afección multisistémica: 3 o más de las siguientes
abscesos (microcápsula polisacárido antifagocítica, facilita la Gastrointestinal; vomito o diarrea al inicio de la enfermedad
evasión de las defensas del huésped, útil en la formación de Muscular; mialgia severa o elevación de CPK, 2 veces arriba de lo
normal
absceso) es común, con un centro necrótico que está formado de
Mucosas; hiperemia en conjuntiva, orofaringe, vagina
pus y una pared de fibrina, que dificulta la penetración de Renal; nitrógeno ureico o creatinina por lo menos dos veces arriba
antibióticos, y protege de defensas del huésped, las bacterias se de lo normal o sedimento urinario con piuria (>5 leucos x campo),
adhieren formando una biopelícula similar a la formada por los en ausencia de infección de vías urinarias
ScoN. La infección puede diseminarse localmente por la formación Hepática; bilirrubina total, transaminasas 2 veces por arriba de lo
de trayectos sinuosos y abscesos secundarios. La diseminación normal
3
hematógena puede infectar articulaciones, válvulas cardiacas y Hematológica: plaquetas <100,000/mm
SNC; desorientación o alteraciones en la conciencia sin focalización,
otros tejidos. Para el desarrollo del choque séptico los monocitos y
en ausencia de fiebre o hipotensión
macrófagos tienen el papel principal, liberan TNF α, IL-1, IL-6, IL-8, Criterios de laboratorio: Estudios con resultado negativo:
posterior al contacto con el estafilococo, peptidoglocano o acido Cultivos de sangre, faringe, LCR (excepto positivo en sangre para S.
lipoteicoico, en respuesta a esto las vías del complemento y aureus)
coagulación se activan, se metaboliza el acido araquidónico y le Títulos de anticuerpos para leptospira o antiestreptolisinas o
libera el factor de activación de plaquetas; provocándose sarampión
hipotensión, fiebre, fuga capilar, CID y disfunción multiorgánica. Los Clasificación de caso:
estafilococos coagulasa negativos (ScoN), se adhieren a cuerpos Probable; 5 de los 6 datos clínicos descritos
Conformado; Los 6 datos clínicos descritos anteriormente,
extraños (a través del acido teicoico en su pared celular y la
incluyendo descamación, a menos que el paciente fallezca antes de
producción de “slime” que recubre el material extraño) y provocan la descamación
infecciones difíciles de tratar (flebitis hasta bacteriemia y choque
séptico) sin retirar el material protésico (catéteres). La bacteriemia por estafilococo puede complicarse con endocarditis o síndrome
séptico. El estafilococo se adhiere ávidamente a la célula endotelial a través de interacciones adhesina-receptor. Algunos productos de
estafilococos son considerados como superantigenos, que provocan enfermedades graves. La toxina-1 del síndrome de choque toxico,
se une a todas la regiones variables de la célula. DIAGNOSTICO: Infecciones de piel y tejidos blandos; impétigo-infección superficial de
piel, buloso (30%) o no buloso (70%), este inicia con una pápula eritematosa, que se presenta en lesiones previas por picadura de
insectos, varicela, eczema, trauma o abrasiones, la pápula progresa a vesícula y después a costra mielisérica, la topografía de las
lesiones es alrededor de nariz y boca raro en extremidades. Foliculitis- pequeñas pápulas o pústulas centradas en el folículo piloso y
rodeado de eritema. Forúnculos-nódulo profundo y doloroso de color rojizo con formación de pus, forma pequeños abscesos por la
necrosis del aparato polisebaceo. Paroniquia-infección que rodea el borde una uña de mano o pie. Celulitis, matitis, infecciones de
heridas quirúrgicas, conjuntivitis purulenta, endoftalmitis. Endocarditis infecciosa-en asociación con el uso de drogas inyectables,
válvula protésica, nosocomial, los pacientes tienen fiebre alta, aspecto tóxico, y dolor torácico pleurítico y producir esputo purulento
que a veces sangriento. La radiografía de tórax puede revelar embolia séptica: lesiones pequeñas y periféricas. Pericarditis como
consecuencia de una diseminación contigua de una infección respiratoria, es rara. Infecciones en huesos y articulaciones; osteomielitis
derivada de la diseminación hematógena o propagación contigua desde un sitio de tejido blando (por ejemplo, úlceras diabéticas o
vasculares). Osteomielitis hematógena en niños frecuentemente en la metástasis de huesos largos y se presenta con fiebre, dolor de
huesos, y la renuencia a soportar peso. Leucocitosis, aumento de la VSG y hemocultivos positivos son típicos. Osteomielitis hematógena
en los adultos es a menudo vertebral. La osteomielitis por infecciones de tejidos blandos contiguos se sugiere por la exposición del
hueso, un trayecto fistuloso drenaje, la falta de cura, el drenaje continuo. S. aureus es también causa común de artritis séptica,
asociada con el trauma o cirugía, o se debida a la diseminación hematógena, hacer hemocultivo y punción articular, la evaluación de
liquido sinovial debe incluir cultivo, tinción de gramm, cuenta celular (75% de PMN), glucosa (2/3 de la sérica) y proteínas. Infecciones
respiratorias; otitis media, traqueítis bacteriana, neumonía nosocomial-se produce principalmente en pacientes intubados, en unidades
de cuidados intensivos, tienen un mayor volumen de esputo purulento, fiebre y nuevos infiltrados pulmonares y puede desarrollar
dificultad respiratoria, rara neumonía de la comunidad. Infecciones en SNC; meningitis-por extensión de un foco parameníngeo,
durante la bacteriemia se puede producir cerebritis y posteriormente formación de un absceso cerebral. La incidencia de bacteriemia
durante la siembra metastásica se ha estimado para ser tan alta como 31 %. Los huesos, las articulaciones, los riñones y los pulmones
son los más comúnmente infectados. Las infecciones por SARM pueden tener muchas presentaciones inusuales (por ejemplo, la fascitis
necrotizante, neumonía necrótica, sepsis, púrpura fulminante) que reflejan el aumento de la virulencia de las cepas CA –MRSA.
ENFERMEDADES MEDIADAS POR TOXINAS; El síndrome de shock tóxico (TSS): Se presentaba en mujeres menstruando que utilizaban
tampones de gran absorción. En la actualidad se conoce que cualquier paciente con infección por estafilococo puede presentar TSS, así

CURSO ENARM CMN SIGLO XXI TEL: 36246001 Pharmed Solutions Institute PÁGINA 130
MANUAL DE TRABAJO DEL CURSO ENARM CMN SIGLO XXI
en la actualidad <50% de casos reportados se relacionan al uso de tampones y alrededor de 10% se relacionan a diversas infecciones
por estafilococo (empiema, bacteremia, abscesos, infecciones quirúrgicas). Más común en mujeres jóvenes. El síndrome se relaciona
con la enterotoxina F o exotoxina pirogénica C o toxina-1 del síndrome de choque toxico, pero las endotoxina A a la E pudieran
contribuir a la presentación del síndrome; puede manifestarse con signos y síntomas leves hasta una enfermedad fatal rápidamente
progresiva, inicio súbito de exantema difuso (difuso, escalatiniforme, predomina en aras de flexión), fiebre, alta, vomito, diarrea,
cefalea, faringitis, mialgias, alopecia, hipotensión, falla multiorgánica, por la pobre perfusión tisular y daño directo de los mediadores,
falla hematológica. Los cambios hemodinámicas incluyen disminución en el tono vasomotor y fuga no hidrostática del liquido del
espacio intravascular al intersticial. Las complicaciones que pueden llevar a la muerte son choque refractario, falla renal oligúrica,
arritmia ventricular, CID y pulmón de choque. La intoxicación alimentaria: La inoculación de la toxina de S. aureus, por los
manipuladores de alimentos, coloniza los alimentos. La toxina crece después en los alimentos que promueven el crecimiento, como la
crema, ensalada de papas, jamón, pollo o carne procesada. La toxina estable al calor no se destruye. Inicio de la enfermedad es rápida y
explosiva, que ocurre dentro de 1-6 h de la ingestión de alimentos contaminados. Los principales signos y síntomas son náuseas y
vómitos, pero pueden ocurrir diarrea, hipotensión y deshidratación. La fiebre está ausente. Los síntomas se resuelven en 8-10 h. El
síndrome de piel escaldada estafilocócica (SSSS): También conocida como enfermedad de Ritter, característico la piel se desprende
como si se hubiese quemado, afecta más frecuentemente a los recién nacidos y los niños. La piel es frágil, se puede formar bullas de
pared delgada que contienen líquido, su manejo es control de electrolitos y lavado de manos estricto así como antibióticos. El signo de
Nikolsky la fricción leve sobre la piel produce un desprendimiento de las capas superficiales de la piel. La bula típica se rompe deja una
área denudada dejando una superficie húmeda y finalmente ocurrirá descamación. TRATAMIENTO: Éxito inicial con penicilinas
semisintéticas resistentes a las penicilinasas como meticilina, oxacilina, nafcilina, cloxacilina y dicloxacilina. Estos agentes fueron
utilizados ampliamente, hasta la aparición de cepas resistentes a meticilina (SARM), actualmente se ha descontinuado su uso debido a
que produce nefritis intersticial. La nafcilina y oxacilina se usan en otros países. En México, la dicloxacilina (250mg a 1 gr en adultos y de
50-400mg/kg/día en niños dividido en 4 dosis), es uno de los fármacos de primera elección para el tratamiento de estafilococos. Una
alternativa son las cefalosporinas de primera generación (cefalozina, cefalotina), segunda generación (cefuroxima). En infecciones
severas se utiliza la terapia combinada, generalmente con un aminoglucosido para sinergia. Los antibióticos combinados con un
inhibidor de beta-lactamasa (ac. Clavulanico, sulbactam, tazobactam), también pueden utilizarse en el tratamiento para infecciones por
estafilococo sensible a meticilina, pero no son la primera elección. La vancomicina no debe utilizarse en forma rutinaria, porque es caro,
toxico y requiere una estrecha vigilancia por efectos secundarios, lo más importante es que favorece la aparición de cepas resistentes.
A pesar de los reportes de resistencia a meticilina, en infecciones comunitarias por S. aureus, donde se sospeche que la resistencia es
elevada, deberán emplearse otras alternativas como clindamicina o trimetropim/sulfametoxaxol. Es indispensable, que se tomen
cultivos antes del inicio de tratamiento, para efectuar modificaciones de acuerdo a los resultados de sensibilidad y la respuesta clínica
del paciente. Cuando el paciente tiene una infección severa y síntomas de sepsis, lo fundamental es reconocer la causa y quitar los
factores precipitantes, el apoyo en un servicio de terapia intensiva y el tratamiento empírico con antibióticos de acuerdo a la sospecha
clínica. En el Síndrome de choque toxico está recomendado: dicloxacilina + aminoglucocido, cefalotina + aminoglucocido,
aminoglucocido + glicopeptidos (amikacina-vancomicina), linezolid. En infecciones de SNC, se ha utilizado tanto el tratamiento
sistémico como el intraventricular, la limitación es la penetración adecuada del fármaco al SNC, los medicamentos habitualmente
utilizados son dicloxacilina, oxacilina y con vancomicina los niveles no son óptimos. Para tratamiento intraventricular se ha utilizado
gentamicina o amikacina.

CASO CLINICO
Paciente varón de 11 años de edad, procedente de Chancay, readmitido después de 19 días de ser sometido a cirugía electiva por un
quiste óseo aneurismático en el fémur izquierdo, realizándole curetaje óseo e injerto autólogo de cresta iliaca. Fue dado de alta al
séptimo día. Posterior al alta, presentó fiebre no cuantificada y malestar general; persistiendo febril los siguientes doce días, motivo de
su readmisión en el servicio de Traumatología. No tenía antecedentes importantes. El niño lucía en buen estado general, presentando
fiebre entre 38 y 38.5 °C. Al examen se evidenciaba discreto aumento de volumen y dolor a nivel sacro y glúteo izquierdo, sin otros
signos de flogosis.Las heridas operatorias habian cicatrizado. El resto del examen físico era normal. El hemograma presentaba 10,200
leucocitos, sin desviación izquierda. No tuvo estudio de VSG ni proteina C reactiva. El examen ecográfico de partes blandas a nivel
sacro-glúteo izquierdo revelaba distorsión de las partes blandas en los planos subcutáneos y musculares adyacentes a la herida
operatoria, sin evidenciar colecciones. En la tomografía pelvica se evidenció imágenes compatibles con abscesos en partes blandas en
pelvis izquierda. No se procedió a punción ni drenaje. Contó con solo un cultivo, el cual fue positivo a S. epidermidis

PREGUNTA
Cual es la conducta terapéutica para el agente etiológico aislado?

RESPUESTA
a.- Cefalotina.
b.- Dicloxacilina.
c.- Ciprofloxacina.
d.- Tazobactam.

CASO CLINICO
Femenino de 63 años, inmunocompetente, que acude por aparición de un bulto en la cara interna del tercio superior del muslo
derecho. En la TAC se observaron extensas colecciones de contenido líquido, desde la pelvis hasta el compartimento intramuscular de
los músculos psoas-ilíaco derecho y obturador derecho. A través de la región inguinal derecha, estas se extendían hasta el tercio medio
del muslo derecho; las colecciones de mayor tamaño se localizaban en el muslo y se asociaban a reacción perióstica femoral, y llegaron
a medir hasta 11 cm de diámetro mayor. Por cultivo del material de drenaje se diagnosticó una infección por Staphylococcus aureus
resistente a la meticilina (SARM).

CURSO ENARM CMN SIGLO XXI TEL: 36246001 Pharmed Solutions Institute PÁGINA 131
MANUAL DE TRABAJO DEL CURSO ENARM CMN SIGLO XXI

PREGUNTA
a.- Rifampicina y minociclina.
b.- Vancomicina y gentamicina.
c.- Ceftriaxiona y amikacina.
d.- Dicloxacilina y vancomicina.

CASO CLINICO
Se trata de un paciente de 79 años con enfermedad renal crónica estadio 5 secundaria a nefroangioesclerosis y/o nefropatía diabética,
en terapia renal sustitutiva con diálisis peritoneal continua ambulatoria. Como otros antecedentes presenta diabetes mellitus tipo 2 de
larga evolución, hipertensión arterial y anemia perniciosa. A los 27 meses del inicio de la técnica, el paciente acude a urgencias con
clínica y recuento de células en el líquido de drenaje, compatibles con una peritonitis. Se inicia el protocolo empírico que incluye
vancomicina y ceftazidima, Se envía muestras para cultivo. Se presentan posteriormente 3 episodios de peritonitis con un intervalo de
15 dias minimo entre cada cuadro.

PREGUNTA
Cual es la conducta mas adecuada a seguir?

RESPUESTA
a.- Cambiar de tipo de catéter.
b.- Vancomicina intraperitoneal.
c.- Daptomicina IV.
d.- Cambiar a hemodiálisis.

CASO CLINICO
Paciente varón de 44 años, con infección por virus de inmunodeficiencia humano (VIH) diagnosticada en marzo de 2005, sin terapia
antirretroviral (linfocitos T CD4/+n 286/mm3, carga viral 89.000 copias/mi). Semanas previas a la consulta realizó viaje a Uruguay y
mantuvo contactos sexuales con más de una pareja. Una semana antes del ingreso comenzó con aumento de volumen labial
diagnosticándose infección por virus Herpes simples, indicándose aciclovir 400 mg/12 h vo. Al día siguiente, se agregó ciprofloxacino
500 mg c/12 h por progresión de lesión. Evolucionó con fiebre 38°C, tos y dolor torácico. Radiografía de tórax reveló neumonía
intersticial bilateral indicándose hospitalización, la que el paciente rechazó. Se indicó levofloxacino 500 mg/día vo. A las 48 h consultó
en por agravamiento de su cuadro clínico y fue hospitalizado. Ingresó febril 38°C, presión arterial 120/70 y taquicardia de 110x'. Al
examen facial presentaba aumento de volumen flegmonoso del labio superior y en examen pulmonar crépitos básales bilaterales.
Hemograma con hematocrito 40%, leucocitos 14.300/mm3, baciliformes 4%, VHS 69 y PCR 24,7 mg/dL. Radiografía y tomografía axial
computarizada (TAC) de tórax mostraron pleuroneumonía bilateral. Punción pleural dio líquido con pH 7,4, glucosa 5 mg/dL, 26.800
células, 61,5% neutrófilos. En cultivos de secreción de labio superior y líquido pleural se aisló Staphylococcus aureus resistente a B-
lactámicos, ciprofloxacino, eritromicina y resistencia inducible a clindamicina.

PREGUNTA
Cual es la conducta terapéutica mas adecuada para el caso?

RESPUESTA
a.- Vancomicina y rifampicina.
b.- Vancomicina y gentamicina.
c.- Ceftriaxiona y amikacina.
d.- Dicloxacilina y vancomicina.

CASO CLINICO
Paciente varón de 23 años, estando en chiapas sufrió picadura de insecto en tobillo derecho. En las horas siguientes, desarrolló
aumento de volumen, dolor y eritema en la zona, asociado a fiebre. Evaluado por médico, se le indicó cefadroxilo 500 cada 12 h vo. A
las 24 h presentó drenaje espontáneo de abundante pus de la herida. Cuatro días más tarde consultó por persistencia de lesión
inflamatoria del tobillo. Al examen de ingreso se constató paciente afebril, presión arterial 134/70 mmHg, FC 64x'. En tobillo derecho
había aumento de volumen eritematoso con úlcera central. El hemograma mostró hematocrito 36,8%, leucocitos 6.680 x mm3,
baciliformes 3%, VHS 60 y PCR 6,3 mg/ dL. El cultivo fue positivo para Staphylococcus aureus resistente a B-lactámicos,
sulfametoxazoltrimetropim y ciprofloxacino.

PREGUNTA
Cual es la conducta terapéutica a seguir mas adecuada para el caso?

RESPUESTA
a.- Cefazolina y clindamicina intravenosa.
b.- Clindamicina via oral.
c.- Ceftriaxiona y amikacina.
d.- Dicloxacilina y vancomicina.

CURSO ENARM CMN SIGLO XXI TEL: 36246001 Pharmed Solutions Institute PÁGINA 132
MANUAL DE TRABAJO DEL CURSO ENARM CMN SIGLO XXI
CASO CLINICO
Paciente varón de 39 años, motociclista. Tres meses previo al ingreso, comenzó con furúnculos en tórax, cuello y extremidades
inferiores las que se trataron con diversos antimicrobianos vía oral, con respuesta parcial y aparición de nuevas lesiones. El estudio
microbiológico de una lesión del muslo, identificó Staphylococcus aureus resistente a cloxacilina y sensible a clindamicina, la que recibió
vía oral, sin respuesta. Por progresión de lesión en muslo izquierdo con celulitis extensa. Al examen físico destacaba lesión pustulosa de
4 cm de diámetro en muslo izquierdo, rodeada de una zona eritematosa extensa que comprometía el tercio inferior del muslo. El
hemograma mostró hematocrito 40%, leucocitos 6.600 x mm3, baciliformes 2%, VHS 32 y PCR 6 mg/dl.

PREGUNTA
Cual es la conducta terapéutica a seguir mas adecuada para el caso?
RESPUESTA
a.- Vancomicina y clindamicina intravenosa.
b.- Clindamicina via oral.
c.- Ceftriaxiona y amikacina.
d.- Dicloxacilina y vancomicina.

CASO CLINICO
Se trata de paciente femenino de 31 años de edad la cual acude a consulta debido a que presenta disnea grave, desorientada,
confundida con tos productiva y fiebre importante, el familiar refiere no tener enfermedades, ser prácticamente sana, soltera y
abogada de profesión, agrega que hace una semana presento un cuadro gripal, caracterizado por fiebre, cefalea, mialgias y malestar
generalizado, sin darle importancia solo toma medidas generales mejorando, dos días después presenta ataque al estado generalizado,
con dificultad respiratoria, a la exploración física se presenta confusa, diaforética, disneica, con temperatura de 39.6 C, pulso de 130
lpm, TA 90/60 mmHg, FR 39 se ingresa y monitoriza con oximetro que reporta 89 % de saturación se coloca oxigeno al 100% sin
cambios, la auscultación pulmonar con sonidos respiratorios anforicos predominante en lóbulo inferior izquierdo, la TAC revela necrosis
del lóbulo inferior izquierdo. Se realiza intubación y soporte de líquidos, se sospecha Staphylococcus aureus como agente causal.

PREGUNTA
Cual es tratamiento mas adecuado antes del reporte de los cultivos.

RESPUESTA
a.- Doxiciclina.
b.- Meticilina.
c.- Vancomicina.
d.- Trimetroprim/sulfametoxazol.

CASO CLINICO
Se trata de masculino de 34 años de edad el cual tiene catéter para hemodiálisis, actualmente con renopatia terminal, acude a consulta
debido a que presenta fiebre y dolor intenso en la región baja de la espalda, a la exploración física se observa ansioso, irritable,
diaforético así como ataque al estado generalizado sin embargo signos vitales 120/80, FC 89, FR 21, temperatura 38.7 C, el sitio donde
se encuentra colocado el catéter se observa rojo y caliente, no se observan secreciones ni olor, la biometría hemática reporta 16 700
con 13 % de bandas, se realiza IRM donde se reporta lesiones compatibles con osteomielitis vertebral.

PREGUNTA
Cual es agente etiológico más probable.

RESPUESTA
a.- Streptococcos.
b.- Stafilococcos.
c.- Mycoplasmas.
d.- Tuberculosis.

CASO CLINICO
Se trata de masculino de 75 años de edad el cual cuenta con antecedentes tales como diabetes mellitus de más de 25 años de
evolución, actualmente acude por segunda ocasión a consulta debido a que presenta una ulcera del pie diabético de 2 meses de
duración la cual no ha respondido a tratamiento previo con metronidazol y dicloxacilina sin especificar dosis, a la exploración física se
observa lesión con olor pútrido, se trata de una lesión de 2,6 cm con pus, la cual se drena e identifica lesión que llega al hueso con una
profundidad de 2.5 cm, se realiza cultivo y tinción, sin embargo se debe iniciar tratamiento, la tinción reporta flora mixta.

PREGUNTA
Cuál es el manejo farmacológico en espera de los resultados del cultivo.

RESPUESTA
a.- Clindamicina.
b.- Ampicilina/sulbactam.
c.- Metronidazol.
d.- Vancomicina.

CURSO ENARM CMN SIGLO XXI TEL: 36246001 Pharmed Solutions Institute PÁGINA 133
MANUAL DE TRABAJO DEL CURSO ENARM CMN SIGLO XXI
CASO CLINICO
Femenino de 23 años de edad, acude a consulta refiriendo fiebre, escalofríos asi como dolor de espalda y malestar generalizado.
Cuenta con antecedente de trastorno bipolar y abuso de sustancias, actualmente bajo tratamiento sin embargo con recaidas. A la
exploración física se observa lesiones tipo puncion en pliegue cutáneo en antebrazo por probable inyección intravenosa de sustancias,
al parecer de drogas sin aceptarlo la paciente, los signos vitales son 38.5 C de temperatura, FC 110 lpm, FR 24, tensión arterial de
110/60 mmHg, Rx de torax con varios infiltrados nodulares periféricos con cavitaciones.

PREGUNTA
Cuál es la patología más grave que la paciente presenta.

RESPUESTA
a.- Neumonia
b.- Endocarditis.
c.- Septicemia.
d.- CID.

INFLUENZA (HAEMOPHILUS INFLUENZAE):


CIENCIAS BASICAS: Infección contagiosa de origen viral, considerada una de las causas más importantes de infecciones de vías
respiratorias. La relevancia de esta infección radica en su frecuente asociación a complicaciones y muerte en poblaciones susceptibles.
Este grupo de virus es capaz de asociarse a epidemias y pandemias con consecuencias devastadoras. El virus de la influenza puede
afectar la mucosa nasal, faringe, bronquios y a veces hasta los alveolos pulmonares. Los síntomas son parecidos a los del catarro común
o resfriado, sin embargo son más severos y su inicio es generalmente abrupto. El Haemophilus influenzae es un pequeño gramm
negativo, ARN, cocobacilo, pleomórfico. La influenza es ocasionada por 3 virus (influenza A,B,C). Las cepas con una cápsula de
polisacárido son más importante clínicamente, causan la enfermedad invasiva sistémica, principalmente en bebés y niños <6 años de
edad. Cepas no tipificables de H. influenzae, que están no encapsulado, provocan la enfermedad mediante invasión local de superficies
mucosas. SALUD PÚBLICA: La enfermedad generalmente requiere atención médica y hospitalización, contribuyendo sustancialmente a
pérdidas económicas y exceso en número de hospitalizaciones y muerte. Por lo menos 4 pandemias han afectado a la humanidad en el
siglo XIX, y 3 en el XX. El virus de la influenza A se caracteriza por causar enfermedad moderada a grave; tiende a afectar a todos los
grupos etarios y tiene la característica de afectar tanto aves como cerdos. El virus B de la influenza cuadros clínicos menos graves, y
tradicionalmente produce infección durante la infancia. La capacidad del virus de la influenza A y B de sufrir cambios antigénicos
graduales en sus dos antígenos de superficie: la hemaglutinina y la neuraminidasa, complica la vacunación contra esta enfermedad.
Afecta a todos los grupos etarios, pero principalmente a <2años y >65años. Ocurre principalmente durante los meses de invierno y
principios de la primavera. En México la neumonía e influenza representan una de las mayores causas de morbilidad y mortalidad. En
>65 años de edad, la neumonía e influenza se reportan como la séptima causa de mortalidad. PATOGENIA: Los virus de influenza se
diseminan de persona a persona a través de los actos de toser o estornudar de personas infectadas o enfermas con los virus de
influenza, gotitas en el aire o fómites, el periodo de incubación es de 1-4 días, la patogénesis de la replicación del virus y su relación al
desarrollo de manifestaciones clínicas no ha sido completamente descifrada, no obstante la replicación viral ocurre primariamente en
el epitelio columnar del aparato respiratorio, pero puede ocurrir en cualquier segmento a lo largo del tracto respiratorio. La
regeneración del epitelio columnar toma de 3-4 semanas, tiempo en el cual las manifestaciones pulmonares pueden persistir.
DIAGNOSTICO: Clínico; Inicio agudo de fiebre, mialgia, cefalea, debilidad extrema, tos no productiva, odinofagia y rinitis, estos síntomas
ocurren en 50-70% de las infecciones. Puede haber dolor retroesternal, fotofobia, dolor abdominal y diarrea, en ancianos puede no
haber fiebre solo delirio y postración, en niños puede asociarse a otitis media y traqueobronquitis. Podemos encontrar linfadenopatía
cervical. Las definiciones clínicas para diagnosticar influenza varían de 63-78% de sensibilidad y de 55-71% de especificidad.
Laboratorio: Aislamiento del virus en muestras de exudado faríngeo o nasofaríngeo obtenido dentro de los primeros 3 días del inicio de
síntomas. Se considera al cultivo viral (permite tipificar y caracterizarlo antigénicamente), como el estándar de oro seguido de
conformación por inhibición de la hemaglutinación. Las técnicas serológicas mas recuente usadas son la fijación de complemento y la
inhibición por hemaglutinación. COMPLICACIONES: Neumonía primaria por influenza o neumonías secundarias bacterianas (S.
pneumoniae, S. aureus), sinusitis bacteriana, bronquitis, traqueobronquitis y otitis media. Puede ocurrir muerte en 0.5-1 en 1,000
casos. Otras; convulsiones, encefalopatía por el virus, o encefalopatía asociada a la utilización de salicilatos (síndrome de Reye=
encefalopatía aguda acompañada de degeneración grasa visceral), miositis, miocarditis y pericarditis. Puede haber diversas
manifestaciones a nivel de SNC, incluyendo mielitis transversa y Sx. de Guillan Barré. Meningitis que se asocia con una alta morbilidad,
6 % de los pacientes tienen una pérdida auditiva neurosensorial, una cuarta parte tienen algunas secuelas importantes, la mortalidad es
de 5 %. Epiglotitis, que ocurre en niños mayores y en ocasiones en adultos, implica la celulitis de la epiglotis y tejidos supraglóticos que
comienza con dolor de garganta y progresa rápidamente a la disfagia, salivación, y obstrucción de vía aérea. TRATAMIENTO: Reposo en
casa, analgésico o antiinflamatorios que no sean de la familia de los salicilatos, no utilizar antibióticos profilácticamente. Los derivados
de amantadino; amantadina (>10 años 100mgs c/12 hrs >65 100mg c/24 hrs) y rimantadina (>10 años 100mgs c/12 hrs >65 100mg
c/24 hrs) y los inhibidores de la enzima neuraminidasa; oseltamivir (>13 años 75mg c/12 hrs) y zanamivir (>10 años de edad 10mg c/12
hrs), los cuatro han demostrado utilidad clínica reduciendo la duración de los síntomas. Meningitis en adultos por H. influenzae tipo B:
ceftriaxona (2 g cada 12 h durante 1-2 semanas). Meningitis por H. influenzae tipo B en niños: ceftriaxona (75-100 mg / kg por día,
divididos en dos dosis cada 12 h) y dexametasona (0,6 mg / kg por día en cuatro dosis divididas durante 2 días a la iniciación del
tratamiento con antibióticos para prevenir pérdida de la audición). La epiglotitis: ceftriaxona (50 mg / kg al día durante 1-2 semanas).
PREVENCION: Vacunación anual de personas en grupos de alto riesgo, utilizando la vacuna inactivada trivalente por su eficacia y baja
reactogenicidad, los grupos son los siguientes; vacunación a personal de salud, personal que labora en asilos de ancianos, personas que
viven en contacto intradomiciliario con personas con alto riesgo de desarrollar complicaciones, mujeres con embarazo de alto riesgo
(2do y 3er trimestre), personas mayores de 65 años, personas entre 50-64 años (enfermedades cronicodegenerativas), pacientes de
cualquier edad con padecimientos crónicos (principalmente respiratorios y cardiovasculares), vacunación de niños de 6-59 meses y

CURSO ENARM CMN SIGLO XXI TEL: 36246001 Pharmed Solutions Institute PÁGINA 134
MANUAL DE TRABAJO DEL CURSO ENARM CMN SIGLO XXI
viajeros (que van a los trópicos o al Hemisferio Sur). La vacuna se administra por vía intramuscular, en región deltoidea (niños región
anterolateral de muslo). Contraindicada en alergia al huevo o a otros componentes de la vacuna o cuadro febril agudo.

CASO CLINICO
Femenino de 46 años con antecedentes de asma bronquial tipo III, para lo cual lleva tratamiento con salbutamol (spray). Acudió por
presentar fiebre de 39 0C y trastornos de la conciencia. Cinco días antes había presentado una crisis asmática acompañada de un
cuadro febril asociado con síntomas respiratorios altos, que coincidió con un cuadro respiratorio alto en su nieto pequeño atendido por
ella; por lo anterior fue vista en su área de salud donde le indicaron tratamiento con penicilina. El día antes presentó vómitos, en
número de 12, de tipo bilioso y acompañados de náuseas, con mucha tos y abundante secreción nasal verdosa y fiebre. En el momento
de ser valorada en el servicio de urgencia llamaba la atención los trastornos de conciencia que presentaba esta paciente, dados por la
agitación psicomotora y el estado de estupor. Al examen físico se constató rigidez nucal y signos de irritación meníngea. Se le realizó
punción lumbar (PL) de donde se obtuvo un líquido cefalorraquídeo (LCR) de aspecto turbio, así como otros exámenes
complementarios, incluidos hemogramas seriados. El examen citoquímico del LCR mostró los valores siguientes: 435 leucocitos/mm3 a
predominio de polimorfonucleares (PMN), hematíes 8/mm3, glucosa en 1,0 mmol/L y proteínas totales en 0,95 g /L. En el examen
bacteriológico del líquido se informó pleomorfismo bacteriano gramnegativo. En el leucograma se constató una cifra de 16 000
leucocitos con predominio de PMN. El cultivo reporto haemophilus influenzae en el LCR.

PREGUNTA
Cual es la conducta mas apropiada a seguir con el diagnostico presuntivo del caso?

RESPUESTA
a.- Rivavirina y amantadina.
b.- Ceftriaxona vía endovenosa.
c.- Ceftriaxona y amikacina IV.
d.- Cloranfenicol y ampicilina.

TOS FERINA O COQUELUCHE (BORDETELLA PERTUSSIS):


CIENCIAS BÁSICAS: Bordetella pertussis causa tos ferina, una infección aguda del tracto respiratorio, caracterizada por accesos de tos
parodia ticos acompañado de estridor al final de la inspiración. B. pertussis es un bacilo aerobio Gramm negativo. SALUD PÚBLICA: La
tos ferina es altamente contagiosa. En los hogares, las tasas de ataque son el 80% de los contactos no vacunados y 20% entre los
contactos inmunizados. La tos ferina sigue siendo una causa importante de morbilidad y muerte infantil en los países en desarrollo. En
los Estados Unidos, la incidencia ha aumentado lentamente desde 1976,
particularmente entre los adolescentes y adultos. Tos persistente de duración> 2 semanas en un adulto puede ser debido a B. pertussis
en 12-30% de los casos. La morbilidad y la mortalidad severa se limitan a lactantes <6 meses de edad. La incidencia en México es de 1-3
por 100,000 habitantes. PATOGENIA: Transmisión por vía respiratoria, al estar en contacto con gotas de secreciones, el periodo de
contagiosidad se presenta durante el periodo catarral y la primera semana del inicio de la fase paroxística. B. pertussis fastidioso se
adhiere a las células epiteliales ciliadas (tropismo) de la nasofaringe, donde prolifera y se dirige al epitelio traqueobronquial, es
internalizado por las células epiteliales, pero no penetra células submucosas, ni invade el torrente sanguíneo, se multiplica localmente,
y produce una amplia gama de toxinas y productos biológicamente activos como toxina pertussis ( la cual actúa como exotoxina y
penetra al torrente sanguíneo, induce linfocitosis, activación de islotes pancreáticos), hemaglutininas filamentosa (adherencia a células
de epitelio), pertactina (adherencia y altamente inmunogenica), BrkA (adherencia y resistencia al complemento), adenilciclasa (inhibe
función fagocitica). DIAGNOSTICO: Enfermedad prolongada de tos. Los síntomas suelen ser más graves en los bebés y niños pequeños.
Tres fases: La fase catarral; posterior a periodo de incubación de 7-10 días es similar al resfriado común (rinorrea, estornudos, lagrimeo,
tos leve progresiva) y tiene una duración de 1-2 semanas y es el periodo de mayor contagiosidad. La fase paroxística o de estado; los
accesos de tos son paroxísticos, característicamente si inspiración y suelen acompañarse de vomito y cianosis y terminar con un
estridor inspiratorio prolongado que le da el nombre de tos coqueluche. El número de accesos le da la gravedad, llegando a 15-20 por
día, sin son frecuentes, puede presentarse sudación, lasitud y somnolencia y son de predominio nocturno, tiene una duración de 2-
6semanas. Fase convaleciente: inicia cuando los accesos de tos dejan de ser espasmódicos y dura varias semanas, durante la fase de
accesos pueden exacerbarse si el paciente presenta infección en vías aéreas superiores. En cuadros más severos predominan datos de
dificultad respiratoria, cianosis y apnea, más que accesos de tos. Laboratorio: Aislamiento de Bordetella pertussis es 100% especifico, la
posibilidad de positividad declina si la muestra se toma >2 semanas del inicio de la sintomatología y la muestra para el cultivo se toma
de la nasofaringe posterior. PCR a aumentado su sensibilidad, pero no se usa como método exclusivo y no sustituye al cultivo.
Detección de anticuerpos fluorescentes (DAF), útil como prueba de detección rápida, pero la sensibilidad es baja. Serología;
demostración de una elevación sustancial 4 veces los títulos de anticuerpos contra diferentes antígenos. COMPLICACIONES: La más
frecuente y causas de muerte es la neumonía bacteriana secundaria, otras son crisis convulsivas y encefalopatía TRATAMIENTO: Del 80-
90% de pacientes con tos ferina sin tratamiento depuran espontáneamente a B. pertussis de la nasofaringeo en las 3-4 semanas.
Manejo principal ente de soporte. La eritromicina es el antibiótico de elección, preferentemente en forma de estolato a dosis de
40mg/kg/día por 14 dias. Alternativas claritromicina por 7 días igual efectividad y mejor tolerancia. Azitromicina y TMP/ SFX. En
menores de un mes se da azitromicina (eritro y claritromicina están contraindicadas). Para la tos se puede manejar benzonatato o
salbutamol en nebulizador. PROFILAXIS POSTEXPOSICION: Contactos asintomáticos de un caso índice que se encuentre dentro de los 21
días del inicio de los accesos de tos, deberá recibir profilaxis con antibióticos antes mencionados. PREVENCIÓN: Vacuna combinada de
toxoide tetánico y difterico (DPT). En México A partir de 1999, poco después de la introducción de la vacuna triple viral, se introduce
una vacuna pentavalente (DTwP-hepatitis B-Hib), en tres dosis a los 2,4 y 6 meses de edad y refuerzos con DPT a los 4 años de edad. A
partir de 2007, se efectuó un cambio biológico a una vacuna pentavalente a base de vacuna acentuar de pertussis (DTaP-IPV-Hib).

CURSO ENARM CMN SIGLO XXI TEL: 36246001 Pharmed Solutions Institute PÁGINA 135
MANUAL DE TRABAJO DEL CURSO ENARM CMN SIGLO XXI
CASO CLINICO
Una mujer de 64 años ingresada por una disnea creciente y dolor torácico. La paciente tenía antecedentes de carcinoma pulmonar no
microcítico y estaba recibiendo tratamiento. Anteriormente había fumado 4-5 cigarrillos al día durante un periodo de hasta 10 años. A
la exploración, se trataba de una mujer con sensación de enfermedad y emaciación, que estaba clínicamente anémica y presentaba
signos de un derrame pleural izquierdo masivo en la TC torácica. La frecuencia del pulso era de 85/min, la presión arterial de 120/85, y
no había signos de insuficiencia cardiaca. La radiografía de tórax confirmó el derrame unilateral izquierdo. La hemoglobina era de
9,1g/100ml, con un frotis de sangre normocrómico y normocítico. La velocidad de sedimentación globular fue de 90mm en la primera
hora. Se realizó una aspiración pleural, en la que se obtuvieron 1.500ml de líquido turbio. Dada la persistencia de las colecciones de
líquido pleural, 2 días después de la segunda aspiración se colocó un drenaje de tórax. El nivel de proteínas fue de 35g/l y la
concentración de glucosa, de 0,3mmol/l. No se observaron células malignas, pero en la PCR se demostró la presencia de B. pertussis. La
paciente no presentó fiebre ni manifestaciones clínicas de infección, y el recuento leucocitario en sangre fue normal. Además, no hubo
clínica de diarrea y los coprocultivos, urinocultivos y hemocultivos fueron repetidamente negativos.

PREGUNTA
Cual es la conducta antibiótica mas apropiada para el caso?

RESPUESTA
a.- Claritromicina.
b.- Azitromicina
c.- TMP/ SFX
d.- Eritromicina.

CASO CLINICO
Mujer de 45 años, fumadora de 6 cig/día desde los 20 años, por malestar general, astenia, fatigabilidad y tos seca de predominio
nocturno de seis semanas de evolución. No refere congestión nasal, rinorrea, descarga nasal posterior, cefalea, pirosis, fiebre,
escalofríos, disnea, dolor torácico, expectoración ni hemoptisis. Antecedentes mórbidos: rinitis alérgica estacional en primavera con
test cutáneos positivos a pólenes y pastos; refujo gastroesofágico en tratamiento con medidas dietéticas y Omeprazol 20 mg/día.
Actividad laboral: enfermera de unidad de hemodiálisis de adultos. En el examen físico, los signos vitales eran normales con una SpO2
de 97% respirando aire ambiente, el examen cardiopulmonar y abdominal eran normales. La paciente fue tratada con medidas
dietéticas y posturales antirefujo. Finalmente, la inmunofuorescencia directa de Bordetella pertussis de hisopado nasofaríngeo fue
positiva.

PREGUNTA
Cual es la conducta antibiótica mas apropiada para el caso?

RESPUESTA
a.- Claritromicina.
b.- Azitromicina
c.- TMP/ SFX
d.- Eritromicina.

MORAXELLA CATARRHALIS:
CIENCIAS BÁSICAS: Moraxella catarrhalis es un coco gram-negativo, que se asemeja a Neisseria. En el género Moraxella podemos
identificar cuatro especies: M. catarrhalis, M. caviae, M. ovis, M. cuniculi. Forma parte de la flora normal de las vías respiratorias
superiores. SALUD PUBLICA: M. catarrhalis coloniza hasta el 50% de los niños sanos (muy relacionados con IRA) y hasta el 3-7% de los
adultos sanos. Las tasas de infección pico son a finales de invierno / principios de primavera. Generalmente no se le considera un
agente primario en las infecciones del tracto respiratorio inferior, esto cambia al referirse a individuos mayores de 50 años, donde es
considerado un patógeno primario de vías respiratorias bajas. Se puede decir que es un agente oportunista que se aprovecha de las
condiciones predisponentes del huésped para causar enfermedad y formar parte de los patógenos humanos
emergentes. DIAGNOSTICO: La otitis media y sinusitis: M. catarrhalis es la tercera causa más frecuente de otitis media en los niños y es
importante aislarlo de los casos de sinusitis aguda y crónica. Conjuntivitis en neonatos. Traqueobronquitis purulenta, bronquitis,
neumonía: La mayoría son pacientes > 50 años de edad y con EPOC (a menudo con el cáncer de pulmón también). Laringitis,
queratinitis, uretritis. Los síntomas son de leves a moderados, la enfermedad invasiva (por ejemplo, enfisema) es poco frecuente.
Laboratorio: Se cultivan muestras de esputo, sangre, secreciones bronquiales, aspirado transtraqueal, lavado broncoalveolar y biopsia
pulmonar. Un método de diferenciación con la Neisseria, es que la Moraxella Catarrhalis no fermenta los carbohidratos y produce
DNasa. Además produce Butirato esterasa, que constituye la base de la prueba fluorométrica rápida para identificarla. En la tinción
aparecen bacilos, cocobacilos o cocos pequeños Gramnegativos. TRATAMIENTO: Este microorganismo presenta una alta resistencia a
beta-lactámicos como la penicilina, ampicilina y amoxicilina. Se puede usar amoxicilina/ac. clavulánico, cefalosporinas de segunda y
tercera generación, TMP-SMZ (mezcla de una parte de trimetoprim y cinco partes de sulfametoxazol). Esta bacteria presenta
sensibilidad a la quinolonas, eritromicina, tatraciclina, amikacina, imipenem, meropenem, cloranfenicol.

CASO CLINICO
Masculino de 15 años, fue ingresado con una historia de 3 días de dolor de cabeza y de 2 días de petequias generalizadas, náuseas y
vómitos. Temperatura 36,9 ℃ , FC 93 latidos / min y la TA 125/82 mmHg. Estaba alerta y se quejó de rigidez en el cuello. No hubo
ninguna secuela motora o sensorial , y el resto de su examen físico sin complicaciones. Leucocitos periféricos de 30.290 / l (96 %

CURSO ENARM CMN SIGLO XXI TEL: 36246001 Pharmed Solutions Institute PÁGINA 136
MANUAL DE TRABAJO DEL CURSO ENARM CMN SIGLO XXI
neutrófilos), hemoglobina de 15,4 g / dl, y plaquetas de 151.000 / l . CSF tenía un aspecto turbio con un aumento de la concentración
de proteína de 575 mg / dl, leucocitos de 35.500 / l ( 90 % de neutrófilos ) y glucosa de 10 mg / dl.
PREGUNTA
Cual es la conducta antibiótica mas apropiada para el caso?

RESPUESTA
a.- Ampicilina y acido clauvulanico.
b.- Ceftazidima y netilmicina.
c.- Quinolonas e imipenem.
d.- Eritromicina y meropenem.

CASO CLINICO
Masculino de 81 años, con antecedentes de cáncer de páncreas y cirrosis hepática por virus de la hepatitis C. Fue ingresado por
inversión del ciclo vigila sueño. A su ingreso, tenía ictericia esclerótica. Leucocitos 6950 / l (73% neutrófilos), hemoglobina de 12,8 g / dl,
plaquetas de 60.000 / l. LCR estaba claro con proteína de 38 mg / dl, glóbulos blancos de 9/μL, glucosa de 76 mg / dl, nitrógeno ureico /
creatinina 41.8/2.2 mg / dl, amoníaco de 287 mg / dl, bilirrubina total de 3,1 mg / dl.

PREGUNTA
Cual es la conducta antibiótica mas apropiada para el caso?

RESPUESTA
a.- Ampicilina.
b.- Ceftaxima.
c.- Imipenem.
d.- Eritromicina.

BRUCELOSIS (BRUCELLA MELITENSIS):


CIENCIAS BÁSICAS: La Brucella es una bacteria gramm negativa, viven en el interior de las células del sistema fagocitico mononuclear
(intracelularfacultativo), por ello tiene un curso recidivante, tratamiento difícil y prolongado. Todas las especies de Brucella tienen
como reservorio las ubres o útero gestante de las hembras y los genitales del macho. La B. melitensis (caprinos y ovinos), es el agente
causal más importante, seguido de B. abortus (bovinos) y B. suis (porcinos) y B. canis (de perros). La brucelosis es una zoonosis
sistémica en el humano puede afectar cualquier órgano o sistema, generalmente adopta un curso crónico, ataque al estado general,
baja letalidad. SALUD PÚBLICA: OMS, la considera la zoonosis de mayor distribución en el mundo. Cada año ocurren medio millón por
lo menos de casos nuevos. En México existe en todo el territorio nacional predominando en un área triangular con la base en la
frontera norte y el vértice en el centro. PATOGENIA: La brucelosis se transmite por ingestión, inhalación o exposición percutánea o
mucosa, la enfermedad en los seres humanos por lo general se asocia con la exposición a animales infectados o sus productos, ya sea
en lugares de trabajo (por ejemplo, el trabajo de los mataderos, la agricultura) o el contexto familiar (por ejemplo, el consumo de
alimentos, especialmente productos como leche no
pasteurizada, queso, mantequiilla) contaminados. BRUCELOSIS TUBERCULOSIS
Ya en el organismo son fagocitados por leucocitos, SITIO Lumbar y otros Dorsolumbar
LAS VÉRTEBRAS Múltiple o contiguas Contiguo
PMN, macrófagos tisulares, donde pueden
DISQUITIS Tarde Temprano
sobrevivir y multiplicarse. Una vez rebasada la CUERPO Intacto hasta el final Morfología de perdida temprana
barrera linfática, llegan a la circulación sistémica, y COMPRESION DEL Raro Común
de esta manera son transportados a los diferentes CANAL
órganos del sistema fagocitico mononuclear EPIFISISTIS Anterosuperior Signo de General: las regiones de disco
Pom´s superior e inferior , central,
(hígado, bazo, medula ósea, ganglios). La
subperióstico
persistencia de Brucella dentro de las células es OSTEOFITOS Anterolateral (pico de Inusual
debido a la inhibición de la fusión fagosoma- loro)
lisosomal, la degranulacion y la activación del DEFORMIDAD En cuña poco comun Cuña anterior, giba
sistema mieloperoxidasa. La liberación de las ABSCESOS Pequeños, bien Pérdida Común y discreta, apófisis
bacterias de las células necróticas puede sobrepasar PARAVERTEBRALES localizados transversa
RECUPERACIÓN Esclerosis, todo el variable
la capacidad fagocitica. Las manifestaciones clínicas cuerpo
están determinadas en gran parte por la liberación
de una endotoxina y el grado de hipersensibilidad a los antígenos brucelares. DIAGNOSTICO: Clínico: El período de incubación de 1
semana hasta varios meses es seguido por el desarrollo de la fiebre ondulante, sudores, aumento de la apatía, fatiga y anorexia y
síntomas inespecíficos como dolor de cabeza, mialgias y escalofríos. La brucelosis menudo se presenta con uno de los tres patrones:
una enfermedad febril similar pero menos grave que la fiebre tifoidea, fiebre y monoartritis aguda, típicamente de la cadera o de la
rodilla, en un niño pequeño (artritis séptica), o la fiebre de larga duración y baja de la espalda o dolor en la cadera en un hombre mayor
(osteomielitis vertebral). Brucelosis puede causar linfadenopatía, hepatoesplenomegalia, epididimoorquitis, compromiso neurológico y
el absceso focal. El microorganismo se cultiva con éxito en el 50-70 % de los casos, pero la cultura de identificación suele tardar hasta 6
semanas. El diagnostico de certeza es el aislamiento de brucella, en hemocultivo o cultivo de medula ósea. Fiebre prolongada
acompañada de ataque articular nos hace sospecharla. Ensayos de aglutinación para IgM son positivos en la infección temprana. Los
títulos individuales de ≥ 1:160 y 1:320-1:640 son diagnósticos en áreas no endémicas y endémicas, respectivamente. La brucelosis se
debe distinguir de la tuberculosis, y si esta distinción no es posible, el régimen debe adaptarse para evitar la monoterapia inadvertida
para la tuberculosis. Brucelosis tiende a causar menos destrucción del hueso y la articulación de la tuberculosis. TRATAMIENTO: Se
recomienda como primera elección estreptomicina a una dosis de 750 mg a 1 g al día (o gentamicina a 5-6 mg / kg al día) durante 14-21

CURSO ENARM CMN SIGLO XXI TEL: 36246001 Pharmed Solutions Institute PÁGINA 137
MANUAL DE TRABAJO DEL CURSO ENARM CMN SIGLO XXI
días más doxiciclina en una dosis de 100 mg C/12 hrs durante 6 semanas. Alternativa: TMP/SFX + Rifampicina por 6 semanas.
Enfermedad compleja (por ejemplo, enfermedad neurológica significativa o endocarditis) requiere al menos 3-6 meses de tratamiento
con múltiples agentes. Alternativa: rifampicina (600-900 mg / d) más doxiciclina (100 mg bid) durante 6 semanas. El trimetoprim
/sulfametoxazol, se puede dar en vez de doxiciclina - por ejemplo, a los niños o las mujeres embarazadas. La recaída se produce en
aproximadamente el 30 % de los casos, por lo general debido a la falta de cumplimiento. El paciente debe controlarse por lo menos 2
años.

CASO CLINICO
Un paciente varón de 20 años de edad, que se acababa de alistar en el ejército, se presentó refiriendo tos, expectoración, dolor
punzante en el hemitórax izquierdo, sudoración nocturna, anorexia, fiebre ondulante y disnea de casi 10 días de evolución. También
refirió la pérdida de 3kg de peso en el último mes. Presentaba fiebre de 38,4°C y su frecuencia respiratoria era de 22/min. La
auscultación torácica reveló estertores inspiratorios basilares bilaterales. Se detectó un recuento de leucocitos de 27.930/mm3 y una
velocidad de sedimentación de 68mm/h. El valor de proteína C reactiva era de 141mg/dl. La radiografía de tórax mostró un infiltrado
neumónico en la zona media derecha y una opacidad homogénea en la zona inferior izquierda. Se documentó el crecimiento de
Brucella spp. en el hemocultivo.

PREGUNTA
Cual es la conducta antibiótica mas apropiada para el caso?

RESPUESTA
a.- Rifampicina
b.- Estreptomicina
c.- Gentamicina
d.- Doxiciclina

CASO CLINICO
Varón de 74 años había trabajado, la mayor parte de su vida, en criaderos de aves de zonas rurales del Área Metropolitana. Su cuadro
clínico se había iniciado aproximadamente tres meses antes con compromiso moderado del estado general, fiebre vespertina y pérdida
de peso, que en ese momento cuantificaba en diez kilos. Sólo se verificó la existencia de fiebre irregular, alrededor de 38°C, y
decaimiento. Se inició estudio del síndrome febril prolongado. El hemograma mostraba anemia moderada (hematocrito 34,1%,
hemoglobina 11,3 gr%), leucocitos en rango normal, neutrofilia de 76%) sin desviación a izquierda y VHS de 36 mm/h. Albuminemia (3,3
gr%), la proteína C reactiva moderadamente elevada.

PREGUNTA
Cual es la conducta antibiótica mas apropiada para el caso?

RESPUESTA
a.- Rifampicina
b.- Estreptomicina
c.- Gentamicina
d.- Doxiciclina

TULAREMIA (FRANCISELLA TULARENSIS):


CIENCIAS BASICAS: Las infecciones humanas causadas por Francisella tularensis, que es una bacteria facultativa intracelular, se
producen a través de la interacción cuando el insecto muerde, o chupa sangre (especialmente las garrapatas y las moscas tabanid),
animales salvajes o domésticas (por ejemplo, conejos salvajes, ardillas), o el medio ambiente. El organismo puede persistir durante
meses en el barro, agua, y los cadáveres de animales en descomposición. Más de la mitad de los casos en Estados Unidos se producen
en Arkansas, Oklahoma y Missouri. PATOGENIA: La F. tularensis entra en la piel o las membranas mucosas a través de mordeduras o
rasguños inaparentes o se adquiere por inhalación o ingestión. Su patogenicicdad se debe a su capacidad para sobrevivir dentro de
macrófagos no estimulados. Los macrófagos fagocitan con facilidad a la F. tularensis pero esta resiste la muerte frente a los radicales
derivados del oxigeno, una respuesta neutrofilica local contraresta la infección, esta respuesta evoluciona a una ulcera. DIAGNOSTICO:
El período de incubación es de 2-10 días. La tularemia a menudo comienza con un comienzo agudo de fiebre, escalofríos, dolor de
cabeza y mialgias. Uno de varios síndromes puede desarrollar: Tularemia Ulceroglandular / glandular (75-85 % de los casos). El sello es
una induración, eritema, úlceras que no sanan 1-3 semanas duraderos (forma ulceroglandular) que comienza como una pápula
pruriginosa, ulcera, bordes drásticamente marcados y un exudado amarillo, y desarrolla una base de negro. Una lesión primaria de piel
puede no ser evidente en el 5-10% de los casos (forma glandular). Linfadenopatía está relacionada con la ubicación de la picadura de la
garrapata; linfáticos inguinales / femoral están afectadas con mayor frecuencia en los adultos debido a la frecuencia de picaduras en las
piernas. Los ganglios linfáticos pueden llegar a ser fluctuante y drene espontáneamente. Tularemia oculoglandular: Infección de la
conjuntiva, por lo general por el contacto con los dedos contaminados, resultados en conjuntivitis purulenta con adenopatía regional y
dolor debilitante. Doloroso linfadenopatía preauricular es único a la tularemia. Orofaríngea y gastrointestinal: Adquirida por ingestión,
la infección se puede presentar con faringitis y adenopatía cervical, ulceraciones intestinales, adenopatías mesentéricas, diarrea,
náuseas, vómitos y dolor abdominal. Tularemia pulmonar: La infección se adquiere por inhalación o por vía hematógena. El paciente se
presenta con una tos no productiva, disnea, dolor torácico pleurítico, infiltrados irregulares o lobar bilaterales, derrame pleural y
empiema ocasionales en la radiografía de tórax. Tularemia tifoidea: Consta de fiebre y signos de sepsis y sin hallazgos focales.
Laboratorio: Tinción policromática de muestras clínicas (tinción de poca ayuda). Serología mediante microaglutinación o la prueba de
aglutinación en tubo. Un solo título de ≥ 1:160 o un aumento de cuatro veces en el título después de 2-3 semanas se consideran

CURSO ENARM CMN SIGLO XXI TEL: 36246001 Pharmed Solutions Institute PÁGINA 138
MANUAL DE TRABAJO DEL CURSO ENARM CMN SIGLO XXI
positivos. La cultura es difícil y supone un riesgo importante para el personal de laboratorio. Métodos de reacción en cadena de la
polimerasa (PCR) se han utilizado para detectar el ADN de F. tularensis en muestras clínicas. TRATAMIENTO: La gentamicina se
considera el fármaco de elección para adultos (5 mg / kg al día C/12hrs) y niños (2,5 mg / kg tres veces al día o 5 mg / kg c/12) con
tularemia. Estreptomicina (1 g cada 12 horas) también es eficaz, pero no es tobramicina. Desaparición de la fiebre por lo general ocurre
dentro de 2 días, pero la curación de las lesiones de la piel y los ganglios linfáticos puede tardar 1-2 semanas. Late supuración de
ganglios linfáticos puede ocurrir, con el tejido necrótico estéril. Enfermedad leve a moderada responden rápidamente a tratamiento
pueden ser tratados durante 5-7 días, de lo contrario, el tratamiento se administra durante 7-10 días. Las alternativas incluyen
tetraciclinas o cloranfenicol (tasas de recaída de hasta el 20 %). Las fluoroquinolonas han demostrado ser prometedores, pero los
ensayos clínicos están pendientes.

CASO CLINICO
Se trata de un paciente varón de 36 años, trabajador en un matadero industrial de aves. Entre sus aficiones destaca la caza de liebres,
conejos y jabalíes, además de su hobby como taxidermista (su última pieza disecada fue un zorro). No tiene perros ni gatos y niega
haber recibido transfusiones o tener otras enfermedades en curso. El paciente acudió al médico por presentar fiebre de 40 grados,
sudoración, quebrantamiento general, mialgias, odinofagia y cefalea. Se interpretó el cuadro clínico como un síndrome gripal y se trató
con paracetamol. La fiebre persistió durante los siguientes 3 días, y por ello acudió al hospital. Tras una exploración física y una
radiografía de tórax normal, continuó con el mismo diagnóstico y tratamiento. Aproximadamente una semana después del comienzo de
los síntomas, presentó una tumoración dolorosa en la axila derecha que aumentó de tamaño progresivamente hasta alcanzar los 5 cm
de diámetro. De las pruebas complementarias que allí se le realizan destacamos las siguientes: la radiografía de tórax es normal, la
analítica presenta neutrofilia con desviación a la izquierda y la ecografía axilar derecha revela un conglomerado de adenopatías de 5 x 3
cm.

PREGUNTA
Cual es la conducta antibiótica mas apropiada para el caso?

RESPUESTA
a.- Rifampicina
b.- Estreptomicina
c.- Gentamicina
d.- Doxiciclina

SINDROME DE INMUNODEDICIENCIA ADQUIRIDA (SIDA); VIRUS DE LA INMUNODEFICIENCIA HUMANA (VIH)


CIENCIAS BASICAS: SIDA; Alteraciones inmunitarias profundas, infecciones bacterianas recurrentes o por gérmenes oportunistas y a
aparición de formas comunes de neoplasias malignas, causada por VIH -1 (América, Europa, Caribe), VIH-2 (oeste de África). El VIH
pertenece a la familia de retrovirus (poseen enzima transcriptasa reversa) citopaticos, no transformantes denominados lentivirus. Las
regiones codificadores del VIH son los genes: gag=poliproteina precursora, que es escindida para proteínas de la capside,
pol=transcripatasa inversa, integrasa y proteasa, env=proteínas de la cubierta. SALUD PUBLICA: Los primeros casos de SIDA, fueron
reconocidos en los Angeles en 1981. Según el CENSIDA, los casos acumulaos suman, hasta junio del 2007, 112,830, de los cuales 2,720,
ocurren en el grupo de los menores de 15 años. Proporción hombre: mujer 6:1, en el año 2003, para el grupo más afectado (25-39
años). El riesgo de transmisión vertical del VIH-1 de una madre infectada al producto varía entre 11-50%. La transmisión sexual en
hombres aun es predominantemente homo-bisexual. PATOGENIA: La infección por VIH, puede adquirirse por transmisión sexual (80-
90%) exposición parenteral a sangre o derivados (por agujas en usuarios de drogas, o trabajado a de la salud), y de madres infectadas a
sus productos, durante el período perinatal. Rutas menos frecuente a través de trasplante de órganos, tejidos y semen contaminado. El
riesgo de adquirir infección depende del tipo de exposición (transfusión de un donador seropositivo es de 100%, exposición única a
aguja contaminada es de 0.4%). La transmisión del VIH de la madre al niño además de in útero, puede ocurrir intraparto, por exposición
del producto a la sangre materna o las secreciones genitales infectadas, esta transmisión se puede evitar mediante estrategias de
profilaxis con antiretrovirales durante el embarazo y parto así como mediante la realización de cesárea programa en mujeres infectadas
con VIH-1, con carga viral de >1,000 copias, a la semana 38 de gestación antes de que se haya desencadenado el TDP y de que se
presente ruptura de membranas. Existe un riego adicional por lactancia materna del 14% en casos de infección establecida y 29% de
infección primaria. No hay evidencia que el VIH se pueda transmitir entre los miembros de una familia, a través de contacto estrecho o
por compartir utensilios. El VIH se une a la célula blanco (tiene tropismo por los T CD4), que tiene un receptor de alta afinidad de la
glicoproteina de la envoltura gp120 del virus, lo que permite su unión, una vez unido se produce la fusión de su envoltura externa del
virus a la membrana celular a través del gp41 con lo que el VIH entra a la célula, pierde su envoltura proteica, libera su ARN y las
proteínas codificadas por la región Pol, como la transcriptasa inversa, en el citoplasma. Esta enzima dirige la construcción de una
cadena de ADN sobre la plantilla de ARN viral, y en segundo paso cataliza la copia de la cadena recién sintetizada para producir un ADN
de cadena doble (proviral). Este ADN proviral se integra en el ADN cromosómico de la célula huésped. Después de su integración, el
ADN proviral del VIH pasa a un estado productivo, en el cual el ADN proviral se transcribe en ARN viral y ARN mensajero, el cual
codifica la si resistí de proteínas virales, necesarias para la replicación viral, mediante el uso de las funciones metabólicas de la célula
huésped. Los estadios finales comprenden el proceso de ensamblaje o encapsulación viral y la protrusión en la superficie celular; en
esta etapa el virus adquiere una envoltura lipidica, siendo finalmente liberado para infectar a otras células y repetir el ciclo. Durante la
fase asintomática de VIH (Clínica latente), los niveles circulantes del virus y las cuentas de CD4 permanecen esencialmente constantes
(equilibrio); el grado de infección de las células es muy semejante al grado de muerte de las células infectadas. Con la replicación viral,
la célula experimenta en pocos días degeneraciones balóninformes (tumefacción celular por alteración en la permeabilidad de la
membrana), lo que conduce finalmente a la muerte celular. Las células infectadas pueden volverse más susceptibles a la súper
infección por otros patógenos, lo cual puede llevar más rápidamente a la depleción de células T, que son las células críticas de la
activación de la respuesta inmunitaria especifica, tanto celular como humoral. Esta inmunosupresión adquirida, explica la gran

CURSO ENARM CMN SIGLO XXI TEL: 36246001 Pharmed Solutions Institute PÁGINA 139
MANUAL DE TRABAJO DEL CURSO ENARM CMN SIGLO XXI
susceptibilidad a presentar infecciones por gérmenes poco frecuentes, como neoplasias malignas. No solo los linfocitos CD4 se afectan,
también monocitos y macrófagos pueden ser infectados por el VIH. La afección de los macrófagos alveolares, explica la susceptibilidad
a neumonía por Pneumocystis jiroveci. La alteración funcional de las células fagociticas también compromete a los neutrófilos PMN, lo
cual explica la susceptibilidad a gérmenes piogenos. También hay alteración en linfocitos B, las cuales consisten en activación policlonal
de estas células, por lo que se presenta hipergammaglobulinemia haciendo susceptible a gérmenes encapsulados como Streptococcus
pneumoniae y Haemophilus influenzas.
DIAGNOSTICO: Clínico: La enfermedad de SIDA ocurre cuando el huésped infectado, ya con graves alteraciones en sus mecanismos de
defensa, no puede controlar microorganismos oportunistas o neoplasias malignas que rara vez causan enfermedad en el individuo
inmunocompetente. Las manifestaciones especificas más frecuentes son: infección primaria=síndrome retroviral agudo,
dermatológicamente eritema maculopapular y ulceración mucocutanea, además, fiebre, faringitis, linfadenopatía, artralgias, mialgias,
anorexia, náusea, pérdida de peso, diarrea intermitente. A nivel neurológico puede haber; Encefalopatía; cuando VIH llega a SNC, en la
fase temprana hay disminución de la capacidad de concentración, perdida de la memoria y lentitud mental, hipertrofia, ataxia,
temblores, en la fase tardía incrementan todos los síntomas y signos, puede haber convulsiones y psicosis, hay datos de extrema atrofia
cerebral y son común es los cambios en la sustancia blanca. La incidencia de gastroenteritis es hasta de 77% cursan con diarrea (crónica
o intermitente) en algún momento, la causa más frecuente es Cryptosporidium. La infección por mycobacterium tuberculosis es la
mayor complicación
oportunista que ESTADIOS CLÍNICOS DE LA INFECCIÓN POR EL VIH POR LA CDC 1993
ESTADIO A ESTADIO B: Denotan progresión de la ESTADIO C: Define un caso de SIDA por medio de las entidades
afecta a los adultos
enfermedad, pero que por sí solas no clínicas asociadas que se presentan
con VIH, con cualquier clasifican al paciente como enfermo de
recuento de CD4 y SIDA
puede incrementar el Infección o Sintomático (sin condición A ni C) Candidiasis de bronquios, tráquea o pulmones
riesgo de Síndrome Candidiasis orofaríngea o vaginal, por más Candidiasis esofágica
retroviral de un mes o que responde pobremente al Cáncer cervical invasivo
complicaciones y agudo (SRA) tratamiento Coccidiodomicosis diseminada
mortalidad asociada a Infección Leucoplasia vellosa de la lengua Criptococosis crónica intestinal (>1mes)
VIH. Muchas de las asintomática Displasia cervical severa o carcinoma in CMV en cualquier órgano (excepto hígado, bazo o ganglios)
infecciones Linfadenopatía situ Encefalopatía asociada a VIH
generalizada Enfermedad pélvica inflamatoria Herpes simplex, causando ulceraciones crónicas por más de un mes
oportunistas en el
persistente Herpes zoster en más de una dermatoma o Histoplasmosis diseminada
adulto son (LGP) por más de 2 episodios Isosporosis crónica por más de un mes
secundarias a Purpura trombocitopenica idiopática Sarcoma de Kaposi
reactivación de Neuropatía periférica Linfoma de Burkitt, inmunoblastico o linfoma primario de cerebro
patógenos Listeriosis Mycobacterium tuberculosis pulmonar o extrapulmonar
Angiomatosis bacilar Mycobacterium avium intracellulare
oportunistas como: Síndromes constitucionales (diarrea por Otras micobacterias diseminadas
Mycobacterium mas de 1 mes, fiebre prolongada de origen Neumonia por P. jiroveci
avium, candida, desconocido) Leucoencefalopatia multifocal progresiva
criptococcus Septicemia recurrente por Salmonella
Toxoplasmosis cerebral
(meningitis con CD4 < Síndrome de emaciación con pérdida de peso de >4.5 kg o más de
50 cels/mcl), herpes 10% del peso usual.
simple, toxoplasma,
histoplasma, CMV (coriorretinitis, perdida de visión, infiltrados amarillentos y/o hemorragias). Toxoplasma (causa mas común de déficit
focal del SNC, cefalea confusión, < 50cels/mcl, TAC lesiones multiples en anillos, tx. Primetamina+ sulfadiazina+ leucovorin por 6 sem),
candida albicans (candidiasis esofagiga definitoria de SIDA, tx. Fluconazol 14-21 dias) adquiridos previamente a la infección por VIH. De
acuerdo al INDRE, las infecciones oportunistas más frecuentes en pacientes con SIDA en México son: 1. Cándidiosis (37.6%) 2.
Neumonía por Pneumocystis jiroveci (12.2%). Tuberculosis (11.1%) herpes simple (7.4%), y Herpes zoster (7.2%). Otras manifestaciones
características son las neoplasias malignas. El Sarcoma de kaposi (neoplasia mas común en VIH, nódulos, placas, alargamiento de
linfonodos), se observa principalmente en adultos, siendo su presentación poco frecuente en niños. Dentro de este grupo se incluyen
las neoplasias del SNC, como los linfomás no Hodking del cerebro (200 veces más común en pacientes con VIH, origen de cel. B,
síntomas fiebre, diaforesis, perdida de peso). Laboratorio: la demostración de anticuerpos anti-VIH puede realizarse por varios
métodos, como ELISA más utilizado, sensibilidad 95%, especificidad 99%, en las primeras 8-12 semanas puede dar negativo , de ahí que
si ELISA sale positiva, se recomienda realizar una segunda prueba más especifica (prueba confirmatoria) y Western el cual reacciona
con los antígenos virales, esta prueba tiene especificidad del 100%. Se considera como persona infectada por VIH o serio positiva
aquella que presente: a) dos resultados positivos de pruebas de Tamizaje positivos (ELISA, aglutinación o pruebas rápidas) y pruebas
confirmatorias de Western Blot positiva. b) dos resultados. Pruebas positivas de Tamizaje, con cuadro clínico sugerente de infección y c)
alguna prueba
suplementaria positiva (cultivo, Ag p24, PCR). EN infecciones oportunistas, Diagnostico de SIDA: infección declaratoria y CD4 <200/ml.
NEUMONÍA POR Pneumocystis jiroveci: Inicio insidioso, fiebre, tos no productiva, disnea, hipoxemia, linfadenopatía generalizada,
diaforesis nocturna. Rx., de tórax infiltrados intersticiales bilaterales, neumótorax espontáneos, DHL elevada, CD4 <200cel/ml, lavado
bronco alveolar 85-90% sensibilidad, tratamiento de
Sistema de clasificación para la infección por VIH/SIDA revisada en 1993. (categoría clínica)
elección TMP/SFX 15mg/kg/d 3-4 dosis por 21 días, se Número de células CD4 A B C
ha demostrado que agregar esteroides al manejo > = 500/mm3 A1 B1 C1
incrementa la sobre ida en pacientes con procesos 200-499/mm3 A2 B2 C2
moderados- severos. Se debe dar profilaxis con <200/mm3 A3 B3 C3
cuentas de CD4 menores de 200/mm3, suspenderla si alcanzan >200. CLASIFICACIÓN:
TRATAMIENTO: Los antiretrovirales (ARV) se debe iniciar en adultos asintomáticos con linfocitos CD4 <500-µl y en aquellos pacientes
sintomático, independientemente de la cifra de estos, en síndrome de infección aguda, en infección crónica (sintomática, o
asintomática con CD4 <350/µl o decreciendo y VIH RNA >50,000 copoias por ml o incrementando), py profilaxis postexposicion.
Actualmente se dispone de los siguientes AVR: 1. Inhibido res de la transcriptasa reversa análogos de núcleosidos; zidovudina 200mg

CURSO ENARM CMN SIGLO XXI TEL: 36246001 Pharmed Solutions Institute PÁGINA 140
MANUAL DE TRABAJO DEL CURSO ENARM CMN SIGLO XXI
c/8hrs o 300 c/12hrs, toxicidad anemia, granulocitopenia, miopatia, didanosina 200 mg c/12hrs, toxicidad pancreátitis, neurópatia
periférica, lamivudina, estavudina, abacavir, emitricitabina 2. Inhibidores no núcleosidos de la transcriptasa reversa: nevirapina (NVP),
toxicidad sx. Steven Johnson, sedación, hepatitis, efavirenz (EFV). Inhibidores de la proteasa; indavir (IDV) 800mg VO c/8hrs, toxicidad
sabor metálico, nausea, dolor abdominal, ritonavir (RTV), sáquinavir, amprenavir, lopinavir. Inhibidores de fusión: efuvirtide o T-20. EN
pacientes con SIDA la fibra es un motivo frecuente de consulta y hospitalización, determinar si la infección no se debe a bacterias, los
agentes frecuentes del proceso infecccioso pueden ser Streptococcus pneumoniae, Haemophilus, Salmonella, Staphilococcus.
Tuberculosis: algunos medicamentos antifimicos interaccionan con los ARV, en este caso retrasar el inicio de los ARV de 2-8 semnas. Se
puede presentar el síndrome de reconstitución inmune: 8-43% de pacientes que inician terapia antirretroviral con Tb activa; se
manifiesta con fiebre, linfadenopatía, empeoramiento de infiltrados pulmonares, derrame pleural.

CASO CLINICO
Se trata de masculino de 31 años de edad el cual acude a consulta a solicitud de su esposa, ya que ha presentado cambios en su estado
cognitivo y alteraciones en sus actividades de la vida diaria, el paciente cuenta con antecedentes de VIH positivo de diagnostico
reciente, la esposa del paciente ha notado una disminución significativa de las actividades de su esposo en la casa, ha sido sancionado
en su trabajo, además se encuentra con afecto embotado, con disminución de la atención, no logra concentrarse, pierde el nucleo del
discurso, actualmente su cuenta de CD4 es de 380 con una carga vírica de 78 000/ml, el paciente presenta a la exploración física signos
vitales sin alteraciones, el examen neurológico no muestra datos de focalización, fondo de ojo normal, minimental 22/30, fue enviado a
IRM con atrofia cerebral desproporcionada e inespecífica sin lesiones focales.

PRECUNTA
Considerando el cuadro clínico cual es la conducta a seguir más apropiada.

RESPUESTA
a.- Terapia antirretrovirica.
b.- PCR para virus JV en LCR.
c.- PCR para micobacteria en LCR.
d.- VDRL para LCR.

CASO CLINICO
Paciente de 30 años, procedente de la ciudad de pergamino, provincia de Buenos Aires, que es ingresado por presentar fiebre, astenia,
tos productiva, disnea progresiva y sudoración nocturna. Heterosexual, niega consumo de alcohol y drogas ilícitas, conoce su condición
de VIH positivo desde hace 7 años. En enero de 2006 se le diagnostica candidiasis esofágica. Recibió diversos esquemas de terapia
antirretroviral de gran actividad (TARGA) con mala adherencia y consecuentes fracasos terapéuticos.

PREGUNTA
Cuál es el agente causal más probable en este caso.

RESPUESTA
a.- Cryptococcus neoformans.
b.- Aspergillus fumigatus.
c.- Mycoplasma kansasii.
d.- Pneumocystis jiroveci

GONORREA (NEISSERIA GONORRHOEAE):


CIENCIAS BASICAS: Enfermedad de transmisión sexual altamente contagiosa y cosmopolita, causada por Neisseria gonprrhoeae, es un
diplococo gramnegativo, aerobio, inmóvil, oxidasa y catalasa positivo, que produce inflamación de los epitelios columnares y
transicionales de la uretra, el recto la faringe y la conjuntiva. Es altamente susceptible a condiciones adversas del ambiente. SALUD
PUBLICA: Gonorrea continúa ocupando el primer lugar entre las infecciones notificadas. En México la tasa de morbilidad por 100,000
hab es de 1.17 en 2006. El grupo etario con mayor morbilidad es de los 15-20 años. PATOGENIA: En la actualidad se tiende a considerar
que el portador “subclinico” (hombre o mujer) transmite la infección a la pareja, que podrá o no desarrollar sintomatología. Factores de
riesgo: trabajadores del sexo, homosexuales, viajeros, trabajadores migratorios, estudiantes, promiscuidad sexual. La gonorrea se
transmite por contacto directo, intimo generalmente sexual (más común de un hombre infectado a una mujer sana, que de una mujer
infectada a un hombre sano). Periodo de incubación de 2-5 días, a los componentes de la superficie del gonococo se les ha hecho
responsables de la integración con las células del hospedero, incluyendo adherencia e invasión a las células epiteliales. La vagina no es
infectada gracias al pH acido del moco vaginal que interfiere con la proliferación del gonococo y a la actividad de un sistema bactericida
mediado por peroxidasa y dependiente de pH. La proctitis gonocócica en la mujer tienen origen en la contaminación a partir de la via
genital o en el coito rectal, que es un mecanismo fundamental en el hombre; otra posibilidad es la ruptura de un absceso pélvico o
prostático. La diseminación a partir del sitio mucoso primario se realiza por dos vías: 1. Linfático; Que lleva a las bacterias a la próstata,
epidídimo, glandula de Skene, Bartholin, Cowper, piel de area genital, trompas de Falopio, peritoneo y por contigüidad al espacio
perihepatico y 2. Hemático; que puede acompañarse de artritis, endocarditis, meningitis y dermatitis séptica. DIAGNOSTICO: Clínica;
en el hombre la uretritis aguda, es la presentación más frecuente, se inicia con descarga uretral y disuria, secreción uretral mucoide,
que en pocas horas se torna purulento y blanco amarillento. La complicación más frecuente es la epididimitis, en casos poco frecuentes
puede haber linfangitis, absceso periuretral, prostatitis aguda, el paciente tienen un curso a febril. En la mujer el sitio principal de
infección es el endocervix, a veces en uretra, glándulas periuretrales y de Bartholin, hasta 90% de las mujeres puede ser asintomático.
Cuando hay síntomas se relacionan con cervicitis y uretritis, los síntomas incluyen leucorrea, disurias y sangrado intermenstrual.
Teniendo como antecedente la exposición orogenital puede encontrarse infección faringea en 10-20% de las mujeres, en 3-7% de

CURSO ENARM CMN SIGLO XXI TEL: 36246001 Pharmed Solutions Institute PÁGINA 141
MANUAL DE TRABAJO DEL CURSO ENARM CMN SIGLO XXI
heterosexuales y 10-25% de homosexuales. La conjuntivitis puede ser severa con exudado purulento y avanza rápidamente a ulceración
cornea. EPI, el riesgo de desarrollarla en una mujer con gonococo es de 10-20% se manifiesta con una combinación de endometritis,
salpingitis, absceso tubovarico. Las manifestaciones agudas y las secuelas a largo plazo, es una se las principales razones por la cual
debe prevenirse entre ellas histerectomías, embarazos ectopícos, con una frecuencia de 7 veces más que alguien sin EPI. La infertilidad
por obstrucción tubaria es de 4%, después de un episodio de EPI, 33% con dos ataques y 60% con tres cuadros. El Síndrome de FItz-
Hugh y Curtís se presenta por extensión de la bacteria de las trompas de falopio a la cápsula del hígado. En embarazo genera abortos
espontáneos, parto, parto preterimos, RPM, incremento en mortalidad perinatal. La bacteriemia se presenta en 0.5-3%, las
manifestaciones comunes son parte del síndrome artritis-dermatitis, con artralgias migratorias en rodillas, codos y articulaciones
distantes. La dermatitis característica se encuentra en 75% de las pacientes, son papilas y pústulas con un componente hemorrágico, en
número de 5-40 y predominan en extremidades. En pediátricos; conjuntivitis (oftalmía neonatorum), es la más reconocida y fue causa
de ceguera antes de la profilaxis con nitrato de plata, oh aplicación tópica con eritromicina y tetraciclina. Laboratorio: tinción de gramm
(diplococos intracelulares) y prueba de estearasa leucocitaria en la descarga uretral si la rincón es positiva y hay más de 5 leucocitos por
campo dar tratamiento para gonorrea y clamidia. Las muestras para aislamiento de N. gonorreae pueden obtenerse de tracto genital,
orina, ano, orofaringe, conjuntiva, glándulas de Bartholin, trompas de falopio, endometrio, liquido auricular, lesiones en piel. Se puede
usar un prueba directa para la detección de gonococo, que es la detección de antígeno y ácidos nucleicos (PACE). Anticuerpos
fluorescentes (prueba polivalente y anticuerpo monóclonal). TRATAMIENTO: El de elección en gonorrea aguda no complicada, es la
cefalosporinas, en uretritis, cervicitis y proctitis, dar tratamiento para clamidia ya que se reporta coinfeccion hasta en 50%. Primera
elección ceftriaxona 125mg IM una dosis o cefixime 400mg VO una dosis (por la alta resistencia a fluroquinolonas, ya no se
recomiendan), si hay resistencia ciprofloxacina 500mg VO una dosis u ofloxacina 400mgs. Para CLamydia una dosis de azitromicina 1 g
VO. Alternativas ceftizoxima, cefotaxima, cefotetan, probenecid. Infección gonococica diseminada (sepsis, artritis, meningitis);
ceftriaxona 1g IM o IV cada 24 hrs, alternativa cefotaxima 1 g IV cada 8 hrs un día o dos y continuar tratamiento por una semana VO
(cefixima 400mg c/12hrs). Próstatitis y epididimitis; ceftriaxona 250mgs IM dosis única mas doxiciclina 100mg VO c/12 hrs por 10 días.
Profilaxis en conjuntivitis neonatal es con eritromicina oftálmica (0.5%) o tetraciclina oftálmica (1%), uno u otro en una sola aplicación.

CASO CLINICO
Paciente mujer de 21 años remitida a nuestro hospital por su médico de familia con el diagnóstico de conjuntivitis aguda purulenta
bilateral, por no responder al tratamiento tópico con la asociación de gentamicina y dexametasona ni con la asociación de neomicina,
polimixina y gramicidina. Como antecedentes personales la paciente refiere que es alérgica a las cefalosporinas. En la exploración
presenta clínica en ambos ojos de edema palpebral severo, abundante secreción purulenta, y epitelitis con lesión superior yuxtalímbica
en la córnea de ojo derecho (OD) que implica capas profundas del estroma, no observándose ésta en el ojo izquierdo (OI). Se remiten
muestras de la secreción conjuntival al servicio de microbiología, y se pauta tratamiento empírico con ofloxacino y la asociación
cloranfenicol y dexametasona tópicos en horas alternas, con limpieza previa de las secreciones con suero fisiológico, y por vía enteral
con ciprofloxacino, betametasona y dexclorfeniramina. En el cultivo se identifica Neisseria Gonorrhoeae, y el antibiograma demuestra
resistencia al ciprofloxacino y tetraciclinas, la paciente es alérgica a las cefalosporinas.

PREGUNTA
Cual es la conducta mas adecuada a seguir?

RESPUESTA
a.- Agregar cloranfenicol.
b.- Agregar vancomicina.
c.- Agregar imipenem.
d.- Agregar doxiciclina.

SIFILIS (TREPONEMA PALLIDUM):


CIENCIAS BASICAS: La sífilis es una Infección de Transmisión Sexual (ITS) causada por la espiroqueta, Treponema pallidum. Mide
aproximadamente en promedio 10 micrones de largo. Esta bacteria solo vive en los humanos. Se multiplica por división simple y tiene
mucho requerimiento de alimentos especiales para crecer en los medios de cultivos para bacterias. SALUD PÚBLICA: Según datos de la
OMS, a nivel mundial existen 12 millones de nuevos casos de sífilis, correspondiendo a una incidencia mundial de la sífilis venérea del
0,4% y la prevalencia del 1%. De estos casos corresponde a Latinoamérica y el Caribe: 3 000 000 de casos por año. El período donde
más personas se contagian es entre los 20 y los 25 años de edad. Se reportan anualmente un promedio de 330.000 casos de sífilis en
mujeres embarazadas en América Latina y el Caribe que no reciben un tratamiento adecuado. Ello significa que todos los años nacen
110.000 niños con Sífilis Congénita. PATOGENIA: La Sífilis es transmitida a través del contacto sexual, ya sea por vía vaginal, anal u oral.
También por el contacto directo con la lesión o herida húmeda de la sífilis, a través de heridas escoriaciones o fisuras no evidentes.No
dispone de toxinas pero su poder invasivo es extraordinario. Las lesiones de sífilis se les conocen como “chancros”, aparecen
principalmente en los genitales y no presentan dolor razón por la cual muchas personas no buscan ayuda médica. También puede
pasarse al besar o a través del contacto manual u otro contacto personal cercano. Esta enfermedad se busca en toda gestante ya que
las mujeres embarazadas con sífilis pueden transmitir esta infección a sus bebés durante el embarazo, antes del parto, a esta forma de
enfermedad se llama Sífilis Congénita. Para la realización de transfusiones sanguíneas también se busca Sífilis en la sangre. Su
transmisión es baja porque solo puede vivir no más de 24 a 48 horas en la sangre que se conserva en el banco de sangre. En las
relaciones entre hombre y mujer es más fácil que se contagie el hombre. Una persona que ha tenido Sífilis se puede volver a contagiar
de esta infección. La sífilis tiene varias etapas: Incubación; 10-90 días con una mediana de 3 semanas. Sífilis Primaria; primera etapa,
chancro de inoculación, es una erosión mucosa superficial, bordes indurados, indolora con secreción serosa, acompañada de
adenopatía regional unilateral, mas frecuentes en genitales externos, cérvix, boca, región perianal y canal anal. Estas lesiones aparecen
en promedio de 2 a 3 semanas después del contacto con una persona infectada y se han descrito hasta 90 días después. Las úlceras
desaparecen en un período de 4 a 6 semanas. La Sífilis Secundaria; se presenta de 2 a 12 semanas después de la aparición del chancro,

CURSO ENARM CMN SIGLO XXI TEL: 36246001 Pharmed Solutions Institute PÁGINA 142
MANUAL DE TRABAJO DEL CURSO ENARM CMN SIGLO XXI
erupción simétrica maculopapular en todo el cuerpo que incluye palma de manos y planta de pies, en la cavidad oral se ulceran.
También pueden aparecer fiebre, inflamación de los ganglios, dolor de garganta, dolor de cabeza, pérdida de peso, dolores musculares
y fatiga. Los signos y síntomas de la Sífilis Secundaria desaparecerán con tratamiento o sin tratamiento, pero la infección progresará
hasta la fase latente (enfermedad más de un año) y terciaria de la enfermedad, si no se administra ningún tratamiento. La persona
infectada seguirá teniendo sífilis aun cuando no tenga ni signos ni síntomas; la infección permanece en el organismo. Sífilis Terciaria; en
ella la infección se puede dirigir al cerebro (sistema nervioso en general), al corazón, a la piel y a los huesos. Entre los signos y síntomas
se encuentran la dificultad para coordinar los movimientos musculares, parálisis, ceguera gradual, demencia y muchos otros signos
neurológicos que pueden causar la muerte de la persona infectada. DIAGNÓSTICO: Para el diagnostico de la Sífilis Primaria es requisito
acudir al médico quien mediante la observación y examen del material obtenido de un raspado de la lesión (chancro) en un microscopio
observara a los treponemas causantes de la sífilis. El microscopio que utiliza se llama microscopio de campo oscuro. También es posible
diagnosticar sífilis mediante exámenes de sangre. Existen pruebas llamadas No treponémicas como el RPR (Rapid Plasma Reagin) o
VDRL (Venerial Disease Research Laboratory) que permiten encontrar anticuerpos (defensas) producidas por el cuerpo contra el
Treponema. Otro examen llamado prueba treponémica FTA Abs (Fluorescent Treponemal Antibody Absorption) es una forma de
confirmar los hallazgos previos; es una prueba que indica que el individuo tuvo o no sífilis. Las mujeres embarazadas pueden infectar a
sus bebes con sífilis y esta infección podría causar la muerte del bebe. Por esta razón se toma a todas las mujeres gestantes estas
pruebas como parte obligatoria del control pre natal que llevan en los hospitales. Durante la gestación la madre puede contraer sífilis
pudiendo infectar así a su bebe. De acuerdo al tiempo de infección el niño puede nacer con múltiples alteraciones, incluso muerto. Las
madres con diagnóstico de sífilis deben de recibir tratamiento inmediatamente. Si tuvo tratamiento irregular (no completo) debe
dársele terapia a ella y al recién nacido. La sífilis del recién nacido recibe el nombre de Sífilis Congénita. Esta infección puede ser
temprana o tardía. La temprana, que se observa antes del segundo año de vida, puede ser fulminante. Puede manifestarse como una
infección generalizada, o por lesiones en piel y en áreas de mucosas (boca, ano), dolores óseos o articulares, anemia, crecimiento del
hígado y del bazo y afectación del sistema nervioso. La forma tardía, con una persistencia de más de dos años, puede originar
deformaciones de huesos y dientes, sordera, neurosífilis (al sistema nervioso) y otras manifestaciones terciarias. Las manifestaciones
clínicas son muy variables. En resumen, el niño puede tener síntomas que lo dejarían con consecuencias graves para su desarrollo
posterior. TRATAMIENTO: En los casos no tratados después de la fase de latencia, ocurren recaidas en una cuarta parte de los casos
que conducen a nueva fase de secundarismo. La sífilis tiene cura. Si una persona ha tenido sífilis durante menos de un año, la
enfermedad se curará con una sola inyección intramuscular de un antibiótico llamado penicilina (2.4 millones de penicilina G
benzatinica en dos dosis separadas, IM profunda). Si una persona ha tenido sífilis por más de un año, necesitará de tres dosis de este
antibiótico. En niños penicilina G benzatinica 50,000/kg IM una sola dosis. En etapas avanzadas, como en Sífilis Terciaria, 2.4 millones
de penicilina G benzatinica, semanalmente durante 3 semanas, no existen estudios que reporten eficacia de tratamientos alternativos o
en alérgicos a penicilina doxiciclina 100mgs c/12 hrs por 14 dias o tetraciclina 500mgs cuatro veces al día por 14 días. Lamentablemente
el antibiótico no revertirá las lesiones ya ocasionadas. Las personas que reciben tratamiento contra la sífilis deben abstenerse de tener
todo tipo de relaciones sexuales con parejas nuevas hasta que las lesiones sifilíticas hayan desaparecido por completo. La pareja sexual
debe someterse a las de pruebas y recibir tratamiento si es necesario. Seguimiento del tratamiento En todos los pacientes con Sífilis
Primaria y Congénita hay que repetir las pruebas no treponémicas cuantitativas (en los que salen valores como por ejemplo 1/8)
llamadas RPR o VDRL al cabo de uno, tres, seis y 12 meses del tratamiento de la sífilis. Al cabo de 12 meses puede haberse negativizado
el 40-75% de casos de sífilis primaria y el 20-40% de las secundarias. Si a los 12 meses siguen siendo positivas, se hace necesaria una re
evaluación.

CASO CLINICO
Masculino de 44 años, alérgico a la penicilina, acude a nuestra consulta con su mujer porque desde hace 2 semanas presenta una úlcera
en el glande no dolorosa y adenopatías inguinales. Niega haber tenido relaciones sexuales extramatrimoniales.

PREGUNTA
Cual es la conducta a seguir?

RESPUESTA
a.- Al negar relaciones sexuales extramatrimoniales podemos descartar enfermedades de transmisión sexual.
b.- Se trata de una sífilis primaria, trataremos con una dosis de penicilina G benzatina 2,4 MU i.m.
c.- Entre el diagnóstico diferencial de la úlcera genital hay las enfermedades de transmisión sexual. Debemos pedir serologías para
sífilis.
d.- Al tratarse de una lesión cutánea derivamos al paciente al dermatólogo de referencia.

PREGUNTA
Si finalmente fuera diagnosticado de sífilis primaria, cual es la conducta prescriptiva mas adecuada?

RESPUSTA
a.- Administraremos una dosis de penicilina benzatina 2,4 MU i.m.
b.- Administraremos penicilina benzatina 2,4 MU i.m. una administración semanal por 3 semanas.
c.- Doxiclina 100 mg vía oral dos veces al día por 14 días.
d.- Doxiciclina 200 mg oral dos veces al día durante 28 días.

CHLAMYDIA TRACHOMATIS:
CIENCIAS BÁSICAS: Es una infección de transmisión sexual bacteriana. La Chlamydia trachomatis, es un parasito intracelular obligado,
tiene al menos 18 serotipos, estos confieren tropismo celular y son específicos de enfermedad: serotipos A, B, Ba y C están asociados
con tracoma, los del D al K están asociados con infecciones de transmisión sexual y la perinatal, los tipos L1, L2 L3 son los más invasivos,

CURSO ENARM CMN SIGLO XXI TEL: 36246001 Pharmed Solutions Institute PÁGINA 143
MANUAL DE TRABAJO DEL CURSO ENARM CMN SIGLO XXI
se extienden al tejido linfático y producen el síndrome clínico de linfogranuloma venéreo y proctocolitis hemorrágica. SALUD PUBLICA:
Causa 30-40% de las uretritis no gonococica. La OMS calcula que existen 89 millones de casos de infección por Chlamydia trachomatis
en el mundo. Las secuelas que puede traer esta infección son las más graves y costosas de todas las ETS, exceptuando la infección por
VIH. Se ha demostrado que la infección por C. trachomatis, puede facilitar la transmisión por VIH. La exposición a chlamydia, es una
causa importante de infertilidad tubaria. Produce un 13.8% de conjuntivitis neonatal. PATOGENIA: la infección se adquiere por
contacto sexual, puede ocurrir colonización en el recién nacido de una madre infectada. Las infecciones más frecuentes son uretritis en
el hombre y cervicitis en la mujer. Estos sitios son la vía de entrada del organismo. Tiempo de incubación de uretritis es de 7-14 días, en
mujeres no está definido. La respuesta inmunológica parece tener un papel principal en la fisiopatogenia. Una proteína de choque de
calor, tiene 50% de homologa con otras proteínas de choque de calor humana. Por lo tanto esta respuesta inmune iniciada por la
infección puede dar lugar a respuestas dañinas. DIAGNOSTICO: Clínico; insidioso, hay secreción uretral o cervical hialina, disuria, ardor
a la micción, dispareunía ocasional, cervicitis y bartolinitis. La C. trachomatis no puede invadir el epitelio vaginal en mujeres adultas,
pero podría causar vaginitis en prepúberes. Exploración física, cérvix con edema, eritema e hipertrofia con descarga mucopurulenta. El
síndrome uretral agudo se define como disuria y polaquiuria, descarga uretral, que es blanca, gris o hialina no purulenta. En
homosexuales es común proctitis y proctocolitis, los síntomas iniciales son prurito anal y descarga rectal mucosa o mucopurulenta. La
mucosa se ulcera y se presenta un proceso inflamatorio crónico en la pared del intestino, con gránulosas no caseosas y abscesos en las
criptas. Hasta 8% de las mujeres desarrolla salpingitis, aunque se prefiere mas EPI, el espectro de EPI por Chlamydia va desde una
enfermedad grave, con perihepatitis y ascitis a la salpingitis silenciosa teniendo como secuela infertilidad, que se atribuye a
inflamación, cicatrización y oclusión tubaria. Los embarazos ectopícos se presentan 8 veces más en pacientes que han tenido infección
por Chlamydia. LINFOGRANULOMA VENÉREO: Es endémico en África, India, Sudamérica y Caribe, tiene 3 estadios: primero pequeña
pápula o ulcera herpetiforme, en la mucosa genital o piel adyacente, aparece entre 22-30 días después de adquirir la infección y
cicatriza sin dejar rastro. Segundo, se caracteriza por linfadenopatía y síntomas sistémicos (fiebre, cefalea, mialgias),en hombres los
linfonodos inguinales son los más afectados. Los linfonodos son blandos, con eritema leve, la inflamación se extiende y forma una
masa, los abscesos dentro de la masa coleasen y forman un bubón que se puede romper espontáneamente. Tercero, cronicidad de la
infección, con desarrollo de abscesos loculados, fístulas, estenosis asi como elefantiasis genital. La ruptura del bubón alivia los síntomas
y puede drenar durante semanas o meses. TRACOMA: Es una queratoconjuntivitis folicular crónica con neovascularizacion corneal, que
se produce por infecciones repetidas, puede producir ceguera como consecuencia. El tracoma es la principal causa de ceguera
infecciosa en el mundo. NEUMONÍA: curso insidioso, más común en niños de 1-3 meses de edad, caracterizados por accesos de tos
paroxistica, general ente sin fiebre. Laboratorio: La C. trachomatis solo se puede cultivar en células McCoy o HeLa, no se visualiza por
tinción gram, por lo que es necesario buscar las típicas inclusiones intracelulares mediante una tinción de Giemsa. ELISA para identificar
antígenos específicos. En caso de identificar IgG se evaluara el incremento de anticuerpos específicos de la fase aguda y en la fase de
convalecencia o se determinara IgM específica en la fase aguda. Cuatro tipo de procedimientos confirmatorios están disponibles: 1.
Evaluación por microscopia directa de los exudados o raspado del tejido utilizando tinción con anticuerpos fluorescentes. 2.
Aislamiento del organismo en cultivos celulares. 3. Detección de antígenos o genes de Chlamydia por medios inmunológicos o pruebas
de amplificación de ácidos nucleícos. 4. Pruebas serológicas en busca de anticuerpos contra C. trachomatis. TRATAMIENTO: para
infección genitourinaria en adolescentes mayores y adultos, se usa tetraciclina 500mg VO, cuatro veces al día por 7-10 días. Como
alternativa se puede utilizar doxiciclina, sulfonamidas o eritromicina, siendo esta ultima el tratamiento de elección en la embarazada.
En linfogranuloma venéreo los mismos medicamentos pero no menor a dos semanas. El recomendado es azitromicina 1gr VO dosis
única o doxiciclina 100mgs VO c/12 hrs por 7 días. Alterativas eritromicina o ciprofloxacina. Para el tracoma tetraciclina oral, mas tópica
o doxiciclina 100mg c/12hrs por 21 días. Para conjuntivitis neonatal 30-50mg/kg/día en 4 dosis durante 14 días, junto con tratamiento
oftálmico tópico con eritromicna o tatraciclina.

CASO CLINICO
Un hombre de 30 años de edad, con presencia de sangrado con moco por via rectal varias veces al dia por tres semanas. La
sigmoidoscopia reporto mucosa granular con varias úlceras a 5-7 cm distales del recto. La biopsia rectal reporto colitis irregular
severamente activa con criptas distorsionadas semejante a la criptitis de la enfermedad de Crohn. El historial clínico adicional reveló
que el paciente tuvo relaciones sexuales anales sin protección durante el viaje a través de Europa tres meses antes. Se confirmó la
presencia de C. trachomatis.

PREGUNTA
Cual es la conducta terapéutica a seguir mas adecuada?

RESPUESTA
a.- Tetraciclina
b.- Doxiciclina
c.- Eritromicina.
d.- Ciprofloxacino.

CASO CLINICO
Masculino de 28 años de edad, VIH positivo se presenta con sangrado rectal. La anoscopía reveló una úlcera en el borde anal. Las
biopsias mostraron fragmentos de piel anal con ulceración, tejido linfoide y tejido de granulación inflamatorio histológicamente
sospechoso de una enfermedad inflamatoria del intestino. La sigmoidoscopia mostró una mucosa eritematosa en el recto distal con
pequeñas áreas de exudados blancos y pliegues apilados. Las biopsias mostraron proctitis linfohistiocitario grave y criptitis. Las pruebas
serológicas fueron positivos para C. trachomatis .

PREGUNTA
Cual es la conducta terapéutica a seguir mas adecuada?

CURSO ENARM CMN SIGLO XXI TEL: 36246001 Pharmed Solutions Institute PÁGINA 144
MANUAL DE TRABAJO DEL CURSO ENARM CMN SIGLO XXI

RESPUESTA
a.- Tetraciclina
b.- Doxiciclina
c.- Eritromicina.
d.- Ciprofloxacino.

CASO CLINICO
Mujer de 48 años, nulípara, con antecedente de un embarazo ectópico tubario derecho, que ingresa para una histerectomía electiva
por adenomiosis sintomática, complicada por una anemia secundaria crónica. Se realizó histerectomía total, con conservación de
ovarios, por vía laparoscópica, sin incidentes. Una vez terminada la cirugía, se realizó la exploración de la cavidad abdominopélvica,
incluyendo los órganos y paredes, tal como lo hacemos en forma metódica y rutinaria en toda cirugía laparoscópica. Al observar la
superficie hepática, se encontraron adherencias en "cuerda de violín", entre el hígado y la pared abdominal. Durante el postoperatorio,
se le comunicó a la paciente el hallazgo y se preguntó a la paciente por antecedente de dolor abdominal en hipocondrio derecho, ante
lo cual relató no tenerlo. Sin embargo, recordó haber sido tratada con antibióticos, por flujo vaginal persistente 15 años antes.

PREGUNTA
Cual es el diagnostico diferencial mas probable, considerando los hallazgos del caso?

RESPUESTA
a.- Síndrome de Beckwith Wiedemann.
b.- Síndrome de Fitz-Hugh-Curtis.
c.- Síndrome de Simpson-Golabi-Behmel.
d.- síndrome de Klippel-Trenaunay-Weber

CASO CLINICO
Una mujer de 23 años presentó ojo irritado y rojo durante 4 semanas, con sensación de pérdida parcial de la audición en el oído
derecho durante 5 días, refiriere además flujo vaginal de color amarillo durante 1 semana. Se diagnostico conjuntivitis folicular del ojo
derecho, cervicitis mucopurulenta, y un derrame en el oído derecho confirmado por timpanometría. Audiometría de tonos puros
mostraron pérdida de audición conductiva en el lado derecho. Se aislo C. trachomatis del ojo derecho, el cuello uterino, y la
nasofaringe, pero no del aspirado del oído medio.

PREGUNTA
Cual es la conducta terapéutica a seguir mas adecuada?

RESPUESTA
a.- Tetraciclina
b.- Doxiciclina
c.- Eritromicina.
d.- Ciprofloxacino.

HERPES GENITAL (VIRUS HERPES SIMPLE TIPO 1 Y 2):


CIENCIAS BASICAS: El Herpes Genital es una Infección de Transmisión Sexual (ITS) causada por el Virus del Herpes Simple, existen 2: el
tipo 2 (VHS-2) que es el principal causante de esta ITS (aproximadamente 90% de los casos) y el Virus del Herpes Simple tipo 1 (VHS-1)
en aproximadamente el 10% de casos. La palabra “herpes” deriva del griego “herpein” que significa “arrastrar”. Los griegos lo refieren
en sus escritos como dolores que se arrastraban por la piel. SALUD PUBLICA: En México ha existido un incremento en las tasas por
100,000 habitantes actualmente es de 4.07 y 4.61. En EE.UU los casos de herpes genital en adulto varían de 200,000- 30,0000. La
infección por VHS-2 genital es más frecuente en las mujeres (aproximadamente una de cada cuatro mujeres) que entre los hombres
(casi uno de cada cinco). Esto puede deberse a que es más probable que ocurra la transmisión de hombre a mujer que la transmisión de
mujer a hombre. La prevalencia de infección herpética cervical o vulvar en la mujer embarazada es de 1%. El riesgo de infección
neonatal asociada a infección materna primaria, al momento del parto alcanza hasta 75%. PATOGENIA: El virus requiere transporte en
fluidos corporales (semen, fluidos de tracto genital femenino)o a través de las lesiones vesiculosas, ingresa a las células subdérmicas e
inicia su replicación en el nucleo, después las partículas virales son transportadas a través de ramas nerviosas a los ganglios neuronales
de las raíces dorsales, aquí es donde permanece de forma latente. DIAGNOSTICO: Clínico; Aproximadamente luego de 2 semanas de
haber adquirido el VHS-2 por contacto sexual, a nivel de la zona genital o región anal, se presenta una sensación de calor o quemazón,
escozor y se torna la piel de un color rosado en algunas ocasiones. Posteriormente aparecen las vesículas o ampollas, pústulas que se
ulceran y forman una herida o llaga llamada úlcera genital que es dolorosa. Esta úlcera se comienza a curar formándose una costra en
su superficie que finalmente desaparece. Acompañando a estas lesiones, la persona con Herpes Genital puede presentar leve sensación
de fiebre, cefalea ataque al estado general, dispareunía, incontinencia urinaria y linfadenopatía inguinal. Desde que aparecen las
lesiones vesiculares hasta que desaparecen pueden pasar de dos a cuatro semanas. En algunas ocasiones se puede presentar otro brote
cercano al primero pero es de menos intensidad y duración. Esta enfermedad es muy contagiosa, especialmente cuando están
presentes las ampollas. Tanto el VHS-1 como el VHS-2 pueden encontrarse en las úlceras causadas por los virus en los genitales y
pueden ser liberados por las mismas, pero entre brote y brote los virus también pueden ser liberados por la piel que no parece afectada
o que no tiene úlceras. Esto significa entonces que existen personas contagiando a sus parejas sin saberlo. Una persona solo puede
infectarse con el VHS-2 durante las relaciones sexuales con alguien que tiene la infección. En conclusión la transmisión puede darse a
partir de una pareja sexual infectada que no tiene una úlcera visible y que no sepa que está infectada. La cervicitis en la mujer puede

CURSO ENARM CMN SIGLO XXI TEL: 36246001 Pharmed Solutions Institute PÁGINA 145
MANUAL DE TRABAJO DEL CURSO ENARM CMN SIGLO XXI
ser sintomática, con secreción vaginal purulenta o asintomática; los signos clínicos en cérvix pueden variar de áreas locales friables
eritematosas hasta lesiones ulcerosas extensas en exocérvix o cervicitis necrótica extensa. Entre las complicaciones más graves de una
infección por herpes genital se encuentran la meningitis aséptica y la mielitis transversa. Laboratorio: debe ser confirmado, ya que el
diagnostico clínico es no sensible e inespecífico. El diagnostico se establece mediante el aislamiento del virus con cultivo celular de una
muestra tomada de las lesiones, observando células gigantes multinucleadas con cuerpos de inclusión eosinofilos intranucleares El
Tzank es rápido pero con baja sensibilidad y especificidad, y no distingue entre tipos de VHS. Anticuerpos monoclonales contra
antígenos específicos de virus de herpes. HERPES Y GESTACIÓN: El Herpes Genital puede causar infecciones potencialmente mortales
en los bebés. Si una mujer tiene el diagnóstico de Herpes Genital, se debe programar un parto tipo cesárea. En 85% de los casos se
adquiere en el canal del parto por contacto con las secreciones maternas infectadas. La forma clínica más frecuente en un 40% es la
infección de piel, ojos y boca, esta forma prácticamente no tiene mortalidad, deja daño en el sistema nervioso en 30% de los recién
nacidos no tratados y en 2% de los que reciben antivirales. El paciente puede presentar ictericia, hepatoesplenomegalia, alteraciones
hematológicas, microftalmia, microcefalia, convulsiones y a veces conjuntivitis, coriorretinitis Los herpes neonatales se presentan como
encefalitis en 35% de los casos y como infección diseminada en 25% de los casos; en ambos tipos de manifestación las complicaciones y
la mortalidad son elevadas. TRATAMIENTO: Hasta el momento no existe cura para esta enfermedad. Se utilizan medicamentos
llamados Antivirales cuya función es acortar y prevenir los brotes. El tratamiento más utilizado es el Aciclovir 200mg VO 5 veces al día
por 7 días (400mg c/8hrs 7-10 días) aunque existen medicamentos más modernos como el Famciclovir (250mg c/8hrs VO 7-10dias) y el
Valaciclovir (1g c/12hrs VO 7-10 días) que son más fáciles de tomar para los pacientes. Se puede alargar si la formación de costras es
incompleta después de 10 días. Para enfermedad diseminada, Herpes Neonatal y pacientes inmunocomprometidos se debe de usar el
Aciclovir endovenoso. Pacientes con cuadros a repetición de Herpes Genital deben de ser evaluados por un médico quien le prescribirá
tratamiento preventivo llamado profiláctico o supresivo (aciclovir 400mgs diarios hasta por 6 años) para reducir a posibilidad de
transmisión a sus parejas sexuales. Neonatos aciclovir 30mg/kg/dia IV dividido en 3 aplicaciones diarias por 10-21 días.

CASO CLINICO
Un hombre de 34 años de edad, ingeniero, previamente sano, presentó en el curso de 24 horas, fiebre, retención urinaria, debilidad de
las extremidades inferiores y déficits sensoriales irregulares en el tórax y abdomen. Como único antecedente, comunicó una tos leve los
últimos 4-5 días antes de admisión a la clínica. Una evaluación urológica descartó uropatía obstructiva. Al examen neurológico estaba
alerta, febril (38°C), sin déficits en nervios craneales. La fuerza en los miembros superiores era normal, pero se observó una debilidad
moderada en las extremidades inferiores con incapacidad para levantarse desde una posición sentada. Los reflejos plantares eran
extensores, había déficit sensorial en la parte posterior del cuello y a nivel del dermatoma D6. El análisis del LCR mostró 35 glóbulos
blancos por mm3, con predominio de células mononucleares. La proteínorraquia fue 75 mg/dl. El LCR fue positivo para VHH-7. La
resonancia magnética (RNM) cerebral fue normal y la RNM de médula evidenció lesiones inflamatorias, difusas, extensas en los
segmentos cervical, y dorsal de la médula.

PREGUNTA
Cual es la conducta terapéutica a seguir mas adecuada?

RESPUESTA
a.- Aciclovir
b.- Famciclovir
c.- Valaciclovir
d.- Ganciclovir

CASO CLINICO
Un hombre de 27 años, publicista, previamente sano, presentó cuadro de de 5 días de duración caracterizada por cefalea persistente,
fiebre (38,5°C), malestar general y diarrea. En el examen estaba alerta, sin anormalidades en el examen neurológico; en particular no se
detectaron signos meníngeos. El hemograma, velocidad sedimentación, proteina C reactiva y otros cultivos fueron normales. El LCR
mostró 160 células, predominio mononucleares y la proteína fue de 75 mg/dl. La deteccion viral por PCR identificó VHH-7 en LCR; el
mismo test fue negativo en sangre y también negativo al repetirlo al cabo de 1 semana en la muestra inicial de LCR. HIV y HTLV-1
fueron negativos.

PREGUNTA
Cual es la conducta terapéutica a seguir mas adecuada?

RESPUESTA
a.- Aciclovir
b.- Famciclovir
c.- Valaciclovir
d.- Ganciclovir

ARTRITIS REUMATOIDE (AR)


CIENCIAS BASICAS: Es las más común de las artritis inflamatorias, asociada a HLA-DR4. SALUD PUBLICA: Relación M:H de 3:1; se
presenta en 0.5-1% de la población. En México 1 millón 700mil personas entre 20 y 50 años padecen artritis reumatoide. 5-20%
presentan curso autolimitado; otro 5-20% forma clínica mínimamente progresivamente; 60-90% forma de artritis de deterioro
progresivo. PATOGENIA: Antígeno desconocido + predisposición genética; genera una respuesta inmunitaria y reacción inflamatoria con
activación de cel. Plasmáticas y linfocitos T CD4 > T CD8, los cuales infiltran la sinovial, forman tejido de granulación por activación de
fibroblastos (pannus), con hiperplasia de células móviles; todos estos cambios generan las manifestaciones de la artritis por citocinas

CURSO ENARM CMN SIGLO XXI TEL: 36246001 Pharmed Solutions Institute PÁGINA 146
MANUAL DE TRABAJO DEL CURSO ENARM CMN SIGLO XXI
secretadas por los macrófagos y destrucción celular y ósea por citocinas formadas en el pannus. DIAGNOSTICO: Cuadro clínico:
poliartrits crónica (>6 sem) simétrica, afecta principalmente manos (metacarpofalangicas 90%, carpo 80%), rodillas 60% y pies
(metacarpo e interfalangicas), más de 4 articulaciones afectadas. Rigidez matutina >1hr y dolor inflamatorio que mejora con la actividad
física. Astenia adinamia, hiporexia, cervicalgia. Desviación cubital de dedos, nódulos de Bouchar (a nivel de interfalangicas proximales),
sinovitis, dedos en cuello de cisne o botonero, hallux valgus (en MsPs), pulgar en “Z”, nódulos reumatoideos (subcutáneos en
superficies de extensión). También hay manifestaciones extraarticulares principalmente a pulmón generando dolor pleurítico y derrame
pleural (25%), hematológicas: anemia normocitica normocromica (30%), queratoconjuntivitis Sicca (25%) ojo seco, Sx. del túnel del
carpo por atrapamiento. Factor reumatoide (FR) positivo en 65% de los pacientes, son autoanticuerpos dirigidos contra IgG, no es tan
específicos porque puede encontrase en oras enfermedades inflamatorias autoinmunes. La presencia de FR muy elevado, se relaciona
con formas mas agresivas de la enfermedad. Anticuerpos anti-peptido cíclico citrulado (Anti-PCC) mas especifico, tiene una
especificidad de 90-95%. VSG y PCR; siempre deben de pedirse, ambos se encuentran elevados. Anti-DNA negativo (especifico para
LES). Cambios Rx: inflamación de articulaciones metacarpofalangicas, pérdida de espacio de articulaciones metacarpo e interfalangicas
y radiocarpiana, erosiones oseas, osteopenia yuxtaarticular, luxación de metacarpofalangicas, nódulos de Bouchar. Criterios Dx: Rigidez
matutina al menos 1 hora o mas. Artritis de 3 o mas articulaciones. Artritis de las manos. Artritis simetrica. Nodulos reumatoideos. FR
positivo. Cambios radiológicos; Diagnostico de artritis con 4 de 7 criterios. COMPLICACIONES: Derrame lateral de rodillas de predominio
izq., puede presentarse quiste de Barker en hueco poplíteo asociado a ruptura e invasión muscular. TRATAMIENTO: Metotrexate piedra
angular 10 mgs inicial, se puede aumentar hasta 25 mgs/sem.. Prednisona no es de elección, se da en dosis bajas y no modifica el curso
de la enfermedad. Los AINES se dan junto con los FARMES (fármacos modificadores de la enfermedad). En artritis activa leve:
Hidroxicloroquina (HCQ) o sulfazalazina (SSZ) ambos tardan 2-3 meses en iniciar su acción. Artritis activa moderada a severa: iniciar con
metotrexate, y considerar prednisona. Si hay una respuesta inadecuada agregar otro FARME (MTX + Abatacep ó MTX + leflunomide ó
MTX + SSZ + HCQ ó MTX + Rituximab). Los biológicos (anti-TNF) no son actualmente de 1ra. Elección. PRONOSTICO: Empeoramiento
progresivo de la salud, ya que daña permanentemente los huesos, cartílagos y articulaciones. Si FR y ANTI-PCC muy elevados indican
mal pronóstico y mayor gravedad. PREVENCION: En los 2 primeros años de inicio de la enfermedad daño severo e irreversible. Un
tratamiento precoz disminuye y previene el daño. Derivación precoz ante sospecha dx. A reumatología. Toda paciente requiere contar
con radiografía de manos y pies en AP y oblicuas. La suspensión del habito tabáquico puede ayudar a prevenir el desarrollo de AR.
CLAVES: Nódulos de Heberden en interfalangicas distales nos habla de osteoartritis. Raro debut con poliartritis aguda con fiebre y
linfadenopatías, pensar en rotavirus, si hay monoartritis en extremidades inferiores pensar espondiloartropatias seronegativas aun mas
si se acompaña de lumbalgía. Síndromes específicos en pacientes con AR: Sx. de Sjogren secundario= AR + Sx. SICCA (xerostomía,
xerostalmía). Sx. de Felty= AR+ esplenomegalia+ leucopenia. Sx. de Caplan= AR + nódulos reumatoideos + neumoconiosis. Amiloidosis =
AR de larga evolución y grave proteinuria e Insf. Renal

CURSO ENARM CMN SIGLO XXI TEL: 36246001 Pharmed Solutions Institute PÁGINA 147
MANUAL DE TRABAJO DEL CURSO ENARM CMN SIGLO XXI
NEOPLASIA DEL SISTEMA NERVIOSO CENTRAL
Epidemiologia: la incidencia de los tumores del SNC oscilan entre el 5.70 y el 9.63 en hombres y entre el 4.71 y el 6.95 en mujeres. Sin
embargo, en todas las series los tumores más frecuentes son los gliomas, y dentro de ellos los glioblastomas. En cuanto a la edad de
presentación, los tumores del SNC muestran una distribución bimodal, con un pico en la edad pediátrica y un aumento progresivo de la
incidencia en los adultos, hasta alcanzar su máximo entre la sexta y la séptima década de la vida. La incidencia combinada de tumores
del SNC en EEUU fue de 6,6 casos nuevos por 100.000 habitantes-año, en tanto que la mortalidad se estimó del 4,7 por 100.000
habitantes/año. Clasificación: El grado I incluye tumores de bajo potencial proliferativo y posibilidad de curación con extirpación
quirúrgica completa. El grado II incluye tumores
con capacidad infiltrativa y capacidad de
recurrencia y progresión a mayor grado de
malignidad, a pesar de presentar un bajo nivel
de actividad proliferativa (por ejemplo, los
gliomas de bajo grado pueden progresar a
gliomas anaplásicos y/o glioblastoma). El grado
III incluye tumores con evidencia histológica de
malignidad, como atipia nuclear y alta actividad
mitótica. El grado IV incluye tumores con
evidencia histológica de malignidad (atipia
nuclear, alta actividad mitótica, necrosis,
proliferación microvascular) y comportamiento
clínico agresivo con rápida progresión,
recurrencia a pesar de tratamiento intensivo y
muy alta mortalidad. Algunos tumores de grado
IV como los meduloblastomas y tumores de
células germinales son rápidamente fatales si
no se tratan, pero se asocian a altas tasas de
respuesta y supervivencia con tratamiento
adecuado. Aunque no son requisitos
indispensables, la capacidad de infiltración
difusa del tejido sano adyacente y la
propensión a diseminación leptomeníngea (e incluso extraneural) son características de algunos tumores de grado IV. Diagnóstico: Los
tumores del sistema nervioso suelen presentarse con síntomas neurológicos de instauración progresiva, aunque también es posible la
presentación aguda (por ejemplo, crisis comiciales o déficit neurológico agudo secundario a hemorragia intratumoral). Así mismo, los
síntomas pueden ser generalizados (por aumento inespecífico de la presión intracraneal) o focales (dependientes de la localización
tumoral).La cefalea es el síntoma de presentación más frecuente de los tumores cerebrales, aunque como síntoma aislado sólo se da
entre un 2 y un 16% de los casos. Las crisis comiciales son el segundo síntoma en frecuencia, pudiendo desarrollarlas hasta un 35% de
pacientes. Son especialmente habituales en gliomas de bajo grado. Pueden tener un inicio focal y reflejar la localización del tumor,
aunque en más de la mitad de los casos se produce una generalización secundaria y el inicio focal puede ser muy difícil de identificar en
base a criterios clínicos. Otros síntomas frecuentes de presentación son la disfunción cognitiva, las nauseas y vómitos, la disfunción
endocrina, los síntomas visuales y los síntomas focales dependientes de la localización tumoral. Diagnóstico por imagen: La técnica de
neuroimagen de elección para el diagnóstico y seguimiento de los tumores del sistema nervioso es la resonancia magnética (RM). La
tomografía computarizada (TC) presenta limitaciones importantes en esta patología, como son su peor resolución anatómica y la
presencia de artefactos de fosa posterior. A pesar de ello, la TC constituye una técnica muy útil para la evaluación de complicaciones
hemorrágicas agudas, efecto masa y extensión del edema vasogénico asociado al tumor. Es importante recordar que la TC craneal
puede aportar falsos negativos en el diagnóstico inicial de un tumor cerebral, en situaciones como tumores pequeños o con poca
densidad celular, masa tumoral isodensa, ausencia de cambios patológicos significativos en la barrera hematoencefálica, o
estabilización de la barrera hematoencefálica por uso previo de corticoides. Existe un conjunto de características radiológicas que
pueden orientar hacia el grado de malignidad en la TC, como son: número, forma y tamaño de las lesiones, localización, márgenes del
tumor, presencia de quistes, calcificaciones o hemorragias, presencia de necrosis intratumoral, edema perilesional, desplazamiento de
estructuras de la línea media y captación o no de contraste la lesión. En la TC la imagen de un tumor puede ser hipo, iso o hiperdensa.
Diagnóstico anatomopatológico y técnicas de biología molecular: Una vez establecida la sospecha clínica y radiológica, el diagnóstico
de certeza de una neoplasia cerebral requiere el estudio directo del tejido tumoral obtenido mediante biopsia o resección quirúrgica. El
objetivo es confirmar el diagnóstico de un tumor y establecer su naturaleza, también dar información pronóstica e incluso predictiva de
respuesta al tratamiento, como en el caso de la codelección 1p 19q en oligodendrogliomas, o la metilación del promotor de la enzima
reparadora de ADN O-6-metil-guanina-ADN-metil-transferasa (MGMT) en glioblastomas. Tratamiento: La cirugía suele ser el primer
paso en el tratamiento de los tumores primarios del sistema nervioso. En toda cirugía debe plantearse cuál es el beneficio esperable,
que deberá compararse con el riesgo existente para poder indicar la intervención. El beneficio esperable puede incluirse en alguno de
estos aspectos: supervivencia, calidad de vida, diagnóstico y obtención de material para investigación. En determinados tumores
(meningiomas, neurinomas, gliomas grado I de la OMS) la cirugía puede ser curativa por sí misma si se extirpa totalmente la lesión. En
gliomas de grados II, III y IV, la evidencia acerca del beneficio de la cirugía sobre la supervivencia es limitada, pero hoy en día se acepta
que los pacientes con una extirpación amplia tienen mayor supervivencia. El efecto de la cirugía sobre la calidad de vida puede ser
positivo o negativo. El principal perjuicio ocurre si se produce un déficit neurológico; por ello el objetivo debe ser siempre conseguir la
máxima extirpación sin provocar déficit. Los índices de morbilidad actuales se han estimado en torno al 13%, mientras que se considera
que la mortalidad en craneotomías por tumor debe ser inferior al 2%. Una cirugía sin complicaciones produce un efecto beneficioso en
la calidad de vida, al permitir la desaparición o rápida mejoría de los síntomas y la rápida retirada de corticoides. La radioterapia (RT) es
un tratamiento útil y eficaz en múltiples neoplasias primarias y secundarias del sistema nervioso central. Se basa en la inducción de

CURSO ENARM CMN SIGLO XXI TEL: 36246001 Pharmed Solutions Institute PÁGINA 148
MANUAL DE TRABAJO DEL CURSO ENARM CMN SIGLO XXI
daño en el ADN y orgánulos de las células malignas, con lo que consigue producir apoptosis y reducción de la masa tumoral. Sin
embargo, es necesario reducir al máximo el área de parénquima sano tratado, pues causa también desmielinización, daño neuronal y
cambios vasculares en el tejido normal. La dosis habitual de los gliomas de alto grado es 60 Gy administrados en fracciones de 1,8-2
Gy/día aplicados al tumor y un área peritumoral de 2 cm. Los gliomas de bajo grado pueden tratarse con dosis más bajas. Las nuevas
técnicas como RT conformada 3D e IMRT consiguen ajustar mejor el área a tratar y la radiocirugía permite tratar de forma muy precisa
volúmenes pequeños con mínima afectación del parénquima circundante. Sin embargo, la radiocirugía tiene un papel muy limitado en
el tratamiento de los tumores primarios, siendo su principal indicación las metástasis cerebrales. Quimioterapia y Agentes Biológicos:
se basa en tres pilares fundamentales, que son la resección quirúrgica, la radioterapia y la quimioterapia. El papel de esta última ha
cambiado en los últimos años, al pasar de su uso limitado en determinados tumores (meduloblastomas, tumores germinales) a formar
parte la temozolomida del tratamiento estándar de primera línea del glioblastoma, el tumor primario cerebral más frecuente.
Adicionalmente, cada vez más estudios muestran que temozolomida es activa en otras neoplasias como los gliomas de bajo grado y los
oligodendrogliomas. Además de la temozolomida, otros fármacos quimioterápicos usados en el tratamiento de los tumores cerebrales
primarios incluyen, entre otros, las nitrosoureas (BCNU, CCNU), procarbazina, vincristina, derivados del platino, metrotrexate y
citarabina. El descubrimiento de las alteraciones existentes en las vías moleculares que regulan la multiplicación celular, el crecimiento
celular, la invasividad de tejidos y la angiogénesis está permitiendo diseñar fármacos que bloquean o revierten dichas alteraciones. Este
tipo de fármacos, creados específicamente para inhibir vías moleculares concretas, son los llamados “targeted therapies” o
“tratamientos anti-diana”. El primero de ellos que ha llegado a la práctica clínica es el anticuerpo monoclonal anti-VEGF bevacizumab
(Avastin®) en gliomas malignos recurrentes, pero existen muchos otros que están siendo evaluados en ensayos clínicos con resultados
prometedores. Tratamiento sintomático: Las crisis comiciales pueden aparecer hasta en el 40% de los casos en el momento del debut
de la enfermedad, y hasta en el 60% a lo largo de su evolución. Todo paciente diagnosticado de un tumor cerebral que ha sufrido
alguna crisis comicial debe recibir tratamiento antiepiléptico. Su uso de forma profiláctica es sin embargo controvertido, y la
recomendación actual de la American Academy of Neurology (AAN) es no iniciar tratamiento antiepiléptico en pacientes con tumores
cerebrales que no hayan presentado nunca crisis. En caso de ser necesario tratamiento antiepiléptico, deben seleccionarse fármacos
con bajo potencial de interacciones (no inductores) y con un buen perfil de efectos secundarios. El edema peritumoral contribuye
significativamente en el deterioro clínico de los pacientes con tumores primarios y metástasis cerebrales. El tratamiento fundamental
del edema peritumoral son los corticoides, siendo el más utilizado en este contexto la dexametasona, por su buena difusión hacia el
parénquima cerebral y su mínima actividad mineralocorticoide. Su uso debe estar guiado por la clínica del paciente, manteniendo
siempre la menor dosis posible para el control de los síntomas e iniciando su retirada progresiva posteriormente para evitar los efectos
secundarios del tratamiento prolongado con corticoides. La trombosis venosa profunda y el tromboembolismo pulmonar ocurren hasta
en el 30% de los pacientes fuera del periodo periquirúrgico. Entre los factores que los favorecen se encuentran la disminución de la
movilidad de los miembros paréticos, decúbito prolongado, uso de quimioterapia, liberación de factores procoagulantes por parte del
tumor y el uso de tratamientos antiangiogénicos. Se recomienda el uso de medias de compresión elástica o de mecanismos de
compresión mecánica secuencial a todos los pacientes tras resección quirúrgica. Las heparinas de bajo peso molecular son el
tratamiento del evento tromboembólico, al haberse comprobado que el riesgo de complicaciones hemorrágicas es bajo. La fatiga, las
dificultades cognitivas y la depresión y ansiedad son también síntomas frecuentes que pueden tener un gran impacto en la calidad de
vida del paciente. Su origen es a menudo multifactorial. El tratamiento incluye la corrección de factores precipitantes si es posible y el
uso de farmacoterapia específica (psicoestimulantes, antidepresivos, ansiolíticos). Tumores Cerebrales Primarios: Tumores
Neuroepiteliales, Tumores Neuroepiteliales: Neuronales y Neurogliales mixtos, Tumores Neuroepiteliales: Tumores no gliales. Tumores
Meníngeos: Tumores de Células Meningoteliales, Tumores Meníngeos: Tumores Mesenquimales, Tumores Meníngeos: Lesiones
Melanocíticas Primarias, Tumores de Células Germinales, Tumores de la Región Selar, Linfoma Primario del SNC. Tumores de los
Nervios Craneales y Espinales, Tumores Medulares Primarios.

CASO CLINICO
Caso clínico. Paciente de 19 años, masculino, consulta por crisis de cefalea intermitente desde hace 5 meses, que aumentaron en
frecuencia e intensidad agregándose vómitos explosivos 3 días previo a la consulta. Al examen neurológico destacó Glasgow 15,
dismetría de extremidad superior derecha y leve paresia facial izquierda. Se le realizó una tomografía computada de encéfalo que
mostró tumor de tipo astrocitoma e hidrocefalia. Se realizó resección del tumor y la biopsia definitiva confirmó el diagnóstico
preoperatorio y determinó bordes libres de tumor.

PREGUNTA
Cual es la sobrevida a 10 año del caso clínico?

RESPUESTA
a.- Más del 90%
b.- Más del 80 %
c.- Más del 70%
d.- Más del 60 %.

CASO CLINICO
Femenino de 60 años de edad quien tiene el antecedente de tiroidectomía total, cinco meses previos a su ingreso por nódulo tiroideo
reportado por histología como hiperplasia multinodular multifocal. Su padecimiento tenía un año de evolución con cefalea
holocraneana persistente, hemiparesia corporal izquierda, así como parestesias mismo lado; presentaba además, cacosmia y déficit
campimétrico visual ipsilateral. La exploración neurológica demostró funciones mentales superiores conservadas. Nervios craneales:
olfación conservada, fondo de ojo sin papiledema, hemianopsia homónima izquierda. Extremidades: hemiparesia corporal izquierda
4/5, reflejos exaltados y Babinski ipsilateral, marcha parética, no se encontraron alteraciones sensitivas del orden interoceptivo,
propioceptivo o exteroceptivo; sin afección meníngea o cerebelosa.

CURSO ENARM CMN SIGLO XXI TEL: 36246001 Pharmed Solutions Institute PÁGINA 149
MANUAL DE TRABAJO DEL CURSO ENARM CMN SIGLO XXI

PREGUNTA
Los glioblastomas multiformes (OMS grado IV) son las neoplasias más frecuentes y malignas del sistema nervioso central, una variante
histológica de éstos, denominada, de células gigantes representa aproximadamente?

RESPUESTA
a.- 1% de todos los tumores cerebrales.
b.- 2% de todos los tumores cerebrales.
c.- 3% de todos los tumores cerebrales.
d.- 4% de todos los tumores cerebrales.

CASO CLINICO
Un hombre de 56 años de edad que sufre de dolor bilateral de las piernas durante 2 meses, lo que empeora sobre todo por las noches.
No había antecedentes de la incontinencia urinaria. En la exploración neurológica fue normal. Los estudios de neuroimagen como la
resonancia magnética (MRI) de la columna lumbar revelaron una masa extramedular intratecal de 23x23x13 mm, en L1-2 Nivel de disco
intervertebral. La masa era isointensa en imágenes ponderadas en T1 e hiperintensa en las imágenes potenciadas en T2 con
componentes quísticos.

PREGUNTA
Cual es el porcentaje que presentan los oligodendrogliomas de todolo los tumores de la meula espilana de origen primario?

RESPUESTA
a.- 1%
b.- 1.5%
c.- 2%
d.- 2.5%

CASO CLINICO
Femenino de 59 años con antecedente de dificultad para la marcha de mas de 5 años de evolución, acompañada de dolor tipo radicular
de miembros inferiores, el dolor era intenso tanto en la region sacrolumbar como en sus extremidades inferiores, que se
incrementaban con las maniobras de valsava. Desde el inicio de los síntomas requirió de andarea para demabular, limitando la marcha
a pocos metros y presentándose un cuadro de urgencia esfentiriana urinaria y fecal desde hacia 15 meses. Referia dos episodios de
empeoramiento, los cuales fueron tratados con AINES, corticoides e inmunoglobulina sistémica, con minima mejoría.

PREGUNTA
Cual es el diagnostico diferencia mas frecuente en este caso?

RESPUESTA
a.- Esclerosis multiple.
b.- Esclerosis lateral amniotrofica.
c.- Mielitis transversa.
d.- Sindrome raquimedular traumatico.

CASO CLINICO
Varón de 17 años y 10 meses, que acudió a su médico por cefalea, vómitos y desorientación. Murió 2 años después tras extensión
medular y metástasis ósea. TC craneal con contraste i.v. en el que se observa una masa hipercaptante del vermix que invade el 4º
ventrículo con pequeños focos quísticos/necróticos. RM axial en secuencia T2 y sagital en T1 con contraste, en el que se observa una
masa muy hiperintensa con edema perilesional en T2, que se tiñe de manera parcheada, comprime el 4º ventrículo generando intensa
hidrocefalia obstructiva.

PREGUNTA
Cual de las siguientes aseveraciones respuesto a los meduloblastomas es menos probable?.

RESPUESTA
a.- La localización del meduloblastoma desmoplásico presenta más frecuentemente una localización fuera de la línea media (50%).
b.- La hemorragia es un hallazgo de imagen frecuente en los meduloblastomas, pero se presenta aproximadamente en el 10%
c.- Se ha descrito que el único subtipo que presentaba ligeras diferencias en su apariencia con RM era el desmoplásico, que
tiende a presentar edema marcado (75%).
d.- Se observan signos radiológicos clásico de hidrocefalia obstructiva.

CASO CLINICO
Niña de 16 años que acude por cefalea, vómitos, diplopia y síndrome cerebeloso. TC con contraste en el que se objetiva una masa
hiperdensa de 4 cms en el vermix cerebeloso, con realce heterogéneo y focos quísticos de necrosis. Masa que comprime el tronco,
hiperintensa en T2 axial e hipointensa en T1 coronal con contraste, de bordes bien definidos, con realce parcheado, y que genera
dilatación ventricular.

CURSO ENARM CMN SIGLO XXI TEL: 36246001 Pharmed Solutions Institute PÁGINA 150
MANUAL DE TRABAJO DEL CURSO ENARM CMN SIGLO XXI
PREGUNTA
Cual de las siguientes aseveraciones respuecto a los meduloblastomas es menos probable?.

RESPUESTA
a.- La apariencia clásica en TC es una masa vermiana hiperdensa, bien delimitada, con edema vasogénico circundante ligero o
moderado.
b.- Presencia de hidrocefalia y realce homogéneo con contraste.
c.- La presencia de calcificaciones y áreas necrótico/quísticas son comunes.
d.- La hemorragia es el signo mas frecuente.

PREGUNTA
Cual de las siguientes aseveraciones respuecto a los meduloblastomas es menos probable?.

RESPUESTA
a.- La apariencia típica en RM es hipointensa en secuencias potenciadas en T1.
b.- La aparencia típica en RM Isointensa en secuencias potenciadas en T2.
c.- Respecto a la substancia gris cerebral, observándose mayores grados de heterogeneidad en la RM que en TC.
d.- Presentan captación intensa del gadolinio de forma principalmente uniforme.

CASO CLINICO
Un hombre de 61 años consulta por un cuadro progresivo de 3 semanas de evolución de debilidad de extremidades inferiores e
inestabilidad de la marcha. No refiere haber presentado fiebre ni perdida de peso, ni historia previa de incontinencia o disfunción
vesical o anal, convulsiones o trauma. Sin embargo, tiene antecedentes de abuso de cocaína e hipertensión, tratada con lisinopril. Al
examen físico, presenta signos vitales normales. Cuando se realiza el examen neurológico, se evidencia debilidad de ambas
extremidades inferiores, hiperreflexia, y debilidad muscular bilateral (puede hacer movimientos en contra de la fuerza de gravedad y
ante una leve resistencia). Presenta marcha atáxica. Se continuó el estudio con una RMN de la médula espinal, que, en fase T1, muestra
una lesión a nivel de de T8, extradural y que comprime la médula. La RMN de cerebro muestra, en fase T1, múltiples lesiones
homogéneas. Al realizar una excisión de la lesión medular, el paciente no vuelve a presentar debilidad muscular en las extremidades
inferiores.

PREGUNTA
Los meningiomas son los tumores benignos del sistema central más comunes, totalizando entre un 15 a 20% de todos los tumores
primarios cerebrales. Cual es la edad mas frecuente de presentación?

RESPUESTA
a.- 10 a 20 años.
b.- 20 a 40 años.
c.- 40 a 60 años.
d.- 60 a 80 años.

CASO CLINICO
Se trata de una paciente mujer de 58 años que consultó por una cervicalgia crónica. La paciente presentó desde dos meses antes de la
consulta episodios de parestesias de miembros superiores que se exacerbaban con las maniobras de valsalva. El examen físico de la
paciente fue normal. Se le realizó resonancia magnética de columna cervical, donde se observó una gran lesión quística que se extendía
desde C2 hasta C3. Con el contraste se observó una captación de un nódulo en la superficie medular que estaba en estrecha relación
con el quiste intramedular. La paciente fue llevada a cirugía, donde se le realizó un abordaje por vía posterior. Se le realizó
laminectomía C2 y C3, y al abrir la dura se encontró un nódulo tumoral vascular en la pared medular. Con técnicas microquirúrgicas se
hizo resección de la lesión espinal medular, y al retirar la lesión se produjo drenaje del quiste. En el postoperatorio la paciente presentó
mejora de la sintomatología. El estudio de patología reportó un hemangioblastoma espinal.

PREGUNTA
Los tumores intramedulares de la columna espinal son raros, cuales su frecuencia dentro de los tumores del sistema nervioso central?

RESPUESTA
a.- 1 a 2 %.
b.- 2 a 8 %.
c.- 8 a 10 %.
d.- 10 a 12 %.

PREGUNTA
Se pueden encontrar a lo largo de toda la médula espinal, cual de los siguientes es el mas frecuente?

RESPUESTA
a.- Ependimomas.
b.- Astrocitomas
c.- Hemangioblastomas

CURSO ENARM CMN SIGLO XXI TEL: 36246001 Pharmed Solutions Institute PÁGINA 151
MANUAL DE TRABAJO DEL CURSO ENARM CMN SIGLO XXI
d.- Menigiomas.

CASO CLINICO
Varón de 16 años con historia de cefalea, episodios de pérdida transitoria del conocimiento de dos años de evolución asociadas a
pérdida bilateral de la visión desde 4 meses. Como antecedente familiar informa hermano mayor fallecido a los 19 años con
enfermedad metastásica por Osteosarcoma de fémur. El TAC demuestra tumoración expansiva en ubicación supraselar, que se refuerza
con el medio de contraste. Le realizan craneotomía para resección biopsia de la tumoración con remisión de muestra a patología con el
diagnóstico de glioma del quiasma.

PREGUNTA
Cual es la localización más predominante de esta patologia?

RESPUESTA
a.- Pineal.
b.- Supraselar.
c.- Tálamo.
d.- Tercer ventrículo.

CASO CLINICO
Varón de 19 años, con historia de 4 años de hipoacusia que evoluciona a pérdida de la audición del oído derecho. Dos años después se
acompaña de cefalea y pérdida de la visión iniciada por el ojo izquierdo que progresa a ceguera total desde 4 meses. Como antecedente
familiar presenta abuela materna fallecida con enfermedad metastásica por carcinoma de cuello uterino. El TAC demostró presencia de
tumoración en región pineal y tercer ventrículo con importante hidrocefalia. Le realizaron craneotomía con colocación de válvula de
derivación revertiendo parcialmente la sintomatología. Dos meses después recurren las manifestaciones clínicas, el estudio de RM
demuestra tumoración expansiva e irregular que compromete regiones selar y pineal, piso del tercer ventrículo y núcleos básales

PREGUNTA
Cual es la tasas de curación mas aproximada de este padecimiento?

RESPUESTA
a.- 60 %.
b.- 70 %.
c.- 80 %
d.- 90 %

CASO CLINICO
Una mujer de 52 años consulta por pigmentación de las uñas de un año de evolución. No presentaba síntomas sistémicos y era
inmunocompetente. Se le había diagnosticado hace 12 años un macro-adenoma pituitario productor de ACTH, que había sido tratado
con neurocirugía endoscópica transnasal. Sin embargo, la cirugía no produjo la cura, por lo que se realizó una adrenalectomía bilateral,
y la paciente recibía mineralo-corticoides y glucocorticoides. Al exámen físico, algunas de las uñas de los dedos y de los pies
presentaban una banda de melanoniquia longitudinal, mientras que otras manifestaban pigmentación gris negruzca difusa. No se
observaron cambios de color en piel ni mucosas. L os estudios de laboratorio mostraron que los niveles plasmáticos de ACTH eran altos,
de 100 pg/mL (normal 10-50 pg/mL). Las imágenes de resonancia magnética (MRI) confirmaron vestigios de adenoma pituitario.

PREGUNTA
Cual de las siguientes afirmaciones relacionadas a la melanoquia es menos probable?

RESPUESTA
a.- Las drogas más comunes que causan melanoniquia son los agentes quimioterápicos.
b.- Las enfermedades endócrinas que causan incremento de los niveles circulantes de ACTH.
c.- Los melanomas son la principal causa de melanoquia.
d.- Frecuentemente los pinealomas no presentan melanoquia.

CASO CLINICO
Paciente de 35 años que presenta desde hace 10 años amenorrea, esterilidad, y galactorrea. A veces alteraciones visuales. Sensibilidad
en las mamas. Disminución del interés sexual, dolor de cabeza, infertilidad. Se le hizo un TAC y se halló un prolactinoma, al tratarla con
dopamina, (carbegolina) el microadenoma remitió.

PREGUNTA
Cual es el porcentaje de cambio de microadenomas a macroadenomas productores de prolactina?

RESPUESTA
a.- 2 %.
b.- 3 %.
c.- 4 %.
d.- 5 %.

CURSO ENARM CMN SIGLO XXI TEL: 36246001 Pharmed Solutions Institute PÁGINA 152
MANUAL DE TRABAJO DEL CURSO ENARM CMN SIGLO XXI

CASO CLINICO
Preescolar masculino de 2 años y 11 meses de edad hospitalizado como desnutrido severo marasmático. Al momento de ingreso la
madre refería un padecimiento de dos meses de evolución caracterizado por vómitos post-prandiales persistentes, hiporexia,
disminución subjetiva y progresiva de peso además de compromiso del estado general. Es producto de primer embarazo, nacido de
término por parto institucional sin complicaciones de una madre de 24 años sin antecedentes patológicos de importancia, con llanto
inmediato al nacer y desarrollo psicomotor adecuado hasta el año de edad, a partir del cual se percibió falta de desarrollo del lenguaje
receptivo y expresivo además de retraso del desarrollo motor caracterizado por imposibilidad de ponerse de pie y caminar sin apoyo. Al
examen físico de ingreso se encontraba en mal estado general y nutricional. El indicador peso para la talla era menor a -3 desvíos
estándar (DE), existía emaciación visible y palidez mucocutánea generalizada. Neurológicamente llamaba la atención una respuesta
verbal inapropiada, imposibilidad de bipedestación y aparente afectación visual manifestada por imposibilidad del paciente de seguir
objetos con la mirada. Su perímetro cefálico fue de 48 cm. (adecuado para la edad) y las pupilas eran isocóricas reactivas al estímulo
luminoso. El informe histopatológico confirmó el diagnóstico de craneofaringioma.

PREGUNTA
Con respecto al diagnostico, cual de las siguientes aseveraciones en menos probable?

RESPUESTA
a.- Tiene un leve predominio en el sexo femenino.
b.- Es un tumor de origen epitelial de la región sellar que se forma a partir de los restos embrionarios de la bolsa de Rathke.
c.- El tumor puede limitarse a la silla turca, o bien puede extenderse a través del diafragma sellar y comprimir la vía óptica, la
protuberancia o el tercer ventrículo, produciendo hidrocefalia.
d.- Los craneofaringiomas representan entre el 5 y el 10% de todos los tumores intracraneales que se presentan durante la niñez.

CASO CLINICO
Mujer de 51 años que padecía desde hacía cuatro meses episodios de repetición de amaurosis fugax en su ojo izquierdo. Mediante
tomografía axial computerizada se diagnosticó una masa orbitaria circunscrita intraconal izquierda sospechosa de hemangioma
cavernoso. Se le practicó una orbitotomía lateral izquierda resecándose una masa rojiza encapsulada, sugestiva de hemangioma
cavernoso. Microscópicamente estaba compuesta por áreas de células fusiformes dispuestas en empalizada (Antoni tipo A),
entremezcladas con áreas quísticas donde las células estaban rodeadas por una matriz mixoide (Antoni tipo B).

PREGUNTA
Cual es la edad de presentación mas frecuente de este padecimiento.

RESPUESTA
a.- entre 20 y 30 años.
b.- entre 20 y 40 años.
c.- entre 20 y 50 años.
d.- entre 20 y 60 años.

INFECCIONES DEL SISTEMA NERVIOSO CENTRAL.


Introducción: constituyen una emergencia médica, pues su
alta morbilidad y mortalidad requieren un diagnóstico y
tratamiento oportuno. Muchos factores se involucran en la
severidad de las infecciones del SNC. Su ubicación anatómica
en un espacio óseo sellado que no permite una expansión
fácil ante un proceso inflamatorio difuso contribuye a que las
altas posibilidades de daño neurológico ocurran por efecto
mecánico, como en los síndromes de herniación. Otros
factores como la competencia inmunológica de cada
individuo, la penetración y concentración de los agentes
antimicrobianos en el sistema nervioso, la edad y las
dificultades diagnósticas contribuyen en la evolución de los
pacientes con infecciones del sistema nervioso. La edad de
aparición más frecuente de meningitis desde la infancia hasta
la segunda y tercera década. Por otro lado ha aumentado la proporción de infecciones nosocomiales. Evaluación de laboratorio. El
líquido cefalorraquideo (LCR) es el pilar fundamental en el diagnóstico de la mayoría de las infecciones del SNC. Las características
iniciales del análisis citoquímico pueden orientar a un diagnóstico específico aunque se requiere la confirmación de éste por otros
medios. Sin embargo una buena correlación clínica inicial es suficiente para tomar una conducta terapéutica. En general las
características típicas de los diferentes tipos de infección meníngea de acuerdo con el citoquímico se muestran en la Tabla. Los cultivos
y el Gram del LCR continúan siendo los exámenes de elección en las meningitis bacterianas. La administración de antibióticos
intravenosos por dos a tres días antes de la punción lumbar no altera el conteo de células o las concentraciones de proteínas y glucosa
en los casos de meningitis bacterianas, pero si reducirán la probabilidad de obtener un cultivo y el Gram del LCR positivos. La
administración de antibióticos orales antes de la punción lumbar no altera ninguno de los parámetros en el LCR pero sí puede disminuir
el porcentaje de neutrófilos y la probabilidad de resultados positivos en el cultivo o el Gram. El Gram tiene una sensibilidad cercana a
60-90% y una especificidad de 100%. Los cultivos se pueden cultivar en platos de agar con adición de nutrientes o en caldos. Los

CURSO ENARM CMN SIGLO XXI TEL: 36246001 Pharmed Solutions Institute PÁGINA 153
MANUAL DE TRABAJO DEL CURSO ENARM CMN SIGLO XXI
cultivos en caldos son más susceptibles de ser contaminados con Stophylococcus epidermidis mientras que son mas útiles en caso de
infecciones de los equipos de derivación ventricular. Los hemocultivos identifican el germen causal en 80% de los casos de neumococo,
90% de los de meningococo y 94% en los casos de Haemophylus influenzal. Los casos de meningitis tuberculosa (TBC) o meningitis
micóticas son difíciles de diagnosticar por cultivo o en el extendido. Son positivos entre 52 a 83% de los casos de meningitis TBC y en
75% de los casos de criptococosis meníngea. La sensibilidad y especificidad de los cultivos en estos dos gérmenes se aumenta hasta en
87% con cultivos repetidos y con grandes volúmenes (hasta 25 ml) de LCR. Las pruebas de látex para identificación de antígenos de los
gérmenes son rápidas requiriendo entre 10 a 15 minutos para realizarlas y sin un entrenamiento riguroso. Además tienen la ventaja de
tener pocos falsos positivos. Por ejemplo, para criptococo puede haber falsos positivos ante la presencia de factor reumatoideo. En el
caso de meningitis por Histoplasma capsulatum, las pruebas son muy sensibles pero la especificidad es baja por tener reactividad
cruzada con criptococo, candida y Coccidiodes inmitis. La identificación de anticuerpos específicos en el LCR es de gran utilidad. En la
neurosífilis un diagnóstico definitivo se obtiene con la demostración de positividad del VDRL en el LCR. Una prueba reactiva en
cualquier título es diagnóstica de neurosífilis. Aunque la prueba es muy específica, la sensibilidad sin embargo varía entre 30-70% y en
muchos casos el diagnóstico se basa en la presencia de pleocitosis o proteínas elevadas en el LCR en un paciente con VDRL reactiva en
sangre. En los pacientes con VIH las dificultades nacen debido a que puede haber pleocitosis con aumento de proteínas en el LCR
debido a la neurosífilis o por la misma infección por el VIH. En estos casos tiene mucha utilidad el FTA-ABS (Treponemal antibody-
absortion test) y el test de hemoaglutinacion para Treponema pallidum (MHA-TP) los cuales son muy sensibles para el diagnóstico de
neurosífilis. Un resultado no reactivo descarta el diagnóstico de neurosífilis en los pacientes con infección por VIH. La PCR es la prueba
más útil para el diagnóstico de meningitis y encefalitis viral. Además tiene la ventaja de cuantificar el ácido nucleico en las muestras de
LCR con lo cual se puede determinar la progresión de la enfermedad y la respuesta al tratamiento. La prueba se fundamenta en realizar
mediante un sistema térmico cíclico una copia y amplificación de hasta un millón de veces del contenido de ADN del germen presente
en el LCR. En los casos de meningitis TBC los resultados de sensibilidad para la PCR en diferentes estudios han sido variables (entre 54-
100%) y la especificidad entre 89-100%. Sin embargo hay evidencia consistente de que la PCR es más sensible que el examen
microscópico y el cultivo para TBC. La adenosin deaminasa (ADA) es una enzima que está asociada con enfermedades que producen
una respuesta inmunológica celular; es de mucha ayuda para el diagnóstico de meningitis TBC. Sin embargo, pueden observarse
resultados positivos de esta prueba en linfomas con compromiso meníngeo, sarcoidosis, neurobrucelosis y hemorragia subaracnoidea.
También pueden haber falsos negativos. Los diferentes estudios muestran una sensibilidad y especificidad para el diagnóstico de 90%
con títulos mayores de 10 unidades internacionales por litro (UI/L) siendo los títulos de corte entre 5-10 UI/L. Los resultados de la ADA
pueden elevarse en las dos primeras semanas de tratamiento. Meningitis: Se define meningitis como la presencia de inflamación
meníngea originada por la reacción inmunológica del huésped ante la presencia de un germen patógeno en el espacio subaracnoideo.
La meningitis es el síndrome infeccioso más importante del sistema nervioso central. El compromiso parenquimatoso adyacente a las
meninges definirá la presencia de meningoencefalitis (encéfalo), meningoencefalomielitis (encéfalo y médula),
meningomielorradiculitis (encéfalo, médula y raíces nerviosas). Los diferentes tipos de meningitis tienen diferente origen y los
patógenos responsables en la mayoría de los casos son predecibles, lo cual permite en muchas situaciones clínicas el inicio de una
terapia empírica mientras se obtiene la confirmación del germen involucrado. La identificación de gérmenes que pueden tener
influencia de tipo endémico es de importancia para las medidas profilácticas y epidemiológicas en determinadas poblaciones (por ej:
meningococo). Los diferentes tipos de meningitis se pueden definir de acuerdo con el perfil clínico, hallazgos de LCR y la etiología. Se
definen como meningitis aséptica aquellos casos de meningitis en los cuales luego de un estudio completo no se identifica una bacteria
y los cultivos y las pruebas inmunológicas deben ser negativos. Es de origen viral por lo general. La meningitis séptica es causada por
una bacteria. Tiene con la meningitis aséptica un perfil diferente del LCR; su tratamiento y pronóstico son completamente distintos a la
anterior. La meningitis crónica se define arbitrariamente como aquella con una evolución mayor de cuatro semanas luego de haberse
documentado clínicamente que el paciente no se encuentra en una fase de recuperación de una meningitis aguda. La meningitis
recurrente se refiere a aquellos casos que clínicamente presenta dos o más episodios de meningitis aguda. El paciente debe haber
evolucionado a la mejoría total tanto clínica como en el LCR entre cada uno de los episodios agudos. Meningitis aséptica: Por lo general
es de curso benigno y en la mayoría de los casos de etiología viral. Puede haber causas no infecciosas de meningitis aséptica. Tiene un
curso clínico bifásico. Primero preceden a su aparición los síntomas o signos de una afección viral respiratoria, gastrointestinal o en la
piel, presentándose posteriormente la fase meníngea con síntomas más específicos como cefalea, fiebre y signos meníngeos. Son poco
frecuentes los síntomas de compromiso encefálico como convulsiones o alteración del estado mental. En algunos países tiene
predilección estacional y es más frecuente en niños. Los enterovirus son los principales agentes causales (entre 55-75%) de los casos de
meningitis aséptica y hasta en 95% de los casos cuando se identifica el patógeno. El virus de la parotiditis fue considerado en alguna
ocasión responsable de muchos casos de meningitis aséptica pero la incidencia ha declinado desde el uso de la vacuna; sin embargo
una meningitis oculta se presenta en más de la mitad de los casos de parotiditis y llega a ser sintomática en solo 30%. Mas aún, puede
ocurrir la meningitis sin parotiditis. Los herpes virus raramente causan meningitis; sin embargo el virus del herpes simple (VHS) es el
responsable de 1 a 3% de todos los casos de meningitis aséptica. De los dos tipos de herpes simple (VHS 1 oral - VHS 2 genital) el VHS 2
produce meningitis en 11% a 33 % de las personas en el momento de la infección genital primaria. El VIH puede causar una meningitis
aséptica principalmente en el período de la infección primaria y durante la seroconversión pero incluso puede producirla muy
temprano en las fases iniciales de la infección por VIH. Raramente la tuberculosis se puede comportar como una meningitis aséptica
autolimitada. En los casos de endocarditis e infecciones parameníngeas puede presentarse un cuadro de meningitis aséptica aunque las
causas sean gérmenes no vírales. La PCR es de gran utilidad con una especificidad del 100% y el resultado es disponible en pocas horas.
El manejo es prácticamente de soporte y de alivio de los síntomas. Meningitis séptica (meningitis bacteriana) implican una mayor
morbi-mortalidad y requieren un rápido diagnóstico y tratamiento para evitar una evolución fatal o secuelas irreversibles, por lo tanto
es una emergencia neurológica. La tasa general de mortalidad es de 25% y de morbilidad hasta 60%. Presentación clínica: los síntomas
clásicos de presentación incluyen cefalea, fiebre, escalofríos, alteración de la esfera mental y la presencia de meningismo. Estos
síntomas pueden variar en los neonatos y en los ancianos. En los neonatos predominan los síntomas inespecíficos como irritabilidad,
vómito, letargia, dificultad respiratoria y síntomas gastrointestinales, siendo el meningismo poco frecuente. Los ancianos presentan con
mayor frecuencia alteración en el estado mental asociado a fiebre; la presencia de cefalea y meningismo resulta exótica. La meningitis
por meningococo tiene frecuentemente una presentación dramática y puede progresar como una enfermedad fulminante muriendo el

CURSO ENARM CMN SIGLO XXI TEL: 36246001 Pharmed Solutions Institute PÁGINA 154
MANUAL DE TRABAJO DEL CURSO ENARM CMN SIGLO XXI
paciente en pocas horas. Muchas veces los pacientes tienen un rash
cutáneo petequial en el tronco y las extremidades inferiores. Este puede,
por contigüidad de las lesiones, formar áreas extensas de equimosis. La
púrpura fulminante es una forma grave de manifestación de la sepsis por
meningococo; es causada por coagulación intravascular diseminada y en
muchas ocasiones causa necrosis distal en las extremidades. Los
marcadores de riesgo incrementado de muerte en los pacientes con meningitis por meningococo son: la presencia de diátesis
hemorrágica, signos neurológicos focales y personas mayores de 60 años. En general muchos síndromes neurológicos diferentes al
meningismo pueden ser la forma de presentación de las meningitis bacterianas. La presencia de signos neurológicos focales por
compromiso cerebrovascular, hipertensión endocraneana, alteración en pares craneales, crisis convulsivas y estado confusional son
algunos de ellos.
Tratamiento: el tiempo del inicio del tratamiento influye significativamente en el pronóstico. El LCR es el principal soporte diagnóstico
en la escogencia de la terapia adecuada, pero en muchas ocasiones la punción lumbar debe diferirse en aquellos casos de pacientes en
estupor o coma, con signos neurológicos focales y ante la presencia de crisis convulsivas. En estos casos las neuroimágenes están
indicadas inicialmente para descartar complicaciones asociadas con la meningitis o hacer un diagnóstico diferencial. Una escanografia
de cráneo (simple) normal permite una mayor seguridad en la realización de la punción lumbar en estos casos y evitar así
complicaciones. Cabe anotar que ante la sospecha diagnóstica debe iniciarse una terapia empírica mientras se realiza la punción
lumbar. La terapia antibiótica no variará significativamente el LCR en las primeras 48 horas. La terapia empírica se escogerá de acuerdo
con los grupos de edad, la sensibilidad antibiótica de los gérmenes
involucrados, su sensibilidad antibiótica y el estado inmune de los
pacientes. El Gram puede ser una guía inicial de utilidad pero no
es tan eficaz en todos los casos; las pruebas inmunológicas son de
gran ayuda para un diagnóstico específico y rápido del germen. La
terapia empírica más conveniente para iniciar el tratamiento.
Luego, al obtener los resultados del cultivo con las pruebas de
sensibilidad antibiótica se decidirá el tratamiento definitivo. El
Streptococcus pneumoniae (neumococo) hoy por hoy es el
principal germen patógeno en la meningitis bacteriana en
cualquier grupo de edad. Penicilina ó cefalosporina con
vancomicina. El Haemophylus influenzae es un coco bacilo
gramnegativo. El único huésped natural de este germen es el
humano y la transmisión persona-persona ocurre por la vía
respiratoria. El trauma de cráneo reciente, la cirugía neurológica previa, las sinusitis paranasales, las otitis media y las fístulas del LCR
son los factores de riesgo más importantes en adultos para padecer una meningitis por H. influenzae. El tratamiento antibiótico es
ceftriaxona ó cefotaxima, por lo tanto son los fármacos de elección. La Listeria monocitogenes es un germen beta-hemolítico
facultativamente anaerobio y grampositivo. Los grupos de edad más frecuentemente involucrados en meningitis por L. monocitogenes
son los neonatos menores de un mes y los adultos mayores de 60 años. En este grupo la listeria es la causa de 20% de las meningitis.
Los principales factores de riesgo para meningitis por L. monocitogenes son el embarazo, inmunosupresión y edad avanzada. El
tratamiento de elección es la ampicilina o la penicilina por un tiempo de cuatro semanas en los pacientes inmunosuprimidos. En los
pacientes alérgicos a las penicilinas el trimetropim-sulfa-metoxazol es la alternativa más recomendada. El estreptococo del Grupo B es
la causa más frecuente de sepsis neonatal y es una causa importante de infección bacteriana invasiva en los adultos. Es el germen
causal en 70% de los casos de meningitis bacteriana en los neonatos menores de un mes, mientras que en adultos es responsable de
menos de 5% de los casos. El estreptococo del grupo B es altamente susceptible a la penicilina y a la ampicilina siendo las drogas de
primera elección. Muchos recomiendan el uso concomitante con gentamicina para el tratamiento de la meningitis neonatal basado en
evidencia de estudios en animales que demuestran sinergismo en la eficacia antibiótica. La Neisseria meningitidis (meningococo) es una
bacteria gramnegativa encapsulada que aparece en pares en el extendido. Generalmente coloniza la nasofaringe de manera
asintomática y la transmisión ocurre de persona a persona mediante secreciones respiratorias. Los individuos esplenectomizados y con
deficiencias de complemento tienen un alto riesgo de infección por meningococo. Causa 60% de las meningitis bacterianas entre la
población de 2 a 18 años de edad y va declinando su frecuencia hasta 5 % en los mayores de 60 años. Es el responsable en 1/3 de los
casos de meningitis en menores de 2 años. El fármaco de elección es la penicilina o la ampicilina. La ceftriaxona y el cefotaxima tienen
también una excelente respuesta y alcanzan buenas concentraciones en el LCR. En Estados Unidos no se han identificados cepas
productoras de beta-lactamasa. Los contactos cercanos en pacientes con meningococemia tienen un riesgo significativamente alto para
desarrollar la infección semanas después de la exposición. Meningitis tuberculosa: Es la manifestación más frecuente de la tuberculosis
en el sistema nervioso. Puede presentarse en muchos casos en forma aislada sin compromiso extrameníngeo. Las manifestaciones
clínicas son diferentes en niños, adultos y en los infectados por VIH. Su epidemiología ha cambiado significativamente aun en países en
donde la incidencia había disminuido de manera importante. Este hecho se explica por la epidemia reciente de infección por VIH y por
la resistencia que ha desarrollado el bacilo a las terapias convencionales. Manifestaciones clínicas: con fines pronósticos se reconocen
tres estadios de la enfermedad. a) Estadio 1: el paciente está consciente y no presenta signos neurológicos focales. b) El estadio dos: el
paciente está confuso pero no está en coma, y tiene signos neurológicos focales como hemiparesia o parálisis de pares craneales. c)
Estadio tres: el paciente se encuentra en coma o en estupor; tiene compromiso múltiple de pares craneales, hemiplejía o paraplejía. En
los niños es más frecuente identificar la fase inicial de infección a nivel respiratorio, mientras que en el adulto en muchos casos no se
puede identificar y puede pasar mucho tiempo desde la infección inicial y la aparición del compromiso del sistema nervioso. Es
frecuente la presencia de hiponatremia en adultos encontrándose en 45% de los casos y se debe a una secreción inadecuada de
hormona antidiurética (SIDHA). En los niños aunque pueden manifestarse también la cefalea, fiebre y meningismo, es común la
presencia de hidrocefalia como primera manifestación de la infección por TBC. Diagnóstico: el diagnóstico recae principalmente en el
LCR. La pleocitosis a expensas de linfocitos, las proteínas altas y la glucorraquia significativamente baja son los hallazgos más

CURSO ENARM CMN SIGLO XXI TEL: 36246001 Pharmed Solutions Institute PÁGINA 155
MANUAL DE TRABAJO DEL CURSO ENARM CMN SIGLO XXI
frecuentemente encontrados. En algunos casos hay un predominio polimorfonuclear en las fases iniciales de la infección, que también
suele observarse cuando se inicia el tratamiento. Este último coincide con el empeoramiento clínico que se observa en algunos
pacientes cuando se inicia la terapia y es muy característico de la meningitis TBC. Por lo general hay un viraje posterior al predominio de
linfocitos pero en algunas ocasiones puede persistir el predominio de polimorfonucleares denominándose meningitis neutrofílica
persistente. Se ha encontrado un mayor contenido de proteínas en el estadio tres de la enfermedad. Como ya se comentó
anteriormente el cultivo y los extendidos para BK tienen una mayor probabilidad de positividad cuando se estudian muestras repetidas
en altos volúmenes. La ADA y la PCR recientemente han sido de gran ayuda para un diagnóstico más rápido y en los casos de cultivos
negativos. Las neuroimágenes brindan apoyo en el diagnóstico. En la RMC se puede observar el compromiso de las meninges basales
que realzan al inyectar el gadolinio; los sitios más comprometidos son la fosa interpeduncular, la cisterna ambiens y la región
quiasmática. Este realce es más frecuentemente observado en los pacientes VIH positivos. Las neuroimágenes también son de ayuda
para el diagnóstico de hidrocefalia, la presencia de vasculitis por la infección y para caracterizar los granulomas. Complicaciones: aún
con tratamiento y cuidados adecuados las complicaciones se pueden presentar. Algunas de ellas son de tipo cerebrovascular por
compromiso de las arterias intracraneales debido a una panarteritis, producto de la infiltración de las paredes de los vasos por el
exudado inflamatorio. Lo anterior produce infartos cerebrales. El territorio carotídeo es el más frecuentemente comprometido. Una
complicación metabólica que puede agravar el cuadro clínico es la hiponatremia por SIDHA; ésta debe corregirse rápida y lentamente
para evitar el riesgo de mielinolisis póntica. Se han informado casos de siringomielia muchos años después de la infección inicial.
Tratamiento: va orientado a eliminar las formas intra y extracelulares del bacilo. Resultan importantes dos factores para evitar la
resistencia: el primero, utilizar múltiples fármacos y el segundo la adherencia al tratamiento. Por lo general el inicio del tratamiento es
empírico, sin obtener una confirmación por cultivo que puede tardar varias semanas. Lo anterior repercute significativamente en el
pronóstico. Se sabe bien que el número de organismos presentes en los casos de meningitis TBC es inferior al de las otras formas de la
infección; esta ventaja potencial se ve disminuida por el hecho de la penetración de los fármacos anti-TBC en el SNC. La isoniacida y la
pirazinamida son bactericidas y penetran las meninges inflamadas o no inflamadas alcanzando buenas concentraciones bactericidas.
Las concentraciones que alcanzan en el LCR la estreptomicina intramuscular, la rifampicina y el etambutol apenas sobrepasan las
concentraciones inhibitorias mínimas para la micobacteria; además no penetran las meninges no inflamadas. Mientras que la
rifampicina es bactericida, el etambutol y la estreptomicina son tuberculostáticos. Se aceptan dos tipos de régimen: uno corto de seis
meses, con cuatro fármacos. En los primeros dos meses: isoniacida 300 mg, rifampicina 600 mg, pirazinamida 1,5 gr al día y
estreptomicina 1g IM (500 mg en ancianos o con peso menor de 50 kg). Luego un peróodo de cuatro meses con isoniacida y rifampicina
dos veces por semana o diario. En los niños este segundo período debe ser de 10 meses. En los casos de resistencia se debe incluir otro
medicamento como etambutol en vez de estreptomicina y seguir luego con dos drogas (isoniacida y rifampicina) por 9 a 18 meses. La
respuesta a la terapia debe evaluarse por cultivos y por la evolución clínica de los síntomas. Meningitis por criptococo: En general todas
las meningitis por hongos se consideran que ocurren en pacientes inmunosuprimidos. La vía de entrada en la mayoría de los casos es a
través de las vías respiratorias superiores, ingresando en forma de levadura en el alvéolo, allí puede producir un proceso inflamatorio
que puede ser completamente asintomático o producir síntomas respiratorios leves y el sistema inmune puede terminar allí todo el
proceso; si esto no ocurre así, pasan a la circulación sistémica. Un defecto en los sistemas celulares permite que los hongos atraviesen
la barrera hematoencefálica. Las infecciones micóticas del sistema nervioso pueden coexistir o no con la infección en otros órganos
blanco en el organismo. En el caso del criptococo, el riñón es el órgano mas frecuentemente comprometido. En el SNC el criptococo
puede comprometer únicamente las meninges o invadir el parenquima y los vasos sanguíneos produciéndose una forma de arteritis
infecciosa. Presentación clínica: la presentación clínica de las meningitis por hongos en general es insidiosa, de instauración subaguda o
crónica; se presenta cefalea, fiebre, escalofríos, alteración del estado general y cambios mentales en un paciente por lo general con
factores de riesgo para adquirir la infección (SIDA, cáncer, terapia inmunosupresora, trasplante de órganos). Diagnóstico: requiere de
sospecha clínica en los pacientes con riesgo y recae principalmente en el aislamiento del criptococo en el LCR. El LCR muestra
generalmente un aumento moderado de la presión; es de aspecto claro y hay pleocitosis que puede alcanzar desde 20 hasta 1.000
células por milímetro cúbico. El predominio celular es de linfocitos, la glucosa es frecuentemente baja (alrededor de 30 mg/dl) y las
proteínas son elevadas entre 50 a 1.000 mg/dl; la tinta china puede mostrar la presencia del criptococo. La prueba debe hacerse con un
control negativo de agua y así estar seguro que la tinta no está infectada por espora. La prueba serológica de látex en el LCR es muy
sensible para el diagnóstico; se puede cuantificar y es de gran ayuda en el seguimiento. Además es rápida y de fácil interpretación.
Tratamiento: el tratamiento de primera línea para la meningitis por criptococo es la anfotericina B a la dosis de 0,7 mg/kg/día más 5
flucytocina a la dosis de 25 mg/kg cada seis horas por vía oral hasta que los cultivos sean negativos o haya mejoría clínica (entre cuatro
a seis semanas). Posteriormente se debe continuar una terapia de mantenimiento con fluconazol por ocho o diez semanas; pero si el
paciente es inmunosuprimido (SIDA, trasplantado, cáncer, etc. ) esta terapia de mantenimiento es de por vida.

CASO CLINICO
Paciente de 30 años, sexo masculino, en tratamiento con prednisona por Lupus Eritematoso Sistémico, que comenzó con un cuadro de
aproximadamente dos semanas de evolución caracterizado por compromiso progresivo del estado general, cefalea holocraneana de
intensidad progresiva, fiebre y diaforesis vespertina por lo que consultó. Se le realizó punción lumbar, obteniéndose LCR de aspecto
opalescente, cuyo estudio citoquímico fue compatible con meningitis bacteriana; el cultivo demostró desarrollo de un bacilo Gram (+)
pequeña, cuya identificación microbiológica confirmó Listeria monocytogenes.

PREGUNTA
Cual es la conducta terapéutica mas apropiada a seguir.

RESPUESTA
a.- Cefotaxima mas vancomicina.
b.- Ampicilina mas gentamicina.
c.- Trimetroprima mas sulfametoxazol y ampicilina.
d.- Gentamicina mas imipenem.

CURSO ENARM CMN SIGLO XXI TEL: 36246001 Pharmed Solutions Institute PÁGINA 156
MANUAL DE TRABAJO DEL CURSO ENARM CMN SIGLO XXI
CASO CLINICO
Paciente de 87 años, sexo masculino, hipertenso, portador de marcapasos definitivo, además de multiinfarto cerebral y bronquitis
crónica. Ingresó con historia de tos productiva y fiebre de aproximadamente 1 semana de evolución, agregándose posteriormente
desorientación progresiva y franco compromiso de conciencia que motivó hospitalización. Desde su ingreso, muy grave, en sopor, con
signos meníngeos esbozados y neumonía confirmada tanto clínica como radiológicamente. Se manejó con antibioticoterapia agresiva y
de amplio espectro para neumonía adquirida en la comunidad con sospecha de aspiración; a pesar de lo anterior evoluciona a la falla
orgánica múltiple.

PREGUNTA
Cual es la conducta terapéutica mas apropiada a seguir.

RESPUESTA
a.- Cefotaxima mas vancomicina.
b.- Ampicilina mas cefotaxima.
c.- Ampicilina mas levofloxacina.
d.- Gentamicina mas imipenem.

CASO CLINICO
Un varón de 56 años de edad, con antecedentes de meningitis criptogena y meningitis aséptica que presenta hace 10 dias fiebre,
escalofríos, rigidez en el cuello y dolor de cabeza. El EF temperatura 36.9 c, FR de 121 lpm. Parecía incómodo, pero sin rigidez de nuca
franca. LCR revelo eritrocitos 34 células/mm3, 154 células/mm3, leucocitos ( 77 % de neutrófilos , 12 % de linfocitos, histiocitos 10 % ),
la glucosa de 63 mg / dl , la proteína de 82 mg / dl. TAC de la cabeza revela estable, leve prominencia difusa del sistema ventricular. El
paciente es alérgico a la penicilina.

PREGUNTA
Cual es la conducta terapéutica mas apropiada a seguir.

RESPUESTA
a.- Cefotaxima mas vancomicina.
b.- Meropenen mas aciclovir.
c.- Amikacina mas levofloxacina.
d.- Tratamiento sintomatico.

CASO CLINICO
Varón de 51 años, ingresó por cefalea de 4 semanas y somnolencia. La EF con deterioro neurológico con 10 de Glasgow. Se realizó una
punción lumbar (PL): 40células/mm3, el 95% mononucleares (MN); glucosa: 4mg/dl, proteínas: 0,68mg/l. En la tinción de tinta china se
apreciaron células levaduriformes. El título de antígeno criptocócico en plasma era superior a 1/256 y en líquido cefalorraquídeo (LCR),
superior a 1/1.024. No se midió la presión intracraneal. En el cultivo de LCR se aisló C. neoformans, se inició tratamiento con
anfotericina B i.v. a dosis de 4mg/kg/día y 5 flucitosina oral a dosis de 100mg/kg/día. El paciente desarrolló sordera bilateral como
complicación. A los 12 días reingresó con afectación de VII par craneal izquierdo.

PREGUNTA
Cual es la conducta terapéutica mas apropiada a seguir.

RESPUESTA
a.- Anfotericina B iv.
b.- Meropenem i.v. a dosis de 1g/8h.
c.- Fluconazol 400mg/día/VO.
d.- Linezolid i.v. a dosis de 600mg/12h.

CASO CLINICO
Varón de 47 años de edad ex-adicto a drogas por vía parenteral y con infección por VIH conocida desde hacía 6 años, en tratamiento
con antirretrovirales que cumplía de forma irregular. Ingresa por cefalea de 3 semanas de evolución de localización holocraneal que
aumentaba tras maniobras de Valsalva. Su intensidad ha seguido un perfil progresivo, interfiriendo el sueño y requiriendo dosis
crecientes de analgesia. Ha asociado vómitos en los últimos días. La familia reconoce enlentecimiento mental y episodios confusionales
recortados durante los cuales realizaba movimientos repetitivos de las manos y de succión. En los 4 días previos a su asistencia en
Urgencias, refería inestabilidad en la marcha, diplopía e hipersomnia diurna. Paciente desorientado parcialmente en el tiempo
(desconocía día del mes y de la semana), aunque orientado en espacio y persona. Bradipsíquico. Con rigidez de nuca. Borramiento de
bordes papilares con incipientes hemorragias peripapilares y desaparición del pulso venoso. Paresia bilateral del VI par. Deficiente
estado nutricional y palidez cutáneomucosa. Hemograma: anemia normocítica y normocrómica (hemoglobina 10.5 g/dl). Leucocitos
3200 con linfopenia (15 %). Serie plaquetaria normal. Velocidad de sedimentación globular 56 mm en la primera hora. Hipernatremia
leve (149 mg/dl), hipertransaminasemia (glutámico-oxalacético transaminasa 78 unidades por litro, glutámico-pirúvico transaminasa
120 unidades por litro, gamma-glutamil-transpeptidasa 320 unidades por litro), resto de parámetros normales. Estudio de coagulación
normal, salvo hiperfibrinogenemia (765). Hipergammaglobulinemia policlonal e hipoalbuminemia discreta.

CURSO ENARM CMN SIGLO XXI TEL: 36246001 Pharmed Solutions Institute PÁGINA 157
MANUAL DE TRABAJO DEL CURSO ENARM CMN SIGLO XXI
PREGUNTA
Cuál de los siguientes diagnósticos le parece el más probable?

RESPUESTA
a.- Complejo demencia-SIDA (Síndrome de Inmunodeficiencia Adquirida)
d.- Enfermedad de Alzheimer en estadio leve
c.- Pseudodemencia depresiva
d.- Meningitis tuberculosa

CASO CLINICO
Agricultor, del sexo masculino, de 64 años, admitido en Urgencias con un cuadro de fiebre baja, mialgias, artralgias e hiporexia, hacía ya
tres semanas y evolucionando hacía ya tres días con tos, disnea y compromiso del estado general. A su llegada, estaba taquipneico,
depletivo, con vómitos, confusión mental y agitación. Presentaba de rigidez cervical terminal. Estertor crepitante en la base del
hemitórax derecho. Soplo diastólico en foco mitral +2/+6, sin irradiación y soplo sistólico en foco aórtico +3/+6, con irradiación cervical.
Leucograma de 18.700 cels/mm³ y 6% de bastones. La radiografía de tórax mostró opacidad alveolar en la base derecha. El paciente
evolucionó con insuficiencia respiratoria aguda, siendo necesario la realización de intubación orotraqueal y ventilación mecánica. Fue
realizada tomografía computadorizada (TC) de cráneo que no arrojó alteraciones, y la punción lumbar con líquido cefalorraquídeo
presentando 78 cels/mm³, siendo 87% polimorfonucleares, hipoglicorraquia.

PREGUNTA
Cual es la conducta terapéutica mas apropiada a seguir.

RESPUESTA
a.- Ceftriaxone mas vancomicina.
b.- Meropenen mas aciclovir.
c.- Amikacina mas levofloxacina.
d.- Bencilpenicilina benzatinica.

ENFERMEDADES VASCULARES DEL SISTEMA NERVIOSO: Comprenden un conjunto de trastornos de la vasculatura cerebral que
conllevan a una disminución del flujo sanguíneo en el cerebro (flujo sanguíneo cerebral o FSC) con la consecuente afectación, de
manera transitoria o permanente, de la función de una región generalizada del cerebro o de una zona más pequeña o focal, sin que
exista otra causa aparente que el origen vascular. La enfermedad cerebrovascular trae como consecuencia procesos isquémicos o
hemorrágicos, causando o no la subsecuente aparición de sintomatología o secuelas neurológicas. La hipertensión arterial (HTA) es el
principal factor de riesgo de la enfermedad cerebrovascular. Epidemiología: Actualmente la cifra de muertes por ECV supera los 5
millones anuales, lo que equivale a 1 de cada 10 muertes. Las enfermedades cerebrovasculares ocupan el tercer lugar como causa de
muerte en el mundo occidental, después de la cardiopatía isquémica y el cáncer, y la primera causa de invalidez en personas adultas
mayores de 65 años. La enfermedad cerebrovascular fue también la quinta causa principal de pérdida de productividad, medido por los
años de vida ajustados por discapacidad. Ello incluye los años de pérdida de productividad por razón de muerte o distintos grados de
discapacidad. Las mujeres son más propensas a padecer ECV, sobre todo cuando han perdido el factor protector estrogénico. Factores
de Riesgo: El consumo de cigarrillos es el factor de riesgo modificable más poderoso que contribuye a la enfermedad cerebrovascular,
independiente de otros factores de riesgo. Otros factores de riesgo demostrados son la hipertensión arterial y la diabetes mellitus. Por
su parte, las dislipidemias suelen ser factores de riesgo más importantes en la enfermedad coronaria que en la cerebrovascular.
Clasificación: Según el tiempo de evolución de la ECV se agrupan en: Accidente isquémico transitorio (AIT). Es cuando los síntomas de la
focalidad neurológica se recupera en menos de 24 horas sin secuelas, de inicio súbito que por lo general dura menos de 15 minutos.
Déficit isquémico neurológico reversible (RIND). Es cuando la duración del déficit persiste por más de 24 h, pero los síntomas
desaparecen en un plazo de 7 a 21 días, sin secuelas. Accidente cerebrovascular (ACV) establecido: el déficit neurológico de origen
vascular persiste y no se modifica por más de tres semanas desde su instauración y deja secuelas. ACV estable: el déficit neurológico
persiste sin modificaciones por 24 horas (en los casos de origen carotídeo) o 72 horas (en los casos de origen vertebrobasilares),
pudiendo luego evolucionar hacia un RIND o ACV establecido. ACV en evolución o progresivo: la focalidad neurológica aumenta y el
cuadro empeora o aparece nueva clínica en 24 a 48 horas. ACV con tendencia a la mejoría o secuelas mínimas: son casos con un curso
regresivo de modo que la recuperación al cabo de 3 semanas es mayor al 80%. Lo más frecuente es dividirlos en dos grupos según el
mecanismo, así suele ser de tipo isquémico o hemorrágico: Isquémico (85% de los casos). Trombótico (Infarto lacunar: oclusión de
vasos cerebrales pequeños ocluyendo la irrigación sanguínea de un volumen pequeño de tejido cerebral. Infarto de un gran vaso
sanguíneo. Embólico (Cardioembólico: la embolia proviene del corazón, con frecuencia, de la aurícula cardíaca). Arteria-arteria.
Criptogénico: la oclusión de un vaso intracraneal sin causa aparente. Hemorrágico: (Intraparenquimatoso, Subdural, Epidural,
Subaracnoideo). También pueden ser clasificados en función del área en el cerebro donde ocurre el mayor daño o por el territorio
vascular afectado y el curso clínico del trastorno. Etiología: Trombosis en pacientes con arteroesclerosis, la hemorragia cerebral
hipertensiva, el accidente isquémico transitorio, el embolismo y la rotura de aneurismas. Las malformaciones arteriovenosas, la
vasculitis y la tromboflebitis también causan con frecuencia ECV. Otras causas menos frecuentes incluyen ciertas alteraciones
hematológicas como la policitemia y la púrpura trombocitopénica, los traumatismos de la arteria carótida, los aneurismas disecantes de
la aorta, la hipotensión arterial sistémica y la jaqueca con déficit neurológico. Fisiopatología: El flujo sanguíneo cerebral (FSC) es
aproximadamente 15-20% del gasto cardíaco total, de 550 - 750 ml de sangre por cada kg de tejido por cada minuto. La disminución o
interrupción del flujo sanguíneo cerebral produce en el parénquima cerebral daño celular y funcional que empeora con el transcurrir
del tiempo. Penumbra isquémica: En el tejido cerebral donde ocurre la isquemia resultado de la disminución del FSC, se distinguen dos
zonas: Un núcleo isquémico de isquemia intensa: A los 10 s de isquemia se aprecia pérdida de la actividad eléctrica neuronal por
alteraciones en los potenciales de membrana notable en el EEG. A los 30 s se observa fallo de la bomba sodio-potasio con alteraciones

CURSO ENARM CMN SIGLO XXI TEL: 36246001 Pharmed Solutions Institute PÁGINA 158
MANUAL DE TRABAJO DEL CURSO ENARM CMN SIGLO XXI
en el flujo ionico y desequilibrio osmótico con pérdida de la función neuronal y edema citotóxico. Al cabo de 1 min y por el predominio
de la glucólisis anaeróbico, aumenta a niveles letales la concentración de ácido láctico y los mediadores de la cascada isquémica.
Después de 5 min se aprecian cambios irreversibles en los orgánulos intracelulares y muerte neuronal. Rodeando a este núcleo
isquémico evoluciona la llamada penumbra isquémica donde el efecto de la disminución en el flujo sanguíneo cerebral, el cual ha
descendido a niveles críticos alrededor de 15 a 20 mL/100 g/min,7 no ha afectado la viabilidad celular. La extensión del área de
penumbra depende del mejor o peor funcionamiento de la
circulación colateral. Vulnerabilidad celular: Las neuronas más
sensibles a la isquemia son las células de la corteza cerebral, del
hipocampo, el cuerpo estriado y las células de Purkinje del
cerebelo. De las neuroglías, se afecta primero los
oligodendrocitos, los astrocitos (gliosis reactiva) y por último la
microglía. Las células del endotelio vascular son las últimas en ser
afectadas. También se ha notado una menor concentración de
células progenitoras endoteliales en pacientes con enfermedad
cerebrovascular. Factores de influencia: Ciertos factores
participan en el daño cerebral progresivo, como el calcio, acidosis
láctica, radicales libres, glutamato, el factor de adhesión
plaquetaria y la descripción genética del individuo. El daño por
isquemia cerebral se verá mayor o menor dependiendo también
de: El estado del flujo sanguíneo cerebral regional; El tiempo que
dura la oclusión vascular; El funcionamiento de la criculación
colateral; El grado de vulnerabilidad celular frente a la isquemia;
La presencia de sustancias vasoactivas como ácidos grasos y
radicales libres en la zona afectada; Hiperglicemia; Hipertermia;
Los valores de la tensión arterial; El grado de hipoxia. Cuadro clínico: La presentación clínica de la ECV se da fundamentalmente de dos
formas: aguda, manifestada por el accidente cerebrovascular y la hemorragia subaracnoidea; y una forma crcónica manifestada por
demencia y epilepsia. Síntomas neurológicos como debilidad, cambios del lenguaje, visión o cambios en la audición, trastornos
sensitivos, alteración del nivel de conciencia, ataxia, u otros cambios en la función motora sensorial. La enfermedad mental puede
también producir trastornos de la memoria. Diagnóstico: Ante la sospecha de enfermedad cerebrovascular, se necesita identificar la
lesión y su ubicación y obtener información sobre el estado estructural del parénquima del cerebro y su condición hemodinámica como
consecuencia de la lesión. La evaluación neuropsicológica de sujetos con daño producido por enfermedad cerebrovascular está
enfocada en conocer las funciones afectadas y depende del tipo de evento. En el infarto cerebral se estudia por imágenes radiológicas
los aspectos topográficos de la lesión, especialmente antes de las 24 horas del inicio del trastorno súbito. Los más utilizados son la
tomografía computarizada, la resonancia magnética y el estudio del flujo sanguíneo regional cerebral. Tratamiento: El tratamiento
debe ser individualizado, según las condiciones de cada paciente y la etapa de la enfermedad cerebrovascular, sopesando los riesgos
frente a los posibles beneficios. En general, hay tres etapas de tratamiento: la prevención del accidente cerebrovascular; la terapia
provista inmediatamente después de la persona sufrir un accidente cerebrovascular; y la rehabilitación del paciente después de sufrir el
accidente cerebrovascular. Para la prevención de eventos cerebrovasculares, un estudio demostró que el uso de ramipril era efectivo
en pacientes de alto riesgo con o sin hipertensión arterial o ECV previo. La vitamina E no parece ser efectivo en reducir el riesgo de
accidente cerebrovascular fatal o no fatal. Terapia farmacológica: El tratamiento médico está destinado a reducir los riesgos y/o
complicaciones de un accidente cerebrovascular a corto y largo plazo. El uso de antitrombóticos se indica tan pronto como se ha
descartado una hemorragia intracraneal. Las enfermedades cerebrovasculares no cardioembólicas, no hemorrágicas son tratadas con
agentes antiplaquetarios, en lugar de la anticoagulación oral como tratamiento inicial. La aspirina, administrada entre 50-325 mg
diarios, o combinada con dipiridamol de liberación prolongada, y el clopidogrel (75 mg diarios) son las opciones recomendadas de
primera línea. La combinación de aspirina y clopidogrel, que bloquea la activación de la plaqueta por el difosfato de adenosina, puede
ser superior a la aspirina sola en la reducción de riesgo de eventos cerebrovasculares isquémicos. La aspirina en combinación con el
clopidogrel aumenta el riesgo de hemorragia y no se recomienda combinada de forma rutinaria para los pacientes con accidente
isquémico transitorio. Para casos con enfermedad cerebrovascular cardioembólico como la fibrilación auricular, prótesis de las válvulas
cardíacas o prolapso de la válvula mitral, se indica la anticoagulación a largo plazo, principalmente con heparina no freccionada,
obteniendo un INR de 1,5-2,5. La administración de 325 mg diarios de aspirina se recomienda para aquellos que no pueden tomar
anticoagulantes orales. En casos de infarto agudo de miocardio con trombo ventricular izquierdo puede combinarse la anticoagulación
oral con aspirina hasta 162 mg diarios. En pacientes con miocardiopatía dilatada también se indican anticoagulantes orales o algunos
clínicos consideran iniciar la terapia antiplaquetaria. Normalmente no se recomienda añadir agentes antiplaquetarios a la warfarina en
casos con enfermedad reumática de la válvula mitral, a menos que el paciente tenga una embolia recurrente a pesar de tener un INR
terapéutico. En casos de calcificación del anillo mitral se suele administrar tratamiento antiplaquetario. Los pacientes con regurgitación
mitral pueden recibir warfarina o aspirina. Cerca de un 4-28% de los pacientes con hemorragia intracerebral presentan convulsiones, las
cuales pueden ser rápidamente controladas con una benzodiazepina, como lorazepam o diazepam, acompañado de fenitoína o
fosfenitoína. El uso de una terapia anticonvulsiva de manera profiláctica en todos los casos de hemorragia intracerebral es
controvertido, ya que no hay ensayos controlados que han demostrado un beneficio claro. La trombólisis con activador tisular del
plasminógeno se ha definido como el tratamiento de primera línea del infarto isquémico agudo, pero debe ser administrada durante las
tres horas posteriores al accidente cerebrovascular. Control de la tensión arterial: Aunque no hay estudios controlados que definan los
niveles óptimos de presión arterial en pacientes con ECV, el seguimiento de los niveles de presión arterial es importante. Se cree que la
presión arterial muy elevada puede conducir a nuevas hemorragias y/o la expansión de un hematoma intracraneal. Por otra parte, la
bajada súbita de una presión arterial elevada puede comprometer la perfusión cerebral. Las dos excepciones al manejo conservador de
la hipertensión arterial son posterior al uso de activador del plasminógeno tisular y ante un concomitante infarto de miocardio. El
nicardipino, labetalol, esmolol, y la hidralazina son agentes que pueden ser utilizados cuando sea necesario el control de la presión

CURSO ENARM CMN SIGLO XXI TEL: 36246001 Pharmed Solutions Institute PÁGINA 159
MANUAL DE TRABAJO DEL CURSO ENARM CMN SIGLO XXI
arterial. No se suele usar el nitroprusiato ya que puede elevar la presión intracraneal. La American Heart Association publica las
siguientes pautas para el tratamiento de la hipertensión arterial: Si la presión arterial sistólica es> 200 mmHg o la presión arterial media
(PAM) es > 150 mmHg, se considera la reducción agresiva de la presión arterial con la infusión intravenosa continua con valoraciones
frecuentes de la presión arterial (cada 5 min). Si la presión arterial sistólica es > 180 mmHg o PAM es > 130 mmHg y hay evidencia o
sospecha de hipertensión intracraneal (PIC), entonces se considera la vigilancia de la PIC y se reduce la presión arterial con el uso de
medicamentos por vía intravenosa de manera intermitente o continua para mantener la presión de perfusión cerebral > 60-80 mmHg.
Si la presión arterial sistólica es > 180 o PAM es > 130 mmHg y no hay pruebas ni sospecha de elevación de la PIC, se considera una
modesta reducción de la presión arterial (PAM diana de 110 mmHg o presión arterial deseada de 160/90 mmHg) con controles de la
presión arterial cada 15 minutos. Pronóstico: La complicación más catastrófica de los diversos tipos de enfermedad cerebrovascular es
la isquemia súbita e irreversible de alguna parte del cerebro, es decir, el accidente cerebrovascular, especialmente frecuente en
ancianos. La severidad varía, desde la recuperación total de las funciones cerebrales y aquellas que del cerebro dependen, en menos de
24 horas, hasta la discapacidad severa y la muerte. La mortalidad por ECV isquémico en el primer mes oscila entre el 17 y 34%, mientras
la del hemorrágico puede ser dos veces mayor.

CASOS CLINICOS
Mujer de 73 años sin factores de riesgo vascular que acudió a urgencias por presentar cuadro de inicio súbito de disminución de la
movilidad en hemicuerpo izquierdo con caída al suelo. A su llegada a urgencias estaba consciente con desviación conjugada de la
mirada a la derecha y hemiplejía completa izquierda, hemihipoestesia y reflejo de Babinsky izquierdo presente. Puntuación en la escala
NIHSS de 16. TC craneal sin lesiones evidentes. Tras consentimiento informado escrito se aplicó tratamiento trombolítico a los 150min
del inicio de los síntomas.

PREGUNTA
Considerando la sintomatologia, cual es la arteria mas probablemente involucrada?

RESPUESTA
a.- Arteria cerebral anterior izquierda.
b.- Arteria cerebral media Izquierda.
c.- Arteria cerebral anterior derecha.
d.- Arteria cerebral media derecha.

CASO CLINICO
Varón de 75 años, con dislipemia en tratamiento con hipolipemiante como único factor de riesgo cardiovascular. Traído a urgencias por
cuadro de hemiparesia izquierda, parálisis facial central izquierda, hipoestesia en hemicuerpo izquierdo, disartria y Babinsky izquierdo.
TC cráneo sin alteraciones. NIHSS 19. Tras consentimiento informado escrito se administró tratamiento trombolítico con 150min de
evolución desde el inicio de los síntomas.

PREGUNTA
Considerando la sintomatologia, cual es la arteria mas probablemente involucrada?

RESPUESTA
a.- Arteria cerebral anterior izquierda.
b.- Arteria cerebral media Izquierda.
c.- Arteria cerebral anterior derecha.
d.- Arteria cerebral media derecha.

CASO CLINICO
Varón de 54 años fumador y con episodio compatible con accidente isquémico transitorio dos años antes. Trasladado a urgencias por
hemiplejía derecha de 5 horas de evolución, objetivándose plejia completa de la extremidad superior y paresia de la extremidad
inferior, NIHSS 7.

PREGUNTA
Cual es la conducta a seguir mas apropiada al caso?

RESPUESTA
a.- Tratamiento sintomático.
b.- Tratamiento trobolitico.
c.- Tratamiento antiplaquetario.
d.- Tratamiento anticoagulante.

CASO CLINICO
Mujer de 52 años fumadora, sin otro factor de riesgo que fue trasladada a urgencias por hemiplejía derecha y afasia de instauración
brusca. Destacó a su ingreso hemiplejía completa derecha y Babinsky derecho, NIHSS de 18. Se realizó TC craneal sin que se objetivaran
alteraciones. Tras realizar consentimiento informado se aplicó trombolisis a los 150min del inicio de los síntomas. A las 24h persistía en
la misma situación neurológica, realizando TC de control que mostraba infarto isquémico extenso con edema y efecto masa, por lo que
se asoció tratamiento antiedematoso. Pese a ello 48h después presentó rápido deterioro neurológico, con nuevo TC mostrando

CURSO ENARM CMN SIGLO XXI TEL: 36246001 Pharmed Solutions Institute PÁGINA 160
MANUAL DE TRABAJO DEL CURSO ENARM CMN SIGLO XXI
progresión del edema y signos de enclavamiento de tronco, desestimándose cirugía descompresiva. Finalmente la paciente fue incluida
en protocolo de donación de órganos.

PREGUNTA
Considerando la sintomatologia, cual es la arteria mas probablemente involucrada?

RESPUESTA
a.- Arteria cerebral anterior izquierda.
b.- Arteria cerebral media Izquierda.
c.- Arteria cerebral anterior derecha.
d.- Arteria cerebral media derecha.

ESCLEROSIS MÚLTIPLE (EM). CIENCIAS BÁSICAS: La esclerosis múltiple es caracterizada por la inflamación crónica (reacción linfocitaria
y mononuclear) y destrucción selectiva de la mielina del SNC, importante con conservación de los axones, el sistema nervioso periférico
está a salvo. Hay daño de los oligodendrocitos con proliferación astrocí ca. Los hallazgos anteriores junto con la pérdida de la mielina
cons tuyen la placa de desmielinización. Aunque pueden verse en la médula espinal, el tallo cerebral y el nervio óptico, la localización
más frecuente es la periventricular (90%). Estas también
pueden encontrarse en la sustancia gris, principalmente en CRITERIOS DE SCHUMACHER:
Examen neurológico que evidencie compromiso del sistema nervioso
localización subpial. Las placas aisladas raramente miden Evidencia clínica de 2 o más lesiones del sistema nervioso
más de 1,5 cm. Etiología se piensa que es autoinmune, Compromiso principalmente de la sustancia blanca
con la susceptibilidad determinada por factores genéticos El compromiso del sistema nervioso debe seguir el siguiente patrón: dos o más
y ambientales. SALUD PÚBLICA: La prevalencia de la episodios de la enfermedad con una duración no menor de 24 hrs y usualmente
menos de un mes, o una lenta progresión de la enfermedad por un periodo hasta
esclerosis m l ple es muy variada en diferentes partes del de 6 meses.
mundo, y es así como en las la tudes norte la enfermedad Inicio de los síntomas en edades comprendidas entre 10-50 años
es más frecuente. Por ejemplo, su prevalencia en algunas Los signos y síntomas no deben ser mejor explicados por otra enfermedad
zonas de Escocia es de 309 por cada 100.000 habitantes, Esclerosis múltiple definitiva apoyada en clínica
Según clínica, la esclerosis múltiple definitiva deberá reunir una de estas
mientras que en el Norte de los Estados Unidos la condiciones:
prevalencia es de 58 por cada 100.000 habitantes y en 2 episodios de la enfermedad y evidencia de 2 lesiones separadas
Japón, África y Sudamérica la prevalencia desciende hasta 2 episodios de la enfermedad, evidencia clínica de una lesión y paraclínica de otra.
cuatro personas por cada 100.000 habitantes. La incidencia
de la enfermedad es casi 1,8 veces mayor en mujeres que en hombres. Compromete más la raza blanca y la edad de aparición más
frecuente está entre los 20 y los 40 años. ASPECTOS CLINICOS: Lo más común son los ataques recurrentes de disfunción neurológica
focal, por lo general duran semanas o meses. Los pacientes con la forma de recaída remisión tienen en promedio un episodio de
disfunción neurológica por año. Cada recaída es seguida por una secuela o recuperación total de su cuadro clínico, las recaídas están
separadas por un periodo asintomático. Cincuenta por ciento de los pacientes con la forma de recaída remisión pueden virar a un
deterioro más progresivo que se denomina la forma secundariamente progresiva. Aproximadamente 10 a 15 % de los pacientes tienen
una forma primariamente progresiva. En esta forma de la enfermedad hay un deterioro progresivo desde su inicio. Esta es quizás la
clasificación clínica más reciente de esclerosis múltiple: Asintomática (hallazgos incidentales en RMN de cerebro) y la sintomática que
son: 1. Recaída remisión; se caracteriza por ataques recurrentes de disfunción neurológica general con o sin recuperación entre los
ataques, no se observa la progresión de la discapacidad neurológica. Representa el 85 % de los casos de EM de nueva aparición. 2.
Secundariamente progresiva; no siempre se presenta inicialmente como recaída remisión sino que evoluciona para ser de forma
progresiva. 3. Primariamente progresiva; se caracteriza por una progresión gradual de la discapacidad desde el inicio sin ataques
discretos, el 15% de los casos de EM de nueva aparición. 4. progresiva con exacerbación; es una forma rara que comienza con un curso
de primaria progresiva, pero se producen recaídas después superpuestas. La EM es una enfermedad crónica, 15 años después del
diagnóstico, sólo el 20 % de los pacientes tienen ninguna limitación funcional, de un tercio a la mitad habrán progresado y requerirán
ayuda para la deambulación. Los síntomas pueden desarrollarse de minutos a horas en 40% de los enfermos, durante varios días en
30% y lentamente durante semanas a meses, en 20% de los pacientes. DIAGNOSTICO: Clínico; El inicio puede ser brusco o insidioso.
Algunos pacientes tienen síntomas que son tan triviales que no pueden buscar atención médica durante meses o años. Algunos
pacientes se presentan inicialmente con deterioro neurológico lentamente progresivo. Los síntomas a menudo empeoran
transitoriamente con fatiga, el estrés, el ejercicio o el calor. Las manifestaciones de la EM normalmente incluyen debilidad y/o síntomas
sensoriales que implican una extremidad, dificultades visuales, alteraciones de la marcha y la coordinación, urgencia o frecuencia
urinaria y fatiga anormal. Participación Motor puede presentarse como una pesada rigidez débil o torpe extremidad. Hormigueo
localizado, "alfileres y agujas" y sensaciones "muertos" son comunes. La neuritis óptica puede resultar en visión borrosa, especialmente
en el campo visual central, a menudo con dolor retroorbital acentuado por el movimiento del ojo. La participación del tronco cerebral
puede causar diplopía, nistagmos, vértigo o dolor facial, entumecimiento, debilidad, hemiespasmo o mioquimia (ondulación
contracciones musculares). Ataxia, temblor, disartria y siempre son representativas de la enfermedad de las vías cerebelosas. Síntoma
de Lhermitte, una sensación momentánea eléctrica similar al shock provocado por la flexión del cuello, indica enfermedad en la médula
espinal cervical. CRITERI S DIAGN STIC S: El diagnóstico de esclerosis múltiple es de predominio clínico, y existen algunos criterios,
como los de Schumacher, que todavía permanecen vigentes. Con el advenimiento de nuevos estudios paraclínicos, como los
potenciales evocados (PE), RMN (revela las áreas brillantes multifocales en las secuencias ponderadas en T2 en > 95 % de los pacientes,
a menudo en una ubicación periventricular) y el estudio de LCR (Hallazgos pleocitosis linfocitaria leve 5-75 células (25 %) , bandas
oligoclonales (75-90 %) , elevación de IgG (80 %) ) TRATAMIENTO: Aunque aún no existe un tratamiento definitivo para la EM. En la
actualidad, para los brotes o periodos de exacerbación de la enfermedad se utilizan la hormona adrenocorticotropa (ACTH) y otros
corticoides. La ACTH puede administrarse siguiendo un protocolo de dosis elevadas de la medicación que incluye 80 UI diarias
intramusculares o intravenosas por siete días, seguidas de 40 UI intramusculares o intravenosas por cuatro días, finalizado con 20 UI
por tres días. En la actualidad, se utiliza en la mayoría de los centros la metilprednisolona, a dosis altas, de 500 mg a 1 g IV diarios por

CURSO ENARM CMN SIGLO XXI TEL: 36246001 Pharmed Solutions Institute PÁGINA 161
MANUAL DE TRABAJO DEL CURSO ENARM CMN SIGLO XXI
tres a cinco días. Se recomienda administrar esta medicación en infusión, disolviéndola en 100 ml de dextrosa al 5%, para ser
administrada en 30 minutos a 1 hora. Si la sintomatología del paciente persiste después de un mes a pesar de este tratamiento, se
recomienda repetir el ciclo de metilprednisolona, seguido por la administración de 1 mg/kg de peso de prednisona interdiaria,
reduciéndola en el curso de 8 a 14 semanas. Se han realizado muchos estudios utilizando azatioprina; sin embargo, los resultados no
son muy buenos. Se han utilizado muchos otros tratamientos, como la ciclosporina, la plasmaféresis, los an cuerpos monoclonales, el
oxígeno hiperbárico y algunos más que están en inves gación sin resultados sa sfactorios. El interferón beta (reduce las tasas de
exacerbación anual aprox. 30 % y también reduce el desarrollo de nuevas lesiones de MRI) ha demostrado ser muy útil en el
tratamiento de la esclerosis múltiple especialmente en la forma de recaídas y remisión, ha sido recomendado por la Academia
Americana de Neurología en sus guías de manejo. Se ha demostrado que la medicación disminuye el número de recaídas y la severidad
de las mismas. Esta medicación debe utilizarse de por vida, si no hay disminución en el número de recaídas y no se observa beneficio
debe retirarse la medicación. Terapia sintomática: la espasticidad puede responder a la terapia física , lioresal (20-120 mg/d) , diazepam
(2-40 mg/d) , tizanidina (8-32 mg/d) , dantroleno (25-400 mg/d) y clorhidrato de ciclobenzaprina (10-60 mg/d) . Disestesia puede
responder a la carbamazepina (100-1200 mg/día) , fenitoína (300 mg/d) , gabapentina (300-3.600 mg/d), pregabalina (50-300 mg/d) o
amitriptilina (50-200 mg/d) .El tratamiento de los síntomas de la vejiga se basa en la fisiopatología subyacente investigados con pruebas
urodinámicas: hiperreflexia vesical es tratado con restricción de líquidos por la noche y la micción frecuente, y si esto no funciona,
anticolinérgicos como oxibutinina ( 5-15 mg/d) puede ser juzgado; hiporreflexia es tratado con el betanecol colinérgica drogas (10-50
mg c/8hrs) , y disinergia debido a la pérdida de la coordinación entre la pared de la vejiga y los músculos del esfínter se trata con
anticolinérgicos y cateterismo intermitente . La depresión debe ser tratada agresivamente.
PROGRESIVA: Esta enfermedad se observa en pacientes con trastornos linfoprolifera vos, como la enfermedad de Hodg in, leucemia
linfocí ca y linfosarcomas. También puede presentarse en pacientes con síndrome de inmunodeficiencia adquirida (SIDA). El JC
papovavirus ha sido encontrado en la mayoría de los casos de leucoencefalopa a mul focal progresiva. La enfermedad cursa en forma
subaguda con cambios de personalidad y demencia. Otros síntomas muy comunes son hemianopsia, disartria y ataxia para la marcha. El
curso de la enfermedad es progresivo e inexorablemente lleva al paciente a la muerte en un periodo de algunos meses. El LCR puede
ser normal, la TAC de cerebro y la RMC ayudan mucho para el diagnóstico y muestran el compromiso de la sustancia blanca. Un
diagnóstico definitivo de la enfermedad solo puede hacerse por patología. : Esta entidad la describieron primero
Adams y Mancall (1959). Clínicamente se caracteriza por una paraparesia o cuadriparesia de evolución rápida, con síntomas
seudobulbares, que se manifiestan por disartria y disfagia, en pacientes con cambios electrolíticos importantes. La causa real de la
mielinolisis no es clara, pero se han postulado teorías vasculares y metabólicas. El cuadro se observa en alcohólicos y pacientes a
quienes se les corrige rápidamente una hiponatremia. Esta entidad también se ha visto asociada a otras patologías, como la
enfermedad de Wilson, la cirrosis, las nefropatías y la enfermedad de Wernicke. Histológicamente la lesión compromete toda o parte
de la base del puente. Puede afectar el tegmento pón co y ascender hasta el mesencéfalo, produciendo desmielinización
importante. Para diagnosticar la enfermedad son de gran importancia los potenciales evocados auditivos y la RMC. El tratamiento de
estos pacientes deberá hacerse con solución salina, administrada con prudencia, y restricción de líquidos. ENCEFALOMIELITIS
DISEMINADA AGUDA (ADEM): Un fulminante, a menudo devastadora, enfermedad desmielinizante que tiene un curso monofásico y
puede estar asociada con la inmunización antecedente o infección. Los signos de enfermedad neurológica diseminada son siempre
presentes (por ejemplo, hemiparesia o tetraparesia, respuesta plantar extensora, perdidos o reflejos tendinosos hiperactivos, pérdida
de la sensibilidad, y la participación de tronco cerebral). Puede presentarse fiebre, dolor de cabeza, meningismo, letargia progresando
al coma y convulsiones. Pleocitosis, generalmente 200 células/l, es común. RM puede revelar extensa realce de la materia blanca en el
cerebro y la médula espinal. El tratamiento inicial es con glucocorticoides a dosis altas. Los pacientes que no responden pueden
beneficiarse de un curso de la plasmaféresis o inmunoglobulinas.

CASO CLINICO
Paciente de 45 años con un cuadro clínico que comenzó hace 2 años con ligera disminución de la fuerza muscular, debilidad e
inestabilidad para la marcha, tuvo una evolución desfavorable y en la actualidad muestra falta de coordinación, alteraciones visuales,
rigidez muscular y trastornos del habla. Acude a nuestro centro para realizarse Resonancia Magnética por Imágenes de Cráneo y
Columna cervical, encontrándose las siguientes alteraciones, en la Resonancia Magnética por Imágenes (RMI) de Cráneo (Secuencias
T1, T2 y Flair axial), observamos imágenes ovoides, algunas miden 6 mm de diámetro, perpendiculares al eje mayor de los ventrículos
que se observan isointensas y algunas hipointensas en T1 y que en T2 y Flair se muestran hiperintensas, localizadas nivel del cuerpo
calloso, en la sustancia blanca profunda periventricular y a nivel de la protuberancia y bulbo; por las alteraciones antes descritas se
solicita realizar Resonancia Magnética por Imágenes (RMI) de columna cervical (secuencias T1 y T2 sagital y axial T2), donde
visualizamos a nivel de médula áreas focales hiperintensas con muy ligero efecto de masa y de características similares a las del
encéfalo.

PREGUNTA
Considerando la clasificación de esta patologia, cual es el grado mas probable del caso?.

RESPUESTA
a.- 1.
b.- 2.
c.- 3.
d.- 4.

CASO CLINICO
Masculino de 68 años de edad quien, previamente sano, sufre un accidente de tránsito al ser atropellado; produciéndose politrauma
severo y fractura de base de cráneo, fractura expuesta del tercio medio de la tibia derecha y trauma de la rodilla izquierda. Al ingreso
Glasgow 3 se describe pupila izquierda media no reactiva, sospechando un III par craneal compresivo se realiza TC que corrobora la

CURSO ENARM CMN SIGLO XXI TEL: 36246001 Pharmed Solutions Institute PÁGINA 162
MANUAL DE TRABAJO DEL CURSO ENARM CMN SIGLO XXI
fractura y demuestra una hemorragia subaracnoidea sin hematoma; se instaura tratamiento con fenitoína y diuréticos; se considera
que presenta un III par craneal periférico traumático. En 12 horas Glasgow 7. A los 2 días movilizaba las 4 extremidades y abría los ojos
al llamado, no acataba órdenes; posteriormente su sensorio mejoró manteniendo un Glasgow de 14. Durante la estancia el paciente
recibió varios esquemas de antibióticos, debido a neumonía nosocomial asociada a ventilación mecánica la cual requirió los tres
primeros días. Al mes de estancia presenta progresivo descenso de la natremia con valores desde 126 mmol/l, hasta 108 a las dos
semanas cuando se inició el tratamiento de reposición; concomitantemente el paciente tenía progresivo deterioro del sensorio.
Durante las primeras horas la corrección se logró llevar a un ritmo adecuado, pero no ocurrió así a las 12 horas de reposición que se
pasó de un sodio de 116 a 125 mmol/l en 5 horas. A los 2 días el paciente nuevamente conciente pero se anota que presenta dificultad
para la deglución; luego presenta un cuadro de diarrea asociada a antibióticos que lo hace llegar a una natremia de 138 mmol/l.

PREGUNTA
Cual es la velocidad de corrección de sodio para evitar esta complicación del caso?

RESPUESTA
a.- 05 mmol/l/dia.
b.- 10 mmol/l/dia.
c.- 15 mmol/l/día.
d.- 20 mmol/l/dia.

CASO CLINICO
Paciente de sexo masculino, 44 años, diestro, infección por VIH, en tratamiento antirretroviral. Presentaba una población linfocitaria
con CD4 inferior a 160 elementos/mm3. Comienza con un síndrome confusional, alteraciones conductuales y cefalea moderada. A los
pocos días de su ingreso se percibe pérdida de fuerza de sus miembros izquierdos y alteraciones del habla que se agravaron en el curso
de los días. No se detecta fiebre. El examen mostraba un paciente irritable, con desorientación temporal y espacial, tendencia al
mutismo, síndrome tónico frontal bilateral predominando a derecha. No se detectó síndrome meníngeo ni fiebre. La TC craneal en fase
de estado mostró una lesión frontal derecha y de cuerpo calloso, con compromiso menor del lóbulo frontal izquierdo que incidía
especialmente sobre la sustancia blanca, sin efecto de masa y con realce con el contraste. La RM encefálica delimitó con mayor
exactitud las lesiones observándose lesiones de baja señal en T1 y de aumento de señal en T2. Se resuelve practicar biopsia
estereotáxica, Hay acúmulos de histiocitos de citoplasmas microvacuolados por fagocitosis de restos mielínicos lipídicos, Existe
moderada astrogliosis con astrocitos de aspecto bizarro símil neoplásicos, con núcleos grandes, algunos multilobulados que no se
acompañan de aumento de densidad celular. La oligodendroglía muestra una cariomegalia hipercuomática y algunos de ellos tienen
gruesas inclusiones intranucleares acidófilas 'en vidrio esmerilado' que contienen el antígeno viral. Hay escaso o nulo exudado
inflamatorio perivascular linfo-plasmocitario".

PREGUNTA
Considerando la clínica y los hallazgos histológicos, cual es el diagnostico mas apropiado?

RESPUESTA
a.- Leucoencefalopa a mul focal progresiva.
b.- Encefalomielitis diseminada aguda.
c.- Mielinólisis pón ca.
d.- Escleroris lateral amniotrofica.

CASO CLINICO
Paciente de 21 años, con antecedentes de pan-colitis ulcerosa, en tratamiento esteroidal con prednisona 60 mg/día y Mesalazina 1 gr
c/6 hrs durante 5 meses previos al ingreso. Días previos al diagnóstico completó tratamiento con ciprofloxacino por síndrome diarreico
presuntamente bacteriano, sin confirmación etiológica y con respuesta favorable. Ingresó al Servicio de Urgencia por cuadro de 48
horas de evolución de compromiso de conciencia caracterizado por desorientación temporo-espacial y somnolencia, además de
deposiciones sanguinolientas. Al examen físico general presentaba parámetros hemodinámicos dentro de rangos normales, afebril y
como único hallazgo se describe candidiasis orofaríngea. Al examen neurológico el paciente se encuentra vigil, con tendencia a la
somnolencia, desorientado en tiempo y espacio, nistagmo vertical, prueba de pequeña paresia positiva a derecha, y signo de
Brudzinski. Resto del examen sin hallazgos. Se realiza TAC de cerebro que no muestra hallazgos patológicos y se realiza punción lumbar
que da salida a líquido cefalorraquídeo claro, con proteínas 74,8/mm3, glucorraquia normal, glóbulos rojos 2/mm3, leucocitos 760/
mm3 de predominio linfocítico. Por antecedente de inmunosupresión crónica y candidiasis orofaríngea al ingreso, se inicia tratamiento
antibiótico con ampicilina+ceftriaxona+aciclovir y corticoides en dosis de estrés (Hidrocortisona 100mg c/8 hrs). Evoluciona en buenas
condiciones generales, estable desde el punto de vista hemodinámico y recuperando conciencia a las 48 horas. Se realiza Resonancia
magnética que muestra imágenes con compromiso talámico bilateral asimético asociado a compromiso capsular posterior izquierdo
con discreta captación de contraste.

PREGUNTA
Considerando la clínica y los estudios de imagen, cual es el diagnostico mas apropiado?

RESPUESTA
a.- Leucoencefalopa a mul focal progresiva.
b.- Encefalomielitis diseminada aguda.
c.- Mielinólisis pón ca.

CURSO ENARM CMN SIGLO XXI TEL: 36246001 Pharmed Solutions Institute PÁGINA 163
MANUAL DE TRABAJO DEL CURSO ENARM CMN SIGLO XXI
d.- Esclerosis lateral amniotrofica.

DEMENCIAS:
CIENCIAS BASICAS: La demencia es un síndrome clínico caracterizado por perdida adquirida de habilidades cognitivas y emocionales, lo
suficientemente importante para interferir con el funcionamiento diario y la calidad de vida. También puede acompañarse de otras
manifestaciones neuropsiquiátricas tales como alteraciones motoras, de la conducta, depresión, ansiedad, alucinaciones y/o delirium.
El término demencia no implica una causa o proceso patológico específico, es de etiopatogenia heterogénea y puede ser mixta. Existen
más de 55 enfermedades que pueden causar demencia. La definición de demencia según Clasificación de la demencia de acuerdo al
DSM IV se divide de la siguiente manera: Demencia en enfermedad de Alzheimer; de inicio temprano, de inicio tardío, mixta o atípica,
inespecífica. Demencia vascular; de inicio agudo, multi-infarto, vascular subcortical, mixta: subcortical y cortical. Demencias propias de
otras enfermedades; Enfermedad de Pick, Enfermedad de Creutzfeldt-Jakob, Enfermedad de Huntington, Enfermedad de Parkinson,
asociada a infección por VIH, asociada a otras enfermedades. Demencia no especificada. Así mismo una vez hecho el diagnóstico de
demencia se puede clasificar: Leve: cuando afecta actividades de la vida diaria sin causar problemas para la independencia del paciente.
Moderado: cuando condiciona incapacidad para vivir de manera independiente. El individuo no recuerda información básica acerca de
su vivienda, actividades recientes o el nombre de ciertas personas familiares a él. Severo: Se caracteriza por la absoluta incapacidad de
retener nueva información y solo se recuerdan fragmentos de experiencias o conocimientos pasados. El paciente no reconoce a sus
familiares. SALUD PUBLICA: La E. Alzheimer en 2050 podría llegar al 30% de la población mundial. A medida que se prolonga la
expectativa de vida también aumenta la prevalencia de enfermedades como las demencias. La frecuencia de E. Alzheimer se duplica
cada 5 años apartir de los 60a. A los 60 años 1%, entre 80-84 años de 16%, >de 85años de 30-40%. Existen factores de riesgo no
modificables como la edad siendo este el más importante para el desarrollo de demencia. En los pacientes con trastorno del
aprendizaje como Síndrome de Down las demencias se presentan más temprano. Estudios de prevalencia muestran un mayor índice de
demencia en mujeres que en hombres, especialmente en enfermedad de Alzheimer (2:1). El número de casos de demencia vascular es
mayor en hombres que en mujeres aunque éstas últimas suelen igualarlos en edades más avanzadas. La historia familiar de demencia
en un pariente de primer grado aumenta cuatro veces el riesgo de desarrollar demencia. Demencia vascular es la segunda causa de
demencia después del Alzheimer representando hasta el 18% de las demencias, se han descrito prevalencias que varían desde el 3 al
21%. DIAGNOSTICO: Para un diagnóstico clínico confiable la pérdida de la memoria debe estar presente al menos durante seis meses; si
la duración de las manifestaciones es menor, el diagnóstico es presuntivo. La severidad de la demencia se establece por el grado de
decremento cognitivo o de la memoria, cualquiera que sea el más predominante. Cuando existen episodios concomitantes de delirium
el diagnostico de demencia debe ser diferido. El diagnóstico de las demencias se debe realizar por un especialista experto en el tema y
podría abordarse de la siguiente establece en dos pasos: se deberá hacer solo tras una extensa evaluación que debe incluir: historia
clínica, evaluación del estado cognitivo, examen físico y otros estudios apropiados, una revisión de los medicamentos tomados por el
paciente en orden de descartar efectos adversos que afecten el funcionamiento y minimizar su uso. ENFERMEDAD DE ALZHEIMER:
Síndrome clínico adquirido y progresivo caracterizado por afectar las funciones intelectuales superiores. Deterioro de la memoria en el
corto y largo plazo. Presencia de por lo menos uno de los siguientes déficits cognitivos: Afasia, Agnosias, Apraxia y/o alteraciones de
funciones ejecutivas. Alteración de conducta: Laboral y social. No debe existir alteración de nivel de conciencia. En su patogenia exiten
Hipótesis colinérgica: hay pérdida de neuronas colinérgicas en corteza e hipocampo con aumento de acción de butirilcolinesterasa.
Hipótesis glutamatergica: L-glutamato neurotransmisor exitatorio SNC, interviene en los procesos de memoria, aprendizaje y
plasticidad neuronal, el aumento parece jugar un rol principal en la patogénesis y en el daño producido por isquemia. Se genera por la
producción anormal y acumulación de ß amiloide, aumenta la producción de ßA1-42 que es el que se deposita. Se generan las placas
de amiloide que desencadenan, formación de ovillos neurofibrilares (por hiperfosforilación de proteína Tau). Oxidación y peroxidación
de lípidos. Excitotoxicidad por glutamato, inflamación, activación de la cascada que lleva a la apoptosis. La muerte neuronal lleva al
déficit de neurotrasmisores. En las guías de diagnóstico, se describe de manera didáctica 10 signos de alerta para pensar en Alzheimer:
1. Pérdida de la memoria 2. Dificultad para realizar tareas familiares 3. Problemas con el lenguaje 4. Desorientación en tiempo y espacio
5. Juicio empobrecido 6. Problemas con el pensamiento abstracto 7. Pérdida de cosas 8. Cambios en el ánimo o el comportamiento 9.
Cambios en la personalidad 10. Pérdida de la iniciativa Alteraciones neuropsiquiátricas y motoras. Conforme la enfermedad avanza se
presentan apraxias y con ello la incapacidad funcional para las actividades cotidianas, como asearse y vestirse, puede haber
desinhibición y agresividad, las alucinaciones son poco frecuentes y si se presentan el diagnóstico debe orientarse en otra dirección, sin
embargo pueden desarrollar delusiones, entre ellas se ha descrito el síndrome de Capgras hasta en el 10% de los pacientes, donde
generan la idea de que su cuidador ha sido sustituido por un impostor, paralelamente hay alteraciones en el patrón de sueño vigilia,
síntomas motores extrapiramidales como rigidez y sacudidas mioclónicas, hasta una etapa final de mutismo y postración. Tratamiento:
Inhibidores de la Acetilcolinesterasa: Donepecilo, rivasigmina, galantamina. Inhibidores de los receptores NMDA: Memantina.
Estabilizadores de membrana: Lamotrigina, carbamacepina, topiramato. Delirios y alucinaciones: Risperidona, olanzapina, haloperidol.
Depresión y ansiedad: citalopram, paroxetina, sertralina. DEMENCIA VASCULAR: Se ha acuñado el término de daño cognitivo vascular
para describir a la pérdida de funciones cognitivas asociada a otras manifestaciones neurológicas y que tienen como base el daño
vascular. Factores de riesgo, la edad, el bajo nivel de escolaridad, la enfermedad hipertensiva, el daño cerebral vascular previo,
especialmente infartos cerebrales, el padecer diabetes, obesidad, síndrome metabólico, niveles elevados de homocisteína y la
ateroesclerosis. Clasificación: Demencia por daño cortical o enfermedad multi infarto relacionada generalmente con el daño de grandes
vasos (23%) y la causada por daño subcortical o enfermedad difusa de la sustancia blanca relacionada a daño de pequeños vasos (50%).
La región más afectada son los ganglios basales, dentro del daño subcortical se hace referencia a tres tipos de entidades, 1) infartos
lacunares (16%), la distribución puede variar, sin embargo el daño se ha descrito con mayor frecuencia en la región frontal, 2) demencia
talámica (8%) donde el daño puede venir desde la arteria basilar y 3) la encefalopatía subcortical ateroesclerótica o síndrome de
Binswwanger donde hay gran daño a la sustancia blanca. El daño vascular, como ya se comentó puede ser secundario a infartos en
grandes vasos corticales o a nivel subcortical predominantemente ya sea por hemorragias o principalmente por eventos vasculares
isquémicos donde la hipoperfusión lleva a la gliosis y al daño en la sustancia blanca que posteriormente se manifestará como
leucoaraiosis, otro tipo de daño es la isquemia crónica subcortical que da afección principalmente a nivel periventricular. Edad de
inicio es en entre los 50 y 59 años, con otro pico entre los 60 y 69 años de edad. Clinica: pueden desarrollan alteraciones en la marcha,

CURSO ENARM CMN SIGLO XXI TEL: 36246001 Pharmed Solutions Institute PÁGINA 164
MANUAL DE TRABAJO DEL CURSO ENARM CMN SIGLO XXI
afectación de primera neurona motora, bradicinesia, rigidez, alteración en los esfínteres, síndrome pseudobulbar, alteraciones de la
personalidad y del estado de ánimo, afasia, agnosias y alteraciones en la memoria. Establecer criterios diagnósticos definitivos ha sido
complicado, las más utilizadas son las de Hachinski, Rosen. Tratamiento: donepezilo con dosis inicial de 5mg al día y de mantenimiento
de 10-20mg después de 4 semanas, rivastigmina con dosis inicial de 1.5mg, incrementando de 1.5mg diarios hasta lograr la dosis de
mantenimiento de 6mg o parches de 4.6mg por día con dosis paulatinas con un máximo de 9.6 mg/día. Dado que se trata de una
demencia que se puede prevenir, es importante dar tratamiento a los factores de riesgo. DEMENCIA FRONTOTEMPORAL: Tercera
demencia más común después de la Demencia tipo Alzheimer. La prevalencia general es de 17,6 por 100.000 habitantes. La edad de
presentación es aproximadamente a los 58 años. Afecta por igual a hombre y mujeres. En el 30-50 % de los casos existe una historia
familiar de Demencia frontotemporal (DFT) principalmente en la variante conductual, mientras que en la demencia semántica y la
afasia progresiva no fluentes menos frecuente. El término de Enfermedad de pick se reserva para casos de DFT con inclusiones
positivas teñidas de plata denominados cuerpos de Pick. Debido a su asociación de atrofia en lóbulos temporales y frontales. La
mutación característica se encuentra en el gen que codifica a la proteína tau en el cromosoma 17, cuyo patrón es de herencia
autosómico dominante. Esta proteína pertenece a la familia de las MAPT (proteínas tau asociadas a microtúbulos) y tiene participación
en la regulación de la polimerización y ensamble de las subunidades de tubulina, por lo que es indispensable para determinar la
estabilidad y función axonal. La DFT es una enfermedad progresiva con inicio insidioso, en etapas iniciales presentan más síntomas
conductuales, sin afectar la memoria ni las capacidades visuoespaciales. Puede ser esporádica o familiar. Pueden presentar diversas
alteraciones de acuerdo al síndrome de DFT, sin embargo se caracteriza por la presencia de afasia, alteraciones en la conducta y del
lenguaje. Las alteraciones en el lenguaje pueden existir con o sin demencia asociada, aunque se estima que la mayora de los pacientes
desarrollara demencia en etapas tardías de la enfermedad. Existe un deterioro de la función ejecutiva que incluye la planificación,
organización, flexibilidad, juicio, resolución de problemas con conservación de la percepción visual y habilidades espaciales. Existe
también una incapacidad para reconocer emociones, en particular las negativas como la ira, el miedo, tristeza. La orientación en
tiempo y lugar, el recuerdo libre demorado son alteraciones frecuentes de la DMF, sin embargo están presentes también durante las
primeras etapas de la enfermedad de Alzheimer, su distinción sigue siendo difícil mediante pruebas neuropsicológicas.

CASO CLINICO
Se trata de masculino de 73 años de edad el cual es originario de Mérida, actualmente vive solo, padece diabetes mellitus desde hace
15 años, hipertensión arterial, triglicéridos, colesterol y acido úrico elevados, al interrogatorio se desconoce la posología para sus
padecimientos, refiere que le indicaron glibenclamida, bezafibrato, enalapril, alopurinol y pravastatina, el familiar refiere que desde
hace mas de dos años presenta dificultad para recordar las cosas que está haciendo, se ha vuelto desconfiado, esconde las cosas para
evitar que se las roben, pobreza de pensamiento, pensamiento con bloqueo e ideas de daño y robo.

PREGUNTA
Debido a las múltiples entidades nosológicas que presenta el paciente cual es complicación neurológica más probable.

RESPUESTA
a.- Demencia vascular es la más probable.
b.- Demencia de Alzheimer es la más probable.
c.- La demencia por priones es la más probable.
c.- La demencia de Pick es la más probable.

CASO CLINICO
Masculino con neumonía por aspiración, con antecedente de pérdida progresiva de funciones mentales tipo demencia, previamente
presento cambio de conducta, pérdida de peso y cefalea continua de tres meses de duración. A la EF alerta, mutista, el examen
neurológico no revela alteraciones sensoriales, pares craneales sin afecciones sin embargo presenta mioclonos a la estimulación y
expontaneos incluyendo durante el sueño, los estudio de laboratorio no muestran datos de importancia, el liquido cefalorraquídeo
normal al igual que la TAC de cabeza.

PREGUNTA
Cuál es el agente causal más probable en este caso.

RESPUESTA
a.- Viral ADN.
b.- Viral ARN.
c.- Fungico.
d.- Prion.

CURSO ENARM CMN SIGLO XXI TEL: 36246001 Pharmed Solutions Institute PÁGINA 165
MANUAL DE TRABAJO DEL CURSO ENARM CMN SIGLO XXI
EPILEPSIA:
CIENCIAS BASICAS: Se define como la tendencia a tener convulsiones recurrentes. La epilepsia es una manifestación de la enfermedad
cerebral subyacente. Convulsiones simples o aquellos que ocurren durante la enfermedad aguda no deben ser clasificados como
epilepsia. Causas: Desconocido en dos tercios de los casos .En Reino Unido, las encuestas de la comunidad muestran: Enfermedad
cerebrovascular, 15 %; tumor cerebral, 6 %; relacionados con el alcohol, 6 %; post-traumático, 2 %; trastornos genéticos, 1 %. Otras
causas incluyen la esclerosis del hipocampo y malformaciones corticales y CLASIFICACION DE EPILEPSIA
vascular. En los trópicos, la neurocisticercosis es una causa común. SALUD EPILEPSIAS GENERALIZADAS Y SÍNDROMES:
PUBLICA: 50/100 000/año. 1 de cada 200 tienen epilepsia activa (en el Reino Idiopáticas de aparición relacionada con la edad
a)Epilepsia de ausencia infantil
Unido, 350.000) .Mayor incidencia en los países en desarrollo. En México las
b)Epilepsia mioclonica juvenil (JME)
crisis convulsivas de origen tardío son en su mayoría debidas a cisticercosis. c)Epilepsia con crisis tónico-clónica
CLASIFICACION: Básica en generalizada (50 %) y focal (50 %), subdividido en Sintomáticas y criptogénicas
categorías etiológicos: Idiopática (predisposición genética con el desarrollo a)Síndrome de West
normal, el examen y EEG). Sintomático (anomalía estructural). Criptogénicas b)Síndrome de Lennox- Gastaut
c)Epilepsias con ausencias mioclónicas
(anomalía estructural supone, pero no probado). Subdivisiones: 1. Parcial Sintomáticas
simple; alteración motora, sensitiva, autonómica o psíquica sin alteración del a)Encefalopatía mioclónica
estado de conciencia. 2. Parcial compleja; alteración del estado de EPILEPSIAS FOCALES Y SÍNDROMES
conciencia, puede comenzar con pródromos o con síntomas motores, Idiopática de aparición relacionada con la edad
a)Epilepsia infantil benigna con puntas centrotemporales
sensitivos, autonómicos o psíquicos, presenta automatismos y es seguida
b)Epilepsia de lectura
por un periodo de confusión (estado posictal). 3. Parcial secundariamente Sintomática:
generalizada: Inicia con alteraciones motoras, sensitivas, autonómicas o a)Epilepsia con convulsiones parciales simples, complejas o
psíquicas, seguidas de pérdida del estado de conciencia, con aumento de secundariamente generalizadas, que surgen de cualquier parte de
tono muscular, contracciones rítmicas (clónicas), al final paciente comatoso la corteza
b)Epilepsia parcial continua
y recuperación lenta. Puede haber incontinencia y/o mordida de lengua. 4. c)Síndrome caracterizado por la activación especifica
Generalizada de ausencia: Inicio súbito con periodo breve de desconexión Epilepsias
con el medio y recuperación rápida, tono muscular puede aumentar o EPILEPSIAS INDETERMINADAS (FOCAL O GENERALIZADA)
disminuir, se acompaña de automatismos o movimientos clónicos leves. 5. Epilepsias con puntas continuas y actividad de las ondas en el
sueño
Generalizada tónico-clónica: Perdida súbita del estado de alerta,
acompañada de contracciones tónico clónica generalizadas y hay estado posictal. STATUS EPILEPTICO: Presencia de una crisis
convulsiva continua o crisis repetitivas, discretas y con alteración del estado de conciencia en el periodo interictal, el límite de tiempo
es de 5 min. La causa más frecuente es abstinencia a fármacos anticonvulsivantes o falta de apego al tratamiento. PATOGENIA: Es
producida por el disparo sincrónico y sostenido de una población de neuronas cerebrales. Las estructuras más susceptibles al desarrollo
de crisis convulsivas recurrentes son la corteza motora y la formación del hipocampo y complejo amigdalino del sistema límbico.
DIAGNOSTICO: Clínico; Los ataques son estereotipados, eventos paroxísticos. El diagnóstico es clínico, testimonios son cruciales. Por lo
general, seguido de un período de somnolencia. Ver ' La pérdida de la conciencia ". Desencadenantes incluyen: alcohol, la fatiga,
privación del sueño, infecciones, hipoglucemia, el estrés, luces estroboscópicas (epilepsia fotosensible), lectura, el agua caliente (raro).
Ausencias infantiles; raras después de 10 años de edad. F> M. Breve pérdida de la conciencia varias veces al día, accionado por la
hiperventilación, remiten en la edad adulta. EEG característico; 3 Hz puntas y olas, sin fotosensibilidad. Epilepsia mioclónica juvenil
(JME): inicio antes de los 30 años, sacudidas mioclónicas por la mañana, ausencias típicas, convulsiones tónico-clónicas generalizadas.
EEG típico generalizado con puntas y ondas puede haber o no fotosensibilidad, remisión raro. Las convulsiones parciales: complejas
asociado con anormalidad estructural que subyace, por ejemplo, esclerosis del hipocampo, automatismos (relamerse los labios, la
masticación, la deglución, movimientos manuales estereotipados).
Déjà vu y jamais vu, auras olfativas (desagradable). Comportamiento TIPO DE EPILEPSIA 1RA. LINEA 2DA. LINEA
Generalizadas Valproato (200mg aumentar Levotiracetam o
inusual o emocionalidad. En aquellos pacientes con epilepsia c/2 sem. maximo 2.5 grs) o Topiramato
previamente diagnosticada y que presenta una crisis epiléptica, se Lamotrigina (25 mg/d, (25mg/d máximo
deberán revisar los niveles séricos del fármaco, obtener electrlitos, máximo 400mgs) 400mg/d)
BH, pruebas de función hepática y renal, así como panel toxicológico, Focal Carbamacepina (200mg/d) o Levotiracetam o
si todo sale normal, pensar en la modificación terapéutica, aumentar Lamotrigina o Topiramato
Valproato
dosis máxima, disminuir dosis de 1er fármaco e iniciar un segundo. Mioclonicas, Valproato o Levotiracetam
Pacientes que no tengan historia previa de epilepsia, solicitar BH, atónicas Lamotrigina (250mgs/d)
electrolitos, calcio, magnesio, glucosa, función hepática renal, EGO y Ausencia Etosuximida
panel de toxicología, si salen normal o negativo hacer RMN o Valproato o
electroencefalograma, para buscar causas focales de las crisis (tumor, EVC, neuroinfección, trauma, enfermedad degenerativa) y tratar
la causa específica así como considerar tratamiento anticonvulsivo. Si todo sale normal se puede pensar en crisis idiopáticas.
TRATAMIENTO: Recomendaciones generales; Evite las actividades peligrosas, por ejemplo, nadar solo, el montañismo. Tome duchas en
lugar de baños. Convulsiones simples: No hay un tratamiento a menos que exista un alto riesgo de recurrencia, por ejemplo, EEG
anormal como en JME o un MRI anormal. Si los factores precipitantes (por ejemplo, alcohol), la evitación pueden prevenir la
recurrencia. Después de un solo ataque no provocado, el riesgo de recurrencia es del 24%, sin causa y EEG normal y 65 % si se asocia
con una anormalidad neurológica + EEG anormal. Profilaxis: No hay indicación para iniciar el tratamiento en pacientes con lesiones en
la cabeza, craneotomía, tumores cerebrales, a menos que se presentan convulsiones. La resistencia al tratamiento farmacológico se
presenta hasta en 20% de los casos. En focos epileptógenos bien definidos se debe considerar el tratamiento quirúrgico con resección
del mismo. Estatus epiléptico: Minutos 0‐5 Documentación clínica. Diagnóstico correcto. Identificación del tipo de SE.
Breve anamnesis y exploración física. Medidas básicas: Mantener función cardiorrespiratoria. Monitorización de constantes vitales.
Vía IV permeable, realización de exámenes analíticos y administración de antiepilépticos. Si es posible, documentación EEG.
Minuto 6‐10 Según sospecha clínica (etilismo, déficit nutricional, hipoglicemia). Tiamina IM (100mg).
Glucosa IV (50 cc al 50% adulto, 2‐4 cc/kg al 25% niño). Finalización del SE clínico y eléctrico (tratamiento farmacológico).

CURSO ENARM CMN SIGLO XXI TEL: 36246001 Pharmed Solutions Institute PÁGINA 166
MANUAL DE TRABAJO DEL CURSO ENARM CMN SIGLO XXI
Diazepam IV (2mg/min hasta que cedan, máximo 0.3 mg/kg).Loracepam IV (0.1 mg/kg, ritmo 2mg/min hasta que cedan, máximo 10m.
Pueden constituir el único tratamiento si las crisis ceden y la causa del SE es corregida. Si no es así, asociar; fenitoína IV (15a20 mg/kg,
en suero salino, ritmo 50 mg/min en adultos y 1 mg/kg/min en niños). Minuto 30-40 Status resistente, ingreso en UCI, opciones:
Diazepam, en perfusión continúa (100mg en 500cc glucosado 5%, ritmo de 40 cc/hr). Fenobarbital IV (20 mg/kg, ritmo 50‐100 mg/min).
Fenitoína IV (hasta 30 mg/kg, dosis añadida de 510 mg/kg). Ácido Valproico 20mg/Kg. Minuto 60: SE refractario. Ingreso en UCI
(intubación orotraqueal, ventilación mecánica, acceso vascular central, monitorización EEG. Anestesia general, elegir una opción:
Midazolam IV/Propofol IV. Barbitúricos (pentobarbital, tiopental).Otros (clonazepam, clormetiazol, lidocaína, valproato sódico,
paraldehido, agentes inhalantes, bloqueantes neuromusculares). Minuto 0‐60/90: Diagnóstico y tratamiento etiológico.
Completar amnamnesis y exploración física detalladas. Exámenes complementarios (TAC, IRM, LCR), pero nunca posponer el tratamien
Decidir tratamiento etiológico o de factores precipitantes. Corrección y prevención de complicaciones sistémicas. Hipoglicemia,
alteraciones hidroelectrolíticas y del equilibrio ácidobase, hipotensión, hipoxia, edema cerebral, HIC, fiebre.
Prevenir la recidiva del SE Tratamiento farmacológico antiepiléptico de mantenimiento. COMPLICACIONES DEL STATUS EPILÉPTICO:
Neurológicas Encefalopatía hipóxica, Edema cerebral, Hipertensión endocraneal, Trombosis senos venosos, Vasculares
(infartos, hemorragias) Cardiovasculares y respiratorias Hipertensión / Hipotensión, arritmias cardiacas, parada cardiaca, insuficiencia
respiratoria / cardiaca, Edema pulmonar Endocrino‐metabólicas Acidosis metabólica / respiratoria, Hiperglicemia / Hipoglicemia,
Alteraciones de la función endocrina, Deshidratación, Hiponatremia, Insuficiencia renal aguda, Pancreatitis aguda, Falla hepática
Otras Coagulación intravascular diseminada.

CASO CLINICO
Hombre de 62 años, debuta con crisis de disnea, visión borrosa, caída al suelo, movimientos repetitivos de los brazos y estado
confusional de duración total de 10 minutos. Fue tratado con fenitoína y ácido valproico con lo que permaneció estable, presentando
sólo 3 años después nueva crisis con similares características. Comienza a notar cambios conductuales, desinterés, irritabilidad y
comportamiento obsesivo. Se hospitaliza por cuadro de compromiso de conciencia progresivo de varios días de evolución, agitación
psicomotora, desorientación, desviación de la mirada hacia arriba y derecha, temblor del mentón y parpadeo. Al ingreso se encontró
además distonías de las extremidades y mantención espontánea de posturas de apariencia catatónica, fenómenos que cedieron con el
uso de diazepam endovenoso. Durante esta hospitalización presentó varios otros episodios catatoniformes que duraban horas o días y
que en algunas oportunidades se asociaron a mioclonías faciales y de la mano derecha. El EEG se mantuvo persistentemente alterado
con actividad lenta generalizada. Se interpretó clínicamente como un cuadro epiléptico, con buena respuesta a los anticonvulsivantes.
El estudio imagenológico mostró una atrofia frontal bilateral. El estudio de laboratorio obtuvo una amonemia alta (44 µmol/L) que fue
interpretada como secundaria al uso de valproato, por lo que debió modificarse el tratamiento anticonvulsivante.

PREGUNTA
La asociación de demencia y epilepsia obliga a pensar en diagnósticos diferenciales, cual de los siguientes es más frecuente?

RESPUESTA
a.- Porfiria aguda intermitente.
b.- Neuroacantocitosis.
c.- Déficit de vitamina B12.
d.- Enfermedad de Lyme.

CASO CLINICO
Se trata de femenino de 84 años de edad la cual ingresa a consulta por presencia de dificultad para deglutir los alimentos, presentando
accesos de tos hemetizante durante las ingestas, y desde hace 48 hs., fiebre, dificultad respiratoria progresiva, somnolencia
permanente y expectoración muco purulenta fétida, se observa pérdida total de la independencia funcional, postrada en cama desde
hace aproximadamente 3 años, al cuidado permanente de 2 de sus hijas, portadora además de miocardiopatía dilatada, cumpliendo
tratamiento en la actualidad con Digoxina, Furosemida, Enalapril, AAS y vasoactivos cerebrales.

PREGUNTA
Considerando los antecedentes y al cuadro clínico, cual es el diagnostico mas probable?

RESPUESTA
a.- Demencia vascular.
b.- Demencia señil.
c.- Demencia alzheimer.
d.- Demencia de pick.

CASO CLINICO
Masculino de 69 años de edad con antecedentes de HTA de carácter leve -moderada desde hace 8 años aproximadamente además de
Hipercolesterolemia desde hace 10 años según refiere, para lo cual ingiere como medicación, Enalapril 10 mg e hidroclorotiazida 12,5
mg una vez por día, complementando dicho tratamiento con dieta hiposódica e hipocolesterolemica. Manifiesta que no cumple
estrictamente con la dieta descripta pero que siempre ingirió la medicación antihipertensiva. Consulta el día de hoy por cefalea a
predominio frontal, la cual se manifiesta desde hace varios días acompañado de sensación de inestabilidad que no sabe referir
exactamente pero si manifiesta sentirse muy extraño, situación que no es habitual en el pero, que lo inquieta y preocupa por lo cual
decidió concurrir a la consulta medica, por propios medios. Manifiesta que últimamente no controló su T A como se le había indicado y
que tampoco respetó la dieta habitual. Como antecedente importante refiere hipercolesterolemia leve. Además expresa que siempre

CURSO ENARM CMN SIGLO XXI TEL: 36246001 Pharmed Solutions Institute PÁGINA 167
MANUAL DE TRABAJO DEL CURSO ENARM CMN SIGLO XXI
llevó una vida activa, que solía concurrir desde el pueblo hasta su domicilio rural en bicicleta hasta hace unos 10 años
aproximadamente.

PREGUNTA
Considerando los antecedentes y al cuadro clínico, cual es el diagnostico mas probable?

RESPUESTA
a.- Demencia vascular.
b.- Demencia señil.
c.- Demencia alzheimer.
d.- Demencia de pick.

CASO CLINICO
Paciente de 73 años de edad que presenta temblor de reposo de predominio en miembro superior derecho, rigidez, bradicinesia y
alteración de reflejos posturales. En la última visita, la familia que lo acompaña relata que a lo largo de los últimos 6-8 meses, lo vienen
notando triste, poco comunicativo, "perezoso" y con escasa ilusión a la hora de iniciar actividades nuevas. Han notado que pasa horas
ensimismado, sin llegar a concluir la tarea que estuviera realizando. Se muestra incapaz de programar las actividades que ha de realizar
al día siguiente o las realiza a destiempo, de forma desorganizada. Parece mostrarse poco reactivo ante los problemas, disgustos o
alegrías cotidianas, como si nada le importara. Se muestra olvidadizo, tiende a usar dietarios y notas cada vez con mayor asiduidad, sin
que esto haya impedido el haber olvidado citas importantes o tareas inexcusables en su negocio.

PREGUNTA
Considerando los antecedentes y al cuadro clínico, cual es el diagnostico mas probable?

RESPUESTA
a.- Demencia por parkinson.
b.- Demencia por prionopatia.
c.- Demencia alzheimer.
d.- Demencia de frontotemporal.

CASO CLINICO
Mujer de 56 años, con 12 años de escolaridad, diestra, dueña de casa, separada, con una convivencia estable. Inicia un delirio
paranoide pero asociado a una gran pasividad (era necesario escogerle la ropa, abandonó las labores domésticas). Se presento
descontrol urinario ocasional. En el examen mostraba una enferma vigil, prescindente, sin contacto visual, que no expresaba afectos y
colaboraba poco al examen (lo que impidió evaluar el campo visual y la sensibilidad). Existía gran hipokinesia pero sin rigidez ni
movimientos anormales; la metría era normal. Existía enganche digital pero no prehensión forzada ni succión. En el Minimental test de
Folstein (MM) obtuvo sólo 10/30 puntos, fracasando en Orientación, Atención y cálculo, Lectoescritura y Dibujo, con respeto relativo de
la memoria. Al preguntarle la fecha dijo "a ver... a ver... a ver"; lo mismo respondió al pedirle restas. Fue capaz de leer la orden escrita,
pero no la cumplió; en vez de una frase escribió su nombre, y el pedirle copiar el dibujo recibió el lápiz pero no rindió, limitándose a leer
una y otra vez fragmentos del test. En la prueba de Matrices Progresivas Coloreadas (MPC) repetía las instrucciones pero no rendía y
musitaba en forma ininteligible; obtuvo 0/12 puntos. En Aprendizaje verbal (AV) decía "ya..." y se quedaba mirando al frente, con rostro
inexpresivo, sin repetir las palabras propuestas. En Memoria semántica (MS) evocó 2 nombres de animales en 60 segundos.

PREGUNTA
Considerando los antecedentes y al cuadro clínico, cual es el diagnostico mas probable?

RESPUESTA
a.- Demencia por parkinson.
b.- Demencia por prionopatia.
c.- Demencia alzheimer.
d.- Demencia de frontotemporal.

CASO CLINICO
Femenino de 47 años acude por fallas de la memoria, errores en la denominación y en actividades domésticas. Aparecieron cambios de
conducta, se puso:"confianzuda", caprichosa, golosa, floja. Se negaba a colaborar en actividades domésticas, e insistía en forma
majadera en diversos temas. Meses después fue evaluada con mayor detalle. En su casa era independiente para vestirse, alimentarse y
asearse, pero a veces se orinaba. No era capaz de retener informaciones nuevas, a veces no reconocía a sus familiares, y mostraba una
proximidad excesiva frente a los extraños. Tenía pérdida de libido y se mostraba inquieta e irritable. Se mostraba ágil y el examen
neurológico básico era normal, no existían reflejos primitivos. Su actitud era inadecuada por animo subido, proximidad excesiva y gran
tendencia al payaseo. Por ejemplo, en una oportunidad interrumpió el examen diciendo "¡apúrate abuelito por favor! porque tengo
que ir a hacerle la comida a mi marido". No mostraba ninguna preocupación por sus rendimientos y con frecuencia no colaboraba; al
pedirle interpretar proverbios sólo dijo "¡no sé!". Al denominar cometía errores caprichosos, pero que sugerían una afasia amnésica.
Por ejemplo, ante el dibujo de un paraguas dijo riendo "¡es un sostén!" y otra vez "¡es una bicicleta!", pero ante el dibujo de un sofá
dijo "es para sentarse". No existían defectos fonológicos ni morfosintácticos. Al pedirle interpretar el proverbio "a quien madruga Dios
le ayuda" dijo "¿a mi marido? No, mi marido tiene negocio y yo trabajo con él, yo de cajera y él de vendedor...". Al dictarle "bajó a la
tierra" lo repitió bien pero escribió -con buena letra- "monono nonino ninino". No existía apraxia constructiva (aunque a veces el dibujo

CURSO ENARM CMN SIGLO XXI TEL: 36246001 Pharmed Solutions Institute PÁGINA 168
MANUAL DE TRABAJO DEL CURSO ENARM CMN SIGLO XXI
era muy descuidado), oral ni ideo-motora; según su familia se vestía bien, pero seleccionando mal su vestimenta. Fracasó en cálculo
oral y escrito; existía una agnosia digital visual y desorientación derecha-izquierda -ni siquiera fue capaz de decir cual era su mano
derecha.

PREGUNTA
Considerando los antecedentes y al cuadro clínico, cual es el diagnostico mas probable?

RESPUESTA
a.- Demencia por parkinson.
b.- Demencia por prionopatia.
c.- Demencia alzheimer.
d.- Demencia de frontotemporal.

CEFALEAS Y DOLOR CRANEO-FACIAL


SALUD PUBLICA: Mas común prevalencia en las mujeres, 18%, hombres, 6%. Edad de inicio promedio 19 años. 46 % tienen
antecedentes familiares. El riesgo de que un niño desarrolle migraña 70% si ambos padres están afectados, el 45%, cuando uno de los
padres afectados. Una condición hereditaria dominante rara, migraña hemipléjica familiar, debido a una mutación en el cromosoma 19
que codifica para una subunidad del canal de calcio dependiente de voltaje.
La arteriopatía cerebral autosómica dominante con infartos subcorticales y leucoencefalopatía (CADASIL) puede presentarse con
migraña hemipléjica y progreso a una encefalopatía isquémica. PATOGENIA: La migraña es un trastorno neurovascular en un individuo
genéticamente predispúesto. Predisposición es una inestabilidad dentro de la red trigéminovascular originada en el tronco cerebral, en
particular, el mesencéfalo dorsal y dorsolateral puente de Varolio. Proyecciones difusas del locus ceruleus y de la corteza cerebral
resultan en deterioro del flujo sanguíneo cortical cerebral que causa la propagación de la depresión asociada con auras migrañosas.
DIAGNOSTICO: Clínico; La migraña es una cefalea episódica generalmente asociada con náuseas (± vómitos) y fotofobia. Puede estar
precedida por síntomas neurológicos focales. El aura no necesariamente puede ser seguida por dolor de cabeza (anteriormente
conocido como equivalentes de migraña). 30 % puede tener otros dolores de cabeza que coexisten, por ejemplo tensión y dolores de
cabeza por uso excesivo analgésicos. Características de dolor de cabeza: Unilateral en 2/3 de los pacientes y bilateral en 1/3. Dolor que
se siente detrás o a lo largo del ángulo interno del ojo o regiones frontotemporal. Irradia a espalda, occipucio o el cuello. Sitio de dolor
de cabeza puede ser tanto ipsilateral o contralateral a la perturbación neurológica focal. Algunos pacientes pueden quejarse de dolor
del miembro ipsilateral del lado del dolor de cabeza. Carácter del dolor de cabeza es aburrido en el inicio y más tarde palpitante (que
aumenta con cada pulsación). Otros pacientes sólo pueden describir un dolor de cabeza constante, o incluso un ligero dolor de cabeza
embotada. Empeora con el movimiento. Características Aura: Auras visuales incluyen: alucinaciones visuales, escotomas, y espectros de
fortificación (zig-zag líneas se asemejan a una muralla cuando se ve desde arriba). Por lo general, blanco y el brillo se mueven a través
del campo visual, dejando un área de discapacidad escotomas centellantes. Otros fenómenos visuales incluyen destellos de luz
(fotopsia). Nota: la epilepsia del lóbulo occipital provoca alucinaciones que son circulares o de formas geométricas y colores. Auras
sensoriales son, por ejemplo parestesias generalmente positiva en lugar de adormecimiento, que se reparten en minutos u horas (5 %).
Otras auras: hemiparesia (de minutos u horas), disfasia, alucinaciones olfativas y gustativas, y la distorsión de las partes del cuerpo,
tales como inflamación de la lengua. Disparadores de migraña: El estrés y la relajación después de estrés. Falta o el exceso de sueño
acostumbrados. Trauma (especialmente en niños).Estimulación sensorial: reflejos, luces parpadeantes, los olores. Los hábitos
alimentarios y la alimentación: falta de una comida (hipoglucemia). Los alimentos como el vino tinto, queso, chocolate. Aditivos
alimentarios: el glutamato monosódico. Ejercicio. El exceso de calor y la deshidratación. Drogas: vasodilatadores como la nitroglicerina.
Variantes de migraña: Migraña vertebrobasilar; síntomas del tronco cerebral: diplopía, vértigo, incoordinación, ataxia y disartria se
producen en posteriores ataques de migraña. También puede ser desmayos o pérdida de consciencia debido a la participación de la
formación reticular del cerebro medio. En casos severos un estado de estupor o coma puede durar una semana (estupor migraña). La
mayoría de los casos están asociados con otros síntomas vertebrobasilar. Migraña oftalmopléjica: Paresia extra-ocular el tercer nervio
es el más afectado. Paresia puede durar días o semanas. Excluir una lesión compresiva tales como aneurisma de la arteria comunicante
posterior. Migraña retiniana: Resultado de la constricción de las arteriolas retinianas, daña la visión en un ojo y se asocia con dolor de
cabeza por detrás del mismo ojo. Vértigo benigno recurrente. Migraña que tiene anormalidades del sistema vestibular. Ataques de
vértigo acompañadas por tinnitus, sordera y cefalea que puede responder con terapia antimigrañosa. Criterios abreviados de Sociedad
Internacional de Cefaleas (IHS) para la migraña: Migraña sin aura;
a) Dolor de cabeza una duración de 4 horas hasta 3 días. b) Náuseas/vómitos y/o sensibilidad a la luz y el ruido. c) Dos de los siguientes:
dolor unilateral dolor moderado o grave intensidad, agravación por la actividad física sencilla, dolor palpitante. Migraña con aura; Al
menos 3 de los siguientes a) tronco cerebral focal reversible o disfunción cortical. c) aura se desarrolla durante > 4 minutos o 2 auras
sucesivas. d) cada aura < 60 min. e) cefalea < 60 min siguiente aura. Los criterios sugeridos para la migraña crónica o transformada:
diario o casi a diario (> 15 días / mes) dolor de cabeza > 1 mes. Promedio de duración de la cefalea > 4 horas / día (sin tratar). Al menos
uno de las siguientes: antecedentes de migraña de IHS. Historia de aumento de frecuencia de cefalea con la disminución de la gravedad
de características migrañosas durante al menos 3 meses. Ataques superpuestas de dolor de cabeza que cumpla con todos los criterios
de la IHS, excepto duración. MIGRAÑA EN EL EMBARAZO: Consejos para no toma de drogas. Para evitar las náuseas y los vómitos
relacionados con el embarazo como resultado hipoglucemia y deshidratación: comer pequeños bocados carbohidratos frecuentes,
ingesta adecuada de líquidos. Descanso adecuado. Minimizar la exposición al fármaco, especialmente durante el primer trimestre. Si es
posible, deje los medicamentos profilácticos. Tratamiento agudo: paracetamol, en todos los trimestres y durante la lactancia, aspirina:
probablemente seguro pero la precaución a corto plazo debido a ↑ riesgo de hemorragia postparto, hemorragia neonatal, y cierre
prematuro del ductus arterioso. Evitar durante la lactancia de riesgo del síndrome de Reye y el sangrado en la mortalidad infantil.
triptanos, no se recomienda,preparaciones de ergotamina contraindicado en el embarazo y la lactancia. MIGRAÑA MENSTRUAL: Gatillo
hormonal es la exposición a los altos niveles de estrógenos seguido por una caída en los niveles. La liberación de prostaglandinas
uterinas que ocurren alrededor de la menstruación un mecanismo adicional. 60 % de las mujeres reportan un aumento en la frecuencia

CURSO ENARM CMN SIGLO XXI TEL: 36246001 Pharmed Solutions Institute PÁGINA 169
MANUAL DE TRABAJO DEL CURSO ENARM CMN SIGLO XXI
de la migraña en torno a la menstruación. 14 % tiene exclusivamente migraña relacionada con la menstruación. Ácido mefenámico 500
mg 3-4 veces al día o naproxeno 500 mg dos veces 1-2 días antes de dolor de cabeza y de la duración del período (de -2 a 3 días de la
menstruación). Ergotamina, 1 mg (o 1/2 supositorios) durante el periodo vulnerable. Naratriptán, 1 mg bd o frovatriptán 2,5 mg bd
durante 3-5 días. TRATAMIENTO: Analgesia simple con antieméticos: si las náuseas y los vómitos no son un síntoma importante como
la motilidad gástrica se ve afectada durante un ataque de migraña. La aspirina, 900-1200 mg (disuelto) + metoclopramida, 10 mg o
domperidona, 10 - 20mg. Los medicamentos alternativos incluyen paracetamol, 1000 mg y los AINE por ejemplo, ibuprofeno,
naproxeno, diclofenaco. Triptanos: Todas las drogas de esta clase (sumatriptán 50-100mgs, zolmitriptán 2.5mgs, naratriptán 2.5mg,
rizatriptán 5-10mgs) tienen una alta eficacia con un máximo de 70 % que tiene una respuesta dentro de 2 horas y el 40 % es libre de
dolor a las 2 horas. Zolmitriptan y rizatriptán disponibles como obleas no tienen una acción más rápida. Sumatriptan disponible como
aerosol nasal e inyección. Las drogas funcionan mejor cuando se toma temprano, pero no durante el pródromo o de la fase de aura.
Recurrencia de la cefalea a las 12-24 horas se produce en el 30%. Consejo habitual es tomar una dosis adicional de los triptanos, quizás
combinarlo con un AINE. Si no hay respuesta intente otra triptanos. El uso excesivo puede dar lugar a dolores de cabeza de rebote de
10 %. Contraindicaciones: enfermedad arterial coronaria, enfermedad cerebrovascular, hipertensión no controlada, enfermedad
vascular periférica, insuficiencia hepática significativa, y el embarazo. Efectos secundarios: molestias en el pecho o sensación de
pesadez, la mandíbula, el hombro y rigidez del cuello, parestesia, fatiga y mareos. Interacciones con otros medicamentos: evitar IMAO.
Propranolol ↑ concentraciones séricas de rizatriptán. Por lo tanto, utilizar dosis de 5 mg. El síndrome de la serotonina posible cuando
se utiliza con los ISRS. Preparaciones de ergotamina: Aun así un papel para la consideración de tartrato de ergotamina en, aquellos
pacientes que no toleran a los agonistas 5 - HT. Solo o en combinación con cafeína 1-2 mg puede administrarse por vía oral en el inicio o
puede ser utilizado en aquellos pacientes que tienen síntomas premonitorios tales como ansiedad, bostezo, o la fatiga. También puede
ser administrado por inhalador o supositorio. La sobredosificación causa náuseas, dolor de cabeza de rebote, y la vasoconstricción
periférica. La dosis máxima recomendada por semana es de 10 mg. Dihidroergotamina (DHE) usaron por vía intravenosa en pacientes
con migraña intratable en dosis de 0,3 a 1,0 mg cada 8 horas hasta una dosis total de 10mg. PREVENCION: La profilaxis de la migraña;
Dolor de cabeza diario útil para controlar la frecuencia y patrones, por ejemplo, relación de los periodos, los fines de semana, el uso
excesivo de analgésicos y de triptanos. La profilaxis ineficaz si el uso excesivo de medicamentos. Evite los desencadenantes dietéticos
sólo en el 10 %. Considere la profilaxis si ≥2 ataques al mes o un ataque prolongado que afecta el estilo de vida. Medicamentos
profilácticos: betabloqueantes (propanolol, timolol, nadolol, atenolol). Los 5-HT2 antagonistas; pizotifen, ciproheptadina, metisergida,
amitriptilina especialmente si la migraña asociada con cefalea tensional. Valproato sódico. Topiramato. Gabapentina.

CASO CLINICO
Paciente 27 Años, femenino, refiere haber comenzado con cefaleas hace 12 hs, de carácter pulsátil, unilateral, que no cede con AINES,
refiere nauseas que atribuye a los medicamentos, esta cefalea es distinta de otros episodios sufridos previamente donde el dolor era
como una presión en zonas parietales y nuca.

PREGUNTA
Cual es el diagnostico mas adecuado en este paciente?

RESPUESTA
a.- Migraña.
b.- Cefalea tipo-tensión.
c.- Cefalea en racimos (cluster).
d.- Cefalae autonómica trigeminal.

CASO CLINICO
Paciente masculino, 62 años, antecedentes de HTA, tabaquismo, EPOC, que es traído por la familia quienes refieren que el paciente
comenzó hace días a consumir AINES por cefalea de reciente comienzo, posteriormente presentó conductas poco habituales y desde
ayer se encuentra desorientado.

PREGUNTA
Considerando los antecedentes y cuadro clínico, cual de los siguientes son factores es el menos críticos para el pronostico?

RESPUESTA
a.- Cefalea subaguda y/o cefalea progresiva que empeora en el tiempo (meses).
b.- Cefalea Nueva o distinta de la Habitual.
c.- Cualquier cefalea de maxima severidad en su inicio.
d.- Nueva cefalea en mayores de 40 años.

CASO CLINICO
Paciente femenino, 42 años que refiere cefalea súbita, intensa, explosiva, “siente que le estalla la cabeza”, refiere haber tenido hace
unos días atrás un episodio similar aunque de menor intensidad, que la obligo a guardar cama por unas horas.

PREGUNTA
Cual es el claso diagnostico mas probable?

RESPUESTA
a.- A trauma de cráneo y/o cuello.
b.- A desórdenes vasculares craneales o cervicales.

CURSO ENARM CMN SIGLO XXI TEL: 36246001 Pharmed Solutions Institute PÁGINA 170
MANUAL DE TRABAJO DEL CURSO ENARM CMN SIGLO XXI
c.- A desorden intracraneal no vascular.
d.- A sustancias o a su supresión.

CASO CLINICO
Paciente de 43 años que acude a urgencias de atención primaria por cefalea. En el último año refiere cefalea basal diaria (Escala Visual
del dolor (EVA) 5-6/10) de característica tensional, sin náuseas, fotofobia ni fonofobia, y aunque no limita por completo su actividad ya
que se ha acostumbrado a ella, tiene episodios de reagudización más intensos, que la obliga a dejar sus labores, como en esta ocasión.
Últimamente estos episodios son más frecuentes, y los relaciona con problemas de salud en un hijo. Refiere además que toma
analgésicos prácticamente a diario y por rachas combinando paracetamol, ibuprofeno y lorazepam, desde hace casi año y medio.
Además está en tratamiento desde hace 2 años con escitalopram.

PREGUNTA
Cual es el diagnostico mas probable?

RESPUESTA
a.- A trauma de cráneo y/o cuello.
b.- A desórdenes vasculares craneales o cervicales.
c.- A desorden intracraneal no vascular.
d.- A sustancias o a su supresión.

CURSO ENARM CMN SIGLO XXI TEL: 36246001 Pharmed Solutions Institute PÁGINA 171
MANUAL DE TRABAJO DEL CURSO ENARM CMN SIGLO XXI
TRASTORNOS DEL MOVIMIENTO Y ATAXIAS:
CIENCIAS BASICAS: Los trastornos del movimiento constituyen una patología relativamente frecuente en la práctica médica diaria,
tanto neurológica como en Atención Primaria, y se pueden presentar como estados hipercinéticos o hipocinéticos. El prototipo de
trastorno hipocinético es la enfermedad de Parkinson y también la representa la patología más frecuente dentro de los trastornos del
movimiento. En general, los trastornos del movimiento tienen como sustrato anatómico común los núcleos de los ganglios basales.
Además, entre la lista de efectos secundarios relacionados con una gran variedad de
fármacos de uso común entre la población, está el desarrollo de trastornos del CLASIFICACIÓN ETIOLÓGICA DE SÍNDROMES COREICOS
1. COREAS HEREDITARIAS
movimiento. SALUD PUBLICA: La enfermedad de Parkinson se presenta con la misma Enfermedad de Huntington
incidencia en todo el mundo y afecta por igual a ambos sexos. En el 5 al 10% de las Coreoacantocitosis
personas que sufren dicha patología, ésta aparece antes de los 40 años de edad, en su Corea hereditaria benigna
forma precoz. La edad media de inicio es aproximadamente de 65 años. Síndrome de Corea paroxística familiar
Enfermedades metabólicas hereditarias (cistinuria,
piernas inquietas afecta al 5-10% de las personas entre 18 y 65 años, y aumenta hasta homocistinuria, fenilcetonuria, galactosemia)
el 15-20% en los mayores de esa edad. Enfermedad de Huntington suele iniciarse de 2. COREAS SECUNDARIAS
forma insidiosa hacia la cuarta década de la vida, tiene una prevalencia de 7-10 1. Fármacos: Neurolepticos, levodopa, agonistas
casos/100.000 habitantes y afecta por igual a ambos sexos ENFERMEDAD DE dopaminergicos, anticolinérgicos, amanrtadina,
antiepilépticos, estimulantes noradrenergicos,
PARKINSON: Idiopática, afección degenerativa que combina una bradicinesia corticoides, opiáceos, metoclopramida,
asimétrica, típica facies inexpresiva, hipocinesia y rigidez (rueda dentada), algunas antidepresivos, reserpina, estrógenos. 2. Tóxicos:
veces acompañadas de temblor en reposo y alteraciones posturales. El temblor, alcohol, monóxido de carbono, manganeso,
mercurio. 3. Alteraciones metabólicas,
inicialmente intermitente, suele comenzar en una mano para extenderse luego a las
hipopglucemia, hiperglucemia. 4. Embarazo. 5.
extremidades restantes y, a veces, a los músculos de la cara y del cuello. El temblor Infecciones y parasitosis de SNC: sarampión,
clásico de la enfermedad de Parkinson es regular y rítmico. Está presente en reposo y varicela, herpes, toxoplasmosis, VIH, sífilis,
disminuye o desaparece con el movimiento de la zona afectada. El temblor aumenta tuberculosis. 6.Neoplasias
3. COREAS DEL DESARROLLO
con la ansiedad y cesa durante el sueño. Con frecuencia, el cuadro se acompaña de
Corea fisiológica de la infancia
alteraciones en el intelecto, aunque de forma leve, lo que incluye afectaciones Discinesia bucolinguofacial del anciano
cognitivas, de percepción, de memoria y de expresión. Un 15% puede verse afectado de
demencia, sobre todo en los casos en los que la enfermedad se ha iniciado tardíamente. La afección del estado emocional, con
tendencia a la depresión, afecta hasta un 40% de los casos. Pueden presentar, en el 40% de los casos, síntomas de disfunción
vegetativa, que incluyen estreñimiento, a veces grave, hiperhidrosis, sofocaciones y alteraciones en la micción (incontinencia,
polaquiuria y enuresis nocturna). La sialorrea es frecuente y se debe al defecto de la deglución. Los pacientes pueden afectarse de
forma diferente debido a una combinación de factores genéticos y ambientales, entre los que podemos citar: virus, toxinas, el consumo
de aguas no potables, la vitamina E y el hábito tabáquico (el tabaquismo no influye de forma negativa al igual que el consumo de
vitamina E), una vez más, presente, como responsable de la puesta en marcha de un pro- ceso patológico. Los familiares en primer
grado de consanguinidad, duplican el riesgo de desarrollar la enfermedad (un 17% de probabilidad, a lo largo de su vida),
comparándolos con la población general. Los criterios de diagnóstico clínico tienen una sensibilidad de 80% y una especificidad del 30%,
comparado con el patrón de oro del diagnóstico de la autopsia. La patología primaria consiste en la pérdida progresiva de células de la
sustancia negra del tallo cerebral donde se produce el neurotransmisor dopamina. El tratamiento está obligado a reemplazar la pérdida
de dopamina. También se ven afectados varios sistemas de neurotransmisores catecolaminérgicos. Las pruebas, como la RMC, suelen
ser normales. Por el contrario, en las pruebas de neuroimagen funcional, como el PET y el SPECT cerebral, se puede observar una
reducción asimétrica de la captación estriatal de 18- fluorodopa o de DAT (transportador de dopamina), respectivamente, alteraciones
que no se observan en pacientes con temblor esencial o con parkinsonismo inducido por fármacos. El tratamiento médico suele ser
eficaz y debe de continuarse durante el resto de la vida del ciudadano. Se fundamenta en las siguientes medidas: Anticolinérgicos (con
la intención de corregir la hiperfunción de las células colinérgicas del núcleo estriado y actúan bloqueando los receptores muscarínicos
centrales)-Amantadina (posee propiedades anticolinérgicas y dopaminérgicas, ya que estimula la liberación de dopamina en las
terminaciones nerviosas) -L-DOPA (fármaco más eficaz, traspasa con facilidad barrear hematoencefálica)-Otros fármacos
dopaminérgicos-Medidas generales-Tratamiento quirúrgico PARKINSONISMOS: producidos por factores etiológicos conocidos, por
ejemplo medicamentosos, reciben el nombre de secundarios o sintomáticos. Los secundarios incluyen aquellos producidos por
infecciones (encefalitis letárgica), intoxicaciones (manganeso, monóxido de carbono, MPTP), administración de fármacos
(neurolépticos, cinarizina), tumores, traumatismos, infartos cerebrales profundos y calcificaciones de los ganglios basales. Un tercer
grupo en el que los síntomas del parkinsonismo aparecen en el contexto de otra enfermedad neurológica y se asocian a otros síntomas
de disfunción neurológica se denominan parkinsonismos plus, con síntomas extrapiramidales, entre los que se encuentra la parálisis
supranuclear progresiva, que se caracteriza por una oftalmoplejia supranuclear progresiva, asociada a rigidez distónica de los músculos
del cuello y del tronco superior, con alteración de equilibrio y una demencia discreta de tipo frontal. Estos síntomas se inician en la
quinta o sexta décadas de la vida y son de evolución progresiva. La falta de equilibrio y las molestias visuales constituyen los síntomas
iniciales más frecuentes. SÍNDROME DE PIERNAS INQUIETAS: Trastorno neurológico del movimiento, caracterizado por la aparición de
molestias inespecíficas en la parte inferior de las piernas durante el reposo, cuando el paciente está sentado o tumbado, es
especialmente frecuente durante el inicio del reposo nocturno, cuando el paciente se acuesta. Consiste en una sensación desagradable
y difícil de calificar, en forma de parestesias, disestesias, hormigueos, pinchazos, desasosiego o dolor que lleva al paciente a la
necesidad irresistible de mover las piernas («inquietud motriz»), levantarse, pasear, darse masajes e incluso baños con agua fría, en un
intento de alivio pasajero. Hay movimientos periódicos en las extremidades, en el que el paciente realiza movimientos bruscos de las
piernas, repetitivos y estereotipados, cada 10-60 seg., a veces durante toda la noche, involuntariamente. Al impedir un descanso
nocturno adecuado puede manifestarse con cansancio y somnolencia diurna e insomnio. Los fármacos con actividad dopaminérgica y
en concreto los agonistas dopaminérgicos (pergolida, pramipexol y ropinirol), se consideran el tratamiento de elección. Ropinirol ha
demostrado en varios estudios que reduce los movimientos periódicos de las piernas, y mejora la eficiencia del sueño en relación con el
período basal. COREAS: Significa «baile» o «danza». Se utiliza para designar movimientos involuntarios, irregulares, de duración breve y
de baja amplitud, que no son predecibles y no tienen una finalidad aparente; fluyen de una parte del cuerpo a otra sin una secuencia
definida. Suelen localizarse en las partes distales de los miembros, generalmente en las manos o en la cara, y suelen asociarse a

CURSO ENARM CMN SIGLO XXI TEL: 36246001 Pharmed Solutions Institute PÁGINA 172
MANUAL DE TRABAJO DEL CURSO ENARM CMN SIGLO XXI
hipotonía o a impersistencia motora: dificultad para mantener una contracción muscular sostenida. En las primeras fases, los
movimientos coreicos pueden aparecer al final de la ejecución de movimientos normales, pero en fases más avanzadas se hacen muy
frecuentes, dispersas y amplias, por lo que terminan siendo muy incapacitantes. Los síndromes coreicos suelen relacionarse con
lesiones o alteraciones funcionales del neoestriado (caudado y putamen). Su fisiopatología, desde el punto de vista neuroquímico,
responde a una situación de hiperactividad dopaminérgica. Esto viene apoyado por la eficacia terapéutica de los fármacos
antidopaminérgicos y por la capacidad de agravar o incluso inducir estos trastornos, que poseen los agonistas de la dopamina.
ENFERMEDAD DE HUNTINGTON: Es la forma de corea hereditaria más frecuente. Enfermedad neurodegenerativa que se transmite de
forma autosómica dominante con penetrancia completa, precisando un solo gen defectuoso en uno de los progenitores. Los hijos de
una persona afectada, tienen el 50% de probabilidad de padecer la enfermedad. Hay atrofia cortical relacionada con el grado de
evolución de la enfermedad, y también atrofia estriatal, fundamentalmente a nivel del núcleo caudado. Las neuronas afectadas por el
proceso degenerativo son las de mediano tamaño. La disfunción estriatal, contraria a la que caracteriza a la enfermedad de Parkinson,
es la responsable de que aparezcan las hipercinesias; y la degeneración neocortical y del núcleo caudado es la responsable de que
aparezca demencia. El cuadro clínico incluye síntomas motores, trastornos psiquiátricos y alteraciones cognitivas: Síntomas motores:
movimientos coreicos, que afectan inicialmente a la parte distal de las extremidades y posteriormente se van extendiendo a la
musculatura craneal, faríngea y laríngea. Las anormalidades en la motilidad ocular, con dificultad para seguir con la mirada un objeto
móvil, posturas distónicas axiales y de las extremidades, así como disfagia y disartria coreica por movimientos coreoatetósicos de los
labios, la lengua, el paladar y el diafragma. Trastornos psiquiátricos: en ocasiones son la primera manifestación de la enfermedad,
cambios en la personalidad, depresión, apatía, agitación, manía, alucinaciones, insomnio. En casos graves es frecuente el suicidio; más
del 30% lo intentan durante el desarrollo de la enfermedad. Déficit de la memoria, la atención, la concentración y el aprendizaje;
pueden llegar a una demencia con afectación de las funciones frontales: trastorno de la atención y de las funciones ejecutivas, sin
afasia, apraxia o agnosia. El diagnóstico de sospecha se realiza mediante la clínica, la historia familiar positiva y la neuroimagen. La
confirmación por genética molecular, que tiene una sensibilidad diagnóstica del 98% y puede utilizarse incluso en fase presintomática.
No existe un tratamiento específico. Se están realizando ensayos con GABA, baclofen, ACTH y antagonistas selectivos de los receptores
D2. COREA DE SYDENHAM: Es un cuadro de corea aguda, que aparece generalmente en la infancia en relación con una infección por el
estreptococo betahemolítico. Está considerada como uno de los criterios mayores de la fiebre reumática. Su fisiopatología parece estar
relacionada con un proceso autoinmune, en el que los anticuerpos antibacterianos provocan reacciones cruzadas con los antígenos
tisulares en el cuerpo estriado. Los síntomas pueden aparecer hasta seis meses después de una infección por el estreptococo, que a
veces no se consigue documentar y que hace el diagnóstico más difícil.

CASO CLINICO
Mujer de 58 años que presenta un cuadro de torpeza generalizada, lentitud y temblor postural en miembros superiores de dos años de
evolución. Dos años presenta de forma fluctuante: conducta impulsiva, cambios de carácter con violencia y agresividad, ataques de
llanto repentino y comportamiento infantil. Presenta asimismo dificultades en concentración, poca capacidad de comprensión y
razonamiento y dificultad para hablar y expresarse. En ocasiones confunde ideas simples, también nombres de objetos y personas.
Comienza a perder las rutinas de su vida diaria, mantiene poca relación con el entorno y muestra falta de cuidado personal. En la
exploración neurológica se objetiva: clara alteración en prueba de secuencias alternantes y en patrones motores alternantes
unilaterales (triple maniobra de Luria), lenguaje muy pobre con carencia de sustantivos y confusiones frecuentes, desorientación parcial
en tiempo y espacio, movimientos extraoculares pobres (aunque sin limitación) junto rigidez y bradicinesia generalizadas.

PREGUNTA
Cual es el diagnostico mas probable?

RESPUESTA
a.- Enfermedad de Huntington
b.- Enfermedad de Sydenham.
c.- Enfermedad de Parkinson.
d.- Enfermedad de Piernas inquietas.

CASO CLINICO
Paciente de 76 años que acude a la consulta diciendo que sufre de temblor en las manos, sobre todo la derecha, desde hace meses. Lo
presenta de forma constante, No refiere otra sintomatología. El paciente presenta temblor en ambas manos, sobre todo la derecha, de
reposo, que mejora con la realización de movimientos voluntarios, inexpresividad facial, dificultad para levantarse del asiento y lentitud
al caminar. La movilización de las extremidades muestra aumento del tono en las piernas y rigidez en rueda dentada en los brazos. Se
trata, por tanto, de un paciente que consulta por temblor en las manos.

PREGUNTA
Considerando el diagnostico cual es el tratamiento mas apropiado?

RESPUESTA
a.- Pergolida.
b.- Penicilina.
c.- Baclofen.
d.- L-dopa.

CURSO ENARM CMN SIGLO XXI TEL: 36246001 Pharmed Solutions Institute PÁGINA 173
MANUAL DE TRABAJO DEL CURSO ENARM CMN SIGLO XXI
CASO CLINICO
Se trata de un paciente masculino de 54 años de edad, el cual es referido por presentar intentossuicidas reiterativos (3 intentos) en un
lapso de 2 meses. El último de ellos realizado en las últimas horas a su envío (03/05/00), al arrojarse a un pozo, sólo provocándose
ligerascontusiones en diversas partes del cuerpo. El padecimiento inicia aproximadamente hace 4 años. De forma paulatina comienza a
presentar miedo a la obscuridad, que cada vez aumentaba en intensidad, desde hace dos años se da la aparición de movimientos
involuntarios en la frente (gesticulaciones), y sin motivos aparentes comienza con “decaimiento” del estado del ánimo, aislacionismo,
mutismo por días, para luego regresar a su estado “normal,” presentó además irritabilidad fácil, así como ideas de desesperanza,
refiriendo que su vida no tenía sentido, transcurre aproximadamente un año y seis meses, con poca afectación en su vida laboral, social
y de familia, durante este tiempo comienza a presentar algunos movimientos involuntarios en manos y en cuello de tipo espasmódicos,
haciéndose más evidentes los movimientos en cara, dichos movimientos se presentan de forma periódica y repetitiva.

PREGUNTA
Cual es el diagnostico mas probable?

RESPUESTA
a.- Enfermedad de Huntington
b.- Enfermedad de Sydenham.
c.- Enfermedad de Parkinson.
d.- Enfermedad de Piernas inquietas.

ENFERMEDADES NEUROMUSCULARES (ENM):


CIENCIAS BASICAS: Las enfermedades neuromusculares son enfermedades de carácter genético y generalmente hereditario que
afectan a la musculatura y al sistema nervioso. También se conocen con el nombre de miopatías. Su aparición puede producirse tanto
en el nacimiento como en otras etapas de la vida. Las ENM son trastornos de la unidad motora, cuyos principales síntomas son la
debilidad muscular, la fatiga, los calambres, el dolor, problemas articulares y la rigidez. Su distribución suele ser simétrica, a diferencia
de lo que ocurre en las enfermedades que afectan al sistema nervioso central (SNC). La debilidad de los músculos respiratorios es la
causa básica que conduce al fracaso de la bomba ventilatoria, y que se traduce en hipoxemia e hipercapnia. Sin embargo, el
compromiso del sistema respiratorio no es igual en todas las entidades sino que está determinado por el grado de afección muscular y
por la progresión de la enfermedad. Existen alrededor de 150 tipos de ENM: 1. Distrofias musculares: afectan predominantemente al
músculo estriado y son debidas a un defecto alguno de las proteínas que forman parte de la fibra muscular, ya sean estructurales o
enzimáticas (ejemplos son la distrofina calpaína, merosina y emerina, entre otras)2. Miopatías distales. 3. Miopatías congénitas. 4.
Distrofia miotónica de Steinert. 5. Miotonías congénitas. 6. Parálisis periódicas familiares. 7. Enfermedades musculares inflamatorias. 8.
Miositis osificante progresiva. 9. Miopatías metabólicas. 10. Enfermedades de la unión neuromuscular. 11. Amiotrofias espinales. 12.
Neuropatías hereditarias sensitivo-motoras (enfermedades de Charcot-Marie-tooth). ESCLEROSIS LATERAL AMIOTRÓFICA (ELA): Tiene
una incidencia anual de 1-2 casos/100.000 habitantes y su sustrato patológico es la degeneración de las neuronas motoras de la médula
espinal, el tronco cerebral y el córtex motor. Clínicamente se caracteriza por espasticidad e hiperreflexia al inicio de la enfermedad,
pero a medida que progresa se establecen otras síntomas, como amiotrofia asimétrica, debilidad muscular, fasciculaciones y síndrome
bulbar. La debilidad de los músculos respiratorios, fundamentalmente de los intercostales y del diafragma, es la causa de la
hipoventilación, y los síntomas respiratorios aparecen cuando la enfermedad está muy evolucionada, a pesar de que numerosos
estudios han comprobado que puede haber alteración de la función ventilatoria incluso cuando la debilidad muscular periférica es
ligera. A pesar de ello, lo más frecuente es que la insuficiencia respiratoria se presente en casos de ELA claramente establecida como
consecuencia de la progresión natural de la enfermedad o de forma aguda, precipitada por una infección de tracto respiratorio. De
forma ocasional, la insuficiencia respiratoria puede ser la primera manifestación de la ELA; en estos casos la lesión afecta
fundamentalmente a las motoneuronas del nervio frénico localizadas en la médula cervical, y la afección bulbar implicará un grave
compromiso de la vía aérea superior. ESCLEROSIS MÚLTIPLE (EM): Es una enfermedad desmielinizante del SNC, y se considera una de
las principales causas de discapacidad neurológica en adultos jóvenes. Como posibles etiologías se han descubierto datos que hacen
referencia a factores genéticos y ambientales. Los síntomas comunes de la enfermedad son debilidad muscular, espasticidad,
incoordinación motora y pérdida de agudeza visual. Habitualmente esta enfermedad tiene dos formas de presentación clínica: en forma
de brotes, en los que puede haber una remisión completa o parcial de los síntomas, o como una enfer medad lentamente progresiva44.
En este sentido, el compromiso del sistema respiratorio en la EM45 está en relación con la forma de presentación clínica de la
enfermedad, con el grado de debilidad muscular y con las estructuras del SNC que se ven afectadas por la desmielinización. En la mayor
serie de pacientes con EM y disfunción respiratoria, las complicaciones respiratorias aparecieron a los 9,5 años de las manifestaciones
neurológicas y se atribuyeron a debilidad muscular, disfunción bulbar, trastornos del control respiratorio, hiperventilación paroxística y
apneas obstructivas. En las lesiones agudas de la médula espinal, las complicaciones respiratorias dependen de la extensión y la
localización de la lesión y se deben a la interrupción de la inervación. Se consideran lesiones altas las que afectan a C1 y C2, y bajas las
que se producen entre C3 y C8. Las motoneuronas que inervan al diafragma se originan entre C3 y C5, por lo que las lesiones medula-
res situadas por encima de C3 implican una parálisis total de la musculatura respiratoria, mientras que en las lesiones entre C3 y C5 la
parálisis muscular es parcial. MIASTENIA GRAVIS (MG): Es una enfermedad autoinmune. Se presentan anticuerpos contra el receptor
de acetilcolina (AchR) que interrumpen la función de la acetilcolina en la unión neuromuscular presentando en debilidad muscular. Los
Músculos bulbar, facial y de las extremidades proximales son los más comúnmente afectados y justifican los síntomas de la debilidad
general, ptosis y diplopía. La debilidad en los músculos de la respiración puede llevar a la falla respiratoria la cual es llamada crisis
miasténica. Las complicaciones asociadas a falla respiratoria son la causa de muerte en paciente con Miastenia Gravis. La incidencia es
de menos de 1 caso por 100 000. Es más común en mujeres durante la segunda y tercera década de la vida. Sin embargo en la Séptima
y Octava década de la vida es más común en hombres. Los Afroamericanos también tienen una incidencia alta. La contracción muscular
ocurre cuando la acetilcolina es liberada por una fibra nerviosa y se une al AchR de una fibra muscular. En la MG, la fibra nerviosa es
normal, sin embargo, el número y función de los AchR nicotínicos del músculo esquelético están disminuidos. Los síntomas aparecen

CURSO ENARM CMN SIGLO XXI TEL: 36246001 Pharmed Solutions Institute PÁGINA 174
MANUAL DE TRABAJO DEL CURSO ENARM CMN SIGLO XXI
cuando el número de AchR esta disminuidos por debajo del 30% de lo normal. Los músculos cardiaco y liso no se afectan por que ellos
tiene diferente antigenicidad que el músculo esquelético. La hiperplasia del timo o el timoma se ha visto en el 75% de los pacientes con
MG. Por lo tanto, se sospecha que el timo es el sitio de producción de anticuerpos, pero el estímulo que inicia el proceso autoinmune
es desconocido. La morbilidad por la debilidad muscular intermitente incluye la neumonía por aspiración y caídas. Los síntomas
empeoran en el día progresivamente con el uso repetitivo de grupos musculares como leer o masticar. Los nervios craneales son los
más comúnmente afectados, resultando en ptosis, diplopía, debilidad facial, disfagia y disartria. La respuesta pupilar a la luz permanece
intacta. La debilidad muscular de las extremidades proximales empeora con el movimiento y mejora con el descanso. Los reflejos
tendinosos profundos pueden estar disminuidos pero nunca ausentes. No hay déficit en la función cerebelar y sensorial. La crisis
miasténica es el grado más severo de MG por que la debilidad extrema de los músculos respiratorios resulta en falla respiratoria
requiriendo soporte ventilatorio. Otras enfermedades que se pueden presentar con síntomas similares son el botulismo, hipotiroidismo
y lesiones de la masa intracraneal. La prueba de diagnóstico para MG incluyen la prueba de edrofonium, electromielograma con
estimulación repetitiva del nervio y prueba serológica para anticuerpos anti AchR. El cloruro de edrofonium es un inhibidor de la
colinesterasa de corta acción que incrementa la cantidad de acetilcolina en la unión neuromuscular. Este incremento vence el bloqueo
de los receptores y la ptosis, oftalmoplejía y la debilidad muscular se corrige en 30 segundos pero regresa a lo basal en 1 a 2 minutos. La
prueba de anticuerpos contra AchR es la prueba más específica para MG. El aspecto más importante para los médicos del
departamento de urgencias es que en todos pacientes gravemente enfermos, la prioridad es establecer y mantener una ruta aérea y
asegurar respiración adecuada. La Intubación endotraqueal por la inducción rápida de la sucesión y intubación (IRS) puede ser
necesario es antes de ser capaz de diferenciar miastenia gravis contra una crisis de colinergica. El preferido es la Succinilcolina. Los
pacientes con MG AchRs resistentes al succinilcolina y dosis más altas (2 mg/kg) debe ser usado para inducir la parálisis. Los agentes de
no despolarizantes como (rocuronium o vecuronium) son los agentes paraliticos preferidos. Estos agentes no causan la parálisis
prolongada en una dosis de 50% la dosis recomendada. La dosis en adultos de edrofonium es 2 mg IV lento. Sin presentar ninguna
respuesta ni ningún efectos adversos de tipo colinergico, administrando 8 mg IV lento para una dosis total de 10 mg, las
contraindicaciones del edrofonium es obstrucción gastrointestinal u obstrucción del trecho urinario. DISTROFIA MUSCULAR
PROGRESIVA, O ENFERMEDAD DE DUCHENNE: Es una enfermedad de herencia recesiva ligada al cromosoma X causada por
mutaciones en el gen de una proteína del citosqueleto: la distrofina. Es una miopatía proximal que comienza en la infancia y en la que,
inicialmente, se afectan los músculos de las extremidades inferiores (cintura pelviana), de modo que el paciente refiere dificultad para
correr o saltar y, a medida que progresa, afecta a los músculos de las extremidades superiores y del tronco; así, los enfermos a los 12
años de edad son dependientes por completo. También es característico de la enfermedad la miocardiopatía primaria, que cursa de
forma asintomática o con trastornos del ritmo cardíaco. La mayoría de estos enfermos presentan un cierto déficit intelectual, y los
pacientes suelen fallecer alrededor de los 20 años por complicaciones respiratorias o cardíacas. Muchas enfermedades
neuromusculares son genéticas, lo que significa que hay una tendencia familiar o existe una mutación en los genes. Algunas veces,
pueden ser provocadas por un trastorno del sistema inmunológico. La mayoría de ellas no tiene cura. El objetivo del tratamiento es
mejorar los síntomas, aumentar la movilidad y el lapso de vida.

CASO CLINICO
Se trata de masculino de 65 años de edad el cual acude debido a que desde hace cuatro meses presenta debilidad distal asimétrica,
además refiere contracturas musculares y fasciculaciones que se presentan con los movimientos voluntarios, a la exploración física se
observa debilidad extensora de las manos así como dificultad para mover la lengua y la cara, los reflejos de estiramiento muscular se
observan incrementados, la exploración de la sensibilidad no se observan alteraciones, al examen mental se encuentra pensamiento y
lenguaje con curso y contenido adecuado, humor y afecto conservados, resto de funciones mentales superiores están intactas.

PREGUNTA
Cuál es el diagnostico clínico más probable.

RESPUESTA
a.- Esclerosis lateral amniotrofica.
b.- Esclerosis multiple.
c.- Mielitis transversa.
d.- Síndrome de guillain-barre.

CASO CLINICO
Masculino de 23 años de edad el cual acude a consulta debido a que desde hace un mes presenta debilidad generalizada, dificultad
para caminar y alzar objetos, estas manifestaciones predominan en la mañana, durante el interrogatorio refiere dormir bien, no
consumo de drogas ó alcohol, no cuenta con enfermedades neurodegenerativas en la familia, actualmente se encuentra terminando la
licenciatura pero le preocupa estos síntomas, a la exploración física usted observa ptosis bilateral, y disminución de fuerza generalizada.

PREGUNTA
Cuál es el diagnostico más probable.

RESPUESTA
a.- Esclerosis multiple.
b.- Esclerosis lateral amniotrofica.
c.- Sindrome de Guillain-Barre.
d.- Miastenia gravis.

CURSO ENARM CMN SIGLO XXI TEL: 36246001 Pharmed Solutions Institute PÁGINA 175
MANUAL DE TRABAJO DEL CURSO ENARM CMN SIGLO XXI
TRASTORNOS TOXICO-METABOLICOS Y CARENCIALES DEL SISTEMA NERVISOSO:
TRASTORNOS TOXICO-METABOLICOS: El sistema nervioso puede verse dañado cuando el fallo de un órgano permite el acúmulo de
sustancias tóxicas que en circunstancias normales son eliminadas del organismo. También como consecuencia de drogas o toxinas
exógenas o endógenas, disfunción de mecanismos homeostáticos o por el déficit de sustratos indispensables. Las principales
herramientas de trabajo serán la historia clínica y la exploración física. El curso de los síntomas (monofásico, progresivo o con recaidas),
la presencia de síntomas sistémicos, enfermedades previas o coexistentes así como el uso de fármacos, hábitos dietéticos o tóxicos y la
exposición a tóxicos serán de gran importancia en la identificación del síndrome y su causa. Hallazgos de laboratorio, estudios más
específicos (autoanticuerpos, estudio del LCR) o pruebas de imagen entre otros nos permitirán completar la aproximación etiológica.
Las encefalopatias se desarrollan habitualmente de manera insidiosa y la norma en todas ellas es la alteración del estado mental, en
ocasiones de forma tan sutil que puede no detectarse con exploraciones rutinarias. La lesión del sistema reticular y la corteza cerebral
inducirán diferentes grados de alteración en la orientación, memoria, percepción, capacidad de concentración, juicio o planificación y
ejecución de tareas. Los trastornos metabólicos sistémicos y la exposición a toxinas provocarán más frecuentemente degeneración
axonal. La musculatura ocular será la más frecuentemente afectada así como los músculos flexores del cuello y de cinturas. Las
enfermedades de los músculos se presentarán con debilidad, dolor y fatiga. Déficits nutricionales y tóxicos exógenos serán los
responsables más frecuentes y el mecanismo patogénico en ocasiones está bien definido. ENCEFALOPATIA HIPÓXICO-ISQUÉMICA: El
cerebro es un órgano con alto requerimiento metabólico y por ello muy susceptible a daño por deprivación del flujo sanguíneo. Las
reservas de glucosa, glucógeno, ATP y posfocreatina se deplecionan a los 10-12 minutos de la isquemia. Después de 15 minutos de
isquemia global con parada cardiaca más del 95% del tejido cerebral estará dañado. Más de la mitad de los supervivientes a una parada
cardiaca presentan algún grado de daño cerebral permanente. La patogenia del daño cerebral dependerá de la etiología del mismo y el
pronóstico del mecanismo implicado. Cualquier mecanismo conduce finalmente a una necrosis y apoptosis neuronal irreversible.
Cambios bioquímicos diversos, daño mitocondrial precoz, alteración de citoesqueleto neuronal y la activación de receptores de
Glutamato son aspectos funcionales subyacentes al daño celular tras un insulto hipoxico-isquémico cerebral. El daño neuronal puede
ocurrir durante el periodo inicial de la isquemia (como consecuencia de los cambios bioquímicos y funcionales), en el periodo de
reperfusión (como consecuencia de la formación de radicales libres y daño tóxico continuo), por deterioro del flujo sanguíneo (al
reanudarse la circulación espontánea y como consecuencia de una microcirculación alterada) y como daño diferido (secundario a
desmielinización). ENCEFALOPATÍA HEPÁTICA: La enfermedad hepática crónica o aguda puede acompañarse de síntomas
neuropsiquiátricos (y neuromusculares) conformando distintos grados de encefalopatía. Es aceptado el papel del amonio como factor
clave dentro de una cascada de sucesos entre los que aparece alteración de neurotransmisores, excesiva producción de glutamina o
estrés oxidativo. El edema de astrocitos resultante se postula como la base fisiopatología subyacente en la forma de HE aguda y
crónica. Factores como la hiponatremia o sedantes como las benzodiacepinas influyen en la aparición de encefalopatía hepática incluso
con cifras de amonio en sangre normales. El aumento de osmolaridad intracelular causado por hiperamonemia produce reducción de
los picos de colina y mioinositol y un aumento en el pico de glutamina y glutamato. Las anomalías metabólicas se correlacionan con la
severidad clínica y son reversibles tras tratamiento. Las opciones terapéuticas dependen del estadio e instauración de la encefalopatía
hepática. El objetivo será reducir la producción de amonio, aumentar su fijación y excreción, controlar los síntomas neurológicos
presentes y modificar el shunt porto-sistémico. Bromocriptina o flumazenilo cayeron en desuso frente a lactulolosa o rifaximina. En
ocasiones el trasplante de hígado será la opción terapéutica a considerar. ENCEFALOPATIA URÉMICA: Los signos de encefalopatía en
pacientes con enfermedad renal no siempre obedecen al fracaso renal progresivo y pueden sumarse al daño neurológico secundario a
tratamientos inmunosupresores, diálisis o trasplante renal. La fisiopatología permanece por determinar. Trastornos del equilibrio acido-
base, alteración en las concentraciones de agua y electrolitos, metabolismo anómalo del calcio y glándulas paratiroides o cambios en
las concentraciones plasmáticas de distintos neurotransmisores están presentes y determinan los síntomas neurológicos detectables.El
tratamiento principal de la encefalopatía urémica es la diálisis. Así mismo existen dos síndromes neurológicos relacionados con ésta, el
síndrome del desequilibrio de la diálisis y la demencia de la diálisis. ENCEFALOPATIA HIPOGLUCEMICA: Glucemia menor de 30mg/dl, o
periodos prolongados de hipoglucemia, para que se produzca daño irreversible. La intensidad del metabolismo cerebral explica la
vulnerabilidad particular del cerebro frente a desórdenes metabólicos. Ante la posibilidad de una disminución importante de glucosa en
el SNC, la actividad de los centros cerebrales superiores disminuye para reducir las necesidades de energía. La hipoglucemia provoca
respuestas nerviosas y hormonales para aumentar la producción de glucosa en el hígado, y reducir su uso en órganos no nerviosos. Se
caracteriza por síntomas de estimulación del sistema nervioso simpático o de SNC, provocados por una concentración plasmática de
glucosa anormalmente baja. Los síndromes hipoglucémicos pueden producirse por fármacos o sustancias como insulina, alcohol o
sulfonilureas, y con menos frecuencia por salicilatos, propanolol, pentamidina, disopiramida, hipoglicina A. Clínica: confusión,
convulsiones, estupor, coma. ENCEFALOPATIA HIPERCAPNICA: Debido a enfermedades respiratorias crónicas retenedoras de CO2,
como fibrosis pulmonar, enfisema. Clínica: Presenta los síntomas de hipertensión craneal, cefalea holocraneal o frontal intensa, edema
de papila, somnolencia hasta coma. ENCEFALOPATÍA SÉPTICA: Disfunción cerebral difusa o multifocal asociada con una infección
sistémica, sin evidencias de infección intracraneal, y que no puede ser atribuida a otros factores tales como efectos farmacológicos o
disturbios metabólicos. La encefalopatía de la sepsis puede ser clasificada como encefalopatía séptica o precoz, que se presenta antes
de que se produzca la disfunción orgánica múltiple, o encefalopatía tardía, que es acompañada por fallo orgánico múltiple, hipotensión
y otros fenómenos sistémicos. Probablemente se origine por la acción de mediadores inflamatorios en el cerebro o por una respuesta
citotóxica de las células cerebrales a estos mediadores. Los efectos de la sepsis sobre el cerebro son detectables en cerebros
previamente sanos, pero son amplificados en casos con lesión cerebral concomitante, como luego de la injuria traumática o la
hemorragia subaracnoidea. La causa más común de encefalopatía en los pacientes con enfermedades médicas críticas, habiendo sido
descrita en el 9 al 71% de todos los pacientes críticos que sufren sepsis. TRASTORNOS CARENCIALES. Ante cualquier trastorno del SN
adquirido, siempre debe tenerse en consideración una carencia vitamínica ya que el tratamiento sustitutivo administrado al inicio del
cuadro puede hacer remitir la sintomatología, mientras que la administración tardía, cuando las lesiones anatomopatologicas ya están
establecidas, no evitara que el paciente quede con secuelas. Los síndromes más importantes y frecuentes son debidos a la carencia de
las vitaminas de grupo B y en general son secunmdarios a estados carencialesde malnutrición, alcoholismo o patología del sistema
digestivo que causen malabsorción. ENCEFALOPATIA DE WERNICKE-KORSAKOFF: Es un síndrome neuropsiquiátrico agudo que aparece
como consecuencia del déficit de tiamina (B1) y se asocia con una morbi-mortalidad significativa. Las causas de depleción de tiamina y

CURSO ENARM CMN SIGLO XXI TEL: 36246001 Pharmed Solutions Institute PÁGINA 176
MANUAL DE TRABAJO DEL CURSO ENARM CMN SIGLO XXI
los mecanismos responsables son múltiples. Las lesiones se localizan de forma simétrica a nivel periacueductal, sustancia gris
subependimaria del III ventrículo, vermis cerebeloso, tubérculos mamilares, núcleos hipotalámicos, porción medial y dorsal del tálamo
o núcleos vestibulares. El cuadro típico clínico de confusión mental con disminución del nivel de conciencia. Puede o no haber parálisis
oculomotora (VI) o nistagmo, oftalmoplejía y trastorno de la marcha aparece tan sólo en el 16% de los pacientes. Puede haber crisis
comiciales y si no se trata al paciente aparece miosis arreactiva a la luz, síntomas vegetativos, piramidalismo, coma y muerte. Una
correcta aproximación facilita el diagnóstico, la prevención en paciente con factores o situación clínica predisponentes y mejora el
pronóstico con el adecuado soporte de tiamina parenteral 500mgs en 100ml de SF a pasar en 30 min 3 veces al dia. Los pacientes
pueden superar la fase aguda con secuelas como el síndrome amnésico de Korsakov. DEFICIENCIA DE VITAMINA B12:
Cianocobalamina, La metilcobalamina actua como factor indispensable para la enzima sintetasa (transformación de homocosteina a
metionina), ruta indispensable para el metabolismo del ácido fólico y síntesis del ADN. La metinina es esencial para la síntesis de colina
y fosfolípidos. La deficiencia se manifiesta con anemia megaloblastica, glositis, atrofia de mucosa intestinal, ocasionalmente vaginitis.
Degeneración combinada subaguda de la medula (alteración de cordones posteriores y laterales, con lo que presenta trastornos
sensitivos, parestesias que ascienden desde los pies hasta el tronco, a la que se añade una ataxia), alteraciones cognitivas/demencia
(irritabilidad, psicosis), polineuropatia mixta sensitivo-motora. En casos de malabsorción o anemia perniciosa: 1000mg por vía IM diaria
durante 2 semanas. Luego seguir con 1000mg IM al mes. DEFICIENCIA DE ACIDO NICOTINICO: Niacina esencial para síntesis de NAD y
NADP. La deficiencia de niacina o su precursor el triptófano, provoca la pelagra. En el mundo occidental la pelagra es muy rara y se
observa en alcohólicos, en el síndrome carcinoide y enfermedad de Harnup. Clínica: triada clásica es dermatitis, diarrea y demencia,
puede haber cuadro neurológico sin manifestaciones cutáneas. De inicio es una encefalopatía inespecífica (síndrome confusional,
alteración del nivel de conciencia, ataxia y mioclonias) que evoluciona a demencia. Tratamiento, administrar entre 100-250mg VO tres
veces al día durante 5 días. La administración de 325 mg de aspirina media hora antes previene el “flushing” facial. DEFICIT DE
VITAMINA E: Debida a defecto de malabsorción intestinal, atresia de vías biliares. Se manifiesta como una degeneración
espinocerebelosa (ataxia, síndrome piramidal), más oftalmoplejía externa, retinitis pigmentaria, polineuropatía. Tratamiento
administración de vitamina E hidrosoluble por VO o IV.

CASO CLINICO
Paciente masculino, 42 años de edad, nacionalidad italiana, con antecedentes de tabaquismo, alcohol esporádico, hernioplastía
umbilical; biopsia hepática cirrosis hepática; familiar de primer grado con hepatitis de causa no filiada. Motivo de ingreso: cuadro agudo
de confusión mas euforia con posterior deterioro del sensorio de 24 hs de evolución más catarsis negativa en la última semana. Al
examen físico se constata somnolencia alternada por episodios de excitación, confusión témporo-espacial; flapping +; spiders en cara
anterior de tronco, ascitis grado II; edemas en miembros inferiores. Laboratorio de ingreso: Bilirrubina Total: 3.27/B.Directa:
1.87/B.Indirecta: 1.4 / FAL: 741/ GGT: 504/GOT : 180/GPT : 148. TP: 44% Plaquetas: 65.000/mm/ Factor V: 34%. Se solicita estudios
para determinar etiología de la misma; Serologías virales (VHC; VHB; HIV; VHA negativos)FAN; ASMA; AMA negativos;Ferremia:169
microg/dL(38-158); Ferritina184 nanogr/ml (8-110);Saturación de Transferrina 98,8%(20-50); Transferrina 171 microg/dL(180-350).
Ceruplasmina 18 mg/dL (5-30); Cupruria 24h 424 microgramos (0-60); Lámpara de hendidura; anillos de Kayser- Fleischer.

PREGUNTA
Considerando el cuadro clínico, que grado de encefalopatía presenta?

RESPUESTA
a.- Grado I.
b.- Grado II.
c.- Grado III.
d.- Grado IV.

CASO CLINICO
Mujer de 52 años con antecedentes de alcoholismo. Es traída al Servicio de Urgencia por cuadro de tres días de compromiso de
conciencia cuali-cuantitativo, vómitos y diarrea. Al ingreso se constata paciente confusa, inatenta, poco cooperadora. Al examen
cognitivo (Mini Mental, Test del Reloj, Test de Generación de Palabras, Test de trecho de digitos) destaca alteración de la atención y
memoria episódica, con amnesia anterógrada y retrógrada. El resto de las funciones cognitivas estaban relativamente conservadas. Se
observa confabulación durante el examen mental. En el examen físico neurológico se encontró dismetría y nistagmo bilateral. La
tomografía computada de cerebro y los parámetros de laboratorio básicos eran normales.

PREGUNTA
Cual es la conducta terapéutica mas apropiada?

RESPUESTA
a.- Cianocobalamina.
b.- Tiamina.
c.- Niacina.
d.- Riboflamina.

CASO CLINICO
Mujer de 60 años sindrome mielodisplásico tipo anemia refractaria simple en tratamiento, presenta cefalea hemicranea izquierda,
pulsatil, severa, súbita (2 dias) agrega fotopsias, nauseas y mareo, no relevante, a la EF se observa confusión, desorientación,
distractibilidad, alteración de la memoria a corto plazo. La resonancia nuclear magnética cerebral evidencia: Alteración en la intensidad

CURSO ENARM CMN SIGLO XXI TEL: 36246001 Pharmed Solutions Institute PÁGINA 177
MANUAL DE TRABAJO DEL CURSO ENARM CMN SIGLO XXI
de la señal de la sustancia blanca con extensión hacia las fibras U de predominio posterior, principalmente en la región parieto-occipital
bilateral y simétrica, con compromiso parcial de la corteza.

PREGUNTA
Cual es el diagnostico mas probable en este caso?

RESPUESTA
a.- Enfecefalopatia hipoxico-isquemico.
b.- Encefalopatia de wernicke-korsakoff.
c.- Encefalopatía hepática.
d.- Encefalopatia hipercapnica.

CASO CLINICO
Paciente del sexo femenino de 26 años de edad, refiere a los 10 minutos de histerorrafia, sentirse mal diciendo que le está dando sueño
y que el dolor no ha cedido y la tensión arterial comienza a disminuir y la frecuencia cardiaca, la paciente se encontraba ligeramente
cianótica y con tensión arterial de 80/40 torr y frecuencia cardiaca de 60 p.m. procedo a intubar sin aplicación de fármacos inductores
ni relajantes y se continuó con la administración de oxígeno al 100% a 5 l/min, observándose movimientos de la bolsa que
corresponden a inspiración y espiración del automatismo respiratorio, sin embargo se aplica ventilación asistida. Posteriormente la
paciente presenta paro cardiorespiratorio detectado con ausencia del pulso carotídeo, por lo cual se comienza a realizar maniobras de
resucitación cardiopulmonar y administramos 1 mg de epinefrina sin respuesta positiva por lo que se procede a administrar otro
miligramo más de epinefrina, saliendo del paro la paciente con tensión arterial de 130/90 y frecuencia cardiaca de 150 p.m., comienza a
disminuir nuevamente la frecuencia cardiaca y la presión arterial, por lo que administramos 0.6 mg de atropina y dopamina a dosis
respuesta; posteriormente, a pesar de tener la paciente tensión arterial de 130/90 torr con apoyo farmacológico con una frecuencia
cardiaca de 150 p.m. la paciente continúa con cianosis por lo que decidimos pasarla a terapia intensiva, practicándose una gasometría
arterial a su ingreso la cual nos reporta los siguientes parámetros: pH 7.27, PCO2 31, PO2 252, CO2 total 14.90, HCO3 14, EB -11.40,
Sat. O2 99.50%. Se practica una tomografía cerebral entre 36 y 48 horas después la cual únicamente observa edema cerebral. Se
extuba dos días después de su ingreso con respuesta neurológica Glasgow 7; a las veinticuatro horas, presenta crisis convulsivas tónico-
clonicas de una hora de duración por lo que sedan profundamente a la paciente y la intuban nuevamente, para que varios días después
se declare coma y lesión cerebral con estado vegetativo.

PREGUNTA
Cual es el diagnostico mas probable en este caso?

RESPUESTA
a.- Enfecefalopatia hipoxico-isquemico.
b.- Encefalopatia de wernicke-korsakoff.
c.- Encefalopatía hepática.
d.- Encefalopatia hipercapnica.

DEPRESION, ANSIEDAD, ESQUIZOFRENIA Y TRASTORNOS DE ALIMENTACION:


DEPRESIÓN: Trastorno del estado de ánimo con repercusión en distintos ámbitos de la persona. Existe una alteración a nivel emocional,
con un sentimiento intenso de tristeza, desesperanza, abandono, inutilidad o culpa. En otras ocasiones, ánimo irritable. A nivel del
contenido del pensamiento, predomina el pesimismo, con abundantes preocupaciones, ideas negativas repetitivas, angustia y falta de
interés con desesperanza en relación al futuro. Desde el punto de vista somático se producen alteraciones del sueño y del apetito, con
pérdida de peso, astenia y alteraciones digestivas que son expresión de la disfunción vegetativa. A nivel conductual se produce una
disminución del rendimiento, con afectación de la memoria, la atención y la capacidad de concentración, apatía, disminución de la
libido y tendencia al aislamiento social. Tanto en el trastorno depresivo como en el síndrome confusional agudo (SCA) puede existir una
afectación de las funciones cognitivas con disminución de la atención y del ciclo vigilia-sueño. Cabe destacar la depresión con rasgos
psicóticos, donde el paciente pierde el contacto con la realidad, combinando síntomas de depresión y psicosis, con aparición de
alucinaciones o delirios. Habitualmente, el contenido de éstos es coherente con la depresión, con una temática relacionada con ideas
de culpa, inducción al suicidio, etc. Existen varios tipos de episodios depresivos en función de su intensidad y curso: el trastorno
depresivo mayor, que generalmente aparece de forma episódica, y el trastorno distímico, menos intenso y con curso crónico. El
episodio depresivo mayor tiende a la recuperación completa, y puede darse de forma aislada en la vida de un sujeto. Sin embargo en
general es una enfermedad con tendencia a la cronicidad y resulta más incapacitante que otras enfermedades médicas como la artritis
o la diabete. Etiopatogenia, podría ser el modelo biopsicosocial, que resultaría de la integración de tres perspectivas diferentes, la
biológica, la psicológica y la social. Los sistemas de neurotransmisión serotoninérgica, noradrenérgica, dopaminergica y peptidérgica
están implicados en la depresión. La serotonina produce la inhibición o la activación de la neurotransmisión en el sistema nervioso
central. Desde las neuronas de los núcleos dorsales y caudales del rafe se extienden numerosas proyecciones serotoninérgicas hacia
áreas cerebrales asociadas a síntomas depresivos. Se ha observado disminución de los niveles de serotonina en el LCR de pacientes con
depresión. El sistema noradrenérgico está también implicado en la depresión. El sistema dopaminérgico implica áreas cerebrales de las
que depende el comportamiento y funciones fisiológicas alteradas en la depresión. La dopamina puede intervenir en la depresión
sobretodo en la manía, ya que algunos agonistas dopaminérgicos se han asociado a la aparición de manía y los antagonistas son
eficaces en su tratamiento. CRITERIOS DSM-IV-TR DEL EPISODIO DEPRESIVO MAYOR: Cinco o más de los siguientes síntomas (entre los
que debe cumplirse los dos primeros) que representan un cambio respecto a la actividad previa, durante un periodo mínimo de dos
semanas. Uno de los síntomas debe ser el estado de ánimo depresivo, o la pérdida de interés o de la capacidad para el placer. Estado
de ánimo depresivo. Disminución del interés o la capacidad para el placer. Aumento o pérdida importante de peso. Insomnio o

CURSO ENARM CMN SIGLO XXI TEL: 36246001 Pharmed Solutions Institute PÁGINA 178
MANUAL DE TRABAJO DEL CURSO ENARM CMN SIGLO XXI
hipersomnia casi cada día. Agitación o enlentecimiento psicomotor casi cada día. Fatiga o pérdida de energía casi cada día.
Sentimientos de inutilidad o de culpa excesivos o inapropiados. Disminución de la capacidad de pensar, concentrarse o indecisión, casi
cada día. Pensamientos recurrentes de muerte, ideación suicida recurrente. El tratamiento en todo tipo de depresiones el uso de
fármacos antidepresivos puede resultar beneficioso, no obstante hay que valorar el riesgo-beneficio especialmente en pacientes con
otras patologías y en pacientes ancianos. Una excepción es la depresión bipolar, por lo que es importante hacer una buena exploración
sobre los antecedentes personales de episodios maníacos e hipomaníacos, y de la historia familiar del paciente. En los pacientes
bipolares, antes de prescribir un antidepresivo es necesario administrar un fármaco aprobado como estabilizador del estado de ánimo.
Tratamiento de la depresión: Inhibidores selectivos de la recaptación de serotonina (ISRS), los inhibidores selectivos de la recaptación
de noradrenalina (ISRSN), los moduladores selectivos duales de los receptores de la noradrenalina y serotonina (NaSSA), inhibidores de
la recaptación noradrenérgica (IRN), inhibidores duales de la recaptación de la noradrenalina y dopamina (IRNaDa), Inhibidores de la
monoaminooxidasa (IMAO) que comprenden varios fármacos. Los nuevos antidepresivos en su conjunto han dado lugar a mejoras en la
farmacoterapia de la depresión por su facilidad de uso, menos efectos secundarios y mayor seguridad en sobredosis.
ANSIEDAD: Estado afectivo fisiológico que consiste en tener una sensación difusa de aprensión. Es una sensación desagradable y vaga,
de aparición aguda y transitoria, que se acompaña de una activación del sistema nervioso autónomo. Este estado actúa como
mecanismo de alerta y adaptación ante una situación amenazante para la integridad física o psíquica del individuo, o percibida como
tal. La ansiedad patológica constituye un estado de ansiedad que se desencadena sin un estímulo amenazante, a diferencia del miedo,
que constituye una situación de ansiedad provocada por un estímulo realmente amenazante. La manifestación de la ansiedad se basa
en tres componentes clínicos fundamentales: un componente somático (síntomas vegetativos), uno cognitivo (sensación de
nerviosismo, aceleración, percepciones de malestar, etc) y uno motor (temblor,etc). Trastornos de ansiedad según la DSM-IV TR: 1.
Ataques de pánico (crisis de ansiedad, crisis de angustia, panic attac ). 2. Agorafobia − Trastorno de angustia sin agorafobia (F41.0) −
Trastorno de angustia con agorafobia (F40.01) − Agorafobia sin historia de trastorno de angustia (F40.00). 3. Fobia específica
(F40.02)4. Fobia social (F40.1). 5. Trastorno obsesivo-compulsivo (F42.8). 6. Trastorno por estrés postraumático (F43.1) 7. Trastorno
por estrés agudo (F43.0)8. Trastorno de ansiedad generalizada (F41.1) 9. Trastorno de ansiedad debido a enfermedad médica (F06.4)
10.Trastorno de ansiedad inducido por sustancias Además de estos trastornos, agrupados bajo el rótulo “trastornos de ansiedad”, en

DIAGNOSTICO DIFERENCIAL DE TRASTORNOS DE ANSIEDAD


TIEMPO
DURA PERIODICIDAD DE DESENCADENANTE MANIFESTACIONES OBSERVACIONES
CION SINTOMAS
Fobia No Episodios esporádicos, Estimulo fóbico (presencia o Miedo a un objeto o situación determinada. Temor o Evitación de la situación que
especifica min/hrs anticipación) síntomas ansiosos provoca el temor
Fobia social No Episodios Situaciones sociales o escrutinio social (presencia o Miedo a colocarse en una situación vergonzosa Evitación de la situación
esporádicos, anticipación). Temor angustioso a en un medio social. Temor o síntomas ansiosos. que provoca el temor
min/hrs determinadas circunstancias, reales o imaginarios Preocupación de que el desempeño pueda ser
Preocupación por que el desempeño sea casa de causa de humillación o burla
humillación.
Trastorno 1 mes Episodios Aparición brusca de un medio intenso. Temblor, sudoración, miedo a morir, sensación de 1 de 3: ansiedad anticipatoria.
de pánico esporádicos 5-20 Ninguno aparente, pero el sujeto lo frio/calor, nauseas, sensación de asfixia, marea, Preocupación constante,
min puede atribuir a múltiples causas miedo a perder el control, dolor de pecho Ataques Repercusión de los AP. Cambio
de pánico (AT)súbitos inesperados y recurrentes conductual significativo
Ansiedad 6 La mayor parte del Aparición progresiva y permanente Fatiga, inquietud, dificultad para dormir, El individuo está preocupado
generalizad meses día casi todos los de síntomas de ansiedad, sin una irritabilidad, tensión muscular, permanentemente sin motivos aparentes y no
a (TAG) días causa real que los provoque preocupaciones, nerviosisimos excesivos lo puede controlar. 3 de 6: intranquilidad,
cuya intensidad han debilitado el control fatigabilidad, concentración, irritabilidad,
sobre ellos tensión muscular, alteración en el dormir
Obsesivo No Min7hrs Obsesiones que pueden ser días o Cada obsesión lleva asociada una compulsión o Un ejemplo es el miedo al
compulsivo pensamientos que se repiten, y no conducta que compensa la angustia que provoca contagio, cuya composición
(TOC) desaparecen de le mente del individuo, correspondiente es lavarse las
aunque lo intente por todos los medios manos rápidamente
Estrés Aparece en aquellos individuos que se Perdida del sueño, irritabilidad, sobresalto, falta de Provoca alteraciones en la
postraumti han visto expuesto a un evento concentración. Re exoperimentracion de la situación vida familiar, laboral y social
cos traumático, que involucra un daño físico traumática: recuerdos intrusivos recurrentes, sueños del individuo
(EPT) o psicológico extremo estresantes recurrentes, flashbacks, temor ansiedad o
reacciones neorovegetativas intensas al exponenrsea
situaciones u objetosrelacionados con el evento.

la DSM-IV TR se incluye finalmente un trastorno de ansiedad infantil, el trastorno de ansiedad por separación.
En el trastorno del pánico se podrán utilizaransiolíticos, y para prevenir la crisis se administrarán antidepresivos. Este tratamiento
tendrá una duración de 6-12 meses. El trastorno obsesivo-compulsivo deberá ser tratado por el psiquiatra. El
tratamiento consiste en la combinación de un fármaco junto a terapia psicológica de modificación de conducta. Las fobias.
Hay que distinguir el tipo. Si es específica, se puede abordar en atención primaria con untratamiento semejante al de
la ansiedad; pero si es grave, como la agorafobia, fobia social generalizada o algún tipo de fobia simple,será tratada por el
especialista. TRASTORNOS DE LA ALIMENTACION: La ANOREXIA NERVIOSA: Se caracteriza por la negativa para mantener el peso
corporal normal, lo que resulta en un peso corporal <8 % del peso esperado para la edad y altura. Signos y síntomas: sensación de frío,
la piel, el cabello, las uñas quebradizos, alopecia , lanugo, acrocianosis , edema, bradicardia , hipotensión, agrandamiento de las
glándulas salivales, el vaciamiento gástrico lento, estreñimiento, enzimas hepáticas elevadas, anemia normocitica normocromica,
leucopenia, aumento de nitrógeno de urea en sangre , aumento de la creatinina, hiponatremia, hipopotasemia, baja la hormona
luteinizante y la hormona folículo-estimulante con amenorrea secundaria, hipoglucemia, la hormona estimulante de la tiroides normal
con baja tiroxina, aumenta el cortisol plasmático. Características de diagnóstico: Rechazo a mantener el peso corporal igual o por
encima del valor mínimo normal. (Esto incluye un fracaso para lograr la ganancia de peso esperada durante un periodo de crecimiento
que conduce a un peso corporal anormalmente bajo). Miedo intenso a la ganancia de peso o la grasa. Distorsión de la imagen corporal
(por ejemplo, sensación de grasa a pesar de un peso objetivamente bajo o minimizar la gravedad de bajo peso). La amenorrea (Este
criterio se cumple si los períodos menstruales se producen sólo después de hormonas, por ejemplo, la administración de estrógenos).

CURSO ENARM CMN SIGLO XXI TEL: 36246001 Pharmed Solutions Institute PÁGINA 179
MANUAL DE TRABAJO DEL CURSO ENARM CMN SIGLO XXI
Tratamiento: Recuperación del peso a 90% de peso predicho es el objetivo principal. La intensidad del tratamiento inicial, incluyendo la
necesidad de hospitalización, se determina por el peso actual, la rapidez de la pérdida de peso reciente, y la gravedad de las
complicaciones médicas y psicológicas. Desequilibrios electrolíticos graves deben ser identificadas y corregidas. Recuperación
nutricional casi siempre se puede lograr con éxito por la vía oral. Las calorías se pueden aumentar gradualmente hasta lograr un
aumento de peso de 1-2 kg/sem (3000-4000 kcal /d). Las comidas deben ser supervisadas. La ingesta de vitamina D (400 UI/d) y calcio
(1.500 mg/d) debe ser suficiente para reducir al mínimo la pérdida de hueso. La ayuda de psiquiatras o psicólogos suele ser necesario.
No hay medicamentos psicotrópicos. Las complicaciones médicas en ocasiones se producen durante la realimentación, la mayoría de
los pacientes transitoriamente retienen el exceso de líquido, a veces resultando en edema periférico. La insuficiencia cardíaca
congestiva y la dilatación gástrica aguda se han descrito cuando la realimentación es rápida. Elevaciones modestas transitorias en los
niveles séricos de enzimas hepáticas ocurren ocasionalmente. Los niveles bajos de magnesio y fosfato deben ser reemplazados. La
mortalidad es del 5% por década, ya sea de hambre crónica o el suicidio. BULIMIA NERVIOSA: Se caracteriza por episodios recurrentes
de atracones seguidos de conductas compensatorias anormales, tales como vómitos autoinducidos, abuso de laxantes o ejercicio
excesivo. Peso es en el rango normal o por encima. Signos y síntomas: agrandamiento de las glándulas salivales, la erosión dental,
hipopotasemia, hipocloremia, alcalosis (vómitos) o acidosis (de abuso de laxantes). Características de diagnóstico: Los episodios
recurrentes de atracones de comida, que se caracterizan por el consumo de una gran cantidad de alimentos en un corto período de
tiempo y la sensación de que la alimentación está fuera de control. Comportamiento inapropiado recurrente para compensar la ingesta
compulsiva, como el vómito autoinducido. La aparición de tanto el trastorno por atracón y el comportamiento compensatorio
inapropiado al menos dos veces por semana, en promedio, durante 3 meses. Preocupación excesiva por la forma y el peso corporal.
Nota: Si se cumplen simultáneamente los criterios diagnósticos para la anorexia nerviosa, sólo se da el diagnóstico de anorexia
nerviosa. Puede ser tratada de forma ambulatoria. La terapia cognitiva conductual y la fluoxetina (Prozac) son tratamientos de primera
línea. La dosis de tratamiento recomendada para la fluoxetina (60 mg/d) es más alta que la normalmente utilizada para tratar la
depresión. Tanto la anorexia nerviosa y la bulimia nerviosa presentan principalmente en mujeres jóvenes previamente sanos que se
convierten en demasiado preocupados con la forma y el peso corporal. Atracones y las purgas comportamiento que puede estar
presente en ambas condiciones, con la distinción fundamental entre los dos que descansa sobre el peso de la persona. Las
características diagnósticas. ESQUIZOFRENIA: Trastorno mental severo que afecta al pensamiento, las emociones y el comportamiento.
Es la forma más frecuente de trastorno mental severo y afecto a una persona de cada 100. Es poco frecuente antes de la pubertad y
más común de 15 a 35 años. Con frecuencia dura toda la vida. Un niño que tiene un padre con esquizofrenia tiene una probabilidad
entre 10 de desarrollar esquizofrenia. Las infecciones virales durante el embarazo, complicaciones en el parto, crecer en el centro de
grandes ciudades y el uso de drogas también parecen jugar un papel en el desarrollo de esta enfermedad. La estructura y química del
cerebro pueden estar afectadas, pero no existe una prueba simple para su diagnóstico hasta el momento. Formas clínicas:
Esquizofrenia paranoide. Esquizofrenia catatónica. Esquizofrenia hebefrénica/desorganizada. Esquizofrenia residual. Existen dos formas
de grupos: “positivos” y “negativos”, normalmente tienen una mezcla de los dos. Síntomas positivos: Delirios (creencias que no sólo
no son ciertas sino que pueden parecer incluso bastante extrañas). Si intentas discutir estas creencias, encontrarás que el que las tiene
las mantendrá a pesar de que exista mucha evidencia en su contra. Trastorno del pensamiento (dificultad para pensar con claridad).
Será difícil entenderlos porque sus frases parecen no tener sentido: pueden saltar de una idea a otra, perdiendo el tema del que están
intentando hablar. Alucinaciones (ves, oyes, hueles o sientes algo que no está ahí). Lo más común oír voces. Estas voces parecen
absolutamente reales, pueden asustar y hacer creer que la gente te está observando, escuchando o molestando. Síntomas negativos:
Las personas jóvenes que padecen esquizofrenia puede volverse muy inactivas, solitarias y parecer desmotivadas. Parecen perder
interés y motivación y pueden dejar de lavarse regularmente o cuidarse de una forma apropiada. Normalmente no pueden concentrase
en un trabajo o en el estudio. Generalmente los síntomas son de una gravedad suficiente como para causar preocupación, tanto en
casa como en la escuela. Los síntomas positivos o negativos por sí solos no necesariamente causan esquizofrenia. Criterios diagnósticos:
A. Síntomas característicos: Dos (o más) de los siguientes síntomas, cada uno de ellos presente durante una porción de tiempo
significativa a lo largo de un mes (o menos en el caso de que hayan sido tratados satisfactoriamente): (1) Ideas delirantes (2)
Alucinaciones (3) Desorganización del habla (4) Conducta catatónica o marcadamente desorganizada (5) Síntomas negativos, i.e.,
embotamiento afectivo, alogia, o abolición B. Deterioro social/ocupacional. C. Duración: Los signos persisten de manera continuada por
lo menos seis meses. Este periodo de seis meses debe incluir al menos un mes de síntomas (o menos si responden a tratamiento) del
criterio A, pudiendo incluir periodos de síntomas prodrómicos o residuales. Durante estos periodos prodrómicos o residuales los signos
del trastorno pueden consistir en sólo síntomas negativos o dos o más síntomas del criterio A presentes de forma atenuada (ej.,
pensamientos extraños, experiencias de la percepción inusuales) D. Exclusión de trastorno del ánimo o esquizoafectivo. E. Exclusión de
causa orgánica o abuso de sustancia o a condición médica general F. Relación con trastorno del desarrollo: Si existe historia de autismo
u otro trastorno del desarrollo, el diagnóstico adicional de esquizofrenia se realizará sólo si existen delirios o alucinaciones marcadas
durante al menos un mes (o menos si responden a tratamiento). El tratamiento se centra principalmente en eliminar los síntomas. Para
obtener el mejor resultado, todo el mundo implicado, incluida la persona afectada, la familia, el equipo psiquiátrico, los docentes y los
servicios sociales, necesitan trabajar juntos desde el inicio. Controla los síntomas de la enfermedad y permite reiniciar una vida normal.
La medicación tiende a ser más efectiva con los síntomas positivos y menos con los negativos. Las alucinaciones y las ideas delirantes
pueden tardar algunas semanas en desaparecer. ANTIPSICOTICOS Muchas divisiones, la más utilizada: convecionales y nueva
generación (típicos y atípicos); Antipsicóticos convencionales: haloperidol, levomepromacina, pimocide, zuclopentixol, sulpiride,
tioridacina. Antipsicóticos de nueva generación: clozapina, risperidona, olanzapina, quetiapina, ziprasidona, amisulpiride y aripiprazol.
Tratamiento esquizofrenia resistente:clozapina (vigilar agranulocitosis). Los antipiscóticos tratan parte de los síntomas de la
enfermedad, mejoran el funcionamiento y calidad de vida y evitan recaidas. Antipsicóticos de nueva generación: clozapina, risperidona,
olanzapina, quetiapina, ziprasidona, amisulpiride y aripiprazol. Antes de cambiar de antipsicótico esperar al menos 3-4 semanas a dosis
terapéuticas: Evaluar el riesgo de efectos secundarios a corto y largo plazo. Especial importancia tiene la dicinesia tardía y el síndrome
neuroléptico maligno. Evaluar la respuesta subjetiva al tratamiento. Reforzar siempre necesidad de tratamiento y evaluar conciencia de
enfermedad. Para la sintomatología extrapiramidal (SEP) se deben dar anticolinérgicos como el biperideno (Akineton), en caso de
distonía aguda dar vía intramuscular y el cuadro revierte a los pocos minutos. Para la acatisia (sensación interna de inquietud, dificultad
para estar sentado, mover las piernas, etc) se puede utilizar propanolol. En algunos casos también responde a anticolinérgicos

CURSO ENARM CMN SIGLO XXI TEL: 36246001 Pharmed Solutions Institute PÁGINA 180
MANUAL DE TRABAJO DEL CURSO ENARM CMN SIGLO XXI
(biperideno) o dosis bajas de benzodiacepinas. La acatisia es especialmente preocupante porque se asocia a abandono del tratamiento,
agresividad y suicidio. TEC especialmente en pacientes con catatonía, gran riesgo de suicidio o importante componente afectivo.
Terapia conductual: Tiene en cuenta las deficiencias que tienen los pacientes. Habilidades sociales en las que se enseña a los paientes a
relacionarse, buscar trabajo, hacer una entrevista, utilizar medios de transporte, etc. Terapia orientada a la familia (sistémica):
Información, identificación de factores precipitantes de recaída, reducir emoción expresada. Importancia de asociaciones de familiares.
Terapia grupal: Centrada en planes de la vida diaria, intercambio de experiencias, conciencia de enfermedad, resolución de problemas.
Psicoterapia individual: Su eficacia se añada a la del tratamiento farmacológico. Importancia de la relación terapéutica.

CASO CLINICO
Mujer de 69 años, traída a la consulta por su hija, por presentar sensaciones de abandono persistente, temor a quedarse sola en su
casa, cambios del humor , insomnio intermedio y final (logra conciliar el sueño, pero se despierta en la madrugada, a veces ya no logra
volver a dormir), la invaden pensamientos catastróficos, miedos sobre todo cuando su hija sale del domicilio a trabajar, piensa que algo
malo le puede suceder y no soporta quedarse sola, llora constantemente, presenta labilidad emocional, su comportamiento ha
regresionado de ser una persona muy independiente, a ser alguien que exige protección constantemente, llegando inclusive a agotar
emocionalmente a su familiar, la sintomatología se evidencia después de recuperarse de una crisis hipertensiva, la hija refiere que
desde entonces su madre ya no fue la de antes. Antecedentes: Siempre fue una persona independiente, su madre falleció cuando era
una adolescente y su padre la abandono con sus tres hermanos, era la segunda de cuatro hermanos, se vio obligada a trabajar desde
muy joven y resolvía los problemas cotidianos sin dificultad llegando incluso a la sobreprotección con sus familiares, su experiencia de
pareja al parecer fue complicada, brinda pocos datos al respecto, decidió vivir sola con su hija la cual tuvo estando soltera, Hasta su
adultez sufrió de cuadros migrañosos con frecuencia

PREGUNTA
Cual es el diagnostico mas probable del caso presentado?

RESPUESTA
a.- Trastorno distimico.
b.- Episodio depresivo.
c.- Trastorno depresivo.
d.- Trastorno de ansiedad.

CASO CLINICO
Mujer de 58 años. Lleva más de 20 años en tratamiento psiquiátrico y ha sido valorada por más de un psiquiatra durante su evolución.
Desde los 20 años de edad, ella se reconoce triste. Excepto por breves temporadas, cada una de las cuales raramente ha durado más de
un mes, en las que se nota prácticamente bien. La intensidad de la sintomatología nunca ha sido severa. En general, con ciertos
reparos, reconoce que siempre ha seguido desarrollando las tares domésticas. Desde hace años es su hija quien se responsabiliza de la
casa, y ella procura ayudarle. Se queja de tristeza, desánimo al afrontar el día desde que se levanta. Pero sobre todo una pobre
confianza en sí misma. Dificultad para concentrarse en cualquier tarea que con dificultad emprende.

PREGUNTA
Cual es el diagnostico mas probable para el caso?

RESPUESTA
a.- Trastorno bipolar.
b.- Depresión mayor.
c.- Trastorno Distímico.
d.- Depresión Psicótica.

CASO CLINICO
Mujer de 43 a. acude al Servicio de Urgencias. Se siente cansada y sin fuerzas para vivir. Responde con lentitud a las preguntas, en un
tono de voz bajo. Dice encontrarse muy triste desde hace dos meses, lo dice con precisión. Ha perdido interés por la vida, se nota
distinta, como si no le importara nada, ni siquiera lo que hagan su marido o sus hijas, aunque no quiere verlos sufrir. Tiene muy poco
apetito, ha perdido casi 5 kg en un mes. Tarda mucho en quedarse dormida y se despierta varias veces por la noche. Se siente inútil y
un estorbo en casa. Piensa en la muerte, aunque no quiere quitarse la vida, porque ello va contra sus creencias religiosas. Nunca antes
le había ocurrido algo parecido.

PREGUNTA
Cual es el diagnostico mas probable para el caso?

RESPUESTA
a.- Trastorno bipolar.
b.- Depresión mayor.
c.- Trastorno Distímico.
d.- Depresión Psicótica.

CURSO ENARM CMN SIGLO XXI TEL: 36246001 Pharmed Solutions Institute PÁGINA 181
MANUAL DE TRABAJO DEL CURSO ENARM CMN SIGLO XXI
CASO CLINICO
La paciente, nacida en 1981. Venía padeciendo ataques de angustia episódicos recidivantes desde el año 2003, sobre todo cuando se
encontraba sola. Tampoco era capaz de recorrer trayectos largos en tren ni en metro. Las crisis comenzaron tras el fallecimiento de su
abuelo, con el cual había tenido supuestamente una relación especial, ya que siempre había sufrido los problemas de relación de sus
padres. Más o menos al mismo tiempo se rompió el antebrazo por una caída, que requirió una complicada operación. Esto acrecentó
aún más sus crisis de angustia. Por lo demás, refirió que gozaba de buena salud, no fumaba ni bebía alcohol, aunque descuidaba las
comidas por motivos de tiempo. Dormía bien, necesitaba de 7 a 8 horas de sueño diarias, en caso contrario no se encontraba en forma.

PREGUNTA
Cual es el diagnostico mas probable en el caso descrito?

RESPUESTA
a.- Trastorno de pánico.
b.- Trastorno de estrés.
c.- Trastorno de ansiedad.
d.- Trastorno obsesivo compulsivo.

CASO CLINICO
Femenino, de 52 años, es ama de casa y madre de cinco hijos ya adultos. La relación con su marido ha perdido mucho con los años pero
se resiste a iniciar la separación. Se ha preocupado en exceso por muchos motivos: sus hijos, su madre, sus nietos... Algunas frases
típicas suyas son:¿Ha llegado Joaquín?; ¿Están bien los críos?; Llevad cuidado con el coche; No salgáis hasta muy tarde, que nunca se
sabe qué puede pasar en la noche. El menor de sus hijos tiene ya 20 años y se ha acostumbrado a las continuas advertencias y
sugerencias para prevenir males posibles. También se ha acostumbrado a llamarla por teléfono en mitad de la noche cuando sale con
los amigos para informarle de que no pasa nada, que todo va bien. Reconoce que le resulta difícil dejar de preocuparse tanto por todos
y por todo. Le resulta muy difícil concentrarse en otra cosa que no sean los peligros que acechan a los suyos, confundiendo con
frecuencia el hecho de que un peligro sea posible con el hecho de que sea probable. Tiene dificultades para dormir y mucha tensión
muscular acumulada. Se resistía a reconocer que lo suyo era un problema de ansiedad “porque los peligros son reales.

PREGUNTA
Cual es el diagnostico mas probable en el caso descrito?

RESPUESTA
a.- Trastorno de pánico.
b.- Trastorno de estrés.
c.- Trastorno de ansiedad.
d.- Trastorno obsesivo compulsivo.

CASO CLINICO
Varón de 33 años, que acude por episodios de agitación psicomotriz en domicilio, heteroagresividad hacia familiares y conductas
acontextuales. Ingresa mostrándose hostil, irritable, suspicaz y autorreferencial. Episodio de agitación psicomotriz en Urgencias y
heteroagresividad dirigida a familia y mobiliario. Es necesaria contención farmacológica y mecánica hasta remisión de la clínica actual
(Olanzapina i.m) y se cursa ingreso involuntario (nula conciencia de enfermedad, riesgo alto para el paciente y terceros y necesidad de
completar estudio) Conductas de riesgo desde la adolescencia (carreras de coches, consumo de tóxicos -THC y cocaína). Personalidad
premórbida impulsiva según su familia y con escasa tolerancia a las normas. Soltero. Padre de una niña que nace en 2000, con la que
mantiene escasa relación. En 2002, a la edad de 23 años, en contexto de intoxicación enólica, sufre accidente de tráfico con
importantes secuelas orgánicas, politraumatizado (incluido traumatismo craneoencefálico (TCE) grave) requiriendo hospitalización
durante 70 días. Permanece en coma según historia clínica durante 15 días hasta recuperación total del nivel de conciencia. Al día de
hoy no secuelas físicas.

PREGUNTA
Cual es el diagnostico mas probable en el caso descrito?

RESPUESTA
a.- Trastorno de personalidad.
b.- Trastorno mental secundario a lesión neurológica.
c.- Trastorno de la conducta secundaria a lesión neurológica.
d.- Trastorno de control de impulsos.

CASOS CLINICOS
Paciente de 50 años, que presenta en la anamnesis entre otros los siguientes síntomas: presencia de obsesiones y de compulsiones o de
actos obsesivos de manera recurrente, con pérdida de tiempo significativa, que supone una alteración en el funcionamiento normal, en
un paciente que comprende lo absurdo de su forma den proceder.

PREGUNTA
Cual es el tratemiento mas adecuado para el caso descrito?

CURSO ENARM CMN SIGLO XXI TEL: 36246001 Pharmed Solutions Institute PÁGINA 182
MANUAL DE TRABAJO DEL CURSO ENARM CMN SIGLO XXI
RESPUESTA
a.- Fluvoxamina.
b.- Fluoxetina.
c.- Sertralina.
d.- Clorimipramina.

CASO CLINICO
Persona de 30 años, que en el curso de menos de 6 meses, acude a la consulta y observamos que el índice de Quetelet es menos (peso
en Kg/altura al cuadrado) que 17,5 (normal de 19 a 24). Ha perdido 10 kilos o más en ese tiempo, presenta amenorrea de tres o más
ciclos consecutivos, y un miedo intenso a ganar eso.

PREGUNTA
Cual es el tratemiento mas adecuado para el caso descrito?

RESPUESTA
a.- Fluvoxamina.
b.- Fluoxetina.
c.- Sertralina.
d.- Clorimipramina.

CASO CLINICO
Paciente con 27 años, que acude en busca de tratamiento para lo que cree bulimia nerviosa, mujer de peso normal, que refiere
episodios de ingestión desenfrenada de comida. Los familiares no saben qué hacer, por qué la bulimia nerviosa le está llevando a otras
adicciones como el alcoholismo.

PREGUNTA
Cual es el tratemiento mas adecuado para el caso descrito?

RESPUESTA
a.- Fluvoxamina.
b.- Fluoxetina.
c.- Sertralina.
d.- Clorimipramina.

CASO CLINICO
Paciente mujer de 42 años acude, acompañada, por presentar episodio de agitación psicomotriz y trastorno de conducta con
heteroagresividad física y verbal en entorno familiar. Se procede a su ingreso para aclaración diagnóstica y contención de la situación,
con orientación diagnóstica de síndrome maniforme. Psiquiátricos: A los 21 años debuta con un episodio depresivo, cuya
sintomatología cedió en pocos meses y fue tratado por Médico de Atención Primaria. Tras un segundo episodio depresivo, presenta a
los 24 años un primer cuadro maniforme que requirió ingreso hospitalario y cuya orientación diagnóstica al alta fue de trastorno
bipolar, episodio actual maniaco.

PREGUNTA
Cual es el tratamiento mas adecuado para el caso descrito?

RESPUESTA
a.- Litio
b.- Olanzapina.
c.- Lamotrigina.
d.- Valproato.

CURSO ENARM CMN SIGLO XXI TEL: 36246001 Pharmed Solutions Institute PÁGINA 183
MANUAL DE TRABAJO DEL CURSO ENARM CMN SIGLO XXI

URGENCIAS

1) MUERTE SUBITA, RCP.


2) ESTADO CHOQUE
3) URGENCIAS, EMERGENCIA HIPERTENSIVA. HIPERTENSION MALIGNA.
4) INSUFICIENCIA CARDIACA AGUDA Y CRONICA AGUDIZADA.
5) TAPONAMIENTO CARDIACO, DISECCION DE LA AORTA, ANEURISMA AORTICO.
6) ANGINA ESTABLE, ANGINA INESTABLE, ANGINA DE PRINZTMETAL.
7) SICA II, SICA II
8) TRASTORNOS DEL RITMO (ARRITMIAS CARDIACAS)
9) FALLA ORGANICA MULTIPLE, COAGULACION INTRAVASCULAR DISEMINADA.
10) ANAFILAXIA Y ALERGIAS.
11) EPOC AGUDIZADO, TROMBOSIS VENOSA PROFUNDA, TROMBOEMBOLIA PULMONAR.
12) SINDROME DE INSUFICIENCIA RESPIRATORIA AGUDA, EDEMA AGUDO PULMONAR.
13) NEUMOTORAX, NEUMOMEDIASTINO, HEMOTORAX, CONTUSION CARDIACA.
14) TRAUMA TORACICO CERRADO Y ABIERTO.
15) INSUFICIENCIA RENAL AGUDA, GLOMERULOPATIAS AGUDAS.
16) SINDROME NEFRITICO Y NEFROTICO.
17) TRASTORNOS ACIDO-BASE AGUDOS.
18) TRASTORNOS ELECTROLITICOS AGUDOS.
19) HIPOGLUCEMIA, HIPERGLUCEMIA, ESTADO HIPEROSMOLAR, CETOACIDOSIS DIABETICA.
20) TRAUMATISMO CRANEOENCEFLICO, ISQUEMIA CEREBRAL TRANSITORIA, ACCIDENTE VASCULAR CEREBRAL.
21) CRISIS CONVULSIVAS, ESTATUS EPILEPTICO, COMA Y MUERTE CEREBRAL.
22) TRASTORNO BIPOLAR, DELIRIUM, PSICOSIS, SUICIDIO.

CURSO ENARM CMN SIGLO XXI TEL: 36246001 Pharmed Solutions Institute PÁGINA 184
MANUAL DE TRABAJO DEL CURSO ENARM CMN SIGLO XXI
MUERTE SÚBITA (MS):
CIENCIAS BÁSICAS: Muerte (ausencia de signos vitales) inesperada sin síntomas precedentes la mayoría de las veces o que en caso de
existir estos, ocurren pocos segundos antes de que la muerte sobrevenga y sin causa traumática aparente o que la explique. Son 4 las
arritmias letales, las de ritmos descargables como; fibrilación ventricular (FV), taquicardia ventricular sin pulso (TVSP), y las de ritmos no
descargables; asistolia y actividad eléctrica sin pulso (AESP). SALUD PÚBLICA: En Estados Unidos de Norteamérica hay hasta 300mil
muertes por año. En México entre 33 y 53 millones anuales, en general ligadas a enfermedad isquémica cardiaca. La cardiopatía
coronaria aumenta 4-6 veces el riesgo de muerte súbita. En 60-70% de los casos de muerte súbita, sobreviene como consecuencia, de
un evento cardiovascular; 1. Cardiopatía coronaria (isquémica) con o sin antecedentes conocidos, es responsable del 70-80%. 2.
Cardiopatía estructural, congénita o adquirida 15-20% (miocardiopatía hipertrófica, dilatada, displasia del ventrículo, estenosis mitral)
3. Arritmias 5% asociadas a fenómenos eléctricos primarios o del sistema
excitoconductor (sx. de QT largo, sx. de Brugada, sx. Wolf-Pakinson-White). Algunos de
estos pacientes concentran una historia familiar positiva, que puede ser el único
elemento que permita identificarlos como grupo de riesgo. La FV causa >90% de MS.
PATOGENIA: Existe un descontrol total del corazón, debido a la presencia del fenómeno
R sobre T (R/T), el cual se presenta cuando llega un estimulo en la parte final de la
repolarización. Es de importancia el periodo refractario relativo (PRR), en el si llega un
estimulo que rebase el umbral, el músculo se excita en su zona lábil, se desencadena una
arritmia cardiaca y puede sobrevenir la muerte. (PRA=aunque el estimulo sea muy fuerte
las células del miocardio no se van excitar). Los que tienen más riesgo de sufrir un
fenómeno de R/T son: post IAM, porque se quedan cicatrices y estas son focos
ectópicos que desencadenan estímulos, en los que tienen hace anómalos, en los
hipertensos con miocardiopatía hipertrófica. Solo 2 arritmias descontrolan el corazón
porque producen el fenómeno de R/T (en el ECG, vemos una extrasístole ventricular) y
lo pueden llevar a la muerte; la taquicardia ventricular (ondas regulares >180 lpm, no hay ondas P, complejos anchos) y la fibrilación
ventricular (desorden y ritmo caótico). Ninguna de las 2 produce gasto cardiaco, el primer signo es el sincope, el cerebro se desconecta
(no llega oxigeno, ni glucosa), también se puede presentar una convulsión como manifestación de anoxia. La persistencia de la falta de
riego lleva a la destrucción irreversible y al fallecimiento del individuo. Así tenemos isquemiaTVFVMS. Muerte súbita es el
resultado de; Condición subyacente (cardiopatía coronaria o estructural) + Susceptibilidad individual (inestabilidad eléctrica, no todos
los pacientes con SCA hacen FV solo el 15%) + Evento gatillante (isquemia, trastorno HE o acido-base, fármaco). DIAGNOSTICO: La
muerte súbita cardíaca no tiene unos síntomas previos como tal, pero sí se pueden enumerar una serie de posibles factores que
pueden influir en su aparición: 1. Que el sujeto haya experimentado un episodio de estas características anteriormente o que tenga
familiares con antecedentes 2. Insuficiencia cardiaca. 3. Haber sufrido un infarto de miocardio. 4. Arritmias cardíacas. TRATAMIENTO:
Sólo hay un tratamiento efectivo para frenar la muerte súbita cardíaca y es la desfibrilación precoz. Este procedimiento consiste en
realizar una descarga eléctrica en el corazón, a través de unas palas o parches, con la que se pretende reiniciar la actividad eléctrica del
corazón. En el caso de no disponer de un desfibrilador a mano, o de no saber utilizarlo, se puede realizar una reanimación
cardiopulmonar. También existe la posibilidad de implantar un desfibrilador cuando se detecta, por ejemplo, un síndrome de Brugada.
Que el porcentaje de sangre bombeada por el corazón en cada latido, denominada 'fracción de eyección', sea igual o inferior a un 40
por ciento. PREVENCION PRIMARIA: Identificar los grupos de riesgo; GRUPO DE MAYOR RIESGO DE SUFRIRI UN EPISODIO DE MS: A).
Cardiopatía coronaria sintomática: con IAM en evolución o reciente. Angina inestable. B). Cardiopatía estructural asociada a cardiopatía
coronaria, ICC  FEVI < 35 %. C). Con arritmias ventriculares, Definen riesgo de MS : TVNS inducible o espontánea, TVS inducible o
espontánea. En este grupo siempre debe corregirse isquemia, ya sea médicamente o con procedimientos de revascularización. GRUPO
DE RIESGO INTERMEDIO: A).Cardiopatía estructural adquirida (HTA, valvular, coronaria) o congénita. Estos grupos son heterogéneos, en
general son pacientes cardiópatas sintomáticos o no sintomáticos, pero sin elementos coronarios activos ni arritmias ventriculares de
alto riesgo, se benefician de terapia farmacológica que detiene o aminora progresión de cardiopatía demostrado por evidencia (IECA,
espironolactona, bloqueadores AT2 ,BB). GRUPO BAJO RIESGO: Población general con factores de riesgo cardiovasculares que
determinan finalmente aparición de cardiopatía coronaria, estructural o ambas , bajo riesgo de MS pero por número de afectados son
los que proporcionalmente aportan más casos al año de MS. Fundamental manejar FR : HTA, DM , sedentarismo, tabaquismo,
dislipidemias. Intervenciones terapéuticas tanto farmacológicas como no farmacológicas aplicadas a esta población son de alto impacto
y relativo bajo costo, por lo que hay alta costoefectividad. A nivel poblacional también son aplicables medidas colectivas para evitar que
aparezcan factores de riesgo, sobretodo en población infantil y juvenil (evitar tabaquismo, evitar sedentarismo, alimentación
saludable), como también se vislumbran intervenciones como introducción de elementos dietéticos específicos. TRATAMIENTO: SI el
paciente tiene una TV o FV, desfibrilar, para que el corazón retome su ritmo, se puede dar reanimación cardiovascular pero con CAB no
con ABC (primero compresiones, para genera gasto cardiaco). Solo los grupos de mayor riesgo se benefician en términos de prevención
primaria de DI (desfibrilador implantable), el resto de los pacientes debieran recibir terapia farmacológica que se ha demostrado que
disminuye el riesgo de muerte atribuible a MS, como amiodarona (prolonga el PRA, reduce posibilidad de fenómeno R/T) o BB, estos
últimos además han demostrado que reducen mortalidad global. Esta estrategia es independiente del manejo adecuado del paciente
con cardiopatía demostrada con fármacos que mejoran sobrevida y progresión de enfermedad como IECA, antagonistas de
Angiotensina 2, espironolactona, etc. El desfibrilador implantable es la terapia preferida en una gran proporción de pacientes

CURSO ENARM CMN SIGLO XXI TEL: 36246001 Pharmed Solutions Institute PÁGINA 185
MANUAL DE TRABAJO DEL CURSO ENARM CMN SIGLO XXI
sobrevivientes a MS. El valor de esta aproximación en pacientes con episodio previo de MS es encontrar condiciones de alto riesgo
donde no cabe duda de la indicación de DI y condiciones donde no hay duda de que el paciente no se beneficiará.

CASOS CLINICOS
Un varón de 26 años de edad acudió por palpitaciones paroxísticas acompañadas de sudoración profusa, náusea improductiva y gran
agitación psicomotriz sin relación con el esfuerzo. Presentó un deterioro inesperado de su estado general, con taquicardia ventricular
por torsade de pointes y una parada cardiorrespiratoria recuperada mediante desfibrilación eléctrica cardiaca (tres choques de 200-200
y 360 J de energía respectivamente) y administración farmacológica (2 g de sulfato magnésico en bolo de 2 min y luego perfusión
intravenosa pautada de 500 ml de SSF con 2 mg de sulfato magnésico, junto con 1,5 μg de isoproterenol/min. Una hora después
presenta parada cardiorrespiratoria sin recuperación a la maniobras.

PREGUNTA
Cuál es la alteración del ritmo más probable que ocaciono la muerte.

RESPUESTA
a.- Bloqueo AV de tercer grado.
b.- Taquicardia ventricular.
c.- Fluter auricular.
d.- Bradicardia ventricular.

CASO CLINICO
Varón de 23 años en parada cardiorrespiratoria no recuperada tras protocolo de reanimación cardiopulmonar avanzada, por lo que se
activa código de donante en asistolia, mientras se mantiene el masaje cardíaco con ventilación mecánica y se conecta al paciente a
circulación extracorpórea. En la radiografía de tórax que se realiza en el proceso de validación pulmonar se evidencia masa mediastínica
radioopaca de localización inferior y media.

PREGUNTA
Cuál es la etiología más probable de este caso

RESPUESTA
a.- Aspiracion de contenido gástrico por maniobras.
b.- Neumonia.
c.- Edema agudo pulmonar.
d.- Tromboembolismo.

CASO CLINICO
Un varón de 26 años de edad acudió por palpitaciones paroxísticas acompañadas de sudoración profusa, náusea improductiva y gran
agitación psicomotriz sin relación con el esfuerzo. Presentó un deterioro inesperado de su estado general, con taquicardia ventricular
por torsade de pointes y una parada cardiorrespiratoria recuperada mediante desfibrilación eléctrica cardiaca (tres choques de 200-200
y 360 J de energía respectivamente) y administración farmacológica (2 g de sulfato magnésico en bolo de 2 min y luego perfusión
intravenosa pautada de 500 ml de SSF con 2 mg de sulfato magnésico, junto con 1,5 μg de isoproterenol/min. Una hora después
presenta parada cardiorrespiratoria sin recuperación a la maniobras.

PREGUNTA
Cuál es la alteración del ritmo más probable que ocaciono la muerte.

RESPUESTA
a.- Bloqueo AV de tercer grado.
b.- Taquicardia ventricular.
c.- Fluter auricular.
d.- Bradicardia ventricular.

CASO CLINICO
Se trata de paciente masculino de 61 años de edad el cual ingresa a urgencia por perdida del estado de despierto de forma súbita,
produciendoce caída desde su propia altura. Se iniciaron maniobras de resucitación con recuperación luego de dos descargas con 360 j,
presentando nuevamente perdida del estado de despierto. Tiene como antecedentes diabetes mellitus 2 desde hace 15 años, tratado
con insulina glargina, actualmente en protocolo de dialisis peritoneal con KDOQI 5, y un infarto hace 6 meses.

PREGUNTA
Considerando la presentación del cuadro clínico y el ECG cual es la conducta a seguir?

RESPUESTA
a.- Descarga de 360 j.
b.- Compresiones toraccicas.
c.- Administracion de amiodarona.
d.- Masaje carotideo.

CURSO ENARM CMN SIGLO XXI TEL: 36246001 Pharmed Solutions Institute PÁGINA 186
MANUAL DE TRABAJO DEL CURSO ENARM CMN SIGLO XXI
PREGUNTA
Cual de las siguientes factores de riesgo es el mas grave para predisponer al paciente de presentar el estado actual?

RESPUESTA
a.- Insuficiencia renal crónica.
b.- Infarto al miocardio antiguo.
c.- Diabetes mellitus de larga evolución.
d.- Hipomagnesemia crónica.

CASO CLINICO
Se trata de paciente masculino de 71 años de edad el cual ingresa a urgencias por perdida del estado de despierto, al ingreso se observa
con palidez generalizada, hipotérmico, sin respuesta a estimulos y sin pulso palpable, se activa el código de alerta y se inicia con
compresiones toraccicas por parte de medico de guardia, se realiza dos ciclos de compresiones con leve respuesta y se observa trazo
fibrilar en ECG.

PREGUNTA
Cual es la conducta a seguir inmediata mas adecuada considerando el trazo.

RESPUESTA
a.- Desfibrilar a 360 j.
b.- Reiniciar compresiones.
c.- Administración de amiodarona.
d.- Intubación orotraqueal inmediata.

PREGUNTA
Considerando la etiología de la FA, cual es la mas probable en este paciente?.

RESPUESTA
a.- Cardiopatia coronaria.
b.- Infarto agudo al miocardio.
c.- Hipertrofia ventricular.
d.- Cardiomiopatia dilatada.

PREGUNTA
Luego de 4 ciclos de compresiones y desfibrilación el paciente presenta ritmo sinusal, cual es la razón de que solo con la descarga se
corrija el ritmo?

RESPUESTA
a.- Periodo refractario.
b.- Periodo refractario relativo.
c.- Periodo refractario absoluto.
d.- Periodo refractario con mecanismo de reentrada.

CASO CLINICO
Se trata de paciente masculino de 21 años de edad, es ingresado a urgencias debido a que perdió el estado de alerta mientras jugaba
futbol, no referio síntoma alguno solo se cayo en la cancha, a la exploración se observa lijera palidez de tegumentos, llenado capilar de
2 segundos, pulso lento y débil, FC90, FR 22, TA 110/70 mmHg, responde a ordenes y refiere no recordar lo que paso. Como
antecedente de importancia refiere que ocacionalmente se ha sentido mareado desde los 12 años, pero nunca se había desmayado, de
niño le mencionaron que tenía un soplo que se quitaría posteriormente, cabe mencionar que su padre murió a los 31 años. Se realizo
ECG en urgencias una hora después de su ingreso.

PREGUNTA
Considerando el cuadro clínico y el ECG cual es la etiología mas probable.

RESPUESTA
a.- Miocardiopatia hipertrófica.
b.- Sindrome de QT largo.
c.- Displasia arritmogenica.
d.- Cardiopatia coronaria.

PREGUNTA
Considerando la condición subyacente cuales son las medidas mas adecuada para evitar otro evento del mismo orden potencialmente
mortal.

RESPUESTA
a.- Evitar eventos gatillantes.

CURSO ENARM CMN SIGLO XXI TEL: 36246001 Pharmed Solutions Institute PÁGINA 187
MANUAL DE TRABAJO DEL CURSO ENARM CMN SIGLO XXI
b.- Considera susceptibilidad individual.
c.- Estudiar antecedentes personales.
d.- Modificación de estilo de vida.

CASO CLINICO
Ingresa a urgencias maculino de 21 años de edad el cual se observa ansioso, agitado con actitud alucinada, a la exploración se
encuentra diaforético con palidez generalizada, súbitamente el paciente pierde el estado de alerta y presenta parada cardiaca, se activa
el código de alerta e inicia compresiones toraccicas, los familiares solo refieren que lo golpearon en una fiesta, 10 minutos después
presento los síntomas. Se observo el siguiente trazo del ECG. No hay respuesta, no se detecta pulso, ni constantes vitales. Se descarga
con 360 j, se mantiene soporte por 20 minutos más.

PREGUNTA
No se presive pulso, cual de las siguientes causas es la mas probable en este caso?

RESPUESTA
a.- Neumotorax a tensión.
b.- Sobredosis por drogas.
c.- Tromboembolismo pulmonar.
d.- Hipovolemia.

CASO CLINICO
Varón de 23 años en parada cardiorrespiratoria no recuperada tras protocolo de reanimación cardiopulmonar avanzada, por lo que se
activa código de donante en asistolia, mientras se mantiene el masaje cardíaco con ventilación mecánica y se conecta al paciente a
circulación extracorpórea. En la radiografía de tórax que se realiza en el proceso de validación pulmonar se evidencia masa mediastínica
radioopaca de localización inferior y media. Se aprecian signos de broncoaspirado, por lo que se desestima la extracción pulmonar.

PREGUNTA
Cuál es la etiología más probable de este caso
RESPUESTA
a.- Aspiracion de contenido gástrico por maniobras.
b.- Neumonia.
c.- Edema agudo pulmonar.
d.- Tromboembolismo.

CURSO ENARM CMN SIGLO XXI TEL: 36246001 Pharmed Solutions Institute PÁGINA 188
MANUAL DE TRABAJO DEL CURSO ENARM CMN SIGLO XXI

RCP BASICA Y AVANZADA:


RCP BASICA: Ante una potencial victima siempre ALGORITMO RCP AVANZADA
verificar la ausencia de respuesta (inconsciencia) y 1.-Paro cardiorrespiratorio, iniciar RCP básico
activar inmediatamente el sistema de respuesta
medica de urgencias para asegurar la llegada de un
desfibrilador. Si hay un desfibrilador presente úselo NO
SI 2.- Checar es un 9.- Asistolia/AESP
en este momento. La RCP básica se mantiene hasta la 3.- FV/TVSP
ritmo a desfibrilar?
llegada de un desfibrilador o hasta que reinicie
ventilaciones espontaneas. BUSCAR RESPUESTA: Ante
4.- Dar una descarga (120- 10.- Reiniciar RCP por 5 ciclos.
una potencial victima de Muerte Súbita (MS) se debe 200J-bifasico o 360J- Admon.: adrenalina 1mg IV,
proceder siempre de la misma manera independiente monofasico. Reiniciar RCP repetir cada 3-5 min.
del lugar de los hechos o los recursos disponibles en el Considerar atropina 1mg IV en
5 ciclos asistolia o AESP lenta, repetir
lugar. El primer paso es asegurarse de que la victima cada 3-5 min, hasta 3 dosis
5.- Checar es un
realmente este inconsciente, hay que ponerla en ritmo a desfibrilar? 5 ciclos
posición adecuada para la RCP (decúbito dorsal,
11.- Checar es un
mirando hacia arriba) y tomándola de los hombros y ritmo a desfibrilar?
hablándole en voz alta al oído buscar alguna respuesta 6.- Continuar RCP mientras
(apertura ocular, emisión de sonidos o movimiento). carga desfibrilador. Dar
una nueva descarga. 12.- Si asistolia, ir a cuadro 10.
Debe evitarse realizar movimientos bruscos que Reiniciar RCP. Admon.: Si actividad eléctrica, checar
puedan producir o agravar lesiones presentes, adrenalina 1mg IV repetir pulso: Si no hay ir a cuadro 10.
especialmente de la columna cervical. Si la víctima no cada 3-5 min durante RCP Si hay, iniciar cuidados post-
resucitación.
responde las posibilidades son que este en coma, este 5 ciclos
en paro respiratorio o paro cardiorrespiratorio (PCR). 7.- Checar es un
Para determinarlo se debe continuar con la evaluación ritmo a desfibrilar? 13.- Cuidados post-
en forma ordenada. PEDIR AYUDA: Como la FV es el resucitación
ritmo más frecuente y tratable en la MS en el adulto,
8.- Continuar RCP mientras carga desfibrilador.
se pide ayuda para asegurar la llegada de un Dar una nueva descarga. Reiniciar RCP.
desfibrilador y ayuda médica avanzada. Si el evento Considerar antiarrítmico, amiodarona: 300mg IV,
fue extrahospitalario y hay otras personas presentes, considerar 150 adicional o lidocaína 1mg/kg.
Considerar Magnesio 1-2g IV en torsade de
el reanimador debe identificar a una de ellas y pointes. Luego 5 ciclos de RCP, ir a cuadro 5
solicitarle que active el sistema de respuesta medica
de urgencias indicando que hay una víctima de muerte súbita y la necesidad de un desfibrilador dando la dirección exacta del evento. Si
el reanimador esta solo debe pedir la ayuda el mismo aun si esto implica dejar a la victima solo por algunos segundos. Si el paro es
intrahospitalario, el reanimador debe activar el sistema local de respuesta médica de urgencias (timbre de alarma, código azul,
etc.). : Permeabilizar la vía aérea de la víctima. Durante el PCR la victima pierde complemente el tono muscular,
asociado al efecto de la gravedad la lengua cae hacia atrás ocluyendo la vía aérea, siendo esta la principal causa de obstrucción. La
maniobra más efectiva para permeabilizar la vía aérea es extender la cabeza y elevar del mentón. Si existe o se sospecha lesión de
columna cervical solo se debe realizar la elevación del mentón manteniendo la cabeza en posición neutra. : Con
la vía aérea abierta el reanimador acerca el oído a la nariz de la víctima, mirando hacia el tórax de la misma, Escuchando si hay
respiración y Sintiendo el aire exhalado en un tiempo igual o menor a 10 segundos. Si la victima tiene una respiración adecuada el
reanimador mantiene la vía aérea abierta y mientras espera la llegada de ayuda médica avanzada inicia la búsqueda de la causa y
reevalúa periódicamente a la víctima. La presencia de respiración adecuada implica presencia de circulación. En ocasiones las victimas
de muerte súbita presentan movimientos respiratorios muy superficiales o respiración agónica que no deben ser confundidos con
respiración adecuada y el reanimador debe actuar de la misma forma que si no hubiera respiración. Con la vía aérea abierta el
reanimador debe mirar, escuchar y sentir (MES), buscando la respiración. : Se debe checar ritmo cada 5 ciclos.
El pulso carotideo ha mostrado ser impreciso. Una víc ma potencial de muerte s bita que efec vamente este en PCR y no recibe RCP
básica morirá con seguridad. La FV progresa hacia asistolia rápidamente, disminuyendo la probabilidad de éxito con la desfibrilación en
un 10% con cada minuto de retraso, la RCP básica disminuye el porcentaje a un 4% por cada minuto, aumentando el periodo en el cual
el ritmo es desfibrilable y por lo tanto reversible. Durante los primeros minutos de una muerte súbita la sangre de la victima contiene
suficiente oxigeno para cumplir con la demanda. Además, la demanda de oxigeno es menor ya que la circulación también esta
disminuida hasta 1/3 parte de lo normal. El problema principal es entonces falta de circulación de sangre más que falta de oxigenación
de la misma. Por lo tanto, si la persona esta inconsciente y sin respiración el reanimador debe iniciar la RCP con 30 compresiones
torácicas externas seguidas de 2 ventilaciones artificiales. Una excepción para el uso precoz de la desfibrilación es la presencia de un
PCR prolongado (>5 minutos) donde la RCP básica, aumentan el éxito de la desfibrilación. Existe un pequeño porcentaje de pacientes
que tienen una muerte súbita de etiología hipoxica (obstrucción de vía aérea por cuerpo extraño) en quienes si se tiene la sospecha o
certeza se podría privilegiar las ventilaciones artificiales y empezar la RCP con estas y continuar con compresiones torácicas
externas. : Las compresiones torácicas se realizan colocando el talón de una mano en el centro del tórax
entre ambas tetillas con la otra mano encima y los dedos entrelazados. Con los codos completamente extendidos se inician las
compresiones empujando fuerte y rápido. El mayor determinante de éxito con la desfibrilación es la presión de perfusión coronaria. Las
compresiones torácicas externas efectivas logran entregar una pequeña pero critica cantidad de oxigeno al cerebro y corazón,
suficiente para mantener una presión de perfusión coronaria. Compresión torácica externa efectiva debe cumplir con las siguientes

CURSO ENARM CMN SIGLO XXI TEL: 36246001 Pharmed Solutions Institute PÁGINA 189
MANUAL DE TRABAJO DEL CURSO ENARM CMN SIGLO XXI
condiciones: 1.- Frecuencia >100 x min. 2. Profundidad entre 4-5 cm. 3.- Equilibrar el tiempo de compresión con el de descompresión
del tórax (50 y 50%). 4.- Permitir el retorno del tórax a su expansión total durante la fase de descompresión. 5.- Minimizar las
interrupciones en la compresión torácica externa. Las 30 compresiones torácicas son seguidas de 2 ventilaciones artificiales de 1
segundo cada una (la misma para uno o dos reanimadores). : Puede ser administrada con distintas técnicas: 1.- Boca a
boca: Abriendo la vía aérea el reanimador toma una respiración normal y a continuación sella su boca alrededor de la de la victima e
insufla lo suficiente como para elevar el tórax de la víctima. 2.- Ventilación con barreras de protección: Estas son dispositivos plásticos
con una válvula unidireccional que en teoría evitan la transmisión de enfermedades infectocontagiosas (no demostrado). 3.- Boca a
nariz: Se utiliza cuando no es imposible ventilar a través de la boca (lesiones bucales, imposibilidad de abrir la boca). 4.- Mascara-
válvula-bolsa: Logra oxigenación y ventilación efectivas pero requiere de entrenamiento y práctica frecuente. En los primeros minutos
de la RCP la máscara-válvula-bolsa es igual de efectiva y segura que un tubo endotraqueal. La intubación endotraqueal no tiene lugar en
la RCP básica. Las compresiones torácicas externas agotan rápidamente al reanimador y cuando esto sucede disminuye notablemente
su efec vidad. Es así que si hay dos o más reanimadores presentes deben rotarse las compresiones torácicas cada 2 minutos para
garantizar que estas sean efectivas. Después de la desfibrilación (200J, 200-300J, 360J si es bifásico o cargas fijas de 360J si es
monofásico) las compresiones torácicas y ventilaciones artificiales se mantienen por 2 minutos. Si la desfibrilación fue exitosa y se logro
un ritmo de perfusión se suspende la RCP básica y se inicia la avanzada según necesidad. En cambio, si la desfibrilación no fue exitosa se
siguen alternando ciclos de compresión torácica con ventilación artificial por 2 minutos y nuevos análisis del ritmo por el DEA y
descargas si este las indica. RCP AVANZADO: A (Airway): asegurar la vía aérea: En el pasado se prefería la intubación oro-traqueal
(IOT), pero la incidencia de complicaciones es muy alta cuando es realizada por inexpertos. Las alternativas serian el Combitube
esófago-traqueal y la mascara laríngea, pero no están disponibles. La IOT permite mantener permeable la vía aérea, aspirar
secreciones, administrar O2 en altas concentraciones, es una vía alternativa para la administración de medicación y cuando se insufla el
manguito, protege la vía aérea de la aspiración. Se debe minimizar el tiempo de interrupción de las compresiones (10seg). Si no fue
exitosa, asegurar un intervalo adecuado de compresiones y ventilaciones. Si el paciente tiene un ritmo con pulso, controlar con
saturometria. Complicaciones de OIT: ruptura de dientes laceración traqueofaríngea, ruptura traqueal, perforación faringoesofagica,
aspiración de contenido gástrico, intubación bronquial. B (Breathing): comprobación de la localización del tubo a través de métodos
clínicos (visualización de expansión torácica y auscultación) y complementarios (detectores de CO2). C (Circulation): Obtener un acceso
venoso de grueso calibre (antecubital), conectar a monitor y administrar drogas según ritmo. Cuatro ritmos producen el paro cardiaco:
la FV, TV sin pulso, asistolia y AESP. Las drogas se administran en bolo seguidas de 20 ml de solución fisiológica y elevar el brazo.
Algunas pueden administrarse por el tubo endotraqueal; atropina, lidocaína, adrenalina y naloxona, pero a 2 veces la dosis y seguida
por 10ml de SF y 3-4 ventilaciones. Los fluidos deben ser administrados cuando se sospecha hipovolemia. Administrar bicarbonato solo
si hiperkalemia persistente. Diagnósticos diferenciales: considerar las posibles causas descompensantes o que complican la
resucitación. Regla de las 5Hs (hipovolemia, hipoxia, H+ (acidosis), Hiper/hipocalemia, hipotermia) y las 5Ts (tóxicos, taponamiento
cardiaco, neumotórax a tensión, trombosis coronaria, TEP).

CASO CLINICO
Varón de 48 años, acude a consulta al hospital y cuando se dirige a recabar su ficha de consulta cae al suelo inconsciente,
Circunstancialmente usted pasa por el lugar y se dispone a brindar auxilio e inicia los pasos del soporte vital básico.

PREGUNTA
Cual es la conducta a seguir inmediatamente?

RESPUESTA
a.- Iniciar compresiones.
b.- Iniciar ventilación.
c.- Activar sistema de emergencia.
d.- Verificar pulso.

CASO CLINICO
Paciente de 47 años, con antecedente de un episodio de fibrilación auricular en que no tuvo mayor estudio y revirtió a ritmo sinusal
(RS) espontáneamente. Sin terapia posterior y asintomático. Estando en reposo, presenta paro cardíaco que fue atendido
inmediatamente por familiares con maniobras básicas de resucitación cardiopulmonar, siendo trasladado en un lapso de 5 a 10 min al
Hospital local donde al ingreso se observa actividad eléctrica sin pulso.

PREGUNTA
Cual es su conducta a seguir inmediatamente?

RESPUESTA
RESPUESTA
a.- Iniciar compresiones.
b.- Iniciar ventilación.
c.- Activar sistema de emergencia.
d.- Verificar pulso.

CURSO ENARM CMN SIGLO XXI TEL: 36246001 Pharmed Solutions Institute PÁGINA 190
MANUAL DE TRABAJO DEL CURSO ENARM CMN SIGLO XXI
ESTADO DE CHOQUE:
CIENCIAS BASICAS: Definición: Reducción de la perfusión tisular sistémica con decremento de la disponibilidad de O2, provocando
hipoxia celular. Estado metabólico en el cual las demandas energéticas celulares de oxigeno, exceden el aporte. Disfunción celular,
orgánica y sistémica. SALUD PUBLICA: Mortalidad mayor al 20%. Corresponde al 1% de las atenciones en el servicio de urgencias. En el
shock anafiláctico cerca de 60% se muere por obstrucción de la vía aérea superior, el resto por hipotensión y arritmias. PATOGENIA:
Varios factores influyen en el desarrollo como: desproporción entre el volumen circulante y el lecho vascular. Trastorno en la
circulación capilar. Trastorno en el intercambio gaseoso. Hipoxia de los tejidos y acidosis metabólica. A 8ml/kg/min se inicia la isquemia
celular
CLASIFICACION DE CHOQUE
HIPOVOLEMICO CARDIOGENICO OBSTRUCTIVO DISTRIBUTIVO
DEFINICION Estado patológico asociado TAS <90mmHg (< de 30 por Obstrucción mecánica al Estado de emergencia, para evitar daño irreversibles.
a procesos cuyo debajo de la basal si se flujo de sangre que Existe una vasodilatación, que conlleva a una
denominador es la hipo conoce) por un periodo de más ocasiona una poscarga hipoperfusión tisular generalizada, porque hay poca
perfusión con hipoxia de 30 min., que no responde a disminuida y una precarga. La falta de respuesta a manejo hídrico y/o
celular, secundaria a la la administración de líquidos y inadecuada perfusión vasopresores, establece el diagnostico definitivo de
perdida de volumen que es secundaria a una tisular choque distributivo
circulante, intra o disfunción cardiaca
extravascular
SIGNOS Hipotensión arterial, Hipotensión arterial, Por lo general hipertérmico, taquicardico (>120 lpm),
SINTOMAS hipotermia, taquicardia, congestión pulmonar, taquipneico e hipotenso.
palidez de tegumentos ingurgitación yugular, ritmo de En el neurogénico dependiendo nivel de lesión puede
galope o tercer ruido, haber hemiplejia superior con incapacidad para
congestión hepática, pulso movilización y sensibilidad de extremidades, y en
paradójico y/o disminución de ocasiones insuficiencia respiratoria. Daño medular
la transmisión de los ruidos bajo, hemiplejia inferior con falta de sensibilidad en
cardiacos extremidades e incontinencia de esfínteres.
Relacionados con isquemia
miocárdica aguda: Dolor
torácico, disnea, diaforesis,
nausea, vomito
TIPOS Hemorrágico Importante: es la máxima Séptico: hipotensión inducida por sepsis que no
No hemorrágico: expresión de la insuficiencia responde a tratamiento a manejo de líquidos o
3er espacio cardiaca por disfunción vasopresores
(pancreatitis, vasculitis, ventricular, es un síndrome Anafiláctico: reacción aguda y generalizada de gran
quemados) clínico que se acompaña de severidad que puede ser potencialmente mortal.
Perdidas disminución persistente y Neurogénico: Alteración hemodinámica por
gastrointestinales progresiva del GC, interrupción de la vía simpática, lo que genera una
Perdidas urinarias (DM acompañado de estado de vasodilatación permanente, antecedente de trauma,
insípida) choque e hipo perfusión con compromiso del SNC, el GC puede estar
tisular, llevado a disfunción comprometido o no, el más común es el traumatismo
celular y falla multiorganica medular
CAUSAS Se da como consecuencia El más común es el choque Lo más frecuente Puede ser inducido por infecciones, reacciones
de la perdida súbita de inducido por IAM, también por Tromboembolia alérgicas o inflamatorias severas, el más común es el
volumen intravascular contusión miocárdica, pulmonar, otras séptico.
miocardiopatía, rotura de neumotórax a tención, El anafiláctico pude ser causado por: antibióticos,
cuerdas tendinosas, tamponade cardiaco, analgésicos, alimentos, picadura de insectos, medios
comunicación IV, aneurisma estenosis mitral o de contraste.
ventricular, arritmias. Hasta en aortica aguda, El Neurogénico puede ser causado por: accidentes
75-80% la disfunción proviene pericarditis obstructiva vehiculares (40%), caídas, violencia, niños durante
del VI deporte o actividades recreativas.
RVP Aumentado (>1500 Aumentado Normal Disminuido
dinas/min/cm-5)
GC Disminuido (<5 L/min) Disminuido Disminuido Aumentado
PVC Disminuida (<8 cm de H2O) Aumentada (>15 cm de H2O) Normal, aumentada o Normal o disminuida
dismi
PCP Baja (<8 mmHg) Aumentada (>15 mmHg) Normal, aumentada o Disminuida
dismi

Es importante diferenciar entre shock séptico y sepsis. Síndrome de respuesta inflamatoria sistémica (SRIS): Cuando se presenta dos
más de los siguientes signos; 1. Distermia: Temperatura corporal >38° 0 <35°. 2 Taquicardia: >90 por minuto. 3. Taquipnea: >20 por
minuto o PaCO2 <32mmHg. Leucocitos: Mas de 12,000 o menos de 4,000 con 10% de bandas. Sepsis: Cuando un paciente tiene SRIS y
un foco infeccioso evidente, con cultivo hemático positivo o negativo. Sepsis severa: Hipotensión inducida por sepsis que responde al
manejo de líquidos y/o vasopresores. ESTRATEGIAS DE TRATAMIENTO: Accesos vasculares; importantes para la administración de
líquidos y elementos de vigilancia. Accesos venosos periféricos, son la primera opción para tratamiento de choque hipovolémico
(catéteres 14,16 o 18), sitios preferidos safena externa, yugular externa, pliegue de ambos antebrazos. Los catéteres cortos no deberán
emplearse en venas centrales a menos que sea indispensable. Accesos venosos centrales, indispensables en todas las formas de
choque, las vías preferidas de acceso central son: yugular interna y subclavia. Métodos de vigilancia, oxigenación tisular, catéter de
presión venosa central (PVC) intraarterial en aurícula derecha, para vigilancia de estado nutricional BH, QS, orina de 24 hrs, vigilancia
respiratoria, gasometría arterial. Apoyo hemodinámica: orientado a la optimización de la oxigenación, mediante fármacos; 1. Terapia
inotrópica: corroborar una adecuada PVC y PCP antes de iniciar tratamiento inotrópico. Una dosis inicial de dobutamina (5-10
microgramos/kg/min), recomendación de inicio en presencia de choque cardiogénico. 2. Terapia vasopresora: de primera línea
dopamina cuando hay RVP disminuidas y GC aumentado, cuando no existe respuesta con dosis elevadas de dopamina (>20
microgramos/kg/min), se recomienda norepinefrina o epinefrina. La noradrenalina es un medicamento vasoconstrictor potente, no
general tantas arritmias, es de primera elección en choque septico. La vasopresina puede ser útil en pacientes refractarios a

CURSO ENARM CMN SIGLO XXI TEL: 36246001 Pharmed Solutions Institute PÁGINA 191
MANUAL DE TRABAJO DEL CURSO ENARM CMN SIGLO XXI
norepinefrina, y se recomienda en el choque en fase
de vasodilatación (choque séptico). La adrenalina es
Manifestaciones de Manifestaciones de bajo Inestabilidad
de primera elección en choque anafiláctico, en bolos ICC gasto hemodinámica
subcutáneos 3. Terapia vasodilatadora: su uso debe Disnea, polipnea, Palidez, oliguria, llenado Hipotensión arterial,
ser sustentado en monitoreo invasivo avanzado, que estertores capilar prolongado, reacción PAS <90mmHg, IC < 2.2
pulmonares, plétora adrenérgica, piloerección, L/min/m2, PCP
provea medición de RVS. Líquidos y fluidos: yugular, S3 diaforesis, cianosis periférica >15mmHg
cristaloides; las mas empleadas solución salina
isotónica y Hartman, clínicamente puede esperarse
de 100-200ml de expansión, después de infundir 1 DIAGNOSTICO CHOQUE CARDIOGENICO
CLASIFICACION DE CHOQUE
litro de cristaloides isotónicos. Las soluciones HIPOVOLEMICO HEMORRAGICO
hipertónicas tienen un potencial benéfico como GRADO I II III IV
incremento en la contractilidad miocárdica y PERDIDA/ML 750 750-1500 1500-2000 >2000
vasodilatación precapilar; sin embargo, pueden PERDIDA % 15% 15-30% 30-40% >40%
FC <100 >100 >120 >140
conducir a estados hiperosmolares con facilidad.
FR 14-20 20-30 30-40 >40
Coloides; en México se cuenta con 4 como albumina T/A NL NL Disminuido Disminuido
(frasco de 50cc con concentración al 20%), gelatina P. pulso Normal Disminuido Disminuido Disminuido
(haemacel), dextranes, almidón (hidroximetilalmidon DIURESIS >30ml/h 20-30 5-15 0
al 10%). Un litro de albumina al 5%, produce una Edo. mental Ansiedad Ansiedad Confusión Letargo
Rest. liquida Cristaloides Cristaloides Crista +sangre Crista +sangre
expansión de plasma de 500-1000 ml. Un litro de
almidón expande 700ml. Tanto gelatinas como almidones son inductores de diátesis hemorrágica, en forma dosis dependiente. El
dextran compromete dramáticamente la hemostasis, la gelatina un impacto moderado, aunque la agregación plaquetaria se ve
moderadamente afectada. Sangre y hemoderivados: no se ha demostrado beneficio en la perfusión tisular, al transfundir pacientes en
estado de choque con niveles de hemoglobina de 8-10mg/dl. Las siguientes condiciones clínicas podrán beneficiarse de
hemotransfusion, taquicardia importante, desaturación venosa mezclada importante, disfunción cardiaca, enfermedad arterial
coronaria coexistente, acidosis láctica persistente, incapacidad otros marcadores de hipoperfusión celular. No existe un nivel de
hemoglobina que indique hemotransfucion con el sustento de mejorar el aporte sanguíneo a los tejidos. Las complicaciones de
hemotransfusion masiva, lleva a mayor riesgo de infecciones y reacciones anafilácticas, puede llevar a SIRA, aquí se llama TRALI, puede
dar a partir de las 6 hrs de trasnfundido. Apoyo ventilatorio; en todo momento asegurase del adecuado funcionamiento de la vía aérea,
siendo la permeabilidad e integridad de la misma una condición indispensable para cumplir con la meta de oxigenación sistémica. Los
elementos que pueden orientar para el empleo de asistencia en la ventilación son: administrar puntas nasales, mascara facial, cánula
faríngea o mascarilla con reservorio en caso de contarse con ella, en cualquier situación clínica que sugiera estado de choque. Criterios
recomendados para decidir intubación endotraqueal, con manejo de ventilador mecánico en un paciente en estado de choque;
hipercarbia >32mmHg, taquipnea >20 rpm, cianosis distal con alguno de los elementos anteriores, acidosis respiratoria con pH=7.2,
paciente con relación PaO2/FiO2 <150, paciente con sospecha de quemadura de vía aérea. MARCADORES SERICOS DE CHOQUE:
Lactato; es un reflejo de hipo perfusión tisular, producto de la glicolisis anaerobia, valores por arriba de 4mmol/l, se asocia con un 90%
de mortalidad, por arriba de 2.5 mmol/l alta incidencia de falla orgánica múltiple (FOM), su medición nos sirve como factor pronostico.
Déficit de base; aproximación global del grado de acidosis e indirectamente el grado de perfusión tisular, nos orienta sobre la severidad
de choque: a) 2-5 leve, b)-6 a -14 moderado, c) mayor a -15.

CASO CLINICO
Se trata de paciente masculino de 23 años de edad el cual ingresa a urgencias presentando orificio de entrada en flanco derecho, por
arma de fuego, sin orificio de salida, a la exploración física se observa con FC 125 lpm, llenado capilar lento, FR 31 rpm, se observa
ansioso con tendencia a la confusión, su gasto urinario fue de 20 ml/h.

PREGUNTA
Considerando las manifestaciones clínicas observadas asi como las variables vitales, en que clase se encuentra el paciente?

RESPUESTA
a.- Clase I.
b.- Clase II.
c.- Clase III.
d.- Clase IV.

PREGUNTA
Cual es la conducta para mantener el volumen circulante mas adecuada?

RESPUESTA
a.- Solución salina 10 ml/kg/hora.
b.- Solución hartman 15ml/Kg/hora.
c.- Crioprecipitados.
d.- Concentrado eritrocitario y cristaloide.

PREGUNTA
Una hora despues, el paciente presenta los siguientes constantes vitales, FC 105 lpm, FR 21 rpm, gasto urinario de 35 ml/h, se
encuentra ansioso, pero no confundido, la hemoglobina fue de 12 y hematocrito de 37, plaquetas de 120,000, asi como DHL 312, AST
435, tiempos de coagulación levemente incrementados, cual es la conducta a seguir en este momento?

CURSO ENARM CMN SIGLO XXI TEL: 36246001 Pharmed Solutions Institute PÁGINA 192
MANUAL DE TRABAJO DEL CURSO ENARM CMN SIGLO XXI
RESPUESTA
a.- Mantener observación estrecha.
b.- Preparar para laparatomia exploratoria.
c.- Incrementar volumen circulante con paquete globular.
d.- Vasopresores, volumen y LAPE.

PREGUNTA
Se realizo LAPE con reparación hepática con buena respuesta, fue necesario crioprecipitados, plaquetas y concentrado eritrocitario, se
administro 3 gramos de gluconato de calcio, cual es la razón de esta indicación?

RESPUESTA
a.- Alteración de coagulación.
b.- Mantener estable la membrana.
c.- Evitar arritmias.
d.- Disminuir la adhesividad plaquetaria.

CASO CLINICO
Se trata de paciente femenino de 48 años de edad, la cual fue ingresada posterior a accidente automovilístico y volcadura, era copiloto
y fue la única sobreviviente, se observa confundida, desorientada, agitada, se presenta diaforesis, con palidez de mucosas, su llenado
capilar lento, FC de 138 lpm, FR 34 rpm, TA 100/65 mmHg, minutos después se estableció un Glasgow de 8, distensión abdominal e
inestabilidad toracicca, su gasto urinario fue de 20 ml/h, Campos pulmonares con matidez a la percusión bibasal, no hay ingurgitación
yugular, pero hay equimosis importante sobre area del cinturón de seguridad, en la radiografia portátil se observo fractura de 5
costillas del lado izquierdo.

PREGUNTA
Cual es la medida inmediata mas adecuada en este momento?

RESPUESTA
a.- Intubación endotraqueal.
b.- Solución hartman, vasopresores.
c.- Solucion salina hipertónica.
d.- Sedacion y relajación muscular.

PREGUNTA
Posterior al decisión anterior se administra 3 paquetes de concentrados eritrocitarios, 3 paquetes de plasma fresco y 2 litro de solución
hartman, sin embargo el gasto urinario es de 25 ml/hr, la tensión arterial se mantiene en 95/60 mmHg durante la primera hora de
estancia, cual es la conducta a seguir mas adecuada?

RESPUESTA
a.- Albumina y expansores.
b.- Infusion de dopamina 5 µg/Kg/min.
c.- Infusion de dobutamina 10 µg/kg/min.
d.- Crioprecipitados y gluconato de calcio.

PREGUNTA
Cual es el objetivo mas importante de la conducta farmacológica tomada anteriormente?

REPUESTA
a.- Mantener el gasto urinario.
b.- Incrementar la fracción de eyección.
c.- Mantener la tensión arterial permisible.
d.- Favorecer el inotropismo.

PREGUNTA
El paciente presenta estabilidad en la tensión arterial 95/70 mmHg y gasto urinario de 35 ml/hr, se prepara para laparatoria
exploratoria, tomando en cuenta la decisión farmacológica anterior, cual de los siguiente efectos no favorece al caso?

RESPUESTA
a.- Incremento mayor del 25 % sobre la tensión arterial basal.
b.- Aumento del flujo sanguíneo renal.
c.- Disminución de la resistencia vascular periférica.
d.- Disminución de la demanda de O2 del miocardio.

CASO CLINICO
Se trata de paciente masculino de 19 años de edad el cual ingresa a urgencias posterior a accidente automovilístico, refiere cefalea y
dolor lumbar leve, permaneció en observación, se coloco catéter periférico 18 fr y solución glucosada al 5 % para 8 hrs, a las 4 horas

CURSO ENARM CMN SIGLO XXI TEL: 36246001 Pharmed Solutions Institute PÁGINA 193
MANUAL DE TRABAJO DEL CURSO ENARM CMN SIGLO XXI
posterior al ingreso refiere ansiedad, sensación de mareo, a la EF se observa palidez de tegumentos, diaforesis, torax sin datos
patológicos, abdomen con ruidos peristálticos disminuidos, SV con TA 100/70 mmHg, FC 105 lpm, FR 32 rpm, no se apresia compromiso
oseo.

PREGUNTA
Se realizan realiza QS, BH, RX de torax y abdomen, Hb 10.1, Hto 30, Rx de abdomen con niveles hidroaereos, imagen despulida, cual es
la conducta a seguir más adecuada?

RESPUESTA
a.- Solución hartman 10 ml/kg/hora.
b.- Solución salina 0.45 % 10 ml/kg/hora.
c.- Solución mixta 10 ml/kg/hora.
d.- Solución glucosada al 10 % 10 ml/Kg/hora.

PREGUNTA
Posteriomente se administra 3 concetrados eritrocitarios y 2 paquetes de plasma fresco, la tensión arterial se mantiene estable en
100/70 mmHg sin embargo la urésis fue de 40 ml/hora, se observa auscencia de ruidos peristálticos, se observa estado de conciencia
adecuado, alerta y tranquilo, cual no es un objetivo primordial en este momento del caso?

RESPUESTA
a.- Elevar la tensión arterial media por arriba de 95 mmHg.
b.- Manterne la hemostasia con gluconato de calcio.
c.- Estabilizar hemostasis por sospecha de hemorragia.
d.- Mantener un adecuado volumen urinario.

PREGUNTA
Cual es el origen menos probable de la perdida del volumen circulante del caso actual?

RESPUESTA
a.- Contusión cardiaca.
b.- Rotura esplenica.
c.- Hemorragia hepática.
d.- Hemorragia retroperitoneal.

PREGUNTA
Se realizo tomografía donde se confirmo hemorragia retroperitoneal y lactato elevado, cual de las siguientes complicaciones de alto
volumen de reposicion es menos frecuente?

RESPUESTA
a.- Acidosis hipernatremica.
b.- Síndrome de fuga capilar.
c.- Perdida de la estabilidad de un coagulo
d.- Disminución de la precarga con aumento de FE.

PREGUNTA
Previo manejo quirúrgico se mantiene al paciente en hipotensión permisiva, cual es el parámetro para este paciente?

RESPUESTA
a.- 100 mmHg.
b.- 90 mmHg.
c.- 80 mmHg.
d.- 70 mmHg.

CASO CLINICO
Mujer de 34 años sin antecedentes patológicos, no hábitos tóxicos, no alergias a fármacos conocidas. Presentó cefalea tensional en
tratamiento con ácido acetilsalicílico con episodios de urticaria aguda en relación con el consumo de marisco, tratado y remitido hace
24 hrs. 4 horas después apareció dolor faringeo, tumefacción facial, de manos y brazos, con disminución de la diuresis. A las pocas
horas de la hospitalización presentó una alteración hemodinámica con taquicardia sinusal a 140 pulsaciones por minuto e hipotensión.

PREGUNTA
Considerando el cuadro clínico cual es la conducta inmediata a seguir, la paciente se encentra con vías de acceso venoso?

RESPUESTA
a.- Administración de volumen con cristaloides.
b.- Administracion de aminas vasoactivas.
c.- Colocación de catéter largo.

CURSO ENARM CMN SIGLO XXI TEL: 36246001 Pharmed Solutions Institute PÁGINA 194
MANUAL DE TRABAJO DEL CURSO ENARM CMN SIGLO XXI
d.- Administración de epinefrina.

PREGUNTA
Posteriormente se deterioró el nivel de conciencia y se produjo un paro respiratorio que precisó intubación y ventilación mecánica. Se
consiguió la estabilización hemodinámica con dopamina y expansores de volumen, crioprecipitados y plasma fresco, cual es la
complicación más probable?

RESPUESTA
a.- Coagulacion por hemodilución.
b.- Sindrome compartimental.
c.- Coagulacion intravascular por consumo.
d.- Coagulopatia dilucional.

PREGUNTA
Considerando el cuadro clínico y la evolución del caso, cual de las siguientes causas es la menos probable?

RESPUESTA
a.- Reacciones a fármacos.
b.- Reacciones a alimentos.
c.- Reacciones transfusionales.
d.- Reacciones a antígenos inhalados.

CASO CLINICO
Paciente varón de 57 años con larga evolución en cuidados intensivos por síndrome de disfunción multiorgánica (SDMO) y síndrome de
respuesta inflamatoria sistémica (SRIS) secundarios a proceso postquirúrgico. Se trata de un paciente gastrectomizado, que desarrolló
en el post-operatorio una doble fístula pleural y digestiva con colocación de prótesis endoscópica y posterior reapertura de fístula. Ha
tenido simultáneamente diversos procesos infecciosos sistémicos (empiema, peritonitis, herida quirúrgica y respiratoria) tratados
empíricamente o mediante antibiograma. Además ha presentado repercusión orgánica persistente con necesidad de soporte
vasopresor para control hemodinámico y respiratorio sin posibilidad de avanzar.

PREGUNTA
Considerando las características actuales que se observan en el paciente que tipo de choque se observa actualmente?

RESPUESTA
a.- Choque distributivo.
c.- Choque restrictivo.
d.- Choque séptico.
c.- Choque oculto.

PREGUNTA
El día 11 de estancia en cuidados intensivos se introduce tratamiento con linezolid por vía intravenosa, 600mg/12h por aislamiento de
Enterococcus faecium resistente a meticilina, sensible a linezolid, en el empiema pleural bilateral. El resto de tratamiento es:
enoxaparina 40mg/24h (11 días), propofol 300mg/h y meperidina 15mg/h en perfusión continua (como sedoanalgesia) (10 días),
imipenem 1g/8h (10 días), amikacina 1,2g/24h (5 días). A las 48 h del inicio de linezolid se observan crisis comiciales en relación con
movimientos tonicoclónicos preferentemente de extremidades superiores y tronco sin llegar a generalizarse, transitorios (pocos
minutos de duración) y repetidos a lo largo del día. Considerando la evolución y los nuevos síntomas cual es su impresión?

RESPUESTA
a.- Choque anafiláctico.
b.- Reaccion adversa a medicamento.
c.- Choque séptico.
d.- Sindrome serotoninergico maligno.

PREGUNTA
Se observa también hiperreflexia, discreta acidosis metabólica y empeoramiento grave de la fiebre, leucocitosis, marcadores de
inflamación y estado hemodinámico (hipotensión). Ante la sospecha de RAM se sustituye meperidino por fentanilo y se añade
midazolam, 10mg/h, en perfusión continua al esquema de sedoanalgesia para la mejora del cuadro y noradrenalina para el soporte
hemodinámico. Considerando las variable vitales, hipotensión refractaria cual de las siguientes medidas es mas adecuada?

RESPUESTA
a.- Incrementar aminas vasoactiva.
b.- Hemodialisis.
c.- Retirar linezolid y administrar metilprednizolona.
d.- Concentrado eritocitario y plasma fresco congelado.

PREGUNTA

CURSO ENARM CMN SIGLO XXI TEL: 36246001 Pharmed Solutions Institute PÁGINA 195
MANUAL DE TRABAJO DEL CURSO ENARM CMN SIGLO XXI
Los cultivos microbiológicos fueron negativas con la excepción de los drenajes torácico y abdominal, en los que se aisló Candida spp. Se
administro anfotericina B liposomal en dosis de 4mg/kg/día. Linezolid fue también suspendido y se pautó daptomicina, 350mg/24h.
Tras el cambio de tratamiento las mioclonías e hiperreflexia mejoraron antes de 24 h, hasta desaparecer por completo al segundo día.
La fiebre, leucocitosis e hipotensión mejoraron también progresivamente, aunque la acidosis metabólica persistió. Cual es la
complicación mas probable?

RESPUESTA
a.- Dudoso por acidosis metabolica.
b.- Coagulopatia por dilución.
c.- Alergia al fármaco.
d.- Falla organica multiple.

PREGUNTA
El diagnóstico de síndrome serotoninérgico suele ser difícil de realizar por la presencia de síntomas inespecíficos y comunes a otras
entidades. Esta problemática puede ser todavía mayor en el caso de un paciente crítico y polimedicado como el que nos ocupa, donde
los síntomas pueden ser atribuidos a múltiples factores de la enfermedad o del abordaje farmacológico. Cual es la base principal para
diagnosticar este caso?

RESPUESTA
a.- Inicio brusco.
b.- Recuperación tras suspensión del fármaco.
c.- La presencia de fiebre, hiperreflexia y acidosis metabolica.
d.- Criterios diagnósticos de Sternbach y Dunkley.

CASO CLINICO
Se trata de un varón de 74 años que ingresó por fiebre y deterioro del estado general de meses de evolución. Como antecedentes
personales, cabe destacar gastritis crónica atrófica, cardiopatía reumática con afectación aórtica y mitral y fibrilación auricular con
tratamiento anticoagulante. En la exploración física no había alteraciones significativas. En la analítica, a reseñar una creatinina de 4,17
mg/dL no conocida y un INR de 7. En los hemocultivos realizados, crecieron en 2/3 L. monocytogenes. En el estudio de la bacteriemia,
se solicitó un ecocardiograma transtorácico que puso de manifiesto una verruga en la válvula aórtica, con fracción de eyección
deprimida, KDOQ III, cual es la conducta farmacológica mas adecuada a seguir?

PREGUNTA
a.- Administrar ampicilina y vancomicina.
b.- Administrar ampicilina y gentamicina ajustada.
c.- Administrar imipenem y amikacina.
d.- Administrar ceftriaxona e imipenem.

PREGUNTA
Se completó el estudio con una RMN craneal, en la que se observaron múltiples infartos cerebrales, y con una ecografía abdominal en
la que se apreció un hígado de estasis. La evolución fue mala, desarrollando insuficiencia cardiaca por lo que se decidió realizar cirugía
cardiaca el 7º día. En la intervención, se encontraron vegetaciones y abscesos que afectaban tanto a la válvula aórtica como a la mitral,
el paciente presento hipotensión refractaria, sin respuesta a aminas vasopresoras, hipotermina, piel marmórea. Cual es el tipo de
choque mas probable en este momento del caso?

RESPUESTA
a.- Choque cardiogenico.
b.- Choque séptico.
c.- Choque restrictivo.
d.- Choque oculto.

CHOQUE SEPTICO
Femenino de 63 años, con antecedentes de hipertensión arterial (HTA), diabetes mellitus (DM) tipo II, dislipidemia, infecciones urinarias
y cólicos nefríticos de repetición, que presenta un cuadro de cinco meses de evolución de dolor abdominal de predominio en fosa ilíaca
izquierda, vómitos y fiebre intermitente. Acude en diversas ocasiones a Urgencias, siendo diagnosticada de infección urinaria y cólico
renoureteral. Recibiendo manejo farmacológico empirico a dosis y tiempo adecuado, debido a la persistencia del dolor se realiza
ecografía abdominal, objetivándose nódulo en segmento IV hepático y quistes renales corticales.

PREGUNTA
Ante dichos hallazgos cual es la conducta mas adecuada a seguir?

RESPUESTA
a.- Bh completa, QS, EGO, Hemocultivo y urocultivo.
b.- TAC de tórax, abdomen y pelvis con contraste.
c.- IRM-F torax y abdomen con marcadores inflamatorios.
d.- Biopsia hepática y renal.

CURSO ENARM CMN SIGLO XXI TEL: 36246001 Pharmed Solutions Institute PÁGINA 196
MANUAL DE TRABAJO DEL CURSO ENARM CMN SIGLO XXI
PREGUNTA
A los dos meses acude de nuevo a Urgencias por dolor intenso en hipocondrio derecho y mal estado general. En la EF destaca signo de
Murphy positivo e hipotensión severa, creatinina 3,68mg/dl, urea 96mg/dl, transaminasa glutámico oxalacética (GOT) 55 UI/l,
transaminasa glutámico-pirúvica (GPT) 92 UI/l, proteína C reactiva (PCR) 451mg/dl. Se realiza TAC de abdomen sin contraste: probable
colecistitis, elevación de hemidiafragma derecho, atelectasia basal derecha, hemangiomas hepáticos, granuloma esplénico y quistes
renales. La paciente presenta deterioro significativo hemodinamico, con obnulización, hipotensión, taquicardia, taquipneica, mal
perfundida con signos de abdomen agudo cual es la conducta a seguir?

RESPUESTA
a.- Preparar para LAPE.
b.- Iniciar dopamina.
c.- Iniciar dobutamina.
d.- Ingresar a UCI.

PREGUNTA
Se realizan laboratorios con los siguientes resultados: hemoglobina 8,7g/dl, leucocitos 2.900 (neutrófilos 77%), actividad de
protrombina 54%, PCR 359mg/ml, procalcitonina 3,8ng/dl, ácido láctico 18mg/dl, creatinina 2,9mg/dl, urea 88mg/dl, GOT 184 UI/l, GPT
111 UI/l, fosfatasa alcalina 229 UI/l, gasometría arterial pH 7,28, pCO2 41; pO2 66, HCO3 19, saturación arterial de O2 91%, cual es la
conducta a seguir menos apropiada ante el estado de choque séptico del caso?

RESPUESTA
a.- Flunitracepam y pancuronio e intubación orotraqueal con conexión a ventilación mecánica.
b.- Estabilización hemodinámica con cristaloides y perfusión de FVA (noradrenalina y dopamina).
c.- Corrección de coagulación con concentrado plaquetario, crioprecipitados y plasma fresco.
d.- Correccion intensiva de valores gasométricos y perfusión de FVA (dobutamina y vasopresina).

PREGUNTA
En LAPE se observa líquido purulento con membranas fibrinopurulentas generalizadas, especialmente en espacio subfrénico y flanco
derecho, congestión de asas y peritoneo, no encontrándose perforación de víscera hueca y vesícula normal. Se realizan lavados y
colocación de drenaje subfrénico derecho. Se inició antibioterapia empírica con piperacilina-tazobactam y a las 48 horas, por mala
evolución clínica con fiebre persistente e hipoxemia importante, se asocia amikacina. Cual de las siguientes diagnosticos diferenciales es
mas probable considerando los hallazgos, evolución y antecedente de DM?

RESPUESTA
a.- Hemangioma, hematoma,
b.- Metástasis, tumor necrótico,
c.- Quiste hemorrágico, quiste hidatídico complicado.
d.- Abseso hepático amibiano ó E. coli ó Klebsiella pneumoniae.

CASO CLINICO
Mujer de 66 años que ingresa por cuadro de shock séptico secundario a pielonefritis aguda derecha con disfunción multiorgánica
(DMO). Requirió apoyo hemodinámico, terapia sustitutiva renal y ventilación mecánica prolongados. Precisó dobutamina (20
mcg/kg/min) y noradrenalina (2-2,5 mcg/kg/min) durante una media de 6 semanas. Recibió además terlipresina (1mg/4h) y azul de
metileno. Se administró hidrocortisona y antibioterapia dirigida. Las dosis elevadas y prolongadas de aminas vasoactivas provocaron
vasoconstricción distal intensa con necrosis isquémica digital que precisó la amputación de todos los dedos de los miembros inferiores.
La retirada de las medidas de soporte orgánico fue lenta así como la recuperación posterior. La estancia en UVI fue de 120 días. En
planta comenzó con dolor en ambas caderas que limitó la rehabilitación. No había antecedentes de traumatismo o caída. Las pruebas
de imagen informaron de lesiones compatibles con necrosis avascular de caderas.

PREGUNTA
Cuales de los siguientes criterios diagnosticos es menos útil para identificar sepsis?

RESPUESTA
a.- PAM < 60 mmHg ó PAS < 90 mmHg.
b.- Hipertermia y/ó hiportermia.
c.- Volumen urinario <20 ml/hora.
d.- Velocidad de llenado capilar.

PREGUNTA
Cuales son los valores analíticos es de poca utilidad dentro de las primeras horas del padecimiento para confirmar el diagnostico de
SRIS y los procesos sépticos, además de los datos clínicos?

RESPUESTA
a.- Hiperglucemia > 120 mg/dl sin DM.
b.- Procalcitonina.
c.- Leucopenia < 4,000.

CURSO ENARM CMN SIGLO XXI TEL: 36246001 Pharmed Solutions Institute PÁGINA 197
MANUAL DE TRABAJO DEL CURSO ENARM CMN SIGLO XXI
d.- Proteina C reactiva 2DS mayor a lo normal.

PREGUNTA
Cual de los siguientes analíticos no es un elemento del sistema PIRO para estimar pronostico del paciente?

RESPUESTA
a.- Grado de agresión de la infección.
b.- Velocidad de instauración.
c.- Numero de órganos con disfunción
d.- Intensidad de la respuesta.

PREGUNTA
Cuál de las siguientes no es una meta adecuada en el tratamiento de la sepsis y choque séptico?

RESPUESTA
a.- Presion venosa central (PVC) 8-12 mmHg.
b.- Presion arterial media (PAM) > 65 mmHg.
c.- Gasto urinario (GU) 0.3 ml/kg/hr.
d.- Saturacion de sangre venosa (SATVO2) > 70 %.

PREGUNTA
Cual de los siguientes criterios de reanimación del paciente con sepsis grave se encuentra fuera de rango?

RESPUESTA
a.- Soluciones cristaloides > 1000 ml/30 min ó 300 a 500 ml coloides.
b.- Trasfusión de concentrados eritrocitarios para lograr un hematocrito > de 30 % con ScvO2 baja.
c.- Administracion de plasma fresco congelado con elevación de TP, INR ó TPT.
d.- Perfusion de dobutamina 50 mcg/kg/min

PREGUNTA
Cual de los siguientes parámetros guarda mayor relación de la principales complicaciones sin embargo menos especifico para vigilar al
caso clínico actual?

RESPUESTA
a.- Proteina C reactiva.
b.- Hiperlacticidemia.
c.- Procalcitonina.
d.- Creatinina serica

PREGUNTA
Cual de los siguientes parámetros guarda menor sensibilidad en relación con el síndrome de respuesta inflamatoria sistémica?

RESPUESTA
a.- Proteina C reactiva +/-1DS mayor.
b.- INR > 1.5
c.- TTPa > 60”
d.- Trobocitopenia < 100,000 celulas/mm3.

CASO CLINICO
Se trata de una mujer de 58 años que ingresó por fiebre y cefalea de aparición brusca. Dentro de sus antecedentes personales,
destacaba la presencia de prótesis mecánica mitral, aórtica secundaria a valvulopatía reumática, y nefrectomía derecha. Se encontraba
recibiendo tratamiento anticoagulante. A la exploración física, únicamente llamaba la atención cierto grado de rigidez nucal. En la
analítica inicial, leve deterioro de su insuficiencia renal de base (creatinina de 1,83 mg/dL).

PREGUNTA
Ante tales manifestaciones, se realizó una TAC craneal así como punción del LCR que no presentaron alteraciones. Considerando los
antecedentes y estado actual cual es la conducta a seguir para establecer un agente etiológico.

RESPUESTA
a.- Ecocardiograma transesofagico.
b.- RMN craneal por síntomas neurológicos.
c.- Cultivo con antibiograma.
d.- BH y tiempos de coagulación.

CURSO ENARM CMN SIGLO XXI TEL: 36246001 Pharmed Solutions Institute PÁGINA 198
MANUAL DE TRABAJO DEL CURSO ENARM CMN SIGLO XXI
CASO CLINICO
Se trata de femenino de 27 años de edad originaria de la sierra de Oaxaca, se encuentra en hospitalización en medicina interna, debido
a que presenta 24 horas con exantema, hipotensión y fiebre, a la exploración física se observa lesión en miembro pélvico, dicha lesión
es ulcerosa, sin eritema, no hay irritación ni secreciones, pero si hay presencia de exantema en palmas, conjuntivas y mucosa bucal,
además descamación en las puntas de los dedos, los resultados de laboratorios reportan creatinina de 2.7 aspartato aminotransferasa
de 250 U/L, alanina aminotransferasa de 328 U/L, aun no se cuenta con resultados de los cultivos, se instaura tratamiento antibiótico y
reanimación con liquidos y vasopresores.

PREGUNTA
Cual es el diagnostico mas probable en este caso.

RESPUESTA
a.- Choque por Fiebre Tifoidea
b.- Choque toxico estafilocócico.
c.- Choque toxico estreptocócico.
d.- Choque toxico botulínico.

CRISIS HIPERTENSIVA:
CIENCIAS BASICAS: Por convención los valores de tensión arterial (TA) severamente elevados se definen por tensión arterial sistólica
(TAS) mayor a 180 mmHg y tensión arterial diastólica (TAD) mayor de 120 mmHg, este corte es meramente arbitrario, con poca
relevancia para el médico, porque la mayoría de los pacientes no requerirá tratamiento urgente para reducción de la TA. De forma
contraria paciente con elevaciones menores, que no reúnen criterios de HTA severa pueden requerir terapia antihipertensiva inmediata
para situaciones de riesgo como en los aneurismas disecantes de aorta. Las crisis hipertensivas incluyen a las emergencias y urgencias.
Emergencia hipertensiva: Presencia de daño de órgano agudo, que requiere rápido descenso de TA, en un intervalo de horas.
Usualmente requiere internación en unidad de cuidados críticos con medicaciones por vía parenteral. En estos casos la TA cumple un
rol patogénico en la génesis y progresión del cuadro clínico. Debemos recalcar que la emergencia está definida por la gravedad del
cuadro clínico y no por los valores de TA, usualmente la TAD es mayor 120 mmHg. Los órganos que son afectado más frecuentemente
en emergencias hipertensivas son el cerebro (EVC 24.5%, encefalopatía 16.3%, hemorragias intracerebrales o subaracnoideas 4.5%),
corazón (EAP 22.5%, ICC 14.3%, IAM 12.0%, disección aortica 2.0%), grandes vasos, riñón y útero gravídico (eclampsia 4.5%,
preeclampsia grave), siendo más frecuente la afectación única en el 83% de los casos, pero en porcentaje variable se pueden presentar
más de un daño de órgano blanco en contexto de emergencias hipertensiva. Urgencia hipertensiva: cuando se produce una elevación
brusca de la presión arterial (PA), en relación a las cifras tensiónales habituales del paciente o cuando la PA ≥ 210/120 mmHg (algunos
consensos actuales hablan de sistólicas ≥180 mmHg), pero sin síntomas específicos ni daño de órganos diana. El objetivo terapéutico
sería reducir gradualmente la PA (en 24-48h) con medicación oral ya que si se baja muy rápidamente puede favorecerse la
hipoperfusión de órganos diana. Debe distinguirse de una pseudocrisis hipertensiva, que es la elevación de la PA reactiva a una
situación de stress o dolor. Hipertensión severa aislada: Elevación de la TAD >120 mmHg, que requiere descenso de valores de TA en
lapso de días con medicación y usualmente sin internación. Por definición se debe establecer previamente la ausencia de daño de
órgano blanco, hay poca evidencia que el control de la TA, en un lapso de tiempo corto sea beneficiosos. SALUD PÚBLICA: La
Hipertensión Arterial (HTA) es un proceso crónico que presenta una prevalencia de más del 25% en la población occidental adulta1.
Aproximadamente un 1-2% de los pacientes hipertensos desarrollaran una crisis hipertensiva (urgencia o emergencia) en algún
momento de su vida. El tratamiento y el adecuado seguimiento de estos pacientes aumentan su esperanza de vida y disminuye la
incidencia de complicaciones. PATOGENIA: T/A = (gasto cardiaco) (resistencias vasculares periféricas). Para que la presión arterial
aumente se requiere desequilibrio en esta relación, como falla de mecanismos autorreguladores, alteraciones en propiedades
contráctiles del musculo arteriolar, activación del sistema SRAA, vasoconstricción y aumento de RVP. El rápido desarrollo de daño de
órgano agudo de las emergencias hipertensivas es atribuido a la falla en el mantenimiento de la normal autorregulación de los lechos
vasculares de órganos de choque (cerebro, riñón, retina y corazón) y por otro lado un aumento abrupto de las resistencias vasculares
sistémicas. Conduciendo a una injuria endotelial con necrosis fibrinoide de arteriolas. Estableciéndose el ciclo isquemia, depósito
plaquetario, y falla de la autorregulación con deterioro clínico del paciente. Los gatillos específicos de este fenómeno no son conocidos.
DIAGNOSTICO: Historia clínica: historia de la HTA (edad de inicio, tiempo de evolución, grado de control, cifras basales, tratamiento
seguido, síntomas acompañantes), momento del inicio de la clínica, evolución hasta el momento del ingreso, síntomas
asociados. Antecedentes personales: hábitos higiénico-dieté cos (alcohol, drogas), enfermedades asociadas, factores de riesgo
cardiovascular, repercusión sobre órganos diana, tratamientos. Antecedentes familiares: HTA, enfermedades cardiovasculares,
enfermedades renales, obesidad, dislipemia. Exploración física: Priorizar en primer lugar la búsqueda de daño a órgano blanco.
Medición de la PA. Peso, talla, índice de masa corporal (IMC) y perímetro abdominal. Exploración sistemática (cardiovascular,
pulmonar, abdominal, neurológica, ocular, cutánea, cuello y palpación tiroidea). Palpación de pulsos periféricos (en las cuatro
extremidades) y centrales valorando simetrías y amplitud. Fondo de ojo (hemorragias, exudados blandos, papiledema). Pruebas
complementarias: En urgencia hipertensiva: sistemático de orina, ECG (isquemia, arritmia, hipertrofia VI), Rx posteroanterior y lateral
de tórax. En emergencia hipertensiva: hemograma, bioquímica con CPK, CPK-MB, troponina I (si existe sospecha de SCA), sistemático de
orina, ECG, gasometría arterial (GSA) (si existe sospecha de EAP o IC), fondo de ojo (grado de retinopatía), Rx posteroanterior y lateral
de tórax, TAC craneal (si existe sospecha de ictus), ecocardiograma (si hay cardiopa a o sospecha de disfunción cardiaca), TAC
tóracoabdominal (si existe sospecha de aneurisma disecante de aorta). TRATAMIENTO: Si daño a órgano blanco: Emergencia
hipertensiva, Objetivo: Disminuir la PAM en un 25% en un periodo comprendido entre minutos y 2 horas o hasta niveles seguros
(excepto en la disección aórtica), si el paciente estable continuar reducción hacia valores normales dentro de 8-24 hrs de admisión.
Tratamiento inmediato y hospitalización. Protocolo A, B, C: soporte vital avanzado (SVA) si es necesario. Mantener vía aérea permeable
y oxigenoterapia. Pulsioximetría, monitorización de la PA, canalización de vía venosa periférica: 1.Labetalol (disección aortica, IAM,
sindromes coronarios agudos): bloqueador mixto β,α, dosis de impregnación 20mg, seguida por una dosis incremental de 20 a 80mg

CURSO ENARM CMN SIGLO XXI TEL: 36246001 Pharmed Solutions Institute PÁGINA 199
MANUAL DE TRABAJO DEL CURSO ENARM CMN SIGLO XXI
con intervalos de 10 min hasta conseguir el efecto deseado. Reducen la mortalidad asociada a arritmias. 2. Nicardipina (encefalopatía
hipertensiva, hemorragia intracraneal, EVC): es un calcio antagonista con alta selectividad vascular y fuerte actividad vasodilatadora
cerebral y coronaria. La infusión inicial es de 5mg/h con aumentos de 2.5mg/h cada 5min, hasta un máximo de 15 mg/h.
3.Nitroprusiato (EVC, isquemia, encefalopatía, hemorragia subaracnoidea): es un vasodilatador arterial y venoso que disminuye la
postcarga y precarga. IV de 0.5.10 mg/kr/min. Es extremadamente potente, útil monitoreo intraarterial, puede generar toxicidad por
cianuros a altas dosis o por el uso prolongado, útil en casi todas las crisis hipertensivas, puede provocar aumento de la presión
intracraneana. 4. Nitroglicerina (ICC aguda o EAP, IAM, síndromes coronarios agudos): es un venodilatador potente. La dosis es 5
mcg/min con un incremento de hasta 20mg. 5. Hidralacina (embarazo): vasodilatador directo de acción predominante arteriolar, droga
de elección en crisis hipertensivas en embarazo. IM de 10-50mgs. Eclampsia, metildopa, MgSO4. NO daño a órgano blanco: Urgencia
hipertensiva: Si PAS ≥ 210 mmHg o PAD ≥ 120 mmHg. Objetivo: Disminuir la presión arterial media (PAM) en un 20% en un periodo de
24-48 horas o disminuir la PAD a valores < 120 mmHg. La disminución debe ser gradual a fin de prevenir isquemia orgánica (cardiaca,
cerebral). Reposo 5-10 minutos en lugar tranquilo y en decúbito supino. Valorar ansiolítico suave (benzodiacepina) si
ansiedad/nerviosismo. Comprobar el nivel de PA. Es importante saber si el paciente estaba recibiendo tratamiento
antihipertensivo previo o no: Si estaba a tratamiento y lo ha abandonado, hay que reiniciarlo de nuevo, pero si continúa con la
medicación hay que aumentar la dosis del que toma o asociar nuevos fármacos. Si no está a tratamiento y es necesario medicación
antihipertensiva, debe emplearse un fármaco de acción lenta (ej. captopril) evitando los de acción rápida porque pueden
inducir isquemia en los órganos diana (nifedipino sublingual). Nunca descender la PA por debajo de las cifras habituales del paciente.
Tratamiento en primer nivel con medicamentos vía oral: 1. Inhibidores de la enzima convertidora de angiotensina: a. Captopril, a dosis
de 25 mg, puede utilizarse por vía oral o sublingual comenzando su acción a los 15-30 minutos, hasta un máximo a los 50-90 minutos,
durando su acción 4-6 horas. Si a pesar del tratamiento la PA ≥ 210/120 mmHg se puede repetir la dosis 2-3 veces a intervalos de 30
minutos hasta un máximo de 100 mg. b. Enalapril: 10mg cada 12 horas, dosis máxima 40mg al día 2.Betabloqueadores: a. Metroprolol:
50-100mg cada 12 horas b. Atenolol: 50mg cada 12-24 horas 3. Antagonistas de los receptores de angiotensina II: a. Losartan: 50mg
cada 12 horas, b. Candesartan: 8-16mg cada 24 horas, c. Telmisartan: 40-80mg cada 24 horas. 4. Calcioantagonistas: a. Amlodipino: 5-
10mg cada 24 horas, b. Nifedipino: 30-60mg cada 24 horas. 5. Diuré cos azídicos: a. Hidroclorotiazida b. Clortalidona.
COMPLICACIONES: Encefalopatía hipertensiva: Ocurre como resultado de un abrupto y sostenido incremento de la TA, que excede los
límites de autorregulación de las arterias de resistencia de pequeño calibre en el cerebro (FSC), es de inicio agudo y reversible, se
presenta con cefalea intensa, vomito, somnolencia, confusión, puede ocurrir convulciones, déficit neurológicos focales, el edema de
papila es frecuente. Se presenta cuando PAM sobrepasa los 160 mmHg aprox., dando lugar a la aparición de isquemia por vasoespasmo
marcado, incremento de permeabilidad vascular, hemorragias puntiformes y edema cerebral. La reducción inmediata en un 30-40%
revierte el vasoespasmo, pero la disminución excesiva debe ser evitada para prevenir mayor isquemia cerebral, que ocurre cuando la
TA cae por debajo del límite inferior de la autorregulación. Debemos tener en cuenta que en sujetos normales este límite se encuentra
alrededor de los 60mmHg. Verdadera emergencia médica, sin tratamiento, la evolución es el coma progresivo y muerte. El régimen
estándar es nitroprusiato IV, hasta reducción de 25% de PAM, en una hora. Isquemia cerebral aguda: Caracterizado por déficit
neurológico de más de 24 hrs de aparición en ausencia de alguna otra causa vascular, se manifiesta por alteraciones visuales, afasia,
paresia, hemiparesia, hemiplejia, ataxia-hemiparesia, disartria. Hemorragia intracerebral, 80% de los pacientes son hipertensos
crónicos, aumenta el riesgo con cifras elevadas de presión diastólica. En EVC isquémico TAD 120-130mmHg, disminuir 10-15% en 24
hrs. En EVC hemorrágico TAS 200 o TAD >110mmHg, disminuir 25% en 6-12 hrs. Hemorragia subaracnoidea, irrupción súbita de sangre
al espacio subaracnoideo craneal o espinal, más común en ganglios basales 40-50%, sustancia blanca subcortical 20%, talamo 15%.
Disección aortica, IAM, EAP, Insuficiencia renal.

CASO CLINICO
Se trata de paciente femenino de 68 años de edad, acude a consulta por presentar cefalea global, acufenos y fosfenos, mareo, refiere
además parestesias de miembro toracicco, además hormigueo facial, a la exploración física se observa ansioso y levemente
desorientado, TA 180/120 mmHg, FC 132 lpm, FR 41. Se envía al servicio de urgencia donde se le administra nifedipino subligual antes
de enviar a segundo nivel.

PREGUNTA
Cual de las siguientes complicaciones por la maniobra realizada es menos probable para el caso.

RESPUESTA
a.- Infarto al miocardio.
b.- Acidente cerebrovascular.
c.- Edema agudo pulmonar.
d.- Bloqueo cardiaco.

CASO CLINICO
Se encuentra paciente de 42 años de edad el cual se conoce hipertenso con buen apego a tratamiento, tiene antecedentes de
endarteritis de takayasu, refiere cefalea global, con acufenos y fosfenos, refiere amaurosis fugax y dolor retrosternal ardoroso y
opresivo, se observo soplo carotidea, disminución de la fuerza muscular, disminución de la sensibilidad en de miembros, disminución
de pulsos, con hipotermia periférica, se observa desviasion de choque de punta, galope, sin de edema pulmonar e imagen de
ensanchamiento mediastinal. Sus constantes vitales fueron TA 210/119 mmHg, FC 123 lpm, FR 31 rpm,

PREGUNTA
Cual de las siguientes complicaciones es la mas probable que se encuentra en este caso?

RESPUESTA

CURSO ENARM CMN SIGLO XXI TEL: 36246001 Pharmed Solutions Institute PÁGINA 200
MANUAL DE TRABAJO DEL CURSO ENARM CMN SIGLO XXI
a.- Insuficiencias cardiaca aguda.
b.- Disección de la aorta.
c.- Bloqueo AV de tercer grado.
d.- Encefalopatia hipertensiva

PREGUNTA
Cual es el objetivo principal del manejo de la presión arterial considerando la complicación con la que cursa actualmente?

RESPUESTA
a.- Disminuir la capacitancia coronaria.
b.- Disminuir el volumen de eyección.
c.- Disminuir el volumen circulante.
d.- Disminuir la poscarga.

CASO CLINICO
Paciente masculino de 61 años de edad hipertenso mal controlado y diabetes mellitus, acude a urgencias por cefalea intensa, vomitos
intenso, el paciente presenta datos compatibles con delirium caracterizado por agitación psicomotriz, presento vomito importante, su
alteraciones neurologías no fueron focalizados y se presento lentamente, durante la exploración clínica presento disartria y
hemiparesia, los signos vitales TA 205/120 mmHg, cual es la complicación mas probable que presento el paciente?

RESPUESTA
a.- Hemorragia cerebral.
b.- Encefalopatia hipertensiva.
c.- Edema pulmonar.
d.- Meningoencefalitis.

PREGUNTA
Cual es la meta sobre la hipertensión arterial?

RESPUESTA
a.- Disminuir 25 % de la diastólica.
b.- Disminuir 50 % de la diastólica.
c.- Disminuir 25 % de la sistólica.
d.- Disminuir 50 % de la sistólica.

PREGUNTA
Cual es tratamiento de primera elección en este caso

RESPUESTA
a.- Betabloqueadores.
b.- Nitroglicerina
c.- Nitroprusiato.
c.- Calcioantagonistas.

PREGUNTA
Se agrego al tratamiento diuretico al manejo, el paciente presenta rahs cutáneo en alas de mariposa, cual es la causa de la presencia de
este sindrome de lupus Like?

RESPUESTA
a.- Hidroclorotiazida.
b.- Clorotiacida.
c.- Furosemide.
d.- Hidralacina.

PREGUNTA
El paciente diminuyo los síntomas durante la estancia en sala de choque, 8 horas después presento incremento de la tensión arterial
nuevamente sin datos de focalización, con datos sugestivos de isquemia miocárdica.

RESPUESTA
a.- Nitroprusiato.
b.- Clonidina.
c.- Nitroglicerina.
d.- Labetolol.

CURSO ENARM CMN SIGLO XXI TEL: 36246001 Pharmed Solutions Institute PÁGINA 201
MANUAL DE TRABAJO DEL CURSO ENARM CMN SIGLO XXI
HIPERTENSIÓN MALIGNA:
CIENCIAS BASICAS: Es una hipertensión arterial rápidamente progresiva caracterizada, desde el punto de vista anatomopatológico, por
lesiones arteriolares graves y clínicamente por la existencia de presión arterial muy elevada, hemorragias y exudados retinianos y, a
menudo aunque no necesariamente, edema de papila. Las cifras de PA suelen ser superiores a 120 mmHg de diastólica, y el concepto
de rápidamente progresiva nos indica que de no tratarse cuidadosamente aparecen sucesivas lesiones vasculares en órganos diana que
condicionan el pronóstico de la enfermedad y puede llegar a ser fatal. Puede aparecer en cualquier forma de hipertensión, ya sea
esencial o secundaria, e incluso puede ser la forma de presentación de una glomerulopatía como la nefropatía IgA o de una vasculitis
necrotizante secundaria a un lupus eritematoso sistémico La encefalopatía hipertensiva es la consecuencia de un aumento súbito y
generalmente intenso de la PA que cursa con intensa cefalea y alteraciones variables del nivel de conciencia y que es reversible al
reducir, mediante el tratamiento, las cifras de PA. Puede acompañar una hipertensión maligna, pero también puede aparecer sin ésta.
Se presenta con más frecuencia en individuos previamente normotensos (embarazadas con eclampsia, jóvenes) que en hipertensos
crónicos cuyas cifras de PA aumentan progresivamente. SALUD PÚBLICA: Si la hipertensión arterial afecta alrededor de la quinta parte
de la población general, la hipertensión maligna apenas alcanza el 1 % de los hipertensos. A medida que la detección y el manejo de la
hipertensión ha ido mejorando, su incidencia va disminuyendo, pudiéndose considerar hoy en día como una patología rara. El 80 % de
las hipertensiones malignas corresponden a una nefropatía o patología vascular renal. PATOGENIA: Los dos factores de mayor
importancia para determinar el inicio de la fase maligna son, probablemente el nivel absoluto de PA y su rapidez de instauración.
Aunque pueden intervenir factores hormonales, la fase maligna es, con seguridad una consecuencia no específica de cifras de PA muy
elevadas. Las lesiones vasculares que se producen conllevan consecuencias isquémicas tanto a nivel de órgano (riñón) como a nivel de
la propia pared vascular, estableciéndose un «círculo vicioso» entre isquemia y PA a ambos niveles. Factores hormonales intervienen en
este círculo vicioso, aumentando no sólo la PA, sino también la permeabilidad de la pared vascular. Al 75 % de los pacientes con
hipertensión maligna se les detecta una actividad renina plasmática elevada, con el consiguiente aumento de la secreción de
aldosterona. La hipersecreción de renina secundaria a los cambios vasculares renales producidos por la elevación intensa de la PA es el
mecanismo que sostiene el círculo vicioso al producir, a través de la angiotensina ll, una ulterior vasoconstricción renal con más
isquemia y más secreción de renina. Además en este momento se produce una «natriuresis de presión», con la consiguiente reducción
del volumen intravascular, que puede a su vez también estimular la secreción de renina y la actividad del sistema nervioso simpático.
Las lesiones arteriolares de la hipertensión maligna afectan a muchos órganos: encéfalo, páncreas, corazón, intestino, bazo y riñón,
siendo este último el más extensamente estudiad. En el riñón las lesiones más características son la necrosis fibrinoide y la endarteritis
proliferativa. La necrosis fibrinoide es posiblemente el resultado de la lesión endotelial por la propia hipertensión, con el depósito de
proteínas plasmáticas (fibrina) en el subendotelio. Esta lesión se reconoce como eosinofílica con detritus de células musculares
necróticas y restos proteicos en la pared arteriolar. La endarteritis proliferativa (hiperplasia o proliferación miointimal) se caracteriza
por un engrosamiento de la íntima, constituido por células musculares lisas, restos de membrana basal y mucopolisacáridos que
conlleva un importante estrechamiento de la luz vascular con el consiguiente aumento de la resistencia al flujo de sangre e isquemia
parenquimatosa. La lesión más frecuentemente observada en el riñón humano es una arteriosclerosis hiperplásica y una esclerosis
glomerular isquémica, y la típica hiperplasia miointimal se observa con relativa frecuencia en el riñón de individuos de raza negra.
DIAGNOSTICO: Clínico: Generalmente se caracteriza por un aumento relativamente brusco de la PA, con encefalopatía hipertensiva o
sin ella. A diferencia de la hipertensión arterial «no maligna», que acostumbra a ser asintomática, la hipertensión maligna
generalmente se acompaña de síntomas diversos como malestar general, cefaleas, visión borrosa, confusión, somnolencia, náuseas,
vómitos, disnea, hematuria y oliguria. La cefalea es, sin duda, el síntoma más frecuente y precoz, y no es raro que preceda al resto de la
sintomatología unos seis meses. En el 75 % de los casos el cuadro se acompaña de pérdida de peso, como resultado de la isquemia del
músculo esquelético y la pérdida excesiva de sal y agua por parte del riñón que se observa en las fases iniciales de la hipertensión
maligna. Sin embargo, tampoco es infrecuente observar pacientes que niegan sintomatología alguna, a pesar de presentar insuficiencia
renal terminal, insuficiencia cardíaca grave o afectación de las capacidades cognitivas. Acostumbra a presentarse en individuos jóvenes
o de mediana edad, y sólo excepcionalmente es una forma de presentación de una hipertensión en edades avanzadas. Gran parte de la
sintomatología que presentan estos pacientes depende de las complicaciones acompañantes. De éstas las más propias de la
hipertensión maligna son: 1. Encefalopatía hipertensiva: alteraciones neurológicas, normalmente transitorias y fluctuantes, reversibles
al controlar la PA, que se manifiesta en forma de cefalea intensa, confusión mental, disfagia, alteraciones visuales y convulsiones.
Puede también manifestar alguna focalidad (hemiparesia), en cuyo caso la posibilidad de una hemorragia intracraneal debe ser
considerada, patológicamente existe edema cerebral más o menos generalizado. El descenso brusco de la PA durante el tratamiento en
estos pacientes con hipertensión maligna puede producir isquemia cerebral por hipoperfusión, si se alcanzan niveles por debajo de una
PA media de 100-120 mmHg pues por debajo de estas cifras el paciente es incapaz de autorregular su flujo cerebral. 2. Insuficiencia
renal: que recibe el nombre de nefroesclerosis maligna, puede presentarse clínicamente como una insuficiencia renal progresiva o
como una insuficiencia renal aguda, fases iniciales estos enfermos pueden presentar poliuria y polidipsia, con hiponatremia e
hipopotasemia. Los factores que inducen estos trastornos son en parte hemodinámicos y en parte hormonales. Entre los primeros
destacan la «natriuresis de presión» y entre los segundos el hiperreninismo e hiperaldosteronismo secundario a la isquemia renal, la
hipertensión maligna puede clínicamente presentarse como un síndrome nefrítico agudo. 3. Anemia hemolítica microangiopática: es la
traducción de un daño endotelial generalizado. Se manifiesta en forma de hemolisis intravascular con esquistocitosis, descenso de la
haptoglobina, aumento de reticulocitos y evidencia de coagulación intravascular. La principal exploración que requiere un paciente con
sospecha de hipertensión maligna es la exploración del fondo del ojo. Los hallazgos de una verdadera retinopatía hipertensiva (grado lll-
IV) son de capital importancia para el diagnóstico, el pronóstico y el tratamiento del paciente. En el fondo del ojo grado lll el calibre
arterial es muy inferior al venoso (espasmo arterial generalizado), con espasmos focales, apariencia de «hilo de plata» en cuanto al
reflejo luminoso, con desplazamiento venoso, cruces en ángulo recto y dilataciones venosas distales. Típicamente aparecen
hemorragias y exudados. Las hemorragias pueden ser de dos tipos: «en llama», que son superficiales y están interpuestas entre las
fibras nerviosas, y en forma de manchas y puntos, de situación más profunda que las fibras nerviosas. Las hemorragias son un signo de
lesión vascular grave y reciente, que desaparecen por lo común a las pocas semanas del control efectivo de la presión arterial. Los
exudados son también de dos tipos: exudados «duros» debidos a extravasación por lesión vascular entre las fibras nerviosas que
pueden tener una distribución radial alrededor de la mácula («estrella macular»), el líquido se reabsorbe, los macrófagos aclaran el

CURSO ENARM CMN SIGLO XXI TEL: 36246001 Pharmed Solutions Institute PÁGINA 202
MANUAL DE TRABAJO DEL CURSO ENARM CMN SIGLO XXI
residuo proteico-lipídico y queda un depósito hialino que algunas veces puede persistir, y b) exudados «blandos» o algodonosos, que
son mayores que los anteriores y de límites bien definidos, no son verdaderos exudados, sino fibras nerviosas infartadas por oclusión
vascular que pueden tener una apariencia lacunar. Estas lesiones desaparecen a las pocas semanas del tratamiento adecuado. En el
grado IV a las lesiones anteriores se añade papiledema. A la distensión venosa le sigue un aumento de vascularización del disco óptico,
que aparece de color rosado, con límites borrosos. Alrededor de la papila hay hemorragias radiales y exudados algodonosos.
PRONÓSTICO: La principal causa de muerte era la hemorragia cerebral o la insuficiencia cardíaca. Con un tratamiento inmediato y eficaz
es posible lograr una supervivencia superior al 70 % a los cinco años, con una remisión, generalmente parcial, del cuadro tanto clínico
como biológico. TRATAMIENTO: En la hipertensión maligna incurren varias circunstancias que la hacen catalogarse como una
emergencia hipertensiva, a saber: encefalopatía hipertensiva, insuficiencia cardíaca, insuficiencia renal aguda y la anemia hemolítica
microangiopática. Ello requiere una actuación inmediata y, por tanto, la administración de fármacos por vía parenteral. El objetivo del
tratamiento es bajar la PA para evitar la aparición o agravación de la repercusión orgánica sin llegar, durante las primeras 24 horas, a
niveles de normotensión. La PA ha de bajarse más o menos rápidamente según la intensidad de los signos y síntomas secundarios a la
lesión orgánica producida por la hipertensión.

CASO CLINICO
Inicio con cefalea, visión borrosa, con dolor toraccico opresivo, el paciente es diabético e hipertenso con mal apego a tratamiento, la
diatolica se observo en 115, refirió disminución de la agudeza visual, con necrosis fribinoide, con hemorragia de la retina, y exsudados
algodonosos, sus antecedentes fueron tabaquismo, anemia hemolítica, con cardiomegalia sin datos de edema pulmonar. Uremia,
Anteriormente presento hemorragia cerebral hace 2 años.

PREGUNTA
El paciente recibió previamente al traslado recibió dihidropiridina sublingual, cual es el tratamiento mas adecuado en este caso?

TRATAMIENTO
a.- Nitroprusiato de sodio.
b.- Nitroglicerina.
c.- Diuretico de asa.
d.- Dihidropiridina.

CURSO ENARM CMN SIGLO XXI TEL: 36246001 Pharmed Solutions Institute PÁGINA 203
MANUAL DE TRABAJO DEL CURSO ENARM CMN SIGLO XXI
CAUSAS FRECUENTES DE DESCOMPENSACION DE IC
INSUFICIENCIA CARDIACA DESCOMPENSADA NO CARDIACAS:
CIENCIAS BASICAS: Se trata de un estado fisiopatológico resultado del Falla de adhesión al tratamiento dietético o farmacológico
fracaso de los mecanismos de compensación ante la dificultad del corazón Aumento de las demandas metabólicas: anemia, fiebre, embarazo,
para llenarse (disfunción diastólica) o bien para bombear sangre (disfunción estrés)
Prescripción reciente de fármacos (amiodarona, AINES)
sistólica), con el objetivo de mantener un gasto cardíaco acorde a los Exceso de alcohol
requerimientos metabólicos y que termina por afectar a otros sistemas Insuficiencia Renal
corporales. La insuficiencia cardíaca (IC) es la vía final común de numerosas Embolismo de pulmón
enfermedades cardiovasculares. Las principales causas de insuficiencia Hipertensión mal controlada (crisis)
cardíaca son la hipertensión arterial y la cardiopatía isquémica, procesos a CARDIACAS:
Taquiarritmias
menudo concomitantes, seguidos de miocardiopatías y disfunciones Bradiarritmias
valvulares, en general, cualquier proceso que altere aguda o crónicamente la Isquemia miocárdica
función cardíaca podría ser causa de IC. SALUD PÚBLICA: Su prevalencia Valvulopatía de nueva aparición o empeoramiento de la existente
aumenta con la edad, siendo por tanto una enfermedad relacionada con el Excesiva reducción de la precarga (diuréticos, IECAS)
envejecimiento poblacional. Constituye en nuestro medio la principal causa de hospitalización en mayores de 65 años. Es el único de los
grandes síndromes cardiovasculares cuya incidencia se encuentra en aumento. Cerca del 1% de la población mayor de 40 años padece
IC, proporción que se incrementa en pacientes mayores de 65 años a un 6-10%. La forma más
CRITERIOS DE GRAVEDAD
común de IC es la crónica, con ocasionales descompensaciones agudas. PATOGENIA: La rapidez de Debut de IC
la instauración del fracaso cardíaco condiciona su presentación. Ante una lesión cardíaca aguda Edad >70 años
pueden aparecer síntomas severos de conges ón (edema agudo de pulmón) o de hipoperfusión Comorbilidades severas
periférica (shock cardiogénico). Sin embargo, en la mayor parte de los casos, nos encontraremos Enfermedad valvular severa conocida
ante un paciente con clínica de descompensación de una IC crónica. DIAGNOSTICO: Según la Disnea de rápida instauración
Angina, sincope o déficit neurológico
sociedad europea de cardiología, se define la IC como el síndrome clínico en el que el paciente Mala perfusión periférica
presenta los siguientes hallazgos: 1. Síntomas típicos manifestados en la anamnesis. Derivados de Oliguria (<30 cc/h o 0.5 cc/kg/h)
congestión pulmonar (IC izq.), disnea, ortopnea o disnea paroxística nocturna. Por congestión Anasarca
venosa, edema en miembros inferiores y zonas declives, distención y dolor abdominal, nicturia. Taquipnea (>30 rpm)
Estertores crepitantes gruesos (medios)
Por bajo gasto, fatigabilidad, astenia, depresión, mareos, sincope. 2. Signos típicos evidenciados en Auscultación R3
la exploración física, por IC izquierda, estertores crepitantes húmedos, bilaterales y de comienzo HTA severa (crisis) o hipotensión
en base, hipoventilación sibilancias. Por IC der., edemas en zonas declives, distención abdominal, Saturación sin O2 >90%
ascitis, hepatomegalia, tinte ictérico, caquexia, ingurgitación venosa yugular, reflujo Cambios en ECG
Taquicardia mayor de 100-120 lpm
hepatoyugular. Signos cardiacos, soplos, arritmias, signo de galope y por bajo gasto (palidez,
Bradicardia extrema
cianosis, sudoración, alteración en la conciencia). La taquicardia, los crepitantes pulmonares y
edema con fóvea, son frecuentes en la IC. 3. Evidencia objetiva de una anormalidad estructural o funcional del corazón en reposo
mediante pruebas complementarias. 4. respuesta al tratamiento con diuréticos y vasodilatadores, en los casos en que existen dudas
con los criterios anteriores. INSUFICIENCIA CARDIACA FORMAS CLINICAS: A) Insuficiencia cardiaca de novo o descompensación aguda
de ICC= disnea o taquicardia con congestión pulmonar o edema intersticial en la RX (65.4%). B) Edema agudo de pulmón (EAP)=IC
acompañada de edema alveolar en Rx de tórax o SAT O2 >90% (16.2%). C) Shock cardiogénico=ICA con hipotensión TAS <90mmHg,
oliguria y bajo gasto cardiaco (3.9%). D) Insuficiencia cardiaca e hipertensión= TA>180/100 mmHg con síntomas de IC (disnea,
taquicardia) y Rx con congestión pulmonar (11.4%). E) Insuficiencia cardiaca derecha= IC con aumento de presión venosa yugular y
hepatomegalia, acompañada de edema pulmonar. TRATAMIENTO EN URGENCIAS DE ATENCION PRIMARIA: A). Sin compromiso vital
inmediato: Grados I-III, sin criterios de inestabilidad y en ausencia de complicaciones agudas. Posición semisentada, examinar vía aérea,
monitorización, ECG (12 derivaciones), TA y Sat O2 (conseguir saturación mayor a 90%). Control, de temperatura. Obtener vía venosa,
restricción de líquidos, tratamiento diurético con furosemida a 40mg (2 ampollas) vía endovenosa. Efecto comienza a los 5-10 minutos
y es máximo a los 30 min. Puede ser IM cuando no hay posibilidad IV. Si paciente mejora y no hay datos de gravedad, estabilidad
hemodinámica (TAS >160mmHg Y >90 mmHg y FC >100 lpm), mejoría sintomática tras tratamiento, y buen soporte familiar y/o social,
se remitirá el paciente a su domicilio (con ajuste de tratamiento diurético, para posterior control ambulatorio. Evitar sobrepeso, dieta
hiposodica. Se puede usar IECAS, ARA II, digoxina, β-Bloqueaores. Si no mejora trasladar a hospital, tratamiento de ingreso Admon.,
oxigeno (>90%), digoxina 1 amp en bolo y luego 1 amp cada 6-8 hrs en 24 hrs, furosemida 1 amp. IV, posteriormente 1 amp c/8-12 hrs o
en perfusión. IECA renitec 5-20mgs c/24 hrs o atacand 16 mg c/24 hrs VO. Betabloquenates contraindicados en descompensaciones
graves. Dobutamina o vasodilatadores IV (nitroglicerina, nitroprusiato), si perfusión no adecuada o no diuresis. Medidas coadyuvantes;
amiodarona único seguro en IC izq. B) Con compromiso vital inmediato: Grado IV, grado III con inestabilidad hemodinámica o
complicación aguda, puede ser IAM, taquicardia ventricular o alguno de los criterios de gravedad, proporcionar medidas para evitar
paro cardiorrespiratorio. Edema agudo de pulmón cardiogénico (EAP); es un cuadro de inicio brusco provocado por el aumento de la
presión en la microcirculación pulmonar, secundario a insuficiencia aguda del ventrículo izquierdo, con la consiguiente acumulación de
líquidos en el intersticio pulmonar y alveolos. En general responde adecuadamente al tratamiento precoz, por lo que es necesaria una
actuación inmediata, sin demorarase en el examen físico, ni en espera de pruebas complementarias. El objetivo es; mantener una Sat02
normal (95-98%), para maximizar la liberación de O2 a los tejidos, ayudando a prevenir la disfunción de órganos distales y el fallo
multiorgánico. Los datos de sospecha son disnea de aparición brusca, ortopnea, disnea paroxística nocturna, tos con expectoración
rosada, oliguria, diaforesis profusa, hipotensión, cianosis periférica, tercer ruido, soplo cardiaco, taquicardia, sibilancias en fase
intersticial, crepitantes bilaterales en fase alveolar, si hay insuficiencia de ventrículo derecho hay IY, hepatomegalia y edemas
periféricos. El tratamiento es posición semisentada/incorporación con piernas colgando. Admon., de O2 (>90%). CPAP; (Funciones: la
presión positiva conlleva a una menor presión intrapleural negativa durante la inspiración, lo que disminuye la presión de llenado del
VD, y con ello mejorando el edema y la congestión pulmonar. La hiperinsuflacion pulmonar que se produce, aumenta el tono
parasimpático, y con ello disminuye la frecuencia cardiaca. En pacientes con función sistólica deprimida va a producir un aumento de la
FE del VI y una disminución de la poscarga. En pacientes con función sistólica normal, va a producir un descenso del volumen diastólico
final); iniciar con 5 cmH2O y aumentos de 2-3 cm, el objetivo es mejoría del disconfort y la disnea, FR >25 rpm, desaparición actividad de
músculos accesorios, disminuir la morbimortalidad, disminuir la necesidad de IOT, estancias prolongadas en pacientes con ventilador

CURSO ENARM CMN SIGLO XXI TEL: 36246001 Pharmed Solutions Institute PÁGINA 204
MANUAL DE TRABAJO DEL CURSO ENARM CMN SIGLO XXI
mecanico posteriormente progresar y retirar continuar con oxigenoterapia convencional o de lo contrario si empeora Intubación
orotraqueal (IOT). Obtener vía venosa, llave de 3 pasos. Monitorización de ECG, TA y Sat O2. Control de diuresis. Furosemida (I-B) 40-
80mgs, repetir en función de respuesta, antes de producir diuresis tiene efecto venodilatador. Primera línea de tratamiento los
nitrataos; Nitroglicerina (I-B) si TAS >90-95mmHg, mejora la congestión pulmonar sin aumentar el consumo de oxigeno miocárdico,
inicialmente se utiliza la via sublingual, aunque se existe HTA severa o isquemia miocárdica refractaria se administrara vía IV. La dosis
sublingual es de 0,4-0,8 mg/5-10min. Máximo de 3 comprimidos con intervalos de 5 minutos o 2 aplicaciones de nitroglicerina en
spray, es mejor que el nitroprusiato por el efecto de “robo coronario”. Levosimedan (IIa-B); es un sensibilizador al calcio, que abre los
canales de K, con efectos inotrópicos, metabólicos y vasodilatadores en el tratamiento de la ICA, con una mejoría clínica y
hemodinámica rápida, esta indicado en la IC de bajo gasto descompensada, IC secundaria a cardiopatía isquémica, IC postoperatoria,
Shock cardiogenico. Morfina es dilatador mixto, fundamentalmente venoso, reduciendo el retorno venoso y la presión de la aurícula
izquierda. Dosis de 3-5mg IV que pueden repetirse en 5-10 min. Diluir una ámpula de cloruro mórfico (1ml) en 9 ml de SF, pasar 3-5 ml
en 2-3 min cada 5 min hasta un máximo de 15 ml. Disponer de antagonistas de opiáceo (naloxona) y de atropina. Valorar
detenidamente la administración de morfina en pacientes con insuficiencia respiratoria crónica y/o si existe disminución del nivel de
conciencia. Dopamina y dobutamina indicado en ICA con hipoperfusión periférica (hipotensión, fracaso renal o signos cutáneos de mala
perfusión renal), uso restringido a UCI. Dopamina diferente efecto según dosis: <2µ/kg/min sobre receptores dopaminergico:
disminución de resistencias vasculares. 3-5µ/kg/min: aumento contractilidad. >5µ/kg/min sobre receptores alfa: aumento de
resistencias vasculares y aumento de TAS.

CASOS CLINICOS
Masculino de 36 años de edad, desempleado. Tabaquista de 20cig/d durante 15 años. Obeso con un índice de masa corporal (IMC)
Dislipidemia relacionada con el aumento del Índice de Masa Corporal) de 39. Sin patologías previas conocidas. Buena clase funcional
habitual. Comienza 15 días previos a la consulta con cuadro de comienzo insidioso, caracterizado por disnea en clase funcional II
(caminar 200 metros), asociado a tumefacción de miembros inferiores a predominio vespertino. También refiere de un mes de
evolución, disnea paroxística nocturna, en número de 2 a 3 episodios semanales, que aumentan en frecuencia en los últimos días. EF:
orientado. Peso: 120 kg Talla: 1,75 metros. (IMC): 39, TA: 130/80 mmHg. (FC): 90 latidos por minuto (lpm). Temperatura: 36,7º C. (FR):
15 respiraciones por minuto. Ingurgitación yugular 3/6 con colapso inspiratorio incompleto. Murmullo vesicular conservado con rales
crepitantes bilaterales hasta tercio inferior. Latido apexiano en 5º espacio intercostal izquierdo, línea hemiclavicular. Ruidos cardíacos
hipofonéticos, sin R3 ni R4. Soplo protomesositólico en foco mitral con irradiación a axila. Abdomen, globuloso, blando, indoloro, con
hepatomegalia a 3cm del reborde costal, no dolorosa. Miembros inferiores con tono, trofismo y pulsos conservados; con edema a
tensión desde raíz de miembros. Electrocardiograma: ritmo sinusal 90 latidos por minuto (lpm). QRS angosto. Sobrecarga de cavidades
izquierdas, aurícula y ventrículo, con trastorno de repolarización en cara lateral secundario a hipertrofia de VI. Figura. Laboratorio:
Hematocrito: 36%- Hemoglobina: 1,9 g/dl- Glóbulos Blancos: 12.840 /mm3 – Plaquetas: 330.000 /mm3 – Glicemia: 93 mg/dl – Sodio:
139 mEq/l – Potasio: 4,04 mEq/l – Urea: 27 mg/dl – Creatinina: 0,72 mg/dl – Bilirrubina total: 0,43 mg/dl – LDH: 348 UI/L – GGT: 39 UI/L
– FAL: 187UI/L – TGO: 14 UI/L – TGP: 20 UI/L. Radiografía de Tórax: relación cardiotorácica conservada. Hipertensión venocapilar
pulmonar grado III. Infiltrados algodonosos bilaterales a predominio de campos pules inferiores y medios. Sin derrame pleural.

PREGUNTA
Cual es la conducta a sintomática seguir mas adecuada?

RESPUESTA
a.- Oxigenoterapia.
b.- Diureticos.
c.- Antiarritmicos.
d.- Digitalicos.

CASO CLINICO
Paciente femenino de 60 años, que presenta dolor de pecho, durante la preparación para colonoscopia, cuadro de deposiciones
líquidas abundantes, con episodio de mareo, ortostatismo, sudoración fría, palpitaciones y dolor opresivo, retro-esternal, irradiado a
región interescapular. Tras la realización del procedimiento e infusión de 2000 cc de cristaloides, la paciente mejora, pero persiste la
molestia precordial. A su ingreso se encuentra hemodinámicamente estable TA 125 / 65 mmHg, (FC) 85 latidos por minuto (regular),
eupneica con buen intercambio gaseoso. En el electrocardiograma (EKG) se evidencia ritmo sinusal, con imagen de hemibloqueo
anterior de rama izquierda (HARI), sin signos de isquemia aguda en el momento actual. Se solicitan pruebas complementarias, con
enzimas de daño miocárdico discretamente elevados (CPK 282, CPK-MB 42; Troponina T 0.67). Ingresa en UCI para monitorización y
tratamiento. EF: Constantes: TA 110 / 65 mmHg; (FC) 80 latidos por minuto, FR: 12 (rpm). Saturación de oxígeno (SatO2) 98% con gases
normales. Bien perfundida, normohidratada, normocoloreada. Buen relleno capilar. Buena perfusión distal. Eupneica, sin sensación de
disnea. Afebril, no signos de sepsis actual. Ligera molestia precordial, que actualmente refiere que aumenta con movimientos
respiratorios. Consciente, orientada, colaboradora. Auscultación cardiaca (AC): Rítmica a 80 latidos por minuto (lpm), soplo sistólico
irradiado a punta cardiaca. (Hb) 14.9; (Htco) 41.2%; Leucocitos 9630 (Neutrófilos: 80%); Plaquetas 210000. Bioquímica: Cr 0.8; K 3.81;
CPK 282; CPK 42.7; Troponina T 0.67. (EKG): ritmo sinusal (RS). PR normal. QRS/QT normal. Hemibloqueo anterior de rama izquierda
(HARI). QS en V1-V2. No alteraciones ST - T sugestivas de isquemia y/o sobrecarga de cavidades. Rx tórax: índice cardiotorácico (ICT)
normal. Estructuras pleuromediastínicas normales. No infiltrados consolidativos ni signos de sobrecarga hídrica.

PREGUNTA
Cual de las siguientes medidas terapéuticas es menos apropiadas al pronostico inmediato en el caso?

RESPUESTA

CURSO ENARM CMN SIGLO XXI TEL: 36246001 Pharmed Solutions Institute PÁGINA 205
MANUAL DE TRABAJO DEL CURSO ENARM CMN SIGLO XXI
a.- Antiagregación con ácido acetilsalicílico (AAS).
b.- Tratamiento antianginoso con B-bloqueantes.
c.- Hipolipemiantes.
d.- Anticoagulación con heparina de bajo peso molecular (HPBPM).

CASO CLINICO
Presentamos a una paciente femenina, blanca, 43 años de edad, con antecedentes personales de salud y madre hipertensa; desde
agosto del año 2009 presenta episodios de tos seca, nocturna, acompañada de falta de aire progresiva a los esfuerzos, hasta
presentarse en el decúbito, con alivio al sentarse, se constatan como elementos positivos al examen físico: edemas de miembros
inferiores, frecuencia respiratoria de 24 inspiraciones por minuto, vibraciones vocales disminuidas hacia la base del hemitorax derecho,
ligera matidez a la percusión a ese nivel, estertores crepitantes en base derecha y sibilantes en ambos hemotórax, latido de la punta
visible y palpable en sexto espacio intercostal izquierdo por fuera de la línea media clavicular de ese lado; ruidos cardíacos
taquicárdicos y de buena intensidad, reforzamiento del segundo ruido en focos pulmonar y mitral, soplo sistólico II/VI en foco
pulmonar. Tensión arterial sistólica (TAS) 120 mmHg y tensión arterial diastólica (TAD) de 75 mmHg, frecuencia cardiaca (FC) de 140
latidos por minuto, ingurgitación yugular, reflujo hepatoyugular, hepatomegalia dolorosa de 4 cm, superficie lisa y borde romo.
Complementarios de hematología y hemoquímica dentro de límites normales. Ultrasonido abdominal: Hepatomegalia de 3 cm,
superficie lisa, con moderado aumento difuso de su ecogenicidad. Rx de tórax con cardiomegalia ligera, congestión hiliar bilateral con
redistribución del flujo hacia los vértices (líneas B de Kerley, borramiento del seno cardiofrénico derecho; en el ecocardiograma
contractilidad global y segmentaria conservada, dilatación de las cuatro cavidades, regurgitación mitral, aparatos valvulares
estructuralmente normales, grosor de las paredes ventriculares normal, flujograma pulmonar grado I, con fracción de eyección del 40%.
En el electrocardiograma (ECG), taquicardia sinusal, frecuencia cardiaca (FC) de 140 latidos por minuto.

PREGUNTA
Con estos elementos cual es la impresión diagnóstica mas adecuada?

RESPUESTA
a.- Miocardiopatia restrictiva.
b.- Miocardiopatia constrictiva.
c.- Miocardiopatía dilatada.
d.- Miocardiopatia mixta.

PREGUNTA
Considerando la ICC cual es el abordaje mas apropiado?

RESPUESTA
a.- Diuréticos, IECA y betabloqueadores.
b.- Diureticos, ARA II y Calcioantagonista.
c.- Diuretico k ahorrador, IECA, Calcioantiagonista.
d.- Diuretico K no ahorrador, ARA II, Calcioantagonista.

CURSO ENARM CMN SIGLO XXI TEL: 36246001 Pharmed Solutions Institute PÁGINA 206
MANUAL DE TRABAJO DEL CURSO ENARM CMN SIGLO XXI
TAPONAMIENTO CARDIACO:
CIENCIAS BÁSICAS: Urgencia vital, es la compresión del corazón que resulta de la acumulación de líquido en el saco pericárdico y que
produce un severo trastorno hemodinámico. La compresión cardíaca lleva a una disminución del gasto cardíaco y de la presión arterial
en un espectro variable en el que el paciente severamente comprometido puede estar en shock cardiogénico. Puede ser de origen
CAUSAS MÉDICAS DE TAPONAMIENTO traumático o secundario a casi cualquier tipo de pericarditis aguda o crónica, aunque es poco
CARDÍACO común en la pericarditis viral y después de IAM. La causa más frecuente de taponamiento
Taponamiento cardíaco agudo cardíaco agudo es el hemopericardio causado por heridas con arma de fuego, arma blanca o
Hemopericardio
Ruptura de la pared libre del ventrículo
trauma cerrado de tórax. Si la herida penetrante es relativamente pequeña suele haber
postinfarto de miocardio taponamiento cardíaco, y el aumento de presión en el saco pericárdico puede ayudar a reducir
Aneurisma aórtico roto a cavidad la severidad de la hemorragia. En el caso del infarto, la ruptura de la pared libre está asociada a
pericárdica la extensión de la necrosis miocárdica y tiene una mortalidad cercana al 90%. PATOGENIA: En
Disección aórtica al pericardio
condiciones normales la presión venosa periférica es superior a la presión venosa central y a su
Neoplasias (CA metastasico de pulmón y
seno, linfoma, leucemias, mesotelioma) vez ésta es mayor que la presión intrapericárdica en 5 Torr. Esta diferencia de presiones permite
Tratamiento anticoagulante y favorece el flujo sanguíneo continuo desde el sistema venoso hacia el corazón derecho. Al
Postoperatorio cirugía cardíaca (fugas, acumularse líquido en el espacio pericárdico se eleva la PVC y disminuye el gradiente entre la
anicoagulacion, aneurismas, disecciones)
presión de la aurícula derecha y el sistema venoso periférico; cuando la presión intrapericárdica
Pericarditis aguda
Bacteriana llega a 10 Torr se iguala a las presiones venosa central y venosa periférica, y de este punto en
Tuberculosa adelante, si se eleva la presión intrapericárdica, las otras dos también. La consecuencia de esta
Urémica igualdad en las presiones es que el ventrículo derecho pierde la presión efectiva de distensión y
Enfermedades del tejido conectivo se colapsa, tanto por acción directa del líquido intrapercárdico como por la pérdida de flujo
Taponamiento cardíaco crónico
Causas infecciosas. Tuberculosis. Algunas
desde la aurícula derecha. En estas condiciones, el flujo hacia la circulación pulmonar, así como
parasitosis. Causa no infecciosa. Urémico. el volumen diastólico que llega al ventrículo izquierdo, disminuyen y se reduce el gasto cardíaco.
Neoplásico (posirradiacion) Mixedematoso. En esta situación los mecanismos homeostáticos para tratar de mantener la presión de
Postirradiación. Síndrome de Dressler. perfusión de los órganos vitales son tres: la utilización del volumen residual diastólico en el
Síndrome postpericardiectomía.
corazón, el aumento de la frecuencia cardíaca y la vasoconstricción arteriolar a nivel del músculo
Quilopericardio
estriado y el riñón. En condiciones normales el volumen del líquido pericárdico es
aproximadamente 20 ml, como una capa de 5 mm que recubre el corazón. El espacio pericárdico, al igual que el pleural, tiene presión
negativa que se aumenta con la inspiración y se hace ligeramente positiva durante la espiración. Cuando se añade líquido al espacio
pericárdico la presión aumenta en forma lenta, mientras la membrana pericárdica permanece distensible, pero una vez que se
encuentra a tensión la presión aumenta en forma rápida. Estos eventos son diferentes dependiendo de si el taponamiento cardíaco es
agudo o crónico. Cuando la acumulación de líquido es aguda, la capacidad de distensibilidad pericárdica es muy baja y con apenas 100
ml de líquido se produce taponamiento cardíaco. Cuando la acumulación de líquido es lenta, como ocurre en los procesos crónicos, el
pericardio se va adaptando y aumenta en forma progresiva su distensibilidad, por lo cual recibe cantidades tan grandes como 2.000 ml
de líquido. Tal capacidad de distensibilidad del espacio pericárdico se debe a su estructura de tejido conjuntivo denso, el cual permite
algún estiramiento al estar sometido a elevaciones de presión progresivas. Esto es especialmente importante en los pacientes jóvenes.
DIAGNOSTICO: Con el aumento de la presión venosa se detecta IY severa, la cual disminuye en forma visible durante la sístole, por lo
cual es posible encontrar la depresión X del pulso venoso magnificada sin que se modifique la depresión Y (signo de Friedrich). Con la
acumulación de líquido en el espacio pericárdico la intensidad de los ruidos cardíacos disminuyen y llegan en ocasiones a no ser
audibles. La presión arterial se encuentra generalmente disminuida y convergente (presión arterial diferencial menor de 30 mmHg) y el
paciente está taquicárdico. El pulso paradójico es un signo casi constante en el taponamiento cardíaco, lo mismo que el signo de
Kussmaul. En el taponamiento cardíaco, generalmente aquejan dolor torácico sordo o constrictivo que puede ser pleurítico. La disnea
es frecuente, síntomas asociados con bajo gasto cardíaco, como frialdad, debilidad muscular y diafóresis, y puede en los casos graves,
exhibir alteración neurológica del estado de conciencia y shock cardiogénico. En el enfermo con taponamiento cardíaco crónico los
síntomas predominantes son los de la enfermedad de base y se pueden encontrar pérdida de peso, anorexia, debilidad marcada y
compromiso del estado general con disnea y dolor torácico crónicos. En estos pacientes es frecuente encontrar signos de congestión
venosa crónica como hepatomegalia, ascitis, y edema de miembros inferiores. Triada clínica del taponamiento agudo por hemorragia
intrapericárdica aguda: 1. descenso de la presión arterial sistémica 2. Aumento de la presión venosa sistémica 3. Corazón chico y
tranquilo. La Rx., del tórax aunque no brinda evidencia directa puede haber cardiomegalia, evidencia de derrame pericárdico y se
puede obtener información de procesos patológicos primarios que son causa de tapónamiento, como neoplasia o tuberculosis. El ECG
tampoco ofrece datos específicos, es usual encontrar taquicardia sinusal, bajo voltaje y anomalías inespecíficas de la repolarización
ventricular como la elevación cóncava del segmento ST. Otros hallazgos pueden ser el desnivel inferior del segmento PQ y la alternancia
eléctrica, la cual indica la presencia de derrame pericárdico, pero no es concluyente. El ecocardiograma es el método diagnóstico que
más información aporta. En primer lugar, detecta la presencia de derrame pericárdico y permite hacer una aproximación de su
cantidad, ofreciendo además múltiples signos de compromiso hemodinámico. La manera más simple de documentar el taponamiento
cardíaco es por medio de un catéter colocado en la aurícula derecha, con el cual se pone en evidencia el aumento de la PVC y se puede
obtener una curva que revele los cambios antes mencionados en el pulso venoso. Con el catéter de Swan-Ganz se pueden obtener,
además, curvas de presión de la arteria pulmonar, mediciones de la presión pulmonar en cuña y una estimación del gasto cardíaco por
termodilución, lo cual es de gran utilidad en el seguimiento y la evolución de los pacientes más graves. TRATAMIENTO: Extracción de
líquido para eliminar la compresión del corazón, por medio de la pericardiocentesis o por métodos quirúrgicos. La elección del tipo
depende de la disponibilidad de personal capacitado, de medios de ayuda pero, principalmente, de la enfermedad que llevó al
taponamiento cardíaco. Se prefiere la cirugía en los casos de hemopericardio en que se desea evitar la repetición de hemorragias y está
indicada en la pericarditis purulenta, con el fin de realizar drenaje completo y controlar la infección. La pericardiocentesis con guía
ecocardiográfica u otro po de imagen, como la fluoroscopia o la TC, es buena alternativa en prácticamente todas las enfermedades
pericárdicas. No debe olvidarse la probabilidad de recurrencia del taponamiento cardíaco cuando se trata con pericardiocentesis, por lo
cual deberá repetirse el procedimiento o hacer cirugía. Son de utilidad medidas terapéuticas de sostén como los líquidos intravenosos,
en especial en casos de hemopericardio traumático agudo, cuando el paciente está hipovolémico. Esta medida aumenta el volumen

CURSO ENARM CMN SIGLO XXI TEL: 36246001 Pharmed Solutions Institute PÁGINA 207
MANUAL DE TRABAJO DEL CURSO ENARM CMN SIGLO XXI
circulante, incrementa las presiones venosa central y pulmonar y la presión arterial. En ningún caso deben aplicarse vasodilatadores,
como el nitroprusiato de sodio por vía intravenosa. Tratamiento quirúrgico por tres métodos. El primero pericardiectomía por
esternotomía vertical o transversa, útil en pacientes con posible constricción (radiación o tuberculosis). El segundo la pericardiectomía
parietal a través de toracotomía intercostal izquierda, que se lleva a cabo bajo anestesia general y posibilita la resección de gran parte
del pericardio parietal anterolateral, lo cual permite una mejoría más definitiva. Es de utilidad en caso de pericarditis purulenta. El
tercero la pericardiostomía subxifoidea, que se efectúa bajo anestesia local con resección del apéndice xifoides y una incisión pequeña
en el pericardio, a través de la cual se drena el líquido y se conoce como procedimiento de ventana se puede liberar adherencias
pericárdicas, obtener biopsias del pericardio y dejar una sonda de drenaje, en caso de taponamiento no constrictivo y reversible
(pericarditis por diálisis o idiopática). La pericardiocentesis tiene ventajas como su rápida aplicabilidad, la facilidad de estudiar el
líquido, la factibilidad de combinarla con estudios hemodinámicos y su eficacia en las 2/3 partes de los casos. Las desventajas consisten
en la necesidad de personal bien capacitado en la técnica y de ecografía de buena calidad para obtener mejores resultados; además, no
siempre permite llegar a un diagnóstico que requiera biopsia pericárdica, no garantiza su eficacia en todos los casos, puede retardar la
intervención quirúrgica por el alivio temporal que proporciona y puede producirse hemopericardio por la punción del corazón. Sin
embargo, la pericardiocentesis puede llegar a ser una medida salvadora en los casos de taponamiento cardíaco agudo. Se aconseja
como premedicación a este procedimiento la administración de atropina (0,8 a 1,0 mg) por vía intravenosa o intramuscular, a fin de
prevenir reacciones vasovagales. La mejor ruta de acceso para la pericardiocentesis es la subxifoidea, pues evita lesiones de las arterias
coronarias. Previa asepsia con alcohol yodado y bajo anestesia local (en los casos urgentes se omite la anestesia) se efectúa la punción
5 cm por debajo de la punta del apéndice xifoides y 1 cm a la izquierda de la línea media, con la aguja en ángulo de 45 grados y dirigida
hacia el hombro izquierdo. Se ejerce succión continua mientras se introduce la aguja, deteniéndose cuando se perciba una sensación de
vencimiento de una resistencia lo cual indica la entrada al espacio pericárdico, hasta que se obtenga líquido o cuando se sientan las
pulsaciones cardíacas transmitidas a la aguja, lo que significa que se ha tocado el miocardio y se debe retirar un poco la jeringa. El
líquido pericárdico de aspirarse con lentitud durante 10 a 30 minutos y enviarse a estudio al laboratorio clínico de acuerdo con el caso.
El condiciones electivas puede emplearse el electrocardiograma para evitar lesiones del miocardio. En caso de taponamiento cardíaco
por herida del corazón, la extracción de algunos centímetros de sangre mejora el retorno venoso y puede salvar al paciente. La sangre
obtenida no coagula porque los movimientos del corazón la desfibrinan rápidamente; si la sangre obtenida se coagula es porque se
puncionó una cavidad del corazón. El tratamiento médico del taponamiento cardíaco agudo, incluido el soporte inotrópico, con o sin
vasodilatadores, es relativamente controversial. La dobutamina, administrada para revertir la hipotensión, es teóricamente ideal.
Durante el taponamiento, de cualquier manera, la es mulación endógena inotrópica del corazón es muchas veces máxima.

CASOS CLINICOS
Mujer de 81 años, sin antecedentes de interés, que ingresó con el diagnóstico de insuficiencia cardiaca desencadenado por infección
respiratoria, con un cuadro inespecífico de 1 mes de evolución, de malestar y astenia, y los días previos al ingreso había aparecido
disnea de esfuerzo, que llegaba a ser de reposo. Refería tos escasa, sin expectoración y sin fiebre. Laboratorios: hemoglobina, 11,9 g/
dl, con hematocrito, 36,8%, y leucocitosis, con 16,8. 705,109 plaquetas/l, y fibrinógeno, 464 mg/dl. La radiografía de tórax mostraba
cardiomegalia con ligera redistribución en ambas bases. En el ECG, reducción en el voltaje del QRS y un aplanamiento difuso de las
ondas T. A las 24 h presentó un deterioro clínico con datos clínicos y hemodinámicos, hipotensión, pulso paradójico y datos de presión
venosa elevada.

PREGUNTA
Cuál es la conducta a seguir mas apropiada.

RESPUESTA
a.- Pericardiocentesis.
b.- Pericardientomia urgente.
c.- Manejo conservador.
d.- Corrección del estado hemodinamico.

CASO CLINICO
Mujer de 69 años de edad que sufre un trauma torácico cerrado por compresión antero posterior al quedar atrapada por las puertas de
un autobús y 30 días después comienza a presentar falta de aire a los esfuerzos, aumentando progresivamente hasta desencadenarse a
los pequeños esfuerzos. Ingresa en el hospital con diagnóstico de cardiopatía isquémica. Se realiza ecocardiograma y se comprueba
gran colección líquida pericárdica que se evacua parcialmente mediante pericardiocentesis, pero al profundizarse los síntomas de
hipovolemia se decide realizar pericardiectomía anterior radical de urgencia a través de una toracotomía antero lateral izquierda y se
constata el pericardio engrosado y tenso. Se comprueba el diagnóstico por punción pericárdica transoperatoria por la presencia de
sangre y se procede a descomprimir lentamente la colección intrapericárdica. Se completa la pericardiectomía y se diagnostica una gran
contusión miocárdica con movimiento cardiaco lento. La evolución temporalmente es satisfactoria con estabilización del
electrocardiograma aunque tres meses después sufre infarto del miocardio y fallece.

PREGUNTA
Cual es el porcentaje de pacientes con aplastamiento torácica desarrollan tamponade cardiaco?

RESPUESTA
a.- 20%.
b.- 30 %.
c.- 40 %.
d.- 50 %.

CURSO ENARM CMN SIGLO XXI TEL: 36246001 Pharmed Solutions Institute PÁGINA 208
MANUAL DE TRABAJO DEL CURSO ENARM CMN SIGLO XXI
DISECCION DE AORTA:
CIENCIAS BASICAS: Consiste en la separación de la capa media de la pared aortica en la que penetra sangre precedente de la aorta con
lo que se establece una falsa luz que puede comprimir la luz verdadera del vaso. Entre ambas encontramos el colgajo intimal. Esta
separación es consecuencia de una rotura o perforación en la intima (puerta de entrada) que se propaga distalmente. La adventicia
puede contener inicialmente el sangrado o evolucionar a la rotura, Alrededor de 95% de roturas ocurren en la aorta ascendente, distal
a la válvula aortica. Dependiendo de la localización y la extensión pueden
aparecer un taponamiento cardiaco, hemotorax, una insuficiencia aortica o un
síndrome de mala perfusión. Este ultimo según los troncos arteriales afectados, se
puede manifestar como un síndrome coronario agudo, una afección neurológica o
visceral (por afección de troncos supraaórticos, arteriales medulares y ramas
viscerales), o una isquemia de los miembros inferiores. Los factores
predisponentes mas frecuentes son: hipertensión (72%) y ateroesclerosis (31%).
En pacientes menores de 60 años encontramos generalmente un aneurisma de
aorta, una cirugía cardiaca previa, un síndrome de Marfan o una válvula aortica
bicúspide.
DIAGNOSTICO: Clínico: el dolor es la característica más importante, es una
sensación de desgarro agudo e intenso, migratorio. La intensidad del dolor es
constante, por lo que se diferencia del dolor secundario a IAM. Un dolor
retroesternal con irradiación a cuello es característico de la afección de la aorta
ascendente, mientras que el dolor dorsolumbar orienta hacia una afección de la
aorta torácica descendente. El sincope puede estar causado por dolor intenso, la
obstrucción de los vasos cerebrales, la activación de barorreceptores aórticos o un
taponamiento cardiaco. En función de los vasos afectados habrá manifestaciones
de mala perfusión. Una disección origina una obstrucción dinámica de los troncos supraaórticos, que se puede manifestar con un
cuadro de isquemia cerebral y asimetría o ausencia de pulsos en extremidades superiores. El desgarro de las arterias intercostales se
puede mostrar como un déficit motor en los miembros inferiores. El compromiso de las arterias viscerales con dolor abdominal
(isquemia medular) o con aligoanuria (arterias renales). Los pulsos femorales pueden estar disminuidos o ser asimétricos. Otras
manifestaciones menos frecuentes son hemoptisis, síndrome de vena cava superior, obstrucción de la vía aérea y síndrome de Horner
(compresión por el aneurisma o hematoma en el cayado), disfonía. La ausencia de pulsos,
PACIENTES DE ALTO RIESGO DE presencia de sincope previo, signos neurológicos, estado de Shock y hemotorax se
DISECCION AORTICA
consideran factores de mal pronóstico. El diagnostico de disección aortica aguda debe
Enfermedad vascular ateroesclerótica
Anuloectasia aórtica considerarse en todo paciente con un cuadro de dolor abdominal agudo, isquemia de las
Aneurisma aórtico extremidades, en caso de sincope o un cuadro de insuficiencia cardiaca aguda no explicada.
Ulcera aortica Recientemente se ha publicado el score de Disección Aortica en Urgencias, valora la
Calcificación de la intima
presencia de dolor en espalada, una relación toraco-mediastinica mayor a 30%, una
Alteraciones genéticas (conectivopatias)
Síndrome de Marfan insuficiencia aortica y un diámetro aórtico mayor de 30mm por ecografía. La sensibilidad y
Síndrome de Ehlers-Danlos especificidad de este score es de 93 y 77% respectivamente, si están presentes 3 o más de
Síndrome de Turner estos signos. Si se sospecha una disección aortica y no hay disponibilidad diagnostica ni de
Enfermedad de Noonan
Osteogénesis imperfecta
tratamiento, debe asegurarse la remisión rápida del paciente. Rx de tórax; suele mostrar
Enfermedades congénitas ensanchamiento mediastinico o derrame pleural asociado. ECG: es normal en 30%, cambios
Coartación aortica isquémicos, signos de hipertrofia VI. Analítica sanguínea: anemia ligera, leucocitosis,
Valvula aórtica unicúspide o bicúspide trombopenia, alteración de pruebas de función hepática, elevación de Cr sérica. Elevación de
Enfermedades degenerativas
reactantes de fase aguda. Acidosis metabólica en casos de mala perfusión. Específicos:
Hipertensión
Causas traumáticas Ecocardiografía transtorácica (ETT), identifica disección aortica proximal, una insuficiencia
Lesión por desaceleración aortica asociada y un taponamiento cardiaco, permite evaluar la función miocárdica y medir
Lesión penetrante los diámetros de la raíz aortica. Ecocardiografía transesofágica (ETE), mas sensible y
Enfermedades inflamatorias especifica, confirma disección en un 90% de los casos y permite localizar la rotura intimal en
Sífilis
Arteritis de células gigantes la mayoría de los casos y valorar el flujo sanguíneo en la verdadera y falsa luz. TAC, tiene
Iatrogenia sensibilidad superior al 95% y una especificidad del 87%, da información precisa de los
Cateterismo cardiaco diferentes diámetros de los segmentos aórticos. Permite evaluar, con precisión la extensión
Canulación aórtica o femoral previa de la aorta afectada y mostrar la implicación de las arterias viscerales e iliacas. Tiene como
Balón de contrapulsación intraaórtico
inconveniente la nefrotoxicidad, el contraste y la necesidad de desplazar al paciente.
TRATAMIENTO: Actuación inmediata: La afectación de la aorta ascendente requiere cirugía, mientras que la disección de la aorta
descendente precisa abordaje médico en ausencia de complicaciones. Farmacológico: Antes de la realización de cualquier medida
diagnostico-terapéutica el paciente debe estar monitorizado con ECG, medida de presión arterial (cada 5min) y canalización de una vía
venosa de buen calibre. La analgesia contribuye a estabilizar al paciente y en, general se da opiáceos a dosis bajas (cloruro mórfico 3
mg IV cada 5-10 min), para evitar la progresión de la enfermedad y el riesgo de rotura aortica, se debe disminuir el estrés parietal
aórtico y controlar la presión arterial (objetivo TAS entre 90-110 mmHg, si se mantiene la diuresis). Los fármacos más usados como
tratamiento antiimpulso, son los bloqueadores β selectivos como: de vida corta esmolol IV carga 0.5mg/kg en 2-5 min, mantenimiento
0,10-0,20 mg/kg/min, vida intermedia atenolol IV, dosis carga 2.5 mg, mantenimiento 0,15 mg/kg/dia. No selectivos como: propanolol
dosis de carga 0.5-1 mg en 5 min, mantenimiento 0.05-0,015 mg/kg cada 4-6 hrs y los bloqueadores α1β2 (labetalol IV de vida media
larga en dosis crecientes 0.5-4 mg/min). En caso de hipertensión arterial se puede iniciar labetalol de primera elección. Si se requiere de
un control adicional de la presión arterial se suelen asociar nitroprusiato, nitratos, agonistas de calcio (nimodipino IV) o incluso IECAS
IV, Si los betas bloqueadores están contraindicados (EPOC) hay que pensar en verapamilo o diltiazem. El tratamiento farmacológico
crónico se reserva para los pacientes ya intervenidos con disección tipo B no complicada y requiere un control estricto de la presión
arterial, manteniéndola por debajo de 130-135/80mmHg. El pilar del tratamiento son los bloqueadores beta, asociando IECA o

CURSO ENARM CMN SIGLO XXI TEL: 36246001 Pharmed Solutions Institute PÁGINA 209
MANUAL DE TRABAJO DEL CURSO ENARM CMN SIGLO XXI
antagonista de calcio, si se precisa o el paciente no tolera lo bloqueadores beta. Se debe realizar revisiones periódicas y pruebas de
imagen, se recomiendan a los 1, 3, 6, y 12 meses y posteriormente anuales. Tratamiento quirúrgico: Objetivo salvar la vida del paciente
previniendo la rotura de la aorta torácica ascendente en el pericardio. El tratamiento de la disección aortica tipo A es quirúrgico (I B),
excepto en situaciones de edad avanzada, comorbilidad importante o daño neurológico. La disección tipo B tiene un mejor pronostico.
Su abordaje inicial es médico y se reserva el tratamiento quirúrgico en la fase aguda a las disecciones complicadas. TIPO A: Reemplazar
la aorta ascendente disecada, con un injerto sintético de dacrón, la zona resecada debe incluir la zona con la rotura intimal. La
mortalidad quirúrgica varía entre un 10-20% en función de la edad, la comorbilidad y la extensión de la disección. TIPO B: El tratamiento
médico tienen buenos resultados, este se centra inicialmente en el control de la presión arterial mediante bloqueadores beta y
vasodilatadores, y el alivio del dolor. Una vez superada la fase aguda, se continúan los bloqueadores beta orales y vasodilatadores. Un
seguimiento radiológico cada 6 meses será necesario para detectar precozmente una evolución tórpida. La cirugía queda limitada a:
dolor torácico persistente o recurrente, expansión aortica, hematoma periaórtico, hematoma mediastínico. En la actualidad, las
intervenciones endovasculares en la disección aortica aguda tipo B suelen limitarse a aliviar las complicaciones con riesgo de muerte.

CASO CLINICO
Se trata de paciente masculino de 46 años de edad, gerente de una tienda departamental, contador, con antecedente de tabaquismo
positivo (una cajetilla diaria) es hipertenso desde hace 5 años con mal apego a su tratamiento, acude a urgencias debido presencia de
dolor torácico que se irradia a la espalda, refiere que nunca había presentado un dolor así de intenso, que inicio hace 6 horas por la
mañana y se retiro de su trabajo porque comenzó a sudar y a presentar nauseas sin llegar al vomito, el dolor se ha vuelto lacerante, el
trazo electrocardiográfico se observa elevación ST de 1 mm en DII y aVF el paciente se encuentra taquicardico con pulso carotideo
salton.

PREGUNTA
Cual es el diagnostico mas probable hasta el momento.

RESPUESTA
a.- Infarto al Miocardio.
b.- Diseccion aortica.
c.- Prolapso valvular.
d.- Ruptura papilar.

ANEURISMA AORTICO (AA):


CIENCIAS BASICAS: Los aneurismas de la aorta (AA) se definen como la “dilatación focal de la arteria que supone un aumento de más
de 50% del diámetro esperado”, basado en medidas medias obtenidas en estudios con TAC en población general. En el caso de la aorta
abdominal correspondería a un diámetro superior a 3 cm. También se habla de AA como una dilatación localizada de por lo menos 1.5
veces mayor al diámetro normal de la arteria; puede ser sacular o fusiforme, y ambas están adyacentes a un segmento de arteria
normal. Esta patología se ha convertido en el motor del tratamiento quirúrgico preventivo de la cirugía vascular. Decimos que existe un
aneurisma verdadero cuando afecta a las tres capas histológicas; pero cuando la íntima y la media están rotas y la dilatación es a
expensas solo de la adventicia, entonces hablamos de seudoaneurisma. La aorta abdominal es el vaso donde se localizan con más
frecuencia los aneurismas arteriales. Hay aneurismas aórticos abdominales (AAA) y aneurismas aórticos de torácica descendente (AAT).
SALUD PÚBLICA: De los aneurismas aórticos 80% se ubican en el abdomen y tienen una presencia en la población de la tercera edad de
2 a 7%. Es mayor en hombres que en mujeres, en una proporción de 4:1. En pacientes con hipertensión arterial, enfermedad vascular
cerebral u otros aneurismas, su presencia aumenta 40%. Los AAA afectan a más de 3% de la población de edades superiores a los 50
años. Los factores de riesgo, como hipertensión arterial, tendencia familiar (se relaciona con cromosoma , autosómico, predisposición
genética), tabaquismo, diabetes, infecciones, dislipidemia y los relacionados con la ateroesclerosis, aumentan su prevalencia
significativamente. El riesgo proporcional para las personas que tienen un familiar de primer grado con AAA, es de 6:1 en comparación
con la población general. Como en un elevado porcentaje son asintomáticos, y la ruptura es la primera manifestación clínica, la
mortalidad real es muy alta. La probabilidad de ruptura al año del AAA menor de 5 cm es de 0.4-5.4%, entre 5-6cmde 20-25%, y de
>7cm entre 40-80%. PATOGENIA: La aorta es un conducto a través del cual la sangre impulsada, desde el ventrículo izquierdo, pasa y se
distribuye en el lecho arterial sistémico. El diámetro máximo en adultos corresponde en su origen a 3 cm, disminuyendo caudalmente,
siendo de 2.5 cm a nivel de la porción de aorta descendente torácica, hasta 1.8 a 2 cm en la porción abdominal de la misma. La pared
vascular está formada por la ín ma delgada, compuesta de endotelio, capa subendotelial de tejido conjuntivo y una capa interna
elástica; la túnica media, de células musculares lisas y matriz extracelular y una adventicia formada fundamentalmente por tejido
conectivo, que engloba los vasa vasorum y la inervación del vaso. Debido a su exposición continúa a la presión pulsátil y fuerzas de
sujeción, está par cularmente expuesta a sufrir lesiones secundarias a trauma mecánico, sobre todo en los casos de desaceleración,
siendo mayor el riesgo de ruptura y de aparición de dilataciones aneurismáticas. Desde el punto de la física, hay dos factores que
generan los AA: la excesiva aplicación de una fuerza interna y la inadecuada resistencia del material. Se piensa que el origen de los AAA
y AAT inespecíficos, es una disminución en la cantidad de elastina de la pared arterial. La cantidad normal de elastina que se encuentra
en las personas normales es de 12%, porcentaje que baja a 1% o menos en los pacientes con aneurismas detectado a través de un
marcador genético; el aumento de la actividad de la elastasa y de la colagenasa afectan al desarrollo del aneurisma. Se ha encontrado
un metabolismo anormal de la elastina y del colágeno, mayor turbulencia o disminución de vasa vasorum en esa zona,. Los AAA y AAT
principalmente, se deben a atero y arteriosclerosis; en menor proporción trastornos del tejido conjuntivo, traumatismos, síndrome de
Marfan, infección, necrosis quística de la media y arteritis. También pueden ser por fibrodisplasias o iatrogénicos. En la mayoría de los
casos no es posible identificar la causa. El AAA inflamatorio, que representa 10% de todos los aneurismas aórticos, puede deberse a una
infección o a alguna otra forma oscura de arteritis. La evolución natural de los aneurismas no tratados es hacia la expansión y la
ruptura siguiendo la Ley de Laplace. CLASIFICACION: En los AAT (Crawford), tiene en cuenta la longitud de la aorta afectada, resultando
en cuatro patrones que determinan la extensión de la intervención quirúrgica, el resultado del tratamiento y la naturaleza de las

CURSO ENARM CMN SIGLO XXI TEL: 36246001 Pharmed Solutions Institute PÁGINA 210
MANUAL DE TRABAJO DEL CURSO ENARM CMN SIGLO XXI
complicaciones. Tipo I; está afectada la mayor parte de la aorta descendente torácica y la parte proximal de aorta abdominal. Tipo II; el
aneurisma afecta gran parte de la aorta descendente y la mayor parte o toda la aorta abdominal. Tipo III; afecta la aorta torácica distal
y la totalidad de aorta abdominal. Tipo IV la mayor parte de la aorta abdominal, incluyendo el segmento de vasos viscerales.
DIAGNOSTICO: Clínico; Una minoría presenta dolor a la palpación abdominal o más com n dolor lumbar confundible con cólico renal.
Un bajo número de pacientes se presentan con dolor abrupto abdominal y en la región lumbar. Al examen físico se puede palpar, en
algunos casos, una masa pulsátil en el abdomen. La ruptura en el aneurisma es una forma de presentación con alta mortalidad, hay
extravasación de sangre, la cual puede ser masiva (hacia la cavidad peritoneal) o contenida (en el retroperitoneo). La tríada de
hipotensión, dolor abdominal y masa pulsátil en el abdomen, es observada sólo en 15% de los pacientes con aneurisma abdominal roto,
donde 78% de ruptura es hacia el retroperitoneo, y sólo 22% se rompe en su pared anterior hacia la cavidad peritoneal. tra
manifestación es la embolización distal de material trombotico que ocupa el saco aneurismá co, y esto ocurre en 3 a 5% de los
pacientes. Es obligada la exploración física del paciente mediante palpación profunda abdominal, buscando en algunos casos una masa
pulsátil en el mesogastrio, especialmente si el paciente es delgado. La presencia de soplos a la auscultación debe hacernos sospechar
enfermedad oclusiva visceral o de aorta terminal y más raramente la presencia de una stula aortocava con datos de falla cardiaca. El
estudio físico debe completarse con palpación de pulsos en extremidades, y en algunos casos pueden estar disminuidos o ausentes. Las
presentaciones clínicas pueden ser de un abdomen agudo por ruptura del AAA, embolias distales o sangrado de tubo digestivo cuando
se comunica la ruptura al duodeno. En los AAT los datos de compresión a bronquios, nervios o tubo digestivo pueden hacer sospechar
su presencia. Dx. De imagen: La mayoría de los AA se diagnostican en forma incidental (80% de AAA), desde la Rx simple en tórax, que
deforma el mediastino, o bien, la sombra de la aorta torácica descendente se hace más evidente. En la Rx de abdomen, además de las
calcificaciones se puede observar “borramiento” de las líneas del m sculo psoas. Asimismo, tanto en una evaluación ultrasonográfica,
TAC, RNM. El ultrasonido representa una modalidad económica, de fácil acceso y es también precisa para la medición del aneurisma.
Este estudio no es sensible para el diagnóstico de hemorragia retroperitoneal ni para la medición de la distancia que separa el
aneurisma de las arterias renales o las arterias iliacas. TRATAMIENTO: La mayoría de los centros quirúrgicos están de acuerdo en que
todo aneurisma de 5 cm o mayor debe ser operado en forma electiva o tratado en forma endovascular. Otra tendencia prefiere el
seguimiento cada seis meses e, independientemente del tamaño del aneurisma, un crecimiento mayor de 0.5 cm en este periodo es
indicación de tratamiento. Es conocido el riesgo quir rgico por enfermedad coronaria coexistente en estos pacientes. La modificación
de los factores de riesgo es fundamental, en el manejo medico de los AAA, la hipercolesterolemia y la HTA deben controlarse
adecuadamente. Los betabloqueadores (propanolol) han sido considerados como terapia para reducir el riesgo de expansión y rotura
del aneurisma. Una vez sospechado el diagnostico debe colocarse un acceso venoso de grueso calibre e infusión de cristaloides,
evitando o corrigiendo la situación de shock inicial; esta medida está directamente relacionada con la morbimortalidad. La mayoría de
los autores coinciden en que deberá tener como finalidad mantener tensión arterial sitolica en torno a 80-100mmHg. En las técnicas
quirúrgicas se puede reparar con injertos sintéticos de dacrón o PTFE (politetrafluoroetileno), en forma recta o bifurcada hacia las
iliacas o hacia las femorales, con resultados variables, este procedimiento quirúrgico seguirá teniendo vigencia, ya que 40% de los
pacientes no son candidatos para cirugía endovascular, por variantes anatómicas que dificulten el implante, pacientes jóvenes con
enfermedades asociadas: renal, pulmonar, cardiaca, carotidea de bajo riesgo que toleren bien la cirugía convencional. Manejo
endovascular: se desarrollo para hacerla vía transfemoral, siendo un procedimiento combinado, a través de la cual pasa la prótesis
montada del sistema para ser colocada bajo fluoroscopia a nivel del aneurisma. La elección de los pacientes es la clave del éxito. Existe
un protocolo estricto que debe incluir mediciones precisas del diámetro de las arterias iliacas.

CASO CLINICO
Se trata de paciente masculino de 59 años de edad el cual se encuentra bajo tratamiento por padecer hipertensión arterial desde hace
10 años, además hipercolesterolemia, actualmente recibe clortalidona, atenolol y pravastatina, se encuentra apegado a tratamiento sin
embargo continua fumando casi una cajetilla diaria. Acude a su consulta de revisión durante la cual le comenta que ha sentido una
bolita que pulsa a la altura del ombligo y quiere descartar un padecimiento cancerígeno ya que en la familia hay esos antecedentes, a la
exploración se encuentra con sus constantes vitales controladas, ninguna alteración cardiopulmonar, al revisar abdomen palpa una
masa depresible no dolorosa pero pulsatil a nivel de cicatriz umblical de al menos 3 dedos de longitud.

PREGUNTA
Cuál es su conducta a seguir.

RESPUESTA
a.- Solicita USG.
b.- Solicita TAC.
c.- Solicita IRM.
d.- Envia para LAPE.

CASO CLINICO
El paciente regresa con los resultados de la tomografía solicitada donde se reporta aneurisma de la aorta abdominal de 4.5 cm
infrarrenal, el paciente agrega que recuerda que desde hace algunos meses ha notado dolor ocasional de espalda que ha sido tratado
por su medico familiar con diclofenaco y medidas locales.

PREGUNTA
Cual es la conducta mas apropiada a seguir en este caso.

RESPUESTA
a.- Realizar estudios para intervención quirúrgica.
b.- Continuar con diclofenaco.

CURSO ENARM CMN SIGLO XXI TEL: 36246001 Pharmed Solutions Institute PÁGINA 211
MANUAL DE TRABAJO DEL CURSO ENARM CMN SIGLO XXI
c.- Vigilar al paciente.
d.- Vigilancia y antiagregante plaquetario.

CASO CLINICO
El paciente ha presentado algunas ocasiones episodios de descontrol hipertensivo, continua su habito de tabaquismo, los niveles de
colesterol no han cedido, el paciente refiere que ha cursado con mareos ocasionales acompañados de zumbido de oídos y luces
brillantes, ha mejorado su alimentación pero refiere aumento del dolor de espalda y sensación de presión interna “como si le pulsara”,
debido a los cambios observados se repite la TAC abdominal donde se reporta un incremento en las dimensiones del aneurisma de la
aorta abdominal pasando de 4.5 cm hace 2 años a 6.7 cm.

PREGUNTA
Considerando las nuevas evidencias el paciente es sometido a cirugía, cual es el criterio más importante para el tratamiento elegido.

RESPUESTA
a.- El tamaño del aneurisma.
b.- La presencia de dolor persistente.
c.- La presencia de descontrol hipertensión.
d.- Incremento de placas ateromatosas.

ANGINA ESTABLE, INESTABLE Y PRINZTMETAL


DEFINICION: La angina estable es un síndrome clínico caracterizado por malestar en el pecho, mandíbulas, hombros, espalda o brazos,
que aparece con el ejercicio o estrés emocional, dura de 2 a 5 minutos, remite espontáneamente, con el descanso o con la
administración de nitroglicerina, (las manifestaciones clínicas aparecen cuando al menos hay una oclusión del 70 % de la luz del vaso
afectado. EPIDEMIOLOGIA: En ambos sexos, la prevalencia de la angina aumenta marcadamente con la edad, de un 0,1-1% en mujeres
de edades comprendidas entre 45 y 54 años a un 10-15% en mujeres de 65-74 años y de un 2-5% en varones de 45-54 años a un 10-
20% en varones de 65-74 años. Con base en estos datos, se calcula que, en la mayoría de los países europeos, entre 20.000 y 40.000
individuos por millón de habitantes sufren angina. El principal factor de riesgo es la aterosclerosis. ETIOLOGIA: No hay factores
espeficificos solo relativos como aterosclerosis, hipertensión, dislipidemia, enfermedad arterial coronaria, alteraciones vasculares,
retinianas, soplos, infarto, presión yugular edema pulmonar, soplo mitral. FISIOPATOGENIA: Los datos anginosos se presentan cuando
la demanda de oxigeno por el miocardio se incrementan, y no existe una respuesta coronaria adecuada para mantener el aporte, por lo
regular se encuentran los síntomas anginosos cuando hay una obtruccion coronaria cercana al 90 % con una distensibilidad reducida de
la coronaria afectada. Frecuentemente los pacientes con obsetrucciones menores a 70 % no muestran síntomas anginosos. CUADRO
CLINICO: Sensacion de opresión, tensión o pesadez y sensación de estrangulamiento, constricción o quemazón. La severidad puede
variar mucho. Falta de aire y malestar torácico, síntomas como fatiga, náusea, eructos, ansiedad ó sensación de muerte inminente.
DIAGNOSTICO: Angina estable: El episodio de angina es breve, generalmente menos de 10 min. Relación con el ejercicio, una actividad
o el estrés emocional y remitan rápidamente en unos minutos cuando desaparezcan los factores causales. Otra característica típica de
la angina es la acentuación de los síntomas después de una comida pesada o a primera hora de la mañana. Los nitratos orales o
sublinguales alivian rápidamente la angina. Angina inestable: a) angina en reposo (durante el reposo y períodos prolongados, 20 min);
b) angina in crescendo (angina estable previa que evoluciona rápidamente en severidad e intensidad y con un umbral más bajo durante
un período corto, de 4 semanas o menos), y c) angina de reciente aparición (episodio reciente de angina severa que limita
marcadamente la actividad diaria del paciente y se presenta durante los 2 meses posteriores al episodio inicial). En la angina inestable
hay evidencia de placas con rotura, hemorragia o laseracion lo cual las vuelve mas inestables y trombogenicas. Angina de Prinzmetal:
Presentan dolor de localización típica, que ocurre en reposo o sólo ocasionalmente, durante el ejercicio. Puede coincidir con la angina
típica de esfuerzo por lesiones coronarias fijas. El vasoespasmo puede ocurrir como respuesta al consumo de tabaco, alteraciones
electrolíticas (potasio, magnesio), consumo de cocaína, estimulación por frío, enfermedades autoinmunitarias, hiperventilación o
resistencia a la insulina. El ECG no es concluyente, prueba de esfuerzo en banda sin fin, ecocardiograma de esfuerzo, ecocardiograma
con estimulación con dobutamina, coronariografia (estándar de oro), cuando hay datos anormales, biomarcadores proteína C de alta
especificidad y péptido natriuretico cerebral. TRATAMIENTO: Antianginosos BB; reducen el consumo de O2, disminuyen TA, FC y
contractibilidad, bloqueadores de canales de calcio, aumentan el flujo coronario y disminuyen el consumo de O2, verapamil o diltiazem
pero con mayor de 30% de FE o disfunción sinusal o nodal; nitritos: vasodilatadores venosos con cierto efecto coronario, disminuyen
precarga, isosorbide o nitroglicerina de 0.2 a 0.6 mg sblingual responde en 1 a 3 minutos y puede repetirse cada 5 minutos. Aspirina
150mg, clopidrogel si hubo intervencionismo, atorvastatina 80 mg/dia e IECA. PRONOSTICO: En varones y mujeres con una
presentación clínica inicial de angina estable, la incidencia de infarto de miocardio no fatal y muerte por enfermedad coronaria a los 2
años era del 14,3 y el 5,5% en varones y del 6,2 y el 3,8% en mujeres, respectivamente. La tasa anual de mortalidad varía entre el 0,9 y
el 1,4% por año 5-9, con una incidencia anual de infarto de miocardio no fatal del 0,5% y el 2,6%. COMPLICACIONES: En caso de no
obtener buenos resultado o si continua la evolución tórpida de la angina se podrá realizar angioplastia o cirugía de derivación cardiaca,
sin haber evidencia de sobrevida mayor con cualquier técnica.

CASO CLINICO
Se trata de paciente masculino de 52 años de edad, el cual acude a consulta externa debido a que desde hace 6 meses presenta
opresión toracicca, refiere que considera que es por las presiones que tiene en el trabajo y en casa se encuentra en proceso de divorcio,
refiere que el dolor dura menos de 5 minutos, desapareciendo respirando profundamente y reposando, agrega que últimamente ha
presentado dolor epigástrico acompañado de nauseas independiente del malestar en el torax para lo cual emplea antiácidos casi diario,
cuenta con antecedentes de importancia por tabaquismo y alcoholismo semanal hasta la embriaguez, asi como padre finado por IAM,
madre viva con hipertensión arterial con tratamiento, tiene habitos alimenticios inadecuados, ricos en grasas y comida rápida, por el
trabajo y no realiza ejercicio a la exploración física se observa con leve rubicundez facial, aliento al parecer alcoholico, presenta signo de

CURSO ENARM CMN SIGLO XXI TEL: 36246001 Pharmed Solutions Institute PÁGINA 212
MANUAL DE TRABAJO DEL CURSO ENARM CMN SIGLO XXI
Levine al interrogar sobre el sitio del dolor, queratosis seborreica importante y xantomas, al parecer tercer ruido en area precordial y
estertores subcrepitantes bibasales. Signos vitales TA 150/95 mmHg, FR 28 rpm, FC 104 lpm.

PREGUNTA
Cuáles son las manifestaciones clínicas de la enfermedad mas importantes para considerar un dianostico presuntivo?

RESPUESTA
a.- Presencia del signo de Levine.
b.- Dolor torácico.
c.- Disminución del dolor con el reposo.
d.- La duración del dolor.

PREGUNTA
Cuál es la manifestacion clínica que no es criterios diágnosticos para establecer la enfermedad del paciente?

RESPUESTA
a.- Molestia toracicca subesternal.
b.- Inicio por estrés o ejercicio.
c.- Disminuye con nitroglicerina.
d.- Sexo, mas frecuente en mujer.

PREGUNTA
Considerando la clasificación de la enfermedad cual de los siguientes diagnosticos es el mas probable para el caso actual?

RESPUESTA
a.- Angina definitiva.
b.- Angina probable.
c.- Angina posible.
d.- Angina no posible

PREGUNTA
Considerando la clasificación funcional de la Canadian cardiovascular society, cual es la clase actual del paciente?

RESPUESTA
a.- Clase I.
b.- Clase II.
c.- Clase III.
d.- Clase IV.

PREGUNTA
Cuál es el diagnostico diferencial mas probable en este caso debido a los factores modificables de riego presentes?

RESPUESTA
a.- Angina de prinzmetal.
b.- Neuritis intersticial.
c.- Sindrome de la pared toracicca anterior.
d.- Enfermedad gastrointestinal funcional.

PREGUNTA
Se realizaron los siguientes axiliares diagnosticos, cual de los siguientes resultados no excluye el diagnostico actual?, “revisar las
imágenes”

RESPUESTA
a.- Prueba de esfuerzo.
b.- Electrocardiograma en reposo.
c.- Radiografia de torax.
D.- Arteriografia coronaria.

PREGUNTA
Antes de realizar al paciente una coronariografia se encontraron los siguientes resultados, previos cual de ellos no es criterio?

RESPUESTA
a.- Depresión del segmento ST >1 mm.
b.- Prueba de esfuerzo anormal.
c.- Sintomatologia presente.
d.- Resultados inconclusos.

CURSO ENARM CMN SIGLO XXI TEL: 36246001 Pharmed Solutions Institute PÁGINA 213
MANUAL DE TRABAJO DEL CURSO ENARM CMN SIGLO XXI

PREGUNTA
Durante las pruebas se encontraron los siguientes resultados de laboratorio en el paciente: Creatinina 1,24, Glucosa 224, Colesterol
310, Trigliceridos 420, HDL 29, Urea 41, Na 138, K 3,42, Troponina T-U: 2,8, CK 105, CK-MB 1,67, ALT-GPT 19, Hemograma y coagulación
normal. En relación a datos de laboratorio obtenidos cuales son los objetivos terapéuticos mas importantes?

RESPUESTA
a.- Incrementar el HDL.
b.- Disminuir triglicéridos y colesterol.
c.- Revascularización inmediata.
d.- Buscar proteína C de alta sensibilidad.

PREGUNTA
Cuáles son las medidas terapéuticas no farmacológicas inicial en este padecimiento, para disminuir la demanda miocárdica de oxigeno?

RESPUESTA
a.- Propanolol.
b.- Hidroclorotiazida.
c.- Losartan.
d.- Enalapril

PREGUNTA
Considerando los datos de lípidos y el informe de la coronariografia, que medida farmacológica es necesario agregar para estabilizar la
placa?

RESPUESTA
a.- Clopidrogel 75 mg cada 12 hrs.
b.- Atorvastatina 40 mg cada 12 hrs.
c.- Enalapril 20 mg cada 12 hrs.
d.- Aspirina 150 mg cada 24 hrs.

PREGUNTA
Cuáles son los siguientes esquemas terapéuticos secuenciales antianginoso tiene un efecto favorable sobre la presión arterial, la
contractilidad y la frecuencia cardiaca más adecuado?

RESPUESTA
a.- Prazocin 1 a 2 mg cada 24 hrs.
b.- Tamsulosina 0.4 mg cada 24 hrs.
c.- Clonidina 0.075 a 0.150 mg cada 24 hrs.
d.- Propranolol 40 a 80 mg en 24 hrs.

PREGUNTA
Cual de las siguientes comorbilidades es la menos frecuentes en la angina típica como es el caso?

RESPUESTA
a.- EPOC.
b.- DM.
c.- Espasmo vasocoronario.
d.- Disfunción eréctil.

PREGUNTA
Considerando la fracción de eyección del paciente, cual fármaco calcio antagonista puede usarse con cuidado comparado con el resto?

RESPUESTA
a.- Amlodipino.
b.- Verapamilo.
c.- Diltiazem
d.- Nifedipina

PREGUNTA
Considerando la fracción de eyección del paciente, cual fármaco calcio antagonista puede usarse con confianza comparado con el
resto?

RESPUESTA
a.- Nitroglicerina.
b.- Isosorbide.

CURSO ENARM CMN SIGLO XXI TEL: 36246001 Pharmed Solutions Institute PÁGINA 214
MANUAL DE TRABAJO DEL CURSO ENARM CMN SIGLO XXI
c.- Nitrito de amilo.
d.- Nitrito de potasio.

PREGUNTA
Considerando la patogenia de la angina de pecho cual de los siguientes síntomas están menos relacionados a la isquemia del miocardio
y si a la necrosis miocárdica?

RESPUESTA
a.- Disconfor o dolor en el pecho.
b.- Maxilar inferior ó brazo.
c.- Hombros y espalda.
d.- Elevación de CPK-MB.

PREGUNTA
Se le solicito al paciente que usara un diario para registra el dolor, cual es la razón fisiopatológica que el paciente presente mayor
frecuencia de dolor en la mañana?

RESPUESTA
a.- Debido a presencia de apnea.
b.- Debido al ritmio endógeno de cortisol.
c.- Debido al ritmo endógeno de secresión de catecolaminas.
d.- Debido a la insensibilidad coronaria a vasoconstrictores.

PREGUNTA
Cual es la frecuencia de angina estable crónica al momento de solicitar consulta al especialista?

RESPUESTA
a.- 50 %.
b.- 60 %.
c.- 40 %.
d.- 70 %.

PREGUNTA
Cuál es la edad más frecuente comparada con el caso del paciente para presentar angina típica vs angina atípica considerando además
en sexo?

RESPUESTA
a.- Hombres de 50 a 59 años.
b.- Hombres de 40 a 49 años.
c.- Hombres de 60 a 69 años.
d.- Hombres de 50 a 69 años

PREGUNTA
Considerando la escala de Duke Treadmill Scores para calificar el riesgo estratificado no invasivo cuál es la calificación que presenta
actualmente?

RESPUESTA
a.- Riesgo alto (3 % de mortalidad anual).
b.- Riesgo muy alto (5 % de mortalidad anual).
c.- Riesgo intermedio (1-3 % de mortalidad anual).
d.- Riesgo bajo (menor a 1% de mortalidad anual).

CASO CLINICO
Se trata de masculino de 68 años de edad al cual se encuentra diagnosticado y tratado por padecer angina típica desde hace 5 años, sin
embargo de 6 meses a la fecha el disconfort del torax ha sido moderado y en ocasiones durante el reposo, acude al servicio de
urgencias debido a que presenta opresión toracicca acompañado de dolor en hombro izquierdo que se iradia a el brazo, agrega que se
encontraba discutiendo cuando comenzó las molestias, no cedió al reposo y duro más del tiempo habitual, cuenta con los antecedentes
de hipertensión arterial desde hace 20 años con tratamiento actual con captopril 25 mg cada 12 hrs, hidralazina 25 mg /12 hrs, hace 5
años fue agregado propranolol, pravastatina y aspirina para el manejo de su angina inicialmente estable, a su ingreso se observa
diaforética, con palidez generalizada, ansiosa, con cianosis distal, sus constantes vitales fueron TA 105/70 mmHg, FC 101 lpm, FR 29
rpm.

PREGUNTA
Considerando el cuadro clínico cuál es diagnostico más probable hasta este momento?

RESPUESTA

CURSO ENARM CMN SIGLO XXI TEL: 36246001 Pharmed Solutions Institute PÁGINA 215
MANUAL DE TRABAJO DEL CURSO ENARM CMN SIGLO XXI
a.- Angina típica descompensada.
b.- Angina atípica inicial.
c.- Infarto agudo al miocardio.
d.- Insuficiencia cardiaca por angina previa.

PREGUNTA
Cuál de los siguientes auxiliares diagnostico es mas útil en este momento para mantener una conducta clinica adecuada?

RESPUESTA
a.- Buscar biomarcadores de isquemia.
b.- Realizar electrocardiograma.
c.- Coronariografia.
d.- Programar prueba de esfuerzo con dobutamina.

PREGUNTA
Cuál cual es su clase funcional con la clínica observada?

RESPUESTA
a.- Clase I.
b.- Clase II.
c.- Clase III.
c.- Clase IV

PREGUNTA
Se practicaron los siguientes estudios al paciente cual es su opinión sobre la radiografia y ECG?

RESPUESTA
a.- Datos de edema pulmonar.
b.- Datos de insuficiencia cardiaca.
c.- Datos de IAM.
d.- Datos de EPOC.

RESPUESTA
Cuál es la conducta inmediata más importante a seguir?

RESPUESTA
a.- Morfina, oxigeno, nitritos y antiagregantes.
b.- Diuretico, morfina, verapamilo y oxigeno.
c.- Diuretico, antiaginoso y trombolisis.
d.- Morfina, nitrito, trombolisis, enoxoparina.

SINDROME CORONARIO AGUDO I IAM SESS


DEFINICION: Conjunto de cuadros clínicos por los que se pone de manifiesto de forma aguda la isquemia miocárdica secundaria en
general, pero no exclusivamente, por ateroesclerosis coronaria, sin embargo tiene su base en la erosion de una placa aterosclerótica de
las arterias coronarias, lo cual activa la coagulación culminando con la formación de un trombo oclusivo. EPIDEMIOLOGIA: 2.5 millones
de ingreso a urgencias, 16.6 millones de muertes, primera causa de muerte, 1 muerte cada minuto. Alta morbilidad e incapacidad. Se
estima que actualmente el 50% de todas las muertes en los países de alto ingreso y el 28% de las muertes en los PBMI son el resultado
de ECV. En 2001, el 31% de todas las muertes en Latinoamérica (LA). Factores de riesgo cardiovascular clasicos: DM, HAS, Tabaquismo,
LDL, estilo de vida, genes. Factores de riesgo cardiovascular actual; Infeccioso, inflamatorio, homocisteina, protrombosis. ETIOLOGIA:
No existe un factor etiológico especifico, son una serie de sucesos concurrentes, la presencia de una placa ateromatosa que se
erosiona, inicia una hemorragia que activa la coagulación generando un fenómeno oclusivo, es conveniente observar que no es
necesario la presencia de angina previa, ni la presencia de grandes placas en las coronarias. FISIOPATOGENIA: Las placas ateromatosas
consideradas como susceptibles a la erosion, en el caso del SICA 1, principalmente la centrolipidicas y con una delgada capa de fibrina,
la desestabilización y consecuente rotura de la placa es consecuencia principal de la circulación de factores inflamatorios locales y
sistémicos, asi como los cambios fisiológicos de las arterias coronarias por demanda u otros fenómenos, al presentarse lisis de la
delgada capa de fibrina se activa la cascada de coagulación, adeshion y agregación plaquetaria por vol willebrand, glucoproteinas
IIb/IIIa, tromboxano A, epinefrina, serotonina y factor activador de plaquetas, aumentando la afinidad de la protrombina y el
fibrinógeno convirtiéndose en trombina y fibrina respectivamente consecuentemente acumulo de eritrocitos y plaquetas formando un
trombo el cual no necesariamente es oclusivo, de ello dependerá en gran medida las modificaciones del ECG. CUADRO CLINICO: El
paciente refiere dolor opresivo o transfictivo retrosternal, con irradiación a cuello, brazo o mandibula, síntomas agregados, nauseas,
vomito, diaforesis, y sensación de muerte inminente. DIAGNOSTICO: Se observan manifestaciones clínicas referidas, alteraciones
electrocardiográficas y biomarcadores, (troponinas y CPK-MB) que no se elevan inmediatamente. TRATAMIENTO: Inmediato:
evaluación general, toma e interpretación de ECG, búsqueda de factores para fibrinoliticos, toma de biomarcadores y rutina.
Secundario: morfina, oxigeno, nitritos y antiagregantes. Terapia de reperfusion: en caso de no existir contraindicaciones y por
factibilidad se indica terapia fibrinolitica; debe realizarse dentro de las primeras 12 horas de iniciado el cuadro. PRONOSTICO: El manejo
firbinolitico favorece del 60 al 80 % de los pacientes, es bueno cuando se realiza mas temprano, además de administrar heparina para

CURSO ENARM CMN SIGLO XXI TEL: 36246001 Pharmed Solutions Institute PÁGINA 216
MANUAL DE TRABAJO DEL CURSO ENARM CMN SIGLO XXI
evitar la reoclusiones. COMPLICACIONES: La arritmias cardiacas sin una de las complicaciones mas importantes, asi como el choque
cardiogenico, otras complicaciones vienen del uso de fibrinoliticos.

CASO CLINICO
Mujer de 66 años, hipertensa de 10 años de evolución con atorvastatina y enalapril actualmente con adecuado control, acudió por
opresión torácica en reposo de 2 h de evolución. Al ingreso se observo con leve palidez de tegumentos con sensación de ahogo y disnea
aun con oxigeno suplementario, TA de 165/88mmHg, obesidad troncular y soplo sistólico II/VI en focos de la base. El ECG mostró
elevación del ST en cara inferior y de V2 a V6. Se realizó fibrinolisis, se encontraba en las primeras 2 hrs de evolución. Se administró
enoxaparina, ácido acetilsalicílico, nitroglicerina y morfina. Un ecocardiograma urgente demostró acinesia del ápex y la cara anterior, y
se estimó una fracción de eyección del 40%. Se observó un ligero derrame pericárdico posterior.

PREGUNTA
Cual de las manifestaciones del cuadro clínico es el menos importante para la decisión para realizar fibrinólisis en este caso?

RESPUESTA
a.- Antecedente de angina inestable.
b.- El tiempo de inicio del cuadro clínico.
c.- La auscencia de factores de riesgo a la fibrinólisis.
d.- La presencia de cambios electrocardiográficos.

PREGUNTA
Cuál de los antecedentes del caso es el de menor importancia para el actual IAM?
a.- Hipertension arterial previa.
b.- Antecedente de angina.
c.- Sobrepeso.
d.- Uso de atorvastatina subterapeutica.

PREGUNTA
Considerando los cambios electrocardiográficos, cuales de las arterias es la menos probable que se encuentre involucrada.

RESPUESTA
a.- Desendente anterior (ramos septales).
b.- Desendente anterior (ramos diagonales).
c.- Circunfleja.
d.- Desendente posterior.

PREGUNTA
Cuál es la complicación mas probable considerando el sitio de compromiso coronario?

RESPUESTA
a.- Bloqueos de rama.
b.- Disfunción ventricular izquierda.
c.- Insuficiencia cardiaca.
d.- Hipotensión.

PREGUNTA
Considerando las características del paciente, cual de los siguiente fármacos favorece la reducción de la extensión del daño miocárdico,
ruptura miocárdica o reinfarto?

RESPUESTA
a.- Nitritos.
b.- Betabloqueadores.
c.- IECAS.
d.- Calcioantagonistas.

PREGUNTA
Cuál de los cambios bioquímicos no son factores que desencadenan los cambios de actividad plaquetaria en este caso?

RESPUESTA
a.- Factor de von Willebrand.
b.- Glucoproteina IIb/IIIa.
c.- Produccion de tromboxano A2.
d.- Leucotrienos.

PREGUNTA
Cuál de las siguientes manifestaciones bioquímicas no espera encontrar en el paciente en cuestión?

CURSO ENARM CMN SIGLO XXI TEL: 36246001 Pharmed Solutions Institute PÁGINA 217
MANUAL DE TRABAJO DEL CURSO ENARM CMN SIGLO XXI

RESPUESTA
a.- Creatinincinasa se eleva entre las 4 y 8 hrs después del comienzo de la sintomatologia.
b.- La fracción MB de la CPK a partir de las 3 del comienzo del infarto.
c.- La troponina I se eleva a partir de que se inicia la sintomatologia.
d.- La mioglobina es la ultima en enzima que se eleva.

CASO CLINICO
Varón de 66 años ingresado por IM lateral evolucionado, no tratado con fibrinolíticos por criterios de tiempo. Tras 72 h sin
complicaciones, bajo tratamiento con nitratos. En el séptimo día de ingreso presentó TV polimorfa sincopal que precisó cardioversión
eléctrica urgente. En las horas siguientes se objetivaron numerosos episodios de taquicardia sostenida y no sostenida, con un
comportamiento incesante y degeneración ocasional en fibrilación ventricular (FV), precisando múltiples choques externos. Estos
episodios no eran precedidos de bradicardia significativa, prolongación anormal del intervalo QT, cambios en el segmento ST o dolor
torácico. Los valores de las enzimas cardíacas dentro de rango normal.

PREGUNTA
Cual es el manejo farmacológico mas adecuado.

RESPUESTA
a.- Lidocaina.
b.- Amiodarona.
c.- Verapamilo.
d.- Propanolol.

SICA II (IAM SESST)


INTRODUCCION: Se trata de proceso coronario isquémico inestable, SICA II o Infarto miocárdico agudo sin elevación del segmento ST,
divido en riesgo alto, medio y bajo, lo que permite considerar tratamiento de reperfusion inmediato o no, condición dinámica que
puede evolucionar de angina inestable a infarto al miocardio, teniendo como punto cardinal la elevación de enzimas cardiacas.
DIAGNOSTICO: Cuadro clínico caracterizado por dolor retrosternal de tipo opresivo, definido como opresivo, acompañado o no con
sensación de pensantez torácica, irradiación a brazo, cuello, hombro, puede presentar dolor en espalda, síntomas generales, debilidad,
mareo y nausea, acompañado por falta de aire. ECG especialmente se considera la falta de elevación del segmento ST, pero podemos
observar alteraciones con depresión o aplanamiento de onda T, o del segmento completo. Finalmente para establecer el diagnostico
deberán buscar enzimas cardiacas, si hay elevación el diagnostico será SICA SESST. Carateristicas del riesgo (Alto) multiples factores de
riesgo para enfermedad coronaria, angina en reposo mas de 20 minutos, o posinfarto, elevación del ST, depresión de ST > 1 mm en 2
derivaciones continuguas, Inestabilidad hemodinámica, signos de insuficiencia cardiaca o disfunción ventricular izquierda,
biomarcadores sericos elevados. Clasificación riesgo (Intermedio) DM antecedentes de IAM, angina prolongada resuelta, Inversion de T
en 5 derivaciones, biomarcadores normales con marcadores inflamatorios elevados. Riesgo (Bajo) Angina inestable o cambiante,
Biomarcadores o marcadores inflamatorios normales, ECG normal o sin cambios. PATOGENIA: El SICA II es casusado por ruptura o
erosion de una placa ateromatosa con formación de trombo plaquetario que genera consecuentemente obstruccion
predominantemente parcial del flujo sanguíneo coronario. Es un fenómeno dinamico que trata de mantener un balance entre la
demanda del miocardio y la demanda que se observa durante la angina inestable y infarto miocárdico sin elevación del segmento ST,
las cuales incluyen, 1.- reducción de la luz arterial coronaria debido a un trombo no oclusivo que se desarrolla seguido a ruptura o
erosion de una placa aterotrombotica. 2.- severo estrechamiento de arteria coronario sin espasmo o trombosis por arteriosclerosis
progresiva o con reestenosis con mas de 6 meses posterior a intervención coronaria percutánea. 3.- Espasmo focal intenso de segento
epicardico coronario arterial (prinzmetal o angina variante) causando obstruccion dinámica del lumen arterial. 4.- Diseccion coronaria
arterial (causado posparto) 5.- En presencia de factores precipitantes extrínsecos a las arterias coronarias que limitan la perfusión
miocárdica, causando una alteración súbita de la demanda miocárdica de oxigeno como (sepsis, fiebre, taquicardia) o reducción de flujo
coronario (hipotensión) o disminución de la entrega de oxigeno al miocardio (hipoxia, anemia severa, etc). TRATAMIENTO: Reposo y
comodidad absoluta, ECG de 12 derivaciones identificando el patron ST posteriormente establecer estratificación del riesgo lo cual
permite decidir el tratamiento, se inicia co
inmediata: angioplastia primaria o trombolisis; AI/SICA SESST (riesgo alto e intermedio) con depresión del ST o cambios dinamicos de T
(inversión) o elevación enzimática
angioplastica coronaria percutánea; IA/SICA SESST (riesgo bajo) con ECG no diagnostico o normal, enzimas normales, se indicara
tratamiento medico, monitorización, estudios de isquemia para estratificación. Analgesicos: morfina indicada si los nitritos no reducen
el dolor, en paciente ansioso o agitado, en edema agudo de pulmon, reduce la precarga y la poscarga, el consumo de oxigeno
miocárdico, sedante, venodilatador. O2 a 4 Ltx´ vigilando saturación (90%), Nitroglicerina via sublingual hasta 3 dosis intervalo de 5
minutos, monitorizar FC y TA, con persistencia de dolor inicio de via IV, su efecto es venodilatador, cuidado en infarto inferior o
derecho, en hipotesion, bradicardia o taquicardia, evitar si hay uso previo de inhibidores de fosfodiesterasa. Aspirina de 160 a 325 mg
de absorción rápida; Clopidrogrel inhibidor irreversible del receptor del adenosindifosfato 300 mg dosis única. Anticoagulantes,
heparina no fraccionada no propaga el trombo sin utilidad lítica a 60 a 70 U por Kg mantenerTPT menor a 75 segundos. Heparina de
bajo peso molecular, enoxoparina mayor seguridad de sangrado. Fibrinoliticos, deben evitarse por riesgo beneficio ya que en SICA II
SESST la arteria se encuentra abierta. PRONOSTICO: SICA II SESST presenta morbilidad y mortalidad expectante por las condiciones de
la enfermedad coronaria de base, la función sistólica del ventrículo izquierdo, la estabilidad de la lesión en si misma, la mortalidad por
angina inestable es de 3.6 % a 6 meses y 6.2 % en caso de infarto al miocardio sin elevación del segmento ST.

CURSO ENARM CMN SIGLO XXI TEL: 36246001 Pharmed Solutions Institute PÁGINA 218
MANUAL DE TRABAJO DEL CURSO ENARM CMN SIGLO XXI
CASO CLINICO
Hombre de 48 años, con antecedente familiar de cardiopatía coronaria, tabáquico y dislipidémico sin tratamiento farmacológico.
Presentó intenso dolor precordial mientras jugaba fútbol, consultando en un servicio de urgencia donde presentó paro
cardiorrespiratorio por fibrilación ventricular. Fue tratado con múltiples desfribilaciones y maniobras de reanimación durante 25 min. El
ECG no mostro datos sugerentes de infarto. Al ingreso se encontraba intubado con asistencia ventilatoria mediante ambú,
hemodinámicamente inestable requiriendo infusión de noradrenalina a 0,1 µg/kg/min. 2 horas después se observo cambios en el ECG
mostró onda Q y supradesnivel del segmento ST hasta 3 mm en VI a V3. Troponina I: 292 ng/mL (normal <0,05).

PREGUNTA
Cual de los criterios es más determinante para considerar infarto al miocardio sin elevación del segmento ST comparado con angina
inestable?

RESPUESTA
a.- Cambios bioquímicos.
b.- Presentación sintomática.
c.- Cambio electrocardiográficos sin elevación de ST.
d.- Factores de riesgo previos.

PREGUNTA
Considerando el tipo de manifestaciones clínicas, electrocardiográficos y biomarcadores cual de las siguientes conductas terapéuticas
no se recomienda en el IAM SESST.

RESPUESTA
a.- Reperfusion inmediata.
b.- Inhibidores de receptores de glucoproteinas.
c.- Heparina de bajo peso molecular.
d.- Angioplastia coronaria percutánea.

PREGUNTA
Cuál es la complicación más probable del paciente.

RESPUESTA
a.- Choque cardiogenico.
b.- Arritmia mortal.
c.- Ruptura de musculos papilares.
d.- Falla organica multiple

CASO CLINICO
Hombre de 72 años, con historia de hipertensión de 22 años de evolución tratada con enalapril 40 mg, diabetes mellitus 2 desde hace
20 años tratada con glibenclamida 50 mg, hipercolesterolemia manejada con atorvastatina 20 mg e hipertrigliceridemia tratada con 200
mg, actualmente se encuentra en cardiología por dolor torácico recurrente, fue admitido con quejas de "opresión en el pecho", que se
irradio a mandibula, hombro y brazo izquierdo de 105 minutos de duración que sedio reposo absoluto y nitroglicerina sublingual, el
comienzo fue súbito y acompañado de disnea, nauseas. Se encontraron los siguientes resultados patológicos 140/95mmHg, FC 107 lpm,
FR 29 rpm, temperatura 37.8 grados, glicemia 235 mg/dl, Colesterol 429 mg/dl, Trigliceridos 650 mg/dl. Electrolitos sericos, K 5.5, El
ECG realizado en la admisión mostró fibrilación atrial con frecuencia ventricular rápida y Segmento ST y Onda T normales. El examen
físico no mostró alteraciones significativas, pero hubo elevación de biomarcadores cardíacos, con pico de troponina I de 1,84 ng/ml y
CKMB-masa 13,4 ng/ml. El diagnóstico inicial no reveló infarto agudo de miocardio con elevación del Segmento ST, y la angiografía
coronaria se muestra. El ecocardiograma reveló una FE >40.

PREGUNTA
Considerando la sintomatología observada en el paciente que clase funcional killip y Kimball.

RESPUESTA
a.- Clase funcional Killip I.
b.- Clase funcional Killip II.
c.- Clase funcional Killip III.
d.- Clase funcional Killip IV.

PREGUNTA
La presencia de DM acelera el proceso de aterogénesis a través de diversos mecanismos, cual de los siguientes mecanismos es mayor
en el paciente actual?

RESPUESTA
a.- Anomalías en concentraciones y la composición de las lipoproteínas.
b.- Asociación con la hipertensión.
c.- Oxidación lipídica.

CURSO ENARM CMN SIGLO XXI TEL: 36246001 Pharmed Solutions Institute PÁGINA 219
MANUAL DE TRABAJO DEL CURSO ENARM CMN SIGLO XXI
d.- Estado procoagulante y proinflamatorio.

PREGUNTA
Cuál es los paraclinicos en el primer nivel de atención es de mayor utilidad para confirmar el diagnóstico de la insuficiencia cardíaca en
el paciente?

RESPUESTA
a.- ECG.
b.- Banda sin fin.
c.- Ecocardiograma.
d.- Arteriografia.

PREGUNTA
Cual de los siguientes factores comorbidos presenta la mayor importancia para desencadenar un evento coronario en el paciente.

RESPUESTA
a.- Diabetes mellitus.
b.- Hipertensión arterial.
c.- Dislipidemia.
d.- Tabaquismo.

PREGUNTA
Cual de los siguientes datos patológicos en el paciente presenta la mayor importancia para desencadenar un evento coronario en el
paciente.

RESPUESTA
a.- Niveles de glucosa.
b.- Hemoglobina glucosilada.
c.- Trigliceridos
d.- Colesterol.

PREGUNTA
Cual es la conducta a seguir con los siguientes criterios es el menos importante en este paciente para envio inmediato a tercer nivel?

RESPUESTA
a.- Dolor toracicco sugerente de angor pectoris.
b.- Dislipidemia.
c.- Hipertensión arterial.
d.- Diabetes mellitus.

PREGUNTA
Cual de los siguientes diagnosticos diferenciales es el menos probable considerando las condiciones del caso?

RESPUESTA
a.- Hipertensión arterial pulmonar.
b.- Pericarditis.
c.- Sindrome de Tietzl.
d.- Reflujo gastroesofagico.

PREGUNTA
Cual de las siguientes medidas debe iniciar antes de enviar al paciente a segundo nivel cuando existe alta sospecha de cardiopatía
isquémica?

RESPUESTA
a.- Dieta y ejercicio progresivo.
b.- Acetilsalicilico y pravastatina.
c.- Diuretico y betabloqueador.
d.- Estudios básicos y gabinete.

CASO CLINICO
Se presenta el caso de un paciente de 17 años de edad, sin antecedentes de interés, que acude a urgencias por un dolor centrotorácico,
opresivo, no irradiado, de 48 hrs de evolución, sin fiebre ni otra sintomatología acompañante. El paciente refiere haber consumido
cocaína y cannabis previamente al inicio de la sintomatología. A su llegada se encuentra estable, sus constantes son correctas y la
exploración física es normal. El electrocardiograma muestra elevación del segmento (ST) en las derivaciones de la cara inferior; la
analítica confirma una elevación de las enzimas cardíacas (creatincinasa [CK] de 1.194 U/l y troponina-I de 19,6 ng/ml).

CURSO ENARM CMN SIGLO XXI TEL: 36246001 Pharmed Solutions Institute PÁGINA 220
MANUAL DE TRABAJO DEL CURSO ENARM CMN SIGLO XXI
PREGUNTA
Cual es terapéutica mas apropiada para este caso.

RESPUESTA
a.- Nitroglicerina, morfina, trombolitico y oxigeno.
b.- Nitroglicerina, morfina, trombolitico y betabloqueador.
c.- Angioplastia percutánea, nitoglicerina, morfina, trombolitico y betabloqueador.
d.- Betabloqueador, morfina, betabloqueador.

CASO CLINICO
Se trata de femenino de 23 años de edad la cual llega a consulta refiriendo dolor torácico intermitente, durante la atención de
urgencias de descarto cuadro isquémico, con marcadores negativos asi como ECG reportado como normal, la prueba de esfuerzo fue
positiva reversible, se administra acido acetilsalicilico pero la paciente refiere malestar continuo por dolor torácico intermitente, sin
embargo continua con sus actividades cotidianas, no cuenta con antecedentes familiares de cardiopatía isquémica, sin embargo es
fumadora social.

PREGUNTA
Cuál es la conducta a seguir para establecer una aproximación diagnostica.

RESPUESTA
a.- Realizar cateterismo cardiaco.
b.- Realizar tomografía computada cardiaca.
c.- Realizar ecocardiograma de estrés.
d.- Evaluación del dolor de origen no cardiaco.

CURSO ENARM CMN SIGLO XXI TEL: 36246001 Pharmed Solutions Institute PÁGINA 221
MANUAL DE TRABAJO DEL CURSO ENARM CMN SIGLO XXI
ARRITMIAS CARDIACAS:
CIENCIAS BASICAS: La arritmia cardiaca se define como la alteración del ritmo cardiaco establecido como normal entre 60-100 lpm.
Menor a 60 lpm, será bradicardia y si es mayor a 100 lpm, será taquicardia. La arritmia más frecuente es la fibrilación auricular. SALUD
PUBLICA: La prevalencia de FA oscila 0,7-17,6%, La prevalencia de las otras 2 taquiarritmias supraventriculares más importantes, el
flutter auricular y la taquicardia paroxística supraventricular (TPSV), es mucho más baja. La incidencia del flutter auricular es de 88 por
1,000,000 de personas/año. ACTUACION INICIAL EN SERVICIO DE URGENCIAS: Hay que sistematizar nuestra actuación para no pasar
por alto aspectos esenciales. 1. Realizar ECG; verlo con calma y valorar los siguientes aspectos: a) si el ritmo es rápido o lento, b) si el
ritmo es regular o irregular, c) si el QRS es estrecho o ancho, d) le presencia o no de ondas “p”, e) la relación de p/QRS, esto nos
orientará la arritmia a la entidad etiológica más probable. 2. Monitorizar las constantes vitales: como la tensión arterial, saturación de
oxigeno, frecuencia cardiaca y respiratoria, vigilar la perfusión tisular, para detectar precozmente los signos de bajo gasto cardiaco.
Todas las alteraciones terapéuticas deben documentarse con tiras de registro de ECG. 3. Canalizar una vía venosa periférica y
administrar oxigeno: es necesario ya que el shock puede no detectarse precozmente y la mala perfusión periférica dificulta obtener una
via. El oxígeno lo administraremos en función de la oxigenación, para evitar hiperoxigenación. 4. Valorar estabilidad hemodinámica-los
signos adversos y que indican mala evolución son; a) shock-palidez, sudoración, por aumento de actividad simpática, bajo nivel de
conciencia, por disminución de FSC o hipotensión arterial. b) sincope-por disminución de FSC. c) insuficiencia cardiaca-edema pulmonar
o fallo del VI o ingurgitación yugular, hepatomegalia por fallo de VD. d) isquemia miocárdica-por aumento de la demanda de oxigeno
del miocardio. Si el paciente tolera bien la arritmia probablemente no haga falta hacerle nada o bien solo fármacos. Si la tolera mal, al
final precisara una cardioversión eléctrica. Dos aspectos importantes en atención primaria: primero no debemos ser más agresivos con
nuestra actuación que la propia arritmia lo es con el paciente y en segundo lugar, los fármacos antiarrítmicos combinados o a dosis
altas pueden ser arritmogénicos: para evitar efectos secundarios es recomendable seguir la máxima de “un paciente, un solo
antiarrítmico”. Adenosina y ATP son los nicos que escapan a esta máxima. DIAGNOSTICO DIFERENCIAL DE BRADICARDIAS:

Ritmo rápido o lento Lento


Ritmo rápido o lento Lento Ritmo regular o irregular Regular
Ritmo regular o irregular Regular QRS estrecho o ancho Estrecho
QRS estrecho o ancho Estrecho nda “p” Presente
nda “p” Presente Relación p/QRS Todas las ondas p conducen, PR alargado (>0.21
Relación p/QRS Todas las ondas p conducen mseg)
Diagnostico probable BRADICARDIA SINUSAL Diagnostico probable BLOQUEO AV de 1°
Manejo Causa más frecuente: síndrome del seno enfermo Manejo Habitualmente no precisa tratamiento

Ritmo rápido o lento Lento Ritmo rápido o lento Lento


Ritmo regular o irregular Regular Ritmo regular o irregular Regular
QRS estrecho o ancho Estrecho QRS estrecho o ancho Estrecho
nda “p” Presente nda “p” Presente
Relación p/QRS Alargamiento progresivo del PR hasta una onda “p” no conduce Relación p/QRS PR, constante, hasta que una onda “p” no
Diagnostico probable BLOQUEO AV 2° WENKEBACK (MOBITZ I) conduce
Manejo Habitualmente no precisa tratamiento Diagnostico probable BLOQUEO AV 2° MOBITZ II
Manejo Derivar a servicio de urgencias, precisara marcapasos

Ritmo rápido o lento Lento


Ritmo regular o irregular Regular
QRS estrecho o ancho Estrecho
nda “p” Hay ondas “p” con ritmo regular
Relación p/QRS Ninguna onda “p” conduce, no hay relación p/QRS
Diagnostico probable BLOQUEO AV 3° ó COMPLETO
Manejo Derivar a servicio de urgencias, precisara marcapasos

MANEJO DE LAS BRADICARDIA: Tratar las bradicardias, mal toleradas o que cumplan criterios de inestabilidad que se numeran a
continuación: 1. Tensión arterial sistólica <90mmHg. 2. Frecuencia cardiaca <40 lpm. 3. Bradicardia sintomática. 4. Bradicardias con QRS
ancho. Los medicamentos de que se dispone para su manejo son; ATROPINA: 0,5mg IV (dosis inferiores producen un efecto paradójico).
Se puede repetir cada 2 minutos, dosis máxima 3 mgs. Usar con precaucion en el síndrome coronario agudo, ya que puede aumentar el
grado de isquemia cardiaca; en transplantados cardiacos causa bloqueo AV de alto grado o paro sinusal. Si tras la medida anterior el
paciente no mejora o hay riesgo de asistolia, precisara un marcapasos transcutáneo, por lo que hay que avisar a servicios de
emergencia. Las situaciones que aumentan el riesgo de que la bradicardia derive en asistolia son: asistolia reciente, paro ventricular
>3seg, BAV Mobitz II, BAV 3° con QRS ancho. Si no disponemos de servicio de emergencias a corto plazo, el paciente se inestabiliza,
podemos administrar una perfusión de ADRENALINA a 1mg en 100cc de SF en 10 min. Si no mejora podemos golpear rítmicamente (60
veces por minuto), con nuestro puño en el margen inferior izquierdo del esternón, a modo de marcapasos externo. Si la causa de la
bradicardia es una intoxicación con betabloqueadores o antagonistas de calcio, administraremos GLUCAGON 1mg.

CURSO ENARM CMN SIGLO XXI TEL: 36246001 Pharmed Solutions Institute PÁGINA 222
MANUAL DE TRABAJO DEL CURSO ENARM CMN SIGLO XXI
DIAGNOSTICO DIFERENCIAL DE TAQUICARDIAS:

CURSO ENARM CMN SIGLO XXI TEL: 36246001 Pharmed Solutions Institute PÁGINA 223
MANUAL DE TRABAJO DEL CURSO ENARM CMN SIGLO XXI

La cardioversion electrica sincronizada, para revertir una arritmia debe ser sincronizada sobre la onda R para evitar el periodo
refractario que sigue a la onda T, ya que podriamos provocar una taquicardia ventricular, de efectos deletereos. Previamente debemos
sedar al paciente con 10-15 mgs de midazolam intravenoso. La energia necesaria para la cardioversion de una arritmia de QRS ancho o
una FA (son las mas frecuentes) es de 70-120J si usamos un desfibrilador bifasico o 100J si es monofásico. En cambio para cardiovertir
arritmias de QRS estrecho o un flutter auricular la energia a palicar es de 120-150J en desfibriladores bifasicos y 200J en los
monófasicos. Si la primera descraga no es eficaz, se puede incrementar la energia (50J) en las siguientes y si hay dificultad para
sincronizar el choque, podemos proceder a choque no sincronizado.

CASO CLINICO
Se trata de paciente masculino de 31 años de edad, licenciado en derecho, originario del Estado de Tabasco, soltero pero con pareja
sexual, de orientación homosexual, con prácticas de riesgo de los 19 a los 27 años, actualmente es negativo a VIH y mantiene una
protección adecuada, consume bebidas embriagantes casi cada semana y también presenta tabaquismo positivo, no más de 5
cigarrillos diarios, cuenta con antecedentes de importancia dengue a los 17 años y apendicetomía a los 19, se le está realizando
evaluación para ingresar a laborar en el sector gubernamental, el paciente refiere excelente estado de salud sin embargo reconoce no
estar en forma, no practica actividades físicas y su trabajo es muy sedentario, refiere uso de drogas con uso recreativo y experimental
de los 21 a los 26 años, solo agrega sentir “brincos” en el corazón en ltimas fechas, tal vez mareo ocasional que no le ha resultado de
importancia, a la exploración física se encuentra un IMC de 24, aparenta buen estado general, resto normal, los parámetros de
laboratorio normales pero el ECG del chequeo muestra un trazo donde se prolonga progresivamente el intervalo PR de 0.20 a 0.28.

PREGUNTA
Cuál de los siguientes diagnósticos es el más probable en este hallazgo.

RESPUESTA
a.- Bloqueo AV de 1er grado.
b.- Bloqueo AV de 2do grado Mobitz I.
c.- Bloqueo AV de 2do grado Mobitz II.
d.- Bloqueo AV de 3er grado.

CASO CLINICO
Se trata de paciente femenino de 72 años de edad, originaria de Guanajuato, viuda, dedicada a su casa, actualmente bajo tratamiento
por insuficiencia cardiaca por hipertrofia ventricular secundaria a hipertensión arterial de 30 años de evolución, cuenta además con
aterosclerosis e insuficiencia intestinal crónica, refiere familiar que encontró a la paciente confundida, desorientada y posteriomente
pierde la conciencia trasladándole a urgencias donde se ingresa a choque, el trazo electrocardiográfico se observa ondas P que no
conducen y latidos de escape ventricular de forma variable (disociación auriculoventricular). Se reanima a la paciente y se prepara para
colocación de marcapasos, previamente se estableció el diagnostico de un Bloqueo AV de 3er grado.

PREGUNTA
Cuál de las siguientes condiciones es la causa más probable del bloqueo de este paciente.

RESPUESTA
a.- Uso de digitalico.
b.- Trastorno electrolítico.
c.- Uso de diurético.
d.- Uso de antihipertensivo.

CASO CLINICO
Se encuentra de guardia en el servicio de urgencias donde llega por la tarde paciente femenino de 49 años de edad la cual refiere que
desde la noche anterior inicio con molestia en el torax con disconfort, ansiedad y sensación de que su corazón latia muy rápido, aun asi
acudió al trabajo, pero se salió porque le faltaba el aire y presencia mareo continuo, teniendo que descansar repetidamente, la
paciente es madre soltera, con tres hijos, emigra al Estado de México hace 15 años por disfunción familiar severa con violencia física,
tiene antecedente de histerectomía total abdominal con ooforectomia bilateral por presencia de cáncer cervicouterino, además de
recibir quimioterapia y radioterapia hace 5 años con excelente respuesta y controles adecuados, además cuenta con antecedente de
procesos infecciosos recurrentes de vías aéreas altas y recuerda escasamente que le diagnosticaron fiebre reumática a la edad de 23
años, pero ya no continuo su atención por sentirse mejor, a la exploración física se encuentra palidez de tegumentos, cianosis
peribucal, no hay datos de ingurgitación yugular, se ve confusa, al examen mental esta desorientada, agitada, incongruente, síntomas
se duran solo minutos sus ruidos cardiacos están disminuidos, constantes vitales son TA 150/60 mmHg, FC 145 lpm, FR 31 rpm,

CURSO ENARM CMN SIGLO XXI TEL: 36246001 Pharmed Solutions Institute PÁGINA 224
MANUAL DE TRABAJO DEL CURSO ENARM CMN SIGLO XXI
normotermica, la paciente refiere que fue diagnosticada recientemente con hipertensión arterial y prescribieron metoprolol 100 mg
cada 12 horas, el ECG presenta ausencia de onda P, con presencia de ondulaciones irregulares y de pequeña amplitud, pero complejos
QRS de 80 a 100 por minuto.

PREGUNTA
Cuál es el manejo más apropiado para evitar complicaciones agudas para el caso.

RESPUESTA
a.- Propafenona.
b.- Amiodarona.
c.- Digoxina.
d.- Verapamil.

CASO CLINICO
Luego de 6 horas de ingreso la paciente presenta datos de focalización neurológica, caracterizado por desviación de la comisura labial,
disartria y dislalia, parestesias de miembro torácico contralateral a la cara, se observa disminución de fuerza en el mismo sitio, a las 24
horas posteriores se observa recuperación paulatina de sensibilidad y fuerza de miembro torácico y discreta mejoría en la cara, al
parecer se trato únicamente de una isquemia cerebral transitoria, actualmente se encuentra con antiarritmico, anticoagulante y
neuroprotector, continua en terapia.

PREGUNTA
Cuál es la causa más probable que condujo a esta arritmia a la paciente.

RESPUESTA
a.- Uso reciente de Betabloqueador.
b.- Valvulopatia crónica.
c.- Sindrome paraneoplasico.
d.- Cardiomiopatia metastasica.

CASO CLINICO
Se encontró en la via publica a sujeto de 50 años aproximadamente el cual se encuentra en condición de calle, se observa desalineado,
con nula higiene, con aliento alcohólico, se observan lesiones dérmica sin tratamiento, el paciente se encuentra desorientado,
confundido, con tendencia a perder el equilibrio con palidez generalizada con pérdida de conciencia por segundos, se toman muestras
para laboratorio, se coloca ECG y se observa un trazo caracterizado por ausencia de ondas p con ondas regulares dentadas en
derivaciones DII, DIII y aVF particularmente negativas.

PREGUNTA
Cuál es la conducta a seguir más adecuada.

RESPUESTA
a.- Cardioversion.
b.- Colocacion de marcapasos.
c.- Administracion de Amiodarona.
d.- Administracion de Digoxina.

CASO CLINICO
Varón de 67 años. Como antecedentes personales de interés destacan hipertensión arterial, diabetes mellitus tipo 2, dislipemia,
bronquitis crónica, cólicos nefríticos de repetición, cirugía de hemorroides y de hernia umbilical. Tratamiento habitual con insulina
aspart, metformina, omeprazol, tiaprida y lorazepam. Ingresa en nuestro centro hospitalario para estudio por posible neoplasia de
cabeza de páncreas. Encontrándose hemodinámicamente estable, se inicia pauta con octreotide 100μg/8h subcutánea. A las 24 horas
presenta bradicardia sinusal a 40 lpm con presión arterial (PA) de 120/60 mmHg, sin sintomatología asociada. En el ECG se observa
bradicardia sinusal y hemibloqueo de rama anterior izquierda, realizan ecocardiograma transtorácico sin alteraciones significativas y
colocan Holter de 24 horas. Presenta deterioro clínico con bradipsiquia e inestabilidad de la marcha; PA, 90/45 mmHg y 35 lpm.
Valorada tira de ritmo en planta presenta bradicardia sinusal a 37 lpm y hemibloqueo de rama anterior izquierda descrito. Tras
administración de atropina hasta 3mg persiste bradicardia sintomática.

PREGUNTA
Cuál es la conducta a seguir más adecuada.

RESPUESTA
a.- Cardioversion.
b.- Colocacion de marcapasos.
c.- Administracion de Amiodarona.
d.- Administracion de adrenalina.

CURSO ENARM CMN SIGLO XXI TEL: 36246001 Pharmed Solutions Institute PÁGINA 225
MANUAL DE TRABAJO DEL CURSO ENARM CMN SIGLO XXI
DISFUNCION ORGÁNICA MÚLTIPLE (DOM):
CIENCIAS BÁSICAS: Disfunción de órganos nace como resultado de la Conferencia de Consenso de la ACCP/SCCM: Es la disminución
potencialmente reversible en la función de uno o más órganos, que son incapaces de mantener la homeostasis sin un sostén
terapéutico. El término disfunción implica un proceso continuo y dinámico en la pérdida de la función de un órgano, que va de menos a
más, siendo la etapa final en la claudicación de la función de dicho órgano lo que denominamos falla. El Síndrome de Disfunción
Orgánica Múltiple (SDOM) se considera como un conjunto de síntomas y signos de patrón diverso que se relacionan en su patogenia,
están presentes por lo menos durante 24 a 48 horas y son causados por disfunción orgánica, en grado variable, de dos o más sistemas
fisiológicos, con alteración en la homeostasis del organismo, y cuya recuperación requiere múltiples medidas de soporte avanzado. Esta
definición además de permitir un diagnóstico clínico precoz, nos da la oportunidad de intervenir activa y rápidamente para tratar de
revertir el proceso que originó el daño Asimismo debemos definir el SDOM primario cuando ocurre una vez que cualquier sistema
orgánico falla, en forma evidente o sea como resultado directo de una lesión conocida, por ejemplo, luego de un episodio de
insuficiencia respiratoria aguda por lesión pulmonar después de un trauma tisular global o contusión pulmonar (En la forma más común
de SDOM, los pulmones son los órganos predominantemente afectados y con frecuencia el nico sistema implicado, hasta etapas
tardías de la enfermedad), la insuficiencia renal aguda por rabdomiólisis y a la coagulopatía por politransfusión. El SDOM secundario es
el que se presenta en el contexto de una respuesta inflamatoria sistémica, o consecuencia de la respuesta del huésped a una agresión
englobándose en el Síndrome de Respuesta Inflamatoria Sistémica (SRIS), es el nexo que media entre la injuria (infecciosa o no) y el
SDOM. El SRIS y el SDMO son la vía final común de muerte en pacientes severamente enfermos o lesionados, en los que la tecnología
avanzada existente en las UCI ha posibilitado su resucitación. SALUD PUBLICA: Las cifras de mortalidad para SIRA por sí solo es de 40 al
50%, una vez que se agrega disfunción orgánica sistémica, la mortalidad se incrementa hasta el 90%. Sus complicaciones son la causa
más frecuente de mortalidad en UCI no coronarias ETIOLOGÍA: Inicialmente se pensó que éste cuadro era la expresión fatal de una
sepsis incontrolada, sin embargo posteriormente se ha demostrado que puede ocurrir en ausencia de infección. La infección no es una
condición indispensable para el desarrollo de FOM. Plantea la existencia de una activación masiva de mediadores de la inflamación
como consecuencia del daño tisular. La incidencia del SDOM en una población de pacientes hetereogénea ó mixta (patologías de tipo
médico y quirúrgico) varía entre 7 y 15%.En pacientes con trauma su incidencia puede ser hasta del 35%, y mucho menor en pacientes
luego de cirugía cardiaca electiva, alrededor del 3%. Es difícil encontrar un elemento causal simple para el SDMO, existiendo en la
mayoría de los pacientes varias causas que se potencian: 1. Traumatismos multisistémicos graves. 2. Postoperatorio. 3. Inestabilidad
hemodinámica. 4. Infecciones severas. 5. Pancreatitis aguda. 6. Quemados. 7. Necesidad de ventilación mecánica prolongada. 8.
Hemorragia gastrointestinal. 9. Disección, ruptura o reparación de aneurisma aórtico. 10. Perforación gastrointestinal. 11. Enfermedad
inflamatoria intestinal. 12. Nutrición parenteral prolongada. 13. Cirugía valvular cardiaca. 14. Transfusiones masivas. 15. Coagulación
intravascular diseminada. PATOGENIA: Se ha acumulado suficiente evidencia de la existencia de una participación activa de la cascada
inflamatoria en la génesis de cuadros de DOM. Es así como noxas de origen infeccioso ó no infeccioso son capaces de iniciar el proceso
inflamatorio, activando vías comunes que a manera de respuesta inespecífica, permiten al huésped reaccionar frente a la injuria. Esta
respuesta inflamatoria se verifica básicamente a cuatro grandes niveles: 1. Activación del sistema Monocito Macrófago. 2. Activación
del sistema del Complemento. 3. Activación del sistema Retículo Endotelial. 4. Activación de la Cascada de la Coagulación. Sin embargo,
todas estas vías están relacionadas entre sí, pudiendo activarse y potenciarse unos a otros desde cualquier punto de esta red
inflamatoria. Es el caso de pacientes politraumatizados en donde la cascada se inicia por la activación del sistema del Complemento y
cascada de la Coagulación, o en las vasculitis en donde el daño inflamatorio endotelial sería el gatillador del proceso. Existe suficiente
evidencia que avala la activa participación de la respuesta inflamatoria en la génesis de la DOM, sin embargo todos los intentos por
bloquear o modular esta respuesta ha fracasado. Luego del fenómeno inicial de up regulation de citoquinas proinflamatorias (TNFa, IL-
1, IL-2, IL-6, IL-8) o concomitantemente con él, se liberan especies anti-inflamatorias (receptores solubles de TNFa, IL-1 ra e IL-10) en
concentraciones 30 a 100.000 veces superiores que la de las moléculas proinflamatorias. Esto podría ser responsable de la disminución
de la activación observada en Monocitos humanos ante la exposición a infecciones de severidad creciente. Es así como el paradigma
actual de la respuesta inflamatoria presente en cuadros de SIRS,
sepsis o MOD es la interacción sistémica entre mediadores pro y anti
inflamatorios que determinarán en definitiva el daño celular ó la
recuperación. En el daño celular, último elemento en el que radica la
DOM, una de las teorías que mayor peso tiene en la actualidad
indicaría que, como consecuencia de la activación de la cascada
inflamatoria, a nivel intracelular la activación de mediadores
determina un stress oxidativo. Es este stress oxidativo el que es
capaz de activar un complejo de mediadores intracelulares
denominado "Complejo Nuclear Factor kb", el cual está compuesto
por tres sub unidades; una molécula pequeña denominada p50, una
mayor, la p65 y una sub unidad inhibitoria. Como consecuencia del
stress oxidativo este complejo es fosforilado, liberando la sub
unidad inhibitoria y permitiendo al dímero p50-p65 libre, migrar al
núcleo y unirse al DNA en los sitios que promueven la codificación
de citoquinas y otras sustancias proinflamatorias. Diversos trabajos
han demostrado que la administración de antioxidantes y que la prevención del stress oxidativo mitocondrial suprimen la activación de
Nuclear Factor kb. DIAGNOSTICO: Clínico: Los registros de temperatura deben ser centrales o rectales. La fiebre se produce por el
efecto de las citoquinas IL-1 y TNFα liberados por los macrófagos activados. Aunque la fiebre y los escalofríos son típicos, algunos
pacientes que desarrollan infecciones bacterianas sistémicas están debilitados y no exhiben cambios llamativos (por ejemplo,
escalofríos) al comienzo de una infección. La causa de la hipotermia es menos conocida y su presencia se asocia con mal pronóstico. En
los pacientes críticos, los mismos síntomas y signos característicos de sepsis pueden aparecer durante la inflamación sistémica de
origen no infeccioso, por lo que el diagnós co y la definición de la severidad del proceso sép co pueden ser di ciles. Durante los
l mos años se ha buscado un marcador clínico o de laboratorio capaz de iden ficar a los pacientes con sepsis, diferenciándolos de los

CURSO ENARM CMN SIGLO XXI TEL: 36246001 Pharmed Solutions Institute PÁGINA 226
MANUAL DE TRABAJO DEL CURSO ENARM CMN SIGLO XXI
portadores de otras patologías que también cursan con SIRS. Entre ellos podemos mencionar: Procalcitonina (PCT); posible marcador
de SIRS en respuesta a infección. Niveles mayores de 10 ng/mL (sepsis) y a veces superiores a 100 ng/ mL(shock séptico). Proteína C
reactiva (PCR); Es un marcador sensible pero tardío y de baja especificidad. Recuento leucocitario; La leucocitosis se interpreta
habitualmente como evidencia de posible infección, pero no es un marcador sensible ni específico. La activación de la coagulación es
un hecho común en el curso de la sepsis, con consumo de factores, aumento del dímero D y sobre todo disminución de la actividad de
los anticoagulantes naturales. Varias citoquinas proinflamatorias, sobre todo los niveles plasmáticos de la IL-6 e IL-8, han mostrado
correlación con el pronóstico en diversos estudios efectuados en pacientes críticos. Pero su determinación tiene varios inconvenientes:
alto costo, la vida media de las citoquinas es muy corta y las concentraciones varían rápidamente por lo que es difícil su interpretación.
No se sugiere su utilización en la práctica clínica. John Marshall crea un Score de Disfunción Orgánica, en el cual considera por medio de
un mecanismo de puntuación, la magnitud de la disfunción de cada uno de los 6 sistemas orgánicos analizados, otorgándoles un valor
entre 0 y 4 según la magnitud de la disfunción. Este sistema de puntuación se correlacionó estrechamente con la mortalidad intra UCI
tanto cuando se utilizaban los valores obtenidos durante las primeras 24 horas desde la admisión como cuando se aplicaba durante la
estadía. Jean Louis Vincent y cols introducen la versión Europea de los Scores de Disfunción (SOFA). Los scores nos permiten hacer un
pronóstico objetivo sobre la evolución del paciente. En la práctica clínica, cuando tenemos más de tres órganos en falla por más de 48
horas, la mortalidad se eleva por sobre el 90%, debiendo plantearse seriamente y con la familia la posibilidad de limitar el tratamiento
en curso. En estos casos, tendrán más posibilidades de sobrevida aquellos pacientes más jóvenes y sin patología debilitante previa.
TRATAMIENTO: Hasta la fecha la terapia de la Disfunción Orgánica continúa siendo una terapia de soporte, una vez eliminada la injuria
inicial (drenaje de colecciones, terapia antibiótica, remoción de tejidos necróticos, restablecimiento de una perfusión adecuada, etc.).
Existen algunos estudios recientes que mostrarían el beneficio de terapias de hemofiltración de alto flujo, como una forma de modular
la respuesta inflamatoria, removiendo aquellas sustancias que se encuentran en exceso. Todos los pacientes con choque séptico deben
tener acceso venoso adecuado para la reanimación con volumen. Un catéter venoso central también puede ser usado para monitorear
la presión venosa central y evaluar el estado del volumen intravascular. Una sonda vesical permanente debe ser usada para monitorear
la producción de orina, marcador de una adecuada perfusión renal y el gasto cardíaco. Los pacientes que desarrollen choque séptico
requieren cateterismo cardíaco derecho en la arteria pulmonar a través del catéter de Swan-Ganz. Este catéter proporciona una
evaluación precisa del estado de volumen de un paciente que está séptico. Mediante la evaluación de la función ventricular tanto
derecha como izquierda, monitoreo de los cambios hemodinámicos, además de brindar información pronóstica. La mayoría de los
pacientes que se encuentren sépticos desarrollan dificultad respiratoria como una manifestación del estado de choque. La disfunción
pulmonar de la sepsis (SDRA) también puede ocurrir. Estos pacientes necesitan intubación y ventilación mecánica para la asistencia
respiratoria óptima.

CASOS CLINICOS
Se trata de paciente femenino de 24 años de edad tercigesta con cesárea y aborto previos, sin antecedentes de importancia, quien
ingresa al servicio de urgencias con embarazo de 36 semanas de gestación + Preeclampsia severa con cifras tensiónales de hasta
190/120mmHg. Presenta además actividad uterina irregular sin datos de vasoespasmo ni datos de eminencia de eclampsia ni
afectación hepática. Frecuencia cardiaca fetal 135 latidos por minuto. Se hospitaliza y maneja con hidratación, antihipertensivo
periférico, calcioantagonista y sulfato de magnesio. Se solicitan laboratoriales. Hemoglobina: 14.2, Hematocrito: 46.2, Plaquetas:
215,000, TP: 12.6, TPT: 36.1, Glucosa: 88, Creatinina: 0.8, Acido Úrico: 5.2, Bilirrubina Total: 0.75, Bilirrubina Directa: 0.16, Bilirrubina
Indirecta: 0.59, Alanina Amino Transferasa (TGP): 35, Aspartato Amino Transferasa (TGO): 20, Deshidrogenasa Láctica (DHL): 350,
Examen General De Orina (EGO): Proteínas 30mg. Se estabilizan cifras tensiónales, motivo por el cual se decide interrupción del
embarazo vía abdominal. Se ingresa a quirófano con hipotensión arterial de hasta 80/40, recuperándose con carga de solución
cristaloide y bolo de efedrina. Se inicia cirugía y al entrar a cavidad abdominal se encuentra hemoperitoneo de 600cc. Se realiza cesárea
Kerr, obteniendo producto único vivo hipotónico. Se amplía incisión y se realiza revisión hepática, encontrándose ruptura hepática del
lóbulo derecho. Se prosigue para realizar empaquetamiento. Paciente en choque hipovolémico, reanimada con soluciones cristaloides
coloides y tres paquetes globulares, así como aminas (dopamina y dobutamina). Se administran esteroides a dosis de choque. No se
cuenta con plasma ni derivados plaquetarios, motivo por el cual se traslada a tercer nivel al servicio de terapia intensiva con sedación y
analgesia con midazolam y fentanil y relajación con vecuronio. Sin embargo, fallece a las 24 horas por falla orgánica múltiple.

PREGUNTA
Cual de las siguientes observaciones relacionadas al DOM es más frecuente en la patología de base?

RESPUESTA
a.- Disminución de la fracción de eyección.
b.- Disminución de la relación PaO2/FiO2.
c.- Ictericia, plaquetopenia, alteración de las PFH.
d.- Ilio paralitico, ulcera por estrés, colecistitis aguda alitiasica.

CURSO ENARM CMN SIGLO XXI TEL: 36246001 Pharmed Solutions Institute PÁGINA 227
MANUAL DE TRABAJO DEL CURSO ENARM CMN SIGLO XXI
COAGULACIÓN INTRAVASCULAR DISEMINADA (CID):
CIENCIAS BÁSICAS: Síndrome patológico adquirido, trombohemorrágico que se produce como resultado de una activación excesiva del
sistema de la coagulación y alteraciones de la fibrinólisis y conlleva el depósito masivo de fibrina en la microcirculación y lesión
orgánica. La CID se produce como consecuencia de una activación excesiva, del mecanismo hemostático, de la incapacidad de los
inhibidores fisiológicos para neutralizar la coagulación y de inhibición de la fibrinólisis mediada por el inhibidor del activador del
plasminogeno de tipo endotelial (PAI-1). La formación de trombos de fibrina y su depósito en los pequeños vasos (microangiopatia
trombotica) produce disfunción multiorgánica, que consume plaquetas y factores de coagulaciones consecuentemente desencadena
una fibrinólisis secundaria. Otros términos que suelen utilizarse para denominar este síndrome son: síndrome de desfibrinación,
coagulopatía por consumo y síndrome de activación sistémica de la coagulación. Frecuencia de CID en condiciones asociadas trauma
severo 50-70%, sepsis o infección severa 30-50%, feto muerto y retenido 50% (>5 sem), embolia de liquido amniótico 50%, abruptio
placentae 50%, HELLP 15%, neoplasias, anormalidades vasculares, circulación extracorpórea, falla hepática severa, shunt arterio-
venoso, rechazo de trasplante. PATOGENIA: La presencia de una condición asociada a CID. Como la sepsis o el trauma, inducen la
activación del sistema de coagulación a
través de la liberación de citoquinas,
como IL-6, IL-1 y/o el factor de necrosis
tu oral (TNF); integrados como parte de
la respuesta inflamatoria e
incrementando la cantidad de factor
tisular asociado a factor VII. La
liberación o exposición en alta
concentración de sustancias
procoagulantes, como el factor tisular
en el SNC. Como ocurre en el
traumatismo de cráneo, puede
desencadenar un trastorno
coagulopatico local y/o generalizarse en una CID. La activación del sistema de coagulación, con exceso de trombina y el subsiguiente
deposito de fibrina en la microcirculación, favorecen la agregación plaquetaria y el consumo de factores de coagulación. Los glóbulos
rojos quedan atrapados en las mallas de fibrina, otros sufren ruptura mecánica al pasar entre la malla de fibrina (responsable de la
esquistocitosis). La microtrombosis intravascular, promovida en la micro-vasculatura parenquimatosa y sistémica, generan un aumento
de plasmina, que degrada el fibrinógeno, la fibrina y otros factores de la coagulación; perpetuándose el circulo vicioso de coagulación-
fibrinólisis. El funcionamiento suboptimo de los sistemas anticoagulantes naturales, proteína C o antitrombina, desbordados por el
incremento de trombina y plasmina, permiten que estas circulen libremente por el torrente circulatorio. El consumo de los factores de
la coagulación y plaquetas, junto a la hiperfibrinolisis favorecen la aparición de hemorragias.
FORMAS DE PRSENTACION: A) según el tiempo de instauración y
progresión: aguda y crónica. B) Por compromiso y extensión:
localozada, sistémica. C) Por sus manifestaciones:
hemorrágica(reacción hemolítica trasnfucional, leucemia
promielocitica), trombótica (síndrome de Trousseau). DIAGNOSTICO:
Clínico; hemorragia, hemorragias cutáneo mocosas generalizadas
(epistaxis, hematuria, petequias, equimosis, gingivorragia, hemorragia
digestiva, hemorragia en SNC), hemorragia tras incisión quirúrgica,
heridas, catéteres, o punciones vasculares, trombosis, púrpura
fulminante, acrocianosis periférica, gangrena de extremidades,
anemia microangiopatica, en SNC-delirio, coma, en pulmón-diestress, en riñón-
insuficiencia renal aguda, necrosis cortical, en tubo digestivo-úlceras
duodenales. La CID aguda se manifiesta como hemorragias, trombosis o ambas. Las hemorragias suelen ser de presentación aguda, en
múltiples sitios, especialmente de punción venosa o arterial, y frecuentemente asociada a petequias y púrpura. La CID crónica se
presenta de manera más solapada, con menores manifestaciones clínicas y de laboratorio, debe considerarse en paciente con
patologías asociadas a esta entidad (neoplasias, feto muerto retenido, aneurisma de aorta, hemangioma, trasplante, HELLP, reacciones
alérgicas), que presentan alteraciones leves de los parámetros de hemostasia o que sangran excesivamente frente a intervenciones
quirúrgicas. El diagnostico de CID se fundamenta en el cuadro clínico: 1. Identificación de la entidad causal de CID. 2. Valoración de los
sangrados y/o trombosis. 3. Alteraciones coaguló-métrica de laboratorio. Pruebas de hemostasia básicas para diagnostico de CID: 1.
Recuento de plaquetas, un recuento bajo o en descenso progresivo, es una determinación sensible aunque no especifica. El 98% de
casos de CID presenta plaquetopenia y el 50% se encuentra por debajo de 50,000 mm3. 2. Producto de degradación del fibrinógeno
(PDF) y dímero D (DD), frecuentemente están elevados como manifestación de hiperfibrinolisis secundario al incremento de plasmina;
son sensibles (95%) aunque poco específicos ya que otras condiciones como el trauma, cirugía reciente o tromboembolismo venoso
pueden incrementarlos, al igual que el deterioro de la función renal o hepática, por una dos inducción de su eliminación. Aun no se han
estandarizado los puntos de corte considerados incremento moderado y alto necesarios para el score diagnostico de CID. 3. Tiempo de
pro trombina (TP) y tiempo de tromboplastina parcial (TTP), se encuentran prolongados en el 50-60% de los casos, aunque cerca de la
mitad de los pacientes pueden presentar tiempos normales o incluso acortados por la presencia de factores de coagulación activados
como la trombina o el factor X. 4. Fibrinógeno, su descenso es poco sensible (28%), se encuentra sobre todo en estadios avanzados de
la enfermedad. 5. Frotis de sangre periférica, la presencia de esquistocitosis, suele encontrarse en el 50% de los casos. No existe un
método diagnostico estándar pero se han desarrollado scores diagnostico. La sensibilidad del score de CID manifiesta la Sociedad
Internacional de Hemostasia y Trombosis (ISTH) es de 91% y tiene una especificidad del 97%. TRATAMIENTO: La clave es la corrección
de la enfermedad desencadenante y dar soporte. No se debe posponer la medida terapéutica requerida, por intentar corregir un
parámetro de coagulación alterado. Hemoderivados; no están indicados para corregir parámetros de laboratorio, pero si en presencia

CURSO ENARM CMN SIGLO XXI TEL: 36246001 Pharmed Solutions Institute PÁGINA 228
MANUAL DE TRABAJO DEL CURSO ENARM CMN SIGLO XXI
de sangrado activo, previo a intervenciones invasivas o en aquellos con alto riesgo de sangrado. Plasma (15 o 30ml/kg), plaquetas (una
unidad de plaquetas cada 10 kg de peso en pacientes sangrando con recuentos <50,000 mm3), crió precipitados (cuando fibrinógeno
esta < 1g/l, sin respuesta a plasma), concentrado de factores (el concentrado de complejo protrombínico, solo tiene factores vitamina K
dependientes y carece de otros factores como el factor V, no pudiendo corregir todos los factores deficientes en la CID). El factor VII
activado recombinante se ha utilizado en casos de sangrado que compromete la vida (su eficacia y seguridad aun no está definida).
Indicaciones de heparina en CID, debido a que la CID está caracterizada por una importante activación de la coagulación, el tratamiento
anticoagulante podría ser racional. No hay ensayos clínicos randomizados que demuestren que la heparina mejora los resultados
clínicos y en la CID aguda podría agravar hemorragias.

CASO CLINICO
Varón de 66 años acude a urgencias por clínica de disnea a mínimos esfuerzos de 10 días de evolución y fiebre de hasta 39°C. Niega tos
y expectoración. Inició tratamiento antibiótico de forma ambulatoria sin presentar mejoría. A su llegada presenta insuficiencia
respiratoria grave que requiere FiO2 elevadas. Presenta rápida progresión de la insuficiencia respiratoria y a las 24h precisa intubación
orotraqueal y ventilación mecánica. La radiografía de tórax inicial muestra un infiltrado alveolar en la base izquierda con aparición de
infiltrados pulmonares bilaterales en controles posteriores. Se realiza cateterización de la arteria pulmonar que descarta fallo cardíaco,
con presión capilar pulmonar inferior a 18mmHg, y que muestra signos de hipertensión pulmonar moderada. Analíticamente, destaca la
aparición de coagulación intravascular diseminada (D-dímero mayor a 20.000ng/ml, trombocitopenia máxima de 47.000/l y tiempo de
protrombina máximo de 1,58 ratio), fracaso renal agudo con cifras de creatinina de hasta 2,33mg/dl y elevación de la lactato
deshidrogenasa (LDH) con valores máximos de 2.305U/l.

PREGUNTA
Cual de los siguientes antecedentes esta mas relacionado con el estado del paciente.

RESPUESTA
a.- Diabetes mellitus.
b.- Hipertension arterial.
c.- Cardiopatia isquémica.
d.- Enfisema pulmonar.

ANAFILAXIA:
CIENCIAS BASICAS: Reacción de hipersensibilidad generalizada o sistémica, grave y que amenaza la vida, consideraremos que la
anafilaxia es una reacción alérgica grave de instauración rápida y potencialmente mortal. Desde el punto de vista clínico se trata de un
síndrome complejo, desencadenado por mecanismos inmunológicos o no, con aparición de síntomas y signos sugestivos de liberación
generalizada de mediadores de mastocitos y basofilos, tanto a nivel cutáneo, como en otros órganos. Las causas más frecuentes son
medicamentos (antibióticos betalactamicos, AINES) y medios diagnostico (46-62%), alimentos (22-24%), picaduras de insectos (8-14%),
factores físicos (3-4%), otros (incluye látex 7%), idiopática (3-5%). La importancia relativa de cada uno de ellos varía considerablemente
en función de la edad, así los alimentos (huevo, leche, frutos secos, pescado y marisco) son la causa más común en niños y los fármacos
son más frecuentes en adultos. SALUD PUBLICA: Cifras de incidencia entre 3,2-30 por 100,000 personas al año, con una mortalidad
entre 0,05-2% del total de las reacciones. En el Shock anafiláctico la incidencia varía entre 3,2-10 por 100,000 personas /año y al menos
1 % es mortal. DIAGNOSTICO: Sospecha clínica; cuando aparece de una manera aguda (en minutos a pocas horas), un síndrome
rápidamente progresivo que afecta a la piel y/o mucosas y que se acompaña de compromiso respiratorio y/o circulatorio. Como las
mayoría de las anafilaxias cursan con síntomas cutáneos (>80%), con este criterio al menos un 80% de las anafilaxias serian
identificadas. Sin embargo existen presentaciones menos típicas, como las que presentan exclusivamente hipotensión. También se ha
descrito que las manifestaciones digestivas se
CRITERIOS CLINICOS PARA EL DIAGNOSTICO DE ANAFILAXIA
asocian con una mayor gravedad. Por último es La anafilaxia es muy probable cuando se cumple uno de los tres criterios siguientes:
importante tener en cuenta que la 1.Inicio agudo (minutos a horas) de un síndrome que afecta a la piel y/o mucosas (urticaria
concurrencia de una exposición a un alérgeno generalizada, prurito, eritema, edema de labios, úvula o lengua) junto con al menos uno de los
potencial o conocido para el paciente apoya el siguientes:
a. Compromiso respiratorio (disnea, sibilancias, estridor, disminución del PEF, hipoxemia)
diagnostico. Por ello se han establecido
b. Disminución de la TA o síntomas asociados de disfunción orgánica (hipotonía, sincope,
criterios, con ellos se espera identificar más del incontinencia)
95% de las anafilaxias. Cuando existe afección 2.Aparición rápida (de minutos a algunas horas) de dos o más de los siguientes síntomas tras la
cardiovascular con hipotensión se habla de exposición a un alérgeno potencial para ese paciente
shock anafiláctico. La dificultad en el a. Afectación de piel y/o mucosas
b. Compromiso respiratorio
diagnostico estriba en que no hay signos o c. Disminución de la TA o síntomas asociados de disfunción orgánica
síntomas patognomónicos; lo que sí es típico es d. Síntomas gastrointestinales persistentes (dolor abdominal, cólico, vómitos)
la rápida progresión en la gravedad e intensidad 3.Disminución de la TA en minutos o algunas horas tras la exposición a un alérgeno conocido para ese
de los síntomas. EVALUACION DE GRAVEDAD: paciente
a. Lactantes y niños: TA baja o descenso superior al 30% de la TA sistólica
Se relaciona con la rapidez en la progresión de b. Adultos: TA sistólica inferior a 90 mmHg o descenso superior al 30% sobre la basal
los síntomas, con el tipo de antígeno, vía de
entrada y órganos afectados. Factores relacionados como edad avanzada, presencia de patología respiratoria o cardiovascular asociada,
tratamiento con IECAS o betabloqueantes o una mastocitosis de base, se han asociado con reacciones graves y mayor mortalidad, por
tanto es fundamental la evaluación ABCD. Las anafilaxias graves vienen definidas por la presencia de cianosis, saturación O2 <92%,
hipotensión, confusión, hipotonía, pérdida de conciencia o incontinencia. Las anafilaxias moderadas presentan signos o síntomas que
sugieren afección respiratoria, cardiovascular o GI, como disnea, estridor, sibilancias, nauseas, vómitos, mareo, sudoración, opresión
torácica, sensación de garganta ocupada o dolor abdominal. Las manifestaciones cutáneas (eritema, urticaria, angioedema) no se
consideran criterios de gravedad. Laboratorio: Los niveles plasmáticos de histamina (se metaboliza rápidamente) y de triptasa total,

CURSO ENARM CMN SIGLO XXI TEL: 36246001 Pharmed Solutions Institute PÁGINA 229
MANUAL DE TRABAJO DEL CURSO ENARM CMN SIGLO XXI
esta es la prueba mas útil para el diagnostico de anafilaxia, puede elevarse en muestras obtenidas entre los 15 y 180 min del comienzo
de los síntomas. Se aconseja un mínimo de 3 muestras seriadas, la concentración normal en suero es inferior a 13.5 µ/l, una elevación
de al menos 2 veces lo normal, es sugestivo de anafilaxia. TRATAMIENTO: 1. Valoración inmediata del sistema cardiovascular y
respiratorio, si el paciente está en paro las medidas de resucitación cardiopulmonar deberá ser de extrema urgencia, ante la inminencia
del shock colocar al paciente recostado con los pies elevados. 2. La primera droga a administrar es la Adrenalina IM es la vía de elección
para el tratamiento inicial, obtiene unas concentraciones plasmáticas mas rápidas y elevadas que la vía SC, presenta un mayor margen
de seguridad que IV. En solución acuosa y una dilución de 1:1000 y a una dosis de 0.01 ml/kg. Subcutánea dosis máxima de 0.3 ml.
cuando la anafilaxia fue secundaria a una inyección o a un piquete siempre y cuando no sea en la cabeza o cuello deberá de recibir una
segunda administración e 0.01 ml/ kg. sin pasar de 0.3 ml en la zona del piquete, esto reduce de una manera considerable la absorción
el antígeno. Esta dosis de Adrenalina puede ser repetida cada 15 o 20 minutos dependiendo de la evolución máxima de 3 dosis. La
administración IV de adrenalina tiene un alto riesgo de arritmias, sin embargo Cuando la evolución es mala y hay shock evidente o
Colapso vascular, podrá administrarse a una dosis de 0.1 ml de adrenalina 1:1000 diluida en 10 ml de solución fisiológica (para quedar
en una dilución de 1:100,000) y administrar IV durante un periodo de 15 a 20 minutos. Considérese esta medida cuando el paciente
está en riesgo de perder la vida. 3. Torniquete En caso de que la reacción de anafilaxia sea debido a un piquete de insecto o por una
inyección se utilizara un torniquete cercano al sitio del piquete o inyección, este podrá liberarse 1-2 minutos cada 10 minutos. 4.
Oxigeno siempre que sea posible hay que administra oxigeno a los pacientes con cianosis, disnea, sibilancias por espasmo bronquial, a
una dosis de 3 a 5 lts. por minuto. 5. Antihistamínicos el Benadryl (difenhidramina) a una dosis de 1-2 mg por kilo deberá ser
administrada IV lentamente en un tiempo no menor de 15 minutos , puede ser administrada IM o hasta vía oral, otra opción es el
clorfeniramina IM, IV u oral a dosis de 0.35mgxkgx24 hs el antihistamínico deberá de continuarse por vía oral cada 6 hs para prevenir
recurrencia de la reacción principalmente de urticaria y angioedema, principalmente para administración oral los antihistamínicos de la
nueva generación pueden ser muy buena opción y continuarlos cuando menos por un lapso de una semana. Corticosteroides .- La
administración temprana después de la adrenalina y del antihistamínico es de mucha ayuda para prevenir la recurrencia de los
síntomas, y la fase tardía, la dosis inicial de la hidrocortisona es de 7-10 mg/kg. y posteriormente 5 mg/kg. cada 6 hs o
Metilprednisolona 2-3 mgx kg. y no deberán ser descontinuados cuando menos por 3 a 4 días. Cuando a pesar de las medidas
enunciadas el paciente no mejora, persiste hipotenso o con problemas de dificultad respiratoria, deberá de internarse inmediatamente
y de preferencia en una unidad de cuidados intensivos para la administración de: 1. Líquidos parenterales de solución glucosada-
fisiológica inicialmente a una dosis de 30 ml por kilo en una carga para la primera hora, deberá de repetirse de acuerdo a su evolución
para mantener una presión por encima de 50 mmHg. 2. Vasopresores: cuando a pesar de las soluciones y manejo persiste hipotenso
deberá de manejarse con vasopresores bitartrato de norepinefrina diluir en 250 ml (glucosado 5% y fisiologico ) 1 ml y pasar 0.5 ml
/minuto. Dopamina 15 mcg /kg/minuto IV, preferible sobre la norepinefrina en los pacientes con insuficiencia cardiaca.
Broncodilatadores.- Regularmente la misma adrenalina alivia la presencia de broncoespasmo sin embargo cuando está presente a pesar
de esta, La nebulizacion de salbutamol 0.5 ml de la solución al 5% ,en 2 ml de agua y dos de fisiológico para nebulizar durante 8-10
minutos y repetir cada hora las primeras tres horas es de muchisimo beneficio para mejorar la permeabilidad de las vías aéreas por
broncodilatacion. En caso de que el broncoespasmo persista deberá de administrarse AMINOFILINA a una dosis de 5-7 mg por kilo en
solución cuando menos 30 ml para pasar en 30 minutos y posteriormente una dosis de 5 mg por kilo para administrar cada 6 hrs
durante 30 minutos. Intubación y traqueostomia.- Cuando a pesar del manejo el paciente no mejora y no se puede restablecer la
permeabilidad de las vías respiratorias por el edema deberá de intentarse la Intubación endotraqueal, Y NUNCA RETRASAR LA DESICION
DE ESTE PROCEDIMIENTO SI SE REQUIERE para el caso de la traqueostomia deberá de ser realizada por personal calificado y en sala de
operaciones de ser posible. Terapia de soporte.- Después de que el paciente ha sido estabilizado, deberá de mantenerse la terapia de
mantenimiento con líquidos y drogas por el tiempo en que se requiera y las funciones vitales se hayan reinstalado. Esto puede ser de
pocas horas a varios días, en los caso más severos la muerte puede ocurrir dentro de los primeros treinta minutos, con el manejo
usualmente la recuperación es completa, a menos que haya sufrido durante la crisis un infarto al miocardio o daño al SNC. Un hecho es
que en estos casos las reacciones subsecuentes con el mismo antígeno pueden dar lugar a reacciones mas severas y mas rápidas, por lo
que un pilar muy importante del manejo de la anafilaxis es la Prevención.

CASO CLINICO
Se trata de una paciente de sexo femenino de 26 años de edad, que consultó por un cuadro clínico de cinco meses de evolución
consistente en pápulas eritematosas, prurito intenso y angioedema asociado al contacto con el frío y, en algunas ocasiones, asociado a
disnea. Los síntomas se presentaban con ambiente frío o con agua fría y sucedían en el área expuesta. Cuando se bañaba en piscina o
con agua fría, la reacción era generalizada; cuando lavaba platos o se lavaba las manos, la reacción era local. Cuando ingería bebidas
frías, no presentaba edema en lengua ni disfagia, pero sí reacción en las manos al sostener la botella o el vaso frío.

PREGUNTA
Cual es el tratamiento farmacologico sistémico mas adecuado.

RESPUESTA
a.- Hidroxicina.
b.- Loratadina.
c.- Prednisona.
d.- Clorfenamina.

CASO CLINICO
Se trata de una mujer de 77 años, obesa, hipertensa y diabética no insulín-dependiente. Había sido intervenida quirúrgicamente en
varias ocasiones sin sufrir accidentes anestésicos, y no era alérgica conocida a fármacos. Tras la inducción anestésica aparecio
inmediatamente un broncoespasmo que dificultaba la ventilación mecánica, (la gasometría arterial durante el episodio agudo mostró
acidosis respiratoria severa con pH: 7,12; pCO2: 71 mmHg; pO2: 63 mmHg; HCO3: 23 mmol/l; EB: -7,7 mmol/l; SaO2 83% con FiO2 al

CURSO ENARM CMN SIGLO XXI TEL: 36246001 Pharmed Solutions Institute PÁGINA 230
MANUAL DE TRABAJO DEL CURSO ENARM CMN SIGLO XXI
100% ), hipotensión extrema (tensión arterial sistólica 50 mmHg) y finalmente disociación electromecánica que requirió resucitación
cardiopulmonar. Durante el episodio aparecieron eritemas en pliegues, habones diseminados y enrojecimiento facial, y la
monitorización cardíaca mostró una bradicardia a 30 latidos por minuto (lpm), y una elevación de ST localizada en la cara inferior y
anterolateral baja en el electrocardiograma (ECG).

PREGUNTA
Cual es el tratamiento farmacológico inicial para manejar la anafilaxia presentada.

RESPUESTA
a.- Metilprednisona.
b.- Prednisona.
c.- Adrenalina.
d.- Dexametazona.

ENFERMEDAD PULMONAR OBSTRUCTIVA CRONICA (EPOC) AGUDIZACIONES:


CIENCIAS BASICAS: La definición de exacerbación de EP C no CLASIFICACION DE LAS EXACERBACIONES DE LA EPOC Y TRATAMIENTO ANTIBIOTICO RECOMENDADO
GRUPO DEFINICION MICROORGANISMOS TRATAMIENTO ORAL TRATAMIENTO IV
está bien establecida y no existen criterios universalmente I EPOC leve <4 reagudizaciones/ Amoxi+clavulanico
aceptados, aunque generalmente suele describirse como el Menor de 65 años año Telitromicina
No comorbilidad H. influenzae Levo o moxifloxacino
empeoramiento agudo de los síntomas respiratorios en un S. pneumoniae Azitromicina
M. catharralis
paciente con EPOC. La combinación de síntomas descrita por IIa EPOC moderado o Igual + Levo o moxifloxacino Levofloxacino
Anthonisen y cols. en 1987 (incremento de la disnea, aumento grave enterobacterias Telitromicna Ceflasporina 3 y 4
No riesgo para P. Amoxi+clavulanico Amoxi +
de la producción y purulencia del esputo) ha sido aeruginosa clavulanico
habitualmente aceptada para la definición de exacerbación IIb EPOC
grave
moderado o >4
año
reagudizaciones/ Ciprofloxacino
Levofloxacino
Betalactamico
parenteral con
pero realmente sólo representa una parte del global. Por ello Riesgo de P. Igual IIa + P. actividad
aeruginosa aeruginosa antipseudomona
nos parece más adecuada la definición propuesta por Desnutricion
Rodríguez Roisín: “Empeoramiento mantenido de la situación
de un paciente con EPOC, desde un estado basal, por encima de las fluctuaciones diarias, que es agudo en su comienzo y que necesita
modificar su tratamiento habitual”. Es importante destacar que la tendencia a sufrir agudizaciones repetidas es característica de la
EPOC. Los pacientes que sufren un número elevado de agudizaciones en un periodo de tiempo siguen con esa tendencia y sufrirán
agudizaciones en el futuro. La mayoría de las agudizaciones (50-70%) se debe a infecciones, fundamentalmente bacterianas, mientras
que el resto puede estar ligado a factores diversos entre los que se encuentran la polución, el incumplimiento del régimen terapéu co o
la toma no controlada de medicación depresora del SNC, hiperrreactividad bronquial, TEP, neumotórax, sedantes. PATOGENIA: La
infección supone la causa más frecuente de las agudizaciones (entre el 50-70% de las agudizaciones podrían ser atribuidas a infecciones
respiratorias). La explicación de que en muchas de las mismas no podamos determinar su etiología puede deberse a la existencia de
infecciones bronquiales no detectadas con las técnicas diagnósticas habituales. En este sentido, el dato clínico que mejor indica que la
exacerbación está producida por una infección bacteriana es la coloración amarillo-verdosa del esputo. En cuanto a los patógenos
bacterianos se ha observado que están presentes entre el 25-80% de las exacerbaciones; pueden tener un papel primario en el
desarrollo de esta o ser una sobreinfección de un proceso viral inicial. El Haemophilus influenzae es la bacteria aislada con mayor
frecuencia en todas las series (2/3 de las exacerbaciones en general), seguido del Streptococcus pneumoniae, Moraxella catarrhalis y
Pseudomona aeruginosa. Se ha demostrado que un 30% de las muestras en las que se aísla H. influenza pertenecían a pacientes
fumadores activos, lo que implica que los fumadores, por presentar una menor movilidad de los cilios, pueden presentar infecciones
respiratorias con mayor frecuencia. Estos mismos gérmenes se han obtenido en cultivo de esputo y en diversas muestras de más
fiabilidad. DIAGNOSTICO: Clínico; Los síntomas más comúnmente referidos por los pacientes con EPOC exacerbada son: incremento de
la disnea, aumento del volumen o purulencia del esputo, aumento
de la tos, síntomas referidos a vías aéreas superiores (odinofagia),
opresión torácica o escasa tolerancia al ejercicio. Además, se puede
añadir la presencia de fiebre, taquicardia, taquipnea, cianosis y
síntomas derivados de la eventual presencia de insuficiencia
respiratoria. En ocasiones aparece sintomatología cardiovascular
(taquicardia, hipo ó hipertensión, arritmias y signos de insuficiencia
cardiaca derecha) ó síntomas neurológicos (cefalea, irritabilidad,
depresión, cambios en el ritmo de sueño, obnubilación y
encefalopatía). En la exploración física los datos a resaltar son: uso
de la musculatura accesoria de la respiración, movimientos
paradójicos de la caja torácica, aparición o empeoramiento de
cianosis, edema periférico, alteraciones auscultatorias (sibilancias,
roncus, crepitantes), signos de insuficiencia cardiaca derecha o en
casos graves inestabilidad hemodinámica y disminución del estado
de alerta. El diagnóstico de la exacerbación de la EPOC es
eminentemente clínico. Las pruebas diagnósticas que a continuación se enumeran van encaminadas a establecer la severidad de la
exacerbación o a descartar procesos que pueden simular la exacerbación y que requieren un tratamiento específico. Gasometría
arterial; La insuficiencia respiratoria (IR) se define como la presencia de niveles de PaO2 menor de 55 mmHg y/o PaCO2 mayor de 50
mmHg. El pH es muy útil para valorar el grado de hipoventilación aguda insertada sobre la hipoventilación crónica. En la acidosis
respiratoria aguda sin compensación renal, el pH cae 0,08 por cada 10 mmHg de subida de la PCO2. En la acidosis respiratoria crónica
con compensación renal normal, el pH cae 0,03 por cada 10 mmHg de subida de la PaCO2. Hay un grupo de pacientes con EPOC que
tienen valores gasométricos alterados de forma basal; en estos casos hablamos de IR cuando se observa un caída de la PaO2 igual o

CURSO ENARM CMN SIGLO XXI TEL: 36246001 Pharmed Solutions Institute PÁGINA 231
MANUAL DE TRABAJO DEL CURSO ENARM CMN SIGLO XXI
mayor a 10-15 mmHg de su valor basal, y/o cualquier valor de hipercapnia asociado a un pH< 7,30. No está indicada la realización de
gasometría de entrada en todos los pacientes con exacerbación; en los casos leves se puede controlar el nivel de oxigenación
determinando la saturación de 2 con pulsioximetría. Solo realizaremos determinación de gases en sangre arterial en aquellos casos de
exacerbación moderada o grave, o en pacientes leves con saturaciones de O2 bajas. RX de tórax, ECG y otras técnicas de imagen. Los
estudios radiológicos convencionales tienen valor para identificar o descartar diagnósticos alternativos de enfermedades que pueden
cursar con síntomas similares a los de la exacerbación de la EPOC. El ECG es útil para demostrar la presencia de hipertrofia de ventrículo
derecho, arritmias o cardiopatía isquémica. El diagnóstico del EPOC agudizado en el entorno extrahospitalario es fundamentalmente
clínico: aumento de la disnea, acompañado o no de aumento de la tos, expectoración o cambios en coloración del esputo.
TRATAMIENTO: Manejo de exacerbación; el paciente EPOC agudizado puede ser tratado o bien de forma domiciliaria o en el ámbito
hospitalario. Como norma general el paciente EPOC leve–moderado será tratado de forma ambulatoria como primera opción, salvo si la
causa de la exacerbación requiere ingreso o si la evolución no ha sido favorable tras tratamiento ambulatorio. El paciente EPOC grave o
muy grave generalmente requerirá ingreso hospitalario, ya que la exacerbación se asocia con frecuencia a complicaciones de peor
pronóstico como insuficiencia respiratoria, insuficiencia cardiaca, neumonía, TEP. Indicaciones para ingreso hospitalario en EPOC
agudizado: EPOC grave o muy grave, edad avanzada, presencia de comorbilidad asociada (incluida neumonía, arritmia cardiaca,
insuficiencia cardíaca congestiva, DM, insuficiencia renal o hepática), mala respuesta al tratamiento ambulatorio, insuficiencia
respiratoria (Sat de O2 <90% o PaO2 <60 mmHg), taquipnea > 25 rpm, uso de musculatura accesoria, hipercapnia, fiebre alta (> 38,5º),
disminución del nivel de consciencia Respiratorias De ingresar se usan dosis elevadas de broncodilatador en forma nebulizada
(asociación de agonista β2 de acción corta (2.5-10mgs) + bromuro de ipratropio (0.5-1mg) cada 4-6hrs), oxigeno suplementario si
insuficiencia respiratoria. Antibioticoterapia si EPOC grave. Glucocorticoides; metilprednisolona IV 0.4-0.6mg/kg cada 6 hrs durante 3-4
días, posteriormente se reducirá en pauta descendente y se pasara vía oral cuando sea posible. Indicaciones de ingreso en UCI:
Insuficiencia respiratoria aguda en la que a pesar de FiO2 40 % la PaO2 es menor de 60 mmHg. Hipercapnia progresiva con / sin acidosis
respiratoria. Apnea. Fatiga muscular respiratoria

CASOS CLINICOS
Mujer de 84 años de edad con asma bronquial desde la infancia e hipertensión arterial en tratamiento con espironolactona y
amlodipino. Un mes antes de su ingreso presentó una reagudización respiratoria, y se le prescribió por vez primera un compuesto
inhalado cada seis horas, que asociaba bromuro de ipratropio y salbutamol. 2 dias después ingresa a urgencias debido a que presenta
tos productiva escasa con dificultad respiratoria, sibilancias crepitantes, diaforesis, palidez generalizada, cianosis central, se realiza tele
de torax donde se observa imágenes sugerentes de consolidación y reforzamiento de la trama broncovascular, rectificación de
diafragma, sin datos de derrame pleural.

PREGUNTA
Cual es la conducta a seguir mas apropiada.

RESPUESTA
a.- Iniciar con antibiótico de amplio espectro.
b.- Microneubilizacion con teofilina.
c.- Aminofilina.
d.- Prednisona.

CURSO ENARM CMN SIGLO XXI TEL: 36246001 Pharmed Solutions Institute PÁGINA 232
MANUAL DE TRABAJO DEL CURSO ENARM CMN SIGLO XXI
ENFERMEDAD TROMBOEMBOLICA VENOSA = TVP (Trombosis venosa profunda) + TEP (Tromboembolia pulmonar aguda)
CIENCIAS BASICAS: Triada de Virchow, base para TVP y TEP=Lesión o trauma endotelial, disminución del flujo sanguíneos o estasis,
hipercoagulabilidad o estado trombofílico. Se requiere un ambiente reológico propicio = velocidad menor de retorno venoso y corriente
local con rotaciones elipsoidales y casi estáticas. Factores predisponentes: Inmovilización prolongada 2-4 días (25%), antecedente
previo de TVP (19%), neoplasia maligna (17%), cirugía (ortopédica de rodilla 70%, abdominal) o traumatismo + miembros inferiores y
huesos largos, en los últimos 3 meses (13%), uso de anticonceptivos orales (4%), IAM, ICC, edad > 40años, embarazo, parto y puerperio,
obesidad, venas varicosas, viajes largos. Dentro de los estados hipercoagulables; están las trombofilias primarias hereditarias
(deficiencia de antitrombina, proteína C y S, mutación del factor V de Leiden) y adquiridas (síndrome antifosfolipido secundario a LES).
SALUD PUBLICA: Afecta aprox. a 2 millones de personas. Incidencia de 145 por 100,000. La TVP va de un caso por cada 10,000 de
adultos jóvenes y 1 por cada 100 de adultos mayores. En personas de 65 a 69 años es de 1.8 habitantes por cada 1,000 y aumenta a 3.1
en el grupoentre 85-89 años. TEP Causa frecuente de mortalidad en pacientes hospitalizados (5-10%). Toda TEP, tiene su origen en una
TVP o por lo menos el 80%. TEP masiva 0.3%. PATOGENIA: La TVP Se inicia con mayor frecuencia en las piernas (venas gemelares y
soleas). SI hay daño a nivel endotelial se activa la cascada de coagulación, el trombo se asienta sobre el seno valvular, las corrientes de
flujo cambian localmente, disminuye el calibre de la vena, se produce estenosis y mayor fuerza de rozamiento, que permite el
crecimiento del trombo y posteriormente adherencia, al disminuir el retorno venoso de la extremidad, hay acumulación de líquido y
aumento de la presión hidrostática, lo que lleva a Edema, en la zona donde ocurre la oclusión aumenta la extracción de O2, de los
tejidos lo que produce cianosis. Lo peligroso no es que se asiente, si no que siga creciendo y se fragmente y produzca embolismo que
llega a obstruir la circulación arterial pulmonar (TEP) en diferentes grados. En la TEP en estadios tempranos puede tener un GC normal
o levemente elevado, hay hipoxemia (disminución de la relación ventilación/ perfusión), esto aumenta la precarga, generando un
gradiente de presión que llevara a isquemia subendocardica y finalmente falla cardiaca derecha con la consiguiente disminución de la
distensibilidad del VI. En la TEP masiva no hay mecanismos compensatorios, manifestándose clínicamente como taquicardia sinusal y
finalmente como hipotensión arterial sistémica, que llevara a la muerte en poco tiempo. DIAGNOSTICO: TVP: No puede ser afirmado, ni
excluido por clínica, la mayoría asintomático. En parte distal a sitio de oclusión; edema, dolor, calor local, cianosis (+ en región plantar),
circulación colateral, cordón venoso palpable. En caso severo flegmasía cerulea dolens (edema severo, cianosis marcada, flictenas y
puede llegar a gangrena por compromiso de retorno linfático). Signo d Homans: dolor en parte alta de pantorrilla al realizar dorsiflexion
del tobillo. Maniobra de Olow: apretar pantorrilla posterior en busca de dolor. TEP: El dx. requerirá de un alto grado de sospecha
clínica, basándose en; a) uno o mas factores de riesgo, b) comorbilidades (ej. cáncer), c) síntomas y signos clínicos, d) hallazgos
paraclinicos, e) demostración objetiva de hipoxemia y trombosis. Sintomatología como: disnea súbita sostenida, taquicardia,
hipotensión arterial sistólica (<100), síncope, dolor precordial o colapso cardiorrespiratorio, se ha correlacionado con obstrucción
mayor a 30%; otros síntomas diaforesis, tos hemoptisis, edema de MsPs. ECG; proporciona datos inespecíficos; taquicardia sinusal, eje
de QRS >90°, T negativa profundas en V1-V4. Gasometría; hipoxemia y alcalosis respiratoria aguda. Dímero D (DD, es un producto de
degradación de los puentes de fibrina, que se libera a la circulación), se eleva cuando existe un proceso trombotico, si el valor es
<500ng/ml y sospecha clínica baja permite excluir el dx. de TVP. Valores elevados de DD nunca serán suficientes para dx. de evento
tromboembolíco, la ausencia de elevación del DD, prácticamente descartaría un evento trómboembolico. Ecodopler; primera elección
en sospecha de TVP proximales. Flebografía; Gold Standart para dx. de TVP para todo territorio venoso ( contraindicado en insf. renal,
hipertiroidismo, embarazo, alergia a medio). En TEP el gammagrama ventilátorio-perfusorio, es el estudio inicial. La angiografía
pulmonar selectiva actualmente se mantiene como "estándar de oró" para el dx. definitivo de TEP. Escala de Wells; estratificación de
riesgo para trombosis proximales; 1. Cáncer +1; 2. Parálisis o inmovilizavion de miembros inf. +1. 3. Encamamiento > 3 días +1. 4. Dolor
localizado +1. Aumento de tamaño de miembros inferiores +1. Edema en pitting unilateral +1. Dilatación venosa superficial +1. Otros
dx. posibles -2. Wells >2 Alta probabilidaf; <2 Baja probabilidad. TRATAMIENTO: HBPM actualmente es la piedra angular y estándar de
oro para tx. inicial, tiene como mecanismo de acción la unión con la antitrombina III e inhibiendo indirectamente a la trombina,
previniendo la formación de trombo adicional y facilitando la fibrinólisis endógena. Vida media 12hrs. SC. No requiere monitoreo,
puede usarse en tx. ambulatorio, reduce mortalidad. Se puede usar durante embarazo porque no atraviesa la placenta. Enoxaparina
1mg/kg/12hrs. HNF (heparina no fraccionada), precisa control con TTpa, riesgo de trombocitopenia y osteoporosis. Iniciar de forma
simultanea con anticoagulación y antagonistas de vitamina k VO , ya que los anticoagulantes orales tienen efecto procoagulante los
primeros 3 días, anticoagulación oral (acenocumarol) control con INR; mantener de 2-3, primero, segundo y tercer día acenocumarol
mas enoxaparina, en cuarto día podemos retirar la enoxaparina, el tiempo de tratamiento con acenocumarinicos, dependerá del tipo
dé evento y la coexistencia de factores de riesgo. La terapia fibrinolitica con estreptocinasa, urocinasa o factor activador de
plasminogeno, se reserva e indica solo en TEP masiva y datos clínicos de inestabilidad hemodinámica. PREVENCION: Paciente con
factor de riesgo transitorio: anticoagular por 3 meses (ej. embarazo). Paciente con TVP idiopático o estado protrombotico: anticoagular
6-12 meses. Paciente con TVP recurrente o persistencia de algún factor de riesgo: anticoagular indefinidamente. En qx. ortopédica
anticoagular por 30 días, en qx. gnral. tratamiento hasta anticoagulación completa. Profilaxis: 1.- Riesgo bajo; qx. sin complicaciones en
<40 años o qx. menos de 30 min. en >40 años sin factores de riesgo en ambos casos. Recomendaciones: no tx. antitrombotico, solo
deambulación precoz e hidratación adecuada. 2.- Riesgo moderado: qx. gnral en > 40a y duración > 30 min. o <40 años n tx. con
anticonceptivos. Incidencia de TVP distal 10-40%; proximal 2-10%; TEP mortal 0.1-0.7%. Recomendaciones: HBPM c/24 hrs o HNF 5000
U c/8-12 hrs SC, 2 hrs antes de la intervención. Si punción peridural, aplicar 8 hrs antes, por riesgo de hematoma peridural que puede
expandirse y causar paraplejía. 3.- Riesgo elevado: qx. gnral o urológica n >40años con historia de TVP o TEP o en qx. abdominal o
pelvica para tx. de neoplasia o qx. ortopédica mayor en miembros inferiores. Incidencia de TVP distal 40-80%; proximal 10-20%; TEP
mortal 1-5%. Recomendaciones: Prótesis de cadera o rodilla, HBPM c/12 hrs SC. profiláctico cada 12 hrs. terapéutica cada 24 hrs.
PRONOSTICO: Dependerá de la sospecha clínica, la instalación de la profilaxis, y recordar que una TVP hasta en 80%, precede a una TEP

CASO CLINICO
Se trata de paciente masculino de 62 años de edad el cual acude a consulta debido a la presencia de dolor en miembros inferiores,
generando dificultad para caminar. Cuenta con antecedentes de tabaquismo positivo hasta la actualidad refiere que con menor
intensidad que antes comenzando a los 20 años de edad, hace dos años ha presentado leve mareos y en una ocasión fue internado por
dificultad para movilizar su cuerpo, pero se recupero sin secuelas aparentes.

CURSO ENARM CMN SIGLO XXI TEL: 36246001 Pharmed Solutions Institute PÁGINA 233
MANUAL DE TRABAJO DEL CURSO ENARM CMN SIGLO XXI
PREGUNTA
Considerando la clasificación de Rutherford cual es la categoría que presenta el paciente?

RESPUESTA
a.- 2.
b.- 3.
c.- 1.
d.- 4.

PREGUNTA
Considerando las manifestaciones clínica del paciente cual es la causa menos probable de este caso?

RESPUESTA
a.- Compresión de una raíz nerviosa.
b.- Proceso artrítico inflamatorio.
c.- Sindrome compartamental crónico.
d.- Insuficiencia venosa.

PREGUNTA
Cual es la conducta a seguir mas adecuada para establecer una terapéutica adecuada en el paciente para identificar factores de riesgo
modificables?

RESPUESTA
a.- Buscar niveles de glucosa, colesterol y triglicérido.
b.- Suspensión de tabaquismo absoluta.
c.- Holter para identificar origen cardiológico.
d.- Ejercicio secuencial programado.

CASO CLINICO
Acude a consulta paciente femenino de 34 años de edad, originaria de yucatan la cual tiene como antecedentes de importancia 7
semanas de posparto y obesidad grado II, viajo mas de 12 horas por carretera para visitar a familiares, solicita atención debido a
presencia de dolor e inflamación de miembro pélvico derecho, a la exploración se observa dicha extremidad con inflamación,
hipersensibilidad y caliente al tacto, con signo de Hoffman positivo, sus signos vitales son estables TA 115/75 mmHg, FC 81, FR 17,
temperatura 37 C, se realiza doppler apoyando el diagnostico de trombosis venosa, se ingresa para tratamiento.

PREGUNTA
Cuál es el manejo farmacológico más adecuado para esta paciente:

RESPUESTA
a.- Heparina IV
b.- Enoxoparina.
c.- Warfarina.
d.- Acecumarol.

PREGUNTA
Cual es la complicación mas importante que debe evitarse en el paciente?

RESPUESTA
a.- Tromboembolia pulmonar aguda.
b.- Evento embolico fatal subsecuente.
c.- Sindrome compartamental.
d.- ECV.

PREGUNTA
Considerando las reglas predictivas de Wells para trombosis venosa profunda, que calificación presenta el paciente en este momento?

RESPUESTA
a.- 2.
b.- 3.
c.- 4.
d.- 1.

PREGUNTA
Considerando las reglas predictivas de Wells para embolia pulmonar?

RESPUESTA

CURSO ENARM CMN SIGLO XXI TEL: 36246001 Pharmed Solutions Institute PÁGINA 234
MANUAL DE TRABAJO DEL CURSO ENARM CMN SIGLO XXI
a.- 6.
b.- 4.
c.- 9.
d.- 3.

CASO CLINICO
Se trata de paciente femenino de 38 años de edad con antecedentes de importancia para el padecimiento actual, usuaria de
anticonceptivo orales desde hace 8 años, tabaquismo positivo desde los 18 años, actualmente con IMC 32. Acude a urgencias refiriendo
que hace 24 horas regreso de un viaje de 12 horas de camino, el cual realizo en auto ella era la conductora, refiere que comenzó con
dolor en las pantorrillas de predominio izquierdo, con leve dificultad para caminar, posteriormente se presenta malestar generalizado,
dificultad para respirar motivo por el cual se traslada al hospital, a la exploración física se observa diaforética con dolor torácico, disnea,
sus constantes vitales son 95/60 mmHg, FC de 115 lpm, FC de 30 rpm, SaO2 de 90%, campos pulmonares si datos por agregar, se realiza
radiografia de torax, y medición de gases arteriales siendo anormales.

PREGUNTA
Cual es el manejo mas apropiado para el caso.

RESPUESTA
a.- Enoxaparina.
b.- Heparina.
c.- Warfarina.
d.- Tecneplasa

SÍNDROME DE INSUFICIENCIA RESPIRATORIA AGUDA (SDRA):


CIENCIAS BÁSICAS: Antiguamente denominado síndrome de dificultad respiratoria del adulto, es un trastorno que se caracteriza por
una insuficiencia respiratoria aguda hipoxémica debida al edema pulmonar causado por el aumento en la permeabilidad de la barrera
alveolocapilar. El SDRA constituye la manifestación más grave de una serie de respuestas al daño pulmonar agudo; estas respuestas
traducen las complicaciones de una reacción sistémica más extensa a la inflamación o agresión agudas. Se dice que ocurre un daño
pulmonar agudo cuando se observa una hipoxemia grave, de comienzo agudo, y opacidades bilaterales difusas en una radiografía
anterior de tórax, después de excluir una hipertensión en la aurícula izquierda o en los capilares pulmonares. El SDRA se diferencia del
daño pulmonar agudo por la gravedad de la hipoxemia y se define como una relación igual o menor a 200 mmHg entre la pO2 arterial y
la fracción de oxígeno en el aire inspirado (PaO2/FIO2). El daño pulmonar agudo y el SDRA deben considerarse como las
manifestaciones primeras y más fáciles de reconocer de un trastorno sistémico infeccioso o inflamatorio. El pulmón adquiere una
enorme importancia en la agresión sistémica por que recibe todo el gasto cardíaco y porque las alteraciones de su función se reflejan
rápidamente en clínica. Más de la mitad de los casos comienzan en las primeras 24 horas desde la primera agresión. El daño pulmonar
agudo se asocia íntimamente a trastornos que producen un daño alveolar directo o un daño indirecto a través del lecho capilar
pulmonar como e indicen a SDRA: Lesión directa del epitelio alveolar, aspiración, infección difusa, semiahogamiento, inhalación
tóxica, contusión de la vía respiratoria, daño pulmonar indirecto, síndrome séptico, traumatismo no torácico grave, derivación
cardiopulmonar. SALUD PÚBLICA: La probabilidad del SDRA varía según la causa desencadenante, desde un 13 % en los casos de
sobredosis farmacológicas hasta un 43 % en la sepsis. PATOGENIA: El daño pulmonar agudo es la consecuencia de la expresión
excesiva, sin ningún tipo de regulación, de las respuestas inflamatorias generales y habituales a la infección, la agresión o ambas. El
daño afecta al epitelio alveolar y al endotelio capilar pulmonar, consecuencia del acontecimiento que inicia la cascada tan compleja de
reacciones celulares y bioquímicas. Estos acontecimientos atraviesan tres etapas: 1. Iniciación; en la que el factor desencadenante
activa la cascada celular. 2. Amplificación: en la que se reclutan y activan las células efectoras. 3. Daño; fase en la que los
acontecimientos se expresan en los tejidos. El daño lo producen los acontecimientos celulares asociados a los neutrófilos, macrófagos,
monocitos y linfocitos, que sintetizan diversas citocinas; estas, a su vez, determinan una activación, quimiotaxis y adherencia celular.
Las células activadas producen una serie de mediadores inflamatorios, como los oxidantes, las proteasas, las caninas, los factores de
crecimiento, los neuropép dos, los activadores de la cascada del complemento, la coagulación intravascular y la fibrinólisis. La
característica fisiopatológica del SDRA es un aumento en la permeabilidad vascular a las proteínas, que determinan la falta de oposición
al gradiente hidrostática; por eso incluso elevaciones discretas de la presión capilar (producidas por una sobrecarga de líquidos por vía
IV o la disfunción cardiaca característica de la sepsis) aumentan de forma considerable el edema intersticiales y alveolar. Este efecto
aditivo de la permeabilidad y los factores hidrostática, se ilustra como la mayor gravedad del proceso en las zonas declives. El aumento
entre la relación del tejido pulmonar y el gas situado en estas zonas determina que las presiones de cierre alveolar excedan de las
presiones transpulmonares locales y se produzca un cierre y un colapso alveolares. La tendencia al colapso se agudiza por la
disminución cuantitativa de la síntesis de sustancia tensioac va, debido a la agresión de los neumocitos de po II y a otras anomalías en
el tamaño, composición y metabolismo del resto de la sustancia tensioac va depositada. Estas zonas atelectásicas del pulmón
contribuyen a reducir la distensibilidad del pulmón en su conjunto, pero las zonas de pulmón, no situadas en declive, poseen
propiedades mecánicas y de intercambio gaseoso normales. Gran parte de la ventilación y del intercambio gaseoso se desplaza hacia
estas regiones pulmonares intactas; para establecer un símil, es como si la función respiratoria del adulto fuera suplida por un par de
pulmones de un bebé. Dada la distensibilidad menor de los pulmones, es necesario que los músculos respiratorios generen altas
presiones inspiratorias, con lo que aumenta el trabajo de la respiración. Esta mayor carga mecánica explica la fatiga de los músculos
respiratorios, la disminución consiguiente de los volúmenes corrientes y el empeoramiento del intercambio gaseoso. La hipoxemia y la
estimulación de los receptores del parénquima pulmonar rígido determinan un aumento de la frecuencia respiratoria., una disminución
del volumen corriente y un deterioro del intercambio gaseoso. La resistencia en las vías respiratorias aumenta porque el volumen
minuto debe sostenerlo un menor n mero de vías respiratorias ven ladas y por el estrechamiento de las vías causado por el exceso de
líquidos y el broncoespasmo. El intercambio gaseoso se caracteriza por relaciones ven lación-perfusión bajas y un cortocircuito amplio,

CURSO ENARM CMN SIGLO XXI TEL: 36246001 Pharmed Solutions Institute PÁGINA 235
MANUAL DE TRABAJO DEL CURSO ENARM CMN SIGLO XXI
acompañado de un gran espacio muerto. El cortocircuito obedece a la atelectasia, colapso vascular, mal funcionamiento de la sustancia
tensioactiva y atenuación de la vasoconstricción hipóxica. El aumento del espacio muerto se debe a la obstrucción y obliteración del
lecho capilar pulmonar. DIAGNOSTICO: Clínico; al inicio puede no mostrar síntomas, ni signos respiratorios. El signo más precoz es el
aumento de la frecuencia respiratoria, seguido poco después de disnea. En el periodo inicial, la determinación de los gases en sangre
arterial muestra disminución de la Po2 a pesar de la disminución de la Pco2, por lo cual aumenta la diferencia de oxígeno alveolo
arterial. En este estado inicial, la administración de oxígeno produce un incremento significativo de la Po2 arterial. La exploración física
puede ser poco llamativa, aunque se pueden auscultar algunos estertores inspiratorios finos. Radiológicamente, los campos
pulmonares pueden ser claros o mostrar sólo mínimos infiltrados intersticiales focales. A medida que avanza la enfermedad, el paciente
se torna cianótico, con disnea y taquipnea crecientes. Pueden hacerse intensos los estertores, que se oirán fácilmente en todos los
campos pulmonares, junto a zonas de ruido tubáricos; la radiografía de tórax muestra extensos infiltrados intersticiales y alveolares de
carácter bilateral y difuso. En este momento, la hipoxemia no se puede corregir simplemente aumentando la concentración de oxígeno
en el aire inspirados y hay que comenzar con la ventilación asistida. En este estadio más avanzado, el mecanismo principal de la
hipoxemia arterial es el cortocircuito de la sangre desde la derecha a la izquierda, a través de los alveolos colapsados u ocupados. El
diagnostico de IRA se basa fundamentalmente en la determinación de gases arteriales al encontrar PaO2 <50 torr o PaCO2 >50 torr. Las
manifestaciones únicas de hipoxemia o hipercapnia, sirven para el reconocimiento de la presencia de anormalidades importantes en el
recambio gaseoso, mas no para el diagnostico, ya que pueden ocurrir tardíamente o aun faltar en presencia de IRA. La falla respiratoria
puede clasificarse en 2 tipos: Tipo I: llamada también oxigenatoria o hipoxemica; hipoxemia con PaCO 2 normal o bajo. Gradiente
alveolo-arterial de O2 incrementado. Tipo II: denominada así mismo ventiladores o hipercapnica; hipoxemia con PaCO2 elevado y
gradiente alveolo-arterial de O2 normal. TRATAMIENTO: Tratamiento de sostén de la hipoxemia. Para alcanzar una Pao2 de 60 mmHg
(saturación de O2 de aprox. el 90 %) debe emplearse el método más sencillo y la fracción más baja de oxígeno en el aire inspirado. Los
niveles más altos apenas añaden más oxígeno y, en cambio, conllevan el riesgo de toxicidad pulmonar por esta sustancia. Los tres
métodos fundamentales para la oxigenación, en orden de eficacia creciente, son las gafas nasales blandas, las mascarillas faciales
simples y las mascarillas faciales con una bolsa de reserva inspiratoria. Parece razonable empezar con flujos moderados (5-10 L/min de
O2 al 100 %) y controlar la gasometría arterial; el flujo y la concentración de O2 se ajustan en función de los resultados. Soporte
ventilatorio mecánico; en presencia de SDRA, estas medidas poco enérgicas no suelen mantener una oxigenación adecuada, sino que se
requiere la intubación endotraqueal y la respiración asistida con un respirador volumétrico. El motivo de aplicar respiración asistida a
un enfermo que hiperventila no es incrementar la ventilación sino el volumen pulmonar medio, abriendo las vías respiratorias
previamente cerradas y mejorando la oxigenación. El objetivo de la ventilación en el SDRA se basa en proporcionar un soporte
fisiológico para el intercambio gaseoso y evitar las consecuencias mecánicas desfavorables de la intervención. Como el pulmón sufre un
daño heterogéneo en el SDRA, el ajuste de los volúmenes corrientes para lograr este objetivo en todo el pulmón acarrea el riesgo de
sobredistender o romper los alveolos menos afectados y ocasionar la entrada de aire fuera del alveolo (barotraumatismo). Para evitar
esta ultima complicación hay que restringir las presiones de distensión alveolares mediante el uso de volúmenes corrientes
relativamente pequeños (aproximadamente 6 a 10 mL/kg de peso corporal magro) y ajustar la presencia respiratoria, de tal suerte que
el volumen minuto se asocie a un pH superior a 7.25 – 7.30. Dado el volumen tan reducido del pulmón aireado, se necesitan frecuencias
elevadas para alcanzar un volumen minuto adecuado. Tratamiento de la sepsis que causa SDRA: Identificar y tratar cualquier proceso
infeccioso, y Tratar de controlar la respuesta inmunitaria, sin regulación, que provoca el daño pulmonar agudo. Todo foco localizado de
sepsis se debe drenar de inmediato. La mortalidad quirúrgica es elevada, pero si no se drena un foco purulento, lo más probable es que
el enfermo muera. Complicaciones: Las medidas empleadas para tratar el SDRA pueden asociarse a graves complicaciones. La toxicidad
del oxígeno, debido al uso prolongado de una Fio2 superior al 50 %, y la hidratación excesiva empeoran a veces los infiltrados
pulmonares. Un volumen y presión corrientes elevados, por parte del respirador, son causa de barotraumatismo. La ventilación
mecánica prolongadas predispone a la neumonía hospitalaria. El SDRA y los trastornos subyacentes favorecen las fístulas
boncopleurales y la coagulación intravascular diseminada. La fibrosis pulmonar, la hipertensión pulmonar irreversible y la insuficiencia
multiorgánica sugieren un mal pronóstico en los enfermos con SDRA.

CASO CLINICO
Se trata de paciente masculino de 54 años de edad el cual es ingresado a urgencia traido por ambulancia desde una distancia de 4 horas
de camino, refieren los paramédicos que es sobreviviente a accidente automovilístico, al parecer era el conductor, a la exploración
conciente, orientado, con facies algicas, se observa contusion en esternón y parrilla costal de predominio izquierdo, discreta disnea,
hematomas multiples en miembros pelvicos, se encuentra fractura de femur no expuesta, signos vitales TA 105/80 mmHg, FC 105, FR
32. Fue estabilizado y se procedió a realizar cirugía de femur, se traslado a piso al 3 dia el paciente inicia con fiebre, disnea progresiva,
edema de miembros inferiores, se realizo radiografia de torax sin observarse hallazgo patológicos, el electrocardiograma mostro un eje
de QRS mayor a 90 oC y ondas T negativas en V1 a V4.

PREGUNTA
Cual es su impresión diagnostica mas probable con el cuadro clínico del paciente hasta el momento?

RESPUESTA
a.- Tromboembolia pulmonar.
b.- SIRA.
c.- Embolia pulmonar grasa.
d.- Neumotorax.

PREGUNTA
Se realiza tratamiento anticoagulante sin aparente mejoría, a las 12 horas posteriores se incrementa la disnea y ortopnea, intolerancia
al decúbito y ansiedad generalizada, tos con expectoración, dificultad respiratoria con uso de musculos accesorio, se realiza Rx de torax
portátil con sospecha de edema agudo de pulmon, cuales de los siguientes hallazgos no corresponden a ese diagnostico?

CURSO ENARM CMN SIGLO XXI TEL: 36246001 Pharmed Solutions Institute PÁGINA 236
MANUAL DE TRABAJO DEL CURSO ENARM CMN SIGLO XXI

RESPUESTA
a.- Opacidades alveolares diseminadas y bilaterales.
b.- Redistribución del flujo pulmonar.
c.- Distribución en ala de mariposa.
d.- Lineas B de Kerley.

EDEMA AGUDO PULMONAR (EAP):


CIENCIAS BÁSICAS: El EAP no es en sí una enfermedad, pero frecuentemente es una complicación de alguna otra anormalidad, la cual
puede tener origen en los pulmones o más comúnmente, en órganos extrapulmonares, (corazón). El edema pulmonar constituye una
verdadera urgencia médica, y por ello debe identificarse rápidamente. El cuadro puede ser de origen cardiogénico o no cardiogénico.
En el EAP cardiogénico, un fallo de bombeo hace que la sangre retroceda a la circulación pulmonar. Cuando la presión hidrostática
capilar pulmonar supera la presión osmótica coloidal, el líquido es impulsado fuera de los capilares pulmonares hacia el espacio
intersticial y el alveolo. En el de origen no cardiogénico, las paredes de las células capilares pulmonares se vuelven más
permeables. Constituye una de las urgencias más graves en pacientes con enfermedades del corazón izquierdo. curre por aumento de
la presión capilar pulmonar secundaria a estenosis mitral o insuficiencia cardiaca izquierda, de ahí que también se le conozca como
“edema pulmonar cardiogénico”. PATOGENIA: El EAP es el acumulo excesivo de líquido extravascular en el pulmón, ya sea en el
intersticio (edema intersticial) o en el alveolo (edema alveolar). Para mantener seco el intersticio pulmonar funcionan varios
mecanismos delicados: 1. Presión osmótica superior a la presión capilar pulmonar; Las fuerzas hemodinámicas básicas opuestas son la
presión capilar pulmonar (PCP) y la presión osmótica del plasma. En los individuos normales la PCP oscila entre los 7 y los 12 mmHg,
siendo la presión osmótica del plasma de 25 mmHg aproximadamente, por lo que .esta fuerza tiende a impulsar el líquido de regreso a
los capilares. 2. Tejido conjuntivo y barreras celulares relativamente permeables a las proteínas plasmáticas; La presión hidrostática
actúa a través del tejido conjuntivo y la barrera celular, que en circunstancias normales son relativamente impermeables a las proteínas
plasmáticas. 3. Extenso sistema linfático; El pulmón posee una extensa red linfática que puede aumentar su flujo en 5 a 6 veces cuando
se encuentra con un exceso de agua en el inters cio pulmonar. Cuando los mecanismos normales para mantener el pulmón seco
funcionan mal o están superados por un exceso de líquidos el edema ende acumularse. Las causas de EAP son:
Cardiogénicas: IAM, arritmias cardiacas, insuficiencia ventricular izquierda grave, shoc cardiogénico. No cardiogénicas: Inhalación de
gases irritantes, neumonía por aspiración, Shock séptico, embolia grasa, síndrome de distrés respiratorio del adulto, administración
rápida de líquidos intravenosos, sobredosis de barbitúricos u opiáceos. DIAGNOSTICO: Agitación, angustia, nerviosismo. La valoración
del edema agudo del pulmón a nivel respiratorio es laborioso; disnea (el edema pulmonar dificulta la distensibilidad pulmonar y hace
más difícil la expansión de los pulmones). Hipoxemia (debido al deterioro del intercambio gaseoso). Existencia de hipoxia con
hipocapnia (hiperventilación) ya que elimina gran cantidad de CO2 al respirar tan dificultosamente, desarrollando una alcalosis
respiratoria. Taquipnea 30-40 min (debido a la hipoxia el paciente respira más de prisa). Crepitantes y sibilancias (al pasar el aire a
través de las vías aéreas llenas de liquido, durante la inspiración). Valoración cardiovascular: Taquicardia y aumento de la presión
arterial (para compensar el déficit de aporte de oxigeno, el sistema simpático aumenta la frecuencia cardiaca). El sistema nervioso
simpático provoca vasoconstricción que aumenta la presión arterial. Distensión de las venas yugulares. Esputo espumoso teñido de
sangre (debido a la presión, algunos vasos pulmonares se rompen haciendo que los hematíes pasen a los pulmones). Piel pálida, fría y
sudorosa (para conservar el oxigeno, el organismo deriva sangre arterial desde la piel hacia órganos vitales). Tercer ruido cardiaco
(ritmo de galope), producido cuando durante el principio de la diástole la sangre entra de prisa en el ventrículo izquierdo
distendiéndolo y da lugar a vibraciones anormales. En el tórax puede observarse el empleo de los músculos accesorios de la respiración
(tiros intercostales). Puede haber estertores audibles a distancia y sin necesidad del estetoscopio. uscar intencionadamente la
presencia de ritmo de galope que permi rá establecer el diagnós co de insuficiencia cardiaca, mientras que con la auscultación de un
ritmo de Duroziez haremos el diagnóstico de estenosis mitral. La radiografía de tórax es el estudio más útil para el diagnóstico en
conjunto con la historia clínica del EAP, ya que muestra la imagen correspondiente al edema alveolar como una opacidad diseminada
en ambos campos pulmonares. Cuando los grados de hipertensión venocapilar pulmonar son menores, podrán observarse otros datos
como son: la redistribución de flujo a los vértices, la cisura interlobar visible, las líneas de erley o un moteado fino difuso y en
ocasiones la imagen en “alas de mariposa”; la presencia de cardiomegalia orientará hacia el diagnós co de insuficiencia cardiaca,
mientras que la ausencia de ella hablará de disfunción diastólica. El electrocardiograma ayuda a identificar la posible cardiopatía
subyacente o los factores desencadenantes, como un infarto del miocardio o una taquiarritmia como la fibrilación auricular.
TRATAMIENTO: El manejo terapéutico de esta grave complicación tiene tres objetivos: 1. Disminuir la presión venocapilar: 2. Mejorar la
ventilación pulmonar. 3. Tratamiento de la enfermedad causal. Medidas que disminuyen la hipertensión venocapilar: a). Sentar al
paciente al borde de la cama con las piernas pendientes para disminuir el retorno venoso al corazón. b). Aplicación de torniquetes
rotatorios en tres de las cuatro extremidades, con la fuerza necesaria para que no desaparezca el pulso arterial y se cambia cada 15
minutos a la extremidad que se encuentra libre (“sangría seca”). c). Isosorbide o nitroglicerina por vía sublingual. La administración de
una tableta de cualquiera de estos dos fármacos tiene un poderoso efecto vasodilatador que reduce drásticamente el retorno venoso y
la congestión pulmonar. d). La administración intravenosa de furosemida a razón de 20 a 60 mg promueve con gran rapidez la
movilización de líquidos del intersticio pulmonar hacia el riñón, por lo que se reduce con gran eficiencia la congestión pulmonar. e).
Nitroprusiato de sodio.- Este poderoso vasodilatador mixto se administra por vía intravenosa a razón de 0.3 a 0.8
microgramos/Kg/minuto, cuando el EAP es consecutivo a insuficiencia ventricular por crisis hipertensiva. En estos casos, esta medida es
de elección para el tratamiento del edema pulmonar. El efecto arteriodilatador reduce significativamente la postcarga y con ello mejora
la función ventricular y el gasto cardiaco, y por otro lado, el efecto venodilatador reduce el retorno venoso al corazón y la congestión
pulmonar. Medidas que mejoran la ventilación pulmonar: a). Administración de oxígeno mediante puntas nasales a razón de 4 litros por
minuto. b). Aminofilina 1 ampolleta de 250 mg muy lenta por vía intravenosa; su efecto broncodilatador mejora la ventilación pulmonar
así como su efecto diuré co potencia la acción de la furosemida. Tratamiento de la enfermedad causal: a). La mayoría de los pacientes
que presentan EAP por estenosis mitral mejoran con las medidas antes anotadas. Cuando a pesar de ellas continúa el cuadro clínico, se
requiere la intubación del paciente para administrarle asistencia mecánica a la ventilación pulmonar, especialmente la presión positiva

CURSO ENARM CMN SIGLO XXI TEL: 36246001 Pharmed Solutions Institute PÁGINA 237
MANUAL DE TRABAJO DEL CURSO ENARM CMN SIGLO XXI
respiratoria final (PEEP) que evita el colapso de las vías respiratorias pequeñas y con ello asegura la ven lación alveolar. En estas
condiciones, el paciente deberá ser llevado al quirófano para realizar comisurotomía mitral de urgencia, que en estos casos es la única
medida que alivia el cuadro de edema pulmonar. b). Cuando la insuficiencia ventricular izquierda es la causa, se requiere la
digitalización rápida con ouabaína o lanatósido C, además de las medidas para reducir la presión capilar y mejorar la ventilación
pulmonar. c). El edema pulmonar que es causado por disfunción diastólica, usualmente se presenta en la evolución de un infarto del
miocardio agudo; debe ser tratado con las medidas que reducen la presión capilar pulmonar asociada a aquellas otras que mejoran la
ventilación pulmonar y ambas a su vez, a las que reducen el efecto de isquemia miocárdica como lo son la administración de
betabloqueadores (propranolol 10 a 20 mg cada 8 horas) o calcio antagonistas (diltiazem 30 mg cada 8 horas), ya que estos fármacos
reducen el efecto que la isquemia tiene sobre la relajación ventricular. En efecto, cuando la relajación se hace más rápida y completa,
se reduce significativamente la presión diastólica del ventrículo izquierdo y con ello la hipertensión venocapilar. Estabilización
hemodinámica: Se considera una buena respuesta si a los 15 a 30 minutos del inicio del tratamiento, mejora la disnea, la frecuencia
cardiaca, disminuyen los estertores pulmonares, aparece una coloración más oxigenada de la piel y se produce una buena diuresis,
después de la cual se le puede ofertar al paciente potasio, 1 ampolleta por vía oral con agua para compensar las pérdidas de de este ion
o adicionárselo a la infusión de la venoclisis

CASOS CLINICOS
Mujer de 25 años de edad, primigesta, admitida por cuadro de preeclampsia severa, con presiones arteriales > 160/110 mmHg,
manejada con sulfato de magnesio y labetalol intravenoso sin obtener adecuada respuesta, por lo que se realiza cesárea, que culmina
con recién nacido sano, se traslada posteriormente a unidad de cuidados intensivos (UCI) para manejo con infusión de vasodilatadores
y vigilancia neurológica por riesgo elevado de eclampsia. Al ingreso en UCI presentó presión arterial de 161/120 mmHg, taquicardia,
inadecuado patrón respiratorio, saturación de oxígeno arterial (SaO2) del 89%, crepitantes bibasales y presión venosa central (PVC) de
16 mmHg. Se realiza radiografía de tórax que muestra cefalización del flujo, con hilios pulmonares congestivos; se considera entonces el
diagnóstico de EAP.

PREGUNTA
Cual es la conducta mas adecuada para el caso?

RESPUESTA
a.- Ventilacion mecánica.
b.- Antibioticoterapia profiláctica.
c.- Furosemide y nitroglicerina.
d.- Sindenafil.

CASO CLINICO
Paciente de 68 años de edad, fumador con criterios clínicos de EPOC, que acudió al servicio de urgencias por disnea progresiva hasta
hacerse a mínimos esfuerzos de 2 semanas de evolución. No refería traumatismo torácico. Radiológicamente, se diagnosticó un
neumotórax izquierdo. Se insertó un tubo de drenaje torácico, se reexpandió completamente el pulmón y mejoró así la clínica del
paciente. Tres horas más tarde tuvo un deterioro clínico consistente en dificultad respiratoria grave. Se auscultaban estertores
húmedos de gruesa burbuja en el hemitórax izquierdo. Una nueva RX de tórax mostró imágenes compatibles con edema pulmonar
unilateral. Fue preciso conectarlo a ventilación mecánica y que recibiera tratamiento vasoactivo.

PREGUNTA
Cual es la etiología mas frecuente del caso.

RESPUESTA
a.- Arritmia cardiaca.
b.- Isquemia miocárdica.
c.- Insuficiencia cardiaca.
d.- Complicaciones del parénquima pulmonar.

NEUMOTÓRAX, NEUMOMEDIASTINO, HEMOTÓRAX, CONTUSION CARDIACA:


NEUMOTÓRAX: Presencia de aire en el espacio pleural que causa un mayor o menor colapso del pulmón, con la correspondiente
repercusión en la mecánica respiratoria e incluso en la situación hemodinámica del paciente. La presión intrapleural es subatmosférica
durante todo el ciclo respiratorio, y es el resultado de las fuerzas elásticas opuestas del pulmón y de la pared torácica. La presión
intraalveolar es relativamente positiva en relación con la intrapleural, ayudando a mantener el pulmón en expansión durante la
respiración. La entrada incidental de aire en el espacio pleural provoca una pérdida de la presión negativa intrapleural y el pulmón
tiende a colapsarse por su propia fuerza retráctil. Si la cantidad de aire es importante, la presión pleural se aproxima a la atmosférica, lo
que puede causar un colapso de todo el pulmón. Es frecuente que exista hipoxemia arterial como consecuencia de la desproporción de
la relación ventilación/perfusión que se produce en el pulmón colapsado (shunt intrapulmonar). La entrada de aire al espacio pleural
desde el pulmón, como consecuencia del desgarro de la pleural visceral, es la causa más frecuente de neumotórax. NEUMOTÓRAX
ESPONTÁNEO (NE): Ocurre en ausencia de antecedente traumático o yatrogénico que lo justifique. El NE se subclasifica en: NE
primario; ocurre en individuos aparentemente sanos, sin enfermedades pulmonares conocidas. Afecta a jóvenes, con un pico de
incidencia entre los 20 y 40 años de edad, y es más frecuente en varones (6:1), con predilección por individuos altos y delgados,
fumadores. El sustrato patológico más frecuente es la presencia de pequeñas bullas subpleurales apicales (blebs) cuya rotura ocasiona
la salida de aire desde el pulmón hacia la cavidad pleural. El mecanismo de formación de estos blebs y los factores que precipitan su
rotura no son bien conocidos. NE secundario Ocurre en pacientes con patología pulmonar previa. Personas de mayor edad, excepto en

CURSO ENARM CMN SIGLO XXI TEL: 36246001 Pharmed Solutions Institute PÁGINA 238
MANUAL DE TRABAJO DEL CURSO ENARM CMN SIGLO XXI
los casos de fibrosis quística, EPOC es la causa más frecuente y la probabilidad de neumotórax es mayor cuanto más avanzada es la
enfermedad. En estos pacientes el neumotórax se debe a la rotura de bullas intrapulmonares y, dado que su reserva funcional es ya
muy limitada. Del 2 al 4% de pacientes con SIDA pueden desarrollar neumotórax. La neumonía por Pneumocystis carinii es la etiología
más probable en estos pacientes. La tuberculosis pulmonar y las neumonías necrotizantes, fundamentalmente las causadas por
estafilococo, también pueden ser causa de neumotórax. Aunque es rara, no debemos olvidar una entidad como el neumotórax
catamenial, que ocurre en mujeres con antecedentes de endometriosis entre otras. NE ADQUIRIDO: Iatrogénico: consecuencia de
procedimientos invasivos, como toracocentesis, biopsia pulmonar transbronquial, biopsia pleural, lavado broncoalveolar, punción
pulmonar transtorácica, el neumotórax por barotrauma (complicación de la ventilación mecánica, por rotura de alveolos).
NEUMOTORAX TRAUMATICO: neumotórax traumático abierto es consecuencia de una herida penetrante en el tórax, que pone en
comunicación el espacio pleural y la atmósfera exterior (entrada de aire atmosférico), y a su vez suele lesionar también el pulmón
(salida de aire alveolar). El neumotórax traumático cerrado está causado habitualmente por una fractura costal, rotura bronquial o
lesión esofágica. Síntomas más frecuentes son: Dolor torácico pleurítico de inicio agudo, que puede ser intenso inicialmente, pero
después de unas horas suele evolucionar a dolor sordo e incluso cede espontáneamente, disnea, que suele aparecer con mayor
frecuencia e intensidad en pacientes con enfermedad pulmonar subyacente. En el NE primario suele ceder habitualmente en el plazo
de 24 horas. Otros síntomas (menos frecuentes): tos improductiva, síncope, hemoptisis, debilidad de extremidades superiores,
sensación de ruido extraño en el tórax. Exploración física: hiperinsuflación y disminución del movimiento del hemitórax afectado,
disminución o abolición del murmullo vesicular en el hemitórax afectado, disminución de la transmisión de la voz, timpanismo,
disminución de las vibraciones vocales. Palpación hepática por aplanamiento diafragmático y desplazamiento del hígado. El
neumotórax a tensión debe sospecharse ante la presencia de taquicardia superior a 135 latidos por minuto, hipotensión, o cianosis.
Tratamiento: se dirige a la eliminación del aire inrapleural, así como la prevención de ataques de recurrencia, se basa en administración
de oxígeno, observación y aspiración de aire intrapleural. NEUMOMEDIASTINO: Aire en el mediastino no acompañado de enfermedad
inflamatoria o infecciosa (también se conoce como enfisema mediastinal) y es primordialmente una complicación de la ventilación
mecánica (barotrauma9, aunque también puede ocurrir espontaneo o secundario a trauma torácico o asma. Ocurre principalmente por
ruptura alveolar microscópica, así como por escape de aire de la vía aérea superior o gastrointestinal. Se presenta con dolor
subesternal, que empeora con los movimientos respiratorios y cambios de posición. También puede presentarse con disnea, disfagia,
enfisema subcutáneo y disfonía. El tratamiento depende de la causa primaria. HEMOTÓRAX: Presencia de sangre en el espacio pleural.
El término hemotórax está reservado para los casos en los cuales el hematócrito del líquido pleural es al menos un 50% del
hematócrito de la sangre periférica. Cuando en una toracocentesis diagnóstica se obtiene líquido hemático, debemos considerar la
medición de su hematócrito. Los podemos clasificar en traumáticos, no traumáticos o espontáneos y iatrogénicos. Traumático; la
sangre puede llegar al espacio pleural por lesión de la pared torácica, diafragma, parénquima pulmonar, vasos sanguíneos o desde
estructuras mediastínicas. Cuando la sangre se acumula en el espacio pleural, ésta tiende a coagularse rápidamente, como resultado
de los movimientos producidos por el corazón y los pulmones. Si el impacto ha sido muy intenso y la situación del paciente lo permite,
la realización de un TAC podrá aportar importante información de posibles lesiones asociadas. De acuerdo a la extensión radiológica el
hemotórax puede clasificarse en tres grados: 1°. El nivel del hemotórax se encuentra por debajo del cuarto arco costal anterior. 2°. El
nivel se encuentra entre el cuarto y segundo arco costal anterior. 3°. El nivel está por encima del segundo arco costal anterior. Esta
clasificación es utilizada como guía por algunos autores para decidir la conducta terapéutica: drenaje con tubo pleural en hemotórax de
primer grado, toracotomía en aquellos de tercer grado y en aquellos de segundo grado se puede comenzar con drenaje torácico y,
eventualmente, toracotomía de acuerdo a la evolución clínica del paciente. Los pacientes con hemotórax traumático deben ser tratados
lo antes posible con drenaje torácico. Esto permite una completa evacuación de la sangre del espacio pleural, cuantificar las pérdidas,
que disminuya la incidencia posterior de empiema, realizar autotransfusión del material obtenido y la rápida evacuación de la sangre
del espacio pleural evitando un fibrotórax. Aproximadamente 20% de los pacientes con hemotórax requiere toracotomía. La
toracotomía inmediata está indicada en: heridas penetrantes con la sospecha o constatación de lesiones viscerales graves y en
pacientes con hemorragia pleural persistente. Las complicaciones más frecuentes de un hemotórax son la retención de coágulos en la
cavidad pleural, el empiema pleural, el derrame pleural persistente y el fibrotórax. Hemotorax iatrogénico; cuando un paciente
ingresado presenta un derrame pleural hemático, se debe considerar el origen yatrogénico. La causa más frecuente de es la perforación
de una vena central o arteria por la inserción percutánea de un catéter, tras una punción o biopsia pleural, punción pulmonar
percutánea, manejo endoscópico para tratar las varices esofágicas. Hemotórax no traumático: son poco frecuentes. La causa más
común son las metástasis pleurales, mientras que la segunda causa más frecuente es la complicación del tratamiento anticoagulante
por embolia pulmonar o patología cardiaca. El hemotórax espontáneo puede ocurrir como resultado de la ruptura anormal de un vaso
sanguíneo intratorácico, como un aneurisma de aorta, aneurisma de arteria pulmonar, un ductus arterioso o una coartación de aorta.
En algunos pacientes la causa permanece desconocida a pesar de la toracotomía exploradora. CONTUCION CARDIACA: Es la más
frecuente de las lesiones que resultan de traumatismos cardíacos contusos. Su intensidad es variable y raramente es fatal. El
mecanismo más postulado para su producción es la desaceleración, donde el corazón golpea contra el esternón siendo comprimido por
la columna. El diagnóstico de esta lesión es demorado o confundido por otras lesiones más graves y agudas que desvían la atención.
Debe plantearse ante un trauma de pared torácica anterior con o sin fracturas costales o de esternón, shock sin taponamiento,
insuficiencia cardíaca, arritmias y frotes pericárdicos a la auscultación. Diagnóstico: ECG; puede estar alterado, principalmente al
ingreso (extrasístoles ventriculares, extrasístoles auriculares, fibrilación auricular, bloqueo de rama, alteraciones del ST). Tiene
sensibilidad de 100% especificidad de 47%. Enzimas Cardíacas (CK-MB, troponina I): se elevan igual que en un IAM y su toma debe de
ser seriada. Valores de troponina I < de 1.05 mcg/L entre el ingreso y las 6 primeras horas descartan CM en un paciente asintomático,
mientras que valores > de 1.05 mcg/L son indicadores de continuar el estudio y la observación. Si un paciente con sospecha de CM
tiene un ECG y una troponina normal a las 8 horas de observación se puede descartar prácticamente con certeza una CM. ECO: se
observan alteraciones del movimiento de las paredes comprometidas, aquinesia, disquinesia e hipoquinesia. Se sugiere realizar
ecocardiografía en los pacientes inestables, en los que presentan arritmias o en los que irán a cirugía. Manejo: Debido al riesgo de que
aparezcan alteraciones del ritmo cardíaco, los pacientes deben estar monitorizados continuamente en una UCI o área controlada
durante las primeras 24 horas, si en las primeras 24 horas no se pesquisa complicación la evolución suele ser favorable. Los pacientes
con contusiones documentadas y compromiso hemodinámico, que requieren cirugía con anestesia general para tratar sus lesiones

CURSO ENARM CMN SIGLO XXI TEL: 36246001 Pharmed Solutions Institute PÁGINA 239
MANUAL DE TRABAJO DEL CURSO ENARM CMN SIGLO XXI
asociadas, deben monitorizarse perioperatoriamente con técnicas hemodinámicas invasivas (línea-A, catéter de Swan-Ganz). En
resumen debe tratarse en forma similar al IAM, dejando en claro que este cuadro no aumentaría la mortalidad perioperatoria en el
caso de ser necesaria la cirugía de urgencia.

CASO CLINICO
Paciente de 68 años de edad, fumador, que acudió por disnea progresiva hasta hacerse a mínimos esfuerzos de 2 semanas de
evolución. No refería traumatismo torácico. Radiológicamente, se diagnosticó un neumotórax izquierdo. Se insertó un tubo de drenaje
torácico, se reexpandió completamente el pulmón y mejoró así la clínica del paciente. Tres horas más tarde tuvo un deterioro clínico
consistente en dificultad respiratoria grave. Se auscultaban estertores húmedos de gruesa burbuja en el hemitórax izquierdo. Una
nueva RX de tórax mostró imágenes compatibles con edema pulmonar unilateral.

PREGUNTA
Cual es la conducta mas apropiada en el estado actual?

RESPUESTA
a.- Furosemide y nitroglicerina.
b.- Sindenafil.
c.- Ventilacion mecánica.
d.- Aminas vasoactivas.

CASO CLINICO
Varón de 30 años de edad. Refería tos seca con una data de un mes, sudoración nocturna, dificultad respiratoria progresiva y baja de
peso estimada en 10 kg. No refería fiebre y una semana antes de ser internado se le agregó diarrea líquida. Antecedente de
promiscuidad homosexual y gonorrea 8 años antes. Ingresó vigil, orientado, temperatura de 37 ºC, presión arterial de 100/65, pulso de
108/minuto, taquipneico con frecuencia respiratoria de 50/minuto. EF: adenopatías pequeñas, móviles, no adheridas a planos
profundos e indoloras, tanto cervicales laterales y posteriores así como axilares e inguinales; boca con placas blanquecinas que
comprometían la lengua, cara interna de mejillas y faringe; pulmones con estertores finos en ambas bases. El examen cardiovascular y
abdominal carecía de hallazgos patológicos. En la exploración de laboratorio tenía: LDH de 833 U/L; hematocrito 44%; leucocitos 6.900
x mm3 (segmentados 87%, linfoncitos 8%). Radiografía de tórax con infiltrados bilaterales y datos compatible con neumotórax.
PREGUNTA
Cual es el agente mas probable relacionado con el desarrollo de neumotórax considerando los antecedentes?

RESPUESTA
a.- M. avium.
b.- P. jiroveci.
c.- C. albicans.
d.- S. pneumonie.

CASO CLINICO
Paciente de 53 años que es traído a Urgencias tras sufrir un accidente de tráfico, al parecer es el que conducía. Refiere que se ha salido
de la carretera y recuerda todo lo que ha pasado (no ha perdido el conocimiento). Exploración física: Llega consciente y orientado,
mueve todos los miembros y las pupilas son normales. Refiere intenso dolor torácico, especialmente en hemitórax izquierdo. Respira
espontáneamente con gran trabajo respiratorio, taquipnea y sudoración. La auscultación pulmonar muestra hipofonesis en la mitad
inferior del hemitórax izquierdo y la percusión es mate. El resto de campos pulmonares es normal. No tiene dolor a otros niveles y la
exploración cardiaca, de abdomen y de miembros es normal.

PREGUNTA
Cual es el diagnostico mas probable?

RESPUESTA
a.- Neumotorax.
b.- Hemotorax.
c.- Hemomediastino.
d.- Neumomediastino.

CASO CLINICO
Se trata de masculino de 27 años de edad el cual cuenta con antecedentes de importancia tales como padre con enfisema pulmonar y
madre cardiópata, presenta tabaquismo positivo desde los 17 años, actualmente consume marihuana y ha probado el crack, refiere que
hace 4 horas inicia súbitamente con dolor en el torax del lado derecho, asi como dificultad para respirar, niega tos, fiebre o malestar
previo, refiere que se encontraba trabajando en su taller mecanico, a la exploración física se observa ansioso, se encuentra afebril, no
se observa edema periférico, su frecuencia respiratoria es de 25 rpm, FC 84 lpm, TA 120/90 mmHg, saturación de oxigeno del 93 %, se
realiza tele de torax donde se observa neumotórax del 50 % del pulmon derecho.

PREGUNTA
Cuál es la conducta a seguir.

CURSO ENARM CMN SIGLO XXI TEL: 36246001 Pharmed Solutions Institute PÁGINA 240
MANUAL DE TRABAJO DEL CURSO ENARM CMN SIGLO XXI
RESPUESTA
a.- Sello pleural.
b.- Aspiracion con aguja.
c.- Observacion.
d.- Toracoscopia.

TRAUMA TORACICO CERRADO Y ABIERTO:


CIENCIAS BASICAS: Es cualquier agresión o trauma sobre las paredes del tórax que producirá un daño en las estructuras sólidas y partes
blandas comprendidas en la caja torácica. Los traumatismos torácicos pueden ser cerrados (contusos) o abiertos (penetrantes).
Traumatismo abierto: se denomina a lesión que rompe la integridad del tejido (atraviesa pleura parietal). Traumatismo cerrado: resulta
por aplicación de energía que provoca lesión sobre los tejidos sin dañar su integridad. SALUD PUBLICA: En México los traumas cerrados
son mayoritariamente por accidente de tránsito, los penetrantes TRAUMA TORACICO CERRADO TRAUMA TORACICO ABIERTO
son por arma blanca, aunque han aumentado las heridas por armas Asociada a compresión y aceleración- Asociada a heridas por arma blanca
de fuego. 8 de cada 100.000 son letales. Principales causas de desaceleración y arma de fuego
Hay fracturas costales múltiples Puede o no haber fracturas costales
traumatismo torácico asociadas: Accidentes de tránsito (43%),
Puede haber hemo o neumotórax Hemo o neumotórax inmediato.
Suicidios (29%), Homicidios (22%.). Nos da gravedad en un Trauma tardío (>24hrs del trauma)
torácico: Impactos de alta energía: caída mayor a 6 metros. El trauma de vía aérea superior se El trauma de vía aérea cursa con
Impactos de alta velocidad. Pasajeros despedidos del vehículo. manifiesta como estenosis gran escape aéreo
Atropello. Lesión penetrante de cabeza, cuello, tórax, abdomen o Tratamiento quirúrgico: requerido en Tratamiento quirúrgico: requeridos
región inguinal. Dos o más fracturas proximales de huesos largos. menos del 10% de lesionados entre 15-30% de lesionados.
Quemaduras mayores al 15% de SC o que afecten cara o vías aéreas. Tórax inestable. TIPOS DE TRAUMA TORÁCICO: Lesiones torácicas
letales: 1.-OBSTRUCCIÓN DE LA VÍA AÉREA; se puede producir por cuerpos extraños, secreciones, sangre, el manejo puede ser invasivo
o no invasivo, otra clasificación es “vía aérea quir rgica” y “no quir rgica”. Consisten en el manejo manual de la vía aérea, intubación
oro o nasotraqueal, cricotiroidotomia por punción o quirúrgica y la traqueotomía. 2.- NEUMOTÓRAX A TENSIÓN; es el escape de aire
hacia la cavidad pleural, mas común neumotórax espontáneos, el mediastino y la tráquea se desplazan hacia al lado opuesto,
comprometiendo la posibilidad de respuesta ventilatoria por parte del pulmón sano, y afectando el retorno venos. Clínicamente se
manifiesta por dificultad respiratoria, taquicardia, hipotensión, desviación de la tráquea, ausencia unilateral de MV, timpanismo del
pulmón afectado, ingurgitación yugular y cianosis tardía. El tratamiento inicial consiste en insertar una aguja, en 2° EIC en LMC del
hemitórax afectado, siempre por el borde superior de la 3° costilla. El definitivo es la inserción de un tubo de tórax en el 5° EIC anterior
a la LAM, siempre por el reborde costal superior de la 6° costilla, en el hemitórax afectado. 3.-NEUMOTÓRAX ABIERTO; Al producirse
una herida en la pared torácica el aire penetrará preferentemente por la herida al igualarse las presiones intratorácica y ambiental y
ofrecer menor resistencia al paso del aire por la herida. El tratamiento inicial será cubrir la herida con un apósito fijado en tres puntas.
El tratamiento definitivo consiste en la instalación de un tubo de tórax, distante de la lesión, y el cierre de la herida será quirúrgico.
Complicaciones: Inadecuada conexión drenaje – aspirador. Inadecuada colocación del drenaje. Oclusión bronquial. (Cuerpo extraño,
coágulo, rotura). Roturas traqueobronquiales. Grandes laceraciones pulmonares. Severa disminución de la distensibilidad pulmonar. 4.-
HEMOTÓRAX MASIVO: Es el resultado de la acumulación de sangre en la cavidad pleural, igual o superior a 1500 ml. La principal causa:
es la lesión de vasos hiliares y mediastinicos generalmente por heridas penetrantes. Clínicamente encontrará un paciente en shock, con
colapso de los vasos del cuello por hipovolemia o con ingurgitación de estos por efecto mecánico de las cavidades. El tratamiento será
simultáneamente con reposición de volumen de forma agresiva (cristaloides, coloides y sangre), y descompresión del hemitórax
lesionado con un tubo de tórax único N° 28 – 32 f. Se hará toracotomía sí: Deterioro hemodinámico sin otra justificación. Persiste un
drenaje superior a 1500 ml en las primeras 12 – 24 horas. Drenaje superior a 200 ml/hora en 4 horas. Persistencia de la ocupación
torácica (hemotórax coagulado). 5.- TÓRAX INESTABLE: Clínicamente puede no ser detectado en primera instancia por la
hipoventilación reactiva al dolor, y por los movimientos del tórax. El tratamiento se basa fundamentalmente en una buena ventilación,
eventualmente mecánica, oxigenación, y tratamiento para el dolor. Si el paciente no está en shock la infusión de fluidos debe ser
cuidadosa para evitar la sobrehidratación y el consiguiente edema pulmonar. Un adecuado y controlado balance hídrico. Cuando
coexisten fracturas costales múltiples en varias costillas consecutivas se produce una inestabilidad de la pared con movimiento
paradojico y alteración de la mecánica respiratoria, con la consiguiente hipoxia. La gravedad de la lesión es directamente proporcional
al grado de alteración del parénquima pulmonar en combinación con el daño de la pared. 6.- CONTUCION PULONAR: Lesión del
parénquima pulmonar que causa hemorragia y edema localizado, producto de traumas en los que hay rápida compresión y
descompresión del tórax. Se observa falla respiratoria tardía, lenta, progresiva y sutil, con o sin tórax inestable. 7.- CONTUCION
MIOCÁRDICA: Difícil de diagnosticar, se sospecha por alteraciones al ECG (arritmias, extrasístoles mono o bifocales, taquicardia sinusal
inexplicable, FA, bloqueo de rama, o claramente un infarto), Eco cardiografía bidimensional e historia compatible. El tratamiento
tratara la manifestación clínica o la arritmia específica. 8.- RUPTURA DE AORTA: Producto de traumatismos cerrados, por laceración o
arrancamiento de los puntos de fijación de la Aorta. Signos clínicos alertan el diagnóstico: Mediastino ensanchado en Rx de Tx. Fx de
1ª y 2ª costillas. Desviación y elevación del bronquio principal, de la tráquea y el esófago hacia la derecha. Depresión del bronquio
principal izquierdo. Opacidad pleural apical. Tratamiento: quirúrgico (reparación o implante). 9.- RUPTURA DIAFRAGMÁTICA: Presente
con más frecuencia en el lado izquierdo ya que se carece de la protección del hígado. Se sospecha el diagnóstico por la presencia de
intestino, estomago en el hemitórax izquierdo. Los traumatismos penetrantes por arma blanca o de fuego a veces pasan inadvertidos y
solo se detectan años después cuando aparece la hernia diafragmática. 10.- LESIÓN TRAQUEOBRONQUIAL: Laringe Diagnóstico:
ronquera, enfisema subcutáneo y crepitación palpable de fractura. Manejo, en caso de vía aérea obstruida: instalación de IOT o
Traqueotomía. Tráquea: las lesiones penetrantes son más obvias que las provocadas por trauma. Se asocia a lesión de esófago y
grandes vasos. Diagnostico a través de broncoscopía. Bronquios: La lesión de un bronquio mayor es rara y mortal, y ocurren a 2-3 cm.
de la carina. Diagnóstico: hemoptisis y enfisema subcutáneo, y se sospecha en neumotórax a tensión con gran escape de aire y es
confirmado por broncoscopía. “Una vez confirmado el diagnostico de lesión traqueobronquial el tratamiento es la reparación
quir rgica”. 11.- RUPTURA ESOFÁGICA: Se sospechara cuando exista neumo o hemotórax a izquierda sin fracturas costales, trauma
directo al esternón o epigastrio con dolor y shock no explicado, además puede haber salida de partículas de contenido digestivo por

CURSO ENARM CMN SIGLO XXI TEL: 36246001 Pharmed Solutions Institute PÁGINA 241
MANUAL DE TRABAJO DEL CURSO ENARM CMN SIGLO XXI
tubo de tórax. Tratamiento: La conducta será la reparación directa, esofagostomía cervical de escape y yeyunostomía de alimentación.
12.- TAPONAMIENTO CARDIACO: Producto de una herida penetrante, en su gran mayoría, pero también puede aparecer por lesiones
de los vasos pericardicos o traumatismo cardíaco en un traumatismo cerrado. Desde el punto de vista clínico se manifiesta por la
“tríada de ec ”, que consiste en el hallazgo de: aumento de la presión venosa central, disminución de la presión arterial, apagamiento
de los ruidos cardiacos. Ingurgitación yugular con la inspiración en un paciente ventilando espontáneamente es signo inequívoco de
taponamiento cardiaco (signo de Kussmaul). En el tratamiento, la pericardiocentesis por vía subxifoidea es de elección en el
prehospitalario, para descompresionar el pericardio, basta extraer 15 – 20 ml, pero es una medida temporal, actualmente se postula
que en él SU y Hospital no deben hacerse pericardiocentesis a menos que sea para dar tiempo a la preparación del pabellón. INSERCIÓN
DE PLEUROTOMÍA; cuidados postoperatorios: El paciente deberá permanecer en el hospital hasta que el tubo torácico sea retirado.
Debe quedar conectado a una trampa de agua donde el pivote del sello de agua debe quedar sumergido a 2cm, y oscilar cuando esté
conectado al paciente. Mientras el tubo está colocado en el tórax del paciente, los enfermeros verifican con cuidado que no haya
escapes de aire, dificultades o problemas al respirar o necesidad de administrar oxígeno adicional al paciente. El paciente tosa y respire
profundo para facilitar que los pulmones se expandan de nuevo, ayudar con el drenaje y prevenir que los fluidos normales se alojen en
los pulmones. ATENCIÓN Y MANEJO: Se realizara la evaluación y el tratamiento de forma paralela y simultánea al momento en que se
avanza en el examen del paciente, esto es ir resolviendo los problemas vitales en la medida que se va encontrando, y siempre en este
orden y no otro. A. Vía aérea y control de columna cervical. Permeabilidad, estabilidad y seguridad de la vía aérea (eventual IOT),
aspiración de secreciones, fijación de columna cervical firme y segura, cricotiroidotomía por aguja o quirúrgica. B. Respiración:
Oxigenación, movimientos respiratorios, ventilación asistida, oclusión de heridas torácicas abiertas, toracocentesis y drenajes torácicos.
C. Circulación y control de hemorragias. Compresión directa de los sitios de hemorragia, evaluación de los pulsos, masaje cardiaco
externo, instalación de dos vías venosas periféricas proximales de grueso calibre para alto flujo, reposición de volumen y uso de
fármacos endovenosos, analgesia y sedación, monitorización cardiaca. D. Déficit neurológicos. E. Exposición corporal y abrigo.

CASO CLINICO

TRAUMA ABDOMINAL ABIERTO Y CERRADO:


CIENCIAS BASICAS: Lesión orgánica producida por la suma de la acción de un agente externo junto a las reacciones locales y generales
que provoca el organismo ante dicha agresión. Todo paciente puede presentar lesiones en múltiples órganos abdominales y, por tanto,
debe ser considerado como un paciente con traumatismo grave, desde el momento del ingreso en la unidad de urgencias. Un tercio de
los pacientes que requieren una exploración abdominal urgente tienen un examen físico inicial anodino, tener en cuenta que puede
tener un comportamiento impredecible y desestabilizarse en el momento más inesperado. Importante conocer el mecanismo lesional
con el fin de anticipar las lesiones esperables. ANATOMÍA: Abdomen anterior: se define como el área localizada entre una línea
superior que cruza por las mamilas, los ligamentos inguinales y la sínfisis del pubis como la línea inferior, y las líneas axilares anteriores
lateralmente. Flanco: Área entre las líneas axilares anteriores y posteriores y desde el cuarto espacio intercostal hasta la cresta ilíaca. El
espesor de la musculatura de la pared abdominal a este nivel, más que las capas aponeuróticas más delgadas de la pared anterior,
actúa como una barrera parcial a las heridas penetrantes, particularmente por arma blanca. Espalda: localizada atrás de las líneas
axilares posteriores, desde la punta de la escápula hasta las crestas ilíacas, el espesor de la espalda y los músculos paravertebrales
actúan como una barrera parcial a las heridas penetrantes. Anatomía interna del abdomen: Cavidad peritoneal: cubierto por la parte
baja de la parrilla costal, el abdomen superior incluye el diafragma, hígado, bazo, estómago y colon transverso. Debido a que el
diafragma en una espiración total se eleva hasta el 4º espacio intercostal, las fracturas de costillas inferiores o heridas penetrantes en la
misma área pueden involucrar estas vísceras abdominales. El abdomen inferior contiene el intestino delgado y el colon ascendente,
descendente y sigmoides. Cavidad pélvica: rodeada por los huesos pélvicos, corresponde a la parte baja del espacio retroperitoneal y
contiene el recto, la vejiga, los vasos ilíacos, y en la mujer los genitales internos. Espacio retroperitoneal: contiene la aorta abdominal,
la vena cava inferior, la mayor parte del duodeno, el páncreas, los riñones, los uréteres, así como segmentos del colon ascendente y
descendente. Las lesiones en las vísceras retroperitoneales son muy difíciles de reconocer porque el área es de difícil acceso al examen
físico y sus lesiones no son detectadas por medio del lavado peritoneal diagnóstico, y difícilmente valoradas por la ecografía, además
de ser de difícil exploración física. SALUD PUBLICA: Traumatismos de los más frecuentes, estimándose en 1 por cada 10 ingresos por
traumatismo en los servicios de urgencias. Las principales causas de muerte en los pacientes con traumatismo abdominal son: 1. Por
lesión de algún vaso principal, como vena cava, aorta, vena porta o alguna de sus ramas, o arterias mesentéricas. Las lesiones
destructivas de órganos macizos, como hígado, bazo o riñón, o sus asociaciones, pueden originar una gran hemorragia interna. 2.
Sepsis: la perforación o rotura de asas intestinales o estómago, supone la diseminación en la cavidad peritoneal de comida apenas
digerida o heces, con el consiguiente peligro de sepsis. Los trastornos de vascularización de un asa intestinal por contusión de la pared
intestinal o de su meso pueden manifestarse tardíamente como necrosis puntiforme parietal y contaminación peritoneal con sepsis
grave. CLASIFICACIÓN: Abiertos (penetrantes y no penetrantes), presentan solución de continuidad en la piel. Cerrados; la piel no tiene
solución de continuidad. PATOGENIA: Las principales causas de abiertos son las heridas por arma blanca (lesiones intrabdominales de
20-30%) y arma de fuego cuya frecuencia es creciente. La principal causa de cerrados son los accidentes de tráfico. Otras causas son los
accidentes de trabajo, accidentes domésticos, accidentes deportivos, siendo estos mucho más frecuentes que los abiertos.
TRAUMATISMO ABDOMINAL ABIERTO: Las heridas por arma blanca y de fuego de baja velocidad (< 600 m/seg) causan daño al tejido
por laceración o corte. Ceden muy poca energía y el daño se localiza en la zona perilesional, afectando habitualmente órganos
adyacentes entre sí, siguiendo la trayectoria de, objeto que penetra. Las heridas por proyectiles de alta velocidad (> 600 m/seg)
transfieren gran energía cinética a las vísceras abdominales, teniendo un efecto adicional de cavitación temporal y además causan
lesiones adicionales en su desviación y fragmentación, por lo que es impredecible las lesiones esperadas. TRAUMATISMO ABDOMINAL
CERRADO: Impacto directo; la energía cinética a los órganos adyacentes a la pared abdominal, puede provocar lesión. Desaceleración:
mientras el cuerpo es detenido bruscamente los órganos intra abdominales animados aún por la energía cinética tienden a continuar

CURSO ENARM CMN SIGLO XXI TEL: 36246001 Pharmed Solutions Institute PÁGINA 242
MANUAL DE TRABAJO DEL CURSO ENARM CMN SIGLO XXI
en movimiento produciéndose una sacudida, especialmente acusada a nivel de los puntos de anclaje, vasos y mesenterio que sufren
desgarros parciales o totales. Compresión o aplastamiento: entre dos estructuras rígidas, estas fuerzas deforman los órganos sólidos o
huecos y pueden causar su ruptura o estallido de estos. Este es el mecanismo típico de lesión del duodeno, en un accidente de
automóvil con impacto frontal, donde aquel es comprimido entre el volante y la columna vertebral. VALORACIÓN INICIAL: El objetivo
en evaluación primaria es evidenciar o descartar lesiones de riesgo vital e instaurar las medidas necesarias de soporte vital para
preservar la vida del paciente. El paciente con traumatismo abdominal debe ser considerado como traumatismo grave o
potencialmente grave y por lo tanto, el manejo de estos pacientes debe seguir las recomendaciones del ABC: A. Asegurar la
permeabilidad de la vía aérea, con control cervical. B. Asegurar una correcta ventilación/oxigenación. Descartar neumotórax a tensión,
abierto, hemotórax masivo. Valorar la necesidad de soporte ventilatorio. Si no es necesario administrar oxígeno a alto flujo con
mascarilla (10-15 l/min). C. Control de la circulación. Detener la hemorragia externa. Identificación y tratamiento del shock.
Identificación de hemorragia interna. Monitorización ECG estable. D. Breve valoración neurológica. E. Desnudar completamente al
paciente, controlando el ambiente y previniendo la hipotermia. DIAGNOSTICO: Valoración clínica del estado de shock: aumento de la
frecuencia del pulso, pulso débil y filiforme, piel pálida, fría y sudorosa, disminución de la presión del pulso, retardo en el relleno
capilar, alteración de la conciencia, taquipnea, hipotensión y oligo anuria., dado que la hemorragia intraabdominal es la causa más
frecuente de shock hipovolémico en estos pacientes. Inicialmente, se asume que el estado de shock es el resultado de la pérdida aguda
de sangre y se la trata con una infusión rápida de volumen: un bolo inicial de 1-2 litros para un adulto de SF al 0’9 % o de solución de
Ringer lactato. La restitución por medio de catéteres intravenosos periféricos de calibre grueso (14G – 16G), dada su mayor rapidez de
canalización. El shock refractario a la infusión rápida de cristaloides sugiere sangrado activo y requiere de una laparotomía urgente.
Primeras medidas: Colocar sonda gástrica: cuyo objetivo es aliviar la dilatación gástrica aguda, descomprimir y reducir el riesgo de
broncoaspiración. Si existen graves fracturas faciales o la sospecha de una fractura de la base del cráneo, la sonda debe introducirse por
la boca para evitar el riesgo del paso del tubo hacia el cerebro a través de la lámina cribiforme. Canalización de dos vías venosas
periféricas con catéter de gran calibre. Sonda vesical para aliviar la retención de orina y descomprimir la vejiga, comprobar la
presencia de diuresis, su flujo horario, descartando previamente la presencia de lesión uretral. Se deben sacar muestras de sangre y
realizar una determinación de BH, tiempos de coagulación, gasometría, amilasa, niveles de alcohol, grupo sanguíneo y pruebas
cruzadas. Evaluación Secundaria: Historia clínica, a partir del propio paciente, de sus familiares y de los profesionales que han llevado a
cabo la atención prehospitalaria. Mecanismo de producción del traumatismo: a) En traumatismos cerrados es importante el tipo de
impacto, daño del vehículo, uso de sistemas de seguridad, el estado de otras víctimas. b) Para las heridas penetrantes, puede ser útil
una descripción del arma y de la cantidad de sangre perdida en el lugar del hecho. Tiempo de evolución desde el trauma hasta la
recepción del paciente. APP: alergias, patologías previas, medicación habitual, cirugía previa, ingesta de drogas. Maniobras realizadas
por los profesionales de la atención prehospitalaria: volumen infundido, vías canalizadas, necesidad de resucitación cardiopulmonar.
Exploración: Inspección; observar el tórax, abdomen, espalda, pelvis, periné, espalada. Hay que observar las huellas en la piel y pared
de los puntos de impacto del agente agresor. Auscultación; confirmar la presencia o ausencia de ruidos intestinales. La presencia de
sangre libre intra peritoneal o contenido gastrointestinal pueden producir un íleo que produce una ausencia de ruidos intestinales. Las
lesiones en estructuras adyacentes, por ejemplo, costillas, columna o pelvis, también pueden producir íleo. Percusión; detectar
matidez (presencia de líquidos) en caso de hemoperitoneo; timpanismo (presencia de aire) si hay dilatación gástrica o desaparición de
la matidez hepática por neumoperitoneo. Palpación; debe repetirse periódicamente, por lo que debe ser realizado por la misma
persona para poder evaluar las diferencias que se originen. Primero, debe dirigirse al plano parietal buscando la presencia de
hematomas, o contusiones musculares. Luego debe investigar la presencia de contractura abdominal refleja, que es un signo fiable de
irritación peritoneal, al igual que el signo del rebote positivo. Finalmente, hay que realizar una palpación más profunda buscando la
presencia de puntos o zonas dolorosa cuya topografía nos oriente a relacionarlas con los posibles órganos lesionados. Evaluación
estabilidad pélvica; La exploración del anillo pelviano debe realizarse mediante una cuidadosa compresión lateral y antero posterior,
siendo dolorosa cuando hay fractura pélvica. Una fractura de pelvis puede ser causa de shock hipovolémico, en ocasiones muy severo.
Produce hematoma perineal y genital a las 24-48 horas del traumatismo y puede acompañarse de hematoma retroperitoneal y
ausencia de hemoperitoneo. Examen del periné y genitales; lesiones externas y la presencia de signos de lesión uretral como sangre en
el meato, hematoma escrotal o desplazamiento hacia arriba de la próstata. La laceración de la vagina puede ocurrir en heridas
penetrantes o por fragmentos óseos de una fractura pélvica. PRUEBAS COMPLEMENTARIAS: Analítica: bioquímica, hemograma,
tiempo de coagulación, pruebas cruzadas, niveles de alcoholemia y análisis de orina. ECG y monitorización de constantes vitales.
Radiografía de abdomen. Radiografía de tórax: es importante para descartar la presencia de hemotórax, neumotórax o fracturas
costales. Radiografía de pelvis. Exámenes complementarios: Lavado peritoneal diagnóstico (LPD); procedimiento invasivo que puede ser
realizado de forma rápida presenta una sensibilidad del 68% y una especificidad del 83%. La indicación principal del LPD son Hallazgos
abdominales equívocos. Exploración física no realizable por traumatismo raquídeo concomitante o alteración de la conciencia
(traumatismo cráneo encefálico o tóxico). Imposibilidad de reevaluación continúa. Hipotensión inexplicable. Pérdida progresiva de
sangre (descenso progresivo del hematocrito). Puede realizarse mediante un método abierto o cerrado. Un LPD negativo no excluye la
presencia de lesiones retroperitoneales o desgarros diafragmáticos. El ultrasonido puede detectar presencia de hemoperitoneo, es un
medio rápido, no invasivo y seguro en el diagnóstico de lesiones intra abdominales (cerrada o penetrante) y puede ser repetido
frecuentemente. Es más sensible que el lavado peritoneal diagnóstico para la determinación de lesiones de vísceras macizas, aunque no
lo es tanto como la TAC. Su indicación es absoluta en casos de embarazo, cicatrices abdominales por cirugías previas y alteración de la
coagulación. La TAC requiere el transporte del paciente a la sala de rayos X, administración oral e intravenosa de contraste. Consume
tiempo y es utilizado únicamente en pacientes hemodinámicamente estables en los que no existe la indicación inmediata de
laparotomía. La TAC proporciona información relativa a la lesión específica de un órgano en particular y también puede diagnosticar
lesiones en el retroperitoneo u órganos pélvicos que son difíciles de evaluar en la exploración física o en el LPD. Laparoscopia la utilidad
de la laparoscopia efectuada bajo anestesia local para identificar lesiones diafragmáticas y cuantificar la cantidad de sangre
intraperitoneal. MANEJO DEL PACIENTE TRAS LA VALORACIÓN INICIAL: 1. Traumatismo Abdominal Cerrado, Hemodinámicamente
inestable: (imposibilidad de mantener una TA sistólica por encima de 90 mm Hg, frecuencia cardiaca inferior a 100 lpm o diuresis de 50
ml/h adulto) Si el paciente presenta signos abdominales patológicos (distensión abdominal, peritonismo, neumoperitoneo en la
radiografía simple), entonces la indicación de laparotomía debe ser inmediata. Si los signos abdominales son dudosos y el paciente

CURSO ENARM CMN SIGLO XXI TEL: 36246001 Pharmed Solutions Institute PÁGINA 243
MANUAL DE TRABAJO DEL CURSO ENARM CMN SIGLO XXI
presenta un traumatismo craneoencefálico o espinal severo, alteraciones de la conciencia por toxicidad, traumatismos toraco-
abdominales, debemos llevar a cabo un estudio rápido del abdomen que nos ayude a descartar la presencia de patología abdominal,
fundamentalmente líquido libre. Para ello disponemos de dos pruebas que no son excluyentes: Ecografía abdominal. Lavado
peritoneal diagnóstico. Otras pruebas radiológicas que precisan mayor infraestructura y tiempo, como la TAC, no son posibles en el
paciente inestable. Hemodinámicamente estable. Se deben tener en cuenta las siguientes premisas a la hora de manejar un paciente
con un traumatismo abdominal cerrado: Las vísceras macizas se lesionan con más frecuencia que las huecas. Traumatismo Abdominal
Abierto; Arma Blanca. - Si el paciente está inestable o presenta signos de irritación peritoneal, debe ser sometido a una laparotomía
urgente. - Si está estable hemodinámicamente y no presenta signos de irritación peritoneal, la primera maniobra que se debe realizar
es la exploración del orificio de entrada del arma, comprobando si la herida es penetrante o no. Arma de Fuego; Puesto que la
trayectoria de una bala es difícil de predecir y dado que el 80-90% de los traumatismos por arma de fuego se asociaran a una o más
lesión visceral, el tratamiento de estos pacientes será quirúrgico, realizándoseles una laparotomía urgente. Recomendaciones basadas
en evidencia, Recomendaciones NIVEL I: Laparotomía exploratoria está indicada para pacientes con LPD (+) TAC está recomendado para
pacientes hemodinámicamente estables con un EF equívoco (lesión neurológica, lesiones extrabdominales). TAC es la modalidad
diagnóstica de elección para manejo no operatorio de lesiones de vísceras sólidas. En pacientes hemodinámicamente estables, LPD y
TAC son modalidades diagnósticas complementarias.

CASO CLINICO

INSUFICIENCIA RENAL AGUDA (IRA) Y GLOMERULOPATIAS AGUDAS:


CIENCIAS BASICAS: Se denomina IRA a la reducción brusca, en horas o días, de la función renal; se produce una disminución del filtrado
glomerular y un acúmulo de productos nitrogenados séricos (urea y creatinina en sangre) con incapacidad para regular la homeostasis
(equilibrio ácido-base e hidroelectrolítico). Aunque se suele asociar a una disminución de la diuresis (IRA oligúrica), hasta un 40% de los
casos no cursan con oliguria e incluso puede existir poliuria (IRA no oligúrica). La IRA suele presentarse como una complicación de
enfermedades graves previas, apareciendo entre el 5 al 30% de enfermos hospitalizados. CLASIFICACION: IRA PRERRENAL O
FUNCIONAL (60-70%); Inadecuada perfusión renal que compromete el filtrado glomerular; sería, por tanto, una respuesta fisiológica a
la hipoperfusión renal pero el parénquima renal está íntegro. Es reversible si se actúa sobre la causa desencadenante de manera
precoz. Causas de IRA prerrenal son: hipovolemia (hemorragias, perdidas GI, perdidas renales, secuestro de líquidos al espacio
extravascular), disminución del gasto cardiaco (ICC, TEP, hipertensión pulmonar), vasodilatación periférica (sepsis, anafilaxia,
antihipertensivos, anestesia), vasoconstricción renal (hipercalcemia, norepinefrina, ciclosporina, anfotericina B, síndrome
hepatorrenal), alteraciones de las respuestas autorreguladoras renales (inhibidores de las PG, como los AINES, y/o IECAS). IRA RENAL,
PARENQUIMATOSA O INTRÍNSECA (25%); Daño en las estructuras anatómicas; se clasifica según la estructura primariamente dañada:
glomérulos, túbulos, intersticio o vasos renales. Se dividen las causas de IRA intrínseca en 4 apartados: 1.Necrosis Tubular aguda (NTA):
Causa más frecuente de IRA intrínseca (70%), afecta a las células tubulares renales, desde lesiones mínimas hasta necrosis cortical, la
lesión de los túbulos renales puede ser por mecanismos isquémicos (cursa con oliguria) principalmente o tóxicos (aminoglucosidos,
cefalosporinas, contrastes radiológicos, AINES, anestésicos, toxinas endógenas), puede cursar con diuresis conservada, incluso
aumentada. 2. Lesión glomerular: glomerulonefritis agudas y rápidamente progresivas, hipertensión maligna, vasculitis, síndrome
hemolítico-urémico, purpura trombocitopénica trombótica, toxemia del embarazo, esclerodermia. 3. Lesión tubulointersticial:
reacciones alérgicas a fármacos (antibióticos, AINES, diuréticos), infecciones (legionella, leptospira, CMV, candidas). 4. Lesión de
grandes vasos: obstrucción de arterias renales (placa ateroesclerótica, trombosis, embolia), obstrucción de venas renales (trombosis,
compresión). IRA POSRENAL U OBSTRUCTIVA (5%): Lesiones que produzcan un obstáculo en la vía urinaria que impida la salida de la
orina formada, provocando un aumento de presión que se transmite retrógradamente, comprometiendo el filtrado glomerular. Pueden
ser lesiones extrarrenales de uréteres, pelvis (litiasis, tumores, fibrosis), vejiga (litiasis, coágulos, tumores, prostatismo, vejiga
neurógena), uretra (estenosis, fimosis) o también lesiones intrarrenales (depósito de cristales, coágulos, cilindros). Para que estas
causas produzcan una IRA es necesario que la obstrucción sea grave, prolongada y que afecte a tracto urinario distal (meato uretral
externo, cuello de la vejiga) o bien a los uréteres de manera bilateral o unilateral en paciente con un único riñón funcionante.
DIAGNÓSTICO: Clínica de las diferentes formas de IRA dependerá de las causas desencadenantes. Así, en la forma prerrenal destacaran
las manifestaciones de reducción verdadera de volumen (sed, hipotensión, taquicardia, disminución de la presión venosa yugular,
disminución de peso, sequedad de piel y mucosas) o de reducción “efectiva” de volumen (en este caso la exploración revelará signos de
hepatopatía crónica, insuficiencia cardíaca avanzada, sepsis). Es importante destacar que en estos casos la IRA desaparece
rápidamente tras reestablecer la perfusión renal. En el caso de la forma renal o intrínseca hay que investigar la presencia de isquemia
renal prolongada (shock hipovolémico, shock séptico, cirugía mayor). En estos casos existe oliguria o incluso anuria (diuresis diaria <
100 ml). La probabilidad de que estemos ante un cuadro de NTA aumenta aún más si la IRA persiste a pesar del reestablecimiento de la
perfusión renal. La posibilidad de IRA nefrotóxica requiere el estudio de los medicamentos que ha recibido recientemente el paciente,
exposición a contrastes radiológicos (sobre todo si se han realizado en pacientes de riesgo: ancianos, diabetes mellitus, deshidratación
previa, mieloma múltiple). Las toxinas también pueden tener un origen endógeno como la mioglobina (tras una rabdomiólisis, por
destrucción muscular aguda) o hemoglobina (tras hemolisis grave). Destacar que en la NTA por tóxicos la diuresis suele estar
conservada. En la forma posrenal la causa más frecuente en el varón es la obstrucción del cuello
de la vejiga por una enfermedad prostática (hiperplasia o carcinoma).La diuresis fluctuante es
característica de la uropatía obstructiva. Laboratorios: BH: urea, creatinina, glucosa, iones. CK. La característica fundamental de la IRA
es la aparición de uremia aguda de rápida aparición. A nivel práctico se considera que esto ocurre cuando la creatinina plasmática
aumenta 0,5 mg/dl/día durante varios días. Si la IRA ocurre en el seno de una IRC, se considera que el aumento debe ser mayor de 1
mg/dl/día. La creatinina es más fiable que la urea para la el diagnóstico de IRA. También puede calcularse el grado de disfunción renal
detectando el deterioro del aclaramiento de creatinina. Para ello, en urgencias, se puede usar esta fórmula, de aclaramiento de

CURSO ENARM CMN SIGLO XXI TEL: 36246001 Pharmed Solutions Institute PÁGINA 244
MANUAL DE TRABAJO DEL CURSO ENARM CMN SIGLO XXI
creatinina (Ccr), es una prueba aceptada como medida de filtrado glomerular. El valor normal de Ccr es de 100-120 ml/min. En el caso
de IRA el Ccr calculado debe reducirse un 50%. Existirá hiperpotasemia en casos de IRA oligúrica o en estados hipercatabólicos, como
sucede en la hemólisis, rabdomiolisis y en los casos de lisis tumoral. La hipopotasemía se da en las formas poliuricas. La hiponatremia es
también un hallazgo frecuente. Un manejo incorrecto del paciente, con un aporte excesivo de agua en proporción a la de sodio, puede
agravar aún más la hiponatremia. El aumento del ácido úrico es característico de la IRA aunque habitualmente es moderado y
asintomático. Suele existir hipocalcemia, hiperfosforemia e hipermagnesemia. La severidad de estas alteraciones será paralela a la del
daño renal que las ha ocasionado. Hemograma: Puede tener gran importancia en el diagnóstico diferencial entre IRA e IRC: así si
aparece una anemia normocítica normocrómica, estará más en concordancia con una IRC. Gasometría arterial: El patrón ácido-base
más frecuente del fracaso renal agudo es la acidosis metabólica ya que el riñón es incapaz de eliminar los ácidos fijos no volátiles.
Volumen urinario: debido a las variaciones en la diuresis de las distintas formas de IRA en general no tiene gran valor diagnóstico,
aunque si sirve para clasificar la IRA como oligúrica y no oligúrica Sedimento urinario: En la IRA prerrenal el sedimento no contiene
células pero si cilindros hialinos. En NTA existen cilindros granulosos, pigmentados y de células epiteliales, generalmente en asociación
con hematuria microscópica. Proteinuria: suele verse en la NTA, es de tipo tubular y menor de 1 gr/24 h. Sistemático de orina: el
estudio de iones, urea, creatinina, osmolaridad y densidad junto al sedimento urinario son fundamentales para el diagnóstico difrencial
de IRA prerrenal de NTA. ECG: Puede orientar hacia trastornos electrolíticos sobre todo la hiperpotasemia (prolongación del intervalo
PR, ensanchamiento del QRS y aplanamiento de la onda T) o hipocalcemia. La Rx. simple de abdomen informa sobre la existencia de
litiasis radiopaca y el tamaño y silueta renal y con la radiografía de tórax se puede valorar la existencia de sobrecarga de líquidos
(edema agudo de pulmón). Ecografía abdominal: diferencial de la IRA. Se puede descartar patología obstructiva así como visualizar el
tamaño renal, dato muy importante para distinguir entre IRA e IRC. TRATAMIENTO: Manejo de IRA prerrenal. Dieta rica en hidratos de
carbono y con aporte de proteínas de alto valor biológico entre 0,6-0,8 gr/Kg/día. Monitorizar tensión arterial, frecuencia cardiaca y
medición de la ingesta y pérdida de agua y sal. El mecanismo de control más simple es el peso diario. El sondaje vesical será necesario si
se precisa la medición de diuresis horaria. Canalizar vía periférica y central y monitorizar presión venosa central (PVC), para ajustar el
aporte de líquidos a una PVC entre 4-8 cmH2O. Si no existe contraindicación, se puede realizar una rehidratación rápida (en unos 30
min) con 500-1000 ml de SF, controlando la presión arterial, la PVC y vigilando la respuesta clínica y diurética. Si existe una pérdida
hemática grave se usará concentrado de hematíes. Una vez corregida la volemia, el volumen urinario aumenta y se debe continuar con
reposición de líquidos a ritmo de diuresis. Si hay una mala evolución en observación será necesaria la consulta con el nefrólogo que
valorará la indicación de diálisis. Manejo de NTA: una serie de medidas conservadoras pueden, sino evitar la necesidad de diálisis, si al
menos transformar una IRA oligúrica en otra no oligúrica lo que permitirá un mejor manejo del enfermo. Para ello, una vez adoptadas
las medidas generales del apartado anterior se forzará la diuresis por medio de diuréticos y dopamina. Así se puede intentar el uso de
furosemida a dosis de 20 a 40 mg cada 6 horas, según los valores de diuresis y creatinina. También pueden usarse diuréticos osmóticos
como el manitol al 20%, a dosis de 80 ml cada 6-8 horas, en función de la respuesta. La dopamina a dosis de 3-5 µg/kg/min puede ser
otra herramienta útil para mejorar la perfusión renal. Para ello se diluye una ampolla de 200 mg en 250 de glucosado y se perfunde a 5-
10 gotas/min. El mecanismo de la dopamina es aumentar el filtrado glomerular y favorecer la acción de los diuréticos. Se deben usar
antagonistas de los receptores de histamina H-2 para la prevención de hemorragias digestivas. No se recomienda el uso de antibióticos
de manera profiláctica. Manejo de la IRA posrenal: Definitivo es desobstruir. De manera transitoria se procederá al sondaje vesical en el
caso de patología prostática. En caso de obstrucción ureteral con hidronefrosis será necesario la realización de nefrostomia percutanea.
Las indicaciones de diálisis en la IRA son: hiperpotasemia o hipontremia graves, acidosis metabólica con bicarbonato plasmático menor
de 10 mEq/l, sobrecarga de líquidos con edema pulmonar o insuficiencia cardiaca, pericarditis urémica, encefalopatía urémica, diátesis
hemorrágica urémica y azotemia severa (urea > 250 mg/dl o creatinina > 10 mg/dl). GLOMERULOPATIAS AGUDAS (GNA): Se caracteriza
por inicio abrupto de hematuria macroscópica, oliguria, falla renal, disminución súbita de las tasa de filtración glomerular con retención
de sodio y agua, manifestando edema e hipertensión y proliferación de células endocapilares del glomerulo. La proteinuria varía
ampliamente en este síndrome y por lo general es menos de 3 g/dl. La principal es la post-infecciosa, donde el daño glomerular resulta
de un compromiso inmune, desencadenado por una variedad de infecciones bacterianas, virales o de protozoarios. La más común es la
postestreptocócica, afecta a niños entre los 2 y 10 años, predomina ligeramente en hombres. Sólo ciertas cepas nefritogénicas de
estreptococo se asocian con la glomerulonefritis. La variedad más común de glomerulonefritis postestreptocócica es usualmente
después de una infección faríngea con estreptococo beta-hemolítico del grupo A. En los países desarrollados, la nefropatía IgA es, en la
actualidad, la más frecuente de las glomerulonefritis primarias. La glomerulonefritis postestreptocócica es una enfermedad aguda y
reversible, caracterizada por recuperación espontánea en la mayoría de los pacientes. Típicamente la hematuria gruesa y el edema se
presentan entre 7 días a 12 semanas después de la infección estreptocócica. La resolución espontánea de las manifestaciones clínicas
es por lo general rápida. La diuresis reaparece en una a dos semanas y la concentración de creatinina sérica retorna a nivel basal dentro
de las cuatro semanas. La hematuria microscópica desaparece dentro de los seis meses pero la proteinuria leve permanece estática en
15 % de los pacientes después de 3 años. El hallazgo de anticuerpos contra antígenos estreptocócicos proporciona evidencia de
infección reciente, pero no es diagnóstica de glomerulonefritis postestreptocócica. Son los más solicitados los anticuerpos
antiestreptolisina o, antiestreptoquinasa, antihialuronidasa y antinicotinamina dinucletidasa. El tratamiento de la glomerulonefritis
postestreptocócica es de soporte, enfocado a la sobrecarga de fluidos con dieta hiposódica (2.4 g/sodio/día); la hipertensión responde
a diuréticos de asa (furosemide) y antihipertensivos tipo IECA, pero se deben dar con precaución si hay insuficiencia renal con
hipercalemia. La terapia antimicrobiana temprana del paciente con infección faríngea o de la piel, puede prevenir la propagación de la
infección estreptocócica y atenúa la severidad de la glomerulonefritis postestreptocócica, pero no previene el desarrollo de la misma.

CASO CLINICO
Varón de 18 años, cuidador de animales, refiere un cuadro febril de larga evolución; en dicho ingreso desarrolla un fracaso renal agudo
de evolución subaguda con creatinina plasmática (crp) de 6 mg/dl. Refería fiebre elevada de dos meses de evolución, que había
aparecido una semana después de sufrir una dudosa picadura de garrapata. La fiebre se presentaba diariamente sin predomino horario
y con sudoración nocturna. Había recibido distintos tratamientos antibióticos de forma empírica. En el momento del ingreso llevaba 20
días sin tratamiento antibiótico persistiendo el cuadro febril. Radiografía de tórax: al ingreso, normal; un mes después presenta

CURSO ENARM CMN SIGLO XXI TEL: 36246001 Pharmed Solutions Institute PÁGINA 245
MANUAL DE TRABAJO DEL CURSO ENARM CMN SIGLO XXI
pequeños infiltrados pulmonares bibasales con imágenes aerolares. La proteinuria que inicialmente fue de rango no nefrótico (1-2 g/24
h), posteriormente alcanza el rango nefrótico.

PREGUNTA
Cual es la conducta a seguir mas adecuada para establecer el diagnostico final.

RESPUESTA
a.- Gamagrama pulmonar.
b.- Biopsia renal.
c.- Urocultivo.
d.- Hemocultivo.

CASO CLINICO
Hombre de 27 años; como antecedentes refería criptorquidia, adenoidectomía y amigdalectomía en la infancia, fumador activo,
bebedor social y homosexual. Dos meses con una discreta faringodinia, aparecieron una induración en la ingle derecha, lesiones
ulceradas en el glande de aspecto serpinginoso y un exudado uretral blanquecino que fue tratado inicialmente con azitromicina. A la
espera de resultados serológicos, se observaron lesiones maculopapulosas en la zona proximal de muslos y tronco, que se extendían a
los pies y a las manos, que evolucionaron en distintas fases sin acompañarse de fiebre, junto con edematización en los miembros
inferiores y en los genitales, con ligero aumento del perímetro abdominal y disminución de la diuresis. Urea 61 mg/dl, creatinina 1,73
mg/dl, iones normales, proteínas totales 4,4 g/dl, albúmina 1,8 g/dl, colesterol total 295 mg/dl, HDL 61 mg/dl, LDL 206 mg/dl,
triglicéridos 140 mg/dl y enzimas hepáticas normales. En la orina, destacaban proteinuria 13,4 g en 24 horas, 250 hematíes por
microlitro y leucocituria negativa. Hemograma normal, coagulación normal.

PREGUNTA
Cual de las siguientes pruebas tiene mayor sensibilidad y especificidad para establecer un diagnostico.

RESPUESTA
a.- ANA.
b.- ANCA.
c.- Anticuerpo FTA.
d.- VHC.

CURSO ENARM CMN SIGLO XXI TEL: 36246001 Pharmed Solutions Institute PÁGINA 246
MANUAL DE TRABAJO DEL CURSO ENARM CMN SIGLO XXI
SINDROME NEFRITICO Y SINDROME NEFROTICO:
SÍNDROME NEFRÓTICO: Criterios diagnósticos Proteinuria >3-3,5g/24 horas o una relación proteína: creatinina >3-3,5 (ambas en
mg/dL) en una muestra aislada. Albuminemia <25 g/L (hipoalbuminemia). Edemas periféricos. Es frecuente la hiperlipidemia severa
(colesterol total >385 mg/dL). Es consecuencia del aumento de la permeabilidad para las proteínas y es expresión de una enfermedad
glomerular cuando existe una alteración funcional o morfológica de la barrera de filtración. Nunca aparece en las enfermedades
extraglomerulares. El síndrome nefrótico constituye el motivo más frecuente de realización de biopsia renal en todos los grupos de
edad. Manifestaciones clínicas: La pérdida de proteínas en orina origina una serie de alteraciones que constituyen las características del
síndrome nefrótico: 1. Hipoalbuminemia: aparece cuando la capacidad de síntesis hepática se ve superada por las pérdidas urinarias de
albúmina y el catabolismo renal. 2. Edema: suele ser la primera manifestación; aparece en partes blandas y en casos graves puede
comportar ascitis, derrame pleural y anasarca, perorbitario, en miembros inferiores. Es debido a la reabsorción de sodio y agua
secundaria a la disminución de la presión oncótica capilar o estimulada directamente por la proteinuria. 3. Hiperlipidemia con aumento
de las cifras de colesterol total, lipoproteínas de densidad baja (LDL), muy baja (VLDL) e intermedia (IDL), lipoproteína A y, con menos
frecuencia, hipertrigliceridemia y descenso de las cifras de lipoproteínas de alta densidad (HDL). La lipiduria se manifiesta con aparición
de cilindros grasos en el sedimento urinario. 4. Pérdida de inmunoglobulinas y de factores del complemento, que conlleva una
tendencia especial a la aparición de infecciones (peritonitis espontáneas, celulitis, e infecciones pulmonares, meníngeas y digestivas). 5.
Trombosis, con una incidencia de entre el 5 y el 60%; suele aparecer en venas renales y de extremidades inferiores, aunque también
pueden afectarse territorios arteriales. 6. Hipertensión arterial, que aparece en el 42,5% de los pacientes. 7. Insuficiencia renal aguda,
que se observa con más frecuencia en pacientes de edad avanzada, con hipoalbuminemia grave, tratados con dosis altas de diuréticos.
Causas de síndrome nefrótico: Enfermedad glomerular primaria; esclerosis glomerular segmentaria focal, enfermedad glomerular
membranosa, enfermedad glomerular con cambios minimos, enfermedad glomerular membranoproliferativa (ej: IgA). Causas
secundarias de síndrome nefrótico: DM, LES, amiloidosis, mieloma multiple, agentes antimicrobianos, AINES, penicilamina, VIH,
hepatitis B y C, mycoplasma, sífilis. La biopsia renal está indicada en el estudio del síndrome nefrótico del adulto, excepto en pacientes
diabéticos en los que no se sospeche otra enfermedad distinta de la nefropatía diabética. En niños con síndrome nefrótico, en principio
no está indicada la realización de biopsia renal (el 90% de los casos se debe a glomerulonefritis de cambios mínimos), salvo en caso de
síndrome nefrótico resistente a los corticosteroides o con recidivas frecuentes. El tratamiento general comprende: reducción del
edema, control de la presión arterial y del perfil lipídico y profilaxis de la trombosis. SÍNDROME NEFRÍTICO: Se caracteriza por edema,
oliguria, hematuria (con cilindros hemáticos en el sedimento urinario), disminución del filtrado glomerular e hipertensión arterial,
normalmente de instauración aguda. Típicamente se ha relacionado con la forma de presentación de la glomerulonefritis aguda
posinfecciosa, sobre todo asociada a infecciones estreptocócicas. También puede aparecer en otras afecciones (tabla 5).
Manifestaciones clínicas: Aparece como macrohematuria en el 30-50% de los casos, con datos de hematuria glomerular. En la
glomerulonefritis aguda posestreptocócica se presenta tras 2-3 semanas de una infección faringoamigdalar o 4-6 semanas de una
infección cutánea. La hipertensión arterial suele ser moderada, con edemas en párpados y extremidades inferiores. En algunos casos
puede producirse insuficiencia cardíaca congestiva. La proteinuria es inferior a 1-2 g/24 h. Es fundamental la realización de una historia
clínica completa, interrogando sobre antecedentes familiares, manifestaciones clínicas sistémicas, infección reciente o consumo de
fármacos. Patogenia: Las cepas nefritogénicas producen proteínas catiónicas identificadas en tejidos renales de pacientes con
glomerulonefritis. Como consecuencia de su carga eléctrica, estas proteínas se depositan en el glomérulo y dan lugar a la formación in
situ de complejos inmunes, además de los complejos inmunes circulantes formados por la unión de inmunoglobulinas con otros
antígenos. Estos complejos circulantes es posible encontrarlos en la primera semana de la enfermedad y se sabe que están en relación
con la gravedad de la enfermedad; de ahí su importancia a la hora de hacer el diagnóstico. Aproximadamente el 90% de los casos con
glomerulonefritis presenta reducción de los niveles séricos de complemento, debido a que después del depósito en la membrana basal
de estos inmunocomplejos se activa la cascada inflamatoria, en un inicio mediada por interleucina 1 con la subsecuente activación de
linfocitos T y posteriormente mediada por interleucina 2, que al unirse con su receptor específico causa proliferación de los linfocitos
activados y posterior depósito de complemento y formación de perforinas que incrementan la lesión en la membrana basal. Todas
estas alteraciones reducen el calibre de los capilares glomerulares, disminuyendo la superficie de filtración, lo que lleva a la
consecuente reducción en la filtración glomerular. Esta disminución tiende a ser compensada por el aumento de la presión de
ultrafiltración que ocurre porque la presión oncótica peritubular tiende a disminuir, con disminución subsecuente de la fracción
excretada de sodio. Tanto la reabsorción de sodio como de agua expanden el volumen del líquido extracelular, expresamente el
volumen circulante efectivo, causando hipertensión arterial secundaria. En la exploración física valoraremos la presencia de edemas,
hipertensión arterial o datos de insuficiencia cardíaca. La ausencia de manifestaciones clínicas sistémicas, el antecedente de infección,
la hipocomplementemia y títulos elevados de anticuerpos antiestreptolisina establecen usualmente el diagnóstico de glomerulonefritis
aguda posestreptocócica. La biopsia renal estaría indicada si existiera algún dato que hiciera dudar del diagnóstico o en caso de
insuficiencia renal progresiva. Si se sospecha otro tipo de infecciones o se observan manifestaciones clínicas sistémicas, estaría
justificado el estudio serológico, la realización de cultivos y el estudio de autoanticuerpos o inmunoglobulinas. Tratamiento: consiste en
tratar la infección, o específicamente el trastorno subyacente si se debe a otra entidad. Las causas más comunes de síndrome nefrítico:
Enfermedades sistémicas; LES, endocarditis, absceso viscerales, nefritis por cortocircuito, crioglobulinemia, poliarteritis nodosa,
vasculitis por hipersensibilidad, granuloma de Wegener, purpura de Henoch-Schonlein. Enfermedades renales; glomerulonefritis aguda
postinfecciosas, glomerulonefritis membranoproliferativa, nefropatía por IgA. El tratamiento general del síndrome nefrítico se basa en
el manejo adecuado del volumen y el control tensional. Para el control del edema se emplean diuréticos de asa y restricción
hidrosalina. En caso de insuficiencia cardíaca o edema agudo de pulmón, se emplea oxígeno, morfina y nitroglicerina. El control
tensional se consigue con bloqueadores del sistema renina-angiotensina y diuréticos. En caso de uremia grave, hiperpotasemia y/o
insuficiencia cardíaca refractaria puede estar indicada la diálisis.

CASO CLINICO

CURSO ENARM CMN SIGLO XXI TEL: 36246001 Pharmed Solutions Institute PÁGINA 247
MANUAL DE TRABAJO DEL CURSO ENARM CMN SIGLO XXI
TRASTORNOS ACIDO-BASE AGUDOS
CIENCIAS BASICAS: Cuando se habla de equilibrio acido-base en realidad se hace referencia a la regulación de la concentración de
hidrogeniones en los líquidos corporales. Pequeños cambios en la concentración de iones hidrogeno pueden producir grandes
alteraciones en las reacciones químicas celulares, aumentando algunas e inhibiendo otras; por este motivo la regulación de la
concentración de iones de hidrogeno es uno de los aspectos más importantes de la homeostasis. El pH sanguíneo arterial se mantiene
entre 7.35-7.45, cualquier desviación de este rango implica desequilibrio acido-base. El pH plasmático se refiere habitualmente a la
relación entre las concentraciones de
TRASTORNOS PRIMARIOS Y RESPUESTAS COMPENSADORAS
bicarbonato/ácido carbónico. El CO2, en
TRASTORNO ALT. PRIMARIA RESPUESTA COMPENSADORA
presencia de anhidrasa carbónica (AC), se
Acidosis metabólica [HC𝑂3−] PaCO2 desciende 1,2mmHg por cada 1
hidrata de la siguiente forma: CO2 +
mEq/l de aumento de la [HC𝑂3−]
H2O H2CO3  H+ + HCO3-. Al valor de Alcalosis metabólica [HC𝑂3−] PaCO2 aumenta 0,7mmHg por cada 1
pH arterial por debajo de 7.35 se denomina mEq/l de descenso de la [HC𝑂3−]
acidemia y cuando está por arriba de 7.45, Acidosis respiratoria PaCO2 Aguda: [HC𝑂3−] aumenta 1mEq/l por cada
alcalemia; la alteración que cursa con niveles 10mmHg de aumento de PaCO2
de pH por debajo de 7.35 se le denomina Crónica: [HC𝑂3−] aumenta 3,5mEq/l por
acidosis y la que cursa con niveles de pH por cada 10mmHg de aumento de PaCO2
arriba de 7.45 se denomina alcalosis. Factores Alcalosis respiratoria PaCO2 Aguda: [HC𝑂3−] desciende 2mEq/l por
de riesgo: Se recomienda tener alto índice de cada 10mmHg de desciende de PaCO2
sospecha para detectar oportunamente Crónica: [HC𝑂3−] desciende 5mEq/l por
trastornos del equilibrio acido-base en las cada 10mmHg de aumento de PaCO2
personas con factores de riesgo para Valores normales: sangre arterial pH 7,36-7,44; PaCO2 36-44mmHg; [HC𝑂3−] 22-26mmHg
Sangre venosa pH 7,31-7,37; PaCO2 42-50mmHg; [HC𝑂3−] 23-27mmHg
desarrollo. El equilibrio ácido-base requiere la
integración de tres sistemas orgánicos: Hígado, Pulmones, Riñón. El hígado metaboliza las proteínas produciendo iones hidrógeno (H+),
el pulmón elimina el dióxido de carbono (CO2), y el riñón generando nuevo bicarbonato (HCO3). Un ácido es una sustancia capaz de
donar un H+ y una base una sustancia capaz de aceptarlo. Por tanto, la acidez de una solución depende de su concentración de
hidrogeniones [H+]. SISTEMAS DE AMORTIGUAMIENTO: El funcionamiento celular requiere mantener la concentración de H+, del
líquido extracelular (LEC) en límites muy estrechos (el pH compatible con la vida esta entorno a 6.80-7.80). Dado que los procesos
metabólicos generan gran cantidad de ácidos, el organismo necesita neutralizar y eliminar los H+ para mantener constante el pH (-log
+ +
[H ]) del LEC. Para ello, dispone de varios medios: 1. Amortiguadores plasmáticos. Bicarbonato El H se une al HCO3- en forma
reversible, cuando el aporte o la producción de H+ aumenta, la reacción se desplaza hacia la derecha, con lo que incrementa la cantidad
de H+ que es captado por el amortiguador, lo que minimiza los cambios de la H +. El HCO3- representa el 50% de la capacidad
amortiguadora del plasma. Si la producción de ácidos no volátiles excede la excreción, el HCO3- disminuye y la H+ aumenta, resultando
en acidosis metabólica, por otra parte si la excreción de HCO3- es mayor que la producción el HCO3- aumenta y la H+ disminuye
resultando en alcalosis metabólica. H + + HCO3- H2CO3 H2O + CO2- Hemoglobina (Hb), Proteínas y fosfatos. Existen otros sistemas de
amortiguamiento como la Hb, proteínas y fosfatos, los cuales proveen de sitios adicionales de unión de H + y por lo tanto
amortiguamiento. La Hb proporciona el 30% de la capacidad amortiguadora del plasma, el restante 20% lo comparten las proteínas y
los fosfatos (13 y 7% respectivamente). 2. Respuesta respiratoria El segundo sistema de amortiguamiento que hace frente a los
trastornos del equilibrio AB es el pulmonar; la disminución en el pH actúa estimulando quimiorreceptores en el tallo cerebral con
incremento en la ventilación minuto y eliminación del CO2. 3. Respuesta renal. El riñón es el tercer sistema de amortiguamiento, para
mantener el equilibrio AB, éstos deben de
excretar aniones de los ácidos no volátiles y
reabsorber el HCO3-, esto lo logran por
medio de tres mecanismos: reabsorción o
excreción del bicarbonato filtrado, excreción
de acidez titulable y excreción de amoniaco.
En definitiva, según la ley de acción de
masas, la acidosis (aumento de H +) puede
producirse por una disminución del
bicarbonato (acidosis metabólica) o por un
aumento de la PaCO2 (acidosis respiratoria);
y la alcalosis (disminución de los H+) por un
aumento del bicarbonato (alcalosis
metabólica) o por una disminución de la
PaCO2 (alcalosis respiratoria). Así pues la
alcalosis o la acidosis son estados en los que
existe un acumulo de bases o ácidos. Por otra
parte en todo trastorno acidobásico se
producen necesariamente respuestas compensadoras (renales en trastornos respiratorios y respiratorios en trastornos metabólicos),
que intentan mantener normal el pH.

CURSO ENARM CMN SIGLO XXI TEL: 36246001 Pharmed Solutions Institute PÁGINA 248
MANUAL DE TRABAJO DEL CURSO ENARM CMN SIGLO XXI
MANIFESTACIONES CLINICAS
ACIDOSIS METABOLICA ACIDOSIS RESPIRATORIA ALCALOSIS METABOLICA ALCALOSIS RESPIRATORIA
Ej: Ph: 7.32; PCO2: 45; HCO3: 16 Ej.: Ph: 7.24; PaCo2: 55; HCO3: Ej: Ph: 7.48; PCO2: 35; HCO3: 26 Ej: Ph:7.50; PaCO2: 30; HCO3:23
(aguda).Ph:7.34; PCO2: 28; HCO3: 18 23 (aguda). pH 7.33; PaCO2: 53; (aguda). Ph: 7.48; PCO2: 50; HCO3: 28 (aguda). Ph: 7.46; PaCo2: 27;
(subaguda). Ph: 7.36; PCO2: 34; HCO3: 19 HCO3: 28 (subaguda). pH: 7.35, (subaguda). Ph: 7.45; PCO2: 50; HCO3: HCO3: 19 (subaguda)
(crónica) PaCo2: 50, HCO3: 28 (crónica) 35 (crónica)
Clínica Causas Clínica Causas Clínica Causas Clínica Causas
Anorexia, Anión GAP elevado Confusión, EPOC, Apatía, Perdidas digestivas Vértigo, Ansiedad
Fatiga, (normocloremicas) Coma, neumonía, Vomito, (vomitos, aspiración Mareo, Hipoxia
Deshidratación, Cetoacidosis diabética Convulsiones, asma, SDRA Bulimia, nsaogastrica, Ansiedad, Hiperventilación
Confusión, Acidosis láctica Alteraciones Sedación Confusión, diarrea) Euforia, Dolor
Letargia, Insuficiencia renal del ritmo Hipoventilación Arritmias Diuréticos Alucinaciones, Fiebre
Estupor, Coma Intoxicaciones (salicilatos, cardiaco, mecánica cardiacas, Hiperaldosteronismo Alteraciones Enfermedad del
Taquicardia, etanol, metanol, Hipotensión Enfermedad Hiperreflexia, 1ro y 2ro. del estado de tronco cerebral
Hipotensión formaldehido) arterial, neuromuscular Clonus, Dieta pobre en sal conciencia,
arterial Rabdomiolisis Cefalea (Guillan-Barre, Convulsiones, Hipercalcemia Mioclonus,
sistémica, Anión GAP normal esclerosis Hiporreflexia Hipopotasemia Asterixis,
Taquipnea, (hipercloremicas) múltiple, crisis Administracion de Taquicardia
Disnea, Fatiga Perdidas digestivas de miastenica) bicarbonato sódico
de músculos bicarbonato (diarrea, Neumonia Transfusiones
respiratorios fistulas pancreáticas, Derrame masivas de sangre
Respiración de biliares) pleural masivo
Kussmaul Perdidas renales de
bicarbonato (acidosis
tubulares renales,
hioperparatiroidismo,
fármacos; acetazolamida,
anfotericina, ciclosporina)

ANION GAP: La neutralidad química del plasma se mantiene por el equilibrio entre aniones (Cl 103; HCO3 24; otros aniones 10;
proteínas 17 =154) y cationes (Na 143; K 4.5; Ca 5.0; Mg 1.5 = 154) excretados y retenidos. El número total de Cationes plasmáticos
debe igualar a los aniones. Los aniones medidos en el laboratorio son inferiores a los cationes medidos. Esta diferencia se denomina
ANION GAP. El anion gap normal es de 12+/- 2. Una AG superior indica la presencia anormal de ácidos. AG = cationes – suma de
aniones. AG= Na – (Cl + HCO3). Según el valor de anión GAP calculado las acidosis metabólicas pueden dividirse en dos grupos: Con
anión GAP elevado (por adición de ácidos fijos). Con anión GAP normal o hiperclorémicas (por perdida de Bicarbonato). El aumento en
la concentración de cloro se debe a que si el bicarbonato desciende, el Cloro aumenta para mantener la electroneutralidad del medio.
Corrección del anión GAP: Según valor de albúmina: por cada g/dl de albúmina por encima de 4 se suma al anión GAP calculado 2
puntos y, por cada g/l por debajo de 4 se restan 2 puntos.

CASO CLINICO
Mujer de 81 años con hipertensión arterial, dislipemia, diabetes tipo 2 y miocardiopatía dilatada (fracción de eyección [FE] del 30%).
Tratamiento habitual: telmisartán, metformina a dosis de 850 mg/8 horas, atorvastatina, bezafibrato y omeprazol. Acude a urgencias
por diarrea mucosanguinolenta y vómitos de una semana de evolución acompañados de oligoanuria en las últimas 24 horas. TA, 120/70
mmHg, FC de 95 lpm, temperatura 36 ºC. Glasgow 12, desorientación temporoespacial y bradipsiquia, sin signos de focalidad.

PREGUNTA
Cual es la prueba mas apropiada para establecer el diagnostico agudo del paciente.

RESPUESTA
a.- Realizar TAC.
b.- Realizar IMR.
c.- QS y BH.
d.- Gases arteriales.

CASO CLINICO
Analítica: hemoglobina 11,7 g/dl, leucocitos 18.030 (78,9% neutrófilos), plaquetas 307.000, glucosa 68 mg/dl, urea 133 mg/dl,
creatinina 6,89 mg/dl, sodio 134 mEq/l, potasio 4,4 mEq/l, pH 6,89, pCO2: 29 mmHg, bicarbonato 6,9 mmol/l, calcio iónico 3,85 mg/dl,
anión gap 28. Coagulación normal. Orina: pH 6, creatinina 71 mg/dl, proteinuria 400 mg/dl, 100 hematíes/campo, 60 leucocitos/campo,
cuerpos cetónicos positivos.

PREGUNTA
Cuál es la conducta terapéutica más adecuada.

RESPUESTA
a.- Bicarbonato.
b.- Solucion salina.
c.- Solucion glucosada.
d.- Dobutamina.

CURSO ENARM CMN SIGLO XXI TEL: 36246001 Pharmed Solutions Institute PÁGINA 249
MANUAL DE TRABAJO DEL CURSO ENARM CMN SIGLO XXI
CASO CLINICO
Varón de 75 años, hipertenso, con enfermedad pulmonar obstructiva crónica. Consultó por cuadro de infección respiratoria y deterioro
funcional asociado consistente en apatía, somnolencia y períodos de agresividad. A la exploración afebril, desorientado, con tendencia
al sueño, taquipnea superficial, roncus dispersos y crepitantes en la base izquierda, con imagen radiológica sugerente de
bronquiectasias sobreinfectadas. Cloro (114 mEq/l), con función renal y resto de iones normales (sodio: 138 mEq/l; potasio: 4,2 mEq/l),
y una gasometría arterial (pH: 7,24; pCO2: 33 mmHg, pO2: 67 mmHg; bicarbonato: 17 mmol/l, y exceso de bases (EB): –9,1 mmol/l). El
valor del anión GAP (diferencia entre el valor del sodio sérico y la suma del cloro y el bicarbonato) fue 7 mEq/l.

PREGUNTA
Cuál es el estado clínico acido base del paciente.

RESPUESTA
a.- Acidosis metabolica.
b.- Alcalosis metabolica.
c.- Acidosis respiratoria.
d.- Alcalosis respiratoria.

CASO CLINICO
Paciente de 61 años de edad ex-fumador, con hipertensión arterial (HTA), diabetes mellitus tipo II, dislipemia, portador de marcapasos
por enfermedad del nódulo sinusal, claudicación intermitente y con valvulopatía aórtica, estenosis carotídea y cardiopatía isquémica
intervenida. Creatinina de 1.56 mg/dl. En la exploración física destaca soplo carotídeo derecho, pulsos poplíteos débiles y ausencia de
pulsos tibiales y pedios como hallazgos patológicos. BH con (Hb 10.4 mg/dl, VCM 91 fl, HCM 30.6 pg y CHCM 31 gr/dl), y en la
bioquímica plasmática potasio 3.7 mEq/l. En la gasometría venosa: (ph 7,461) y (CO3H 29.5 mEq/l), con (pCO2 medida 42.3 mmHg, con
PCO2 estimada de 43,85 +/- 2 mmHg).

PREGUNTA
Cual es el manejo más adecuado para corregir los gases.

RESPUESTA
a.- Disminuir Fi02
b.- Disminuir FR.
c.- Disminuir volumen ventilarotio.
d.- Disminuir volumen residual.

CURSO ENARM CMN SIGLO XXI TEL: 36246001 Pharmed Solutions Institute PÁGINA 250
MANUAL DE TRABAJO DEL CURSO ENARM CMN SIGLO XXI
TRASTORNOS ELECTROLITICOS AGUDOS:
CIENCIAS BASICAS: El riñón desempeña un papel esencial en el equilibrio hidrosalino del organismo porque regula la excreción de
sodio y del agua. La osmolalidad del sector extracelular es estrictamente una función de la natremia. La natremia depende de: el capital
sódico, el capital potásico y el agua total. En situaciones normales los riñones TIPOS DE SOLUCIONES INTRAVENOSAS
regulan la osmolalidad sérica entre 295 a 300 mosm/kg. de agua. La SOLUCION CONCEPTO/ACCIONES
homeostasis del agua depende de la normalidad del mecanismo de la sed y de ISOTÓNICOS
Suero Expansor de volumen extracelular para la
la secreción de la hormona antidiurética (ADH) por la hipófisis posterior. Sobre fisiológico al hipovolemia
estos últimos mecanismos interactúan la presión arterial, la osmolalidad 0.9% Un exceso puede provocar hipervolmia y
celular y sérica, la angiotensina II y las prostaglandinas. La osmolalidad sérica Na: 154mEq/l acidosis por exceso de cloro
se puede estimar a partir de las siguientes formulas: Osmolalidad sérica= 2Na Cl: 154mEq/l Solo aporta iones cloro y sodio
308 mOm/l Se puede usar en hiponatremia
(mEq/L)+ K(mEq/L)+Urea (mg/dl)/2.8 +Glucosa (mg/dl)/18 o si la urea y la
Único que se puede usar conjuntamente con
glucosa son normales se puede utilizar la siguiente formula simplificada: sangre y derivados
Osmolalidad= (Na+10) x 2. La osmolalidad sérica se expresa en mosm/kg. de Suero Igual que SF al 0.9%, pero la mezcla de
agua. Las principales manifestaciones de un trastorno en el equilibrio del agua glucosado al dextrosa en sangre provoca hipotonicidad
+ 5% Aporta muchas calorías (170/l)
son la hiponatremia y la hipernatremia. HIPONATREMIA: Sodio (Na ) <135
278 mOm/l
mEq/L o de 135 mmol/L y como consecuencia existirá disminución de la Ringer-lactato Contienen múltiples electrolitos y en
osmolalidad plasmática. Sus causas pueden ser: hipovolémicas con sodio (Hartmann) concentraciones similares a la plasmática
corporal total disminuido y se produce pérdida de líquidos y sodio extracelular Para tratar hipovolemia
como en el caso de la fase poliúrica de la IRA, uso de diuréticos, vómitos, El lactato se metaboliza rápido y se convierte
en bicarbonato (riesgo de alcalosis)
diarreas, sudoración profusa, fístulas digestivas y quemaduras; hipervolémicas
Ringer-lactato Aporta 170 calorías por litro y agua libre
con sodio corporal total aumentado. Existe exceso de líquidos y de sodio glucosado al No se debe utilizar para reponer volumen,
extracelular como se observa en la insuficiencia cardiaca congestiva, síndrome 5% pues diluye el plasma y altera concentraciones
nefrótico, insuficiencia renal y cirrosis hepática; endocrinas como en el caso de HIPOTONICOS
la secreción inapropiada de la hormona antidiurética por tumores, Suero Se emplea para potenciar la eliminación de
traumatismos, SIDA y en la enfermedad de Addison. Clínica: aparece cuando la fisiológico al solutos por los riñones debido a su cantidad
0,45% de agua libre y para reponer el líquido
natremia es inferior a 120 mEq/L de manera cronica y se caracterizan por Na: 76.5mEq/l extracelular
anorexia, náuseas, vómitos, calambres musculares, hipotermia, letargo, Cl: 76.5mEq/l Tratamiento de la hipernatremia y de otros
confusión, convulsiones y coma. Con natremias de 128-130mEq/l, síntomas 153 mOm/l estados hiperosmolares
pueden ser graves si se produce de manera aguda (<48hrs), tienen alto riesgo Solo aporta cloro y sodio
Si se emplea la formula con dextrosa 5%, se
de edema cerebral. Laboratorio: electrolitos en sangre y orina, osmolalidad puede provocar hipertonicidad en relación con
plasmática y urinaria, glicemia, proteínas y lípidos plasmáticos. Se debe el plasma
recordar que 100 mg/dl de glicemia superior a lo normal disminuye el sodio HIPERTONICOS
sérico en 1.6 mEq/L. Tratamiento, se debe tratar la causa desencadenante de Suero Para tratar hiponatremias en situaciones
manera tal que en los estados hipovolémicos se utilizan soluciones fisiológicas. fisiológico al graves
3% Precaución por su contenido alto en sodio y
Cuando existe retención hidrosalina y edema se restringen los líquidos y el Na: 513mEq/l cloro, que pueden dar lugar a exceso de
sodio y se administran diuréticos tipo furosemida y en caso de Cl: 513mEq/l volumen intravascular y edema de pulmón
sobrehidratación, restricción de la ingesta de líquidos. En la práctica para la 1,026 mOm/l
corrección de una hiponatremia aguda hipovolémica se debe elevar el sodio Suero Igual que el anterior pero más concentración
fisiológico al de sodio y cloro
sérico de 1 a 2 mEq/l x hora y se calcula el déficit de sodio según la siguiente
5%
formula: (Na+ plasmático deseado-Na plasmático real) x 0.6 x Kg peso=déficit
de sodio corporal total (en mEq). Se utilizarán soluciones salinas isotónicas al 0.9% que contienen 154 mEq/L. También en la práctica
para tratar una hiponatremia severa en un paciente con insuficiencia renal y realizarla a una velocidad de 0.5 mEq/L/hora de elevación
de la natremia ha sido utilizada la hemodiálisis continua venovenosa con un líquido de diálisis con un contenido de sodio menor que lo
normal. La hiponatremia crónica puede ocasionar mielinolisis pontina si se corrige rápidamente. En las formas agudas puede elevarse
el Na+ hasta 2mEq/l, mientras en las formas crónicas no debe superar los 1-1.5mEq/l. El uso de furosemida favorece la eliminación de
agua libre, siendo especialmente útil en estados edematosos o si la osmolalidad en orina es >400mOsm/kg. En el síndrome de secreción
inadecuada de vasopresina (SIADH): el tratamiento crónico se basa en la restricción de líquidos debe recordarse que: 1) la restricción de
líquidos afecta a todos los líquidos no solo al agua. 2) el grado de restricción depende de la diuresis más las perdidas insensibles. 3)
suelen ser varios días antes de que se modifique la osm. 4) no debe restringirse el Na+, fármacos solo para casos refractarios,
antagonista V2 (tolvaptan). HIPERNATREMIA: Sodio (Na+)>145mmol/l, y por tanto existirá un aumento de la osmolalidad plasmática
superior a 300 mosm/L. Menos frecuente que hiponatremia, más común en niños y adultos mayores. Puede ser resultado de perdida
de agua (frecuente) o de un aporte de sodio (raro). La pérdida de agua desencadena dos mecanismos para evitar la hipernatremia: la
sed y la liberación de vasopresina (ADH). Siempre hay que tener en mente la sed es primordial, ya que incluso la máxima secreción de
ADH puede no lograra retener agua suficiente para compensar las pérdidas si no se aumenta el aporte de agua. La hipernatremia con
hipervolemia es poco frecuente y se produce por una ganancia neta de sodio. Es mucho más frecuente encontrar situaciones de normo
o hipovolemia. Sus causas pueden ser: euvolémicas con Na+ normal y se observa en los estados febriles severos, hiperventilación
pulmonar; hipovolémica con Na disminuido se observa en la deshidratación por sudoración profusa, uso de manitol y soluciones
glucosadas, diarreas, quemaduras, diabetes insípida(central o nefrogénica su diferencial mediante test de deprivación de agua,
midiendo la ADH plasmática); hipervolémica con Na aumentado la cual se observa cuando se utilizan en forma indiscriminada
soluciones de bicarbonato de sodio, cuando se utilizan la carbenicilina y los esteroides y en el coma hiperosmolar no cetogénico.
Clínica: sed, astenia, debilidad muscular, somnolencia, irritabilidad, letargia, confusión, temblor, convulsiones y coma. Laboratorio:
sodio sérico y osmolalidad plasmática. El tratamiento consiste en administrar soluciones glucosadas al 5% en los pacientes
deshidratados con sodio normal. En los pacientes con volumen intravascular aumentado se deben utilizar los diuréticos más soluciones
glucosadas al 5%. En la diabetes insípida central se utilizará la vasopresina exógena y en nefrogénica tiazidas. En forma práctica, para
calcular la cantidad de líquido a administrar se utiliza la siguiente formula: Agua corporal total (ACT) real = peso corporal (Kg) x 0.6.
Na+ plasmático real/Na+ plasmático deseado X ACT real = ACT deseada. ACT deseada- ACT real = déficit de líquido. Es prudente

CURSO ENARM CMN SIGLO XXI TEL: 36246001 Pharmed Solutions Institute PÁGINA 251
MANUAL DE TRABAJO DEL CURSO ENARM CMN SIGLO XXI
administrar el 30% de la solución calculada en las primeras 24 horas. La corrección rápida de hipernatremia puede producir edema
cerebral, convulsiones, daño neurológico permanente e incluso la muerte. Para disminuir riesgo se aconseja corregir Na + lentamente.
En casos graves (>170mEq/l), no corregir a menos de 140mEq/l en primeras 48-72hrs y en formas crónicas no debe corregirse mas de 8-
12 mEq/l. HIPOPOTASEMIA: Potasio (K+) <3.5 mEq/L. La hipokalemia es leve cuando el K+ se encuentra entre 3.5 a 2.8 mEq/L, moderada
de 2.8 mEq/L y severa menor de 2.8 mEq/litro. Causa son: vómitos, diarreas, fístulas gastrointestinales, uso de diuréticos, diuresis
osmótica, fase poliúrica de la IRA, uso de esteroides, hiperaldosteronismo primario o secundario, acidosis tubular renal, alcalosis.
Clínica: decaimiento, letargia, predisposición a la intoxicación digitálica, anorexia, náuseas, vómitos, distensión abdominal, íleo
paralítico, paresias, hiporeflexia osteotendinosa, rabdiomilosis la cual puede originar IRA. Se deben solicitar el potasio sérico y urinario
así como los gases arteriales. Tratamiento corrección de la hipokalemia, cuando el K+ sérico es superior a 2.8 mEq/L se utilizará la vía
oral sobre la base de alimentos ricos en potasio sobre todo frutas cítricas, de manera tal que la ingesta diaria sea de unos 40 a 120 mEq.
Sí el K+ sérico es inferior a 2.8 mEq/L se utilizará la vía IV a razón de 40 mEq de KCL por litro en solución fisiológica. Es prudente un
tratamiento preventivo de la hipopotasemia en pacientes que reciban diuréticos, esteroides, digitálicos, así como en la fase poliúrica de
la IRA y en pacientes que reciban tratamiento para una cetoacidosis diabética. HIPERPOTASEMIA: Potasio (K+) >5.5 mEq/L y constituye
una verdadera emergencia médica ya que el paciente corre el riesgo de morir por paro cardiaco. La hiperkalemia puede ser leve hasta
6.5 mEq/L, moderada hasta 7.5 mEq/L y severa superior a 7.5 mEq/L. Las causas más frecuentes dehiperkalemia son: insuficiencia renal
aguda y crónica, uso de diuréticos ahorradores de potasio, insuficiencia suprarrenal aguda o crónica (enfermedad de Addison),
hipercatabolismo, acidosis metabólica y la pseudohiperkalemia por muestra de sangre hemolizada, trombocitosis y leucocitosis por
encima de 100.000 mm3. Se deben solicitar potasio sérico y electrocardiograma, ya que éste es fundamental para evaluar la
hiperkalemia. Cuando es leve al ECG se observan ondas T altas picudas y simétricas, cuando es moderada se observa ensanchamiento
del complejo QRS, disminución de la amplitud de la onda P y prolongación del intervalo PR y cuando es grave desaparición de la onda P,
bradicardia, extrasistoles ventriculares, fibrilación ventricular y asistolia. Tratamiento, además del preventivo (evitar frutas cítricas,
evitar uso de diuréticos ahorradores de potasio, no administrar sangre almacenada por largo tiempo). Moderada: se utilizan resinas de
intercambio catiónico (poliestirensulfonato cálcico) que eliminan el potasio del tubo digestivo. Diuréticos. Grave: glucosa intravenosa
junto con insulina de acción rápida (10UI de insulina en 500ml de glucosa al 10%). La insulina favorece la entrada de potasio a la célula y
la glucosa previene la aparición de hipoglucemia. Administración de bicarbonato de sodio vía intravenosa para corregir la acidosis.
Furosemida: 40 mg IV a repetir cada 4 horas o bumetanida 1 mg IV cada 4 hrs. Gluconato de calcio: 10 ml al 10% IV en 20 minutos.
Bicarbonato de sodio: 60 ml IV rápidamente cada 8 horas. Solución glucosada al 10% 250 ml más 10 unidades de insulina cristalina IV.
Sulfonato de poliestireno sódico (Kayexalate) intercambia sodio por potasio a nivel intestinal en enema a retener 50 a 100 gramos en
100 a 200 ml de agua o por vía oral 20 a 40 gramos más 20 ml de sorbitol al 50%. De ser necesario y en medio especializado
hemodiálisis o diálisis peritoneal. HIPOCALCEMIA: Calcio (Ca 2+) <8,5mg/dl. Causas: Hipoparatiroidismo: una de las causas más
frecuentes de hipocalcemia crónica, déficit de vitamina D, hipomagnesemia (en esta situación se suprime la secreción de PTH),
insuficiencia renal, hipoalbuminemia (calcio circulante se halla unido a la albumina), pancreatitis (aumenta la lipolisis, los acidos grasos
se unen a calcio y disminuye), politransfusiones sanguíneas (elcitrato utilzado como anticoagulante se une al calcio), alcalosis, abuso de
laxantes o síndromes de malaabsorcion. Clinica: hiperreflexia, calambres musculares, la tetania por irritabilidad neuromuscular, es el
signo clínico fundamental. La tetania se puede poner de manifiesto mediante dos maniobras: Signos de Chevostec (estimular nervio
facial; contracción de musculos faciales), signo deTrousseau (se infla manguito de presión por encima de presión arterial sistólica y se
produce un espasmo carpal. Adormecimiento alrededor de la boca, hormigueo, espasmo faríngeo, convulsiones, psicosis, demencia.
Tratamiento, tratar la causa, en situaciones agudas, se administra gluconato de calcio de forma intravenosa. Cuando hipocalcemia es
crónica aporte de vitamina D y suplementos de calcio oral. HIPOMAGNESEMIA: Es una situación muy frecuente en los pacientes
gravemente enfermos, de sintomatología vaga e inespecífica y frecuentemente se asocia a la hipocalcemia. Los niveles séricos de
magnesio normales son de 1.4 a 2 mEq/l. Puede observarse cuando se utilizan diuréticos, en los alcohólicos crónicos, en la pancreatitis
aguda, en la hipomagnesemia familiar con hipercalciuria, intoxicación digitálica, IAM, insuficiencia cardiaca. Clínica: disfagia, diplopía,
debilidad, psicosis, depresión, nistagmo, signos de Chevostek y Trousseau, hiperreflexia osteotendinosa y convulsiones. El tratamiento
consiste en administrar magnesio a la dosis de 2 gramos de sulfato de magnesio diluidos en 100 ml de solución glucosada al 5% IV para
pasar en 5 a 10 minutos y se puede repetir cada 8 horas por 5 días. HIPERMAGNESEMIA: Se acompaña de arreflexia, parálisis
respiratoria y paro cardiaco se trata con calcio IV 100 a 200 mg en 20 minutos.

CASO CLINICO

CURSO ENARM CMN SIGLO XXI TEL: 36246001 Pharmed Solutions Institute PÁGINA 252
MANUAL DE TRABAJO DEL CURSO ENARM CMN SIGLO XXI
HIPOGLUCEMIA, HIPERGLUCEMIA:
HIPOGLUCEMIA: Niveles de glucemia < 50 mg/dL reversible con la administración de glucosa. Se trata de una urgencia muy peligrosa ya
que puede ocasionar lesiones irreversibles e incluso la muerte por falta de aporte de glucosa a nivel de SNC. La hipoglucemia es la
complicación aguda más frecuente del tratamiento de la DM. Más del 35% de pacientes con DM tipo 1 experimentan durante la
evolución de la enfermedad, al menos, un episodio de hipoglucemia grave que requiere atención por otra persona. La incidencia de
hipoglucemia grave en los pacientes con DM tipo 2 es de 1 a 3 por 100 pacientes por año. Esta frecuencia se triplica en pacientes
mayores de 75 años, con ingresos hospitalarios recientes y que reciben múltiples fármacos. Si bien la incidencia de hipoglucemia es
inferior en pacientes con DM tipo 2 que en los de tipo 1. Ante una situación de disminución de las concentraciones de glucosa en
sangre se produce una serie de acontecimientos tendentes a normalizar la glucemia. En sujetos diabéticos que sufren episodios
repetidos de hipoglucemia se ha observado que el umbral de respuesta secretora de catecolaminas, así como de la aparición de
síntomas autónomos, desciende de forma importante. Estos pacientes no presentan síntomas neuroglicopénicos hasta que sus valores
de glucemia descienden a valores en torno a 40 mg/dl. Es decir, estos pacientes soportan bien valores bastante bajos de glucemia
(mecanismo de adaptación cerebral), lo que resulta perjudicial, ya que le introduce en un círculo vicioso de hipoglucemias no
reconocidas. ETIOLOGÍA: La causa más frecuente de hipoglucemia por exceso de insulina es la producida por antidiabéticos orales (más
frecuente las sulfonilureas) e insulina exógena, seguida de la discutible hipoglucemia reactiva y los casos de hiperinsulinismo endógeno
por tumores (insulinoma y tumores extrapancreáticos productores de factores insulin-like). Pacientes diabéticos: Dosis excesivas de
insulina o sulfonilureas. Dieta inadecuada (horarios alterados, baja ingesta,...). Exceso de ejercicio físico. Enfermedades que disminuyen
las necesidades de insulina: Insuficiencia renal (aumenta la vida media de los hipoglucemiantes), hepática, déficits hormonales.
Interacciones medicamentosas (salicilatos, clofibratos, fenilbutazona, sulfinpirazona). Estrés. Pacientes no diabéticos: Hiperinsulinismo
endógeno (insulinoma, fármacos). Comidas poco frecuentes. Hipoglucemia postprandial postcirugía gástrica. Abuso de alcohol. Una
causa inusual, pero con una incidencia cada vez mayor, es la producida por fenómenos autoinmunes contra la insulina, su receptor o la
propia célula beta, conocido como síndrome autoinmune por insulina (tiroideas como tiroiditis de Hasimoto y enfermedad de Graves,
etc). Otras causas: La sintomatología propia de la hipoglucemia reactiva se presenta en el período postprandial, especialmente si se ha
efectuado una ingesta de hidratos de carbono de absorción rápida. El insulinoma es un raro tumor neuroendocrino con una incidencia
aproximada de 1 a 2 por cada millón de habitantes/año. Típicamente se presenta como un tumor solitario, pero puede aparecer de
forma multifocal en el contexto de una neoplasia endocrina múltiple tipo 1 o como un tumor metastásico en el insulinoma maligno. En
el diagnóstico de un insulinoma, la confirmación bioquímica de hiperinsulinismo debe preceder a cualquier técnica de localización. La
sospecha clínica es importante para su diagnóstico, y así, es fundamental la existencia de la triada de Whipple: la clínica sugestiva de
hipoglucemia debe asociarse con unos niveles de glucosa bajos en sangre y mejorar con
la administración de glucosa. Con la sospecha clínica de insulinoma la prueba
diagnóstica de elección es un ayuno de 72 horas. Esta prueba tiene que realizarse bajo
supervisión médica y por tanto será necesario hospitalizar al enfermo. Por último, ante
una hipoglucemia inexplicable, debemos pensar en la posibilidad del diagnóstico de
hipoglucemia facticia para realizar las pruebas de confirmación pertinentes y así evitar
otras que pudieran resultar innecesarias. DIAGNOSTICO: Síntomas adrenérgicos:
Sudoración, palpitaciones, ansiedad, temblor, hambre, predominan cuando la glucemia
desciende rápidamente pero pueden enmascararse con la toma de B-bloqueantes o si
existen neuropatías. Síntomas neuroglucopénicos: trastornos de conducta, agresividad,
confusión focalidad neurológica, somnolencia, convulsiones, coma. Exploración física:
TA, Tª, FC, FR, SAT O2, glucemia capilar, hidratación o perfusión cutánea, fetor etílico,
nivel de conciencia, focalidad neurológica o crisis convulsivas. Glucemia capilar , BH,
QS, electrolitos, osmolaridad, coagulación, EGO, ECG: podrían producirse por el efecto
directo de la hipoglucemia, el aumento en la secreción de adrenalina, la hipopotasemia
secundaria o la disfunción autonómica, todo ello sobre un posible sustrato de
cardiopatía: alteraciones de la repolarización, como depresión del segmento ST,
aplanamiento e inversión de la onda T y prolongación del intervalo QT, trastornos del ritmo, como taquicardia y bradicardia sinusal,
bloqueo auriculoventricular. TRATAMIENTO: LEVE: nivel de conciencia conservado y tolera vía oral se administrarán líquidos azucarados
con 20gr de glucosa. MODERADO: disminución leve del nivel de conciencia o no tolera la vía oral: glucagón 1mg (1amp) IM o SC (pauta
habitual en domicilio) o seguir pauta siguiente. En un estudio se recomiendan 20 gramos de D-glucosa como lo más eficaz, ya que
corrigen la hipoglucemia de grado moderado a grave en 20 minutos, sin causar hiperglucemia prolongada. A los adultos diabéticos en
estado de coma u otros pacientes que tienen coma de causa imprecisa se les debe administrar 50 ml de una solución de glucosa al 50 %
en bolo intravenoso, después de obtener sangre para estudios apropiados. En caso de confirmarse la hipoglucemia, se debe iniciar
venoclisis con solución de glucosa al 5, al 10 o al 20 %. La glucosa intravenosa continua por 4 a 6 horas es necesaria para la mayor parte
de las reacciones hipoglucémicas. GRAVE: disminución del nivel de conciencia severo, coma o PCR: Vía venosa periférica y administrar
glucosa hipertónica (glucosmon*) 30ml al 33% (3 amp) y sueros glucosados (SG) al 10%, si no posible canalizar vía: glucagón 1mg
(1amp) IM o SC, si tras 1000ml de SG continua con cifras bajas de glucemia administraremos hidrocortisona 100mg y 1mg de glucagón a
cada litro de solución, adrenalina 1mg diluido 1/1000 SC, protocolo de RCP si precisa. En pacientes alcohólicos administrar previamente
tiamina 100mg, IM. Mantenimiento: Control horario de glucemia capilar hasta cifras 120mg/dl y a partir de entonces cada 4-6 horas
durante 24 horas. Continuar con SG 5-10% hasta tolerancia oral. HIPERGLUCEMIA: Se define como el nivel sérico superior a 140mg/dl.
La liberación de glucógeno por parte del hígado y los músculos, combinado con la glucogénesis y el metabolismo de los ácidos grasos
libres, aumentara el nivel de glucosa en sangre. El que este aumento alcance el nivel de hiperglucemia dependerá de la capacidad del
organismo para utilizar la glucosa a nivel celular. Hiperglucemia grave, es la elevación de la glucemia plasmática casi siempre superior a
250mg/dl, que se asocia con alteración de otros órganos en el organismo. Las dos principales causas de deterioro en la utilización de
glucosa son los medicamentos y la diabetes. También puede producir hiperglucemia cualquier enfermedad que aumente el estrés
corporal. El estrés aumenta la producción de citosinas y hormonas contrarreguladoras de insulina (catecolaminas, cortisol, glucagón y
hormonas del crecimiento) que alteran el metabolismo de los carbohidratos, incluidas la resistencia a la insulina, la gluconeogénesis,

CURSO ENARM CMN SIGLO XXI TEL: 36246001 Pharmed Solutions Institute PÁGINA 253
MANUAL DE TRABAJO DEL CURSO ENARM CMN SIGLO XXI
glucogenolisis y la disminución de la secreción de la misma debida al mal funcionamiento de la células beta del pancreática. Otros
factores que participan en la elevación de glucosa son: la administración exógena de glucocorticoides a dosis elevada, el tratamiento
subóptimo del control glucémico por falta de estrategias o algoritmos de manejo (pobre adherencia 20-40%) y la nutrición enteral o
parenteral, infecciones (20-50%), embarazo, entre otros. La hiperglucemia se ha asociado con aumento de la respuesta proinflamatoria,
una función alterada del sistema inmunitario, disfunción en la quimiotaxis de neutrófilos con la consecuente alteración en la fagocitosis,
disfunción endotelial, estado protrombótico, daño neuronal asociado con la isquemia cerebral y con aumento del estrés oxidativo. Los
pacientes diabéticos pueden presentar hiperglucemia si no se aplican su insulina o no toman hipoglucemiantes orales, se alteran las
restricciones dietéticas o están sometidos algún tipo de estrés ya sea fisco o emocional. Clínica: polidipsia, polifagia, poliuria,
frecuentemente acompañadas de pérdida de peso y fatiga. La hiperglucemia es un problema que se manifiesta en un porcentaje
importante de pacientes hospitalizados y constituye un factor de riesgo para IAM, infarto cerebral, sepsis, infecciones nosocomiales,
insuficiencia cardiaca y renal. Además, complica una gran variedad de enfermedades, aumenta el tiempo de estancia hospitalaria y es
un factor de riesgo independiente de complicaciones que producen una importante morbilidad y mortalidad en los servicios
hospitalarios, sin mencionar su gran impacto económico. La administración de insulina puede prevenir varios de los efectos adversos
asociados a hiperglucemia. La insulina puede suprimir la generación de especies reactivas de oxígeno, inducir vasodilatación, inhibir la
lipolisis, reducir los ácidos grasos libres, inhibir la agregación plaquetaria y disminuir la respuesta inflamatoria. Por lo tanto la insulina
juega un papel primordial en la prevención de los riesgos asociados con la hiperglucemia. Si la hiperglucemia no se trata aparecerá:
cetoacidosis. En la valoración posterior hay que controlar los medicamentos que puedan causar hiperglicemia: corticoides,
anticonceptivos, antihipertensivos (furosemida, tiacidas, ácido etacrinico).

CASO CLINICO
Mujer de 36 años con diabetes de larga evolución con polineuropatía, retinopatía y nefropatía. Padeció además un síndrome de
Cushing por microadenoma hipofisario que requirió cirugía, una infección por VHC crónica e HTA, para la que había recibido
tratamiento con amlodipino 10mg/24h. Intolerancia al ortostatismo y síncopes, que se agravó en las dos últimas semanas, por lo que la
paciente había estado confinada en la cama. A la exploración se observo desorientación, agitación, desviación de la comisura labial
acompañada de ptosis parpebral unilateral, ausencia de reflejos osteotendinosos en las extremidades inferiores y perdida de control de
impulsos.

PREGUNTA
Cual es la conducta a seguir mas apropiada.

RESPUESTA
a.- Glucosa al 50 %.
b.- Ringer latato.
c.- Solucion harman.
d.- Solucion mixta.

CASO CLINICO
Mujer de 46 años con hipertensión arterial, diabetes mellitus insulinodependiente con insuficiencia renal crónica en diálisis peritoneal
que ingresa en planta de medicina interna con el diagnóstico de cetoacidosis diabética. Se inicia tratamiento y el segundo día se reinicia
la diálisis peritoneal con su pauta habitual. El cuarto día presenta disminución del nivel de conciencia, aumento de presión arterial,
sudoración profusa y al final una parada respiratoria por lo que es intubada. La glucemia capilar es de 122mg/dl y la gasometría arterial
muestra una glucemia de 20mg/dl. Se administra glucosa intravenosa. Al día siguiente se decide reducir la sedación, presentando una
hora más tarde convulsiones clónicas, desviación de la mirada y taquipnea. La glucemia capilar en ese momento es de 59mg/dl.

PREGUNTA
Cual es la conducta mas adecuada a seguir para establecer el diagnostico de esta complicación?

RESPUESTA
a.- Realizar EEG.
b.- Realizar PEV.
c.- Realizar TAC.
d.- Realizar IRM

PREGUNTA
Entre otros estudios se realiz un EEG en donde se observa signos de sufrimiento cerebral difuso de grado muy severo y una TC craneal
que descarta afección intracraneal aguda. Se decide no retirar el líquido de diálisis peritoneal e iniciar diálisis convencional. La escala de
Glasgow fue de 4, Cual es el diagnostico mas probable en este momento?

RESPUESTA
a.- Muerte cerebral.
b.- Encefalopatia hipoglucemica.
c.- Acidente vascular cerebral.
d.- Encefalopatia hipoxico isquémica.

CURSO ENARM CMN SIGLO XXI TEL: 36246001 Pharmed Solutions Institute PÁGINA 254
MANUAL DE TRABAJO DEL CURSO ENARM CMN SIGLO XXI
ESTADO HIPEROSMOLAR HIPERGLUICEMICO (EHH), CETOACIDOSIS DIABETICA (CAD):
CIENCIAS BASICAS: Las complicaciones hiperglucemicas son causantes de un gran número de hospitalizaciones en diabéticos,
presentándose entre 4 y 8 por 1000 pacientes con diabetes, con una mortalidad que varía entre 4 y 20%. De su adecuado diagnóstico y
tratamiento dependerá una considerable disminución de las hospitalizaciones y días de estancia en esta población. PATOGENIA: Tanto
la CAD como EHH, son el resultado de la combinación del déficit absoluto o relativo de insulina y el aumento de las hormonas
contrareguladoras. Este estado lleva a un aumento de la producción hepática de glucosa y disminución en el consumo periférico,
produciendo un aumento exagerado de la glucosa sanguínea; esta a su vez induce glucosuria, diuresis osmótica y deshidratación. En los
pacientes con DM tipo I el déficit severo de insulina y el aumento en las catecolaminas, cortisol y hormona del crecimiento estimulan la
lipasa sensible a hormonas, aumentando la producción de ácidos libres derivados de triglicéridos, los cuales son metabolizados en el
hígado y convertidos en cuerpos cetónicos, proceso estimulado por el aumento de glucagón y el déficit de insulina que activa la enzima
carnitina palmitoil transferasa I que permite la entrada de los ácidos grasos libres a la mitocondria donde se convierten en ácido β-
hidroxibutirico, acetona y ácido acético. Estos cuerpos cetónicos son los responsables de la acidosis en los pacientes con DM1. El β-
hidroxibutirico y el acetoacético se disocian, produciendo exceso de iones de DIAGNOSTICO DIFERENCIAL ENTRE CETOACIDOSIS DIABETICA
hidrogeno que consumen el bicarbonato, disminuyendo sus niveles séricos. A (CAD) Y ESTADO HIPEROSMOLAR HIPERGLUCEMICO (EHH)
PRESENTACION CLINICA CAD EHH
medida que se van acumulando los cuerpos cetónicos, el pH sanguíneo
Dolor abdominal +++ -
desciende. Cuando el pH desciende hasta cerca de 7.2, el centro respiratorio Hiperventilacion + -
eleva frecuencia e intensidad de las respiraciones (respiración de Kussmaul) en Vomito +++ -
el intento de eliminar con las reparaciones el exceso de ácido carbónico. La Deshidratación ++ +++
acetona un componente de los cuerpos cetónicos, también se elimina a través Signo de Kussmaul +++ -
Alteración del edo. de alerta + +++
de la respiración, lo que origina un olor característico afrutado en el aliento del Aliento cetonico ++ -
paciente. Los riñones a su vez también eliminan el exceso de cuerpos cetónicos, Poliuria + +++
dando lugar a la cetonuria. Como la cetoacidosis diabética altera la Polidipsia + +
homeostasia, el organismo intenta restablecer el orden mediante una serie de Fiebre + +
Pérdida de peso + +
mecanismos compensatorios. Para compensar la diuresis osmótica, el centro Taquicardia + -
de la sed del cerebro estimula al paciente para que beba más. Los riñones DIAGNOSTICO
comienzan a segregar renina, poniéndose en marcha el sistema renina Glucemia >250mg/dl >600mg/dl
angiotensina, aldosterona, este complejo proceso hace que los riñones Cetonuria +++ +
reabsorban sodio y agua. También la hipófisis posterior segrega hormona Bicarbonato sérico <15 >15
pH <7.30 >7.30
antidiurética que ayuda también a conservar el agua y el sodio. La diuresis Brecha aniónica Na- (Cl+HCO3) >12 <12
osmótica deriva de la hiperglucemia lleva a un déficit severo de líquidos que Osmolaridad >320
puede estar entre 5 y 7 litros. Además se produce también déficit de cloruro de 2Na+(glu/18)+(BUN/2.8)
sodio entre 3-10mmol/kg, los niveles de sodio pueden estar falsamente
alterados por la hiperglucemia
presente. El potasio se encuentra
también severamente disminuidos,
sin embargo los niveles de potasio en
suero pueden estar normales o
incluso elevados durante el episodio,
debido a la acidosis y a la
hiperglucemia presente,
disminuyendo severamente el
potasio intracelular. Otros elementos
que pueden estar alterados son el
fosfato, el magnesio y el calcio.
CAUSAS: Diagnostico de novo,
infecciones, enfermedades
intercurrentes, falla en la aplicación
de insulina o en la toma de
medicamentos orales (corticoides),
excesos alimentarios, cirugía,
traumatismo, desconocida. En los
diabéticos diagnosticados, su causa
desencadenante suele ser una
situación estresante que incrementa
las necesidades de insulina, aunque
también puede obedecer a una
descompensación de la enfermedad
por no haber seguido correctamente el tratamiento prescrito CLINICA Y DIAGNOSTICO: En cuadro. Iniciar con Bh completa y gases
arteriales, EGO, glucosa plasmática, BUN, electrolitos, cretinina, electrocardiograma. TRATAMIENTO: El objetivo principal e inicial es la
corrección del déficit hídrico para expandir el volumen intra y extracelular y asegura una adecuada perfusión renal. Si no existen
trastornos cardiacos se inicia con solución salina al 0.9% normal; si el sodio es mayor de 155meq/l se recomienda utilizar solución salina
al 0,45 normal. COMPLICACIONES: La aparición de edema cerebral es raro afortunadamente, múltiples factores influyen en su origen,
incluyendo la aparición de idiosmoles que causan un gradiente y una desviación del agua hacia las células, la terapia con insulina por si
sola aumenta la entrada de sustancias osmóticamente activas en el espacio intracelular y un aumento rápido del déficit de sodio.
Síndrome de dificultad respiratoria del adulto; esta complicación puede producirse por el aumento en el volumen de agua en los

CURSO ENARM CMN SIGLO XXI TEL: 36246001 Pharmed Solutions Institute PÁGINA 255
MANUAL DE TRABAJO DEL CURSO ENARM CMN SIGLO XXI
pulmones y una disminución en la adaptabilidad pulmonar. Acidosis metabólica hipercloremica; el mecanismo principal es la perdida de
cetoácidos por la orina, los cuales son requeridos para la generación de bicarbonato

CASO CLINICO
Una mujer de 76 años que no se conocía diabética ingresó por desihidratacion grave, refieren los familiares que vive sola y desconocen
su tratamiento, refieren que la paciente presentaba desorientación, letargia, falta de respuesta a estimulos, se ingresa a la paciente con
datos de desihidratacion glucemia de 596mg/dl pH 7.29, glucosuria y cetonuria, con osmolalidad en suero calculada de 318 mOsm/l.

PREGUNTA
Cual de las siguientes condiciones incrementan el riesgo de cetoacidosis.

RESPUESTA
a.- Infeccion de vías urinarias.
b.- Desequilibrio hidroelectrolitico.
c.- Osmolaridad alterada.
d.- Dieta cetogenica.

CASO CLINICO
Una mujer de 76 años que no se conocía diabética ingresó por una descompensación hiperglucemica cetosica. Durante el ingreso sufrió
4 crisis parciales motoras con generalización secundaria, de inicio motor hemicorporal izquierdo y generalización tónico-clónica, con
una duración de entre 1 y 3 minutos, con recuperación del nivel de conciencia entre las crisis y en un periodo de 5h, seguidas de un
intenso déficit poscrítico hemisférico derecho, con hemiparesia, hemihipoestesia, hemianopsia y heminegligencia izquierdas,
recuperándose en las siguientes 24h. La analítica mostró una glucemia de 596mg/dl pH 7.30, glucosuria y cetonuria, con osmolalidad en
suero calculada de 318 mOsm/l. Se inició tratamiento con anticonvulsivante), sin recurrencia de las crisis. Se le realizó una RM craneal
18 horas después de la primera crisis, donde se apreció una hipointensidad en sustancia blanca subcortical parietal derecha en T2 con
ligera hiperintensidad cortical en FLAIR, tenue captación giriforme de contraste y leve restricción de la difusión en dicha localización. El
electroencefalograma (EEG) evidenció un foco de ondas delta frontotemporal derecho.

PREGUNTA
Considerando las complicaciones que se presentan en esta patología cual es su pronostico mas probable?

RESPUESTA
a.- Bueno ya que se resolvió adecuadamente.
b.- Moderado posibles secuelas.
c.- Puede continuar con crisis convulsivas.
d.- Para evitar recurrencias es conveniente dejar anticonvulsivo.

CASO CLINICO
Mujer de 42 años con antecedentes de tiroiditis crónica de Hashimoto, 3 partos normales, sin macrosomía y una enfermedad mixta del
tejido conjuntivo (EMTC), ante la aparición de una poliartritis de pequeñas articulaciones, esclerodactilia, fenómeno de Raynaud,
miopatía, anticuerpos antinucleares (+) 1/5120 y anticuerpos antiENA (++++). Una tía era diabética tipo 2. Usaba levotiroxina 50 µg/día,
prednisona 7,5 mg diarios y MTX 25 mg intramusculares semanales. Hace tres meses, se espació la dosis de MTX a 25 mg cada dos
semanas, apareciendo debilidad muscular progresiva, mialgias y posteriormente sed y poliuria. La glicemia fue 286 mg/dl (glicemias

CURSO ENARM CMN SIGLO XXI TEL: 36246001 Pharmed Solutions Institute PÁGINA 256
MANUAL DE TRABAJO DEL CURSO ENARM CMN SIGLO XXI
previas siempre normales), prescribiéndose dieta y glibenclamida 5 mg/día. Tres semanas después consultó en un Servicio de Urgencia
por sed intensa. La glicemia fue 550 mg/dl con cetonemia (-). Se aumentó la glibenclamida a 10 mg/día e inició metformina 850 mg y
antiinflamatorios. Una semana después, consultó nuevamente por vómitos, sed, poliuria, mialgias, artralgias y compromiso importante
del estado general. Fue hospitalizada, destacando deshidratación marcada, normotensión y polipnea. IMC: 27,3 kg/mt2. La piel de la
cara estaba acartonada, enrojecida, violácea y tenía aspecto cushingoide. No existía acantosis nigricans. El tiroides se palpaba normal y
el examen cardiopulmonar y abdominal era normal. Existía falta de fuerzas en las extremidades. La glicemia era 414 mg/dl, cetonemia
(++), hemoglobina glicosilada A1C 12%, existía acidosis metabólica (pH 7,0, bicarbonato 2,9 mEq/L) con lactacidemia normal, creatinina
1,3 mg/dl y potasio 2,6 mEq/L. Se administró insulina cristalina subcutánea y luego en bomba de infusión en dosis crecientes, hasta 520
unidades en 24 h, sin lograr controlar la hiperglicemia ni la cetoacidosis. Recibió bicarbonato de sodio y potasio. Se descartó la
existencia de una infección. A pesar de la administración de grandes dosis de insulina cristalina, persistieron la hiperglicemia y la
cetosis.

PREGUNTA
Considerando las condiciones del caso cual de las siguientes medidas serian mas útiles para identificar la resistencia al tratamiento?

RESPUESTA
a.- Buscar anticuerpos antiinsulina.
b.- Buscar anticuerpor antireceptores.
c.- Buscar niveles de CD8.
d.- Buscar anticuerpos antimicrosomales tiroideos

TRAUMA CRANEOENCEFALICO (TCE):


CIENCIAS BASICAS: En adultos, es definido como un intercambio brusco de energía mecánica que genera deterioro físico y/o funcional
del contenido craneal. Se consigna como alteración del contenido encefálico el compromiso de conciencia, la amnesia postraumática
y/o un síndrome vertiginoso o mareos persistentes. También debe considerarse como un signo de disfunción del contenido craneal la
aparición de una cefalea holocránea persistente y progresiva que puede o no acompañarse de vómitos. Se distingue de la Contusión de
cráneo, que corresponde a un impacto mecánico sobre la bóveda craneana que no produce alteración del contenido craneano, y que
puede asociarse a dolor local. SIGNOS DE ALARMA, en la evaluación prehospitalaria o la llegada a urgencias: Deterioro progresivo de la
conciencia (disminución de Glasgow), signos de focalidad neurológica, cefalea progresiva, vómitos explosivos recurrentes, agitación
psicomotora, convulsiones, amnesia anterógrada de más de 30 min, cambio en el tamaño de las pupilas, sospecha de herida craneal
penetrante, intoxicación, evidencia clínica o radiológica de fractura de cráneo, sat. O2 <80%, hipotensión. SALUD PUBLICA: Constituye
una de las principales causas de mortalidad e incapacidad en la población menor de 40 años. Cada año fallece 1,000 000 de personas
víctimas de trauma craneal severo. Los accidentes con vehículo automotor son la principal causa de traumatismos. El trauma causa
150,000 muertes en EU y un tercio se acompaña por TCE severo. Entre 2-10% de pacientes con TCE tienen asociada lesión cervical.
PATOGENIA: Se distinguen 2 mecanismos básicos: la colisión o traumatismo directo, en el cual actúan como formas lesivas la energía
cinética y la deformante. Y el traumatismo indirecto en el que no existe contacto previo, cabeza/objeto agresor, siendo las fuerzas
lesivas la aceleración angular pura y la hiperpresión transmitida pura. El impacto mecánico origina la degeneración neuronal mediante
3 mecanismos básicos: 1. Mecanismo lesional primario, son lesiones nerviosas y vasculares producidas inmediatamente por la agresión
biomecánica, las lesiones resultantes de la agresión primaria son: fracturas craneales, contusiones, laceraciones, hematomas
intracerebrales, lesión axonal difusa. 2. Mecanismo lesional secundaria; en los traumatismos cerrados, el impacto de las fuerzas se
produce en las zonas donde el cráneo se pone en contacto directamente con el encéfalo, pudiendo producir lesiones en la zona de
impacto (lesión por golpe) y en la zona diametralmente opuesta (lesión por contragolpe).,también lesiones por cizallamiento, las
GRAVEDAD DE TCE lesiones resultantes son; hipotensión, hipercapnia, hipoxemia, CLASIFICACION DE TCE SEGÚN OMS
Leve 14-15 puntos hipertermia, hipoglucemia, acidosis, hiponatremia, hipertensión Fracturas de cráneo:
Moderado 9-13 puntos intracraneal, hematoma cerebral tardío, edema cerebral, Fracturas de la bóveda
Severo Menos de 9 puntos Fracturas de la base
convulsiones, vasoespasmo. 3. Mecanismo lesional terciario;
Fracturas de los huesos de la cara
engloban una serie de procesos neuroquímicos y fisiopatológicos complejos, concatenados, con Otras y las fracturas inclasificables
posibilidad de retroalimentación positiva, entre sí que inician inmediatamente tras el TCE. Múltiples fracturas que afectan el
CLASIFICACION: En la escala de coma de Glasgow se puede obtener una calificación máxima de 15 y cráneo o la cara con otros huesos
un mínimo de 3. Un paciente que presenta cualquiera de los siguientes signos debe considerarse Lesión intracraneal
que sufre un TCE severo: Anisocoria, déficit motor localizado, fractura abierta del cráneo con Conmoción
Laceración cerebral y contusión
exposición de masa encefálica o salida de LCR, deterioro neurológico, fractura deprimida de la Hemorragia subaracnoidea,
bóveda del cráneo. Otra clasificación es en: BAJO RIESGO; Asintomática, cefaleas, mareos, subdural y extradural
hematoma, laceración o scalp de cuero cabelludo, ausencia de criterios de moderado o alto riesgo. Hemorragias intracraneales
Actitud a seguir; Si no presentan otras lesiones asociadas que requieran ingreso hospitalario u postraumáticas inespecíficas
Lesión intracraneal de naturaleza
observación se envían a su domicilio siempre que una segunda persona pueda observar la evolución inespecífica
del paciente en las próximas horas. Debe informarse por escrito sobre signos o síntomas de alarma,
ante la presencia de las cuales debe consultar de nuevo al hospital. MODERADO RIESGO; Historia de perdida transitoria de la
conciencia, intoxicación por alcohol y drogas, cefalea progresiva, vómitos persistentes, amnesia peritraumatica, politraumatismo que
impide adecuada valoración clínica del TCE, traumatismo facial severo, sospecha de niño maltratado, edad menor de 2 años excepto
lesión trivial. Actitud a seguir; deben permanecer en observación al menos 24 hrs, pueden reducirse a 12 hrs la observación si no hay
sintomatología neurológica, TAC de cráneo normal, no existen otras lesiones asociadas que requieran ingreso hospitalario. Si existe
sintomatología neurológica evidente, el periodo de observación debe prolongarse y valorar TAC craneal de control a las 12-24 hrs.
ALTO RIESGO; Disminución del nivel de conciencia actual o progresivo no claramente debido a otras causas (metabólico, epilepsia),
signos neurológicos de focalidad, hundimiento o herida penetrante de cráneo, sospecha de fractura de la base del cráneo (otorrea,
hemotímpano, rinorrea, hematoma en anteojos, hematoma retroauricular), convulsiones postraumáticas, respiración irregular o

CURSO ENARM CMN SIGLO XXI TEL: 36246001 Pharmed Solutions Institute PÁGINA 257
MANUAL DE TRABAJO DEL CURSO ENARM CMN SIGLO XXI
apnéica. Actitud a seguir Una vez diagnosticados y estabilizados, deben pasar al área de tratamiento definitivo (quirófano o UCI). Desde
el ´punto de vista patológico, pueden existir tres tipos fundamentales de lesiones cerebrales: 1. Conmoción o concusión cerebral;
caracterizada clínicamente por una breve perdida de conciencia, con un corto periodo de amnesia seguida de una recuperación rápida y
total, sin ningún signo neurológico focal. No hay lesión estructural macroscópica del cerebro, tan solo se producen lesiones por
estiramiento de los tractos axonales de la sustancia blanca, con perdida reversible de su función, responsables de la pérdida de
conciencia transitoria. 2. Contusión cerebral; sobre todo en polos frontales y temporales, por el contacto entre la superficie cerebral y
el interior del cráneo, que abarcan desde una simple magulladura en una pequeña área cortical, hasta lesiones extensas, a menudo
hemorrágicas, de gran parte de la superficie cerebral, con daño en la sustancia blanca y el mesencéfalo, clínicamente alteración del
nivel de conciencia, desde confusión, inquietud y delirio y grados variables de coma, estos pacientes deben ser hospitalizados para
observación, dado el desarrollo tardío de edema cerebral. Realizar TAC, valora necesidad de iniciar tratamiento para HIC. Las
contusiones que producen efecto de masa requieren cirugía urgente. 3. Lesión cerebral difusa; presencia de un coma prolongado de
días o semanas, lesión frecuente con alta mortalidad. Su diagnóstico es presumible cuando en la TAC cerebral no se aprecia una lesión
ocupante de espacio en un paciente con coma profundo, además de la situación de coma suelen presentar posturas de descerebración
o decorticación, y frecuentemente presentan signos de disfunción autonómica. DIAGNOSTICO: Clasificación de los hallazgos de la
primera tomografía tras el TCE: A) Lesión difusa I; sin patología visible. B) Lesión difusa II; cisternas visibles, con desviación de la línea
media hasta 5mm y/o sin lesión mayor de 25 ml. C) Lesión difusa III; cisternas comprimidas o ausentes, con desviación de la línea
media hasta 5 mm y/o sin lesión mayor de 25ml. D) Lesión difusa IV; desviación de la línea media mayor a 5 mm, sin lesión mayor de
25ml. E) Toda lesión evacuada quirúrgicamente. F) Lesión mayor de 25ml no evacuada quirúrgicamente. TRATAMIENTO: inicial de las
situaciones de amenaza vital: 1. Asegurar la permeabilidad de la vía aérea con control de la columna cervical. 2. Oxigenación y
ventilación adecuada. 3. Control de la hemorragia externa y mantener la presión arterial. 4. Evaluación del estado neurológico. 5
Investigar otras lesiones traumáticas. Objetivo del tratamiento en TCE: Posición de la cabeza a 30° sobre el plano horizontal, analgesia
eficaz, normotermia, PaO2 >70mmHg, normocapnia, presión arterial media >90mmHg, euvolemia, Hb >10mg/dl, osmolaridad
plasmática >290mOsm, glucemia <200mg/dl, profilaxis de convulsiones precoces. Farmacología de TCE; MANITOL, su efecto rápido
sobre la PIC es especialmente útil en situaciones de urgencia, su acción es rápida y fugaz, obteniéndose el efecto máximo a los 40
minutos de infusión en bolo, en urgencias, redujo con mayor frecuencia la dilatación pupilar y mejoro la evolución a los 6 meses,
tienden a mostrar mayor sobrevida y mejor pronóstico. Indicaciones: a todo paciente hemodinamicamente estable con signos de
herniación cerebral (anisocoria, signos de decorticación-descerebración) se le debe administrar manitol en bolo rápido, realizando a
continuación una TAC craneal urgente. Dosis se comienza con un bolo de 1-2 g/kg, se puede repetir cada 6 hrs. FUROSEMIDA, su
ventaja sobre el manitol es su efecto sobre la osmolaridad es menos marcado, aunque puede producir trastornos electrolíticos, la
desventaja es que es menos efectivo en el tratamiento del edema cerebral. Indicaciones: lesiones hemorrágicas cerebrales, debería
reservarse para pacientes en los que la expansión de volumen producida por el manitol pudiera resultar perjudicial (cardiopatías,
insuficiencia renal). CORTICOIDES, son eficaces para disminuir el edema cerebral en los procesos tumorales o inflamatorios pero en el
edema, debido al traumatismo, no parecen ser útiles. La administración de pre-hospitalaria de sol.de cloruro de Na al 7.5% para trauma
e hipotensión se asocia con un incremento significativamente mayor en la presión arterial comparado con la infusión de Ringer-lactato.
Profilaxis antibiótica, siempre en los traumatismos abiertos y penetrantes, cuando existan signos clínicos cardiológicos de fractura de la
base de cráneo, se emplearan antibióticos de amplio espectro. Profilaxis de crisis convulsivas; se establecerá precozmente un
tratamiento de fondo preventivo de las mismas siempre que la lesión sea supratentorial con afectación del parénquima cerebral
(hundimientos, contusiones, dislaceraciones, hematomas intraparenquimatosos). Los anticomiciales usados son; fenobarbital (100mg
c/8hrs) tanto enteral como parenteral y fenitoina (100mg c/8 hrs). TCE grave; el exceso de tono simpático en la dinámica vascular
cerebral y sus efectos sobre la PÍC debe ser controlado con una adecuada sedación y analgesia; opiáceos 8fentanilo, morfina),
benzodiacepinas, propofol. La intubación debe ir siempre acompañada de una adecuada sedación, una correcta relajación muscular ya
que si esta no se consigue durante la maniobra de la PIC esta aumentaría, el relajante muscular ideal para estos pacientes es la
succinilcolina. COMPLICACIONES: Hematoma extradural o epidural: colección de sangre entre el cráneo y la duramadre cuya causa más
frecuente es la lesión traumática de la arteria meníngea, sospechar en paciente que ha sufrido TCE, que tras un periodo de 1-24 hrs
entra en estado de coma, pudiendo haber dilatación pupilar del lado lesionado y hemiparesia contralateral. Este cuadro requiere
cirugía inmediata. Hematoma subdural: Debidos a roturas de las venas comunicantes entre corteza cerebral y duramadre, incidencia
mayor en pacientes etílicos y ancianos. Si aparecen en las primeras 24 hrs son agudos, entre 24hrs y 2 semanas subagudos y crónicos
cuando aparecen más tardíamente. El hematoma subdural agudo requiere cirugía urgente. Hemorragia subaracnoidea: se acompaña a
menudo de hematoma subdural concomitante o de una contusión cerebral, el diagnostico se realiza mediante TAC y si es normal el
diagnostico se realizara mediante la demostración de un LCR hemorrágico, no requiere tx., quirúrgico urgente. Hematoma
intraparenquimatoso: pueden manifestarse como lesiones rápidamente expansivas o ser asintomáticas, en la mayoría de los casos
existe fractura craneal asociada por golpe o contragolpe.

CASO CLINICO
Varón de 41 años que bajo el efecto de sustancias tóxicas sufre una caída de 5 metros de altura que le provoca traumatismo
craneoencefálico y parada cardiorrespiratoria (PCR). La parada revierte tras reanimación cardiopulmonar con una duración estimada de
anoxia total de 20 minutos. Ingresado en la UCI, el estudio toxicológico fue positivo para etanol, cocaína y cannabis. 2 horas se presenta
crisis convulsivas tónico clónica presentando disminución del estado de conciencia con respuesta parpebral expontanea,
desorientación, habla incongruente no obedece ordenes, con respuesta de retirada por estimulo doloroso, no localizado.

PREGUNTA
Considerando el estado clínico actual cual es la conducta inmediata a seguir.

RESPUESTA
a.- Administracion de bicarbonato.
b.- Colocación de tubo endotraqueal.

CURSO ENARM CMN SIGLO XXI TEL: 36246001 Pharmed Solutions Institute PÁGINA 258
MANUAL DE TRABAJO DEL CURSO ENARM CMN SIGLO XXI
c.- Colocar sonda orofaringea.
d.- Hiperventilar con bolsa-mascarilla.

CASO CLINICO
Paciente de 51 años, con antecedentes de traumatismo craneoencefálico, contusión cerebral frontobasal, hemorragia subaracnoidea
postraumática y meningitis aséptica, hace 10 meses, por accidente laboral. Desde el traumatismo presenta: impotencia sexual y
disminución de la libido, astenia, anorexia, así como poliuria, polidipsia y nicturia. El paciente refiere asimismo tendencia al sueño,
abotargamiento, caída de pelo y disminución del ritmo de afeitado, cuadro que ha ido progresando hasta acentuarse a raíz de un
catarro. Exploración física: temperatura, 36 ºC; presión arterial, 60/40 mmHg, y frecuencia cardíaca, 40 pulsaciones/min. Se trata de un
paciente joven, con mal estado general, pálido, con piel seca y áspera, ausencia de pelo en cola de las cejas, disminución del vello axilar
y pubiano, abotargado y con tendencia al sueño. La auscultación cardíaca revela bradicardia.

PREGUNTA
Cuál es la causa más probable que presenta la paciente.

RESPUESTA
a.- Isquemia hipofisiaria.
b.- Necrosis hipofisiaria.
c.- Sindrome de secreción inapropiada de ADH.
d.- Panhipopitituarismo.

CASO CLINICO
Mujer de 74 años con antecedentes de cardiopatía valvular con prótesis biológicas en posición mitral y aórtica, hipertensión arterial
bien controlada con enalapril y propanolol y fibrilación auricular paroxística en tratamiento con acenocumarol. Acude por dolor cervical
y debilidad en los miembros inferiores sin traumatismo previo. En el momento del ingreso la paciente está afebril y normotensa, así
como consciente y orientada aunque discretamente bradipsíquica. En la exploración neurológica se aprecian signos de irritación
meníngea y una parálisis completa de ambas extremidades inferiores con hipotonía y reflejos rotulianos y aquíleos abolidos, y reflejo
cutáneo plantar bilateralmente extensor. Se evidencia una anestesia táctil y dolorosa con nivel dorsal bajo.

PREGUNTA
Cuál es la arteria más probable que se compromete.

RESPUESTA
a.- Cerebral anterior.
b.- Arteria basilar.
c.- Comunicante posterior.
d.- Cerebral media.

CASO CLINICO
Mujer de 28 años presenta vómitos y cefalea de varias horas de evolución. Dado el mal estado general de la paciente se decide su
ingreso. Como únicos antecedentes de interés destacaba el tabaquismo, una cirugía previa de tiroides por bocio y consumo de
anticonceptivos orales desde hacía un mes. Al día siguiente la paciente inicia un cuadro progresivo de somnolencia, desorientación y
hemiparesia izquierda, es ingresada a terapia intensiva posterior a deterioro neurológico.

PREGUNTA
Cuál es la complicación mas frecuentes que este paciente desarrolle.

RESPUESTA
a.- Neumonia.
b.- Urosepsis.
c.- Neuroinfección.
d.- Sindrome ADH.

CURSO ENARM CMN SIGLO XXI TEL: 36246001 Pharmed Solutions Institute PÁGINA 259
MANUAL DE TRABAJO DEL CURSO ENARM CMN SIGLO XXI
ISQUEMIA CEREBRAL TRANSITORIA (ICT):
CIENCIAS BASICAS: Se define como un episodio breve de disfunción neurológica causado por daño cerebral focal o isquemia retiniana,
con signos que típicamente duran hasta una hora y sin evidencia de infarto cerebral agudo. SALUD PUBLICA: Se calcula que en Estados
Unidos de Norteamérica hay 300,000 casos nuevos de ICT por año y alrededor de 15-20% de los pacientes con un infarto cerebral
tienen historia de ICT. Recientemente se ha demostrado que la ICT es un fuerte predictor a corto plazo de infarto cerebral, enfermedad
cardiovascular y muerte. Presentar un ICT conlleva un riesgo de ACV en el primer mes de 8% y al año de 5%, junto con 5% de riesgo de
infarto de miocardio al año. PATOGENIA: Se basa especialmente en un mecanismo de producción aterotrombotico, que sin duda es el
principal responsable de la mayoría de los casos de ICT. Otros mecanismos como el cardioembolismo, la arteriopatía no ateroesclerosa
y las vasculopatías han sido descritas también como ICT. Las observaciones iniciales mediante examen oftalmoscópico de la amaurosis
monocular transitoria, sirvió para ilustrar el compromiso del flujo sanguíneo en las arterias retinianas y el rompimiento de las columnas
venosas en patrón cuadrado, con material "blanco" que obstruía la arteria retiniana. La razón íntima relacionada con las lesiones en ICT
tiene que ver con la estenosis vascular y ulceración de placas ateroscleróticas con formación de trombos, con subsecuente
embolización de material de fibrina y plaquetas desde estos sitios ateroscleróticos, como el arco aórtico, carótida interna extracraneal e
intracraneal y la arteria cerebral media y la vertebral. Igualmente, la formación de coágulos de fibrina y glóbulos rojos formados en la
circulación rápida o las cavidades cardiacas también cuenta como causal de las lesiones en ICT. Recientemente, el estudio para el
tratamiento de ACV Agudo definió las anormalidades cardiacas de alto y mediano riesgo de embolización. Las anormalidades de alto
riesgo (mayor de 5% anual) son: la fibrilación auricular (FA), válvulas protésicas, enfermedad reumática cardiaca, endocarditis
bacteriana, mixoma auricular y cardiomiopatía dilatada. Los de riesgo moderado (2% anual) son: personas mayores de 65 años sin
factores de alto riesgo; y pacientes con riesgo bajo (1% anual) los menores de 65 años y sin factores de riesgo concomitantes.
CARACTERISTICAS CLINICAS: Debe referirse exclusivamente al principal territorio arterial afectado, es decir carotideo o vertebrobasilar
(tallo cerebral). La característica principal es la constelación de signos y síntomas de déficit neurológico focal que son: alteraciones de
la conciencia o sincope, mareo monosintomático, amnesia o confusión aislada, crisis convulsivas, vértigo aislado, diplopía aislada,
escotomas cintillantes, disfagia aislada, disartria aislada, incontinencia. Los síntomas aislados no deben ser considerados como eventos
de ICT. En el territorio carotideo las más frecuentes son: déficit motor, sensitivo o trastornos del lenguaje e incluyen; disfunción
sensitiva y motora de extremidades contralaterales, seguidas de afección motora o sensitiva pura o bien con menos frecuencia disfasia
o afasia aislada. Duración de síntomas menos de 15 min. La amaurosis fugax o ceguera monocular transitoria (visión borrosa, nebulosa
o vidrio empañado) es una de las manifestaciones clínicas tradicionales de enfermedad carotidea ateroesclerosa, se atribuye a
embolismo arteria-arteria, duran de 1-5 min., y rara vez exceden los 15 min. La visión se restablece por completo al terminar el evento.
En el territorio vertebrobasilar las manifestaciones pueden ser más variadas lo que provoca con mucha frecuencia que síntomas
aislados se confundan con ICT. Lo más recuente es debilidad o torpeza que pueden cambiar de un lado a otro, alteraciones sensitivas
que pueden ser bilaterales, hemianopsia homónima o ceguera total transitoria, ataxia, diplopía; o bien por lo menos dos de los
siguientes; disartria, diplopía, vértigo o disfagia. DIAGNOSTICO: Se obtiene por historia clínica, ya que al momento de la exploración es
frecuente que el paciente se encuentre recuperado completamente. Es importante realizar un adecuado examen físico general
buscando cardiopatías o enfermedades de origen vascular y un examen neurológico exhaustivo ya que permiten identificar signos
persistentes. Laboratorio: Nos ayuda a identificar causas metabólicas como hipoglicemia, hiponatremia y trombositosis, una VSG
elevada puede sugerir endocarditis bacteriana o arteritis temporal. Dado que una de las causas de ICT es la embolia de origen cardiaco,
uno de los estudios iniciales debe ser un ECG, que puede revelar fibrilación auricular o IAM silente. La TAC y RM pueden revelar
patologías que simulan a ICT. El doppler carotideo o angioIRM permiten identificar enfermedad ateroesclerosa. TRATAMIENTO: La
presencia de ICT ofrece la oportunidad de iniciar tratamientos que limiten el posible inicio de un infarto cerebral. No existe tratamiento
específico para la ICT, debe individualizarse en base a los factores de riesgo y alteraciones encontradas en cada paciente. La
identificación de factores de riesgo tanto modificables (HTA, DM, cardiopatía isquémica, hipercolesterolemia, tabaquismo, alcoholismo
y sedentarismo) como no modificables (edad, genero, raza, herencia) debe ser una prioridad. En la HTA reducciones de 10mmHg en
sistólica, así como 5 en diastólica se asocian con un 30-40% de reducción de riesgo de EVC, por ello es uno de los principales elementos
en la prevención de nuevos eventos de ICT y de infarto cerebral. Los niveles de glucosa en pacientes con ICT deben ser <126mg/dl, y en
casos ya confirmados de DM, el tx., con hipoglucemiantes orales y/o insulina debe ser iniciado de inmediato y control estricto. Manejo
farmacológico: Antiagregantes plaquetarios está indicado en los casos de ICT de origen no cardioembólico ya que se ha demostrado
disminución en el riesgo de EVC recurrente en alrededor de 22%. Los aprobados como medida de prevención secundaria son: aspirina
(75-325mg/d), clopidogrel y la combinación de aspirina-dipiridamol de liberación prolongada. El tratamiento con anticoagulantes orales
está indicado en los casos de ICT secundarios a fibrilación auricular (ICT cardioembólica) u otras patologías cardiacas potencialmente
emboligénas. Se sugiere mantener valores de INR en promedio de 2.5 (2-3).Otro grupo de alto riesgo en quienes está indicada la
anticoagulación son los pacientes con ICT “in crescendo”. En estos casos la presentación de los episodios de ICT es repetitiva, ya que se
ha observado cese de la sintomatología de manera dramática después del inicio de la anticoagulación. La endarterectomía carotidea o
stent carotideo se reserva para pacientes con criterios establecidos para dicho manejo. Dado el alto riesgo de desarrollar isquemia
cerebral en las horas o días que siguen al ICT, esta debe considerarse como una urgencia neurológica. Los objetivos de su evaluación
oportuna incluyen: confirmar que se trata de ICT, definir su mecanismo de producción, para establecer un tratamiento adecuado.

CASO CLINICO
Mujer de 53 años de edad que fue evaluada debido a un trastorno del lenguaje de 12 h de evolución e inicio brusco. La paciente refería
dificultades para encontrar las palabras durante el discurso. El examen neurológico mostró una afasia de Broca moderada y una pérdida
parcial de la sensibilidad del brazo y la pierna derechos. El examen del fondo de ojo era normal. No existía síndrome confusional ni
signos de meningismo. La paciente presentaba un historial oncológico de larga evolución. Fue diagnosticada de carcinoma de cérvix
uterino a los 26 años, y de neoplasia mamaria a los 43. Un mes antes del episodio actual se realizó una tomografía computarizada
toracoabdominal debido a un incremento de los marcadores tumorales en sangre. Éste mostró una masa ovárica, múltiples
adenopatías abdominopélvicas y un tromboembolismo pulmonar bilateral.

PREGUNTA

CURSO ENARM CMN SIGLO XXI TEL: 36246001 Pharmed Solutions Institute PÁGINA 260
MANUAL DE TRABAJO DEL CURSO ENARM CMN SIGLO XXI
Cuál es la frecuencia de presentar EVC dentro de los primeros 6 meses.

RESPUESTA
a.- 10 al 20 %
b.- 20 al 30 %
c.- 30 al 40 %
d.- Más del 40 %

CASO CLINICO
Mujer de 73 años sin factores de riesgo vascular que acudió a urgencias por presentar cuadro de inicio súbito de disminución de la
movilidad en hemicuerpo izquierdo con caída al suelo. A su llegada a urgencias estaba consciente con desviación conjugada de la
mirada a la derecha y hemiplejía completa izquierda, hemihipoestesia y reflejo de Babinsky izquierdo presente. TC craneal sin lesiones
evidentes. Se aplicó tratamiento trombolítico a los 150min del inicio de los síntomas. En su evolución en las primeras horas se objetivó
clara mejoría de la movilidad de hemicuerpo izquierdo, persistiendo al alta hemiparesia izquierda.

PREGUNTA
Cuál es la conducta a seguir para mejorar la secuela del paciente.

RESPUESTA
a.- Rehabilitación física.
b.- Rehabilitacion neuropsicológica.
c.- Se evolución es favorable.
d.- Rehabilitación en casa.

CASO CLINICO
Varón de 75 años, dislipidemia en tratamiento hipolipemiante como único factor de riesgo cardiovascular. Traído a urgencias por
cuadro de hemiparesia izquierda, parálisis facial central izquierda, hipoestesia en hemicuerpo izquierdo, disartria y Babinsky izquierdo.
TC cráneo sin alteraciones. Se administró tratamiento trombolítico a 150 min de evolució. Posterios al tratamiento se mantiene
semiología neurológica, con TC de control a las 24h de la trombolisis con infarto en territorio de arteria cerebral media con edema
citotóxico y desviación de línea media. Se añadió tratamiento con manitol, con mejoría del nivel de conciencia

PREGUNTA
Cuál es el tiempo máximo para administración de trombolisis.

RESPUESTA
a.- 100 minutos.
b.- 150 minutos.
c.- 200 minutos.
d.- 250 minutos.

CURSO ENARM CMN SIGLO XXI TEL: 36246001 Pharmed Solutions Institute PÁGINA 261
MANUAL DE TRABAJO DEL CURSO ENARM CMN SIGLO XXI
COMA Y MUERTE CEREBRAL:
CIENCIAS BASICAS: Dentro de las alteraciones agudas de la vigilia; la obnubilación significa literalmente embotamiento mental o
torpeza, se aplica a enfermos con reducción leve o moderada del estado de vigilia. Es distintivo en ellos lograr la reacción de despertar
con estímulos sonoros más o menos intensos. En el estupor el paciente no responde y se despierta solo con estímulos verbales o
dolorosos repetidos y fuertes, tras los cuales vuelve a sumirse en un sueño profundo. En el coma, ningún estímulo despierta al sujeto;
incluso la estimulación dolorosa no provoca respuestas intencionadas y pueden producir postura refleja de decorticación o
descerebraciones, se debe a una disfunción cerebral orgánica difusa. Definición de COMA; Máxima degradación del estado de
conciencia. Síndrome clínico caracterizado por una pérdida de las funciones de la vida de relación y conservación de las de la vida
vegetativa, como expresión de una disfunción cerebral aguda y grave. Teasdale y Jennet definen el coma como la incapacidad de
obedecer órdenes, hablar y mantener los ojos abiertos. PATOGENIA: La alerta se mantiene por el Sistema Reticular Activador
Ascendente (SRAA) localizado entre el tercio medio de la protuberancia y la porción más alta del mesencéfalo. Este sistema es una
estructura polisináptica que se puede afectar por procesos intrínsecos del tallo cerebral que lo destruyen, por procesos extrínsecos que
lo comprimen o desplacen y por procesos metabólicos que lo alteran o inhiben. Una lesión COMA POR LESIÓN ANATÓMICA
hemisférica produce coma directamente por su volumen o de manera indirecta por SUPRATENTORIALES (15 A 20 %)
• Intracerebrales: Hemorragia cerebral,
compresión, isquemia o hemorragia en el mesencéfalo y tálamo. Esta lesión por crecimiento
Hemorragia intraventricular, Infarto
radial crea un cono de presión transtentorial y comprimen el SRAA en la parte rostral del tronco cerebral extenso (arterial o venoso),
encefálico. CLASIFICACION: Existen múltiples clasificaciones del estado de coma. Según el Tumores, Infecciones (Encefalitis
porcentaje de daño cerebral, el sitio y la causa del coma tenemos, ver cuadro. DIAGNÓSTICO: focal, absceso cerebral)
La historia clínica de los pacientes en coma sigue siendo el elemento fundamental de su • Extracerebrales: Tumores,
Hidrocefalia, Hemorragia intracraneal
diagnóstico. Ante todo es preciso interrogar a las personas que traen al enfermo para extraer postraumática (epidural, subdural),
los datos posibles sobre las circunstancias de aparición del evento. El inicio del cuadro puede Empiema subdural
ser súbito (paro cardiaco, hemorragia o embolias cerebrales) o progresivo (intoxicaciones, INFRATENTORIALES (10 A 15 %)
tumores, trombosis cerebral, meningoencefalitis, encefalopatía hepática, encefalopatía • clusión basilar
• Hematoma subdural y extradural de la
urémica). Un TCE reciente puede sugerir un hematoma epidural o una contusión cerebral y si es fosa posterior
de más tiempo orienta hacia un hematoma subdural crónico. Se recogerán los antecedentes • Hemorragia pontina primaria
inmediatos y antiguos, el antecedente reciente de cefalea puede orientar hacia una masa • Hemorragia cerebelosa
expansiva intracraneal (tumor, hematoma, absceso); el de epilepsia, a un coma postcrítico y la • Infarto cerebeloso
• Malformaciones arterio venosas del
existencia de focalidad neurológica, a un tumor o isuqemia cerebral. Exploración física siempre
tronco
debe realizarse completa por aparatos, ya que puede aportar datos muy útiles. Nivel de encefálico
conciencia la profundidad del coma se explora aplicando al paciente estímulos de intensidad • Aneurisma de la arteria basilar
creciente (verbal, táctil y dolorosa) y se clasificará según la mejor respuesta obtenida durante la • Abscesos
exploración. Para valoración del estado de COMA utilizamos la escala de Glasgow. A todo • Granulomas
• Tumores primarios o metastásicos
paciente en coma se le deberá realizar un estudio analítico básico para descartar una causa • Mielinolisis central pontina
metabólica de coma (Diabetes mellitus, hipoglucemia, coma urémico, encefalopatía hepática o COMA POR LESIONES DIFUSAS (TÓXICO-
coma por diselectrolitemia). Ante la sospecha clínica de coma exógeno y en todos los casos de METABÓLICAS, 65 A 75 %)
coma sin diagnóstico evidente debería realizarse un estudio toxicológico de sangre y orina. Es EXÓGENOS
aconsejable determinar la presencia de alcohol, barbitúricos, benzodiacepinas, antidepresivos • Fármacos
• Tóxicos
tricíclicos, fenotiazinas y opiáceos. TRATAMIENTO: Medidas generales: Asegurar la oxigenación, • Trastornos físicos
dada la necesidad de un aporte continuo de oxígeno al cerebro es necesario priorizar y ENDÓGENOS
garantizar la función respiratoria. En caso de coma profundo se puede intubar al enfermo como • Hipoglicemia
profilaxis de la broncoaspiración. Mantener la circulación: Con el fin de mantener flujo • Hipoxia: Disminución de la tensión de
oxígeno: PaO2 35mmHg, enfermedades
sanguíneo cerebral adecuado. Si deterioro hemodinámico, obrar según corresponda con aporte pulmonares, alturas, Hipoventilación.
de volumen, drogas vasoactivas, etc. Evitar hipotensión brusca en caso de emergencia Disminución del contenido sanguíneo de
hipertensiva y coma. No bajar TA diastólica por debajo de 100 mmHg. Si alcoholismo crónico o oxígeno: Anemia, intoxicación por CO2,
desnutrición se administrará 100 mg intramuscular y 20 mg endovenoso de Tiamina y luego 50 metahemoglobulinemia
• Shoc : Cardiogénico, hipovolémico o
mL de dextroza al 50 % IV. (25 gramos). Si se administra esta antes de la Tiamina se puede séptico
precipitar una encefalopatía de Wernicke. Otras medidas; Vaciamiento del contenido gástrico: • Alteraciones metabólicas: Hiper o
Sonda nasogástrica a bolsa previo lavado gástrico. Sonda vesical: Medir diuresis horaria. hiponatremia,
Considerar antídotos y corregir causas: Tratar las causas de reversibilidad inmediata. Glucosa 50 hipercalcemia, hiper o hipomagnesemia,
% IV. si hipoglucemia sospechada o constatada. Tiamina (B1) 100 mg IM si alcoholismo. acidosis metabólica o respiratoria, hiper
o hipoosmolaridad, hipofosfatemia
Flumazenilo 0,25 mg IV. si sospecha intoxicación con benzodiacepinas. Fisostigmina 5 mL = 2
mg. Administrar lentamente 1 Ámp. cada 30-60 min. en intoxicaciones graves por antidepresivos tricíclicos Naloxona, 1 mL = 0,4 mg.
Dosis: 10 mcg/kg o 400 mcg /dosis única, si se sospecha intoxicación por Opiáceos. Tratamiento del edema cerebral: Dexametasona 10
mg IV y seguir con dosis de 4 mg IV. cada 6 horas. Manitol al 20 %: 1g/kg IV. en 20 min. Iniciar cuando se aprecia deterioro rostrocaudal.
Evitar soluciones hipotónicas y de glucosa puras- Tratamiento anticonvulsivo: Diazepam 2 mg/min. IV. hasta 20 mg, Clonazepam
1mg/min. IV. hasta 6 mg, Fenitoina (250 mg-5 mL) 2,5 cc IV. c/8 horas. MUERTE CEREBRAL: Es la expresión con la que se designa la
perdida de todas las funciones del encéfalo. Se declara cuando los reflejos del tronco cerebral, las respuestas motoras y la actividad
respiratoria están ausentes en un individuo comotaso normotérmico, con lesión cerebral masiva e irreversible, que no haya recibido
drogas o fármacos que actúen en el sistema nervioso central y que no tenga otros factores que contribuyan que contribuyan al cuadro
clínico=coma estructural, origen conocido, daño irreversible, hemodinamicamente estable, temperatura mayor de 34°C, ventilación y
oxigenación adecuada, ausencia de enfermedades metabólicas importantes, ausencia de fármacos o drogas que actúen sobre el SNC.
Esta definición permite certificar la muerte aunque se encuentre funcionando la mayor parte de sus órganos del cuerpo. Solo es posible
establecer este diagnóstico en el medio hospitalario (unidades de reanimación o de cuidados intensivos), y requiere un protocolo
extremadamente riguroso, la intervención de personal experto y la realización de pruebas confirmatorias objetivas y fiables. Además,
para aumentar la seguridad, se fija un periodo de observación cuya duración depende de la edad del paciente. PATOGENIA: La muerte
cerebral implica siempre la lesión estructural del encéfalo, en gran parte de los casos suele ser primaria, inicialmente localizada y

CURSO ENARM CMN SIGLO XXI TEL: 36246001 Pharmed Solutions Institute PÁGINA 262
MANUAL DE TRABAJO DEL CURSO ENARM CMN SIGLO XXI
supratentorial, por ejemplo: hemorragia intracerebral espontanea, traumatismo cerebral, infarto cerebral, hemorragia subaracnoidea y
tumores cerebrales. Menos de 15 % de las lesiones encefálicas son globales, supratentoriales e infratentoriales, casi siempre
secundarias a un paro cardiaco o respiratorio que provoca anoxia encefálica y con menor frecuencia debidas a infección del sistema
nervioso central y edema celular ocasionando por tóxicos o trastornos hidroelectrolíticos. Se produce edema global del parénquima
encefálico, con el consiguiente aumento de la presión intracraneal, que iguala a la presión de perfusión encefálica. Esto ocasiona paro
circulatorio intracraneal y conduce a infarto encefálico total y a isquemia global del encéfalo; el estadio final es la necrosis de toda la
masa encefálica=muerte cerebral. La pérdida de funciones del tronco del encéfalo sigue casi siempre a una secuencia rostrocaudal. Se
inicia en el mesencéfalo y finaliza en el bulbo. Su fase clínica final suele distinguirse por la caída brusca y significativa de la presión
arterial sistémica, tanto sistólica como diastólica. El paro circulatorio intracraneal completo puede ocurrir simultáneamente o más
tarde, pero nunca antes. DIAGNOSTICO: Exige una certeza absoluta, por lo cual debe seguirse el protocolo sistemático, estricto y
riguroso. Los tres pilares son: a) conocer la causa de la lesión encefálica, b) descartar los trastornos que pudieran simular la muerte
encefálica (condiciones hemodinámicas, metabólicas, farmacológicas y toxicas actuales o relativamente recientes y c) efectuar una
exploración neurológica reglada. La ley General de Salud, dedica su título decimocuarto a la donación, trasplantes y pérdida de vida.
Específicamente los artículos 343 y344 del capítulo IV se refieren a la perdida de vida. Artículo 344: La muerte cerebral se define como
la ausencia total e irreversible de todas las funciones cerebrales. Se puede considerar muerte cerebral cuando existen los siguientes
signos: I. Perdida permanente e irreversible de conciencia y de respuesta a estímulos sensoriales. II. Ausencia de automatismo
respiratorio. III. Evidencia de daño irreversible del tallo cerebral, manifestado por arreflexia pupilar y ausencia de movimientos oculares
en pruebas vestibulares y de respuesta a estímulos nociceptivos. En la legislación mexicana se ha determinado que la muerte cerebral
es irreversible y que puede considerar el fin de vida. De acuerdo con los criterios del artículo 344, no hay diferencia entre el estado
vegetativo persistente y la muerte cerebral. El personal de salud debe involucrarse en la creación de una normatividad y lenguaje
común sobre muerte cerebral, mientras que la ley debe adecuarse a los avances tecnológicos y humanísticos.

CASO CLINICO
Varón de 48 años que acude a nuestro por náuseas y malestar de 2 días de evolución. Desarrolla rápidamente coma, acidosis
metabólica severa (pH 6,8) con anión GAP elevado e infiltrado pulmonar bilateral que requiere soporte ventilatorio invasivo. Se realiza
TAC craneal, que es normal. En las siguientes 12 horas presenta rigidez de descerebración. En un nuevo TAC se observa hipodensidad
putaminal bilateral y edema cerebral difuso. Ante la sospecha de intoxicación por metanol se inicia tratamiento con diálisis y etanol. La
evolución neurológica es desfavorable, produciéndose muerte encefálica en 24 horas. Posteriormente se documentaron niveles de
metanol de 0,4g/l.

PREGUNTA
Cual de las siguientes pruebas es menos útil para establecer el diagnostico de muerte cerebral?

RESPUESTA
a.- EEG
b.- PEV
c.- Retiro de apoyo ventilatorio.
d.- IRM

CASO CLINICO
Paciente joven de sexo femenino que tras consumir 50mg de éxtasis ingresa con una encefalopatía hiponatrémica e insuficiencia
respiratoria aguda severa por edema pulmonar, destacándose una natremia de 109mEq/l y una osmolaridad urinaria de 360mOsm/l. Se
realizó la corrección de la hiponatremia mediante la infusión de cloruro sódico hipertónico al 3%, presentando una mala evolución con
fallo respiratorio y shock. A las 9h la ecografía doppler transcraneal evidenció un patrón compatible con muerte encefálica. La necropsia
mostró edema cerebral con signos de herniación y hepatización del parénquima pulmonar.

PREGUNTA
Cual es la casua mas probable de esta condición?

RESPUESTA
a.- Síndrome de secreción inadecuada de hormona antidiurética.
b.- Pérdida de fluidos hipotónicos (hipertermia, sudoración).
c.- Pérdida de la capacidad de dilución urinaria por lesión tubular renal proximal.
d.- Reabsorción de fluidos hipotónicos desde la luz intestinal.

CURSO ENARM CMN SIGLO XXI TEL: 36246001 Pharmed Solutions Institute PÁGINA 263
MANUAL DE TRABAJO DEL CURSO ENARM CMN SIGLO XXI
DELIRIUM:
CIENCIAS BASICAS: Alteración transitoria del estado mental, caracterizada por la presencia de diferentes manifestaciones clínicas,
principalmente alteración del nivel de conciencia y atención, junto con otras alteraciones de esferas cognitiva y no cognitiva. La causa
siempre es orgánica y multifactorial, el inicio agudo y subagudo y el curso fluctuante a lo largo del día, con alternancia de intervalos
lucidos diurnos y empeoramiento nocturno. Delirium tremens: (cuadro confusional agudo) secundario a la privación alcohólica.
Complicación grave del síndrome de abstinencia alcohólica. Este aparece en casos de dependencia a alcohol, entre 4-12 horas después
de la última ingesta. La ingesta habitual de alcohol para desarrollarlo es muy variable. SALUD PUBLICA: Su incidencia aumenta su con la
edad y es más prevalente en la población anciana. Es mucho más frecuente a nivel hospitalario. Originado por hiperreactivación de
receptores NMDA, hiperactividad del sistema Noradrenérgico y Dopaminérgico. Hipoactivación del sistema GABA. PATOGENIA: No está
bien definida, pero el sustrato básico, consiste en un desorden generalizado del metabolismo cerebral y la neurotransmisión que afecta
a estructuras corticales y subcorticales encargadas de mantener el nivel de conciencia y la atención. En esta disfunción se encuentran
implicados neurotransmisores como acetilcolina, dopamina, GABA y serotonina, los cuales unos por exceso y otros por defecto,
justifican los síntomas. Estas alteraciones bioquímicas explican la efectividad de fármacos utilizados en el tratamiento o el efecto
adverso de otras sustancias que, indicados en determinados procesos, podrían desencadenar un episodio de delirium. El delirium es un
síndrome que sigue un modelo multifactorial, especialmente en ancianos, ya que representa la compleja interrelación entre factores
predisponentes intrínsecos (envejecimiento cerebral, patología orgánica cerebral, episodios previos de delirium, déficit funcionales y
factores estresantes) a un paciente vulnerable y factores precipitantes externos como: infecciones respiratorias y urinarias, alteraciones
metabólicas, endocrinopatías, trastornos cardiopulmonares y gastrointestinales, el perioperatorio y medicamentos; principalmente los
anticolinérgicos (difenhidramina, hidroxizina, levodopa, amantadina, escopolamina, atropina, amitrptilina, imipramina,
antiespasmódicos), antiarritmicos (propanolol, digoxina, procainamina, lidocaína), antihipertensivos (reserpina, metildopa),
psicotrópicos (amitriptilina, litio, benzodiacepinas, neurolépticos, opiáceos, hipnóticos), antagonistas H2 (ranitidina, cimetidina,
famotidina). También hay que recordar los estados de intoxicación por fármacos, en estrecha relación con los cambios farmacocinéticas
y farmacodinamicos asociados a la edad, como disminución de la masa corporal con
aumento de materia grasa, disminución de la filtración glomerular y aclaramiento de CUADRO CLINICO
DELIRIUM DEMENCIA
la creatinina, deterior de función hepática y disminución de proteínas, que hacen que
Comienzo Súbito Insidioso
dosis terapéuticas sean toxicas para ancianos. Otros factores son las lesiones que Curso Fluctuante Estable
produce la hospitalización (deprivación del sueño, interrupción de rutinas, cambios Conciencia Disminuida Normal
del ambiente, inmovilización y uso de restricciones físicas etc…). DIAGNOSTICO: Es Atención Alterada Normal, salvo en
clínico; la anamnesis y exploración física son fundamentales, debemos revisar el nivel globalmente casos graves
de conciencia, atención (respuesta a estímulos, destruido, respuesta a instrucciones), Cognición Alterada Deteriorada
globalmente globalmente
memoria (perturbación de memoria, inmediata y reciente), orientación (esfera
Alucinaciones Visuales A menudo ausentes
temporal, segunda de espacial y personal), pensamiento y lenguaje (lenguaje, Ilusiones Fugaces A menudo ausentes
irrelevante y repetitivo, incoherente, con circunloquias, la capacidad de abstracción Orientación Deteriorada Deteriorada
es nula), percepción (interpretación errónea de un objeto real, alucinaciones, Lenguaje Incoherente Pobre,
principalmente visuales) conducta (actividad psicomotora disminuida, con apatía e perseveración
inmovilidad o bien inquietud y agitación), estado afectivo (euforia o ansiedad, que Enf. orgánica Siempre Ausente
alternan con apatía, indiferencia y depresión), ciclo sueño-vigilia (insomnio con empeoramiento de la confusión durante la noche e
hiperinsomnia durante el día). Las pruebas complementarias básicas y las técnicas de imagen permiten hacer una valoración etiológica,
como BH, QS, VSG, Función renal, hepática, electrolitos, glucosa, calcio, EGO, determinación de niveles de medicamento, ECG,
radiografía de tórax. CRITERIOS DIAGNOSTICOS DE DELIRIUM según DSM-IV: 1. Alteración de la conciencia, con disminución de la
capacidad para centrar o mantener o dirigir la atención. 2. Cambios en las funciones cognoscitivas o alteraciones de la percepción. 3. La
alteración se presenta en un corto periodo de tiempo (horas o días) y tiende a fluctuar a lo largo de la vida. 4. Demostración a través de
la historia, de la exploración física y de las pruebas complementarias de que la alteración tienen una causa orgánica. TRATAMIENTO:
Medidas preventivas, ya que reducen la incidencia y reducción del cuadro clínico. Identificación y tratamiento de causas subyacentes,
ya que el tratamiento etiológico en algunas ocasiones conlleva, la resolución del delirium. Cuidados de soporte y rehabilitadores,
nutrición, hidratación, facilitar y promover la movilización, proteger al paciente de caídas, minimizar el riesgo de neumonías, cuidados
intestinales y de tracto urinario. La utilización de psicofármacos requiere la valoración riesgo beneficio, están indicados, en síntomas
como trastornos de la conducta e insomnio, los neurolépticos son de elección para el control de la agitación (haloperidol), los de última
generación o atípicos (risperidona, olanzapina, clozapína), provocan menos efectos extrapiramidales y han demostrado similar eficacia
al haloperidol, por lo que su utilización se está extendiendo en la actualidad, no hay que olvidar sus efectos adversos como hipotensión
ortostatica, sedación y agranulocitosis en el caso de clozapina. Las benzodiacepinas constituyen la medicación indicada en casos de
delirium secundarios a síndromes de abstinencia por sedantes y alcohol y el control del insomnio. Paciente alcohólico que acude a
urgencias con síntomas de delirium tremens establecido: Este tipo de pacientes requiere ingreso hospitalario en una unidad de
medicina interna o de cuidados intensivos. Son de especial importancia las medidas de soporte, el control hidroelectrolítico, la diuresis
y la prevención de complicaciones. Desde el punto de vista psiquiátrico, puede valorarse a necesidad de contención mecánica debido a
la agitación, que en algunos casos puede ser extrema y que debe ser de 5 puntos. Debe mantenerse al paciente en un ambiente bien
iluminado, tranquilo, con escasos estímulos sensoriales: Dosis de ataque de 20mg de diazepam por vía intramuscular, repitiendo cada
30 minutos hasta conseguir la sedación Posteriormente se administrarán 10-20 mg cada 6h que se irán reduciendo de manera
progresiva; 50-75mg de cloracepato por vía intramuscular, repitiendo cada 30 minutos hasta conseguir la sedación y posteriormente,
50mg cada 6 horas reduciendo progresivamente

CASO CLINICO
Masculino de 79 años de edad el cual se encuentra diagnosticado con demencia tipo Alzheimer desde hace 5 años, actualmente
permanece solo en casa, con supervisión nocturna únicamente, se observa con signos de desnutrición, con mal higiene generalizada, a
la exploración física se observa con ruidos cardiopulmonares adecuados, se observa con moderada agitación psicomotriz que se alterna

CURSO ENARM CMN SIGLO XXI TEL: 36246001 Pharmed Solutions Institute PÁGINA 264
MANUAL DE TRABAJO DEL CURSO ENARM CMN SIGLO XXI
con periodos de somnolencia, no hay datos de focalización, se realiza BH y EGO sin datos de procesos infecciosos, se mantiene en
observación por la noche los síntomas se agudizan con presentación de ideas delirantes de daño y alucinaciones visuales.

PREGUNTA
Cuál es la conducta a seguir en este paciente.

RESPUESTA
a.- Realizar IRM de Craneo.
b.- Administrar haloperidol.
c.- Realizar hemocultivo.
d.- Cuantificacion de electrolitos sericos.

TRASTORNO BIPOLAR (TB):


CIENCIAS BASICAS: Es una enfermedad psiquiátrica recurrente y severa que consiste en la alteración cíclica y recurrente del estado de
ánimo, entre episodios de depresión (depresión intensa desesperanza), de manía (felicidad extrema) y mixtos (depresión y exceso de
actividad en una fase maniaca). Estos cambios duran normalmente varias semanas o meses. TRASTORNO BIPOLAR según DSM-IV
SALUD PUBLICA: Existe una prevalencia a lo largo de la vida de 1,6% para trastorno bipolar tipo TIPO I: Episodio maniaco único
I y 0.5% para tipo II. La prevalencia por trasto en personas mayores de 65 años que se Episodio más reciente hipomaniaco
Episodio más reciente maniaco
encuentran en la comunidad se estima en el 0.1-1%. Generalmente empieza durante o después
Episodio más reciente mixto
de la adolescencia. Es raro que empiece después de los 40 años de edad. En relación con el Episodio más reciente depresivo
comportamiento suicida en TB, la prevalencia del intento suicida fue de 17% para tipo I y 24% Episodio más reciente no
para tipo II. PATOGENIA: Posibles factores etiológicos: Endógeno o constitucional relacionado especificado
con el biotipo pícnico en un 70%. Hereditario, es de tipo autosómico dominante. Biológico TIPO II: Trastorno depresivo mayor
recurrente con episodios hipomaniacos
(catecolaminicas, indolaminicas). Toxico (alcohol, cocaína, anfetaminas. Hormonales (tiroides,
Trastornos ciclotímicos: Presencia de al
hipófisis, gónadas). Psicológico (shocks emocionales). La fisiopatología, esta sin duda mediada menos dos años de reiterados síntomas
por una red de circuitos límbicos, estriatales y fronto-corticales interconectados, así como por hipomaniacos y síntomas depresivos
sistemas colinérgicos, catecolaminérgicos y serotoninérgicos de neurotransmisores, además de alternados
alteraciones de neuroplasticidad y resiliencia celular DIAGNOSTICO: El diagnostico de Trastorno bipolar no especificado:
Trastornos con características bipolares,
bipolaridad se hace cuando se observa, que la depresión alterna con un episodio maniaco., de que no cumplen los criterio para ninguno
intensidad variable y que alternan de manera cíclica en el tiempo. Generalmente la depresión especifico
aparece primero y en pocas ocasiones el primer episodio de trastorno bipolar es de tipo
maniaco. Factores predictivos de bipolaridad: inicio precoz de episodio depresivo (mujeres <25años) con inhibición psicomotriz e
hipersomnia, historia familiar de TB, antecedente de depresión postparto y antecedentes de hipomanía al iniciar el tratamiento con
antidepresivos. Episodio maniaco: elevación del estado de ánimo, por sobre los niveles considerados normales (euforia) o en otras
ocasiones irritable, con incremento de la energía, aumento de la actividad psicomotora en general; habla excesiva y acelerada
(verborrea), pensamientos veloces y apresurados, saltando de un tópico al siguiente sin aparente conexión entre ellos. La capacidad de
atención y concentración suele estar disminuida y el paciente se distrae con facilidad. Impulsividad y desinhibición, disminución de la
necesidad de dormir. En ocasiones es posible observar, desorganización del pensamiento y franca psicosis con alucinaciones y delirio.
La duración varía, pero debe mantenerse por al menos 7 días para constituir un episodio de manía según DSM IV, o menos si fue
requerida hospitalización. Episodio hipomaníaco: euforia o irritabilidad, con incremento de la energía, aumento de actividad física y
mental, pero con intensidad subumbral para constituir manía. Características de manía pero los rasgos parecen ser mucho más
adaptativos, el pensamiento es creativo, se disfruta más de actividades cotidianas, el pensamiento nunca se desorganiza y o se
observan síntomas psicóticos ni se presenta impedimento del funcionamiento. Episodio depresivo: bajo estado de ánimo (tristeza),
disminución de la capacidad de experimentar placer y perdida de interés (anhedonia), disminución de energía, actividad física y mental,
habla infrecuente y enlentecida, pensamientos lentos, aparición de ideas pesimistas, desesperanza, irritabilidad, disminución de libido
aumento o disminución del apetito, dependiendo de la intensidad, es posible observar ideación suicida y aparición de franca psicosis
con alucinaciones y delirio. Características clínicas de la depresión bipolar: antecedentes historia de abuso sexual, explosividad e
irritabilidad en la infancia, antecedentes de trastorno de déficit de atención, interrupción de la vida académica, historia familiar de
bipolaridad. Inicio precoz y súbito, crisis depresiva separada de la experiencia existencial, hipersomnia, hiperfagia y aumento de peso,
delirio no congruente con el estado afectivo, confusión mental. Inhibición psicomotriz, agitación, ansiedad, síntomas más intensos en la
manía, cambios en la polisomnografia. Episodio mixto: aparición simultanea de síntomas maniacos y depresivos, mas frecuente en
mujeres, responden de manera menos eficaz al litio que al ácido valproico, tienen mayor comorbilidad con el uso de sustancias y
alcohol y mayor suicidalidad que la manía pura TRATAMIENTO: La meta es dar una respuesta adecuada a ambas fases de la
enfermedad (manía, depresión), prevenir la frecuencia y mejorar la calidad. Las intervenciones psicosociales estructuradas se han
validado como un medio efectivo. El manejo psicofarmacológicos es condición fundamental pero no única. Los fármacos incluyen a los
que disminuyen la sintomatología efectiva, los que previenen el episodio agudo y los que son complemento de distintas circunstancias
clínicas en el curso de la enfermedad. Fármacos estabilizadores del ánimo: litio (tiene eficacia antimaníaca y estabilizadora del ánimo,
dosis de 600-2400mg), Ac. Valproico, carbamacepina, lamotrigina, oxcarbamazepina. Antipsicóticos típicos: haloperidol. Antipsicóticos
atípicos: risperidona, clozapina, olanzapina. Benzodiazepinas: clonazepam, lorazepam.

CASOS CLINICOS
Paciente de 17 años, portadora de un trastorno bipolar, trastorno de personalidad limítrofe y con antecedente de abuso de marihuana,
cocaína, éxtasis y heroína. Después de cuatro días de ser dada de alta desde una clínica psiquiátrica, donde estuvo dos meses en
tratamiento con clozapina, ácido valproico, carbonato de litio y lorazepam, inició un cuadro de confusión, letargia, conducta catatónica,
rigidez, mialgias y fiebre, que mantenía por cinco días, razón por la que ingresó a la Unidad de Cuidados Intensivos (UCI) con el
diagnóstico presuntivo de meningoencefalitis. En el examen físico se constataron fiebre, taquicardia, normotensión, rigidez

CURSO ENARM CMN SIGLO XXI TEL: 36246001 Pharmed Solutions Institute PÁGINA 265
MANUAL DE TRABAJO DEL CURSO ENARM CMN SIGLO XXI
principalmente del esqueleto axial, y gran agitación psicomotora. El hemograma de ingreso mostró 17.000 leucocitos/mm3 con 14% de
baciliformes, plaquetas normales, VHS de 31, PCR de 19,8 mg/dl, CK de 920 mg/dl con fracción MB de 22 mg/dl. Se tomaron
hemocultivos, cultivo de orina, radiografía de tórax y punción lumbar que fueron normales, pese a lo cual recibió antibióticos por
sospecha de un cuadro pulmonar aspirativo.

PREGUNTA
Cual es la complicación mas probable que presenta el caso?

RESPUESTA
a.- Sindrome serotoninergico maligno.
b.- Sindrome neuroléptico maligno.
c.- Síndrome neurotóxico por litio.
d.- Encefalopatia por hepatotoxicidad.

PSICOSIS:
CIENCIAS BASICAS: La Asociación Psiquiátrica Americana estableció la diferenciación entre trastornos psicóticos y trastornos no
psicóticos, puesto que, ante una agresión exógena, es posible hallar, cuadros acreedores del término psicosis. También se estableció la
distinción entre trastorno "orgánico" y trastorno "sintomático". Esta subdivisión se apoya en el hecho de que en los trastornos
orgánicos existiría una lesión histopatológica cerebral de carácter permanente, con sintomatología crónica y estable. En los trastornos
sintomáticos existiría una alteración fisiopatológica cerebral sin lesiones orgánicas pesquisables, perturbando el organismo en forma
aguda y generalmente transitoria, con un cuadro clínico agudo y reversible. En algunos casos, si la afección se prolongase o fuera
demasiado intensa, o si existiera una labilidad previa del terreno, este cuadro podría evolucionar hacia un trastorno orgánico. De este
modo, el trastorno sintomático sería la manifestación nerviosa central de una enfermedad sistémica o de una intoxicación que no
habría producido una lesión histopatológica por el momento. Así, aunque la sintomatología confusional es preferentemente de origen
"sintomático" y la semiología demencial de procedencia "orgánica" es posible encontrar cuadros confusionales en una demencia,
alteraciones irreversibles de memoria después de algunos delirium tremens o cuadros neurotiformes en muchos trastornos "orgánicos"
o "sintomáticos". Por esta razón, DSM III entre otros, se refieren siempre a "trastornos orgánicos". 1.- Demencia de aparición senil o
presenil. 2.- Trastornos mentales orgánicos inducidos por sustancias psicoactivas. 3.- Trastornos mentales orgánicos asociados con
trastornos físicos del eje III, o cuya etiología es desconocida. Dentro de estos: a.- Delirium, b.- Demencia, c.- Síndrome amnésico, d.-
Trastorno delirante orgánico, e.- Alucinosis orgánica, f.- Trastorno orgánico del estado de ánimo, g.- Trastorno orgánico de ansiedad, h.-
Trastorno orgánico de la personalidad. La proclividad o la resistencia a hacer una psicosis somática varía mucho de un sujeto a otro. Los
factores individuales que intervienen en la etiopatogenia son: psíquicos, endógenos y somáticos. Los factores más influyentes son los
de orden somático, dentro de los cuales debe considerarse: la vía de penetración de la noxa, los dispositivos básicos en la defensa
metabólica e inmunitaria y el estado del sistema nervioso. Un factor individual importante de considerar es la edad del enfermo. En los
niños, las psicosis sintomáticas son mucho más frecuentes que entre los adultos y a menudo adoptan la forma de "delirium". Entre los
adolescentes abundan las psicosis sintomáticas esquizofreniformes. A medida que la edad es más avanzada, aumenta la frecuencia del
síndrome de Korsakow. Generalmente el EEG no se altera en las psicosis endógenas, mientras que a menudo muestra anomalías
persistentes de tipo difuso o central en las psicosis sintomáticas, las alteraciones EEG en las psicosis sintomáticas carecen de
especificacidad, varían según las oscilaciones del nivel de conciencia. A medida que progresa el compromiso de conciencia aumenta la
desorganización del trazado y la lentificación y la amplitud de la actividad bioeléctrica. PSICOSIS EXOGENAS AGUDAS: Se refiere a las
perturbaciones psiquiátricas agudas que aparecen en relación directa con alteraciones somáticas. Dichas alteraciones somáticas son de
la más diversa índole: afecciones cerebrales agudas, traumatismo y heridas cerebrales; epilepsia, enfermedades generales (infecciones,
intoxicaciones, anemias, etc.); trastornos endocrinos; puerperio; intoxicaciones por sustancias extrañas al organismo (alcohol,
anfetaminas, etc.). Para que se produzca la perturbación psiquiátrica, las alteraciones mencionadas o las noxas que de ella provengan
deben actuar sobre el cerebro, de manera directa o indirecta. Bonhöffer estableció los fundamentos del concepto de psicosis exógena y
demostró que los cuadros psicopatológicos resultantes se reducen a un escaso número a los que denominó "tipos de reacción exógena
aguda", destacando los siguientes síndromes o "formas de presentación": estado delirioso, epileptoide-angustioso, crepuscular,
amencial, alucinósico y estuporoso. El concepto de tipos de reacción exógena aguda lleva implícita la idea de una falta de especificidad
en la relación existente entre la naturaleza de la enfermedad somática y el cuadro psicopatológico resultante. Esto significa que
enfermedades corporales completamente diferentes pueden dar lugar al mismo cuadro psicopatológico. El síntoma central de estos
trastornos es el enturbiamiento de conciencia en sus diversos grados (excepto en el caso de la alucinosis). Bonhoffer también mencionó
las "formas de transcurso" de las reacciones exógenas agudas, es decir, conjunto de síntomas enmascarados entre los otros,
pertenecientes al síndrome hiperestísico emocional o al síndrome de Korsakow. El síndrome hiperestísico-emocional incluye: "aspecto
fatigado y marchito, color pajizo de la cara, hiperestesia a los ruidos, a la luz y el dolor, tristeza, mal humor, cefalea, mareos, astenia de
predominio vesperal y nocturno, labilidad de la memoria, labilidad emotiva, susceptibilidad a flor de piel, incapacidad de concentrarse,
tendencia marcada a las autorreferencias paranoicas las cuales nunca se configuran acabadamente, insomnio o sueño muy
interrumpido, no reparador y con actividad onírica laboral o terrorífica. El síndromde Korsakow está constituido por: desorientación en
tiempo y lugar, falso reconocimiento y fabulación de perplejidad. DIAGNOSTICO: Estos cuadros se inician bruscamente, aunque pueden
no ser evidentes desde el principio cuando son de ligera intensidad. Generalmente son reversibles si remite la patología subyacente,
pero algunos pueden evolucionar hacia un síndrome orgánico crónico, como cuando una psicosis post traumática aguda deja al
descubierto una demencia o cuando la encefalopatía de Wernicke se transforma en un síndrome amnésico duradero. Los cuadros
clínicos resultantes se deben fundamentalmente a la perturbación de la función cerebral normal por alteraciones bioquímicas,
eléctricas o mecánicas. Los síntomas comunes más importantes son los siguientes: Alteración del nivel de conciencia, alteraciones
psicomotoras, trastornos del pensamiento, alteraciones de la memoria, alteraciones perceptivas, alteraciones emocionales. Otras
manifestaciones que aparecen en estadías poco intensas de la enfermedad: síntomas neurotiformes de apariencia depresiva,
hipocondríaca, fóbica, histriónica, paranoide o esquizofreniforme. Los trastornos cuantitativos de conciencia corresponden a la

CURSO ENARM CMN SIGLO XXI TEL: 36246001 Pharmed Solutions Institute PÁGINA 266
MANUAL DE TRABAJO DEL CURSO ENARM CMN SIGLO XXI
obnubilación, que agrupa los cuatro trastornos cuantitativos, desde el más leve al más grave: embotamiento (disminución o retardo en
el ritmo de las elaboraciones psíquicas. Fatigable; la captación de los estímulos es trabajosa porque la percepción es lenta, imperfecta,
imprecisa y carente de nitidez; se altera la memoria), somnolencia (tendencia al sueño, pesadez, tiende a quedarse en la cama y
dormitar en el día. Hay disminución del ritmo alfa al EEG) o adormecimiento, sopor (sólo logra despertarse parcialmente, reacciones
psicomotoras rudimentarias y presenta respuestas reflejas. El EEG muestra ondas delta) y coma (EEG tiende a ser isoeléctrico). El
término obnubilación significa que la conciencia está nublada o empañada en relación con la función de alerta. TRASTORNOS
CUALITATIVOS DE CONCIENCIA: 1.- ESTADO CREPUSCULAR: Lo más característico de este trastorno es circunscribirse sólo a ciertas
manifestaciones de la vida psíquica, durante el cual el paciente es cogido por un afecto exaltado que tiñe de tal manera el campo de su
vivenciar que da la impresión de que toda la actividad psíquica del sujeto se concentra sólo en aquello que tiene relación con su rabia,
su angustia, su odio o su éxtasis. Presenta ilusiones y alucinaciones, principalmente visuales aunque también auditivas. El afecto
desmandado puede llevar al enfermo a incurrir en actos de violencia o a estados beatíficos de éxtasis. Los estados crepusculares en
general son breves, de minutos hasta días, raramente duran semanas o meses. Debido al fondo obnubilatorio presente en todo estado
crepuscular, hay amensia parcial o total del episodio vivido, pero mientras esté dentro de él, el enfermo recuerda lo que ha sucedido.
Se tiende a denominar estado crepuscular epiléptico a todo cuadro psicótico epiléptico con compromiso de conciencia. La psicosis
epiléptica que presenta fenómenos deliriosos no es un estado crepuscular epiléptico, sino un estado delirioso epiléptico. Los estados
crepusculares aparecen con mayor frecuencia en la epilepsia, en las psicosis sintomáticas y en la histeria. La embriaguez patológica es la
forma de estado crepuscular más frecuente de las psicosis sintomáticas, y se presentan generalmente en epilépticos, orgánicos o
psicópatas, con dosis pequeñas de alcohol. Son estados crepusculares desorientados acompañados de agitación psicomotora, con
amnesia posterior. Todas las causas de hipoxia cerebral pueden provocar un estado crepuscular. Los estados crepusculares de causa
orgánica son desorientados. Pueden ser debidos a repercusiones patológicas de las crisis epilépticas, el edema cerebral, el proceso
fundamental del cerebro, o bien a dosis excesivas de anticonvulsivantes. 2.- LA AMENCIA: Hay formas alucinatorias, catatónica e
ideofugal-incoherente. La obnubilación es un embotamiento leve. La psicomotrocidad da la impresión de intencionalidad, aunque los
actos son muy cambiantes e incompletos. El pensamiento es incoherente y la percepción de la realidad dificultosa. El sentimiento que
coge al paciente es de miedo, desamparo y angustia, que impregna todo el cuadro clínico. El miedo expectante lo lleva a sobresaltarse
al primer ruido, originando así percepciones deliriosas paranoídeas. El desamparo se observa en la expresión llorosa y los sentimientos
de soledad y abandono mencionados por el paciente. El enfermo está muy angustiado y perplejo, presentando de forma característica
cambios afectivos bruscos. A ratos da la impresión de que se sumerge en un mundo de vivencias deliriosas, alucinaciones y falsos
reconocimientos, todo esto vivido con gran sentimiento de realidad, que lo lleva a arrancar, defenderse, gritar, hacer movimientos
bruscos, tornarse negativista. Dentro del cuadro, el enfermo recuerda difícilmente lo sucedido horas antes o el día anterior; una vez
pasado el episodio puede haber amnesia completa o recuerdos fragmentarios y borrosos. Dura semanas o meses. 3.- ALUCINOSIS: Se
caracteriza por la presencia de alucinaciones auditivas en forma de voces, un delirio persecutorio, en directa relación con las voces,
generalmente no se presenta alteración de conciencia (el paciente está orientado, atento y reflexico). El contenido de las voces es
habitualmente de reproche y condenación, lo que en una atmósfera de intensa angustia lleva muchas veces al paciente al suicidio.
También se presentan voces que discuten entre sí, que comentan los propios actos o repiten lo que el enfermo piensa. Habitualmente
se presenta en el alcoholismo y puede durar días o semanas, pero también puede orientarse hacia la cronicidad. Conviene subrayar
algunas diferencias entre la alucinosis alcohólica y el delirum tremens: 1.- El delirium tremens generalmente es desencadenado por
abstinencia previa. La alucinosis suele desencadenarse por un aumento del consumo. 2.- En el delirium tremens hay alteración de
conciencia y la alucinosis aparece con conciencia clara. 3.- En el delirium tremens las alucinaciones son principalmente visuales y en la
alucinosis hay alucinaciones auditivas. 4.- El correlato somático agudo del delirium tremens no se encuentra presente en la alucinosis.
La alucinosis también es conocida como "delirio alucinósico de los bebedores", de Kraepelin o "alucinosis aguda de los bebedores" de
Wernicke. 4.- EL ESTADO EPILEPTOIDE ANGUSTIOSO: Para Roa; "el núcleo de la exacerbación lo forma casi siempre un sentimiento
indefinible de miedo, desamparo o soledad, con oscurecimiento de la situación y viva petición mímica de ayuda. El enfermo demanda
auxilio con las manos, se reclina sobre alguien, abre desmesuradamente los ojos, emite algunas interjecciones ansiosas, palidece o
enrojece y se recupera a los pocos minutos. En lugar de petición anhelante de ayuda puede haber violencia y agresividad. La
exacerbación, que puede terminar en una crisis convulsiva de acento epiléptico, puede repetirse muchas veces, sobre todo en
presencia de terceros. El estado básico entre las exacerbaciones es de euforia banal, con precipitación en las respuestas, sentimiento
íntimo de depresión, falta de concentración y quizás cierta pesadez corporal. Este estado dura días, semanas o meses. A veces precede
a los primeros ataques de la epilepsia genuina, acompaña los tumores encefálicos o matiza el curso de la histeria y la esquizofrenia". 5.-
EL ESTADO ESTUPOROSO: El estado estuporoso con compromiso de conciencia es una de las formas de manifestación de un estado
delirioso o crepuscular, en su inicio o durante el transcurso. "los estados estuporosos se dan con claridad u oscuridad de conciencia. El
primer caso es muy propio de la esquizofrenia catatónica". Los enfermos que quedan de pie, sentados o acostados, ya en posiciones
fijas, ya acomodándose ligeramente de vez en cuando y sin obedecer órdenes. Algunos caen en gatismo urinario y fecal, otros en
retención y unos terceros evacuan si son llevados al retrete. No rara vez el estado estuporoso es el comienzo de un estado delirioso,
crepuescular, amencial o uno de los momentos del curso de dichos estados. 6.- EL DELIRIO AGUDO O DELIRIUM: El delirium es el
síndrome psiquiátrico orgánico más frecuentemente observado por psiquiatras y por otros médicos. Se presenta en aproximadamente
el 10% de todos los pacientes ingresados a servicios médicos y quirúrgicos agudos, y se cree que su incidencia es aún mayor. Se ha
informado de cifras de hasta 80% en los servicios geriátricos agudos. El término sugiere un descarrilamiento o separación de una ruta o
vía, es decir, una desviación brusca del estado habitual del individuo. El DSM III R lo denomina delirium, pero también se ha propuesto
los siguientes términos: estado confusional agudo, psicosis tóxica, encefalopatía metabólica, enfermedad metabólica cerebral exógena,
síndrome de insuficiencia cerebral, síndrome cerebral agudo y estado disergástico. TRATAMIENTO: Manejo de antipsicóticos casos
leves (olanzapina, risperidona), en casos graves (haloperidol, levopromcina). Búsqueda de la causa y manejo de la misma.

CASOS CLINICOS

CURSO ENARM CMN SIGLO XXI TEL: 36246001 Pharmed Solutions Institute PÁGINA 267
MANUAL DE TRABAJO DEL CURSO ENARM CMN SIGLO XXI
SUICIDIO:
CIENCIAS BASICAS: Diversos estudios dan cuenta del incremento de suicidios relacionados con la vejez en muchos países, sobre todo
los industrializados. Son factores de riesgo la ancianidad, el sexo masculino, la raza blanca, una historia familiar de suicidios, los
conflictos familiares y de vidas severas y recientes, los trastornos de personalidad y la enfermedad mental. SALUD PUBLICA: En relación
con la gente joven se cita que la proporción de suicidas entre ancianos es 50% mayor. Además, las autopsias psicológicas de las
víctimas revelan que más de 90% presentaba un trastorno psiquiátrico en el momento de su muerte. ETIOLOGIA: Las causas habituales
para suicidio son: los trastornos afectivos; de ellos, el más frecuente es la depresión, con una prevalencia aproximada de 50-60%; le
siguen los trastornos por abuso de sustancias 19-60% y la esquizofrenia 2-13%. También se cita el trastorno de personalidad como
factor de riesgo para suicidio. Con respecto a este último los rasgos caracterológicos conectados con la conducta suicida son rigidez,
autosuficiencia, dificultad para aceptar la ayuda de los demás y narcisismo. De los problemas existenciales, relacionados con el suicidio,
se describen diferencias entre los jóvenes y los viejos. En los primeros los principales factores desencadenantes de estrés, previos al
suicidio, están relacionados con el trabajo, las finanzas y las dificultades en las relaciones interpersonales. Mientras tanto, entre los
adultos mayores el suicidio se comete en el contexto de enfermedades médicas, generalmente crónicas o invalidantes y pérdidas. El
suicidio en esta etapa de la vida puede ser silente, expresado mediante conductas pre suicidas como dejar de comer o medicarse. Sin
embargo, es común el acto impremeditado e impulsivo, en ocasiones precipitado por acontecimientos menores que no alertan a la
familia. Es usual que el suicida elija métodos drásticos tales como, el ahorcamiento o las armas de fuego. También hay quienes antes
del suicidio, mientras están deprimidos, cometen homicidios: para estos casos se cita una incidencia de 0.4-0.9 por 100, 000. Lo
referido destaca la importancia del reconocimiento precoz y el tratamiento oportuno del trastorno afectivo, además de la adopción de
medidas preventivas. Por último, es necesario subrayar que cerca de un tercio o la mitad de los suicidas hicieron contacto con
psiquíatras antes del acto, y que aproximadamente tres cuartas partes de ellos lo hicieron con un médico general en el mes previo al
acto, u horas antes de suicidarse. La ideación suicida supone entre el 5-20% de las consultas psiquiatricas urgentes (media=10%). 1% de
admisiones a urgencias en hospitales generales. El 2% fallecen tras la tentativa. El suicidio supone 8,5 por mil fallecidos. Métodos:
Suicidio consumado: ahorcamiento, precipitación, ahogamiento. Tentativas: 90% son sobredosis farmacológicas. Mas intentos de
suicidio en mujeres (9/1), más suicidios consumados en hombres (3/1). DIAGNOSTICO: Evaluacion de riesgo suicida de (SAD person):
¿El paciente tiene ideación suicida?: SI valorar (cada item positivo de los siguientes vale 1 punto): Sexo: varón. Edad: < 19 ó > 45 años.
Depresión clínica. Intentos de suicidio previos. Abuso de alcohol. Trastornos cognitivos. Bajo soporte social. Plan organizado de suicidio.
Sin pareja estable. Enfermedad somática. TRATAMIENTO: Modalidad de intervención: 0-2: Alta al domicilio con seguimiento
ambulatorio. 3-4: Seguimiento ambulatorio intensivo. Considérese el ingreso. 5-6: Hospitalización si no hay control familiar adecuado.
7-10: Hospitalización. Elevado riesgo de suicidio. Abordaje verbal del paciente suicida General: Preocuparse por las necesidades del
paciente y transmitir que se intenta ayudarle. Identificarse y explicar los objetivos. Mostrarse sereno, firme, seguro y honesto. Evitar
conductas hostiles, desafiantes, amenazantes o irrespetuosas. Específico (hablar de la problemática suicida). Cuidar la
contratransferencia (evitar o controlar reacciones de angustia, preocupación, hostilidad). Insistir en la dimensión temporal (la tendencia
suicida como crisis delimitada en el tiempo). Resaltar la ambivalencia de toda conducta (vivir vs.o morir como cuestión normal).
Intervención farmacológica a corto plazo en el paciente suicida. Tratamiento específico a corto plazo: Ansiedad: Benzodiacepinas de
vida media media/larga (diazepam). Insomnio: Benzodiacepinas vida media corta (Lorazepam). No utilizar fármacos potencialmente
letales. No utilizar antidepresivos en urgencias (incrementan el riesgo de suicidio)

CASO CLINICO
Ingresa a urgencias femenino de 27 años la cual se encuentra bajo tratamiento por trastorno de personalidad limite, ha sido tratada por
múltiples psiquiatras y esquemas terapéuticos, al ingresar el esposo de la paciente la encontró en la bañera con cortes superficiales en
ambas muñecas y falta de respuesta a los estímulos sin embargo respira superficialmente, en la sala de urgencias usted identifica
respiración lenta y superficial sin percibir olores específicos, observa ROT´s disminuidos, con respuesta ha estímulos dolorosos, su
tratamiento actual es clonacepam, clozapina y citalopram.

PREGUNTA
Considerando el manejo farmacológico cual sería la mejor intervención para revertir el efecto de una dosis excesiva ingerida en este
intento suicida.

RESPUESTA
a.- La administración de naloxona está indicada por polifarmacia.
b.- Un agonista antagonista parcial competitivo como flumacenil.
c.- Naltrexona es un fármaco que revierte la dosificación de benzodiacepinas.
d.- La paciente debe recibir biperiden para revertir los efectos de clozapina.

CURSO ENARM CMN SIGLO XXI TEL: 36246001 Pharmed Solutions Institute PÁGINA 268
MANUAL DE TRABAJO DEL CURSO ENARM CMN SIGLO XXI

CURSO ENARM CMN SIGLO XXI TEL: 36246001 Pharmed Solutions Institute PÁGINA 269
MANUAL DE TRABAJO DEL CURSO ENARM CMN SIGLO XXI

GINECOLOGIA Y OBSTETRICIA

1. PLANIFICACION FAMILIAR, ANTICONCEPCION, INFERTILIDAD, ESTERILIDAD.


2. AMENORREA PRIMARIA Y SECUNDARIA, ENDOMETRIOSIS.
3. CERVICOVAGINITIS, ENFERMEDAD PELVICA INFLAMATORIA, ENFERMEDADES DE TRANSMISION SEXUAL.
4. INFECCION POR VIRUS DEL PAPILOMA HUMANO. DISPLASIAS, CANCER CERVICOUTERINO Y OVARICO.
5. SANGRADO UTERINO ANORMAL Y DISFUNCIONAL. CANCER ENDOMETRIAL, SINDROME DE OVARIO POLIQUISTICO.
6. MASTOPATIA FIBROQUISTICA, CANCER DE MAMA, MENOPAUSIA Y CLIMATERIO
7. CONTROL PRENATAL Y DEL EMBARAZO.
8. EMBARAZO ECTOPICO. AMENAZA DE ABORTO, ABORTO, INCOMPETENCIA CERVICAL.
9. HIPEREMESIS GRAVIDICA, ENFERMEDAD TROFOBLASTICA.
10. HIPERTENSION ARTERIAL, ENFERMEDAD HIPERTENSIVA DEL EMBARAZO, PREECLAMPSIA Y ECLAMPSIA.
11. DIABETES MELLITUS, INTOLERANCIA A LA GLUCOSA, DIABETES GESTACIONAL.
12. POLIHIDRAMNIOS, OLIGOHIDRAMNIOS, PLACENTA PREVIA, DESPRENDIMIENTO DE PLACENTA.
13. ANEMIAS, IVU, PARTO PRETERMINO, PARTO NORMAL Y CESAREA.
14. COMPLICACIONES DEL PUERPERIO, HEMORRAGIA OBSTETRICA, PLACENTA ACRETA.
15. SINDROME DE TENSION PREMENSTRUAL, PSICOSIS POSPARTO Y DEPRESION POSTPARTO.

CURSO ENARM CMN SIGLO XXI TEL: 36246001 Pharmed Solutions Institute PÁGINA 270
MANUAL DE TRABAJO DEL CURSO ENARM CMN SIGLO XXI

PLANIFICACION FAMILIAR, ANTICONCEPCION Y CONTRACEPCION:


CIENCIAS BASICAS: Planificación familiar: es el derecho de toda persona a decidir de manera libre y responsable e informada sobre el
número y espaciamiento de sus hijos y a obtener la información especializada y los servicios idóneos. Métodos anticonceptivos:
procedimientos que se utilizan para impedir el embarazo en forma temporal o permanente y se clasifican de la siguiente manera;
Temporales hormonales (orales, inyectables, parches) y no hormonales (barrera, naturales, DIU). Definitivos vasectomía, salpingoclasia.
CRITERIOS para iniciación y seguimiento de todos los métodos anticonceptivos; 1. Conocer la eficacia del método. 2. Ventajas y
desventajas. 3. Efectos colaterales y complicaciones. 4. Prevención de ETS. 5. seguimiento
METODOS ANTICONCEPTIVOS
CLASIFICACION METODO DESCRIPCION COMO FUNCIONA EFICACIA OBSERVACIONES
Temporal Orales Estrógeno (etinilestradiol o Evita la liberación de >99% (uso Disminuye el riesgo de cáncer endometrial
hormonal combinados mestranol) y progestágeno óvulos por los ovarios correcto y y ovárico: no tomar en la lactancia
oral (norentindrona o acetato de (ovulación) sostenido) Reducción de la frecuencia de spotting
norentindrona o noretinodrel o 92% como se Disminución del riesgo de EPI (cambia la
norgestrel o levonorgestrel) usa consistencia del moco y protege)
comúnmente No usar en Ca de mama, endometrio,
CaCu, hepatopatías, varices extensas,
mujeres de 35-40años
Temporal Orales Contiene únicamente Hace más espeso el moco >99% (uso Puede usarse durante la lactancia: tomar
hormonal minipastillas progesterona (levonorgestrel), del cuello uterino, lo que correcto y todos los días a la misma hora
oral minidosis 350mcg impide entrada de sostenido) Pueden inducir amenorrea a lo largo del
espermatozoides y 92% como se tiempo
previene la ovulación. usa
Modifican endometrio comúnmente
para disminuir
probabilidades de
implantación
Temporal Anticoncepción Levonorgestrel 1,5 mg. Se Evita la ovulación Reduce de un No altera el embarazo si este ya se ha
hormonal emergencia toman para prevenir embarazo, 1ra dosis dentro de las 72 60% as un producido
oral hasta 5 días después de la hrs poscoito y 2da dosis, 90% el No protegen de ETS. Causan más efectos
relación sexual sin protección 12 hrs después de la embarazo colaterales
primera
Temporal Implantes Cilindros (silastic, Mismo mecanismo de las >99% Debe ser insertado y extraído por personal
hormonal Subdérmicos y etonorgestrel)o capsulas pastillas de progestágeno sanitario, se puede usar por 3-5 años,
pequeñas y flexibles que se según el tipo. Puede generar spotting
colocan debajo de la piel del
brazo, contienen únicamente
progestágeno
Temporal Parches Sistema trasndérmico >99% Menor perdida sanguínea, ciclos
hormonal (norelgestrinoma y mentsruales predecibles, disminuye acné,
etinilestradiol) hirsutismo, disminuye riesgo de ca de
Se cabia cada semana x 3 sem, ovario y de endometrio. Contraindicado en
la sem 4 no se utiliza flebitis, enf. Coronaria, EVC, ca de mama,
tumores hepático, embarazo
Temporal Inyectable de IM profunda cada 2-3 meses. Mismo mecanismo de >99% Al cesar el uso, la fecundidad tarda en
hormonal progestágeno Se inicia en los primeros 7 días pastillas de progestágeno reaparecer (1-4meses); Puede haber
inyectable del ciclo. Enantato de solo spotting. Indicado en nuligestas, nulíparas
noretisterona o multíparas, antes del 1er embarazo aun
200mg/1ml=c/2meses. Acetato cuando estén contraindicados los
de medroxiprogesterona=c/3 estrógenos (VPH), puérperas lactando
meses150mgs/3ml (usar después de 6ta sem).
Temporal Inyectable IM profunda cada mes, se Mismo mecanismo de >99% (uso Por ser de depósitos tienen más efectos
hormonal mensual inicia en primeros 5 días del anticonceptivos orales en correcto y adversos
inyectable ciclo, contiene estrógeno y combinación. Su sostenido) Hemorragias vaginales comunes. Indicado
progestágeno (Cipronato de protección anticonceptiva 97% como se en nuligestas, nulíparas y multíparas, antes
estradial +Acetato de se extiende hasta 33 días usa de 1er embarazo, posaborto, postparto y
Medroxiprogesterona. comúnmente postcesarea, si no está lactando.
Valerianato de estradiol+ Contraindicado en la lactancia en primeros
Enantato de norestisterona) 6 meses posparto, embarazo o sospecha
de él, en ca mamario o de cérvix, enf.
Tromboembolica, hepatopatías
Temporal no Dispositivo Dispositivo de plástico flexible y El cobre daña los >99% Disminuye la frecuencia del cólico
hormonal intrauterino pequeño que contienen una espermatozoides e menstrual y los síntomas de endometriosis
(DIU) asa o cubierta de cobre y se impiden que se junten amenorrea, en un grupo de usuarias
inserta en el útero con el ovulo
Temporal no Condón Vaina o cubierta que envuelve Forma una barrera que 98% usado de También protege de las ETS, en particular
hormonal de masculino el pene erecto impide el encuentro de manera la causada por VIH
barrera los espermatozoides con correcta y
el ovulo sostenida
85% como se
usa
Temporal no Condón Vaina o forro que se adapta Forma una barrera que 90% También protege de las ITS, en particular la
hormonal de femenino holgadamente a la vagina; esta impide que los causada por VIH
barrera hecho de un material plástico espermatozoides y el
transparente, fino y suave ovulo se junten
Temporal no Diafragma Cubre el cérvix. También hay Forma una barrera que 90% El riesgo de embarazo aumenta, si no se
hormonal de (capuchón de diafragmas de plástico y impide que los usan espermaticidas juntos. No interfiere
barrera látex blando) silicona disponibles. El borde espermatozoides y el la lactancia. Puede introducirse hasta 6 hrs
contiene un resorte firme y ovulo se junten antes. Puede brindar cierta protección

CURSO ENARM CMN SIGLO XXI TEL: 36246001 Pharmed Solutions Institute PÁGINA 271
MANUAL DE TRABAJO DEL CURSO ENARM CMN SIGLO XXI
flexible que mantiene el contra algunas ITS. Extremadamente
diafragma en su lugar raros: síndrome de shock tóxico
Temporal no Espermaticida Nonoxinol-9.Detergentes no Funcionan rompiendo la 70-80% Uso correcto en cada acto sexual. Pueden
hormonal de iónicos. Los espermicidas se membrana de las células utilizarse como método principal o como
barrera introducen profundamente en del esperma, matándolas método de respaldo. Pueden irritar el pene
la vagina poco antes de tener o enlenteciendo su o la vagina, alergia local. Favorecen IVU.
sexo. movimiento. No usar pacientes con SIDA o VIH.
Temporal no Coito El hombre retira el pene fuera Evita que los 96% uso de Es uno de los métodos menos eficaces,
hormonal interrumpido de la vagina y eyacula fuera de espermatozoides manera porque a menudo resulta difícil determinar
natural esta, con lo que el semen no penetren en la vagina, correcta el momento, preciso de la retirada
tienen contacto con los con lo que se evita la 73% como se
genitales externos de la mujer fecundación usa
Temporal no Métodos que Método de calendario (Ogino- La pareja evita tener 70-80% uso El uso correcto y regular exige la
hormonal requieren Knauss): temperatura corporal embarazo, evitando tener de manera colaboración del varón. Cuando hay
natural conocimiento basal (TCB), método del moco coito, sin protección correcta infección vaginal, es difícil interpretar el
de la cervical (Billings), sintotérmico durante los días fecundos 63% como se moco. Por lo menos hay que observar 6
fecundidad y/o (TCB+moco cervical) generalmente, mediante usa ciclos menstruales, mientras abstinencia o
abstinencia la abstinencia comúnmente método de barrera. De las 2 fases del ciclo
periódica menstrual, solo la 2da es la constante
Temporal no Amenorrea en Protección prolactina. Para Impide que los ovarios 99% si se Esta basado en el efecto natural del
hormonal la lactancia mujeres recién paridas que no liberan óvulos (ovulación) aplica de amamantamiento sobre la fecundidad
natural han vuelto a menstruar, exige manera
el amamantamiento exclusivo correcta
día y noche, de una criatura 98% como se
menor de 6 meses usa
comúnmente
Definitivo Vasectomía Se bloquean o cortan los Impide que haya >99%. Tarda en actuar unos 3 meses debido a
conductos deferentes que espermatozoides en el Después de la que quedan espermatozoides
transportan los semen eyaculado evaluación almacenados; no afecta el funcionamiento
espermatozoides desde los del semen a sexual del hombre
testículos los 3 meses
Definitivo Salpingoclasia Se bloquean o cortan las Los óvulos no pueden >99% Es fundamental que sea una elección
trompas de Falopio juntarse con los voluntaria y con conocimiento de causa
espermatozoides

CASO CLINICO
Se trata de paciente femenino de 23 años de edad la cual acude a consulta para planificación familiar, solicita un método
anticonceptivo, refiere estar casada desde hace 3 meses y desea no embarazarse por lo menos en 3 años más, niega antecedentes
heredofamiliares como hipertensión, diabetes mellitus, así como otras relevante, al interrogatorio niega tabaquismo y alcoholismo, no
toma medicamentos, a la exploración física no se agregan datos de relevancia, IMC 24, TA 100/65 mmHg, FC 78, FR 19. Aparentemente
la paciente se encuentra sana.

PREGUNTA
La paciente solicita un método anticonceptivo oral, cuál sería el método de contracepción más apropiado para esta paciente desde el
enfoque de riego, así como en función del tiempo de espera que quiere para embarazarse.

RESPUESTA
a.- No hay contraindicación para el método anticonceptivo oral por carencia de factores de riesgo.
b.- No hay contraindicación para el método anticonceptivo oral ya que es seguro y efectivo por el tiempo que desea usarlo.
c.- Igualmente podría usarse anticonceptivo inyectable mensual o trimestral.
d.- Considerando su edad y inicio de vida sexual activa, un método de barrera sería la mejor opción.

CASO CLINICO
Paciente femenino de 28 años de edad la cual se encuentra en pos-aborto de 5 dias, refiere que ya no presenta sangrado, acude a
solicitar método anticonceptico, refiere que no quiere tener mas hijos por lo menos en 3 años, tiene 2 hijos, anteriormente empleaba
hormonales orales y durante este método se embarazo, no cuenta con antecedentes de cervicovaginitis ni EPI.

PREGUNTA
Cual de los siguientes métodos de planificación familiar es el mas adecuado para este caso?

RESPUESTA
a.- Hormonal inyectado.
b.- Implante hormonal.
c.- Presenvativo.
d.- Dispositivo intrauterino.

PREGUNTA
La paciente acude ha consulta por cambios del flujo menstrual, el cual refiere es obscuro y ha presentado manchado intermenstrual, a
la exploración se obseva moco cervical con sangre moderada, cual es la conducta a seguir?

RESPUESTA
a.- Retirar el DIU.

CURSO ENARM CMN SIGLO XXI TEL: 36246001 Pharmed Solutions Institute PÁGINA 272
MANUAL DE TRABAJO DEL CURSO ENARM CMN SIGLO XXI
b.- Indicar un AINES.
c.- Indicar antibiótico.
d.- Indicar DOC.

PREGUNTA
La paciente acude 5 años después por flujo abundante con olor desagradable, a la exploración se observa un proceso compatible con
cervicovaginitis, agrega que hace un año se divorcio por infidelidad por parte del esposo y actualmente ya tiene otra pareja, cual es la
conducta a seguir más adecuda.

RESPUESTA
a.- Retirar el DIU.
b.- Cambio de método a hormonal.
c.- Indicar acido mefenamico.
d.- Indicar antibioticoterapia y preservativo.

PREGUNTA
La paciente acude 6 meses después por dolor abdominal en fosa iliaca izquierda que se irradia a hipocondrio y hacia tras, a la palpación
se presenta dolor abdominal de predominio pélvico, acude además porque su DOC presento cambios inflamatorios inespecíficos, cual
es la conducta a seguir por en este caso?

RESPUESTA
a.- Retirar DIU por probable EPI.
b.- Retirar DIU por resultado de DOC.
c.- Retirar DIU por probable embarazo.
d.- Retirar DIU por cambio de método.

CASO CLINICO
Se trata de paciente femenino de 22 años de edad la cual acude a solicitar método de planeación familiar, actualmente vive en casa con
sus padres, refiere un IVSA a los 20 años, agrega que ha utilizado preservativo pero lo considera con fallas, además no desea que sus
familiares se enteren que tiene vida sexual activa, por otro lado menciona que mantiene una relación de dos años de duración y no
desea embarazarse, a la exploración física se observa con leve sobrepeso, con hirsutimo y acné presente, refiere que sus periodos
menstruales son irregulares.

PREGUNTA
Considerando la solicitud de la paciente y su juicio clínico cual sería su conducta a seguir:

RESPUESTA
a.- Considera suficiente la información y hallazgos para indicarle el método solicitado.
b.- Considera que necesita mayor información por lo que envía a realizar un USG.
c.- Considera que debe realizar un PIE por la irregularidad menstrual.
d.- Considera que debe esperar el próximo periodo para tomar la decisión con la paciente.

CASO CLINICO
Masculino de 31 años de edad el cual acude para realizarse vasectomía, refiere que tiene una paternidad satisfecha, a la exploración
clínica se observa aparentemente sano, a la exploración genital se observa bolsa escrotal con aspecto normal, a la palpación hay leve
dolor y se aprecia una masa irregular depresible, tortuosa, refiere el paciente que desde los 25 años se había percatado, sin embargo
no le prestó importancia solo ocasionalmente le duele, principalmente cuando camina.

PREGUNTA
Considerando las manifestaciones clínicas, cual es la conducta a seguir respecto al procedimiento que solicita el paciente.

RESPUESTAS
a.- No presenta contraindicación de relevancia para poder realizar la vasectomía.
b.- Debe descartarse un proceso neoplasico antes de realizar la vasectomía.
c.- Debe realizar un USG para descartar un varicocele antes de realizar la vasectomía.
d.- Debe cuantificar factores tumorales antes de realizar la vasectomía.

CASO CLINICO
Femenino de 21 años de edad la cual se encuentra en procedimiento cesarea transoperatorio, actualmente se realiza el procedimiento
por cesarea previa y periodo intergenesico de 10 meses, su primer embarazo fue a los 16 años el cual fue interrumpido mediante
legrado, actualmente es su tercer hijo y refiere maternidad satisfecha al preguntarle, sin embargo refiere que es muy joven aun, y
podría utilizar un dispositivo intrauterino.

PREGUNTA
Considerando el caso y tomando en cuenta la condición quirúrgica en la que se encuentra la paciente, cuál sería la mejor opción de
esta.

CURSO ENARM CMN SIGLO XXI TEL: 36246001 Pharmed Solutions Institute PÁGINA 273
MANUAL DE TRABAJO DEL CURSO ENARM CMN SIGLO XXI

RESPUESTA
a.- Solicitar autorización y colocar DIU ya que la paciente es joven aun.
b.- Solicitar autorización y realizar OTB ya que la paciente tiene riesgo obstétrico.
c.- Le explica brevemente las ventajas y la dice que podría pensarlo.
d.- El GO continúa el procedimiento y la podría realizarse OTB por endoscopia posteriormente.

INFERTILIDAD, ESTERILIDAD. CIENCIAS BASICAS: Esterilidad; incapacidad de una pareja para lograr la cocenceppcion. La esterilidad se
clasifica en: primaria (cuando la pareja, tras un año de relaciones sin tomar medidas de protección, no ha conseguido un embarazo) y
secundaria (la de la pareja que, tras la consecución del primer hijo, no logra una nueva gestación tras 2 o más años de intentarlo). El
tiempo mínimo a partir del cual se habla de esterilidad se fija en un año de relaciones sexuales con deseo de descendencia. Infertilidad;
es la incapacidad para producir un hijo vivo. Así, se denomina infertilidad primaria la que padece una pareja que consigue una gestación
que no llega a término con un recién nacido normal, e infertilidad secundaria cuando, tras un embarazo y parto normales, no se
consigue una nueva gestación a término con recién nacido normal. Infecundidad; incapacidad de una pareja de llevar a termino un
embarazo. Fertilidad humana: En comparación con otras especies animales, el proceso reproductivo en humanos resulta altamente
ineficaz. Existe un momento de máxima fecundidad en los primeros meses (alrededor de 20-30% de embarazos por ciclo) de forma que
al cabo de tres ciclos, dos terceras partes de las mujeres han logrado el embarazo. Dicha fecundidad se reduce paulatinamente en ciclos
sucesivos. Globalmente, la tasa acumulada de embarazo entre parejas de fecundidad probada es del 90% aproximadamente a los 12
meses y de alrededor del 95% a los 24 meses. SALUD PUBLICA: Aproximadamente nueve de cada diez parejas en edad fértil que
mantienen relaciones sexuales regulares consiguen un embarazo durante el primer año. Las cifras de prevalencia de esterilidad que
manejan los especialistas sitúan entre un 12 y un 20% la imposibilidad de tener un hijo. La edad de la mujer es importante: se estima
que las posibilidades de que una mujer de 20-24 años no se quede embarazada están entre el 4-5%. Estas posibilidades aumentan
lentamente con la edad, de tal forma que a los 35 años estarían alrededor del 14%. Este suave aumento de la esterilidad en la mujer
sufre un brusco cambio a partir de los 35 años, de tal forma que a los 40 años la esterilidad en la mujer rondaría cifras del 65-70%. Este
incremento se ve agravado por el hecho de que también aumentan las tasas de aborto, que a partir de esta edad son aproximadamente
del 71%. La esterilidad e infertilidad suponen una situación carencial que no compromete la integridad física del individuo ni supone un
riesgo vital, sin embargo, dado que la mayoría de los adultos contemplan como objetivo vital esencial el tener hijos, esta carencia
puede incidir negativamente en el desarrollo de la persona, produciendo frustración y desmoralización. Por ello suponen un problema
de salud pública de trascendencia. Las técnicas de reproducción humana han alcanzado un auge importante en las dos últimas décadas.
PATOGENIA: Un 40% de los casos de esterilidad son de causa masculina: por alteraciones en el ámbito testicular, obstrucción de
conductos, patologías en la próstata, alteraciones en la eyaculación o erección y alteraciones en el semen. Otro 40% se debe a
causas femeninas, como la menopausia precoz, la endometriosis, las obstrucciones o lesiones de las trompas de Falopio, anomalías
uterinas y cervicales o los problemas ovulatorios. El 20% restante corresponde a causas mixtas o combinadas, en las cuales ambos
cónyuges son responsables. Aborto de repetición: Posiblemente pocos problemas ginecológicos resulten tan frustrantes, tanto para la
pareja como para el médico, como el aborto de repetición, equivalente al término castellano «infertilidad». Para la pareja, por lo
traumático que resulta la pérdida repetida de gestaciones que, en general, se suelen conseguir con facilidad, se malogran sin
justificación clara y carecen de expectativas satisfactorias de tratamiento en algunas ocasiones. El término «aborto de repetición» se
aplica en la actualidad a aquellas situaciones en las que se han producido al menos dos abortos consecutivos o más de dos alternos.
Factor vaginal: las infecciones que se presentan pueden reprimir el coito (por dispareunia), las infeccionescausads por Chamydia
trachomatis, ureaplasma urealyticum, mycpoplasma, traen problemas de esterilidad, ya que ejercen accion nocisa sobre los
espermatozoides, ovulos y embriones.. Factor cervical: incopentencia istimico-cervical. Factor uterino; responsable del 5-10% de los
casos de infertilidad; por presenia de polipos, miomas intracavitarios y algunas de las malformaciones Muellerianas (bidelfo, bicorne).
Factor tubo-peritoneal: responsable de 15-20% de todos los casoso de infertilidad, agenesia , estenosis, a tresias, duplicacione y
orificios accesorios en salpinges. Factor endocrino-ovarico: Responsable del 25-30% de los caosos de infertilidad. Fcator inmunologico:
es bajo solo representa de 3-5% de todos los casos. Las causas que podemos buscar ne el varon son: infecciones, problemas
inmnologicos, tumores, alteraciones neurlgicas, ambientales, toxicas, idiopatica, disfuncion erectil. DIAGNOSTICO DE LA PAREJA
ESTÉRIL: Se han descrito multitud de pruebas diagnósticas encaminadas al despistaje de cada uno de los factores implicados en la
reproducción. Por ello, y en un intento de simplificar el camino a estas parejas que tanta carga psíquica, física y económica han de
soportar, se ha simplificado al máximo el estudio. Pruebas diagnósticas esenciales son: Evidencia de que la mujer ovula, para lo cual es
suficiente comprobar que menstrúa cada 28 ± 7 días. Anamnesis e historia clínica para detección de enfermedades concomitantes
(diabetes, hipertensión, alteraciones tiroideas, etc.) e historial médico familiar sobre patologías y enfermedades hereditarias. Evidencia
de una reserva ovárica suficiente: se determina sistemáticamente, de forma basal (tercer día del ciclo), la FSH (hormona
folículoestimulante) y el estradiol sérico. Deeterminar prolactina, ya que la hierprolactinemia se encuentra entre 10-30% decasos de
infertilidad. Debe encontrase <20ng/ml. Evidencia de que el semen es normal, para lo cual se realiza un seminograma. Evidencia de que
el canal genital es normal, para lo cual basta inicialmente con una histerosalpingografía (HSG) y una ecografía transvaginal (de eleccion
para patolgia ginecologica) que explore también los ovarios, en busca de quistes endometriósicos o endometriomas y miomas uterinos
fundamentalmente. La HSG es una prueba molesta, por lo que siempre se realiza tras evidenciar un espermiograma normal o suficiente
para la realización de coitos programados o inseminaciones artificiales, muestra alteraciones morfologicas de la cavidad y area corneal.
Durante el screening inicial se evidencia la existencia de las siguientes patologías: Alteraciones en la ovulación: síndrome de ovarios
poliquísticos, amenorreas de causa central o periférica, fallos ováricos precoces o menopausia precoz, fallos ováricos ocultos, etc.
Según la patología encontrada se completan las pruebas pertinentes: cariotipo, resistencia a la insulina, anticoagulantes circulantes,
autoanticuerpos, etc. Presencia de quistes endometriósicos, miomas uterinos, pólipos endometriales, alteraciones mullerianas en la
anatomía genital (útero doble, tabiques, ausencia de vagina, etc.), alteraciones en las trompas de Falopio (obstrucciones, inflamación y
presencia de hidrosálpinx). En estos casos se considera la necesidad de realizar una valoración endoscópica por vía laparoscópica o
histeroscópica para definir mejor el diagnóstico o para realizar las correcciones quirúrgicas pertinentes. Alteraciones en el
espermiograma: se valora la concentración, la motilidad y la morfología de los espermatozoides. En ocasiones se objetiva una ausencia

CURSO ENARM CMN SIGLO XXI TEL: 36246001 Pharmed Solutions Institute PÁGINA 274
MANUAL DE TRABAJO DEL CURSO ENARM CMN SIGLO XXI
de espermatozoides en el eyaculado (azoospermia), por lo que se deberá recurrir a la biopsia testicular para completar el estudio y
obtener espermatozoides testiculares o de epidídimo con los que poder trabajar a posteriori. Valores recomnedados por OMS:
volumen: 1,5-5,0ml, pH; >7.2, viscocidad; <3 (escala 0-4), espermatozoides; >20 millones/ml, N. total de espermatozoidez; >40
millones/eyaculado, porcentaje demotilidad >50%, progresion progresiva; >2 (escala de 0-4), Morfologia normal; >50% normal,
aglutinacion de espermatozoides <2 (escala de 0-3). Laparoscopia: para valorar factor tubo-peritoenal, es el elemento mas valioso para
el diagnostico y tratamiento mas valioso, nos yauda a corroborar la permeabilidad de las salpinges y permeabilizar las mismas.
TRATAMIENTO: partir de ese momento, se decide ya un tratamiento para la pareja. Si todas las exploraciones resultan normales se
indica la realización de coitos programados con estimulación de la ovulación (si el problema es una disfunción ovulatoria, como ocurre
en los ovarios poliquísticos) o de inseminaciones artificiales (cuando supuestamente los ciclos de la mujer han sido regulares). La
experiencia demuestra que la inseminación conyugal proporciona tasas acumuladas de embarazo > 60%, por lo se aconseja instaurar
este tipo de terapia durante 4 ciclos. Si la mujer no gesta, el camino a seguir dependerá mucho de su edad, pero en casi todos los casos
se decide practicar fecundación in vitro (FIV) para estudiar ambos gametos en el laboratorio y la interacción de ambos (fecundación y
desarrollo embrionario). En el 20% de los casos en los que las inseminaciones fallan y se realiza fecundación in vitro, se ha observado
que no se produce la penetración del espermatozoide en el interior del ovocito u óvulo y por lo tanto se produce un fallo de
fecundación, responsable de la esterilidad en estas parejas. Por ello, en un primer ciclo tras fallo de inseminaciones, se realiza con la
mitad de los óvulos FIV y con la otra mitad ICSI o microinyección intracito plasmática de espermatozoides (1/2 FIV- 1/2 ICSI), lo que
permite decidir sobre la técnica más adecuada en el futuro y al menos asegurar fecundación en la mitad de los ovocitos, para poder
realizar transferencia embrionaria y rentabilizar el ciclo con posibilidad de gestación. FÁRMACOS EMPLEADOS: La terapia
farmacológica contempla las siguientes opciones: 1) Citrato de clomifeno: Se emplea por vía oral. Su acción se basa en la unión del
fármaco con los receptores estrogénicos de la hipófisis, que provoca el bloqueo de la retroalimentación negativa del estradiol. Ello
induce un incremento de la secreción de gonadotropinas por la hipófisis. Es un fármaco de fácil uso y barato, pero el efecto
antiestrogénico a nivel endometrial y del cuerpo lúteo puede ser responsable de las bajas tasas de implantación que permite obtener.
2) Gonadotropinas humanas: Las primeras estimulaciones de la ovulación se realizaron con gonadotropina hipofisaria humana, pero
tuvo que ser abandonada por la aparición de la enfermedad de Creutzfeldt-Jakob. Años más tarde se aislaron gonadotropinas en las
orinas de las mujeres menopáusicas (gonadotropina coriónica humana o HMG) con escasa pureza y conteniendo FSH y hormona
luteinizante (LH). Desde entonces ha habido una intensa investigación farmacéutica para aislar FSH de alta pureza sin carga de LH y con
posibilidad de administración subcutánea, y desde 1998 existe la FSH recombinante obtenida por ingeniería genética. En breve saldrá al
mercado LH recombinante. 3) Análogos de la GnRH: Uno de los avances más importantes en el campo de la reproducción asistida ha
sido el descubrimiento de los análogos de la GnRH. Administrados en distintas fases del ciclo menstrual y combinados con la
estimulación de la ovulación con gonadotropinas, proporcionan una mayor sincronía en el crecimiento de los folículos, impiden el pico
espontáneo de LH que provocaría una luteinización prematura de los mismos (impiden la ovulación espontánea) y disminuyen el
número de cancelaciones de procedimientos. Con su uso se han mejorado las tasas de fecundación, de número de ovocitos obtenidos y
ovocitos maduros obtenidos, así como de las tasas de gestación. Su mecanismo de acción es en dos tiempos: en un primer momento
por su unión al receptor estimulan la producción (efecto flare up), para que horas después desensibilicen los receptores por saturación
(hipofisectomía médica reversible). 4) Inductores de la ovulación HCG: La HCG u hormona del embarazo farmacológica induce la
ovulación 37 horas tras su administración. Obtenida de la orina de mujeres embarazadas, su acción es equivalente a la de la LH
hipofisaria. La LhRH recombinante está en fase de ensayo clínico. 5) Antagonistas de la GnRH: Bloquean los receptores de la GnRH,
suprimiendo la secreción de gonadotropinas por la hipófisis de forma inmediata y evitando el efecto flare up provocado por el uso de
análogos de la GnRH. En breve aparecerán en el mercado farmacéutico español preparados de administración subcutánea.

CASO CLINICO
Femenino de 37 años de edad la cual acude a consulta debido a que no ha logrado embarazarse, refiere que lleva 4 años de
intentándolo, con vida sexual activa desde hace 5 años con la misma pareja, no usa método anticonceptivo, como antecedentes refiere
menarca a los 19 años, gesta 0, para 0, abortos 0, a la exploración física se observa con caracteres sexuales secundarios adecuados, a la
exploración GO sin secreción vaginal blanquecino sin olor desagradable, a la especuloscopia con normal, acude con resultados de
Papanicolaou con datos inflamatorios crónico, colposcopia negativo para VPH y perfil hormonal normal (FSH, LH, P4, E2).

PREGUNTA:
Tomando en cuenta el perfil hormonal de la paciente que intervención en primera instancia tomaría usted:

RESPUESTA:
a.- Iniciar clomifeno 5 mg cada 24 hrs durante 14 días.
b.- Solicitar una espermatobioscopia.
c.- Realiza USG de anexos para buscar causa obstructiva.
d.- Indicar medroxiprogesterona 25mg y etinilestradiol 2mcg im.

CASO CLINICO
El resultado de laboratorio reporta 40 % de espermatozoides viables, tomando en cuenta estos resultados, decide continuar el
abordaje mediante estudio de ultrasonografia pélvica, donde se reporta ovarios de características adecuadas, sin embargo se reporta
alteraciones de oviductos, la paciente agrega que en los últimos 2 años ha presentado cuadro repetidos de infecciones cervicovaginales
por lo que indica un cultivo vaginal, recibiendo resultado positivo para garnerela y cladmidia tracomatis resistente a doxiciclina.

PREGUNTA:
La paciente es referida a biología de la reproducción, donde se indica histerosalpingografia, la cual reporta obstrucción bilateral del 75
% de oviductos, tomando en cuenta los últimos reportes de laboratorio y gabinete, cuál sería la complicación más importante para
conseguir la concepción en este caso:

CURSO ENARM CMN SIGLO XXI TEL: 36246001 Pharmed Solutions Institute PÁGINA 275
MANUAL DE TRABAJO DEL CURSO ENARM CMN SIGLO XXI
RESPUESTA:
a.- Nivel de espermatozoides viables disminuido.
b.- Secuelas de Enfermedad Pélvica Inflamatoria.
c.- Resistencia a los antibióticos de primera línea.
d.- La condición mixta como causa de la infertilidad.

CASO CLINICO
Varón de 25 años de edad, sin antecedentes personales de interés, que fue remitido para la realización de una ecografía escrotal por
presentar dolor testicular izquierdo de 4 meses de evolución; en la exploración física se pone de manifiesto la presencia de un
varicocele. Se practica ecografía con escala de grises y ecografía Doppler color con un transductor lineal con una frecuencia 10 MHz. La
prueba se realiza con el paciente en posición de decúbito supino. El teste derecho es normal ecográficamente. Se observa un varicocele
extratesticular izquierdo en un testículo de tamaño normal, así como estructuras hipoecoicas tubulares en su interior con flujo venoso
que aumenta durante la maniobra de Valsalva en la ecografía Doppler; estos hallazgos se corresponden con un varicocele
intratesticular, con resultado de laboratorio donde se reporta azoospermia y reporte de urocultivo positivo a Chlamydia.

PREGUNTA:
Cual de las siguientes condiciones afecta de manera significativa a la fertilidad del paciente con las condiciones que presenta:

RESPUESTA:
a.- El varicocele.
b.- La Chamydia
c.- La azoespermia.
d.- Obtructivo

CASO CLINICO
Se trata de pareja que consultó con una historia de infertilidad primaria de 2 años. Ella de 36 años y su marido de 37 años. Dentro del
estudio inicial se encontró oligoastenozoospermia moderada. Ella además presentaba anovulación secundaria a un síndrome de ovario
poliquístico. Luego de intentar 4 ciclos de superovulación con inseminación intrauterina sin éxito, Biología de la Reproducción decidió
optar por un ciclo de alta complejidad.

PREGUNTA:
Cual es la complicación más probable para la paciente por la exposición a los ciclos de superovulación a la que fue sometida:

RESPUESTA:
a.- Sindrome de Hiperestimulacion Ovarica.
b.- Gestación Múltiple.
c.- Embarazo Extrauterino.
d.- Aborto Recurrente.

CASO CLINICO
Femenino de 31 años de edad la acude a consulta debido a que presenta amenorrea secundaria, agrega que desea embarazarse, como
antecedentes cuenta con menarca a 21 años con oligomenorrea, se observa con clasificación de Taner grado 3, refiere presencia de
leucorrea frecuente ha recibido tratamiento, a la exploración observa Talla 1.47 mts, Peso 48 kg. Además cuello alado, torax en escudo.

PREGUNTA:
Cuál es la condición más frecuente de origen genético que presenta con los datos observados en este caso.

RESPUESTA:
a.- Sindrome de Kallman.
b.- Sindrome de Turner.
c.- Sindrome de Klinifelter.
c.- Sindrome XXX.

CURSO ENARM CMN SIGLO XXI TEL: 36246001 Pharmed Solutions Institute PÁGINA 276
MANUAL DE TRABAJO DEL CURSO ENARM CMN SIGLO XXI
AMENORREAS. CIENCIAS BASICAS: Ausencia temporal o permanente de menstruación. AMENORREA PRIMARIA: se define como la
ausencia de menstruación después de los 15 años de edad cuando coincide con la falta de la aparición de los caracteres sexuales
secundarios (infantilismo sexual) y por arriba de los 18 años en presencia de desarrollo sexual. La amenorrea primaria frecuentemente
se acompaña de infantilismo sexual y talla baja, lo que hace sospechar en primer término la existencia de una disgenesia gonadal (1ra
causa de amenorreas primarias); además si existen malformaciones somáticas se hace aparente un síndrome de Turner. En otras
ocasiones se puede encontrar el antecedente en la infancia de un tumor hipofisario que fue sometido a cirugía y/o radiación. En forma
excepcional la amenorrea
primaria se encuentra asociada
a una talla normal, desarrollo
mamario, pero ausencia de vello
sexual e infantilismo sexual con
dispareunia; todo ello hace el
diagnóstico de un síndrome de
testículo feminizante ó Sx., de
Morris (enfermedad recesiva
ligada al cromosoma X,
insensibilidad a andrógenos), es
la 3ra causa de las amenorreas
primarias. Agenesia del
conducto de Muller o Sx. de
Rokitansky-Kuster-Hauser;
segunda causa mas frecuente de
amenorrea primaria, hay utero
rudimentario no canalizado o
solo cuernos uterinos, cariotipo
46XX. Finalmente, aunque raro, es que se presente en forma normal el desarrollo de los caracteres sexuales secundarios, pero no la
menarquia y esto hace sospechar una malformación congénita del útero y la vagina o un himen imperforado. La confirmación
diagnóstica utiliza esencialmente estudios citogenéticos y de imagenología, pasando a segundo término los hormonales. Se puede
establecer este diagnóstico de acuerdo al algoritmo de la figura 1. En los casos en que se presenta amenorrea y además ausencia de
caracteres sexuales secundarios, el diagnóstico se establece en base a los resultados de laboratorio y el estudio del cariotipo. La
pubertad retardada es una entidad familiar caracterizada por hipogonadismo-hipogonadotrópico, por lo tanto los antecedentes
familiares detallados ayudan a establecer esta etiología, ya que es difícil de distinguir entre la falla hipotalámicao hipofisaria. La
recomendación en estos casos es la vigilancia estrecha de los cambios fenotípicos. Otra entidad conocida como síndrome de Kallmann,
la cual se asocia a anosmia, también puede causar hipogonadismo-hipogonadotrópico. Los casos de hipogonadismo-
hipergonadotrópico y amenorrea primaria están causados por disgenesia gonadal o falla ovárica prematura. La forma más común de
disgenesia es el síndrome de
Turner que se caracteriza por
la presencia cromosómica 45
X0, pliegue en el cuello,
amplio espacio entre los
pezones y estatura corta. Sin
embargo, se debe recordar
que puede existir mosaicismo
como en todo síndrome
cromosómico. AMENORREA
SECUNDARIA: es la
desaparición de la
menstruación, por más de
tres meses, en una mujer que
haya tenido varios ciclos
previos. La duración de la
amenorrea secundaria es muy
variable y puede ser
permanente. La primera
posibilidad de amenorrea
secundaria es el embarazo
como norma clínica; una vez
descartado lo anterior, las
causas más frecuentes son el
síndrome de ovarios

CURSO ENARM CMN SIGLO XXI TEL: 36246001 Pharmed Solutions Institute PÁGINA 277
MANUAL DE TRABAJO DEL CURSO ENARM CMN SIGLO XXI
poliquísticos (Sx de Stein-Leventhal) y los trastornos hipotalámicos que son diagnósticos por exclusión (eliminando la
hiperprolactinemia debida
coincidiendo o no con un tumor
hipofisario, así como un trastorno
tiroideo). Una prueba que mantiene
su vigencia y utilidad es la
administración de progesterona
para inducir un sangrado menstrual
y cuando esto ocurre se confirma la
presencia de un nivel aceptable de
estrógenos circulantes; en cambio
cuando no hay menstruación la continuación diagnóstica es el ciclo completo de estrógenos y progesterona, si consigue la
menstruación entonces se puede sospechar un defecto severo de producción endógena de estrógenos. Si un ciclo combinado de
estrógenos y progesterona no consigue inducir la menstruación se debe sospechar una ausencia virtual del útero por sinequia uterina
ocasionada por un legrado o una infección postparto o más raro por tuberculosis. En la actualidad aparece más frecuentemente la
menopausia precoz ya sea quirúrgica o idiopática que se puede diagnosticar con la prueba de estrógno-progesterona y si es necesario
con las determinaciones de FSH y estradiol circulantes. Los exámenes de laboratorio de mayor utilidad como una segunda etapa a la
clínica y orientados por ésta son la prolactina (PRL), las gonadotropinas (LH, FSH), TSH (hormona estimulante deltiroides) y de menor
valor estradiol y andrógenos. Los avances en la imagenología son ahora de gran ayuda para conocer la hipófisis y los órganos
intrapélvicos (ovarios,útero, endometrio y anexos). Existen rutas diagnósticas que han permitido la elaboración de algoritmos útiles en
la clínica. Se ha establecido un algoritmo para llegar a este diagnóstico después de haber descartado el embarazo como potencial
diagnóstico, como se muestra en la figura 2. Hipotiroidismo: Cuando existe enfermedad tiroidea se presentan cambios menstruales
antes de presentarse la amenorrea, como en el caso de hipotiroidismo leve, donde se puede presentar disminución o aumento del
sangrado menstrual, pero después del tratamiento del hipotiroidismo se restablece el ciclo menstrual aunque después de varios meses.
Hiperprolactinemia: En las pacientes con elevadas concentraciones de PRL (> 100 ng/mL), galactorrea, cefalea y trastornos visuales se
deben realizar estudios de imagenología para confirmar un tumor de hipófisis, ya que el prolactinoma es la principal causa de trastorno
de la hipófisis anterior. En caso de encontrarse un microadenoma (< 10 mm) se deben tratar con agonistas dopaminérgicos, además de
vigilar y tratar el problema de infertilidad. Los macroadenomas también deben tratarse con agonistas dopaminérgicos o extirparlos
quirúrgicamente a través de resección transesfenoidal. Cuando se ha excluido el adenoma, se debe tener presente la posibilidad de
hiperprolactinemia causada por medicamentos, aunque los medicamentos no logran concentraciones elevadas como un adenoma, es
decir, las concentraciones siempre estarán por debajo de 50 ng/mL, la suspensión del medicamento mejorará este trastorno. Ausencia
de endometrio: La incapacidad uterina para producir endometrio se conoce como síndrome de Asherman, causado por curetaje
excesivo del endometrio durante un procedimiento de limpieza obstétrica o por infección intrauterina, la histerosapingografía,
histeroscopia y el ultrasonido pueden ayudar a establecer el diagnóstico (Figura 3). Otras causas menos frecuentes la constituyen: la
estenosis cervical, la fibrosis obstructiva o los pólipos. Falla ovárica prematura y menopausia precoz: Está caracterizada por
hipogonadismo hipergonadotrópico, causado por depleción de los folículos ováricos que lleva al hipoestrogenismo y finalmente
amenorrea. Esta entidad está asociada a trastornos autoinmunes, por esta razón los exámenes de laboratorio resultan normales, por lo
tanto debe tratarse la enfermedad de fondo (ej. enfermedad de Addison). En el dagnostico podemos tener: Gonadrotrofinas
aumentadas= FSH>40 y LH >25Mui/ml= Trastorno reside en ovarios. Gonadotrofinas disminuidas= FSH<5 y LH <5mUI/ml= Tratsorno del
sistema hipotálamo-hipofisis.

CASO CLINICO
Se trata de femenino de 24 años de edad la cual acude a consulta debido a que anteriormente se encontraba bajo tratamiento con
clomifeno ya que desea embarazarse sin embargo no sabe el diagnostico establecido, tiene un USG donde se observa gónadas con
hipoplasicas además de niveles hormonales gonadales por debajo de la media.

PREGUNTA
Considerando los datos observados, anatómicos y fisiológicos, cual es la indicación más adecuada para favorecer la ovulación y de esta
manera mejorar la posibilidad de embarazo.

RESPUESTA
a.- Debemos primeramente realizar una prueba con bromocriptina.
b.- La groserelina nos favorecería una estimulación gonadal.
c.- El citrato de clomifeno es el mas indicado para estimular hipotalámicamente.
d.- Por protocolo deberá iniciarse manejo con gonadotrofinas.

CASO CLINICO
Se trata de femenino de 24 años acude por presentar alteraciones en su ciclo menstrual, refiere periodos menstruales irregulares que le
duran uno o dos días y en ocasiones solo es manchado, a la exploración física se observa paciente con deficiencia en caracteres
sexuales femeninos que corresponden a menor edad aparente contra la cronológica.

PREGUNTA
Cuál es el patrón hormonal más probable que esperaría encontrar.

RESPUESTA
a.- Aumento de FSH y LH con estrógeno y progesterona alta.

CURSO ENARM CMN SIGLO XXI TEL: 36246001 Pharmed Solutions Institute PÁGINA 278
MANUAL DE TRABAJO DEL CURSO ENARM CMN SIGLO XXI
b.- Aumento de FSH y LH con estrógeno y progesterona baja.
c.- Andrógenos elevados y estrógenos y progesterona normal.
d.- Andrógenos normales con estrógeno y progesterona baja.

CASO CLINICO
Se trata de paciente femenino de 21 años de edad la cual acude por falta de menstruación durante los últimos 6 meses, niega vida
sexual activa, sus antecedentes GO son menarca 11 años, G: 0, a la exploración se encuentra caracteres sexuales normales para edad y
sexo, se observa sobrepeso por IMC, niega flujo, refiere que en los 3 últimos años sus periodos menstruales han sido irregulares, niega
cefalea u otros síntomas.

PREGUNTA
Cuál de los siguientes diagnósticos es el más probable.

RESPUESTA
a.- Microadenoma PRL productor.
b.- Adenoma GH productor.
c.- Microadenoma ACTH productor.
d.- MEN.

ENDOMETRIOSIS. CIENCIAS BASICAS: La endometriosis es una de las enfermedades más enigmáticas que afecta a las mujeres en edad
reproductiva. Esta enfermedad se define por la presencia de tejido endometrial (epitelio glandular y estroma) fuera de la cavidad
uterina, localizándose habitualmente sobre la superficie peritoneal y sobre el ovario. Otras posibles localizaciones de esta enfermedad
son: sistema gastrointestinal, pulmón, sistema genitourinario, pared abdominal.... Puede manifestarse por trastornos funcionales,
relacionados con esterilidad, es un distrofia benigna, es hormonodependiente, la menopausia es una forma de curación fisologica, y el
tratamiento sustitutivo la reactiva. La loscalizaciones mas frecuentes son: ligamentos utero-sacros, fondo de saco posterior, fondo de
saco vesicouterino, ovario (el mas frecuente), tubas uterinas, extragenialtes. SALUD PUBLICA: Aunque la prevalencia exacta de
endometriosis en la población general
es desconocida se estima que entre el
20 y 90% de mujeres con dolor
pélvico y/o infertilidad presentan esta
enfermedad. En mujeres
asintomáticas, la prevalencia se sitúa
entre 3 y 30%. Un 10% en el grupo de
eda de los 30-40 años de edad.
PATOGENIA: En la actualidad existen
cuatro teorías para explicar la
patogénesis de la endometriosis:
metaplasia celómica, restos de células
embrionarias, diseminación linfática y
vascular y trasplante de tejido
endometrial. 1) Metaplasia celómica:
A principios del siglo pasado, fue
considerada la primera teoría que
explicaba la patogenia de la endometriosis.Se sugirió que la endometriosis se originaba en la membrana celómica a través de un
proceso de metaplasia seguido de una inducción metaplásica, lo cual ha sido confirmado. El epitelio celómico, que surge de una capa de
células epiteliales del conducto mulleriano, también se diferencia en epitelio pleural y peritoneal y en células de la superficie de los
ovarios. Este hecho puede explicar las localizaciones inusuales de la endometriosis. Existen otros argumentos que defienden la teoría
de la metaplasia celómica como son la presencia de esta enfemedad en hombres, en mujeres etapa prepuberal, en aquellas que jamas
han menstruado y en adolescentes sin anormalidades del conducto de mullerianos y pocos años después de la menarquia. 2) Teoría de
los restos embrionarios: Esta teoría fue acreditada por Von Recklinghausen, quien se basó en la identificación de restos del conducto
Wolfiano en el ligamento ancho y porciones antero-laterales de la vagina y el cérvix, que podrían dar lugar a la endometriosis. La
presencia de estos restos embrionarios puede explicar la existencia de
endometrio ectópico. Áreas adyacentes a los conductos mullerianos y
duplicaciones del sistema mulleriano, permitirían que células de origen
mulleriano se conviertan en endometrio funcionante. Sin embargo, la
presencia de los mismos no se ha encontrado en la pelvis o en la
cavidad torácica. La endometriosis tiene su mayor incidencia en
mujeres de a partir de los 25 años. 3) Diseminación linfática y vascular:
Sampson fue el primero en sugerir la diseminación linfática y
hematógena de la endometriosis al encontrar tejido endometrial en
venas de pacientes con adenomiosis. Posteriormente Halban confirmó
esta teoría postulando la viabilidad de las células endometriales que
entraban a través de la membrana basal de los vasos sanguíneos y
linfáticos. Estas células endometriales mediante la formación de
microémbolos provocarían el desarrollo de la enfermedad en lugares a
distancia. 4) Teoría de la implantación: Según los hallazgos de

CURSO ENARM CMN SIGLO XXI TEL: 36246001 Pharmed Solutions Institute PÁGINA 279
MANUAL DE TRABAJO DEL CURSO ENARM CMN SIGLO XXI
Sampson el contenido menstrual con fragmentos de endometrio pasa a través de las trompas de Falopio alcanzando la cavidad
peritoneal en lo que conocemos como menstruación retrógrada. Ese endometrio posteriormente se implanta en la superficie peritoneal
y en los órganos del abdomen y de la pelvis, donde en los próximos ciclos menstruales se produce el sangrado menstrual. Esta teoría, se
basa en tres suposiciones: la primera, que hay menstruación retrógrada a través de las trompas; la segunda, que las células
endometriales contenidas en el material de reflujo son viables en la cavidad uterina; la tercera, que dichas células pueden adherirse al
peritoneo, y favorecer los procesos de invasión, implantación y proliferación subsiguientes. La teoría de la implantación fue rechazada
durante muchos años por la suposición de que la menstruación era infrecuente. DIAGNOSTICO: Se hace aparente en los años de la
etapa reproductiva debido a que las hormonas producidas por el ovario estimulan al endometrio. Los síntomas son intensos en el
periodo premenstrual y moderados al término de la menstruación, por lo tanto, el dolor pélvico es el síntoma más frecuente, pero
además puede aparecer dolor de espalda, dispareunia, dolor a la defecación, dolor al cambio de posición y disuria. La mayoría de las
mujeres en quienes se ha diagnosticado endometriosis ha tenido previamente síndrome de vejiga irritable. La asociación de
endometriosis con infertilidad es debido a las adherencias en la cavidad peritoneal, lo cual distorsiona la anatomía pélvica y trastorna la
liberación del óvulo; sin embargo, los trastornos anatómicos no son el único motivo de la infertilidad, ya que se ha descrito que la
endometriosis causa baja reserva ovárica y de la calidad del óvulo. Además, en los estudios de fertilización se ha encontrado baja
calidad del embrión, lo que disminuye la capacidad de implantación y diferenciación celular. Existen pocos estudios clínicos y maniobras
exploratorias que ayuden a diagnosticar de forma cetera la endometriosis. Hay signos que pueden estar ausentes, pero deben estar
incluidos nódulos en la parte posterior de la vagina, dolor al movimiento uterino, útero en retroversión y fijo, y dolor de masa anexial
por endometriomas. Se ha determinado la utilidad de los signos clínicos y de los síntomas en el diagnóstico de la endometriosis en
mujeres que presentan infertilidad. El diagnóstico de la endometriosis se basa en la visualización del endometrio ectópico por
laparoscopia o laparotomía, acompañado de confirmación histológica de macrófagos con hemosiderina, epitelio endometrial y
glándulas o estroma endometrial. El ultrasonido intravaginal ha sido el de mayor uso en el diagnóstico de lesiones retroperitoneales y
uterosacras, pero no tiene la exactitud para identificar lesiones peritoneales o endometriomas pequeños. TRATAMIENTO: Quirúrgico
consiste en la ablación del tejido endometrial ectópico a través de laparoscopia, aumenta la fertilidad en mujeres con endometriosis,
aunque se ha reportado que la exéresis de los quistes endometriósicos otorga un mejor resultado en la disminución del dolor
comparado con la eletrofulguración y drenaje del tejido endometrial ectópico. Por otro lado, la histerectomía acompañada de salpingo-
ooforectomía bilateral se considera el tratamiento definitivo para la endometriosis severa, aunque en estudios retrospectivos se ha
observado que 10 % de las mujeres con histerectomía y salpingectomía bilateral tuvieron recurrencia de los síntomas, mientras que las
mujeres que solo tenían histerectomía presentaron recurrencia de los síntomas en 62 % de los casos. La vertiente actual cuando hay
infertilidad es tratar la endometriosis aunque ésta sea mínima, dbido a que los cambios presentes por este tejido ectópico son a nivel
inmunológico y en los órganos genitales, lo que trastorna todo el proceso de la concepción. El tratamiento médico para el dolor incluye
analgésicos (antiinflamatorios no esteroideos), anticonceptivos orales, danazol, análogos de la hormona liberadora de gonadotropinas
(GnRH = leuprolide, goserelina, triptorelina, nafarelina). Se ha demostrado que el uso de GnRH para el tratamiento del dolor causado
por endometriosis no mejora el efecto que los anticonceptivos o los progestágenos, además, tiene el inconveniente de causar
hipoestrogenismo como efecto secundario, que cesa con la suspensión del tratamiento. Recientemente se han utilizado los inhibidores
de aromatasa con respuesta prometedora para tratar la endometriosis, con lo cual se ha visto efecto sobre el dolor pélvico y la
producción de endometrio ectópico, ya que bloquea la producción en secuencia de ciclooxigenasa-2 (COX2), producción de
prostaglandinas y producción local de estradiol por el endometrio ectópico, el cual es estrógeno-dependiente.

CASO CLINICO
Femenino de 41 años de edad. Entre sus antecedentes mencionó el diagnóstico de tuberculosis intestinal tres años antes, siendo
tratada durante 8 meses con tratamiento específico tetra-asociado, periodos menstruales regulares y un parto por cesárea a los 32
años. Se hospitalizo por cuadro de varios años de evolución, con frecuentes episodios de dolor abdominal difuso, distensión abdominal,
flatulencia, nauseas, vómitos y constipación, mayor durante ciclos menstruales. El cuadro que se intensifico durante el último año,
asociándose esporádicamente deposiciones con estrías de sangre viva y perdida de 4 a 5 Kg de peso. Al ingreso, paciente en regulares
condiciones generales, presión arterial 120/80 mmHg, frecuencia cardiaca 88 latidos por minuto, temperatura 36,6 ºC, el abdomen se
encontraba distendido, timpanizado a la percusión, chapoteo en flanco y fosa iliaca derecha y borborigmos generalizados a la
auscultación. Los laboratorios, rayos X de tórax, abdomen y ecografía abdominal de ingreso fueron normales, exceptuando la sangre
oculta en heces que fue (+). Las revisiones ginecológica y proctológica no encontraron alteraciones.

PREGUNTA
Cuál es el estadio en el que se encuentra la paciente?

RESPUESTA
a.- Estadio I (Mínimo) - 1-5 puntos.
b.- Estadio II (Leve) - 6-15 puntos.
c.- Estadio III (Moderada) - 16-40 puntos.
d.- Estadio IV (Severa) - más 40 puntos.

CASO CLINICO
Una paciente de 47 años que consultó por dolor intenso, de carácter cíclico, de aproximadamente 1 año de evolución, localizado en la
FID. Los dolores comenzaban 3 o 4 días antes de cada menstruación y precisaba de grandes dosis de analgésicos para su control. Como
antecedentes de interés refería una salpingectomía y una cuña ovárica izquierdas por vía laparoscópica, probablemente por quistes
endometriósicos, así como la resección por histeroscopia de un endometrioma uterino 3 años antes. En las exploraciones
complementarias, que incluyeron resonancia nuclear magnética y tránsito gastrointestinal baritado, únicamente destacó un
endometrioma en ovario izquierdo de 17 mm de diámetro, que el servicio de ginecología desestimó para cirugía. Ante el diagnóstico de
dolor crónico recurrente en FID de origen desconocido y dados los antecedentes médico-quirúrgicos de la paciente, se planteó realizar

CURSO ENARM CMN SIGLO XXI TEL: 36246001 Pharmed Solutions Institute PÁGINA 280
MANUAL DE TRABAJO DEL CURSO ENARM CMN SIGLO XXI
una laparoscopia exploradora, en la que se evidenció un quiste seroso ovárico izquierdo y un apéndice vermiforme normal, pero no se
apreciaron tumores ni lesiones intraabdominales, por lo que se optó por llevar a cabo la apendicectomía profiláctica.
PREGUNTA
Cuales son las evidencias diagnosticas de laboratorio y gabinete mas frecuentes.

RESPUESTA
a.- Niveles plasmáticos de CA 125.
b.- Glándulas endometriales.
c.- Estroma endometrial
d.- Macrófagos con inclusiones de hemosiderina

CASO CLINICO
Enferma de 34 años de edad que consulta por clínica de síndrome miccional y sensación de disconfort hipogástrico de varios meses de
evolución con urocultivos realizados en Atención Primaria negativos. En una ocasión tuvo un episodio de hematuria autolimitada y no
tiene antecedentes médicos ni ginecológicos de interés, salvo un parto vaginal no instrumentado. En los estudios analíticos practicados
no se observan hallazgos dignos salvo una discreta anemia y una elevación mínima de la V.S.G. La ecografía abdominal practicada se
informa como compatible con la normalidad, por lo que se decide la práctica de una cistoscopia donde se aprecia una lesión quística en
pared posterior no sugerente de neoformación papilar típica. En la R.T.U. practicada se informa como endometriosis vesical

PREGUNTA
Cuál es la conducta más adecuada para este caso.

RESPUESTA
a.- Danazol a dosis de 600 a 800 MG.
b.- Análogos de GnHR 3.5 MG IM mensual.
c.- Medroxiprogesterona oral.
d.- Anticonceptivos hormonales combinados.

CERVICOVAGINITIS. CIENCIAS BASICAS: El término “cervicovaginitis” se refiere a la inflamación del epitelio escamoso de la vagina y el
cuello uterino. En este cuadro, la mucosa cervical y vaginal responde a la infección con una reacción inflamatoria que se caracteriza por
el daño a las células superficiales. Vaginitis: Corresponde a la enfermedad inflamatoria del cérvix y la vagina cuya etiología son Cándida
albicans y Trichomonas vaginalis. Vaginitis recurrente: Presentación de cuatro o más episodios en un año. SALUD PUBLICA: La
inflamación de la vagina y la presencia de flujo vaginal sustentan el diagnóstico ginecológico más frecuente en mujeres en edad
reproductiva que asisten a clínicas de primer contacto. La vaginosis o vaginitis bacteriana se ha encontrado en el 15-19% de las
pacientes ambulatorias. Durante el embarazo, las cervicovaginitis se asocian en 11.5 % con el parto pre término y en 11.6 % con
ruptura prematura de membranas y como factor predisponente para esterilidad de origen tubárico y cáncer cervicouterino.
PATOGENIA: Dentro de la etiología de las cervicovaginitis, 22.6 % de los casos son producidos por Gardnerella vaginalis, 19.1 % por
Candida spp., 7.8 % por Candida albicans y 1.5 % por tricomonas, estas en conjunto causan el 90% de los casos. Otras causas que
forman el 10%, son: alérgicas (a la proteína del semen), irritativa ( ducha vaginal, jabones, desodorantes, medicamentos tópicos). En
este cuadro, la mucosa cervical y vaginal responde a la infección con una reacción inflamatoria que se caracteriza por el daño a las
células superficiales. Este daño conduce a la descamación y ulceración, que causan una reducción del espesor epitelial debido a la
pérdida de células de la capa superficial y de parte de la capa intermedia (que contienen glucógeno). En las capas más profundas, las
células sufren tumefacción con infiltración de neutrófilos en el espacio intercelular. La superficie del epitelio se halla cubierta por
desechos celulares y secreciones inflamatorias mucopurulentas. Existe congestión del tejido conjuntivo subyacente con dilatación de
los vasos superficiales y papilas estrómicas hipertróficas y dilatadas. VAGINOSIS BACTERIANA (VB): Es la causa más frecuente de
exudado vaginal y de mal olor de vagina. La VB es una alteración de la flora vaginal, en la que la flora bacteriana normal, constituida
por bacilos gram positivos (Lactobacillus spp.), se halla sustituida por cocobacilos gramnegativos (Gardnerella vaginalis) y una flora
variada que comprende diversas especies anaerobias (prevotella, movilluncus, mycoplasma). Su importancia viene dada por su relación
directa con la enfermedad
pélvica inflamatoria,
interfiere en la fertilidad. En
su patogénesis intervienen
sinérgicamente G. vaginalis y
los anaerobios que
producen exudado vaginal y
el mal olor. La causa de esta
disbacteriosis es
desconocida, y se asocia con
la existencia de múltiples
parejas sexuales, duchas
vaginales y pérdida de
Lactobacillus, lo que provoca
una elevación del pH vaginal.
Criterios diagnósticos de
Amsel: 1) Flujo homogéneo, delgado blanco-grisáceo. 2) pH >4.5, 3) prueba de aminas positiva, 4) células “clave” en frotis en seco. Con
3 de 4 criterios son diagnostico; con sensibilidad de 73% y especificidad de 94%. La tinción de Gramm es el gold estándar. El

CURSO ENARM CMN SIGLO XXI TEL: 36246001 Pharmed Solutions Institute PÁGINA 281
MANUAL DE TRABAJO DEL CURSO ENARM CMN SIGLO XXI
tratamiento de las parejas no resulta eficaz para prevenir las recidivas. Tratamiento: Se recomienda en la mujer embarazada, pues
reduce el riesgo de sufrir complicaciones como parto prematuro y endometritis puerperal. En la no embarazada también se
recomienda, pues elimina los síntomas y signos de VB y reduce las complicaciones en caso de aborto o histerectomía. El tratamiento se
realiza con metronidazol en dosis de 500 mg cada 12 horas por vía oral, durante 7 días. Existen tratamientos alternativos por vía vaginal
como clindamicina al 2% en crema vaginal o metronidazol al 0,75% en gel, ambos se administran con un aplicador, antes de acostarse,
durante 7 días. No se recomienda el tratamiento de las parejas. En el embarazo: metronidazol 250mg VO c/8h por 7 días o cindamicina
300mg VO c/12 por 7 días. VULVOVAGINITIS POR CANDIDA: La colonización vaginal por Candida es relativamente frecuente. Muchas
de ellas también presentan colonización en la zona anorrectal. No presenta sin sintomatología. Sin embargo la mitad que presenta
vulvovaginitis candidiásica presenta inflamación vulvar y vaginal, fisuras y existencia de un exudado adherente a la mucosa,
blanquecino y amarillento, con grumos (cottage cheese). El pH vaginal se mantiene en 4,5. En la patogenia de esta infección actúan
como factores predisponentes la diabetes, el embarazo, el uso de contraceptivos orales, la obesidad, el empleo reciente de
antimicrobianos y la utilización de corticoides. La forma de presentación se clasifica en complicada o no complicada. La primera es
recurrente, grave, puede estar producida por otras especies de Candida diferentes a C. albicans y se produce en diabéticas no
controladas, inmunodeprimidas y embarazadas. Tratamiento tópico: Existen numerosos tratamientos tópicos mediante aplicación
intravaginal. Referimos solo algunos: 1. Butoconazol 3% en crema, 5 g durante 3 días. 2. Clotrimazol 100 mg en óvulos vaginales, 2
óvulos durante tres días. 3. Clotrimazol 500 mg en óvulos, una sola aplicación. 4. Miconazol 100 mg un supositorio vaginal al día
durante 7 días. 5. nistatina 100.000 unidades en tableta vaginal, una tableta durante 14 días. Tratamiento oral fluconazol 150 mg, dos
dosis separadas 72 horas. VAGINITIS POR TRICHOMONAS: La infección por T. vaginalis constituye una de las ETS más frecuentes en el
mundo, en la embarazada se ha asociado a parto prematuro y recién nacido de bajo peso. Se identifica en 30-80%de parejas masculinas
de mujeres infectadas, la mayoría se mantienen en estado de portador solo el 10% presenta uretritis no gonocócica. Produce leucorrea
profusa, espumosa, amarillo-verdosa y maloliente, con abundantes polimorfonucleares, pH alcalino y prurito vaginal. En la exploración,
la vagina está inflamada y el cérvix enrojecido y edematoso con aspecto de frambuesa. En el varón, la infección por T. vaginalis es
menos frecuente. La sintomatología que produce es de uretritis y, con frecuencia, la infección es asintomática. El diagnóstico por
microscopia en fresco es un método específico en caso de vaginitis purulenta, pero poco sensible para detectar a las pacientes
asintomáticas. El cultivo es un método específico y sensible, superado por los métodos moleculares, especialmente por la reacción en
cadena de la polimerasa (PCR). La citología, especialmente el Papanicolau, se considera un método inadecuado por su baja sensibilidad
y pobre valor predictivo positiva. Un tratamiento eficaz es el metronidazol que, en dosis única de 2 g o dosis múltiples de 500 mg cada
12 horas durante 7 días, se ha mostrado eficaz en más del 90% de las pacientes, al igual que los nuevos compuestos tinidazol y
ornidazol. Este tratamiento se recomienda también en las mujeres embarazadas, en las que no se ha mostrado teratogenico.
DIAGNOSTICO: Es importante hacer historia clínica minuciosa, interrogar sobre las características de flujo, cambio de pareja sexual
recientemente, número de parejas sexuales, tollas vaginales, duchas vaginales, uso de tampones, tratamientos previos, etc.. Fortis en
seco: Laminilla N.1; aplicar secreción vaginal+2gotas de sol. Salina 0.9%, observar en campo seco, vemos flagelados móviles con
abundantes PMN, caracteristicos de tricomoniasis o bacterias cocoides y celulas “clave” (cel.epiteliales descamativas cubiertas por
bacterias, Gardnerella), características de vaginosis bacteriana. Laminilla N.2; secreción vaginal+ KOH al 10%, al microscopio vemos
levaduras o pseudohifas de candida, características. Laminilla N.3; secreción vagunal + tinción gramm, vemos morfotipos gramm
variables de vaginosis bacteriana.

CASO CLINICO
A 30 años mujer blanca de edad presentó al Centro de Salud, refiriendo una secreción maloliente vaginal, ardorosa o quemante durante
4 meses. La paciente fue diagnosticada por primera vez con tricomoniasis vaginal por su médico de atención primaria y que había
recibido cinco ciclos de metronidazol 500 mg por vía oral dos veces al día, la mayor duración del tratamiento fue de 10 días. Su marido
se trató con 3 días y 7 ciclos de un día de metronidazol. Ella admitió haber tenido otra pareja sexual que no habían recibido tratamiento
con metronidazol. Su último curso de metronidazol fue de 1 mes antes de su presentación. No había antecedentes de otras
enfermedades de transmisión sexual. Ella estaba tomando anticonceptivos orales. En el examen, vulva con eritema, edema y
excoriación. El examen con espéculo, la pared vaginal era eritematosa, edematosa y con secreciones anormales.

PREGUNTA
Cuál es al hallazgo mas frecuente en esta patología?.

RESPUESTA
a.- Desaparición de lactobacilos y bacterias anaerobias.
b.- Células epiteliales con bordes mal definidos.
c.- Abundantes cocobacilos.
d.- Presentación de cuatro o más episodios en un año.

CASO CLINICO
Paciente de sexo femenino, 28 años de edad, sin antecedentes patológicos previos. Consultó por episodios repetidos de vulvovaginitis.
Estos episodios habían comenzado dos años antes, se repetían casimensualmente y aportó estudios microbiológicos previos, tres de los
cuales mostraron pseudohifas y elementos levaduriformes brotantes en el examen microscópico directo, con desarrollo de Candida
spp. Sus ciclos menstruales eran regulares y no tenía antecedentes de embarazos previos. Como método anticonceptivo empleaba un
dispositivo intrauterino. Fumadora y bebedora. No tenía antecedentes de diabetes ni había recibido antibióticos. El examen físico no
acusó anormalidades y ginecológico apreció aumento de la secreción vaginal, congestión de la mucosa y pseudomembranas
blanquecinas en vulva y vagina.

PREGUNTA
Cuál es la conducta más adecuada para este caso.

CURSO ENARM CMN SIGLO XXI TEL: 36246001 Pharmed Solutions Institute PÁGINA 282
MANUAL DE TRABAJO DEL CURSO ENARM CMN SIGLO XXI

RESPUESTA
a.- Metronidazol.
b.- Nistatina.
c.- Itraconazol.
d.- Nitrofurntoina.

CASO CLINICO
Femenino de 42 años de edad paciente presentó una historia de 10 meses de la descarga vaginal persistente por el que había recibido
ciclos intermitentes de eritromicina. Ella se encontró que tienen tricomoniasis y se le prescribió metronidazol 400 mg dos veces al día
durante 7 días. Ella negó cualquier actividad sexual posterior. Su compañero sexual anterior también fue tratado con metronidazol. T.
vaginalis se aisló repetidamente sobre el seguimiento de la cultura sobre las próximas semanas a pesar de nuevos cursos de
metronidazol intravaginal tanto oral como tópica.

PREGUNTA
Cuál es la conducta más adecuada para este caso.

RESPUESTA
a.- Metronidazol 500 mg cada 12 h, vía bucal. 7 días
b.- Metronidazol 500 mg cada 24h vía vaginal. 7 días.
c.- Nistatina, 100 mil unidades cada 24h, Vía vaginal. 14 días
d.- Metronidazol 500 mg cada 24h, vía vaginal, 7 días.

CASO CLINICO
Una mujer de 28 años refiere que durante los últimos tres años, la había estado tratando sin éxito una recurrente descarga vaginal
desagradable. En su primera visita a la clínica, Trichomonas vaginalis se encontró en una preparación húmeda de las secreciones
vaginales y se indico metronidazol 200 mg tres veces al día durante una semana fue recetado. Durante los próximos seis meses que
tenía tricomoniasis sintomática recurrente.

PREGUNTA
Cuál es la conducta más adecuada para este caso.

RESPUESTA
a.- Baño diario.
b.- Enjuague anogenital al medio día.
c.- Uso de toallas sanitarias solo durante la menstruación.
d.- Evitar el uso diario de pantiprotectores y tampones.

CASO CLINICO
Se trata de paciente femenino de 23 años de edad la cual acude a consulta debido a que presenta desde hace tres semanas secreción
vaginal blanquecina de mal olor, refiere relación sexual con nuevo compañero sin protección, cuenta con antecedente de uso de
anticonceptivo orales, menarca 10 años, gesta 1, para 0, abortos 1, tabaquismo y alcoholismo positivos desde los 17 años, a la
exploración se observa secreción blanca leve que se encuentra recubriendo la vagina de forma uniforme con olor a “pescado”.

PREGUNTA
Cual es tratamiento más adecuado para este caso.

RESPUESTA
a.- Metronidazol 2 g VO dosis única.
b.- Metronidazol 500 mg VO cada 12 hrs x 7 dias.
c.- Fluconazol 100 mg VO dosis única.
d.- Clindamicina 300 mg VO cada 12 hrs por 5 dias.

ENFERMEDADES DE TRANSMISION SEXUAL (ETS). CIENCIAS BASICAS: Son conocidas también como enfermedades venéreas (en honor
a la diosa romana Venus que es el equivalente a la diosa griega del amor y la belleza Afrodita) y su definición obedece al modo de
transmisión de las mismas ya que se adquieren mediante el contacto sexual: vaginal, anal u oral. Todas las conductas sexuales que
involucran contacto con otra persona o con los fluídos de la misma se considera como factor de riesgo para la adquisición de ETS.
SALUD PUBLICA: En México y también a nivel mundial, las ETS han representado un problema de salud pública; a nivel nacional son una
de las diez primeras causas de morbilidad, con un promedio de 220,000 casos anuales. La frecuencia de ETS en la población general
fluctúa entre 0.1 y 0.5%; en cambio, para la población considerada como de alto riesgo, se calcula una frecuencia que va de 10 a 20%.
La población expuesta se compone en su mayor parte de jóvenes, el 34% corresponde al periodo entre la 2da y 3ra década de la vida,
seguido por el grupo de 25 a 44 años que integran dos décadas en un 41%. La relación hombre mujer fue de 2,8:1 GONORREA O
BLENORRAGIA (Neisseria gonorrhoeae): El agente causal es Neisseria gonorrhoeae (el cual tiene tropismo por las mucosas), el humano
es el único huésped conocido. Puede causar uretritis, cervicitis, epididimitis, faringitis, proctitis, EPI y en los casos graves diseminación
sistémica. En México se reporta 1.0 por cada 100´000 hab. La transmisión más frecuente es mediante el acto sexual vaginal, anal u oral
y se hace a través del contacto de la superficie mucosa de un individuo infectado. También puede ser contagiada mediante fómites a

CURSO ENARM CMN SIGLO XXI TEL: 36246001 Pharmed Solutions Institute PÁGINA 283
MANUAL DE TRABAJO DEL CURSO ENARM CMN SIGLO XXI
otras áreas, por ejemplo a un ojo con la mano o con otra parte del cuerpo humedecida con líquidos infectados. La transmisión vertical
ocurre cuando la madre infecta al recién nacido mediante el paso por el canal del parto, el producto adquiere la infección
manifestándose como conjuntivitis, vulvovaginitis e inclusive neumonía, por lo tanto, en estos casos estaría indicado el parto por
cesárea. Patogenia: Este microorganismo se adhiere a la superficie de los epitelios uretral, endocervical, vaginal, rectal, faríngeo e
incluso a los espermatozoides humanos y a las células ciliadas de las trompas de Falopio, pueden expresar simultáneamente varios
tipos de cadenas de LPS antigénicamente diferentes, que median en gran parte su toxicidad. Las manifestaciones aparecen de 2-5 días
posterior al contacto de sexual, desde la infección asintomática, síntomas locales en mucosas hasta la diseminación sistémica. Se inician
con leve molestia uretral, seguida de disuria de grado variable y a veces urgencia miccional. Hay eritema y edema del meato urinario y
salida de material purulento por él, sin embargo la infección puede ser asintomática en 5-7% de los afectados. Puede complicarse con
epididimitis, prostatitis, tromboflebitis, absceso periuretral o fístulas, vesiculitis seminal, estenosis uretral y esterilidad. En mujeres los
síntomas suelen aparecer entre 7-21 días después de la infección, disuria, descarga vaginal y a veces fiebre. En los hombres, lo más
común es que el paciente presente una uretritis anterior con disuria (que es más frecuente durante la mañana) y una descarga uretral
purulenta característica. La complicación local más común es secundaria al ascenso de los microorganismos y se presenta como una
salpingitis aguda o EPI en el 10%-20% de los casos. El diagnóstico de gonorrea es fundamentalmente clínico y se comprueba con la
identificación de N. gonorrhoeae mediante tinciones de frotis de las mucosas infectadas. El diagnóstico de uretritis se hace con el
hallazgo de 4 o más PMN por campo por 1000X; el de uretritis gonocócica en tanto es altamente sugerente por la presencia al gram de
cocobacilos gram negativos en granos de café. Cuando el examen directo es negativo y la sospecha es alta, debe realizarse el cultivo en
medio de Thayer Martin, medio enriquecido y con antimicrobianos que inhiben el crecimiento de otros microorganismos. El
tratamiento recomendado para gonorrea genital, rectal y faríngea no complicada en adultos y adolescentes es con cefixima vía oral o
ceftriaxona intramuscular. Otros regimenes recomendados son la ofloxacina, ciprofloxacina y levofloxacina. Si se comprueba la
infección conjunta con C. trachomatis el paciente puede ser tratado con azitromicina o doxiciclina. Es importante mencionar que debe
darse tratamiento a las parejas sexuales de los pacientes, y recordarles que no pueden tener relaciones sexuales hasta que se haya
completado el tratamiento y las parejas sexuales hayan sido tratadas. URETRITIS NO GONOCÓCICA (UNG) (Chlamydia Tracomatis y
Mycoplasmas genitales): Estos son los patógenos más frecuentes. Corresponden al 40 a 50% de las uretritis. En un 30% de los casos no
se encuentra el agente etiológico. La C. trachomatis se ha asociado a uretritis, proctitis y conjuntivitis en ambos sexos, con epididimitis
en el hombre, y con cervicitis mucopurulenta, salpingitis aguda, bartolinitis y el Sd. de Fitz Hugh Curtis en la mujer. También puede
asociarse a artritis séptica. La coinfección con N. Gonorrhoeae es frecuente, y en general, si bien puede comprometer los mismos
órganos, las manifestaciones clínicas producidas por C. trachomatis suelen ser menos sintomáticas y muchas veces asintomáticas. La
infección asintomática en mujeres puede provocar daño tubario e infertilidad. Los mycoplasmas que con mayor frecuencia infectan el
aparato genital son el Mycoplasma Genitalium y el Ureaplasma Urealyticum (UU). Ambos pueden adquirirse como colonización al
momento del parto, sin embargo menos del 10% de pre-púberes están colonizados con Ureaplasmas. La tasa de UNG se ha mantenido
relativamente estable, con un valor actual de 5.1 casos por 100000 habitantes. La edad pico de la infección por C. trachomatis es la
adolescencia tardía y primeros años de la tercera década. El uso de anticonceptivos orales y la ectopía cervical son factores de riesgo
para infección por C. trachomatis, es un patógeno intracelular obligado de tamaño pequeño. Las manifestaciones clínicas de la infección
derivan de la toxicidad directa por estallido celular y de la respuesta inmune, siendo ésta última la responsable del daño tubario
después de infecciones repetidas. El comienzo de los síntomas suele ser insidioso, por lo que el paciente consulta con frecuencia
enforma tardía. El período de incubación va de 2 a 3 semanas. Clásicamente se describe una secreción uretral escasa de tipo muco-
serosa, fundamentalmente matinal. Hay leve disuria y prurito y escasa congestión del meato uretral. Puede ir desde un cuadro
asintomático a un cuadro similar a la uretritis gonocócica. El cérvix en fresa es característico de chalamydia. Diagnóstico: Anamnesis y
examen físico concordante, frotis uretral para gram y cultivo de N. Gonorrhoeae: la UNG se demuestra por: ausencia de diplococos
gram negativo intracelulares, al menos 4 PMN por campo de inmersión 1000X, cultivos de gonococo negativos, exámenes para C.
Trachomatis y UU específicos. El estudio de C. Trachomatis se realiza principalmente mediante inmunoflurescencia directa con un 90%
de sensibilidad, y no necesita viabilidad del germen. Actualmente se cuenta también con PCR, con un 95% de sensibilidad. El cultivo es
caro y complejo por lo que no se realiza en clínica. Tratamiento: Azitromicina 1 g VO DU o Tetraciclina 500 mg VO c/6 h por 7 días o
Doxiciclina 100 mg VO c/12 h por 7 días. Mujeres embarazadas o lactancia: Azitromicina 1 g VO DU o Eritromicina 500 mg VO c/6 h por
7 días Recurrencia y persistencia: Metronidazol 2 g VO DU o 500 mg c/12 h por 7 días + eritromicina 500 mg VO c/6 h por 7 días. SIFILIS
(Treponema pallidum): La sífilis se conoce también como lúes, mal gálico, es una enfermedad crónica adquirida por contacto sexual
(aunque también debe tenerse en cuenta la infección prenatal), caracterizada por una variedad de manifestaciones clínicas ya que
puede afectar prácticamente todas las estructuras del organismo; es una enfermedad intermitente que tiene períodos de actividad
(fase primaria, secundaria y terciaria) y periodos prolongados de latencia. El Treponema pallidum es una espiroqueta, incapaz de
sobrevivir fuera del huésped. La transmisión ocurre mediante la penetración de la espiroqueta en las membranas mucosas o abrasiones
en las superficies epiteliales. El tiempo de incubación desde la exposición a la aparición de las lesiones primarias es en promedio de 3
semanas con un rango de 10 a 90 días. La lesión primaria es una pápula asintomática que al paso del tiempo sufre necrosis superficial
convirtiéndose en una ulceración bien circunscrita denominada chancro, ésta siempre aparecerá en el sitio de contacto primario o de
inoculación, y puede acompañarse de linfadenopatía inguinal. Este período conocido también como primario es autorresolutivo, es
decir, el chancro desaparece en 3 a 12 semanas en forma espontánea Posteriormente hay diseminación linfática y sanguínea del
microorganismo y la enfermedad pasa a una etapa latente asintomática. Dos a tres meses después aparece la expresión de esta
septicemia, el secundarismo, que dura varias semanas y se manifiesta por un amplio espectro de lesiones en piel y síntomas generales
con linfadenopatía generalizada (Figura 2 y 3). En este momento el enfermo es altamente contagioso aún al tacto de las lesiones ya que
estas contienen una gran cantidad de espiroquetas. Es también en este periodo que las mujeres embarazadas y contagiadas pueden
infectar al feto vía transplacentaria. A continuación se presenta un período de latencia que es el tiempo entre la resolución de las
lesiones clínicas y la aparición de las manifestaciones tardías de la enfermedad, y puede durar años inclusive. Aproximadamente el 70%
de los individuos que no han recibido tratamiento permanecerán en este estadio por el resto de sus vidas. La sífilis latente, se divide en
temprana (cuando dura menos de un año) o tardía (duración de mas de un año) y típicamente encontramos serología positiva con
anticuerpos específicos al T. pallidum y el paciente se encuentra asintomático. Durante la etapa terciaria encontramos muy pocos
microorganismos y una respuesta inmune celular aumentada al microorganismo. La espiroqueta en este periodo puede invadir el

CURSO ENARM CMN SIGLO XXI TEL: 36246001 Pharmed Solutions Institute PÁGINA 284
MANUAL DE TRABAJO DEL CURSO ENARM CMN SIGLO XXI
sistema cardiovascular (80%-85%), sistema nervioso central (5%-10%), piel y otros órganos; debido a la respuesta de hipersensibilidad
retardada se producen áreas de inflamación local y formación de gomas en el tejido afectado. La penicilina continúa siendo el
tratamiento de elección para todos los estadios de sífilis. En sífilis temprana (primaria, secundaria y latente temprana adquirida de
menos de 1 año de duración) en pacientes inmunocompetentes se recomienda una aplicación única de 2.4 millones de unidades de
penicilina benzatínica intramuscular o 600´000 unidades de penicilina procaínica intramuscular durante 10-14 días. En personas
alérgicas a la penicilina se puede utilizar doxiciclina 200mg diariamente por 14 días; tetraciclina 500mg cuatro veces al día durante 14
días; eritromicina 500mg cuatro veces al día durante 14 días o ceftriaxona intramuscular por 10 días. En sífilis tardía latente (adquirida
de más de un año de duración o cardiovascular) se recomiendan 2.4 millones de unidades de penicilina intramuscular en tres dosis o
penicilina procaínica 600´000 intramuscular por 17 a 21 días. Las alternativas son doxiciclina 200mg diariamente por 21-28 días;
tetraciclina 500mg cuatro veces al día por 28 días y eritromicina 500mg cuatro veces al día durante 28 días. En neurosífilis el esquema
terapéutico es de 24 millones de unidades de penicilina G acuosa intravenosa por día dividida en 6 dosis por 10 a 14 días o penicilina
procaínica 2.4 millones de unidades intramuscular diariamente junto a probenecid 500mg cuatro veces al día durante 10-14 días. El
régimen alternativo es doxiciclina 200mg diariamente por 28-30 días. LINFOGRANULOMA VENÉREO (Chlamydia trachomatis): También
conocido como enfermedad de Durand-Nicolas-Favre o como linfogranuloma inguinal es una enfermedad causada por Chlamydia
trachomatis. En México es poco frecuente con una tasa de incidencia de 0.2% por cada 100´000, sin embargo en algunos países de
África y Asia se considera una enfermedad endémica. La infección ocurre después del contacto directo con la piel o las membranas
mucosas de la pareja sexual infectada. El microorganismo (C. trachomatis) no penetra la piel intacta, se necesita que exista una solución
de continuidad para que pueda hacerlo; viaja por los vasos linfáticos hasta los ganglios donde se replica dentro de los macrófagos e
inicia la respuesta inflamatoria. A pesar que la mayoría de los casos son predominantemente por contacto sexual, también hay casos de
infección por accidentes de laboratorio, fomites o contacto directo. El linfogranuloma venéreo ocurre en tres etapas. En la primera, que
puede ser asintomática y pasar desapercibida, aparece una pápula no dolorosa en el sitio de inoculación que rápidamente se convierte
en pústula y se ulcera (chancro de inoculación) con resolución espontánea de la misma en una semana aproximadamente. La segunda
etapa se presenta de 2 a 6 semanas después de la lesión primaria y consiste en linfadenopatía inguinal. Finalmente, la tercera, que es
mas frecuente en mujeres y aparece años después de la infección, se manifiesta como proctocolitis y otros padecimientos rectales. El
diagnóstico se basa principalmente en las características clínicas del paciente, sin embargo es necesario realizar estudios paraclínicos
para confirmar el agente etiológico. La aspiración del material purulento de los bubones es la mejor muestra que se puede obtener
para realizar la siembra de C. trachomatis que generalmente crece en medio de cultivo con cerebro de ratón El tratamiento de elección
recomendado es con azitromicina 1g en dosis única. Otra opción es doxiciclina dos veces al día vía oral durante 7 días o eritromicina
500mg cuatro veces al día vía oral durante 7 días. Finalmente se pueden indicar quinolonas del tipo de ofloxacino 300mg vía oral
durante 7 días o levofloxacino 500mg vía oral diariamente por 7 días. El drenaje de los bubones puede ayudar en la disminución de la
sintomatología y prevenir la formación de úlceras. CHANCROIDE (Haemophilus ducreyi): Es conocido también como chancro blando
o ulcus molle, se caracteriza por úlceras genitales dolorosas e invasión regional a los ganglios linfáticos con formación de bubones sin
manifestaciones sistémicas. La característica de estas úlceras es que son blandas (de ahí su nombre) con fondo sucio y rápidamente
necrosantes. Las lesiones se pueden autoinocular a otras áreas del cuerpo muy fácilmente. El agente causal es Haemophilus ducreyi,
una bacteria Gram negativa anaerobia facultativa. En México, en el 2006 se reportó una tasa de incidencia de 0.5% por cada 100´000
habitantes. El periodo de incubación es entre 3 y 10 días. Se inicia con la aparición de una pápula rodeada de eritema la cual se
transforma con rapidez en pústula y deja una ulceración muy dolorosa, bien limitada y no indurada que tiene aspecto de tejido de
granulación con base necrótica en la mayoría de los casos y un exudado de aspecto sucio. Se localiza principalmente en genitales y
región perianal. En hombres se puede encontrar en prepucio, en el surco coronal o en el frenillo, y no es raro que exista un edema
importante del prepucio. En mujeres, la localización más frecuente es el introito vaginal, aunque también se encuentra en el cérvix o el
área perianal. En una frecuencia menor se encuentra en localización extragenital. Puede ocurrir linfadenopatía dolorosa regional hasta
en el 50% de los casos. El regimen antibiótico recomendado actual para el tratamiento es con azitromicina 1gr vía oral en dosis única;
otra opción es ceftriaxona 250mg vía intramuscular en dosis única, se puede utilizar también ciprofloxacino 500mg dos veces al día
durante 3 días o bien eritromicina 500mg tres veces al día durante 3 días. DONOVANOSIS: Esta ETS también conocida como granuloma
inguinal, se caracteriza por la presencia de úlceras genitales que tienden a sangrar con facilidad. El agente etiológico
es Calymmatobacterium granulomatis, una bacteria pleomórfica gramnegativa; sin embargo, y dada la similitud filogénetica que tiene
con el género Klebsiella, se ha propuesto el cambio de nombre a Klebsiella granulomatis. En México es muy rara, y los casos que se
logran detectar son en personas que han viajado a zonas endémicas. El modo de transmisión es primariamente sexual, aunque la
infección también puede adquirirse mediante fómites, vía oral-fecal por alimentos contaminados o a través del canal del parto. El
periodo de incubación varía dependiendo de la fuente que se cite, se reporta desde 1 día hasta 1 año. La lesión inicial es una pápula
firme, usualmente asintomática, que se expande lentamente en semanas o meses; debido a que contiene muchos elementos
vasculares, sangra con traumatismos mínimos y se ulcera. Una vez que se ha formado la úlcera, inicia la destrucción del tejido
adyacente con exudado maloliente y presencia de tejido necrótico. La confirmación diagnóstica se logra mediante la identificación de
cuerpos de Donovan (que son colonias de C. granulomatis) dentro de los macrófagos en un frotis de la úlcera teñido con Giemsa,
Wright o en biopsia de las lesiones. El tratamiento antibiótico recomendado es a base de doxiciclina 100mg diariamente vía oral por lo
menos durante 3 semanas o hasta que las lesiones hayan cicatrizado por completo. Como alternativa se puede emplear azitromicina
1gr vía oral una vez a la semana durante 3 semanas. Así también se puede utilizar ciprofloxacino 750mg dos veces al día durante por lo
menos 3 semanas. INFECCIÓN POR VIRUS DEL HERPES SIMPLE: El virus del herpes simple es un microorganismo patógeno que causa
infección orolabial (VHS-1) y genital (VHS-2) caracterizadas por erupciones vesiculares primarias y recurrentes. Estos dos tipos: el virus
del herpes simple tipo 1 y 2, que aunque están estrechamente relacionados en cuanto a morfología y patogenicidad, difieren
epidemiológicamente. Las infecciones por el VHS-1 se presentan con gran variabilidad y la regla es que se observen hasta en el 80% de
los casos en forma asintomática; se desconoce el número exacto de personas infectadas ya que se considera que más del 95% de la
población mundial tiene anticuerpos contra este virus que contrajo en el 1er año de vida. En México se desconoce la prevalencia de la
infección por el VHS tipo 2 que es considerado como ETS; sin embargo se sabe que en el 2006 hubo una tasa de incidencia de herpes
genital de 1.4% por cada 100´000 habitantes. El espectro clínico del herpes simple genital va desde la primo- infección primaria que
raramente se observa para posteriormente darnos los episodios de recurrencia que son variables en frecuencia e intensidad. En el sitio

CURSO ENARM CMN SIGLO XXI TEL: 36246001 Pharmed Solutions Institute PÁGINA 285
MANUAL DE TRABAJO DEL CURSO ENARM CMN SIGLO XXI
de entrada del virus, que generalmente son los genitales externos aparecen vesículas que se agrupan sobre una base eritematosa, éstas
fácilmente progresan a pústulas y úlceras. No es infrecuente encontrar durante la primoinfección una balanitis erosiva, vulvitits o
vaginitis. En las mujeres las lesiones también pueden encontrarse en cérvix, nalgas y perineo. Se requiere de un diagnóstico certero que
es fundamentalmente clínico, recordando que cada episodio es autorresolutivo en 3 a 7 días. El estándar de oro para el diagnóstico es
el cultivo del virus, sin embargo es una técnica difícil de lograr y muy costosa por lo que se reserva solo para casos especiales. Al ser una
ETS recidivante pero autorresolutiva a corto plazo, el tratamiento es generalmente sintomático con analgésicos, aplicación de
compresas frías, y medidas adecuadas de higiene. Debe hacerse hincapié en explicar amplia y detalladamente la evolución de la
infección, es decir que habrá periodos en que el paciente se encuentre asintomático y sin lesiones y otros en los que aparecerán los
brotes que tendrán la duración ya mencionada y puede acompañarse de ardor discreto. El empleo de aciclovir y otros antivirales
similares ayudarán a reducir la severidad y la duración de las recurrencias solamente. No existe hasta la actualidad ningún fármaco o
vacuna curativos. Debemos hacer hincapié en que los antivirales por vía tópica son totalmente ineficaces.

CASO CLINICO
Es ingresada paciente femenino de 21 años que se encuentra en condición de calle, fue traída por una patrulla que refiere se
encontraba solicitando ayuda, la paciente se encuentra con alteración del sensorio solo dice estar embarazada y que tiene dolor tipo
contracciones, no sabe cuantas semanas de embarazo tiene a la exploración se observa paciente con estado de desnutrición moderada,
alteración del estado de alerta, con actitud alucinada, huele a tiner, presenta además escoriaciones e irritación peribucal, fondo uterino
de 25 cm, contracciones irregulares, una contracción cada 10 minutos aproximadamente, presenta lesiones en labios mayores y
linfadenopatias en región inguinal y axilar.

PREGUNTA
Cuales son las manifestaciones mas importantes para el diagnostico clínico de esta patología.

RESPUESTA
a.- Vesiculas bucal o/y genital.
b.- Ulcera genital limpia, única y elevada.
c.- Ulcera purulente, destructiva y signo de Follman.
d.- Ulcera limpia, vesículas, disuria y prurito.

CASO CLINICO
A primigesta de 24 años de edad con amenorrea secundaria sin determinar fecha y dolor abdominal y lumbar que irradia en su muslo
derecho. Durante los tres meses anteriores, el paciente descrito sangrado vaginal irregular y flujo vaginal acuoso. IVSA a los 14 años de
edad y se observó un mínimo de 15 parejas sexuales anteriores. Resultados positivos de examen toxicológico de drogas ilegales.
Durante los últimos 6 años, los resultados del cultivo y serología documentó cuatro infecciones previas por separado con Chlamydia
trachomatis, Neisseria gonorrhoeae, herpes simplex virus 2 y Treponema pallidum, respectivamente. El paciente informó que completó
el tratamiento para las cuatro infecciones.

PREGUNTA
Cuál es el padecimiento infeccioso que más probable presenta.

RESPUESTA
a.- Amoxicilina 500 mg + probenecid 500 mg diario.
b.- Doxiciclina 2 mg diario.
c.- Eritromicina 500 mg diario.
d.- Ceftriaxona 1 g diario.

CASO CLINICO
Acude a consulta paciente femenino de 24 años de edad, refiere que inicia padecimiento hace 2 semanas aproximadamente
caracterizado por flujo amarillento, en cantidad moderada, agrega que su esposo tiene secreción amarillenta proveniente del meato
urinario la cual es ardorosa.

PREGUNTA
Cual es la conducta a seguir más apropiada para el manejo de incapacidad y referencia.

RESPUESTA
a.- Incapacidad y referencia a segundo nivel.
b.- 7 dias de incapacidad y referencia a segundo nivel.
c.- Iniciar el tratamiento e incapacidad de 24 hrs.
d.- Incapacidad hasta resolución del padecimiento.

CURSO ENARM CMN SIGLO XXI TEL: 36246001 Pharmed Solutions Institute PÁGINA 286
MANUAL DE TRABAJO DEL CURSO ENARM CMN SIGLO XXI
ENFERMEDAD PELVICA INFLAMATORIA (EPI). CIENCIAS BASICAS: Comprende las alteraciones inflamatorias e infecciosas que afectan
los órganos genitales situados en la pelvis menor. Incluye combinaciones de cervicitis, endometritis (utero), salpingitis (salpinges),
anexitis, parametritis (ligamentos de soporte) y pelviperitonitis, así como abscesos tubo ováricos. SALUD PUBLICA: Es más frecuente y
grave en mujeres de 15-25 años. 1 de cada 4 EPI experimentan secualeas. Su importancia radica no sólo en la morbilidad aguda, sino en
su capacidad de producir secuelas como esterilidad, gestación ectópica, recidivas y dolor abdominal crónico. Tasa de incidencia anual
de 17.2 por mil mujeres entre 15-44 años. Afecta cada año del 1-2% de mujeres jóvenes sexualmente activas. PATOGENIA: La EPI es la
complicación más frecuente de las enfermedades de transmisión sexual bacterianas. Suele ser polimicrobiana. Los patógenos más
frecuentes son: Neisseria Gonorrhoeae (5-36%) y Chlamydia Trachomatis (17-70%), Aerobios (Estreptococos del grupo B, Escherichia
coli 30%, Gardenella vaginalis, Mycoplasma hominis 37-75%), anaerobios (Peptoestreptococos, Bacteroides fragilis 60%), y un 5% de
los casos patógenos respiratorios (Haemophilus influenzae 5%, Streptococcus pneumoniae, Streptococcus pyogenes). Las bacterias
pueden acceder a la porción superior del aparato genital por diseminación de órganos adyacentes infectados (apendicitis, diverticulitis),
por diseminación hematógena de focos distantes (tuberculosis) y fundamentalmente por diseminación ascendente transuterina a
partir del tracto genital inferior. Como resultado de esta infección ascendente que alcanza las trompas y de la reacción inflamatoria que
se produce, van a ocurrir dentro de la trompa fenómenos de vasodilatación, trasudación de plasma y destrucción del endosálpinx con la
consiguiente producción de un exudado purulento. En los primeros estadíos de la enfermedad, la luz de la trompa permanece abierta
con lo cual se permite que este exudado salga por la fimbria hacia la cavidad pélvica, lo que produce peritonitis pélvica. Como resultado
de esta inflamación peritoneal, las estructuras adyacentes como ovarios, ligamentos anchos, omento, intestino delgado, sigmoide y
ciego se comprometen en el proceso inflamatorio. La tensión de oxígeno en estos tejidos inflamados y necróticos disminuye lo que
favorece el crecimiento de flora anaerobia. La destrucción del tejido con la posterior degradación lleva a la formación de abscesos. El
flujo menstrual puede facilitar la inserción del tracto superior debido a la pérdida del revestimiento endometrial. Los factores de riesgo
principales son: Edad inferior a 25 años, múltiples compañeros sexuales, ETS, no utilización de métodos de barrera, historia previa de
EIP, historia de vaginosis, cervicitis, dispositivos intrauterinos, solo tiene relación con la EPI en los 3 meses posteriores a la inserción por
la manipulación, abortos, instrumentación uterina, cirugía cervical, término de embarazo. CLASIFICACIÓN: Se basa fundamentalmente
en el grado evolutivo de la enfermedad, en la sintomatología y en los datos recogidos en la exploración. Podemos diferenciar cuatro
estadios (Monif): Estadio I (leve): Salpingitis aguda sin pelviperitonitis. Estadio II (moderada): Salpingitis aguda con pelviperitonitis.
Estadio III (severa): Salpingitis con formación de abscesos tuboováricos. Estadio IV (muy severa): Rotura de absceso y choque septico.
Clasificacion laparoscópica: Leve; eritema, edema, las trompas se mueven libremente, no hay exudado purulento. Moderada; eritema,
edema mas marcado, material purulento evidente. No hay movimiento libre de las trompas. La fimbria puede no ser evidente. Severa;
presencia de piosalpinx y/o absceso. DIAGNOSTICO: puede cursar con los siguientes síntomas: dolor abdominal bajo (95%), y su
intensidad varía de totalmente ausente (EIP silente) a muy intenso en cuadros con un componente peritoneal importante. Aumento del
flujo vaginal, flujo de características anormales (74%). Sangrado anormal (intermestrual, poscoital) (45%). Síntomas urinarios (35%),
como disuria y polaquiuria. Síntomas digestivos (14%), como nauseas, vómitos, diarrea,... Es posible la ausencia de síntomas. Y en ella
podemos encontrar estos signos: dolor a la movilización del cuello, dolor anexial en la exploración vaginal bimanual (99%). En el
examen con espéculo observamos cervicitis y descarga endocervical purulenta (74%). Fiebre (> 38º C) (< 47%). Masa pélvica: sugiere
absceso tuboovárico (ATO). Signos de peritonitis. Las pacientes con infección por Chlamydia pueden cursar con salpingitis subclínica o
subaguda, con secuelas de adherencias e infertilidad. El diagnóstico clínico es con frecuencia dificultoso por la poca especificidad y
sensibilidad de la historia clínica y los estuios de laboratorio, sin embargo el retraso en el diagnóstico y tratamiento puede producir
secuelas importantes. Ante la sospecha de EPI
debemos realizar: Hemograma y bioquímica
general, con determinación de VSG y PCR, test de
embarazo en orina o βHCG sérica., tomas
vaginales y endocervicales para detección de
gonococo y clamidias. Ecografía, sobre todo en su
forma transvaginal, ya que se obtendrá
información sobre la existencia o no de abscesos
tuboováricos. La laparoscopia es el único método
fiable para el diagnóstico, que permite una visión
directa y la toma de cultivos. Se reserva en casos
seleccionados (diagnóstico dudoso o fracaso del
tratamiento). Biopsia endometrial con cánula de
aspiración para cultivo microbiológico y
diagnóstico anatomopatológico. Serología
completa. El diagnóstico clínico se basa en los
criterios de Hager modificados por la SEGO. Se
requiere la presencia de todos los criterios mayores y de al menos uno menor. CRITERIOS MAYORES: Dolor en abdomen inferior. Dolor
a la movilización del cervix. Dolor anexial a la exploración abdominal. Historia de actividad sexual en los últimos meses. Ecografía no
sugestiva de otra patología. CRITERIOS MENORES: Temperatura > 38ºC. Leucocitosis >10500 /µl. VSG elevada. GRAM de exudado
intracervical con diplococos intracelulares (gonococo), cultivo positivo para N.Gonorrhoeae o C.Trachomatis. SINDROME FITZ-HUGH-
CURTIS: Es una perihepatitis, se asocia con EPI, principalmente por Chlamydia, hay desarrollo de adherencias y fibrosis perihepaticas,
dolor agudo en cuadrante superior derecho, no hay alteración de enzimas hepáticas. TRATAMIENTO: Ante la sospecha de EPI se debe
instaurar tratamiento antimicrobiano empírico, aun en ausencia de causa confoimada si hay: dolor a la palpación en abdomen inferior,
en anexos o con el movimiento del cuello uterino. Debe realizarse precozmente para prevenir las secuelas que pueden producirse
incluso en casos de infección leve. En las pacientes con estadio I sin criterios de ingreso hospitalario se prescribirá tratamiento
ambulatorio con: Régimen A: Cefotaxima 2g IM + 1g de probenecid VO DU (tratamiento de primera elección) ó Ceftriaxona 250mg IM +
doxiciclina 100mg VO c/12h por 14 dias (mismo tratamiento para la pareja sexual) Régimen B: Ofloxacina 400mg VO c/12 hrs por 14
dias + clindamicina 450mg VO c/6h ó metronidazol 500mgs VO c/12h por 14 dias. Las pacientes que no respondan al tratamiento

CURSO ENARM CMN SIGLO XXI TEL: 36246001 Pharmed Solutions Institute PÁGINA 287
MANUAL DE TRABAJO DEL CURSO ENARM CMN SIGLO XXI
antibiótico ambulatorio en 48 h. deben ser hospitalizadas para confirmar el diagnóstico y realizar terapia parenteral. Tratamiento
hospitalario: Régimen A: Cefoxitina 2g IV c/6h ó cefotetan 2g IV c/12h + doxiciclina 100mg IV o VO c/12h. Regimen B: Clindamicna
900mg IV c/8h + gentamicina 2mg/kg (1.5mg/kg c/8h) ó doxiciclina 100mg VO c/12h óclindamicina 450mg VOc/6h por 14 dias. Regimen
C: Aztreonam 2-4g/dia IV + clindamicina 600mg IV c/6h. Este régimen debe administrarse hasta 48 horas después de que la paciente
muestra mejoría clínica evidente. En caso de presencia de DIU es preciso la extracción y cultivo del mismo. Si existe absceso mayor de 8
cm. o en estadio IV se procederá al drenaje quirúrgico. En ocasiones, es necesaria la histerectomía y la anexectomía. Las parejas
sexuales de las pacientes con EIP deben ser estudiadas y tratadas si han mantenido relaciones en los 2 meses previo a la aparición de
síntomas de EIP con 2 gr. de Azitromicina en dosis única vo. o Ceftriaxona 250mg im. en dosis única y Doxiciclina 100mg/12 horas vo. 7
días. CRITERIOS DE INGRESO: Requieren ingreso hospitalario las pacientes con EIP en estadio II, III y IV, y en estadio I cuando concurra
alguna de las siguientes circunstancias: Falta de respuesta a la antibioterapia después de 48 h. Sospecha de incumplimiento
terapéutico, temperatura > 38ºC, diagnóstico incierto, riesgo quirúrgico, náuseas y/o vómitos. Intolerancia oral, embarazo, signos de
reacción peritoneal alta, sospecha de piosálpinx, absceso ovárico o tuboovárico, prepúber o adolescente, gran interés en mantener la
fertilidad.

CASO CLINICO
Paciente de 19 años de edad con antecedentes de rara malformación uterina y vaginal, consistente en útero doble, el derecho
rudimentario, con hipoplasia cervical y agenesia de vagina. Fue intervenida a los 16 años, practicando extirpación de hemiútero
rudimentario, creación de neovagina con piel del abdomen, identificación de cérvix y colocación de dispositivo intrauterino (DIU). Se
comprobó durante la intervención la existencia de ovarios y trompas normales. Posteriormente siguió controles y presentaba ciclos
regulares con menstruaciones normales. Inicia coitos sin uso de preservativo, practicando controles citológicos sin hallazgos
patológicos. El episodio motivo de esta presentación se inicia tras la última regla, como cuadro de inicio brusco de dolor hipogástrico,
leucorrea y fiebre de 38 ºC. USG con piosálpinx. Se partió de una analítica inicial de 25.000 leucocitos/ml, proteína C reactiva de 63,6
mg/dl y cultivos del DIU positivos para Streptococcus spp., Proteus spp. y Escherichia coli, con urocultivo negativo.

PREGUNTA
Cuál de los siguientes síntomas clínicos es menos frecuente al dianostico de esta patología:

RESPUESTA
a.- Uso de DIU.
b.- Dolor abdominal.
c.- Leucorrea.
d.- Sangrado irregular.

CASO CLINICO
Acude a consulta femenino de 33 años de edad, refiere que hace 10 días fue diagnosticada con enfermedad inflamatoria, menciona que
ha tomado el tratamiento de forma irregular, regreso debido a que desde hace 24 horas inicio con dolor abdominal intenso, que se
incrementa cuando camina o hace algún movimiento, fatiga, adinamia, a la exploración física observa TA 110/70, FC 89, FR 23,
Temperatura de 38.5 grados, mal estado generalizado, abdomen con datos de irritación peritoneal, decide ingresarla y envía estudios
de laboratorio y gabinete debido a que considera que la paciente presenta una complicación:

PREGUNTA
Cuál es la complicación más probable que presenta la paciente:

RESPUESTA
a.- Peritonitis espontanea.
b.- Absceso tubo-ovárico.
c.- Salpingitis aguda.
d.- Ooforitis aguda.

CASO CLINICO
Una mujer de 35 años de edad, con alcohol crónico y abuso de nicotina fue admitida con un historial de dos días de evolución de dolor
en fosa iliaca izquierda, así como las temperaturas febriles de 38.8∘C y los parámetros de inflamación elevados. Una historia previa de
tuberculosis fue negada por el paciente. En un examen por ultrasonido se observo lesión de 5x6 cm masa quística en el área del anexo
izquierdo.

PREGUNTA
Cuál es el agente etiológico mas frecuente.

RESPUESTA
a.- Neisseria g.
b.- Chalmydia.
c.- Mycoplasma.
d.- Ureaplasma .

CURSO ENARM CMN SIGLO XXI TEL: 36246001 Pharmed Solutions Institute PÁGINA 288
MANUAL DE TRABAJO DEL CURSO ENARM CMN SIGLO XXI
SANGRADO UTERINA DISFUNCIONAL (HUD). CIENCIAS BASICAS: La hemorragia uterina anormal es la causa más común de pérdida
hemática en la mujer en edad reproductiva. Las mujeres con hemorragia pueden
padecer anemia crónica, dolor pélvico e incapacidad, enfrentando de tal forma un
problema médico debilitante que afecta de manera adversa sus responsabilidades
laborales y familiares. HUD: se define como aquella en la que se producen cambios
en la frecuencia del ciclo menstrual, en su duración, o en la cantidad de la pérdida
sanguínea; su diagnóstico es de exclusión, lo que obliga al clínico a descartar
inicialmente alguna patología orgánica (Tabla 1). Ésta puede catalogarse en 2
grandes rubros: la debida a causas orgánicas (hemorragia uterina anormal) y la que
es producida por anovulación (hemorragia uterina disfuncional). Las principales
causas de hemorragia uterina disfuncional o anovulatoria se enlistan en la Tabla 2.
SALUD PUBLICA: HUD, es la segunda causa de consulta ginecológica, después de la
infecciones cervicovaginales, es la principal causa de hemorragia en la mujer adulta. Se estima que alrededor de 10 millones de mujeres
en México sufren hemorragia uterina y anualmente, solo 6 millones de ellas buscan atención médica. CLASIFICACION: Por la FIGO
sistema PALM-COEIN: El sistema básico engloba cuatro categorías que se definen
por criterios estructurales visualmente objetivos (PALM: pólipos, adenomiosis,
leiomioma y malignidad o hiperplasia); cuatro afecciones no estructurales (COEI:
coagulopatia, disfunción ovulatoria, disfunción endometrial, iatrogénica), que no
están relacionadas con anomalías estructurales y una (N), reservada para las
afecciones no clasificables. La categoría leiomioma (L), se subdivide en las
pacientes que tienen al menos un mioma submucoso y que tienen miomas que no
tienen efecto en la cavidad endometrial. PATOGENIA: En relación a su
fisiopatología, en un ciclo anovulatorio el cuerpo lúteo no se desarrolla, el ovario
pierde su capacidad para producir progesterona pero la producción estrogénica
continúa; esta situación trae como consecuencia una proliferación endometrial sin
una descamación inducida por progesterona que culmine en una menstruación. El
resultado clínico de esta eventualidad es una hemorragia no cíclica, impredecible e
inconsistente en cuanto a volumen. La estimulación estrogénica continua y sin oposición (de la progesterona) produce un crecimiento
endometrial inestable excesivamente vascular, sin la suficiente capa estromal que lo soporte, tornándolo frágil y vulnerable; en este
contexto la descamación endometrial es irregular, prolongada e impredecible. En el endometrio de las mujeres con hemorragia uterina
se han encontrado grandes cantidades de prostaglandinas (PGE2 y PGF2α) cuando se les comparó con mujeres con ciclos menstruales
regulares. Incluso existe evidencia deque en los trastornos de la homeostasis (coagulación anormal) la proporción de PGE2/PGF2 y la de
prostaciclina (PGI2)/tromboxano (TXA2) están elevadas. Estas prostaglandinas están presentes, tanto en el endometrio como en el
miometrio, y el mecanismo exacto por el cual producen pérdida sanguínea elevada aún es especulativo. DIAGNOSTICO: La hemorragia
uterina disfuncional es un diagnóstico de exclusión, por lo cual el clínico debe descartar inicialmente cualquier patología orgánica o
endocrinológica. Para una adecuada evaluación clínica conviene estratificar por edad a las pacientes, debido a que las de mayor edad
incrementan el riesgo de patologías malignas o premalignas. EDAD REPRODUCTIVA (19 A 39 AÑOS DE EDAD): Aproximadamente, entre
un 6 a 10% de las mujeres con HUD tienen hiperandrogenismo con anovulación crónica (síndrome de ovarios poliquísticos), lo cual
incluye trastornos en el ciclo menstrual, hirsutismo y obesidad (IMC>25 kg/m2). El 65% de las mujeres con hirsutismo y anovulación
crónica son obesas. En nuestro país, el 37.4% de las mujeres tienen sobrepeso y el 34.5% obesidad, por lo que al sumar ambas
prevalencias, tenemos un 71.9% de mujeres de 20 años y mayores (esto es en mujeres en edad reproductiva) con trastornos en la
alimentación. En las mujeres con obesidad, irregularidades menstruales (oligo u anovulación) y datos de hiperandrogenismo deberá
descartarse síndrome de ovarios poliquísticos. En mujeres con una rápida progresión de hirsutismo acompañada de virilización, debe
sugerir al clínico descartar un tumor suprarrenal. En la mayoría de los casos, la cuantificación de los niveles de testosterona, de sulfato
de dehidroepiandrosterona y de 17-hidroxiprogesterona, puede guiarnos al diagnóstico. La evaluación debe considerar también la
valoración de biometría hemática, una prueba de embarazo, de los niveles de prolactina y de la hormona estimulante del tiroides (TSH).
La evaluación debe considerar también la existencia de embarazo, hiperprolactinemia y trastornos Tiroideos. Cuando se sospecha de
falla ovárica precoz la estimación de los niveles de FSH serán de utilidad. La anovulación es la causa más frecuente de amenorrea en las
mujeres que experimentan amenorrea secundaria. La anovulación crónica que resulta de una disfunción hipotalámica se diagnostica
por niveles bajos o normales de FSH, y puede ser el resultado de estrés fisiológico, ejercicio en exceso o pérdida de peso. A las mujeres
con amenorrea que presentan una prueba negativa de embarazo, niveles normales de FSH, TSH y prolactina podemos catalogarlas
como anovulación. MUJERES EN EDAD REPRODUCTIVA TARDÍA (DE LOS 40 AÑOS HASTA LA MENOPAUSIA): La incidencia de HUD se
incrementa de manera paralela con la edad, y los ciclos anovulatorios continuos representan la declinación de la función ovárica. En
estas mujeres, la causa más frecuente de hemorragia uterina no es precisamente la hiperplasia endometrial ni el cáncer endometrial,
sino las patologías intracavitarias como pólipos endometriales y miomas submucosos. En las mujeres de esta edad con HUD a las cuales
se les ha descartado inicialmente alguna causa orgánica, y que persisten con episodios de hemorragia a pesar de haberse instaurado un
tratamiento adecuado, deberán revalorarse en la búsqueda de causas malignas o premalignas. El clínico no debe olvidar la estrecha
relación de algunos de los parámetros clínicos presentes en la mujer con HUD, como la obesidad y la anovulación en la génesis del
cáncer endometrial. La biopsia de endometrio es una técnica sencilla, relativamente simple y que puede realizarse en el consultorio; las
pacientes menores a 35 años de edad con HUD que no respondan a la terapia médica instaurada son candidatas a la biopsia de
endometrio y las mujeres mayores de 40 años de edad y HUD deberán someterse a una biopsia de endometrio. La histeroscopia de
consultorio es una técnica de visualización directa que detecta un alto porcentaje de anormalidades intracavitarias. Ultrasonido
transvaginal (UTV), la sonohisterografía (SHG) y la histeroscopia de consultorio en la detección de lesiones intracavitarias, se reportan
una sensibilidad y especificidad del UTV en un 56.3 y 100%; para la SHG 72% y 87.5%; finalmente, para la histerosopia un 100% y 100%.
Los autores concluyen que la certeza diagnóstica de la SHG es equiparable a la histeroscopia. Sin embargo, se reporta menos dolor con
la SHG que con la histeroscopia. Desafortunadamente, es una técnica que requiere entrenamiento, y el instrumental necesario para su

CURSO ENARM CMN SIGLO XXI TEL: 36246001 Pharmed Solutions Institute PÁGINA 289
MANUAL DE TRABAJO DEL CURSO ENARM CMN SIGLO XXI
realización es caro. El objetivo de la evaluación de la cavidad uterina en las mujeres con HUD incluye la detección de lesiones focales
(miomas o adenomiosis) y alteraciones a nivel endometrial (engrosamiento endometrial o pólipos). El ultrasonido transvaginal (UTV) es
una herramienta útil para el diagnóstico de lesiones focales como miomas, ya que logra detectar alguna patología hasta en un 97% de
los casos en mujeres premenopáusicas, sin embargo, pierde certeza diagnóstica en lesiones intracavitarias. La evaluación por UTV del
endometrio que resulte en un grosor mayor de 18 mm sugiere, fuertemente, alguna patología a este nivel. TRATAMIENTO: No existe
evidencia suficiente para afirmar que el uso de anticonceptivos orales (AOC), solos o comparados con otros tratamientos (AINES,
danazol, DIU) sean benéficos para la mujer con HUD en relación a la mejoría de su sintomatología. No existen ventajas con el uso de
progestinas para el control de HUD si se les compara con danazol, AINES o con el dispositivo impregnado con levonorgestrel; sin
embargo, el uso de progesterona por 21 días parece reducir de manera significativa la pérdida sanguínea en pacientes con HUD, por lo
que este régimen se puede administrar de manera inmediata para el control de la hemorragia y solo como un tratamiento temporal. Es
insuficiente la evidencia para señalar que el uso de DIU-IL sea la mejor opción, en comparación con el uso de noretindrona continua en
las mujeres con HUD; pero se reportarán mayores efectos adversos en el grupo de DIU-IL. El danazol es una mejor alternativa para el
tratamiento de HUD, en comparación a placebo, progestinas, AINES y AHO, pero presentan mayores efectos adversos. Debido al
pequeño número de trabajos, no es posible emitir una recomendación adecuada para su uso en la práctica clínica en las pacientes con
HUD. Tratamiento quirúrgico: El legrado uterino instrumentado es la intervención diagnóstica o terapéutica más empleada en nuestro
medio, no cuenta con la suficiente evidencia científica sobre su utilidad, sola o comparada contra intervenciones. La ablación
endometrial solo está indicada en mujeres con paridad satisfecha y que no desean histerectomía total abdominal. Si se le compara con
el tratamiento médico, resulta ser significativamente más efectiva para controlar la hemorragia a 4 meses de seguimiento, sin evidencia
de efectividad a más largo plazo. No hay diferencia en un año respecto a la calidad de vida, al compararse con el DIU-IL. Para las
mujeres que tienen satisfecho su deseo reproductivo, y en las que se han utilizado las diferentes opciones terapéuticas (tanto médicas
como quirúrgicas) sin haberse encontrado una respuesta satisfactoria a su problema, la histerectomía representa la mejor opción al ser
curativa y mejorar la calidad de vida, aunque tenga mayor riesgo de complicaciones.

CASO CLINICO
Mujer de 27 años con antecedentes de migraña y colecistectomía laparoscópica 2 años antes, presenta alteraciones en la cantidad y
frecuencia del ciclo menstrual asi como flujo intermenstrual recurrente, como antecedentes la paciente le fue realizada cesarea por
placenta de implantación baja total hace 6 meses, hay antecedentes familiares de cáncer endometrial y quistes ováricos.

PREGUNTA
Cual es la conducta a seguir para establer un escrutinio diferencial.

RESPUESTA
a.- USG enfocándose en anexos.
b.- Realizar PIE.
c.- Realizar evaluación de gonadotrofina.
d.- Biopsia endometrial.

CASO CLINICO
Paciente de 26 años de edad se presenta a la consulta refiriendo que desde hace un año posterior a un parto via vaginal inicia
padecimiento caracterizado por alteraciones de su periodo mentrual, refiere que anteriormente era regular pero ahora no, además
agrega infecciones vaginales frecuentes, es usuaria de DIU su DOC fue normal a la exploración presenta leve dolor a la movilización de
cérvix.

PREGUNTA
Cuál es la conducta a seguir.

RESPUESTA
a.- Iniciar método de barrera.
b.- Realizar USG.
c.- Retirar DIU.
d.- Envio a colposcopia.

MIOMATOSIS UTERINA. CIENCIAS BASICAS: Los miomas son una enfermedad benigna común del útero, son tumores que se inician en
una sola célula de músculo liso uterino y pueden crecer en cualquier parte del útero bajo la influencia de factores de crecimiento local,
ctocinas y hormonas sexuales; ocasionan trastorno a nivel endometrial. La miomatosis uterina es la presencia de uno o varios tumores
benignos formados por fibras musculares del útero (matriz) y se considera la tumoración más frecuente del aparato genital femenino.
También se le denomina leiomiomas o fibromas uterinos. SALUD PUBLICA: La prevalencia de la miomatosis uterina en mujeres en edad
fértil se calcula entre 20 y 40%, siendo más común en mujeres afroamericanas, y contando como factores de riesgo la edad,
nuliparidad, tabaquismo y otros. Su incidencia acumulada para la edad de 50 años es de más del 80% en mujeres afroamericanas y 70%
en mujeres caucásicas. Se han asociado con infertilidad en 5 a 10% de los casos, y se estima que se encuentran como factor único
presente en 1 a 3% de los casos. CLASIFICACION: Se ha clasificado a los miomas uterinos como submucosos si distorsionan la cavidad
uterina, intramurales si residen predominantemente dentro de la pared miometrial uterina y subserosos si protruyen fuera de la
superficie uterina. El número y localización de los miomas correlaciona con los síntomas y el efecto en la fertilidad. Existen otras
localizaciones poco comunes: cervicales (en el cuello de la matriz), intraligamentarios (dentro de los ligamentos que sostienen al útero)
o incluso miomas parásitos, que son aquéllos que al ir creciendo van tomando el aporte sanguíneo de estructuras vecinas, pudiendo
incluso llegar a quedar completamente separados del útero. PATOGENIA: Predisposición del sitio anatómico a formar miomas. Se cree

CURSO ENARM CMN SIGLO XXI TEL: 36246001 Pharmed Solutions Institute PÁGINA 290
MANUAL DE TRABAJO DEL CURSO ENARM CMN SIGLO XXI
que son causados por una excesiva estimulación de los estrógenos, que actúan sobre una o varias células de las fibras musculares del
útero susceptibles a responder en forma exagerada, dando lugar a la proliferación desmedida y la formación de tumores. Se piensa que
es un tumor estrógeno-dependiente, ya que crece durante la edad reproductiva y en ocasiones revierte o disminuye en la menopausia
cuando ya no hay el estímulo estrogènico. El uso de hormonales orales en forma constante incrementa su desarrollo DIAGNOSTICO:
Dependerá en gran medida de la localización, tamaño y número de los miomas: Subserosos: Su sintomatología se relaciona con la
compresión a órganos vecinos. Si comprimen la vejiga, pueden dar síntomas de infección urinaria (ardor al orinar, micción frecuente y
en poca cantidad, etc.), o incluso incapacidad para contener la orina. Si comprimenal intestino grueso podrán ocasionar constipación,
colitis, dolor durante la evacuación o incluso oclusión intestinal. La compresión sobre las arterias o venas de la pelvis condicionará el
desarrollo de varicosidades, inflamación de miembros inferiores o sensación de dolor y de pesantez en las piernas. Intramurales:
Interfieren con la contractilidad uterina, comprimen los plexos venosos y arteriales y, a la larga, condicionan la presencia de
menstruaciones prolongadas y abundantes, sangrado ínter menstrual y dolor durante la menstruación, además de estar relacionados
en muchos casos con esterilidad. Submucosos: Deforman el interior de la cavidad uterina y son los que con mayor frecuencia ocasionan
aumento en el sangrado menstrual (hiper-poli menorrea) además de cólicos. Son un importante factor causal de esterilidad y pueden
crecer tanto que salen a través del cuello uterino (miomas abortados). La exploración bimanual clínica es el segundo paso para un
diagnóstico adecuado, pudiendo detectarse un crecimiento uterino exagerado o la delimitación de la tumoración. De ahí la importancia
de que la paciente haya vaciado la vejiga antes de la exploración y de que se encuentre en una posición cómoda, con los músculos
abdominales completamente relajados, para permitir un examen adecuado. Siempre que se sospeche la presencia de miomas, está
indicada la realización de un ultrasonido pélvico, lo que permitirá determinar las dimensiones del útero, así como el número,
localización, tamaño y relación del (o los) miomas con estructuras cercanas. Si después de los puntos anteriores existe alguna duda
diagnóstica, se podrá recurrir a otros estudios de gabinete como son: histerosalpingografía, tomografía axial computarizada, y realizar
diagnóstico diferencial con algún otro tipo de tumor benigno o maligno de la cavidad abdominal. TRATAMIENTO: Miomectomía:
Consiste en la resección únicamente de los miomas a través de una incisión en la pared abdominal (si son tumores de gran tamaño) o
mediante cirugía de mínima invasióncomo lo es la laparoscopia. Esta miomectomía se indica siempre que la paciente tenga deseos de
embarazos futuros y sea menor de 35 años. Hasta 80% de las mujeres refiere disminución de los síntomas después de la miomectomía.
Histerectomía: Es la extirpación total del útero, también a través de una incisión abdominal o por cirugía laparoscópica. Será el
procedimiento de elección en aquéllas que no deseen tener más hijos, estén cerca de la menopausia o si el crecimiento tumoral ha sido
muy acelerado. Tratamiento Hormonal: El objetivo es no bloquear al endometrio, el objetivo será regular la tortuosidad de los vasos, la
estimulación a nivel endometrial y que el endometrio este creciendo de manera regular en toda la cavidad uterina. Se puede usar
ergotrate (vasoconstrictor especifico), los primeros días del ciclo, en cada menstruación o ac. Mefenamico (seguro). Lutoral o provera
en la segunda parte del ciclo. En miomatosis uterina de grandes elementos, donde no hay sangrado no se da tratamiento. Se pueden
utilizar hormonas inhibidoras de la producción de estrógenos en forma “paliativa” o si existe alguna contraindicación para realizar los
procedimientos anteriores, también se utilizan si se desea disminuir el tamaño de los tumores previo a la cirugía o si coexiste
endometriosis severa. El éxito del tratamiento médico-hormonal es menor del 10%.

CASO CLINICO
Mujer de 49 años con 3 hijos, viene a verlo debido en a que presenta períodos abundantes, desde hace dos años ha notado que esto se
ha incrementado de tal forma que le ha generado incomodidades ya que afirma que “inunda” la toalla los primeros días y duran hasta
10 dias además agrega cansancio crónico, astenia y adinamia. A la exploración física se observa palidez de tegumentos, uñas
quebradizas, pelo frágil asi como piel fría. A la exploración GO se observa sangre obscura en vagina con algunos coagulos, dolor a la
palpación y movilización uterina, refiere que en su anterior DOC se reportaron cambios inflamatorios recibiendo tratamiento el cual no
especifica. Cuenta con OTB, finalmente refiere sangrado poscoital.

PREGUNTA
Cuál es la conducta a seguir.

RESPUESTA
a.- Realizar USG.
b.- Biometria hemática y ferritina serica.
c.- Histeroscopia.
d.- Biopsia endometrial.

CASO CLINICO
Mujer de 31 años de edad, refiriendo presencia de irregularidades de su periodo menstrual, agrega que no se ha podido embarazar
desde hace 6 años que inicio su vida sexual activa. No usa método anticonceptivo. Actualmente toma 37,5 mg de venlafaxina dos veces
al día por episodio depresivo que inicio hace un año. Su examen físico se observa presencia de hematomas refiriendo que esto le ha
ocurrido desde la adolescencia, se observa además un IMC de 34, con hirsutismo en cara, espalda y piernas, además vello púbico tipo
romboide.

PREGUNTA
Cuál es la conducta a seguir.

RESPUESTA
a.- USG transvaginal.
b.- USG ginecologico abdominal.
c.- USG de glándulas suprarrenales.
d.- Biopsia endometrial.

CURSO ENARM CMN SIGLO XXI TEL: 36246001 Pharmed Solutions Institute PÁGINA 291
MANUAL DE TRABAJO DEL CURSO ENARM CMN SIGLO XXI
VIRUS DEL PAPILOMA HUMANO (VPH), DISPLASIA, CANCER CERVICOUTERINO (CACU). CIENCIAS BASICAS: Los papilomavirus son
pequeños virus de DNA, de doble cadena, cuya actividad transformante se explica principalmente por la actividad de sus oncoproteínas
E6 y E7. Estas proteínas se unen a un sin número de reguladores celulares importantes en el control de procesos biológicos como: la
apoptosis, proliferación celular, estabilidad cromosómica, transcripción de genes (oncogenes y genes supresores de tumor),
diferenciación celular y la respuesta inmunológica, entre otros. El VPH se encuentra ampliamente distribuido en todo el mundo y
provoca un amplio espectro de enfermedades epiteliales, desde verrugas a papilomas en los epitelios de distintas mucosas (lesiones
preinvasores de cérvix, la principal), ya que poseen un trofismo específico. También VPH se ve involucrado en la patogenia de diversos
tumores benignos y malignos, y constituye el factor de riesgo más importante para el desarrollo de cáncer cérvico uterino. SALUD
PUBLICA: El VPH está fuertemente asociado al desarrollo de displasia, neoplasia intraepitelial, y cáncer del cuello uterino. Más del 95%
de los cánceres de cérvix poseen DNA de VPH de alto riesgo. 1% de las displasias leves evolucionan a invasividad. De 5-15% de las
displasias graves progresa a cáncer en 3 años. El virus puede estar latente hasta 10-15 años y no ocasionar cambios. El papilomavirus
tipo 16 es el más prevalente de los VPH oncogénicos, responsable de mas de la mitad de los tumores, mientras que el papilomavirus
tipo 18 está involucrado en el 20% de los mismos. PATOGENIA: Dependiendo del tipo de VPH y de la lesión clínica, puede transmitirse
por contacto cutáneo, relación sexual, transmisión perinatal, por escamas infectadas directa o indirectamente, y posiblemente por
fómites. El período de incubación varía desde tres semanas a 8 meses, con un promedio de tres meses. En cuanto al genotipo del VPH y
su localización más frecuente en piel se tiene: Verruga Vulgar: 2,4,7, verruga plantar: 1,4, verruga plana: 3, 10, 28, 41,
epidermodisplasia Verruciforme: 5, 8, 9, 12, 14, 15, 17, 19, 36, 46, 47, 49, 50. En Mucosas (bajo riesgo): Papilomas anogenitales,
cervicales, orofaringe y tracto respiratorio: 6, 11, 30, 34, 40, 42, 44, 55, 57, 59. En Mucosas (alto riesgo): papilomas anogenitales y
cervicales, papulosis Bowenoide, displasia cervical, orofaringe, cáncer cervical y anogenital: 16, 18, 31, 33, 35, 39, 45, 51, 52, 56. Los
principales factores de riesgo del VPH genital son: Tener muchas parejas sexuales, tener menos de 25 años de edad, IVSA a una edad
temprana (<16 años), sin embargo, incluso las mujeres que sólo han tenido una pareja sexual pueden infectarse con el VPH,
multigestas, antecedente de ETS, tabaquismo, anticonceptivos orales. Sólo un 10% de las infecciones por VPH tiene manifestación
clínica, ya sea en la forma de verrugas, papilomas o displasias. El virus se replica en el estrato granuloso y es detectado en el estrato
córneo, no así en el estrato basal. La infección por VPH no tiene rol oncogénico per se, sino que juega un rol que es potenciado por
factores físicos y químicos. En los carcinomas no se encuentran partículas virales activas, pero sí su DNA y sus genes tempranos. Los
productos de los genes E5, E6 y E7 tienen actividad oncogénica, ya que sus proteínas están involucradas en el control del ciclo celular y
estimulan la proliferación o interfieren con la diferenciación de células infectadas. El blanco de estas proteínas virales está
representado por las proteínas retinoblastoma (Rb), y p53. Normalmente retinoblastoma actúa inhibiendo la transcripción de genes
como c-myc, ras, entre otros que estás encargado de la proliferación celular, de manera que la inactivación de Rb provoca una
replicación celular descontrolada. La p53 se encarga de promover la transcripción de genes para reparar el DNA dañado o inducir
apoptosis, de modo que su inactivación provoca la pérdida de la capacidad de bloquear la proliferación celular como respuesta al daño
del DNA. El resultado es la inestabilidad genética y el desarrollo de mutaciones críticas que favorecen el desarrollo de tumores. En la
zona de transición (unión escamocolumnar) de exocervix y endocervix hay dos epitelios, por eso es un lugar ideal, para el virus del VPH.
DIAGNOSTICO: Clínica; La principal manifestación clínica son las verrugas del área genital externa y el condiloma acuminado. El
condiloma acuminado se presenta como lesiones papulares con superficie lobulada e irregular, de color rosado oscuro, con
prolongaciones digitiformes con aspecto de coliflor. El número de lesiones es variable, su tamaño ve de 2mm a 1 cm, pero si son
numerosas pueden confluir comprometiendo grandes áreas genitales que se traumatizan durante el coito. En la mujer se ubican con
mayor frecuencia en los labios mayores y menores, parte posterior del introito, clítoris, monte de Venus, paredes vaginales y en el
cuello uterino. En el hombre son comunes en la cara interna del prepucio, en el frenillo y en el surco balanoprepucial. Con menor
frecuencia se presentan en el glande y en el meato uretral. Las lesiones perianales y rectales pueden verse en varones homosexuales o
pueden ser el resultado de la propagación perineal en las mujeres. En personas que practican el sexo oral puede encontrarse
condilomas orales. En el embarazo e inmunodeprimidos, especialmente los transplantados renales las lesiones tienden a ser más
numerosas y exuberantes. El VPH puede infectar el epitelio escamoso de cérvix, vagina, vulva, periné, pene y región perianal, y
determinar lesiones como verrugas genitales, condilomas acuminados, lesiones precancerosas intraepiteliales y cáncer. La detección de
HPV va de 40 a 90% en las neoplasias intraepitelial (NIE) de bajo grado, a 95% en los pacientes con cáncer invasor. Los VPH también han
sido implicados en el desarrollo de tumores malignos en sitios distintos a la región anogenital, como ano, vagina, vulva y pene e incluso
cavidad bucal, pero con una fracción atribuible considerablemente menor a la del cáncer de cérvix, en el cual virtualmente el 100% de
los cánceres son causados por VPH. Laboratorio y gabinete: Papanicolaou se utiliza para detectar cambios celulares o células anormales
en el cuello uterino, (estas células anormales pueden ser precáncer o cáncer, u otras cosas) es una prueba de screening. Se extraen
células del cuello uterino y se procesan. Luego se observan con un microscopio para ver si las células son normales o si se pueden
observar cambios en ellas. La prueba de Papanicolaou es una excelente prueba para encontrar células cancerosas y células que se
pudieran convertir en cáncer. El resultado anormal más común en la prueba de Papanicolaou se llama ASC-US. Las células ASC-US
generalmente no indican precáncer, pero tampoco son del todo normales. Actualmente con la captura de hibridos, es más fácil
detectarlo, solo nos dice si hay virus o no. Hasta 7 años antes, a diferencia del papanicolao normal donde a veces lo encontramos en
NIC I, porque no siempre vemos los coilocitos, este detecta lesion los hibridos no. Las lesiones subclínicas deben ser visualizadas con
colposcopía y con la aplicación de ácido acético al 3-5% que ayuda a delimitar la lesión mediante la reacción blanco acética. Sin
embargo es un examen de baja especificidad, ya que se altera en otras enfermedades, como por ejemplo el líquen plano, la candidiasis
etc. En todas las mujeres con condiloma acuminado debe realizarse citología con técnica de Papanicolau anual. Colposcopia: identifica
lesiones sopechosas, delimita la zona de transformación, determina extensión de la enfermedad, útil para toma de biopsias en zonas
sospechosas. Clasificacion por hallazgos colposcopicos: Grado I; insignificante no sospechoso, epitelio aceto-blanco delgado, vasos no
atípicos, distancia intercapilar pequeña. Correlacion histológica: metaplasia escamosa y LIEBG. Grado II: significativo, sospechoso,
epitelio aceto-blanco mayor opacidad y grosor, sin vasos atípicos, distancia intercapilar aumentada. Correlacion histológica: NIC II y NIC
III. Grado III: Altamente significativo, muy sospechoso, epitelio aceto-blanco grueso irregular y opaco, vasos dilatados irregulares y
atípicos, distancia intercapilar variable, contorno superficial irregular. Correlacion histológica: LIEAG e invasión temprana. Todas las
lesiones que encontremos por colposcopia se biopsian. Debe biopsiarse verrugas resistentes al tratamiento, atípicas o pigmentadas.
CLASIFICACION: Bethesda, para pronostico y tratamiento: LIEBG (lesión intraepitelial de bajo grado): IVPH y displasia leve (NIC I). LIEAG

CURSO ENARM CMN SIGLO XXI TEL: 36246001 Pharmed Solutions Institute PÁGINA 292
MANUAL DE TRABAJO DEL CURSO ENARM CMN SIGLO XXI
(lesion intraepitelial del alto grado): NIC II, NIC III. ASCUS: Células que no son claramente displasicas, cambios inflamatorios. 20-50%
desarrollan displasia. Se da seguimiento como si se hubiera descubierto un NIC. TRATAMIENTO: Químico; Podofilino al 10%-30%-45%
en solución alcohólica, aplicación por profesional médico en lesiones de
genitales externos y perianales. La aplicación debe repetirse
semanalmente por 3 a 4 semanas hasta la desaparición de las lesiones.
Si no mejora considerar otra posibilidad diagnóstica o la presencia de
una cepa más agresiva. La aplicación de grandes cantidades de
podofilino puede provocar toxicidad sistémica. Está contraindicado en el
embarazo y la lactancia. Podofilotoxina al 0.5% Puede aplicarse
directamente por el paciente dos veces al día por 3 días. No requiere
lavarse como la podofilina. Se descansa 4 días y luego se repite por 3
días más. Las respuestas se ven habitualmente a las 6 semanas. Su
eficacia es similar a la de la podofilina, pero tiene menos toxicidad
sistémica. Aproximadamente la mitad de los pacientes muestra algún
grado de inflamación, quemaduras o erosiones. Ácido Tricloroacético al
80-90%, aplicación local por el médico 2-3 veces por semana por un
máximo de tres semanas. Se forma una erosión que sana en unas 3
semanas sin cicatriz. Puede usarse como terapia combinada con
podofilino al 40% en pacientes inmunodeprimidos y es el tratamiento de
elección en el embarazo. Tratamientos Físicos; Crioterapia (Nitrógeno
Líquido), cura aproximadamente el 90% de las lesiones, aunque a veces
se requiere varias aplicaciones. Electrocirugía Eventualmente
desaparecen todas las lesiones, aunque el 20-30% desarrolla nuevas
lesiones en los bordes quirúrgicos o en sitios alejados. Extirpación
Quirúrgica; 1) Láser. 2) Inmunomoduladores: factor inmunomodulador
inductor de la síntesis de interferón gamma, TNF alfa, IL 1,6,8,10 y factor
estimulante de colonias granulocíticas. Estimula la inmunidad celular y
carece de actividad antiviral directa in vitro. La presentación en crema al
5% se utiliza en el tratamiento de los condilomas y verrugas planas, se
aplica cada 48 horas y se deja actuar por 2 a 4 horas. La respuesta se
observa a las seis semanas. Es bien tolerado incluso en pacientes
inmunodeprimidos, aunque dos tercios de los pacientes presentan
eritema y ardor. Produce curación en el 60% de los casos y las mujeres
responden mejor que los hombres. 3) Imiquimod (Aldara). Tratamiento
específico: NIC I o LIEBG: Tratar procesos infecciosos concomitantes y
repetir papanicolao y colposcopia, a intervalos de 3-4 meses con examen
pélvico. Terapia ablativa en personas poco confiables para seguimiento. En pacientes inmunocompetentes hay remisión hasta en 50%.
NIC II, NIC III ó LIEAG: Terapia ablativa la criocirugía elimina lesión hasta 95%, cirugía láser de CO2, vaporización del tejido tratado con
éxito de 95%, se hace en lesiones que se extienden. Escisión quirúrgica con asa diatermica, solo cuando esta confinada la lesión al
cérvix. Cono cervical: Se realiza cuando hay cáncer in situ, con deseos de conservar la fertilidad, de lo contrario se realizara
histerectomía. La curación es de 87-97%. También se realiza cuando hay ca microinvasor, menor a 3mm, sin compromiso linfovascular.
Con este procedimiento el seguimiento es más riguroso, realizar citología cada 3 meses. Histerectomia: Extrafascial, cuando no está a
discusión la fertilidad. En casos de ca in situ el seguimiento de la citología es trimestral primer año, posteriormente anual. LESIONES
PREINVASORAS: Abarcan solo epitelio, sin membrana basal. NIC I, II, III (Neoplasia intraepitelial cervical). DISPLASIA: Cambios
morfológicos celulares, núcleos hipercromaticos, aumento de la relación núcleo y citoplasma, mayor índice mitótico. CANCER
CERVICOUTERINO: EL CaCu ocupa los primeros lugares como causa de muerte por cáncer en mujeres mexicanas. Constituye el 34.2%
de las neoplasias malignas. Una infección persistente de virus del papiloma humano (VPH) de tipos virales de alto riesgo oncogénico, es
el factor etiológico principal en el desarrollo de esta neoplasia. Se conoce que solamente una pequeña fracción de lesiones cervicales
infectadas con VPHs de alto riesgo evoluciona a lesiones de alto grado o cáncer. Las mujeres sexualmente activas, de cualquier edad,
pueden infectarse con VPHs oncogénicos. Sin embargo, el cáncer de cérvix invasor en mujeres jóvenes infectadas con virus oncogénicos
es raro y la prevalencia de VPH en mujeres de 40 años o mayores no se correlaciona con la alta tasa de cáncer cervical. Es la persistencia
de VPHs oncogénicos lo que da lugar al desarrollo de lesiones precancerosas y potencialmente al cáncer invasor, lo que puede llevar
varios años para su desarrollo. El cáncer de cérvix ocurre en dos formas predominantes: carcinoma epidermoide y adenocarcinoma,
carcinoma adenoescamoso 1.7%, carcinoma verrugoso, carcinoma indiferenciado de células pequeñas, tumor carcinoide, melanomas.
El tipo histológico más comúnmente encontrado en las mujeres es el carcinoma epidermoide (90% de los casos) y está más
frecuentemente asociado al VPH 16. El adenocarcinoma 3.7% es el segundo tipo histológico más común y aunque el VPH tipo 16
también es el más frecuente, la proporción de los genotipos 18 y 45 aumenta significativamente en este tipo de tumores. Los estudios
de citología, que incluyen a la prueba Pap convencional, se utilizan para detectar lesiones precancerosas, pero no es suficiente para
detectar infecciones por VPH. Es claro que, el adenocarcinoma es más difícil de detectar por Pap que el carcinoma escamoso.
Manifestaciones clínicas: sangrado (lo más importante), predominantemente poscoital, también puede ser intermenstrual y
posmenopáusico (siempre pensar en cáncer cervical y después cacu), flujo seroso, fétido ocasionado por la necrosis tumoral, dolor
pélvico (territorio de nervio ciático), edema de extremidades inferiores, dolor lumbar, pueden presentarse fistulas por infiltración a
vejiga o recto. En estadios avanzados; pérdida de peso, anemia, síndrome urémico. Marcadores tumorales: Antigeno del carcinoma de
células escamosas, lo podemos encontrar hasta en 50% de los primarios y 75% en los recurrentes, tienen baja utilidad. El diagnóstico de
esta neoplasia se realiza con estudio histopatológico mediante una biopsia dirigida, ya sea mediante colposcopia en caso de no
observarse una lesión o mediante toma directa si existe tumor visible. Los estudios de extensión a realizar en cada caso, dependerá de

CURSO ENARM CMN SIGLO XXI TEL: 36246001 Pharmed Solutions Institute PÁGINA 293
MANUAL DE TRABAJO DEL CURSO ENARM CMN SIGLO XXI
la etapa clínica obtenida en el examen inicial Estadificacion según la FIGO; ver cuadro anexo. TRATAMIENTO: Recomendaciones de
tratamiento en el Instituto Nacional de Cancerología 1) Cáncer Cervico uterino In Situ; Preservación de Fertilidad, cono terapéutico.
Fertilidad satisfecha (<50 años o premenopausia): cono terapéutico o histerectomía Tipo I con preservación de anexos. Fertilidad
satisfecha (>50 o post menopausia): cono terapéutico o histerectomía Tipo I con salpingo-ooforectomía bilateral. 2) CaCu IA 1; Cono
terapéutico o histerectomía tipo I. Preservación de anexos en <50 años o premenopáusica Salpingo-ooforectomia bilateral en >50 años
o postmenopáusica. 3) CaCu IA 2; Histerectomía Radical tipo II. Preservación de anexos en <50 años. Linfadenectomía pélvica en caso
de PVL. Braquiterapia ± radioterapia externa a pelvis total (dosis total de 75-80Gy a punto A) en caso de pacientes médicamente
inoperables. 4) CaCu IB1: Histerectomía Radical tipo II (en tumores menores de 2 cm) y linfadenectomía pélvica bilateral. Histerectomía
Radical tipo III y linfadenecto mía pélvica. Preservación de anexos en <50 años o premenopáusicas. En caso de contraindicación médico
quirúrgica la opción terapéutica es la Radioterapia externa a pelvis total + braquiterapia (dosis total de 80 85Gy a punto A). 5) CaCu
IB2–IVA. Radioterapia externa a pelvis total concomitante con quimioterapia basada en cisplatino + braquiterapia (dosis total >85Gy a
punto A). Cis platino 40 mg/m2 en forma semanal. Exenteración pélvica para pacientes con fístula vesico-vaginal y/o recto-vaginal sin
infiltración a la pared pélvica y con ECOG I y K más del 90%. Nota: En caso de paciente con hidronefrosis co locación de catéter JJ o
Catéter de nefrostomía antes de iniciar el tratamiento con radioterapia. Con fístula vésico-vaginal no exenterables derivación urinaria.
Con fístula recto vaginal no exenterable colostomía. 6) Ca Cu II, III, IVB; Tratamiento sistémico paliativo, radioterapia paliativa exclusiva,
superviviencia a 5 años 65-75%, 30-50% y 20% respectivamente. Vigilancia: 80-90% presentan recaidas en los 2 primeros años.
Exploración pélvica y citológica; mensual o bimensual el primer año. Cada 2-3 meses el segundo año. Cada 4-5 meses el tercer año.
Cada 6 meses el cuarto año y quinto año. Cada año a partir del sexto año. PAPULOSIS BOWENOIDE (PB): Es una forma de cáncer
espinocelular in situ asociado al VPH. Se caracteriza por múltiples pápulas asintomáticas, ligeramente solevantadas, de 2 a 20 mm que
pueden confluir formando placas de color marrón violáceo y con tenue descamación, ubicadas en la base del pene, vulva y periné en
pacientes jóvenes. Se considera un factor de alto riesgo de desarrollo de cáncer de cérvix. Su curso es habitualmente benigno.
CONDILOMA ACUMINADO GIGANTE: o tumor de Buschke-Lowenstein es una lesión precancerosa. Se caracteriza por condilomas de
gran tamaño que causan daño tisular local. Son más frecuentes en la ingle, región perianal y en el surco balanoprepucial. No metastisa,
se maneja con cirugía y crioterapia y recurre con frecuencia. Si se detecta la presencia de VPH 16 o 18 debe considerarse la progresión a
la malignidad.

CASO CLINICO
Mujer de 29 años de edad la cual refiere sangrado poscoital la cual ha resultado levemente dolorosa, como antecedentes presento
menarca a los 9 años, IVSA a los 14 años, ha tenido 4 parejas sexuales, es usuaria de método hormonal oral, agrega que ha presentado
varios cuadros de cervicovaginitis, la ultima DOC presento cambios inflamatorios, tabaquismo y alcoholismo positivo. A la exploración
ginecológica se observa cérvix afresado, doloroso a la movilización.

PREGUNTA
Cuál es la conducta a seguir.

RESPUESTA
a.- Repetir DOC.
b.- Realizar colposcopia.
c.- Enviar a clínica de displasias.
d.- Realizar citología de base liquida.

CASO CLINICO
Paciente de 45 años, con antecedentes de hipertensión arterial crónica en tratamiento con atenolol (50 mg/día) e hidroclorotiazida
12,5 mg cada 24 hrs. Multípara, usuaria de dispositivo intrauterino (T de cobre, por 7 años). Consultó por hipermenorrea y
dismenorrea, de intensidad progresiva y resistente al tratamiento médico (antiinflamatorios no esteroidales y retiro del dispositivo
intrauterino). Al examen físico presentaba un útero aumentado de tamaño, con anexos normales. Se solicitó ultrasonografía
transvaginal que confirmó la presencia de miomas múltiples, el mayor en el fondo uterino de 4 cm de diámetro. El hemograma
demostró anemia ferropriva significativa (Hto. 26%). Dado el tamaño de los miomas y síntomas asociados (hipermenorrea, anemia
severa secundaria, dismenorrea y dispareunia intensa), se decidió realizar un tratamiento quirúrgico subtotal. Evolucionó
satisfactoriamente, dándose de alta en buenas condiciones con suplemento oral de hierro.

PREGUNTA
Cuál es la conducta a seguir.

RESPUESTA
a.- Realizar citología cervical 3 veces anualmente.
b.- Realizar citología cervical normalmente.
c.- Se puede descontinuar la citología cervical.
d.- Realizar citología vaginal cada 2 o 3 años.

CURSO ENARM CMN SIGLO XXI TEL: 36246001 Pharmed Solutions Institute PÁGINA 294
MANUAL DE TRABAJO DEL CURSO ENARM CMN SIGLO XXI
CANCER ENDOMETRIAL (CE). CIENCIAS BASICAS: Es una neoplasia glandular maligna que se origina en el endometrio, generalmente
parece estar relacionado con una estimulación estrogenica crónica del endometrio no contrabalanceada, por una fuente de estrógenos
endógenos o exógenos. La mayoría de los canceres de endometrio son adenocarcinomas (canceres que derivan de células que
producen moco y liberan moco u otros liquidos) SALUD PUBLICA: Es la segunda neoplasia ginecológica más frecuente a nivel mundial,
siendo más común en la perimenopausia. La incidencia del CE es seis veces mayor en países desarrollados que en los menos
desarrollados. La supervivencia en las etapas clínicas tempranas es del
80%, mientras que las pacientes con enfermedad avanzada tienen una
supervivencia a largo plazo menor al 50%. El tipo histológico más
frecuente es el adenocarcinoma endometroide 75-80%. CLASIFICACION:
Basado en el perfil histológico, molecular y clínico el cáncer de
endometrio se divide en dos tipos: TIPO I: variedad endometrioide (90%)
frecuentemente de bajo grado, el más frecuente, se presenta entre los
55-65 años de edad, relacionado a estrógenos, diagnosticado en forma
temprana y con buen pronóstico en supervivencia global, el cual se
origina de una lesión precursora (hiperplasia atípica o neoplasia
intraepitelial endometrial), en donde está implicado la mutación como
inactivación de PTEN (una proteína natural), gen supresor de tumor. TIPO
II: variedad no-endometrioide, no hormonodependiente, se presenta en
mujeres de >65 años, son pacientes delgadas, papilar seroso, células
claras y carcinosarcomas. Puede con llevar una enfermedad extrauterina,
se relaciona con escasa supervivencia, representa el 10% de todos los canceres de endometrio., Invade el espacio vascular y linfático,
hasta 36% presenta ganglios positivos sin tener invasión en miometrio. PATOGENIA: Factores de riesgo: historia de terapia hormonal
estrogenica sustitutiva sin oposición progestacional, tratamiento con tamoxifeno, menopausia tardía, nuliparidad, infertilidad o falla
terapéutica de ovulación, y obesidad. La DM e hipertensión, también son considerados factores de riesgo. Todas las lesiones
endometriales se originan del componente glandular del endometrio,
formando una lesion polipoide en la cavidad uterina hasta que la lesión se
vuelve friable y necrótica, consecuentemente, el sangrado postmenopáusico
constituye el 90% de los síntomas al inicio de la enfermedad. La invasión local
y linfática son los pasosos siguientes en la historia natural: mientras que la
porción superior uterina drena, através de los vasos ováricos a los relevos
paraaórticos, las porciones inferiores drenan a través de los vasos uterinos a
los pélvicos. Resultando que del 10-35% de los tumores tempranos tengan
adenopatías paraaorticas sin relevos pélvicos positivos. Una vez que el tumor
rompe la serosa, puede haber invasión a órganos vecinos. DIAGNOSTICO: El
cuadro clínico, generalmente se caracteriza por la presencia de sangrado
vaginal anormal (75%), frecuentemente posterior a la menopausia, en etapa
temprana. Otros síntomas son: dolor pélvico, piometra y hematómetra. Deben incluirse en un programa de escrutinio a partir de los 45
años a las pacientes con 2 ó más factores de riesgo dentro de los cuales se encuentran: Diabetes mellitus, obesidad, exposición crónica
a estrógenos sin oposición, uso de tamoxifeno. De manera similar, las mujeres posmenopáusicas que presenten sangrado transvaginal,
se les debe realizar biopsia endometrial preferentemente mediante histeroscopía. En mujeres consideradas de alto riesgo por: historia
familiar de cáncer de colon no polipósico y pacientes con síndrome de Lynch tipo II, deben iniciar un programa de escrutinio a los 35
años. Lo mismo que pacientes con antecedentes de cáncer de mama, tiroides y ovario. Estudios de extensión: La evaluación de la
paciente incluye examen clínico completo y exploración ginecológica incluyen do tacto recto-vaginal para evaluar los parametrios y el
tabique rectovaginal. Ultrasonido de alta resolución preferentemente transvaginal (especialmente en pacientes en las que se realizó la
biopsia endometrial sin ultrasonido previo). Citología cervicovaginal, histeroscopía con toma de biopsia dirigida para determinar el
tamaño y localización del tumor (afección a istmo, canal cervical). Además de lo anterior, es necesario incluir: Radiografía de tórax.
Estudios de laboratorio: biometría hemática, química sanguínea de 26 elementos, tiempo parcial de tromboplastina (TPT), tiempo de
pro trombina (TP), INR y examen general de orina. En histología no endometrioide: se solicita CA-125. Tomografía computada para
determinar la extensión de la enfermedad en pacientes con evidencia de cáncer de endometrio órgano no confinado. Resonancia
magnética para evaluar invasión miometrial y afectación del estroma del cérvix. Manejo de padecimientos concomitantes y valoración
preoperatoria. Gradación histológica: Grado Arquitectural: G1: No más del 5 % del tumor constituido por masas sólidas. G2: Entre el 6 y
50 % del tumor constituido por masas sólidas. G3: Más del 50 % del tumor constituido por masas sólidas. Grado Nuclear; G1: Núcleo
oval. Cromatina uniformemente distribuida. G2: Núcleos con características intermedias entre G1 y G3. G3: Núcleos de gran tamaño,
pleomórficos, cromatina irregular, nucleolos eosinófilos. TRATAMIENTO: la cirugía de cáncer endometrial debe realizarse para; la
estadificación y tratamiento primario. En el momento de la cirugía estadificadora debe realizarse estudio transoperatorio (ETO) de
útero en forma rutinaria. La cirugía implica la realización de laparotomía exploradora y estadificadora para cáncer de endometrio la cual
debe incluir: 1) lavado peritoneal y 2) histerectomía extrafascial con salpingo-ooforetctomía bilateral; linfadenectomia pélvica bilateral
y para-aortica. En caso de estirpe de células claras, serosa-papilar o indiferenciados, se debe realizar además omentectomia y toma de
biopsias peritoneales y de cúpula diafragmática. Las modalidades terapéuticas son: 1) Cirugía. 2) radioterapia (adyuvante, indicada en
estadios de riesgo intermedio o alto de recurrencia). 3) Quimioterapia (doxorrubicina/cisplatino=AP,
cisplatino/doxorrubicina/ciclofosfamida=CAP, cisplatino/epirrubicina/ciclofosfamida=CEP). 4) Hormonoterapia (manejo conservador en
mujeres con deseo de fertilidad menores de 40 años y valorando los demás factores). ENFERMEDAD RECURRENTE: Evaluar cirugía de
salvamento (incluyendo la posibilidad de exenteración pélvica) en caso de recaída pélvica. Se utilizarán los esquemas de QT (a base de
platino por cuatro a seis ciclos: Paclitaxel + carboplatino cada 21 días o Doxorubicina + cisplatino cada 21 días) y/u hormonoterapia (a
base de acetato de medroxiprogesterona 200mg diarios, reservado para tumores de bajo grado y con receptores progestacionales
positivos), siempre que no sean candidatas a cirugía o RT. Al igual que ocurre con el cáncer endometrial avanzado, la enfermedad

CURSO ENARM CMN SIGLO XXI TEL: 36246001 Pharmed Solutions Institute PÁGINA 295
MANUAL DE TRABAJO DEL CURSO ENARM CMN SIGLO XXI
neoplásica endometrial recurrente también ha sido tratada con radioterapia y quimioterapia, pero existen estudios que sugieren que la
cirugía radical en pacientes seleccionadas puede mejorar la supervivencia a largo plazo. Como se ha demostrado en el carcinoma de
ovario, las pacientes con cáncer endometrial recurrente en quienes se consigue una resección quirúrgica completa, se beneficiarán de
una mayor supervivencia global. La exenteración pélvica, por lo general reservada para pacientes con recurrencia central de carcinoma
de cérvix, ha sido también aplicada en pacientes con carcinoma endometrial recurrente. SEGUIMIENTO: 1) Los primeros tres años
(Cada tres o cuatro meses). Evaluación clínica con examen físico y ginecológico. Citológico vaginal. Laboratorios: Ca 125 en variedad no
endometrioide. Gabinete: Tele de tórax (PA y/o lateral) semestral. Tomografía computada abdminopélvica en caso de sospecha de
recurrencia y de forma anual en pacientes de alto riesgo. 2) Después del tercer año (Cada seis meses).Evaluación clínica con examen
físico y ginecológico. Citológico vaginal. Laboratorios: Ca 125 en variedad no endometrioide. Gabinete: Tele de tórax (PA y/o lateral)
semestral. Tomografía computada abdominopélvica en caso de sospecha de recurrencia y de forma anual en pacientes de alto riesgo. 3.
Después quinto año (Cada año). Mismos estudios. PRONOSTICO: Los factores pronósticos desfavorables son la presencia de variedad
no endometrioide, invasión vas cular y linfática, tumores G 3, penetración mayor al 50% y pacientes mayores de 70 años.

CASO CLINICO
Se trata de paciente de 60 años de edad la cual refiere sangrado transvaginal, refiere que hace un par de meses inicio con manchado
transvaginal, el cual se convirtió en sangrado franco, la paciente cuenta con antecedentes de inicio de vida sexual activa a la edad de 14
años, menarca a los 9 años, gesta 5 para 3 abortos 2, además de 3 parejas sexuales, es hipertensa desde hace 10 años con adecuado
control, diabetes mellitus desde hace 8 años y dislipidemia. A la exploración física se observa paciente con leve palidez de tegumentos,
con índice de masa corporal de 31, signos vitales de TA 135/95, FC 83, FR 21, normotermica, torax sin fenómenos agregados, abdomen
depresible, no doloroso al tacto vagina bimanual utero de características normales, no doloroso, con presencia de manchado del
guante.

PREGUNTA
Cuál es la conducta a seguir más adecuada.

RESPUESTA
a.- Realizar USG.
b.- Indicar progestágenos.
c.- Realizar biopsia.
d.- Realizar legrado hemostático.

CASO CLINICO
Se trata de paciente de 60 años de edad la cual refiere sangrado transvaginal, refiere que hace un par de meses inicio con manchado
transvaginal, el cual se convirtió en sangrado franco, la paciente cuenta con antecedentes de inicio de vida sexual activa a la edad de 14
años, menarca a los 9 años, gesta 5 para 3 abortos 2, además de 3 parejas sexuales, es hipertensa desde hace 10 años con adecuado
control, diabetes mellitus desde hace 8 años y dislipidemia. A la exploración física se observa paciente con leve palidez de tegumentos,
con índice de masa corporal de 31, signos vitales de TA 135/95, FC 83, FR 21, normotermica, torax sin fenómenos agregados, abdomen
depresible, no doloroso al tacto vagina bimanual utero de características normales, no doloroso, con presencia de manchado del
guante.

PREGUNTA
Cuál es la conducta a seguir más adecuada.

RESPUESTA
a.- Realizar USG.
b.- Indicar progestágenos.
c.- Realizar biopsia.
d.- Realizar legrado hemostático.

CURSO ENARM CMN SIGLO XXI TEL: 36246001 Pharmed Solutions Institute PÁGINA 296
MANUAL DE TRABAJO DEL CURSO ENARM CMN SIGLO XXI
SINDROME DE OVARIO POLIQUISTICO (SOP). CIENCIAS BASICAS: El SOP es una de las alteraciones endocrinas más comunes en la
mujer, en edad reproductiva y es la principal causa de esterilidad por anovulación. Representa un trastorno heterogéneo que se
distingue por una combinación de irregularidades menstruales, hirsutismo o acné y obesidad, que suele diagnosticarse en la
adolescencia, pero aparentemente tienen su origen en la vida intrauterina. Actualmente la relación entre SOP y síndrome metabólico
(obesidad, hipertensión, dislipidemia y disglucosis), y subsecuente diabetes mellitus tipo II, lo convierten en un grave problema de
salud pública con un alto coste económico. SALUD PUBLICA: Tiene una prevalencia de 4-8%. Tan solo en Estados Unidos se considera
entre 7-10 millones de mujeres afectadas por este padecimiento. Se ha asociado con un riesgo mayor de cáncer de endometrio y
probablemente de glándula mamaria. El 50-65% de las pacientes con SOP son obesas y se considera que el 35-45% de ellas presentará
intolerancia a la glucosa durante una curva de tolerancia a la glucosa, con un riesgo de desarrollar DM tipo 2 del doble a lo esperado
para su edad. PATOGENIA: En la aparición de SOP existen componente sobre los que no podemos influir: preconcepcionales (genética)
y postconcepcionales (peso al nacer o exposición intraútero a andrógenos) y otros aspectos que si pueden ser modificables como son
los hábitos de vida (dieta y ejercicio). Hay evidencias que parece seguir un patrón de herencia autosómico dominante. En mujeres con
SOP, se ha descrito una alteración en el eje hipotálamo-hipófisis-ovario, que se distingue por aumento de la actividad del hipotálamo
que produce un mayor numero de pulsos de la hormona liberadora de gonadotrofinas (GnRH), lo que a su vez aumenta la hormona
luteinizante, cambiando la relación LH/FSH a favor de la primera. Al predominar la LH se sintetizan preferentemente andrógenos en el
ovario. La resistencia a la insulina puede desempeñar una función central en la causa del síndrome: el musculo esquelético es
profundamente resistente, mientras que otros tejidos, como el hipotálamo, la glándula suprarrenal y el ovario, conservan la
sensibilidad a la insulina. La hiperinsulinemia compensadora resulta en una disminución en la globulina transportadora de esteroides
sexuales (SHBG) y simultáneamente sirve como estímulo trófico en la producción de andrógenos por parte de las glándulas
suprarrenales y el ovario. La insulina también tiene efectos directos en el hipotálamo, ya que estimula el apetito y la secreción de
gonadotrofinas. Existen otros efectos importantes de la insulina que contribuyen al hiperandrogenismo: inhibición de la producción
hepática de la SHBG y de la proteína transportadora tipo de IGF-1 (1GFBP-1), lo cual aumenta las concentraciones circulantes de
andrógenos libres y produce una mayor actividad ovárica. DIAGNOSTICO: La mayoría de los datos clínicos que aparecen en pacientes
con SOP pueden formar parte de otras patologías no necesariamente relacionadas con alguna disfunción hormonal y no los hechos
aislados deben siempre sugerir el diagnóstico. La severidad o la rápida evolución debe alertar a los clínicos a la realización de un
diagnóstico diferencial más extenso. Acné: Su permanencia de los 20 años en adelante debe considerarse sospechosa. Es imperativo
interrogar acerca de irregularidades menstruales, hirsutismo o cualquier otro dato de hiperandrogenismo. En mujeres con acné se ha
encontrado datos de SOP hasta en el 45% de los casos. Hirsutismo: Se define como el crecimiento excesivo del vello corporal terminal
en mujeres, en áreas anatómicas donde el desarrollo de los folículos depende de la estimulación androgénica: Tercio proximal en cara
interna de muslos, abdomen, pecho, parte baja de la espalda y cara entre otros. El grado y la severidad se evalúan de acuerdo a la
escala de Ferriman-Galwey. Virilización: La aparición de hipertrofia clitorídea, amenorrea prolongada, aumento de la musculatura,
atrofia de los senos, hirsutismo severo y habitus masculino obliga a descartar la presencia de hiperplasia adrenal, hipertecosis o
tumores ováricos o adrenales. Irregularidad menstrual y anovulación: Las mujeres con SOP presentan grados variables de disfunción
ovulatoria, manifestada como oligomenorrea, dismenorrea y amenorrea. Incluso, frecuentemente presentan infertilidad (17.5% vs 1.3%
en normales). La relación entre el exceso de insulina y la anovulación se atribuye en primer lugar a la hiperandrogenemia y en segundo
a la estimulación del inhibidor 1 del activador del plasminógeno (PAI-1). Este inhibidor no solo regula la eliminación de los depósitos de
fibrina de los vasos sanguíneos sino que a nivel del ovario, inhibe a las colagenasas responsables de la ruptura folicular. Acanthosis
Nigricans: Se trata de una hiperplasia hiperpigmentada de la piel, la cual aparece predominantemente en el cuello y en pliegues
cutáneos como axilas y codos. La importancia de su detección radica en que su presencia correlaciona significativamente con los
estados de resistencia a la insulina e hiperinsulinemia compensatoria. Dependiendo de las poblaciones estudiadas aparece ligada a
obesidad hasta en el 74% de los casos constituyéndose como un factor de riesgo independiente para DM tipo 2. Aproximadamente el
30% de las pacientes con SOP lo presentan. El SOP es un diagnóstico de exclusión y no es necesaria la presencia de alteraciones en los
niveles hormonales o quistes en los ovarios para considerar su presencia. Escrutinio: Determinación en ayunas y durante los primeros 7
días del ciclo de: a) LH y FSH en pool (el promedio de 3 muestras recolectadas con 20 minutos de diferencia) y Prolactina. b)
Testosterona total y libre. c) Dehidroepiandrosterona sulfato y 17 alfahidroxiprogesterona. En el 66% de los pacientes se documenta
una relación LH:FSH (>2.5:1), la cual es característicade SOP. La prolactina debe ser determinada en la evaluación de cualquier paciente
con amenorrea; muchas de las causas de hiperandrogenismo pudieran incrementarla pero valores >100 sugieren patología hipofisiaria.
La testosterona total es la prueba más validada para evaluar la presencia de tumores secretores de andrógenos: niveles >150 ng/dL
fuertemente sugieren la posibilidad de tumores ováricos o adrenales. Cuando la paciente desea procrear es fundamental determinar si
los ciclos son ovulatorios. Los niveles de progesterona <2 ng/mL después del día 21 del ciclo son interpretados como anovulación.
Estudios de Imagen: La ultrasonografía (USG) pélvica es el método de elección para la evaluación inicial de los anexos.
Característicamente se trata de folículos de 8 mm con aumento del estroma central, aunque con la técnica vaginal pueden detectarse
quistes de 3-5 mm. Los quistes ováricos aparecen irregularmente en las pacientes con SOP. El estudio de imagen de elección para el
estudio de las suprarrenales es la tomografía axial computarizada de alta resolución. TRATAMIENTO: Pérdida de peso. La obesidad es
prevalente en mujeres con SOP, y se asocia con un empeoramiento de los síntomas. La pérdida de peso de un 5-7% con respecto al
basal produce un descenso en la concentración circulante de andrógenos, insulina y lípidos, lo cual se acompaña de una mejoría de la
sintomatología y de las posibilidades de presentar ciclos ovulatorios. Tratamiento hormonal. Los anticonceptivos hormonales
combinados proveen varios beneficios en las pacientes con SOP y durante mucho tiempo han sido la piedra angular del tratamiento.
Restituyen los ciclos menstruales con eficiencia y mejoran el hirsutismo en más del 60% de los casos por su efecto inhibidor de la LH, lo
que aumenta los niveles de SHBG. También suprimen el metabolismo de los andrógenos en las adrenales y disminuyen el número de
receptores de la 5a-reductasa en la piel, lo que beneficia al acné. En general, las progestinas protegen al endometrio oponiéndose a los
efectos proliferativos de los estrógenos pero tienen un efecto negativo en los parámetros metabólicos ya que aumentan la resistencia a
la insulina y los niveles de triglicéridos. Agentes sensibilizadores de la insulina. Está documentado que la disminución en los niveles de
insulina mediante el uso de este grupo de fármacos se acompaña de una mejoría del cuadro clínico y de las anormalidades metabólicas.
Metformina: Actúa principalmente en el hígado inhibiendo parcialmente la gluconeogénesis, lo que reduce en un 17-25% la producción
de glucosa y en el músculo esquelético donde incrementa la captación de glucosa estimulada por insulina en un 29% y disminuye la

CURSO ENARM CMN SIGLO XXI TEL: 36246001 Pharmed Solutions Institute PÁGINA 297
MANUAL DE TRABAJO DEL CURSO ENARM CMN SIGLO XXI
oxidación de los ácidos grasos en un 10-20%. En mujeres con SOP el metformín ha demostrado inducir la ovulación y mejorar las
posibilidades de lograr un embarazo, incluso en pacientes sometidas a fertilización in vitro. La combinación de metformín con
anticonceptivos orales o acetato de ciproterona, se ha acompañado de una mejoría en los parámetros clínicos y metabólicos.
Tiazolidinedionas: En pacientes con SOP la troglitazona, el primer medicamento del grupo y el más estudiado (actualmente no
disponible para su venta en México) mejora la resistencia a la insulina disminuyendo la dehidroepiandrosterona sulfato, la testosterona
libre, la androstenediona y la LH, e incrementando la proteína transportadora de hormonas sexuales. Tratamiento de la anovulación. El
citrato de clomifeno, solo o en combinación con la pérdida de peso, sigue siendo, dada su seguridad y simplicidad, el tratamiento de
primera elección en la infertilidad de origen anovulatorio asociado al SOP. Acetato de ciproterona, para tratamiento de hirsutismo.
Agonistas de la GnRh. Los agonistas de la GnRH como el leuprolide, disminuyen la producción ovárica de esteroides por supresión de LH
y FSH. Este tratamiento es altamente efectivo en mujeres con SOP severo o hipertecosis ovárica. Tratamiento Quirúrgico. La resección
en cuña bilateral de los ovarios es un procedimiento quirúrgico que se realizó con éxito en pacientes con SOP durante muchos años.
Esto provoca una reducción en los niveles de LH y producción de andrógenos.

CASO CLINICO
Acude a consulta femenino de 23 años de edad que presenta falta de periodo menstrual y desea saber si esta embarazada, la paciente
se dedica a su casa, vive y tiene relaciones con su pareja desde hace 2 años y quiere tener su primer hijo, refiere que nunca ha sido
regular, a la exploración física se observa con una talla de 156 cm y peso de 74 kg, se observa acné en cara y espalda con hirsutismo, se
observa obscurecimiento de cuello y axilas.

PREGUNTA
Cuál es la causa más probable de la amenorrea en este caso.

RESPUESTA
a.- Hiperandrogenismo.
b.- Incremento de progesterona.
c.- Disminución de estrógenos.
d.- Aumento de gonadotrofinas.

CASO CLINICO
Se trata de femenino de 27 años de edad la cual cursa con esterilidad primaria de dos años de evolución, sin diagnostico definitivo aun,
acude a la consulta debido a que se han acentuado sus síntomas, la cual refiere presencia de periodos menstruales irregulares, a la
exploración usted observa hirsutismo, acné en frente y espalda, obesidad con datos de acantosis nigrans.

PREGUNTA
Envía a la paciente para realizar estudios de imagen, cual es el diagnostico que espera encontrar más probablemente:

RESPUESTA
a.- Hiperplasia Suprarrenal.
b.- Sindrome de Silla Turca Vacia.
c.- Sindrome de Ovarios Poliquisticos.
d.- Sindrome de Cushing.

CANCER OVARICO. CIENCIAS BASICAS: Entre los cánceres del tracto genital femenino, el de ovario es el tercero en frecuencia por
detrás del cáncer de cérvix y de endometrio, pero es responsable de casi la mitad de las muertes por cáncer genital, debido a la
dificultad para el diagnóstico precoz. El ovario puede originar tumores histológicamente muy diferentes y siempre ha sido un problema
su clasificación porque la patogenia está menos clara que en otros tumores. Factores de riesgo: Adquieren cada vez más importancia
los antecedentes familiares y las mutaciones hereditarias como las del BRCA1 y BRCA2, igualmente se ha descrito concentraciones
elevadas del oncogén HER2/neu y mutaciones en el p53, hiperestrogenismo, uso de anticonceptivos, disminuye con la paridad. SALUD
PUBLICA: Los tumores ováricos constituyen el tercer grupo de tumores en la mujer. Aproximadamente 1 de cada 10 féminas lo tendrá a
lo largo de la vida, la mayoría de las cuales requerirá de una evaluación quirúrgica. Estos se presentan desde edades tempranas hasta
avanzadas. La experiencia clínica revela la alta incidencia en la etapa del climaterio, comprendida entre los 35 y 65 años de edad.
CLASIFICACIÓN: La OMS clasifica los tumores ováricos de acuerdo con el tejido de origen. Ver cuadro anexo. Es necesario puntualizar
que los tumores “borderline” o tumores en el límite de malignidad histológica, son formas tumorales con entidad propia, no un estadio
evolutivo. Histológicamente se diferencian de los benignos por: estratificación epitelial, aumento de la actividad mitótica, atípia nuclear
con ausencia de invasión estromal, y porque pueden metastatizar. PATOGENIA: En los ovarios pueden desarrollarse tumores benignos
o malignos, quísticos o sólidos, de tipo epitelial, conjuntivo, funcionantes o no o teratomatosos. Los tumores más frecuentes no son
neoplásicos, sino que derivan del desarrollo folicular; son formaciones quísticas originadas por un estímulo anormal del folículo o
alteraciones en el proceso de involución. Los verdaderos procesos neoplásicos presentan una constitución histológica muy diversa, de
origen epitelial, le continúan los procedentes del estroma gonadal y, finalmente, los tumores de las células germinales. Se conoce que
los tumores benignos del ovario no constituyen un grupo bien definido y algunos de ellos pueden malignizarse en su evolución.
Aproximadamente de 75 a 85 % de los tumores son en principio benignos. Hoy día, los factores causales del cáncer de ovario son
desconocidos, aunque se plantea que en el ámbito celular es el resultado de la acumulación de múltiples defectos genéticos menores.
Los estudios epidemiológicos han permitido enunciar 3 teorías con respecto a las causas de esta enfermedad: La división celular
ininterrumpida y la continua regeneración del epitelio ovárico con cada ovulación proporcionan la oportunidad para una mutación y
transformación maligna por la estimulación de los ovarios y por hormonas gonadotrópicas, que de forma mantenida pueden inducir a la
malignización. La exposición del ovario a diferentes agentes carcinogenéticos. Los eventos moleculares que dirigen el desarrollo del

CURSO ENARM CMN SIGLO XXI TEL: 36246001 Pharmed Solutions Institute PÁGINA 298
MANUAL DE TRABAJO DEL CURSO ENARM CMN SIGLO XXI
cáncer epitelial de ovario son desconocidos. A pesar de esto, en investigaciones epidemiológicas se han identificado algunos factores
causales en la carcinogénesis del cáncer de ovario: factores endocrinos, ambientales y genéticos, aunque se incluyen otros, tales como:
nuliparidad, historia familiar, menarquia temprana, menopausia tardía, raza blanca y aumento de la edad. La incidencia específica
aumenta según la edad de la paciente, con un pico en la sexta década de la vida. Es infrecuente en mujeres menores de 40 años. El
riesgo de que se presente esta enfermedad disminuye con la paridad, con el uso de contraceptivos hormonales y la lactancia; también
parece que la histerectomía o la ligadura de trompas con conservación de los ovarios disminuyen la posibilidad de aparición. El cáncer
de ovario con historia familiar constituye apenas 2,5 % del total. Al respecto, se han identificado 3 tipos de cánceres hereditarios, a
saber: 1. Cáncer en sitio específico: Existe alto riesgo si hay cáncer en 2 familiares de primer grado (50 % de probabilidades de ser
afectado). 2. Síndrome de cáncer familiar mama-ovario: Patrón de transmisión dominante; asociado a los cromosomas 16q y 17q. Se
considera este síndrome cuando hay 5 casos de cáncer de mama y ovario en la familia, incluyendo 2 de cada uno de estos 3. Cáncer
colorrectal no polipósico (síndrome de Lynch de tipo II): Incluye múltiples adenocarcinomas, cáncer de colon familiar, de ovario,
endometrio, mama, así como de los sistemas gastrointestinal y urinario. DIAGNOSTICO: Entre los datos clínicos de sospecha figuran:
crecimiento lento del abdomen, síntomas compresivos, dolor abdominal agudo o subagudo, sobre todo cuando se complican por
torsión (frecuente en los benignos), rotura, hemorragia o infección, rara vez datos indirectos de actividad especial endocrina o
metabólica, tales como: pubertad precoz, hemorragia uterina disfuncional, amenorrea, hirsutismo, tirotoxicosis, síndrome de Cushing,
galactorrea, policitemia, hipoglucemia e hipercalcemia, por citar algunos. Por lo general, los síntomas que más refieren las pacientes
incluyen cierto malestar abdominal, seguido de distensión abdominal, a causa de la presencia de ascitis o una masa tumoral que
paulatinamente aumenta de tamaño, así como también síntomas gastrointestinales (como náuseas, dispepsias, saciedad precoz y
constipación); los urinarios son menos frecuentes, al igual que la metrorragia. Estos síntomas tardíos ocurren con más frecuencia en
mujeres premenopáusicas. Es difícil de diagnosticar en etapas tempranas, pues las manifestaciones clínicas son escasas y fácilmente se
confunden con las de otras afecciones, además, no es fácil crear un sistema de detección precoz (tamizaje).

CASO CLINICO
Se trata de paciente femenino de 51 años de edad la cual es originaria del estado de México, comerciante ambulante, cuenta con
antecedentes de familiares con cáncer que ya murieron, no especifica qué tipo de cáncer, acude a consulta debido a que desde hace
varios meses ha presentado pérdida de peso de más de 15 % total, agrega fatiga, malestar generalizado, refiere que se realiza detección
oportuna de cáncer mediante papanicolao hace dos años sin datos de importancia y detección de cáncer de mama sin datos
patológicos, acude a urgencias debido a que se encuentra con distención abdominal importante, se realiza paracentesis donde se
encuentra datos sugerentes de adenocarcinoma, se realiza rastreo tomografico de mama y pelvis sin encontrar datos patológicos, sin
embargo se encuentra CA-125 elevada, posteriormente se toma biopsia dirigida a ovarios con reporte de patología positivo para cáncer
ovárico.

PREGUNTA
Cuál es la conducta más apropiada para seguir.

RESPUESTA
a.- Cirugia citorreductora.
b.- Cirugia citorreductora mas cisplatino y paclitaxel.
c.- Mastectomia bilateral y ooforectomia bilateral.
d.- Mastectomia y ooforectomia bilateral mas tamoxifen y radiación.

CURSO ENARM CMN SIGLO XXI TEL: 36246001 Pharmed Solutions Institute PÁGINA 299
MANUAL DE TRABAJO DEL CURSO ENARM CMN SIGLO XXI
MASTOPATIA FIBROQUISTICA (MFQ). CIENCIAS BASICAS: También conocida como enfermedad fibroquística de la mama, cambios
fibroquísticos, displasia mamaria y mama nodular dolorosa, es una entidad clinicopatológica crónica no maligna, que se evidencia como
una zona dura, debido a la proliferación del tejido conectivo, del epitelial y a la presencia de quistes, de forma focal o difusa. Consisten
en abultamientos circunscritos a la mama que pueden confundirse con neoplasias. La diferencia es que son bilaterales y tienen
variaciones en su tamaño de acuerdo con la fecha del ciclo en que se examinan; su volumen es mayor en etapa premenstrual. La
naturaleza precancerosa de la MFQ es muy discutida, aceptándose que su transformación maligna es baja. La MFQ simple no tiene
riesgo de malignización; sí lo tiene la mastopatía proliferativa con atipia. SALUD PUBLICA: Entre un 45-85% de las pacientes que acuden
a una consulta de mama lo hacen por este cuadro clínico. Constituye el proceso benigno más frecuente de la mama. Produce síntomas
incapacitantes en el 5-10% de las pacientes. Se observa, clínicamente, en un 50% de las mujeres adultas y se detecta en un 90% de los
estudios histológicos. Representa la alteración más común evidenciada en la mama de la mujer durante la madurez sexual. PAOGENIA:
En el origen de la mastopatía fibroquística se involucran factores hormonales, como: desequilibrio estrógeno-progesterona, con
concentraciones en exceso de estrógenos y disminución en las concentraciones de progesterona, y concentración elevada de
prolactina en la sangre. DIAGNOSTICO: Clínicamente, se manifiesta con molestias en una o ambas mamas y con dolor a la palpación. El
dolor suele ser uni o bilateral, generalmente de carácter cíclico, premenstrual. Otros síntomas son nodularidad y/o induración de forma
variable y secreción por el pezón; la telorrea serosa suele aparecer en la MFQ y en los quistes simples y suele ser bilateral y pluriorificial,
igual que la de aspecto lechoso. La sintomatología es frecuente que mejore con el embarazo y lactancia y también tras la menopausia.
A la exploración, se palpa un engrosamiento en forma de placa o múltiples irregularidades en una parte de la mama, sobre todo en
cuadrantes superiores y externos, normalmente bilateral. Si hay un nódulo dominante, se presenta de forma regular, borde y superficie
definidos y con poca movilidad. Los síntomas en pacientes con mastopatía fibroquística coinciden en mastodinia y nódulos no
adheridos en ambas mamas, que aumentan de volumen en etapa premenstrual. Para su diagnóstico, además de la exploración y
clínica, contamos con las pruebas de imagen, cuyo objetivo fundamental es confirmar o descartar la existencia de una lesión maligna.
Las lesiones de mama se clasifican mediante el sistema I-RADS, que caracteriza los hallazgos de la imagen en varias categorías que van
desde la normalidad hasta la confirmación histológica de malignidad. Se considera el idioma universal en el diagnóstico de la patología
mamaria. Este sistema fue disenado ˜ en principio para clasificar las lesiones
detectadas mediante mamografía; actualmente se han incorporado la ecografía
y la RM a esta clasificación. Las pruebas de imagen son: Mamografía: zonas
densas, difusas y con quistes. Los nódulos descritos (BI-RADS 2) son los quistes
simples demostrados en el estudio ecográfico. La placa fibrosa se manifiesta en
el estudio mamográfico como una densidad focal asimétrica de márgenes mal
definidos ( I-RADS 3), precisando de la ecografía para completar el estudio.
Ecografía: lo más habitual es que nos podemos encontrar en la MFQ imágenes
de similares características ecográficas que en las tumoraciones quísticas,
diferenciándose en su número y en su diámetro máximo. Atendiendo a su
diámetro máximo, dividimos la MFQ en: Tipo I (de pequeñas formaciones):
tumoraciones quísticas de diámetro < 0,5 cm. Tipo II (de medianas formaciones):
entre 0,5---2 cm. Tipo III (de grandes formaciones): > 2 cm. Como todas las
tumoraciones quísticas, ecográficamente se presentan redondeadas u ovaladas,
con ecoestructura generalmente anecoica (BI-RADS 2), aunque pueden presentar
otros aspectos como hipoecogénico con finos ecos internos, de contenido turbio
o espeso, a veces con finos tabiques en su interior (BI-RADS 3). Sus bordes están bien definidos, su eje mayor suele ser horizontal y, con
gran frecuencia, están presentes los signos indirectos (refuerzo acústico posterior y sombra acústica lateral), pudiendo ser, a veces,
bilobuladas. No tienen ninguna señal Doppler interna. En muchas ocasiones, tras el estudio ecográfico, se modifica la categoría I-RADS
previamente asignada por mamografía. Está indicada la realización de una ecografía mamaria ante la presencia de una alteración
mamográfica para una caracterización más precisa, como es la presencia de una densidad asimétrica focal que, cuando no es palpable,
la lesión es categoría BI-RADS 3. También, ante la evidencia de una distorsión arquitectural en el seno de áreas densas de tejido
mamario, catalogada como BI-RADS 4. En general, se puede afirmar que la RM tiene una alta sensibilidad y baja especificidad en la
evaluación de las lesiones mamarias. También se dispone de las pruebas diagnósticas intervencionistas, como la punción-aspiración con
aguja fina (PAAF)/biopsia con aguja gruesa ( AG): fibrosis, proliferación epitelial, adenosis, quistes y, en ocasiones, hiperplasia
intraductal o lobulillar típica o atípica. Ante un cuadro clínico---eco y radiológico dudoso (BI-RADS 3) o sugerente de malignidad (BI-
RADS 4 y 5), hay que hacer una PAAF y/o BAG como primera medida. La MFQ asintomática no requiere seguimiento; la sintomática se
seguirá con ecografía y/o mamografía. TRATAMIENTO: Los AINES tópicos en gel como el piroxicam o diclofenaco al 2% deben ser
considerados para el control localizado del dolor en el tratamiento de la mastalgia. Debe iniciarse con el uso de preparados
hormonales existentes para el tratamiento sintomático, tales como los progestágenos (de uso tópico o por vía oral o vaginal),
antiprolactínicos (bromocriptina), antiestrógenos, fundamentalmente el tamoxifeno, medicamento no autorizado en nuestro país para
el tratamiento de la mastalgia pero sí en otros países, y antigonadotropos (danazol). Si dichos preparados fracasan, se adoptarán otras
diversas medidas paliativas. Actualmente, se propone alfa dihidroergocriptina por su alta efectividad, mínimas reacciones secundarias y
fácil dosificación. El alfa dihidroergocriptina es un derivado hidrogenado del alfa ergocriptina, que es un alcaloide natural del Ergot
utilizado en el tratamiento de la hiperprolactinemia. Tiene un efecto dopaminérgico significativo en el cuerpo estriado y en la hipófisis
y es capaz de disminuir las concentraciones séricas de prolactina mediante la inhibición de la adenilato-ciclasa dependiente de los
receptores dopaminérgicos D2 en los lactotropos. PREVENCION: La edad recomendable para el inicio del examen medico mamario
rutinario es a partir de los 19 años. Los pacientes con BCRA1 y BCRA2, deben inicar la exploración médica mamaria entre 18-21 años de
edad. La autoexploración aumenta la probabilidad de que la mujer encuentre anormalidad en las mamas.

CURSO ENARM CMN SIGLO XXI TEL: 36246001 Pharmed Solutions Institute PÁGINA 300
MANUAL DE TRABAJO DEL CURSO ENARM CMN SIGLO XXI
CASO CLINICO
Una mujer de 43 años de edad con un historial de cambios fibroquísticos de la mama, que se presentan con una masa palpable en la
mama derecha. No tenía antecedentes familiares de cáncer de mama. El examen clínico reveló una masa de 4x3 dolorosa, movil, bien
circunscrita en el cuadrante inferior interno de la mama derecha. La mamografía mostró una masa bien delimitada con pared
engrosada. En la ecografía, la masa era hipoecoica y claramente definidas con pared gruesa e irregular. La lesión fue extirpada. El
examen macroscópico mostró una lesión quística bien delimitada de 3.3x1.7 cm de tamaño, con aspecto gelatinoso espeso de material
homogéneo, de color amarillo oscuro en su lumen. El espesor de la pared del quiste fue de aproximadamente 3 mm, y la superficie
interior tenía plegamientos papilares reminiscentes de papiloma quística.

PREGUNTA
Cuál es la diagnostico mas probable?

RESPUESTA
a.- Galactocele.
b.- Fibroma.
c.- Lipoma.
d.- Fibrosarcoma.

CANCER DE MAMA. CIENCIAS BASICAS: Es un padecimiento crónico, heterogéneo con una evolución irregular y lenta. Es la primera
causa de muerte por neoplasia en la mujer en el ámbito mundial. SALUD PUBLICA: Cerca de 500 mil muertes cada año a nivel mundial,
de las cuales el 70% ocurre en países en desarrollo. Las tasas de incidencia tienen una variación considerable en todo el mundo; las
más altas se encuentran en Europa y Norteamérica con cifras estandarizadas de 99.4 por 100 mil mujeres. En cuanto a la magnitud
actual del cáncer de mama en México, a partir de 2006 éste ocupa el primer lugar de mortalidad por tumor maligno en las mujeres
mayores de 25 años, desplazando de esa posición al cáncer cervicouterino. En el año 2010 la tasa estandarizada de mortalidad fue de
18.7 por 100 mil mujeres de 25 y más años, lo que representa un incremento del 49.5% en los últimos 20 años. PATOGENIA: Los
factores de riesgo para el desarrollo del cáncer de mama son los siguientes: A) Biológicos: Sexo femenino, envejecimiento: A mayor
edad mayor riesgo, antecedente personal o familiar de cáncer de mama en madre, hijas o hermanas, antecedentes de hallazgos de
hiperplasia ductal atípica, imagen radial o estrellada, así como carcinoma lobulillar in situ por biopsia. Vida menstrual mayor a 40 años
(menarca antes de los 12 años y menopausia después de los 52 años), densidad mamaria, ser portador conocido de los genes BRCA1 o
BRCA2. B) Iatrógenos o ambientales: Exposición a radiaciones ionizantes, principalmente durante el desarrollo o crecimiento (in utero,
en la adolescencia), tratamiento con radioterapia en tórax. C) Factores de riesgo relacionados con los antecedentes reproductivos:
Nuliparidad, primer embarazo a término después de los 30 años de edad, terapia hormonal en la perimenopausia o posmenopausia por
más de cinco años. D) Factores de riesgo relacionados con estilo de vida: Alimentación rica en carbohidratos y baja en fibra, dieta rica
en grasas tanto animales como ácidos grasos trans, obesidad, principalmente en la posmenopausia, sedentarismo, consumo de alcohol
mayor a 15 g/día, tabaquismo. DIAGNOSTICO: El cáncer de mama en etapas iniciales se presenta de manera subclínica en la mayoría
de los casos, es decir que solamente es detectable por estudios de imagen (mastografía, ultrasonido y resonancia magnética), en
menor proporción por clínica (tumores palpables); sin embargo otra forma de presentación común es como un tumor no doloroso que
hasta en 30% se asocia a adenopatías axilares. Los tumores localmente avanzados en nuestro país representan 70% de las etapas
clínicas al diagnóstico, pueden incluir cambios cutáneos como edema, ulceración, cutánides, así como afectación de ganglios como los
supra e infraclaviculares homolaterales El carcinoma inflamatorio de la mama, un tipo de presentación poco común pero de mal
pronóstico y que por lo general progresa rápidamente, se caracteriza por una induración difusa de la mama con eritema, edema y
aumento de la temperatura local en al menos un tercio de la glándula, en la mayoría de los casos no existe una tumoración franca
palpable. Todavía menos frecuente es el diagnóstico de cáncer de mama por los síntomas de la metástasis y no por el tumor primario.
Es de suma importancia tener en cuenta dentro del abordaje diagnóstico los factores de riesgo del paciente, sus condiciones generales
y antecedentes heredo-familiares. Estudios de imagen: La mastografía es hasta ahora el mejor método de detección, tiene una
sensibilidad diagnóstica de 80 a 95%, aunque 10 a 15% de los tumores puede ser oculto sobre todo en mujeres con mamas densas (con
el uso de mastografía digital mejora la sensibilidad diagnóstica en este grupo de pacientes). El ultrasonido es en algunos casos una
herramienta complementaria para diferenciar masas quísticas de sólidas, para caracterizar lesiones benignas y malignas y como guía
para la realización de biopsias de lesiones no palpables. La imagen por resonancia
magnética (IRM) con gadolinio tiene sensibilidad diagnóstica de 94 a 100%, pero baja
especificidad (37 a 97%) y valor predictivo positivo de 44 a 96%.Las indicaciones
actuales de este estudio son: a) como estudio de detección en mujeres con alto riesgo
(como aquellas portadoras de mutaciones BRCA 1 y 2), b) búsqueda de tumores
ocultos mamarios de presentación axilar, c) mujeres portadoras de implantes o
prótesis mamarias, d) evaluación de la respuesta al tratamiento sistémico
neoadyuvante, e) evaluación complementaria para determinar multicentricidad y
bilateralidad. Biopsia: El fundamento del diagnóstico del cáncer de mama es la
confirmación histológica del mismo, para esto se prefiere la realización de biopsias de
mínima invasión con la obtención de material tisular que per mite determinar factores
pronósticos y predictivos de suma importancia en el manejo integral de las pacientes,
por ejemplo la determinación de receptores hormonales y de Her2/neu. El
procedimiento de elección es la toma de biopsias con aguja de corte (trucut) tanto en
lesiones palpables como en las no palpables; esta forma diagnóstica se asocia con una
exactitud del 98.5% (7). En lesiones no palpables, la biopsia debe ser realizada bajo la guía de algún método de imagen (ultrasonido,
mastografía, resonancia, etc). Las biopsias quirúrgicas previo marcaje (arpón, radiocoloide, etc.) están indicadas cuando no es factible el
diagnóstico mediante un procedimiento menos invasivo. En la actualidad la biopsia por aspiración con aguja fina (BAAF) generalmente

CURSO ENARM CMN SIGLO XXI TEL: 36246001 Pharmed Solutions Institute PÁGINA 301
MANUAL DE TRABAJO DEL CURSO ENARM CMN SIGLO XXI
está reservada para la confirmación de metástasis en adenopatías loco-regionales y tiene poca utilidad como método diagnóstico en la
lesión primaria. La evaluación patológica del cáncer de mama debe incluir de manera indispensable tipo histológico, grado, permeación
vascular y linfática, tamaño del tumor, márgenes, número de ganglios y tamaño de la metástasis ganglionar, estudios de
inmunohistoquímica que evalúen la presencia o no de receptores hormonales para estrógenos y progesterona, Ki67 (>14%), la
sobreexpresión del gen ErbB2 (Her2/neu) o su amplificación por FISH o CISH, además de estudios complementarios como
citoqueratinas y factores de crecimiento epidérmico, etc. ESTADIFICACIÓN: La estadificación del cáncer de mama proporciona
información respecto al pronóstico y orienta el trata miento. Los estudios de imagen en la actualidad son un complemento para evaluar
el tamaño del tumor, la presencia de los ganglios y las metástasis. Ver cuadro anexo. TRATAMIENTO: Integral y multidisciplinario, los
manejos locorregionales son cirugía y radioterapia en cualquiera de sus tres modalidades (neoadyuvante, adyuvante y paliativa) y el
tratamiento sistémico incluye la quimioterapia, la terapia endocrina y la terapia dirigida a blancos moleculares. El tratamiento
quirúrgico del tumor primario en el cáncer de mama ha pasado por múltiples modificaciones, en la actualidad se divide en cirugía
conservadora y mastectomía con sus múltiples variedades. . Las indicaciones para mastectomía incluyen: 1) contraindicación para
recibir radiote rapia; 2) enfermedad multicéntrica e 3) dificultad para obtener márgenes adecuados y resultado cosmético favorable
después de un intento de cirugía conservadora. Existen varios tipos de mastectomía, la mastectomía total extirpa la totalidad del tejido
mamario incluyendo el complejo areola-pezón (CAP), en la mastectomía total preservadora de piel se realiza una incisión circundante a
la areola y se remueve solo el CAP, la ventaja de esta modalidad es favorecer los resultados cosméticos de la reconstrucción inmediata.
La mastectomía radical modificada es un procedmiento realizado en nuestro país debido a los estadios localmente avanzados en los
que realizamos el diagnóstico en nuestras pacientes e incluye la resección de la totalidad del tejido y piel mamaria así como la disección
de los niveles ganglionares I y II, procedimientos más extensos como la mastectomía Halsted están reservados solo en casos
seleccionados. Por muchos años, la disección de la axila ha sido el estándar de tratamiento. En la actualidad la disección del ganglio
centinela es el abordaje quirúrgico de elección en los casos en que la axila es clínicamente negativa. Tratamiento Adyuvante;
quimioterapia, eliminar la enfermedad micrometastásica antes del desarrollo de clonas resistentes. La quimioterapia basada en
antraciclinas disminuye 33% las recaídas y 27% la mortalidad en mujeres menores de 50 años. Terapias biológicas: Aproximadamente
15 a 25% de los cánceres de mama tendrá sobreexpresión del gen HER2/neu (ErbB2). El Trastuzumab es un anticuerpo monoclonal
humanizado dirigido en contra de la proteína HER2. Terapia Endocrina: Entre 50 y 70% de los pacientes con cáncer de mama, el tumor
será hormono sensible por lo que se podrán beneficiar de una de las siguientes modalidades de manejo: Tamoxifén; ha demostrado
una disminución en el riesgo a recurrencia de 40% y en riesgo de muerte de 35%, así como reducción en el desarrollo de otro cáncer de
mama. Inhibidores de aromatasa: exclusivo de mujeres posmenopáusicas, ha demostrado mejorar la supervivencia libre de
enfermedad, no la global. El tratamiento con radioterapia está indicado en todos los pacientes que hayan sido sometidos a cirugía
conservadora. Tratamiento Neoadyuvante Sistémico: Esta modalidad terapéutica se ha utilizado desde hace muchos años y se
considera el estándar en los tumores localmente avanzados. Se recomienda un esquema basado en taxanos y antraciclenos durante
seis a ocho ciclos. El objetivo principal es facilitar las diferentes modalidades quirúrgicas. En términos generales, la enfermedad
metastásica puede clasificarse como visceral (pulmón, hígado y sistema nervioso central) y no visceral (incluye hueso, tejidos blandos y
derrame pleural), con supervivencia global para el primer grupo de 12 meses y para el segundo de tres a cinco años. Seguimiento:
Consiste en valoración clínica periódica (trimestral los primeros dos años, del año 3 al año 5 es semestral, y posteriormente, anual), los
objetivos son: detectar recaída local, regional o sistémica y la presencia de un segundo primario. Se debe realizar una masto grafía y
tele de tórax anuales así como densitometria ósea semestral (en posmenopáusicas o tratadas con inhibidores de aromatasa).
PREVENCION: La promoción de las conductas favorables a la salud para la prevención del cáncer de mama puede disminuir hasta en
30% la incidencia en la población. Se debe orientar a las mujeres acerca de su responsabilidad en el autocuidado de la salud, disminuir
los factores de riesgo cuando sea posible y promover los estilos de vida sanos como: Dieta rica en frutas y verduras y baja en grasas
animales. Práctica de ejercicio físico moderado, que se asocia con una disminución del riesgo de cáncer de mama en la población
general. Consumo de ácido fólico. Amamantar, por lo que debe incluirse entre las ventajas de la lactancia materna. Mantener un
adecuado índice de masa corporal pues el elevado (> 30) se asocia con un incremento significativo en el riesgo de cáncer de mama en
posmenopáusicas. La prevención secundaria para la detección del cáncer de mama debe incluir la autoexploración, el examen clínico y
la mastografía. La autoexploración se debe recomendar a partir de los 20 años; el objetivo es sensibilizar a la mujer sobre el cáncer de
mama, lograr que tenga un mayor conocimiento de su propio cuerpo y que identifique cambios anormales para la demanda de
atención médica apropiada. El examen clínico debe ser practicado anualmente a partir de los 25 años, por personal de salud capacitado
en la exploración de las mamas. Detección anual con mastografía a partir de los 40 años. Se recomienda el empleo de
quimioprevención, el empleo de tamoxifeno1-3 y exemestano se asocia con una reducción en el riesgo de carcinoma ductal invasor
Para decidir el uso de estos agentes se deben tomar en cuenta las contraindicaciones relacionadas con su empleo y los antecentes de
la paciente tales como: Historia de eventos tromboembólicos o hiperplasia atípica del endometrio en el caso de tamoxifeno o el
diagnóstico de osteopenia/osteoporosis o enfermedades cardiovasculares con el uso de exemestano. La cirugía profiláctica debe tener
un abordaje multidisciplinario considerando que es una cirugía reductora de riesgo de cáncer de mama contralateral. Los pacientes con
antecedente de radioterapia (enfermedad de Hodgkin en mediastino), susceptiblidad genética, e historia de cáncer de mama familiar
son los grupo más recomendados para este tratamiento.

CASO CLINICO
Se trata de femenino de 48 años de edad originaria del estado de puebla, con el antecedente de importancia de ser portadora de
cáncer de mama actualmente bajo tratamiento con tamoxifen, alendronato, acetato de megestrol, acude a urgencia debido a que
presenta nauseas y vomito, fatiga y cansancio generalizado que se acompaña con dolor abdominal difuso, los síntomas no se presentan
relacionados a ingesta de alimento u otras actividades cotidianas, a la exploración física se observa con un índice de masa corporal de
23, leve deshidratación de mucosas, palidez generalizada moderada, cardiorespiratorio sin compromiso aparente, abdomen plano
depresible difusamente doloroso, no hay signos de irritación peritoneal, ruidos intestinales normales, los estudios de laboratorio
revelan biometría normal, química sanguínea y electrolitos con los siguientes resultados: sodio 130 meq/L, potasio 4,8 meq/L, cloruro
98 meq/L, Bicarbonato 29 meq/l, BUN 15mg/dl, creatinina 0.7 mg/dl.

CURSO ENARM CMN SIGLO XXI TEL: 36246001 Pharmed Solutions Institute PÁGINA 302
MANUAL DE TRABAJO DEL CURSO ENARM CMN SIGLO XXI
PREGUNTA
Cuál es la conducta más adecuada a seguir para establecer un diagnostico con los datos que presenta el caso.

RESPUESTA
a.- Realizar laparoscopia.
b.- Cuantificar cortisol serico.
c.- Transito gastrointestinal.
d.- Endoscopia alta.

CANCER DE MAMA
Se trata de paciente femenino de 49 años de edad la cual acude a consulta debido a que desde hace 6 meses ha presentado dificultadas
para caminar y caídas frecuentes por lo mismo, cuenta con antecedentes de importancia como alcoholismo social crónico, tabaquismo
positivo desde los 17 años hasta la actualidad, hace 2 años fue realizada histerectomía por sangrado uterino anormal, sin especificar el
diagnostico histopatológico, a la exploración física se observa incoordinación, marcha inestables, nistagmus, disartria leve y alteración
en la prueba dedo a nariz.

PREGUNTA
Considerando el historial clínico cual es la fuente primaria mas probable en este caso.

EXPLICACION
a.- Cáncer de mama.
b.- Cáncer cervicouterino.
c.- Cáncer de colon.
d.- Cáncer pulmonar.

MENOPAUSIA Y CLIMATERIO. CIENCIAS BASICAS: Menopausia: es un evento natural, inevitable en la vida de la mujer, que lleva al cese
de la función reproductora. Menopausia literalmente significa "cese permanente de la menstruación", se debe a la pérdida de la
función folicular ovárica y el diagnóstico se hace en forma retrospectiva después de un período de amenorrea de 12 meses. El
promedio de edad de la menopausia en estudios realizados en E.U. y Europa varía entre 48 y 52 años. Cifras similares se han reportado
en África, México y Japón. El factor más importante que determina la edad de la menopausia es el número de folículos ováricos, cuyo
número es determinado en la fase temprana de la embriogénesis. Climaterio o perimenopausia: comienza varios años antes de que se
produzca la menopausia, incluye el período inmediatamente anterior a la menopausia (cuando comienzan las manifestaciones
endocrinológicas, biológicas y clínicas) como mínimo se prolonga hasta el primer año siguiente a la menopausia. Postmenopausia: se
refiere al período que comienza a partir de la menopausia, si bien este momento no se puede determinar hasta que se hayan
observado doce meses de amenorrea espontánea. Se ha utilizado el término de menopausia quirúrgica para referirse al cese de la
menstruación posterior a la histerectomía sin ooforectomía, pero se recomienda utilizarlo únicamente cuando la operación va
acompañada de la extirpación de ambos ovarios con o sin útero. SALUD PUBLICA: Debido a diferentes factores como la disminución de
la mortalidad materna y de las enfermedades infecciosas, la esperanza de vida se ha prolongado y actualmente es de más de 70 años.
Esto ha provocado un aumento del número de mujeres que llegan a la menopausia y de los años que viven como postmenopáusicas ya
que, si tomamos en consideración que la edad en que se produce aquélla es alrededor de los 50 años. PATOGENIA: Durante la
transición de la etapa reproductiva a la no reproductiva, la mujer experimenta una serie de eventos, los cuales se inician varios años
antes de que ocurra la menopausia (cese de la
menstruación). Generalmente después de los 40
años la fase folicular del ciclo se acorta, los niveles
de estradiol, inhibina y de folículoestatina
disminuyen y los de la FSH comienzan a elevarse.
Posteriormente los niveles de la hormona
luteinizante (LH) también aumentan aunque en
menor proporción que la anterior. Los folículos
disminuyen y es más frecuente la ovulación
prematura y la fase lútea insuficiente con
disminución de la producción de progesterona, lo
que produce un exceso de estrógenos en relación
a la hormona anterior. Cerca de la menopausia, los
ciclos menstruales a menudo son irregulares con
anovulación intermitente, los niveles de las
gonadotrofinas son erráticas, es frecuente el hiperestrogenismo relativo con disminución de la progesterona. Clínicamente esto se
manifiesta con sangrados vaginales irregulares, pueden existir períodos de amenorrea con valores de FSH y LH elevados, similares a los
hallados en la menopausia, pero cuando se produce la ovulación, los niveles hormonales pueden volver a límites normales. Finalmente,
los folículos ováricos dejan de responder a las FSH y LH; el estradiol desciende a menos de 20 pg/mL, la progesterona es indetectable y
clínicamente hay cese de la menstruación. Ver cuadro anexo. Después de la menopausia cambia el origen y la naturaleza del estrógeno
circulante, ya que durante la vida reproductiva predomina el estradiol y en la postmenopausia la estrona. El primero es producido por
el ovario y la segunda proviene en su mayoría de la conversión periférica de los precursores androgénicos en el tejido adiposo, músculo
e hígado. DIAGNOSTICO: El climaterio se asocia con signos y síntomas típicos relacionados con la deficiencia de estrógenos. Típicos:
Amenorrea, bochornos, crisis de sudoración, alteraciones del tracto genitourinario. Generales: Alteraciones psicológicas, alteraciones
emocionales, alteraciones psicosomáticas. Síntomas vasomotores: Los bochornos o sofocos se definen como un aumento en la

CURSO ENARM CMN SIGLO XXI TEL: 36246001 Pharmed Solutions Institute PÁGINA 303
MANUAL DE TRABAJO DEL CURSO ENARM CMN SIGLO XXI
percepción del calor dentro del cuerpo, se manifiestan por una elevación en la temperatura cutánea, vasodilatación periférica,
aceleración transitoria de la frecuencia cardíaca y modificaciones en la actividad electrodérmica. Se inician varios años antes de la
menopausia, aun en mujeres con ciclos regulares, generalmente aumentan conforme éstos se vuelven irregulares, alcanzando su
máxima intensidad y mayor frecuencia 1 a 2 años después de la menopausia y pueden persistir hasta por 5 ó 15 años después de ésta.
Los episodios nocturnos de sofocos se refieren como crisis de sudoración. Debido al mismo origen embriológico de vagina, trígono
uretral y vejiga, las vías urinarias inferiores presentan cambios atróficos por falta de estrógenos. Son comunes las quejas tanto de
síntomas urinarios (disuria, infecciones urinarias frecuentes) como los propios de vaginitis atrófica: prurito, sequedad vaginal,
dispareunia, sangrado vaginal etc. Las mujeres que no reciben tratamiento con estrógenos, al paso del tiempo pueden desarrollar
uretritis causada por atrofia de la mucosa uretral que produce disuria y vaginitis atrófica. La atrofia de la vagina produce
adelgazamiento del endotelio, pérdida de los pliegues, acortamiento de la misma y disminución de la lubricación, lo que se relaciona
con dispareunia. Antes de la menopausia las células epiteliales de la vagina son ricas en glicógeno, el cual es metabolizado por
lactobacilos, lo que contribuye a formar un ph ácido que protege contra las infecciones, posterior a la misma se pierde este mecanismo
protector y puede presentarse mayor frecuencia de infecciones así como tendencia a las ulceraciones. Síntomas psicológicos, somáticos
y emocionales: ansiedad, aumento de la tensión, cambios de humor, depresión, irritabilidad, cansancio, insomnio, dificultad para
concentrarse, ataques de pánico, cefalea, dolores articulares, palpitaciones, parestesias. Después de la menopausia, el adelgazamiento
del epitelio de la vagina no afecta la función sexual cuando hay relaciones sexuales en forma regular. En los casos de atrofia severa o
vaginitis atrófica puede existir dispareunia. Recientemente se ha sugerido que la falta de estrógenos que se produce durante la
menopausia contribuye al desarrollo de la enfermedad de Alzheimer y se ha reportado que la terapia de reemplazo con estrógenos
puede ser útil para prevenir o retardar el inicio de esta enfermedad. La osteoporosis y la enfermedad cardiovascular constituyen las
principales causas de morbimortalidad en las mujeres después de la menopausia. Varios estudios han reportado alteraciones en el
metabolismo de lípidos, principalmente disminución de los niveles plasmáticos de lipoproteínas de alta densidad (HDL-c) y aumento de
lipoproteínas de baja densidad (LDL-c) tanto en mujeres con menopausia fisiológica como quirúrgica. Después de la enfermedad
cardiovascular, la osteoporosis es el problema más significativo relacionado con la deficiencia de estrógenos. TRATAMIENTO:
Farmacológico: 1) Terapia hormonal (TH): se debe ofrecer de primera línea con estrógenos ó estrógeno/progestageno. Es la más eficaz
para el control de síntomas vasomotores (hasta en 75%) y atrofia urogenital del climaterio, así como prevención de la osteoporosis.
Esta contraindicada en las pacientes con manifestaciones clínicas de hipoestronismo en la perimenopausia o post menopausia. Terapia
cíclica: estrógenos de 1-25 días más progestágeno los últimos 10-14 días de haber iniciado los estrógenos. Ciclico-combinado:
estrógenos 1-25 días y progestágenos de 1-25 días junto con estrógenos. Continuo cíclico: estrógenos diariamente y progestágeno
durante 14 días durante 2-6 meses. Continuo combinado: estrógeno más progestágeno. Diariamente sin descanso. Intermitente
combinado: estrógenos diariamente y progestágenos 3 dias con 3 de descansos. La principal función de los progestágenos es la
protección endometrial, ya que los estrógenos solos, tienen riesgo de desarrollo de cáncer. Los estrógenos utilizados en la TH solos o
combinados más conocidos son: Orales; Estrógenos conjugados derivados de equinos simples o combinados con acetato de
medroxiprogesterona (0.625mg), estradil 1mg, etinilestradil + acetato de noretindrona. Estradiol-drosperinona. Transdermica:
estradiol-levonorgestrel, estradiol-acetato de noretindrona. Al utilizarse un progestágeno es recomendable que sea por lo menos 12-
14 días de cada mes. En México el acetato de clormadinona es uno de los progestágenos que se usan frecuentemente como terapia
combinada en el síndrome climatérico. 2) Terapia no hormonal: inhibidores de la recaptura de serotonina y norepinefrina. Clonidina,
gabapentina o veraliprida. Cuando haya contraindicación para hormonal.

CASO CLINICO
Una mujer de 48 años de edad, con menstruaciones regulares mensuales refiere frecuentes despertares nocturnos durante los últimos
3 años, seguido por una gran dificultad para volver a dormir. Estos despertares inexplicables ocurrieron 2-3 veces por noche ≥ 3 noches
a la semana. La interrupción del sueño causaba somnolencia diurna excesiva, dificultad para concentrarse en el trabajo, por lo que se
encuentra irritable. Ella había estado tomando anticonceptivos orales diariamente por cerca de 12 años. Le prescribieron durante 6
meses trazodona antes de dormir sin resultado favorable y posteriomente 4 meses de amitriptilina al acostarse ifualmente ineficaz.

PREGUNTA
Cuál es la conducta a seguir más adecuada.

RESPUESTA
a.- Terapia hormonal de remplazo.
b.- Clonacepam.
c.- Venlafaxina.
d.- Zolpiden.

CURSO ENARM CMN SIGLO XXI TEL: 36246001 Pharmed Solutions Institute PÁGINA 304
MANUAL DE TRABAJO DEL CURSO ENARM CMN SIGLO XXI
OBSTETRICIA
CONTROL PRENATAL Y DEL EMBARAZO: La mayoría de los daños obstétricos y los riesgos para la salud de la madre y del niño pueden
ser prevenidos, detectados y tratados con éxito, mediante la aplicación de procedimientos normados para la atención, entre los que
destacan el uso del enfoque de riesgo y la realización de actividades eminentemente preventivas y la eliminación o racionalización de
algunas prácticas que llevadas a cabo en forma rutinaria aumentan los riesgos. ESPECIFICACIONES: En la atención a la madre durante el
embarazo y el parto debe de vigilarse estrechamente la prescripción y uso de medicamentos, valorando el riesgo beneficio de su
administración. La unidad de atención deberá disponer de un instrumento que permita calificar durante el embarazo, el riesgo
obstétrico en bajo y alto, el cual servirá para la referencia y contrarreferencia. CONTROL PRENATAL: Las actividades son; Elaboración de
historia clínica. Identificación de signos y síntomas de alarma (cefalea, edemas, sangrados, signos de infección de vías urinarias y
vaginales). Medición y registro de peso y talla, así como interpretación y valoración. Medición y registro de presión arterial, así como
interpretación y valoración. Valoración del riesgo obstétrico. Valoración del crecimiento uterino y estado de salud del feto.
Determinación de biometría hemática completa, glucemia y VDRL (en la primera consulta; en las subsecuentes dependiendo del
riesgo), la embarazada debe ser informada a cerca del propósito de las pruebas de laboratorio. Determinación del grupo sanguíneo
ABO y Rho, (en embarazadas con Rh negativo y se sospeche riesgo, determinar Rho antígeno D y su variante débil Dµ), se recomienda
consultar la Norma Oficial Mexicana para la disposición de sangre humana y sus componentes, con fines terapéuticos. Examen general
de orina desde el primer control, así como preferentemente en las semanas 24, 28, 32 y 36. Detección del virus de la inmunodeficiencia
adquirida humana VIH en mujeres de alto riesgo (transfundidas, drogadictas y prostitutas), bajo conocimiento y consentimiento de la
mujer y referir los casos positivos a centros especializados, respetando el derecho a la privacidad y a la confidencialidad. Prescripción
profiláctica de hierro y ácido fólico. Prescripción de medicamentos (sólo con indicación médica: se recomienda no prescribir en las
primeras 14 semanas del embarazo). Aplicación de al menos dos dosis de toxoide tetánico rutinariamente, la primera durante el primer
contacto de la paciente con los servicios médicos y la segunda a las cuatro u ocho semanas posteriores, aplicándose una reactivación en
cada uno de los embarazos subsecuentes o cada cinco años, en particular en áreas rurales. Orientación nutricional tomando en cuenta
las condiciones sociales, económicas y sociales de la embarazada. Promoción para que la mujer acuda a consulta con su pareja o algún
familiar, para integrar a la familia al control de la embarazada. Promoción de la lactancia materna exclusiva. Promoción y orientación
sobre planificación familiar. Medidas de autocuidado de la salud. Establecimiento del diagnóstico integral. Con el apoyo de los datos
anteriores, se deben establecer los criterios de referencia para la atención de las gestantes a las unidades de primero, segundo y tercer
niveles. La unidad de atención debe proporcionar a la mujer embarazada un carnet perinatal que contenga los siguientes datos:
identificación, antecedentes personales patológicos, evolución del embarazo en cada consulta, resultados de exámenes de laboratorio,
estado nutricional, evolución y resultado del parto, condiciones del niño al nacimiento, evolución de la primera semana del puerperio,
factores de riesgo y mensajes que destaquen la importancia de la lactancia materna exclusiva, planificación familiar y signos de alarma
durante el embarazo. Se utilizará éste, como documento de referencia y contrarreferencia institucional. Para establecer el diagnóstico
de embarazo no se deben emplear estudios radiológicos ni administrar medicamentos hormonales. El control prenatal debe estar
dirigido a la detección y control de factores de riesgo obstétrico, a la prevención, detección y tratamiento de la anemia, preeclampsia,
infecciones cérvicovaginales e infecciones urinarias, las complicaciones hemorrágicas del embarazo, retraso del crecimiento
intrauterino y otras patologías intercurrentes con el embarazo. La unidad de salud debe promover que la embarazada de bajo riesgo
reciba como mínimo cinco consultas prenatales, iniciando preferentemente en las primeras 12 semanas de gestación y atendiendo al
siguiente calendario: 1ra. consulta: en el transcurso de las primeras 12 semanas. 2a. consulta: entre la 22 - 24 semanas. 3a. consulta:
entre la 27 - 29 semanas. 4a. consulta: entre la 33 - 35 semanas 5a. consulta: entre la 38 - 40 semanas. La prolongación del embarazo
después de las 40 semanas requiere efectuar consultas semanales adicionales con objeto de vigilar que el embarazo no se prolongue
más allá de la semana 42. DETECCIÓN DE FACTORES DE RIESGO: 1) Asociación entre exposición a sustancias toxicas y efectos adversos
en la madre o el feto (partos prematuros, hipertensión preeclampsia, bajo peso al nacer). 2) El exceso de alcohol tienen efecto adverso
en el feto, por lo que se sugiere que la mujer embarazada limite su consumo. 3) El tabaquismo se ha asociado a un incremento en el
riesgo de mortalidad perinatal, muerte súbita infantil, RPM, embarazo ectópico, placenta previa, parto prematuro, bajo peso al nacer,
desarrollo de labio y paladar hendido; por lo cual debe ser evitado. 4) El fumar marihuana se asocia a bajo peso al nacer, muerte
perinatal, parto prematuro, evitar su uso. 5) Se ha demostrado la utilidad de la suplementación con Yodo en poblaciones con alto riesgo
de cretinismo. 6) Tamizaje para preeclampsia, con toma de presión, pruebas de orinas para detectar proteinuria, capacitación para
reconocer los síntomas de alarma de preeclampsia. 7) Las vacunas con virus vicos atenuados están contraindicadas durante la gestación
y solo se recomienda el toxoide tetánico. 8) Los profesionales de la salud deben estar alertas sobre datos que orienten hacia violencia
intrafamiliar, ya que se presenta hasta en 17% en el embarazo y ello dificulta la atención de la mujer. CONTROL EN LAS CONSULTAS:
De acuerdo a guías de práctica clínica: La primera cita debe ser antes de las 12 semanas: Identificar mujeres con riesgo alto de resultado
adversos del embarazo (DM, HTA, preeclamsia, síndrome de Down, nefropatías, enfermedades de la colágena). Descartar
incompatibilidad a grupo y Rh, anemia hipocromica y megaloblasrtica; sífilis. Solicitar grupo y Rh, BH, EGO y VDRL. Detección de VIH y
hepatitis B. Búsqueda de bacteriuria asintomática y proteínas en orina. Envió de pacientes con factores de riesgo a segundo nivel.
Calcular fecha probable de parto por fecha de último periodo menstrual, en caso de duda sobre la edad gestacional solicitar
ultrasonido. Calcular y registra IMC y presión arterial. A las 16 semanas: revisar, discutir y registrar los resultados de todos los
exámenes, plantear el diseño de cuidados. El embarazo no complicado, dar nueva cita, el embarazo complicado o factores de riesgo,
enviarlas a segundo nivel. En hemoglobinas menores de 11g/dl considerar suplemento de hierro. Medir y registrar presión arterial. En
18-20 semanas: Debe realizarse un ultrasonido para detectar anormalidades estructurales. Para las pacientes en que se encuentre la
placenta cerca del OCI, realizar otro USG a la semana 36 medir y registra presión arterial. En 25 semanas en primigestas: Medir fondo
uterino, presión arterial y descartar la presencia de proteínas en orina. En semana 28: un nuevo examen para detectar anemia y células
atipias. Nivel menor a 10.5 g/dl considerar implementación de hierro. Ofrecer vacuna anti Rh de ser necesario. Medir fondo uterino,
presión arterial y descartar la presencia de proteínas en orina. Realizar tamiz de diabetes gestacional. En 31 semanas: medir fondo
uterino, presión arterial y descartar la presencia de proteínas en orina. Identificar mujeres embarazadas que requieren cuidados
especiales. En semana 34: ofrecer segunda dosis de vacuna anti Rh en mujeres Rh negativo. Medir fondo uterino, presión arterial y
descartar la presencia de proteínas en orina. En pacientes con tratamiento por anemia revisar sus estudios de laboratorio de control,
ajustar tratamiento. En embarazos con evolución normal informar fecha probable de parto con medidas preventivas para embarazo

CURSO ENARM CMN SIGLO XXI TEL: 36246001 Pharmed Solutions Institute PÁGINA 305
MANUAL DE TRABAJO DEL CURSO ENARM CMN SIGLO XXI
posmaduro y detección de preeclamsia. En semana 36: medir fondo uterino, presión arterial y descartar presencia de proteínas en
orina. Determinar la posición fetal, en caso de duda realizar USG. Mujeres con producto en presentación pélvica envió a
ginecoobstetricia. En semana 37: se debe descartar presentación anormal del producto y asegurar atención oportuna para evitar pos
madurez. En semana 38: medir fondo uterino, presión arterial y descartar la presencia de proteínas en orina. Insistir en medidas
preventivas de embarazo pos maduro. En semana 40 o más: medir fondo uterino, presión arterial y descartar presencia de proteínas en
orina, envió a urgencias GO para evaluar inducción. La ganancia de peso total durante el embarazo en una mujer sana 7-18 Kg, se
relaciona con neonatos de 3-4 kg. El incremento en el IMC se ha asociado con preeclampsia y con cesárea. El peso y la estatura se
deben medir y registrar en cada cita de atención prenatal. Se debe calcula el IMC. En cada cita se debe determinar la altura de fondo
uterino para detectar productos mayores a menores para su edad gestacional; de preferencia utilizar tablas estandarizadas y la
aplicación correcta de la técnica. Aunque la auscultación del foco fetal confirma que el producto está vivo, parece no ofrecer otros
datos; en general médicos y enfermeras suponen que es satisfactorio para el paciente. Se recomienda registrar la frecuencia del foco
fetal en cada consulta. El examen pélvico se ha utilizado para identificar condiciones clínicas como: anormalidades anatómicas, ETS,
evaluar tamaño de la pelvis de la mujer y evaluar el cérvix uterino, incompetencia cervical o para predecir parto pretermino. Sin
embargo la RPM aumenta 3 veces más cuando se realiza comparación con las mujeres en las que no se realiza exámenes pélvicos. No
se recomienda el tacto vaginal en forma rutinaria, pues no predice con exactitud la edad gestacional, nacimientos pre término o
desproporción cefalo-pelvica y si favorece la RPM.

CASO CLINICO
Se trata de femenino de 23 años de edad la cual acude a consulta por odinofagia, malestar generalizado, cefalea, mialgias y artralgia, a
la exploración física observa amígdalas hiperemicas e hipertróficas con secreción blanquecina, adenopatía cervical, refiere que se
enferma con poca frecuencia, durante el interrogatorio la paciente le menciona su interés por quedar embarazada ya que lleva casada
2 años y así lo desea, usted amplia su interrogatorio dirigido y no encuentra hallazgos de importancia.

PREGUNTA
Cuál es la mejor conducta a seguir con esta paciente para tener las mejores condiciones para tener un embarazo con adecuada
planeación.

RESPUESTAS
a.- Es conveniente indicar suplemento de Hierro y Vitaminas para prevenir defectos.
b.- Realizar estudios tales como BH, QS y EGO para descartar padecimiento no visibles.
c.- Realizar Papanicolaou y colposcopia para descartar virus del papiloma humano.
d.- Realizar serología para ETS y VIH ya que es una etapa donde se podrían evitar la transmisión al producto.

CASO CLINICO
Se trata de paciente femenino de 29 años de edad con amenorrea de 10 semanas, se realiza PIE resultando positivo, al interrogatorio la
paciente refiere nauseas, cefalea, vomito, pérdida de peso aproximadamente 5 kilos, refiere tabaquismo y consumo de alcohol
ocasionalmente, agrega que pocas ocasiones acude a consulta médica, sus antecedentes GO son Gesta 2, Para 1, Abortos 1, refiere que
su primer embarazo no fue planeado, actualmente vive con su segundo esposo, el cual consume drogas inyectables, tienen 16 meses
de relación, lo conoció en su trabajo donde labora como mesera, el esposo es originario de Tijuana, a la exploración física palpa una
masa en mama derecha la cual no es dolorosa, esta superficial, en espalda y muslos observa equimosis en proceso de resolución, la
paciente refiere que se pego accidentalmente, a la exploración vaginal se observa leucorrea con olor desagradable.

PREGUNTA
Considerando los factores de riesgo y la sintomatología observada cual es su conducta a seguir para identificar otros riesgos no visibles,
potencialmente prevenibles.

RESPUESTAS
a.- Realiza una búsqueda intencionada de ETS y VIH por los datos indirectos aportados de la paciente.
b.- Envía interconsulta a trabajo social para identificar violencia intrafamiliar ya que las lesiones que observa no se explican con la
información proporcionada.
c.- Indica suplemento de hierro y complejo vitamínico siguiendo la normatividad.
d.- Considera que no tiene información necesaria para modificar su manejo prenatal de forma distinta al de rutina.

CASO CLINICO
Se trata de paciente de 41 años de edad la cual refiere 5 meses de no presentar periodo menstrual, sin embargo decide acudir a
consulta porque tiene ardor al orinar y cefalea ocasional de intensidad leve, la paciente refiere presentar estrés laboral y posiblemente
perdida del empleo, cuenta con antecedente de Hipertensión Arterial diagnosticada a los 38 años, actualmente tratada con ARA con
buena respuesta, a la exploración física usted identifica crecimiento de pezones, a la palpación abdominal identifica presencia de útero
a la altura de cicatriz umbilical, ausculta y percibe frecuencia cardiaca fetal de 140 lx´.

PREGUNTA
Cuáles son los factores de riesgo más importantes que se deberá mantener controlado por el embarazo actual.

RESPUESTA
a.- Hipertensión arterial preexistente.
b.- Preeclapsia.

CURSO ENARM CMN SIGLO XXI TEL: 36246001 Pharmed Solutions Institute PÁGINA 306
MANUAL DE TRABAJO DEL CURSO ENARM CMN SIGLO XXI
c.- Glomerulopatia
c.- Descontrol hipertensivo.

CASO CLINICO
Paciente femenino de 21 años de edad la cual acude a consulta debido a que presenta un atraso del periodo menstrual de 4 semanas,
refiere nausea, mareo leve en dos ocasiones, cefalea pulsátil en región occipital, sensibilidad mamaria, al interrogatorio refiere IVSA a
los 20 años sin método contraceptivo, a la exploración vaginal se observa reblandecimiento del cuello uterino, a la exploración de fondo
uterino se palpa a nivel de sínfisis púbica.

PREGUNTA: Integrando signos y síntomas cual sería la edad gestacional.

RESPUESTA:
a.- 4-6 semanas de gestación.
b.- 6 a 12 semanas de gestación.
c.- 12 a 18 semanas de gestación
d.- 16 a 20 semanas de gestación.

CASO CLINICO
Acude a la consulta paciente con 10 semanas de amenorrea, refiere que se realizo una prueba de embarazo en casa, resultando
positiva, refiere nausea matutina y vomito ocasional, durante la exploración se observa crecimiento mamario y pigmentación, se
observa línea alba pigmentada, la paciente presenta tabaquismo positivo y un índice de masa corporal de 17, usted indica medidas
correctivas a estos factores de riesgo,

PREGUNTA:
Considerando los factores de riesgo cual es su conducta a seguir para mantener un buen control prenatal?

RESPUESTA
a.- Incrementa la dieta y vigila su estado nutricional para obtener un producto con peso adecuado.
b.- Indica a la paciente que definitivamente debe suspender el consumo de tabaco por el riesgo de bajo peso al nacer.
c.- Envia a la paciente a realizar BH, QS para valorar su estado general y como protocolo según la norma.
d.- Envia USG para identificar la edad gestacional y el estado del producto ya que hay riesgos para cierre de tubo inadecuado.

CASO CLINICO
Paciente originaria de Chiapas la cual actualmente vive en el estado de México, refiere tener 27 años, acude para control prenatal,
presenta 6 semanas de amenorrea, niega uso de método anticonceptivo, sus antecedentes GO, gesta 3, para 2, abortos 0, menarca a
los 12 años, IVSA a loa 19 años.

PREGUNTA
Considerando la edad gestacional y el correcto control prenatal, cual es la inmunización adecuada para esta paciente:

RESPUESTA
a.- Usted indica vacuna de influenza.
b.- Toxoide tetánico.
b.- Vacuna de Hepatitis.
c.- Vacuna DPT.

EMBARAZO ECTOPICO (EE). CIENCIAS BASICAS: Se define como la implantación del óvulo fecundado fuera de la cavidad uterina.
Representa una de las causas más importantes de abdomen agudo en ginecología. Alrededor del 95% de los embarazos ectópicos se
localizan en la trompa (ampular 79,6 %, istmico 12,3 %, fímbrico 6,2 %, intersticial 1,9 %). Las formas no tubáricas del embarazo
ectópico incluyen el embarazo cervical (0,1%), el embarazo ovárico (0,5%), el embarazo abdominal (1,3%), el embarazo cornual (3%), el
embarazo intraligamentario (0,1%), y el embarazo en cuerno uterino rudimentario. SALUD PUBLICA: El embarazo ectópico es la causa
más frecuente de morbilidad y mortalidad materna en el primer trimestre de la gestación, siendo responsable del 9% de las muertes
durante este periodo. En cuanto a la edad, afecta en el 48’2% a mujeres entre los 30-39 años y en el 36’8% entre los 20-29 años,
representando las nulíparas el 34’4% y las que
han tenido 1-2 hijos el 23,6%. PATOGENIA:
Está determinada por todos aquellos factores
que impiden o dificultan la migración normal
del huevo hacia la cavidad uterina. Se pueden
distinguir entre: 1) Factores tubulares: pueden
ser debidos a) EPI y dentro de ella la causada
por la salpingitis gonocócica y la TBC genital
(asintomáticas en el 80-90% de casos) y la
salpingitis posabortiva. Deterioran la actividad
ciliar y la motilidad tubárica y generan adherencias en la mucosa tubárica que conducen a la estenosis. b) cirugía tubárica previa contra
la infertilidad (21% riesgo relativo), c) cirugía tubárica para la esterilidad permanente (9.3%), d) endometriosis tubárica: favorece la
nidación precoz, e) anomalías congénitas: hipoplasias (observada en la exposición intraútero al dietilestilbestrol) o trompas

CURSO ENARM CMN SIGLO XXI TEL: 36246001 Pharmed Solutions Institute PÁGINA 307
MANUAL DE TRABAJO DEL CURSO ENARM CMN SIGLO XXI
excesivamente largas y tortuosas, f) alteraciones funcionales del peristaltismo tubarico o de la actividad ciliar. 2) Factores ovulares:
aquellos que conducen a la; a) Nidación precoz del huevo, bien sea debido a huevos excesivamente pesados que maduran con rapidez y
no pueden progresar, o al desprendimiento precoz de la capa pelúcida del huevo capacitando la implantación, b) Transmigración: se ha
sugerido la posibilidad de la hipermigración del huevo y su implantación en la trompa contralateral. En el 20% de los casos el cuerpo
lúteo se encuentra en el ovario contralateral al de la implantación, c) Técnicas de Reproducción Asistida (TRA): de todos los embarazos
clínicos resultantes de las TRA, el 5-5,7% fueron ectópicos y el 1% fueron embarazos heterotópicos, debido a la hiperestimulación
ovárica y a las implantaciones múltiples resultantes de la transferencia de varios embriones. Algunos autores afirman que la
implantación ectópica está estrechamente ligada a la insuficiencia del cuerpo lúteo, y que esta insuficiencia es un factor patogénico
común a muchas anomalías de la reproducción, entre ellas algunas cromosomopatías y abortos. Una vez implantado el huevo en la
trompa, las vellosidades invaden rápidamente el endosalpinx, alcanzando la parte tubárica y el peritoneo. Esta penetración va
acompañada de una proliferación vascular y de un hematoma peritubarico e hematosalpinx, que a menudo afecta la trompa
contraleteral, y que se propaga hacia el extremo distal de la trompa, entre la pared y la serosa. A partir de aquí puede evolucionar hacia
el: 1) Aborto tubarico, el huevo carece de vellosidades suficientes y está implantando en el segmento distal de la trompa. Se atrofia, se
desprende y es expulsado por el orificio peritoneal de la trompa. Esta expulsión suele ir acompañada de hemorragia moderada, que
ocupa el fondo de saco de Douglas formando hematocele. En raras ocasiones puede convertirse secundariamente, en embarazo
abdominal. El aborto tubárico se da con más frecuencia en la implantación ampular y fímbrica de la trompa. 2) Rotura tubarica, las
vellosidades son suficientes y enérgicas, pero el aumento de la presión intatubarica llega a dificultar el riego sanguíneo de la trompa, lo
que produce necrosis tisular que conduce a la ruptura por corrosión vascular (no por estallido). Esta rotura suele acompañarse de
hemorragia intensa, que da lugar al hemoperitoneo y shock hipovolémico, ya que suele afectar las anastomosis de las arterias tubáricas
con la uterina ascendente. La rotura tubárica se da con más frecuencia en la implantación ístmica e intersticial de la trompa. La
metrorragia se manifiesta generalmente antes de la rotura, debido a la insuficiencia placentaria (esteroidogénesis reducida), que
precede las manifestaciones clínicas. 3) Ectópico viable, lo habitual del embarazo ectópico es que progrese hasta el segundo o tercer
mes, momento en que se interrumpe, y rara vez llega a ser viable. DIAGNOSTICO: En algunas ocasiones, la interrupción del embarazo
es tan precoz, que se resuelve por reabsorción espontánea, sin llegar a dar manifestaciones ni de amenorrea. El embarazo ectópico
accidentado no precisa mucha pericia diagnóstica, la paciente suele presentar un cuadro de schock hipovolémico y dolor abdominal
agudo, confirmándose el diagnóstico en quirófano, previa punción fondo de saco de Douglas. Las manifestaciones clínicas del
embarazo ectópico no accidentado en evolución, se caracterizan en el 90% de los casos por pérdidas hemáticas irregulares y dolor
abdominal. La triada clásica compuesta por dolor, sangrado y masa anexial solo se presenta en 45% de las pacientes. A la exploración
física 30% de pacientes, no presenta sangrado transvaginal, y 10% pueden tener masa anexial palpable y 10% tener un examen pélvico
normal. La localización más frecuente del embarazo ectópico es en la tuba uterina, manifestándose con dolor y sangrado, sin evidencias
de embarazo intrauterino y con una masa anexial. El sangrado del embarazo tubario es lento, gradual y algunas pacientes pueden
permanecer hemodinamicamente estables a pesar de hemoperitoneos de 1000-1500ml. En toda paciente en edad reproductiva y con
vida sexual que presenta retraso menstrual, dolo y sangrado transvaginal debe descartarse la presencia de embarazo tubario a través
de ultrasonografía pélvica transvaginal y determinación sérica de β-HCG. La detección por ultrasonido transvaginal de una masa anexial,
combinada con una concentración de β-HCG de 1000mUI/ml tienen una sensibilidad de 97%, una especificidad de 99% y unos valores
predictivos positivo y negativo de 98% para diagnóstico de embarazo tubario. Para la detección temprana del embarazo tubario
temprano el US transvaginal ofrece mayor sensibilidad que el pélvico abdominal por lo cual debe realizarse el primero si se cuenta con
el recurso de lo contrario utilizar el pelvicoabdominal. Ante la duda diagnostica de embarazo tubario, la evaluación laparoscópica está
indicada. Los niveles séricos de β-HCG por si solos no diferencian un embarazo tubario de un intrauterino. Las determinaciones seriadas
de β-HCG tienen 36% de sensibilidad y 65% de especificidad para la detección de embarazo tubario. Los rangos de β-HCG de la zona de
discriminación son de 1500-2000mUI y estos se asocian con la presencia ultrasonografica del saco gestacional intrauterino. La
determinación sérica β-HCG es más sensible que la determinación en orina. TRATAMIENTO: Puede ser médico, quirúrgico y expectante.
El tratamiento médico consiste en uso de metotrexate con o sin ácido folinico, está recomendado en pacientes con: estabilidad
hemodinámica, embarazo tubario no roto, sin datos de sangrado activo intra-abdominal, pacientes con niveles séricos de β-HCG
menores de 2000mUI/ml, tamaño del saco gestacional menor a 3.5cm, ausencia de latido cardiaco embrionario. Existen tres protocolos
de administración de metotrexate para el tratamiento de embarazo ectópico monodosis, doble dosis y multidosis. Se recomienda
monodosis ya que los resultados son similares y con menos toxicidad. Contraindicaciones absolutas: lactancia, disfunción renal,
hepática, pulmonar o hematológica, hipersensibilidad a metotrexate, enfermedad acido-péptica. Relativas: saco gestacional mayor a
3.5cm, actividad cardiaca embrionaria. El tratamiento quirúrgico puede consistir en laparoscopia o laparotomía. Sin indicaciones para el
tratamiento quirúrgico y embarazo tubario: pacientes no candidatas a tratamiento médico con metrotexate, falla al tratamiento
médico, embarazo heterotopico con embarazo intrauterino viable, pacientes hemodinamicamente inestables, pacientes con
inestabilidad hemodinámica deben ser estabilizadas antes de entrar a quirófano, en pacientes con embarazo tubario
hemodinamicamente estables, se recomienda el abordaje laparoscópico, siempre y cuando se cuente con el recurso material y humano
necesarios. Ante las siguientes circunstancias se considera de primera elección la laparotomía exploradora: Antecedentes de cirugía
abdominal, presencia de adherencias pélvicas, inexperiencia del cirujano laparoscopia. Las indicaciones de salpingectomia incluyen:
daño severo de la tuba uterina, embarazo tubario recurrente en la misma tuba uterina, sangrado persistente después de la
salpingostomia, embarazo tubario mayor a 5cm, embarazo heterotopico, pacientes con paridad satisfecha. El manejo expectante del
embarazo tubario no está claramente establecido. En pacientes con embarazo tubario que son Rh negativo no sensibilizadas, se
recomienda la aplicación de inmunoglobulina anti D a razón de 250UI. Un embarazo ovárico es difícil de distinguir de un quiste
hemorrágico de cuerpo lúteo, incluso durante el acto quirúrgico.

CASO CLINICO
Se trata de femenino de 19 años de edad la cual se encuentra en control prenatal, acude a su consulta de control, refiriendo que desde
hace una semana presenta dolor en fosa iliaca derecha, sensación de pesantez y distención abdominal, agrega que ha tenido dificultad
para la evacuación, refiere que el día previo a la consulta observa sangrado leve transvaginal, la paciente es ingresada a urgencias por

CURSO ENARM CMN SIGLO XXI TEL: 36246001 Pharmed Solutions Institute PÁGINA 308
MANUAL DE TRABAJO DEL CURSO ENARM CMN SIGLO XXI
sospecha de embarazo se realiza PIE resultando positiva, por lo que es ingresada para observación ya que el sangrado no ha disminuido
y continua el dolor abdominal.

PREGUNTA
Cuál es la su conducta a seguir, considerando la evolución del paciente.

RESPUESTA
a.- Realizar Bh en busca de anemia y leucocitosis.
b.- Realizar USG transvaginal.
c.- Realizar cuantificación de Gonadotrofinas.
d.- Cuantificación de progesterona.

ABORTO, AMENAZA DE ABORTO. CIENCIAS BASICAS: Aborto es la expulsión o extracción del producto de la concepción antes de las
22SDG, el feto habrá de pesar menos de 500gr. SALUD PUBLICA: A mediados de los años 90 se calculaba una cifra anual de un millón
700 mil abortos, estimándose que alrededor de la mitad eran inducidos Se presenta aborto espontaneo en 15% de los embarazos
reconocidos y del 13-26% de los no reconocidos. En general 80% de los abortos espontáneos ocurren en las primeras 12 semanas de
gestación. PATOGENIA: Factores de riesgo: Edad materna avanzada (en mayores de 40 años se duplica el riesgo). Aborto recurrente
(más de dos pérdidas consecutivas aumenta el riesgo al 43%). Período intergenésico largo (mayor a 10 años). Bajo peso y sobrepeso
materno. Intervalo de ovulación-implantación prolongado (mayor a 72 horas). Bajo nivel de folatos durante la gestación (nivel sérico
</= 2.19ng/ml). Alcohol (no ha sido establecido nivel seguro en ninguna etapa del embarazo). Cocaína. Cafeína (consumo de más de
100mg por día = 4 a 5 tazas). Cigarrillo (padre o madre): consumo de más de 10 cigarrillos por día. AINES (consumo cerca a la
concepción por su mecanismo de acción sobre las prostaglandinas). Etiología: Anomalías genéticas 50-60% (la más frecuente,
aneuploidías: trisomías autosómicas 52%, monosomía X 19%, poliploidías 22% y otras 7%). Anomalías endocrinas 10-15%,
desprendimientos corio-amnióticos 5-10%, cérvix incompetente 8-15%, implantación anormal de la placenta 5-15%, infecciones
maternas agudas 3-5%: listeria monocitogenes, toxoplasma, parvovirus B19, rubeola, herpes simplex y CMV. Anomalías inmunológicas:
3-5%. Anomalías anatómicas uterinas: 1-3% (malformaciones: septo uterino, leiomiomas, adherencias). Causas desconocidas: < 5%.
DIAGNOSTICO: Clínico; amenorrea + sangrado vaginal + dolor hipogástrico. Historia clínica. Examen físico: revela la fuente de sangrado.
Ecografía transvaginal: hallazgos de saco vitelino normal y actividad cardíaca fetal (5,5-6 SDG). Es tranquilizador en etapas tempranas
de la gestación. El diagnostico definitivo de embrazo intrauterino no viable puede ser hecho con los siguientes criterios: ausencia de
actividad cradica en un embrión con una longitud caudal >5mm, saco vitelino irregular y ausencia de polo fetal con un diámetro
promedio de saco gestacional >25mm transabdominal o 18mm transvaginal. Cuando cualquiera de estos hallazgos este presente
repetir el US en 1 semana: muy probablemente mostrara la perdida gestacional. Estudios de laboratorio: Medición de β-HCG
8mediciones seriadas). RECOMENDACIONES GENERALES: Realizar ecografía transvaginal después de la expulsión espontánea o manejo
médico. Observación al menos 30 min posterior al procedimiento, en caso de curetaje. Inmunoglobulina anti-D (después de evacuación
o al momento del diagnóstico en caso de manejo expectante) con incompatibilidad Rh. Vacunación antirrubéola y sarampión posaborto
si no ha sido vacunada. Solicitar VDLR. La menstruación debe reorganizarse en 6 semanas. Los niveles de B-hCG retornan a lo normal a
las 2 o 4 semanas posaborto. Iniciar anticoncepción de inmediato. AMENAZA DE ABORTO: Es la presencia de hemorragia genital
indolora, dolor leve suprapúbico, cérvix cerrado, útero de tamaño adecuado para la edad gestacional y actividad cardíaca fetal
demostrada por ecografía. Diagnóstico: Amenorrea secundaria, prueba inmunológica de embarazo positiva, presencia de vitalidad fetal
por ultrasonido, hemorragia de magnitud variable, dolor tipo cólico en hipogastrio de magnitud variable, volumen uterino acorde con
amenorrea, sin dilación cervical. Solicitar prueba de embarazo y básicos. Tratamiento: Disminuir la actividad física, psíquica y sexual,
reposo en cama, hidratación vía oral, se puede administrar antiespasmódicos. Si continua con hemorragia referir a segundo nivel de
atención. No hay intervenciones terapéuticas para prevenir la pérdida en el 1er trimestre. Se hospitaliza a todas las pacientes que
continúen con sangrado transvaginal. Realizar β-HCG y ultrasonido pélvico para completar evolución clínica. Hidratación oral y
parenteral. ABORTO INEVITABLE: Sangrado abundante, contracciones uterinas dolorosas, cérvix abierto, tejido gestacional puede
visualizarse a través del orificio interno, y ruptura prematura de membranas antes de las 20 semanas. Variedad que hace imposible la
continuación de la gestación generalmente por la existencia de hemorragia genital intensa o ruptura de membranas, aun sin
modificaciones cervicales o actividad uterina reconocible. Diagnóstico: Dolor tipo cólico en hipogastrio, volumen uterino igual o menor
que por amenorrea, hemorragia abundante o ruptura de membranas, puede o no haber dilación cervical. Manejo quirúrgico: curetaje
(preferiblemente evacuación por aspiración) Manejo expectante: realizar en paciente estable, sin signos de infección y con gestaciones
menores a 13 semanas. La expulsión espontánea ocurre en las primeras 2 semanas del diagnóstico en el 80% (puede prolongarse a 3 o
4 semanas). Manejo médico con misoprostol: I trimestre: 800mcg vía vaginal cada 6 a 12 horas, hasta 3 dosis, u 800mcg sublingual cada
3 a 4 horas hasta 3 dosis Iniciando II trimestre: 400 mcg vaginal entre 13 y 15 semanas, y 200mcg vaginal entre 16-20 SDG. No olvidar
las reacciones adversas del misoprostol: diarrea y dolor abdominal (10%), cefalea, constipación, flatulencia, vomito, dispepsia y en <1%
ansiedad, anafilaxia, fiebre, trombocitopenia, hipotensión. ABORTO EN EVOLUCION: Se caracteriza por la presencia de hemorragia
genital persistente, actividad uterina reconocible clínicamente y modificaciones cervicales incompatibles con la continuidad de la
gestación. Diagnóstico: expulsión del tejido ovular inminente, dolor tipo cólico progresivo en intensidad y frecuencia, volumen uterino
menor que por amenorrea, hemorragia persistente, dilatación cervical. Tratamiento: Hospitalizacion, se realizara la AMEU y el LUI
como procedimientos de primera y segunda opción respectivamente. ABORTO INCOMPLETO: Cuando ha ocurrido la expulsión de una
parte del huevo y el resto se encuentra aún en cavidad uterina. Diagnóstico: expulsión parcial del producto de la concepción,
hemorragia y dolor tipo cólico de magnitud variable, dilación cervical y volumen uterino no acorde con amenorrea. Tratamiento:
Legrado si el diámetro del tejido retenido es igual o mayor a 15mm, para evitar complicaciones hemorrágicas e infecciosas; además, es
la elección si ocurre antes de 12 semanas. Algunos estudios han sugerido que el manejo expectante del aborto incompleto en pacientes
estables puede ser seguro, o se puede dar manejo médico con Misoprostol 600 mcg vía oral, o 400 mcg vía sublingual, dosis única y
control en 7 días. ABORTO COMPLETO o consumado: Aquel en el que la expulsión del huevo ha sido total. Diagnostico expulsión
completa del producto de la concepción, disminución de la hemorragia y el dolor, se favorece el cierre del orificio cervical. Tratamiento:

CURSO ENARM CMN SIGLO XXI TEL: 36246001 Pharmed Solutions Institute PÁGINA 309
MANUAL DE TRABAJO DEL CURSO ENARM CMN SIGLO XXI
Generalmente no necesita manejo adicional, si se tienen duda manejarlo como aborto incompleto. ABORTO DIFERIDO o huevo muerto
retenido: Se presenta cuando habiendo ocurrido la muerte del producto de la concepción, no se expulsa en forma espontánea. Esta
entidad presupone un lapso entre la muerte ovular y la elaboración del diagnóstico. Generalmente existe el antecedente de amenaza
de aborto. Diagnóstico: volumen uterino menor que por amenorrea, ausencia de vitalidad fetal (latido cardiaco fetal), no hay
modificaciones cervicales. Los abortos del segundo trimestre de este tipo pueden asociarse a trastornos de la coagulación. Tratamiento:
Manejo quirúrgico: curetaje (preferiblemente evacuación por aspiración) Manejo expectante: realizar en paciente estable, sin signos de
infección y con gestaciones menores a 13 semanas. La expulsión espontánea ocurre en las primeras 2 semanas del diagnóstico en el
80% (puede prolongarse a 3 o 4 semanas). Manejo médico con misoprostol: I trimestre: 800 mcg vía vaginal cada 6 a 12 horas, hasta 3
dosis, u 800 mcg sublingual cada 3 a 4 horas hasta 3 dosis Iniciando II trimestre: 400 mcg vaginal entre 13 y 15 semanas, y 200 mcg
vaginal entre 16-20 SDG. No olvidar las reacciones adversas del misoprostol: diarrea y dolor abdominal (10%), cefalea, constipación,
flatulencia, vomito, dispepsia y en <1% ansiedad, anafiulaxia, fiebre, trombocitopenia, hipotensión. ABORTO HABITUAL: Es la perdida
repetida y espontanea del embarazo en 3 o más ocasiones, o de 5 embarzos en forma alterna. Requiere por ser de origen multifactorial,
de una adecuada valoración clínica y estudios de laboratorio y gabinete para determinar su causa, de un tratamiento especializado y
posteriormente de una atención prenatal precoz y con calidad durante todo el embarazo. ABORTO SEPTICO: Cualquiera de las
variedades anteriores a las que se agrega infección intrauterina. Diagnóstico: Escurrimiento de mal olor e incluso secreción hemato-
purulenta a través del cérvix. Fiebre sin ningún otro sitio evidente de infección, hipersensibilidad suprapubica, dolor abdomino-pelvico
a la movilización del cérvix y útero, ataque al estado general. El examen físico se encuentra taquicardia, taquipnea, dolor hipogástrico,
útero friable y doloroso con cérvix dilatado. Es una complicación frecuente de abortos ilegales. Agentes involucrados: staphylococcus
aureus, bacilos gram negativos, cocos gram positivos, infecciones mixtas, organismos anaerobios y hongos. Clasificación: Leve: infección
ovular. Moderado: respuesta inflamatoria sistémica secundaria a enfermedad localizada. Severo: sepsis - inflamación sistémica +
bacteriemia + Choque séptico à síndrome de falla multiorgánica. Tratamiento: Referir al paciente a segundo nivel de atención,
independientemente de la edad gestacional. Soporte hemodinámico. Antibióticos de amplio espectro; Leve: cefalosporina de 1ª
generación. Moderado: clindamicina + gentamicina, Ampicilina-sulbactam. Severo: penicilina cristalina + clindamicina (600mgs IV c/8h)
+ gentamicina (80mg IV o IM c/8h). Hidrocortisona 3 gr IV dosis inicial y repetir 1g IV c/8h en las siguientes 24h. Si no hay respuesta
histerectomía o anexohisterectomía. Evacuación del contenido uterino. Profilaxis - tétanos

CASO CLINICO
Paciente femenino de 24 años de edad, originaria de Campeche, en comunidad rural productora de aves, acude debido a que presenta
amenorrea secundaria de 8 semanas de evolución, ella refiere que desde hace tres días ha presentado dolor lumbosacro y presencia de
sangre roja rutilante, por lo que se mantiene en reposo; cuenta con los antecedentes GO de Gesta 4, para 0, abortos 3, ella refiere que
sus embarazos han terminado en abortos a las pocas semanas.

PREGUNTA
Tomando en cuenta el cuadro clínico actual y los antecedentes GO cuál es su conducta a seguir para establecer una aproximación
diagnostica:

RESPUESTA
a.- Realizar USG en búsqueda de anomalías uterinas.
b.- Buscar anticuerpos antifosfolipidicos y anticardiolipinas.
c.- Prueba TORCH.
b.- Determinación de gonadotrofina corionica.

CASO CLINICO
Se trata de femenino de 23 años de edad que acude a control prenatal, actualmente cuenta con 22 semanas de gestacion, refiere que
es empleada de una granja y que ha presentado molestias tales como dolor en la región lumbo-sacra que se extiende hacia la región
abdominal baja, cuenta con antecedentes de dos abortos expontaneos y por eso se mantiene preocupada cuando presenta estas
molestias.

PREGUNTA
En caso de llegar a término el producto y considerando la exposición laboral, cuál sería la complicación más probable en el Recien
Nacido.

RESPUESTA
a.- Malformacion Cardiaca.
b.- Sordera.
c.- Malforacion Oftalmica.
d.- Malformacion Renal.

CASO CLINICO
Se trata de femenino de 25 años de edad la cual acude al servicio de urgencias con dolor en ambas extremidades inferiores, cólicos,
ardor al orinar, presencia de manchado transvaginal, refiere que tiene tres días con esta sintomatología y decidió acudir porque se
incrementaron los síntomas, al interrogatorio refiere contar con 6 semanas de embarazo por FUM, actualmente bajo control prenatal
con médico familiar, sus antecedentes GO son Gesta 2 para 1, Abortos 0, refiere que el embarazo previo fue sin complicaciones al igual
que el parto, niega toxicomanías ni presencia de otra patología.

PREGUNTA

CURSO ENARM CMN SIGLO XXI TEL: 36246001 Pharmed Solutions Institute PÁGINA 310
MANUAL DE TRABAJO DEL CURSO ENARM CMN SIGLO XXI
Luego de ingresar a la paciente a observación, se realiza cuantificación de gonadotrofinas observandoce dentro del rango,
disminuyendo la sintomatología al mantenerse en reposo, cual es la conducta a seguir:

RESPUESTA
a.- Mantener a la paciente en observación debido a que puede progresar la amenaza de aborto.
b.- Iniciar con medicación antiespamodica y vigilancia en casa con cita abierta.
c.- Dar indicación de reposo absoluto y signos de alarma con cita abierta.
d.- Indicar antibiótico, indicar reposo, dar signos de alarma y cita abierta.

CASO CLINICO
Se trata de paciente femenino de 28 años de edad la cual acude al servicio de urgencias ya que refiere que aproximadamente 4 horas
en el trayecto a su trabajo presenta dolor intenso en región lumbar con irradiación a miembros pélvicos, y sangrado transvaginal
moderado, refiere que cuenta con 9 semanas de embarazo aparentemente normal y bajo control médico, al ingreso a urgencias
observa el cuello reblandecido y 2 centímetros de dilatación, y salida de material, se indica USG transvaginal y se observa saco
amniótico abierto sin presencia determinada de producto en su interior.

PREGUNTA.
Cuál es su conducta a seguir.

RESPUESTA
a.- Envio a segundo nivel.
b.- Ingreso para vigilancia estricta.
c.- Manejo ambulatorio.
d.- Determinación de hGC.

INCOMPETENCIA CERVICAL (IC). CIENCIAS BASICAS: Es la entidad clínica en la cual el cuello uterino es incapaz de ejercer su función
hasta el término del embarazo, con perdida repetida del producto en el segundo trimestre (parto prematuro). Un cérvix incompetente
es aquel que, por una anomalía estructural o funcional, es incapaz de mantener un embarazo hasta su término. En realidad esta
incompetencia del cérvix se localiza en el orificio cervical interno y el istmo, de ahí que muchos hablen de insuficiencia ístmico cervical.
SALUD PUBLICA: Su incidencia varía entre el 0,005 y el 1% de todos los embarazos, y se considera la principal causa de abortos tardíos.
PATOGENIA: Se caracteriza por la dilatación cervical pasiva y por la sucesión de pérdidas gestacionales durante el segundo trimestre.
No se acompaña de metrorragia ni de rotura prematura de membranas. Aunque no suele encontrarse una historia obstétrica clásica en
estas pacientes, deben investigarse los factores etiológicos previos. En realidad la IC es un síndrome al cual se puede llegar por
diferentes causas: clásicamente se admite la 1) Causa congénita; puede darse en los casos de malformaciones uterinas (mujeres
expuestas al dietilestilbestrol durante su embarazo). 2) Causa traumática. En los casos de partos traumáticos vaginales instrumentales
o no, pueden producirse desgarros que afecten al canal cervical y sean la causa de la IC. Otras veces son los legrados con dilatación los
responsables del cuadro. Por último pueden incluirse en este apartado las intervenciones quirúrgicas sobre el cérvix; cada día son más
frecuentes las conizaciones en edad de procrear y, dependiendo de la técnica y el tamaño del cono, la secuela de la IC puede
producirse del 5 al 15% de estas mujeres. 3) Causa oculta. Incluimos aquí todas las IC sin aparente lesión pero a pesar de lo cual se
producen de forma reiterada abortos del segundo trimestre o partos muy prematuros. Son posiblemente casos en los cuales hay una
alteración histoquímica de los tejidos que constituyen el cuello y de una forma muy especial del colágeno. DIAGNOSTICO: Es una
entidad en la que el diagnóstico precoz es fundamental, ya que cuando se objetiva una protrusión cervical de membranas en reloj de
arena, el pronóstico es peor. En la anamnesis es esencial buscar factores de riesgo de la incompetencia cervical, como antecedentes de
partos distócicos con fórceps o ventosa, en los que se hayan podido producir desgarros cervicales extensos que hayan dejado al cérvix
incompetente para sus funciones; dilataciones cervicales traumáticas para legrados por abortos o en interrupciones voluntarias de
embarazo, o intervenciones quirúrgicas sobre el cérvix, como conizaciones o tratamientos destructivos por patología cervical. En
algunos casos, la etiología del cérvix incompetente no es de origen traumático sino congénito, entidad en realidad extremadamente
rara y a cuyo diagnóstico se llega tras constatar la existencia de una incompetencia cervical sin antecedentes lesivos cervicales. La
incompetencia cervical congénita se ha relacionado con la toma materna de dietilestilbestrol durante la gestación y con la existencia de
posibles malformaciones uterinas. La ecografía transvaginal tiene un papel de primer orden en la valoración de las características
cervicales en el diagnóstico de incompetencia cervical y en la predicción de parto prematuro, que afecta al 8-10% de los embarazos con
o sin rotura prematura de membranas. Su aportación al diagnóstico de la modificación cervical sin la existencia de dinámica uterina es
fundamental. Desde el punto de vista ecográfico se ha demostrado que cuanto menor es la longitud cervical más posibilidades de parto
prematuro existen. En la evaluación ecográfica cervical, entienden las siguientes como imágenes de normalidad: cérvix formado, mayor
de 30 mm; OCI cerrado, menor de 5 mm; canal cervical con bordes paralelos; canal en forma de cono, con angulación dirigida al OCI, y
ausencia de herniación de la bolsa amniótica. Estos criterios de normalidad siempre se deben tener en cuenta en la evaluación cervical
para realizar el diagnóstico de incompetencia cervical, además del diagnóstico ecográfico, que también es fundamental en el control y
seguimiento de esta patología. Clásicamente, el diagnóstico de incompetencia cervical se realizaba mediante exploración, por la
detección de repetidas pérdidas gestacionales o de antecedentes traumáticos cervicales. Hoy día, la cervicometría y la observación del
orificio cervical interno (OCI) mediante ecografía son imprescindibles para su diagnóstico y seguimiento. Se sabe que la longitud cervical
no es constante durante todo el embarazo, sino que disminuye a medida que éste avanza. Para la apertura del OCI deberán
considerarse los valores patológicos que, antes de la semana 28, sean inferiores a 30 mm y superiores a 10 mm. TRATAMIENTO: 1)
Tratamiento antes del embarazo: Es siempre quirúrgico y tiene como objetivo reconstruir la normal anatomía del cuello. Se han
propuesto diferentes técnicas: traquelorragia, reducción del calibre ístmico cervical, reparación de desgarros y cerclaje. Todos estos
procedimientos han sido prácticamente desechados dados los malos resultados obtenidos. 2) Tratamiento durante el embarazo. Se ha
propuesto tratamiento médico y tratamiento quirúrgico. El tratamiento médico consiste en reposo, abstinencia sexual y administración

CURSO ENARM CMN SIGLO XXI TEL: 36246001 Pharmed Solutions Institute PÁGINA 311
MANUAL DE TRABAJO DEL CURSO ENARM CMN SIGLO XXI
de tocolíticos o de progesterona, en este tipo de tratamiento no ha demostrado, ningún tipo de eficacia, por lo que a excepción del
reposo, hoy día no se utiliza. Tratamiento quirúrgico, es de elección y consiste en el cerclaje. Hay numerosas técnicas de cerclaje pero
las utilizadas casi exclusivamente son: la de Mc Donald y Shirodkar, ambas consisten en hacer un lazo alrededor del cuello por vía
transvaginal y el período más apropiado para su realización es entre las semanas 13 y 16 del embarazo. Excepcionalmente se preconiza
la realización del cerclaje por vía transabdominal. Esta técnica se realiza en el quirófano, ya que se requiere el uso de anestesia general,
buenas condiciones de asepsia y control ecográfico para valorar la correcta realización del cerclaje. El cerclaje cervical se debe realizar si
la gestación es inferior a 20 semanas y el diagnóstico está confirmado mediante ecografía. Si la gestación es superior a 26 semanas, el
cerclaje no estaría indicado en ningún caso. La actitud correcta que se debe adoptar es estar en reposo absoluto, llevar a cabo un
control exhaustivo del embarazo y valorar tocólisis mientras se realiza maduración pulmonar fetal con betametasona. El cerclaje puede
ser utilizado en tres circunstancias clínicas: cerclaje en casos diagnosticados de insuficiencia ístmico-cervical, el cerclaje en caso de IC
oculta (cuando los datos ecográficos se alteran haciendo temer el parto pretérmino) y, por último, lo que se denomina cerclaje de
emergencia que es aquel que se realiza cuando ya existe dilatación e incluso prolapso de las membranas en vagina. Cuando se practica
el cerclaje entre las semanas 12 y 18 cuando aún no existe dilatación, se logra un feto viable del 70 al 80% de los casos, los resultados
suelen ser mejores cuando más precozmente se realiza. El cerclaje está contraindicado en las malformaciones fetales graves, cuando
existen contracciones, en los casos de coriamnioitis y es muy discutible cuando hay rotura de las membranas. Existen algunas
publicaciones de abortos tardíos de uno de los gemelos en la que se ha realizado cerclaje logrando prolongar la gestación hasta la
viabilidad del feto. Estos casos son excepcionales y la indicación muy discutible.

CASO CLINICO
Se trata de paciente femenino de 25 años de edad la cual tiene 21 semanas de gestación actualmente bajo control prenatal, refiere
presentar cólicos tipo menstrual niega sangrado transvaginal, sin embargo el dolo se extiende a las piernas y región sacra, que
disminuye al reposo, se realizo USG con diagnostico de embarazo de 22 SDG, al tacto se percibe remblandecimiento de cuello uterino,
no se observa dilatación cervical, por lo que se envía a domicilio con indicación de reposo absoluto, a la semana vuelve la paciente a
revisión, donde refiere presencia de leve sangrado transvaginal y cólicos de las mismas característica, se realiza valores de
gonadotrofina con datos normales, al tacto se observa un cm de dilatación y remblandecimiento de cuello cervical.

PREGUNTAS
Considerando el cuadro clínico y considerando las complicaciones potenciales durante el embarazo, que procedimiento es conveniente
realizar.

RESPUESTA
a.- Indica antibiótico profiláctico.
b.- Envio a segundo nivel para cerclaje cervical.
c.- USG para establecer la viabilidad fetal.
d.- Evaluación de fibronectina fetal.

CASO CLINICO
Femenino de 31 años de edad la cual acude a consulta para control prenatal, refiere 4 semanas de retraso menstrual, muestra un
reporte de laboratorio con PIE positivo, se encuentra ansiosa debido a que ha presentado 3 abortos espontáneos, fue enviada a GO
para ser diagnosticada, es referida a la consulta de control prenatal en medicina familiar con diagnostico de incompetencia cervical y
con tratamiento de cerclaje cervical.

PREGUNTA
Con el actual tratamiento, cual es la probabilidad de llegar a un embarazo a término con producto vivo.

RESPUESTA
a.- 70 a 80 %
b.- 60 a 70 %
c.- 80 a 90 %
d.- 50 a 60 %.

HIPEREMESIS GRAVIDICA. CIENCIAS BASICAS: Las náuseas y vómitos son una situación clínica muy frecuente en el embarazo que
afecta al 75-80 % de las gestantes (1-2 vómitos al día con tolerancia correcta a la ingesta, que inician sus síntomas entre la semana 6 y
la 9 de gestación). La hiperemesis gravídica representa el espectro más grave de una situación clínica que aparece de forma gradual.
Son pacientes que han ido empeorando clínicamente de forma progresiva hasta presentar vómitos persistentes con intolerancia parcial
o total a la ingesta y con pérdida ponderal superior al 5%. SALUD PUBLICA: El espectro más grave de esta situación clínica conocido
como hiperemesis gravídica es mucho menos frecuente y afecta en realidad al 0,5-2 % de todas las gestaciones. No existe consenso
acerca de la definición de la enfermedad, pero la mayoría de las definiciones incluyen vómitos persistentes en ausencia de otras
patologías que los expliquen con pérdida ponderal importante (superior al 5% del peso inicial). PATOGENIA: La etiología de la
hiperemesis gravídica es desconocida, aunque diferentes causas han sido propuestas: Niveles elevados de Beta-hCG y estrógenos, la
“masa placentaria” (las mujeres con embarazos múltiples o enfermedad trofoblástica presentan con mayor frecuencia sintomatología
de náuseas y vómitos), factores psicológicos y factores familiares (mujeres que tienen madres o hermanas que hayan presentado
náuseas y vómitos tienen más probabilidad de presentarlas). La hieremesis parce relacionarse con concentraciones séricas altas o en
aumento rápido de gonadotropina corionica, estrógenos o ambos. Se ha demostrado que el receptor hCG/LH el cual se encuentra
presente en las células del cuerpo lúteo del ovario produciendo el estímulo de la progesterona sobre la decidua para la comunicación
inicial entre el blastocisto y el endometrio, además se encuentra presente en distintas áreas del cerebro como en el hipocampo y tallo

CURSO ENARM CMN SIGLO XXI TEL: 36246001 Pharmed Solutions Institute PÁGINA 312
MANUAL DE TRABAJO DEL CURSO ENARM CMN SIGLO XXI
cerebral lo cual explica la hiperémesis gravídica. También se ha postulado que la HCG causa hiperémesis gravídica estimulando la vía
secretora del tracto gastrointestinal superior. Se ha informado que las mujeres con enfermedad grave tienen aumento 1.5 veces de la
probabilidad de tener un feto del sexo femenino, lo que apoya la hipótesis de los estrógenos (retraso del vaciamiento gástrico y
enlentecimiento de la motilidad gastrointestinal). DIAGNOSTICO: Es muchas veces un diagnóstico de exclusión y básicamente clínico: 1.
Paciente que explica varios vómitos al día en relación o no con las comidas. Intolerancia total o parcial a la ingesta. 2. Ausencia de otra
sintomatología: No fiebre, no dolor abdominal, no cefalea, no alteraciones neurológicas. No todas las pacientes que vomitan al inicio de
la gestación presentan una hiperemesis 3. Alteraciones analíticas: Hemograma (hemoconcentración), Alteraciones electrolíticas
(hiponatremia, hipopotasemia, hipocloremia y alcalosis metabólica), alteración del perfil hepático (elevación leve de GOT y GPT <
300U/L, y Bilirrubina < 4 mg/dl) y alteración del perfil tiroideo (aumento de la T4 y la TSH). Ecografía: Las pacientes con embarazos
múltiples o enfermedad trofoblástica presentan vómitos con mayor frecuencia. PREVENCION: Existen diferentes estrategias de
prevención de las náuseas y los vómitos del embarazo. 1. La toma de complejos multivitamínicos de manera periconcepcional han
demostrado disminuir la incidencia de náuseas y vómitos del embarazo. Así que parece razonable recomendar su administración
sobretodo en aquellas pacientes que han presentado náuseas y vómitos en gestaciones anteriores. 2. Comidas frecuentes, poco
abundantes (repartir la ingesta en un mínimo de 5 comidas de menor cantidad), sólidas y evitar las comidas muy grasas y picantes.
TRATAMIENTO: Farmacológico: Puesto que la aparición de la hiperemesis gravídica se produce de manera escalonada en la mayoría de
las pacientes, la manera de tratarlas es ir añadiendo los fármacos que se refieren a continuación en el orden en el que se citan, a
medida que la sintomatología vaya empeorando. 1. Doxilamina 10 mg + piridoxina 10 mg: La dosis habitual es un comprimido cada 6-8
horas, aunque se puede ajustar la dosis en función de la sintomatología hasta alcanzar una dosis máxima de 70 mg/d. Hay que tener en
cuenta que ante la persistencia de sintomatología en una franja horaria determinada la dosis a aumentar es la inmediatamente anterior
(p. Ej. si persisten las náuseas y vómitos matutinos hay que aumentar la dosis de la noche). Importante tener en cuenta la somnolencia
como efecto secundario de la doxilamina. 2. Añadir Dimenhidrinato 50-100 mg/4-6 horas: La vía de administración puede ser oral o
rectal. Dosis máxima 400 mg /d. 3. Añadir Metoclopramida 5-10 mg/8 horas: La vía de administración es oral en comprimidos o
suspensión. Este fármaco puede causar sintomatología extrapiramidal (espasmos en cara, cuello y lengua). Cuando persiste la clínica a
pesar de la asociación de 2 ó más tratamientos y la paciente presenta intolerancia total a la ingesta, nos encontraremos ante el
espectro más severo de la enfermedad y será necesario un ingreso hospitalario para rehidratación endovenosa. 4. Dieta famis y
fluidoterapia: Suero Glucosado 10 % 500 cc /8 horas alterno con Ringer Lactato o Suero Fisiológico 500 cc/8-12 horas hasta corregir el
trastorno electrolítico. En pacientes que requieren fluidoterapia y que presentan clínica de vómitos durante más de 3 semanas se
recomienda añadir suplementos de tiamina (vitamina B1) intravenosa a dosis de 100 mg/d durante 2-3 días 1 vial de 100 mg/d. En
pacientes que presenten alteración en el ionograma se asociará 10-20 mEq de ClK en cada suero glucosado durante 24-48 h o hasta
corregir el trastorno hidroelectrolítico 5. Añadir Metoclopramida 5-10 mg/8 horas IV. 6. En casos resistentes a todos los tratamientos
mencionados o que requieren varios ingresos, además se puede asociar: a. Ondansetrón 8 mg/12 h IV. b. Metilprednisolona 16 mg/8h
vía oral o endovenosa durante 3 días, seguido de dosis decrecientes durante 15 días hasta desaparición de los síntomas (Prednisona vo:
40 mg/día 1 día, 20 mg/día 3 días, 10 mg/día 3 días y 5 mg/día 7 días). COMPLICACIONES: La encefalopatía de Wernicke es poco
frecuente pero se asocia a las principales causas de mortalidad en las pacientes, se presenta por la deficiencia de tiamina posterior a 3
semanas de vómitos persistentes caracterizada por la triada de anormalidades oculares, confusión y ataxia de la marcha se presenta en
el 46.9% de los casos.

CASO CLINICO
Paciente femenino de 24 años de edad la cual acude a control prenatal, refiere que es su segunda consulta, presenta desde hace una
semana nausea intensas de predominio matutino, actualmente tiene 8 semanas de gestación, ha vomitado hasta 3 veces al día, agrega
que en su embarazo anterior presento los mismo síntomas y luego de un realizarse un USG fue diagnosticada con embarazo molar
tratada sin complicaciones, y reporte de patología negativo a proceso neoplasico.

PREGUNTA
Cuál es la conducta a seguir, considerando sus antecedentes y cuadro actual,

RESPUESTA
a.- Considerando la edad gestacional puede indicarse piridoxina 50 a 100 mg al día.
b.- La modificación de los hábitos alimenticios puede mantenerse adecuadamente.
c.- Solo requiere mantenerse en control continuo del estado de liquidos y electrolitos.
d.- Realizar a la brevedad un USG para descartar un nuevo embarazo molar.

CURSO ENARM CMN SIGLO XXI TEL: 36246001 Pharmed Solutions Institute PÁGINA 313
MANUAL DE TRABAJO DEL CURSO ENARM CMN SIGLO XXI

ENFERMEDAD TROFOBLASTICA GESTACIONAL (ETG). CIENCIAS BASICAS: Las anormalidades proliferativas del trofoblasto
(citotrofoblasto, sincitiotrofoblasto y trofoblasto intermedio) conocidas como Enfermedad Trofoblástica gestacional, describe un grupo
de patologías relacionadas con la fertiolizacion y se derivan de una proliferación anormal del trofoblasto de la placenta humana
(hiperplasia) y del genoma paterno, con una contribución materna ocasional, tendencia variable a la invasión local y a metástasis, y
cuyo denominador común es la hipersecreción de beta HCG y responden a la quimioterapia. La ETG comprende a la mola hidatiforme
(MH), mola completa y mola parcial, las cuales se consideran enfermedades benignas. La neoplasia trofoblastica gestacional (NTG)
incluye a la mola invasora, coriocarcinoma y tumor
del sitio placentario (TSP), los cuales se consideran
patologías malignas. SALUD PUBLICA: La MH se
presenta aproximadamente en 1 de cada 1000
embarazos en EUA y Europa; en Japón la incidencia
es 3 veces mayor. En México es de 2.4 por cada
1000 embarazos. La incidencia de mola invasora
ocurre en 1 de cada 40 embarazos molares y en 1
de cada 150,000 embarazos normales. PATOGENIA:
Factores de riesgo y probabilidad de desarrollar
ETG: mola previa 50%, aborto previo 25%,
embarazo ectópico previo 5%, embarazo previo de
término 20%. En mujeres mayores de 40 y 50 años,
la incidencia de NTG es de 40% y 50%
respectivamente. La ETG son alteraciones de la
gestación, generadas por anormalidades en el desarrollo del tejido trofoblastico con características biológicas y patológicas
particulares, puesto que son lesiones en las cuales hay representación del genoma paterno, lo que las diferencia de los tumores no
gestacionales. CORIOCARCINOMA: Tumor maligno del epitelio trofoblastico. Está compuesto por sincitio y citotrofoblasto anaplasico y
células gigantes multinucleadas, que invade y puede dar metástasis a sitios distantes. En el coriocarcinoma la invasión vascular ocurre
tempranamente resultando en metástasis hacia el pulmón, vagina, cerebro, riñón, hígado y aparato gastrointestinal. MOLA
HIDATIFORME (MH): Se caracteriza histológicamente por la presencia de vellosidades coriónicas con grado variables de proliferación
trofoblástica anormal y edema estromal (degeneración hidrópica), adquiriendo la morfología de bandas y cúmulos de vesículas que
confieren el típico aspecto de “racimos de uvas”. Dentro de los factores de riesgo se pueden mencionar: Los extremos de la vida
reproductiva y el antecedente de Mola previa. MOLA COMPLETA: Ausencia de tejido embrionario o fetal. Degeneración hidrópica de
vellosidades coriales y son avasculares, hiperplasia trofoblastica difusa con atipia difusa y marcada del trofoblasto en el sitio de la
implantación molar. El factor de riesgo más asociado, es la edad, ya que mujeres mayores de 40 años tienen un riesgo 5-10 veces
mayor, que mujeres jóvenes. El síntoma y signo clínico más importante en el embarazo molar es el sangrado o manchado vaginal, que
se presenta entre un 89 a 97% de las pacientes, esta patología se debe sospechar cuando encontramos una Beta-hCG mayor de 100.000
UI en embarazos tempranos. Otro signo importante es una altura uterina mayor a la esperada para la edad gestacional, lo cual se
produce por una amplia proliferación trofoblastica secundaria a un elevado nivel de Beta-hCG; esto ocurre en cerca del 50% de las
pacientes. La ausencia de actividad cardiaca fetal ocurre a pesar del tamaño uterino francamente suprapúbico. Los quistes
tecaluteinicos, están presentes hasta en un 46% de los embarazos molares completos, y generalmente son mayores de 5 cm (6-12 cm),
pero pueden verse hasta de 20 cm de diámetro, suelen ser bilaterales, su contenido es serosanguinolento, y su aparición se explica por
hiperestimulación ovárica de la β-hCG circulante. La hiperémesis gravídica también es más frecuente en estos embarazos, se ha
identificado hipertiroidismo (7%). MOLA INVASORA: (coriocracinoma destruens) La definen el crecimiento trofoblástico excesivo y su
capacidad invasora local, con una penetración extensa por los elementos trofoblásticos, que incluyen vellosidades completas, en el
miometrio (sin involucrar el estroma endometrial) e incluso hasta el peritoneo parametrios y cúpula vaginal adyacente. Las metástasis a
distancia son excepcionales. MOLA PARCIAL: Presencia de tejido embrionario o fetal. Hiperplasia trofoblastica focal, variabilidad
marcada en el tamaño de las vellosidades con importante edema y cavitación, presentando inclusiones prominentes en el estroma
trofoblastico de las vellosidades, presentándose ocasionalmente atipia focal y leve de trofoblasto en el sitio de implantación. La
presentación clínica del embarazo molar parcial es similar a la de un aborto incompleto dado por sangrado genital y dolor tipo cólico en
hipogastrio de hecho, el diagnóstico, en su gran mayoría, se realiza en el estudio histopatológico. TUMOR DEL SITIO PLACENTARIO: Es
un tumor extremadamente raro. Se origina en el sitio de implantación de la placenta y deriva de células de trofoblasto intermedio de la
placenta, las cuales se identifican por la secreción de lactogeno placentario y pequeñas cantidades de fracción beta de HCG. No
contiene vellosidades coriales. DIAGNOSTICO: Los síntomas son más evidentes en la mola completa, y se presentan entre el primero y
el segundo trimestre. Los síntomas clásicos de la mola completa son: la hemorragia vaginal (97%), el aumento del tamaño uterino de
manera más rápida del habitual (50%); la hipertensión arterial en embarazos menores de 24 semanas (27% de las pacientes con mola
completa), embolismo pulmonar (2% en pacientes con mola completa), tirotoxicosis (2-7%). La mola hidatiforme parcial no produce el
cuadro clínico espectacular de la mola completa, generalmente se presentan los síntomas de aborto (amenaza de aborto o aborto
incompleto), y se puede establecer el diagnostico después del estudio histopatológico del material obtenido. Las características
histopatológicas permiten hacer el diagnóstico diferencial de la diversidad de las presentaciones de ETG. Cuando exista sangrado
uterino anormal por más de 6 semanas posteriores a cualquier embrazo, descartar ETG. Luego de la sospecha clínica el examen de
primera línea es la ecografía. La mola completa tiene un aspecto ecográfico vesicular característico en “lluvia de nieve”. En el embarazo
molar parcial se puede ver como una imagen indistinguible de un aborto incompleto, aunque se pueden observar espacios quísticos
focales en el tejido placentario y aumento del diámetro transversal del saco gestacional; si se encuentran los 2 hallazgos el valor
predictivo positivo para la mola parcial es de 90%. La ecografía asociada a la dosificación de BhCG, cuando esta es mayor de 82350
mUI/ml y no existe actividad cardiaca visible en el embrión da una probabilidad de estar frente a una mola hidatiforme del 89%. La
medición de los niveles séricos de BhCG con valores mayores de lo esperado para la edad gestacional son fundamentales para el
diagnóstico. La hiperemesis gravídica y la aparición de preeclampsia temprana son otros de los cuadros clínicos que pueden

CURSO ENARM CMN SIGLO XXI TEL: 36246001 Pharmed Solutions Institute PÁGINA 314
MANUAL DE TRABAJO DEL CURSO ENARM CMN SIGLO XXI
presentarse. En la evaluación inicial se debe hacer búsqueda rápida de enfermedad metastásica, solicitando RX de tórax y pruebas de
función hepática y renal. TRATAMIENTO: El tratamiento de la MH tiene dos aspectos: evacuación y seguimiento. La evacuación
inmediata de la mola preferiblemente se debe realizar mediante aspiración, previa maduración cervical con misoprostol en caso de
encontrarse un cérvix cerrado. La evacuación debe realizarse en instituciones de segundo o tercer nivel de atención, lo de elección es el
AMEO, o histerectomía en bloque, con reserva de sangre, uso de oxitócicos en caso de ser necesario, en casos especiales evaluación
ecográfica transoperatoria y aplicación de inmunoglobulina humana antiD si es Rh negativo. La quimioterapia profiláctica en MH ha
sido controversial, está indicada en aquellas mujeres que tienen molas de alto riesgo. La histerectomía puede ser una opción preferible
al curetaje por aspiración en caso de mujeres con paridad satisfecha y en especial en mujeres mayores de 40 años o más, dada la mayor
frecuencia de enfermedad trofoblástica maligna en este grupo etareo. Aunque la histerectomía no elimina del todo la posibilidad de
metástasis, reduce apreciablemente la enfermedad recurrente. Los quistes tecaluteínicos grandes y sintomáticos se pueden aspirar con
guía ecográfica. Una alternativa del AMEO es el LUI, considerando el mayor riesgo de perforación uterina. En MH con altura uterina
similar a 16SDG o mayor existe riesgo de embolizacion pulmonar. Para descartar el desarrollo de NTG, las pacientes se siguen luego de
la evacuación molar con niveles de β-hCG semanales, el seguimiento debe realizarse cada 2 semanas con examen clínico completo y
mediciones seriadas de BhCG hasta tener 2 mediciones seguidas negativas; luego cada 3 meses durante un año. En la enfermedad
trofoblástica benigna la regresión de la BhCG ocurre entre los dóas 11 y 229, con un promedio de 74 días. Encontrar niveles séricos de
25.000 mUI/ml a la cuarta semana es signo de alta acividad trofoblástica.

CASO CLINICO
Acude a la consulta femenino de 36 años de edad a solicitar método de planificación familiar ya que tiene 2 años de su ultimo parto y
no se ha cuidado debido a que su esposo se encuentra trabajando en estados unidos y va a regresar pronto, la paciente es originaria de
Michoacán y tiene estudios de primaria, dentro de sus antecedentes cuenta con historia de tabaquismo positivo, ingesta de alcohol
ocasional, presento menarca a los 10 años, gesta 4, para 3, abortos 1, refiere que el ultimo embarazo fue hace 4 años, le realizaron un
legrado porque no se había formado el producto y presento sangrados al inicio con salida de tejido como burbujas, le indicaron volver a
consulta y ya no regreso, ahora ya no quiere otro embarazo. A la exploración física la paciente solo se observa con obesidad y
levemente desalineada.

PREGUNTA
Considerando que la paciente presento con mucha probabilidad una enfermedad trafoblastica, que método de planificación es el más
adecuado.

RESPUESTA
a.- Inyectable combinado mensual.
b.- Oral con microdosis.
c.- Método de barrera.
d.- Obstrucción tubarica bilateral.

POLIDRAMNIOS. CIENCIAS BASICAS: Se define como un volumen de líquido amniótico mayor a 1500-2000ml, secundario a un aumento
en la producción o deficiencia en la eliminacion. Según el índice de Phelan, definimos polihidramnios con un índice de líquido amniótico
>25 cm (8-18 normalidad ecográfica del ILA, 18-25 valores límite superior, >25 polihidramnios). El líquido amniótico (LA) es el fluido que
ocupa la cavidad amniótica y que contribuye en importantes funciones para un adecuado desarrollo del embarazo. Sirve de protección
para el feto frente a traumatismos, mantiene una temperatura adecuada, permite el desarrollo de determinados órganos vitales y
puede aportar información acerca del estado y madurez fetal. Su volumen varía a lo largo de la gestación. Aumenta de 50 ml en la
semana 12 a 400 ml en la semana 20. Hacia la 38 semana puede alcanzar valores de 1000 ml y al término es aproximadamente de 800
ml, oscilando entre 300 y 1500 ml. El LA renueva a un ritmo de 500 ml/hora y que en un período de 2-3 horas todo el contenido de agua
del LA se ha renovado. Se ha visto que la cuarta parte de esta circulación se realiza a través del feto y el cordón umbilical; y el resto, a
través de las membranas ovulares y superficie placentaria. El intercambio de LA a través del feto puede realizarse por las siguientes
vías: aparato digestivo, respiratorio, urinario y piel. Se calcula que el feto puede deglutir de 5 a 7 ml hora, cantidad escasa en
proporción a la circulación total de líquido amniótico. SALUD PUBLICA: Incidencia oscila entre 0.13-3.2% de los embarazos. Se asocia
con una alta mortalidad perinatal (60%). PATOGENIA: Se puede deber a causas Idiopático (65%), maternas (7%): DM (en estos casos, el
polihidramnios se atribuye a diversas causas, como un aumento de la osmolaridad del LA debido a una elevación de la glucosa, una
disminución de la deglución fetal y poliuria fetal por la hiperglucemia), isoinmunización al grupo y Rh) como fetales (13%): del tracto
gastrointestinal (atresia esofágica, atresia duodenal, agnatia, fisura palatina, labio leporino, artrogriposis mandibular, obstrucción
intestinal, hernia diafragmática, páncreas anular, gastrosquisis, peritonitis meconial, onfalocele), defectos del tubo neural,
enfermedades cardiovasculares fetales, patología del tracto urinario(riñón poliquístico, obstrucción ureteropélvica), embarazo múltiple,
hidropesía fetal inmune y no inmune) y por ultimo causas placentarias (corioangioma placentario: el tumor benigno más frecuente en la
placenta). CLASIFACION: Según la severidad se clasifica de acuerdo al ILA en LEVE; cuando el índice de líquido amniótico (ILA) se
encuentra entre 25-30cm. MODERADO; se encuentra entre 30.1-35cm. GRAVE; por arriba de 35 cm. Por su evolución se clasifica en
agudo (2%), de inicio súbito, aparece en el segundo trimestre, se asocia a malformaciones congénitas no compatibles con la vida y
rápidamente evoluciona a parto pretérmino. Crónico (98%), aparece en el tercer trimestre, se asocia a factores maternos y es
principalmente idiopático. DIAGNOSTICO: La sintomatología generalmente es secundaria a la sobresdistensión abdominal y uterina, y
dependiendo del grado de extensión puede ir desde fatiga leve, hasta la presencia de disnea, cianosis y edema (miembros inferiores,
vulvar y de pared abdominal), aumento de peso, desarrollo de preeclampsia, así como trabajo de parto. Se sospecha ante la presencia
de un fondo uterino mayor al esperado, con abdomen a tensión y dificultad para palpar partes fetales, dificultad para auscultar foco
fetal y palpación subjetiva de aumento de líquido amniótico. El USG nos brinda el diagnostico con la medición de líquido amniótico, la
técnica más utilizada es la de Phelan. La medida de la cantidad del líquido amniótico se puede realizar utilizando diferentes técnicas, las
dos más utilizadas son la máxima columna vertical y el índice de líquido amniótico (ILA). Máxima columna vertical: Se realiza midiendo

CURSO ENARM CMN SIGLO XXI TEL: 36246001 Pharmed Solutions Institute PÁGINA 315
MANUAL DE TRABAJO DEL CURSO ENARM CMN SIGLO XXI
la máxima columna vertical de líquido libre de partes fetales y de cordón de manera vertical. Es el método de elección en gestaciones
múltiples y en gestaciones únicas de menos de 24 semanas. Se considera normal cuando es superior a 2 cm en todas las edades
gestacionales y cuando es inferior a 8 cm por debajo de la semana 20 o a 10 cm a partir de la semana 21. ILA: Es el valor obtenido a
partir de la suma de las máximas columnas verticales de líquido, libre de partes fetales o cordón umbilical, en cada uno de los cuatro
cuadrantes que se delimitan por la intersección de dos líneas perpendiculares en el abdomen materno: la línea media longitudinal con
la línea transversal media entre la sínfisis púbica y el fondo uterino. El transductor se coloca en posición sagital y lo más perpendicular
posible al suelo. Se consideran normales valores de ILA entre 5 y 25 centímetros. La medida del ILA ha demostrado tener una superior
reproducibilidad y mejor detección de patología por lo que se considera técnica de elección en la medida de la cantidad de líquido
amniótico en las gestaciones únicas de más de 24 semanas. También el USG puede detectar malformaciones fetales. Los exámenes de
laboratorio son importantes para poder determinar una causa no idiopática; se debe tomar en cuenta la curva de tolerancia a la
glucosa, los anticuerpos anti toxoplasma, CMV, tipo sanguíneo y Rh y el cariotipo fetal analizado a través del líquido amniótico.
TRATAMIENTO: El manejo depende dela etiología, la edad gestacional y la intensidad, llegando a requerirse amniocentensis
evacuadora en casos con compromiso materno. El medicamento de elección es la indometacina su uso se limita a partir de 28-32 SDG,
es un inhibidor de las prostaglandinas vasodilatadoras, por lo cual produce una vasoconstricción arteriolar en el riñón fetal
disminuyendo el filtrado glomerular con la subsecuente disminución de la producción de orina fetal que es la principal fuente de
líquido amniótico a partir del segundo trimestre. La dosis usada varía entre 1,5 y 3 mg/Kg por día, siendo la más usada de 25 mg cada
seis horas. Su empleo debe ser controlado por el riesgo de cierre del conducto arterioso fetal. La vía de interrupción dependerá de las
condiciones obstétricas y fetales. Debe monitorizarse el ILA bisemanalmente de tal modo que cuando llegue a 18cm se suspende.
También es importante realizar ecocardiograma fetal cada semana, para observar el agujero oval, en caso de cierre de este debe
suspenderse la indometacina ya que el proceso generalmente es irreversible. En pacientes con polihidramnios esta incrementado el
riesgo de parto pretérmino, rotura prematura de membranas, prolapso de cordón, presentaciones distosicas, rotura uterina, atonía
uterina y desprendimiento prematuro de placenta normoinserta. La amniocentesis es un procedimiento invasivo, que alivia las
molestias maternas, provee líquido que puede estudiarse (cariotipo, madurez pulmonar, gramm y cultivo), y al disminuir la compresión
uterina, mejora el flujo de las arterias espirales. Las complicaciones de la amniocentesis pueden ser parto pretérmino, DPPN, punción
del feto e infecciones que son raras. El tratamiento dependerá de la severidad del caso. En casos leves puede utilizarse únicamente la
indometacina. Cuando es moderado a severo, puede utilizarse la indometacina junto con la amniocentesis.

CASO CLINICO
Una mujer de 20 años de edad, gesta 1 para 0, con 28 SDG con un alto riesgo debido a la presencia de un patrón de crecimiento fetal
discordante y embarazo gemelar. A la exploración física se observo altura del fondo uterino de 35 cm, palidez de tegumentos, edema
de miembros inferiores, TA 90/70 mmHg, el orificio externo del útero cerrado, sin secresión vaginal anormal.

PREGUNTA.
Cuál es su conducta a seguir.

RESPUESTA
a.- Envio a segundo nivel.
b.- Realizar preparativos para cesaria.
c.- Indica datos de alarma y continua ambulatoria.
d.- Suplementación energética y proteica.

CASO CLINICO
Femenino de 24 años de edad, gesta 3, para 2, actualmente con 28 SDG por USG que fue realizado hace 6, la paciente refiere que se
incremento el tamaño del abdomen. Su embarazo había sido sin incidentes. No hay otras pruebas anormales durante embarazo.
Durante una visita para un chequeo de rutina 6 semana antes, una ecografía reveló amniótico normal, la ecografía al ingreso reveló
actual polihidramnios grave, alcanzando un índice de líquido amniótico de 40 cm. Un examen Doppler del cordón umbilical
descubiertos bucles de cordón múltiples envuelto alrededor del cuello fetal.

PREGUNTA.
Cuál es su conducta a seguir

RESPUESTA
a.- Incrementar el número de visitas.
b.- Envio a segundo nivel.
c.- Realizar prueba de grupo y Rh.
d.- Realizar BH, VDRL.

OLIGODRAMNIOS. CIENCIAS BASICAS: Se define como un volumen de líquido amniótico menor a 500ml. Presencia de un ILA inferior a
5 o de una máxima columna vertical inferior a 2 cm. Se define como oligohidramnios severo/anhidramnios la presencia de una columna
máxima ≤ 1 cm. SALUD PUBLICA: Se reporta una incidencia de 5-37%. En el segundo trimestre de la gestación la presencia de una RPM
explica el 50% de los casos de oligohidramnios, seguido por el RCIU y las malformaciones fetales en el 20% y el 15% respectivamente,
siendo un 5% de los casos idiopáticos. PATOGENIA: Podemos dividir las causas de Oligohidramnios en tres grandes grupos: Causas
fetales: Crecimiento intrauterino restringido (CIR), gestación cronológicamente prolongada (GCP), infección fetal por CMV, obstrucción
tracto urinario (ureteral bilateral, valvas uretrales posteriores), patología renal (agenesia renal bilateral, displasia renal multiquística
bilateral, riñones poliquísticos) y defectos del tubo neural. Causas placentarias-membranas: rotura prematura de membranas (RPM).
Causas maternas: medicación materna (inhibidores de la síntesis de prostaglandinas, IECA). DIAGNOSTICO: Datos clínicos sugestivos:

CURSO ENARM CMN SIGLO XXI TEL: 36246001 Pharmed Solutions Institute PÁGINA 316
MANUAL DE TRABAJO DEL CURSO ENARM CMN SIGLO XXI
Disminución de la motilidad fetal, fondo uterino menor al esperado para la edad gestacional, copn fácil palpación de partes fetales,
memebranas aplandas a la exploración vaginal y con alteraciones en la fecuencia cradiaca fetal. Datos paraclínicos: El método de mayor
sensibilidad es sin duda la ultrasonografía, Índice de líquido amniótico con técnica de Phelan, un índice menor de 5 se considera
oligohidramnios. Ante el diagnóstico de oligohidramnios debemos realizar las siguientes pruebas diagnósticas: 1. Descartar RPM:
Mediante anamnesis y exploración. Realizar PROM test si no existe hidrorrea franca. Si existe historia clínica sugestiva y el PROM test es
negativo valorar la posibilidad de instilación de fluoresceína intraamniótica mediante amniocentesis. 2. Descartar CIR: Valoración del
peso fetal estimado así como realización de estudio Doppler. 3. Descartar malformaciones fetales: Estudio morfológico dirigido a
descartar la presencia de malformaciones nefro-urológicas y del tubo neural. 4. Descartar la infección fetal por CMV: serologías
maternas (IgG/IgM), marcadores fetales ecográficos (microcefalia, ventriculomegalia, focos parenquimatosos hiperecogénicos,
hiperrefringencia intestinal). 5. Descartar toma de fármacos: inhibidores de la síntesis de prostaglandinas y IECAs. TRATAMIENTO: No
existe tratamiento específico para esta complicación por lo que la asistencia se basa en la vigilancia y terapéutica de la causa que lo
originó y la edad gestacional en la que se encuentra. El manejo clínico del Oligohidramnios depende principalmente de la causa del
mismo: A) En aquellos casos en los que se diagnostique una RPM o un CIR se aplicara el protocolo específico de cada patología. B) En el
caso de toma de fármacos se interrumpirá la toma de los mismos de forma inmediata. Si la paciente ha consumido inhibidores de la
síntesis de prostaglandinas se realizará valoración del ductus arterioso. Si existiera una restricción (IP<1 o insuficiencia tricuspídea
significativa (holosistólica, ≥150 cm/s)) se realizaría control cada 48 h hasta su normalización. C) En aquellos casos en los que el feto
presente una malformación, se informará del pronóstico de la misma y del riesgo de hipoplasia pulmonar y en función de esta
información los padres podrían acogerse a la interrupción legal del embrazo. Si los padres deciden seguir adelante con la gestación
debemos realizar amniocentesis/cordocentesis para estudio de cariotipo y valorar el estudio de la función renal en orina fetal. D) La
evidencia disponible indica que las pacientes con oligohidramnios idiopático no presentan peores resultados neonatales en
comparación con la población con líquido amniótico normal. Manejo anteparto: en casos menores de 34 SDG se deben emplear
esteroides para maduración pulmonar y pruebas de biestar fetal. Realización de Perfil biofísico y estudio Doppler, semanal hasta las
36.6 semanas y a partir de la semana 37.0 cada 72 horas. Estimación de peso fetal cada dos semanas. Finalización de la gestación: Se
mantendrá una conducta expectante hasta las 40 semanas de gestación si el control de bienestar fetal es normal. Por encima de las 37
semanas ante condiciones cervicales favorables valorar la finalización de la gestación. No existe contraindicación para el uso de
prostaglandinas. Manejo del parto: Se debe realizar monitorización continua. Realizar amnioinfusión.

CASO CLINICO
Femenino de 28 años, gesta 3, para 1, cesarea 1, acude a consulta a las 34 SDG. Presenta un USG realizado a las 24 SDG, con peso fetal
aproximado de (650 g) apropiados para la edad gestacional. Después del examen, el paciente no recibió atención prenatal durante dos
meses. Se realizó nuevamente un USG donde observó restricción del crecimiento intrauterino 1,440 g, con bajo nivel de liquido
amniótico, temperatura 36,0 grados, la frecuencia del pulso 84/min, frecuencia respiratoria 20/min y la presión arterial 120/80 mmHg.

PREGUNTA.
Cuál de las siguientes observaciones es falsa respecto a esta patología.

RESPUESTA
a.- Constituye un elemento accesorio del feto
b.- Depende del desarrollo, crecimiento y maduración.
c.- Su volumen varía fisiológicamente.
d.- El apgar depende del volumen del liquido amniótico.

CURSO ENARM CMN SIGLO XXI TEL: 36246001 Pharmed Solutions Institute PÁGINA 317
MANUAL DE TRABAJO DEL CURSO ENARM CMN SIGLO XXI
PLACENTA PREVIA (PP). CIENCIAS BASICAS: Se define como aquélla implantada en el segmento inferior del útero. Pero una definición
actual establece que es la implantación en el segmento inferior del útero, ocluyendo a veces el orificio cervical interno (OCI). Las
hemorragias del embarazo son causales de importante morbimortalidad. Se las divide en aquellas que afectan la primera mitad y
segunda mitad de éste. Entre las últimas destacan la placenta previa (PP), el desprendimiento prematuro de placenta normoinserta
(DPPNI) y rotura uterina. En el puerperio se agregan el acretismo, y la inercia uterina. La hemorragia del post parto por anomalías de la
inserción placentaria es la principal indicación de histerectomía (HT) obstétrica. SALUD
PUBLICA: La hemorragia obstétrica es la causa más importante de mortalidad materna en
los países desarrollados. Su incidencia es de 3,8/1.000 embarazos (rango: 1,4 a 7/1.000).
Para nulípara, la incidencia es de 0.2%, mientras que en multíparas, puede ser mayor a 5%
y la tasa de recidiva es de 4% a 8%. El factor de riesgo más importante para placenta previa
es tener una cesárea previa. La placenta previa ocurre en el 1% de las embarazadas
después de una cesárea. La incidencia después de 4 o más cesáreas se incrementa a 10%
Se asocia a riesgo significativo de hemorragia, hospitalización, transfusión, parto
prematuro, alta frecuencia de cesárea, y de HT post cesárea. La mortalidad perinatal (MPN)
está aumentada 3 a 4 veces, dada principalmente por parto prematuro. CLASIFICACION:
Basada en la localización relativa de la placenta en el OCI: 1) COMPLETA O TOTAL: cuando
la placenta cubre enteramente el OCI. 2) PARCIAL: cuando la placenta cubre parte pero no
todo el OCI. Es la más peligrosa. 3) MARGINAL: cuando el borde de la placenta está en
contacto con el OCI, pero sin cubrirlo. 4) DE INSERCIÓN BAJA: cuando la placenta está
localizada cerca (a 3 cm.), pero no está en contacto con el OCI. PATOGENIA: El segmento
inferior es una región inadecuada para la inserción placentaria, por presentar: a. Endometrio: de menor grosor que determina una
decidua más delgada y con menor vasculatura, por lo que la placenta tiende a ser más extendida, aplanada e irregular, con escaso
desarrollo de tabiques entre cotiledones. Debido a lo anterior el trofoblasto puede invadir decidua, pared uterina (más delgada) y aún
órganos vecinos (acretismo placentario). La inserción
placentaria en el segmento inferior permite que actúe como
tumor previo. b. Musculatura: menos fibras musculares en
relación al segmento superior y con mayor cantidad de fibras
colágenas, lo que lo hace distensible, pero con menos
potencia para colapsar vasos sanguíneos, dificultando la
hemostasia si hay desprendimiento parcial de la placenta y
por supuesto, durante el alumbramiento. c. Membranas: en
el borde placentario son más gruesas y menos elásticas,
existe mayor frecuencia de RPM. d. Cordón: Por la atrofia de
cotiledones, secundario al desarrollo insuficiente de decidua,
es frecuente la inserción velamentosa del cordón. La
placenta previa sangra sólo si se desprende. Las causas de
desinserción son: En el embarazo: Por el crecimiento uterino
y por las contracciones de Braxton-Hicks, se forma y
desarrolla el Segmento uterino Inferior, cuya capacidad de
elongación es mayor y supera al de la placenta; En el parto:
Por las contracciones del trabajo de Parto que causan la
formación del canal cervico-segmentario y la dilatación
cervical. DIAGNOSTICO: En la Placenta Previa, la hemorragia
genital se caracteriza por ser indolora, con expulsión de
sangre liquida, roja rutilante, que aparece en forma brusca e
inesperada, la mayoría de las veces en reposo e incluso
durante el sueño. El 35% de las pacientes con placenta previa presentan el primer episodio de hemorragia genital antes de la semana
30, 33% entre las semanas 30 y 35, y 32% después de la semana 36. El promedio de edad estacional en el que aparece el primer
episodio de hemorragia es de 29, no se asocia con mortalidad materna y se resuelve espontáneamente. A veces es posible escuchar el
soplo placentario bajo. El sangrado genital está presente en el 80% de los casos; hemorragia + dinámica uterina en un 10-20% y un 10%
son asintomáticas, detectadas sólo por ecografía. En el examen físico, la anemia materna se correlaciona con la cuantía de la
metrorragia. A la palpación, el útero presenta consistencia normal. Si existe trabajo de parto, la dinámica es normal. El dolor, es el
habitual durante las contracciones. Como ya dijimos, la Placenta Previa actúa en la cavidad uterina como tumor previo, por lo que las
presentaciones altas son frecuentes, al igual que las presentaciones de tronco, nalgas (30%) y las posteriores (15%). A la inversa, una
presentación de vértice, con buen apoyo en la pelvis, nos debe hacer pensar en otra causa de metrorragia. La fetocardia es normal. El
compromiso fetal depende del compromiso materno, que junto al ritmo sinusoidal en el monitoreo fetal indicaría que el feto también
ha sangrado al desprenderse la placenta. Es norma en todo estudio ultrasonográfico informar la localización de la placenta y su relación
con el orificio cervical interno (OCI). Actualmente el diagnóstico de la mayoría de las PP es realizado mediante ultrasonografía rutinaria
del segundo trimestre, por vía transvaginal (TV). El USG TV en cualquier edad gestacional debe considerarse el Gold Standard para
diagnosticar PP, ya que la vía transabdominal (TA) presenta inconvenientes en la correcta localización placentaria, describiéndose hasta
60% de relocalización por vía TV. La vía TV tiene una sensibilidad de 87,5%, especificidad de 98,8%, y un valor predictivo positivo de
93,3%, y valor predictivo negativo de 97,6%. Además, la vía TV ha demostrado ser segura en presencia de metrorragia estable, al no
agravar el sangrado vaginal. TRATAMIENTO: El manejo médico de la placenta previa sangrante es hospitalario, se debe efectuar
evaluación completa de los factores etiológicos que favorecen la presencia del sangrado: infecciones cervicales, infecciones urinarias o
vaginales, sobredistensión uterina, factores precipitantes de la actividad uterina de pretérmino. Los exámenes paraclínicos incluirán:
hemograma, hemoclasificación, gram y cultivo de secreción vaginal, parcial de orina (urianálisis), urocultivo, VDRL, ecografía

CURSO ENARM CMN SIGLO XXI TEL: 36246001 Pharmed Solutions Institute PÁGINA 318
MANUAL DE TRABAJO DEL CURSO ENARM CMN SIGLO XXI
transabdominal y/o transvaginal para confirmar el diagnóstico y evaluar la biometría fetal, así como determinar el peso fetal; en caso
de que se considere muy posible la terminación del embarazo, monitoría fetal electrónica bisemanal. Tratamiento quirurgico o cesarea:
La manera de terminar el embarazo esta basado en el juzgamiento clínico sumada la exploración ecografíca. Una placenta con un borde
placentario a menos de 2 cm del orificio cervical externo es probable que requiera una cesárea, especialmente si esta es posterior.
Recomendación B. Recientemente un estudio prospectivo observacional que incluyó 63 pacientes con placenta previa , se demostró
que en todas a las que se le dio parto vaginal , la distancia del borde placentario al orificio cervical interno fue de 2 cm cuando esta era
anterior y 3 cm cuando esta era posterior. Nivel de evidencia IIb.

CASO CLINICO
Se trata de femenino la cual cuenta con 36 semanas de gestacion la cual inicia con trabajo de parto desde hace 6 horas, actualmente
presenta contracciones cada 3 minutos aproximadamente y con duración de 50 a 60 segundos, a la exploración se observa producto
transverso, a la exploración vaginal se observa 10 % de borramiento y 1-2 centímetros de dilatación, no se palpan estructuras oseas, sin
embargo se observa sangre fresca y en cantidad moderada, la paciente cuenta con antecedentes gineco-obstetricos de cesarea previa
con periodo intergenesico de 10 meses.

PREGUNTA
Considerando el cuadro clínico, cual es la complicación más probable que se presentara en este caso.

RESPUESTA
a.- Desprendimiento de Placenta.
b.- Ruptura Uterina.
c.- Sufrimiento Fetal.
d.- Cesarea por Placenta Previa.

CURSO ENARM CMN SIGLO XXI TEL: 36246001 Pharmed Solutions Institute PÁGINA 319
MANUAL DE TRABAJO DEL CURSO ENARM CMN SIGLO XXI
DESPRENDIMIENTO PREMATURO DE PLACENTA NORMOINSERTA (DPPNI). CIENCIAS BASICAS: DPPNI, abruptio placentae o accidente
de Baudelocque es el proceso de separación parcial o total de la placenta de su lugar de inserción normal que corresponde al fondo
uterino, se produce a partir de las 20 semanas de gestación hasta antes del nacimiento del feto, pudiendo ocurrir incluso durante el
trabajo de parto. SALUD PUBLICA: Aproximadamente 1% de los partos. El 50% ocurre en embarazos con síndrome hipertensivo del
embarazo (SHE). Existe un riesgo de recurrencia que va desde 5.5 hasta 30 veces. La tasa de mortalidad perinatal es entre un 20 y un
35%, dependiendo de la severidad del cuadro y de la edad gestacional y determinados básicamente por la prematuridad y la hipoxia.
Actualmente la mortalidad materna es infrecuente, pero la
morbilidad es común y puede ser severa. Casi todas las
complicaciones maternas graves del DPPNI son consecuencia
de la hipovolemia, y de la patología de base asociada al DPPNI.
CLASIFICACION: Grado 0: asintomático y generalmente se
puede diagnosticar en periodo postparto. Grado I: pacientes
quienes presentan solo hemorragia vaginal. Grado II: pacientes
que se presenta con hemorragia vaginal, hematoma
retroplacentarios, sensibilidad uterina (a veces sin hipertonía),
y signos de sufrimiento fetal. Grado III: paciente que presenta
hemorragia vaginal, hematoma retroplacentario, sensibilidad
uterina (con o sin hipertonía), Choque materno, muerte fetal y
signos de coagulopatia de consumo. La evidencia ha mostrado
que hay más casos de Desprendimiento de placenta Grado III
asociado a Hipertensión arterial en el embarazo que en
pacientes que no la padecen y que además el resultado perinatal era más pobre en estas pacientes. PATOGENIA: La gran mayoría es
multicausal y plantean la posibilidad de un mecanismo isquémico a nivel decidual como factor involucrado, asociado con patologías
variadas. De estas la más común es la Enfermedad Hipertensiva asociada al Embarazo, con la cual se presentan los casos de DPPNI más
graves (45% de los casos). Otros trastornos descritos: Antecedente de DPPNI en embarazos previos, RPM, Traumatismo abdominal
grave, descompresión uterina brusca (polihidramnios o salida del primer gemelar), leiomiomas uterinos, consumo de cocaína, bajo
incremento ponderal materno, tabaquismo. Causas directas: Constituyen entre el 1% y 5% de todas las causas y especialmente se
refieren a: trauma directo (accidentes de tránsito, caídas violentas, etc.), disminución súbita del volumen uterino como puede suceder
después de una perdida rápida y abundante de líquido amniótico o el parto del primer gemelo, o un cordón umbilical anormalmente
corto (lo cual puede ocurrir usualmente durante el parto). La causa precisa que conduce al abrupcio de placenta en la mayoría de los
casos es desconocida. La formación del hematoma retroplacentario originado por el DPPNI produce la separación de la placenta lo que
deteriora el intercambio feto-materno dando lugar a una pérdida del bienestar fetal o incluso a la muerte fetal si el desprendimiento es
>50%. También la lesión de los vasos placentarios induce la producción de sustancias vasoactivas (prostaglandinas y endotelina 1) que
pueden originar por si mismas una disminución del intercambio gaseoso fetomaterno materno. Las prostaglandinas y la distensión
uterina originada por el hematoma, provocan una hipertonía uterina que es típica del DPPNI. La formación del coágulo retroplacentario
supone una hemorragia materna oculta. Además, el hematoma retroplacentario (HRP) y la infiltración sanguínea de la pared muscular
estimulan las contracciones colapsando el retorno venoso, y persistiendo el aporte arterial, con presiones superiores a las del útero, por
lo que el hematoma sigue creciendo. Si el coágulo es de gran tamaño dará lugar a hipotensión y shock materno. Además el secuestro de
factores de coagulación en la formación del coágulo y el paso de tromboplastina al torrente circulatorio materno podrán determinar la
aparición de una coagulación intravascular diseminada. El hematoma retroplacentario es el principal responsable de la clínica y de las
complicaciones maternas y fetales del DPPNI. DIAGNOSTICO: El motivo de consulta es el dolor abdominal, de comienzo brusco, intenso
y localizado en la zona de desprendimiento, que se generaliza a medida que aumenta la dinámica uterina y se expande el HRP. Existe
compromiso del estado general, palidez taquicardia, pero las cifras de presión pueden aparentar normalidad si existe SHE. La
hemorragia genital (78%) es rojo oscura, sin coágulos o muy lábiles; es posterior a la presencia del dolor y decididamente menor que el
compromiso del estado general, ya que la sangre proveniente del HRP debe buscar camino, separando las membranas de la pared
uterina para salir al exterior. El sangrado es de inicio súbito y cuantía variable (lo que no guarda necesariamente relación con la
gravedad del cuadro). Según su ubicación, si el HRP aumenta, es posible observar en horas que el útero crece. El dolor uterino se
expresa con reblandecimiento y dolor de espalda en el 66% de los casos. La irritabilidad del útero va progresando: contracciones
uterinas, polisistolía e hipertonía (20%), contractura, palpándose finalmente un útero de consistencia “leñosa”, típico de este cuadro. Es
difícil palpar al feto y precisar su presentación debido a la irritabilidad uterina. La auscultación muestra sufrimiento fetal (60%) o
muerte fetal (15-35%). En el tacto vaginal dilatación cervical, la que progresa rápidamente debido a la hiperactividad uterina. Las
membranas están tensas y al romperse, el líquido amniótico presenta color “vinoso”, al estar mezclado con sangre y hemoglobina
procedente del HRP. Ecográfico. Su utilidad es limitada, ayudando al diagnóstico diferencial con la placenta previa dado que permite
descartarla. También tiene utilidad para el seguimiento de los hematomas retroplacentarios en pacientes con DPPNI asintomáticos en
el segundo trimestre ó inicios del tercero. Un nuevo campo de investigación es el intento de mejorar la sensibilidad de este método
diagnóstico mediante el uso del doppler, para ver la vascularización y áreas de perfusión placentaria. Pruebas diagnósticas
complementarias. Se han buscado pruebas que nos ayuden en el diagnóstico, ninguna de ellas es de utilidad clínica. Signos
cardiotocográficos, un buen registro no es motivo de tranquilidad, pues se puede producir un rápido deterioro del estado fetal. Niveles
de Ca125. Niveles de dímero-D. Trombomodulina. Es un marcador de daño endotelial. α-fetoproteína en suero materno. Se encuentra
aumentada. TRATAMIENTO: La interrupción de la gestación se realizará por la vía más rápida sin tener en cuenta la edad gestacional, el
útero debe ser evacuado lo más rápido posible, la decisión de realizar parto vaginal dependerá de que las condiciones obstétricas sean
ideales para una terminación rápida y sobre todo cuando existan signos de coagulopatía, se realizará tacto vaginal para estimar las
horas que faltan para la terminación del parto, de acuerdo a las condiciones halladas (borramiento, dilatación, altura de la
presentación). Además con el tacto, si hay trabajo de parto, se descarta la presencia de placenta previa. Se romperá las bolsas de las
aguas tan pronto como sea posible aun sin tener en cuenta el método de parto a emplearse, con ésta maniobra disminuye la presión
intraamniótica, se reduce la extravasación sanguínea y se abrevia significativamente la duración del parto. Si éste no hubiera

CURSO ENARM CMN SIGLO XXI TEL: 36246001 Pharmed Solutions Institute PÁGINA 320
MANUAL DE TRABAJO DEL CURSO ENARM CMN SIGLO XXI
comenzado, se puede intentar la inducción mediante la infusión intravenosa continua de oxitocina. Ambas conductas, parto
espontáneo o inducido, están justificadas por la gran rapidez con que se produce el parto en estas pacientes. Si en el transcurso del
trabajo de parto se constata una alteración de los latidos fetales, se interrumpirá el trabajo de parto y se indicará la cesárea sin
dilación. La cesárea abdominal se realiza: Si la condición de la paciente se agrava, si el feto está vivo, para prevenir su muerte, si fracasa
la inducción del parto o si el parto se prolonga, en los casos más graves, cuando se sospecha una apoplejía uterina, ya que permite no
solo evacuar el útero con rapidez, sino también decidir si éste podrá conservarse o no. La precaución del obstetra no termina con la
evacuación del útero, ya que debe controlarse todavía la correcta retracción uterina, debiendo recordar la posibilidad de instalación de
un cuadro de atonía. En los casos severos de apoplejía, o si después del alumbramiento no hubiera respuesta a los ocitócicos y continúa
la hemorragia por atonía uterina se efectuará inmediatamente la histerectomía. El puerperio inmediato es el momento más crítico de
esta complicación. Se pondrá especial atención en: Mantener la volemia y las constantes hematológicas con soluciones salinas y sangre
preferentemente fresca. Se evitaran los expansores plasmáticos, por ser antiagregantes plaquetarios. Control estricto de signos vitales.
Control de la diuresis horaria. Observación rigurosa de los valores de crasis sanguínea. COMPLICACIONES DE LA FORMA GRAVE:
Accidente de Couvelaire: apoplejía uteroplacentaria; La sangre derramada invade la pared uterina con zonas de extensas hemorragias
miometriales que disocian los haces musculares y puede difundirse hacia el tejido subperitoneal, trompas, ovarios y a veces ligamentos
anchos. A causa de esta infiltración sanguínea el miometrio pierde su propiedad contráctil. CID; Se da en el 30 % en los DPPN masivos
con muerte fetal. Se produce por el paso de tromboplastina a la circulación materna y activación de la coagulación. La baja
concentración de fibrinógeno plasmático depende del atrapamiento de fibrina en el hematoma retroplacentario, esto provoca que la
hemorragia sea incoercible y la sangre no coagule. Insuficiencia Renal Aguda; l colapso circulatorio periférico y la sobredistención
uterina provocan isquemia en las zonas corticales de los riñones. Shock Hipovolémico; Se produce por la hemorragia brusca y masiva,
con o sin exteriorización de la sangre por los genitales. Necrosis de la Hipófisis: (Síndrome de Sheehan). La hipófisis anterior puede
sufrir una necrosis total o parcial debido a trombos o espasmos de los vasos del sistema porta. Se manifiesta en el puerperio por
agalactia como síntoma inicial, ya que son las células secretoras de prolactina las que primero se ven afectadas por la isquemia,
posteriormente se evidencian los síntomas de esfera gonadal, tiroidea y suprarrenal. Aunque es poco frecuente.

CASO CLINICO
Se trata de femenino con 40 semanas de gestación la cual acude a consulta debido a que inicia trabajo de parto, refiere que desde hace
2 horas inicia con dolor en la región abdominal baja tipo contracciones, además de presencia de sangrado leve pero continuo, a la
exploración física usted palpa contracciones de 2 a 3 en 10 minutos con una duración de 40 segundos aproximadamente, al realizar
tacto encuentra borramiento del 40 % y dilatación de 3 cm, sin embargo presenta sangrado importante, las constantes vitales de la
madre se encuentra dentro de parámetros normales, no así del producto el cual observa frecuencia cardiaca aproximadamente de 160
a 180 lpm.

PREGUNTA
Cuál de los siguientes factores de riesgo esta altamente relacionado con desprendimiento de placenta normoinserta.

RESPUESTA
a.- Mayor de 35 años de edad.
b.- Muerte Materna.
c.- Cesarea.
d.- Miomectomia.

CASO CLINICO
Paciente de 37 años de edad, acude por abdominalgia de 1 día de evolución, a las 25,3 semanas de gestación por una rotura prematura
de membranas (RPM). Las ecografías y analíticas practicadas hasta ese momento eran normales. Como antecedentes obstétricos
destacan una cesárea (incisión transversal) 2 años antes, a las 28 semanas de gestación por riesgo de pérdida del bienestar fetal y un
aborto espontáneo previo. A la exploración presenta abdomen ligeramente doloroso a la palpación profunda en hipogastrio y en el
tacto vaginal se aprecia cuello uterino acortado un 20% y cerrado. La paciente refiere pérdida de líquido por genitales externos, que se
confirma con el espéculo con el que se observa salida de líquido amniótico claro a través del cérvix. En la ecografía abdominal se
observa feto único con latido cardíaco positivo, biometrías de 26 semanas, placenta anterior normoinserta y líquido amniótico en
cantidad normal. Al tratarse de una RPM pretérmino de 25 semanas, se deriva a nuestro centro previa administración de una dosis de
betametasona 12 mg intramuscular.

PREGUNTA
Cuál de los siguientes factores de riesgo esta altamente relacionado con desprendimiento de placenta normoinserta.

RESPUESTA
a.- Mayor de 35 años de edad.
b.- Ruptura prematura de membranas
c.- Cesarea.
d.- Miomectomia.

CURSO ENARM CMN SIGLO XXI TEL: 36246001 Pharmed Solutions Institute PÁGINA 321
MANUAL DE TRABAJO DEL CURSO ENARM CMN SIGLO XXI
ANEMIAS EN EL EMBARAZO. CIENCIAS BASICAS: La anemia es el problema hematológico más común en el embarazo. Es referida como
un proceso dilucional secundario al aumento del volumen plasmático. Sin embargo existen deficiencias nutricionales, hemólisis y otras
enfermedades que pueden causar anemia significativa y ser capaces de afectar a la madre como al feto. La anemia de acuerdo a la OMS
en la mujer embarazada se define como la concentración de hemoglobina al nivel del mar menor a 11g/dl y el hematócrito menor que
33 % durante el tercer trimestre de la gestación. La anemia puede relacionarse con muerte fetal nacimientos de bajo peso y
anormalidades del feto. La anemia sin embargo puede ser un marcador de factores nutricionales, sociales o ambientales más que la
causa de esos problemas. Cuando la Hb cae a niveles inferiores a 6-7g/dL se pueden tener efectos adversos en la madre y el feto. La
anemia menos severa (8-10g/dL) es de poco riesgo para la madre pero puede poseer mayor riesgo para el infante. Un Hto elevado
puede asociarse no sólo con partos prematuros, baja de peso fetal o muerte perinatal sino también con hipertensión materna y
toxemia. SALUD PUBLICA: Existe una prevalencia de anemia entre las mujeres embarazadas del 42%.4. En México se encontró anemia
en el 21.6% de 500 mujeres embarazadas. Tomando en cuenta mujeres con embarazo normal y complicado se encontró la prevalencia
del 22.4%. Ciertos grupos de mujeres están en mayor riesgo para desarrollar anemia durante el embarazo, probablemente en parte a
factores familiares, sociales, económicos, nutricionales y a falta de cuidado prenatal. CLASIFICACION: Durante la gestación, creemos útil
tener en cuenta que las anemias que acompañan al embarazo pueden ser agrupadas en 2 categorías: 1. Directamente relacionadas con
la gestación: a) Ferropénicas, b) Megaloblásticas, c) Hipoplásicas. 2. Que no guardan relación directa con la gestación: a) Anemias por
hematíes falciformes. b) Otras anemias hemolíticas y raras. ANEMIA FERROPENICA: En países en desarrollo el 83-95% de las mujeres
embarazadas anémicas tienen deficiencia de hierro. El embarazo y el parto representan una pérdida de1 a 1,3 g de hierro, que se extrae
fundamentalmente delos depósitos de hierro en el sistema reticuloendotelial y en el parénquima hepático, en forma de hemosiderinao
ferritina. Con frecuencia, las embarazadas enfrentan estas necesidades con las reservas de hierro exhaustas. Entre los factores que
llevan a ello se encuentran: menstruaciones abundantes embarazos con escaso periodo intergenesico, dietas con bajo contenido en
hierro, embarazos anteriores sin un adecuado suplemento férrico, partos con sangramientos durante el alumbramiento o el puerperio,
parasitismo intestinal, baja absorción del hierro y otros. Estas anemias son pobres en signos y por lo general, son asintomáticas; puede
observarse palidez cutaneomucosa y cierta tendencia a la fatiga. Las formas más severas presentan un síndrome anémico dado por:
laxitud, "cansancio de muerte”, irritabilidad, astenia, nerviosismo, cefalea, anorexia y otros. En los casos de anemias muy severas
alteraciones del apetito, pirosis, ardor lingual y bucal, flatulencia, constipación y es posible la aparición de glosistis. En ocasiones puede
haber manifestaciones de insuficiencia cardiacas y cardiomegalia. A veces las pacientes pueden
tener dolores de tipo neurálgico, adormecimiento de las extremidades, sensación de hormigueo,
trastornos vasomotores y otros. Al realizar el examen físico, se detecta palidez cutaneomucosa; las
uñas de las manos, y a veces las de los pies, aparecen opacas y sin brillo, y se rompen con facilidad.
Con frecuencia, la auscultación permite escuchar soplos anémicos funcionales. Diagnóstico: durante la atención prenatal el estudio
sistematico de la Hb y Hto que deben hacerse cada 6-12 semans, permitirá el diagnostico precoz de anemia. Si la hemoglobina está por
debajo de 110 g/L, se considera que hay anemia. Independientemente de la clasificación dada por la OMS (1991), creemos que desde el
punto de vista práctico conviene considerar 3 grados: ver cuadro anexo. Las anemias ferriprivas se caracterizan por tener: 1.
Hemoglobina y hematocrito disminuidos. 2. Hierro sérico disminuido. 3. Discapacidad total elevada. 4. Índice de saturación disminuido.
5. Protoporfirina eritrocitaria elevada. 6. Lámina periférica normocítica hipocrómica. Tratamiento profiláctico: administración de hierro
VO desde la primera consulta prenatal. Ingestión diaria de 60mg de hierro elemental, en pacientes con feto único. Debe administrarse
en forma de sales ferrosas: 1. Sulfato ferroso: tableta de 300 mg = 60 mg de Fe elemental. 2. Gluconato ferroso: tableta de 300 mg = 36
mg de Fe elemental. 3. Fumarato ferroso: tableta de 200 mg = 65 mg de Fe elemental. La administración de hierro debe proveer al
organismo la cantidad suficiente de este elemento para lograr la regeneración de la hemoglobina y para la reserva. Esto se consigue
administrando 2 a 3 tabletas diarias de sales ferrosas durante el embarazo y hasta 6 meses después del parto. Tratamiento curativo: La
VO es de elección siempre que sea posible en dosis de 600 a 1 200 mg/día, que equivaldrían a 120 o 180mg, preescrito en 1-2 tabletas
media hora antes de desayuno, almuerzo y comida, ya que es preferible separarlo de los alimentos. Puede indicarse además la
administración de 100mg de ácido ascórbico diariamente. La administración durante las comidas presenta mejor tolerancia, aunque es
menor su absorción. Las sales de hierro no deben acompañarse de leche, té, café o huevo, ya que éstos interfieren en su absorción. El
tratamiento debe ser sostenido por lo menos de 2 meses después de normalizado el hematocrito y la hemoglobina. Una forma práctica
de calcular la dosis total de hierro en miligramos sería: Hemoglobina normal-hemoglobina de la paciente × 255 mg de Fe. La vía IM sería
necesaria en las circunstancias siguientes: 1. Intolerancia gástrica al hierro oral. 2. Cuando este contraindicada su administración como
en los casos de gastritis, úlcera, diverticulosis y otras afecciones digestivas. 3 Falta de respuesta al tratamiento oral. 4. Síndrome de
malabsorción intestinal. 5. Anemia intensa (85 g/L o menos) después de las 34 semanas. ANEMIA MEGALOBLASTICA: En el embarazo,
existe un aumento de necesidades de ácido fólico y Vit., B12 para la síntesis del ADN y del ARN, debido al rápido crecimiento celular del
embrión y del feto en desarrollo. La anemia megaloblatica del embarazo es causada por deficiencia de ácido fólico y vit., B12. La
gestante también puede sufrir una deficiencia de ácido ascórbico, que se asocia con la de ácido fólico. En la lámina periférica se
encuentra macrocitosis, punteado basófilo (policromatofilia), leucocitos de Pitaluga y macroplaquetas. La determinación de ácido fólico
ofrece cifras por debajo de 4µg/l. la medula osea es megaloblastica. La deficiencia de ácido fólico y de ácido ascórbico determinan un
aumento de las complicaciones infecciosas de madre, abortos, partos prematuros, RPM y otros. Tratamiento: profiláctico 1. Ácido
fólico: 1 mg/día (tabletas). 2. Ácido ascórbico: 200 mg/día (tabletas). Tratamiento específico: Se administra ac. fólico 5-10mg/día.
Además del ácido fólico debe administrarse hierro en dosis terapéutica, ya que la transformación de la médula ósea megaloblástica en
normal, requiere gran cantidad de hierro. Antes de iniciar el tratamiento debe buscarse la existencia de una infección que condicione la
anemia (la infección urinarias es la mas frecuente) y puede hacerla refractaria a tratamiento. Una característica de esta anemia es su
remisión espontánea después del parto. ANEMIA HIPOPLÁSICA: Se le relaciona con el embarazo, y se considera por algunos como una
manifestación de toxemia. Es rara y de gravedad variable. Puede tener remisiones parciales o completas y en algunas ocasiones,
desaparecer espontáneamente después del parto. Puede provocar muerte fetal y parto pretérmino. Diagnóstico: la anemia es de
desarrollo rápido, con palidez, fatiga y taquicardia. Las manifestaciones clínicas dependen de los grados de la anemia, la
granulocitopenia y la trombocitopenia. Puede haber formas globales, con caída de los 3 sistemas o formas parciales con la afección de 1
solo de ellos. Laboratorio: fundamentan el diagnóstico las determinaciones de: 1. Hemoglobina (muy baja). 2. Hematocrito (reducido).
3. Trombocitopenia. 4. Hierro sérico (elevado). 5. Médula ósea hipocelular con depresión selectiva o de los 3 sistemas (pancitopenia).

CURSO ENARM CMN SIGLO XXI TEL: 36246001 Pharmed Solutions Institute PÁGINA 322
MANUAL DE TRABAJO DEL CURSO ENARM CMN SIGLO XXI
Tratamiento: Para tratar la anemia hipoplásica se dispone de recursos muy limitados. El tratamiento con hierro, ácido fólico y vitamina
B12 ha resultado ineficaz. En el aspecto médico, se aconseja: una serie de medidas para prolongar la vida de la paciente, como son: 1.
Transfusiones de concentrados de glóbulos rojos, si la anemia fuera lo fundamental. 2. Transfusión de plaquetas. 3. Administración de
antibióticos (no profilácticos y con antibiograma). 4. Administración de anabólicos: nerobol: 1 a 3 mg/kg por vía oral, diariamente.
ANEMIA POR HEMATÍES FALCIFORMES O DREPANOCITEMIA: de acuerdo con los síntomas clínicos de la Hb.S parece trasmitirse por
medio autosómicos recesivo (solo están afectados los hemocigotos SS), se considera más bien el resultado de herencia autosómica
codominante, puesto que el genotipo heterocigoto ATS (rasgo de células falciformes), puede bajo ciertas circunstancias, producir
morbilidad grave e incluso la muerte. En los individuos SS, la hemólisis es consecuencia del secuestro y de la destrucción de los
hematíes falciformes en el sistema reticuloendotelial, así como su destrucción intravascular originada por trauma mecánico; de ahí que
la anemia crónica sea la regla. La hemoglonina S y la hemoglobina C son resulta do de la sustitución del ácido glutámico en la posición 6
de la cadena de la globina, por valina y lisina respectivamente. En la crisis drepanocítica se produce el fenómeno de deformación
semilunar de los hematíes y puede hacerse irreversible. El feneomeno de falciformes es debido a que la propiedad fundamental que
diferencia a la hemoglobina S de la hemoglonina A es la baja solubilidad de la primera en su forma desoxigenda, lo cual provoca su
precipitación debido a la formación de polimeros, agergados constituidos por la formación de varias moléculas de hemoglobina S, que
forman un gel semisólido que se extiende a lo largo de los hematíes y da lugar a la defiormacion característica.

CASO CLINICO
Femenino de 16 años de edad originaria de tabasco, la cual acude por cansancio, fatiga, adinamia, actualmente cursa con embarazo de
28 semanas de gestacion, en control de rutina muestra Hematocrito de 39 y hemoglobina de 10, desde la semana 20 fue diagnosticada
con anemia, fue suplementada con hierro, sin embargo comparado con el estudio anterior no hay mejoría, actualmente se observa
edema de miembros pélvicos ++,

PREGUNTA
Considerando los datos de laboratorio cual es su conducta a seguir para resolver el presente caso.

RESPUESTA
a.- Transfusión Sanguínea.
b.- Complejo B
c.- Hierro.
d.- Realizar un estudio de tinción.

CASO CLINICO
Mujer de 24 años, Área gineco-obstétrica: menarquía a los 12 años; ritmo menstrual de 28x3-4 días, en ocasiones con coágulos; inicia
vida sexual activa a los 22 años y cursa el 6º mes de su primer embarazo de evolución normal aunque tuvo emesis gravídica leve
durante el primer mes pero no acude a control gineco-obstétrico hasta el sexto, días antes de la consulta hematológica refiriendo
entonces astenia, somnolencia y se le observa pálida. En la consulta de hematología se confirman tales datos y además menciona “pica”
y “pagofagia” de varios meses o incluso un año de evolución. Signos vitales normales a la exploración, observando pelo frágil y
platoniquia, se tiene la impresión de palpar el bazo al inspirar profundo. Citología hemática: Hemoglobina (Hb): 10.5 g/dL; Hematocrito:
0.32/l; Eritrocitos: 5.000.000/mm3; Volumen eritrocitario medio: 70 fl; Hemoglobina corpuscular media: 26pg; Concentración de
hemoglobina corpuscular media: 30g/dL; Leucocitos y fórmula diferencial normales; Plaquetas: 480.000/mm3

PREGUNTA
Cual es el diagnostico mas probable?

RESPUESTA
a.- Anemia normocítica normocrómica
b.- Anemia megaloblástica normocrómica
c.- Anemia microcítica normocrómica
d.- Anemia microcítica hipocrómica

INFECCIONES DE VIAS URINARIAS (IVU). CIENCIAS BASICAS: La infección urinaria (IVU) es una entidad clínica que se asocia
frecuentemente al embarazo; las variantes clínicas son la bacteriuria asintomática, la cistouretritis y la pielonefritis. Durante la
gestación se producen una serie de cambios fisiológicos que aumentan el riesgo de presentar infecciones del tracto urinario: Dilatación
ureteral secundario a la acción de progesterona y a la compresión uterina, reflujo vesico-ureteral, estasis vesical, aumento del filtrado
glomerular con glucosuria y amnioaciduria con elevación del pH urinario. SALUD PUBLICA: La infección urinaria es una de las
complicaciones médicas más frecuentes del embarazo, únicamente superada por la anemia y la cervicovaginitis; si no es diagnosticada
y adecuadamente tratada, puede llevar a un incremento significativo en la morbilidad en la madre y el feto. El 20% de las embarazadas
presentan una infección vías urinarias bajas (ITU) en el curso de la gestación. Un 10% de los ingresos hospitalarios en gestantes se
deben a IVU. La bacteriúria asintomática no tratada es un factor de riesgo de pielonefritis, bajo peso al nacer y amenaza de parto
prematuro. CLASIFICACION: La infección en el embarazo es clasificada según el sitio de proliferación bacteriana, de la siguiente manera:
Bacteriuria asintomática=Infección urinaria baja. Cistitis, Pielonefritis=Infección urinaria alta. PATOGENIA: Constituyen factores de
riesgo de desarrollar una infección urinaria en la gestación los siguientes factores: 1. Bacteriúria asintomática 2. Historia de ITU de
repetición 3. Litiasis renal 4. Malformaciones uroginecológicas 5. Enfermedades neurológicas (vaciado incompleto, vejiga
neurógena…) 6. Reflujo vesico-ureteral 7. Insuficiencia renal. 8. Diabetes mellitus 9. Infección por Chlamydia trachomatis 10.
Multiparidad 11. Nivel socioeconómico bajo. Entre los diferentes agentes etiológicos que se conocen como causantes de las IVU, la
Escherichia coli, procedente de la flora enterobacteriana es el microorganismo más habitual y causante de la mayor parte de estas

CURSO ENARM CMN SIGLO XXI TEL: 36246001 Pharmed Solutions Institute PÁGINA 323
MANUAL DE TRABAJO DEL CURSO ENARM CMN SIGLO XXI
infecciones (80-90% casos). Le siguen por orden de importancia: Proteus mirabilis, Kelbsiella pneumoniae, Enterococcus spp y
Staphylococcus saprophyticus. A mayor edad gestacional, mayor probabilidad de Gram positivos principalmente SGB. BACTERIÚRIA
ASINTOMÁTICA: Se denomina bacteriuria asintomática (BA), a la presencia de bacterias en orina cultivada (más de 100.000 colonias
por ml) sin que existan síntomas clínicos de infección del tracto urinario. La mayoría de bacteriurias asintomáticas se dan en el primer
trimestre de la gestación. Aparece en un 5-6% de los casos. El 25% de las bacteriúrias asintomáticas no tratadas desarrollaran una
pielonefritis aguda vs el 3% de las tratadas. Hasta un 30% de las bacteriúrias asintomáitcas tratadas presentan una recaída a pesar del
correcto tratamiento antibiótico. Diagnóstico: es microbiológico: cultivo orina 100.000 unidades formadoras de colonias (UFC). El
diagnóstico se establece mediante el urocultivo cuantitativo. La muestra de orina debe ser obtenida bajo determinadas condiciones de
asepsia: después de limpieza de genitales externos y recogida del chorro medio de la micción. NO es necesario para realizar un
urocultivo el sondaje vesical. Si el resultado del urinocultivo es de “orina contaminada”, debe repetirse, haciendo hincapié en las
medidas de asepsia para una correcta toma de la muestra. Tratamiento: Normalmente se dispone de antibiograma. Utilizar el
antibiótico de espectro más reducido: empezar por amoxicilina, después amoxicilina-clavulánico....etc. En caso de NO disponer de
antibiograma se propondrá como tratamiento empírico de la bacteriúria asintomática: Amoxicilina-clavulánico 500 mg/8 h vo x 7 dias.
En caso de alergia a betalactámicos: Fosfomicina trometamol 3 g vo (unidosis o pauta corta dos dosis separadas por 3 días). Otras
alternativas terapéuticas de mayor espectro que deben reservarse para casos de resistencias són: Cefuroxima 250 mg/12 h vo x 7 días ó
Cefixima 400 mg/24 h vo x 7 días ó En caso de alergia a betalactámicos: Nitrofurantoína 50-100 mg/6 h vo x 7 días. Seguimiento:
Comprobar curación con urinocultivo a los 15 días después de tratamiento. En caso de recidiva, actuar según antibiograma y si no se
dispone de él, ampliar el espectro. Repetir urinocultivo cada trimestre de gestación. CISTITIS AGUDA: Es un síndrome caracterizado por
una clínica de urgencia miccional, disuria y dolor suprapúbico en ausencia de síntomas de afectación sistémica (fiebre) y dolor lumbar.
Aparece en un 1.3% de las gestaciones. La mayoría de cistitis agudas se presentan en el segundo trimestre del embarazo. Diagnóstico:
clínica sugestiva y cultivo orina positiva (≥100.000 UFC). Puede aparecer hematuria macro/microscópica. Recordar normas de recogida
de muestra de orina para urinocultivo: NO requiere necesariamente de sondaje vesical, pero debe recogerse en determinadas
condiciones de asepsia: después de limpieza de genitales externos y recogida del chorro medio de la micción. Tratamiento: idealmente
según antibiograma específico, pero puede tratarse empíricamente si la clínica es muy sugestiva y no se dispone de resultado de
urinocultivo. El tratamiento suele ser empírico porque no solemos disponer de antibiograma Cefuroxima 250 mg/12 h vo x 7 días. Si
alergia a betalactámicos: Fosfomicina trometamol 3 g vo (unidosis o pauta corta dos dosis separadas por 3 días). Si disponemos de
antibiograma, se debe seguir la misma estrategia terapéutica que en el caso de bacteriúria asintomática y empezar con el antibiótico de
menor espectro: Amoxicilina 500 mg/8 h vo x 7 días. Amoxicilina-clavulánico 500 mg/8 h vo x 7 dias. Cefixima 400 mg/24 h vo x 7 díaS.
En caso de alergia a betalactámicos: Nitrofurantoína 50-100 mg/6 h vo x 7 días. Comprobar curación con urinocultivo a los 15 días
después de tratamiento. En caso de recidiva, actuar según antibiograma y si no se dispone de él, ampliar el espectro. Repetir
urinocultivo cada trimestre de gestación. PIELONEFRITIS AGUDA: Infección del parénquima renal que aparece en 1-2% de las
gestaciones y cuya prevalencia aumenta al 6% en gestantes en las que no se ha realizado cribado de bacteriúria asintomática durante el
embarazo. El 80-90% de las pielonefritis aparecen en el 2º, 3º trimestre de la gestación y en el puerperio. La vía de entrada más
frecuente es la ascendente a través de la vía urinaria aunque en pacientes inmunodeprimidos la vía de diseminación puede ser la
hematógena (principalmente en diabéticos, tratamiento con corticoides, enfermedades sistémicas...). La localización más frecuente es
la afectación renal derecha (50% de los casos); en un 25% de casos la localización es izquierda y en un 25%, bilateral. Si la paciente tuvo
una bacteriúria asintomática, es posible que el germen causante de la pielonefritis sea el mismo. Clionica; Dolor costovertebral con
puñopercusión lumbar positiva, fiebre, náuseas y vómitos, en general, NO cursa con clínica de cistitis aguda. Tratamiento ambulatorio:
Si NO cumple criterios de hospitalización, es posible el control ambulatorio en dispensario de curas de enfermería (CENF) cada 24h: De
elección ceftriaxona 1 g /24 h ev o im, en caso de alergia a betalactámicos, gentamicina 240 mg/24h ev o 80 mg/8h im. Si mejoría
clínica del cuadro febril, a las 48-72 h podrá pasarse a terapia oral. Si se dispone de antibiograma, pasar el antibiótico de menor
espectro: amoxicilina, después amoxicilina-ac. Clavulanico. Si NO se dispone de antibiograma, iniciar tratamiento empírico con
cefuroxima 250mg/12 h vo hasta cumplir 14 días de tratamiento. Sólo se utilizará como antibiótico alternativo la cefixima 400 mg /24 h
vía oral x 14 días en caso de sospecha de resistencias a la cefuroxima (por antibiogramas de urinocultivos anteriores). En caso de
persistencia de síndrome febril y no respuesta clínica tras 72h (3 días) de tratamiento médico ambulatorio, indicar ingreso hospitalario.
Tratamiento hospitalario: En general el tratamiento de la pielonefritis será ambulatorio a excepción de aquellas pacientes que cumplan
algunos de los criterios de ingreso hospitalario: Criterios de ingreso hospitalario fiebre ≥ 38ºC, sepsis, deshidratación, edad gestacional
≥ 24 semanas, cuadro clínico de amenaza de parto prematuro, pielonefritis recurrente, comorbilidad, intolerancia oral, fracaso de
tratamiento ambulatorio tras 72 h (3 días), no posibilidad de tratamiento ambulatorio. 1.- Hidratación agresiva: Primer día:
Sueroterapia a una perfusión de 150 ml/h. Segundo día perfusión de100 ml/h. 2.- Tratamiento parenteral antibiótico hasta 48 horas
afebril: En gestante: Primera elección: 1.- Ceftriaxona 1 g/24 h IV Alergia a betalactámicos: 2.- Gentamicina 80 mg/8 h IV. En puérpera:
Primera elección: 1.- Ceftriaxona 1 g/24 h ev + ampicilina 1 g/6 h ev (se debe cubrir también el Enterococo faecalis). Alergia a
betalactámicos: 2.- Gentamicina 240 mg/24 h IV. En casos de sepsis y/o pielonefritis con sospecha de gérmenes multiresistentes o
pacientes con sondajes o vías con tiempo prolongado: El tratamiento antibiótico deberá cubrir Pseudomona auroginosa u otros
gérmenes multiresistentes: 1.- Ceftacidima 1 g/8 h IV. Alergia a betalactámicos: 2.- Amikacina 15 mg/kg/24h IV. En el caso de sospecha
de infección por Enterococco faecalis añadir ampicilina 1 g/6 h IV o iniciaremos tratamiento en monoterapia con piperacilina-
tazobactam 4g/8 h IV.

CASO CLINICO
Mujer de 25 años de edad con 34ª semana de gestación. Antecedentes de aborto espontáneo cuatro años antes y un parto hacía dos
años. Hacía tres días inicia con disuria, prurito, urgencia urinaria, dolor suprapubico, con descarga vaginal grumosa.

PREGUNTA
Cual es el tratamiento antibiótico mas adecuado para este caso.

RESPUESTA

CURSO ENARM CMN SIGLO XXI TEL: 36246001 Pharmed Solutions Institute PÁGINA 324
MANUAL DE TRABAJO DEL CURSO ENARM CMN SIGLO XXI
a.- Ampicilina.
b.- Cefalexina.
c.- Nitrofurantoina.
d.- Trimetropin con sulfametoxazol.

CASO CLINICO
Paciente de 17 años, nulípara, con antecedentes mórbidos de bacteriuria asintomática con tres episodios de cervicovaginitis. Ingresó a
las 31 semanas de gestación por trabajo de parto prematuro. Al ingreso se le practicó una amniocentesis obteniéndose líquido
amniótico con meconio y estudio citoquímico sugerente de infección intraamniótica. Se tomó cultivo del líquido amniótico y se inició
tratamiento. El trabajo de parto fue acelerado con oxitocina, resolviéndose por vía vaginal.

PREGUNTA
Cual de las siguientes complicaciones maternas fetales se asocia más frecuentemente?

RESPUESTA
a.- Corioamnionitis.
b.- Pielonefritis.
c.- Sepsis puerperal.
d.- Urosepsis.

CASO CLINICO
Paciente de 17 años primigestante con embarazo de 36 semanas por fum cierta y confiable y 38 semanas por ecografia de segundo
trimestre. Quien refiere cuadro clinico de 1 mes de evolucion caracterizado por edema de miembros inferiores. niega cefalea, niega
acufenos, niega fosfenos. refiere dolor en region lumbar de moderada intensidad no irradiado y disuria. refiere movimientos fetales
presentes. niega genitorragia, niega hidrorrea.

PREGUNTA
Cual es la conducta a seguir mas adecuada?

RESPUESTA
a.- Ceftriaxona.
b.- Gentamicina
c.- Amoxicilina.
d.- Cefuroxima.

CURSO ENARM CMN SIGLO XXI TEL: 36246001 Pharmed Solutions Institute PÁGINA 325
MANUAL DE TRABAJO DEL CURSO ENARM CMN SIGLO XXI
ESTADOS HIPERTENSIVOS EN EL EMBARAZO. CIENCIAS BASICAS: Definición: Se habla de hipertensión en el embarazo cuando la
tensión arterial diastólica es > 90mmHg y la sistólica es >140mmHg, o un incremento en la TA sistólica de por lo menos 30mmHg del
valor basal o de diastólica de por lo menos 15mmHg sobre el valor basal. Proteinuria: Excreción urinaria de proteínas mayor de 30mg/dl
o más en tiras reactivas (se requieren 2 determinaciones o más en un lapso de 6 horas en ausencia de infección). 300mgs o más en una
colección de orina en 24h. SALUD PUBLICA: Los trastornos hipertensivos durante la gestación, son la primera complicación médica en
muchos países del mundo, constituyendo una de las principales causas de morbilidad y mortalidad materna, fetal y neonatal. 7-10% de
los embarazos se complican con HTA. La preeclampsia/eclampsia origina el 70% de los estados hipertensivos, y el 30% lo representan
pacientes con hipertensión crónica preexistente durante el embarazo. En México su prevalencia varía entre 7-10% de la población
gestante, existe mayor incidencia antes de los 20 años y después de los 35; aproximadamente 75% de los casos corresponden a
primigestas. CLASIFICACION, de los trastornos hipertensivos durante el embarazo: A) Enfermedad hipertensiva inducida por el
embarazo: Preeclampsia leve. Preeclampsia severa. Inminencia de eclampsia. Eclampsia. Síndrome de HELLP. Preeclampsia recurrente.
Enfermedad hipertensiva no clasificable (imposibilidad de clasificar por carecer de elementos necesarios o por haberse instituido
tratamiento previo a su estadificacion). Hipertensión transitoria (HTA después de la semana 20 o en las primeras horas posparto sin
otros signos de preeclampsia). B) Enfermedad vascular crónica hipertensiva: Hipertensión sistémica esencial (HTA independiente de la
gestación o anterior a las 20 semanas y que persiste más de 6 semanas posparto y que no sea a consecuencia de lesión de alteración
anatómica o funcional renal). Hipertensión crónica con enfermedad hipertensiva agregada (HTA previa al embarazo agregándose
preeclampsia, puede haber elevación de ácido úrico igual o mayor de 6mg/dl). De acuerdo a la Clasificación del Colegio Americano de
Obstetricia y Ginecología, se identifican 4 categorías: HIPERTENSION CRONICA: Coexistencia de hipertensión de novo antes de las 20
SDG, al igual que si tiene el antecedente de hipertensión preexistente antes del embarazo. Puede ser: primaria (esencial) o secundaria
(renal, suprarrenal, etc). Las mujeres con hipertensión crónica tienen un riesgo de 10-20% de desarrollar preeclampsia. Es conveniente,
además, en las pacientes con hipertensión crónica, establecer si ya existe daño en algún órgano blanco antes del embarazo, incluyendo
hipertrofia ventricular izquierda, retinopatía y/o enfermedad renal. En canto a los medicamentos, suspender el tratamiento con
medicamentos del grupo de los IECA, así como ARA II. Existen comunicaciones sobre restricción del crecimiento fetal intrauterino,
oligohidramnios, insuficiencia renal y muerte neonatal. Hay que reemplazarlos por alfametil-dopa, labetalol o los antagonistas de
calcio. HIPERTENSION ARTERIAL CRONICA CON PREECLAMPSIA SOBREAGREGADA: Pacientes con hipertensión crónica, que presentan
descompensación de las cifras tensionales y aparición o incremento de proteinuria después de las 20 SDG. El pronóstico es peor que
con cada entidad por separado. Criterios diagnósticos: Aparición de proteinuria por primera vez, después de las 20 semanas de
gestación. Incremento súbito de la proteinuria en pacientes que la presentaban antes del embarazo. Incremento súbito de la presión
arterial en la gestante que la tenía bien controlada. Trombocitopenia (menos de 100 000 plaquetas/mL). Incremento de enzimas
hepáticas por encima de valores normales. PREECLAMPSIA: la describimos más adelante y por ultimo HIPERTENSION GESTACIONAL:
Presión arterial sistólica mayor o igual a 140mmHg y/o presión arterial diastólica mayor o igual a 90mmHg en una mujer previamente
normotensa, después de la semana 20 de gestación. En dos tomas separadas por un intervalo de 4 a 6h entre una y otra con ausencia
de proteinuria y se recupera después del parto. Las mujeres con hipertensión gestacional con inicio antes de la semana 34 de gestación,
tienen probabilidad de 35% de desarrollar preeclampsia.

CASO CLINICO
Paciente femenino de 37 años de edad actualmente cursando su tercer embarazo con 14 semanas por FUM, acude a visita de control
prenatal cuenta con 17 SDG por USG, antecedentes personales patológicos de pre-eclampsia, además obesidad previa al embarazo,
cuenta con antecedentes familiares de diabetes y obesidad por parte de la madre e hipertensión por parte del padre, el cual fallece por
infarto al miocardio. A la exploración física se observa paciente gestante con obesidad grado I, sin tratamiento previo, se envía EGO, QS
y BH, donde se observa triglicéridos de 350, colesterol de 290, glucosa de 105 mg/dl, hemoglobina glucosilada de 7, tensión arterial en
dos ocaciones de 135/95 mmHg. Tratada con dieta hiposodica, hipocalorica y restricción de liquidos.

PREGUNTA
Cuál es el factor más importante que presenta la paciente, para considerar Hipertension Arterial Pre-existente.

RESPUESTA
a.- Trigliceridemia.
b.- Hipercolesterolemia.
c.- Obesidad.
d.- Cifras tencionales altas.

CASO CLINICO
Se trata de paciente femenino de 26 años de edad la cual acude a consulta de control prenatal actualmente contando con 21 semanas
de gestación, refiere que ha presentado cefalea ocasional, mareo y cansancio a la exploración física se observa edema de miembros
inferiores, la consulta anterior usted envió estudios de EGO, QS y BH, los datos de relevancia fue Hematocrito de 38, las constantes
vitales son de FR 18, FC 89, TA 160/90. MmHg. Con edema leve sin hematuria ni proteinuria.

PREGUNTA.
Cuál es la conducta a seguir mas adecuada.

RESPUESTA
a.- Restrición de liquidos y sal.
b.- Vigilancia estrecha de tencion arterial.
c.- Monitorización de vitalidad fetal.
d.- Iniciar con alfametildopa.

CURSO ENARM CMN SIGLO XXI TEL: 36246001 Pharmed Solutions Institute PÁGINA 326
MANUAL DE TRABAJO DEL CURSO ENARM CMN SIGLO XXI
PREECLAMPSIA. CIENCIAS BASICAS: Se define como la aparición de hipertensión y proteinuria después de la semana 20 del embarazo.
Se suele acompañar de edemas pero no es necesaria la presencia de éstos para ser diagnosticada. Es una enfermedad característica y
propia del embarazo de la que se pueden tratar los síntomas, pero sólo se cura con la finalización del mismo y si no se trata
adecuadamente puede ser causa de graves complicaciones tanto para la mujer embarazada como para el feto. CLASIFICACION:
Preeclampsia leve; presión arterial de 140/90mmHg o mas o elevación de 30mmHg en la sistólica y 15mmHg en la diastólica, cuando se
conocen las cifras basales previas, se presenta después de la semana 20 hasta 30 días posparto, existe proteinuria de más de
300mg/24h, ausencia de síntomas de vasoespasmo. Preeclampsia severa; presión arterial de 160/110mmHg o más, después de la
semana 20 hasta 30 días posparto, existe proteinuria mayor de 5g/24h, presencia de cefalea, acufenos, fosfenos, edema generalizado.
Inminencia de eclampsia; se establece el diagnóstico cuando después de la semana 20 de gestación, parto o puerperio (no más de 30
días), aparece uno o más de los siguientes datos: Presión arterial sistólica mayor a 185mmHg con presión arterial diastólica >115mmHg,
proteinuria mayor a 10g, estupor, pérdida parcial o total de la visión, dolor epigástrico en barra, hiperreflexia generalizada.
Preeclampsia recurrente; presencia de cualquiera de los tipos de enfermedad hipertensiva inducida por el embarazo, que aparece por
segunda ocasión o más en embarazos consecutivos o no. PATOGENIA: Factores de riesgo: Ausencia o deficiencia de control prenatal,
desnutrición, obesidad, intervalo intergenésico menor a dos años, muer menor de 18 y mayor de 35 años, primigesta o nuligesta,
preeclampsia/eclampsia en embarazos anteriores o antecdenetes familiares repetidos, hipertensión arterial crónica o cualquier otro
trastorno hipertensivo durante lagestacion, sobredistension uterina de cualquier origen (embarazo gemelar o multiple, polihidramnios),
infeccion en vías urinarias recurrentes, DM tipo 1 y 2, enfermedad renal previa o recurrente, enfermedad trofoblastica, enfermedades
autoinmunes, factores psicosociales. Es un estado de vasoconstricción generalizado secundario a una disfunción en el epitelio vascular,
en lugar de la vasodilatación propia del embarazo normal. Ello se asocia a isquemia placentaria desde mucho antes de la aparición del
cuadro clínico, en lo que parece ser uno de los orígenes de los factores tóxicos para el endotelio vascular. Dicha isquemia parece ser
debida a una deficiente placentación en la que no se produciría la habitual substitución de la capa muscular de las arterias espirales
uterinas por células trofoblásticas, que es lo que produce una vasodilatación estable que permite aumentar varias veces el caudal de
sangre asegurando así el correcto aporte sanguíneo a la unidad fetoplacentaria, su etiología última que sigue siendo desconocida,
aunque vamos identificando factores. Así podemos citar factores hereditarios por vía materna pero también paterna, factores
inmunológicos que explican que se produzca más en la primigesta y más si no ha estado expuesta antes al contacto directo con los
antígenos paternos, a factores nutricionales como la falta de ingesta de calcio en algunas poblaciones. Todo ello se manifiesta por un
desequilibrio entre prostaglandinas vasodilatadoras (prostaciclina) frente a las vasoconstricciones (tromboxano), quizá mediado por la
aparición de exceso de producción de citoquinas proinflamatorias (IL-2, TNF), y de producción de lipoperoxidos capaces ambos de e
inducir alteraciones endoteliales, esta disfunción endotelial general, que afecta a muchos órganos distintos como son el riñón, el
cerebro, el hígado pero en especial al sistema cardiovascular por lo que la hipertensión es su expresión más genuina, que con
frecuencia se acompaña de edema. El vasoespasmo y el edema facilitan la reducción del volumen plasmático que lleva a la
hemoconcentración característica y a la hipercoagulabilidad. Todo ello hace que exista una hipoperfusión multiorgánica que empeora
el cuadro a nivel general y fetoplacentario. DIAGNOSTICO: Se establece el diagnostico de preeclampsia leve cuando se presentan 2 o
mas de los siguientes signos: 1) Presión sistólica >140mmHg o elevación >30mmHg sobre la presión habitual. 2) Presión diastólica,
>90mmHg o elevación >15mmHg sobre la presión habitual. Proteinuria. Edema. La medición de la presión deberá realizarse con la
paciente sentada y requiere de dos tomas consecutivas con un intervalo de 6h o mas en este lapso de horas debe permanecer en
reposo. El incremento de proteinuria y de hipertensión en una paciente nefrópata o hipertensa previa se denomina preeclampsia
sobreañadida y el manejo clínico es parecido. Se establece el diagnostico de preeclampsia severa cuando se presentan 2 o más de los
siguientes signos: 1) presión sistólica >160mmHg y presión diastólica >110mmHg. 2) proteinuria >5g en orina de 24h o su equivalente
en tiras reactivas (más de 3+). 3) Oliguria de menos de 500ml/24h. 4) Trastornos cerebrales o visuales. 5) Edema generalizado. Por otra
parte también es grave cuando aparece Síndrome de HELLP (plaquetopenia, elevación de enzimas hepáticas y hemolisis), de
insuficiencia cardiaca (edema agudo de pulmón), o de insuficiencia renal (creatinina>1,2 mg/dL), o dolor epigástrico. La inminencia de
eclampsia se diagnostica con uno o más de los siguientes datos: 1) Presión arterial sistólica >185mmHg con presión arterial diastólica
>115mmHg. 2) Proteinuria >10g. 3) Estupor. 4) pérdida parcial o total de la visión. 5) Dolor epigástrico en barra. 6) Hiperrreflexia
generalizada. Por la gravedad de esta variedad clínica las pacientes deben ser manejadas como eclámpticas. Todas suelen acompañarse
de signos de afectación fetal por insuficiencia placentaria crónica en forma de signos de restricción del crecimiento intrauterino o
agudo con signos de Riesgo de Pérdida de Bienestar Fetal. Con el agravante de que la situación fetal suele empeorar al tratar la
hipertensión materna grave ya que al descender sus valores se disminuye la perfusión placentaria, y de que los fármacos administrados
a la madre dificultan la valoración del estado fetal a través del estudio de la frecuencia cardiaca fetal basal o test no estresante (NST en
inglés), por lo que las unidades que traten los casos graves tendrían que disponer de la posibilidad de ecografía Doppler de forma
continuada. TRATAMIENTO: Debe recordarse que existe una hemoconcentración a pesar de que puedan existir edemas, por lo que NO
deben administrarse diuréticos para tratarlos o para disminuir la TA, excepto en los casos de edema agudo de pulmón o de oliguria. Por
la misma razón tampoco se debe hacer una dieta hiposodica estricta. Los mejores resultados actuales se deben a la posibilidad de
terminar la gestación en los casos graves a partir de la semana 32 o antes en caso necesario, con buenos resultados perinatales gracias
a la posibilidad de madurar los fetos con corticoides (no contraindicados con buen control en la preeclampsia), y especialmente a las
mejoras en el manejo neonatal de los prematuros extremos. Preeclamsia leve. Terminar la gestación al llegar a término sin sobrepasar
la semana 40. No se ha demostrado que se mejore con reposo, ni con tratamiento hipotensor. Se debe controlar la aparición de signos
de gravedad para poder iniciar el tratamiento en el momento oportuno. Antihipertensivos: alfametil-dopa 250-500mgs VO cada 8 h, si
es preciso asociados a hidralazina 10-50mgs VO c/8hrs. Preeclampsia grave. En todos los casos se debe: 1) Controlar la TA con
hipotensores manteniéndola de ser posible por debajo de los valores indicados como de gravedad, pero por encima de 140/90 para no
disminuir la perfusión placentaria en exceso. Se usara, hidaralazina bolo inicial de 5 mg IV si diastólica >110mmHg, repetir en 20
minutos (5-10mg) de acuerdo a respuesta hasta un máximo de 30 mg (de no contar con hidralazina se recomienda nifedipino VO 10mg,
repetir cada 10-30min). Labetalol, 20 mg por vía intravenosa, seguidos de 40 a 80 mg cada 10 minutos, hasta una dosis máxima de 220
mg. Alfametil-dopa 500mgs VO cada 6 h, si es preciso asociados a hidralazina 50mgs VO c/6hrs. No usar betabloqueantes (atenolol) ni
IECAs, ni ARA-II. Indicado para evitar edema cerebral dexametasona (16mg IV dosis única). 2) Hacer prevención del riesgo de Eclampsia
y la posible la hiperreflexia, con sulfato de magnesio (SO4Mg, la dosis de ataque es de 6g. en 100 de dextrosa al 5% pasar en 15 min.

CURSO ENARM CMN SIGLO XXI TEL: 36246001 Pharmed Solutions Institute PÁGINA 327
MANUAL DE TRABAJO DEL CURSO ENARM CMN SIGLO XXI
Continuar con dosis de mantenimiento a 1-2 g por hora administrados en infusión IV continua (preparar sol. Glucosada de 900cc al 5%
mas 10 ampolletas de 1g. Pasar 100-200ml de solución por hora igual a 24-48 gotas por minuto). En caso de intoxicación utilizar 1 g de
Gluconato de calcio IV diluido en 100cc de solución fisiológica y pasar en 15 min. 3) Vigilar la aparición de otros signos de gravedad
Síndrome de HELLP, CID clínica, insuficiencia cardiaca, renal, etc. 4) controlar el bienestar fetal con NST periódico, perfil biofísico y
Doppler para comprobar el grado de afectación fetal y si existen signos de redistribución vascular, para poder indicar la extracción fetal
antes de la afectación de los vasos venosos, momento que por lo que sabemos hasta ahora, coincide con el inicio de la acidosis fetal. 5)
Finalizar la gestación: 1) A termino: en cuanto la situación materna se estabiliza. 2) Pretérmino: >32 semanas, en cuanto se compruebe
la madurez pulmonar fetal o antes si hay indicación materna o fetal. 3) Pretérmino <32 semanas: siempre que exista indicación
materna por aparición de complicaciones graves o fetales con signos de redistribución ante la hipoxia. 28-32 semanas madurara con
corticoides y valoración del riesgo de prematuridad correspondiente a cada semana y peso fetal frente a los signos de redistribución
ante la hipoxia con afectación de vasos venosos o indicación materna. A partir de la semana 30 probablemente nos inclinaremos por la
extracción fetal, con feto madurado.

CASO CLINICO
Se trata de femenino de 27 años de edad la cual cuenta con 22 semanas de embarazo, acude a su tercera visita de control prenatal, al
revisar su expediente usted observa que ha presentado por tercera ocasión de forma independiente una tensión arterial de 140/90,
usted realiza una prueba de rutina para apoyar su sospecha diagnostica de pre-eclampsia, se observa edema de miembros inferiores,
proteinuria, cefalea constante.

PREGUNTA
Cual de las medidas no es adecuada.

RESPUESTA
a.- Manejo ambulatorio.
b.- Hidralacina 50 mg cada 12 hrs.
c.- Dieta hiposodia y restricción de liquidos.
d.- Captopril 25 mg cada 24 hrs.

ECLAMPSIA. CIENCIAS BASICAS: Se denomina así a las convulsiones similares a una crisis epiléptica aparecida en el contexto de una
preeclamsia e hiperreflexia, después de la semana 20 de gestación, parto o puerperio en ausencia de otras causas de convulsiones.
Suponen un riesgo vital y pueden ocurrir, antes, durante o después del parto. TRATAMIENTO: 1) Soporte vital (vía aérea, constantes,
vía intravenosa...). 2) Tratamiento de la hipertensión: hidaralazina bolo inicial de 5 mg IV si diastólica >110mmHg, repetir en 20 minutos
(5-10mg) de acuerdo a respuesta hasta un máximo de 30 mg (de no contar con hidralazina se recomienda nifedipino VO 10mg, repetir
cada 10-30min). Labetalol, 20 mg por vía intravenosa, seguidos de 40 a 80 mg cada 10 minutos, hasta una dosis máxima de 220 mg. 3)
Tratamiento anticonvulsivante con SO4Mg, la dosis de ataque es de 6g. en 100 de dextrosa al 5% pasar en 15 min. Continuar con dosis
de mantenimiento a 1-2 g por hora administrados en infusión IV continua (preparar sol. Glucosada de 900cc al 5% mas 10 ampolletas
de 1g. Pasar 100-200ml de solución por hora igual a 24-48 gotas por minuto), ajustando los niveles a 4.8-9.6 mg/dl de magnesemia,
siendo necesario mantener los controles clínicos continuados (reflejo patelar, respiraciones /minuto, diuresis) para evitar que una
sobredosificación pueda producir un paro cardiorrespiratorio. Esta medicación se ha comprobado en estudios prospectivos y
randomizados más eficaz que cualquier otra, pero los límites terapéuticos y tóxicos están muy cercanos. También está indicado el uso
de diazepam, como droga de segunda elección, que actuaría disminuyendo el consumo de oxigeno por el tejido cerebral, pero que
posee efectos nocivos sobre el feto (depresión respiratoria, hipotonía). Las dosis recomendada son: bolo de 10 mg por vía IV y luego
continuar con 50 mg en 500 cc de solución dextrosada al 5%. 4) Acabar la gestación en cuanto la situación clínica materna este
estabilizada y lo permita. COMPLICACONES: De preeclampsia/eclampsia; Sindrome de HELLP. Desprendimiento prematuro de
placenta. Insuficiencia renal aguda. Edema pulmonar agudo. Edema cerebral. Coagulacion intravascular diseminada. Ruptura hepática
(Raro, alta letalidad, se manifiesta por dolor epigastrico en barra o en el cuadrante superior derecho, irradiado a hombro, región
iterescapulovertebral derecha, en hemicinturon, pudiendogeneralizarse a todo el abdomen). Hemorragia cerebral.

CASO CLINICO
Se trata de paciente de 19 años de edad la cual acude por primera vez a consulta por amenorrea secundaria de 30 semanas por FUM,
acude hasta ahora ya que desconocían su embarazo en casa, la paciente refiere mareo, dolor abdominal en barra, opresión de torax,
cefalea e irritabilidad, a la exploración física se observa escleras hiperemicas, leve rubicundez facial, rot´s incrementados y edema de
miembros inferiores, proteinuria, se mide fondo uterino con 28 cm, con disminución de actividad fetal, los signos vitales de la madre
fueron FR 21, FC 96, TA 130/100 en 3 ocaciones.

PREGUNTA
Cuál es la conducta inmediata a seguir con la paciente.

RESPUESTA
a.- Identificar la viabilidad fetal.
b.- Busqueda de síndrome de HELLP.
c.- Sulfato de magnesio 4 g IV.
d.- Prepara a la paciente para Cesarea.

CURSO ENARM CMN SIGLO XXI TEL: 36246001 Pharmed Solutions Institute PÁGINA 328
MANUAL DE TRABAJO DEL CURSO ENARM CMN SIGLO XXI
CASO CLINICO
Se trata de femenino de 27 años con 32 semanas de gestación que llega al servicio de urgencias por presentar cefalea intensa, acufenos
y fosfenos, dolor abdominal en barra, además dolor en región lumbosacra y sensación de pesantes pélvica, a la exploración física se
observa fondo uterino de 28 cm, obesidad grado I, ROT´s incrementado, edema de miembros inferiores +++, al tacto observa dilatación
de 2 cm, con presencia de moco con sangre, se observan proteínas en orina ++, nivel de conciencia disminida, leve desorientación
temporo-espacial, el familiar refiere que la encontró en el suelo con somnolencia.

PREGUNTA
Cuál es la conducta a seguir de forma inmediata.

RESPUESTA
a.- Indica Sulfato de Magnesio.
b.- Indica Hidralacina.
c.- Preparar para Cesarea.
d.- Preparar para Parto.

CASO CLINICO
Se trata de femenino de 18 años de edad con 32 semanas de gestación por fecha de última menstruación, sin atención prenatal, es
llevada a urgencias por presentar en casa perdida del estado de alerta y con movimientos tipo crisis convulsivas, a la exploración se
observa somnolienta con respuesta a estímulos dolorosos, ha sido estabilizada en sus constantes vitales, los resultados de laboratorios
son los siguientes, plaquetas de 45,000, proteinuria +++, DHL de 590, AST 239, ALT 432.

PREGUNTA
Tomando en cuenta la gravedad del caso, cual es la mejor conducta a seguir.

RESPUESTA
a.- Realizar Cesárea.
b.- Sulfato de magnesio.
c.- Estabilizar plaquetas.
d.- Maduración Pulmonar.

CURSO ENARM CMN SIGLO XXI TEL: 36246001 Pharmed Solutions Institute PÁGINA 329
MANUAL DE TRABAJO DEL CURSO ENARM CMN SIGLO XXI
DIABETES MELLITUS, INTOLERANCIA A LA GLUCOSA, DIABETES GESTACIONAL. GENERALIDADES: Cambios fisiológicos en el embarazo
normal: El embarazo normal se considera un estado diabetogénico o de resistencia progresiva al efecto de la insulina, durante el primer
trimestre y las etapas iniciales del segundo se eleva la sensibilidad a la insulina, lo que se ha atribuido a las mayores concentraciones de
estrógenos circulantes. Este fenómeno incrementa el depósito de energía, sobre todo en el tejido adiposo, con expansión del mismo;
pero a partir de las 24 a 28 semanas de gestación aumenta paulatinamente la resistencia a la insulina, que puede alcanzar los niveles
que se observan en pacientes diabéticos tipo 2. Más evidente al final del segundo trimestre del mismo; el organismo de la madre se
prepara almacenando nutrientes y en esta etapa la sensibilidad a la insulina es mayor; sin embargo, en la segunda mitad de la
gestación hay una resistencia progresiva al efecto de la insulina, lo que causa una liberación de los nutrientes (glucosa, los aminoácidos,
los ácidos grasos los triglicéridos y los oligoelemntos) para que el feto tenga un desarrollo adecuado. Las células β del páncreas elevan
la secreción de insulina en un intento de compensar la resistencia a la insulina del embarazo, lo que origina pequeños cambios en la
concentración de insulina en el curso de la gestación, comparados con los grandes cambios en la sensibilidad de la misma. El músculo
esquelético es el sitio principal para utilizar la glucosa corporal, y junto con el tejido adiposo, empiezan a ser resistentes al efecto de la
insulina, lo que es más evidente durante la segunda mitad del embarazo. Un embarazo normal se caracteriza por aproximadamente un
50% de disminución en la disponibilidad de glucosa mediada por insulina. Barbour señala un incremento en la secreción de insulina
hasta de 200% para tratar de mantener euglucémica a la madre. Una gran cantidad de sustancias producidas por la placenta y por los
adipocitos, (lactógeno placentario, hormona placentaria de crecimiento, prolactina, hormona liberadora de corticotropina-cortisol,
insulinasa, FNT y adipocitocinas [leptina, resistina, visfatina, adiponectina]), son las que reprograman la fisiología materna y causan
este estado de resistencia a la insulina para dirigir los nutrientes hacia el feto en desarrollo, sobre todo en la segunda mitad del
embarazo. El lactogeno placentario se eleva unas 30 veces durante la gestación. En los obesos hay una correlación positiva entre el
FNTa y el IMC e hiperinsulinemia. RESISTENCIA A LA INSULINA: En esta resistencia a la insulina participan numerosas sustancias
producidas por la placenta y por los adipocitos, entre las cuales destacan la hormona de crecimiento placentaria y el FNTa, que por
diversos mecanismos disminuyen el efecto de la insulina a nivel intracelular. La primera aumenta la cantidad de la subunidad p85a de la
PI-3K (fosfatidil-inositol 3 cinasa), lo que al final disminuye un factor de trascripción AKT que altera el desplazamiento de los
transportadores de glucosa (GLUT) hacia la membrana celular y disminuye la entrada de glucosa a la célula; el segundo altera la
fosforilación de la subunidad beta del receptor de insulina y del IRS-1, ya que fosforila residuos de serina y treonina en lugar de los de
tirosina. Este evento conduce a la mayor degradación de ambos y modifica la acción de la insulina. Otro mecanismo implicado es que en
el musculo esquelético el GLUT-4 esta disminuido en el tejido adiposo en mujeres embarazadas, y es aún menor en la diabetes mellitus
gestacional; además, está alterada la traslocación de estos trasportadores. Los cambios moleculares en el adipocito durante el
embarazo muestran reducción en la trascripción del PPARg, receptor nuclear que regula la trascripción de varios genes centrales en el
metabolismo del adipocito (adiponectina, la lipoprotein lipasa, la proteína P-2 fijadora de ácidos grasos intracelulares y la proteína no
acoplada mitocondrial). Barbour halló hasta 40 a 50% de reducción en el ARNm del PPARg (un factor que suprime considerablemente a
este receptor nuclear es el FNTa) en embarazadas obesas y DM gestacional; esto altera la concentración de ácidos grasos en la sangre, y
las lipoproteínas ricas en triglicéridos (VLDL) con mayor resistencia para el efecto de la insulina ya que la adiponectina es un
sensibilizador de esta hormona y estos ácidos grasos entran a la célula como fuente energética. SALUD PUBLICA: La prevalencia de
diabetes gestacional a nivel mundial se ha estimado en 7%. En México se ha mencionado una frecuencia que varía entre 4 y 19% de la
población obstétrica. Respecto a la edad de la madre, se ha señalado que la incidencia es de 0.4 a 0.5% en menores de 25 años y de 4.3
a 5.5% en mayores de esa edad. Lo cierto es que la frecuencia de este trastorno se ha duplicado en la última década, en forma paralela
a la llamada pandemia metabólica que afecta a las sociedades modernas. Más de 90% de los casos de diabetes que complican a un
embarazo son casos de diabetes gestacional. La creciente prevalencia de diabetes, que se diagnostica en mujeres a edades tempranas,
favorece la presencia de diabetes y embarazo. Las mujeres que desarrollan diabetes gestacional tienen alto riesgo de desarrollar
diabetes tipo II, en los 10 años que siguen al embarazo. PATOGENIA: Factores de riesgo: 1) Edad mayor de 25 o 30 años. 2) IMC > 25
kg/m2 SC. 3) Antecedentes de hijos macrosómicos. 4) Diabetes mellitus en familiares de primer grado 5) Antecedentes de intolerancia a
la glucosa. 6) Glucosuria. 7) Ganancia de más de 20 kg de peso en la actual gestación. 8) Antecedentes de problemas obstétricos serios.
9) Peso bajo del feto al nacimiento (<10º percentil) para la edad gestacional. 10) Peso alto del feto al nacer (> del 90o percentil). 11)
Origen étnico de riesgo alto de obesidad o diabetes mellitus. Los más importantes son: mayor edad en la madre, familiares de primer
grado con diabetes y mayor índice de masa corporal pregestacional. En la segunda mitad del embarazo se requiere un estado fisiológico
de resistencia a la insulina para dirigir los nutrientes almacenados en la madre hacia la unidad fetoplacentaria y dar un crecimiento
adecuado al feto; sin embargo, cuando las mujeres desarrollan diabetes mellitus gestacional, la resistencia a la insulina es más
acentuada, lo cual modifica el medio intrauterino y causa crecimiento acelerado del feto, con riesgo elevado de macrosomía. Los datos
disponibles en la bibliografía muestran un aumento lineal en el índice de complicaciones en el feto conforme se eleva la cifra de la
glucosa en la sangre materna: índices de mortalidad del feto de 4% cuando la glucosa es menor de 100 mg y hasta de 24% cuando la
cifra excede 150 mg. DIABETES PREGESTACIONAL: o preexistente se refiere a pacientes conocidas con diabetes mellitus tipo I o II que
se embarazan. Los hijos de mujeres con diabetes pregestacional tienen alto riesgo de sufrir malformaciones estructurales congénitas,
aborto espontaneo y restricción del crecimiento intrauterino, lo que se debe a que la hiperglucemia y la hiperinsulinemia en el feto
pueden ser teratogénicas, ya que existe una fuerte asociación entre un pobre control glucémico en el periodo periconcepcional y el
riesgo de tales malformaciones. La incidencia de riesgo de estas malformaciones es de 6 a 8% y la mayoría ocurre en el sistema nervioso
central, el corazón, el sistema urogenital y el tubo gastrointestinal. DIABETES GESTACIONAL: Es un padecimiento caracterizado por la
intolerancia a los carbohidratos con diversos grados de severidad, que se reconoce por primera vez durante el embarazo y que puede o
no resolverse después de este. Existe mayor riesgo de macrosomía y polihidramnios. Se incrementa el riesgo de prematurez,
preeclamsia/eclampsia, trauma obstétrico, cesárea y mortalidad perinatal. DIAGNOSTICO: Escrutinio; prueba de escrutinio con 50 g de
glucosa oral entre las semanas 24 a 28 de la gestación. Una cifra de 140 mg con diabetes mellitus gestacional o bien 130 mg a la hora,
que identificaría al 90%. Sin embargo, a pesar de tener la mayor sensibilidad (79%) y especificidad (87%) de todas las pruebas de
escrutinio disponibles, ésta se reserva para pacientes con alto riesgo para padecer la enfermedad, más que para la población general.
Existen 4 formas de realizar el diagnostico de Diabetes Gestacional: 1) Glucemia en ayuno >126mg/dl en dos ocasiones. 2) Glucemia
casual >200mg/dl. 3) Prueba de tamiz con 50g con resultado >180mg/dl (170mg/dl en mujeres mayores de 30 años). 4) Curva de
tolerancia a la glucosa con 100g o 75 g. Se realiza el diagnostico al tener alterados 2 o más de los siguientes valores: ver cuadro anexo.

CURSO ENARM CMN SIGLO XXI TEL: 36246001 Pharmed Solutions Institute PÁGINA 330
MANUAL DE TRABAJO DEL CURSO ENARM CMN SIGLO XXI
En caso de tener alterado un solo valor se diagnostica intolerancia a los carbohidratos. COMPLICACIONES MATERNO-FETALES: En el
feto; Macrosomía, muerte intrauterina, asfixia neonatal, distocia de hombro y lesión nerviosa,
hipoglucemia, hipocalcemia, ictericia, SIRPA, Taquipnea transitoria, cardiomiopatía (hipertrofia
septal), eritrocitosis, trombosis, sobre todo de la vena renal. Cuando son adultos: obesidad, diabetes
mellitus, alteraciones neuropsicológicas. En la madre; Preeclampsia, eclampsia, polihidramnios,
desgarros perineales, mayor número de cesáreas, riesgo de padecer diabetes mellitus II, parto
prematuro. TRATAMIENTO: La meta terapéutica de glucosa sanguínea durante el embarazo es:
Glucemia central en ayuno entre 60-90mg/dl y menor de 140mg/dl una hora pospandrial o menor a 120mg/dl a las 2 h pospandrial. El
objetivo fundamental del tratamiento es mantener la normoglucemia con una dieta adecuada, reduciendo el consumo de
carbohidratos a 40-45% de la ingestión calórica total. Las pacientes con IMC superior a 30 kg/m2 de SC deben disminuir su ingestión
calórica en 30 a 33% o alrededor de 25 kcal/kg de peso corporal. El peligro de la restricción calórica muy estricta en la embarazada es la
producción de cetosis, que puede alterar el desarrollo psicomotor del feto. La actividad física adaptada a la fisiología materna de
acuerdo con la edad gestacional. Si con los dos recursos terapéuticos señalados no se obtiene un control adecuado, será necesaria la
administración de insulina en forma juiciosa para mantener cifras de glucosa dentro de límites normales para la mujer embarazada. Se
debe evitar un tratamiento intenso que cause hipoglucemia, lo que ocurre hasta en 60% de las pacientes, en muchos casos
asintomática para la madre, pero con diversas repercusiones para el feto. En pacientes con insulina la dieta fraccionada con una
colación nocturna, disminuye la probabilidad de hipoglucemias. Los análogos de insulina de acción rápida lispro y aspart son seguros en
el embarazo y tienen algunos beneficios comparados con la insulina rápida, como, menos episodios de hipoglucemia, un mejor control
de la glucemia postpandrial y mayor satisfacción del paciente. La insulina NPH es también segura y se utiliza en combinación con las
anteriores. El esquema de tratamiento con insulina puede consistir en: Se indica el fraccionamiento de la dosis de insulina en dos
aplicaciones cuando: 1) Continua glucemia en ayuno elevada a pesar de incremento de la dosis de insulina matutina. 2) Se requiere más
de 20 U de insulina para lograr control metabólico. 3) Se adiciona insulina rápida cuando no se alcanza la meta terapéutica de la
glucemia pospandrial. Esquema: Combinación de insulina rápida e intermedia administrada 30min antes del desayuno y de la cena. Los
requerimientos varían de 0.3-1.5 UI por kg de peso real y de acuerdo a descontrol glucémico, así como a la presencia de sobrepeso,
edad gestacional o enfermedades que modifiquen la glucemia como la hiperémesis gravídica o infecciones. La dosis total calculada se
dividirá en 2/3 pre-desayuno y 1/3 pre-cena (30 min antes de los alimentos). La razón será NPH/rápida 2/1 pre-desayuno y 1/1 pre-
cena. Se recomienda iniciar con la mínima dosis de insulina e incrementarla gradualmente de acuerdo al automonitoreo con glucemias
capilares pre y postpandriales, por lo menos 3 veces al día. Los análogos de insulina deben ser utilizados hasta que se disponga de
mayores datos o exista respuesta a las interrogantes que aún existen acerca de su uso, como: teratogenicidad, incremento en el daño a
la retina o incluso a la formación de anticuerpos y reactividad inmunológica de la insulina. Sólo dos de los fármacos hipoglucemiantes
orales han sido utilizadas: una es un secretagogo de insulina (glibenclamida) y otro un sensibilizador de insulina (metformina). Aunque
diversos grupos de expertos restringen su uso, los datos disponibles muestran que en algunas se logra un control adecuado, además
que se encuentran dentro de la clasificación C o D, lo que significa que nos son seguros. La elevación posprandial de la glucosa causa
alteraciones en el feto. Un fármaco que puede utilizarse es la acarbosa, que se absorbe en muy poca cantidad y, según algunos
trabajos, ha logrado mejor control de la glucemia posterior a la ingestión de alimentos. El control adecuado de la glucosa sanguínea es
el objetivo más importante del tratamiento de la diabetes mellitus gestacional y, en teoría, este control óptimo disminuiría la
morbilidad y mortalidad maternofetal. PREVENCION: La Federación Internacional de Diabetes y la Asociación Americana de diabetes
proponen los siguientes puntos para el consejo preconcepcional: 1) En toda mujer en edad fértil en etapa reproductiva con diabetes,
identificar anualmente por interrogatorio directo el deseo de embarazo, en caso de no desearlo proporcionar consejo sobre método
anticonceptivo. 2) En la eta preconcepcional y de embriogénesis, lograr un control glucémico estricto con una cifra ideal de Hb
glicosilada menor a 6.1%, evitando la presencia de hipoglucemia, para disminuir la probabilidad de malformaciones. 3) Con base, en
que a mayor daño vascular mayor riesgo de complicaciones materno-fetales, definir el riesgo establecido si existe daño por micro o
macroangiopatia utilizando la clasificación de Priscilla White. 4) En caso de diabetes tipo I, determinar la función tiroidea asociada, ya
que hasta un 17% pueden cursar con esta co-morbilidad. 5) En el momento en que se establece el diagnostico de embarazo, suspender
hipoglucemiantes orales o insulina glargina si la paciente los estaba utilizando. 6) Suspender los IECA y ARA II, si la paciente los estaba
utilizando y usar alfametildopa, nifedipina, labetalol o hidralazina. 7) Mantener las cifras de tensión arterial por debajo de
130/80mmHg. 8) Suspender estatinas y fibratos. 9) Suspender el consumo de alcohol y tabaco. 10) Iniciar ácido fólico tres meses antes
del embarazo. No es recomendable el embarazo si la mujer diabética presenta cualquiera de las siguientes situaciones: HbA1c >10%.
Cardiopatía isquémica. Nefropatía avanzada (depuración de creatinina <30ml/min y Cr sérica >1.4mg/dl). Retinopatía proliferativa.

CASO CLINICO
Paciente femenino de 34 años de edad la cual acude a consulta debido a que presenta amenorrea de 8 semanas de gestacion, agrega
que ha presentado leve fatiga, anorexia con nausea matinal, usted envía laboratorios de rutina presentando glucosa en sangre de 200
mg/dl y hemoglobina glucosilada de 7.5 refiere la paciente que fue diagnosticada con diabetes hace 5 años por lo que recibe
Tolbutamida, además recibe captopril por hipertensión arterial diagnosticada hace 3 años. Al interrogatorio la paciente refiere
antecedentes de madre diabética e hipertensa, sus antecedentes GO refiere menarca a 13 años, gesta 3, para 2, abortos 0, además
refiere que su embarazo anterior hace 5 años presento un incremento de 14 kilogramos con producto que preso 3,750 grs. Por vía
cesárea a las 36 semanas de gestación.

PREGUNTA
Cuál es la conducta menos adecuada a seguir.

RESPUESTA
a.- Suspender hipoglucemiente.
b.- Suspender IECA.
c.- Indicar interrupción de embarazo.

CURSO ENARM CMN SIGLO XXI TEL: 36246001 Pharmed Solutions Institute PÁGINA 331
MANUAL DE TRABAJO DEL CURSO ENARM CMN SIGLO XXI
d.- Indicar alfametildopa.

CASO CLINICO
Paciente femenino de 21 años de edad, la cual presenta 24 semanas de gestación, acude a control prenatal, refiere ardor al orinar,
deseos continuos de orinar, con urgencia, usted identifico un incremento de peso hasta el momento a partir de su embarazo de 10
kilos. Sus antecedentes presentan, obesidad previa al embarazo, padre diabético actualmente con insuficiencia renal en tratamiento
con hemodiálisis, madre hipertensa bajo tratamiento.

PREGUNTA
Considerando el cuadro clínico cuales son las complicaciones que el producto puede presentar.

RESPUESTA
a.- Producto Macrosomico.
b.- Diestres respiratorio.
c.- Hipoglucemia.
d.- Parto pretermino.

PARTO PRETERMINO. CIENCIAS BASICAS: El diagnóstico de parto pretérmino se obtiene en pacientes con ≤37 semanas (o un peso igual
a mayor de 500g, que respira o manifiesta signos de vida) y que presentan contracciones dolorosas y regulares que ocurren al menos
cada 10 minutos. Esto debe estar asociado con dilatación cervical y/o descenso. El diagnóstico se realiza al demostrar una dilatación
progresiva del cérvix debido a la presencia de contracciones entre las semanas 20-37, lo que incluye hasta la semana 37. La definición
de puede dividir a saber: Pretérmino: 33-37 semanas. Pretérmino moderado: 28-32 semanas. Pretérmino extremo: 20-27 semanas.
SALUD PUBLICA: El parto pretérmino es la mayor causa de mortalidad perinatal en el mundo desarrollado y ocurre en
aproximadamente un 7% de todos los nacimientos. Los niños pretérmino presentan causas severas de morbilidad como dificultad
respiratoria del recién nacido, hemorragias intraventriculares, displasia broncopulmonar y enterocolitis necrotizante. También se
pueden presentar complicaciones a largo plazo como parálisis cerebral, perdida visual y auditiva. Cerca de un 30% de los parto
pretérmino son idiopáticos y espontáneos. PATOGENIA: La incidencia del parto pretérmino es generalmente más alta en los grupos
socioeconómicos bajos en comparación con los grupos de mayor ingreso económico. De manera que se han establecido relaciones
entre la incidencia de amenaza de parto, parto pretérmino y algunas condiciones asociadas con los problemas socioeconómicos, a
saber: Bajo cumplimiento de la consulta prenatal, edades maternas extremas (adolescentes y madre añosa), fumado, condiciones
laborales extremas, pobre educación materna, estrés en la vida diaria, ansiedad relacionada al embarazo, violencia doméstica. Ante la
presencia de un parto pretermino se debe buscar de forma intencionada la presencia de infecciones a través de urocultivo y cultivo de
secreción vaginal con el objetivo de especificar el uso de antibiótico. Es necesario corroborar por USG la edad gestacional. Para
establecer el pronóstico de sobrevida del neonato y sus posibles complicaciones. El factor de riesgo más importante para predecir parto
pretermino es el antecedente de parto pretermino. Las pacientes con antecedente de parto pretermino, deben recibir consejo
preconcepcional o atención medica temprana en el embarazo. DIAGNOSTICO: No hay datos clínicos específicos y la exploración de
cérvix es subjetiva e inexacta. Identifique los factores de riesgo de parto pretérmino en la historia clínica de las mujeres en control.
Algunos síntomas tempranos sugestivos de amenaza de parto pretérmino son: Dolor abdominal bajo y/o calambres, dolor lumbar,
presión pélvica, flujo vaginal aumentado, manchado o sangrado. Síntomas definitivos son: actividad uterina regular acompañada de
borramiento y dilatación del cérvix. Criterios diagnósticos: A) actividad uterina regular entre las 20 y 37 semanas de gestación más
contracciones uterinas clínicamente documentadas (1/10 minutos, 4/20 minutos o 6/60 minutos). B) Dilatación cervical igual o mayor a
2cm. C) Borramiento cervical igual o mayor a 80%. Laboratorio y gabinete: la evaluación ultrasonografica de la longitud cervical y
prueba de fibronectina fetal tienen elevado valor predictivo negativo; por lo tanto, el uso único o de ambas, permite determinar qué
tipo de pacientes no requieren tocolisis. En pacientes con riesgo de parto pretermino la medición de la longitud cervical se realizara
entre la semana 20.1-34 de gestación y fibronectina fetal entre las semanas de 24-34 de gestación. La amniocentesis puede ser usada
en mujeres con amenaza de parto pretermino para valorar la madurez pulmonar fetal e infección intraamniotica. FIBRONECTINA
FETAL: La FNf es una glicoproteína producida por la membrana coriónica y está localizada en la decidua basal adyacente al espacio
intervelloso. Su objetivo principal parece el de una molécula de adhesión (pegamiento), la cual favorece la unión entre las membranas
corionicas con la decidua materna contigua. Puede ser encontrada, normalmente, en SCV en las primeras 22 SDG. Virtualmente, nunca
es encontrada en la misma secreción entre las 24 y 34 SDG, a menos que el cérvix haya desarrollado borramiento y dilatación
prematura, generalmente en asociación con contracciones uterinas sintomaticas. Existe una fuerte asociación entre la expresión de FNf
en SCV y PP.(1,2,7) Numerosos ensayos han mostrado una asociación entre la presencia de FNf y nacimientos pretérmino, así como una
reducción en el riesgo de nacimientos pretérmino cuando el resultado de esta prueba es negativo Los beneficios con el uso de la
prueba de FNf(1) son: Disminución de ingresos hospitalarios y de la duración de los días de hospitalización. Identificación más
apropiada de pacientes que necesitan esteroides y tratamiento con agentes tocolíticos. Disminución del uso de agentes tocolíticos en
pacientes que no están en riesgo de nacimiento pretérmino. Reducción del estrés y ansiedad para la mujer embarazada y su familia
debido a la tranquilidad de no realizar una transportación u hospitalización innecesaria. Las indicaciones para FNf son: embarazos de 24
a 34 SDG, síntomas de parto pretérmino (contracciones uterinas regulares >6 por hora y/o presión pélvica) membranas amnióticas
intactas, cérvix <3 cm de dilatación y bienestar fetal documentado. TRATAMIENTO: Los antibióticos no parecen prolongar la gestación y
solo debe usarse para la profilaxis del estreptococo del grupo B en pacientes en las cuales el parto es inminente. No está definido un
tocolitico de primera elección para el manejo del parto pretermino. Las drogas tocoliticas pueden prolongar el embarazo de 2-7 días
permitiendo la administración de esteroides para mejorar la madurez fetal y considerar la referencia de la madre a un tercer nivel. Usar
de terbutalina 0.25mg subcutánea cada 20min a 3 h controlando intervalos regulares de la presión arterial y la frecuencia cardiaca en la
madre, en conjunto con un control estricto de líquidos. Se recomienda el uso de indometacina 100mg rectal de dosisi inicial, con 25-
50mg VO o VR cada 4-6 horas por un máximo de 48h. Indicado si la paciente presenta enfermedad cardiovascular, hipertiroidismo, DM
o hipertensión. Solo utilizar en embarazos con menos de 32 SDG. La administración de 2 dosis de 12mg de betametasona o

CURSO ENARM CMN SIGLO XXI TEL: 36246001 Pharmed Solutions Institute PÁGINA 332
MANUAL DE TRABAJO DEL CURSO ENARM CMN SIGLO XXI
dexametasona IM, con 12-24h de intervalo, se recomienda como esquema de inducción de madurez fetal. La edad gestacional
recomendada para la apliacion de corticoide para inducción de madurex pulmonar fetal en pacientes con riesgo de parto pretermino es
de 24-34 SDG. Los tocoliticos no deben utilizarse como terapia de mantenimiento de repetición. Tratamiento no farmacológico: El
reposo en cama y la hidratación no parecen mejorar la incidencia de parto pretermino, por lo cual no debe ser una recomendación
rutinaria. El internamiento prenatal para guardar reposo no ha mostrado ser efectivo en disminuir el parto pretermino y la morbilidad
perinatal, por lo cual se recomienda individualizar cada caso. El reposo absoluto puede presentar efectos adversos en las mujeres y
debe evitarse en la medida de lo posble. Sera motivo de referencia a tercer nivel, embarazos con fetos potencialmente viables (27-
32SDG), en caso que la unidad de referencia no cuente con unidad de terapia intensiva neonatal. Contraindicaciones de la inhibición del
parto pretérmino; La inhibición del parto pretérmino está contraindicada en las siguientes condiciones cuando el parto es inminente o
cuando otros factores obstétricos indican que el parto no debe ser retrasado, a saber: Retardo del crecimiento intrauterino, pre-
eclampsia fulminante, abrupto placentario severo, sufrimiento fetal, corioamnioitis severa ante una ruptura de membranas,
anormalidad mortal del feto, desarrollo de efectos adversos severos con el tratamiento. Evite inhibir el parto si está en el mejor interés
de la madre o el niño. Debe detenerse el tratamiento si el parto continúa a pesar de la terapia. PREVENCION: En general se dice que
cerca del 50% de los partos pretérmino son potencialmente prevenibles. Algunas estrategias utilizadas para la reducción de los factores
de riesgo incluyen estrategias basadas en la población, a saber: Preparación para el parto: estado nutricional, evitar el tabaco y drogas.
Ambiente de apoyo. Dar mayor poder a la mujer en su consulta prenatal, incluyendo apoyo continuado, adecuada respuesta a
preguntas e información para evitar factores de riesgo. Educación al paciente sobre el reconocimiento de los síntomas tempranos
(dolor abdominal bajo, dolor lumbar, presión pélvica, flujo vaginal aumentado, sangrado o manchado). Tratamiento de las infecciones
vaginales sintomáticas antes de las 32 semanas. Tamizaje por vaginosis bacteriana en mujeres con antecedente de parto pretérmino.
Cerclaje cervical en mujeres con historia de incompetencia cervical.

CASO CLINICO
Se trata de paciente femenino de 29 años de edad la cual acude a urgencias refiriendo contar con 34 semanas de gestación por FUM,
no cuenta con cuidados prenatales, agrega que este es su tercer embarazo, previamente dos partos, agregando que no presenta
complicaciones en sus embarazos o partos previos, a la exploración usted observa borramiento del 100% y dilatación de 8 cm, decide
dar continuidad al trabajo de parto, ya que considera que en caso de enviarla a unidad de mayor complejidad colocaría en riesgo al
binomio.

PREGUNTA
Considerando que la paciente cursa con un trabajo de parto pretermino, cual es la complicación más frecuente que el producto puede
presentar y por lo tanto tendrá que enviar a un segundo nivel posterior a asistir el parto.

RESPUESTA
a.- Hemorragia Periventricular.
b.- Enfermedad de Membrana Hialina.
c.- Enterocolitis Necrotizante.
d.- Sepsis Neonatal.

CASO CLINICO
Paciente de 34 años de edad, casada, con antecedentes de incompetencia cervical, colecistectomía laparoscópica debido a colecistitis
crónica litiásica de cinco meses de evolución y timectomizada a los 18 años de edad por miastenia gravis, sin tratamiento farmacológico
actual. Al momento de su estudio cursaba la vigésima cuarta semana del quinto embarazo. Los partos previos fueron vaginales,
menores de 25 semanas, de hijos nacidos vivos pero que fallecieron a las pocas horas de nacidos, el alumbramiento fue normal, con
una placenta discoide, a la revisión se obserrvaron dos zonas discoides, hemorrágicas además signos de envejecimiento placentario.

PREGUNTA
Cual de las siguientes medidas presentan mayor efecto para modificar el curso de esta entidad?

RESPUESTA
a.- Cerclaje temprano.
b.- Reposo absoluto.
c.- Antibiotico profiláctico.
d.- Progesterona micronizada.

PARTO NORMAL. CIENCIAS BASICAS: Se define como aquel de comienzo espontáneo, de bajo riesgo desde el comienzo del trabajo de
parto, hasta la finalización del nacimiento el niño nace en forma espontánea, en presentación cefálica, entre las 37-41 SDG. Luego tanto
la madre como el niño están en buenas condiciones. Presentación: parte del feto que se presenta al canal del parto y es capaz de
desencadenar el TDP. El 95 % de las presentaciones son cefálicas, 4 % son presentaciones podálicas o pelvianas, que pueden ser de
cara, frente y muy rara, de hombros. Situación: relación que existe entre el eje axial del feto y el eje del útero, por lo que existen dos
tipos de situación: longitudinal y transversa. Posición fetal de una presentación particular: relación existente entre el dorso del feto y el
lado derecho o izquierdo de la madre. En la variedad de posición en cada tipo de presentación va a tener un punto de referencia. En la
presentación cefálica el punto de referencia es el occipucio; en la presentación de cara es el mentón y en la pelviana o podálica es el
sacro. TRABAJO DE PARTO (TDP): Por definición, el comienzo es espontáneo, por lo tanto se inicia en la casa, y es la propia mujer y su
familia quienes controlan sus condiciones y evolución durante las primeras etapas. Causas que desencadenan el TDP; 1. Musculares. 2.
Hormonales. 3. Nerviosas. 4. Placentarias. 5. Fetales. Reflejo de Ferguson 1: La distensión progresiva del útero estimula los receptores
locales que por vía neuronal estimulan los núcleos supraopticos y paraventriculares que liberarían oxitocina. Reflejo de Ferguson 2: La

CURSO ENARM CMN SIGLO XXI TEL: 36246001 Pharmed Solutions Institute PÁGINA 333
MANUAL DE TRABAJO DEL CURSO ENARM CMN SIGLO XXI
mayor distención del segmento inferior y el borramiento y dilatación cervical llevaría nuevos estímulos reflejos hacia la neurohipofisis lo
que estimularía la secreción de oxitocina. FASES DE TDP: FASE 1 ó LATENTE: Ocurre al final del embarazo hasta el inicio de las
contracciones del TDP. FASE 2 ó ACTIVA: Se compone de 3 períodos: 1) Período de dilatación: comprende desde el comienzo de la labor
del parto hasta la dilatación completa (10 cm). Es la fase más larga del TDP y tiene una duración de 8-12 h en las nulíparas y de 6-8h en
las multíparas. Consta de 2 fases: A) Fase latente: comienza con las contracciones y se prolonga hasta los 2,5 cm de dilatación; es de
unas 8 horas de duración y no debe pasar de 20 horas en nulíparas y de 14 en multíparas. B) Fase activa: fase de aceleración inicial,
fase de máxima inclinación, fase de desaceleración. 2) Período de expulsión: comprendido desde la dilatación completa hasta el
nacimiento del bebé y varía desde pocos minutos hasta 2h, sobre todo en las nulíparas. 3) Período de alumbramiento: transcurre desde
el nacimiento hasta la expulsión de la placenta, y se espera que esto ocurra en un período de 30 a 40 min. Algunos plantean un cuarto
período que sería la hora después del alumbramiento en que existe gran peligro de que ocurra una hemorragia; pero la mayoría asume
que éste sería la prolongación del tercer período. Es difícil determinar en muchas ocasiones cuándo comenzó la labor de parto, pero
por regla general esto ocurre cuando las contracciones tienen una frecuencia cada 2 ó 3 min de aparición entre ellas y duran de 30 a
45seg, y como resultado existen modificaciones del cuello uterino. Los 3 factores que contribuyen a que las contracciones borren y
dilaten el cuello son: 1. La presión hidrostática de la bolsa de las aguas. 2. La tracción que sobre las fibras del cuello uterino ejercen las
del cuerpo. 3. La dilatación activa por la disposición espiral de las fibras musculares del útero. La bolsa de las aguas, impulsada por la
presión hidrostática, se introduce en el orificio cervical interno al comienzo de la dilatación, y al mismo tiempo expulsa el tapón mucoso
endocervical. El sangramiento que ocurre en un parto normal se encuentra entre los 400 a 500 mL. Se plantea que después del
alumbramiento el útero se contrae y se retrae para mantener la hemostasia por medio del estrechamiento de las fibras musculares que
comprimen los vasos venosos, a lo que se ha denominado ligaduras vivientes de Pinard. CONTRACCIONES UTERINAS: La medida de la
contracción uterina y su registro gráfico tiene mucha importancia en la clínica obstétrica moderna, porque constituyen la base de un
control más científico del seguimiento del TDP, que recibe el nombre de monitorización obstétrica. Se ha determinado que la
intensidad de las contracciones en el parto normal es de 30-50 mmHg y el tono muscular normal está comprendido entre 8 y 12 mm.
Antes de las 30 SDG: Tipo A, poca intensidad y limitado a pocas áreas del útero (Álvarez). Tipo B, Braxton Hicks mayor intensidad, baja
frecuencia y se propagan. Preparto en ultimas semanas: Aumento de las contracciones de Braxton Hicks, se caracteriza por la
maduracion progresiva del cuello uterino. Cuando el cuello presenta 2cm de dilatación o más, la onda contráctil normal comienza en las
zonas denominadas “marcapasos” ubicadas cerca de la implantación de las trompas y se caracteriza por el triple gradiente descendente
(TGD). 1. Sentido descendente 2. Duración: disminuye a medida que se aleja de los marcapasos 3. La intensidad disminuye a medida
que se aleja de los marcapasos. Clínica de las contracciones durante el TDP: Frecuencia: comienzo c/ 20 o 25 minutos final c/ 2-3
minutos. Duración: Comienzo 15 a 20 segundos final 40-45seg, intensidad: Aumento a medida TDP, hay dolor: por la dilatación y
contracciones. MECANISMO DEL PARTO: En la mayoría de los partos se realiza por parte del objeto del parto (el feto) una serie de
evoluciones que van a ir sorteando los diferentes diámetros y curvatura de la pelvis ósea y de las partes blandas del canal, y que tendrá
su mecanismo bien establecido en cada una de las presentaciones. Por ser la presentación de vértice la más frecuente, describiremos
estos mecanismos: 1) Acomodación Estrecho Superior: para facilitar su entrada en la excavación pélvica, la cabeza se reduce por flexión
y cabalgamiento de los huesos del cráneo y orienta sus diámetros mayores anteroposteriores hacia el diámetro mayor del estrecho
superior. Al comienzo del parto la cabeza colocada transversalmente se halla en actitud indiferente y presenta su diámetro
frontooccipital de 12 cm al estrecho superior de la pelvis. En virtud de las contracciones uterinas y de la resistencia que ofrece el
estrecho superior, la cabeza se flexiona y sustituye su diámetro anteroposterior por otro menor, el suboccipitofrontal de 10,5 cm. 2)
Encaje y descenso: La contracción uterina hace que la cabeza realice un movimiento de lateroflexión, en virtud del cual el parietal
posterior desciende a la excavación pelviana y rebasa el promontorio, mientras el parietal anterior está detenido al nivel del pubis
(asinclitismo posterior). Al descender la cabeza por debajo del promontorio y alojarse el parietal posterior en la concavidad del sacro,
permite el descenso a su vez del parietal anterior hacia la cavidad pelviana. Cuando ambos parietales han franqueado el estrecho
superior de la pelvis se puede decir que la cabeza se ha encajado. La cabeza también puede encajarse en asinclitismo anterior e incluso
sinclíticamente. 3) Rotación interna: Al llegar aquí se encuentra con una situación completamente distinta: los diámetros mayores del
estrecho inferior son los anteroposteriores, por lo cual tiene que realizar una rotación de 90º para acoplar así sus diámetros mayores
(anteroposteriores) a los del estrecho inferior que también son los anteroposteriores. 4) Desprendimiento: La cabeza tiene que vencer
la resistencia del perineo y de la extremidad inferior del sacro y del coccix. El sacro puede realizar movimientos muy limitados, no
mayores que 5 a 6 mm en su parte inferior (nutación y contranutación del sacro); sin embargo, el cóccix puede retropulsar 30-40mm la
cabeza va dilatando el anillo vulvar y aparecen, en secuencia, las eminencias frontales, la raíz de la nariz, la boca, y, por último, el
mentón, hasta completar su desprendimiento total, mientras la cabeza se está desprendiendo en sentido anteroposterior, los hombros,
con diámetro biacromial y situados en el estrecho superior, van descendiendo, en la excavación pelviana (encajamiento de los
hombros). 5) Rotación externa: Los hombros, que han llegado en transversa al estrecho inferior, tienen que realizar un movimiento de
rotación interna de 90º para llevar su diámetro mayor biacromial al diámetro mayor anteroposterior del estrecho inferior. Los
hombros, al realizar su rotación interna en el estrecho inferior, hacen que la cabeza, fuera de la vulva y sin nada que la obstaculice,
realice su rotación externa. Sexto tiempo: expulsión de los hombros y deslizamiento del resto del cuerpo. Una vez expulsados los
miembros inferiores del feto por la hendidura vulvar, termina el mecanismo del parto. PERÍODO DE EXPULSIÓN: Cuando la dilatación
se ha completado y la cabeza llega al plano perineal, la parturienta experimenta deseos de pujar en cada contracción y se abomba el
perineo, por lo que sabemos que la paciente está en período expulsivo. Se colocará a la paciente en posición ginecológica en una mesa
de partos. Auscultar frecuencia cardíaca fetal. Asepsia del perineo, la vulva y la raíz de los muslos. Colocarán los paños estériles y
medias ginecológicas. Asepsia del médico (manos). Se vestirá con bata estéril, gorro, tapabocas, botas, guantes estériles. La mano
izquierda se coloca sobre el saliente de la cabeza y con la mano derecha se cogen, entre el pulgar por un lado y los dedos índice y medio
por el otro, las eminencias frontales a través del perineo tenso. Con ambas manos se controla la salida de la cabeza para evitar que ésta
lo haga bruscamente y produzca un desgarro perineal. Aunque la protección del perineo es conveniente, no debe prolongarse
exageradamente. En estos casos se realizará su infiltración local con lidocaina a 1% u otro anéstesico similar y se procederá a realizar la
episiotomía mediolateral derecha o izquierda. La episiotomía siempre es más ventajosa que un desgarro perineal (no abusar de este
proceder). Una vez expulsada la cabeza fetal, se aspiran las flemas de las fosas nasales de la orofaringe mientras se va produciendo
espontáneamente el resto del mecanismo del parto. Si existiera alguna circular del cordón, se libera. Después que dejamos de sentir las

CURSO ENARM CMN SIGLO XXI TEL: 36246001 Pharmed Solutions Institute PÁGINA 334
MANUAL DE TRABAJO DEL CURSO ENARM CMN SIGLO XXI
pulsaciones en el cordón umbilical, se pinza éste con tres pinzas de Kocher y se secciona entre las dos más próximas. ATENCIÓN DEL
PERÍODO DE ALUMBRAMIENTO: Una vez terminado el período de expulsión, debe asistirse con especial atención al alumbramiento. Se
observarán los signos clínicos del desprendimiento placentario: 1. Elevación del útero por encima del ombligo y desviación hacia la
derecha; esto se puede observar a través de las cubiertas abdominales Signo de Küstner, el cual consiste en el ascenso del cordón
umbilical al colocar nuestra mano por encima de la sínfisis del pubis y rechazar el útero hacia arriba, lo que significa que la placenta aún
está adherida al útero. En caso de que la placenta se haya desprendido, el cordón no se moverá. 3. Descenso espontáneo de la pinza
colocada en el cordón al nivel de la vulva. El desprendimiento placentario por lo general ocurre espontáneamente en los 10 min
siguientes a la expulsión del recién nacido, pero se debe esperar hasta 30 min antes de actuar. Una vez desprendida la placenta. Una
vez expulsada la placenta se debe evitar el desgarro de las membranas, para lo cual se le imprimen movimientos de torsión a la
placenta (maniobra de Dublin), de manera que las membranas se hagan más resistentes a los desgarros y vayan saliendo lentamente,
pero íntegras. Después de extraída la placenta se procede a su revisión.

CASO CLINICO
Se trata de femenino de 29 años de edad, la cual acude a urgencias debido a que desde hace 10 horas inicia con contracciones
dolorosas, la paciente cuenta con antecedentes GO gesta 3, para 2, abortos 0, a la exploración física se observa producto cefálico,
occipito anterior derecho identificado por palpación abdominal y tacto vaginal, se observa a nivel de segundo plano, se encuentra 100
% de borramiento y dilatación de 7 cm, la actividad uterina se observa 3 a 4 contracciones cada 10 minutos con duración de 50 a 60
segundos, durante la exploración observa presencia de cordon umblical y liquido amniótico de características adecuadas, el producto
presenta frecuencia cardiaca de 150 con descensos variables durante la contracción de menos 20 latidos.

PREGUNTA
Considerando el cuadro clínico, cual es la conducta a seguir.

RESPUESTA
a.- Realizar Bloqueo y Realizar Cesarea.
b.- Realizar Bloqueo y Continuar Trabajo de Parto.
c.- Continuar Trabajo de Parto.
d.- Indicar Tocoliticos.

CASO CLINICO
Se trata de femenino de 19 años de edad la cual es originaria del Estado de México la cual acude a urgencias, refiere que desde hace 6
horas presenta dolor abdominal los cuales son esporádicos, aproximadamente cada 10 minutos, de intensidad moderada, agrega que
desde hace 4 horas arrojo moco con sangre por vía vaginal, durante la exploración se encuentra cuello cervical reblandecido, con un
centímetro de dilatación con producto libre, usted observa presencia de secreción verdosa con olor desagradable, la madre refiere que
ha presentado cuadro repetidos de infecciones cervicovaginales con presencia de VPH por colposcopia reportado a las 18 semanas de
gestación, sin tratamiento, actualmente cuenta con 40 semanas de gestación.

PREGUNTA
Considerando el cuadro actual, cual es la conducta a seguir:

RESPUESTA
a.- Esperar inicio de trabajo de parto.
b.- Inducir trabajo de parto.
c.- Prepara a la paciente para cesárea.
d.- Conducir el trabajo de parto.

CASO CLINICO
Se trata de paciente femenino de 32 años de edad la cual acude por tercera vez a consulta debido a que se encuentra embarazada y no
se ha sentido bien, cuenta con antecedente de G; 3 Para; 2 Abortos 0; al interrogatorio se identifica 42 semanas de gestación por FUM,
sin embargo no hay inicio espontaneo de trabajo de parto, por lo indica USG.

PREGUNTA
Cuál es el factor más importante que busca para tomar una decisión quirúrgica.

RESPUESTA
a.- Calcificación placentaria.
b.- Características del líquido amniótico.
c.- Cantidad de líquido amniótico.
d.- Características del cordón umbilical.

CESAREA. CIENCIAS BASICAS: La cesárea es la intervención quirúrgica que tiene como objetivo extraer el producto de la concepción y
sus anexos ovulares a través de una laparotomía e incisión de la pared uterina. TIPOS DE CESÁREA: A) Según antecedentes obstétricos
de la paciente: Primera: Es la que se realiza por primera vez. Iterativa: Es la que se practica en una paciente con antecedentes de dos o
más cesáreas. Previa: es cuando existe el antecedente de una cesárea anterior. B) Según indicaciones: Urgente: Es la que se practica
para resolver o prevenir una complicación materna o fetal en etapa crítica. Electiva: Es la que se programa para ser realizada en una
fecha determinada por alguna indicación médica y se ejecuta antes de que inicie el trabajo de parto. C) Según técnica quirúrgica:

CURSO ENARM CMN SIGLO XXI TEL: 36246001 Pharmed Solutions Institute PÁGINA 335
MANUAL DE TRABAJO DEL CURSO ENARM CMN SIGLO XXI
Corporal o clásica. Segmento corporal (Tipo Beck). Segmento arciforme (Tipo Kerr). CORPORAL O CLASICA: La incisión es vertical se
realiza en el cuerpo uterino. Sus indicaciones más frecuentes son: cáncer cérvico-uterino invasor, embarazo pretérmino, situación fetal
transversa con dorso inferior, histerorrafia corporal previa, procesos adherenciales o varicosos importantes en el segmento inferior,
placenta previa en cara anterior, cesárea posmortem, miomatosis uterina de grandes elementos y cuando después de la cesárea se
realizará una histerectomía. Sus desventajas son: apertura y cierre más difícil, mayor hemorragia, adherencias más frecuentes,
histerorrafia menos resistente que puede hacerse dehiscente durante un nuevo embarazo. SEGMENTO CORPORAL (Beck): La incisión
es vertical y se realiza sobre el segmento y parte del cuerpo uterino. Sus principales indicaciones son: embarazo pretérmino, embarazo
gemelar, situación fetal transversa con dorso inferior, presentación pélvica, placenta previa en la cara anterior del útero, anillo de
retracción e histerorrafias corporales previas. Las desventajas de esta técnica no difieren de la anterior. SEGMENTO ARCIFORME O
TRANSVERSAL (Kerr): Es la técnica quirúrgica más usada por sus múltiples ventajas. Al realizarse una incisión transversal del segmento
inferior tiene las ventajas de producir menos hemorragia, y permitir una fácil apertura y cierre de la pared uterina, formación de cicatriz
uterina muy resistente con poca probabilidad de dehiscencia y ruptura en embarazos subsecuentes y así como pocas adherencias
postoperatorias. INDICACIONES DE LA OPERACIÓN CESÁREA: La indicación para realizar una operación cesárea puede ser por causas:
Maternas, Fetales o Mixtas. A) Causas maternas; 1. Distocia de partes óseas (desproporción cefalopélvica): Estrechez pélvica. Pelvis
asimétrica o deformada. Tumores óseos de la pelvis. 2. Distocia de partes blandas: Malformaciones congénitas. Tumores del cuerpo o
segmento uterino, cérvix, vagina y vulva que obstruyen el conducto del parto. Cirugía previa del segmento y/o cuerpo uterino,
incluyendo operaciones cesáreas previas. Cirugía previa del cervix, vagina y vulva que interfiere con el progreso adecuado del trabajo
del parto. 3. Distocia de la contracción: Hemorragia (placenta previa o desprendimiento prematuro de la placenta normoinserta).
Patología materna incluyendo nefropatias, cardiopatías, hipertensión arterial o diabetes mellitus, etc. B) Causas fetales: Macrosomía
fetal que condiciona desproporción cefalopélvica. Alteraciones de la situación, presentación o actitud fetal. Prolapso de cordón
umbilical. Sufrimiento fetal. Malformaciones fetales incompatibles con el parto. Embarazo prolongado con contraindicación para parto
vaginal. Cesárea posmortem. C) Causas mixtas: Síndrome de desproporción cefalopélvica. Preeclampsia/eclampsia. Embarazo múltiple.
Infección amniótica. Isoinmunización materno-fetal. INDICACIONES MÁS FRECUENTES: Las principales indicaciones de la operación
cesárea son: 1. Desproporción cefalopélvica: La desproporción cefalopélvica ocurre en las siguientes circunstancias: Pelvis estrecha,
deformada u obstruida y feto normal con buena actitud. Pelvis normal y feto grande, anómalo (gigantismo o hidrocefalia) o con mala
actitud. Pelvis y feto normales con diámetros fetopélvicos limítrofes, después de una prueba de trabajo de parto infructuoso. El
diagnóstico de la DCP se establece mediante una valoración clínica adecuada de los estrechos superior, medio e inferior de la pelvis
materna y su relación con el feto, con el apoyo de estudios de inmagenologia. Ante la duda diagnóstica se deberá utilizar la prueba de
trabajo de parto. 2. Cesárea previa: la estrategia de permitir un parto vaginal a pacientes con cesárea previa bajo condiciones
controladas, logra un incremento en los nacimientos por vía vaginal sin complicaciones. Contraindicaciones para permitir el parto
vaginal en mujeres con cesárea previa: Antecedente de dos o más cesáreas previas segmento arciforme Antecedentes de cesárea
previa tipo segmento-corporal, corporal o presencia de otras cirugías uterinas. Antecedente de cesárea previa reciente (menor a 6
meses). Antecedentes de ruptura uterina o dehiscencia de histerorrafia. Pelvis no útil clínicamente 3. Sufrimiento fetal: Los factores
causales del sufrimiento fetal pueden ser reversibles (hipotensión materna, efecto Poseiro, hipercontractilidad por uso de oxitócicos), o
irreversibles (causa fetal, placentaria o del cordón umbilical). Se considerarán los siguientes parámetros para evaluar la frecuencia
cardíaca: Basal (normal): 120-160 lpm. Bradicardia: leve: entre 110-119 lpm, moderada: entre 100-109 lpm, severa: por debajo de 100
lpm. Taquicardia: leve: entre 160-169 lpm, moderada: 170-179 lpm, severa: mayor de 180 lpm. En las unidades de atención que
disponen de equipos de cardiotocografía se registrará la frecuencia cardíaca fetal y su relación con las contracciones uterinas. Las
principales alteraciones que pueden observarse son: Bradicardia menor de 100 lpm. mantenida durante 5 minutos o más. Presencia de
desaceleraciones tardías o dips tipo II en el 30% o más de las contracciones. Desaceleraciones variables graves (menos de 70 lpm de
más de 60 seg. de duración) en el 50% de las contracciones. Variabilidad de la curva de 5 lpm o menos por más de 30 minutos.
Taquicardia mayor de 170 lpm que persisten por más de 10 min. Patrón sinusoidal (con amplitud menor de 5 lpm), sin observarse
movimientos fetales, ni aceleraciones. Se considerará la operación cesárea, cuando exista la asociación de dos o más de las alteraciones
de la frecuencia cardíaca fetal antes señaladas. 4. Ruptura prematura de membranas: Es la salida de líquido amniótico a través de una
solución de continuidad de las membranas ovulares en embarazos mayores de 20 semanas y/o por lo menos 2h antes del inicio del
trabajo de parto. La conducta obstétrica en los casos de ruptura prematura de membranas, es la siguiente: a) Menor de 26 semanas de
gestación: interrupción del embarazo por vía vaginal con el consentimiento de la mujer. b) Entre 26 y 28 semanas de gestación: cuando
se cuente con los recursos mínimos para la atención del neonato, se realizará operación cesárea. c) Entre 29 y 32 semanas de
gestación: se realiza operación cesárea. d) Entre las semanas 33 y 36 de gestación: se valorarán las condiciones obstétricas y se
resolverá el embarazo por la vía más favorable. 5) Presentación pélvica: ocurre en el 3-4 % del total de los embarazos. En la mayoría de
las unidades obstétricas, esta modalidad de presentación constituye una indicación de cesárea (excepto cuando la paciente acude en
periodo expulsivo). ANTIBIOTICOTERAPIA EN LA OPERACIÓN CESÁREA: Como en otras intervenciones quirúrgicas, la realización de la
operación cesárea conlleva riesgo de infección; la frecuencia y severidad de las infecciones depende de las condiciones en las que se
realiza y con base en ello, puede considerarse como: a) Limpia: Se considera que una operación cesárea es limpia cuando se realiza en
condiciones asépticas, sin defectos de la técnica, ni lesiones en los aparatos gastrointestinal o urinario. No hay reacción inflamatoria en
la vecindad y la cavidad uterina no está contaminada. En ésta no hay necesidad de utilizar antibióticos. b) Contaminada: Se considera
de este modo cuando existen: Trabajo de parto de más de 6h. Más de 6 tactos vaginales. Membranas rotas entre 6 y 24 hrs.
Antecedentes de aminiocentesis o instrumentaciones. Alto grado de dificultad en la técnica quirúrgica. Anemia severa (Hb. menor de 9
mg /dl). Líquido amniótico meconial. Se utilizarán antibióticos de manera profiláctica desde el momento en el que se realiza el
pinzamiento del cordón umbilical. c) Séptica: Cuando hay evidencia de infección clínica, supuración o presencia de materia fecal; en
esta categoría se incluyen las cesáreas con uno o más de los siguientes factores: Cuadro febril. Líquido amniótico fétido o caliente.
Ruptura de las membranas ovulares de más de 24h. Cesáreas con alto grado de dificultad en la técnica quirúrgica. La antibioticoterapia
se mantendrá por vía endovenosa 72h y se completará el esquema según la evolución de la paciente. REQUISITOS PREOPERATORIOS:
Antes de la operación cesárea deberán cubrirse los siguientes requisitos preoperatorios: Historia clínica perinatal completa, incluyendo
exámenes de laboratorio y gabinete. Consentimiento informado de la mujer o de un familiar responsable, y firma de la autorización
para realizar la operación cesárea. En el caso de la oclusión tubaria bilateral, por ser un método anticonceptivo permanente se requiere

CURSO ENARM CMN SIGLO XXI TEL: 36246001 Pharmed Solutions Institute PÁGINA 336
MANUAL DE TRABAJO DEL CURSO ENARM CMN SIGLO XXI
además la firma de consentimiento de la paciente en el formato institucional específico. Nota preoperatoria en el expediente clínico en
la que se fundamente la indicación de la operación cesárea. CUIDADOS PREOPERATORIOS Y POSTOPERATORIOS: Preoperatorios
incluyen: Valoración preanestésica de acuerdo a la NOM-170-SSAl-1998, Para la Práctica de Anestesiología. Ayuno preoperatorio de 8h
o más (cuando el caso lo permita). Rasurado suprapúbico y aseo completo, con especial énfasis en la vulva y perineo. Colocación de
sonda vesical. Disponibilidad de sangre segura y compatible. Postoperatorios consistirán en: Medición horaria de signos vitales durante
las primeras cuatro horas y posteriormente cada 8h hasta el alta de la paciente. Vigilancia de la diuresis durante las primeras 12h y del
inicio de la micción espontánea, después de retirar la sonda vesical. Administración de líquidos por vía oral después de 8h; una vez
iniciado el peristaltismo intestinal se indicará dieta blanda.

CASO CLINICO
Acude a consulta paciente femenino gestante con 39 semanas de gestación por USG la cual refiere tener 12 horas con dolor en región
abdominal y lumbo-sacra, agrega que cuenta con antecedentes de gesta 4 para 2 cesárea 1, aborto 0, a la exploración identifica
producto podálico con borramiento del 30 % y dilatación de 2 cm, la paciente cuenta con antecedentes de polidramnios en embarazo
previo con producto con atresia gastroesofagica con muerte perinatal, envía a su casa y cita en 6 horas, a la exploración nuevamente
observa a la paciente la cual se encuentra fatigada, se observa actividad uterina con 3 contracciones en 10 minutos aproximadamente,
con duración de 50 segundos, la paciente refiere ruptura de membranas con presencia de liquido verdoso, usted explora al producto el
cual se encuentra transversal con frecuencia cardiaca de 100 a 120 lpm.

PREGUNTA
Cuál es su conducta más adecuada a seguir.

RESPUESTA
a.- Prepara a la paciente para cesarea.
b.- Prepara a la paciente para conducción.
c.- Espera a que el trabajo de parto continue.
d.- Inicia manejo con tocolitico.

COMPLICACIONES DEL PUERPERIO. CIENCIAS BASICAS: Se entiende por puerperio al periodo de tiempo que sigue al momento del
parto durante el cual los cambios fisiológicos del embarazo, trabajo de parto y parto se resuelven y la mujer retorna al estado
pregravídico y en la glándula mamaria se producen una serie de cambios fisiológicos que permiten la lactancia. Usualmente este
periodo de tiempo es de 6 semanas. El puerperio es un periodo con gran número de particularidades, donde pueden quedar
enmascaradas serias complicaciones por hechos fisiológicos, por lo cual se debe distinguir bien entre los límites del puerperio
normal y los signos o síntomas que deben alertar. En el puerperio inmediato se aconsejan las siguientes acciones, que pueden disminuir
el riesgo de complicaciones: Vigilancia de la adecuada involución uterina; masaje uterino. Lactancia precoz, limpieza del pezón. Insistir
en la higiene adecuada de la episiorrafia o de la herida quirúrgica si la paciente tuvo una cesárea. Deambulación precoz (en las primeras
24 horas) para disminuir la posibilidad de presentación de eventos tromboembólicos y mantener activo el tránsito intestinal y vesical.
La paciente puede iniciar su alimentación idealmente 2 horas después de un parto vaginal; no antes porque dentro de este lapso
pueden ocurrir las complicaciones inmediatas del posparto que requieran anestesia. La revisión de la paciente en puerperio se puede
realizar a la semana del posparto, cuando se puede identificar cualquier anomalía e iniciar el método de planificación familiar.
CLASIFICACION: Cronológicamente se divide en: Puerperio inmediato: primeras 24h posparto. Puerperio mediato: desde el segundo
hasta el décimo día posparto. Puerperio tardío: desde el día 11 hasta el día 42 posparto. HEMORRAGIA POSPARTO: Se define como una
pérdida sanguínea mayor de 500cc luego del parto vaginal y 1000cc luego de la cesárea. De una manera objetiva ha sido descrito como
la disminución del hematocrito en 10% o más en comparación con el valor inicial. Su diagnóstico se debe realizar de una manera
oportuna para darle un manejo adecuado. La frecuencia de la hemorragia posparto es de 3% – 4% en las pacientes que han tenido un
parto vaginal, hasta 9% en quienes han tenido cesárea. Todo esto aumenta si está asociado a factores de riesgo. Usualmente se clasifica
en dos tipos: Hemorragia posparto inmediata: se llama así la que tiene lugar en las primeras 24 horas del puerperio; la mayoría ocurre
en las primeras 2 horas. Ésta es la hemorragia más frecuente y la más importante. Hemorragia posparto tardía: Se presenta a partir del
segundo día del puerperio hasta 6 semanas después del parto. La frecuencia oscila entre 5 a 15 %. A. HEMORRAGIA TEMPRANA: 1.
RETENCIÓN DE PLACENTA: El principal mecanismo del alumbramiento está dado por la contracción uterina y la formación del
hematoma retroplacentario. El tiempo recomendado para que se produzcan los mecanismos del alumbramiento es de 15 a 20 minutos;
más tiempo se considera retención de la placenta. El manejo activo del alumbramiento se ha considerado una manera adecuada para
evitarla (se realiza la maniobra de Brandt: el descenso de la pinza que ligó el cordón indica cuando empieza a desprenderse la placenta),
uso de oxitócico desde el tercer estadio del parto. 2. PLACENTA ANORMALMENTE ADHERIDA: Es un diagnostico que se realiza desde el
tercer trimestre o en el momento del parto. Consiste en una adherenciaanormal de las vellosidades a la pared uterina; existe una
deficiencia o ausencia de la decidua basal, y es difícil que se realice el alumbramiento normal, con la consecuente retención de
placenta. Se obtiene una placenta con sangrado de difícil control y precoz. Clasificación: Ácreta: la placenta se encuentra adherida al
endometrio Íncreta: penetran las vellosidades en el miometrio Pércreta: la invasión alcanza la serosa peritoneal y puede llevar a
ruptura del útero. ATONIA UTERINA: Se define por la inadecuada involución uterina, por la no contracción miometrial y cambios
vasculares intramiometrales y ausencia de un adecuado sistema coagulación que permita su control. Es la responsable del 80 % de
todas las hemorragias posparto, considerando que en el embarazo a término la suplencia sanguínea de la placenta es de
aproximadamente 600 a 800 mL/min. Se debe estar preparado para realizar esta reposición de la volemia y desencadenar los
mecanismos de adaptación materna. Factores de riesgo: Durante el embarazo: Multigestantes, infección intraamniotica, fibromas
uterinos, sobredistencion uterina, polihidramnios. Macrosomia fetal. Durante el trabajo de parto: Trabajo de parto precipitado, trabajo
de parto prolongado. Relajación uterina por medicamentos como sulfato de magnesio, halogenados anestésicos, nitroglicerina.
Hemorragia del tercer trimestre (diagnóstico final del embarazo) Tratamiento: 1. Inicialmente reemplazar la volemia con soluciones
cristaloides y dependiendo del volumen de sangre perdido y el estado clínico de la paciente, administrar glóbulos rojos empacados. 2.

CURSO ENARM CMN SIGLO XXI TEL: 36246001 Pharmed Solutions Institute PÁGINA 337
MANUAL DE TRABAJO DEL CURSO ENARM CMN SIGLO XXI
Cuando la hemorragia se presenta antes de que se expulse la placenta se debe realizar un alumbramiento artificial. 3. Si ya ha salido la
placenta, entonces se procede a revisar el útero y el canal de parto. 4. La aplicación de útero-tónicos: Oxitocina 10 a 20 U en infusión
continua. Los efectos adversos son poco: dolor con las contracciones, nausea, vómito, en casos extremos intoxicación hídrica.
Prostaglandinas. Disponible aplicación de Misoprostol (análogo sintético de la prostaglandina E 1). Dosis: 5 tabletas de 200
microgramos por vía rectal, con muy buenos resultados reportados en la literatura. Efectos adversos: hipertermia transitoria,
broncoespasmo. Vasopresina: efecto vasoconstrictor: en casos extremos de no respuesta y si está disponible. Inyección directa no
vascular de 0.2 unidades (dilución de 20 unidades en solución salina 10occ, aplicar 1 cc ). INVERSIÓN UTERINA: Es la invaginación del
útero sobre sí mismo. El fondo del útero es impulsado hacia abajo y la parte superior de la cavidad endometrial se expone a través del
cuello uterino y aparece en los genitales externos. El control y manejo incorrecto del alumbramiento es el principal factor
desencadenante de esta complicación. Es muy raro que ocurra espontáneamente. Se produce al traccionar en forma brusca y sostenida
el cordón umbilical, combinado con la presión sobre el fondo uterino, tratando de abreviar el tercer período del parto. Su presentación
es súbita, acompañada de la hemorragia, dolor pélvico intenso, hasta el shock neurogénico por la gran tensión de todo el complejo de
ligamentos de sostén uterino hasta la zona genital externa. Cuando ocurre inmediatamente después de la expulsión de la placenta se
debe tratar: Colocar el útero en forma rápida de una manera anatómica. Introducir una mano en la vagina y hacer presión suave y
directa hacia arriba en el mismo eje del estrecho pélvico superior. Aplicar Clorhidrato de Ketamina IV por parte del anestesiólogo para
una relajación materna. Estimular la contracción uterina con infusión continua de oxitocina y masaje sobre el fondo uterino. Si se
produce la inversión antes de la salida de la placenta se debe considerar una normalidad de la inserción placentaria y manejarse como
acretismo placentario. B. HEMORRAGIA TARDÍA: SUBINVOLUCIÓN UTERINA: Es una detención o retardo de la involución. La
subinvolución va acompañada de una prolongación del periodo de eliminación de los loquios y a veces, de una hemorragia uterina
irregular o excesiva. Clínicamente el útero se encuentra de mayor tamaño, más blando que lo normal para un momento determinado
del puerperio. Entre las causas reconocidas de subinvolución están la retención de fragmentos de placenta y la infección pélvica.
Diagnóstico: Examen clínico detallado. Realización de ecografía trasvaginal. Tratamiento: Si se confirman restos placentarios: curetaje.
Útero tónico: (antes descritos). Si se diagnostica infección inicio de antibióticos de amplio espectro. HEMATOMA VULVAR: Se
diagnostica fácilmente por el intenso dolor y la aparición súbita de un tumor sensible, fluctuante e intenso, de tamaño variable y
cubierto por una piel cuya coloración es diferente de lo normal, violáceo. Puede presentarse también en el posparto inmediato. Su
manejo es la Incisión y evacuación de sangre, seguidas de las ligaduras de los puntos que sangran. La cavidad debe obliterarse
empleando una sutura adecuada, usualmente catgut cromado 2 /0. Se administran antibióticos de amplio espectro si hay signos de
infección. INFECCIÓN PUERPERAL: Las infecciones pélvicas son las más comunes y serias complicaciones del puerperio. Se define como:
temperatura mayor de 38 grados, excluyendo las primeras 24 horas. Las causas más frecuentes de fiebre en el puerperio son
endometritis, infección urinaria y mastitis. ENDOMETRITIS: Es la forma más común de infección puerperal, se relaciona con la decidua,
el miometrio adyacente y los tejidos parametriales. Su vía de Infección es ascendente, endógena y polimicrobiana (Gram positivos del
endometrio en el 40% de los casos, los gérmenes aerobios gram negativos se aíslan del endometrio en el 22%, Las bacterias anaerobias
representan el 38% de los patógenos, La Chlamydia Trachomatis es responsable del 2 al 4% de la endometritis tardía, hasta 6 semanas
posparto. Frecuencia de endometritis posterior a cesárea 7 a 30 veces más frecuentecomparada con el parto vaginal. Depende de los
factores de riesgo asociados. Puede ser hasta un 90% si no se usan antibióticos profilácticos. Complica el 5 a 6 % de las cesáreas
electivas y el 22 a 85% de las urgentes Los síntomas iniciales aparecen generalmente en las primeras 48 a 72 horas. La paciente
consulta usualmente a los 4 a 5 días después del parto por fiebre alta de 39 a 40 grados, (este es el signo cardinal de la endometritis, de
ahí que el aumento de la temperatura en el puerperio, se debe considerar endometritis hasta que se pruebe lo contrario), síntomas
generales, taquicardia, escalofrío, expulsión de loquios que tienen un aspecto ajamonado (color rosado brillante) y de muy mal olor .Se
queja de dolor abdominal, principalmente en hipogastrio. La involución del útero puede estar retardada. Si se encuentra fiebre de 38.5
a 39 grados centígrados durante las primeras 24 horas posparto, acompañada de distensión abdominal, taquicardia y loquios claros
acuosos, se debe considerar el diagnóstico de endometritis por Estreptococo hemolítico del grupo B o del grupo A; el cuadro clínico
puede progresar a celulitis pélvica, peritonitis y septicemia. Tratamiento: Teniendo en cuenta la microbiología, los antibióticos usados
deben ser de amplio espectro. Para casos leves, usualmente después de parto vaginal, un antimicrobiano de amplio espectro por vía
oral puede ser suficiente. Hay mejoría en 48 a 72 horas en el 90% de las pacientes. Los esquemas pueden ser: Penicilina procaínica.
800.000 unidades IM cada 12 a 24 horas ó Ampicilina 500 mg a 1 gr. Vía oral cada 6 horas, ó Cefalosporínas de primera generación. 500
mg a 1 gr. Vía oral, cada 6 horas, más Un Aminoglicósido tipo Gentamicina 160 mg/día. Sin embargo, para casos moderados a severos,
incluyendo la mayoría de aquellos que tuvieron parto por cesárea, se inicia terapia parenteral. Hospitalizar y utilizar un esquema
biconjugado ó triconjugado Emplear antibióticos de amplio espectro: Penicilina Cristalina 5.000.000 Unidades IV cada 6 horas ó
Cefalosporínas de primera generación 1 gramo IV cada 6 horas ó Ampicilina 1 gr., IV cada 6 horas, más Gentamicina 80 mg IV cada 8
horas ó 240 mg IV ó IM. , dosis única al día Este esquema no cubre Bacteroides Fragilis, no previene las complicaciones de
tromboflebitis pélvica y absceso pélvico, hay un mayor tiempo de hospitalización. En un 20 y 40% de los casos posoperatorio de cesarea
no responden a este tratamiento. Por eso se adiciona un tercer antibiótico que cubra anaerobios. Metronidazol 1 gr., inicial seguido
500mg IV ó vía oral cada 8 horas ó Ornidazol 1 gr., cada 12 horas IV ó Clindaminica 600 cada 6 horas a 900 mg IV cada 8 horas (no sólo
cubre anaerobios, también estafilococo, estreptococo y Chlamydia), con este esquema sólo se añade la Gentamicina. INFECCIÓN E LA
HERIDA QUIRÚRGICA: La incidencia de infección de la incisión abdominal luego de la cesárea es de un 5 a 15% con un promedio de 7%.
Cuando se administra antibióticos profilácticos la incidencia es de un 2% o menos. Los factores de riesgo para la infección de la herida
abdominal son: Obesidad, diabetes, terapia con esteroides, inmunosupresión, anemia y pobre hemostasia con formación de
hematoma. El tratamiento consiste en drenaje quirúrgico y antibióticos. Evaluar que la fascia este intacta, ó si no el cierre secundario se
debe realizar. MASTITIS: Cuando la congestión mamaria o las grietas del pezón no mejoran, puede ocurrir una infección superficial o
profunda de la glándula. La inflamación va precedida casi siempre de una ingurgitación, fiebre, taquicardia, la mama se torna dura,
enrojecida, y la paciente se queja de dolor. El microorganismo responsable más frecuente es el Estafilococo Áureos. También se ha
aislado el Estafilococo coagulasa negativo y el Estreptococo viridans. La fuente inmediata de estafilococos que provocan mastitis está
casi siempre en la nariz y garganta de la madre. En el momento de la lactancia la bacteria penetra en la mama a través del pezón, en el
lugar donde existe alguna grieta o fisura, que puede ser mínima. La bacteria se puede cultivar de la leche materna. TRATAMIENTO:
manejo quirúrgico; Taponamiento uterino: es necesario cuando el tratamiento quirúrgico no sea posible en el sitio actual o la paciente

CURSO ENARM CMN SIGLO XXI TEL: 36246001 Pharmed Solutions Institute PÁGINA 338
MANUAL DE TRABAJO DEL CURSO ENARM CMN SIGLO XXI
este demasiado inestable para una cirugía. Requisitos: 1. Eliminar todo el tejido placentario a excepción de los fragmentos
correspondientes a una placenta ácreta. 2. Descartar una rotura y una inversión uterina. 3. Usar previamente los oxitócicos y el masaje
uterino. 4. Descartar otra causa de sangrado. 5. Un ayudante expone el cuello traccionando los labios anteriores y posteriores. Se
emplea una pinza para extraer restos ovulares con el fin de empacar gasa seca en el fondo uterino de modo que quede bien apretada,
luego se completa el procedimiento dejando comprimida la vagina. 6. Durante el procedimiento se debe aplicar una infusión continua
de oxitocina. 7. La paciente debe recibir antibióticos 8. Se remite con sonda vesical abierta. 9. Si fracasa no debe intentarse
nuevamente, y se debe realizar una laparotomía sin pérdida de tiempo. Si este procedimiento falla, se pasa al siguiente paso: ligadura
de los vasos uterinos: La arteria es ligada donde corre al lado del borde uterino, por encima del segmento uterino inferior. Si es
posterior a una cesárea, la ligadura se realiza 2 a 3 cm. por debajo del nivel de la incisión uterina. Puede ser necesario descender la
vejiga. La arteria y la vena pueden ser ligadas pasando una aguja 2 a 3 cm. medial a los vasos pélvicos, incluyendo la mayoría del grosor
del miometrio, y atravesando el ligamento ancho en el área avascular lateral a los vasos. Histerectomía: Desafortunadamente no
garantiza el control de la perdida sanguínea severa. El sangrado puede persistir de las superficies pélvicas debido a alteraciones de la
coagulación combinado con el trauma de la manipulación prolongada. Se debe continuar con empaquetamiento abdominal.
Empaquetamiento abdominal: Aplicación de compresas estériles en la cavidad pélvica, para controlar el sangrado continúo de las
superficies peritoneales cuando ya se realizó la histerectomía y se inició una Coagulopatía. Son retiradas 24 horas luego de corregida la
Coagulopatía. Embolización: de la arteria uterina / iliaca interna: cuando las opciones quirúrgicas se han agotado y existe la posibilidad
de intervención por radiología.

CASO CLINICO
Acude a consulta paciente con 48 horas posparto, refiere que presenta dolor en región pélvica de características punzante e intensidad
moderada a severa, agrega que presenta cefalea continua, malestar generalizado, fatiga, adinamia, cuenta con antecedentes GO de
gesta 3 para 3 abortos 0, refiere que presento flujo amarillento en varias ocasiones en este ultimo embarazo, a la exploración se
observa TA 100/70, FC 89 FR 24, Temperatura de 38.5 grados, a la palpación refiere dolor, al tacto se identifica liquido
serosanguinolento fétido y útero con flacidez, decide ingresar a la paciente para manejo de la complicación posparto que presenta
(corioamnionitis), a las 3 horas de ingreso se presenta dificultad para respirar, a la auscultación se encuentra estertores bilaterales y
ataque al estado generalizado, TA de 70 /50 FC 102, FR 31, Temperatura de 36.1 grados.

PREGUNTA
Considerando la gravedad del cuadro cual es la complicación más probable en esta paciente.

RESPUESTA
a.- Tromboembolia pulmonar.
b.- Coagulacion intravascular diseminada.
c.- Sepsis abdominal.
d.- Choque distributivo.

HEMORRAGIA OBSTETRICA. CIENCIAS BASICAS: Se define como aquel sangrado que se produce a través de la vagina en cualquier
momento del embarazo y cuya presencia no está justificada. Durante las primeras 20 SDG el aborto, el embarazo ectópico y la mola
hidatiforme, son las principales causas de sangrado transvaginal. Las hemorragias que se presentan en el tercer trimestre del embarazo
suelen ser todo un reto, pues estos sangrados son los que con mayor frecuencia ponen en riesgo la vida de la madre y del feto. SALUD
PUBLICA: Hemorragia obstétrica causa importante de morbimortalidad materna y perinatal. La hemorragia obstétrica se ha identificado
como la causa única más importante de decesos maternos en todo el mundo; explica casi la mitad de los fallecimientos postparto. En
México en el 2005, las hemorragias durante el embarazo se encuentran como la
segunda causa de mortalidad materna con un porcentaje de 26.1%.
CLASIFICACION: 1) Hemorragias preparto: pueden deberse a embarazo
ectópico, aborto espontaneo, enfermedades del trofoblasto, lesiones vaginales y
cervicales locales, trastornos de la coagulación, cáncer. A la mitad del embarazo
se deben: anomalías de la implantación placentaria (placenta previa),
desprendimiento prematuro de placenta normoinserta (DPPNI), trabajo de
parto pre término y 2) Hemorragias posparto: trauma del canal del parto,
desgarros cervicales, atonía uterina, inversión uterina, retención de restos
placentarios. PLACENTA ACRECTA. CIENCIAS BASICAS: El acretismo placentario
es la inserción anormal de parte o de toda la placenta, con ausencia total o
parcial de la decidua basal y anormalidad de la caduca verdadera con
penetración de las vellosidades coriales al miometrio. Es una de las principales causas de hemorragia obstétrica, principalmente del
post parto inmediato, y que condiciona un importante riesgo demorbimortalidad materna, por la patología misma, como también por
el tratamiento aplicado. SALUD PUBLICA: La incidencia de placenta acreta está aumentando, especialmente por la mayor incidencia de
operación cesárea; el mayor riesgo de morbimortalidad materna disminuye con un adecuado diagnóstico prenatal y planificando la
interrupción del embarazo con un equipo quirúrgico multidisciplinario, en una institución que disponga de banco de sangre. La edad
media de las pacientes fue de 31 años, y el grupo de mayor riesgo fue el de 35 a 39 años con 26% de acretismo placentario.
CLASIFICACION: Existen tres variedades de adherencia anormalde la placenta: Placenta acreta: es la adherencia anormal de una parte o
de la totalidad de la placenta a la pared uterina, sin que las vellosidades coriales penetren el miometrio. Placenta increta: las
vellosidades coriales penetran el miometrio. Placenta percreta: es la penetración de los elementos coriales hasta sobrepasar la serosa
del útero, pudiendo alcanzar órganos vecinos. Por su extensión se reconocen tres tipos: a) Focal: solo involucra pequeñas áreas de la
placenta; b) Parcial: uno o más cotiledones se involucran en el proceso; c) Total: la superficie completa de la placenta esta
anormalmente adherida. PATOGENIA: Los principales factores de riesgo identificados para acretismo placentario de acuerdo a los

CURSO ENARM CMN SIGLO XXI TEL: 36246001 Pharmed Solutions Institute PÁGINA 339
MANUAL DE TRABAJO DEL CURSO ENARM CMN SIGLO XXI
antecedentes son: Edad (mayores de 30 años), Gestas (multíparas de 2 a 3), Placenta previa con antecedente de cesárea (35%),
Placenta previa con antecedente de 3 o más cesáreas (67%), Historia de legrado uterino (18 a 60%), Antecedente de extracción manual
placentaria. Historia de retención placentaria. Antecedente de infección intramniótica. Se piensa que el acretismo placentario es
debido a una ausencia o deficiencia de la capa de Nitabuch, que es una capa de fibrina que se ubica entre la decidua basal y el corion
frondoso. Benirschke y Kaufman, sugieren que esta situación es consecuencia de una falla en la reconstitución del endometrio/decidua
basal, especialmente después de la reparación de una operación cesárea. El acretismo placentario puede llevar a hemorragia obstétrica
masiva, y como consecuencia alteraciones de la coagulación por pérdida, y porcoagulación intravascular diseminada. DIAGNOSTICO:
Básicamente el diagnóstico se limita al empleo de métodos como es el ultrasonido y la resonancia magnética. Sin embargo, el
diagnóstico definitivo de acretismo placentario es por medio de histopatología, al comprobar la invasión de las vellosidades coriales en
el miometrio. Ultrasonografía. El ultrasonido es una útil herramienta para el diagnóstico de acretismo placentario así como de sus
distintas variantes en el segundo y tercer trimestre del embarazo. Finberb y Williams, establecieron en 1992, la utilidad de esta técnica
de imagen; entre los criterios diagnósticos establecidos fueron: pérdida de la zona hipoecoica miometral retroplacentaria,
adelgazamiento o disrupción de la serosa uterina hiperecoica y la interfaz con la vejiga, la presencia de masas exofíticas, presencia de
una gran área de sonolucencias placentarias. Alfa-feto proteína fetal sérica materna. Actualmente se ha propuesto que existe una
anormalidad en la interfaz placeta-útero, lo cual promueve una fuga de alfa-feto proteína fetal hacia la circulación materna. Histología.
Los criterios microscópicos para el diagnóstico de acretismo placentario es la ausencia de la decidua basal. Sin embargo su diagnóstico
no resulta del todo fácil por la distorsión que ocurre en el momento del alumbramiento, salvo que se deje la pieza in situ y se realice
una histerectomía y así se obtenga la muestra del tejido. En el caso de placenta increta o percreta el diagnóstico es más fácil y consiste
en la presencia de tejido trofoblástico en el espesor uterino o en la estructura invadida. TRATAMIENTO: El principal tratamiento es la
histerectomía obstétrica total abdominal, debido a su frecuente asociación con placenta previa y cicatrices de cesáreas previas .Este
procedimiento es un desafío a las habilidades quirúrgicas del médico obstetra, por lo que es necesario conocer otras alternativas de
tratamiento. Manejo conservador. Es una medida que consiste en evitar la histerectomía, al menos en ese momento, y tratar de
preservar la fertilidad. Existen diversas modalidades de manejo conservador,que incluyen: Dejar la placenta en su lugar (totalmente o
en fragmentos). Resección del lecho placentario y su reparación. Extracción y legrado obstétrico. Empleo de medicamentos asociados a
cualquiera de los anteriores puntos. Empleo de algún medio que cause isquemia (embolización, ligadura de vasos, etc.) del lecho
placentario. El metotrexate es un medicamento quimioterápico el cual se encuentra catalogado dentro del grupo de los antagonistas
de los folatos. Su empleo en el acretismo placentario tiene como fundamento la efectividad en contra del trofoblasto proliferativo. Sin
embargo, de forma más reciente se ha argumentado que después del nacimiento del feto la placenta detiene su división y por lo tanto
pierde su utilidad.

SINDROME DE TENSION PREMENSTRUAL. CIENCIAS BASICAS: La American Psychiatric Associationlo denomina DDPM (desorden
disfóricopremenstrual) o DFL (desorden de fase lútea).Se caracteriza por ser una entidad con sintomatología somática y afectiva
(trastornos anímicos) que produce disfunción e incapacidad significativa. Posiblemente el DDPM no sólo representa el extremo más
grave de la sintomatología premenstrual sino quizás una modalidad de trastorno depresivo o de ansiedad. Durante la vida
"reproductiva" de la mujer, son frecuentes una serie de cambios tanto físicos como psicológicos (molimia premenstrual), que
acompañan a los distintos ciclos menstruales; tan sólo, una pequeña proporción de mujeres (3-5%) llegan a presentar el llamado
síndrome premenstrual (SPM) definido como la recurrencia cíclica de una combinación de malestar físico, psicológico y/o cambios en la
conducta presentes en la fase luteínica (aumento de estrógenos y progesterona) de la mayoría de los ciclos, que remiten al final de la
menstruación, con un intervalo libre de síntomas de al menos una semana por ciclo y de tal severidad que producen un deterioro de las
relaciones interpersonales y/o interfieren con otras actividades cotidianas. SALUD PUBLICA: 3-10% de las mujeres en edad reproductiva
son asintomáticas. 30-90% presentan síntomas premenstruales leves. 20-30% presentan síntomas moderados a graves que alteran su
forma de vida. 3-8% padecen síntomas graves e invalidantes. Las causas por las que las mujeres solicitaban ayuda médica eran la
depresión severa, fátiga y disminución de la concentración. PATOGENIA: En la actualidad es desconocida; se han propuesto tanto
biológicas como psicológicas para explicar el origen del SPM. A) Hipótesis biológicas: postulan que los síntomas pueden surgir como
resultado de alteraciones neuropsicológicas causadas por la acción a nivel del SNC de esteroides gonadales, neurotransmisores o
sustancias neuromoduladoras. Ciertos hallazgos fisiológicos apoyan el origen orgánico-biológico: Alteraciones de la conductividad
cutánea. Cambios a nivel del EEG ciclo-dependiente, con aparición de ondas alfa y aumento de la duración de la fase REM durante el
sueño, siendo directamente proporcional a la severidad de los síntomas. Otros hallazgos en el EOG (Electrooculograma). Se ha
intentado relacionar cambios hormonales con alteraciones en las monoaminas (5HT, NA), centrándose en algunos trabajos en las
alteraciones del sueño y de la termoregulación (aumento de la temperatura corporal nocturna). Se sabe que la serotonina (5HT)
disminuye los niveles de estrógenos y de progesterona al inhibir la LHRH, y que la noradrenalina (NA) modula los receptores de
progesterona situados en los núcleos septales y en la amígdala (implicados en la termorregulación, conducta sexual y emociones).
Algunos autores han encontrado otros hallazgos como: Alteraciones en los niveles de prolactina, aumento de los niveles de
prostaglandinas (PG), disminución de los niveles de PG, disminución de los niveles de Piridoxina, B) Hipótesis psicológicas: El SPM es
más frecuente en mujeres con un elevado nivel de neuroticismo, que a su vez muestran un pobre ajuste psicológico y una gran
sensibilidad a la frustración. Dada la frecuencia de síntomas afectivos (disforia, ansiedad e irritabilidad) y la ciclicidad inherente en su
definición, se ha hecho hincapié en la relación de este cuadro y de otros
trastornos psiquiátricos (ciclotimia y distimia). En este sentido, se han dado
tres teorías básicas que intentan relacionarlos: 1.- Episodios afectivos
tempranos, anteriores a la menarquía, que determinarían las características
de los cambios del humor posteriores (menstruales). 2.- Cambios en el
humor menstrual pueden servir como desencadenante de otras patologías
afectivas mayores en personas predispuestas genéticamente. 3.- la teoría de la "indefensión aprendida”. DIAGNOSTICO: Las
pacientes informan 1 ó más de los siguientes síntomas afectivos o somáticos durante los 5 días anteriores a la menstruación durante los
últimos 3 ciclos. Los síntomas mejoran dentro de los 4 días de ocurrida la menstruación, sin recurrencia por lo menos hasta el día 13 del
ciclo. Irritabilidad (85%), angustia (83%) y labilidad emocional (77%) son los síntomas más reportados. Según Sampson (1989) deben

CURSO ENARM CMN SIGLO XXI TEL: 36246001 Pharmed Solutions Institute PÁGINA 340
MANUAL DE TRABAJO DEL CURSO ENARM CMN SIGLO XXI
cumplirse algunas características para su diagnóstico: Periodicidad de los síntomas, aumento de la gravedad en la medida que el ciclo
progresa, existencia de un período postmenstrual de 7 días, como mínimo, libre de síntomas, alivio al inicio de la menstruación y
desaparición al cabo de 3 días posteriores al comienzo, presencia de los síntomas durante 3 ciclos consecutivos como mínimo, con
posibles variaciones de intensidad, interferencia de los síntomas con el trabajo, las actividades sociales y las relaciones interpersonales.
Criterios diagnósticos del SPM o del Trastorno Disfórico Premenstrual (son criterios de investigación)del DSM: A) Cinco (o más) de los
siguientes síntomas durante la mayor parte del día de la última semana de la fase lútea de la mayoría de los ciclos menstruales del
último año, que empiezan a remitir 2 días después del inicio de la fase folicular y que desaparecen completamente en la semana
siguiente a la menstruación, teniendo en cuenta que al menos uno de estos síntomas debe ser alguno de los cuatro primeros: 1) estado
de ánimo deprimido, sentimientos de desesperanza e ideas de autodesaprobación acusadas; 2) ansiedad, tensión, sensación de agobio
o de estar "al limite"; 3) labilidad emocional evidente; 4) enfado, irritabilidad o aumento de los conflictos interpersonales de forma
acusada y persistente; 5) pérdida del interés por las actividades cotidianas; 6) sensación subjetiva de dificultad de concentrarse; 7)
letargia, fatigabilidad fácil o falta evidente de energía; 8) cambios significativos del apetito, atracones o antojos por determinadas
comidas; 9) hipersomnia o insomnio; 10) sensación subjetiva de estar rebasada o fuera de control; 11) otros síntomas físicos como
hipersensibilidad o aumento del tamaño mamario, dolores de cabeza, molestias articulares o musculares, sensación de hinchazón o
ganancia de peso. B) Estas alteraciones interfieren acusadamente con el trabajo, la escuela, las actividades sociales habituales o las
relaciones interpersonales. C) La alteración no representa una simple exarcebación de síntomas de otro trastorno. D) Los criterios A, B y
C deben ser corroborados por técnicas de valoración diaria y prospectiva de los síntomas en al menos dos ciclos sintomáticos
consecutivos. TRATAMIENTO: El objetivo principal del tratamiento es disminuir los síntomas de SPM o DDPM para que la paciente
pueda realizar una vida normal, lo cual incluye entre otras medidas: Modificaciones de los hábitos alimentarios (Nivel B),
vitaminoterapia, fármacos, psicoterapia (Nivel C). Educación: es importante que la mujer conozca la fisiología del ciclo menstrual para
comprender lo que le está ocurriendo. El ACOG recomienda los cambios en el estilo de vida como la 1ra línea de tratamiento. Dieta: Se
recomienda restringir el consumo de: Sal (3g/día) para reducir la retención de agua, azúcares refinados, alimentos con Metilxantinas
para no aumentar la ansiedad y la irritabilidad, tabaco y alcohol, carnes rojas, se recomienda incrementar la ingesta de: Carbohidratos
complejos, pescados y aves como fuente de proteínas, fibras, vegetales, legumbres y cereales. Ejercicio físico: Aeróbicos: caminata,
trote, natación, remo, ciclismo y danza (nivel C), meditación, yoga. Es controvertido y debe reservarse a las pacientes que no mejoran
con las medidas mencionadas. Vitaminoterapia: Vitamina B6, la administración diaria de 50 ó 100 mg podría tener efectos beneficiosos
sobre los síntomas depresivos.(nivel B). Vitamina E: 400 a 600 UI/día. Mineraloterapia: El calcio y el magnesio son cofactores en la
síntesis de neurotransmisores, lo que sugiere una posible asociación entre SPM y la deficiencia de serotonina (nivel B). AINE Mejoran la
mastalgia, la dismenorrea, la migraña y los dolores musculares. Anticonceptivos Hormonales: Se ha sugerido un efecto beneficioso
sobre el bienestar psicológico y se los ofrece como una buena alternativa para el tratamiento (nivel B). FDA aprobó un esquema
terapéutico de ACO con bajas dosis de EE y DRSP con 24 comprimidos activos y 4 placebos para los síntomas físicos y emocionales,
mediantela reducción de la fluctuación hormonal a lo largo del ciclo y las propiedades antiandrogénicas y antimineralocorticoideas.
Joffe y colaboradores informaron que los ACO no modifican los cambios de humor premenstrual, pero, en aquellas mujeres con
antecedentes de depresión, tienen un gran riesgo de empeorar estos síntomas. Agonistas de GnRH: Producen alivio significativo de las
manifestaciones de la enfermedad, pero inducen síntomas menopáusicos que limitan su uso en la mayoría de las pacientes (nivel B).
Inhibidores de las Gonadotrofinas: El danazol parece ser efectivo en el tratamiento del SPM, especialmente en el alivio de la mastalgia
(por acción directa) y de la irritabilidad. No se recomienda en pacientes con depresión y ansiedad. La dosis propuesta es de 200 mg dos
veces por semana, sólo en la fase lútea. Gestágenos: No se pudo demostrar su efectividad para el tratamiento del SPM, no se
recomienda su uso (nivel A). Diuréticos: Las candidatas para el uso de diuréticos son aquellas pacientes que manifiestan aumento de
1.5 kg de peso durante el período premenstrual, tensión mamaria y otros síntomas de retención hídrica. La espironolactona es el
diurético recomendado, ya que es un agonista de la aldosterona. La dosis es de 25 mg, 2-3 veces por día, durante los días 18 al 26 del
ciclo. Inhibidores Selectivos de la Recaptación de Serotonina (ISRS): Primera línea para el tratamiento del DDPM, pues son muy eficaces
para el alivio de los síntomas físicos y emocionales (nivel A).Fluoxetina, sertralina, citalopram, paroxetina) actúan sobre el humor, la
saciedad y la agresividad.Son eficaces cuando se usan en forma continua o sólo en fase premenstrual.

CASO CLINICO
Se trata de femenino de 21 años de edad la cual acude a consulta debido a que refiere mastalgia y sensación de plenitud mamaria,
irritabilidad, inestabilidad emocional, refiere que desde los 17 años se presento más intensos los síntomas, agrega dismenorrea
ocacional con rechazo al periodo menstrual, al interrogatorio usted observa irregularidades menstruales, la paciente presenta un IMC
de 18.

PREGUNTA
Cuál es la conducta más adecuada para este caso.

RESPUESTA
a.- La conducta mas adecuada es iniciar un esquema de contraceptivo oral.
b.- Iniciar un Inhibidor Selectivo de la Rescaptura de Serotonina.
c.- Indicar la calendarización de la sintomatología previa a la terapia.
d.- Derivar a la paciente al servicio de psiquiatría.

CASO CLINICO
Paciente de 39 años, madre de dos hijos, primer parto con cesárea por falta de dilatación, el segundo con fórceps. Sus molestias
habían comenzado siete años antes, en el período premenstrual, manifestándose como dolores de cabeza. Experimentaron con
el tiempo progresiva prolongación e intensificación. Por la jaqueca consultó a neurólogo, quien pidió un electroencefalograma, que
resultó normal. Le fueron recetados analgésicos. Se presentaron molestias mamarias que se hicieron francamente severas y
duraban casi todo el mes. Tuvo diagnóstico de Mastopatía Fibrosa Bilateral de grado moderado. Fue tratada con un derivado

CURSO ENARM CMN SIGLO XXI TEL: 36246001 Pharmed Solutions Institute PÁGINA 341
MANUAL DE TRABAJO DEL CURSO ENARM CMN SIGLO XXI
sintético de la progesterona durante diez días cada mes; hubo alivio parcial de las molestias mamarias. Por otra parte, se le
diagnosticó "úlcera al cuello del útero", lo que se trató con dos cauterizaciones, sin proporcionar ningún beneficio a nivel general. La
paciente manifestaba sufrir molestias en las relaciones sexuales y además frigidez, razón por la cual procuraba evitarlas. Sus
molestias generales persistían y los síntomas mamarios se acentuaron. Al exámen físico se observaba una paciente decaída, palidez de
piel y mucosas, lengua sucia, conjuntivas irritadas, mirada triste. Además distensión abdominal, edema (hinchazón) de manos y
párpados, piel y cabello secos. Las mamas presentaban aumento de volumen, congestión, sensibilidad, ardor y aumento de la red
venosa.

PREGUNTA
Cual es la ducta a seguir mas adecuada?

RESPUESTA
a.- Acido mefenamico.
b.- Ibuprofeson.
c.- Progesterona
d.- Citalopram

PSICOSIS POSPARTO. CIENCIAS BASICAS: La psicosis posparto o puereral se define como un episodio de manía o de psicosis precipitado
por el nacimiento de un bebé: “Es una enfermedad severa cuyas características clínicas incluyen todas las variantes de la manía, psicosis
cicloides o esquizofreniformes.” Se caracteriza por pérdida de contacto con la realidad, agitación, confusión, alucinaciones vividas,
delirios paranoides y comportamiento violento. Muy excepcionalmente se ha descrito suicidio e infanticidio. SALUD PUBLICA: El riesgo
general es de 1-2 casos por cada 1.000 nuevos nacimientos. Se suelen describir algunos factores de riesgo. Así, se ha comprobado que
el 50-60% de los casos ocurren en primíparas, y que en el 50% de los casos se asocian a complicaciones psiquiátricas perinatales. En
cuanto a los antecedentes familiares, un 50-60% de las mujeres afectadas tienen antecedentes familiares con trastornos afectivos. El
riesgo está muy aumentado en las mujeres que tienen antecedentes personales de trastornos del humor unipolares o bipolares
(especialmente un trastorno bipolar I). En estos pacientes el riesgo pasa de 2/1000 al 20 a 25%. Las enfermedades psiquiátricas del
posparto son 5 veces más frecuentes que las del embarazo, de ahí la hipótesis del papel protector del embarazo frente a las
enfermedades mentales graves. Esta patología psiquiátrica del posparto debuta en la primera semana que sigue al parto en el 40% de
los casos, y en el primer mes postparto en el 80% de los casos. PATOGENIA: Los factores que pueden influir en el inicio de un cuadro de
las características que nos ocupa se pueden agrupar en: 1.- Factores heredo-constitucionales: Se insiste en el papel del "terreno",
siendo el grupo de familia con historia de bipolaridad el que ofrece las relaciones genéticas más significativas. Cuando una mujer ha
presentado crisis maníaco depresivas, existe un elevado porcentaje de riesgo de recaída después del parto. 2.- Factores biológicos: Los
trastornos endocrinos que aparecen antes, durante e inmediatamente después del parto son de una amplitud y rapidez únicas. En
efecto, en los días siguientes al parto las concentraciones plasmáticas de estrógenos y progesterona caen de manera considerable para
alcanzar tasas 100 veces más bajas que aquellas que existen en el preparto. 3.- Factores psicodinámicos: autores hacen intervenir el
rechazo del niño como uno de los factores esenciales de las melancolías puerperales: rechazo consciente (niño no deseado) o más a
menudo inconsciente, que emana de la ambivalencia profunda que marca la relación madre e hijo, "rechazo combatido tanto por la
culpabilidad como por el amor maternal. Entre los factores psicosociales figura, en primer término, el conflicto provocado por la
aparición de una tercera persona en el grupo familiar. La madre tiene que asumir ahora una mayor responsabilidad. Tiene que dejar de
ser hija para ser ella, a su vez, madre. DIAGNOSTICO: Clinico; Las psicosis puerperales se caracterizan por depresión e ideas delirantes
que aparecen después del parto. El estado de ánimo puede estar elevado o deprimido y con frecuencia es lábil; puede también haber
agitación, aumento de la actividad e insomnio. La enferma puede ver al recién nacido como la personificación del diablo o como el
Mesías; también pueden existir alucinaciones auditivas que pueden ser órdenes, en ocasiones para que la paciente mate al recién
nacido. Pueden producirse infanticidios.
El cuadro clínico tiene unas
características específicas: a) Rápido
cambio de sintomatología, b) Labilidad
de humor, c) Signos confusionales. Los
síntomas iniciales más frecuentes son
ansiedad, cansancio, humor depresivo,
disturbios del sueño, alteraciones del
comportamiento, agitación catatónica,
ilusiones y alucinaciones. El comienzo
ocurre en un período muy poco tiempo
después del parto y existe una serie de
precisiones:Deben ser excluidas las
psicosis asociadas con eclampsia,
infección, deprivación de alcohol o con
problemas del puerperio. Los pacientes con trastornos delirantes monosintomáticos y con psicosis psicógena deben ser excluidos. Si se
decide incluir como tal a las psicosis depresivas, debieran restringirse a aquellas que presentan delirios, alucinaciones, confusión,
estupor, catatonía o síntomas maniformes transitorios. El intervalo entre el parto y la aparición de la psicosis puede ser de 14 días, un
mes o tres meses. No es necesario que la madre esté libre de patología psiquiátrica durante el embarazo. El DSM−IV no contiene
criterios diagnósticos específicos para este trastorno. El diagnóstico puede hacerse cuando la psicosis ocurre en estrecha relación
temporal con el nacimiento del niño. Los síntomas característicos son ideas delirantes, déficits cognoscitivos, alteraciones motoras,
alteraciones del estado de ánimo y, de forma ocasional, alucinaciones. El contenido del material psicótico hace referencia al embarazo y
la maternidad. TRATAMIENTO: Los tratamientos farmacológicos son comparables a aquellos utilizados para los síndromes similares que

CURSO ENARM CMN SIGLO XXI TEL: 36246001 Pharmed Solutions Institute PÁGINA 342
MANUAL DE TRABAJO DEL CURSO ENARM CMN SIGLO XXI
aparecen fuera del contexto del posparto. Al comienzo, es necesario un tratamiento farmacológico enérgico, y en la mayoría de los
casos, se recomienda la hospitalización. Dependiendo de cada mujer, suelen prescribirse antipsicóticos y estabilizadores del ánimo. Por
otra parte, se tiende a utilizar benzodiacepinas para controlar los episodios de agitación y de ansiedad. Si la paciente no responde
favorablemente a la farmacoterapia o los síntomas van en aumento, puede ser necesaria la terapia electro convulsiva. Un estudio
demuestro que las fenotiazinas son más utilizadas en las mujeres que presentan una psicosis del posparto. La psicoterapia debe estar
centrada en las áreas particularmente conflictivas. Hay que ayudar a la paciente a aceptar su rol femenino y la rabia y los celos que
surgen a raíz de la dependencia que tiene respecto de su madre. Va de suyo la necesidad de tener en cuenta al grupo familiar entero.
También puede estar indicado el introducir algunos cambios en la estructura del hogar.

CASO CLINICO
Femenino de 28 años, casada desde hacía cinco años, estaba esperando el nacimiento de su primer hijo. Nunca había tenido ningún
problema emocional particular y había estado siempre físicamente bien. El parto de una niña de 3,2 kg, saludable y sin complicaciones,
dejó cansada pero feliz durante los tres días siguientes a la paciente. Dejó el hospital con ánimos y todo fue bien durante la primera
semana en casa. Repentinamente, se puso ansiosa y no era capaz de estar quieta. Empezó a pensar que alguien intentaba hacer daño al
bebé. Reconocía que era ridículo pero no podía parar de pensar que alguien estaba intentando envenenar al bebé o entrar en la casa.
Empezó a preocupar al marido al decir cosas extrañas, como que habían tenido niños gemelos, en vez de una única niña. Algunas veces
se encontraba normal, mientras que otras, estaba inquieta y preocupada con sus ideas extrañas. Durante la semana siguiente se
encontró tan mal que no pudo hacerse cargo del bebé y en ocasiones, no podía ni levantarse por las mañanas. Se sentía culpable de
“todo”, no podía dormir por las noches y no tenía apetito. Con frecuencia, estaba confundida, no sabiendo dónde estaba o lo que le
estaba pasando.

PREGUNTA
Cual es la conducta farmacológica mas adecuada?

RESPUESTA
a.- Clorimipramina.
b.- Risperidona.
c.- Clonacepam.
d.- Valproato de sodio.

CASO CLINICO
Femenino de 27 años, Un mes antes de la internación abandonó a sus hijos y a su marido porque no los reconocía como propios. El
esposo refiere que notó 30 días después del parto que la paciente comenzaba a desatender a sus hijos, hablaba “en lenguas extrañas”,
no dormía, se enojaba con facilidad y cantaba a los gritos durante todo el día cánticos religiosos. En la misma época comienzan las
huidas del hogar. Examen al ingreso: Desconfiada, alucinada, “escucha la voz de un hombre que dice cosas feas”. Refiere sentir dolores
de cabeza y de vientre, sensación que vivencia como una agresión de ese mismo hombre. Reticente. Pueril. Con conductas impulsivas
de huida. Durante la internación la paciente está excitada, con desorientación y falsos reconocimientos. Por momentos permanece sin
ningún contacto con el entorno y al rato está atenta y dispuesta al diálogo, persisten las alucinaciones. Logorreica, con logoclonías,
responde a todo estímulo del ambiente, lee láminas de las paredes, toca historias clínicas, quita hojas de las plantas, intensa actividad
motora sin objeto. Insomne, canta o grita de día y de noche. La afectividad está polarizada hacia el placer con estados anímicos
fluctuantes. Escaso capital ideativo. Ocurrencias delirantes polimorfas, místicas, eróticas y megalómanas. Cree ser la Virgen María, tiene
falsos reconocimientos con el personal del servicio médico.

PREGUNTA
Se le ingreso e inicio tratamiento antipsicoticco, a las 48 años refiere acatisia y rigidez, cual es la conducta ha seguir mas adecuada?

RESPUESTA
a.- Clonacepam.
b.- Trihexifenidilo.
c.- Biperiden.
d.- Difenidramina.

DEPRESION POSTPARTO (DPP). CIENCIAS BASICAS: Dentro de la nomenclatura psiquiátrica, la DPP es definida rigurosamente como un
trastorno/episodio depresivo de inicio en el postparto. En el Manual Diagnóstico y Estadístico de los Trastornos Mentales (DSM IV-TR)
la clasificación es la siguiente: F32.x Trastorno depresivo mayor, episodio único (296.2x). SALUD PUBLICA: Existen diferentes
investigaciones a nivel mundial sobre la prevalencia de este padecimiento, las cuales fluctúan entre el 6 y 34%. En nuestro país, los
estudios reportan una prevalencia que va desde 13.9 hasta 32.6%. A pesar de esta situación, se podría hablar de que cerca del 20% de
las mujeres que dan a luz pueden presentar síntomas de DPP. En cuanto a la recurrencia, existe una probabilidad del 10 al 35%.
PATOGENIA: Hasta el momento, no se conocen las causas de la depresión postparto (DPP); únicamente se han podido observar
factores de riesgo, este padecimiento se puede presentar incluso cuando la madre no ha mostrado ningún antecedente. Factores de
riesgo: Historia pasada de psicopatología (especialmente depresión y ansiedad), presencia de alteraciones psicológicas durante el
embarazo, DPP previa, complicaciones obstétricas durante el embarazo, el parto o después del mismo (principalmente embarazos de
alto riesgo), embarazo no planeado y/o deseado, falta de apoyo familiar y/o social, problemas familiares y/o sociales, pobre relación
marital, abandono o separación de la pareja, no contar con un compañero, pareja presente, pero que no proporcione apoyo emocional,
falta de redes de apoyo, situaciones estresantes, dificultades económicas, abuso físico, emocional o sexual en la infancia, fallecimiento,

CURSO ENARM CMN SIGLO XXI TEL: 36246001 Pharmed Solutions Institute PÁGINA 343
MANUAL DE TRABAJO DEL CURSO ENARM CMN SIGLO XXI
abandono o separación de un familiar o ser querido, mala relación con la madre, antecedente de pérdidas perinatales, sensibilidad a los
cambios hormonales, alteración en la función tiroidea, dificultades en la lactancia, mala salud del recién nacido, bebé difícil de cuidar.
Suele comenzar en cualquier momento posterior al parto, ya sea días, semanas, meses y hasta un año después. Puede afectar en
cualquier tipo de parto o número de embarazo. DIAGNOSTICO: En relación a la intervención, el primer paso es detectarla,
especialmente en sus inicios, cuando se perciban síntomas o se presenten signos de alarma, como quejas psicosomáticas, cansancio,
dolores, múltiples llamadas para pedir ayuda, o se observe que la mujer está triste, preocupada o con miedo. A continuación, es
necesario canalizarla con un especialista (psicólogo, terapeuta o psiquiatra) para que sea evaluada y reciba el tratamiento adecuado,
buscando el apoyo multidisciplinario y una atención integral. Síntomas depresión posparto: Estado de ánimo deprimido, tristeza o
llanto persistentes, disminución del interés o de la capacidad para disfrutar de casi todas las actividades, cambios en los patrones de
alimentación y sueño (principalmente insomnio), cansancio o fatiga crónica, ansiedad que puede llegar hasta los ataques de pánico,
sentirse abrumada, indefensa e incapaz, autodesvalorización y autorreproches, generalmente relacionados a su competencia como
madre, dificultad, reticencia o imposibilidad de buscar ayuda y apoyo para sí misma, irritabilidad, problemas de atención, concentración
y memoria, dificultad para vincularse con el bebé, sentimientos de ambivalencia hacia el hijo, sobreprotección del menor, dificultades
en la lactancia, dificultad y/o imposibilidad de cuidar al recién nacido, pensamientos obsesivos y conductas compulsivas, relacionados
con el hijo (p. ej. revisarlo constantemente cuando está dormido para ver si respira) y con su cuidado (p. ej. lavar y desinfectar varias
veces sus artículos), pensamientos negativos hacia el bebé, especialmente creer que lo puede dañar, sentimientos de culpa,
pensamientos de dañarse a sí misma o al bebé, pensamientos atemorizantes, ideación suicida. TRATAMIENTO: El tratamiento tiene dos
opciones de intervención terapéutica: 1) terapia (individual o de grupo) y/o 2) medicación. En aquellos casos en que se presenta una
depresión de moderada a grave, lo ideal es combinar ambas. La psicoterapia, ya sea interpersonal o con un enfoque cognitivo-
conductual, debe basarse en los siguientes aspectos: a) Educación e información a la mujer, la pareja y los familiares sobre la DPP. b)
Control, disminución y extinción de los síntomas. c) El nuevo rol como madre, las dificultades, mie dos o angustias que esto puede
generar. d) Uso de estrategias para proteger, mejorar o fortalecer el vínculo madre-hijo, así como la relación de pareja. En esta última,
se buscará una adecuada comunicación, así como tener espacios de calidad sin el bebé. e) Promover un buen funcionamiento familiar,
especialmente cuando existan otros hijos. f) Disminución de las situaciones estresantes actuales. g) Búsqueda y fortalecimiento de
redes de apoyo (familiares, sociales e institucionales). Tratamiento medico: En cuanto al tratamiento psiquiátrico, dependiendo de cada
caso, los medicamentos más usados son los antidepresivos tricíclicos y los inhibidores selectivos de la recaptación de serotonina (ISRS),
para combatir los síntomas depresivos. Por otra parte, se tiende a utilizar benzodiacepinas para tratar la ansiedad y el insomnio. Si la
mujer tiene una historia previa de DPP, lo más recomendable es iniciar la medicación inmediatamente después del parto, para prevenir
una posible recaida. Cuando la mujer con DPP no recibe tratamiento, puede presentar dificultades y estrés para cuidar al recién nacido,
lo que tiende a generar problemas en el vínculo materno-infantil. Dichos bebés pueden llegar a presentar retraso en el desarrollo
psicomotor, temperamento difícil o irritabilidad, así como mostrar un apego inseguro. Así mismo, se ha observado que algunos no son
tan atentos y activos, y sonríen menos, lo que puede generar déficits en el área social y cognitiva. Con una intervención adecuada, la
DPP es temporal y se puede alcanzar una completa recuperación.

CASO CLINICO
Paciente femenina de 36 años de edad, con 5 dia de puerperio, parto normal, producto a término, primer hijo, madre soltera, que vive
con su propia madre únicamente, la cual es divorciada desde hace 20 años. La madre de la paciente la lleva a consulta por presentar
conductas extrañas, refiere que no se ha bañado desde que salio del hospital, la observa apática, callada, poco interesada en el hijo, y
finalmente decidio ir a consulta al escuchar a su hija decir “tal vez estaríamos mejor si no estubieramos” a la EF se observa paciente
desalineada, desaseada, poco cooperadora, desinteresada, niega ideas suicidas ni homicidas pero acepta haber dicho lo que la madre
refiere, se apresia melancolica mas que ansiosa.

PREGUNTA
Cual es la conducta a seguir mas adecuada?

RESPUESTA
a.- Ingreso psiquiátrico involuntario.
b.- Envio a consulta externa de psiquiatría.
c.- Imipramina 50 mg cada 8 hrs.
d.- Haloperidol 5 mg cada 8 hrs.

PREGUNTA
Cual de los siguientes antidepresivos es el mas adecuado considerando la sintomatología del caso?

RESPUESTA
a.- Velanfaxina.
b.- Paroxetina.
c.- Escitalopram.
d.- Fluvoxamina.

CURSO ENARM CMN SIGLO XXI TEL: 36246001 Pharmed Solutions Institute PÁGINA 344
MANUAL DE TRABAJO DEL CURSO ENARM CMN SIGLO XXI
PEDIATRIA

1) SX DE ADAPTACION PULMONAR, SX DE DIFICULTAD RESPIRATORIA (EMH), TAQUIPNEA TRANSITORIA DEL RN,


2) ASPIRACION DE MECONIO, NEUMONIA NEONATAL.
3) SEPSIS NEONATAL, ENTEROCOLITIS NECOTRIZANTE, MENINGITIS NEONATAL.
4) HIPOGLUCEMIA, ICTERICIA, KERNICTERUS.
5) ATRESIA ESOFAGICA, FISTULA TRAQUEOESOFAGICA, HERNIA HIATAL, ESTENOSIS PILORICA.
6) CARDIOPATIAS CONGENITAS Y ADQUIRIDAS
7) KWASHIORKOR, MARASMO, AVITAMINOSIS,
8) OBESIDAD, VACUNACION.
9) DERMATITIS, PENFIGO, MOLUSCO CONTAGIOSO, TIÑA CORPORIS
10) HIPERQUERATOSIS PALMOPLANTAR, FOLICULITIS, PEDICULOSIS.
11) PURPURA DE SCHÖNLEIN-HENOCH, ANEMIAS,
12) LEUCEMIAS, RETINOBLASTOMA, TUMORES DEL SNC.
13) PAROTIDITIS, HEPATITIS, INFLUENZA, MENINGITIS
14) OTITIS, SINUSITIS, FARINGITIS, AMIGDALITIS
15) EPIGLOTITIS, LARINGOTRAQUEITIS, BRONQUITIS.
16) BRONQUIOLITIS, BRONCONEUMONIA, NEUMONIAS
17) ALERGIAS, ASMA, FIBROSIS QUISTICA.
18) URETRITIS, CISTITIS, PIELONEFRITIS.
19) GLOMERULO NEFRITIS, SINDROME NEFRITICO, SINDROME NEFROTICO.
20) GASTROENTERITIS, PARASITOSIS.
21) DESHIDRATACION, LIQUIDOS Y ELECTROLITOS
22) QUEMADURAS, ACCIDENTES
23) PICADURAS, MORDEDURAS
24) INTOXICACIONES, ENVENENAMIENTOS
25) TRAUMATISMO CRANEOENCEFALICO, CEFALEA Y EPILEPSIAS
26) ENURESIS, ENCOPRESIS, TDAH, TRASTORNOS DEL DESARROLLO.

CURSO ENARM CMN SIGLO XXI TEL: 36246001 Pharmed Solutions Institute PÁGINA 345
MANUAL DE TRABAJO DEL CURSO ENARM CMN SIGLO XXI
SINDROME DE ADPATACION PULMONAR (SAP): CIENCIAS BASICAS: Los cambios que implican adaptarse a la nueva vida se producen
de manera simultánea desde el nacimiento con la primera respiración y terminan aproximadamente a las 15 hrs de vida,
denominándose período de transición. Involucra cambios cardiorrespiratorios, metabólicos, neurológicos y hormonales. Los
componentes del período de transición son: Esfuerzo respiratorio inicial. Aumento unas cinco veces en el flujo linfático pulmonar.
Depuración del líquido pulmonar desde la vía aérea a los canales linfáticos y vasculares pulmonares. Establecimiento de un área de
superficie pulmonar estable. Reducción de los altos niveles de resistencia vascular pulmonar. Aumento del flujo sanguíneo pulmonar.
Aumento de la presión de oxígeno arterial. Cierre del conducto arterioso. La eliminación del líquido pulmonar se lleva a cabo en 2-6hrs.
PRIMER PERÍODO DE REACTIVIDAD: 30-60 minutos de vida. Movimientos de succión y deglución. Temblores finos en extremidades y
mandíbula. Cierre y apertura de los párpados. Movimientos espasmódicos rápidos y breves de los globos oculares. Reflejo espontáneo
de Moro. Llanto de inicio y detención súbita. Cianosis leve y breve. Enrojecimiento con el llanto. Abundantes secreciones en vías aéreas.
FR 100x´, FC 200´, quejidos inspiratorios. Estertores bilaterales. TA elevada. Peristalsis ausente. Hipotermia. PERÍODO DE SUEÑO O
TRANQUILIDAD: Después de los 30-60 minutos, dura hasta las 2 horas de vida. Puede permanecer dormido, pero responde a estímulos
en forma brusca. FR es rápida y superficial. Desaparecen los quejidos inspiratorios y estertores. FC regular 120-140x´, desciende 100 por
breves períodos. TA se normaliza. Aparece la peristalsis. Se observan ondas peristálticas. Temblores y sacudidas espontáneas.
SEGUNDO PERÍODO DE REACTIVIDAD: Ocurre entre las 2-6hrs de vida, es breve o persiste por 15hrs. Se reanuda la actividad motora,
exagerada. Inicia nuevamente taquicardia, polipnea. Respiraciones periódicas. Secreciones abundantes. Reflejo nauseoso, vómitos,
meconio. Muy sensible estímulos externos. Inestabilidad vasomotora. Causas que alteran la eliminación del líquido pulmonar:
Nacimiento por cesárea, parto inducido, prematurez, nacimiento sin trabajo de parto, asfixia, drogas en las madres, hijo de madre
diabética. Causas que alteran la adaptación pulmonar: Hipoxia y acidosis, hipotermia, período expulsivo prolongado, RN con dificultad
respiratoria, RN de término o pretérmino limítrofe, acrocianosis, elevación de la FC, remite entre las 8 y 12 hrs. DIAGNOTICO: Rx de
Tórax es normal o con leve aumento de la trama vascular, los gases sanguíneos con acidosis respiratoria e hipoxia leves. Diagnóstico
diferencial: Taquipnea Transitoria del RN. Enfermedad de la Membrana Hialina. Alteraciones metabólicas. Alteraciones de la
temperatura. Anemia. TRATAMIENTO: Ambiente térmico neutro para reducir el consumo de O2. Aspiración de secreciones.
Oxigenoterapia: mantener la PaO2 entre 50-80mmHg. Vía oral en cuanto sea posible, si no, iniciar LIV con glucosada al 10% a
requerimientos.

CASO CLINICO
Se trata de paciente masculino RN de 60 minutos, obtenido por cesarea iterativa, demás de presentar sufrimiento fetal, por
disminución de actividad intrauterina y disminuciones de FC de hasta 110 lpm, se observa con movimientos muy finos, al inicio
presento cianosis leve y breve principalmente periungeal y oral, se apresiaron secresiones orales abundantes, a la exploración física se
auscultan estertores bilaterales normales, peristaltismo poco audible y temperatura de 36.1 grado, se apresió expulsión de secresiones
con tinte meconial durante la exploración.

PREGUNTA
Considerando el cuadro clínico cual es su conducta a seguir?

RESPUESTA
a.- Mantener al paciente en observación continua.
b.- Buscar psobles patologías ocultas.
c.- Realizar radiografia de torax.
d.- No hay datos patológicos son adaptativo.

PREGUNTA
Cual es la conducta terapéutica menos apropiada en este momento del caso?

RESPUESTA
a.- Mantener ayuno hasta identificar causa.
b.- Colocar en ambiente neutro.
c.- Administrar oxigeno por casco cefálico 40 %.
d.- Gasometria y oximetría de pulso.

PREGUNTA
Cual de los siguientes parámetros gasométrico no es normal esperarlo en el caso?

RESPUESTA
a.- pH < 7.34.
b.- PaCO > 45 mmHg.
c.- HCO3 > 40mEq/L.
d.- SpO2 < 95.

PREGUNTA
Se realizaron electrolitos sericos por conituar con rechazo al alimento y secresion moderada oral, todos los resultados resultaron dentro
de parámetro normales, cual de las siguientes diagnosticos diferencias del acidosis respiratoria es el mas probable presentarse?

RESPUESTA
a.- Ventilación inadecuada alveolar.

CURSO ENARM CMN SIGLO XXI TEL: 36246001 Pharmed Solutions Institute PÁGINA 346
MANUAL DE TRABAJO DEL CURSO ENARM CMN SIGLO XXI
b.- Desordenes musculares.
c.- Defectos pulmonares.
d.- Trastornos de las vías aéreas.

PREGUNTA
Considerando los valores gasométricos y de eletrolitos, cual es la causa de la modificación del anio gap, del caso?

RESPUESTA
a.- Error de laboratorio.
b.- Toxinas no identificadas.
c.- Mielomas.
d.- Exceso de soluciones.

PREGUNTA
El paciente presenta persistencia de los síntomas por 3 horas más, se reporta que incrementa con la alimentación, con dificultad
continua succionando, usted considera realizar laboratorios, cual de los siguientes resultados es el menos probable encontrar en este
caso?

RESPUESTA
a.- Aumento de la trama vascular.
b.- Presencia de infiltrados pulmonares.
c.- Acidosis respiratoria.
d.- Hipoxemia leve.

CURSO ENARM CMN SIGLO XXI TEL: 36246001 Pharmed Solutions Institute PÁGINA 347
MANUAL DE TRABAJO DEL CURSO ENARM CMN SIGLO XXI
SINDROME DE DIFICULTAD RESPIRATORIA (ENFERMEDAD DE MEMBRANA HIALINA). CIENCIAS BASICAS: El síndrome de dificultad
respiratoria por deficiencia de surfactante es la principal patología respiratoria entre los recién nacidos; ocupa un papel preponderante
por su alta morbimortalidad. Ha recibido diversos nombres a través del tiempo siendo los más com nmente reportados, “síndrome de
insuficiencia respiratoria idiomática”, “síndrome de hipoperfusión pulmonar” y “síndrome de microatelectasias m ltiples”. El término
de síndrome de dificultad respiratoria tipo I, se otorgó por las alteraciones radiológicas presentadas en esta patología, el Doctor Swis-
chuk reportaba en las radiografías de esta patología pequeñas burbujas de tipo esférico de 1 a 1.5 mm. de diámetro a las cuales
denomino burbujas tipo I, en otras patologías encontró a nivel radiográfico burbujas de mayor tamaño a las que denomino burbujas
tipo II y otras de tamaño mucho mayor (quísticas) las denomino tipo III. El término más aceptado actualmente es el de síndrome de
dificultad respiratoria por deficiencia de surfactante, ya que define sus principales características fisiopatológicas como la congestión
difusa pulmonar, notoria disminución de la distensibilidad pulmonar y la presencia de tejido necrótico y membranas de aspecto hialino
en bronquiolos y alvéolos. SALUD PUBLICA: Este síndrome continúa siendo una de las primeras causas de ingreso a las unidades de
terapia intensiva neonatal a nivel mundial. La incidencia exacta de esta patología es difícil de precisar. Las tendencias actuales reportan
incidencia de 60 a 80% en los recién nacidos menores de 30 semanas de gestación, un 15 a 30% de los de 32 a 36 semanas de gestación
y solo 5% de los que tienen 37 semanas o más de gestación. La frecuencia también aumenta cuando son hijos de madre diabética (de
evolución corta la diabetes materna) en asfixia perinatal, y algunos casos en que por error en la determinación de edad gestacional, se
realiza operación cesárea antes de lo debido. Discreta tendencia a ser más frecuente en recién nacidos de sexo masculino que en sexo
femenino. PATOGENIA: En el síndrome de dificultad respiratoria, la frecuencia respiratoria se encuentra elevada por lo que a pesar del
volumen corriente (Vt) disminuido, la ventilación minuto inicialmente esta incrementada. Debido a la deficiencia o disminución en la
cantidad o calidad del surfactante pulmonar la mayor parte del pulmón está colapsado o poco ventilado y la mayor parte de la
ventilación alveolar se deriva a una región muy pequeña del pulmón lo que conlleva a una disminución de la capacidad residual
funcional (CRF). Asimismo la distensibilidad está muy disminuida, no tanto por el tórax del recién nacido que es fácilmente distensible,
sino que por los pulmones que con esta deficiencia de surfactante llegan a tener mediciones de la distensibilidad de 0.3-0.6
ml/cmH2O/kg en lugar de 2.0-2.5 ml/cmH2O /kg que es lo normal, lo cual traduce una distensibilidad menor al 30% de los normal. La
resistencia de la vía aérea es normal pero con tendencia a incrementarse hasta en 40-50% como resultado de todo esto el trabajo
respiratorio se verá incrementado en poco más del 50%. Edberg y colaboradores encontraron disminución de la distensibilidad,
incremento de la resistencia, reducción del volumen pulmonar y disminución en la eficacia para la mezcla de gases en los recién nacidos
prematuros con síndrome de dificultad respiratoria. A partir de estos datos y dado que la constante de tiempo (CT) depende de la
distensibilidad y la resistencia (CxR=CT) , se verá afectada y como consecuencia se presentará un inadecuado intercambio del volumen
alveolar. Esta disminución o acortamiento de la constante de tiempo no es uniforme en toda la vía respiratoria, se aprecia sobre todo
en las áreas más distales, por lo que , en un mismo pulmón habrá constantes de tiempo diferentes lo que lleva a una ventilación no
uniforme con riesgo de dañar a aquellas vías aéreas con constantes de tiempo normales, que son sometidas a la ventilación mecánica
necesaria para forzar a abrirse a las vías aéreas con constante de tiempo acortada. DIAGNOSTICO: Los signos pueden manifestarse
desde los primeros minutos de vida o después de algunas horas, y por lo general son de incremento gradual. Aunque en ocasiones
estos signos de dificultad respiratoria son menos marcados debido a la debilidad de la musculatura respiratoria, lo que los llevará
rápidamente a una falla respiratoria con hipoventilación y apna (conocido como respuesta paradójica a la hipoxemia). Los signos de
dificultad respiratoria se manifiestan con taquipnea, tiraje intercostal, retracción xifoidea, disociación toracoabdominal, aleteo nasal, y
quejido espiratorio, este último uno de los más frecuentes y es motivado por el cierre de la glotis en su afán de realizar un auto PEEP
(presión positiva al final de la espiración) para conservar los alvéolos abiertos y aumentar el volumen residual pulmonar para un
adecuado intercambio gaseoso. A la auscultación de campos pulmonares encontraremos disminución del murmullo vesicular
habitualmente en forma bilateral. Además de los signos de dificultad respiratoria se puede presentar cianosis central que obligará al
uso de oxigenoterapia en cualquiera de sus modalidades y que podrá variar desde los casos leves que solo requieran apoyo con oxígeno
en fase de casco cefálico o bien casos moderados y severos que requerirán CPAP (presión positiva continua en vías aéreas) o ventilación
mecánica en su diversas modalidades y según lo requiera cada paciente. Otro grupo de manifestaciones clínicas estará determinado por
los trastornos a nivel hemodinámico que se derivan de los cortos circuitos de derecha a izquierda así como por la repercusiones por la
acidosis y la hipoxemia, todo esto conllevará a trastornos de perfusión a todos los niveles con repercusiones y manifestaciones clínicas
de cada uno de los órganos afectados (falla renal, falla cardiaca, trastornos por hipoperfusión a nivel cerebral, intestinal, etc.). La misma
hipoxemia favorecerá incremento en las resistencias pulmonares y como consecuencia datos de hipertensión pulmonar con lo que se
agravará la hipoxemia y acidosis, y ameritará manejos más enérgicos para poder restituir la oxigenación adecuada de todos los tejidos.
La misma prematurez de estos pacientes favorece más complicaciones como la hipotermia que deberá ser corregida en forma
oportuna ya que de lo contrario generará acidosis y está a su vez vasoconstricción e hipoxemia. El diagnóstico se podrá sustentar en
base al cuadro clínico así como a los hallazgos a nivel laboratorial y radiográfico. Después del nacimiento, el surfactante pulmonar
puede evaluarse en el líquido amniótico, ya que parte del líquido pulmonar fetal entra en la bolsa amniótica y por lo tanto medirse la
lecitina de manera conjunta con la SP-C, pero con algunos cambios en la cantidad de líquido amniótico puede alterar la concentración
de la SP-C. Otra prueba de laboratorio que puede ser utilizada es la relación entre la lecitina y la esfingomielina que permanece
relativamente constante a lo largo de toda la gestación y se expresa como el índica L/S el cual se incrementa en forma lenta y gradual
de la primera a la semana 32 de gestación, el índice es de 2 hacia la semana 28 y de l hasta cerca del término de la gestación; la
incidencia de síndrome de dificultad respiratoria es sólo de 0.5% cuando el radio es de 2, pero es cercana al 100% si el radio es menor
de 1; entre 1 y 2 , el riesgo disminuye de modo progresivo. También resulta útil la medición de los niveles de fosfatidilinositol el cual
aumenta rápidamente en el líquido amniótico hasta la semana 36 y después disminuye, por medio de cromatografía se puede
determinar su porcentaje del total de los lípidos, si es menor de 1% se correlaciona con síndrome de dificultad respiratoria. Estas dos
últimas pruebas de laboratorio son los mejores predictores de esta patología con una correlación conjunta del 80%.También se han
realizado mediciones de la SP-A y SP-C pero sus resultados no son tan confiables. Por otra parte, sin que sean indicadores del síndrome
de dificultad respiratoria, se deberá determinar estudio gasométrico el cual nos mostrará diversos grados de acidosis, hipoxemia e
hipercapnia, los cuales nos indicaran respuesta al manejo instalado. El estudio por medio de radiografías de tórax nos reportara gran
utilidad, y nos mostrará un aumento difuso de la densidad en ambos campos pulmonares con una apariencia granular muy fina dando
la apariencia de un “vidrio esmerilado” o de “vidrio despulido” , así como también se aprecian a las vías aéreas con mayor densidad que

CURSO ENARM CMN SIGLO XXI TEL: 36246001 Pharmed Solutions Institute PÁGINA 348
MANUAL DE TRABAJO DEL CURSO ENARM CMN SIGLO XXI
los campos pulmonares produciendo una imagen de broncograma aéreo y los diafragmas se muestran habitualmente elevados , todos
estos cambios se deben a la pérdida de volumen pulmonar por colapso alveolar. TRATAMIENTO: Debe ser integral respetando siempre
la secuencia recomendada y altamente probada como es: pasos iniciales de la reanimación, ventilación, compresión torácica, así como
la intubación y uso de medicamentos. El soporte ventilatorio deberá ser oportuno y el necesario para cada paciente en particular y
podrá ser: Oxigenoterapia en fase I (casco cefálico, puntas nasales) con oxígeno a flujo libre y en las concentraciones necesarias de la
fracción inspirada de oxígeno (FiO2) para dar un soporte real al paciente y lograr una oxigenación tisular adecuada. Manejo con presión
positiva continua en vías aéreas (CPAP), donde la presión suministrada así como la fracción inspirada de oxígeno deberán ser
controladas para proporcionar un adecuado intercambio gaseoso y evitar en lo posible complicaciones. Ventilación mecánica en sus
diversas modalidades: Ventilación convencional, sincronizada, activada por el paciente, de volumen controlado, de ayuda proporcional,
de alta frecuencia. La finalidad del soporte ventilatorio es lograr un adecuado intercambio gaseoso reclutando los alvéolos colapsados
por el déficit de surfactante, y de esta manera mejorar la acidosis, la hipoxemia y la hipercapnea. Este reclutamiento alveolar se logra
manteniendo una presión positiva continua al final de la expiración. (PEEP) debido a que el llamado PEEP fisiológico es de 2, se
recomienda rangos por arriba de 4 cmH2O. El tratamiento de restitución de surfactante ha logrado disminuir la mortalidad hasta en un
50% de los casos del síndrome de dificultad respiratoria; en la actualidad hay dos grupos de surfactante aprobados por la FDA: 1.-
surfactante natural (se obtiene de fuentes humanas o animales). 2.- surfactante sintético. Las modalidades del tratamiento con
surfactante pulmonar exógeno incluyen la modalidad profiláctica (Admón. En UTQ (100 mg/kg), valorar de requrirse Administrar en
primera hr vida 100 mg/kg) y la de rescate (100-200 mg/kg si clínica y FIo2 >40%, Rx Compatible).

CASO CLINICO
RN masculino de edad gestacional de 39 semanas, de 3,8 kg hijo de madre diabética el cual se obtuvo por cesaria previa administración
de esterioides a la madre, el cual requirió de estimulación vigorosa cursando con hipotermia que mejoro en incubadora, se observo
posteriormente, aleteo nasal, retracción esternal, quejido respiratorio, cianosis y polipnea, la Rx mostro un patro retículo granular
difuso bilateral y disminución de la expansión pulmonar, PaO2 < 50 mmHg.

PREGUNTA
Cual es la conducta a seguir.

RESPUESTA
a.- Presión positivo nasal cotinua.
b.- Oximetria de pulso.
c.- Intubación traqueal.
d.- Ventilación mecánica.

CASO CLINICO
RN de 31 semanas que presenta dificultad respiratoria. Tiene 2 horas de nacido por parto vaginal. La dificultad respiratoria tiende a
incrementar. Madre diabética, salvo la edad gestacional no se identificaron otras complicaciones durante el embarazo. EF: Temp. 37.5
ºC, TA:86/58, FR:60/min, FC:148/min. Marcadamente taquipneico, con retracciones supraesternales y supraclaviculares, aleteo nasal,
quejido. Faringe normal. CP normal. Usted decide intubación endotraqueal, toma laboratorios y RX de tórax. RX de tórax: Infiltrado
bilateral, difuso, “vidrio despulido” en ambos pulmones, sin datos de atrapamiento de aire.

PREGUNTA
Cual de los siguientes resultados de la gasometría es menos frecuente observar?

RESPUESTA
a.- Hipoxemia.
b.- Hipercapnia.
c.- Acidosis metabólica.
d.- Acidosis respitoria.

CURSO ENARM CMN SIGLO XXI TEL: 36246001 Pharmed Solutions Institute PÁGINA 349
MANUAL DE TRABAJO DEL CURSO ENARM CMN SIGLO XXI
TAQUIPNEA TRANSITORIA DEL RECIEN NACIDO (TTRN): CIENCIAS BASICAS: Es un proceso respiratorio no infeccioso que se presenta
con más frecuencia en los recién nacidos de termino o cercanos a término, donde hay retención de líquido pulmonar. Se inicia en las
primeras horas y se caracteriza por la presencia de taquipnea (FR >60x´), insuficiencia respiratoria y aumento de requerimiento de
oxígeno, con niveles de CO2, normales o ligeramente aumentados. Es un proceso generalmente benigno y autolimitado que se resuelve
aprox. De 24-72hrs. Factores que favorecen la aparición de TTRN: maternos; asma, DM, tabaquismo, manejo abundante de líquidos,
sedación por tiempo prolongado, RPM >24hrs. Recién nacido; nacimiento por cesárea sin trabajo anterior, parto precipitado,
nacimiento cercano a término o termino , asfixia perinatal, sexo masculino, macrosomia, demoras en el pinzamiento del cordón
umbilical, sedación materna excesiva, embarazo múltiple. SALUD PUBLICA: Incidencia: 1 a 2% de los recién nacidos. Recién nacidos
término, cercanos a término, o prematuros grandes. La TTRN abarca del 35-50% de los casos de insuficiencia respiratoria no infecciosa
de los recién nacidos que ingresan a los servicios de neonatología. PATOGENIA: 3 factores: 1. Retraso en la resorción de líquido
pulmonar fetal 2. Inmadurez pulmonar 3. Deficiencia ligera de surfactante. Al momento del nacimiento, el epitelio pulmonar del RN que
durante el embarazo es un activo secretor de cloro (Cl-) y liquido hacia los alveolos, tiene que cambiar para convertirse en un activo
absorbedor de sodio (Na++) y liquido con el objeto de remover este último, que está condicionado por la presencia de catecolaminas
secretadas durante el trabajo de parto que estimulan los llamados canales epiteliales de Na++. La TTRN es el resultado de alveolos que
permanecen “h medos” al no producirse esta reabsorción en forma adecuada. El niño nacido por cesárea o el que nace
precipitadamente por vía vaginal tiene mayor riesgo de tener exceso de líquido pulmonar como resultado de no haber experimentado
las fases de la labor y la falta de exposición a las catecolaminas mencionadas. El resultado final son alveolos que retienen líquido
(comprometiendo el intercambio gaseoso que favorece la hipoxemia), el cual se acumula poco a poco en el intersticio, hasta que es
removido por los vasos linfáticos o pasa al torrente sanguíneo. El acumulo de líquido produce edema intersticial y disminución de la
distensibilidad pulmonar, siendo esto último la causa de taquipnea (compensatoria), y colapso parcial bronquiolar que condiciona a su
vez atrapamiento aéreo. Durante el transcurso de las siguientes horas el líquido es removido progresivamente, mejorando la
oxigenación y disminuyendo la frecuencia respiratoria. DIAGNOSTICO: Taquipnea (>60 x´), dificultad respiratoria de cualquier grado
evaluado de acuerdo a la escala de Silverman, cianosis en casos graves, campos pulmonares sin estertores. Laboratorio y gabinete:
Gasometría arterial; hipoxemia, CO2 en limite normal o ligeramente aumentado, acidosis respiratoria compensada. Radiografía de
tórax; Imágenes de atrapamiento aéreo (rectificación de arcos costales, herniación de parénquima, hiperclaridad, aumento de espacios
intercostales, aplanamiento de diafragmas), cisuritis, congestión parahiliar, cardiomegalia aparente. Si después de 72 hrs la taquipnea
no remite o incrementa pensar en otra patología y/o en complicaciones. TRATAMIENTO: Mantener en todo momento ambiente
térmico neutro. La forma e inicio de alimentación se determina de acuerdo al estado clínico de RN (alimentación por succión, sonda
orogástrica). Se mantendrá en ayuno en los siguientes casos: FR >80x´y dificultad respiratoria con Silverman mayor de 3. El volumen de
líquidos IV será de acuerdo a las necesidades para la edad gestacional y el peso. Si existiera alguna patología asociada a la TTRN el inicio
y forma de alimento será a juicio del médico. Manejo y estimulación mínimos. Farmacológico: oxigenoterapia; habitualmente no
requieren FiO2 mayores al 40%. El objetico es obtener saturaciones por púlsometria en el rango de 88-95%. No se recomienda el uso de
medicamentos ya que no existe evidencia suficiente de su eficacia y seguridad en RN. La administración postnatal de epinefrina con el
fin de estimular la reabsorción de líquido pulmonar, ha sido motivo de estudios experimentales sin que exista al momento evidencia.
Los esteroides antenatales a madres entre las 34-37 SDG. Podría tener un efecto benéfico al disminuir la morbilidad respiratoria de los
RN, aun se requieren estudios para establecer su recomendación.

CASO CLINICO
Se trata de paciente masculino de 15 horas de vida estrauterina, el paciente nació via cesarea, nació a la 35 semanas de gestacion, se
administro maduradores a la madre debido a trabajo de parto distócico con ruptura de membranas 12 horas antes de la intervención
quirúrgica, a la exploración física se observo taquipnea de 101 rpm, y signo moderados de dificultad respiratoria.

PREGUNTA
Considerando un propable diagnostico clínico realizado, cual de los diagnosticos diferenciales es menos probable encontrar?

RESPUESTA
a.- Sindrome de adaptación pulmonar.
b.- Sindrome de aspiración de meconio.
c.- Enfermedad de membrana hialina.
d.- Neumonia neonatal hospitalaria.

PREGUNTA
Se decide realizar radiografia de torax, cuales son los datos no es probale espera encontrar para confirmar el diagnostico?

RESPUESTA
a.- Broncograma aéreo.
b.- Hilio congenstivos.
c.- Derrame cisurales.
d.- Sobredistencion pulmonar.

PREGUNTA
Cuales son los factores de riesgo inducido para la taquipnea trasitoria del recién nacido, cual es la mas probable en el caso clínico?.

RESPUESA
a.- Sexo masculino
b.- Grandote.

CURSO ENARM CMN SIGLO XXI TEL: 36246001 Pharmed Solutions Institute PÁGINA 350
MANUAL DE TRABAJO DEL CURSO ENARM CMN SIGLO XXI
c.- Cesarea.
d.- Sedación materna.

PREGUNTA
Cual es el fenómeno fisiopatológico no es la mas adecuada para el caso?

RESPUESTA
a.- Falta de absorción de líquido amniotico.
b.- Falta administración de esteroides.
c.- Deficienca ligera de surfactante.
d.- Edema pulmonar transitorio.

PREGUNTA
Considerando el diagnotico actual, el cual muestra singos de dificultad respiratoria, resultados de laboratorio normales al momento,
FiO2 < 0.40 descartando los diagnosticos diferencias, cuál es la conducta a seguir en este caso es menos adecuada?

RESPUESTA
a.- Monitoreo de signos vitales.
b.- Monitoreo de gases por puncion cada 6 horas.
c.- Mantener glucosa y electrolitos.
d.- O2 por casco al 70 %

PREGUNTA
Tomando en cuenta la evolución del caso, cual es su pronostico?

RESPUESTA
a.- Bueno.
c.- Malo.
b.- Fatal.
d.- Excelente.

PREGUNTA
Considerando un propable diagnostico clínico realizado, cual de los diagnosticos diferenciales es menos probable encontrar?

RESPUESTA
a.- Sindrome de adaptación pulmonar.
b.- Sindrome de aspiración de meconio.
c.- Enfermedad de membrana hialina.
d.- Neumonia neonatal hospitalaria.

PREGUNTA
Se decide realizar radiografia de torax, cuales de los datos no es probale espera encontrar para confirmar el diagnostico?

RESPUESTA
a.- Broncograma aéreo.
b.- Hilio congenstivos.
c.- Derrame cisurales.
d.- Sobredistencion pulmonar.

PREGUNTA
Cuales son los factores de riesgo inducido para la taquipnea trasitoria del recién nacido cual es la mas probable en el caso clínic?.

RESPUESA
a.- Sexo masculino
b.- Grandote.
c.- Cesarea.
d.- Sedación materna.

PREGUNTA
Cual es el fenómeno fisiopatológico no es la mas adecuada para el caso?

RESPUESTA
a.- Falta de absorción de líquido amniotico.
b.- Falta administración de esteroides.
c.- Deficienca ligera de surfactante.
d.- Edema pulmonar transitorio.

CURSO ENARM CMN SIGLO XXI TEL: 36246001 Pharmed Solutions Institute PÁGINA 351
MANUAL DE TRABAJO DEL CURSO ENARM CMN SIGLO XXI

PREGUNTA
Considerando el diagnotico actual, el cual muestra singos de dificultad respiratoria, resultados de laboratorio normales al momento,
FiO2 < 0.40 descartando los diagnosticos diferencias, cuál es la conducta a seguir en este caso es menos adecuada?

RESPUESTA
a.- Monitoreo de signos vitales.
b.- Monitoreo de gases por puncion cada 6 horas.
c.- Mantener glucosa y electrolitos.
d.- O2 por casco al 70 %

PREGUNTA
Tomando en cuenta la evolución del caso, cual es su pronostico?

RESPUESTA
a.- Bueno.
c.- Malo.
b.- Fatal.
d.- Excelente.

CASO CLINICO
Un RN de 3,6 kg nació a las 37 semanas de gestación hijo de diabética con un agente hipoglicémico oral. Obetnido por cesárea electiva
con Apgar de 6/9. Desarrollo taquipnea inmediatamente después de su nacimiento y requirio oxígeno suplementario. En la gasometria
presento; fueron PO2 de 57 mm Hg, PCO2 de 52 mm Hg, y pH de 7,31. El niño se mantuvo en oxígeno por campana. A las 2 horas el
paciente se encuentra hipotónico, cianótico, con saturación del 70 %. La Rx de observa rectificación, hiperclaridad e incremento de los
espacios intercostales y congestión parahiliar.

PREGUNTA
Cual es la medida inmediata a seguir.

RESPUESTA
a.- Intubacion orotraqueal.
b.- Alimentación por sonda orogastrica.
c.- Mantener un ambiente neutro.
d.- Realizar medidas de reanimación.

PREGUNTA
Cual de las siguientes datos radiológicos es menos probable para el diagnostico radiológico?

RESPUESTA
a.- Aumento de la trama vascular con imágenes algodonosas.
b.- Liquido en cisusras interlobales.
c.- Abatimiento de diafragma.
d.- Cardiomegalia.

PREGUNTA
Cual de los siguientes diagnostico diferenciales es mas frecuente?

RESPUESTA
a.- Sepsis.
b.- Neumonía
c.- Asfixia perinatal.
d.- Cardiopatía congenita.

CASO CLINICO
Un RN de 3,4 kilos nació a las 40 semanas de gestación con madre diabética tratada con insulina. El niño fue obtenido por cesárea.
Puntuación de Apgar fue 5/6. Se presento taquipnea una hora después en observación, el niño fue colocado en oxígeno con casco
cefálico. Las determinaciones de gases en sangre arterial en ese momento eran de PO2 52 mm Hg, pCO2 de 48 mm Hg, y pH de 7,31.

PREGUNTA
Cual es la conducta inicial a seguir.

RESPUESTA
a.- Iniciar con bicarbonato.
b.- Verificar laboratorios.
c.- Realizar estudios de gabinete.

CURSO ENARM CMN SIGLO XXI TEL: 36246001 Pharmed Solutions Institute PÁGINA 352
MANUAL DE TRABAJO DEL CURSO ENARM CMN SIGLO XXI
d.- Colocacion de factor surfactante.

CASO CLINICO
RN femenino de 31 SDG "después de un embarazo sin complicaciones”, con peso de 1480 g, y Apgar de 2/6. Fue obtenido por cesaria
electiva por cesaria previa por periodo intergenesico de 6 meses. A los pocos minutos de nacer, su frecuencia respiratoria fue de 61,
con significativo aleteo nasal, uso prominente músculos accesorios y retracciones subcostales. Su ritmo cardíaco era normal (121
latidos / min). Había signos de cianosis.

PREGUNTA
La paciente se encuentra en un hospital rural de concentración, cual es su conducta a seguir.

RESPUESTA
a.- Trasladarla a una unidad de cuidados intensivos.
b.- Vigilancia estrecha ya que su evolución es buena.
c.- Los datos radiográficos no son importantes.
d.- Verificar el diagnostico inicial

CURSO ENARM CMN SIGLO XXI TEL: 36246001 Pharmed Solutions Institute PÁGINA 353
MANUAL DE TRABAJO DEL CURSO ENARM CMN SIGLO XXI
SINDROME DE ASPIRACION MECONIAL (SAM). CIENCIAS BASICAS: El síndrome de aspiración meconial (SAM) se manifiesta con distrés
respiratorio y es producido por la aspiración de líquido amniótico (LA) teñido con meconio intra útero o intra parto. Constituye una
causa de morbimortalidad en el recién nacido, principalmente en el niño a término y postérmino. Factores de riesgo: Embarazo
postérmino, preeclampsia- eclampsia, hipertensión materna, DM materna, oligohidramnios, tabaquismo intenso, puntaje de Apgar
(igual o menor a 6) bajo a los 5 minutos. Hipoxia aguda intraparto, hipoxia perinatal crónica, frecuencia cardiaca fetal anormal, RN
pequeños para la edad gestacional. SALUD PUBLICA: Se reporta con frecuencia líquido meconial 14%. 10% SAM. Mortalidad 12 % y
además deja secuelas. Se presenta 9-15% de nacidos vivos. La expulsión de meconio rara vez es antes de las 37 semanas. Puede ocurrir
30% de los embarazos que continúan después de las 42 SDG. PATOGENIA: 1. Evacuación del meconio in-útero: La asfixia y otras formas
de estrés intrauterino pueden causar un aumento del peristaltismo intestinal, con relajación del esfínter anal externo y evacuación de
meconio. El efecto de la hipoxia intrauterina sobre el peristaltismo y el tono esfinteriano parece aumentar con la edad gestacional. 2.
Aspiración de meconio: Después de la evacuación de meconio en el líquido amniótico, las respiraciones jadeantes del feto asfixiado, ya
sea in útero o durante el trabajo de parto, pueden determinar la aspiración del líquido amniótico teñido con meconio hacia las vías
aéreas grandes del pulmón. El meconio espeso provoca obstrucción de la vía aérea, lo que ocasiona dificultad respiratoria. Con la
aspiración distal de meconio puede ocurrir una obstrucción parcial o total de la vía aérea. En las áreas de obstrucción total se
desarrollan atelectasias; en cambio, en las áreas de obstrucción parcial ocurre un fenómeno valvular que ocasiona atrapamiento aéreo
e hiperinsuflación pulmonar. El atrapamiento aéreo aumenta el riesgo de escape de aire del 21 al 50%. Finalmente, se desarrolla
neumonitis intersticial y química, con edema bronquiolar y estrechamiento de las vías aéreas de pequeño calibre. La ventilación
desigual debida a áreas de obstrucción parcial y neumonitis sobreagregada produce retención severa de dióxido de carbono e
hipoxemia. La resistencia vascular pulmonar aumenta como resultado de la hipoxia, la acidosis y la hiperinsuflación de los pulmones. El
aumento de la resistencia vascular puede conducir a un cortocircuito de derecha a izquierda auricular o ductal y a una mayor
desaturación. CLASIFICACION: Clínicamente se clasifica en: a) Leve: discreta polipnea, hiperinsuflación torácica. No existe alteración en
la PO2 ni en la PCO2 FiO2 <0,4. b) Moderada: hipercarbia, cianosis. Necesidad de FiO2 progresivamente creciente en las primeras 12-24
horas, incluso VM. Pueden desarrollar neumotórax o insuficiencia cardiaca hipóxica. c) Grave: Hipoxemia e hipercarbia desde el
nacimiento, que precisa VM con FiO2 altas y medidas de soporte cardiovascular, auscultación roncus y estertores diseminados.
DIAGNOSTICO: Clínico; Los síntomas dependen de la severidad de la lesión hipóxica y de la cantidad y de la consistencia del meconio
aspirado. Es frecuente que su inicio clínico sea precoz, progresivo a lo largo de 12 a 24 horas, con hipoxemia, dificultad respiratoria,
taquipnea, cianosis persistente, hipoxemia, de saturaciones frecuentes, labilidad en el manejo clínico. Los neonatos con líquido
amniótico teñido con meconio suelen mostrar signos de posmadurez; son pequeños para la edad gestacional y tienen uñas largas, piel
descamada teñida con pigmento amarillo o verde y cordón umbilical teñido de meconio, mancha o unto sebáceo. Pueden presentar
depresión respiratoria en el momento de nacer, con escaso esfuerzo respiratorio y tono muscular disminuido si ha existido una asfixia
perinatal importante. Laboratorio; niveles de gases en sangre arterial revelen hipoxemia. La hiperventilación puede producir alcalosis
respiratoria en los casos leves; pero los lactantes con enfermedad grave suelen manifestar acidosis respiratoria con retención de
dióxido de carbono, debido a obstrucción de la vía aérea y neumonitis. La radiografía de tórax típica muestra hiperinsuflación de los
campos pulmonares y diafragmas aplanados. Hay infiltrados focales irregulares y gruesos con líquido pulmonar aumentado alternando
con zonas hiper aireadas (imagen en panal de abejas). Puede haber neumotórax o neumomediastino en el 10-40% de los casos.
TRATAMIENTO: En la sala de parto; el obstetra deberá aspirar en contenido de la nariz y oro faringe antes del parto de tórax con
perilla. Si hay aspiración de meconio y el RN no está vigoroso (respiración inadecuada, tono muscular disminuido, FC <100 lpm), está
indicada aspiración traqueal directamente. Antes de que establezca las respiraciones. Introducir laringoscopio y utilizar sonda de
aspiración de 12 F a 14F para aspirar la boca y la faringe posterior y así visualizar la glotis. Introducir el tubo endotraqueal en la
tráquea, conectar a equipo de aspiración, aspirar a medida que se retira la sonda. Po otra parte aumentar la concentración de oxigeno
inspirado, monitorear gases sanguíneos, CPAP (presión de la vía aérea positiva continua) individualizar cada caso, ventilación mecánica,
ventilación de alta frecuencia (ventiladores oscilatorios o jet). Corticoesteroides: no hay evidencia. En recién nacidos con SAM masivo e
hipertensión pulmonar persistente, el manejo con ventilación de alta frecuencia y óxido nítrico ha mejorado su pronóstico; la
incorporación de oxigenación con membrana extracorpórea, (no disponible aún en nuestro medio) se muestra como un arma
prometedora en el manejo de este tipo de pacientes. La inactivación del surfactante por el propio meconio y/o por la coexistencia de
edema pulmonar, hace suponer que algunos de estos RN se beneficiarían de la administración de surfactante. En este sentido, están en
marcha estudios corroborativos para demostrar la efectividad de esta nueva arma terapéutica. COMPLICACIONES: Neumonía por
aspiración de meconio. Neumonitis. Hipoxemia. Di estrés respiratorio. Acidosis metabólica. Obstrucción mecánica de las vías aéreas.
Escape aéreo: 10-20% neumotórax o neumomediastino. Hipertensión pulmonar persistente del recién nacido.

CASO CLINICO
RN femenino de término obtenido por parto vaginal espontáneo con un peso al nacer de 3.500 g. el monitoreo intraparto no revelaron
evidencia de sufrimiento fetal. Poco después del parto, fue ingresado en una unidad de cuidados intensivos debido a la presencia de
meconio, vómitos y succión debil, su temperatura era inferior a 36,0 ° C, el pulso fue de 148 lat / min, y su frecuencia respiratoria era de
72 respiraciones / min. El tiempo de protrombina y el tiempo parcial de tromboplastina activada eran 20,7 y 54,6 s, respectivamente. La
orina y sangre se sometieron a la cultivo. La radiografía de tórax mostró infiltrados y rayas gruesas del campo pulmonar derecho.

PREGUNTA
Considerando la gravedad del cuadro cual es la complicación más probable en esta paciente.

RESPUESTA
a.- Sepsis.
b.- Acidosis respiratoria.
c.- Neumonia.
d.- Coagulopatia.

CURSO ENARM CMN SIGLO XXI TEL: 36246001 Pharmed Solutions Institute PÁGINA 354
MANUAL DE TRABAJO DEL CURSO ENARM CMN SIGLO XXI
PREGUNTA
Cual de las siguientes manifestaciones es mas importante para el pronostico?

RESPUESTA
a.- La cantidad de meconio aspirado.
b.- La edad gestacional.
c.- Manifestaciones de neumonitis química.
d.- Manifestaciones de posmadurez.

PREGUNTA
Cual de las siguientes alteraciones es importante mantener en la estabilización mediata del paciente?

RESPUESTA
a.- Taquipnea.
b.- Cianosis persistentes
c.- Desaturaciones.
d.- Cambios minimos se inestabiliza.

PREGUNTA
Cual de las siguientes medidas es menos importantes en la reanimación del paciente?

RESPUESTA
a.- No estimular.
b.- No frotarlo.
c.- No voltearlo ni manipular.
d.- Aspirar primero la nariz y la orofaringe,

PREGUNTA
Cual de las siguientes manifestaciones es mas frecuente encontrar en esta patologia?

RESPUESTA
a.- Infiltrados.
b.- Hiperinsuflacion.
c.- Atelectasia semegtadas o globales
d.- Puede haber neumotórax.

PREGUNTA
Cual de las siguientes complicaciones es la menos frecuente en esta patologia.

RESPUESTA
a.- Hipertension pulmonar persistente.
b.- Acidosis con lactato bajo.
c.- Neumonitis química.
d.- Sindrome de escape aéreo.

CURSO ENARM CMN SIGLO XXI TEL: 36246001 Pharmed Solutions Institute PÁGINA 355
MANUAL DE TRABAJO DEL CURSO ENARM CMN SIGLO XXI
NEUMONIA NEONATAL (NNN). CIENCIAS BASICAS: Ocurre como una complicación de infecciones connatales o de infecciones
nosocomiales. Neumonía intrauterina: se adquiere vía intrauterina o durante el paso de canal de parto. Neumonía nosocomial: se
adquiere en su estancia hospitalaria, procesos invasivos, intubación, asistencia ventilatoria, intubación, permeabilización de vasos.
Después de las 72 h. Neumonía adquirida durante el nacimiento, por contaminación durante el paso a través del canal de parto, aspirar
meconio o líquido amniótico infectado. La neumonía puede aparecer 2ª 4ª semana de vida. Factores de riesgo: maternos; RPM, IVU
materna dentro de 15 días antes del parto, colonización vaginal patológica. En el RN; menor diámetro de árbol bronquial y escaso
desarrollo de aparato ciliar, bajos niveles de IgM, complemento, opsoninas y función linfocitaria, trauma de la vía aérea (intubación
prolongada, aspiraciones profundas) y presencia de meconio en vía aérea, corioamnioitis. SALUD PUBLICA: La incidencia en el RNT es
menor de 1% y en el RNPT de 10-20%, dependiendo de cada UCI Neonatal. El pulmón es el órgano que con mayor frecuencia se
compromete en las infecciones que se desarrollan en las primeras 24 horas de vida. Mortalidad: 20-50%, según la edad gestacional y
complicaciones asociadas. PATOGENIA: Etiología: NNN de inicio precoz = Complejo TORCH, E. Coli, Streptococo del grupo B, listeria y
menos frecuentemente agentes virales (herpes simple, CMV, influenza, rubéola, ADV y echovirus. NNN de inicio tardío = Staphylococos,
Klebsiella, Pseudomona, Enterovirus, E. Coli, Cándida Albicans y algunos virus como CMV (por transfusiones). DIAGNOSTICO: Clínica;
Inespecífica, se manifiesta por dificultad respiratoria (polipnea, quejido y cianosis) asociado o no a un signo clínico de infección
(palidez, mal llene capilar, alteraciones de termorregulación, etc.) que se agrava rápidamente si no se instaura un tratamiento
adecuado. Considerar factores de riesgo descritos. Laboratorio: hemograma (leucocitos fuera de rangos normales con predominio de
neutrófilos en infecciones bacterianas, linfocitario o de monocitos en infecciones virales, eosinófilos por chlamydia trachomatis),
proteína C reactiva, gasometría, glicemia, calcemia, Test Látex, antígenos capsulares (en orina, LCR). Cultivos: de sangre, secreción
traqueal (útiles si se toman precozmente). Radiografía de tórax. Radiológico: inespecífico, infiltrado denso alveolar en uno o ambos
pulmonares. Un dato importante es la persistencia de la imagen radiológica por más de 48 horas. TRATAMIENTO: Medidas generales:
monitorización, ambiente térmico neutral, balance H-E y ácido base. Antibióticos: inicia terapia de amplio espectro y luego, si existe
confirmación adecuar terapia específica según la epidemiología de cada unidad de RN. Neumonía intrauterina: ampicilina-amikacina 7-
10 días. Neumonía nosocomial: vancomicina, cefotaxima. NNN inicio precoz: Ampicilina y Gentamicina. NNN inicio tardío: Cloxacilina y
Amikacina. Manejo respiratorio según gravedad, oxigenoterapia, ventilación mecánica y otras medidas más recientes cuando se
obtengan (óxido nítrico, intubarlo). COMPLICACIONES: Choque séptico. Foco infeccioso en meninges, articulaciones. Hipertensión
pulmonar.

CASO CLINICO
Paciente masculino 3 dias que presenta datos de dificultad respiratoria con rechazo a la alimentación, su nacimiento fue a las 35
semanas de gestacion, cabe destacar presencia de conjuntivitis, parto distocico, por via vaginal, la radiografia es inespecíficas, solo un
infiltrado denso alveolar, la imagen fue persistente, los laboratorios mostraron eosinofilia, no recibió factor surfatante pero recibió
oxigeno al 100 %, cabe destacar que mientras se encontraba en unidad de cuidados intensivos presento distención abdominal.

PREGUNTA
Cual es el agente etiológico mas probable?

RESPUESTA
a) Pseudomona.
b) Candida
c) e. coli
d) Cloacale

PREGUNTA
Cual es la conducta farmacología a seguir considerando que los cultivos aun no tienen datos para ser dirigido?

RESPUESTA
a.- Vancomicina 74mg/Kg y cefotaxima 50mg/kg
b.- Cefotaxima 20 mg/ Kg y metronidazol 30 mg/Kg
c.- Ampicilina 100 mg /kg mas amikacina 15mg/kg
d.- Doxiciclina 15 mg/kg mas gentamicina 15 mg/kg

NEUMONIA NEONATAL TARDIA


Se trata de paciente masculino de 4 semanas de nacido, el cual nación prematuro por ruptura de prematuras de membrana el cual
presento taquinea transitoria del recién nacido, inicia progresivamente con irritabiliad

PREGUNTA
Cual es el agente causal mas frecuente en esta patologia?

ETIOLOGIA
a.- klepsiella.
b.- Stafilococo.
c.- Psudomona.
d.- E. coli.

CURSO ENARM CMN SIGLO XXI TEL: 36246001 Pharmed Solutions Institute PÁGINA 356
MANUAL DE TRABAJO DEL CURSO ENARM CMN SIGLO XXI
CASO CLINICO
Se trata de recién nacido de 3 horas de nacido, el cual nace prematuro, de madre adolecente con cervicovaginitis y parto distócico,
inicia con taquipnea, dificultad respiratoria, con aleteo nasal, con disociación toraco abdominal, con mala tolerancia a la via oral, con
hipotermia, Los datos radiográficos de control se observo persistencia de imágenes radiológicas en ambos campos pulmonares tipo
infiltrado, con síndrome de fuga, el recuento de leucocitos fueron predominio eosinofilos.

PREGUNTA
Cual es el agente etiológico mas probable?

RESPUESTA
a- Clanmydia
b.- Klepsiela
c.- Candida
d.- Garnerella

PREGUNTA
Cual es el tratamiento farmacológico en espera de los resultados de cultivo?

a.- Ampicilina y amikacina.


b.- Ampicilina y gentamicina.
c.- Ceftriaxona y vancomicina.
d.- Imipenem y Cefotaxima.

SEPSIS NEONATAL. CIENCIAS BASICAS: Se define como un Síndrome de Respuesta Inflamatoria Sistémica (SRIS) en la presencia o como
resultado de infección probada o sospechada durante el primer mes de vida extrauterina. Según la edad de presentación puede ser
clasificada de manera arbitraria en sepsis temprana, si aparece en los primeros 3 días de vida (para algunos autores hasta los 7 días de
vida), que es debida generalmente a microorganismos adquiridos de vía materna y sepsis tardía, la cual se presenta después de los 3
días (algunos autores >5 dias) de vida extrauterina y es causada frecuentemente por microorganismos adquiridos después del
nacimiento; esta última puede ser de adquisición nosocomial o de la comunidad. Factores de riesgo: Sexo masculino, prematurez y bajo
peso al nacer, ruptura de membranas > 18 hrs, reanimación al nacer y procedimientos invasivos, fiebre materna periparto.
Dependiendo de la gravedad de la infección y la intensidad de la respuesta inflamatoria, esta se presenta como: Sepsis (síndrome de
respuesta inflamatoria sistémica más hemocultivo positivo) Sepsis grave (sepsis más disfunción orgánica, hipotensión o hipoperfusión)
Choque séptico (sepsis grave con hipotensión que no responde a carga de líquidos) y Falla orgánica múltiple. SALUD PUBLICA:
Incidencia: 1 a 5 por cada 1000 nacidos vivos. 4 millones mueren de sepsis en las primeras 4 semanas. Cerca de un 85% de los pacientes
requieren ventilación mecánica. PATOGENIA: Etiologia: Sepsis neonatal temprana: E. Coli, Klebsiella sp, Enterococcus sp. Streptococcus
agalactiae y Listeria monocytogenes. Sepsis neonatal tardía: Staphylococcus coagulasa negativa, staphylococcus aureus, Enterococcus
sp, E Coli, Klepsiella, Candida spp, Streptococcus del grupo B. La presencia de los mediadores hormonales e inmunológicos que
interviene directamente en la modulación de la respuesta inflamatoria a la infección, definen más claramente los mecanismos
involucrados en la respuesta inflamatoria aguda a la infección, además de la respuesta de anticuerpos, la activación de las dos vías del
complemento, la participación del sistema de coagulación, la respuesta fagocitica mononuclear y polimorfonuclear y de la activación
linfocitaría, ciertos mediadores que son sintetizados principalmente por monocitos, linfocitos, fibroblastos y células endoteliales actúan
de manera determinante en la regulación y modulación de la respuesta inflamatoria inmunológica a la infección. Las citocinas actúan
como mediadores de la comunicación intercelular y efectores de la activación de funciones de las células que tiene receptores para
estas proteínas. Las cuales son: interleucinas, interferones y factores de crecimiento. Según sea su función se pueden encontrar las de
actividad proinflamatoria (IL-1, IL-6, TNFalfa, IL-2, IL-8, IL-12) otras antiinflamatorias o inhibitorias (Antagonistas, Receptores solubles,
IL-4, IL-10, IL-13) y factores de crecimiento (GM-CSF, G-CSF, M-CSF, IL-3, IL-6, IL-5, IL-7). La magnitud de citosinas proinflamatorias que
se producen en etapa temprana de la sepsis se relaciona con la intensidad del daño pulmonar. DIAGNOSTICO: El diagnóstico temprano
y oportuno de sepsis neonatal no es fácil porque las manifestaciones clínicas son inespecíficas y pueden avanzar rapidamente a
estadíos más avanzados. Los signos de alarma identificados por la Organización Mundial de la Salud (OMS) son los siguientes:
convulsiones, rechazo al alimento, dificultad respiratoria, hipoactividad, polipnea. Las manifestaciones clínicas son inespecíficas y muy
variadas dependiendo de la gravedad de presentación. Algunas de las principales son: distermias, dificultad respiratoria, ictericia,
apneas (con más frecuencia en prematuros), distensión abdominal, hepatomegalia, letargia, sangrados, hipoactividad, palidez, oliguria,
cianosis, piel marmórea, crisis convulsivas, irritabilidad, esplenomegalia, vómito, diarrea, hipotensión arterial, petequias o equimosis,
trombocitopenia y acidosis. La sospecha clínica es lo principal para poder llegar al diagnóstico de sepsis neonatal e idealmente
confirmarse con cultivos positivos en sangre, orina, líquido cefalorraquídeo (LCR) u otros sitios normalmente estériles. Ante cualquier
cuadro sugestivo de sepsis, se debe realizar estudio diagnóstico completo. En sepsis temprana se deben incluir hemocultivos (central y
periféricos) y cultivo de líquido cefalorraquídeo. En sepsis tardía se debe incluir además urocultivo. En casos de infecciones localizadas
(por ejemplo osteoartritis), se debe cultivar el sitio de infección. Prácticamente cualquier problema neonatal puede presentarse como
sepsis, debido a esto es importante que se realice una historia clínica y exploración física adecuadas, toma de biometría hemática
completa. Si bien no existe un biomarcador de sepsis ideal, existen múltiples estudios que apoyan la utilidad de procalcitonina y
proteína C reactiva para el diagnóstico de sepsis neonatal. Hemocultivo: sensibilidad 82%, especificidad 96%, BH: Leucocitos anormales:
< 6 000 células/mm o > 30 000 células/mm en las primeras 24 hrs. 50% RN puede tener conteos normales. TRATAMIENTO: Primera
elección: Ampilicina- amikacina. El manejo empírico inicial de antibióticos debe hacerse con base en la experiencia de cada hospital,
siempre teniendo en cuenta el patrón de resistencia y sensibilidad. En sepsis neonatal temprana el tratamiento debe iniciarse con
ampicilina y un aminoglucósido (gentamicina ó amikacina), en ocasiones especiales se puede sustituir el aminoglúcosido por
cefotaxima, sobre todo si existe la sospecha de neuroinfección (está demostrado que esteriliza el LCR con mayor rápidez). En recién

CURSO ENARM CMN SIGLO XXI TEL: 36246001 Pharmed Solutions Institute PÁGINA 357
MANUAL DE TRABAJO DEL CURSO ENARM CMN SIGLO XXI
nacidos con sepsis tardía adquirida en la comunidad, es posible utilizar el mismo esquema, sin embargo en sepsis nosocomial, el
tratamiento debe estar orientado a combatir los microorganismos presentes en cada institución. La evidencia actual de ensayos
clínicos controlados aleatorizados no apoya el uso rutinario de inmunoglobulina intravenosa y factor estimulante de colonias de
granulocitos. El uso de pentoxifilina como adyuvante en el manejo de sepsis reduce la mortalidad en neonatos pretérmino, sin embargo
debido a debilidades metodológicas de los estudios al respecto, no es adecuado utilizarlo de manera rutinaria hasta la obtención de
mejor evidencia. Dependiendo de las condiciones clínicas del paciente en el caso de sepsis grave o choque séptico se deberá
proporcionar apoyo ventilatorio, suministro de líquidos, aminas e incluso corticoesteroides en el caso de hipotensión refractaria a las
mismas o en caso de sospecha de insuficiencia suprarenal. Es indispensable realizar la corrección del equilibrio ácido base y
proporcionar apoyo calórico y nutricional ya sea por vía enteral o parenteral según sea el caso. Pacientes con riesgo de sepsis: iniciar
tratamiento revalorar 48 hrs. (el 95% de las infecciones se presenta en las primeras 24 hrs). Si no hay desarrollo de cultivos, biometría
hemática normal y PCR normal suspender el tratamiento.

CASO CLINICO
Primigesta de 15 años de edad, con gestación de 40 semanas, presenta trabajo de parto en fase de dilatación rápida (5 cm de
dilatación) y expulsión de líquido amniótico (LA) con tinte meconial. Tuvo una rotura precoz de membranas 1 h 20 min antes del
ingreso. Se indicó anestesia de conducción con peridural continua y monitorización electrónica que reveló una variabilidad menor de 5
LCF, una frecuencia cardíaca fetal media de 150 latidos por min y una desaceleración variable prolongada (hasta 120 LCF por minuto
durante 3 min). Se reevaluó luego de nueva desaceleración (60 LCF por minuto durante 7 min) y se encontró 8 a 9 cms de dilatación,
presentación cefálica en II plano y LA con grumos de meconio con leve mal olor. Se trasladó inmediatamente a sala de partos con
dilatación completa y presentación cefálica en tercer plano. Se obtuvo un recién nacido (RN) de 3,440 g de sexo masculino, con muy
mal olor, con Apgar 1/3. El RN evolucionó grave, con compromiso del SNC, encefalopatía hipóxica isquémica grado III, asociado a shock
séptico precoz, con compromiso multiorgánico y necesidad de apoyo ventilatorio y de drogas vasoactivas.

PREGUNTA
Cual es la complicación más probable en esta paciente.

RESPUESTA
a.- Coagulacion intravascular diseminada.
b.- Falla multiple.
c.- Neurosepsis.
d.- Neumonia asociada.

CASO CLINICO
Se trata de paciente de 35 semanas de gestacion de 5 dias de nacido, con ruptura prematura de membrana el cual requirió reanimación
intensiva debido a pagar 2/6, se observo fiebre materna durante el trabajo de parto, la madre vive en medio rural, apnea, crisis
convulsivas, vomito, distermia, taquicardia, disnea y deterioro, hiporreactivo con piel marmórea, distención abdominal, se diagnostico
sepsis neonatal.

PREGUNTA
Cuales de los diagnosticos diferenciales es el menos frecuente?

RESPUESTA
a.- Sindrome de adaptación del recién nacido.
b.- Taquicardia transitoria del recién nacido
c.- Hemorragia ventricular.
d.- Neumonitis

PREGUNTA
Cual de los siguientes valores de laboratorio es mas probable esperaría?.

RESPUESTA
a.- PCR elvada y neutrofilia.
b.- Leucocitosis con eosinofilia.
c.- Linfocitosis mas plaquetopenia.
d.- VSG elevada con leucopenia.

PREGUNTA
Cual es la conducta farmacológica empirico de primera eleccion en espera de los resultados de cultivos.

RESPUESTA
a.- Ampicilina mas gentamicina.
b.- Ampicilina mas amikacina.
c.- Inmoglobulina y cefotaxima.
d.- Ampicilina y FEC.

CURSO ENARM CMN SIGLO XXI TEL: 36246001 Pharmed Solutions Institute PÁGINA 358
MANUAL DE TRABAJO DEL CURSO ENARM CMN SIGLO XXI
ENTEROCOLITIS NECROTIZANTE (ECN). CIENCIAS BASICAS: Se presenta como un síndrome gastrointestinal y sistémico que comprende
síntomas variados y variables, como distensión e hipersensibilidad abdominal, sangre en heces, intolerancia alimentaria, apnea, letargia
y en casos avanzados acidosis, sepsis, CID y shock. Constituye la urgencia gastrointestinal más frecuente en las UCI neonatales. SALUD
PUBLICA: Es una enfermedad grave que afecta a recién nacido, en especial prematuros, con una incidencia y morbilidad elevados.
Incidencia 1-3 por 1000 RN vivos y 1-7.7% de los RN ingresados a unidades neonatales. Aumenta la incidencia en los menores de 1500g,
hasta un 2-10%. La edad gestacional media oscila en torno a las 31 semanas, con un peso medio al nacimiento de 1.460 g. PATOGENIA:
Se acepta un mecanismo multifactorial en el huésped predispuesto. Entre los factores propuestos implicados en la patogénesis de la
ECN se han descrito la prematuridad, alimentación láctea, inestabilidad hemodinámica, infección y alteración de la mucosa intestinal.
Solo la prematuridad y la alimentación láctea tienen una base epidemiológica consistente. En prematuros, la predisposición de estos
niños pudiera explicarse por la inmadurez de su tracto gastrointestinal con función luminal limitada que conlleva una absorción parcial
de carbohidratos y grasas así como proliferación bacteriana, mayor permeabilidad de la mucosa e hipomotilidad. Junto a ello existe
inmadurez de los sistemas defensivos sistémicos y de la mucosa intestinal, entre otros la IgA secretora y la barrera de mucina. Se ha
especulado la posibilidad de un efecto protector de los glucocorticoides administrados prenatalmente. Una de las principales hipótesis
es que la ECN proviene de efectos nocivos de la microflora intestinal sobre una mucosa entérica lesionada o isquémica. La lesión
mucosa puede provenir de distintos procesos, incluyendo la asfixia o la isquemia secundaria a la derivación de la sangre lejos del
intestino. Se desconoce la especificidad de este proceso. La flora microbiana involucrada en la ECN no es única, aunque representa
organismos intestinales predominantes presentes en el recién nacido en el momento de la aparición de la enfermedad. Estos
organismos pueden incluir las bacterias anaeróbicas, como las especies de Clostridium. Por el momento no se ha definido un papel
especifico del Clostridium difficile o de una toxina especifica. Algunos virus pueden estar incluidos en este cuadro microbiológico.
DIAGNOSTICO: El hallazgo más precoz suele ser un cambio en la tolerancia alimentaria en un niño prematuro, con buena evolución
hasta ese momento y que comienza a presentar restos gástricos. Los síntomas sistémicos asociados son inespecíficos, desde aparición
de apneas, alteración del patrón respiratorio, distermia, inestabilidad hemodinámica con bradicardias, hasta hipotensión, letargia o
shock séptico y CID. Desde el punto de vista gastrointestinal, la ECN se presenta con distensión abdominal, restos gástricos, abdomen
doloroso, vómitos, diarrea o hematoquecia. El curso de la enfermedad varía según recién nacidos. Con mayor frecuencia aparece como:
1) una presentación fulminante, rápidamente progresiva de signos congruentes con necrosis intestinal y sepsis, y 2) como una
presentación lenta, paroxística, de distensión abdominal, íleo y posible infección. Laboratorio y gabinete: La radiografía abdominal
(confirma diagnostico) suele revelar un patrón gaseoso anormal congruente con el íleo. Estas radiografías pueden revelar un edema de
la pared intestinal, la posición fija de un asa en estudios seriados, la aparición de una masa, la neumatosis intestinal (95%) más
frecuente en cuadrante inferior derecho, el aire venoso portal o hepática, la neumobilia o el neumoperitoneo (cuando hay perforación
intestinal). Estudios sanguíneos. La trombocitopenia, la acidosis metabólica persistente y la hiponatremia refractaria grave son la triada
más frecuente y ayudan a confirmar el diagnóstico. La trombocitopenia se asocia a necrosis intestinal y empeoramiento clínico. El
análisis de las heces para detectar sangre e hidratos de carbono, si los hay, se ha utilizado para diagnosticar a los recién nacidos afectos
de ECN, basado en los cambios de la integridad intestinal. La malabsorción de los hidratos de carbono, según refleja un Clinitest positivo
(cuerpos reductores) en las heces, puede ser un indicador frecuente y precoz de ECN en el marco de los signos. Otros hallazgos
bioquímicos inespecíficos son la
elevación sérica de PCR y alfa-1-
glicoproteina o de la alfa-1-antitripsina
en heces. Ante la posibilidad de sepsis
deben recogerse cultivos (hemocultivo,
cultivo de LCR, coprocultivo). Si se
sospecha Clostridium son necesarios
cultivos específicos y determinación de
toxina. La visualización directa de la
mucosa entérica mediante endoscopia
se ha utilizado para detectar a los recién
nacidos afectos de ECN, aunque la experiencia con esta técnica es demasiado limitada para ser de aplicaci6n directa. Los avances
tecnológicos pueden hacerla posible en el futuro. Debe considerarse la ultrasonografía como método útil en el diagnostico cuando
existe sospecha clínica, no confirmada por radiología. CLASIFICACION: ETAPA I: sospecha a.- signos sistémicos leves: apnea,
bradicardia, inestabilidad térmica, letargia. b.- signos intestinales leves: distensión abdominal, restos gástricos (pueden ser biliosos),
sangre oculta en heces. c.- radiografía de abdomen: normal o con signos no específicos. ETAPA II: enfermedad definida (signos
radiológicos positivos) a.- signos sistémicos moderados. b.- signos intestinales adicionales: silencio abdominal, dolor a la palpación del
abdomen. c.- signos radiológicos específicos: neumatosis intestinal o gas en el sistema portal. d.- alteraciones analíticas: acidosis
metabólica, leucopenia, trombocitopenia. ETAPA III: enfermedad avanzada: shock séptico y neumoperitoneo. a.- afectación sistémica
grave: hipotensión arterial, signos evidentes de shock. b.- signos clínicos de peritonitis. c.- signos radiológicos de gravedad:
neumoperitoneo. d.- alteraciones analíticas: acidosis metabólica y respiratoria, leucopenia y neutropenia, trombocitopenia, coagulación
intravascular diseminada, proteína C muy elevada. TRATAMIENTO: Medico: consiste en medidas de soporte, reposo intestinal,
nutrición parenteral, antibioterapia y corrección de alteraciones hematológicas y/o electrolíticas que pudieran estar presentes.
Medidas específicas son la dieta absoluta, descompresión intestinal con aspiración, reposición de líquidos considerando pérdidas a un
tercer espacio, aporte calórico adecuado mediante nutrición parenteral y antibióticos endovenosos de amplio espectro. Deben
suspenderse todos los fármacos relacionados como posibles factores de riesgo. Como medidas de soporte se incluye la asistencia
respiratoria temprana ante la aparición de apneas o patrón respiratorio acidotico, la corrección de la acidosis, hiponatremia o
trombopenia. La acidosis metabólica persistente es un indicador de progresión de la lesión intestinal e incluso necrosis. Se debe
asegurar una perfusión y transporte de oxigeno adecuados, con un aporte suficiente de líquidos y manteniendo el nivel de hematocrito
>35%. Puede ser necesario el uso de agentes inotrópicos. La dopamina a dosis bajas puede ser de ayuda para mejorar la perfusión
sistémica y aumentar el flujo mesentérico. Habitualmente la ampicilina y gentamicina constituyen el tratamiento adecuado, debiendo
asociar clindamicina o metronidazol ante sospecha de gérmenes anaerobios, si bien la clindamicina ha sido asociada con un aumento

CURSO ENARM CMN SIGLO XXI TEL: 36246001 Pharmed Solutions Institute PÁGINA 359
MANUAL DE TRABAJO DEL CURSO ENARM CMN SIGLO XXI
de la frecuencia de estenosis postenterocoliticas. La dieta y la antibioticoterapia han de mantenerse durante 10-14 días, con
introducción progresiva de aporte enteral, con fórmulas hipoosmolares de hidrolizado de proteínas. Quirúrgico: El momento ideal sería
aquel en que se ha producido una gangrena intestinal pero todavía no existe perforación, ni peritonitis secundaria. Indicada en
pacientes que progresa con necrosis de pared intestinal y perforación debe ser urgente debido al rápido deterioro. La decisión es clara
en aquellos en que destaca la presencia de neumoperitoneo. El empeoramiento progresivo nos alerta sobre necrosis intestinal como:
signos de peritonitis como edema y eritema de pared, masa abdominal, trombopenia y acidosis persistentes. Paracentesis: un resultado
positivo es altamente específico de necrosis intestinal.

CASO CLINICO
RN con parto en casa con control prenatal con 34 semanas de gestación por FUM, con un peso 2100 g. 7 dias despues desarrolló sepsis
y fue hospitalizado en la unidad de cuidados intensivos. El examen clínico encontró una distensión abdominal y una radiografía
abdominal mostró aire en las paredes del intestino y un neumoperitoneo grande (aire libre subdiafragmático, aire perihepático libre,
doble signo de la pared). Requirio intubacion y se proporciono soporte inotrópico y antibióticos.

PREGUNTA
Cual es la causa más probable de la complicación.

RESPUESTA
a.- Nacimiento en casa.
b.- Edad gestacional
c.- Comorbilidad pulmonar.
d.- Sepsis abdominal.

CASO CLINICO
Al segundo día de vida presenta resolución de su dificultad respiratoria con disminución de requerimientos de oxígeno y tolerancia al
destete, por lo cual se inicia estímulo enteral con adecuada tolerancia. Los laboratorios iniciales hemograma normal y PCR negativa. Se
continúo vigilancia clínica y aumento progresivo de la vía oral. A los 5 días de vida presenta cuadro de episodio emético de contenido
alimentario sin otra sintomatología el cual se interpretó como reflujo gastroesofágico. Al día siguiente presenta deterioro clínico dado
por taquicardia, distensión abdominal, persistencia de episodios eméticos postprandiales, residuo gástrico del 70%, deposiciones con
sangre macroscópica e hipoglicemia.

PREGUNTA
Considerando los hallazgos clínicos cual es el estadio en el que se encuentra?

RESPUESTA
a.- Enterocolitis necrotizante I.
b.- Enterocolitis necrotizante II.
c.- Enterocolitis necrotizante III.
d.- Enterocolitis necrotizante IV.

PREGUNTA
Cual es la conducta a segur mas adecuada en este momento?

RESPUESTA
a.- Se suspende la vía oral, BH, QS, cultivos, ampicilina y gentamicina.
b.- Ampicilina + amikacina, Bh, QS, cultivos, ayuno.
c.- Alimentacion parenteral, antibioticoterapia empirica, BH, QS, cultivos.
d.- Ampicilina, cefotaxima, alimentación por sonda, BH y QS.

PREGUNTA
Tres días después el paciente presenta hipotensión e hipoperfusión asociado a falla respiratoria por lo que requiere soporte inotrópico
y ventilatorio. Presenta acidosis metabólica, trombocitopenia y alteración en pruebas de coagulación, cultivos con E.coli
multiresistente, considerando los cambios en de las imágenes de actuales cual es la conduca a seguir en el paciente?

RESPUESTA
a.- Transfusión de plasma y plaquetas.
b.- Crioprecipitados, albumina y cristaloides.
c.- Piperacilina tazobactam y nutrición parenteral.
d.- Piperacilina tazobactam, amikacina y nutrición parenteral.

CASO CLINICO
Recién nacido (RN) varón, con antecedentes de parto de pretérmino a las 32 semanas de gestación, con peso de 1 300 g y Apgar 8 y 10
al nacer. Estando relativamente bien, a las 72 horas de vida comienza con rechazo alimentario y distensión abdominal. Un día más
tarde se agregan vómitos biliosos, deposiciones con sangre e inestabilidad hemodinámica.

PREGUNTA

CURSO ENARM CMN SIGLO XXI TEL: 36246001 Pharmed Solutions Institute PÁGINA 360
MANUAL DE TRABAJO DEL CURSO ENARM CMN SIGLO XXI
Se realizo una radiografia para confirmar diagnostico, cual de los siguientes datos no esta presente en esta?

RESPUESTA
a.- Dilatación de asas intestinales de colon e intestino delgado.
b.- Engrosamiento de paredes y extensa neumatosis intestinal.
c.- Múltiples imágenes aéreas pequeñas, en aspecto de "burbujas" .
d.- Neumoperitoneo.

PREGUNTA
Considerando las imágenes observadas, cual es diagnostico final y complicaciones asociadas.

RESPUESTA
a.- Enterocolitis necrotizante complicada, con perforación intestinal y neumoperitoneo secundario.
b.- Enterocolitis necrotizante complicada, sin perforación intestinal ni neumoperitoneo secundario.
c.- Enterocolitis necrotizante simple, con preforacion intestinal.
d.- Enterocolitis complicada con peritoneo secundario.

PREGUNTA
Cual de las siguientes complicaciones agudas es mas frecuente?

RESPUESTA
a.- Necrosis intestinal con perforación
b.- Peritonitis secundaria.
c.- Formación de abscesos intraabdominales.
d.- Coagulación intravascular diseminada.

CASO CLINICO
Se trata de un paciente varón de 5 meses de vida que ingresó en nuestro hospital por presentar vómitos proyectivos en la mitad y al
final de las tomas, acompañados de una pérdida de peso de 350g en el transcurso de una semana. Como antecedentes destacaba que
era un exprematuro moderado de 30 semanas de gestación que al quinto día de vida desarrolló una EN con neumatosis gástrica e
intestinal, por lo que precisó una resección de 10cm del yeyuno y 25cm del íleon. Tras la intervención quirúrgica presentó varios
episodios de retención gástrica y vómitos, con resolución espontánea y adecuada tolerancia posterior, por lo que se le dio el alta a su
domicilio.

PREGUNTA
Cual es la conducta a seguir en este caso, con la sintomatologia presente cual es estudio de primera elección para identificar la causa de
la condución actual?

RESPUESTA
a.- Tránsito esofagogastroduodenal.
b.- Radiografia de abdomen.
c.- Ultrasonograma abdominal.
d.- Endoscopia digestiva.

PREGUNTA
Cual de los siguiente diagnostico diferenciales del estado actual, es el menos frecuente?

RESPUESTA
a.- Ingestion de causticos.
b.- Anomalias del ligamento falciforme.
c.- Enfermedad granulomatosa caustica.
d.- Gastroenteritis eosinofilica.

CURSO ENARM CMN SIGLO XXI TEL: 36246001 Pharmed Solutions Institute PÁGINA 361
MANUAL DE TRABAJO DEL CURSO ENARM CMN SIGLO XXI
MENINGITIS NEONATAL. CIENCIAS BASICAS: Es una enfermedad infecciosa con secuelas a corto y largo plazo, siendo su tratamiento
costoso, con aumento de los días de hospitalización y número de días recibiendo antibióticos. Meningitis bacteriana: síndrome clínico
compatible con meningitis, más aislamiento de un organismo por cultivo del líquido cefalorraquídeo o aislamiento de un organismo por
cultivo de sangre y anormalidad del LCR consistente con infección bacteriana. SALUD PUBLICA: Frecuencia variable. 25-30% de los
casos de sepsis neonatal se complica con meningitis. Meningitis bacteriana tiene incidencia hasta de 1 x 1000 nacidos vivos.
CLASIFICACION: Meningitis temprana: Primera semana transmisión vertical (gérmenes localizados en el canal vaginal materno)
gérmenes involucrados, Streptococcus del grupo B, E. coli, Listeria monocytogenes, Enterococcus. Meningitis tardía: Nosocomial
adquirida en la comunidad gérmenes involucrados, transmisión horizontal Gram negativos, Estafilococos. PATOGENIA: Etiología: gram(-
): E. coli, Klebsiella, Serratia marcescens, Proteus, Enterobacter, Neisseria, Pseudomona, Salmonella, Citrobacter, etc. Gram(+): Stp,
Enterococos, Staphylococcus aureus, estafilococo coagulasa negativo. Otros: Lysteria monocytogenes. Más frecuente: Enterobacterias
gram negativas (30-50%). Estreptococo beta-hemolítico grupo B (30-40%). Lysteria monocytogenes (10%). S. pneumonie, H. influenzae.
Es la diseminación hematógena en la mayoría de los casos, sin embargo ocasos se producen por propagación por contigüidad como
consecuencia de la contaminación de defectos del tubo neural, fístulas congénitas o de heridas penetrantes causadas al obtener
muestras de sangre, partos sépticos o domiciliarios. La cerebritis y los infartos sépticos son frecuentes en la meningitis bacteriana. La
formación de abscesos, la ventriculitis, la hidrocefalia y los derrames subdurales aparecen más a menudo en recién nacidos que en
niños mayores. Los hallazgos patológicos son similares entre los diferentes agentes bacterianos. El más común en autopsias es
exudado purulento en meninges y superficie ependimal de los ventrículos. Se evidencia también inflamación perivascular y gran
respuesta inflamatoria. Hidrocefalia y encefalopatía no infecciosa puede ser demostrada en aproximadamente 50% de los neonatos
con meningitis. La efusión subdural raramente ocurre en neonatos. Varios grados de flebitis y arteritis de vasos intracraneales pueden
presentarse. Ventriculitis puede ser demostrada en virtualmente todos los neonatos y en 75% al momento del diagnóstico. Factores de
riesgo: Infección perinatal e
intrauterina (corioamnioitis,
endometritis, IVU, infección vaginal),
prematurez, ruptura prematura de
membranas, peso bajo al
nacimiento, vía hematógena,
aspiración, inhalación.
Corioamnionitis clínica: presencia de
fiebre materna >38° C, con dos o más de los siguientes hallazgos: taquicardia fetal (>160), sensibilidad uterina a la palpación, descarga
vaginal fétida o leucocitosis materna. DIAGNOSTICO: Clínica: fiebre o hipotermia 60%, Irritabilidad 60% pobre alimentación/vomito
48% crisis convulsivas 42% dificultad respiratoria 33% apnea 31% diarrea 20% fontanela abombada 25% rigidez de nuca 13%. La
sintomatología de un paciente con meningitis es muy inespecífica y puede ser muy similar a la de un paciente con sepsis y las
manifestaciones como fontanela abombada, alteración del estado de conciencia, convulsiones y coma, cuando se presentan se asocian
a pobre pronostico; por lo que es importante que dentro de la evaluación del neonato infectado se incluya la punción lumbar incluso en
la ausencia de signos neurológicos evidentes. Laboratorios: 1. BH, recuento de leucos y porcentaje de PMN: < 20% ó >80% Índice
inmaduros/ maduros totales: ≥ 0.2 - Plaquetas <100.000 2. PCR: Es significativo cuando esta sea > 10mg/dl 3. Glicemia: si presenta
valor <40mg/dl ó >120mg /dl 4. Hemocultivos: Siendo positivos hasta un 50 %. Citoquímico, Gram y cultivo de LCR: Se tendrán los
siguientes criterios en el líquido cefalorraquídeo: 1) Glucosa menor a dos tercios la glucosa sérica medida por glucometria simultánea.
2) Relación glucosa LCR/ sérica mayor de 0.5. 3) Proteínas: Se considerará positivo para infección valor mayor de 90 mg/dl para recién
nacido a término y de más de 150md/dl en prematuros. 4) Celularidad: Mayor de 10 células blancas. 5) Cultivo positivo.
Contraindicaciones para la punción lumbar (LCR: 10-30 ml prematuro, RN termino 40 ml): compromiso cardiorespiratorio, CID, lesiones
en el sitio de la punción, prematuros extremos (menores de 1000 gramos). Imagenología: Hacerla para detecta complicaciones,
pacientes choqueados, falla respiratoria, déficit neurológico, cultivo + después de 48-72 Hs con apropiada terapia. TAC para descartar
abscesos. TRATAMIENTO: Sepsis temprana: ampicilina+amikacina. Sepsis tardía: cefotaxima+amikacina. Específico para el agente
identificado. Otros: Meropenem, Ticarcilina, Vancomicina. Duración terapia: 14-21 días. Esteroides. Inmunoglobulina. Aunque el
manejo final deberá basarse en el resultado del antibiograma de las muestras de los hemocultivos, el tratamiento empírico iniciado con
penicilinas más una cefalosporina de tercera generación y/o un aminoglucósido tipo gentamicina preferencialmente, después de
haberse tomado las muestras, es el que ha mostrado la mejor de las respuestas clínicas. El tratamiento endovenoso deberá continuarse
con el antibiótico apropiado para el germen aislado, hasta dos semanas después de la negativización del hemocultivo.
Desafortunadamente, las secuelas severas a largo plazo son frecuentes (12% a 29% de los afectados), especialmente en las infecciones
asociadas a Streptococcus del grupo B y a bacilos gram negativos, aún en los casos en los que el tratamiento ha sido oportuno y
adecuado

CASO CLINICO
Paciente masculino 10 dias de nacimiento, que se obtuvo en casa sin control prenatal, ingresa por diarrea, llanto, e irritabilidad, vomito
en proyectil, en casa, refiere la madre que solo fue en una ocacion, al ingreso se observo hipotónico, llanto agudo, mal estado
generalizado, con tono cervical no se observo rigidez, pero fontanela abombada, su peso fue de 2050 grs, laboratorios con 150,000 de
plaquetas, antecedentes de rinorrea hialina, se presento 35 grados, irritabilidad, durante la exploración se observo pedaleo y
chupeteo, dificultad respiratoria.

PREGUNTA
Para establecer el diagnostico y conducir su terapéutica para el caso?

RESPUESTA
a.- Hemocultivo.
b.- Punsion lumbar.

CURSO ENARM CMN SIGLO XXI TEL: 36246001 Pharmed Solutions Institute PÁGINA 362
MANUAL DE TRABAJO DEL CURSO ENARM CMN SIGLO XXI
c.- Tomografia.
d.- Biometria hemática.

PREGUNTA
Se encuentra en espera de resultados, sin embargo el paciente se va deteriorando, cual es la agente etiolohico mas probable para dar
una terapéutica dirigida?

a.- H influenza
b.- E Coli.
c.- Listeria monocitogenes
d.- Estreptoco beta hemolito.

PREGUNTA
Cual es la conducta farmologica más adecuada considerando la etiología?

RESPUESTA
a.- Ampicilina, amikacina.
b.- Ampicilina mas metronizadol.
c.- Ampicilina mas vancomicina.
d.- Ampicilina mas cefotaxima.

PREGUNTA
Se administro esteroides en el paciente, cual es la razón mas correcta para realizar esta conducta en el caso?

RESPUESTA
a.- Disminuir los efectos de los factores inflamatorios.
b.- Favorece la penetración del fármaco.
c.- Disminuir la probabilidad de crisis convulsivas.
d.- Prevenir el edema cerebral.

PREGUNTA
Cual es la secuela mas frecuente que los pacientes con esta patologia desarrollan?

RESPUESTA
a.- Empiema cerebral
b.- Retazo en el neurodesarrollo
c.- Hidrocefalia.
d.- Paralisis cerebral.

CASO CLINICO
Recién nacido de 12 días de vida con fiebre de 38,5ºC de una hora de evolución y rechazo de la alimentación. Como antecedentes;
ruptura espontánea de membranas intraparto, parto instrumental con fórceps a las 41 semanas de gestación y peso al nacimiento de
3880 g. Apgar 8/9. Comenzó su esquema de inmunización. Alimentación con lactancia materna exclusiva. La exploración física mostró
decaimiento y regular perfusión periférica con constantes vitales normales. La analítica inicial mostró una determinación de proteína C
reactiva cuantitativa (PC-R) 133,4 mg/L, con un hemograma coagulado y un sedimento de orina normal. La punción lumbar resultó
hemorrágica. Ingresó con diagnóstico de síndrome febril sin foco y se inició antibioterapia empírica con ampicilina y cefotaxima
intravenosas tras la toma de cultivos. En el hemograma repetido se observaron 5800 leucocitos/mm3, con fórmula normal, 13,4 g/dl de
hemoglobina, 32 000 plaquetas/mm3. A las 18 h del ingreso empeoró su estado general y presentó crisis convulsivas generalizadas que
requirieron tratamiento con fenobarbital y perfusión de midazolam. En la evolución desarrolló afectación sistémica con hipotensión
arterial, acidosis metabólica, hiponatremia, oliguria y plaquetopenia grave; requirió ventilación mecánica convencional, transfusión de
hemoderivados, soporte inotrópico con dopamina y diurético con furosemida. El tercer día del ingreso se informó el aislamiento en el
hemocultivo de Streptococcus pyogenes sensible a penicilina, por lo que se suspendió la ampicilina y se mantuvo el tratamiento con
cefotaxima intravenosa.

PREGUNTA
Cual es la causa más probable de las crisis convulsivas.

RESPUESTA
a.- Hemorragia cerebral.
b.- Alteracion hidroelectrolitica.
c.- Neurosepsis.
d.- Encefalopatia hipoxico-isquemica.

CURSO ENARM CMN SIGLO XXI TEL: 36246001 Pharmed Solutions Institute PÁGINA 363
MANUAL DE TRABAJO DEL CURSO ENARM CMN SIGLO XXI
CASO CLINICO
Neonato de 11 días de vida, previamente sana, ingresada por dificultad respiratoria, mala coloración, rechazo de tomas y decaimiento.
Constantes vitales; TA de 95/65 mmHg, FR de 160 lpm, FR de 90 rpm y saturación de oxígeno del 94% con FiO2 del 30%. Se auscultan
ruidos crepitantes e hipoventilación basal izquierda. La radiografía de tórax evidencia infiltrado basal izquierdo.

PREGUNTA
Cual es la conducta a seguir.

RESPUESTA
a.- Amoxicilina mas gentamicina.
b.- Trimetoprim mas sulfametoxazol.
c.- Cetriaxona.
d.- Vancomicina.

CASO CLINICO
Se trata de paciente que se encuentra en la UCIN con diagnostico de sepsis neonatal, prematuro, bajo peso y masculino con 10 dias de
nacido, inicia con llanto continuo, irritable, sin rigidez de nuca, se observa hipotónico, rechazo al alimento, vomito en proyectil.
Antecedentes con aspiración de meconio con sufrimiento e hipoxia, por parto prolongado y distocia, se observo disminución de
complemento y linfocito,

PREGUNTA
Cual es el estándar de oro para el dianostico mas adecuado?

a.- Hemocultivo y urocultivo.


b.- Liquido cefaloraquideo.
c.- Biometria hemática y EGO.
d.- Marcadores inflamatorios.

PREGUNTA
Cual es el agente etiológico mas probable en el caso clínico actual?

a.- E. coli
b.- Klebsiella
c.- Pseudomona
d.- Salmonela

PREGUNTA
Cuales son las siguiente manifestaciones es la que presenta mayor frecuencia y mayor valor diagnostico?

a.- Fiebre e irritabiliad


b.- Vomito y crisis convulsivas.
c.- Alteraciones metabolicas
d.- Fontanela abombada.

PREGUNTA
Cuales son los valores que no es probable observar en este paciente.

RESPUESTA
a.- Proteína de aumentada
b.- Glucosa 2/3 del central.
c.- Leucocitos elevados
d.- Acido láctico disminuido.

PREGUNTA
Cual es la complicación anatomopatologia mas frecuente?

RESPUESTA
a.- Hemorragia ventricular.
b.- Ventriculomegalia
c.- Encefalomacia.
d.- Ventriculitis.

PREGUNTA
Cual es el tratamiento empirico de primera elección?

RESPUESTA

CURSO ENARM CMN SIGLO XXI TEL: 36246001 Pharmed Solutions Institute PÁGINA 364
MANUAL DE TRABAJO DEL CURSO ENARM CMN SIGLO XXI
a.- Ampicilina mas gentamicina
b.- Ampicilina mas amikacina
c.- Amikacina mas cefotaxima.
d.- Vancomicina, ampicilina mas cefotaxima.

PREGUNTA
Al paciente se le administro esteroides, cual es el objetivo farmacológico para esta indicación?

RESPUESTA
a.- Como medida antiedema.
b.- Para sinergismo con inmunoglobulina.
c.- Sinergismo con los antibióticos.
d.- Estabilización de membrana.

PREGUNTA
Cual de las siguientes complicaciones es menos frecuente esperar?

RESPUESTA
a.- Ventriculitis
b.- Hidrocefalia.
c.- Pérdida auditiva y ceguera.
d.- Sindrome de Rett.

PREGUNTA
Condiderando los criterior para definir sepsis es un síndrome de respuesta inflamatoria sistémica, aunado a un agente infeccioso en el
RN?

RESPUESTA
a.- Horas de nacido, >130 FC.
b.- FR mas de 50
c.- Leucocitos mas de 16,500.
d.- Temperatura 37.8 y 10% de bandas 10 %

PREGUNTA
Cual es el agente causal mas probable en este caso especifico?
a.- Listeria monocitogenes.
b.- Stafilococco
c.- Enterococcus.
d.- Candida A.

HIPOGLUCEMIA. CIENCIAS BASICAS: Hipoglucemia neonatal: debajo de 45 mg/dl (2.5 mmol/L) para prematuros como de término y a
cualquier edad extrauterina. Hipoglucemia neonatal transitoria: se auto limita durante los primeros 7 días de vida extrauterina, es
consecuencia de reserva energética limitada, excesivo consumo periférico, agotamiento precoz de reservas energéticas y a inmadurez
del sistema hipotálamo-hipofisiario, responsable de la secreción de hormonas contra regulación. Hipoglucemia neonatal persistente o
recurrente: <45 mg/dl en 3 ocasiones o persiste por más de 72 hrs a pesar de tratamiento con soluciones glucosadas. Hipoglucemia
severa: aporte glucosada kg min > 10 ml/kg/min persisten cifras menores de 45 mg/dl. Factores de riesgo: Madre diabética durante el
embarazo o ingestión de betabloqueadores o hipoglucemiantes orales. Prematurez. Peso bajo para la edad gestacional. Peso grande
para la edad gestacional. Detección tardía y manejo inadecuado de la hipoglucemia neonatal tiene un impacto sobre el desarrollo.
Causas de hipoglucemia transitoria: Estrés perinatal, septicemia, asfixia, hipotermia, policitemia, choque. SALUD PUBLICA: En RN a
término la incidencia está en un rango de 5-7% y pue de variar entre valores de 3,2 % a 14,7 % en recién nacidos pretérminos. Casi el
40% de los neonatos hijos de madre diabética tienen hipoglucemia. PATOGENIA: La etiología más frecuente de hipoglucemia en el
recién nacido está de manera general asociada a incremento de la utilización de glucosa, a un aporte inadecuado de glucosa endógeno
o exógeno o a una combinación de ambos. Una de las causas más frecuentes de hipoglucemia en el RN ligada a hiperinsulinismo fetal es
el hijo de madre diabética mal controlada. En estos niños la hipoglucemia frecuentemente se produce a las 4-6 hrs después del
nacimiento, tienen una reducción incrementada de insulina secundaria a sensibilidad aumentada de células beta del páncreas a la
glucosa; que persiste durante varios días después del parto. Síntomas clásicos se deben a activación del SNA con la libera catecolaminas
(hormona contrarreguladora de la hipoglucemia). Deprivación de glucosa al cerebro con alteración función neurológica. La
administración antenatal de clopropamida, benxotiazidas, beta-simpaticomiméticos, propanolol o la administración de glucosa a la
madre a alta concentración y supresión brusca de su administración puede inducir insulinismo fetal transitorio y por lo tanto
hipoglucemia neonatal. La eritroblastocisis fetal en RN con incompatibilidad Rh y el síndrome de Beckwith-Wiedemann, se caracteriza
por RN con macrosomia, onfalocele, macroglosia, visceromegalia e hipoglucemia. Se ha explicado la hipoglucemia por hipertrofia de las
células beta del páncreas y por lo tanto hiperinsulinismo. Otras causas: por deficiencias hormonales: déficit de hormonas del
crecimiento, deficiencia tiroidea. Causas hereditarias: enfermedad de orina de jarabe de arce, intolerancia a la fructosa, galactosemia.
DIAGNOSTICO: Apnea, hipotonía, reflejo de succión inadecuada, irritabilidad, somnolencia, respiraciones irregulares, cianosis,
temblores, palidez, crisis convulsivas, letargia, cambios en el nivel de conciencia, inestabilidad de la temperatura, coma. Triada de
Whipple: Características clínicas, glucosa sérica baja, resolución de síntomas con corrección de glucemia. Estándar de oro:

CURSO ENARM CMN SIGLO XXI TEL: 36246001 Pharmed Solutions Institute PÁGINA 365
MANUAL DE TRABAJO DEL CURSO ENARM CMN SIGLO XXI
Determinación enzimática de los niveles de glucosa en laboratorio por el método de la hexocinasa. TRATAMIENTO: Debe iniciarse una
alimentación precoz en las dos primeras horas de vida (de preferencia en los primeros 30-60 minutos) y establecerse intervalos de
alimentación cada 2-3 horas. Hipoglcemia asintomática: 1. Si mediante tira reactiva los niveles de glucosa periférica se encuentran
entre 27mg y 45 mg hay que tomar muestra sanguínea por punción venosa y corroborar niveles de glucosa, ofrecer inmediatamente sin
esperar el resultados alimentación al seno materno y posteriormente cada hora. Si el RN no puede recibir seno materno de manera
adecuada, suplementar con sucedáneo de leche materna cuyo volumen se ajustara con base al peso y se ofrecerá cada 3 hrs. 2. Si
mediante tira reactiva los niveles de glucosa periférica se encuentran por debajo de 27mg/dl, corroborar con una glucosa central.
Indicar infusión de glucosa IV para propiciar una infusión de glucosa kilo minuto de 6mg/kg/min. No suspender alimentación enteral. Si
el tratamiento inicial fue ofrecer únicamente alimentación enteral y con ello se normalizo la glucosa, se recomienda: Indicar infusión de
glucosa IV con líquidos a 80ml/kg/día para proporcionar una infusión de glucosa kilo minuto de 6 mg/kg/min. Se sugiere que en RN con
hipoglucemia asintomática que requieran tratamiento con soluciones IV, el volumen de leche materna o sucedáneo de leche materna
sea de 10-15 ml/kg/día. Hipoglucemia sintomática: Tomar glucosa central para corrobora. Administrar solución glucosada al 10% IV en
bolo: Si el paciente presenta crisis convulsivas administrar 4ml/kg. Si el paciente no presenta crisis convulsivas, administrar 2ml/kg.
Después de administrar el bolo establecer infusión continúa de glucosa IV de mantenimiento de 6mg/kg/min, una vez controlado
aumentar 2mg/kg/min hasta 12 mg/kg/min. Si las condiciones lo permiten continuar alimentación oral con leche materna o sucedáneo
de la leche materna en volumen de 10-15 ml/kg/min. Cada vez que se detecte hipoglucemia asintomática se deberá administrar bolo
de solución glucosada al 10%. Si a pesar de tratamiento apropiadamente instaurado, el paciente persiste con hipoglucemia durante 3
determinaciones consecutivas, se recomienda referencia a tercer nivel en cualquiera de las siguientes situaciones: persistencia de
hipoglucemia a pesar de recibir aporte de glucosa kilo minuto de 10 mg/kg/min. Reaparición de hipoglucemia al disminuir de manera
apropiada el aporte de glucosa kilo minuto. Se recomienda referencia a tercer nivel cuando exista hipoglucemia asociada a:
Antecedente familiar de muerte neonatal súbita o síndrome de Reye, crisis convulsivas o alteración del estado de consciencia asociados
a hipoglucemia, alteraciones de la termorregulación, defectos de la línea media, exoftalmos, micropene. En hipoglucemia refractaria o
persistente: Glucagón (0.1mg/kg IM, máximo 1 mg) o hidrocortisona. Referir a tercer nivel: Persista hipoglucemia a pesar de recibir
aporte de glucosa kilo minuto de 10 mg. Reaparición de hipoglucemia al disminuir el aporte de glucosa. Crisis epilépticas. Alteraciones
de la termorregulación.

CASO CLINICO
RN de 36 SDG obtenido por cesarea por desprendimiento de placenta. Al nacimiento presenta Apgar 8/9, peso en -0,89 DE, longitud en
+1,75 DE, sin datos del perímetro cefálico. Con 48 horas de vida ingresada por ictericia en su hospital de referencia presenta
hipoglucemias no cetósicas (glucemias entre 25-45mg/dl), insulinemia máxima registrada: 7μU/ml coincidiendo con glucemia de
33mg/dl.

PREGUNTA
Cual es la conducta a seguir para identificar la causa de la hipoglucemia.

RESPUESTA
a.- Verificar Diabetes Mellitus en la madre.
b.- Realizar escaneo abdominal.
c.- Verificar funcionamiento hepático.
d.- Verificar funcionamiento suprarrenal.

CASO CLINICO
Se trata de reciencia nacido de 24 horas de edad, presenta en cunero las siguientes alteraciones: reflejo de succion inadecuado, crisis
convulsivas letargia, inestabilidad de la temperatura.

PREGUNTA
Cual es la conducta mediata mas adecuada para este caso?

RESPUESTA
a.- Administracion rectal de diacepam.
b.- Administración oral de glucosa.
c.- Intubacion orotraqueal.
d.- Glucosa periférica.

CASO CLINICO
Se trata de paciente masculino de 37 semanas de gestacion obtenido por cesarea, la madre presento cuadro de pre-eclampsia, luego de
24 horas de nacido presenta disminución de la succion, letargia, falta de respuesta a estimulos con disminución de tono muscular, asi
como alteraciones de la temperatura con tendencia a la hipotermia, los laboratorios reportaron 35 mg/dl de glucosa periférica, resto de
datos de laboratorio y gabinete dentro de parámetro normales, sin embargo posterior a la administración de glucosa mediante catéter
periférico presenta nuevos cuadro de hipoglucemia?

PREGUNTA
Cual de los siguiente antecedentes es menos frecuente para la patologia que presenta el paciente?

RESPUESTA
a.- Displasia de células beta.

CURSO ENARM CMN SIGLO XXI TEL: 36246001 Pharmed Solutions Institute PÁGINA 366
MANUAL DE TRABAJO DEL CURSO ENARM CMN SIGLO XXI
b.- Enfermedad de orina de arce.
c.- Galactosemia.
d.- Hipotiroidismo congénito.

ICTERICIA. CIENCIAS BASICAS: La ictericia es una de las condiciones más comunes que requieren atención médica en los RN y se refiere
a la coloración amarillenta de piel y mucosas causada por la fijación de bilirrubina en el tejido graso subcutáneo; generalmente, se
observa cuando los niveles séricos de bilirrubina son mayores o iguales a 5-7 mg/dl.. Las causas de ictericia neonatal son múltiples y
producen hiperbilirrubinemia directa, indirecta o combinada, de severidad variable. La hiperbilirrubinemia se refiere al aumento de los
niveles de bilirrubina total en sangre (≥ 2-5 mg/dl), como producto final de metabolismo del heme, componente esencial de la
hemoglobina. En la mayoría de los casos, suele ser benigna, y autolimitada, pero por el efecto neurotóxico de la bilirrubina, los
neonatos de riesgo deben ser vigilados para evitar hiperbilirrubinemia severa que produzca alteraciones neurológicas como
encefalopatía aguda y kernicterus, los cuales son causas prevenibles de parálisis cerebral. Los neonatos amamantados son más
propensos a desarrollar, ictérica fisiológica (aparece después de las 24 hrs y desaparece antes de los 10 días) en la primera semana de
vida y cerca del 10% son ictéricos al mes de vida. Ictericia patológica; cuando inicia en las primeras 24 hrs, se acompañe de otros
síntomas, la bilirrubina aumente >5mg/dl diarios, sobrepase los 15mg% o 10mg% en enonatos a término y pretermino, la fracción
directa sea superior a 2mg/dl o dure más de una semana en el RN a término (excepto si recibe lactancia materna). SALUD PUBLICA: La
ictericia fisiológica es una situación frecuente, aparece después del segundo día de vida como expresión de una condición fisiológica
hasta en un 80% de los prematuros y en un 60% de los a término. Ictericia patológica en 6% de RN. PATOGENIA: Factores de riesgo:
Alimentación a pecho. Mayor pérdida de peso (más de 5%). Sexo masculino. Edad gestacional < 35 semanas. Diabetes materna.
Hematomas. Raza Oriental. La principal causa de la apricion de la ictérica fisologica en el RN es la inmadurez del sistema enzimático del
hígado, a esto se le suma una menor vida media del glóbulo rojo, la poliglobulia (el RN produce el doble de bilirrubina que el adulto), la
extravasación sanguínea frecuente, el RN reabsorbe gran parte de la bilirrubina a través de la circulación enterohepatica y la ictérica
por lactancia. Ictericia patológica, las causas más frecuentes son 1. Enfermedad hemolítica (incompatibilidad sanguínea materno-fetal,
esferocitosis familiar, déficit de enzima G-6-PD), las cuales disminuyen la vida media de los eritrocitos. 2. Hematomas y hemorragias
(cefalohematomas), cuya reabsorción aumenta la oferta de bilirrubina. 3. Incremento en la
reabsorción intestinal como sucede en el retraso en la alimentación gástrica en RN
enfermos o la presencia de obstrucción intestinal. 4. Policitemia, por mayor volumen y
destrucción globular. 5. Defectos enzimáticos congénitos como Sx. De Crigler-Najjar. 6.
Ictericia acolurica familiar transitoria (Sx. De Lucey-Driscoll), se presenta en RN cuyas
madres son portadores de factor inhibitorio en el suero que impide la conjugación.
ICTERICIA POR INCOMPATIBILIDAD DE FACTOR Rh: Es la causa más frecuente de ictérica
patológica y el 97% de los casos se debe a isosensibilizacion para el antígeno Rh D. Un alto
título materno de anticuerpos anti-Di ≥ 64 de se asocia a un riesgo elevado de
hiperbilirrubinemia severa para los recién nacidos. La administración de profiláctica de
inmunoglobulina Anti-D. DIAGNOSTICO: El tinte ictérico no solo está en la piel y
conjuntivas, sino que también puede apreciarse en el LCR, lágrimas, saliva y especialmente
en los casos patológicos. Es conveniente valorar la presencia de coluria y acolia, ya que son
datos de gran valor diagnóstico. La presencia de hepatomegalia precoz es sugestiva de
infección prenatal o de enfermedad hemolítica por incompatibilidad Rh; cuando esta es
dura y de aparición más tardía, hará pensar en la posibilidad de afectación hepática primitiva (hepatitis, atresia), si bien en estos casos
el resto de signos clínicos son distintos. La esplenomegalia hará sospechar que actúa como un foco hematopoyético extramedular o
bien que el RN padece una infección prenatal con
manifestaciones clínicas. Por la frecuencia con la que
se presenta al tercer día de vida una
hiperbilirribinemia secundaria a la reabsorción de
hematomas, se deberán buscar colecciones de
sangre extravasada. La ictericia inicia en cara u
tienen una progresión cefalocaudal, útil para valorar
el grado de ictericia. Otro síntoma frecuentemente
asociado a hemolisis es la hipoglicemia, como
resultado de la hiperpalsia pancreática. La presencia
de petequias y purpura sugieren la posibilidad de
infección connatal. Laboratorio: Dosaje de
bilirruibinemia total y directa, si hay elevación de
bilirrubina indirecta, sugiere hemólisis. Si hay
elevación de la bilirrubina directa, sugiere
enfermedad hepatobiliar. Reacción de Coombs
directa e indirecta. Hematocrito y hemoglobina (para
valorar la presencia de anemia asociada). Recuento
de reticulociotos. TRATAMIENTO: Las alternativas
son: 1. Fototerapia; la de elección y más difundida,
su administración redujo en gran medida el uso de
exanguineotransfusion, actúa por fotooxidacion
(destrucción física de la bilirrubina en productos más pequeños y polares para ser excretados) y fotoisomerización, el RN debe estar
desnudo, cubrir los ojos, y control térmico, actualmente existe fototerapia en fibra óptica 2. Exanguinotransfusion, se basa en la
remoción mecánica de sangre del RN por sangre de un donador, ha sido reemplazada por la fototerapia, se reserva en especial para

CURSO ENARM CMN SIGLO XXI TEL: 36246001 Pharmed Solutions Institute PÁGINA 367
MANUAL DE TRABAJO DEL CURSO ENARM CMN SIGLO XXI
enfermedades hemolíticas severas, cuando la fototerapia no ha resultado eficaz. Terapia farmacológica; mesoporfirina, inhibe el
catabolismo del hemo y por lo tanto la producción de bilirrubina. Fenobarbital, es un inductor enzimático que estimula las etapas de
captación, conjugación y excreción de bilirrubina. Por estar asociado a un potencial desarrollo de adicción, sedación excesiva y efectos
metabólicos adversos, han limitados su aplicación en el RN. Administración Oral de sustancias No absorbibles: estos al captar bilirrubina
en la luz intestinal, reducen la absorción enteral de ésta y, así se puede disminuir los niveles de bilirrubina sérica. TOXICIDAD DE LA
BILIRRUBINA: Hay dos fases en la neurotoxicidad de la bilirrubina una temprana y aguda que es reversible si el pigmento es removido y
una lenta y tardía cuyos efectos son irreversibles. Los signos clínicos de toxicidad aguda son apatía, somnolencia o insomnio, junto con
la alteración de los potenciales evocados auditivos, pero que luego revierten, una vez que los valores de bilirrubina descienden. La
encefalopatía por bilirrubinas es un síndrome neurológico que resulta del deposito de bilirrubina no conjugada en el SNC,
especialmente en los ganglios basales y nucleos del tallo cerebral. La causa de esta encefalopatía es de origen multifactorial relacionada
con niveles elevados de bilirrubina no conjugada libre, cantidad de bilirrubina que se une a la albúmina, alteración de la barrera
hematoencefálica (BHE) por otras enfermedades y susceptibilidad neuronal.

CASO CLINICO
RN presenta ictericia, que se hizo evidente en el segundo día de la vida. Nació de padres no relacionados a los 36 semanas de gestación
con un peso al nacer 2800 g, después de embarazo y el parto expontaneo y normal aparentemente. Al examen físico, paciente estaba
activo y cómodo. Sus signos vitales eran estables. En recuento de admisión de sangre completa fue en normal límites.

PREGUNTA
Cual es la conducta a seguir.

RESPUESTA
a.- Alta y medidas en casa.
b.- Se ingresa para fototerapia.
c.- Verifica niveles de bilisrrubinas.
d.- Verifica grupo y factor en ambos padre.

KERNICTERUS. CIENCIAS BASICAS: Es la coloración amarilla de los ganglios basales producida por impregnación de bilirrubina, descrita
en autopsias de RN fallecidos con severa ictericia. Constituye la complicación más grave de la ictericia neonatal. El kernicterus es la
secuela más importante de la encefalopatía bilirrubínica. Es una enfermedad devastadora, una entidad previsible cuando la
hiperbilirrubinemia es tratada agresivamente. SALUD PUBLICA: Su incidencia aumento con las nuevas políticas de alta prematura, esto
causa mayor riesgo de complicaciones debidas a ictericia temprana no detectada. PATOGENIA: Hay varias situaciones que alteran la
barrear hematoencefalica y facilitan la entrada de bilirrubina al SNC, aumentando notablemente el kernicterus, como son; bajo peso al
nacer, hipoglucemia, asfixia neonatal, acidosis metabólica, hemolisis, hipotermia-frio, hipoalbuminemia, drogas que compiten por la
unión a albumina, diestres respiratorio. La bilirrubina no conjugada penetra en el cerebro y actúa como una neurotoxina, a menudo
ésta se asocia con condiciones que dificultan la función de la barrera hemato-encefálica (ejemplo, sepsis), inhibiendo varios procesos
bioquímicos muy importantes, como la fosforilación oxidativa de las mitocondrias y la síntesis proteica. Es necesario anotar que se
desconocen los niveles toxicos de bilirrubina para SNC. Revisiones recientes han sugerido que 25mg% y aun unos puntos más serían los
tóxicos. DIAGNOSTICO: Puede Ser asintomático en prematuros pequeños. Se caracteriza por atetosis, sordera neorosensorial parcial o
completa, limitación de la mirada vertical, déficit intelectual, displasia dental. En la forma clásica se reconocen 3 estadios: Primera fase;
caracterizada por inicio con vómitos, letargia, hipotonía, rechazo al alimento, succión débil y llanto agudo. Segunda fase; se caracteriza
por irritabilidad, hipertonía y opistótonos. Tercera fase; observada en sobrevivientes de las 2 anteriores y caracterizada por la triada de
hipertonía, atetosis y otros movimientos extrapiramidales y retardo psicomotor. Pueden quedar secuelas alejadas siendo las más
frecuentes la sordera, los trastornos motores y los problemas de conducta. Las regiones del cerebro más comúnmente afectadas son
los ganglios basales, particularmente los núcleos subtalámicos y el globo pálido, el hipocampo, el cuerpo geniculado, varios núcleos
cerebrales, incluyendo el colículo inferior, vestibular, oculomotor, coclear y olivar inferior, y el cerebelo, especialmente el núcleo
dentado y el vérmix. La necrosis posnatal es el hallazgo histopatológico dominante después de los 7-10 días de vida posnatal. El
diagnóstico puede ser confirmado por resonancia nuclear magnética (RNM) cerebral, cuya imagen característica es de tipo bilateral,
con alta señal de intensidad en el globo pálido, vista en los cortes de T1 y T2. Estas imágenes también se pueden observar en el
hipocampo y el tálamo.

CASO CLINICO
RN de 35 SDG, vigoroso, peso 2,400 g, perímetro cefálico 34 cm, talla 47 cm, sin patología perinatal, alimentado a pecho direcho
exclusivo, alta a las 48 horas, regreingresa a los 6 dias de vida por ictericia generalizada, con peso al ingreso de 2,130 grs. Succion
vigorosa pero breve e ineficaz, escasas diuresis y deposiciones. Bilisrrubinas totales 32,52 mg/dl, bilisrrubina indirecta 32 mg/dl, Bh
normal, glicemia normal. La EF se observa leve temblor distal

PREGUNTA
Cual es la conducta mas adecuada a seguir en este caso?

RESPUESTA
a.- Colchon de fibra óptica de luz continúa.
b.- Exanguinotransfusion.
c.- Fototerapia.
d.- Luz halogenada.

CURSO ENARM CMN SIGLO XXI TEL: 36246001 Pharmed Solutions Institute PÁGINA 368
MANUAL DE TRABAJO DEL CURSO ENARM CMN SIGLO XXI

ATRESIA ESOFAGICA (AE) Y FISTULA TRAQUEOESOFAGICA (FTE). CINECIAS BASICAS: La atresia de esófago es una anomalía congénita
en la cual la porción media del esófago está ausente (atresia); en un porcentaje importante se acompaña de una comunicación anormal
entre la traquea y el segmento distal del esófago llamada fistula traqueoesofágica. La atresia de esófago es una malformación
incompleta de la luz esofágica. La variante más frecuente es la tipo C de la clasificación de Gross o la III de Voght en 87% de los casos.
Factores de riesgo: y agravantes del pronóstico: malformaciones congénitas, neumonías, bajo peso al nacer. Se encuentran anomalías
cromosómicas entre 6–10% de los casos; las más frecuentes son las trisomías 18 y 21. SALUD PUBLICA: Es la malformación esofágica
más frecuente. La atresia esofágica con o sin fístula traqueoesofágica es una alteración que aparece en 1:3,000 a 1:4,500 recién
nacidos. El 95% de la AE tiene fístula traqueoesofágica asociada. El 50% de los casos se asocia con otras malformaciones congénitas y
existe riesgo de recurrencia de 2 a 3%, y mayor riesgo relativo si se tienen hijos o familiares afectados. En virtud que en México nacen
aproximadamente 2 millones de niños por año, se estima que cada año hay entre 500 y 600 casos nuevos de niños con atresia de
esófago. PATOGENIA: Malformaciones congénitas asociadas: Cardiacas (35%): comunicación interventricular, conducto arterioso
persistente, tetralogía de fallot, coartación de aorta. Gastrointestinales (20%): ano imperforado, atresia duodenal, malformaciones
intestinales. Genitourinarias (20%): reflujo ureteral, agenesia renal. Músculo esquelético (13%): vertebrales, costales, defecto de las
extremidades y del sistema nervioso central (10%). La asociación de algunas de estas anomalías se denomina asociación VACTERL
(Vertebrales, Anoreactales, Cardiacas, Traqueales, Esofagicas, Radiales, renales y de extremidades (L; limbs en inlges) y se presenta con
una frecuencia de 10%. El origen de la AE es poco claro aún pero se atribuye a una alteración en la migración de los pliegues laterales o
a una detención del crecimiento en el momento de la evaginación. En la mayor parte de los casos el esófago posterior no se separa
totalmente de la tráquea, lo que da lugar a distintas variedades de fístula traqueoesofágica o a hendiduras Esta alteración se produce
entre la tercera y sexta semana de gestación. Fístula traqueoesofágica (TEF) Una fístula es una conexión entre el esófago y la tráquea.
Esta conexión permite que la comida (desde el esófago) entre en los pulmones (aspiración). La comida en los pulmones puede causar
neumonía (referida a neumonía por aspiración), lo cual puede ser muy serio. Si hay una fístula traqueoesofágica con atresia esofágica,
toda la comida ingerida terminará en los pulmones, como el esófago no está conectado con el estómago, porque finaliza cerrado. El
tipo más difícil para diagnosticar es la fístula traqueoesofágica tipo H, donde el esófago no termina cerrado, pero hay una fístula o
conexión entre la tráquea y el esófago. En la fístula traqueoesofágica tipo H, el aire que entra al estómago (desde la tráquea) y la
comida puede entrar a los pulmones. El aire en el estómago puede causar hinchazón en el abdomen del bebé y puede hacer que el
bebé está molesto. DIAGNOSTICO: El diagnóstico prenatal se establece con ultrasonografía y puede sospecharse durante el embarazo
por polihidramnios, incapacidad de identificar el esófago fetal o ausencia de burbuja gástrica. Diagnostico en sala de parto, la
complicación al paso de la sonda para verificar la permeabilidad del esófago; sialorrea, distensión abdominal, dificultad respiratoria, tos
o cianosis son algunas manifestaciones en el recién nacido. Al darle de comer: vómitos/atragantamiento (no pasa el contenido a
estómago), crisis de sofocación con tos tras las tomas lo que sugiere fístula superior. Puede apreciarse
abdomen escavado a la inspección o abdomen distendido si fístula inferior. De persistir la alimentación
sin diagnosticarse: vómitos o neumonías de aspiración. Ante la sospecha debe realizarse sonda
nasogástrica: no pasa hasta estómago; se enrolla en bolsón. No necesario contraste por que las sondas
son radiopacas y hacer estudio estudio radiográfico para confirmar el diagnóstico; el estudio muestra
el fondo de saco, ciego, del esófago atrésico o, bien, puede realizarse un examen endoscópico para
confirmar la fístula. Se han evaluado la resonancia magnética, TAC, estudios de medicina nuclear,
angiografía y ultrasonografía sin resultados concluyentes. CLASIFICACION: Clasificación de Vogt
modificada por Ladd de la atresia de esófago: AE TIPO I (5-8%): Ambos cabos esofágicos ciegos sin
fístula traqueoesofágica: Es de fácil diagnostico en el embarazo por que cursa con polihidramios y
ausencia de imagen gástrica. Abdomen excavado por falta de pasaje de aire al intestino. Ambos cabos
se encuentran muy separados entre sí. AE TIPO II (0.5-1%): Fístula traqueoesofágica superior y cabo
inferior ciego. AE TIPO III (80-85%): Fístula traqueoesofágica inferior y cabo esofágico superior ciego:
En éstos pacientes el reflujo gastroesofágico puede ser lesivo para los pulmones, son propensos a sufrir
Neumonías Químicas. AE TIPO IV (0.5-1%): Fístula traqueoesofágica en ambos cabos del esófago. AE
TIPO V (3-5%): Fístula en H o N. Es una fístula traqueoesofágica sin atresia de esófago: El diagnostico se
realiza en la infancia ya que aparecen los síntomas en ese periodo. AE TIPO VI (0.-1%): Estenosis
esofágica aislada. Tipos de atresia esofágica, clasificación de Gross: A. Atresia esofágica sin fístula: 3-5%. B. Atresia esofágica con fístula
proximal: 2%. C. Atresia esofágica con fístula distal: 80-90%. D. Atresia esofágica con fístula proximal y distal: 3-5%. E. Fístula
traqueoesofágica sin atresia: 6%. F. Estenosis esofágica. TRATAMIENTO: El adecuado ambiente térmico, el suministro de líquidos,
glucosa y electrólitos, la posición adecuada del paciente (semisentada, con la cabeza elevada en 30° a 40°), sonda doble lumen. Drenaje
del cabo proximal con sonda de doble lumen (sonda de Repogle) para aspiración constante), la aspiración cuidadosa y frecuente de la
saliva y las secreciones acumuladas en el cabo ciego, si existe neumonía, iniciar doble esquema de antibiótico (ampicilina mas
aminoglucocido). Intervención quirúrgica, interrumpir la comunicación entre la tráquea y el esófago. Reestablecer la continuidad del
Esófago. Preservar el esófago existente. Gastrostomía y esofagectomía: Plastia esofágica, plastia esofágica y cierre fístula, sustitución
esofágica, cierre fístula. Atresia tipo III: dentro de las primeras 24 hrs de vida se realizara cierre de FTE inferior y, si la distancia entre los
cabos es menor de 3 cm, anastomosis termino-terminal (T-T) entre los cabos esofágicos a través de una toracotomía con abordaje
extrapleural. COMPLICACIONES: Reflujo gastroesofágico. Refistulación traqueo esofágica. Estreches anastomótica. Traqueomalasia.
Dismotilidad esofágica.

CASO CLINICO
Recién nacida, gemela I, de madre sana de 35 años de edad, producto del segundo embarazo, de 35 semanas de duración; el embarazo
anterior terminó con aborto espontáneo en el segundo trimestre, seis años antes. El embarazo actual tuvo adecuado control prenatal,
recibió ácido fólico, calcio, hierro y polivitaminas; se complicó por amenaza de aborto en el primer trimestre, pero se recomendó
reposo. En la semana 31 del embarazo se le realizó ultrasonografía obstétrica que reportó. Embarazo gemelar con fetos vivos, placenta
única, corporal, posterior, grado II y edad gestacional de 31 semanas, 3 dias antes del parto la madre tuvo infección de las vías urinarias,

CURSO ENARM CMN SIGLO XXI TEL: 36246001 Pharmed Solutions Institute PÁGINA 369
MANUAL DE TRABAJO DEL CURSO ENARM CMN SIGLO XXI
por lo que se hospitalizo para tratamiento y recibir esquema de tres dosis de inductores de la maduración pulmonar, tuvo rotura de
membranas 4 horas antes del parto, se realizo bloqueo peridural y se realizo cesarea, al nacimiento el Apgar fue de 6/8 con peso de
2,000 g, dificultándose la aspiración de secresiones, continuando con tos, sialorrea, y cianosis leve.

PREGUNTA
Cual es la conducta a seguir inmediata mas adecuada?

RESPUESTA
a.- Colocar en silla porta-bebe a 45 grados.
b.- Colocar sonda de doble lumen.
c.- Prepara para cirugía.
d.- Aplicación de oxigeno a 3 lt/x´.

HERNIA HIATAL CONGENITA. CIENCIAS BASICAS: Sede be al cierre incompleto de la membrana pleuroperitoneal (abertura o
dehiscencia congénita del diafragma) o al retorno prematuro del intestino a la cavidad abdominal con herniación de las vísceras
abdominales a la cavidad torácica. SALUD PUBLICA: La hernia diafragmática congénita ocurre entre 1 en 2,000 a 1 en 5,000 recién
nacidos vivos. La mortalidad varía entre un 40-70% dependiendo el grado de hipoplasia pulmonar y el desarrollo de hipertensión
pulmonar. La relación hombre: mujer es de 1:1,8. El 20-53% presentan malformaciones asociadas, sobre todo defectos cardiacos (9-
23%), defectos del tubo neural (28%), trisomías y ciertos síndromes bien definidos. PATOGENIA: Existen dos teorías acerca de la
embriogénesis de la hernia diafragmática: 1. Crecimiento pulmonar anormal que produce desarrollo diafragmático anormal. 2. Defecto
diafragmático con hipoplasia pulmonar secundaria. La etiología de la hernia diafragmática no es clara, han sido reportados casos
familiares sugiriendo predisposición genética. En 20% de los casos se ha presentado polihidroamnios y es de mal pronóstico. Talidomida
y quinidina se han reportado como causantes de hernia diafragmática en humanos. Las anomalías en la formación anatómica normal
del diafragma explican la aparición de defectos congénitos. Si la detención del desarrollo diafragmático se produce precozmente en
útero, el RN presenta una amplia comunicación entre tórax y abdomen. Si la detención de la formación se produce después de
formado el tabique membranoso pero antes de formarse la caja muscular, el niño presenta un saco herniario que contiene los órganos
desplazados hacia arriba. Esta patología se asocia prácticamente siempre con efecto de masas, que se manifiesta en el feto por
desplazamiento del mediastino, compresión pulmonar y reducción del tejido pulmonar antes de la semana 16, momento en el que
desarrollo bronquial es completo determina reducción del número de bronquios y alveolos del pulmón en desarrollo que conlleva a la
hipoplasia pulmonar. El pulmón hipoplásico en hernia diafragmática presenta unareducción cuantitativa y cualitativa de surfactante y
un compromiso de la distensibilidad pulmonar. CLASIFICACION: De acuerdo a su localización anatómica: 1. Hernia posteroexterna de
Bochdalek es el tipo más común representa 85-90%, siendo la mayor ubicación izquierda 80%, derecha 15%, y mixta 5%. 3. Hernia de
hiato esofágico. 4. Hernia retroesternal de Morgagni. DIAGNOSTICO: Algunos neonatos con este defecto no tienen manifestaciones
clínicas perceptibles y se desarrollan sin problemas excepto cuando por alguna razón por lo general una infección respiratoria o
problemas gastrointestinales se descubre en un estudio radiológico que tienen hernia diafragmática. El diagnóstico prenatal se realiza
por ecografía, se basa en la visualización de órganos abdominales en el tórax y el signo ecográfico distintivo es una masa ocupada por
líquido inmediatamente por detrás de la aurícula y el ventrículo izquierdos, en la parte inferior del tórax visualizando en una vista
transversal. Otros signos ecográficos que hacen sospechar el diagnóstico son la ausencia del estómago en el abdomen, desplazamiento
del mediastino, perímetro abdominal fetal pequeño y polihidramnios. TRATAMIENTO: El tratamiento incluye terapias como el uso de
corticosteroides, la ventilación de alta frecuencia, oxigenación con membrana extracorpórea (ECMO), terapia con surfactante, óxido
nítrico y cirugía fetal. HERNIA DE BOCHDALEK: Puede ser de presentación esporádica o familiar. El defecto ocurre cuando la membrana
pleuroperitoneal no se fusiona con las otras porciones. La fusión de la membrana pleuroperitoneal ocurre primero del lado derecho y
luego del izquierdo. Además la protección del hígado en el lado derecho, hace que sea más frecuente la hernia de Bochdalek del lado
izquierdo. La hipoplasia pulmonar se debe a que los pulmones se encuentran en la fase glandular del desarrollo y el intestino torácico
ocupando espacio, impide el normal desarrollo. Por lo tanto existe una disminución del lecho vascular y de la segmentación bronquial.
Es frecuente la mal rotación de los intestinos debido a que los intestinos se desplazan al tórax antes de la fijación del ciego al cuadrante
inferior derecho del abdomen. HERNIA DE MORGAGNI: La hernia de Morgagni es una anomalía congénita del diafragma, causada por
un defecto anteromedial del diafragma, entre sus inserciones costal y esternal, que fue descrito por primera vez por Morgagni, Los
pacientes con hernia de Morgagni se diagnostican a cualquier edad en forma accidental al efectuar un examen radiológico de tórax por
otra patología, tal como infecciones respiratorias o molestias gastrointestinales, un considerable número de pacientes se diagnostica en
la edad pediátrica por presentar dificultad respiratoria. El diagnóstico de hernia de Morgagni se lo efectúa en la edad adulta, la mayoría
de las veces en forma incidental, y en los niños por presentar dificultad respiratoria. Su patogenia es desconocida, aunque actualmente
se sugiere una etiología multifactorial en la que se implican factores hereditarios en relación con otros síndromes malformativos como
los síndromes de Down, Turner, Prader-Willi y Nooan. El tratamiento es eminentemente quirúrgico en todos los casos y consiste en la
plastia del defecto diafragmático; hoy en día varios centros han reportado exitosas reparaciones por vía laparoscópica.

CASO CLINICO
Lactante de 5 meses de edad, femenino, eutrófico, con historia de cuadros de aparente dolor abdominal a repetición. Consultó por
presentar síntomas respiratorios, fiebre e irritabilidad, se solicitó radiografía de tórax en proyecciones anteroposterior y lateral que
muestra reducción del parénquima pulmonar en hemitórax derecho y desplazamiento del mediastino y del corazón hacia la izquierda.

PREGUNTA
Cual es la conducta diagnostica siguiente mas adecuada?

RESPUESTA
a.- Serie gastroesofágica.

CURSO ENARM CMN SIGLO XXI TEL: 36246001 Pharmed Solutions Institute PÁGINA 370
MANUAL DE TRABAJO DEL CURSO ENARM CMN SIGLO XXI
b.- Serie gástrica completa.
c.- Tomografia de abdomen.
d.- Resonancia magnética.

CASO CLINICO
Recién nacido varón, que nace por Cesárea, SGB +, Transaminasas Maternas elevadas. Apgar 6/7, Peso 3670 gramos, talla 51
centímetros. Presenta, en el postparto inmediato, dificultad respiratoria con cianosis, aleteo nasal, quejido, tiraje intercostal,
crepitantes bibasales, disminución de entrada de aire y del murmullo vesicular en hemitorax Izquierdo.

PREGUNTA
Cual es la conducta diagnostica mas adecuada?

RESPUESTA
a.- Serie gastroesofágica.
b.- Radiografia de torax.
c.- Tomografia de abdomen.
d.- Endoscopia superior.

PREGUNTA
Cual de las siguientes manifestaciones es menos frecuente en esta patología?

RESPUESTA
a.- Insuficiencia Respiratoria Severa desde el nacimiento.
b.- Disminución o ausencia de murmullo vesicular.
c.- Ruidos hidroaéreos (RHA) en tórax.
d.- Desplazamiento de ruidos cardiacos al lado contralateral.

PREGUNTA
Cual es la trisomía que no se ha relacionado en esta patología?

RESPUESTA
a.- X.
b.- 18.
c.- 21.
d.- 13.

ESTENOSIS HIPERTROFICA DEL PILORO. CIENCIAS BASICAS: La estenosis hipertrófica del píloro anteriormente se conocía como
hipertrofiacongénita del píloro; también se le ha denominado estenosis pilórica hipertrófica infantil, para diferenciarla de la estenosis
adquirida que se observa en el adulto. La estenosis hipertrófica del píloro es la causa más frecuente de cirugía en los lactantes menores
de 6 meses, superada solamente por las hernioplastias. SALUD PUBLICA: Se estima que hay entre uno y cinco pacientes con esta
patología por cada 1,000 recién nacidos vivos. 7% asociado a malformaciones: malrotación intestinal, uropatía obstructiva, atresia
esofágica, hernia hiatal. Se presenta entre las 2 y 8 semanas de edad, con un pico entre las 3 y las 5 semanas. Es 4 a 5 veces más común
entre varones que en mujeres, con una mayor incidencia en primogénitos. PATOGENIA: La causa exacta de la estenosis del píloro no se
ha determinado todavía, pero se han desarrollado algunas teorías relacionadas con un desequilibrio neurohormonal, o de mediadores
neuroendocrinos, en el control del tono del esfínter pilórico, en la producción de gastrina y la motilidad del estómago. Una propuesta
es una descoordinación entre el peristaltismo gástrico y la relajación pilórica, lo que lleva a una contracción gástrica contra un píloro
cerrado, que causaría hipertrofia en el músculo pilórico. Otras teorías proponen una elevación en las concentraciones de gastrina
(hipergastrinemia), debido a un aumento hereditario en el número de células parietales de la mucosa gástrica que llevan a un ciclo de
aumento en la producción de ácido gástrico, contracciones cíclicas periódicas en el píloro y vaciamiento gástrico lento, esto lleva a
hipertrofia e hiperplasia de las fibras musculares del esfínter pilórico. Se ha propuesto herencia autosómica dominante cromosoma 16
q24 y herencia “multifactorial”. Administración de eritromicina/ azitromicina los primeros días de vida, se ha encontrado que la
administración de macrólidos a las madres que dan alimentación al pecho también puede ser un factor para que los lactantes
presenten estenosis pilórica. El hábito de fumar materno se ha reportado como posible factor de riesgo para estenosis pilórica. Otras
investigaciones señalan que al estudiar muestras de las capas musculares depacientes afectados de estenosis pilórica, en comparación
con controles, se hanencontrado cantidades disminuidas de las terminales nerviosas y de los neurofilamentos; disminución en los
marcadores para células de soporte; disminución en las células intersticiales de Cajal; disminución en la actividad de la sintetasa de
óxido nítrico, el cual actúa como relajante del músculo liso en diversos tejidos; disminución en la producción del ARN mensajero para la
sintetasa de óxido nítrico. DIAGNOSTICO: El síntoma más característico son los vómitos posprandiales, no biliosos, progresivos hasta
ser incluso en proyectil, que eventualmente impiden la alimentación adecuada del lactante. Esto lleva a pérdida de peso por
disminución del aporte de calorías y por deshidratación. Por lo general el recién nacido ha tenido previamente un período libre de
vómitos. De los antecedentes es importante indagar por el uso de eritromicina o macrólidos en las dos primeras semanas de vida,
además de los antecedentes familiares. Conforme continúan los síntomas el paciente adelgaza (pierde peso) y se torna
hambriento,toma con avidez la leche, pero rápidamente la vomita; se puede observar el vómito explosivo o en proyectil. Algunos
pacientes presentan ictericia (10%) de grado variable. También se pueden encontrar signos clínicos de deshidratación como mucosas
con saliva filante o secas, depresión de la fontanela, llenado capilar lento y signo del pliegue presente (se suma a la pérdida de peso). El
abdomen no está distendido; cuando la emaciación ha progresado, es posible observar las ondas peristálticas en el epigastrio. El signo

CURSO ENARM CMN SIGLO XXI TEL: 36246001 Pharmed Solutions Institute PÁGINA 371
MANUAL DE TRABAJO DEL CURSO ENARM CMN SIGLO XXI
patognomónico es la masa palpable en el cuadrante superior derecho del abdomen, en forma de aceituna u oliva, que corresponde al
píloro engrosado (oliva pilórica). Para encontrar la oliva es necesario que el paciente esté tranquilo y la musculatura abdominal
relajada. Los porcentajes del hallazgo varían entre los autores y dependen de la experiencia del explorador; algunos indican cifras que
van de 85 a 100 %. Signo del biberón: al tomar líquidos por biberón se observan ondas peristálticas. Signo de la pelota de golf: ondas
peristálticas de izquierda a derecha hacia la zona pilórica. El hallazgo más característico es la alcalosis metabólica, por la pérdida de
hidrogeniones a través del contenido gástrico; con el vómito se pierde también cloruro lo que lleva a hipocloremia, que completa el
panorama de la bioquímica sanguínea. También puede haber hipokalemia. Se puede solicitar hemograma completo para descartar
algún proceso séptico como causa de los vómitos. En la estenosis del píloro el hemograma está normal; se describe sin embargo que
puede haber hemoconcentración por la deshidratación. Por ello puede encontrarse elevación de la creatinina; la densidad urinaria
también puede estar elevada, sin datos de infección. Otro hallazgo es la elevación de la bilirrubina indirecta, que se puede explicar por
la disminución del tránsito gastrointestinal que produce aumento de la circulación enterohepática de bilirrubina, aunque otros autores
mencionan disminución de la actividad de la glucoroniltransferasa por el ayuno. La radiografía simple de abdomen muestra distensión
de la cámara gástrica con poco o escaso aire distal. La serie esófago gastroduodenal muestra un estrechamiento del canal pilórico (cola
de ratón) con una o varias imágenes de «cuerda»; es el medio de contraste que dibuja los pliegues hipertrofiados de la mucosa pilórica;
se describe también un efecto de masa que se proyecta hacia el antro gástrico (signo del hombro); se observa falta de progresión de las
ondas peristálticas del estómago en la fluoroscopia. En la actualidad, el método más utilizado para confirmar el diagnóstico clínico es el
ultrasonido abdominal. Se logra evidenciar engrosamiento de la capa muscular del píloro mayor de 2.5 mm, aumento en la longitud del
canal pilórico mayor de 15 mm y estrechamiento del canal; se logra visualizar también hipertrofia de la mucosa en grados variables,
corte axial Imagen “ojo de bovino” “dona” Imagen tiro al blanco. La endoscopia tiene sensibilidad y especificidad 100%. TRATAMIENTO:
El tratamiento actualmente es quirúrgico. Sin embargo se deben corregir los trastornos hidroelectrolíticos (alcalosis, hipocloremia,
hipokalemia y deshidratación) antes de que el paciente sea llevado al quirófano. Se pueden suministrar bolos de solución de cloruro de
sodio al 0,9 % (solución salina normal) a 20 mL/kg para restituir volumen y electrolitos. Luego se puede dejar una solución de
mantenimiento, con dextrosa al 5 % y cloruro de sodio al 0,45 % o al 0,9 %, según el centro hospitalario, además se debe agregar
cloruro de potasio a razón de 20 mEq/L (3.4mEq/kg/día) una vez asegurada la diuresis. La estabilización puede demorar unas 24 a 48 h.
La cirugía no es urgente; si el paciente es llevado a sala de operaciones con alcalosis metabólica se aumenta el riesgo de apnea
posoperatoria; además, el estrés quirúrgico puede empeorar los trastornos electrolíticos. Actualmente es raro observar desnutrición
grave como en años atrás, cuando el diagnóstico se retrasaba. Se debe mantener una sonda nasogástrica a drenaje que ayuda a
descomprimir el estómago, previene la aspiración posoperatoria y también la atonía gástrica. Niño con electrólitos normales menos del
5% de la deshidratación cirugía inmediata. Vitamina K: 0.4 mg/kg en niños con peso inferior a 2.5 kg. > 1 año 5-10 mg/día por vía I.V. o
I.M. Ranitidina 1 mg/kg/dosis cada 8 hrs. Una vez compensado el paciente se lleva a cabo la Piloromiotomía de Fredet- Ramsted, se
realiza el corte de la serosa y luego de la capa muscular circular del píloro, sin cortar la mucosa; luego se deja sin suturar la muscular y
se cierra la serosa. La operación generalmente se hace por laparotomía, aunque también se puede a través de técnicas laparoscópicas.
COMPLICACIONES: Las complicaciones están relacionadas con los trastornos electrolíticos y con la cirugía. Se citan principalmente:
apnea posoperatoria, hipoglicemia, obstrucción posoperatoria (vómitos), perforación de la mucosa (duodenal), miotomía incompleta,
eventración de la herida quirúrgica; el fallecimiento ocurre en menos del 1 % de los casos

CASO CLINICO
Se trata de un paciente masculino de cuatro semanas de edad, el cual acude por historia de 2 dias de evolución caracterizado por
vomitos postpandriales abundantes, se establecen medidas antireflujo, sin embargo continuo la sintomatología aumentando el vomito
en proyectil de contenido alimentario.

PREGUNTA
Cual es la conducta diagnostica mas adecuda en este momento?

RESPUESTA
a.- Radiografia de torax.
b.- USG abdominal.
c.- Radiografia de abdomen.
d.- Endoscopia.

PREGUNTA
Cual de los siguientes diagnosticos diferenciales es el mas frecuente en esta patología?

RESPUESTA
a.- Intolerancia a la lactosa.
b.- Reflujo gastroesofágico.
c.- Meningitis.
d.- Volvulus.

CURSO ENARM CMN SIGLO XXI TEL: 36246001 Pharmed Solutions Institute PÁGINA 372
MANUAL DE TRABAJO DEL CURSO ENARM CMN SIGLO XXI

CARDIOPATIAS CONGENITAS
CIENCIAS BASICAS: Corazón primer órgano que alcanza desarrollo (de 3ra-8va semana de gestación) funcional completo, el día 17 de
gestación escuchamos latido cardiaco. Definición: Conjunto de enfermedades caracterizadas por anomalías estructurales del corazón o
de los grandes vasos intratorácicos. Los cambios en la transición de la circulación fetal a la neonatal; en el feto existen 3 estructuras
únicas: el conducto venoso, el agujero oval y el conducto arterioso. La sangre oxigenada llega de la vena umbilical a la vena cava inferior
a través del conducto venoso, evitando el hígado. Gracias a la anotomía de la aurícula derecha, la mayor parte de la sangre proveniente
de la vena cava inferior (sangre oxigenada) se dirige al corazón izq. Por medio del agujero oval, mientras que la sangre proveniente de
la vena cava superior (sangre desoxigenada), se dirige al ventrículo derecho a través de la válvula tricúspide. La sangre que irriga el
pulmón tienen un contenido de oxigeno sumamente bajo y esto produce una constricción intensa de los vasos pulmonares que eleva la
resistencia vascular. El ventrículo der. bombea la sangre hacia la arteria pulmonar; sin embargo la alta resistencia pulmonar hace que la
sangre fluya principalmente a través del conducto arterioso hacia la aorta descendente. Los cambios fisiológicos y mecánicos que
ocurren al nacer y que propician el cierre del conducto arterioso son: la disminución de la resistencia pulmonar (por la distención
alveolar y el incremento del contenido de oxígeno en la sangre), la secreción de bradicinina por el endotelio pulmonar y disminución de
la contracción sérica de las prostaglandinas placentarias. Junto con el flujo pulmonar, estos cambios incrementan el volumen y la
presión en la aurícula izq., rebasando la de la aurícula derecha y cerrando el agujero oval. SALUD PUBLICA: Su incidencia es del 1%, lo
que las coloca entre las malformaciones más frecuentes. En cerca de 90% de los casos no existe una causa identificable, pero existen
ciertos factores como como la prematurez, cromosomopatías, infecciones congénitas, diabetes, alcohol, entre otros.
CLASIFICACION:
CARDIOPATIAS POR FISOPATOGENIA (por orden de incidencia)
Obstructivas CORTOCIRCUITOS
Coartación de la Derecha a izquierda (cianóticas) Izquierda a derecha (acianóticas)
aorta (5-7) Sangre pobremente oxigenada pasa a la circulación sistémica, Propician aumento del volumen circulatorio
Estenosis pulmonar ello condiciona a hipoxemia y cianosis en el sistema pulmonar, a expensas de una
Estenosis aortica disminución del gasto cardiaco sistémico, no
cianosis pero si hipertrofia del VD y vasos
pulmonares
Flujo pulmonar aumentado Flujo pulmonar disminuido Comunicación interventricular CIV (25-30)
Transposición de grandes vasos Tetralogía de Fallot (5-7) Comunicación interauricular CIA (6-8)
(3-5) Atresia tricuspidea Conducto arterioso persistente CAP (6-8)
Tronco arterioso (1-2) Atresia pulmonar Defecto de cojinetes endocardicos
Ventrículo único
Hipoplasia de VI

TETRALOGIA DE FALLOT: Cardiopatía congénita más común de las cianóticas (3-6 por 10,000). Implica obstrucción del tracto de salida
del VD (estenosis pulmonar); comunicación interventricular (CIV); hipertrofia del VD y dextroposición de la aorta con cabalgamiento
sobre la CIV, todo esto lleva a obstrucción del flujo del VD, hacia la arteria pulmonar y cortocircuito de derecha a izquierda. Leve no
muestran cianosis hasta después del año, en grave cianosis poco después de nacimiento, de no corregirse el paciente presenta color
azulado, acropaquias de pies y manos (dedos en palillo de tambor) y disnea, pueden sufrir ataques hipercianóticos paroxísticos con el
esfuerzo que lo pueden llevar al sincope, si se ponen en cunclillas mejora su estado. La viabilidad del producto depende del conducto
arterioso y del agujero oval. DIAGNOSTICO: Las Rx: corazón con aspecto de bota, trama vascular pulmonar disminuida, ECG: hipertrofia
del VD y desviación del eje eléctrico, Ecocardiograma: “Estándar de oro”, para dx y gravedad. TRATAMIENT : Cirugía correctiva,
inicialmente PG E1 para mejorar síntomas. TRANSPOSICION DE LOS GRANDES VASOS: Aquí la arteria pulmonar nace del VI y la aorta
del VD. La cianosis y la taquipnea suelen ser evidentes en las primeras horas o días de vida y representan una urgencia médica.
DIAGN STIC : La Rx: normal o datos de hiperflujo pulmonar. Ecocardigrafía es “estándar de oro” para dx. TRATAMIENT : Dar PG E1
para mantener abierto el conducto arterioso, la corrección quirúrgica final es la conmutación o switch arterial. TRONCO ARTERIOSO:
Defecto en el desarrollo del tronco arterioso en aorta y arteria pulmonar, dejando un tronco arterial único que nace del corazón, para
ambas circulaciones. COMUNICACIÓN INTERVENTRICULAR: Cardiopatía congénita más común en la infancia, durante los dos primeros
años de vida casi la mitad de estos defectos se cierran espontáneamente y la mayoría lo hace para los 10 años. Se dividen en;
membranoso (más común) y muscular. El cuadro clínico depende de la cantidad de sangre que fluye hacia la circulación pulmonar, en la
mayoría es pequeño y permanecen asintomáticos. DIAGNOSTICO: Defectos grandes disnea, dificultad para la alimentación, infecciones
pulmonares recurrentes e IC. Auscultación soplo holosistólico, a veces frémito. La Rx: cardiomegalia y aumento de trama vascular. La
CIV muscular es menos común y su cierre espontaneo es más probable. TRATAMIENTO: Cirugía, solo si hay retraso en el crecimiento,
grandes defectos o hipertensión pulmonar. COMUNICACIÓN INTERAURICULAR: Cardiopatia congénita más comun en el adulto (1 de
cada 3 casos). En cualquier parte del septo, los principales tipos son: ostium secundum (90%), ostium primum y seno venoso, el grado
de cortocircuito y mezcla de sangre depende del tamaño del defecto, generalmente no presentan síntomas hasta 4ta década de vida, la
presentación más común es la falla cardiaca derecha. DIAGNOSTICO: Auscultación, S2 con desdoblamiento fijo, por hiperflujo
pulmonar, un soplo de eyección de bajo grado. TRATAMIENTO: Corrección quirúrgica de defectos grandes, contraindicada cuando la
relación de las resistencias arteriales pulmonar: sistémica es mayor a 0.7. Requiere profilaxis para endocarditis bacteriana. CONDUCTO
ARTERIOSO PERSISTENTE: Su cierre normal ocurre entre 4-10 días de nacimiento, si no sucede la sangre aortica se desvía hacia la
arteria pulmonar, el cierre después de la infancia es infrecuente, el principal factor de riesgo es la premadurez e infección materna por
rubeola. DIAGNOSTICO: los conductos grandes pueden producir presión de pulso amplio, pulsos periféricos saltones, insuficiencia
cardiaca y retraso en el crecimiento a largo plazo. Auscultación: soplo característico es sistólico; suele describirse como “ruido de
maquinaria”. La Rx: incremento en la trama vascular y una arteria pulmonar ensanchada. Ecocardiograma estudio de elección.
TRATAMIENTO: Las probabilidades de que cierre espontáneamente son escasas, para ayudar a que se cierre se da indometacina
(inhibidor de la sintesisi de PG). En casos extremos es necesaria la cirugía, ya que disminuye el riesgo de endocarditis infecciosa.

CURSO ENARM CMN SIGLO XXI TEL: 36246001 Pharmed Solutions Institute PÁGINA 373
MANUAL DE TRABAJO DEL CURSO ENARM CMN SIGLO XXI

COARTACION DE LA AORTA: Causada por el estrechamiento de la aorta en algún punto desde el cayado hasta la bifurcación de las
arterias iliacas. Este trastorno se relaciona con válvulas aorticas bicúspides (70%). Relación importante con el Sx. de Turner. Hay dos
tipos: coartación juxtaductal antes conocida como del adulto y la infantil (coartación preductal) DIAGNOSTICO: Cuadro clínico depende
de la localización; la presión arterial esta aumentada en los vasos proximales a la coartación y conlleva una disminución distal a la
estenosis, signo clásico es la disparidad de la presión arterial entre los brazos y las piernas. La Rx: congestión pulmonar, cardiomegalia.
Ecografía cardiaca “estándar de oro” para dx. El tratamiento de la forma infantil se administra PG E1, la cirugía correctiva es necesaria,
en la juxtaductal se deben controlar la hipertensión y la falla cardiaca y después realizar la cirugía. PREVENCION: La profilaxis está
indicada en todos los casos de cardiopatías cianóticas, durante los primeros 6 meses después de una reparación quirúrgica completa de
un defecto y en todos los casos de reparaciones incompletas en las que exista material prostético cerca del defecto. Las opciones mas
comunes son: 1. Amoxicilina 2 gr VO; 1 hr antes de procedimiento quirúrgico. 2. Ampicilina 2gr IV en caso de intolerancia a la VO. 3.
Macrólidos o cefalosporinas de 3ra generación en alegría a las penicilinas. PRONOSTICO: Los avances en el tratamiento médico y
quirúrgico de las cardiopatías congénitas han permitido que los pacientes que las padecen lleguen a la vida adulta

CASO CLINICO
Una recién nacida (RN) de 19 días de vida es admitida en la sala de emergencias en estado grave. En el examen físico, la paciente se
presentaba hipotónica, con hipotermia, gimiente, cianótica con aleteo de la nariz, taquipneica (80 rpm), con tiraje y severa retracción
subcostal y esternal, murmullo vesicular (MV) presente con crepitantes difusos, roncus, taquicárdica (190 lpm), pulso débil, ritmo
cardíaco regular (RCR), ruidos cardíacos normofonéticos (RCNF), soplo sistólico +++/4+, abdomen distendido, hígado a 4 cm por debajo
del reborde costal derecho. La paciente es remitida en la Unidad de Cuidados Intensivos Neonatales y se realizó intubación orotraqueal
(IOT), con FiO2 al 100%, estableciéndose un acceso periférico e iniciándose corrección con solución de bicarbonato de sodio debido a la
presencia de acidosis metabólica, disección venosa de la vena axilar derecha y la siguiente medicación: furosemida, dobutamina,
milrinona, fentanilo y midazolam. Se realizaron exámenes (PCR negativa, cultivo de orina con presencia de Staphylococcus coagulasa
resistente a ampicilina/penicilina), se realizó una radiografía de tórax que mostró la presencia de cardiomegalia.

PREGUNTA
Cual es su diagnostico mas probable?

RESPUESTA
a.- CIV.
b.- CIA.
c.- CAP
d.- TGV

CURSO ENARM CMN SIGLO XXI TEL: 36246001 Pharmed Solutions Institute PÁGINA 374
MANUAL DE TRABAJO DEL CURSO ENARM CMN SIGLO XXI

KWASHIORKOR. CIENCIAS BASICAS: Forma de desnutrición que existe cuando no hay suficiente proteína en la dieta, es agudo. La
palabra ' washior or', fue usada inicialmente en Ghana y traduce “enfermedad del niño destetado”, término utilizado por las madres
para describir la enfermedad del primer hijo cuando nace el segundo. La desnutrición tipo kwashiorkor (edematosa) es más frecuente
en lactantes con déficit de proteínas pero adecuada ingesta de carbohidratos y se caracteriza por depleción de proteínas séricas,
principalmente la albumina, que tiende a ser más severa en esta forma de desnutrición. La gravedad o grado de desnutrición se
determina siguiendo la clasificación de la OMS, basada en el grado de pérdida de peso: Desnutrición grado I, hay pérdida del 15-25% de
peso. Grado II, hay pérdida de 25-40% de peso. Grado III, hay pérdida de más de 40% de peso. SALUD PUBLICA: Este tipo de DNT es
más común en las zonas rurales de países en vías de desarrollo y afecta principalmente a los niños de 2 años de edad, asociado al
periodo de destete e inicio de dieta rica en almidón y pobre en proteínas. Afecta aproximadamente una décima parte de los niños
<5años. PATOGENIA: Es una condición compleja, multifactorial, en la cual existe una combinación de factores geográficos, climáticos,
educacionales, psicosociales, culturales, nutricionales, y otros factores. Puede ser considerado el punto final de la combinación de
todos estos en un organismo que no tiene la capacidad de adaptarse a un cambio de condiciones, que lleva a que se desarrollen las
características clínica. Cuando un niño nace, recibe ciertos aminoácidos vitales para el crecimiento procedentes de la leche materna.
Cuando el niño es destetado, si la dieta que reemplaza a la leche tiene un alto contenido en carbohidratos, y es deficiente en proteínas,
como es común en diferentes partes del mundo donde el principal componente de la dieta consiste en almidones vegetales, o donde el
hambre hace estragos, los niños pueden desarrollar Kwashiorkor. El origen del edema en el paciente con kwashiorkor es multifactorial.
La teoría clásica postula que una ingesta inadecuada de proteína lleva a bajas concentraciones de albumina en plasma que llevan a
edema. Sin embargo, esta hipótesis ha cambiado, pues se ha encontrado que no existe diferencia en la concentración de albumina en el
plasma antes y después de la disminución del edema. El edema nutricional está asociado con un aumento en la secreción de hormona
antidiurética (ADH) que previene la respuesta excretora normal a la administración de agua. La ferritina activada es liberada por el
hígado dañado, llevando a aumento de la secreción de ADH cuyo resultado neto es la retención de agua. La aldosterona también es
normalmente inactivada en el hígado, y se ha reportado un metabolismo inadecuado en los pacientes con enfermedades hepáticas,
llevando a hiperaldosteronismo que podría ser el responsable de la retención de sodio. Otras características intrínsecas del individuo
como variaciones en isoenzimas o en concentración de enzimas podrían influir. Altos niveles de glutamato y bajos o indetectables de
alanina son la característica del kwashiorkor, asociado a menores niveles de aminotransferasas. Es posible que estas diferencias
influyan en la disímil respuesta frente al estrés de una dieta inadecuada y pueden ser responsables del desarrollo de kwashiorkor.
DIAGNOSTICO: Clínico; es anoréxico, irritable, con facies abotagada “cara de
luna llena”, edema peri orbitario y edema con fóvea en extremidades inferiores
y manos, abdomen globoso con hepatomegalia (infiltración grasa del hígado;
hígado graso)) y dilatación de asas intestinales. La piel es seca (dermatosis) y el
cabello se cae fácilmente, perdida del interés y del apetito. El peso
generalmente es adecuado para la edad. El edema, signo cardinal del
kwashiorkor, típicamente inicia en el dorso de los pies y en las piernas. Después
se extiende a otras partes del cuerpo incluyendo manos, antebrazos, espalda,
extremidades superiores y en casos severos en la cara, principalmente en las
mejillas y alrededor de los ojos. Generalmente corresponde al 5-20% del peso
corporal. Puede desarrollarse ascitis (por ausencia de proteínas en la sangre) o
derrame pleural como manifestaciones tardías del edema. Otros cambios
fisiológicos incluyen atrofia del musculo cardiaco con disminución del gasto
cardiaco, insuficiencia circulatoria y bradicardia. Tienen elevación de TSH
(hormona estimulante tiroides) con función tiroidea normal, pero con
disminución de proteínas plasmáticas que se unen a la tiroxina que llevan a bajas
concentraciones plasmáticas de tiroxina .Las células de la mucosa intestinal y del
páncreas se encuentran atróficas llevando a alteración de la síntesis y actividad de disacaridasas, disminución del área de absorción que
se traduce en alteración en la utilización de nutrientes, e intolerancia a la lactosa .Presentan además disminución de la tasa de filtración
glomerular, menor capacidad de concentrar la orina, trastornos hidroelectrolíticos principalmente hipokaliemia, atrofia del timo con
compromiso de la respuesta inmunológica principalmente la medicada por células que lleva a mayor susceptibilidad a infecciones .La
hipoglicemia es un fenómeno común en pacientes con kwashiorkor, aunque también se ha descrito en pacientes con emaciación
severa. Teóricamente la hipoglicemia puede estar causada por alteración en la producciónhepática endógena de glucosa o por aumento
de la eliminación de la misma. Esta complicación aporta de manera importante a la mortalidad. Para realizar el diagnóstico de
desnutrición severa, la OMS define malnutrición aguda severa como una circunferencia superior del brazo (MUAC por sus siglas en
ingles) < 11.5 cm, un puntaje z de peso para la talla (P/T) < -3 o la presencia de edema bilateral pedio (kwashiorkor). En ausencia de
evaluación antropométrica, también puede diagnosticarse desnutrición severa por la presencia de emaciación severa visible, definida
como la presencia de emaciación muscular en la región glútea, perdida de la grasa subcutánea o prominencia de las estructuras óseas
particularmente en el tórax Se debe tener en cuenta que el peso puede estar afectado por diferentes variables como el edema o la
deshidratación, ambos presentes con alta frecuencia en estos pacientes, y que pueden llevar a clasificaciones inadecuadas si solo se
tiene en cuenta este parámetro. En las alteraciones bioquímicas, las más comunes son los bajos niveles de albumina y hemoglobina.
TRATAMIENTO: Generalmente, la enfermedad puede ser tratada añadiendo a la comida alimentos energéticos y proteínas; sin
embargo, la letalidad puede ser tan alta como del 60% y puede haber secuelas a largo plazo como niños con talla corta, y en casos
severos, desarrollo de retraso mental. El manejo en casa se recomienda para aquellos pacientes sin complicaciones. Los niños con
kwashiorkor severo o asociado a complicaciones requieren tratamiento intrahospitalario. Generalmente 5-30% de los niños requieren
hospitalización. Como las principales causas de muerte son la deshidratación, alteraciones electrolíticas, hipoglicemia e infecciones, el
tratamiento debe empezar con la corrección de estas. La sepsis se presenta en el 15-60% de los niños con malnutrición severa
complicada, por lo tanto es necesario iniciar antibióticos de amplio espectro. Estos pacientes generalmente tienen niveles bajos de
hemoglobina (6-10 g/dl), aunque rara vez tienen perdida aguda de sangre. La transfusión puede empeorar la falla cardiaca y se ha

CURSO ENARM CMN SIGLO XXI TEL: 36246001 Pharmed Solutions Institute PÁGINA 375
MANUAL DE TRABAJO DEL CURSO ENARM CMN SIGLO XXI
identificado como un factor de riesgo para muerte. La OMS recomienda transfundir solo si la hemoglobina es < 4 g/dl . Debe
administrarse suplementos de vitaminas A, D, ácido fólico y algunas del complejo B. En cuanto a la rehabilitación nutricional, la OMS en
el año 2000 definió las orientaciones para el manejo hospitalario de estos niños, las cuales periódicamente han sido actualizadas. Se
basa en recomendar formulas especiales (F75/F100).

CASO CLINICO
Niño de 3 años de muy bajo nivel socioeconómico, Cursa desde hace dos años con poliuria, polidipsia, astenia, y perdida de peso.
Ingresa por presentar estado de estupor y signos de deshidratación severa. Peso: 9 kg, Talla: 92 cm. Frecuencia respiratoria: 25 por
minuto, Tº corporal y PA: normal. Malas condiciones generales, marcada palidez muco-cutánea, perdida de tejido subcutáneo,
secreción purulenta en conductos auditivos, lesiones ulcerosas en lengua, membranas blanquecinas en cavidad oral, quelitis sangrante.
Abdomen globoso, distendido sin megalias, edema +++.

PREGUNTA
Considerando el estado de desnutrición y las complicaciones que potencialmente desarrollará, cual de los siguientes es el mas
probable?

RESPUESTA
a.- Alcalosis metabolica
b.- Acidosis metabolica
c.- Acidosis respiratoria.
d.- Alcalosis respiratoria.

PREGUNTA
Cual es su grado de desnutrición?

RESPUESTA
a.- Grado I.
b.- Grado II.
c.- Grado III.
d.- Grado IV.

PREGUNTA
Cual es la complicación endocrinológica mas probable para el caso?

RESPUESTA
a.- Hipogonadismo hipogonadotrofico.
b.- Diabetes mellitus.
c.- Hipotiroidismo yodoprivativo.
d.- Cetoacidosis diabética.

MARASMO. CIENCIAS BASICAS: Es una forma de desnutrición crónica (no edematosa) caracterizada clínicamente por emaciación
severa (flaqueza exagerada) del tejido subcutáneo, músculos y grasa, que da lugar a un aspecto, envejecido o arrugado, debida a la
carencia de proteínas, su causa primaria es el aporte inadecuado de calorías en la dieta alimentaria, si diferencia en la relación
proteínas/energía. Un niño con marasmo aparece escuálido y su peso corporal puede reducirse hasta menos del 80% de su peso
normal para su altura. SALUD PUBLICA: Habitualmente en menores de 18 meses. PATOGENIA: Como la disminución del aporte
energético no puede compensar el requerimiento, se utiliza grasa corporal como sustrato de energía, con la consiguiente disminución
del tejido celular subcutáneo. Como hay un déficit de aporte tanto de proteínas como
de energía, el fenómeno de adaptación eleva niveles de glucagón y cortisol y
disminuye la insulina con los efectos orgánicos pertinentes. Los músculos son los más
afectados por la expoliación ya que proporcionan los aminoácidos esenciales, para el
mantenimiento de la síntesis proteica visceral y así producir cantidades adecuadas de
albumina sérica y beta-lipoproteínas, que provienen de la presencia de edemas o
infiltración de grasa del hígado. En el intestino delgado existe daño de la mucosa de
carácter transitorio con alteraciones mitóticas. Hay déficit de producción de tripsina,
quimiotripsina, amilasa y lipasa, originando una malabsorción. Quizás las causas
precipitantes más importantes del marasmo son las infecciones y enfermedades
parasitarias de la infancia. Estas incluyen sarampión, la tosferina, diarrea, malaria, y
otras debidas a parásitos. Las infecciones crónicas como la tuberculosis pueden
también llevar al marasmo. Otras causas comunes del marasmo son el parto
prematuro, la deficiencia mental y las molestias digestivas, como malabsorción o
vómito. Una causa muy común es también la interrupción temprana de la lactancia.
DIAGNOSTICO: Crecimiento deficiente: En todos los casos el niño no crece en forma
adecuada. Si se conoce la edad, el peso será muy bajo. En los casos graves la pérdida
muscular es obvia: las costillas sobresalen; el estómago, en contraste con el resto del cuerpo, puede ser protuberante; la cara tiene una
característica simiesca (como un mono); y las extremidades inferiores son muy delgadas. El niño parece ser sólo piel y huesos. Un caso
avanzado de la enfermedad es inconfundible, y una vez que se ve, jamás se olvida. Emaciación: Los músculos siempre se encuentran

CURSO ENARM CMN SIGLO XXI TEL: 36246001 Pharmed Solutions Institute PÁGINA 376
MANUAL DE TRABAJO DEL CURSO ENARM CMN SIGLO XXI
muy disminuidos. Hay poca grasa subcutánea, si es que queda algo. La piel cuelga en arrugas, sobre todo alrededor de las nalgas y los
muslos. Cuando se toma la piel entre el índice y el pulgar se nota la ausencia de la capa habitual de tejido adiposo. Estado de alerta: Los
niños con marasmo raramente son desinteresados como los que sufren kwashiorkor. Los ojos profundamente hundidos les dan una
apariencia bastante despierta. También se puede manifestar en modo menos infeliz e irritable. Apetito: El niño por lo general tiene
buen apetito. En realidad, como cualquier individuo hambriento, el niño puede ser voraz. Los niños con marasmo a menudo se chupan
las manos violentamente o la ropa o cualquier otra cosa a su alcance. Algunas veces emiten sonidos de succión. Anorexia: Algunos
niños son anoréxicos. Puede aparecer pero solo algunos casos, muy pocos. Diarrea: La materia fecal puede ser suelta, pero no es una
característica constante de la enfermedad. La diarrea de naturaleza infecciosa, como ya se mencionó, puede comúnmente haber sido
un factor precipitante. Anemia: Casi siempre se encuentra anemia. Ulceraciones en la piel: Puede haber úlceras por presión, pero por lo
general están sobre las prominencias óseas, no en áreas de fricción. En contraste con el kwashiorkor, no existe edema y en el marasmo
no hay dermatosis en copos de pintura. Cambios del cabello: Puede haber cambios semejantes a los del kwashiorkor. Es más común un
cambio en la textura que en el color. Deshidratación: Aunque por sí misma no es una característica de la enfermedad, es común que la
deshidratación acompañe al marasmo; como resultado de una fuerte diarrea (y algunas veces del vómito). TRATAMIENTO: Es esencial
tratar no sólo los síntomas sino también las complicaciones de estos desórdenes tales como infecciones, deshidratación y trastornos del
aparato circulatorio que frecuentemente son letales y provocan una alta mortalidad si son ignorados. Consiste en la aplicación de una
dieta equilibrada, en la que el componente más importante sea la leche, que en un principio se administra de forma diluida y poco a
poco se va haciendo más concentrada. Una vez que la ingestión de proteínas es adecuada, se añaden calorías en forma de azúcar y
cereales. El tratamiento debe ser establecido poco a poco para lograr la readaptación de las funciones metabólicas e intestinales en los
seres humanos.

CASO CLINICO
Un varón de 6 meses de edad, nacido a término, que presenta desnutrición severa. Nació de una madre alimenta normalmente, tras un
embarazo sin complicaciones y peso al nacer 2,680 g,). Exclusivamente amamantado y parecía estar bien alimentado. En el segundo
mes de vida, empezó a mostrar falta de crecimiento, sin antecedentes de vómitos o diarrea. Dos semanas más tarde, desarrollo
erupcion difusa no pruriginos. A la exploración se oservo pelo escaso con un aspecto plateado rubio. Se observa irritable, con tos no
productiva. Estaba alerta pero caquéctico severamente. Su peso (3.245 kg), longitud (51,5 cm), y circunferencia de la cabeza (37,5 cm).

PREGUNTA
Cuál de lo siguientes hallazgos de laboratorio es menos probable encontrar.

RESPUESTA
a.- Anemia normocítica.
b.- Leucopenia.
c.- Hipoalbuminenia.
d.- Neutrofilia.

PREGUNTA
Cuál de las siguientes dosificaciones esta contraindicada por generar mayor deterioro respecto al estado nutricional.

RESPUESTA
a.- Vitamina A 5mg primera dosis luego 1mg/dia.
b.- Zinc 6mg /kg/dia.
c.- Cobre 0.3 mg/Kg/dia.
d.- Hierro 3mg/Kg/dia. A partir de la segunda semana.

PREGUNTA
Cual de las siguientes complicaciones se presenta más frecuentemente en el paciente desnutrido que por general pasa desapersibidas.

RESPUESTA
a.- Infección de vías urinarias.
b.- Otitis media.
c.- Tuberculosis.
d.- Guardiasis.

CASO CLINICO
Un varón de 11 meses alimentado exclusivamente a seno materno, vive en zona rural y es el ultimo hijo de 8 en total, madre dedicada
al hogar con educación primaria, reingresó 10 días después de un episodio previo de gastroenteritis con diarrea durante 2 días y
deshidratación, Después de la rehidratación estaba por debajo del 3er percentil para el peso. 2 semanas después de la admisión la
diarrea es severa. Una semana más tarde la diarrea era con sangre, fiebre. Egresa una semana después de tratamiento
intrahospitalario.

PREGUNTA
Cual es la conducta a seguir.

RESPUESTA
a.- Suplementar alimentación.

CURSO ENARM CMN SIGLO XXI TEL: 36246001 Pharmed Solutions Institute PÁGINA 377
MANUAL DE TRABAJO DEL CURSO ENARM CMN SIGLO XXI
b.- Integración a la dieta familiar.
c.- Alimentacion cada dos horas suplementada.
d.- Referencia a segundo nivel.

PREGUNTA
El paciente anterior reingresa por presentar irritabilidad, llanto sin lagrimas e incontrolable, hipotónico, hipotérmico con lienzo húmedo
y reporta hipoglucemia la cual es controlada oralmente. Considerando la comorbilidad de la desnutrición cual es la conducta
profiláctica a seguir.

RESPUESTA
a.- Administrar trimetropin – sulfametoxazol.
b.- Administrar cefotaxima.
c.- Administrar ampicilina más gentamicina.
d.- Administrar cloranfinicol IV.

PREGUNTA
Considerando el estado hídrico del paciente, los antecedentes y la comorbilidad cual es su manejo de mantenimiento de liquidos.

RESPUESTA
a.- 100 ml/kg/dia.
b.- 130 ml/kg/dia.
c.- 160 ml/Kg/dia.
d.- 180 ml/Kg/dia.

PREGUNTA
El paciente es egresado por mejoría después de 7 dias de tratamiento, con respecto a la recuperación ponderal usted suplementa
micronutrientes y aporte proteico de 1 a 1.5 g/kg/dia hasta 4 a 6 g/kg/dia. Cuál es el incremento ponderal que esperaría usted con las
indicaciones al egreso considerando que lo debe de citar semanalmente.

RESPUESTA
a.- 100 g semanal.
b.- 150 g semanal.
c.- 200 g semanal.
d.- 250 g semanal.

CURSO ENARM CMN SIGLO XXI TEL: 36246001 Pharmed Solutions Institute PÁGINA 378
MANUAL DE TRABAJO DEL CURSO ENARM CMN SIGLO XXI

AVITAMINOSIS:
CIENCIAS BASICAS: Déficit de vitaminas, ya sea por falta de ingesta, absorción inadecuada a nivel intestinal, su mala utilización
metabólica o el aumento de la demanda (como en el embarazo por ejemplo). Como las vitaminas participan en las reacciones como
coenzimas, diversos procesos metabólicos podrían verse afectados. Ejemplos son el raquitismo por déficit de vitamina D, ceguera
nocturna por déficit de vitamina A. Las vitaminas son nutrientes esenciales, orgánicos, de bajo peso molecular, imprescindibles para el
crecimiento, desarrollo y mantenimiento del organismo, y que el hombre necesita adquirir a través de los alimentos. Aunque los
alimentos de origen animal pueden ser fuentes inmediatas de vitaminas, las plantas son sus principales suministradores, a causa de su
mayor capacidad de síntesis de los precursores metabólicos de las vitaminas. Los requerimientos de vitaminas dependen de la
composición de nutrientes de la dieta y de las diferentes condiciones de vida del individuo. Asimismo, deben adecuarse a las
poblaciones en particular, a sus características físicas, gastos de energía, fuentes alimentarias, hábitos, actitudes alimentarias y a las
deficiencias nutricionales detectadas. Así, por ejemplo, las pérdidas de vitaminas por sudor pueden alcanzar cifras importantes en
países tropicales. También algunas vitaminas se necesitan en mayores cantidades por individuos que realizan trabajos físicos intensos,
así como durante el embarazo, la lactancia y el crecimiento, y en estados febriles y algunas enfermedades crónicas. Las vitaminas son
absorbidas, en general, de 20 a 95 %; así por ejemplo, la vitamina A se absorbe de un 70 a 90 %, la vitamina D entre 80 y 90 %, la
vitamina C entre 80 y 95 % y la vitamina B6 entre 95 y 98 %, mientras que la vitamina E solo se absorbe entre 20 y 40 % y los carotenos
entre 20 y 50 %.
VITAMINAS LIPOSOLUBLES
VITAMINA FUNCION DEFICIENCIA FUENTES DIETETICAS
A Intervienen en la percepción visual. XEROFTALMIA (caracterizada por una sequedad del globo ocular y una falta Vegetales verdes,
Necesaria para el mantenimiento de los de brillo en la superficie ocular, se acompaña de una leve pérdida de visión, legumbres frescas
tejidos epiteliales especialmente por la noche). La carencia de vitamina A es una enfermedad
general que afecta a células y órganos de todo el cuerpo; los cambios
resultantes de la arquitectura epitelial reciben la denominación de
"metaplasia queratinizante", la cual se produce con relativa rapidez en las
vías respiratorias y urinarias. Según estimaciones de la OMS, alrededor de
250 000 000 de niños menores de 5 años sufren carencia de vitamina A en
todo el mundo. En los países en desarrollo la carencia de este nutriente tiene
carácter grave como para provocar daños oculares permanentes en unos 13
000 000 de niños y ceguera a 5 000 000, causante de 25 % de las muertes en
la población infantil;
D Estimula la absorción intestinal de Ca. RAQUITISMO: Caracterizada por reblandecimiento de los huesos del cráneo, Leche, huevos,
Condiciona depósito de Ca y P en los asociado generalmente a un agradamiento exagerado de este, el tórax se mantequilla
huesos aplasta trasversalmente, el esternón es prominente y las articulaciones de la
rodilla se abultan; en las extremidades inferiores, por la acción del peso
corporal, aparecen incurvaciones de los huesos largos y desviaciones del eje
longitudinal de los mismos. Esta deformación se manifiesta cuando los niños
empiezan a andar; es frecuente la aparición de fracturas de hueso y la
movilidad de las articulaciones es muy amplia debida a la laxitud de los
ligamentos, por lo que estos niños adoptan posiciones raras e inverosímiles.
Son frecuentes las lesiones musculares debido a la debilidad de los músculos.
Otras manifestaciones del raquitismo son: abultamiento del vientre, retraso
en la dentición, implantación defectuosa de los dientes, disminución de la
talla en relación con la normal. TRATAMIENTO: Se basa en la administración
diaria de vitamina D por vía oral. La dosis será de 2 500 a 5 000 U (62 a 125
mg), y puede llegarse a 10 000 U/día (250 mg), lo que corresponde a una
cantidad de 5 a 20 gotas de preparado que contenga 10 000 U/mL, durante 6
u 8 semanas.
E Impide la autooxidación de los ácidos Puede ocasionar anemia hemolítica (destrucción de los glóbulos rojos de la o
Tocoferol grasos insaturados. Impide deterioro de sangre), degeneración muscular y desórdenes en la reproducción.
las membranas celulares
K Intervienen en la síntesis de Pueden producirse hemorragias nasales, en el aparato digestivo o el genito- Vegetales verdes,
Filoquinona protrombina (coagulación) urinario. tomates, aceites
vegetales
VITAMINAS HIDROSOUBLES
C Antioxidante, importante para la síntesis ESCORBUTO; Se caracteriza por una debilidad muscular progresiva, dolores Frutos cítricos,
Ac. de colágeno en los brazos y en las piernas, adelgazamiento progresivo, palpitaciones, verduras frescas
ascórbico aceleración de pulsaciones cardiacas y sensación de ahogo; son frecuentes las
hemorragias especialmente en las encías, que están hinchadas y sangran con
facilidad y en la piel aparecen manchas sanguíneas; los dientes caen
precozmente; es constante la anemia y diversas alteraciones óseas que se
ponen en evidencia radiológicamente. La resistencia a las infecciones está
muy disminuida.
B1 Coenzima que interviene en las BERIBERI: Se caracteriza por trastornos nerviosos, circulatorios y generales Levaduras, germen y
Tiamina reacciones de transferencia de grupos (astenia, debilidad general, marcha tambaleante, anemia). Si el déficit es salvado de arroz,
aldehído de dos carbonos severo puede aparecer el síndrome de Korsakoff, caracterizado por la pérdida hígado, carnes
de memoria y confusión o la encefalopatía de Wernicke, trastornos oculares,
confusión
B2 Constituyente de las coenzimas FMN y Lesiones en la piel boca y ojos Mismas que B1, leche,
Riboflavina FAD que intervienen en el metabolismo queso, huevos
energético como transportadores de H+
y electrones
B6 Coenzima que intervienen en las El déficit importante provoca irritabilidad, debilidad, mareos, depresión, Levaduras, verduras
Piridoxina desaminaciones. Relacionada con el neuropatía periférica y espasmos, alteraciones del crecimiento, acrodinia y frescas, leche, carne,
metabolismo de las proteínas anemia. huevos
Niacina Forma parte de las coenzimas NAD y PELAGRA: Las principales manifestaciones son dermatitis, inflamación y dolor Trigo integral, levadura
Ac.nicotinico NADP que interfiere en la boca y en la lengua, diarrea, marcha tambaleante, anemia. de cerveza, verduras,

CURSO ENARM CMN SIGLO XXI TEL: 36246001 Pharmed Solutions Institute PÁGINA 379
MANUAL DE TRABAJO DEL CURSO ENARM CMN SIGLO XXI
hígado

Ac. fólico Coenzima que interviene en el Anemia


metabolismo de los ácidos nucleicos
B12 Forma parte de una coenzima necesaria Anemia perniciosa Carne, leche. Huevo,
cobalamina en el metabolismo de las proteínas y de pescado
ácidos nucleicos
TRATAMIENTO: El tratamiento se basa en la identificación de la vitamina o vitaminas deficitarias, para luego proceder a su reposición
mediante suplementos vitamínicos y raramente, en síndromes de mala absorción, mediante cargas inyectables.

CASO CLINICO
Se trata de un paciente masculino de 1 año 3 meses al ingreso. Los antecedentes heredo familiares eran negativos. Como antecedentes
perinatales se documentó: Madre de 17 años. Se trataba de suprimera gesta por cesárea por parto distósico con líquido amniótico
mecanizado. Se clasificó al recién nacido de término adecuado para la edad gestacional. Su peso al nacer fue de 2720 g. Perímetro
Cefálico 33 cm y APGAR 9-9. Sus inmunizaciones estaban completas. Según la historia de la madre se documentó un retardo de
desarrollo psicomotor. Los médicos tratantes consideraron que su estado de desnutrición podía ser una causa contribuyente. La
alimentación se describió como deficiente, con mucha leche o atoles. Descrípción del estado socio-económico: Familia inmigrante,
matrimonio joven, con baja escolaridad y problemas económicos importantes. La madre ama de casa y padre recolectar de café sin
trabajo fijo.) Cambios frecuentes de vivienda. Motivo de ingreso: Enfermedad diarreica aguda de 8 días de evolución, con 3 o 4
deposiciones por día, verde-amarillenta, sin moco, sin sangre. Vómitos desde 5 días prevíos al ingreso de contenido alimentario. Al
ingreso se planteó como problema falla para progresar con un peso de 8.3 kilogramos, talla de 73 centímetros y retardo del desarrollo
psicomotor. Se describieron datos cínicos de deshidratación moderada. Se describe al paciente en este ingreso irritable, apático,
hipoactivo, con lesiones de piel se describen como hiperpigmentadas en todo el cuerpo, múltiples, con distribución de predominio en
cara, en forma de alas de mariposa y en las manos con distribución de "guantes", las cuales se desprendían fácilmente con el roce,
hiperqueratosis, resequedad en la piel, fisuras generalizadas de predominio en labios y en codos, glositis, hiperplasia gingival, flictenas
en brazo izquierdo, bajo reborde costal y edema en mano y piel. Se describe también hígado a 4 centímetros bajo el reborde costal.

PREGUNTA
Considerando la sintomatología cual es el déficit vitamínico mas probable?

RESPUESTA
a.- Vitamina A.
b.- Complejo B.
c.- Vitamina C.
d.- Vitamina D.

CASO CLINICO
Femenino de 6 años de edad, que acude por resfriado común y a la exploración se observa dificultad en la marcha, con sensación
dolorosa en el lado derecho de la cadera y refiere la madre que se ha caído espontáneamente en alguna ocasión, se encuentra sin
vacunación, desarrollo psicomotor dentro de limites.

PREGUNTA
Cual es su deficiencia vitamínica mas probable en este caso?

RESPUESTA
a.- Vitamina A.
b.- Complejo B.
c.- Vitamina C.
d.- Vitamina D.

CURSO ENARM CMN SIGLO XXI TEL: 36246001 Pharmed Solutions Institute PÁGINA 380
MANUAL DE TRABAJO DEL CURSO ENARM CMN SIGLO XXI
OBESIDAD. CIENCIAS BASICAS: La obesidad es una condición caracterizada por el cúmulo de grasa corporal, más allá de los límites
esperados para la edad y el sexo de un sujeto dado. Ella constituye un problema de salud creciente y es prevenible. La principal
vinculación de este estado con la salud, estriba en su asociación con enfermedades que ocupan los primeros lugares dentro de los
cuadros de morbilidad y mortalidad de la población, como son la cardiopatía isquémica, la hipertensión arterial, la DM tipo II, las
enfermedades cerebrovasculares y algunos tipos de cáncer. El sobrepeso que acompaña a la adiposidad excesiva puede producir
diversos trastornos como son los problemas ortopédicos y la reducción de la capacidad de trabajo físico. Todo esto, unido a
consideraciones estéticas, produce, además, trastornos psicológicos que se derivan no solo de la distorsión de la autoestimación, sino
de las relaciones sociales del individuo. Factores de riesgo: Padres obesos (hijos tienen riesgo de 80% de ser obesos), niños alto peso al
nacer tienen alto riesgo de sobrepeso en la infancia, niños pequeños para la edad gestacional tiene riesgo de desarrollar obesidad.
SALUD PUBLICA: Prevalencia niños 5 a 11 años 46%, 2010 43 millones en niños menores de 5 años tenían sobrepeso. Primer lugar
obesidad infantil mundial. En los adolescentes, la obesidad constituye la causa principal de hipertensión mantenida y se ha señalado
que aproximadamente el 25 % de los adolescentes hipertensos obesos pueden sufrir complicaciones cerebrovasculares o
cardiovasculares en los siguientes 7 años. PATOGENIA: El depósito de grasa en exceso es multifactorial. Obesidad como una forma de
mala nutrición por exceso, básicamente cuantitativa (con un balance energético positivo). El mecanismo mediante el cual se produce el
exceso de deposición de grasa en el cuerpo es el desbalance energético, o sea un ingreso excesivo de energía en relación con el gasto,
lo que se produce por 3 mecanismos diferentes: aumento en el ingreso; reducción del gasto y afectación de la regulación del balance
energético. En el obeso hay una mayor eficiencia térmica, lo que infiere que, es menor la cantidad de energía perdida en forma de calor
y mayor la acumulada en las reservas en forma de grasa. También suele ser menor la energía utilizada en actividad física. La evidencia
de la herencia como factor condicionante o predisponente de la obesidad se deriva de estudios realizados en gemelos monocigóticos y
dicigóticos, de la comparación entre hijos biológicos y adoptados en una familia, y del estudio de patrones familiares de adiposidad.
También la edad del rebote adiposo o sea del inicio del cúmulo de grasa prepuberal preestablece la magnitud de la adiposidad en la
adolescencia y así los niños que muestran un rebote temprano corren un alto riesgo de convertirse en obesos si viven en un medio que
favorece el consumo de alimentos por encima de sus necesidades. La dieta es determinante en la regulación energética y de hecho
constituye el principal factor desencadenante del desequilibrio entre el ingreso y el gasto de energía. Las prácticas de alimentación del
lactante han sido con frecuencia relacionadas con el desarrollo de la obesidad: la prevalencia de obesidad parece ser mayor en niños
que recibieron lactancia artificial y en aquellos con destete temprano o en los que se introdujeron sólidos antes del tercer mes.
CLASIFICACION: Obesidad; GRADO I: IMC: 30-34.9 KG/m2, GRADO II: IMC 35- 39.9 KG/m2, GRADO III: 40 KG/m2. Sobrepeso; SOBREPESO
I: IMC 25- 26.9 KG/m2, SOBREPESO II: IMC 27-29.9 KG/m2. DIAGNOSTICO: En la etapa de lactante el incremento fisiológico de la grasa
corporal puede ser excesivo, por lo que se produce un aumento en el tamaño de las células adiposas, pero la probabilidad de que la
hiperadiposidad persista hasta la adultez es muy baja. Sin embargo, cuando esta aparece entre los 5 y l0 años de edad se observa un
incremento gradual en la adiposidad y es frecuente que persista a lo largo de la adolescencia y se mantenga en la adultez,
caracterizándose por el incremento del número de adipocitos. El tejido celular subcutáneo aumenta y es característico la acumulación
en la cara, la región mamaria, el abdomen y la región pubiana, en la cual el exceso de grasa puede disimular el tamaño real del pene en
los varones, lo cual puede dar lugar a interpretaciones erróneas de hipogonadismo. Recientemente se le confiere a la distribución de la
grasa una particular importancia clínica. En aquellos sujetos en los cuales la grasa se acumula en el tronco (obesidad superior o
androide) se ha observado una mayor asociación con la presencia de enfermedades crónicas degenerativas, lo cual también se observa
cuando se produce un incremento de la grasa visceral (obesidad central). Cuando la grasa tiende a acumularse en las extremidades,
particularmente en el tren inferior (inferior o ginecoide), la probabilidad de asociación de la obesidad con dichas afecciones es mucho
menor. En la piel del abdomen de sujetos obesos moderados o severos suelen verse estrías atróficas. Igualmente pueden observarse
lesiones de Acantosis nigricans, localizados sobre todo en la piel del cuello y las extremidades. En el sistema osteomioarticular pueden
verse genu valgo, coxa vara y deslizamientos epifisarios de la cabeza del fémur. En los obesos son frecuentes los trastornos
psicológicos, en ocasiones como elementos causales y en otras, como resultado de la distorsión que sufre el sujeto de su imagen y de
los conflictos que determina su condición dentro de su grupo. En una proporción de escolares y adolescentes es posible encontrar
hipertensión arterial, por lo general, ligera a moderada, hipercolesterolemia, hipertrigliceridemia y elevación de LDL y VLDL, así como
caída de las HDL. También puede observarse intolerancia a la glucosa e incluso hiperglicemia, manifestaciones que suelen responder
favorablemente al tratamiento de la obesidad. La inspección de un sujeto es el método más simple y económico para el diagnóstico
cualitativo. Ha sido empleado, inclusive, como parte de un sistema de valores de referencia en lactantes y escolares. La medición de la
masa corporal total y su comparación con los valores de referencia expresados en porcentajes o en percentiles de peso para la edad,
han sido utilizadas como criterio diagnóstico, a considerar obesos a aquellos que se hallan en el percentil 97 o más. INDICE DE MASA
CORPORAL o índice de Quetelet (que es el resultado de dividir el peso corporal (kg) por la talla (m2), según la expresión; IMC = peso
(kg)/talla m2): entre 85-95 es indicativo de sobrepeso en niños IMC: encima de percentil 95 es indicativo de obesidad en niños. IMC: >
= 30 kg/m2 = obesidad adultos. Pero, al no tomar en cuenta la talla del sujeto, puede incluir como obesos a individuos normales de talla
elevada o excluir a individuos de baja talla. El método más difundido ha sido la determinación del peso corporal y su relación porcentual
con el peso ideal o esperada para la talla del sujeto. Así, valores entre 110 y 120 % se denominan sobrepesos y valores, superiores a 120
% obesos. Este criterio ha sido fuertemente criticado, ya que considera el peso para la talla independiente de la edad, lo que introduce
un sesgo importante. Otro procedimiento de utilidad práctica es la medición de los espesores de los pliegues cutáneos en diversas
zonas del cuerpo con un compás destinado al efecto; las más utilizadas son los pliegues tricipital, bicipital, subescapular y suprailíaca.
Estos pliegues brindan información sobre la cantidad de grasa subcutánea, la que expresa indirectamente el estado de las reservas
energéticas del organismo. Para medir la distribución de la grasa el indicador más práctico y útil es el índice cintura/cadera que es la
resultante de dividir la circunferencia de la cintura por la cadera. SINDROME DE CARPENTER Obesidad, retraso mental, puente nasal
plano, sindáctilia, craneosinostosis, hipogonadismo. SINDROME DE PRADER WILLI Hipotonía neonatal, baja estatura,
hipogonadismo, hiperfagia compulsiva, retraso mental obesidad, cromosoma 15. TRATAMIENTO: El objetivo, es lograr un cambio en
la composición corporal con reducción del peso en grasa y mantenerla después dentro del rango adecuado para su edad y sexo. El
tratamiento no deberá interferir con el incremento de masa magra, es decir, con el crecimiento. La terapéutica está dirigida a modificar
el desbalance energético para reducir el peso en grasa y a modificar el estilo de vida para garantizar que esta reducción sea duradera.
Para ello, se sustenta en 3 pilares fundamentales: La dieta, el ejercicio, la inducción de cambios de conducta. Farmacológico: limitado

CURSO ENARM CMN SIGLO XXI TEL: 36246001 Pharmed Solutions Institute PÁGINA 381
MANUAL DE TRABAJO DEL CURSO ENARM CMN SIGLO XXI
en niños. Las dietas muy bajas en calorías solo deben hacer especial manejo en obesidad y nunca será la terapia indicada en niños.
Reducir azúcares, agua. Frutas y cereales granos enteros Leche descremada, no comidas rápidas. Educar a las personas para lograr que
ellas distribuyan el consumo de energía diario de forma tal que el 20 % se consuma en el desayuno, el 20 % entre comidas y el 30 % en
almuerzo y comida, respectivamente. Solo al constituirse ese hábito con carácter familiar podrá ejercerse influencia sobre los miembros
más jóvenes de la familia. Identificar los sujetos en riesgo y brindarles una atención especial, tomando en cuenta, entre otros, los
factores siguientes: Obesidad en los padres, los hermanos y otros familiares cercanos. Obesidad en convivientes no familiares. Bajo
nivel educacional. Sobreprotección: hijos únicos o valiosos, separación de los padres o muerte de uno de ellos. Tratamiento con
antihistamínicos, anabólicos o esteroides. Alto consumo de azúcar y/o leche (más de un litro diario). Poca actividad física. Promover la
lactancia materna extendiéndola por el mayor tiempo posible, y con carácter exclusivo hasta los 5 meses. No introducir ningún otro
alimento en la dieta del lactante hasta esa fecha. Cuando se inicie la ablactación, no acostumbrar al bebé al consumo excesivo de
alimentos dulces. Tratamiento farmacológico metformina: obesos adolescentes no diabéticos pero con comorbilidad, hiperinsulinemia
o resistencia a la insulina. Efectos secundarios: náuseas, vómito Ovarios poliquísticos. Tratamiento quirúrgico: Cirugía gástrica y
cirugía baipás: reduce la comorbilidad. Última opción la cirugía bariatrica.

CASO CLINICO
Niña de 10 años de edad, acompañada por su madre para una visita de niño sano. El peso del paciente, la altura y los signos vitales,
fueron 42,1 kg, su altura es de 1,40 m, y su presión arterial es de 98/50 mmHg. IMC 21,6 kg/m2, que está en el percentil 91o para la
edad y el género.

PREGUNTA
Cuál es el factores de riesgo más importante que buscaría para establecer un diagnostico adecuado.

RESPUESTA
a.- Familiares con sobrepeso y obesidad.
b.- Interrogar sobre tiempo de ver televisión.
c.- Actividad física.
d.- Peso al nacimiento.

PREGUNTA
Cual es la comorbilidad más frecuente que el paciente desarrollará en caso de persistir el sobrepeso.

RESPUESTA
a.- Diabetes mellitus.
b.- Esteatosis hepática.
c.- Baja autoestima.
d.- Desplazamiento epifisiario.

PREGUNTA
Cuál es el exámen no básico para pesquizaje intensional de complicaciones.

RESPUESTA
a.- Perfil lipidico.
b.- Valores antropométricos.
c.- Glucosa en ayunas e insulina.
d.- USG por poliquistosis ovárica.

PREGUNTA
Considerando el cuadro clínico cual es la medida mas adecuada.

RESPUESTA
a.- Sibutramina.
b.- Orlistat.
c.- Metfomina
d.- Dieta y estilo de vida.

PREGUNTA
Cual no es un criterio de referencia para segundo nivel inmediato para el tratamiento.

RESPUESTA
a.- Sobrepeso u obesidad con retrazo mental.
b.- Lesion hipotalámica por cualquier causa.
c.- Falla en el tratamiento instaurado.
d.- Falla en el crecimiento y desarrollo.

CURSO ENARM CMN SIGLO XXI TEL: 36246001 Pharmed Solutions Institute PÁGINA 382
MANUAL DE TRABAJO DEL CURSO ENARM CMN SIGLO XXI
CASO CLINICO
Un niño nacido de padres no relacionados, sanos después de un embarazo a término y el parto normal (peso al nacer 3200 g, longitud
50 cm, 34,5 cm OFC). Fue hipotónica en los primeros meses de vida, pero no se observaron problemas de alimentación en un principio.
Excesivo aumento de peso con aumento del apetito y el comportamiento de búsqueda de alimento se observó a los 30 meses de edad.
A los 5 años, su peso era de 26 kg (> 3 SD), longitud 112 cm (1 SD) y OFC 48 cm (-2DE) a los 9 años y su peso es de 72 kg (> + 6 SD),
longitud 144 cm (+2 SD) y OFC 52,5 cm (-0,5 SD). Distintivos rasgos faciales incluyen braquicefalia, cara plana, la frente alta,
hipertelorismo, nariz antevertidas, labio superior delgado, prognatismo, manos cortas, sindactilia de los dedos 2-3, y genitales
anormales (criptorquidia e hipospadias).

PREGUNTA
Cual es la conducta a seguir.

RESPUESTA
a.- Tratamiento farmacológico.
b.- Enviar a segundo nivel.
c.- Enviar a tercer nivel.
d.- Tratamiento quirúrgico.

CURSO ENARM CMN SIGLO XXI TEL: 36246001 Pharmed Solutions Institute PÁGINA 383
MANUAL DE TRABAJO DEL CURSO ENARM CMN SIGLO XXI
VACUNACION. CIENCIAS BASICAS: Las vacunas son sustancias
que nos ayudan a prevenir muchas enfermedades infecciosas,
las cuales son contagiosas y provocan discapacidad grave o
incluso la muerte. Cartilla Nacional de Vacunación (0 a 19
Años): Es el documento Oficial para dar seguimiento al
esquema de vacunación a los niños menores de 5 años,
escolares y adolescentes hasta los 19 años de edad. Siempre
que se acude a vacunación se debe de entregar para ver el
esquema o anotar la vacuna que se está aplicando en ese
momento. BCG: Vacuna contra la tuberculosis. Dosis única de
0.1 ml, RN, hasta los primeros 5 años, vía intradérmica, en la
región deltoidea del brazo derecho, sin prueba tuberculínica
previa, deja una cicatriz en el brazo posterior a su aplicación.
Contraindicaciones: Fiebre > 38.5 °C. RN peso inferior 2 kg,
dermatitis progresiva, pacientes con cuadro clínico de SIDA, Tx
Transfusiones, o inmunoglobulina, esperarán cuando menos
tres meses para ser vacunadas, no aplicar durante el
embarazo, no aplicar BCG con profilácticas de medicamentos antituberculosos. HEPATITIS B: Se aplican 3 dosis, 10 mcg en 0.5 ml. IM
profunda, en la cara anterolateral externa del muslo izquierdo en los menores de 18 meses de edad, a partir de los 18 meses de edad,
en la región deltoidea del brazo derecho , la primera al nacer y a los 2 y 6 meses de edad. Previene este tipo de hepatitis, que afecta
principalmente al hígado. Contraindicaciones: Fiebre > 38.5 °C. RN peso inferior 2 kg, enfermedad grave, Tx Transfusiones, o
inmunoglobulina, esperarán cuando menos tres meses para ser vacunadas, hipersensibilidad a cualquier componente. PENTAVALENTE:
Es la vacuna que previene la difteria, tos ferina y tétanos, además también a la poliomielitis y a las bacterias del Haemophilus Influenzae
del tipo b, que provocan neumonías y meningitis. Se aplica en 4 dosis a los 2, 4, 6, y 18 meses de edad. Se aplica por vía IM profunda,
las primeras tres dosis deben aplicarse en el tercio medio de la cara anterolateral externa del muslo derecho. A los 18 meses se debe
aplicar en la región deltoidea del brazo izquierdo. ROTAVIRUS: Previene la gastroenteritis o la diarrea causada por el rotavirus en sus
formas graves. Se aplican 3 dosis 0.1 ml sobre la parte interna de las mejillas, a los 2, 4 y 6 meses de edad y nunca después de los 8
meses de edad. Contraindicaciones: Hipersensibilidad después de la administración de la vacuna, antecedente de enfermedad
gastrointestinal crónica, malformación congénita no corregida en el tracto gastrointestinal. SABIN: Ayuda a prevenir la poliomielitis. Se
aplican 2 gotitas en los menores de 5 años a partir de los 6 meses como dosis adicional (de 0.1 ml) en cada Semana Nacional de Salud,
siempre y cuando hayan recibido dos dosis previas de vacuna de poliovirus inactivada, uso de esta vacuna en actividades de control de
casos y brotes. Contraindicaciones: Infección por VIH, tener inmunodeficiencia congénita, tener hemopatía o tumor sólido o terapia
inmnunosupresora. NEUMOCÓCICA: Contra el neumococo. Se aplican 3 dosis, 0.5 ml, a los 2, 4 y 12 meses de edad. Vía IM en el
tercio medio de la cara anterolateral externa del muslo derecho. Contraindicaciones: Procesos febriles de más de 38.5 °C, con
enfermedad grave, reacción alérgica severa (anafiláctica) hacia un componente de la vacuna incluyendo toxoide diftérico, antecedentes
de Síndrome de Guillain-Barré, antecedentes de hipersensibilidad al látex. No revacunar antes del tiempo establecido, ya que puede
provocar reacciones locales severas. Aplicar un refuerzo antes del tiempo establecido puede provocar el desarrollo de neumonía
neumocóccica posvacunal severa. ANTI-INFLUENZA: Previene el
virus de la influenza y se aplica a niños de 6 a 35 meses de edad
desde octubre a febrero, 0. 25 ml IM basto lateral izquierdo. En la
primera ocasión se aplican dos dosis con intervalo de 1 mes y
después cada año. Contraindicaciones: No aplicar a personas con
alergia a las proteínas del huevo o a algún componente de la
vacuna, fiebre, haberse aplicado el biológico en menos de un año,
en lactantes menores de 6 meses, en personas que hayan tenido
una reaccIón de hipersensibilidad, incluyendo una reacción
anafiláctica al huevo o a una dosis previa de vacuna contra
influenza, ante la presencia de fiebre mayor o igual a 38.5°C,
pacientes con antecedente de síndrome de Guillain Barré.
VACUNA TRIPLE VIRAL (SRP): Previene el sarampión, la rubeola y
las paperas. Se aplica 2 dosis de 0.5 ml de vacuna reconstituida:
12 meses de edad y los 6 años de edad vía SC en el área superior
externa del tríceps del brazo Izquierdo, inscritos o no en primer
año de primaria. Contraindicaciones: Fiebre > 38.5, alérgicos al
huevo, neomicina, antecedente de aplicación de inmunoglobulina
o transfusiones 3 meses previos a la vacunación, neoplasias,
inmunodeficiencias, tratamiento con corticoesteroides,
citotóxicos, embarazo. Poco frecuentes, pueden presentarse: Parotiditis uni o bilateral, después de doce días y dura menos de cuatro
días, meningitis aséptica, entre la segunda y la cuarta semanas posteriores a la vacunación, púrpura trombocitopénica. DPT: Sirve como
un refuerzo que previne a la difteria, tos ferina y tétanos. Se aplica dosis de 0.5ml a los 4 años, se aplica por vía IM profunda en la
región deltoidea del brazo izquierdo. Contraindicaciones: No mayores a 5 años, reacción anafiláctica a una aplicación previa,
antecedentes de crisis convulsivas o alteraciones neurológicas, fiebre > 38. 5°C. Toxoide Tetánico Diftérico (TD): Se aplica a las
personas desde los doce años hasta los adultos mayores, hombres y mujeres, especialmente a las embarazadas, dosis de 0.5ml.
Previenen el tétanos en los recién nacidos y en los adultos. Las personas no vacunadas o con esquema incompleto de vacuna
pentavalente o DPT recibirán al menos dos dosis con intervalo de 4 a 8 semanas entre cada una, y revacunación cada 10 años.
Contraindicaciones: No suministrar a personas con hipersensibilidad a algún componente de la vacuna, a personas con

CURSO ENARM CMN SIGLO XXI TEL: 36246001 Pharmed Solutions Institute PÁGINA 384
MANUAL DE TRABAJO DEL CURSO ENARM CMN SIGLO XXI
inmunodeficiencias, a excepción de la infección por el VIH asintomática, fiebre superiores a 38.5°C y enfermedades graves, historia de
reacción grave de hipersensibilidad o eventos neurológicos relacionados con la aplicación de una dosis previa, las personas
transfundidas o que han recibido inmunoglobulina, deberán esperar tres meses para ser vacunadas, salvo en aquellos casos de
traumatismos con heridas expuestas, ya que puede aplicarse, simultáneamente con antitoxina, independientemente de transfusión o
aplicación de inmunoglobulina. EVENTOS TEMPORALMENTE ASOCIADOS A LA VACUNACIÓN ETAVS: Se define como eventos
temporalmente asociados a la vacunación a todas aquellas manifestaciones clínicas que se presentan dentro de los 30 días posteriores
a la administración de una o más vacunas y que no pueden ser atribuidos inicialmente a alguna entidad nosológica especifica (para la
vacuna Sabin el periodo puede ser hasta de 75 días y para la vacuna BCG hasta seis meses ó un año). Falsa contraindicación para todas
las vacunas: Reacción local de leve a moderada, enfermedad leve con o sin fiebre, tratamiento antimicrobiano actual, fase de
convalecencia de alguna enfermedad, exposición reciente a enfermedades infecciosas, historia de alergia a penicilina u otras alergias
inespecíficas o el hecho de que sus familiares tengan dichas alergias, alergia a proteínas del huevo, a neomicina manifestada sólo por
rash.

CASO CLINICO
Un niño de 24 meses de edad, nació a término, cuenta con el antecedente que después de haber recibido la vacuna antipoliomielítica,
fiebre (38,5 º C y 40,0 º C), al día siguiente presento vómitos y convulsiones tónico-clónicas. El líquido cefalorraquídeo mostró
pleocitosis y aumento moderado de proteínas. El LCR fue positivo por PCR para enterovirus y una pleocitosis. Las muestras de heces
tomadas los días 5 y 9 fueron positivos para enterovirus, que se caracterizó posteriormente como poliovirus tipo 3.

PREGUNTA
Cual es la conducta a seguir.

RESPUESTA
a.- Aplicar SRP
b.- Vacuna pentavalente acelular.
c.- Evitar DPT a los 4 años.
d.- Revacunación anual.

DERMATITIS HERPETIFORME (DH). CIENCIAS BÁSICAS: Es una enfermedad pápulovesicular, muy pruriginosa, crónica, de distribución
característica. PATOGENIA: Se ha especulado sobre el papel de los anticuerpos del gluten en la etiopatogénesis de esta enfermedad. El
gluten es una fracción proteica remanente de la harina después que ha sido lavada y eliminado el almidón. Las principales fuentes de
gluten en occidente son: trigo, centeno, cebada; el maíz y la avena no parecen contener cantidades importantes. El 75% de los casos de
DH tienen aplanamiento de las vellosidades intestinales y 20% presenta infiltrado linfocitario en el epitelio como evidencia de
enteropatía sensible al gluten (ESG). Es bien establecido, que las manifestaciones cutáneas e intestinales mejoran con la dieta estricta,
libre de gluten, sin embargo, el mecanismo fisiopatológico se desconoce. La asociación inmunogenética apoya la naturaleza
autoinmune de la DH a otras enfermedades autoinmunes. Aunque no se ha descrito todavía el antígeno diana, hay asociación de la DH
con otras enfermedades autoinmunes, como lupus eritematoso, enfermedades tiroideas y diábetes mellitus. DIAGNOSTICO: La DH se
ha reportado raramente en niños, pero la edad de presentación es de 2 a 7 años. Las lesiones cutáneas características son pápulas y
placas eritematosas, urticarianas, edematosas sobre las cuales hay vesículas, distribuidas simétricamente en superficie extensora de las
extremidades. Los sitios característicos son: codos, rodillas, glúteos, nuca y cuero cabelludo. Una sensación de ardor o escozor suele
preceder las lesiones. El prurito es intenso haciendo que el paciente esté inquieto. Hay lesiones secundarias al rascado como
excoriaciones, costras y máculas hiper e hipopigmentadas residuales. Las membranas mucosas no están comprometidas. Sin embargo,
se han descrito defectos en el esmalte dental similar a lo observado en la enfermedad celíaca, cambio de coloración y textura de la
superficie dentaria. Los síntomas gastrointestinales pueden ser muy leves o estar ausentes, aunque por lo menos un tercio de los
pacientes sufre de esteatorrea y anemia. La patología es característica. Se describen microabscesos en la punta de las papilas dérmicas
y formación de ampolla subepidérmica. En la dermis superior y media se observa un infiltrado de linfocitos, neutrófilos e histiocitos
alrededor de los vasos sanguíneos. Los eosinófilos son escasos. El diagnóstico se basa en criterios clínicos, histológicos, así como la
respuesta favorable a las sulfonas y a la dieta libre de gluten. TRATAMIENTO: El tratamiento de elección, como en los adultos, es la
dieta libre de gluten. Se describe una respuesta mucho más rápida en este grupo con control de las lesiones cutáneas a los 11 meses,
aproximadamente. La dapsona a la dosis de 1.5-2 mg/kg/día, es efectiva para el control de las lesiones cutáneas y del prurito intenso.
La suspensión de ésta, produce la reaparición rápida de las lesiones, por lo que el tratamiento debe ser combinado con la dieta libre de
gluten y disminución progresiva y suspensión de la dapsona al lograr control, con lo que disminuye la aparición de los efectos
secundarios de la droga: metahemoglobinemia, anemia hemolítica, agranulocitosis, y complicaciones neurológicas. Antes de su uso, los
niveles de glucosa-6-fosfato deshidrogenasa deben ser determinados. En esos casos, en los que no se pueda utilizar la dapsona, se
recomienda la sulfapiridina a 250 mg/día con ajuste de acuerdo a respuesta terapéutica. El curso de la DH en niños no está bien
definido, la ESG persiste toda la vida, aunque con períodos de remisiones espontáneas en algunos casos. Sin embargo, hay que informar
a la familia sobre la naturaleza de la enfermedad y su duración, así como, la importancia de la dieta estricta libre de gluten de por vida.

CASO CLINICO
Paciente varón de 13 años de edad que consulta por lesiones muy pruriginosas, que se localizan fundamentalmente en codos, rodillas y
región lumbosacra. Se ha tratado con diversas pomadas de corticoides tópicos sin mejoría. En la exploración cutánea se observa en los
codos, rodillas y región lumbosacra la presencia de pápulas eritematosas, excoriadas y alguna pequeña vesícula intacta. Se realizó
biopsia cutánea de una de las lesiones y el estudio histopatológico demostró: microabscesos de neutrófilos a nivel de papilas dérmicas,
El estudio de inmunofluorescencia directa demostró depósito de Ig A granular en las papilas dermicas.

CURSO ENARM CMN SIGLO XXI TEL: 36246001 Pharmed Solutions Institute PÁGINA 385
MANUAL DE TRABAJO DEL CURSO ENARM CMN SIGLO XXI
PREGUNTA
Considerando la patología que presenta el caso cual de los siguientes diagnosticos diferenciales es el menos probable?

RESPUESTA
a.- Dermatitis alérgica de contacto
b.- Dermatitis atópica
c.- Sarna
d.- Penfigoide ampolloso

PENFIGO. CIENCIAS BÁSICAS: Es una enfermedad ampollar autoinmune, del epitelio estratificado, que normalmente afecta a individuos
de edad media y mayores, pero puede ocurrir raras veces en niños, relacionado con factores genéticos y ambientales secundario a la
producción de IgG. Las dos variantes del pénfigo son el vulgar (PV) y foliáceo (PF), que difieren en su presentación clínica, histológica y
en los antígenos patogénicos. El pénfigo vegetante y eritematoso son variantes clínicas más raras. En Brazil existe la forma endémica
del pénfigo foliáceo, denominada fogo selvagem. Más recientemente se describió el pénfigo paraneoplásico y el pénfigo IgA. SALUD
PUBLICA: Es una enfermedad que afecta todas las razas sin distinción de sexo. Sin embargo, se reporta una mayor incidencia en judíos
ashkenazi y menos en la raza negra. La edad de presentación en niños va desde 3 hasta 17 años y adultos pico de 50-60 años. En 1955
se publicó el primer caso de pénfigo vulgar en un niño. Mortalidad 17.7% (choque séptico, embolia pulmonar). CLASIFICACION: Pénfigo
vulgar (80%), pénfigo violáceo, pénfigo inducido por fármacos, pénfigo paraneoplasico, pénfigo IgA. PATOGENIA: El pénfigo es una
enfermedad autoinmune que afecta la piel y las mucosas, de etiología desconocida. Existe un aumento en la expresión de antígenos del
complejo mayor de histocompatibilidad HLA-A, DR4, Dw10 y Dw6. Los pacientes tienen autoanticuerpos que reaccionan con los
desmosomas, moléculas de adhesión localizadas en la superficie de los queratinocitos. Los autoanticuerpos detectados son de la clase
IgG. En los últimos años se ha logrado un avance en la determinación de los antígenos responsables en todas las formas de pénfigo. De
la unión de este autoanticuerpo con las desmogleína antigénica se produce la acantolisis, con pérdida de la cohesión de las células y la
formación de las ampollas. DIAGNOSTICO: Clínicamente, suele comenzar en la mucosa bucal con erosiones del epitelio. Las lesiones
cutáneas consisten en vesículas, ampollas fláccidas que se rompen con facilidad resultando en extensas erosiones. Se localiza
predominantemente en cara, cuello, tronco, extremidades y áreas intertriginosas. Las erosiones causan dolor y ardor y
secundariamente pueden infectarse con bacterias u hongos. El signo de Nikolsky es positivo (eritema y formación de ampollas como
resultado de fricción o presión sobre la piel), signo de Asboe-Hansen (aumento periférico del tamaño de la ampolla al presionar
verticalmente su superficie), no es especifica. Se ha reportado onicolisis y distrofia ungueal. En el PF infantil es endémico en Brasil, en la
forma de fogo selvagem, pero su incidencia en otros países es menor a la del PV. Clínicamente se presentan erosiones superficiales con
descamación y costras, aisladas o generalizadas (eritrodermia). En ocasiones se inicia en cuero cabelludo o áreas seborreicas, luego se
extienden al tronco y extremidades. No se aprecian lesiones orales. Se deben escoger las vesículas intactas para el examen histológico.
En el PV se evidencia la ampolla intraepidérmica suprabasal, con acantolisis (desaparición de puentes intracelulares) marcada. El piso
de la cavidad está constituido por queratinocitos basales y leve infiltrado inflamatorio, compuesto por eosinófilos. En lesiones
tempranas los eosinófilos invaden la epidermis en racimo. Este fenómeno se denomina espongiosis eosinofílica. La ampolla en el PF se
localiza en la subcorneal de la epidermis. La acantolisis está presente inmediatamente por encima o por debajo de las células de la capa
granulosa. Este patrón se ha denominado “panal de abejas” y constituye la herramienta diagnóstica en estas enfermedades, cuya
positividad alcanza hasta el 100%. El diagnóstico se basa en la combinación de criterios clínicos, histológicos. Si hay duda observar la
celularidad, células Tzank, tiene 100% sensibilidad y 43% especificidad. Se observan células acantoliticas (zona blanca). TRATAMIENTO:
Las lesiones se alivian en 1-2 años y dejan hiperpigmentacion pósinflamatoria. En manejo de los pacientes pediátricos en PV es similar
al adulto. Los casos con enfermedad localizada pueden ser tratados con corticoesteroides tópicos. Sin embargo, la mayoría de los casos
desarrollan formas más severas que ameritan el uso de corticoesteroides sistémicos a altas dosis (2 a 3 mg/kg/día). Con la finalidad de
disminuir los efectos adversos de éstas, se usan drogas adyuvantes como: azathioprina, ciclofosfamida, dapsona y metotrexato, sales de
oro.

CASO CLINICO
Un niño de 12 años de edad se presentó con lesiones generalizadas en la piel eritematosas y costras de 4 meses de duración. Las
lesiones aparecieron por primera vez en su cuero cabelludo y dentro de unos días, eritrodérmica se generalizó y, a continuación.
Escalado y exudación también fueron vistos. Diferentes antibióticos y esteroides tópicos fueron prescritos sin mejora significativa. El
niño también se quejo de pérdida de la audición desde un mes. En el examen físico, eritrodermia con escalamiento grave y maloliente
con descarga y leve queratodermia palmoplantar todo el cuero cabelludo. Había dos pequeñas vesículas a lo largo del lado cubital de la
palma derecha. Las superficies de las mucosas y las uñas eran normales. Tenía ganglios linfáticos submandibulares. El conducto auditivo
estaba lleno de escamas y costras. pabellón de la oreja era sensible a la palpación. Después de quitar las costras, canal auditivo externo
se encontró que era roja e hinchada

PREGUNTA
Cual es la conducta a seguir.

RESPUESTA
a.- Prednisona oral de 0.5 a 1 mg/kg al día.
b.- Dapsona dosis de 100 mg/día.
c.- Azatioprina es de 2.0 a 2.5 mg/kg,
d.- Ciclofosfamida a dosis oral de 1 a 3 mg/kg al día

CURSO ENARM CMN SIGLO XXI TEL: 36246001 Pharmed Solutions Institute PÁGINA 386
MANUAL DE TRABAJO DEL CURSO ENARM CMN SIGLO XXI
HIPERQUERATOSIS EPIDERMOLITICA. CIENCIAS BASICAS: También llamada Eritrodermia ictiosiforme congénita ampollosa, es una
genodermatosis infrecuente, con patrón de herencia autosómico dominante, aunque 50% de los pacientes experimentan mutaciones
de novo. Se distingue por eritrodermia y ampollas secundarias a traumatismos mínimos, que se manifiestan desde el nacimiento y que
evolucionan a ictiosis generalizada a lo largo de la vida, predominantemente en zonas de flexion. SALUD PUBLICA: No hay predilección
por sexo o raza. Se manifiesta desde el nacimiento en un caso por cada 200,000 a 300,000 nacidos vivos. PATOGENIA: La causa son los
defectos en las citoqueratinas que forman los filamentos intermedios. Las citoqueratinas afectadas son la K1 y la K10, las cuales
predominan en los estratos suprabasales de la epidermis. Cuando las citoqueratinas experimentan mutaciones puntuales, se asocian
con la hiperqueratosis epidermolítica y la eritrodermia ictiosiforme congénita ampollosa, y generan células suprabasales que se
fragmentan fácilmente, con epidermis hiperproliferativa e hiperqueratósica. Forman parte del túbulo de las glándulas sudoríparas
ecrinas y son marcadores de queratinización por
excelencia. La hiperqueratosis y la ictiosis
subsecuentes que se observan en estos pacientes se
deben a un mecanismo de compensación para
proteger contra la fragilidad cutánea, y así disminuir
la probabilidad de formación de ampollas.
CLASIFICACION: En 1994 DiGiovanna distinguió
clínicamente los dos tipos de hiperqueratosis
epidermolítica con base en la presencia o ausencia
de queratodermia palmoplantar. DIAGNOSTICO: Se
basa en manifestaciones clínicas y confirmación histológica. Aparece desde el nacimiento e inicia como eritrodermia generalizada, con
fragilidad cutánea que causa descamación, así como ampollas, incluso con traumatismos menores, en la mayoría de los casos. Las
ampollas son superficiales y grandes, y al romperse ocasionan grandes zonas de exulceración. En los pliegues de flexión aparecen
ulceraciones superficiales y zonas extensas de piel denudada. Debido a la interrupción de la barrera epidérmica, en estos niños existe
un mayor riesgo de infecciones severas, desequilibrio hidroelectrolítico y sepsis. Hay disminución gradual de la eritrodermia y de las
ampollas, que a los pocos meses evolucionan a placas hiperqueratósicas, de aspecto verrugoso, en las zonas de flexión, la piel cabelluda
y los glúteos. La hiperqueratosis inicia a los tres meses de edad, con engrosamiento poco evidente de la piel o con escamas finas y
pequeñas. Con la edad algunas de las escamas se van haciendo más gruesas (adquieren aspecto de “cartón corrugado”) y la
probabilidad de manifestar ampollas disminuye, aunque incluso 20% de los pacientes sigue teniendo ampollas hasta la vida adulta. La
hiperqueratosis se agrava con el sol y el calor. La afección de las palmas y las plantas ocurre en 60% de los pacientes y se manifiesta
con grados variables de hiperqueratosis, asociada, sobre todo, con mutaciones de CK1. La queratodermia palmoplantar puede originar
fisuras y contracturas recurrentes y dolorosas, que pueden ocasionar incapacidad funcional. La biopsia de piel, confirma el diagnostico
si observamos epidermis con hiperqueratosis ortoqueratósica, acantosis irregular moderada con áreas de degeneración de
queratinocitos, espinosos, dermis con papilomatosis y focos mínimos de linfocitos perivasculares. La inmunohistoquímica muestra
anormalidades de expresión de la citoqueratina 1 o 10. TRATAMIENTO: La piedra angular del tratamiento son los emolientes tópicos.
Existen reportes de mejoría clínica con dosis altas de betacarotenos, retinoides sistémicos o tópicos, glicerina a 10%, ácido láctico, alfa-
hidroxiácidos, calcipotriol y urea; sin embargo, no existe un consenso sobre la administración de estos fármacos. Los agentes
queratolíticos son poco tolerados por estos pacientes debido a que las escamas se desprenden en bloques extensos. El
sobrecrecimiento bacteriano es común, sobre todo por Staphylococcus aureus, secundario a la acumulación y maceración de la escama,
por lo que pueden usarse jabones antibacterianos suaves o limpiadores con clorhexidina. PRONOSTICO: Es bueno para la vida, sin
embargo los pacientes pueden padecer alteraciones funcionales, secundarias a queratodermia palmoplantar y contracturas en flexión
de las manos.

CASO CLINICO
Lactante mujer, de un año de edad, sin antecedentes patológicos de interés, que consulta por aparición progresiva de lesiones cutáneas
a nivel palmoplantar. El cuadro se inició a la semana del nacimiento con discreto eritema en dicha localización que evolucionó
progresivamente hacia placas hiperqueratósicas amarillentas, simétricas y confluentes. En el momento de la consulta presentaba una
hiperqueratosis difusa palmoplantar bien delimitada por un borde eritematoso en los márgenes laterales de las manos y los pies, sin
extensión a la superficie dorsal de estos.

PREGUNTA
Cual es la conducta a seguir.

RESPUESTA
a.- Envio a segundo nivel.
b.- Tratamiento farmacológico.
c.- Tratamiento quirúrgico.
d.- Terapia fotodinámica.

PREGUNTA
Cual de los antecedentes personales patológicos incrementa más esta patologia.

RESPUESTA
a.- Xeroderma pigmentoso.
b.- Albinismo.
c.- Inmunosupresion.
d.- Sindrome de Bloom.

CURSO ENARM CMN SIGLO XXI TEL: 36246001 Pharmed Solutions Institute PÁGINA 387
MANUAL DE TRABAJO DEL CURSO ENARM CMN SIGLO XXI
FOLICULITIS. CIENCIAS BASICAS: Foliculitis Bacterianas: Se definen como el conjunto de infecciones originadas por bacterias que
afectan al folículo piloso con una expresión clínica que varía de acuerdo a la profundidad y extensión. Foliculitis: Se definen infecciones
superficiales localizadas a nivel del ostium folicular, acompañada en ocasiones de una reacción inflamatoria perifolicular. PATOGENIA:
El agente causal más frecuente es Staph. aureus, pero también pueden infectar bacterias Gram negativas ó S. Aureus coagulasa
negativo. Los folículos pilosos están en toda la superficie de la piel excepto en palmas y plantas, pero son abundantes sobre todo en piel
cabelluda, zonas de varaba y bogote, axilas, pubis. Son estos los sitios más afectados por la foliculitis. DIAGNÓSTICO: Es clínico y de ser
necesario se realizará gram y cultivo. Se presentan en cualquier zona pilosa, sobre todo en ambiente húmedo, maceración, falta de
higiene, aplicación del emoliente oclusiva y drenaje de heridas adyacentes y abscesos, uso de agentes oclusivos o presencia de
infecciones adyacentes, y más frecuentemente en pacientes con inmunosupresión local ó sistémica. Se presentan clínicamente como
una pústula de base eritematosa, apenas de unos milímetros, a nivel de cada folículo, en ocasiones se dejan atravesar un pelo, son de
color amarillento se abren rápidamente y dejan salir una gotita de pus que al secarse formara una costra melicerica. En general no es
un proceso pruriginoso, ni doloroso.Las lesiones se extioenden rápidamente en superficie y mientras unas van curando, otras nuevas
aparecen. Al desaparecer no dejan cicatriz. Se localizan con mayor frecuencia en zonas de pelo terminal; si la infección se extiende en
profundidad formará Abscesos. A las formas más superficiales también se les denomina Impétigo de Bockhart. TRATAMIENTO: En
primer lugar eliminar factores desencadenates, antisépticos n(fomentos de sulfato de cobre y aplicación de toques con solución yodada
al 1%), antibióticos tópicos y sistémico (para evitar recidiva dicloxacilina, TMP/SFX, por 3-4 semanas) sabiendo que el agente causal es
Staph. Aureus.

CASO CLINICO
Niña de 8 años de edad. Consultó por aparición en los 3 días previos de un brote de lesiones papuloeritematosas, algunas de aspecto
seudovesiculoso o pustuloso, pruriginosas, en el tronco (de predominio en caderas y glúteos) y las extremidades inferiores. Con la
sospecha inicial de exantema periflexural asimétrico de la infancia frente a picaduras, se inició tratamiento con antihistamínico por vía
oral y metilprednisolona por vía tópica, con mejoría aparente de las lesiones. Seis días más tarde consultó por aparición progresiva de
nuevas lesiones de similares características. En la exploración física presentó máculas hiperpigmentadas marrones en la zona abdominal
inferior y en la región anterior de ambos muslos, junto con múltiples pápulas eritematoedematosas, algunas con centro pustuloso, en la
zona lumbar y en ambos glúteos y otras con aspecto vesiculoso en costado izquierdo. La niña negaba ir a piscinas, o hacer cualquier tipo
de actividad recreativa relacionada con agua.

PREGUNTA
Cual es el agente causal mas frecuente.

RESPUESTA
a.- Staphylococcus aureus.
b.- Pseudomona aeruginosa.
c.- Estreptococos
d.- Proteus.

IMPETIGO. CIENCIAS BASICAS: Es la infección bacteriana superficial y contagiosa más frecuente en los niños, producida por Estafilococo
Aureus, Estreptococo B hemolítico del grupo A (pyogens) o ambos; puede ser primario en caso de invadir piel sana ó secundario cuando
existe daño de la barrera cutánea previamente, como ocurre en la dermatitis atópica, escabiosis. El impétigo contagioso es la más
común. Estas infecciones pueden dividirse en: 1. infección cutánea primaria 2. Infección secundaria de una enfermedad cutánea
primaria y 3. lesiones cutáneas como manifestación de infección primaria en otro órgano, ejemplo de este último grupo tenemos:
S.aureus causando síndrome de shock toxico y síndrome de piel escaldada estafilocócico, S. pyogenes escarlatina, N. meningitides,
sepsis por meningococo (petequias), S. thiphy, fiebre tifoidea (roséola tifoidea), Rickettsia conorii, Tifus africano (rahs macular)
IMPÉTIGO CONTAGIOSO: Costroso ó no ampollar, infección muy frecuente representa el 70% de los casos; predomina en niños de
todas la edades , aunque también se puede ver en adultos; el contagio se origina sobre todo por rascado. La infección puede ser
originada por Strep. Pyogenes (15%), Staph. Aureus (70 a 80 %) o ambos (15 a 100%). La cara es la zona de más frecuente de
localización, sobre todo en las uniones mucocutáneas, por ejemplo: introito nasal, boca y en todas las zonas periorificiales, manos, etc,
pero como es contagioso, se puede ver en cualquier zona corporal. La lesión inicial son pápulas eritematosas pequeñas que
rápidamente evolucionan a una vesícula que se rompen rápidamente dando paso a erosiones cubiertas por costras melicéricas y lesión
residual hipocrómica temporal (lesiones primitivas, ampollas, pústulas que al romperse dejan salir exudado que se seca, costra
melicerica), sin dejar cicatriz; si hay dolor es leve y puede presentar linfadenopatía regional, pero no hay síntomas sistémicos. Las
lesiones suelen propagarse por autoinoculación originando lesiones satélites. La resolución es espontánea entre la segunda y tercera
semana de evolución, pero debe ser tratada con prontitud y así se acorta el período de enfermedad y el contagio, así como las
eventuales complicaciones glomerulonefritis post estreptocócica, escarlatina ò psoriasis. Diagnóstico: se basa en la clínica pero se
puede confirmar con un gram y cultivo. IMPÉTIGO AMPOLLAR: Representa el 30% restante, es observa más frecuentemente en
neonatos y lactantes. Es ocasionado por la toxina exfoliativa tipo A del Staph. Aureus; esta toxina actúa como una serina proteasa de
desmogleína 1, ocasionando de esta manera la acantólisis subcorneal que conduce a la formación de ampollas. Las lesiones se pueden
localizar en cualquier área de la superficie corporal, pero lo más frecuente es alrededor de los orificios de la boca, nariz, área genital y
en las extremidades Clínica:pueden presentar fiebre hasta de 40°C, ampollas flácida, que se rompen con facilidad en el curso de 1 a 2
días, dejando erosiones cubiertas por una costra periférica fina, de ahí que también se conozca como impétigo circinado; las lesiones
aparecen usualmente agrupadas. Existe una tendencia a la curación espontánea sin cicatriz, pero pueden dejar hipopigmentación
residual temporal. TRATAMIENTO: Es igual en ambos tipos clínicos: 1) Realizar Antisepsia, por ej con Clohexidine, sulfato de cobre l
1x1000 con eliminación de las costras., Mejorar la higiene persona y recortar las uñas. 2) Terapia Tópica, con antibacterianos
preferiblemente de origen sintético, con espectro sobre los agentes causales más frecuentes: por ej mupirocina, ácido fusídico o más
recientemente retapamulina. 3) Si hay más de 2 áreas corporales afectadas , presencia de muchas lesiones, enfermedad sitémica

CURSO ENARM CMN SIGLO XXI TEL: 36246001 Pharmed Solutions Institute PÁGINA 388
MANUAL DE TRABAJO DEL CURSO ENARM CMN SIGLO XXI
subyacente, como la Dermatitis Atópica, Inmunosupresión ó varios familiares afectados , se debe agregar terapia sistémica, por ej:
amoxilinia con Ac Clavulanico , dicloxacilina, cefalexina, cefradina , macrólidos u otros si hay resistencia bacteriana. Si no hay una buena
respuesta clínica en forma rápida se debería cultivar y realizar antibiograma

CASO CLINICO
Se trata de escolar masculino de seis años de edad quien es traído por su madre por presentar lesiones ampollosas en las manos,
pecho, espalda y brazos de color amarillento con ardor e hipertermia de 39ºc;.

PREGUNTA
Cual es la conducta a seguir inmediata mas adecuada?

RESPUESTA
a.- Iniciar antibiótico.
b.- Administrar antipirético.
c.- Hidratación adecuada.
d.- Laboratorio de rutina.

PREGUNTA
Considerando la exploración del caso cual es la conducta terapéutica para la etiológica mas adecuada?

RESPUESTA
a.- Mupirocina.
b.- Acido fusidico.
c.- Penicilina.
d.- Amoxicilina.

ERISIPELA. CIENCIAS BASICAS: Es una Infección bacteriana profunda dermoepidérmica pero más superficial que la Celulitis, localizada
en dermis profunda y zona superior del tejido celular subcutáneo con afectación linfática significativa. Casi siempre muy bien
delimitada y con tendencia a progresar hacia la periferia. Es una enfermedad potencialmente transmisible y reviste mayor gravedad en
los niños. PATOGENIA: Los agentes causales más comunes son: Streptococccus pyogenes o Streptococcus beta-hemolítico del grupo A,
con menor frecuencia por Streptococcus de los grupos B, C, D o G, sobre todo en pacientes postquirúrgicos ó con otras enfermedades.
Los del grupo B suelen ser responsables de la erisipela del recién nacido. Otros gérmenes que pueden ocasionar erisipela: son
Staphylococcus aureus, Pneumococcus, Klebsiella pneumoniae, Yersinia enterocolítica y Haemophilus influenzae tipo b. La lesión
comienza a través de una pequeña solución de continuidad de la piel o de las mucosas, que constituye la puerta de entrada y que en
muchas ocasiones pasa inadvertida, por ejemplo en pacientes con ulceras, prurigo, intertrigo, tiña pedis, etc ó paciente diabéticos o
imunosuprimidos. De forma más excepcional se debe a una diseminación hematógena, procedente de focos a distancia, por ejemplo en
dientes o faringe. El periodo de incubación es de 2 a 5 días, la localización más frecuente son los miembros inferiores, puede localizarse
en cara y área periumbilical en caso de los lactantes. DIAGNOSTICO: Precede a la lesiones cutáneas síntomas como fiebre, escalofrío ,
náuseas , cefaleas y vómitos, malestar general y dolor local ; las lesiones tienen comienzo abrupto y se presentan como una placa
ardorosa, eritematosa de color rojo intenso, indurada, edematosa, caliente, dolorosa a la palpación y bien delimitada , su crecimiento
es centrífugo con un borde activo y bien definido, con la zona central de la placa aclarándose ; en algunas formas menos usuales se
pueden presentar vesículas, ampollas, pústulas y lesiones necróticas. Su topografía más común es en las extremidades inferiores y en la
cara a nivel de las mejillas. Se acompaña con frecuencia de adenopatías satélites regionales. Habitualmente las lesiones inician su
involución en 10-12 días descamándose la región afectada sin dejar cicatriz. Habitualmente el diagnóstico es clínico pero si es posible
obtener material de la puerta de entrada se debe realizar Gram y cultivo. TRATAMIENTO: Antisepsia. Antibiótico Tópico. Antibiótico
Parenteral se puede iniciar con Penicilina (800,000 UI de penicilina procainica c/24 hrs por 8-10 días, continuar con benzatinica
1200,000 UI cada 8 días por 3-4 semanas), Cefalosporinas, Amoxicilina + Ac. Clavulánico, y en los países con cepas Meticilino-
resistentes se pueden utilizar de acuerdo a estudios epidemiológicos previos: Trimetoprim-sulfametoxazole, Clindamicina, Rifampicina-,
Linezolid y Vancomicina. Si hay alergia a penicilinas se puede usar eritromicina. Para evitar recidivas usar sultrim dos tabletas diarias por
8-10 dias. COMPLICACIONES: incluyen bacteremia, abscesos, gangrena, linfangitis, celulitis, septicemia, tromboflebitis y
glomerulonefritis. Cuando se trata de brotes repetidos de erisipela la afección de los vasos linfáticos que al fibrosarse originan estados
elefantiásicos, conocidos como elefantiasis nostra.

CASO CLINICO
Se trata de femenino de 2 años de edad la cual cuenta con el antecedente de haber jugado en la tierra y producirse escoriasiones en
miembros pélvicos, a las 48 hrs posteriores inicia con fiebre no cuantificada, enrojecimiento y edema en la zona, la cual se encuentra
caliente y eritematosa a la exploración.

PREGUNTA
Cual es la conducta a seguir.

RESPUESTA
a.- Limpieza y tratamiento ambulatorio con penicilina.
b.- Debridación y tratamiento hospitalario.
c.- Limpieza y alta con signos de alarma.
d.- Limpieza local, vendaje y penicilina.

CURSO ENARM CMN SIGLO XXI TEL: 36246001 Pharmed Solutions Institute PÁGINA 389
MANUAL DE TRABAJO DEL CURSO ENARM CMN SIGLO XXI

PIODERMIA GANGRENOSA. CIENCIAS BASICAS: Fagadenismo geométrico de Brocq, a pesar del nombre este cuadro de causa
desconocida, dista mucho de ser una enfermedad bacteriana, probablemente sea más una inmunopatia que una verdadera infección
aun cuando no pueda descartarse del todo la intervención de bacterias. PATOGENIA: Su etiología es desconocida y se invocan
alteraciones inmunológicas aun no bien determinadas, se piensa en una reactividad tisular local, necrótica a antígenos microbiana.
DIAGNOSTICO: En cualquier área del cuerpo sobre todo en las extremidades, aparece repentinamente en personas en aparente estado
de salud, un área eritematosa que pronto se cubre de pústulas o ampollas y se ulcera rápidamente. Esta ulceración se extiende con
contornos geográficos, sus bordes son levantados y pueden aparecer abscesos en la periferia habitualmente estériles, el fondo es sucio,
sangrante o necrótico. En poco tiempo estas lesiones que profundizan por lo general poco, han abarcado extensas zonas de la piel. La
sintomatología no corresponde a la intensidad del proceso, pero puede haber dolor y ataque al estado general del paciente. Es
frecuente que este proceso coincida con artritis, colitis ulcerosa, enteritis regional de Crhon o enfermedades hematológicas con las
cuales no hay necesariamente una relación causa-efecto. TRATAMIENTO: Es empírico, se han usado los corticoesteroides, la colchicina,
la diaminodifenilsulfona, la clofazimina, la talidomina. Algunos responden bien a determinados fármacos y otros casoso a otros o no
responden. El proceso de repente y sin causa aparente se detiene y se inicia la cicatrización

CASO CLINICO
Femenino de 17 años que consultó por lesiones ampollosas dolorosas, de contenido claro, en rodilla derecha, acompañada de fiebre
con escalofríos, de 1 semana de evolución. Las mismas lesiones aparecen gradualmente en toda la pierna derecha, antebrazo derecho y
glúteo izquierdo, que revientan y dejan la piel exulcerada. Ingresó febril, estaba pálida. En piel se observan múltiples lesiones
ampollosas exulceradas, con fondo fibrinoso, muy dolorosas, ubicadas en rodilla y pierna derecha, otras más pequeñas en muñeca
derecha y glúteo izquierdo.

PREGUNTA
Cual es la conducta a seguir ahora?

RESPUESTA
a.- Limpieza y tratamiento ambulatorio con penicilina.
b.- Debridación y tratamiento hospitalario con corticoides y antibióticos.
c.- Limpieza y alta con signos de alarma.
d.- Limpieza local, vendaje y penicilina.

TUBERCULOSIS (TB) CUTANEA. CIENCIAS BASICAS: La TB cutánea se considera en la mayoría de los casos de reinfección, la
primoinfección es excepcional, aunque posible presentándose el complejo primario en la piel: nódulo cutáneo, linfangitis y
adenopatías. SALUD PUBLICA: Mycobacterium tuberculosis, quizá el germen de mayor ubicuidad en el mundo y en el organismo pues
no hay órgano que se escape a su agresión. La tuberculosis sigue siendo un problema de salud pública, especialmente en la forma
pulmonar. La TB cutánea ocupa el 4to o 5to lugar, en cuanto a localización del bacilo de Koch. Estadísticas mexicanos en varios años
muestran cifras que van de 1-3% de Tb cutánea por cada 114 pacientes y cerca de 20% son niños menores de 15 años. PATOGENIA:
Casi en todos los casos, los pacientes han recibido ya la primoinfección pulmonar inoculado con M. tuberculosis bovina y hominis,
presentando una respuesta positiva al PDD. A partir de este foco original mediante reinfeccionendogena.se diseminan los bacilos a la
piel, otras veces la reinfección es exógena por llegada de nuevos bacilos del exterior. De acuerdo con esta vía de reinfección, de la
virulencia del bacilo y sobre todo de la respuesta inmunológica (hipersensibilidad) del organismo ante el bacilo, se van a producir
lesiones en la piel y que parecen entidades diferentes. CLASIFICACION: de Latapí, Escalona y Estrada; se consideran 2 grupos básicos, el
primer grupo está formado por entidades en las cuales hay respuesta normérgica al bacilo. El segundo grupo comprende las llamadas
tuberculoides, que como su nombre indica son lesiones resultados más de la respuesta exagerada, hiperérgica al bacilo. En este caso no
es posible encontrar bacilos en las lesiones y la respuesta a los antígenos bacilares es muy intensa. TB COLICUATIVA: También conocida
como escrufulosis o escrofulodermia, en México es la variedad más frecuente. Se presenta sobre todo en niños y jóvenes y predomina
en las áreas de población más desprotegidas. Es secundaria habitualmente a TB de ganglios, huesos y articulaciones. Su topografía
habitual es donde hay ganglios, como regiones supraclaviculares, las axilas, las ingles, uni o bilateral. Puede estar en región esternal,
codos, rodillas. Las lesiones son siempre nódulos y gomas, refieren “pequeñas bolitas en el cuello”, es decir ganglios infartados, no
dolorosos, no móviles, un buen día se fijan a la piel y esta se pone eritematosa e infiltrados y se inicia la formación de un nódulo, que se
reblandece y se abre al exterior dejando salir pus de color amarillo claro y espeso, pronto toda la región se convierte en un plastrón
endurecido (nódulos, gomas, abscesos fríos), es posible que dejen cicatrices deformantes y retractiles. Puede haber febrícula vespertina
o franca fiebre, anorexia y adelgazamiento, y a veces síntomas de TB activa. TB VERRUGOSA: A menudo inoculada en personas que
manejan material contaminado como mozos de anfiteatro, matanceros, carniceros, laboratoristas. Se localiza en partes distales, de os
miembros como los pies y las manos y algunas veces en nalgas. En sitio de inoculación aparece el tubérculo anatómico, que es nódulo
verrugoso, que es punto de partida de las lesiones que son siempre placas verrugosas o vegetante de tamaños variables circulares u
ovales, bien limitadas y cuya superficie siempre es áspera, con costras melicericosanguineas, a veces se observan pústulas. En algunas
placas se ve cicatrización central con crecimiento periférico y puede haber compromiso de los linfáticos superficiales y profundos de la
región y producirse una estasis linfática que ocasiona edema y más verrugosidades. TB LUPOSA: Era muy frecuente en el siglo pasado.
Es más frecuente en niños y jóvenes y su clásica topografía es en la cara; mejillas y dorso de nariz, disposición en “mariposa”, también
puede afectar a los pabellones auriculares. La lesión fundamental es el nódulo pequeño llamado lupoma que se cubre de escamas y
verrugosidades que lo ocultan, de tal manera que se forman placas eritematosas, escamosas, verrugosas, circulares, bien limitadas de
crecimiento periférico y cicatrización central y con tendencia a la ulceración. En el centro solían producir extensas y profundas
destrucciones, el subtabique nasal se destruía y la nariz tomaba el aspecto de “pico de loro”. TB MILIAR AGUDA: Muy rara se
presentaba en niños y jóvenes con TB avanzada y baja reactividad a los antígenos. Son pequeños nódulos rojizos acuminados, a veces
ulcerado y cubiertos de costras en diferentes partes del cuerpo, en especial en el centro de la cara, que cicatrizan espontáneamente.

CURSO ENARM CMN SIGLO XXI TEL: 36246001 Pharmed Solutions Institute PÁGINA 390
MANUAL DE TRABAJO DEL CURSO ENARM CMN SIGLO XXI
TB NODULAR PROFUNDA: Descrita por Bazin, con el nombre de eritema indurado, casi exclusiva de mujeres de localización única en las
piernas sobre todo en las caras posteriores y caracterizada por nódulos profundos, de evolución crónica y recidivante, muy dolorosos y
que en su evolución dejan zonas atróficas depreimidas que deforman las piernas. TB NODULO NECROTICA: Es menos frecuente que la
nodular profunda, aparece en personas jóvenes con topografía en partes salientes; codos, rodillas, nalgas y en ocasiones en la cara,
sobre todo en los pabellones auriculares. La lesión es un pequeño nódulo que sufre una necrosis central y así se cubre de una costra
negruzca que al caer deja una cicatriz varioliforme. Son asintomáticos y evolucionan por brotes. TB MICRONODULAR: Es muy rara y
pasa inadvertida, son nódulos de 1-2mm que forman placas de aspecto folicular en las regiones lumbares u otras partes del tronco, son
asintomáticas. DIAGNOSTICO: Es más bien de eliminación, dado que la comprobación del agente causal no es fácil en todos los casos. El
laboratorio nos puede ayudar sobre todo para descartar procesos que suelen semejarse a las lesiones tuberculosas. Hallazgo del bacilo;
puede hacerse por medio de baciloscopia y de biopsia difícil de encontrar en TB colicuativa y ulcerosa. El cultivo en medios apropiados
como Lowestein es difícil de lograr. Histopatología; por definición la imagen de las lesiones tuberculosas es el clásico granuloma
tuberculoide, constituido por linfocitos, células epiteloides y células gigantes tipo Langhans, resultado de la unión de las segundas., sin
embargo no es patognomónica de la tuberculosis, por lo que solo será sugestiva de tuberculosis. Intradermoreacción con PPD; no es
diagnostica ya que solo nos indica un contacto con el bacilo de Koch y un alto porcentaje de la población da positivo. Tiene importancia
cuando es negativa, lo cual significa que no tienen un cuadro de origen tuberculoso. TRATAMIENTO: Se usan las mismas drogas que en
la tuberculosis en general y a dosis semejantes. La estreptomicina es muy útil a dosis de 1 gr diario o cada tercer día, 50-60grs como
dosis total. La isoniacida que se dice bacteriostática se usa a 5-8mg/kg. El etambutol a dosis de 15-20mg/kg, es más costoso y tienen
efectos colaterales en nervio óptico, es reversible. Rifampicina a dosis de 10-20 mg/kg, para un adulto se aconsejan 600mgs, para niño
la mitad. No usar monoterapia. Tratamiento ideal es; 1 gr de estreptomicina cada tercer día, 600mgs de isoniacida diarios o bien
estreptomicina + etambutol. La TB colicuativa tarda de 3-6 meses en curar. La TB verrugosa y luposa requieren menos tiempo. En la
actualidad se está aplicando con éxito un triple tratamiento, isoniacida, rifampicina, pirazinamida.

CASO CLINICO
Niña de 10 años que consulta por una lesión en mejilla izquierda seguida de la aparición de adenopatía cervical, de dos meses de
evolución. Al examen físico, se observa una pequeña úlcera de 1 cm de diámetro, con leve descamación periférica y adenopatía satélite.

PREGUNTA
Cual es la conducta a seguir?

RESPUESTA
a.- Biopsia de la lesión.
b.- PPD.
c.- BAAR.
d.- Cultivo de la lesión.

PITIRIASIS ALBA (MAL DEL PINTO). CIENCIAS BASICAS: Es una condición benigna de lapiel común en la infancia, descrita por primera
vez hace más de 80 años y su importancia radica en que es un motivo de consulta frecuente debido a su curso crónico, tendencia a
reaparecer y su impacto estético. SALUD PUBLICA: La pitiriasis alba se encuentra en todas partes del mundo. Afecta a entre 1.9-5.25%
de los niños en edades entre 6-16 años. En general, no hay predisposición en cuanto a sexo; sin embargo, algunas series describen una
mayor afección de los varones, así como a individuos de color de piel oscura.3 Se cree que la mayor frecuencia de pitiriasis alba en
personas de piel oscura es debido al hecho de que su piel se tiñe más fácilmente excepto en las áreas afectadas. Hay reportes que
muestran mayor incidencia en niños escolares de baja situación socioeconómica. Desde el punto de vista clínico afecta la cara en el 50%
de los casos. PATOGENIA: La etiología no está bien establecida y a la condición se le han adjudicado diversas denominaciones, como
eritema streptogenes, pitiriasis streptogenes, e impetigo furfuracea, usualmente relacionadas con los agentes etiológicos propuestos,
sin embargo, no se ha identificado una asociación causal definitiva. Microorganismos como Pityrosporum, Streptococcus, Aspergillus y
Staphylococcus, se han implicado en su etiología pero ninguno de ellos se ha confirmado como agente causal. Varios factores además
de los infecciosos, se han asociado con la etiología, la exposición solar, la humedad relativa del aire, la altitud y el viento. El uso de
jabones abrasivos, duchas excesivamente largas y la temperatura caliente del agua durante el baño también se han relacionado con la
xerosis que se aprecia en esta entidad. En cuanto a la exposición solar, se plantea que la radiación ultravioleta induce una irritación
excesiva de la piel que conlleva a una reacción inflamatoria y que además la acción directa de la luz solar sobre los melanocitos
modifica su número y su función conduciendo a una disminución de la intensidad del color de las lesiones en la pitiriasis alba. Esto
además se apoya en el hecho de presencia de lesiones en áreas fotoexpuestas y en niños en edades en las cuales hay mayor número de
actividades al aire libre. Se ha observado que el uso de protector solar frecuente FPS > 15 reduce el desarrollo de pitiriasis alba. La
condición dermatológica que se asocia principalmente con la pitiriasis alba es la dermatitis atópica y la presencia de deficiencias
nutricionales así como la xerosis es común a ambas, por lo que se ha implicado en la patogénesis de la misma. Las deficiencias
nutricionales involucradas son de vitaminas y bajos niveles de cobre, el cobre es un cofactor para la tirosinasa, enzima necesaria para la
producción de melanina, por lo que su deficiencia puede jugar papel patogénico en esta entidad. DIAGNOSTICO: Frecuentemente se
inicia como placas rosadas con un borde elevado que luego de varias semanas se desvanece dejando una mancha pálida cubierta por
una descamación blanquecina polvorienta. Posteriormente progresa a máculas hipopigmentadas de bordes difusos, de tamaño variable
entre 0.5 a 5 cm de diámetro. Aunque puede haber ligero prurito en general las lesiones son asintomáticas, por lo que generalmente el
paciente no consulta por ellos y se observan incidentalmente en el examen físico. Aunque en el caso de los niños, puede ser motivo de
preocupación común en las madres, ya que la mayoría de las lesiones se localizan en la cara, principalmente en la frente y la zona
malar, pero también pueden encontrarse en las extremidades superiores y ocasionalmente en las inferiores. Las lesiones pueden
persistir por 6 meses a 7 años, pero este curso puede prolongarse en los pacientes atópicos; en general es una condición autolimitada.
Por otro lado, las lesiones pueden ser más visibles en el verano, cuando la piel circundante está bronceada. Luego de la resolución las
lesiones pueden reaparecer en la misma localización. Variantes clínicas: PITIRIASIS ALBA EXTENSA: esta entidad se caracteriza por

CURSO ENARM CMN SIGLO XXI TEL: 36246001 Pharmed Solutions Institute PÁGINA 391
MANUAL DE TRABAJO DEL CURSO ENARM CMN SIGLO XXI
lesiones de pitiriasis alba generalizadas y no precedidas de eritema. Se observa más frecuentemente en los adultos. Generalmente se
afectan la porción inferior del tronco de manera simétrica. Es asintomática y los pacientes usualmente no tienen antecedentes o
historia de atopia. La histología no es específica, se observa un número reducido de melanocitos funcionales con menor número de
melanosomas, pero su distribución y transferencia a los queratinocitos no está perturbada. PITIRIASIS ALBA PIGMENTADA: es una
variante que puede estar asociada con una infección por dermatofitos superficial y la forma clásica de PA. Se observa una
hiperpigmentación azulada rodeada de un área hipopigmentada y descamativa muy parecida a la pitiriasis alba clásica. Casi siempre
aparece en la cara y rara vez hay compromiso extrafacial. El área pigmentada se atribuye a depósitos de melanina en la dermis, lo que
no se observa en la pitiriasis alba clásica. En general no es necesario ya que el diagnóstico clínico suele ser obvio, además de que la
histología no es específica. Lo destacable es una menor cantidad de melanocitos y melanosomas en la capa basal de la epidermis, sin
embargo, esto se ha asociado a inflamación determinada por cierto grado de espongiosis, lo que interfiere con la producción de
melanina. Desde el punto de vista histológico puede llegarse a un diagnóstico si se observa: pigmentación irregular de la melanina en la
capa basal, taponamiento folicular, espongiosis folicular y glándulas sebáceas atróficas. Además de infiltrado linfocitario y edema en la
dermis. Estos cambios son variables dependiendo de la etapa en la que se encuentre la enfermedad. TRATAMIENTO: Inicialmente en
convencer al paciente de que la enfermedad es benigna y autolimitada ya que hasta ahora, ninguna terapia es completamente exitosa.
Se recomienda limitar la exposición solar, uso regular de protector solar FPS > 15 y reducir la frecuencia y la temperatura de los baños.
Los emolientes y cremas hidratantes ayudan a disminuir la xerosis y la irritación. En los casos en etapa inflamatoria la desonida o la
hidrocortisona tópica al 1% pueden ayudar a la resolución de las lesiones. En los niños solo deben prescribirse esteroides de baja
potencia no halogenados. La forma extensa de PA no responde a los esteroides tópicos pero si lo hace a terapia con PUVA.
Recientemente se publicó la efectividad de tacrolimus al 0.1% en el tratamiento de esta entidad. El tacrolimus es un macrólido que
bloquea la función catalítica de la calcineurina, inhibiendo la síntesis y la liberación de IL-2. La eficacia y la seguridad de tacrolimus en
ungüento en el tratamiento de la dermatitis atópica está bien documentada. Sin embargo, el mecanismo de acción en la pitiriasis alba
no es bien entendido. Una intervención terapéutica definitiva será posible cuando se comprenda mejor la patogénesis de esta entidad,
a la cual no se le da mucha importancia en la literatura médica, lo que ha conllevado a la escasez de conocimientos precisos en cuanto a
su patogenia y de estudios controlados para dilucidar una terapéutica totalmente efectiva.

CASO CLINICO
Niño de 6 años de edad el cual acude a consulta de rutina, muestra en la cara manchas blancas, bordes irregulares no bien definidas,
con leve descamamiento a la exploración, no son pruriginosas?

PREGUNTA
Cual es la conducta a seguir menos adecuada?

RESPUESTA
a.- Protectores solares.
b.- Baños cortos con agua templada.
c.- betametasona.
d.- Tacrolimus.

HERPES SIMPLE. CIENCIAS BASICAS: Los virus de herpes se encuentran ampliamente distribuidos en la naturaleza y la mayoría de las
especies animales son hospederos naturales de más de uno. Se han aislado y caracterizado más de 100, varios de los cuales afectan al
humano: herpes simple tipo 1 y tipo 2 (VHS-1, VHS-2), varicela zoster (VVZ), citomegalovirus (CMV), Epstein Barr, (VEB), virus herpes
humano 6 (VHH6), virus herpes humano 7 (VHH7), virus herpes humano 8 (VHH8) y otros. SALUD PUBLICA: La infección por VHS-1 es
frecuente en sitios hacinados y con condiciones precarias de higiene se tienen porcentajes de 90% de la población tienen anticuerpos
antivirales El más frecuente en niños. El VHS-1 en ubicación oral se transmite por saliva, besos, por compartir vasos, cepillos de dientes
y en otras partes del cuerpo se debe a contacto del virus con la piel, se autotransmite con frecuencia, principalmente a los ojos. La
infección por VHS-2 depende de la actividad sexual. El VHS-2 se transmite por secreciones vaginales, contacto sexual (prepuberes) y al
neonato durante el paso por el canal de parto infectado. La posibilidad de que VHS-1 y VHS-2 establezcan infecciones latentes con
recidivas asintomáticas favorece su transmisión, ya que un individuo infectado puede ser transmisor durante toda su vida. Los virus
infectantes se encuentran en el líquido de las vesículas. PATOGENIA: La patogenia de VHS-1 y VHS-2 es similar, con infección primaria
generalmente asintomática, aunque pueden presentarse lesiones vesiculares. El virus inicia la infección en las membranas de las
mucosas, se replica en las células mucoepitelilales originando infección lítica y se disemina a las células adyacentes y neuronas que
inervan el sitio donde se inició la infección aguda. La infección latente en la neurona no produce daño aparente, pero diferentes
estímulos la pueden reactivar. Una vez reactivado, el virus se multiplica, viaja a lo largo del nervio en forma centrifuga y ocasiona lesión
en la terminal del nervio, por lo tanto todas las recidivas se producen en el mismo sitio. La expresión del genoma se requiere para la
reactivación, pero no para el establecimiento de la latencia. El mecanismo para su establecimiento se desconoce, sin embargo, se
piensa que para la expresión del genoma se requiere una proteína celular, tan es así que no en todas las estirpes celulares establece
latencia. El tipo de infección que resulta depende del estado inmune del individuo; los sujetos susceptibles desarrollan infección
primaria después de la primera exposición al virus. Sujetos seropositivos pueden ser reinfectados con virus de otro tipo. VHS-1 y VHS-2
se transmiten por diferentes vías e infectan diferentes sitios del cuerpo. A grandes rasgos se considera que el HSV-1 infecta de la
cintura para arriba y el VHS-2 de la cintura para abajo, sin embargo esta diferenciación no es estricta. DIAGNOSTICO: La primoinfección
por VHS-1 (ORAL): puede presentarse de distintas formas: La gingivoestomatitis herpética; forma más frecuente de infección primaria o
inicial en niños. Se trata de un cuadro agudo con fiebre y mal estado general, aparición de lesiones vesiculosas y erosivas en mucosa
oral que ocasionan odinofagia y halitosis características, dificultando la alimentación. Pueden existir lesiones vesiculosas, en labios e
incluso en la cara, agrupadas en racimos (típicas). Podemos encontrar adenopatías submandibulares y el cuadro clínico es auto limitado
(7-10dias). Otras formas de primoinfección, pueden aparecer lesiones en cualquier localización, solo es precisa una fuente de contagio
(familiar), una puerta de entrada (una erosión, arañazo) y un huésped susceptible (niño). Podemos encontrar casos de primoinfecciónes

CURSO ENARM CMN SIGLO XXI TEL: 36246001 Pharmed Solutions Institute PÁGINA 392
MANUAL DE TRABAJO DEL CURSO ENARM CMN SIGLO XXI
con lesiones faciales extensas, en un miembro, oculares, y lo que es más frecuente y bien definido, el panadizo herpético, caracterizado
por la aparición de lesiones cutáneas vesiculosas, que afectan al pulpejo y región periungueal de un dedo, muy dolorosas, que cursan
con fiebre y adenopatías satélites. El cuadro también es autoinoculativo, pero dura algo más; unas 2 semanas. Es bien conocida por
todos la capacidad de los virus simple de permanecer en el huésped, acantonados y recurrir o producir síntomas ante diversas
circunstancias; enfermedades debilitantes, infecciones, fiebre, exposición solar. La clínica es variable pero suele localizarse en la cara;
labios, región perioral, mejillas. Otras localizaciones son más raras en la infancia. El aspecto clínico es muy característico con la
aparición de lesiones vesiculosas, agrupadas en racimo, sobre una piel eritematosa, que con el tiempo dan lugar a una costra. El cuadro
puede ir precedido de sensación prodrómica de dolor, quemazón o disestesias de difícil descripción e interpretación por parte de
nuestros pacientes y no suele acompañarse de fiebre, mal estado general, ni adenopatías. La infección por VHS-1 puede originar
cuadros clínicos de variada severidad, que oscilan desde la gingovoestomatitis, herpes labial, panadizo herpético, meningitis, encefalitis
con alta mortalidad y queratitis herpética que a su vez puede originar ceguera. VHS-2 (GENITAL): En general no ocurre en edad
pediátrica, cuando aparece en un menor debe investigarse la posibilidad de abusos sexuales. La primoinfección es superponible a las
formas orofacilaes, pero en la región genital. El cuadro también es agudo se acompaña, de dolor, fiebre, dificultad para la micción y
adenopatías. Las lesiones vesiculosas, erosivas, típicamente agrupadas en racimos nos darán sospecha diagnostica. El diagnóstico de
laboratorio económico y rápido se realiza mediante la prueba de Tzank que consiste en hacer una impronta de las células y teñirlas con
el colorante de Wright o Giemsa, y observar células fusionadas con varios núcleos, sincitios, así como inclusiones nucleares de Cowdry.
No es posible confirmar la presencia del virus por medio de esta técnica ya que otros virus producen el mismo efecto en las células,
inclusive el VVZ. Actualmente se hace uso de técnicas inmunoenzimáticas, biológicas, bioquímicas y de biología molecular para detectar
anticuerpos o antígenos virales. La detección de anticuerpos solo es de utilidad para detección de la primo infección y estudios
epidemiológicos. TRATAMIENTO: Existen antivirales efectivos utilizados en el tratamiento de infecciones por VHS-1 y VHS-2, entre ellos
famciclovirn (125mgs c/12hrs por 5 dias), Aciclovir (200mg 5 veces al dia por 5 dias), valaciclovir (500mgs c/8 hrs por 5 dias). Los
antivirales no eliminan las partículas virales que se encuentran en ganglios neurales, solo impiden su replicación, por lo que pueden
presentarse reactivaciones. En estos casos, el uso de dosis mínimas de los fármacos, durante un tiempo prolongado, tratamiento
denominado supresivo, se indica en los pacientes con brotes constantes, prolongados o intensos, aunque debe contemplarse la
posibilidad de resistencias. Los VHS se transmiten a través de las secreciones de lesiones, por lo es recomendable evitar contactos
durante la lesión activa. El tratamiento tópico debe hacerse mediante fomentos con sulfato de cobre 1/1000 o sulfato de cinc 1/1000 y
antibiótico tópico (mupirocina, ac. Fusidico) COMPLICACIONES: La más frecuente es la impetiginizacion o sobreinfección bacteriana,
fundamentalmente estreptocócica y estafilocócica. Las lesiones oculares en particular, la queratitis puede llevar a ceguera. INFECCION
HERPETICA NEONATAL: La infección herpética durante la gestación puede dar lugar a manifestaciones en el RN y en el feto. La vía de
contagio puede ser transplacentaria o genital (en el canal del parto, o ascendente, con o sin rotura de membranas). Existen 2
posibilidades: infección primaria antes de la 20 SDG, aborto, muerte fetal, hidrocefalia y coriorretinitis. Herpes simple neonatal, en más
de 90% de los casos es en relación con la primoinfección herpética materna durante la gestación, solo 5% se produce afección fetal en
las recurrencias. Cuando la infección ha sido adquirida en la vida intrauterina (5-15%), la sintomatología aparece en las primeras 24-
48hrs. En caso de infecciones intrapartum, generalmente por VHS-2 (madre) o en el caso de VHS-1 (padres, familiares) la
sintomatología se produce entre los 2-20 días. Clínica: afectación severa del estado general en un neonato aparentemente sano,
lesiones cutáneas, coriorretinitis, y afección al SNC, afectación sistémica-visceral, lesiones cutáneas en más de 50% de los casos:
vesículas, ampollas, pústulas, cicatrices, eritema generalizado, erosiones, anoniquia. Mortalidad elevada (40%) y elevada morbilidad.
Tratamiento precoz cesárea.

CASO CLINICO
Femenino de 15 años de edad la cual acude a consulta por presentar desde hace dos días, vesículas agrupadas en el labio inferior, que
se transforman en pustulas y costras hemorrágicas, cuenta antecedente de prurito y sensación de hormigueo en la zona. Comenta la
paciente que en el último año se han repetido episodios de vesiculas similares en varias ocacion, antecentes de importancia
actualmente embarazo de 22 semanas, ha tenido multiples parejas y es madre soltera.

PREGUNTA
Cual es la conducta a seguir mas adecuada?

RESPUESTA
a.- Tratamiento antiviral doble esquema.
b.- Realizar USG para identificar anormalidades.
c.- Tratar y esperar parto nornal.
d.- Tratamiento especifico y programa cesarea aportuna.

HERPES ZOSTER. CIENCIAS BASICAS: El virus de la varicela (VVZ) origina varicela y su reactivación causa herpes zoster. SALUD PUBLICA:
Más frecuente en el adulto que en la infancia (0.74 casos por 1000 individuos al año). Suele ocurrir en niños que pasaron varicela
precoz (primeros 2 años de vida). El virus se transmite por contacto directo, por gotitas de aerosol y por vía área en comunidades;
también puede haber contagio por contacto con vesículas cutáneas. Es altamente transmisible. Se dispone de eficaz vacuna atenuada.
PATOGENIA: Se caracteriza por una infección neurológica y dermatológica, posterior a un episodio de varicela (primoinfección) en los
ganglios sensitivos espinales y craneales (donde esta latente el VVZ). Una vez reactivado, el virus se mueve a lo largo de las fibras
sensitivas hasta él área de piel que inervan, que usualmente afectan de 1-3 dermatomas. Dicha reactivación ocurre, por lo general, una
vez en la vida, sobre todo en pacientes de la tercera edad o en aquellos inmunodeprimidos (por HIV, cáncer, tratamiento con
corticoesteroides, fármacos inmunosupresivos, por ejemplo). DIAGNOSTICO: Las lesiones cutáneas consisten en lesiones papulosas
iniciales agrupadas en racimos sobre la piel eritematosa, las pápulas se convierten en vesículas en 12-24 hrs y luego estas se rompen y
dan lugar a costras. La erupción cura en 7-14 días y suele acompañarse de dolor. El dolor puede ser prodrómico, es decir preceder a las
lesiones cutáneas, acompañarlas e incluso persistir durante meses tras la resolución clínica dermatológica, en este último supuesto se

CURSO ENARM CMN SIGLO XXI TEL: 36246001 Pharmed Solutions Institute PÁGINA 393
MANUAL DE TRABAJO DEL CURSO ENARM CMN SIGLO XXI
habla de neuralgia postherpética, esta implica la pérdida de autonomía y una calidad de vida pobre. La infección por varicela puede
ocasionar ocasionalmente que el virus atraviese la placenta, dando lugar al síndrome de varicela congénita que puede ser leve o severo.
Cuando la madre ha estado en contacto con casos de varicela es recomendable determinar el nivel de anticuerpos anti-varicela. El
diagnóstico se basa en la prueba de Tzank, con las dificultades mencionadas; el virus se puede aislar de las vesículas y recientemente se
utilizan técnicas de biología molecular para su identificación. COMPLICACIONES. La neuralgia postherpética. En individuos
inmunocomprometidos puede dar lugar a enfermedades progresivas y graves, principalmente encefalitis post-infección y neumonía, en
algunos casos fatales. La respuesta inmune celular que generalmente es más potente en adultos que en niños causa durante la
infección primaria lesiones y cuadros graves, principalmente en pulmones, y da lugar a 20-30% de los pacientes con neumonía
intersticial. TRATAMIENTO: Resultados satisfactorios se han obtenido con inhibidores de la síntesis del ADN (aciclovir y derivados), sin
embargo se requieren dosis más altas que las recomendadas para HSV-1 y 2. HERPES ZOZTER EN LA INFANCIA: Las principales
características son: 1. Suele presentarse en niños que presentaron varicela en los 2 primeros años de vida. 2. Su curso es benigno y sin
complicaciones. 3. Es poco doloroso (no pródromos), no suele complicarse con neuralgia postherpetica. 4. Se recomienda investigar
existencia de inmunodepresión. 5. El tratamiento oral con antivirales solo está indicado en casos seleccionados, pero debe ser lo más
precoz posible.

CASO CLINICO
Masculino de 6 meses de edad quien manifestó una dermatosis diseminada al tronco, región inguinal derecha y la cara lateral interna
del muslo derecho en el tercio distal, asi como en el tercio medio y proximal de la pierna. La dermatosis era unilateral, constituida por
vesículas agrupadas sobre una base eritematosa y escasas costras hemáticas que confluida entre si, formando varias placas de tamaño
variable y de evolución aguda y dolorosa, se realizo diagnostico clínico de herpes zoster.

PREGUNTA
Cual es la conducta a seguir menos apropiada?

RESPUESTA
a.- Aciclovir 400 mg/kg/dosis.
b.- Fomentos con sultafo de cobe y oxido de cinc.
c.- Profilaxis antibiótica profiláctica.
d.- Cerco sanitario a contactos.

VERRUGAS VIRALES. CIENCIAS BASICAS: Son los virus del papiloma humano capaces de producir las verrugas virales y algunos cuadros
relacionados con la carcenogénesis. Se han descrito ya 25 tipos de virus. La palabra verrugosidad indica solamente una lesión dura,
anfractuosa, seca que histológicamente presenta hiperqueratosis y papilomatosis. La vegetación en cambio es blanda, anfractuosa,
friable y solo presenta la papilomatosis, por ello estas lesiones solo se presentan en zonas húmedas de la piel y en mucosas. Las
verrugas virales son neoformaciones epiteliales benignas que pueden afectar a cualquier persona, niños, adultos, hombres y mujeres,
constituyendo una de las 5 entidades que con más frecuencia se ven en la consulta. Su transmisibilidad es baja y sabemos que son
autoinoculables. Un estado inmunológico deteriorado permite sui extensión y persistencia. TIPOS: Vulgares, planas, plantares,
acuminadas y filiformes. VERRUGAS VULGARES: Son los populares “mezquinos” y son las más frecuentes. Predominan en los niños. Son
neoformaciones verrugosas solitarias o múltiples, de superficie anfractuosa, secas, duras, de color de la piel o más oscuras,
semiesféricas y bien limitadas, presentan en su superficie un fino puntilleo oscuro. Son indoloras a menos que se les traumatice y su
desaparición es espontánea y no deja secuelas, habituales en extremidades superiores. VERRUGAS PLANAS o juveniles: Son muy
pequeñas y numerosas, no más de 1mm y muy aplanadas, levantan de la superficie, así es que a primera vista parecen machas. De color
de la piel, llegan a sumar más de 100, localizadas de preferencia en las mejillas aunque también pueden verse en extremidades
superiores; dorso de manos o antebrazos. También son asintomáticas. VERRUGAS PLANTARES: Se les conoce vulgarmente como “ojos
de pescado” y como su nombre o indica se presentan en la planta del pie, la presión de peso del cuerpo las hunde en la gruesa capa
cornea, por lo que solemos ver “la base de pirámide”, y ello explica el dolor que las acompaña, únicas o múltiples, con pequeñas
hemorragias postraumáticas en su superficie. VERRUGAS ACUMINADAS: También llamadas condilomas acuminados o papilomas
venéreos, la mayoría se adquieren por la relación sexual, en niños a veces por desaseo. Asientan en sitios húmedos y calientes,
alrededor de orificios naturales; glande, labios mayores y menores, ano recto, boca, no se produce hipertrofia como en otras verrugas,
la lesión toma más bien un aspecto vegetante, como coliflor, pueden ser pequeñas a penas salientes (crestas en el término) o alcanzar
dimensiones monstruosas, deformando la región, se maceran e infectan y adquieren un olor muy desagradable. Debemos
diferenciarlas siempre de condilomas planos o sifílides papuloerosivas, muy ricas en treponemas. Son pápulas aplanadas y erosionadas
y no neoformaciones vegetantes, no tiende a persistir, evolucionan hacia la desaparición. Los condilomas acuminados no tienden a la
involución, persisten y crecen. VERRUGAS FILIFORMES y digitiformes: Se presentan en los pliegues: cuello axilas, ingles en forma de
neoformaciones alargadas, como hilos oscuros, algunas pediculadas o con prolongaciones transparentes. Se discute su etiología viral.
PRONOSTICO: Aunque se menciona el papel oncogénico de algunas variedades de parvovirus, no debe tomarse a las lesiones
verrugosas comunes como precancerosas, sin embargo en el caso de lesiones acuminadas, se piensa que pueden relacionarse con el
cáncer cervicouterino y se ha observado que las mujeres que presentan esta neoplasia tienen con más frecuencia verrugas acuminadas
y no solo eso, sino que se ha observado también que las parejas sexuales han padecido la infección genital por estos virus.
TRATAMIENTO: Involución excepto en las acuminadas. Es una crueldad innecesaria traumatizar a un niño quemándole sus verrugas con
ácidos mediante electrofulguración. Se han visto antiestéticas cicatrices, a veces queloides como resultado de estas iatrogénicas
maniobras. La destrucción de pocas lesiones puede hacerse con electrofulguración o congelándolas con nitrógeno líquido. Las
acuminadas requieren aplicación de soluciones de podofilina al 20-40% en solución alcohólica o en aceite, protegiendo tejidos vecinos
con vaselina o colodión. Igualmente son molestas las elctrofulguraciones de las verrugas plantares, pues solo se destruye una mínima
porción y es inútil poner al paciente en cama varios días, cuando la recidiva es la regla. Ya se sabe que las verrugas tienden a la
involución espontáneamente, está probado científicamente y con estudios estadísticos, la utilización de los placebos (en dosis mínimas,

CURSO ENARM CMN SIGLO XXI TEL: 36246001 Pharmed Solutions Institute PÁGINA 394
MANUAL DE TRABAJO DEL CURSO ENARM CMN SIGLO XXI
magnesia calcinada, glucosa, cloruro de sodio, BCG, levamisol) en estas afecciones tan orgánicas Debe tenerse en mente que las
verrugas virales tarde o temprano desaparecerán, no así las cicatrices que perduran toda la vida y recordaran al paciente los malos
tratamiento recibidos.

CASO CLINICO
Se trata de masculino de 17 años con dermatosis localizada en cara, área de la barba con extensión al cuallo constituida por numerosas
neoformaciones (90) que varia de 1 a 3 mm de diámetro con proyecciones digitiformes que dan aspecto verrugoso y papiliforme, del
color de la piel, solo presentaba prurito ocasional, estudios de rutina dentro de parámetros normales.

PREGUNTA
Cual es la consuta a seguir mas adecuada?

RESPUESTA
a.- Electrofulguracion.
b.- Nitrógeno líquido.
c.- Podofilina.
d.- 5-fluorouracilo.

MOLUSCO CONTAGIOSO (MC). CIENCIAS BASICAS: El molluscum contagiosum (MC) es una patología cutánea benigna ocasionada por
un virus DNA de la familia de los poxvirus. A pesar de tener una distribución por todo el mundo, su morbilidad es baja. No obstante,
actualmente existen más individuos afectos al añadirse los pacientes inmunocomprometidos. SALUD PUBLICA: El MC tiene una
distribución mundial y una incidencia de 4,5%. No tiene predilección racial o entre géneros. Los estudios epidemiológicos realizados
sugieren que la transmisión podría estar relacionada con factores tales como la humedad y calor del clima, la pobreza, hacinamiento y
la falta de higiene. Sin embargo, en otros trabajos se apoya más la idea del contacto entre personas, ya que se ha encontrado que se
puede desarrollar en el 35% de los miembros expuestos de una familia y que en la mayoría de los casos las familias poseen excelentes
estándares de higiene. En niños la transmisión se produce por contacto piel-piel, por fómites o por auto inoculación. La infección por
MC ocurre en todos los grupos etáreos, y al parecer ha venido aumentando en la última década. La mayor incidencia se observa en
niños menores de 5 años, por el mayor contacto físico casual entre ellos, y a la auto inoculación. La infección en lactantes es rara, esto
se explica por la persistencia de anticuerpos maternos. Otro pico en la incidencia se observa en adultos jóvenes, debido a la
propagación secundaria al contacto sexual. El MC está incrementándose cada vez más dentro de las enfermedades diagnosticadas en la
población sexualmente activa. La transmisión en adultos normalmente es vía sexual. Esta vía se apoya en la localización mayoritaria
encontrada en el área genital, la frecuencia de encontrar las parejas afectadas y la coexistencia de otras enfermedades de transmisión
sexual entre los pacientes afectos de MC. Otras vías posibles han sido a través de saunas, masajes, cirugía. Desde 1983 se han recogido
diversos casos de MC en pacientes con infección HIV. La prevalencia en este grupo de población es tan alta como del 5-18% .
PATOGENIA: El molluscum contagiosum es producido por un miembro del grupo de los poxvirus, no clasificado, de ultraestructura
análoga a la del virus de la viruela. El virus del MC es uno de los más grandes que causan enfermedad en los humanos, el vibrión
maduro es una partícula que mide 150x350 mm2. El genoma, al igual que en los demás poxvirus, es una única molécula linear de DNA
de doble hélice. Estudios recientes del DNA de los molluscum aislados confirman la presencia de dos diferentes subtipos, MCV I y MCV
II, con genomas de 185 kb y 195 kb respectivamente. Las lesiones producidas por cada subtipo son indistinguibles. No existen
diferencias en cuanto a la distribución de ambos subtipos, encontrándose ambos a nivel de lesiones genitales y fuera de esta zona, pero
no se hallan MCV tipo II en ningún paciente de edad menor a 15 años. Todos los MC aislados de un mismo paciente pertenecen al
mismo genotipo así como los aislados de los miembros infectados de su familia. Se ha observado la existencia de MC en pacientes
inmunocomprometidos. La mayoría de los pacientes muestran una deficiencia en su función o número absoluto de linfocitos T. Un
grupo heterogéneo de enfermedades se han descrito en asociación al MC: atopia, neoplasias, estados yatrógenos de inmunosupresión,
los pacientes atópicos un incremento en la susceptibilidad a la infección cutánea viral podía estar relacionado con la piel eccematosa
que abriría una puerta de entrada a la colonización vírica. DIAGNOSTICO: El periodo de incubación de la infección es de 14 a 50 días,
aunque hay datos de recién nacidos con lesiones al cabo de 7 días postparto. Las lesiones se inician como neoformaciones que miden
generalmente de 2 a 6 mm, aunque pueden llegar a medir 3 cm, son hemisféricas, cupuliformes, lisas, del color de la piel o perladas,
algunas (20%) tienen una umbilicación central; la base es levemente eritematosa y son de consistencia firme, cuando se rompen. Se
localizan en cualquier parte del cuerpo e incluso pueden afectar mucosas, generalmente se agrupan en un área específica, pero pueden
estar diseminadas en personas infectadas con el virus de la inmunodeficiencia humana adquirida, siendo un marcador de enfermedad
avanzada, el material es blanquecino y cremoso. En el caso de los niños las lesiones se localizan normalmente en cara, tronco, brazos y
piernas a diferencia de los adultos jóvenes en quienes el molusco contagioso se adquiere por transmisión sexual, las lesiones tienen
predilección por genitales, abdomen y cara interna de los muslos. Las lesiones del molusco suelen aparecer entre los 14 días a 6 meses
después de la exposición, hay datos de recién nacidos con lesiones al cabo de 7 días postparto. Se pueden propagar por auto
inoculación, pero es típico que se resuelvan espontáneamente en pocos meses. Las lesiones son asintomáticas en la mayoría de los
pacientes, aunque en el 10% de los casos puede haber prurito y desarrollarse una reacción eccematosa. El diagnóstico se hace
clínicamente y en algunos casos dudosos puede efectuarse biopsia con tinción H-E donde se encuentran los cuerpos de molusco
(inclusiones intracitoplasmáticas grandes) o de Hendersen-Paterson; el 90% de los pacientes posee Ac tipo IgG. Puede realizarse
microscopía electrónica, PCR, Elisa, e inmunohistoquímica. TRATAMIENTO: Enfermedad autolimitada que eventualmente se resolverá
en la mayoría de los pacientes pero en niños atópicos e inmunosuprimidos la evolución es larga y tórpida. Primera elección es
imiquimod 5% crema cada 2 dias hasta que desaparezcan las lesiones (aprox. 4 sem). Curetaje: Es un método sencillo y disponible de
forma rápida. Se realiza con una cureta simple, doloroso por lo que se recomienda el uso de algún anestésico tópico previo a la
realización del mismo. Tiene la ventaja de proveer de una muestra de tejido para la confirmación del diagnóstico en caso de dudas.
Criocirugía: Es un método eficiente y rápido para el tratamiento de las lesiones. El nitrógeno líquido se aplica sobre cada lesión por
pocos segundos. Se repiten las sesiones cada 2 a 3 semanas según sea necesario. Entre las desventajas se encuentran el dolor, hiper o

CURSO ENARM CMN SIGLO XXI TEL: 36246001 Pharmed Solutions Institute PÁGINA 395
MANUAL DE TRABAJO DEL CURSO ENARM CMN SIGLO XXI
hipo pigmentación y cicatriz residual. La podofilina se aplica en tintura al 25% en alcohol una vez a la semana en cada lesión. Los
efectos secundarios descritos son locales como erosión de la piel sana circundante y sistémicos como daño renal, neuropatía, íleo
paralítico, leucopenia y trombocitopenia. El podofilotoxina es uno de los componentes activos de la resina de podofilina, es más seguro
que la podofilina y puede ser usado por el paciente en casa. La aplicación recomendada es 0,05 ml de podofilotoxina al 5% en etanol
tamponado en lactato dos veces al día por 3 días. Está contraindicado durante el embarazo. Se coloca sobre las pápulas una solución
de yodo al 10% y al secar se cubren con ácido salicílico al 50% y cubren con adhesivo. El proceso se repite cada 24 horas hasta que las
pápulas se tornen eritematosas lo cual ocurre entre el 3er día y el 7mo, después de lo cual solo se coloca solución yodada. Se ha
reportado resolución de las lesiones con una media de 26 días. Puede ocurrir maceración y erosión.

CASO CLINICO
Un varón de 10 años de edad, nacido del matrimonio consanguíneo fue llevado con múltiples lesiones elevadas blancas como perlas en
todo el cuerpo desde hace 6 meses, asintomáticas que se fueron generalizando. No había antecedentes de múltiples y recurrentes
lesiones llenas de pus en el pecho, las ingles, junto con infecciones pulmonares recurrentes. No había antecedentes personales o
familiares de atopia. No había antecedentes de ningún dentales, problemas óseos, neurológicos o cualquier facies distintivas. De los
tres hermanos, un hermano más joven tenía antecedentes de lesiones de la piel similar en todo el cuerpo.

PREGUNTA
Cual es la conducta a seguir.

RESPUESTA
a.- Crioterapia y curetaje.
b.- Conducta expectante.
c.- Nitrito acidificado.
d.- Imiquimod tópico.

ESCABIASIS. CIENCIAS BASICAS: También llamada sarcoptosis, sarna. Es una enfermedad altamente transmisible, cosmopolita, cuyo
agente etiológico es el ácaro Sarcoptes scabiei var. hominis, del grupo Arachnida, del orden Astigmata. Los factores más importantes en
la transmisión son la pobreza, hacinamiento y la promiscuidad sexual. Se le asocia a estratos socioeconómicos bajos en los que además
es frecuente observar higiene y nutrición deficientes. SALUD PUBLICA: Se transmite principalmente por contacto directo, estrecho, sin
descartar la evidencia de transmisión por fomites (por ejemplo, ropa de cama o personal en infestaciones severas) y contacto sexual.
Debe tenerse en cuenta el contacto frecuente en algunas instituciones (asilos, escuelas, cuarteles, hospitales). Los perros y gatos y
otros animales también padecen la parasitosis. Los animales de compañía con S. scabiei var canis pueden ser responsables de
infestación y sensibilización, causando una dermatitis papular limitada, aunque generalmente existe especificidad de especie.
PATOGENIA: El artrópodo adulto mide 1 - 3 mm de longitud, tiene un cuerpo aplanado, ovalado y 4 pares de patas. La hembra
fertilizada se instala en la superficie de la piel y en un lapso de tiempo menor a una hora excava un túnel en el estrato córneo y
granuloso. Puede desplazarse unos 2 - 5 cm/min sobre la piel cálida. El promedio de vida del artrópodo es de unos 25 - 30 días, durante
los cuales el túnel excavado puede extenderse unos milímetros o hasta centímetros. El ácaro deposita durante su avance 2- 5
huevos/día y pequeñas pelotitas de excremento, oscuras y ovales, irritantes. La respuesta inflamatoria y los productos del ectoparásito
son responsables, en gran medida, del prurito. Las larvas liberadas alcanzan la madurez unos 15 días después de la oviposición; los
nuevos adultos copulan y el ciclo se repite; al cabo de varias semanas la infestación todavía es baja y los signos y síntomas leves. Se
requieren unos 15 - 20 ácaros adultos y su diseminación para provocar el intenso prurito. DIAGNOSTICO: Los síntomas iniciales son
leves y habitualmente se atribuyen a la picadura de algún insecto. El prurito es un síntoma cardinal, con agravamiento nocturno
(reacción de hipersensibilidad a saliva, huevos, materia fecal de los ácaros). El rascado, que inicialmente alivia al paciente porque
destruye los túneles y algunos parásitos se erige como una forma de diseminación de los mismos (y elimina las lesiones primarias,
constituidas por túneles y vesículas escasas y puntiformes, con ácaros, y pápulas pequeñas). Los túneles son delgados, elevados,
curvados o en forma de S y miden 2 - 20 mm de longitud. Su localización más frecuente se encuentra en pliegues interdigitales,
muñecas, en los bordes de manos y pies, superficies extensoras de codos y rodillas, pene, escroto, pliegues mamarios e incluso
pezones, glúteos, axilas, cintura y pliegues poplíteos (zonas incluidas por las "líneas de Hebra", divisiones imaginarias a nivel de
hombros y rodillas). En niños pequeños también se ubican con frecuencia en palmas de las manos, plantas de los pies, cabeza y cuello,
con vesículas y pústulas. Además pueden presentar nódulos en la zona del pañal y en la axilar. En adultos también es posible identificar
nódulos escabióticos. Las lesiones secundarias dominan el cuadro clínico y son ocasionadas por el rascado, infección bacteriana
secundaria y la automedicación: pústulas, excoriaciones, eccema, costras. Se desarrollan nódulos en codos, axilas, pene, escroto (masas
firmes, oscuras, que persisten durante meses aún en ausencia de infestación activa). Datos de eczema inflamatorio se atribuyen a la
automedicación. Diagnóstico epidemiológico: Cuadros similares en el ambiente familiar o comunal, en instituciones. Localización de
lesiones y morfología de las lesiones. Biopsia de piel. Infiltrado inflamatorio compuesto por eosinófilos, linfocitos, histiocitos. Pueden
llegar a observarse larvas, ninfas, hembras. Se ha reportado el empleo de PCR en pacientes con probable escabiasis y exámenes
dermatológico y microscópico negativos. TRATAMIENTO: Debe ser aplicado a todos los miembros de la familia y contactos cercanos.
Incluye aseo personal y de la ropa. Se sugiere lavar la ropa en agua caliente y guardar en bolsas de plástico aquéllo que no pueda
lavarse durante 2 semanas. Antibióticos por vía sistémica en el tratamiento de impétigo bacteriano secundario y anthistamínicos para
el control del prurito. La dosis única de ivermectina es efectiva. En casos severos se aconsejan hasta tres administraciones, c/1 - 2
semanas. Los agentes tópicos incluyen: Permetrina en crema o emulsión al 5% (Scabisan), benzoato de bencilo en emulsión al 25% para
adultos (Hastilan), crotamitón en crema (Eurax), muy eficaz ante el prurito. El lindano ha sido prohibido en varios países. La FDA reporta
que el 70% de las reacciones adversas por lindano son de origen neurológico, e incluyen ataxia, desorientación, tremores, convulsiones
y muerte. La exposición crónica deriva en efectos sistémicos graves y/o mortales. En pacientes con escabiasis noruega se utiliza
ivermectina y tratamiento tópico simultáneamente, anthistamínicos y emolientes. El tratamiento es difícil, con recaídas. Debe
monitorearse a los pacientes. El prurito puede persistir en las 2 - 4 semanas posteriores, lo cual puede significar: tratamiento exitoso,

CURSO ENARM CMN SIGLO XXI TEL: 36246001 Pharmed Solutions Institute PÁGINA 396
MANUAL DE TRABAJO DEL CURSO ENARM CMN SIGLO XXI
recaída, nueva infección. SARCOPTOSIS COSTROSA O NORUEGA: Se presenta en sujetos con inmunodepresión (SIDA, virus linfotrópico
o leucemia), ancianos institucionalizados, en pacientes con trastornos neurológicos, principalmente síndrome de Down, demencia,
trastornos nutricionales y enfermedades infecciosas. Las lesiones, hiperqueratósicas, son extensas, semejantes a las que se presentan
en la psoriasis, con escaso prurito en la mayor parte de los casos. Hay formación de costras gruesas que involucran manos y pies,
escamas grisáceas y costras en tronco y extremidades, descamación en la zona facial, lesiones semejantes a verrugas en dedos y sitios
de trauma, y caída profusa de cabello. La observación de escamas aclaradas con hidróxido de potasio permite apreciar abundantes
ácaros en todas las fases de crecimiento. Un cuadro que se presenta con cierta frecuencia, es el de "las personas limpias", quienes
presentan lesiones escasas, en pliegues. Un buen número de pacientes presentan títulos elevados de IgE, eosinofilia, activación de
mastocitos y una reacción del tipo de hipersensibilidad inmediata. Ante una reinfestación los síntomas aparecen con mayor rapidez.
COMPLICACIONES: impétigo secundario, hiperinfección. Las infecciones bacterianas secundarias se relacionan con frecuencia con
Streptococcus del grupo A y Sytaphylococcus aureus, por lo que se estima que la ectoparasitosis es un factor de riesgo importante en
zonas endémicas en el desarrollo de glomerulonefritis, fiebre reumática, celulitis y otras infecciones invasivas.

CASO CLINICO
Paciente de sexo masculino, de 15 meses de edad, previamente sano. Consulta a su pediatra de cabecera por presentar pápulas
eritematosas en muñeca izquierda. Algunas de ellas se tornaron costrosas y, en pocos días, se sumaron pápulas, placas y pequeños
nódulos eritematosos con escamas en axila derecha, región supraumbilical y axila izquierda. Se encontraba en buen estado general y
afebril.

PREGUNTA
Cual de las siguientes medidas es conveniente prescribir como final de tratamiento mas adecuado?

RESPUESTA
a.- Cefalexina por vía oral
b.- Permetrina al 5% (crema fluida)
c.- Hidroxicina 2 mg/kg/día 3 veces/día.
d.- Aplicación diaria de mometasona (crema) en los nódulos.

PEDICULOSIS. CIENCIAS BASICAS: Es una ectoparasitosis ocasionada por los piojos, los cuales no reconocen barreras geográficas,
socioeconómicas ni culturales. Es la parasitación del hombre y los animales por insectos del genero pediculus del cual existen 3
principales: el P. capitis (pediculosis de piel cabelluda), el P. vestimenti (pediculosis del vestido y el cuerpo) y el P. pubis o ladilla (en
vello púbico). El piojo se alimenta succionando sangre; su saliva contienen sustancias vasodilatadoras y anticoagulantes. Los huevos del
piojo, conocidos como liendres, están finamente adheridas a pelos individuales. SALUD PUBLICA: Es un problema cosmopolita. Su alta
prevalencia ha sido relacionada, con la limitada disposición de agua y deficientes prácticas de aseo personal. Hay preferencia con el
sexo femenino relacionado con el cabello largo, ya que estos facilitan las formas de transmisión. Aproximadamente 6-12 millones de
casos/año se presentan entre niños de 3-12 años de edad en Estados Unidos. En México se reportó una prevalencia entre 18-33%, en
algunas poblaciones. PEDICULOSIS DE PIEL CABELLUDA: El llamado piojo negro o gris es el causante y parasita sobre toda la piel
cabelluda aunque puede observarse en la barba y el bigote. Es más frecuente en niños entre 5 y 15 años de edad. El piojo en realizad es
gris, pero toma a menudo el color del cabello que parasita y pone sus huevecillos que originan larvas (liendres), las cuales se adhieren
fuertemente al pelo y son difíciles de desprender. El piojo mide unos 3mm y su cuerpo es alargado, se sitúa sobre todo en la región
occipital y sus movimientos originan intenso prurito, lo cual provoca no en pocas ocasiones un impétigo secundario en esta región que
es a menudo el motivo de consulta, por ello ante un impétigo sobre todo en niño, en la región occipital debe pensarse en la posibilidad
de pediculosis previa. Deben diferenciarse las liendres de simples escamas que en general se dejan atravesar por el pelo, mientras que
la liendre está adherida a su costado y avanza con el pelo al crecer este. La hembra dura poco menos de 30 días, pero en ese tiempo es
capaz de depositar entre 50-150 huevecillos. PEDICULOSIS DEL CUERPO Y DEL VESTIDO: En realidad los piojos llamados blancos se
encuentran en la ropa de personas sucias, vagabundos, alcohólicos, que nunca se bañan, ni cambian de ropa y ahí mismo el piojo
deposita sus huevos que pueden observarse junto con los adultos en las costuras de la ropa. Las lesiones que producen son paulas,
costras hemáticas, manchas hipercromicas en abdomen, nalgas, muslos, con intenso prurito. El impétigo y la eczematización son
complicaciones habituales. El piojo blanco transmite el tifo exantemico y otras ricketsias. PEDICULOSIS DEL PUBIS: El P. pubis
vulgarmente llamado ladilla, es más corto y ancho que los anteriores y en sus extremidades presenta unos ganchos que le dan aspecto
de cangrejo y que lo adhieren fuertemente al pelo del pubis, periné, pliegue intergluteo y en ocasiones puede subir hasta la ceja,
pestañas y vello axilar. El parasito es sedentario, se mueve poco, se alimenta continuamente y deposita sus deyecciones en el sitio y en
la ropa interior por lo que el paciente cuando llega habitualmente al consultorio ya tienen el diagnostico, porque ha sentido prurito en
la región del pubis y hasta ha logrado visualizar y extraer el piojo. Al examinar la región se observan los parásitos y las liendres y en la
trusa o pantaleta se puede observar un fino puntillo producido por el depósito de las deyecciones del piojo (signo de la trusa) dato muy
seguro para el diagnóstico. En la piel se pueden ver costras hemáticas y manchas hemorrágicas de color azulado de medio a un cm,
llamadas manchas cerúleas. La hembra dura unos 30 días y llega a depositar en ese tiempo unos 30huevecillos que se transformaran
en adultos. La transmisión es en la mayoría de las veces por transmisión sexual. TRATAMIENTO: Sera necesario destruir los parásitos
adultos sin olvidar las liendres que no son fácilmente desprendibles del pelo, de otro modo en pocos días, estas darán origen a nuevos
adultos. La permetrina al 1% en loción o crema, es el tratamiento de elección para pediculosis, debido a su eficacia y ausencia de
toxicidad, esta actúa como una neurotoxina, bloquea los canales de sodio ocasionando parálisis del sistema nervioso y
musculoesqueletico e impidiendo la respiración del piojo. La piretrina mas el piperonyl es un insecticida de origen vegetal extraído del
crisantemo, el cual presenta un efecto ovicida del 70% y pediculicida del 97%. Se debe repetir 7 días para asegurar su eficacia. Los
adultos pueden destruirse con DDT, benzoato de bencilo al 25%, gamexano o lo más útil y menos irritante: vaselina con xilol que los
ahoga rápidamente. Se aplica tal mezcla en la cabeza o en el pubis y se deja toda la noche con baño al día siguiente. Para destruir las

CURSO ENARM CMN SIGLO XXI TEL: 36246001 Pharmed Solutions Institute PÁGINA 397
MANUAL DE TRABAJO DEL CURSO ENARM CMN SIGLO XXI
liendres es necesario desprenderlas mediante la aplicación con fomentos de ácido acético o simplemente vinagre blanco en agua, que
disuelve la sustancia que las adhiere al pelo. Al día siguiente baño y peinado con el peine de dientes apretados especial para piojos.

CASO CLINICO
Niña de 2 años que tras una extensa pediculosis en la cabeza tratada con 2 ciclos de permetrina al 1% en loción acude por parasitación
en las pestañas. La exploración mediante lámpara de hendidura pone de manifiesto la presencia de algunas liendres y de un piojo
maduro (Pediculus humanus capitis) adheridos a las pestañas. Se realiza retirada directa de gran parte de los parásitos y se inicia
tratamiento con vaselina en pomada 3 veces al día durante una semana. No vuelve a presentar recurrencias tras el tratamiento.

PREGUNTA
Cual es la conducta a seguir.

RESPUESTA
a.- Permentina 1%.
b.- Piretroides.
c.- Permetrina 5 %.
d.- Malathion 0.5 %.

TIÑAS. CIENCIAS BASICAS: Bajo el término dermatofitosis se engloban una serie de dermatosis producidas por un grupo determinado
de hongos que se conocen como dermatofitos y que se caracterizan por parasitar, y digerir, las estructuras córneas (piel, uñas y pelo).
También se conocen como micosis superficiales o tiñas y comprenden seis grupos, en función del área corporal afecta. Generos:
Trichopyton, Microsporum, Epidermophyton. SALUD PUBLICA: T. rubrum 36-52%, M. canis 14-24%, T. tonsurans 15-18%. El cpontagio
es persona a persona, por animales, artículos contaminados a través del suelo y albercas. TIÑA DE LA CABEZA: La más frecuente en
niños. Afecta a los cabellos de cuero cabelludo que se rompen a nivel del ostium folicular lo que determina zonas de alopecia (tiñas
tonsurantes). La afectación puede realizarse implicando a todos los pelos de una zona determinada (tiñas tonsurantes microspóricas) o
a unos pelos sí y otros no (tiñas tonsurantes tricofíticas). Estas formas clínicas afectan predominantemente a niños, curan
habitualmente en la pubertad y no dejan lesiones residuales. La tiña fávica o favus se caracteriza por estar producida por T. schönleini, y
producir una inflamación folicular profunda que desemboca en una alopecia cicatricial. En contra de lo que ocurre en las formas
tonsurantes el favus no cura con la pubertad. Es poco frecuente en países desarrollados pero la emigración determina la aparición de
casos en estas zonas. Existen procesos inflamatorios en forma de foliculitis supurativa, no exclusiva de cuero cabelludo, conocidas como
Querion de Celso, que están producidas por hongos zoofílicos (T. rubrum en 75%) y que presentan exudación abundante a nivel
folicular especialmente cuando se presiona la placa (signo de la espumadera). Dependiendo de la intensidad del componente
inflamatorio podremos encontrar, o no, la existencia de alopecia cicatricial. El tratamiento de primera elección es la terbinafina. Niños
10mg/Kg/dia por 6 semanas. Otra opción es itraconazol VO 5mg/día por 4 semanas. TIÑA DEL CUERPO: Cualquier zona corporal no
incluida en esta clasificación puede incluirse en este grupo. Su clínica es muy evocadora en forma de lesiones papulosas que tienden a
crecer de forma excéntrica determinando “círculos” en los que existe una zona central sana o discretamente escamosa, y un borde
inflamatorio, activo, papuloso o pustuloso. Al igual que en cuero cabelludo pueden existir formas inflamatorias cuando el agente
etiológico corresponde a un hongo de origen animal. Se puede usar terbinafina tópica, cada 24 hrs por 4 semanas. Alkternativa,
miconazol. TIÑA CRURAL: Se caracteriza por la aparición de placas anulares, semejantes a las descritas en la tiña corporal, pero que
tienden a localizarse a nivel inguinal. Es excepcional en niños pero sí suele afectar a jóvenes que practican deporte de forma habitual, y
no es rara su asociación a tiña de los pies. Su posible transmisión por contacto sexual es siempre necesario tenerla en cuenta. TIÑA DE
LAS MANOS: Es un proceso poco frecuente, y excepcional en la edad pediátrica, que casi siempre aparece en mujeres con trastornos
circulatorios, diabetes o en profesiones que maceran sus manos bien por el uso continuado de guantes o por precisar mantenerlas en
agua. La forma clínica más frecuente es la aparición de descamación y maceración interdigital, no obstante pueden aparecer cuadros de
descamación palmar. Sin duda alguna las levaduras son más frecuentes en las micosis de manos que los dermatofitos. Tratamiento con
terbinafina tópica. 2 semanas. TIÑA DE LOS PIES: Rara en la etapa infantil pero frecuente en la pubertad, y en ocasiones asociada a tiña
crural. Es debida a la parasitación por dermatofitos en el pie pudiendo adoptar morfologías variables. Forma interdigital (pliegues de
4to y 5to dedo): incluida dentro del cuadro genérico de “pie de atleta” se caracteriza por la aparición de descamación y fisuración a
nivel de los espacios interdigitales. El proceso puede ser poco llamativo cuando la infección es exclusiva de dermatofitos (Tinea pedis
simplex) o ser muy inflamatoria o exudativa cuando existe infección combinada de dermatofitos y bacterias (Tinea pedís complex).
Examen microscópico directo con hidróxido de potasio 20%; observamos filamento slargos delgados o gruesos, artroconidias,
blastoconidias, onicomicosis candida. Tratamiento con ternbinafina tópica por 2 semanas. TIÑA DE LAS UÑAS: Es un proceso habitual
en las consultas de dermatología pediátrica. El patrón diagnóstico es la presencia de una hiperqueratosis subungueal que determina un
levantamiento de la uña del lecho (onicolisis). Generalmente la lesión comienza en el pliegue subungueal distal (onicomicosis
subungueal distal), aunque puede hacerlo en el proximal (onicomicosis subungueal proximal) o incluso acabar afectando a toda la uña
(onicomicosis distrófica total). En contadas ocasiones se produce una parasitación exclusivamente de la lámina ungueal que presenta
una coloración blanquecina (Onicomicosis blanca superficial).

CASO CLINICO
Niña de 7 años, que consultó de urgencias por aparición progresiva en los últimos 10 días de una placa anular eritemato-descamativa
muy pruriginosa, de 3 x 3cm de diámetro y crecimiento centrífugo, con borde levemente sobreelevado. Clínicamente planteaba el
diagnóstico diferencial entre una tinea corporis y un eczema numular. Para confirmar el diagnóstico se realizó un examen directo de
escamas cutáneas, obtenidas después de raspar el borde de la lesión con un bisturí e incubadas con 0,5ml de KOH al 30% sobre una
lámina portaobjetos.

PREGUNTA

CURSO ENARM CMN SIGLO XXI TEL: 36246001 Pharmed Solutions Institute PÁGINA 398
MANUAL DE TRABAJO DEL CURSO ENARM CMN SIGLO XXI
Cuales son los factores de riesgo más importante para sospechar de este padecimiento.

RESPUESTA
a.- Deportes y recreación en equipo.
b.- Exposición a lugares húmedos sin proteción
c.- Compartir objetos personales.
d.- Uso de ropa sintetica.

PREGUNTA
Cual de los siguientes factores de riesgo es el más frecuente que están relacionados con las tiñas.

RESPUESTA
a.- Uso de corticoides.
b.- Inmunocompromiso.
c.- Queratodermia.
d.- Ictiositocis hereditaria.

PREGUNTA
Cual es la conducta terapéutica más adecuada.

RESPUESTA
a.- Terbinafina 250 mg por 12 semanas.
b.- Itraconazol 10 mg/Kg/dia
c.- Fluconazol 150 mg VO por 24 semanas.
d.- Miconazol crema al 2 %

PREGUNTA
Al revalorar la paciente posteriormente la paciente continua con el cuadro clínico con complicaciones, cual de las siguientes no es
indicativo de envio a segundo nivel.

RESPUESTA
a.- Diseminacion dentro de las 4 primeras semanas.
b.- Presencia de onicomicosis comorbida.
c.- Alteración de enzimas hepáticas.
d.- Inmunosupresion de origen idiopático.

PTIRIASIS VERSICOLOR. CIENCIAS BASICAS: Es una infección micótica del estrato córneo de la piel, es la micosis más superficial que se
conoce, causada por Malassezia furfur (ptiriosporum furfur), caracterizada por lesiones discrómicas, que pueden manifestarse como
manchas hipercrómicas o hipocrómicas irregulares y en ciertos casos, de manera vitiligoide, variedad conocida como acromiante, todas
con descamación fina. Además del clima, se han señalado otros factores predisponentes como aplicación local de corticoesteriodes,
desnutrición, recambio lento de la epidermis y predisposición genética. SALUD PÚBLICA: La infección se presenta a partir de la
adolescencia y es raro encontrarla en personas de edad avanzada. Esta micosis es más frecuente en personas que habitan lugares de
clima cálido y húmedo, tales como las regiones costeras tropicales, en donde la frecuencia puede llegar a ser hasta del 50%. No existe
predominio de género. PATOGENIA: El agente etiológico es M. globosa que puede encontrarse como especie única o asociada con a
otras especies, entre ellas M. sympodialis, M. slooffiae. DIAGNOSTICO: Las lesiones generalmente son asintomáticas, se inician como
discretas manchas eritematosas cubiertas con una escama muy delgada (como salvado), que pronto toman dos aspectos
hipocromiantes o hipercromiantes. En México predomina la primera, son manchas un poco más claras que la piel raras veces
acromicas, lenticulraes confluentes, bien delimitadas de bordes no activos como deshilachados, con esa fina escama y prácticamente
sin prurito. Se localiza con mayor frecuencia en el tronco, cuello, cara, piel cabelluda y los brazos, aunque se pueden observar en otras
regiones corporales de acuerdo a los factores predisponentes de cada paciente. Técnicas para la toma de muestra: raspado de la lesión,
de preferencia de la periferia. Se pueden utilizar dos portaobjetos estériles, uno para raspar y el otro para recibir las escamas o bien se
puede emplear una caja de Petri estéril. Producto biológico: escamas. Observación microscópica: hidróxido de potasio del 15-30%, azul
de algodón, tinta o tinta azul de marca Parker. En escamas de lesiones de pitiriasis versicolor la morfología de las estructuras
parasitarias es diagnóstica de esta enfermedad. Las levaduras son esféricas de 2-8 µm de diámetro, agrupadas, asociadas con hifas de
10-25 µm de largo y 2-5 µm de ancho, las hifas pueden estar alineadas o ramificadas (conocida imagen de spagueti con albóndigas). El
cultivo no es un procedimiento que se practique rutinariamente en el laboratorio clínico, sin embargo se pueden obtener los cultivos a
partir del producto biológico. Cuando se toma una porción de la colonia, es difícil preparar una suspensión o estriar sobre el agar, ya
que las células permanecen juntas formando pequeños grupos (o racimos). TRATAMIENTO: Ya que la mayoría de patologías causadas o
asociadas a Malassezia son superficiales, el tratamiento tópico es lo más recomendado. Habitualmente se recomiendan lociones,
cremas o jabones con ácido acetil salicílico al 5% o azufre al 1-3%; ungüento de Whitfield, hiposulfito de sodio al 20% en solución
acuosa, tolnaftato, tolciclato, crema de piroxolamina al 1%; champú de disulfuro de selenio al 2.5%, con efecto antimicótico,
bactericida y anfi-inflamatorio. Las lociones y cremas deben aplicarse diariamente durante 3 a 4 semanas. El champú se aplica
diariamente, dejando actuar algunos minutos antes de enjuagar el pelo, durante 2 a 3 semanas. Para casos graves, está indicado el
ketoconazol por vía oral; uno de los esquemas es 400 mg en una sola dosis; o bien 200 mg diarios por 10 a 30 días dependiendo de la
gravedad del caso. El itraconazol también ha dado resultados satisfactorios: 100 a 200 mg/día durante 3 a 15 días, dependiendo de la

CURSO ENARM CMN SIGLO XXI TEL: 36246001 Pharmed Solutions Institute PÁGINA 399
MANUAL DE TRABAJO DEL CURSO ENARM CMN SIGLO XXI
extensión de las lesiones. En el caso de la pitiriasis versicolor, las manchas pueden persistir varios meses después del tratamiento, por
lo que el paciente debe ser informado.

CASO CLINICO
Se trata de paciente masculino de 18 meses de edad el cual ha presentado diversos cuadros de dermatitis aatopica con importante
rascada, acude a consulta de control donde observa placas blanquecinas peribucales sin comprometer mucosa.
PREGUNTA
Considerando su diagnostico y estadio del padecimiento cual es conducta terapéutica menos adecuada?

RESPUESTA
a.- Evitar sol e indica protectores solares.
b.- Baños cortos y con agua templada.
c.- Indica corticoide en crema.
d.- Indica tacrolimus.

CANDIDIASIS. CIENCIAS BASICAS: La candidosis o candidiasis es una micosis causada por diversas especies de levaduras del género
Candida. Cualquier tejido puede ser afectado por lo que se presentan diversos cuadros clínicos, cada uno de ellos asociado
directamente al estado inmunológico del paciente. Las candidosis de mucosas y piel son las más frecuentes, mientras que las sistémicas
son de evolución aguda o crónica y generalmente severas. SALUD PUBLICA: La distribución geográfica de esta micosis es universal y
más de 70 % de ellas son producidas por C. albicans observándose un porcentaje mayor por el serotipo B. Los casos de candidiasis
sistémica están relacionados a pacientes con severas deficiencias en su sistema inmune. C. krusei y C. glabrata son habitualmente
resistentes a los compuestos azólicos y su hallazgo como agentes infecciosos involucrados en enfermedades sistémicas
intrahospitalarias ha aumentado en los últimos años. PATOGENIA: La C. albicans se ha considerado el ejemplo más clásico de lo que es
un parasito oportunista, vive habitualmente en forma saprofita en las mucosas orales, nasal, vaginal, tracto gastroinetstinal, en cambio
no es habitual encontrarla en piel. El niño al nacer puede recibir este parasito al pasar por el tracto vaginal de la madre. Cuando se
convierte de saprofita a patógena, puede producir desde una simple algondoncillo hasta graves endocarditis o septicemia. Factores
predisponentes: 1. Fisiológicos que suelen transformar el pH de las mucosas como la infancia y el embarazo. 2. Maceración, humedad y
traumatismos, (pliegues interdigitales, submamarios, uñas y rebordes ungueales). 3. Dermatosis inflamatorias previas (como dermatitis
del pañal que se complica con candida). 4. Mal estado de la dentadura y prótesis. 5. Enfermedades metabólicas (DM y obesidad). 6.
Enfermedades inmunosupresoras (leucemias, Enf. De Hodking, SIDA). 7. Medicamentos que alteran la flora bacteriana (corticoides,
citotóxica). DIAGNOSTICO: La manifestación más conocida es el algodoncillo que aparece en la boca de RN por su bajo pH. Las lesiones
son placas cremosas, blanquecinas, como residuo de leche, que pueden afectar la mucosa de los carrillos, la lengua, el paladar, encías e
incluso invadir toda la boca hasta la tráquea o salirse de la boca y producir fisuras cubiertas de material blanquecino en las comisuras
labiales. Estas lesiones son dolorosas e impiden la alimentación al niño. La Candida albicans no es habitual en la piel, pero puede
producir enfermedad cuando aumenta la susceptibilidad del in dividuo. Las lesiones a parecen a nivel de los pliegues: interdigitales en
manos y pies, inguinales y submamarios, axilas, intergluteos y periné. Se trata de fisuras y erosiones eritematosas, maceración, vesícula
y pústulas y algunas costras y escamas. En los pies semejan lesiones de tiña, frecuentemente producen mal olor y son pruriginosas. En
los niños pequeños contribuyen a formar la llamada dermatitis por pañal que se observa sobre todo por la orina, la aplicación de
pomadas, la maceración, todo lo cual favorece el desarrollo de Candida. El niño presenta entonces en regiones inguinales, glúteas y
genitales, extensas zonas eritematosas con vesículas y pústulas, costras y escamas con intenso ardor y prurito. La perionixis y la
afección de la uña se ha observado sobre todo en personas que mantienen mucho tiempo las manos dentro del agua, como
empacadores de frutas, pescados y mariscos. El reborde ungueal se observa inflamado, eritematoso, desprendido de la uña y esta se
afecta principiando de la matriz al borde libre hacia adentro. Poco aparecen estrías en las uñas, la cual se vuelve amarillenta y opaca y
se empieza a despulir. La lesiones son habitualmente superficiales, es poco frecuente que al igual que los dermatofitos se introduzcan
más allá de la capa cornea, dependiendo del estado inmunológico del huésped originándose granulomas tricofíticos o candidósicos. El
simple hallazgo de Candida albicans no significa de ninguna manera su intervención patógena, es necesario encontrarla en grandes
cantidades y correlacionar su hallazgo con lesiones clínicas. . Es fácil buscarla de manera directa, colocando material recolectado en un
portaobjetos con solución de potasa al 20% y se podrá encontrara las clásicas formas levaduriformes, que a veces producen
pseudofulamentos. El cultivo en medio Saboraud produce colonias típicas de aspecto cremoso. TRATAMIENTO: Corregir enfermedad
de base. En boca usamos buches con agua de bicarbonato de sodio. Localmente puede usarse violeta de genciana al 1% que mancha
mucho o la nistatina, antibiótico que solo es activo para levaduras de Candida. Los imidazoles tanto tópicos como sistémicos son
altamente efectivos en candidiosis: miconazol, ketoconazol, clotrimazol.

CASO CLINICO
Paciente femenino de 10 años de edad en consulta por presentar engrosamiento y cambio de coloración en la uña del pulgar derecho,
al examen físico, en cuero cabelludo, se observan tras placas de alopécicas, con cicatrización en la periferia y escasas costras
serosanguinolentas en el centro, dos de estas placas se localizan alrededor del apice del cráneo y la tercera en región parietal derecha.
En cavidad oral y labios se observan placas blanquecinas que cubren el centro y los bordes de la lengua, asi como ambas comisuras
labiales; al remover estas placas se observa eritema y fisuras de mucosas.

PREGUNTA
Cual es la conducta a seguir mas adecuada?

RESPUESTA
a.- Itraconazol.
b.- Miconazol.

CURSO ENARM CMN SIGLO XXI TEL: 36246001 Pharmed Solutions Institute PÁGINA 400
MANUAL DE TRABAJO DEL CURSO ENARM CMN SIGLO XXI
c.- Ketoconazol.
d.- Clotrimazol.

MICETOMA. CIENCIAS BASICAS: Es una infección crónica de la piel y de los tejidos subyacentes con tendencia a afectar los huesos. Se
caracteriza por un aumento de volumen relativamente indoloro y fístulas a través de las cuales se elimina pus y granos constituidos por
filamentos. Los agentes causales son de origen exógeno y pueden ser hongos (eumicetoma) o actinomicetales (actinomicetoma).
Agentes etiológicos: a) Bacterias (ACTINOMICETALES); Nocardia brasiliensis 86.0 %. Actinomadura madurae 10.2 %. Streptomyces
somaliensis 1.3%. N. asteroides, A. pelletieri, N. otitidiscaviarus. b) Hongos (EUMICETOMA). Los hongos dematiaceos forman granos
negros visibles a simple vista, mientras que los de filamento hialino forman granos blancos o blanco-amarillentos. Hongos hialinos:
Acremonium falciforme, A. recifei, A. kiliense, Aspergillus flavus. Hongos dematiaceos: Curvularia geniculata, C. lunata, Exophiala
jeanselmei, Leptosphaeria senegalensis. PATOGENIA: El micetoma se adquiere por inoculación traumática de agentes etiológicos a
través de la piel. En México existe gran número de cactáceas, las espinas son un mecanismo de infección, que probablemente tengan
importancia en la transmisión de la enfermedad. Una vez que la bacteria o el hongo se localizan en los tejidos y sobrevive, se presenta
una inflamación aguda a nivel local con llegada de PMN y activación de macrófagos, aunque la mayoría de bacterias son fagocitadas y
destruidas, algunas logran sobrevivir hasta 16 días y pueden reproducirse. Los agentes no destruidos proliferan y forman colonias en el
interior del tejido conocidas como "granos"; el desprendimiento de los pseudofilamentos de la periferia del grano ocasiona una
inflamación continua con formación de nuevas estructuras parasitarias y colecciones de pus, que al fusionarse forman los trayectos
fistulosos generalmente limitados por tejido fibroso; cuando las fístulas alcanzan la superficie, forman un nódulo que posteriormente se
reblandece y abre liberando pus conteniendo colonias parasitarias del agente. El tejido conjuntivo formado alrededor de los
microabscesos y de las fístulas produce una retracción que ocasiona el aspecto deprimido de las fístulas antiguas. La presencia de micro
abscesos, fístulas, inflamación, edema y fibrosis provoca externamente la deformidad y dureza del área afectada e internamente
obstrucción de la circulación por compresión mecánica y por arteritis. Estos cambios histológicos influyen en la pobre respuesta
terapéutica en los pacientes con evolución prolongada. DIAGNOSTICO: Por la naturaleza saprofita de los agentes causales de micetoma
que posibilita la infección principalmente por traumatismo en pie o pierna, todas las casuística de frecuencia muestran un franco
predominio de afección a extremidades inferiores, pero de acuerdo con las costumbres de trabajo o de vestido y a las condiciones
socioeconómicas, los porcentajes de frecuencia presentados en cada área topográfica por diferentes autores, pueden tener alguna
variación. Examen directo; La observación del material purulento que drena a través de las fístulas permite identificar los granos
macroscópicos en los eumicetomas y, al examen microscópico, en la mayoría de los actinomicetomas. Cultivo; Cuando es posible, los
granos de actinomicetales se mezclan y se lavan con solución salina isotónica (SSI), se centrifugan a 3000 rpm y posteriormente el
sedimento se siembra, el desarrollo de la mayoría de los agentes se presenta después de dos o tres semanas de incubación a 25°C. Otro
medio adecuado para su desarrollo es el de Lowenstein-Jensen que se incuba en las mismas condiciones. Las colonias tienen bordes
bien definidos e irregulares, aspecto céreo o membranoso, generalmente de superficie plegada. Los cultivos de Nocardia spp. Son de
color blanco amarillento o anaranjado las colonias de A. madurae son blanco amarillentas, A. pelletieri forma colonias rojas y S.
somaliensis tienen color oscuro, algunas casi negras. La identificación precisa de los agentes se realiza por medio de pruebas
bioquímicas. Los granos de eumicetoma se siembran y se incuban a 25°C. La velocidad de crecimiento es variable y depende de cada
agente. Así. Fusarium spp, Aspergillus spp. o Scedosporium apiospermum se desarrollan en menos de ocho días; mientras que
Madurella spp, Leptosphaeria sp y Pyrenochaeta romeroi son de crecimiento lento, es decir más de tres semanas. Cuando las biopsias
se toman de sitios con gran actividad parasitaria, los granos de todos los agentes se visualizan con relativa facilidad con la tinción de
hematoxilina eosina (H-E). Los estudios radiológicos no permiten establecer el diagnóstico preciso de esta patología, sin embargo son
de suma importancia para determinar el grado de afección ósea, permiten evaluar la respuesta terapéutica y emitir un pronóstico. Los
cambios observables en estudios radiológicos simples pueden ser en los casos iniciales únicamente periostitis, o bien, destrucciones
osteolíticas extensas con formación de geodas, destrucción de superficies articulares y lisis en huesos del pie que ocasionan el aspecto
de 'caries', datos conocidos desde las primeras descripciones del padecimiento y que se presentan en pacientes con varios años de
evolución. Otro estudio radiológico necesario, principalmente en los pacientes con más de cinco años de evolución del micetoma, es la
arteriografía del área afectada. El aumento de volumen y la fibrosis, condicionan que algunas zonas del área afectada tengan un aporte
sanguíneo deficiente, de tal manera que este procedimiento es un valioso auxiliar para el pronóstico. TRATAMIENTO: Debido a la
posibilidad de que los micetomas sean causados por hongos o por bacterias, antes de prescribir la terapéutica es indispensable conocer
el tipo de agente causal (micótico o bacteriano); esto puede lograrse mediante a la observación de las características de los granos en el
examen directo y en el estudio histológico. Actinomicetoma; La primera elección es la combinación de TMP/SMX 800 mg/160 mg c/12
hrs asociada a la administración de 100 mg de diaminodifenilsulfona (DDS), cada 24 horas. La mayoría de micetomas causados por
actinomicetales responden a este tratamiento y la mejoría es evidente después de dos meses de tratamiento. Los pacientes deben ser
valorados periódicamente debido a que estos medicamentos pueden ocasionar diversos efectos adversos, entre los que se encuentran
reacciones medicamentosas graves a nivel cutáneo como es el síndrome de Stevens-Johnson, necrólisis epidérmica tóxica, aplasia
medular, anemia, intolerancia gástrica, alteraciones hepáticas, etc. Como segunda elección se emplea la asociación de TMP/SMX a la
dosis mencionada previamente en forma continua, y agregando amikacina, aminoglucósido que se administra por vía intramuscular en
ciclos de 21 días de tratamiento a dosis de 7.5 mg/kg de peso cada 12 horas. Entonces, se suspende el aminoglucósido durante ocho
días. Considerar ahora el posible daño renal o auditivo causado por la amikacina; por lo tanto, antes de iniciar el tratamiento y después
de cada ciclo de amikacina, se debe valorar la función de esos órganos. La curación generalmente se alcanza con tres ciclos del
aminoglucósido, pero el TMP/SMX debe continuarse durante 12 a 24 meses de acuerdo con la evolución clínica. Aunque el estudio de
sensibilidad antibacteriana debería ser una práctica habitual en todas las cepas aisladas de casos de actinomicetoma, estos estudios
generalmente no se realizan, dando como resultado que se utilicen tratamientos basados únicamente en la experiencia clínica. Estos
esquemas de tratamiento (cuasiempíricos) generalmente mejoran el estado del paciente, pero en caso de no haber respuesta, retrasan
la curación. Entre los antibióticos utilizados en estas condiciones tenemos: tetraciclina, amoxicilina, estreptomicina o los antifímicos
como la isonizida. Desde los primeros estudios inmunológicos relacionados con actinomicetoma fue evidente que algunos pacientes
presentan inmunosupresión inespecífica severa, y no responden al tratamiento médico aun cuando se les administren los antibióticos
adecuados. En estos casos es necesario valorar la respuesta inmunológica y dar los antibióticos efectivos basados en un estudio de

CURSO ENARM CMN SIGLO XXI TEL: 36246001 Pharmed Solutions Institute PÁGINA 401
MANUAL DE TRABAJO DEL CURSO ENARM CMN SIGLO XXI
sensibilidad acompañados de un esquema terapéutico inmunomodulador, como puede ser la administración de antígeno bacteriano y
levamisol con lo que se logró la curación en sólo dos meses de un paciente con actinomicetoma causado por N. brasiliensis localizado
en el abdomen, después de siete años de evolución sin respuesta al tratamiento tradicional. Se ha reportado la utilidad de la
oxigenoterapia hiperbárica en el tratamiento de un caso de micetoma sin afección ósea, causado por N. brasiliensis; el paciente
presentó mejoría a los cinco días de tratamiento y las fístulas cicatrizaron un mes después del inicio de la terapia. Eumicetoma: Este
tipo de micetoma representa un grave problema de tratamiento, ya que no existen drogas altamente específicas. Durante muchos años
la medida terapéutica empleada con mayor frecuencia fue la amputación de la extremidad afectada. Posteriormente se hicieron
ensayos con griseofulvina a dosis de 500 a 1000 mg por día sin obtener curación; con base en estudios de sensibilidad in vitro la
anfotericina B fue utilizada en el tratamiento de micetomas causados por M. grisea y M. mycetomatis. En opinión de algunos
investigadores, la fibrosis característica que rodea los granos de los agentes etiológicos, impide que se alcancen las concentraciones
adecuadas para la curación y, en consecuencia, el tratamiento no es efectivo. El advenimiento de los compuestos azólicos proporcionó
nuevas alternativas para el manejo de esta patología, se hicieron pruebas con ketoconazol solo o asociado a cirugía; esta última
alternativa mejoró el pronóstico de manera substancial. En la última década, se ha utilizado el itraconazol a dosis de 300 a 400 mg por
día, reportándose algunos casos de curación y otros con mejoría. Algunos autores como Smith y Kutbi, mencionan que en casos de
eumicetoma de corta evolución la cirugía acompañada de tratamiento antimicótico es la única posibilidad de curación definitiva.

CASO CLINICO
Masculino de 8 años, originario de zona rural de guerrero, fue llevado a consulta por una dermatosis localizada en el tronco que se
extendia al toras y a la cara anterior del hombro derecho. La dermatosis era de aspecto monomorfo y estaba constituida por algunas
fistulas, numerosos nódulos eritematoso de 0.3 a 2 cm aproximadamente, telangiectasias y costras sanguíneas y meliceras que
formaban una placa mal circunscrita, de 6 x 4 aproximadamente, crónica y asintomática. Se realizaron cultivo los cuales mostraron
colonias de aspecto yesoso, rogosas y de color blanco. La prueba de la hidrolisis de caseína resulto positiva y la colonia se identifico
como Norcardia brasiliensis. No se observo compromiso oseo a los rayos x.

PREGUNTA
Cual es el manejo mas adecuado para el caso?

RESPUESTA
a.- Anfotericina.
b.- Trimetroprim-sulfametoxazol.
c.- Itraconazol.
d.- Clotrimazol.

URTICARIA. CIENCIAS BASICAS: Es una reacción vascular de la piel, caracterizada por la presencia de habones y prurito intenso donde
ocurren procesos inflamatorios importantes, hay una alteración del equilibrio de sustancias vasoactivas, causando vasodiltaacion y
edema a nivel de la piel (urticaria), o del tejido subcutáneo (angioedema). En la mayoría de los pacientes, una liberación masiva de
histamina es la causa del desequilibrio. El nombre de urticaria
se deriva de la palabra ortiga, una planta cuyas hojas causan
lesiones urticariales pasajeras. SALUD PUBLICA: La prevalencia
de la urticaria-angioedema es de 10-20% de la población lo
sufrirá en su vida. En la edad pediátrica, la urticaria aguda es la
forma que más prevalece. CLASIFICACION: Urticaria Aguda <
de 6 semanas de evolución. Urticaria Crónica > de 6 semanas
de evolución. Urticaria Aguda Recurrente: Episodios repetidos
de brotes agudos con períodos asintomáticos prolongados
entre ellos. PATOGENIA: Alimentos: Huevo, leche, nueces
maní, mariscos, pescados, fresas, kiwi, otros. Drogas:
Penicilinas, ASA, AINES, vacunas, medios de contraste,
productos sanguíneos. Insectos: Abejas, avispas, hormigas.
Infecciones: Parásitos, bacterias, virus, hongos. Estímulos
Físicos: Frio, calor, presión, ejercicios, agua, sol. Aeroalergenos,
alérgenos de contacto: Polvo casero, pólen, esporas, epitelios
de mamíferos. Enfermedades sistémicas LES, ARJ, enfermedad
del suero, vasculitis, cáncer, endocrinopatías, Enf celíaca. Otros
A. Hereditario, amiloidosis, deficiencia de C3b, estrés,
idiopático. El evento central en la urticaria-angioedema es la
liberación de histamina desde unos gránulos preformados en
las células cebadas. Existen múltiples factores que pueden provocar la degranulacion de las células cebadas. En la urticaria aguda, el
mecanismo principal es el de hipersensibilidad tipo I, IgE mediada, también conocida como la reacción alérgica clásica. En las primeras
exposiciones que el paciente tuvo alérgeno se inició una respuesta Th2 mediada que resulto en la formación de abundantes cantidades
de IgE específica por parte de las células plasmáticas específicas. Esta IgE es liberada a la circulación sistémica y se fija de preferencia a
receptores de alta afinidad para IgE (FcR1), en la superficie de la célula cebada (CC). Un subsecuente contacto con el mismo alérgeno
provoca directamente el puenteo de dos IgE especificas vecinas en la membrana de la CC, y esto es la señal para la degranulación de la
CC. Así se liberan grandes cantidades de sustancias vasoactivas, especialmente histamina, pero también leucotrienos, factor derivado
de plaquetas. Las respuestas principales de estos mediadores son: prurito, vasodilatación (eritema), aumento en la permeabilidad
vascular, reflejo axonal. La exposición al alérgeno de la urticaria puede ser por cualquier vía, incluyendo la oral, transdermica e

CURSO ENARM CMN SIGLO XXI TEL: 36246001 Pharmed Solutions Institute PÁGINA 402
MANUAL DE TRABAJO DEL CURSO ENARM CMN SIGLO XXI
inhalatoria. URTICARIA AGUDA: En niños la urticaria aguda es más frecuente que la crónica. Es frecuentemente por una reacción
alérgica mediada por IgE a infecciones agudas generalmente del TRS y drogas. Infecciones. Virus: Enterovirus, Parainfluenza 1,2,3, Virus
respirat. Sincitial, Adenovirus, rinovirus, Influenza A y B, VEB, CMV, Herpes simple, Parvovirus B19, Hepatitis A, B, C, Mycoplasma
Neumoniae. Parásitos: Oxiuro, Toxocara, Giardia lamblia. Bacterias: Estreptococo beta hemolítico, H. pylori. URTICARIA CRONICA: En la
Urticaria crónica los factores físicos como presión y frío son la causa principal. Luego siguen infecciones: tracto urinario, tracto
respiratorio y las idiopáticas. Existe un porcentaje de Ac antitiroideos antiperoxidasa positivos en niños con urticaria crónica y también
hay mayor incidencia de enfermedad celíaca en estos niños. DIAGNOSTICO: La urticaria se identifica por lesiones típicas que tienen 3
caracteristicas básicas: consisten en una inflamación central eritematosa, levemente elevada sobre el nivel de la piel, casi siempre
rodeada por una piel eritematosa (reflejo axonal). Las lesiones son en esencia evanescentes, con una duración de menos de 24 hrts,
que al desaparecer dejan una piel intactas. Con frecuencia son intensamente pruriginosas. La urticaria puede tomar diferentes formas
variando de lesiones puntiformes de 2 milimetros (point lesions), que cubren grandes áreas de la piel (típico aspecto de urticaria
colinérgica), lesiones aisladas de 1-2 cm o hasta lesiones lesiones confluyentes de varios centímetros. Historia y examen físico. Pruebas
para urticarias físicas, prueba de suero autólogo, examen simple de orina y seriado de heces, hematología completa y VSG, pruebas
tiroideas y Ac antitiroideos. Otros: según cuadro clínico. TRATAMIENTO: 1. Medidas generales como evitar contacto con alérgeno,
eliminar el alérgeno si es posible: aditivos, alimentos, medicamentos, frío, calor, picaduras. Implementar dieta de eliminación. 2.
Antihistamínicos: Primera línea en el tratamiento de la urticaria. Actúan por inhibición competitiva de la histamina bloquean
receptores H1. Control del prurito. Primera generación sedantes (Dexclorfeniramina 0.15-0.2 mg/día QID, Hidroxicina 2 mg/día QID,
Ciproheptadina 0.25 mg/día BID, Difenidramina 5 mg/dia TID) y Segunda generación No sedantes (Loratadina: 1-2 años 1cc/dia; 2-5
años 5mg/dia, Cetirizina : 6m-1año 0,5 mg/dia; 2-5años 2.5 mg/dia, Fexofenadina: 6-12 años 120 mg/día, Ebastina : 6-12-años 5-10
mg/dia, Desloratadina: 2-5 años 1,25 mg/dia; 6-11 años 2,5mg, Levocetirizina: >2 años 5/mg dia). Los de segunda generación provocan
liberación de aminas vasoactivas. Desloratadina: Tratamiento de rápida acción y bien tolerado y por largo tiempo. Usado en casos de
Urticaria crónica idiopática. Levocetirizina: Enantiómetro o metabolito activo de certirizina. Receptor Antagonista H1. Rápido inicio de
acción, pocos efectos colaterales. Indicado en rinitis alérgica y en UCI. Efecto antiinflamatorio importante que aumenta su efecto
terapéutico en enfermedades alérgicas. Uso por tiempo prolongado desde el año de edad. Dosis: 5mg/dia. 3. Antileucotrienos: Segunda
línea en urticaria que no responde a antihistamínicos. Previene o mejora los síntomas asociados a procesos mediados por leucotrienos
C4, D4, y E4. Util en urticaria por medicamentos ASA y AINES. Montelukast 2-5 años 4 mg/dia; 6-14 años 5 mg/dia. Zafinlukast 5-11 años
10 mg/BID. 4. Corticoides: Esteroides Sistémicos ayudan a disminuir la inflamación. Solo deben usarse en casos extremos y de
angioedema, por corto tiempo para evitar efectos colaterales. Urticaria por presión y urticaria vasculitis. Prednisona 1mg/Kg/dia por 5-
7 dias. 5. Ciclosporina: En la tercera línea de tratamiento. Es un inmunosupresor. Controles de funcionalismo renal. Dosis: 3 mg/Kg de
peso. 6. Omalizumab. Urticaria Aguda con síntomas sistémicos: Dificultad respiratoria-Evidencia angioedema, Adrenalina 1/1000
subcutánea a 0.01 mg/Kg, repetir cada 15 a 20 minutos, hidrocortisona 7-10 mg/Kg/dósis y luego instaurar ciclo corto, menor a 10 días,
clorfeniramina: IV. Dieta de exclusión de alimentos disparadores (maní, piña, chocolate). Combinación de anti-H1 (1ra y 2da) o
Combinación de anti-H1 y H2 (o anti-LT).

CASO CLINICO
Paciente de 4 meses de edad, sin antecedentes patológicos conocidos, traido a consulta por presentar papulas generalizadas
pririginosas de 2 meses de evolución, al EF mostraba papulas eritematosas generalizadas en torax y miembros superiores e inferiores
que respetaban palmas y plantas. Dichas papulas, al ser frotadas, tomaban aspecto de roncha rodeada por un halo eritematoso. Según
refirió su madre sobre una de ellas se había formado previamente una ampolla, el paciente se encontraba afebril y en buen estado
general.

PREGUNTA
Cual es la conducta terapéutica mas adecuada?

RESPUESTA
a.- Clorhidrato de hidroxicina.
b.- Maleato de clorfeniramina.
c.- Loratadina.
d.- Cetirizina.

PRURIGOS. CIENCIAS BASICAS: El prurigo infantil o también llamado urticaria papular es una reacción alérgica producida por la
picadura de insectos. Esta afección netamente pediátrica no debe ser confundida con una alergia a los alimentos; se caracteriza por el
prurito intenso de difícil control, con brotes frecuentes y sucesivos que por lo general se autolimitan en el tiempo. SALUD PUBLICA:
Afecta en su mayoría a niños entre los dos y diez años, frecuentemente en los meses de primavera y verano. TIPOS: La palabra prurigo
define un proceso patológico cuya lesión elemental es la pápula y el síntoma principal es el prurito. El espectro clínico abarca un rango
que va desde las pápulas (prurigo papular), nódulos (prurigo nodular) entre otros tipos de prurigo podemos mencionar al actínico, el
atópico o de Besnier, el de Sutton y el pigmentoso. DIAGNOSTICO: Se observa una erupción estacional y recurrente, con presencia de
grupo de pápulas y vesículas pruriginosas de localización frecuente en antebrazos, abdomen, área lumbar, glúteos, muslos y piernas. La
respuesta va a depender del tipo de paciente y la calidad del inóculo, pudiendo también observarse formas ampollares. PRURIGO
SIMPLE POR INSECTOS O URTICARIA PAPULOSA: Es causada como su nombre lo indica por la picadura de algún insecto. No está
relacionada por factores dietéticos. La naturaleza alérgica de la condición fue demostrada cuando los cambios histológicos producidos
por picaduras de insecto a nivel experimental demostraron que eran idénticos a los de la urticaria papular. Afecta principalmente a
niños de 1 a 7 años, siendo una de las causas más frecuentes en la dermatología pediátrica, afecta igual a ambos sexos y a cualquier
etnia. Numerosos insectos causan prurigo, siendo los más frecuentes el Cemex lectularius (chinche) produciendo la Cimiasis; la pulga
(Pulex irritans) que ocasiona la puliciasis y las picaduras por mosquitos (Culicidae). Los alérgenos presentes en la saliva del insecto
inducen una sensibilización del paciente con formación de anticuerpos específicos. El tipo e intensidad de la reacción originada por la

CURSO ENARM CMN SIGLO XXI TEL: 36246001 Pharmed Solutions Institute PÁGINA 403
MANUAL DE TRABAJO DEL CURSO ENARM CMN SIGLO XXI
picadura dependerá de si el paciente ha estado ya expuesto al insecto y de la capacidad del huésped de responder al estímulo
antigénico. Las lesiones tempranas de prurigo por insecto se deben a una respuesta de hipersensibilidad tipo I causada por la liberación
de IgE. Posteriormente interviene un mecanismo de hipersensibilidad tipo IV dependiente de linfocitos T que produce las lesiones
tardías. El prurigo por insecto afecta cualquier área corporal, se presentan vesículas en la fase inicial posteriormente aparecen
pequeñas pápulas eritematosas, las lesiones son muy pruriginosas por los que se observan costras hemáticas por rascado. Se observan
lesiones en diferentes estadios y evolucionan por brotes. PRURIGO NODULAR: Es una dermatosis crónica caracterizada por nódulos
muy pruriginosos que aparecen principalmente en las superficies extensoras de extremidades inferiores, puede desarrollarse luego de
una picadura por insecto ó de otras formas de inflamación localizada. La lesión por sí misma es muy pruriginosa, lo cual es el resultado
de un círculo vicioso establecido por el rascado, trauma mecánico e infección, etc. Diversas investigaciones señalan que existe un
número elevado nervios dérmicos y células de Merkel en estas lesiones, sugiriendo que este cuantitativo aumento puede estar
directamente involucrado en la etiología. Se ha reportado que la terapia antituberculosa produce una aclaración parcial o completa de
las lesiones. El prurigo nodular puede ocasionalmente ser el resultado del prurito que acompaña a algunas enfermedades sistémicas
como los procesos obstructivos biliares, anemia, enfermedad renal crónica, policitemia vera, DM, parasitosis, erupciones por droga y en
los niños el más común el Linfoma de Hodgkins. Las lesiones del prurigo nodular presentan un diámetro aproximado de 0,5cm a 3cm, la
superficie de las lesiones puede ser queratósica ó deprimida en el centro. La lesión inicial es eritematosa y puede semejar a la urticaria,
sin embargo todas las lesiones tienden a pigmentarse con el tiempo. El número de lesiones varía desde 2 a 200 de las cuales algunas
mejoran espontáneamente. PRURIGO DE SUTTON: Es una erupción liquenoide de los codos en niños, pitiriasis de codos y rodillas. Esta
entidad clínica es una dermatosis eccematosa papular pruriginosa, la cual está limitada a codos, pudiendo extenderse a nudillos, manos
y tórax. Se asocia a una historia personal de atópia o familiar, así como a niveles elevados de IgE sérica. Se ha descrito en niños de 3 a
13 años, siendo la lesión inicial una pápula eritematosa de 1-2mm de diámetro. PRURIGO PIGMENTOSO: Es una dermatosis rara,
ocurre en Japón, se describe el trauma físico o fricción de la ropa como gatillo. Se ha detectado que puede ocurrir en adolescentes, se
caracteriza por pápulas eritematosas pigmentadas que confluyen formando un patrón reticulado, las lesiones se presentan en verano y
más frecuentes en mujeres jóvenes. El rash es de distribución simétrica en tronco, área cervical, región lumbosacra, abdomen y cara.
PRURIGO DEL ATOPICO O DE BESNIER: También llamado neurodermatitis, o eccema del lactante. Es la dermatosis más frecuente en
población pediátrica. La prevalencia ha mostrado incremento en las últimas décadas, siendo del 18-20%. Es más frecuente en áreas
urbanas de países industrializados, especialmente en inmigrantes provenientes de países con menor prevalencia. Es una variante poco
común en la infancia, se puede observar en adolescentes. De acuerdo con la morfología, pueden definirse tres tipos de lesiones, sin ser
el tiempo de evolución determinante para su definición: 1. Aguda. Caracterizada por pápulas y vesículas muy pruriginosas, sobre un
área de piel eritematosa, asociada a escoriaciones, erosiones, exudado seroso y costras melicéricas (eccema). 2. Subaguda.
Caracterizada por eritema, pápulas, descamación y escoriaciones. 3. Crónica. Placas de piel engrosada, con liquenificación y pápulas
fibróticas. Existen además tres fases cronológicas, con características específicas: 1. Lactante (2 semanas a 2 años). Afecta la cara,
predominando en mejillas y respetando el triángulo central. Puede extenderse a piel cabelluda, pliegues retroauriculares y de flexión,
tronco y nalgas; a veces puede generalizarse. Predominan las lesiones de dermatitis aguda por lo que se le ha denominado “eccema del
lactante”. Se puede asociar con dermatitis seborreica en un 17%. Aparece por brotes, siendo frecuente la desaparición de a los 2 años
de edad. 2. Escolar o infantil (3 a 14 años). Afecta pliegues de flexión (antecubitales y huecos poplíteos), cuello, muñecas, párpados,
región peribucal y genitales. Puede presentarse con lesiones agudas o crónicas, evolucionando en brotes, con prurito intenso. Puede
desaparecer (75-90%) o progresar a la última fase. Existen formas localizadas como el mal plantar juvenil, que afecta plantas y dorso de
los pies, forma periorbitaria, queilitis exfoliativa, intertrigo auricular y eccema del pezón. Por otro lado, se describen formas atípicas
como lo son la papular y folicular. 3. Adulto. (15 a 23 años de edad). PRURIGO ACTINICO: Es una erupción papular o nodular frecuente,
persistente, pruriginosa y escoriada de la piel expuesta al sol y en menor medida de la no expuesta. Es frecuente en verano y en
ocasiones no desaparece con el invierno, por lo general aparece en la niñez y a veces remite en la pubertad. Parece ser una variante
persistente y algunas veces coexístente de la erupción de lumínica polimorfica (ELPM) aunque sus características clínicas son diferentes.
La exposición de radiación UV sería el inductor del prurigo actínico (PA) ya que el trastorno es más intenso en primavera y en verano y
las respuestas cutáneas anormales a la radiación están presentes en dos tercios de los pacientes, más a menudo con radiación UVA que
con UVB. El PA podría ser una forma de ELPM exagerada, de evolución lenta y por lo tanto ser una reacción de hipersensibilidad de tipo
retardado. Esto se sustenta que más pacientes con PA de los esperados presenta familiares cercanos con ELPM, el antígeno linfocitario
humano (HLA) DR4, presentan el 30% de los sujetos normales, se encuentra en el 80-90% de los pacientes con PA y el subtipo DRB1
0407 de DR4 presentan en el 6% de los sujetos normales y frecuentan en indios estadounidenses se encuentra alrededor del 60% de
aquellos con PA. El PA es más común en el sexo femenino y suele comenzar a los 10años tiende a mejorar y desaparecer en la
adolescencia, aunque en ocasiones puede persistir en la vida adulta. La erupción por PA está presente durante todo el año aunque por
lo general empeora en el verano, y en raros casos se produce en invierno. Así mismo las exacerbaciones de las lesiones tienden a
producirse durante el clima soleado, en lugar de hacerlo después de episodios de exponerse al sol, aunque los brotes tipo ELPM son
posibles. En los casos típicos las lesiones son pápulas, nódulos pruriginosos, excoriaciones asociadas con eccematización liquenificación
o formación de costras, también se describe queilitis sobre todo en el labio inferior. El PA puede asociarse a una tendencia a la
eccematización. TRATAMIENTO: Las medidas generales son de gran importancia, la explicación a los padres acerca del padecimiento,
su predisposición atópica, la cronicidad y evolución por brotes. Se recomienda además el uso de pijamas largos, mosquiteros,
insecticidas cuando no están los niños. Tópicamente se utilizan lociones antipruriginosas y esteroides tópicos. Por vía oral se utilizan
antihistamínicos no sedantes. En algunas ocasiones se utiliza Tiamina por vía oral, la cual se ha recomendado empíricamente por
mucho tiempo "tal vez sea un repelente natural que se excreta por la piel", dosis 200mg a 600mg por día, lo más importante es evitar la
infección secundaría. Se utilizan emolientes, capsaicina tópica, radiaciones UV y sistémicamente Talidomida, la cual está contraindicada
cuando hay riesgo de embarazo y de producir neuropatía periférica.

CASO CLINICO
Niña de 11 años de edad que consulta por haber presentado lesiones eccematosas en el brazo izquierdo, donde hace un mes, en un
puesto ambulante, le realizaron un tatuaje de henna de color negro. A los 10 días, comenzó con eritema y vesículas muy pruriginosas,
que cubrían toda la zona del tatuaje. Las lesiones se resolvieron tras un mes de tratamiento con corticoide tópico, con persistencia de

CURSO ENARM CMN SIGLO XXI TEL: 36246001 Pharmed Solutions Institute PÁGINA 404
MANUAL DE TRABAJO DEL CURSO ENARM CMN SIGLO XXI
hipopigmentación residual limitada a la zona del tatuaje. La paciente se hacía, desde hace tres veranos, tatuajes transitorios de henna
color marrón en la playa.

PREGUNTA
Cual de los siguientes factores esta mas relacionado con el padecimiento.

RESPUESTA
a.- Infecciones de la piel.
b.- Exposición a alérgenos.
c.- Irritantes como jabon, shampoo y cremas.
d.- Inhalacion ingestión de alérgenos.

PREGUNTA
Considerando el caso clínico cual es la conducta terapéutica mas apropiada.

RESPUESTA
a.- Indica emolientes.
b.- Corticoides tópicos de leve potencia.
c.- Corticoides tópicos de moderada potencia.
d.- Corticoides tópicos de alta potencia.

CASO CLINICO
Masculino de 8 meses de edad que es llevado por su madre por presencia de tos y secresion nasal desde hace dos días y a la
exploración se observa en cara, brazos enrojecimiento, la madre agrega que le pica y que se presentan y luego desapareces solos, al
tacto la piel se siente aspera.

PREGUNTA
Cual es la conducta a seguir menos adecuada?

RESPUESTA
a.- Evitar ambiente caluroso, ropa de lana, platicos y otras fibras.
b.- Bañarlo con agua templada y de preferencia de periodos largos para mitigar lesiones.
c.- Aplicar crema hidratante y aceites varias veces al dia.
d.- Indicar crema con corticoide.

ERITEMA POLIMORFO. CIENCIAS BASICAS: El eritema multiforme (EM) o eritema polimorfo es una condición reactiva aguda,
autolimitada, en ocasiones recidivante, mediada inmunológicamente, que afecta la piel y las membranas mucosas. Desencadenada
fundamentalmente por el virus del herpes simple (VHS), presenta lesiones en diana y un patrón histopatológico inflamatorio. Se
cataloga como una reacción inflamatoria aguda, autolimitada, con diversos niveles de gravedad y variado conjunto sintomático.
CLASIFICACION: Eritema multiforme maculopapuloso o “eritema polimorfo menor (menor intensidad en los cambios
anatomopatologicos, escasa o nula participación mucosa). Representa el 80 % de los casos. Eritema multiforme vesiculoampolloso o
“eritema polimorfo mayor”. Las formas graves, copn afeccion del estado general y posible participación de órganos internos
corresponderían al Sindrome de Steven-Jhonson. La Necrolisis toxica epidérmica puede coinsiderase como un proceso aparte aunque
algunos autores lo consideran dentro del mismo espectro. SALUD PUBLICA: Predomina en adultos jóvenes, de 20-40 años, con afección
de niños y adolescentes en menos del 20% de los casos. Es algo más frecuente en varones y no presenta predilección racial. Se observa
con mayor frecuencia durante la primavera y verano, probablemente secundario a la reactivación del VHS desencadenada por la
exposición solar. Entre un 25% y un 50% de casos no puede establecerse el agente desencadenante. PATOGENIA: Se han reportado
numerosos factores desencadenantes, entre los cuales se encuentran infecciones, enfermedades malignas, enfermedades
autoinmunes, radiación, inmunizaciones, medicamentos (AINE, sulfonamidas, anticonvulsivantes (30%), penicilinas, doxiciclina,
tetraciclinas) y menstruación. El VHS es la causa más común y está involucrado en un 80-90% de los casos, con mayor asociación al VHS
tipo 1. Se obtiene una historia de herpes labial en un 50% de los pacientes en las 2 semanas previas al desarrollo del EM; sin embargo,
la lesión herpética puede ocurrir simultáneamente o desarrollarse luego de las lesiones en diana. Los mecanismos patogénicos han sido
estudiados en los casos de infección por el VHS. En la recurrencia, material genético es fagocitado y transportado a la circulación
periférica por monocitos/macrófagos que portan el antígeno linfocitario cutáneo, un antígeno que les permite transportarse a la piel,
en donde se adhieren a las células endoteliales de la microvasculatura dérmica, y el ADN viral fragmentado es finalmente transferido a
los queratinocitos del estrato basal y de las capas inferiores del estrato espinoso. La inflamación en la lesión cutánea se cree es debida a
una respuesta inmune celular TH1 específica contra los queratinocitos que contienen el gen de la polimerasa de ADN (Pol), con la
liberación de citoquinas efectoras como el interferón γ (IFN-γ), que lleva a la amplificación de una respuesta inflamatoria no específica a
través de células T autorreactivas y que resulta en la apoptosis de queratinocitos aislados (necrosis celular satélite) y el resto de los
hallazgos patológicos que observamos en el EM. Finalmente, aditivos de alimentos, como los benzoatos y terpenos, y agentes tópicos
como los tatuajes y la dermatitis por contacto con hiedra venenosa, han sido reportados como causas de EM. DIAGNOSTICO: Tal como
indica su nombre de la enfermedad, ls lesiones clínicas son polimorfas e incluyen maculas, papulas y lesiones vesiculo ampollosas. Las
lesiones más características muestran una apariencia anular concéntrica “en iris” o “en diana” Generalmente no se observan
pródromos; algunos pacientes pueden presentar fiebre, cefalea, malestar y mialgias leves una semana antes de la erupción. Las
lesiones cutáneas se desarrollan en forma súbita, completándose el brote en 3-5 días. Las lesiones pueden diferir entre un paciente y
otro, y variar durante la evolución de la enfermedad. Hay sensación de prurito o ardor. La erupción inicia como pápulas

CURSO ENARM CMN SIGLO XXI TEL: 36246001 Pharmed Solutions Institute PÁGINA 405
MANUAL DE TRABAJO DEL CURSO ENARM CMN SIGLO XXI
eritematoedematosas que simulan picadas de insectos, que permanecen fijas por más de una semana. Algunas lesiones aumentan de
tamaño, formando placas menores de 3cm, redondeadas, de bordes bien definidos, que desarrollan anillos concéntricos (eritema iris).
La lesión en diana típica consiste en una porción central de color rojo sucio, rojo vinoso o purpúrico, inmediatamente rodeada por una
zona externa de color rojo intenso; le sigue un anillo edematoso pálido, rodeado finalmente de un halo eritematoso. La porción central
en ocasiones puede presentarse vésico-ampollar, formando el herpes iris de Bateman, o adquirir un aspecto costroso. La distribución es
acral y simétrica, con predilección por el dorso de las manos y superficies extensoras de las extremidades superiores; con menos
frecuencia se afectan las palmas, cara, cuello, tronco y extremidades inferiores. Se han descrito lesiones siguiendo las líneas de
Blaschko. Las lesiones mucosas se presentan en un 25-60%, y ocurren conjuntamente con las lesiones cutáneas. La mucosa oral es la
más afectada, principalmente la mucosa no queratinizada del tercio anterior. Inicia de forma rápida como edema y enantema que
afecta los carrillos, gingival y lengua, que progresa a erosiones superficiales con formación de seudomembranas; en los labios se
observan fisuras, sangrado y costras serohemáticas. El prurito, la sensación de quemazón, así como la pobre alimentación y baja ingesta
de líquidos debida al dolor de las erosiones mucosas, que puede ser severo, son causas importantes de morbilidad en el EM
Laboratorio: No hay pruebas específicas para el diagnóstico. En casos severos puede encontrarse aumento de la velocidad de
eritrosedimentación, leucocitosis o elevación de las pruebas de función hepática. El diagnóstico del eritema multiforme es clínico. Los
cambios histológicos dependen de la morfología clínica, la duración y el área de la lesión de donde se toma la biopsia. El hallazgo más
precoz es la apoptosis de queratinocitos. Se observa necrosis celular en satélite, con linfocitos unidos a queratinocitos necróticos
aislados. ERITEMA MULTIFORME MENOR: Originado principalmente por infecciones virales herpéticas (herpes simple) o Mycoplasma
pneumoniae y solo en 1 % por productos farmacéuticos. La erupción se produce en el término de 12 a 24 horas y en la mitad de los
casos hay un pródromo similar a una infección de las vías respiratorias altas. Las lesiones cutáneas típicas son máculas rojo- azuladas,
con 3 anillos concéntricos denominados en diana o arco de tiro y una ampolla central, si bien pueden estar afectadas las mucosas. De
hecho, 20 % de los eritemas multiformes afecta a niños y adultos jóvenes; su inicio es “abrupto”, con fiebre alta y síntomas
prodrómicos intensos. Se observan lesiones cutáneas en el tronco, similares al eritema multiforme menor, pero más extensas y
necrosantes. Daña gravemente a más de 2 mucosas y ocasiona erosiones costrosas profundas en los labios; su evolución es más
prolongada, especialmente con fármacos de vida media más larga y sus manifestaciones clínicas duran entre 3 - 6 semanas. Las formas
menores de eritema multiforme desaparecen espontáneamente en alrededor de 3 semanas sin secuelas. SÍNDROME DE STEVEN-
JOHNSON: Es una forma grave de eritema multiforme habitualmente de ori gen tóxico. El grado de afectación cutánea varía desde
formas con gran lesionalidad hasta casos con escasas o nulas lesiones cutáneas. Sin embargo, se caracteriza por la severa participación
de mucosas (oral, nasal, conjuntival, genital) y afectación del estado general. NECRÓLISIS TÓXICA EPIDÉRMICA: La característica clínica
más importante es la formación de grandes ampollas flácidas que condicionan el desprendimiento de láminas de epidermis, dejan do
extensas áreas denudadas. Se trata de una situación grave, en ocasiones mortal. TRATAMIENTO: La aplicación del tratamiento puede
mejorar el cuadro sintomático y acortar la evolución del proceso; pero las formas mayores suelen ser mortales o causar graves
secuelas. La tasa de mortalidad para el síndrome de Stevens-Johnson se sitúa entre 1 y 5%, según distintos autores. La existencia de una
gran extensión de zonas denudadas, la edad avanzada, la insuficiencia renal concomitante y la afectación pulmonar empeoran el
pronóstico. Dado el curso autolimitado del proceso, en eritema multiforme menor, solo se requiere tratamiento sintomático. En los
casos postherpeticos recurrentes puede plantearse tratamiento con Aciclovir u otros agentes antiherpeticos. El Síndrome de Stevens
Jhonson y La Necrolisis toxica epidérmica son procesos potencialmente graves que requieren de ingreso del paciente y manejo en
cuidados intensivos, en condiciones de asepsia. Existe controversia respecto al empleo de corticoides sistémicos pues si bien suelen
condicionar un alivio de sintomatología subjetiva no hay evidencia de que reduzcan la morbimortalidad. En ocasiones se precisa
antibioticoterapia para combatir complicaciones sépticas.

CASO CLINICO
Preescolar de 3 años de edad, de sexo femenino, previamente sana, que consulta por cuadro de dos días de evolución, caracterizado
por fiebre, odinofagia y discreto compromiso del estado general. Evaluada en el Servicio de Urgencia se diagnosticó amigdalitis aguda
purulenta. Se indicó penicilina benzatina 600 000 UI intramuscular por 1 vez. Posterior a la administración y en el transcurso del mismo
día, comenzó con prurito generalizado, más intenso en la región dorsolumbar. Horas más tarde, la madre notó aparición de lesiones
maculares eritematosas en extremidades, cara y tronco, por lo cual consultó nuevamente, 48 horas después. En ese momento, al
examen clínico se constató una paciente en buenas condiciones generales con temperatura de 38,5° C axilar y lesiones papulares,
eritematosas, redondeadas, distribuidas en forma simétrica y comprometiendo todo el cuerpo, más marcado en cara y extremidades
inferiores. Las lesiones presentaban aspecto de anillos concéntricos, con un halo periférico eritematoso y un área más central pálida y
en algunas lesiones una pequeña vesícula. La mucosa oral presentaba petequias escasas en forma aislada. No había otras mucosas
comprometidas. El hemograma mostró GB 11 700 (75% segmentados, 5% baciliformes), hematocrito 38%, plaquetas 384 000 x mm3.
Además VHS 80 mm x hr, PCR 10,9, creatininemia 0,61 mg/dl, nitrógeno ureico 0,22 mg/dl. Radiografía de tórax y sedimento de orina
fueron normales.

PREGUNTA
Dentro de los siguientes diagnosticos cual es el menos probable?

RESPUESTA
a.- Eritema Multiforme.
b.- Stevens-Jonhson.
c.- Necrolisis Epidérmica Tóxica
d.- Vasculitis.

CURSO ENARM CMN SIGLO XXI TEL: 36246001 Pharmed Solutions Institute PÁGINA 406
MANUAL DE TRABAJO DEL CURSO ENARM CMN SIGLO XXI
ERITEMA NODOSO. CIENCIAS BASICAS: Es un síndrome caracterizado por una erupción cutánea nodular, eritematosa, caliente y
dolorosa, que se localiza preferentemente en la región pretibial, aunque también en ocasiones en muslos, brazos, antebrazos y otras
áreas corporales. PATOGENIA: La etiología de este síndrome es múltiple, pero podemos globalmente dividirla en dos causas
fundamentales: 1. De causa infecciosa: Bacterianas; tuberculosis (bacilo de Koch), estreptococicas, por yersinia, salmonela, shigela,
campilobacter, brucelosis, tularemia, enfermedad por arañazo de gato, rickettsias, clamidias, mycoplasma, leptospirosis. Virica; VEB,
hepatitis B, parvovirus B19. Protozoos; Giardia lamblia, amebas, toxoplasma. 2. De causa no infecciosa: Medicamentos;
antoconceptivos orales, sulfamidas, penicilinas, salicilatos, TMP, bromuros, yoduros, sales de oro. Las lesiones del eritema nodoso
representan una reacción frente a varios estímulos, se consideran como un tipo de vasculitis cutánea alérgica, pudiendo, por lo tanto,
ser consecutivas de cierto número de estímulos provocadores. DIAGNOSTICO: En ocasiones se acompaña de fiebre, con aceptable
estado general, puede haber artralgias de localización en grandes articulaciones de extremidades inferiores, que puede ser la
manifestación de un proceso subyacente causa de esta patología. Las lesiones cutáneas, son bastante características; se trata de
nódulos de 1-3 cm de diámetro, dolorosos, indurados, brillantes, enrojecidos, calientes y sobreelevados. Aparecen en las áreas
pretibiales fundamentalmente, pero también pueden tener otras localizaciones, como las nalgas, pantorrillas y también en las
extremidades superiores. La forma evolutiva de estas lesiones es bastante característica; durante un periodo de varios días, se hacen
prominentes y de color violáceo; después de 1-2 semanas van disminuyendo su prominencia e induración, modificando el color,
pasando por purpura oscuro y al final desaparecen dejando un rastro parduzco, sin dejar ulceración, ni cicatriz. Estas lesiones aparecen
por brotes, en un periodo entre 3-6semanas, para determinar desapareciendo definitivamente. El diagnóstico es clínico, pero debemos
hacer el diagnostico etiológico se refiere a la necesidad de descartar una enfermedad asociada; esto puede ser en ocasiones difícil y se
puede no llegar al mismo en 20% de los casos. Hay que practicar pruebas complementarias como PPD, analítica básica general,
coprocultivo, serologías, cultivos, radiografía de tórax, tránsito intestinal si es necesario, etc. TRATAMIENTO: Generalmente tienen una
resolución espontanea, y la adopción de medidas generales como el reposo y las medidas físicas es suficiente. El tratamiento debe ir
dirigido a tratar la enfermedad de base que sea la causante de este trastorno. Las medidas generales van encaminadas a disminuir el
dolor y el acortamiento de la enfermedad. Dentro de ellas es importante insistir en el reposo y en la elevación de las extremidades
inferiores. Diversos fármacos antiinflamatorios han demostrado ser útiles en el tratamiento del eritema nodoso, pero especialmente los
salicilatos, la indometacina y el ibuprofeno. El empleo de corticoides sitemicos se debe reservar para aquellos casos con
manifestaciones sistémicas graves (artritis, fiebre muy elevada), enfermos con sarcoidosis y pacientes con eritema nodosp persistente y
ausencia de respuesta a los demás tratamientos mencionados. ERITEMA NODOSO MIGRATORIO: Es una variante clínica de eritema
nodoso, también denominada Paniculitis migratriz de Bäfverstedt o Vilanova. Presenta nódulos unilaterales; en general menos
numerosos, dolorosos y persistentes que los de la forma clásica. Al progresar tienden a dividirse y extenderse en la periferia, esto les
confiere un patrón arciforme con bordes eritematosos, brillantes y un centro más violáceo o pardo.

CASO CLINICO
Masculino de 12 años de edad acudió a consulta debido a que 15 dias previas inicio con lesiones eritematosa violácea nodulares y
dolorosas en los miembros inferiores de predomio en la parte posterior de las piernas. De 1 a 2 cm de diámetro, no referia ingesta de
fármacos, fiebre, síntomas respiratorios, digestivos ni articulares, en la anamnesis no se encontraron antecedentes personales ni
familiares de interés.

PREGUNTA
Considerando la etiología mas frecuente de esta patología, cual es la conducta diagnostica mas apropiada.

RESPUESTA
a.- Biopsia.
b.- PPD.
c.- Hemocultivo.
d.- Panel viral

DERMATITIS DE CONTACTO. CIENCIAS BASICAS: La dermatitis de contacto o eczema de contacto constituye un síndrome motivado por
la reacción cutánea de una sustancia aplicada en la piel. Pue de ser una reacción eczematosa aguda o crónica y está causada por un
irritante primario o por un mecanismo de sensiblización a un alérgeno tópico Es una dermatosis frecuente, afecta a ambos sexos y a
todas las razas y puede presentarse a cualquier edad. Es la causa dermatológica más frecuente de enfermedad profesional. SALUD
PUBLICA: Es una de las enfermedades de la piel más frecuentes, alrededor de 10% de todas las dermopatías. Principales sensibilizantes:
Medicamentos (antibióticos, sulfonamidas, mercuriale, antihistamínicos, anestésicos, psoralenos), cosméticos (parafenilenodiamina,
peróxidos, formaldehido, colorantes azoados, perfumes), metales (niquel, cromo, cobalto), ropas y zapatos (telas sintéticas, hules,
cueros), plásticos (epoxi, resinas, acrílicos, nylon), remedios caseros (ajo, limón, yerbas). PATOGENIA: Cualquier sustancia u objeto que
este en contacto con la piel puede ocasinar una dermatitisde contacto. El mecanismo puede ser: A) Alérgico, por el modelo clásico de
inmunidad celular (hipersensibilidad tipo IV). Es necesaria fase de sensibilización, fase de latencia y de reexposición o desencadenante.
El antígeno suele ser un hapteno que una vez en la epidermis, previa unión a una proteína, conforma un antígeno completo que,
mediante células de Langerhans, es presentado a los linfocitos T que proliferan como células T efectoras y de memoria penetrando en
la circulación sanguínea. En la fase de desencadenamiento, 48 hrs después de la exposición, los linfocitos sensibilizados reconocen a los
alérgenos dando lugar a una transformación blástica y proliferación clonal con liberación de citocinas mediadoras de la inflamación.
También participan otras células sanguíneas. Es posible la sensibilización por vía oral o parenteral. El fenómeno de autoeczematizacion
por proteínas epidérmicas es frecuente en lesiones en piernas. B) Irritativo que origina un inflamación de la piel secundaria a la
exposición de un agente irritante, en concentración y tiempo suficiente o contactos repetidos del mismo que da lugar a lesión celular
sin que exista mecanismos inmunológicos. DIAGNOSTICO: Localización variada y siempre en relación al agente causal. La clínica puede
ser: 1. Aguda, con eritema, edema, vesículas, ampollas y en casos extremos necrosis. Existe prurito quemante. 2. Subaguda, con placas
eritematosas con descamación. 3. Crónica con lesiones en placas liquenificadas y zonas de descamación y excoriación. A veces grietas y

CURSO ENARM CMN SIGLO XXI TEL: 36246001 Pharmed Solutions Institute PÁGINA 407
MANUAL DE TRABAJO DEL CURSO ENARM CMN SIGLO XXI
fisuras. En casos muy intensos y de exposición prolongada al antígeno, pueden aparecer lesiones a distancia o “ides”. El diagnóstico se
basa en la clínica, profesión y hábitos del paciente. Las pruebas epicutáneas sirven para diferenciar el eczema de contacto de origen
irritativo del de origen alérgico y, en esta caso identificar el antígeno. Los diferentes tipos de eczemas o dermatitis de contacto tienen
los mismos rasgos histopatológicos y van a depender de la fase evolutiva en la que se biopsien. Como rasgos generales evidenciaremos
espongiosis, exocitosis y en dermis superficial infiltrado perivascular, fundamentalmente de tipo linfocitario. TRATAMIENTO: Antes que
nada debe indicvarse al paciente lo que no debe de hacer: No aplicarse nada en la piel, eliminar toda clase de pomadas,
remedioscaseros, objetos, ropas cuando se sospecha de ellas. Evitar jabones y detergentes. No hay dietas. Puede comer de todo. Evitar
el sol si hay fotosensibilidad. El tratamiento es sobre todo tópico y dependerá del estado de la piel. Una piel eczematosa requiere
primero medicamentos para secarla, así están indicados los fomentos con agua de manzanilla o agua de vegeto al 50% (subacetato de
plomo). Se aplican a la temperatura ambiente o fríos, cada 3 o 4 hrs, durante las primeras 24-48 hrs, hasta que la piel se seque y las
costras melicericas se desprendan, cuando ello se logre se podrá aplicar las conocidas pastas inertes a base de óxido de znc y/o
calamina. El uso de sedantes y antipruriginosos puede ser necesario en algunos casos y a veces hasta psicotrópicos del tipo de la
clorpromazina. Los corticoides por vía tópica del tipo de la hidrocortisona están indicados solamente en las dermatitis por contacto por
irritantes primarios y en casos muy limitados de sensibilización. Si hay mucha sensibilización y muchas ides, el esteroide mejorara de
inmediato el cuadro, pero vendrán los rebotes cada vez más intensos y más ides y el resultado final será un estado eritrodermico difícil
de controlar.

CASO CLINICO
Un niño de 12 años, en buen estado general y afebril, consultó, por presentar una placa eritematosa única, amplia, de bordes definidos,
sobre la que asentaban pequeñas pápulas y pústulas, no fluctuante, localizada en la región lateral derecha del cuello, de 3 días de
evolución; se acompañaba de leve prurito, pero era indolora. No presentaba adenopatías regionales. Había acudido a una guardia,
donde se le indicó cefalexina en dosis de 100 mg/kg/día; en el momento de la consulta cursaba el primer día de tratamiento. En el
interrogatorio se destacaba que, 48 horas antes a la aparición de las lesiones, el niño había participado en actividades recreativas al aire
libre en la colonia de verano del club, y no se había aplicado repelente.

PREGUNTA
Cual es la conducta a seguir mas adecuada en este momento?

RESPUESTA
a.- Continuar con antibiótico por 5 dias.
b.- Corticoides topicos.
c.- Prednisona oral.
d.- Cetirizina únicamente.

DERMATITIS DEL PAÑAL. CIENCIAS BASICAS: Dermatitis del pañal, dermatitis amoniacal, “rozadura”. Es una erupción inflamatoria
aguda del área cubierta por el pañal. SALUD PUBLICA: Afecta con mayor frecuencia a recién nacidos y lactantes, con un pico de
incidencia a los 9 a 12 meses, aparentemente en relación con el cambio en la dieta. Se presenta además en niños mayores y adultos con
incontinencia urinaria y/o fecal secundaria a alteraciones urológicas o neurológicas. La prevalencia es del 7 al 35% en población
pediátrica, ocupando el 3ª a 4ª causa de consulta dermatológica en niños. No se ha descrito diferencia racial ni en cuanto al sexo.
PATOGENIA: Koblenzer clasificó la dermatitis de esta zona en tres grupos: Grupo 1. Enfermedades que se producen en forma
independiente del uso de pañales (epidermolisis bulosa, histiocitosis de células de Langerhans, etc.). Grupo 2. Enfermedades agravadas
por el uso de pañales (dermatitis atópica, dermatitis seborreica, psoriasis, etc.). Grupo 3. Enfermedades provocadas por el uso de
pañales, en niños no predispuesto y como consecuencia directa de su uso, es el más frecuente y ocupará nuestra revisión. Lo más
característico es la dermatitis de contacto por irritante primario en la cual la humedad persistente provoca maceración de la piel; lo
anterior aunado a la fricción generada por los movimientos del bebé altera la función de barrera y permite la acción de irritantes como
son: heces (proteasas, lipasas y sales biliares), orina (urea y amoniaco), jabones, lociones, cremas y productos empleados para el lavado
de los pañales; por lo tanto, los irritantes actúan en forma secundaria en piel ya dañada previamente. Por otro lado, la piel así dañada,
es susceptible de infección por Candida albicans con mayor frecuencia (50%) y más raramente, agentes bacterianos (S aureus,
estreptococos y enterobacterias). La dermatitis por contacto alérgica no es común el área del pañal, sin embargo algunas sustancias
como gomas, plásticos, neomicina y mercurio pueden actuar como sensibilizantes. DIAGNOSTICO: Existen diferentes patrones de
afección clínica: Dermatitis eritematosa simple. Es el más frecuente. Afecta áreas convexas (en W), respetando los pliegues. Las formas
más leves se caracterizan por eritema y descamación; el eritema puede desarrollar una apariencia vidriosa y aparecer pápulas.
Variedad sifiloide pápulo-erosiva o poserosiva de Sevestre-Jaquet. Es una forma moderada de la dermatitis eritematosa simple. Se
presenta sobre las superficies convexas en que se observan pápulas del color de la piel o eritematosas, generalmente menores de 5 mm
de diámetro, que por acción de la fricción o maceración se erosionan en la cúpula. Pueden volverse violáceas y liquenificadas. Ulcerosa.
Se caracteriza por pérdida cutánea epidérmica o más profunda, siendo la presentación más grave de las anteriores; surge como
consecuencia de la potencia y el tiempo de exposición a la agresión y sensibilidad individual de la piel. Dermatitis perianal. Limitada a
esta área, se manifiesta como eritema, erosiones o pústulas; es más frecuente en el periodo neonatal. Puede deberse a cambios
químicos (alteración del pH en procesos diarreicos o enteropatías) o infecciosos (dermatitis estreptocóccica perianal, parasitosis). El
diagnóstico es clínico; puede solicitarse examen directo con KOH y cultivo para corroborar infección micótica. La biopsia se reserva a
casos resistente al tratamiento, en que se sospecha otra enfermedad de base. TRATAMIENTO: Medidas preventivas, como son el
cambio frecuente de pañal, recomendándose pañales desechables con geles absorbentes (alginatos), de mayor tamaño, para evitar el
roce y facilitar la absorción; realizar el aseo del área con agua, dejando el área sin pañal el mayor tiempo posible. Las pastas con talco y
óxido de zinc son útiles como aislantes y para reducir la fricción y se aplican posterior a cada cambio de pañal. En las lesiones con más
de 72 horas de evolución o datos de infección por candida debe aplicarse algún agente antimicótico, 2 veces al día, por 3 semanas. En
caso de impétigo segundario se recomienda el uso de antibióticos tópicos o sistémicos. La utilización de corticoesteroides deberá ser

CURSO ENARM CMN SIGLO XXI TEL: 36246001 Pharmed Solutions Institute PÁGINA 408
MANUAL DE TRABAJO DEL CURSO ENARM CMN SIGLO XXI
racional, restringiéndose a casos resistentes a las terapias convencionales, utilizando esteroides de baja potencia y nunca más de 2
semanas. COMPLICACIONES: Candidosis. Es la complicación más frecuente. Compromete los pliegues inguinales e interglúteos y se
caracteriza por pápulas y pústulas sobre una base eritematosa, intensa, brillante, con bordes netos, elevados, geográficos y escama
blanquecina; asimismo lesiones satélites constituidas por pápulas y pústulas que se presentan en la periferia. Se ha demostrado
colonización por C albicans en la dermatitis de la zona del pañal con más de 72 horas de evolución (45-75%), aún en ausencia del cuadro
clínico característico. Impétigo secundario. Agregándose pústulas y costras melicéricas. Granuloma glúteo infantil. Pápulas y nódulos
rojizos o purpúricos de 0.5 a 4 cm, en región perianal y glútea; se ha asociado a irritación y fricción crónica, C. albicans y uso de
esteroides fluorinados.

CASO CLINICO
Niña de 4 meses de edad que la trae su padre porque al ir a cambiarle el pañal le ha visto que tenía la zona muy eritematosa y además
al limpiarle la niña se queja.

PREGUNTA
Considerando el agente etiológico mas frecuente, cual es la conducta especifica mas adecuada?

RESPUESTA
a.- Desecantes.
b.- Emolientes.
c.- Antifungico.
d.- Antihistaminico.

PSORIASIS. CIENCIAS BASICAS: Es una enfermedad autoinmunitaria eritematodescamativa muy frecuente de evolución crónica que
cursa a brotes. Se caracteriza por lesiones bien definidas con escamas típicas blanco-nacaradas. SALUD PUBLICA: Afecta a 1-3% de la
población general. Aproximadamente el 25% de los casos de psoriasis se inicia durante la infancia, y aumenta su incidencia con la edad
hasta la adolescencia. PATOGENIA: Es una enfermedad hereditaria aunque no se conoce todavía el modo de herencia. Existe una
predisposición genética compleja y probablemente multifactorial. Hay una frecuencia aumentada de algunos haplotipos HLA. Los
factores genéticos influyen en el patrón de la psoriasis, severidad y edad de inicio. El riesgo para los familiares de primer grado de un
caso aislado es menor de 10%, pero si los dos padres tienen psoriasis los hijos tienen un riesgo de un 50% de padecerla. Existen unos
factores desencadenantes conocidos capaces de precipitar o exacerbar un brote. Las infecciones (particularmente por Streptcoccus), el
frio, el estrés emocional, los traumatismos cutáneos repetidos y ciertos fármacos (cloroquina y corticoides sistémicos). DIAGNOSTICO:
Las lesiones de psoriasis infantil son placas eritemato-escamosas similares a las de la edad adulta, de tamaño y forma variables, con los
mismos signos característicos al raspado y con tendencia también a desarrollar lesiones de psoriasis en las zonas de roce o de rascados
y traumatismos (fenómeno isomórfico de Koebner). La biopsia cutánea muestra una epidermis engrosada con acantosis y
papilomatosis, elongación y edema de la dermis papilar y un adelgazamiento de la epidermis por encima de las papilas. Se han realizado
numerosos estudios sobre la calidad de vida en pacientes con psoriasis3, y se ha demostrado que repercute en actividades cotidianas y
en el bienestar físico y psicológico de los enfermos. TRATAMIENTO: En niños se basa en la experiencia y en las pautas terapéuticas
empleadas en los adultos, con las limitaciones propias de sus efectos secundarios. Tópico: Emolientes y queratolíticos: No tienen
efectos secundarios, alivian el prurito y disminuyen la descamación, aunque es el tratamiento menos eficaz. Son útiles para conseguir
eliminar escamas importantes y así facilitar la penetración de otros productos más efectivos. Uno de los más utilizados es la vaselina
salicílica al 3-5%. El alquitrán de hulla es la brea más utilizada; en niños se emplean los baños de breas sólo en casos muy extensos, por
su efecto calmante y reductor. La antralina (dithranol) al 0,1% parece que es menos irritante y es útil en placas muy queratósicas;
carece de toxicidad general. Se ha demostrado además que los tratamientos en aplicaciones cortas son efectivos y menos irritantes,
mediante aplicaciones diarias de 10-30 min y aumentando progresivamente la concentración del 0,1 hasta el 2%. Corticoides: tienen en
la psoriasis una triple acción: antiinflamatoria, inmunosupresora y antiproliferativa. Su uso se ve limitado por los efectos secundarios:
efectos locales como atrofia, estrías, telangiectasias e hipertricosis a menudo irreversibles; las erupciones acneiformes son también
frecuentes cuando se aplican en la cara corticoides de alta potencia, y aparecen pápulas y pústulas redondeadas y rojizas y
posteriormente comedones; a diferencia del acné juvenil, las lesiones se encuentran todas en el mismo estadio evolutivo. En pliegues,
genitales y cara causan más efectos secundarios locales que en el resto del tegumento. Efectos sistémicos: la absorción de corticoides
potentes a través de la piel puede provocar un Cushing iatrogénico e incluso detención del crecimiento. La vitamina D como
tratamiento para la psoriasis, dado que induce la diferenciación de queratinocitos y disminuye su proliferación. Calcipotriol.
Proporciona buenos resultados en niños sin alterar la concentración sérica de calcio, siempre que no se sobrepasen los 50 g/semana.
Inmunomoduladores tópicos: El tacrolimus y el pimecrolimus actúan inhibiendo la calcineurina y mejoran las placas de psoriasis poco
hiperqueratósicas. Si son muy hiperqueratósicas, se requiere la aplicación previa de un queratolítico tópico para facilitar su absorción.
Psoralenos + radiación ultravioleta A (PUVA): La PUVAterapia está basada en la interacción entre la radiación ultravioleta A (UVA) y un
agente fotosensibilizante químico: los psoralenos, pero no se recomienda su empleo en niños menores de 10 años, ya que se acumulan
en el cristalino y si éste está aún en desarrollo puede causar daños irreversibles; los rayos UVA penetran hasta el cristalino y pueden
inducir la aparición de cataratas, por lo que hay que extremar la protección ocular durante la radiación. La alta permeabilidad del
cristalino en los jóvenes contraindica este tratamiento antes de los 12 años. Por otra parte, son ya conocidos los efectos carcinogénicos
a largo plazo de la radiación UVA15-17. Tratamiento sistémico. Retinoides, metotrexato, ciclosporina. Tratamientos biológicos: Son
anticuerpos monoclonales o proteínas humanas recombinantes contra algunas citocinas: etanercept, infliximab, adalimumab
(antifactor de necrosis tumoral), efalizumab. Hay pocos casos en niños, aunque con buenos resultados, pero de uso experimental y
todavía no está aprobado

CURSO ENARM CMN SIGLO XXI TEL: 36246001 Pharmed Solutions Institute PÁGINA 409
MANUAL DE TRABAJO DEL CURSO ENARM CMN SIGLO XXI
CASO CLINICO
Varón de 17 años que, tras pasar una amigdalitis, presenta este exantema cutáneo. A la EF se observan papulas y pequeñas placas
eritematosas con minima descamaciones distribuidas de forma generalizada, pero principalmente en el tronco.

PREGUNTA
Considernado el tipo de psoriasis que presenta el caso, cual de las siguientes medidas es la mas adecuada para intervenir en el agente
casual?

RESPUESTA
a.- Corticoide tópico de mediana potencia, en excipiente no demasiado graso.
b.- Realizar baños con derivados de alquitrán.
c.- Antibiotico especifico preferentemente por cultivo.
d.- Exposición solar moderada y frecuencia.

DERMATITIS SEBORREICA. CIENCIAS BASICAS: Es una dermatosis inflamatoria eritemato-descamativa de evolución crónica y
recurrente que se localiza en cara, cuero cabelludo, pliegues y regiones medio-torácicas e interescapulares (zonas con mayor cantidad
de glándulas sebáceas activas). También se le conoce como eczema seborreico o pitiriasis seca y esteatoide. SALUD PUBLICA:
Enfermedad frecuente en países desarrollados, no existiendo diferencias de género. La dermatitis seborreica infantil aparece en la
lactancia, habitualmente entre la 2ª y la 8ª semanas de vida Puede afectar tanto a niños como a adultos o ancianos. PATOGENIA: La
causa es desconocida. Se consideran factores predisponentes, genéticos, emocionales, Atópicos, neurológicos, bacterianos, endocrinos,
alimentarios, medicamentosos, estrés y alcoholismo. También se ha relacionado con la disminución de ácidos grasos esenciales y
biotina. Aunque se conoce que existe un cambio cuantitativo y cualitativo en la composición del sebo con transformación de
triglicéridos en ácidos grasos más irritantes con mayor cantidad de colesterol y menor de escualeno, no se sabe cuál es la causa de la
citada alteración aunque se implica en ello la acción del Pityrosporum ovale, levadura lipófila que forma parte de la flora cutánea
normal y se convierte en oportunista con potencial patógeno. En pacientes con SIDA hay disminución de linfocitos CD4. En niños se
desconoce la causa aunque se sabe que del 15-35% de os pacientes durante la infancia padecieron dermatitis seborreica, padecen en la
edad adulta, psoriasis o dermatitis atópica. Un grupo de autores suecos afirma que la dermatitios seborreica infantil (DSI) podría
deberse a un déficit de la enzima d-6-desaturasa, que convierte el ácido linoléico en ácido gammalinolénico (AGL). Este déficit pare ce
normalizarse hacia los 6 ó 7 meses de edad, coincidiendo con la desaparición de la DSI. La asociación entre las levaduras de Malassezia
y la patogenia de esta enfermedad ha sido causa de controversia desde los años setenta. Evidencia reciente relaciona a la dermatitis
seborreica con metabolitos del hongo y con la inducción de citocinas. El hallazgo de las levaduras en escamas de los pacientes es lo
habitual. Las especies más frecuentes en cuero cabelludo son: M. globosa y M. restricta, y en rostro M. globosa y M. furfur.
DIAGNOSTICO: En todas las áreas, las lesiones de DSI consisten en placas eritematosas de tinte rosado, con morfología redondeada o
incluso circinada, con bordes bien definidos, aisladas o confluentes, que se cubren de una descamación amarillenta de aspecto
grasiento, que merece la denominación de ‘seborreica’. La DSI no se acompaña de síntomas generales ni prurito. Para muchos autores,
la forma más precoz de aparición de la DSI es la costra láctea. Otros, consideran que la costra láctea es una entidad diferenciada, sin
ninguna relación con la DSI. Sea como fuere, la costra láctea se desarrolla en la primera o segunda semanas de vida, y se manifiesta
como una gran placa descamativa, con escamas grasientas y adherentes, de color amarillento, bajo las cuales se puede apreciar un
eritema más o menos vivo. Asienta sobre todo en el vértex, y puede extenderse hacia áreas frontales con menor frecuencia, puede
ocupar una amplia superficie del cuero cabelludo. Las lesiones clásicas de DSI suelen afectar, además del cuero cabelludo, a la frente,
pabellones auriculares, pliegues retroauriculares, nasogenianos, cejas, parpados y cuello. También puede extenderse a el tronco, con
predominio en la zona periumbilical, axilas, inglés, tórax anterior y espalda En los miembros, suele afectar a los pliegues antecubitales y
poplíteos. En ocasiones, puede ser un trastorno generalizado, de tipo eritrodérmico. La afectación de la zona del pañal es muy
frecuente, especialmente en las áreas inguinales, pliegue interglúteo y genitales externos. El trastorno conocido como eritrodermia
descamativa de Leiner cursa con eritrodermia descamativa ‘seborreiforme’, retraso del crecimiento y diarrea. No hay estudios a gran
escala para establecer la histopatología de la DSI, aunque parece similar a la DS del adulto. Se observa paraqueratosis focal, acantosis y
espongiosis. En la desembocadura de los folículos pilosos dilatados se observan polimorfonucleares neutrófilos. La dermis superficial
muestra un infiltrado linfohistiocitario perivascular. TRATAMIENTO: No tienen tratamiento definitivo. Dependiendo del grado de
afectación se puede utilizar desde jabones no alcalinos o sustitutivos del jabon, a lociones o crema a base de ácido salicílico, azufre,
selenio, ketoconazol u otros derivados azolicos, ciclopiroxolamina, terbinafina, hidrocortisona, tacrolimus. A nivel general se puede
complementar con biotina a altas dosis.

CASO CLINICO
En la revisión de un bebe al mes de nacido, la madre refiere que el niño presenta costras que le sale sobre todo en la cabeza y en la
frente y cejas.

PREGUNTA
Cual de las siguientes medidas terapéuticas es la menos adecuada?

RESPUESTA
a.- Aplicar aceites y sustancias emoliente.
b.- Champues con antifungicos.
c.- Dieta rica en biotina.
d.- Corticoide de baja potencia.

CURSO ENARM CMN SIGLO XXI TEL: 36246001 Pharmed Solutions Institute PÁGINA 410
MANUAL DE TRABAJO DEL CURSO ENARM CMN SIGLO XXI
ACNE. CIENCIAS BASICAS: Sinónimos acné vulgar, acné polimorfo o acné juvenil. Es la inflamación crónica folicular producida por
retención de sebo, debida a obstrucción y distensión de la unidad polisebacea. SALUD PUBLICA: El “acné vulgaris” es la enfermedad
más común de la piel, afectando al 80% de las personas entre los 11 y los 30 años de edad en algún momento. Además, a nivel de los
adolescentes afecta más del 85%. Puede persistir por años y generar cicatrices permanentes y conllevar un daño importante a nivel del
desarrollo psicosocial y un problema de autoestima. Aunque se observa en ambos sexos, predomina ligeramente en el hombre.
PATOGENIA: La susceptibilidad a la seborrea está determinada genéticamente. Los factores patógenos fundamentales son:
queratinización folicular anormal, secreción sebácea aumentada, colonización bacteriana e inflamación local. No hay influencia práctica
de la dieta. Al llegar la pubertad, por aumento en la sensibilidad de las glándulas sebáceas, las hormonas andrógenas generan aumento
de tamaño y de la secreción de las mismas, lo que aunado a la hiperqueratosis de la pared y el poro foliculares, originan la lesión
fundamental e inicial del acné, el comedón (“espinilla”). Se ha demostrado la presencia de Propionibacterium acnes, quien transforma
los triglicéridos del sebo en ácidos grasos libres, que tienen efecto irritante local, favorece
la inflamación por este mecanismo. Si la inflamación se realiza en la parte terminal del
conducto folicular, a nivel de la epidermis, se origina una pústula, pero si se hace en la
parte proximal del folículo y en la misma glándula sebácea, se originan los abscesos. Los
factores psicógenos pueden agravar esta dermatosis de modo secundario, sobre todo en
aquellos pacientes que presentan excoriaciones y ulceraciones autoprovocadas.
DIAGNOSTICO: Afecta cara (99%), cuello y tórax (espalda 60%, pecho 15%), sobre todo en
su parte superior. En cuanto a la morfología, el acné se caracteriza por polimorfismo
lesional encontrándose lesiones no inflamatorias que son los comedones abiertos y
cerrados; el comedón abierto se reconoce como una concreción gris oscuro, café o negro
dentro de un orificio folicular dilatado y el comedón cerrado es una pequeña pápula del
color de la piel. Las lesiones inflamatorias son pápulas eritematosas, pústulas y nódulos.
Las secuelas del acné son cicatrices y senos epiteliales. TIPOS: De lesiones acneicas en
cuadro anexo. CLASIFICACION: De acuerdo al tipo de lesión predominante, el acné se
clasifican en: comedónico, papuloso, pustuloso, quístico, conglobata (abscesos y fístulas),
queloideo y fulminans (pápulas, nódulos y quístes acompañados de datos de infección
sistémica). De acuerdo a la gravedad del acné, se clasifican en: a) ACNE LEVE: Predominan
los comedones. b) ACNE MODERADO: Con predominio de pápulas y pústulas inflamatorias. c) ACNE GRAVE: Que consta de nódulos
(abscesos}) y de pápulas, pústulas y comedones. TRATAMIENTO: medidas generales; explicar al paciente claramente la naturaleza de la
enfermedad, los posibles efectos adversos de la terapia y su duración habitualemnte
prolongada, para evitar las expectativas a corto plazo y el abandono terapéutico. Detectar
posibles factopres gravantes o predisponenetes (medicaments, sustancias químicas).
Desaconsejar la manipulación de las lesiones e insistir en una adecuada limpieza e higienen
cutánea. Evitar el uso de cosméticos comedogenicos (cremas, lociones, geles, maquillajes) y
recomendar como alternativa interesante los cosméticos libres de grasa (“oil free”). El
tratamiento tópico es el pilar fundamental en el que se debe basar el manejo del acné en
atención primaria. Considerando la clasificación clínica anterior, el manejo se efectuará de la
siguiente manera. Acné leve. Jabones neutros o azufrados para el lavado dos veces al día y aplicación de lociones desengrasantes como
el licor de hoffman, a las que pueden agregarse ácido salicílico, resorcina o azufre a 1 y 3%. Pueden usarse comedolíticos como
retinoides tópicos o peróxido de benzoilo. Acné moderado: puede usarse clindamicina o eritromicina tópicas, en solución o gel, dos
veces al día o valorar el uso de antibióticos sistémicos aunado al tratamiento tópico. Acné grave: Los antibióticos sistémicos son de
primera elección para el acné moderado. Acné grave: los antibióticos sistémicos son de primera elección para el acné moderado a
grave, los más recomendados son las tetraciclinas, eritromicina, minociclina y el TMP/SFX, por uno a tres meses. La
diaminodifenilsulfona (DDS) da muy buenos resultados en acné conglobata. La isotretinoina oral se utiliza en acné conglobata y nódulo
quístico, pero su toxicidad es muy alta. Tratamiento psiquiátrico: La dismorfofobia es una entidad que se asocia con frecuencia en el
acné y consiste en la percepción exagerada de los padecimientos cutáneos. Los pacientes se preocupan por su “fealdad imaginaria”.
Esto lleva a depresión clínica y a un trastorno obsesivo compulsivo o fobia social.

CASO CLINICO
Paciente de 15 años de edad, de sexo masculino estudiante, que consulta por presentar compromiso sistemico, con fiebre, astenia,
mialgias, anorexia y pérdida de peso. El cuadro fue de inicio súbito; presenta lesiones dermatológicas que afectan pecho y espalda.
Estas lesiones son de aspecto granulomatosas que sangran fácilmente, muy secretantes y necrosis dolorosas. Se solicitan análisis de
laboratorio. Los resultados de estos estudios evidenciaron leucocitosis, anemia, eritrosedimentación elevada (VSG), cultivos
bacterianos de secreción y sangre negativos.

PREGUNTA
Cual de las siguientes medidas tiene la menor evidencia dentro del tratamiento de esta entidad?

RESPUESTA
a.- Eritromicina.
b.- Isotretinoína
c.- Dapsona.
d.- Prednisona.

CURSO ENARM CMN SIGLO XXI TEL: 36246001 Pharmed Solutions Institute PÁGINA 411
MANUAL DE TRABAJO DEL CURSO ENARM CMN SIGLO XXI
PÚRPURA DE SCHÖNLEIN-HENOCH (PSH). CIENCIAS BASICAS: Es una vasculitis sistémica de vaso pequeño y curso habitualmente
agudo. Es una vasculitis leucocitoclástica, de mecanismo inmunologico la más común en la infancia. La causa es desconocida. Precedida,
en ocasiones, por una infección de vías respiratorias que sugiere un posible agente infeccioso desencadenante, siendo larga la lista de
patógenos implicados. Otros desencadenantes pueden ser fármacos (penicilina, ampicilina, eritromicina, quinina), alimentos,
exposición al frío o picaduras de insectos. SALUD PUBLICA: Afecta en el 90% de los casos a niños, preferentemente varones (2:1), con
una edad media de 6 años. PATOGENIA: Es una vasculitis mediada por IgA de los pequeños vasos. Hay aumento en la producción de
IgA, aumento de inmunocomplejos circulantes de IgA y depósitos de IgA en las biopsias de piel y de riñón. La lesión renal de la PSH es
indistinguible histopatológicamente de la nefropatía por IgA de la enfermedad de Berger. Ambas pueden producir insuficiencia renal.
DIAGNOSTICO: Usualmente se presenta con la tétrada: púrpura palpable, artritis, dolor abdominal y enfermedad renal. La púrpura
aparece en el 100% de los casos pero en una cuarta parte no es la manifestación inicial. Se caracteriza por lesiones palpables de 2-
10mm, de color rojo-violáceo (90%), que se concentran característicamente en los glúteos y las extremidades inferiores aunque, en
ocasiones, pueden afectar a los brazos y la cara, y rara vez al tronco. La afectación articular (50-85%) se manifiesta habitualmente como
oligoartritis de miembros inferiores, afecta principalmente la articulación del tobillo y la rodilla. Las alteraciones gastrointestinales (50-
70%) se caracterizan por dolor abdominal cólico o anginoso (empeora con la ingesta) que puede acompañarse de náuseas, vómitos,
diarreas, rectorragias y melenas Los síntomas gastrointestinales se deben al edema de la pared intestinal y a la hemoragia propia de la
vasculitis, cuando estas lesiones son muy intensas pueden dar lugar a un infarto intestinal y perforacion. En la gastroscopia y/o
colonoscopia se observan petequias, hemorragias o erosiones. Son complicaciones posibles una hemorragia intestinal grave y la
invaginación o perforación intestinal. La enfermedad renal (20-50%), normalmente se presenta como una glomerulonefritis moderada
que cursa con hematuria, habitualmente microscópica, y con menos frecuencia proteinuria leve y raramente como síndrome nefrítico,
nefrótico y/o insuficiencia renal. En los adultos hay un mayor riesgo de afectación renal crónica que puede llegar a insuficiencia renal
terminal. Aunque se trata de una vasculitis autolimitada hay recurrencias en la tercera parte de los casos. El diagnóstico es clínico y se
utilizan los criterios del American College of Rheumatology: 1. Inicio de los primeros síntomas a los 20 años o menos. 2. Lesiones
purpúricas sobreelevadas sin trombocitopenia. 3. Dolor abdominal anginoso (empeora con comidas), vómitos, diarreas, rectorragias y
melenas. 4. Biopsia cutánea: neutrófilos peri o extravasculares, en arteriola y/o vénula. 5 Biopsia cutánea: neutrófilos en pared vascular
de arteriola y/o vénula. La presencia de ≥ 2 criterios clasifica de PSH con sensibilidad: 87,1% y especificidad: 87,7%. Las crioglobulinas,
FR, ANA y ANCA suelen ser negativos o positivos a títulos bajos. En los adultos la biopsia de piel y el control de la posible afectación
renal son obligatorios. La biopsia cutánea muestra una vasculitis leucocitoclástica que afecta a los pequeños vasos (arteriolas, vénulas y
capilares). El hallazgo más característico es la infiltración de neutrófilos de los pequeños vasos de la dermis, con frecuente
fragmentación del núcleo de los mismos. Estudios mediante inmunofluorescencia muestran depósitos granulares de IgA y, en menores
cantidades, C3 y fibrina en la pared de los vasos. La microscopia renal convencional suele mostrar una proliferación mesangial aislada y
sólo en casos graves una glomerulonefritis con semilunas. TRATAMIENTO: Se resuelve espontáneamente en el 94% de los niños y en el
89% de los adultos, por lo que el primer objetivo es tranquilizar al paciente y utilizar tratamiento sintomático para la afectación
articular y el dolor abdominal. El tratamiento con corticoides (1-2mg/kg) es controvertido. Una revisión sistemática indica que los
corticoides reducen la duración del dolor abdominal y disminuyen el riesgo de invaginación intestinal, afectación renal y recurrencias.
Su uso en la PSH no complicada no mejoró el curso de la enfermedad, según un ensayo clínico8. La afectación renal grave precisaría de
corticoides, inmunosupresores y/o plasmaféresis. Lo único que tiende la cronicidad es la nefritis.

CASO CLINICO
Varón de 7 años de edad ingresado por presentar un cuadro de dolor abdominal difuso y una deposición melénica. La semana previa
había consultado por varicela y había tomado ibuprofeno, 20 mg/kg/día. No presentaba antecedentes de interés, salvo dolor abdominal
recurrente. En la exploración al ingreso no se objetivaron hallazgos patológicos. Al ingreso la coagulación era normal así como el
sistemático de sangre. Al noveno día apareció un exantema eritematoso violáceo palpable, simétrico, en extremidades inferiores, y se
estableció el diagnóstico púrpura de Schönlein-Henoch.

PREGUNTA
Cual el predominio vascular primordial en esta patologia.

RESPUESTA
a.- Vasculitis de grandes vasos.
b.- Vasculitis de vasos medianos.
c.- Vasculitis de pequeños vasos.
d.- Vasculitis asociadas a conectinopatias.

CASO CLINICO
Niño de 6 años de edad, sin antecedentes personales ni familiares de interés, que fue valorado en el servicio de urgencias por
presentar, desde hacía 24 h, lesiones cutáneas maculopapulosas y purpúricas distribuidas simétricamente por las nalgas, miembros
inferiores y superiores asociadas a signos inflamatorios en las rodillas y tobillos, muy indicativo de púrpura de Schönlein-Henoch. Los
análisis de orina, de función renal y la presión arterial eran normales. Se inició tratamiento con antiinflamatorios no esteroides y se dio
de alta.

PREGUNTA
Considerando el cuadro clínico cual de las manifestaciones es la menos frecuente para orientar el diagnostico de PSH.

RESPUESTA
a.- Fiebre prolongada de origen desconocido.
b.- Lesiones cutáneas sugestivas.

CURSO ENARM CMN SIGLO XXI TEL: 36246001 Pharmed Solutions Institute PÁGINA 412
MANUAL DE TRABAJO DEL CURSO ENARM CMN SIGLO XXI
c.- Artralgias, artritis, miositis y serositis.
d.- Parametros de laboratorio de inflamación.

PREGUNTA
Cual de los siguientes factores etiológicos puede estar más relacionado con esta patologia.

a. Estreptococo del grupo A beta hemolítico o M. pneumoniae,


b.- Virus de hepatitis A, citomegalovirus ó virus de Ebstein Barr.
c.- Vacuna del sarampión, influenza, rubéola, neumococo.
d.- Antibióticos betalactámicos, macrólidos ó AINES.

PREGUNTA
Cual de las siguientes manifestaciones es la menos frecuente para hacer el diagnostico.

RESPUESTA
a.- Manifestaciones cutáneas.
b.- Manifestaciones digestivas.
c.- Manifestaciones articulares.
d.- Manifestaciones renales.

ANEMIAS. CIENCIAS BASICAS: Es la disminución de la hemoglobina o el hematocrito, por debajo de los valores normales de la localidad
y de acuerdo a la edad y sexo, con la consecuente hipoperfusión tisular de oxígeno. A través de la BH (Hb, Hto, reticulocitos, VCM,
CHCM), se obtienen los valores que indicaran si corresponde a un síndrome anémico. SALUD PÚBLICA: En México 67% de los menores
de 2 años presentan ferropenia y en edad escolar 34%. En escolares y adolescentes la falta de una ingesta adecuada de carne de res y
en el caso de las mujeres con la menstruación, favorecen esta entidad. De 2-11% de los niños menores de 12 años de edad presentan
deficiencia de folatos y aquellos que ingieren vegetales verdes solo 0.22%. PATOGENIA: La anemia puede deberse a falta de
producción (aplasia, nutricional), perdida (hemorragia) o destrucción de los eritrocitos (hemolisis). DIAGNOSTICO: En el RN a término
que presenta anemia al nacimiento o durante los primeros días de vida, clínicamente puede “lucir bien” y ser un hallazgo por estudio
rutinario de H o presentar manifestaciones de leves a severas: “lucir mal”, palido con ingesta inadecuada de leche, irritable. En los
lactantes y preescolares puede observarse palidez, irritabilidad, somnolencia, astenia, menor incremento ponderal, infecciones de
repetición y en algunos casos pica o geofagia. En escolares o adolescentes puede manifestarse además cefalea: acufenos, fosfenos,
lipotimia, palpitaciones, disnea, parestesias, calambres. La palidez como dato objetivo de anemia, es valido en los lechos ungueales, ya
que la mucosa oral o la conjuntiva pueden manifestar otras alteraciones que la eclipsan y la palidez global puede ser expresión de otras
alteraciones como estrés coartación aórtica, estado de choque. Se puede encontrar de acuerdo a la intensidad y cronicidad de la
anemia, manifestaciones de insuficiencia cardiaca como: taquicardia, diferencial amplia en la tensión arterial, edema de miembros
inferiores, hepatomegalia. La ruta diagnostica inicia al determinar el porcentaje de reticulocitos corregidos: arregenerativa o falta en la
producción cunado están por debajo de 3% y regenerativa cuando están por arriba de 3%. Habitualmente las segundas tienen cifras pór
arriba de este porcentaje y es debida a hemorragia aguda o hemolisis. ANEMIA FERROPENICA: La mayor causa por falta de producción
de eritrocitos, en la mayor parte de los países es la ferropenia. Se inicia con la depleción de eritrocitos de hierro (ferropenia latente),
pasa por eritropoyesis ferropénica (inicialmente microcitosis y posteriormente hipocromía) y termina con la anemia. En el neonato este
evento es poco frecuente porque la reserva de hierro es directamente proporcional a su peso corporal. El prematuro no recibe la
cantidad de hierro suficiente de Fe y debe reponerse el mismo en las primeras semanas. Las complicaciones del embarazo que originan
perdidas sanguíneas crónicas, disminuyen la dotación de hierro al neonato, como la hemorragia vaginal, amniocentesis traumática,
varices aneurisma del cordón umbilical, inserción velamentosa, placenta previa, desprendimiento prematuro de placenta. ANEMIA
MEGALOBLASTICA: Hay una eritropoyesis ineficaz, puede deberse a deficiencia nutricional por deficiencia de folatos y de vitamina B12.
En el prematuro la deficiencia se presenta en el primer mes de nacido y requiere reposición de este elemento. Las reservas de folatos
se mantienen por 2 semanas en cualquier etapa del desarrollo, la ingesta de vegetales verdes mantiene estos niveles, por lo que los
lactantes alimentados solo con leche, desarrollan esta deficiencia. Algunos medicamentos interfieren con el metabolismo de los folatos
como los barbitúricos, difenilhidantoina, TMP/SFX, excepcionalmente pueden llevar a anemia megaloblastica. En el caso de B12 el
hígado almacena hasta por 6 meses, por lo que es raro observar anemia por este elemento. ANEMIAS CONGENITAS: Como la aciduria
orótica hereditaria y la anemia diseritropoyética hereditaria se presentan excepcionalmente. En los RN si hay anemia normocitica,
pudiese corresponder a infecciones de tipo congénito como la rubeola, parvovirus B19, CMV, toxoplasmosis, ingesta demediacmentos
durante el embarazo o bien aplasia pura de la serie roja congénita. En el resto de las edades pediátricas debe descartarse aplasia pura
de la serie roja adquirida, enfermedades crónicas como colagenopatias, endocrinopatías, infección o insuficiencia renal. Anemia
arregenerativa: DFe acuerdo a las características morfológicas de los eritrocitos, se estudia por la presencia de hipocromía, macrocitosis
o normocitosis. DISQUERATOSIS CONGÉNITA: (síndrome de Zinsser -Cole-Engman) es una enfermedad hereditaria caracterizada por
hiperpigmentación de la piel, leucoplasia de las membranas mucosas y distrofia ungueal, que en aproximadamente la mitad de los
casos se acompaña de pancitopenia con hipoplasia medular. Se ha demostrado que esta enfermedad se transmite en forma recesiva
ligada al cromosoma X, habiéndose observado en algunas madres portadoras, lesiones de leucoplasia en la mucosa bucal. También se
han descrito otros probables patrones de transmisión que incluyen la herencia dominante ligada al cromosoma X y autosómica
dominante. Lo anterior marca una diferencia con la anemia de Fanconi que es una enfermedad heredada con carácter autosómico
recesivo y que además se presenta asociada a múltiples alteraciones esqueléticas y cromosómicas. ANEMIAS HEMOLÍTICAS
HEREDITARIAS: la alteración en el eritrocito se puede presentar a diferentes niveles; a. Defectos en la membrana del eritrocito. La
forma más frecuente es la esferocitosis hereditaria, la cual ocupa el primer lugar entre las anemias hemolíticas en México. Otros
variantes incluyen la eliptocitosis, ovalocitosis y la estomatocitosis hereditaria. b. Capacidad limitada para la síntesis de las cadenas
normales de la hemoglobina. En este grupo se incluyen las talasemias y las alteraciones en la secuencia de los aminoácidos de las

CURSO ENARM CMN SIGLO XXI TEL: 36246001 Pharmed Solutions Institute PÁGINA 413
MANUAL DE TRABAJO DEL CURSO ENARM CMN SIGLO XXI
cadenas polipeptídicas lo cual determina un gran número de variantes moleculares; las más frecuentes de estas variantes son la
hemoglobinopatía S o anemia africana (ocupa el segundo lugar en frecuencia dentro de las anemias hemolíticas hereditarias en
México), la hemoglobina C, D y E. c. Deficiencias enzimáticas de los eritrocitos. Incluyen principalmente las deficiencias de
piruvatoquinasa y de glucosa-6-fosfato deshidrogenasa. ANEMIA HEMOLÍTICA AUTOINMUNE: Destrucción exagerada de eritrocitos,
producida por una actividad inmune aberrante que se dirige contra los glóbulos rojos del propio huésped. Clínicamente puede ser
clasificada en primaria o secundaria, de acuerdo a si se halla asociada o no a otros estados patológicos. Estos últimos pueden incluir
infecciones virales o bacterianas, enfermedades asociadas con producción de autoanticuerpos, síndromes de inmunodeficiencia y
neoplasias. Se desconoce el mecanismo por el cual el organismo forma anticuerpos contra sus propios glóbulos rojos, pero sí se
conocen los mecanismos inmunológicos capaces de destruir los glóbulos rojos in vivo. La anemia hemolítica autoinmune primaria
(AHAIP) se caracteriza por la presencia de anticuerpos capaces de actuar contra los eritrocitos del propio paciente. Los anticuerpos han
sido caracterizados como inmunoglobulinas G(IgG) o M(IgM). Éstas actúan directamente contra los antígenos de la membrana del
eritrocito o bien, mediante la formación de complejos inmunes dirigidos contra ella. En otros casos, la lisis de la membrana eritrocitaria
se produce por activación del sistema del complemento. Habitualmente el inicio es agudo con descenso rápido en los niveles de
hemoglobina. Asimismo, la mayor parte de los pacientes presentan periodos breves de evolución, menores de tres a seis meses. Clínica;
decaimiento, anorexia, palidez, dolor abdominal e ictericia de intensificación progresiva; puede ocurrir además emisión de orina de
color pardo obscuro a consecuencia de la hemoglobinuria. A la exploración física puede observarse hepatomegalia y esplenomegalia de
intensidad variable. Con menos frecuencia se manifiestan fiebre, tendencia hemorrágica, disnea, taquicardia y linfadenopatías. Los
hallazgos de laboratorio característicos de los pacientes con AHAIP: anemia, macrocitosis, reticulocitosis, hiperbilirrubinemia indirecta y
concentración de hemoglobina libre en plasma en valores superiores al normal. TRATAMIENTO: Anemia ferropenica sulfato ferroso:
20mg/kg/día, fumarato ferroso 15mg/kg/día, el tiempo de administración va en relación a corregir la anemia, y completar las reservas,
se puede uno guiar con la BH y suspender cuando ha desaparecido la microcitosis o bien completarse por 6 meses. Excepcionalmente
se requiere la administración por vía parenteral y solo cuando existe intolerancia gástrica. Anemia por deficiencia de folatos; ácido
fólico 1 mg por VO, por un mes. Anemia por deficiencia de B12, es conveniente su administración por vía parenteral, ya que la
absorción intestinal es insuficiente, cuando hay deficiencia. Se indica 100mg IM al día por 10 días. En la anemia hemolítica congénita el
consumo exagerado de folatos por la reprodccion celular incrementada, requiere mantener niveles adecuados de este elemento. Se
administra ácido fólico a 1mg/día VO por día. Anemia hemolitca autoinmunitaria; se indican folatos por la misma razón que en la
congénita y esteroides por vía endovenosa si hay crisis hemolítica o por VO prednisona, iniciando 2 mg/kg/dia, con disminución
paulatina cada 2 semanas, hasta completar 2 meses de tratamiento. Tratamiento quirúrgico; en el caso de talasemia o esferocitosis
hereditaria, que cursen con esplenomegalioa o hemolisis intensa, está indicada la esplenectomía, a partir de los 4 años de edad.
Tratamiento nutricional; en las anemias carenciales es necesario realizar cambios en los hábitos alimenticios, como el inicio temprano
de la ablactación, con frutas, verduras y carne de res por lo menos 4 días de la semana y en especial las mujeres adolescentes por las
pérdidas sanguíneas debidas a la menstruación. El trasplante alogenico de medula ósea está indicado en anemia hemolítica congénita,
ya que permite la curación en la mayor parte de los casos, principalmente en pacientes con alfa talasemia, beta talasemia mayor y
anemia falciforme.

CASO CLINICO
Lactante de 2 años von 6 meses con letargia, hiporexia, retraso ponderoestatural e involución psicomotriz (hipotonía, irritabilidad y
pérdida de sonrisa social) de un mes de evolución. Como antecedentes personales destacaban un crecimiento intrauterino retardado
disarmónico, una serología materna positiva para lúes durante la gestación y anemia perniciosa mal controlada. No había
consanguinidad. La alimentación era con lactancia materna exclusiva. En la exploración física presentaba un estado general regular,
hipoperfusión periférica, palidez e ictericia mucocutáneas, un soplo sistólico eyectivo, taquicardia, papilitis lingual, aftas palatinas.

PREGUNTA
Considerando el cuadro clínico cual de las manifestaciones es la menos importante para el diagnostico.

RESPUESTA
a.- Antecedentes personales.
b.- Antecedentes maternos.
c.- Soplo sistólico eyectivo.
d.- Aftas palatinas.

PREGUNTA
Cuales son las manifestación para hacer el diagnostico diferencial para anemia ferropenica.

RESPUESTA
a.- Volumen corpuscular medio disminuido.
b.- Hemoglobina corpuscular media normal.
c.- Amplitud de la distribución eritrocitaria disminuido.
d.- Recuento de plaquetas mormales o aumentadas.

PREGUNTA
Cual no es una indicación para referir al paciente antes referido.

RESPUESTA
a.- Ferritina serica no concluyente.
b.- Perfiel de hierro no concluyente.

CURSO ENARM CMN SIGLO XXI TEL: 36246001 Pharmed Solutions Institute PÁGINA 414
MANUAL DE TRABAJO DEL CURSO ENARM CMN SIGLO XXI
c.- Paciente con buen apego que no responde.
d.- Pérdida sanguínea aguda.

LEUCEMIAS. CIENCIAS BASICAS: La leucemia aguda es el padecimiento más frecuente en pediatría, no solo México, sino en todo el
mundo. Es un trastorno genético de las células hematopoyéticas. La característica principal de este padecimiento, es la proliferación de
la célula más primitiva de la medula ósea, que puede ser desde la más indiferenciada hasta la mayor grado de diferenciación, proviene
tanto de linfocitos B como T. Las leucemias crónicas son mucho más comunes en los adultos que en los niños. Suelen crecer más
lentamente que las leucemias agudas, aunque también son más difíciles de curar. Las leucemias crónicas también se pueden dividir en
dos tipos. SALUD PUBLICA: En el niño existen varios tipos de leucemias siendo la más frecuente leucemia aguda linfoblastica (LAL),
comprende más de 65% de los casos, le sigue la leucemia mieloblastica aguda (LAM) en un 25% frecuencia (es la responsable del 30%
de las muertes por leucemia en la edad pediátrica) y la leucemia granulocitica crónica en menos de 5% de los casos. Se estima
alrededor de 1 caso por 2880 niños sanos, en pacientes con síndrome de Down la frecuencia es de 1 en 95 casos. La OMS señala que en
México es más frecuente en el sexo femenino. El pico máximo de frecuencia es el alrededor de los 3-5 años de edad. PATOGENIA: La
etiología de esta enfermedad es desconocida, existen factores predisponentes como: radiación (in útero, con las placas simples de
radiografía), anomalías cromosómicas (Sx. Bloom, anemia de Fanconi, Sx de Down), otras entidades (Sx. de Poland, de Rubistein-Taybi,
neurofibromatosis), fármacos (cloranfenicol, agentes citotoxicos), insecticidas, la frecuencia de leucemia aguda es mayor en los
familiares de pacientes con LA. Las células leucémicas se pueden reproducir rápidamente, y puede que no mueran cuando deberían
hacerlo, sino que sobreviven y se acumulan en la médula ósea, desplazando a las células normales. Un tipo común de anomalía del ADN
que pueden dar lugar a la leucemia se conoce como translocación cromosómica. El ADN humano está empacado en 23 pares de
cromosomas. En una translocación, el ADN de un cromosoma se desprende y se une a un cromosoma diferente. El punto en el
cromosoma donde ocurre el desprendimiento puede afectar
los oncogenes o los genes supresores de tumores. Por
ejemplo, una translocación vista en casi todos los casos de
leucemia mieloide crónica (CML) infantil y en algunos casos
de leucemia linfocítica aguda (ALL) infantil es el intercambio
de ADN entre los cromosomas 9 y 22, lo que conduce a lo
que se conoce como cromosoma Philadelphia. Esto crea un
oncogén conocido como BCR-ABL. También se han
descubierto en leucemias infantiles muchos otros cambios
en cromosomas o en genes específicos. Algunos niños
heredan mutaciones del ADN de uno de sus padres que
pueden aumentar su riesgo de desarrollar cáncer.
CLASIFICACION: Citomorfológica, la clasificación Franco-
americana-británica (FAB) reconoce 3 tipos morfológicos: L1;
tamaño célula pequeño, forma del nucléolo regular,
nucléolos ausentes y relación núcleo-citoplasma es elevada.
L2; población celular más heterogénea e irregular, la relación
núcleo-citoplasma es mayor y a menudo se perciben
nucléolos. L3; células morfológicamente idénticas a las de
linfoma de Burkit, grandes, homogéneas con nucléolo
redondo y ovalado, así como presencia de vacuolas. En los
niños L1 80-85%, L2 15% y L3 menos de 3%. Clñasificacion
tipo celular afectado: Leucemia linfocítica crónica (LLC).
Afecta a las células linfoides y es por lo general de
crecimiento lento. Hay más de 15 000 casos nuevos de
leucemia cada año. A menudo, las personas que son
diagnosticadas con esta enfermedad son mayores de 55
años. Casi nunca afecta a niños. Leucemia mieloide crónica
(LMC). Afecta a células mieloides y por lo general es de
crecimiento lento al principio. Hay aproximadamente 5 000 casos nuevos de leucemia cada año. Afecta principalmente a adultos.
Leucemia linfocítica (linfoblástica) aguda (LLA). Afecta a células linfoides y es de crecimiento rápido. Hay más de 5 000 casos nuevos de
leucemia cada año. La LLA es el tipo de leucemia más común entre niños pequeños. También afecta a adultos. Leucemia mieloide aguda
(LMA). Afecta a células mieloides y es de crecimiento rápido. Hay más de 13 000 casos nuevos de leucemia cada año. Afecta tanto a
adultos como a niños. En forma general las LAL son positivas a la reacción de PAS, mientras que las LAM son negativas. La clasificación
inmunológica en que los linfoblastos expresan antígenos correspondientes a las células linfocitarias B o T en distintas fases de
maduración, aproximadamente 80% de las LAL. DIAGNOSTICO: Clínica; Inicio insidioso desde asintomático o presentar síntomas y
signos atribuibles a cuadros virales de vías respiratorias altas, sospecha por descubrimiento incidental en una BH. Más de 50%
presentan síntomas generales como fiebre elevada, anorexia, adinamia y malestar general con palidez de tegumentos. En 30%
manifestaciones de hemorragia como petequias, equimosis y epistaxis. Las manifestaciones de dolor óseo son frecuentes y no siempre
corresponden con los hallazgos radiológicos. Las manifestaciones neurológicas o de tumores es poco frecuente y se ha asociado con
algunos tipos de específicos de leucemias como las LAL de células T. Laboratorio: Presencia de anemia, trombocitopenia y las cifras de
leucocitos pueden ser normales. El estudio de medula ósea por aspiración es la prueba diagnóstica (90%), preferentemente se deberá
efectuar en la espina iliaca posterosuperior, sin embargo se puede realizar en la espina anterosuperior, en general se muestra
celularidad aumentada, con sustitución casi por linfoblastos. Después de realizar los frotis se deberá teñir con tinción de Wright. Si en
la cuenta diferencial existen más de 25% de linfocitos, se establece el diagnostico de leucemia. Además se deben realizar pruebas de
funcionamiento hepático. Otro estudio obligado para el diagnóstico y seguimiento es el de LCR en el que se buscan linfoblastos

CURSO ENARM CMN SIGLO XXI TEL: 36246001 Pharmed Solutions Institute PÁGINA 415
MANUAL DE TRABAJO DEL CURSO ENARM CMN SIGLO XXI
preferentemente, proteínas y glucosa. Es obligado realizar estudio de inmunofenotipo para confirmar el diagnóstico y linaje celular B o
T. TRATAMIENTO: Para LAL comprende 3 fases de tratamiento, va de la inducción a la remisión el objetivo es erradicar la carga de
linfoblastos de la MO, al término de esta fase que dura 28 días, la MO debe encontrase con menos de 5% de blastos, para considerar
que el paciente está en remisión; la segunda fase es de consolidación; la tercera fase es la de mantenimiento en la que se indican
reducciones con la finalidad de que al término de esta no exista enfermedad medible en la MO. La duración dependerá del grupo de
riesgo si es de bajo o alto grado, actualmente para una LAL de alto riesgo es de 30 meses, mientras que opera leucemias de bajo riesgo
es alrededor de 24 meses. La base del tratamiento es la quimioterapia: Algunos de los medicamentos comúnmente usados para tratar
la leucemia en niños incluyen: Vincristina (Oncovin, Daunorubicina, también conocida como daunomicina (Cerubidina), Doxorrubicina
(Adriamycin), Citarabina, también conocida como arabinósido de citosina o ara-C (Citosar), L-asparaginasa (Elspar), PEG-L-asparaginasa
(pegaspargasa, Oncaspar), Etopósido (VePesid, otros). Hay que dar profilaxis al SNC, que se establece con la aplicación de
quimioterapia intratecal. Actualmente se acepta que la indicación para trasplante de MO para el paciente de LAC es en aquellos que
presentan una segunda remisión, esto quiere decir, pacientes que tienen una recaída a la MO; pacientes con factores de riesgo elevado
como las alteraciones citogenéticas del cromosoma Philadelphia t (9,22), t (4,11) entre las más importantes, la indicación de trasplante
debe ser una vez alcanzada la remisión. La cirugía tiene una función muy limitada en el tratamiento de la leucemia en niños. Como las
células leucémicas se propagan por toda la médula ósea y a muchos otros órganos a través de la sangre, no es posible curar este tipo de
cáncer con cirugía.

CASO CLINICO
Niño de 10 años que acude a urgencias por dolor en hombro y codo izquierdos desde hace 4 días, con signos de inflamación en codo
izquierdo. El dolor en hombro izquierdo cede pero a las 24 h aparece dolor en hombro derecho, con limitación para la abducción y
rotación interna. No refiere traumatismos. El día anterior hizo ejercicio (waterpolo). Hace un mes, presentó un episodio de ojo rojo
diagnosticado de conjuntivitis aguda, que cedió con colirio antibiótico. No refiere procesos infecciosos previos. Antecedentes
personales sin interés. En los antecedentes familiares destacan madre y tío materno con espondilitis anquilosante HLA B27 positiva.
Exploración física: Temp. 38°C. Afectación discreta del estado general. Inflamación del codo izquierdo con derrame y limitación
funcional de la flexo-extensión. Impotencia funcional de abducción de hombro derecho. El resto de la exploración es normal.

PREGUNTA
Cual es la conducta a seguir.

RESPUESTA
a.- Realizar BH, QS, EGO.
b.- Realizar pruebas CPK, GPT, GOT, GGT y DHL.
c.- Realizar serología para micoplasma, Borrelia, Yersinia enterocolitica (serotipo O:3), parvovirus, VHA, VHC, RPR con rosa de Bengala.
d.- Realizar artrocentesis e iniciar antibiótico e anti-infamatorio.

PREGUNTA
Se obtiene los siguientes resultados: Hemograma: leucocitos 9.930/mm3 (neutrófilos 53,3%, linfocitos 35,8%, monocitos 6%,
eosinófilos 2,1%), hemoglobina 14,2g/dl, VCM 80,6, plaquetas 308.000/μl, proteína C reactiva 5mg/dl; VSG 26mm/h. ioquímica: CP ,
GPT, GOT y GGT normales, factor reumatoide 14 U/ml; anticuerpos antiestreptolisina (ASLO), inmunoglobulinas y C3 y C4 normales.
Serologías para Micoplasma, Borrelia, Yersinia enterocolitica (serotipo O:3), parvovirus, VHA, VHC, RPR con rosa de Bengala, negativas.
HLA B27 positivo. Radiografías de tórax, codo izquierdo, hombro derecho y sacro-ilíacas, normales. Hemocultivos y cultivo de líquido
articular, negativos.

Fue diagnosticado con artritis reumatoide juvenil, se ajusta tratamiento, cual es la conducta a seguir.

RESPUESTA
a.- Indica ibuprofeno.
b.- Indica prednisona.
c.- Inicia con sulfaxalazina.
d.- Inicia con cloroquina.

PREGUNTA
Tras 6 meses de seguimiento se inicia tratamiento con metotrexato por persistencia de los síntomas. Pasados 10 meses comienza con
astenia y dolor abdominal. En control analítico se objetiva neutropenia (320/μl) y aumento de LDH hasta 588 U/l. Se realiza un nuevo
hemograma por fiebre que objetiva un 36% de blastos. Cual es la conducta diagnostica a seguir.

RESPUESTA
a.- Aspirado de medula osea.
b.- Escaneo con resonancia magnetica.
c.- Frotis y gota gruesa.
d.- Realizar Gamagrama.

RETINOBLASTOMA (RB). CIENCIAS BASICAS: Es un tumor intraocular embrionario de origen retiniano que se presenta generalmente en
niños menores de 5 años. Dejado a su evolución, el retinoblastoma es casi siempre fatal y un retraso en su tratamiento suele suponer
un pronóstico visual precario, de ahí la transcendencia del diagnóstico precoz. SALUD PUBLICA: Representa aprox. El 3% de todos los
canceres presentes en niños menores de 15 años. Su incidencia es de 1:15-20.000 recién nacidos vivos y su etiología esporádica en el

CURSO ENARM CMN SIGLO XXI TEL: 36246001 Pharmed Solutions Institute PÁGINA 416
MANUAL DE TRABAJO DEL CURSO ENARM CMN SIGLO XXI
70-75% de los casos o hereditaria, en el 25-30%. De acuerdo con la Dirección General de Epidemiología de la Secretaría de Salud, podría
representar la neoplasia sólida más frecuente después de los tumores del sistema nervioso central. Generalmente aparece antes de los
2 años de edad y 95% de los casos se diagnostican
antes de los 5 años. PATOGENIA: El gen
responsable, el RB1, está localizado en el
cromosoma 13 y actúa de forma dominante; es
decir, en células donde las dos copias del gen estén
dañadas. El 35-40% de los pacientes corresponden a
casos hereditarios que son portadores de una
mutación germinal. De ellos, más de dos tercios
representan nuevas mutaciones (“mutación de
novo”) sin historia familiar previa. Fenómenos
inflamatorios y glaucoma, ambos secundarios a un tumor que empuja hacia adelante el diafragma cristaliniano o a células tumorales
que azolvan la malla trabecular. Los sitios afectados más frecuentes por metástasis son el SNC y el hueso (60%), dentro de la afecciono
sea el sitio más frecuente fue la órbita y huesos craneales (90%), huesoso largos (18%). DIAGNOSTICO: Se presenta clínicamente con
leucocoria 50-60% (reflejo blanco en la pupila), estrabismo y signos inflamatorios, más raramente con glaucoma, celulitis orbital, uveitis
y hemorragia vítrea. El 60% de los RB son unilaterales con una edad media al diagnóstico de 24 meses, mientras que Los RB bilaterales
son menos frecuentes, suelen ser multifocales y de más temprana edad de comienzo (15 meses). En la mayoría de los niños con
tumores bilaterales, ambos ojos están afectados al diagnóstico. Sólo en algunos casos de RB unilateral se desarrolla un tumor
contralateral más tarde. Las características de inicio precoz, bilateralidad y multifocalidad apuntan a un probable origen hereditario del
proceso. Los retinocitomas o retinomas son tumores benignos resultantes de la regresión espontánea de retinoblastoma y que
raramente pueden ser descubiertos en la edad adulta como lesión residual de un RB que cursó de forma subclínica en la infancia. Otros
tumores asociados: Los portadores de la mutación germinal en RB1 tienen un exceso de riesgo de desarrollar otros tumores. La
presencia de tumores extraoculares (Pinealomas, osteosarcomas, sarcomas de tejido blando, melanomas) llamados segundos tumores
primarios, se manifiesta en la adolescencia o al inicio de la edad adulta, siendo el tiempo medio de aparición del segundo tumor 10-13
años. El diagnóstico de RB se establece mediante oftalmoscopia indirecta. La CT, MRI y ecografía sirven para confirmación del
diagnóstico y estadio tumoral. En base a la focalidad tamaño y localización de los tumores existe un estadiaje de Reese-Ellsworth (en
tabla adjunta) que evalúa el pronóstico, I Muy favorable para la conservación de vista. II Favorable para la conservación de vista. III
Posible conservación de vista. IV Desfavorable para la conservación de vista. V Muy desfavorable para la conservación de la vista.
TRATAMIENTO: La elección de la terapia adecuada depende no solo del estadio del tumor sino de factores tales como la focalidad
(unifocal, unilateral multifocal, o bilateral, localización del tumor dentro del ojo y edad del paciente. Las opciones incluyen: Enucleación
está indicado en todos aquellos casos unilaterales que cumplen alguno de los criterios siguientes: (1) Ocupar más de la mitad del ojo.
(2)Extensión al humor vítreo. (3)Desprendimiento total de retina. (4)Crecimiento de nuevos vasos en el iris y (5) Implicación del tumor
en otras estructuras del ojo. La Radioterapia externa incrementa en un 50% la aparición de otros tumores. Sin embargo, está indicado
en RB bilaterales no susceptibles de tratamiento local. Este método se aplica en tumores mayores de 16 mm que se extienden a la
órbita, cercanos a la fóvea, múltiples o cuando se extienden al humor vítreo. La radioterapia local puede ser efectiva en tumores
grandes (>16 mm) no susceptibles a la crioterapia o fotocoagulación. Aunque reduce la probabilidad de aparición de tumores inducidos,
está asociado a retinopatías y papilopatías. Fotocoagulación, este tratamiento se aplica en tumores pequeños (menos de 3 mm de
diámetro y 2 de grosor) que no implican el disco óptico o la mácula. Llega a ser muy efectivo en tumores muy pequeños, con
localización anterior al ecuador y en tumores donde la altura es igual o menor que la mitad del diámetro de la base. Crioterapia, es
efectiva para el tratamiento de tumores de 5mm de diámetro y 3mm de grosor. Habitualmente son necesarias varias sesiones. Es de
creciente interés el empleo de la Quimioterapia sistémica como terapia coadyuvante en niños previamente candidatos a enucleación o
external beal radiation bilateral. El empleo de agentes alquilantes puede incrementar la incidencia de segundos tumores y está
asociado al desarrollo de leucemia no linfoblástica e infertilidad. RETINOBLASTOMA TRILATERAL: Es un síndrome bien reconocido que
se presenta en 5 a 15% de los pacientes con la forma hereditaria del retinoblastoma y que se define mediante la formación de un tumor
neuroblástico de la línea media intracraneal, típicamente hasta más de 20 meses después del diagnóstico del retinoblastoma. Los
pacientes que son asintomáticos al momento del diagnóstico de un tumor intracraneal tienen un pronóstico más favorable que
aquellos pacientes sintomáticos. Considerando que el retinoblastoma trilateral tiene un pronóstico precario, y el corto intervalo entre
el diagnóstico del retinoblastoma y la aparición de la enfermedad trilateral, el uso de exámenes mediante neuroimagenología de rutina
podrían, potencialmente, detectar la mayoría de los casos dentro de los primeros 2 años del diagnóstico inicial. Ya que no es claro si el
diagnóstico temprano puede incidir en la supervivencia, la frecuencia de los exámenes de detección con imágenes de resonancia
magnética se recomienda para aquellos presuntos de tener una enfermedad hereditaria o para aquellos con enfermedad unilateral y
antecedentes familiares positivos, con una frecuencia de cada 6 meses por 5 años. No es claro si esto tendrá un efecto en el desenlace
o en la supervivencia. Se deben evitar las exploraciones por tomografía computarizada como métodos de detección de rutina en estos
niños debido a la percepción de riesgo sobre la exposición con radiación ionizante.

CASO CLINICO
Niña de 7 meses, previamente sana, presenta leucocoria de 2 meses de evolución, no hay presencia de dolor ocular ni síntomas
concomitantes. Al examen oftalmológico se constata ausencia de fijación y esotropia del ojo derecho con ausencia de reflejo rojo.

PREGUNTA
Cual de los siguientes diagnosticos diferenciales es el mas frecuente?

RESPUESTA
a.- Enfermedad de Coats.
b.- Catarata.

CURSO ENARM CMN SIGLO XXI TEL: 36246001 Pharmed Solutions Institute PÁGINA 417
MANUAL DE TRABAJO DEL CURSO ENARM CMN SIGLO XXI
c.- Persistencia de vítreo primario hiperplásico.
d.- Estadio V de la retinopatía de la prematuridad

TUMORES DEL SNC. CIENCIAS: Representan la segunda causa de neoplasias en la infancia. Los tumores cerebrales primarios son un
grupo diverso de enfermedades que, constituyen el tumor solido más común de la niñez. Los tumores más frecuentes son tres:
astrocitoma cerebeloso (33.9%), el meduloblastoma (26.3%) y el glioma del tronco cerebral. Otros tumores importantes de considerar
son el craneofaringeoma, tumeor de la región pineal y ependimoma. SALUD PUBLICA: En México no existen estadísticas, pero se estima
12%. En el INP, estos tumores ocupan el tercer lugar. Los tumores del sistema nervioso central (SNC) constituyen la segunda causa de
muerte en los menores de 15 años, tan sólo superada por la leucemia. PATOGENIA: Se desconoce la causa de los tumores cerebrales
infantiles pero existe una serie de factores predisponentes como: 1.- Genéticos.- Neurofibromatosis, Esclerosis Tuberosa , Enfermedad
de von Hippel-Lindau, Síndrome de Li-Fraumeni, Síndrome de Turcot y el Síndrome de Nevo con Carcinoma de Células Basales, este
último y aquellos que cursan con Tumor de Wilms son más propensos a desarrollar meduloblastoma. 2.- Inmunológicos.-
Inmunosupresión congénita o adquirida, individuos que serán sometidos a transplante renal ya sea antes o durante el mismo tienen
350 veces más riesgo de desarrollar Sarcoma de células reticulares y Linfomas, los pacientes con ataxia-Telangiectasia cursan con
alteraciones inmunológicas y mayor riesgo de desarrollar neoplasias. 3.- Ambientales.- Exposición a Hidrocarburos Aromáticos,
compuestos N-nitrosos, triazinas e hidrazinas. Una asociación menos documentada es el consumo, durante el embarazo de
barbitúricos, exposición prenatal a rayos X, trauma, infección y anestésicos. El cáncer se produce por mutaciones de genes que regulan
la proliferación y muerte celular. Las mutaciones genéticas pueden ocurrir dentro de la línea germinal o como mutaciones somáticas
exclusivamente dentro de células tumorales. Solo una pequeña fracción de niños con Tumores cerebrales tienen mutaciones
germinales adquiridas de sus padres o mutaciones nuevas. La causa de las mutaciones somáticas en la mayoría de todos los tumores
cerebrales son desconocidas. Existe evidencia de que el crecimiento y diseminación de una neoplasia es dependiente de la angiogénesis
(formación de nuevos capilares sanguíneos a partir del propio tumor lo que incrementa las posibilidades de incorporación de células
neoplásicas a la circulación). Cada incremento sucesivo en el volumen del tumor es precedido por una fase de angiogénesis.
CLASIFICACION: De acuerdo a la localización: existe un predominio de los tumores infratentoriales (43.2%), seguido de los tumores
supratentoriales (40.9%), los de médula espinal (4.9%) y de sitios múltiples (11%). Infratentoriales y, de estos, 75% están situados en el
cerebelo o cuarto ventrículo. Entre los tumores infratentoriales comunes (fosa posterior) se encuentran los siguientes: 1.-
ASTROCITOMA CEREBELAR: tumor de mejor pronóstico, generalmente de bajo grado. Existe un tumor específico denominado
astrocitoma Pilocitico Juvenil, de excelente pronostico con terapia quirúrgica. No requiere radioterapia, estos tumores se ubican en los
hemisferios cerebelosos y generalmente son quísticos. La porción tumoral en realidad es un nódulo mural. La cavidad quística visible en
TAC es reactiva. 2.- MEDULOBLASTOMA: forma parte de los tumores neuroectodérmicos primitivos, por originarse de esta célula
primordial, son muy agresivos y por ende de mal pronóstico, puede dar metástasis incluso vía LCR, se origina en la primera década de la
vida en el techo del IV ventrículo, por lo cual puede causar hidrocefalia precoz, clínicamente se presenta con cefalea y vómitos, los
signos serán relacionados con un síndrome cerebeloso, un niño con frecuentes caídas puede cursar una ataxia. Es de los tumores que
con más frecuencia presenta diseminacionm extraneural (menos del 4%), siendo el hueso, médula ósea, linfáticos, hígado y pulmones
los sitios más comunes. Tratamiento quirúrgico, radio y quimioterapia. 3.- EPENDIMOMA: ocurren mas frecuente en la región del torax.
En raras ocasiones presentan malignizacion y en muchas ocasiones presentan asociación con quistes intramedulares. Es un tumor
relativamente avascular y son encapsulados, es decir tienen un plano de clivaje que permite su extirpación completa y curación de los
pacientes. Un grupo de ellos crece en el interior de la medula y otro en el cono medular. 4. GLIOMA DEL TALLO ENCEFÁLICO: puede ser
de distintos grados, en general de mal pronóstico dado su natuiraleza infiltrativa. Puede estar a nivel alto manifestándose por síndrome
cerebeloso e hidrocefalia. Pueden ubicarse a nivel bajo (bulbo medular), manifestándose por compromiso multiple de pares craneales
bajos y por compromiso motor y sensitivo de extremidades. La radioterapia asociada a corticoides, mejoran la calidad de vida. 4. Tumor
teratoide atípico. Los tumores supratentoriales incluyen aquellos que ocurren en la región selar o supraselar del cerebro y en otras
áreas del mismo. Los tumores selares y supraselares representan aprox., el 20% de los tumores cerebrales infantiles, incluso los
siguientes: 1. Craneofaringoma. 2. gliomas diencefalicos. 3 Tumores de células germinales. Otros tumores que ocurren de forma
supratentorial incluyen los siguientes: 1. Astrocitoma o glioma de bajo grado. 2. Astrocitoma de alto grado o maligno. 3. Glioma mixto.
4. Oligodendroglioma. 5. Tumor neuroectodermico primario. DIAGNOSTICO: No existen cuadros patognomónicos para el diagnóstico
de tumores cerebrales. Generalmente el inicio es insidioso y progresivo. Las manifestaciones clínicas de los tumores cerebrales
infantiles depende de: 1.- Localización del tumor (zonas elocuentes, obstrucción o desplazamiento) 2.- Tipo histológico del tumor. 3.-
Edad y desarrollo del paciente. Los tumores intracraneales pueden manifestarse como déficit neurológico focal, crisis convulsivas o no
convulsivas, aumento de la presión intracraneal, que puede deberse a efectos directos del tumor u obstrucción del líquido
cefalorraquídeo. Un análisis de los síntomas tempranos de los tumores cerebrales infantiles mostró que los tumores supratentoriales se
presentan con vómito en el 46 % de los casos, con cefalea en el 43% de los casos; mientras que los Tumores Infratentoriales se
presentan con dificultades para la coordinación en el 59 %, vómito 76% y cefalea 56 %. Debe sospecharse de un tumor cerebral en todo
niño con datos clínicos neurológicos insidiosos y progresivos. DESCRIPCION SINDROMÁTICA: I) Hipertensión Intracraneana: este cuadro
puede manifestarse de manera aguda o crónica: Agudo: < 1 año incremento inesperado del perímetro cefálico, separación de suturas,
alteraciones del estado de alerta (somnolencia, irritabilidad) disminución en la ingesta. En preescolares, escolares y adolescentes puede
manifestarse con cefalea y vómito generalmente matutino que se exacerba con maniobras de Valsava y disminuye en el transcurso del
día. El vómito puede ser en proyectil, irritabilidad, letargia, edema de papila, discromatopsia (pérdida de la visión de colores),
escotomas centrales y la paresia del sexto par, este último inespecífico de topografía y explicado por la longitud del trayecto (falso
focalizador). Crónico o Intermitente: irritabilidad, letargia, vómito, atrofia de papila o pérdida progresiva de la visión, cambios
conductuales, de personalidad, del rendimiento académico, anorexia y pérdida o ganancia ponderal. II) Síndrome cefalálgico: La cefalea
en tumores clásicamente se describe con un curso subagudo de 4 a 6 meses de duración, de tipo pulsátil, continua o intermitente.
Despierta al niño por la noche, frontal, occipital o universal, de moderada intensidad medida por escala análoga visual del dolor, o
porque el niño deja de hacer sus actividades. III) Crisis: Las crisis en los tumores generalmente son focales y pueden asociarse a
patrones bioelétricos anormales focales. IV) Síndrome cerebeloso: Este puede ser vermiano con ataxia troncal o de la marcha,
hemisférico con lateropulsión derecha, izquierda o indistinta, dismetría, disdiadococinesia, lenguaje escándido (disartria), reflejos

CURSO ENARM CMN SIGLO XXI TEL: 36246001 Pharmed Solutions Institute PÁGINA 418
MANUAL DE TRABAJO DEL CURSO ENARM CMN SIGLO XXI
osteotendinosos pendulares con o sin nistagmus horizontal. V) Neuropatía craneal: El involucro de pares craneales dependiendo de
cuál sea, es sugestivo de la topografía de la lesión sin embargo esto puede ser muy sutil en virtud de que los tumores son más
compresivos que destructivos. Entendiendo por neuropatía del III al XII par craneal. Los estudios necesarios para realizar el diagnostico
son la TAC, siendo posible realizar el diagnostico hasta en 90%, la imagen por IRM, mielografía, mielotomografía, en caso de tumor
como el meduloblastoma y ependimoma: siendo el diagnóstico definitivo el estudio histopatológico, obteniéndose a través de
resección quirúrgica, con una toma de biopsia y exéresis del tumor, así como por biopsias por esterotaxia. Existe una ruta crítica para la
evaluación de un niño con sospecha de neoplasia en SNC: Sospecha de tumor primario  estudios de laboratorio y gabinete
interconsulta con neurocirugía y oncología procedimiento diagnostico-quirúrgico  clasificación histopatológica estadificacion
 valoración radioterapia  valorar inicio de esquema de quimioterapia  seguimiento. TRATAMIENTO: Los cánceres dejados a su
evolución natural son incontrolables y mortales en la mayoría de los casos. Se basa en la desaparición de los síntomas, así como el
tratamiento específico, por lo que es importante individualizar cada caso, algunos pacientes requerirán el empleo de anticomisiales
(Fenitoína 20 mg/kg/dosis de impregnación y mantenimiento 7 a 10mg/kg/día en dos dosis), así como de esteroides para el manejo de
hipertensión craneal, ya que algunos autores mencionan que los esteroides tienen efecto vaso génico, aumentando la permeabilidad
vascular, corrigiendo e edema cerebral y por tanto mejorando el flujo circulatorio de forma transitoria. En cuanto al tratamiento
específico, la cirugía debe realizarse por el especialista experto en patología tumoral. En ocasiones los pacientes requieren colocación
de válvulas ventriculoperitoneales para el manejo urgente de la hidrocefalia y posteriormente la cirugía de resección que siempre es
recomendable sea completa, pero suele ser imposible. La radioterapia es una modalidad de tratamiento necesaria para los tumores
malignos de SNC, los pacientes menores de 3 años presentarán detención del crecimiento y la posibilidad de retardo psicomotor,
disfunción endocrina y radionecrosis. La quimioterapia es usada para las neoplasias del SNC, pero no todas son sensibles debido a la
barrera hemartoencefalica

CASO CLINICO
Paciente de 9 años de edad, sexo masculino, que consultó por cuadro de 3 días de evolución caracterizado por cefalea holocránea de
tipo opresivo y vómitos explosivos aproximadamente 6 veces al día. Durante la anamnesis la madre refirió que el paciente presentaba
crisis de cefalea intermitente hace aproximadamente 5 meses, las que con el tiempo fueron aumentando en frecuencia e intensidad
hasta constituir el cuadro de consulta actual. Al examen físico general el niño impresionaba en buenas condiciones generales, y se
apreciaron varias lesiones vesiculares y costrosas en todo el cuerpo, compatibles con cuadro de varicela en sus últimas etapas. Al
examen neurológico destacó paciente con un puntaje de 15 en la Escala de Glasgow, dismetría de extremidad superior derecha y leve
paresia facial izquierda.

PREGUNTA
Cual es la conducta inmediata a seguir mas adecuada?

RESPUESTA
a.- Realizar puncion lumbar.
b.- Realizar tomografía computada.
c.- Medidas anti-edema.
d.- Colocacion de derivación ventrículo peritoneal.

CASO CLINICO
Se trata de RN femenino, nacido por cesarea con 38 SDG por Capurro, apgar 8/9, sin datos de hidrocefalea, perímetro cefálico de 39.5
cm. Fontanelas anterior y posterior ampliar y plenas, con diastasis de la sutura sagital, tono muscular cervical disminuido,
cardiopulmonar sin compromiso. Movilidad expontanea en las cuatro extremidades, discreta hiperreflexia miotatica, babinski
espontaneo bilateral y reflejos primitivos globales disminuido, La TC mostro hidrocefalia supratentorial sin edema transependimario,
dilatación de tercer ventrículo y en la fosa posterior una imagen hiperdensa ovalada de 46 x 42 mm que obliteraba el cuarto ventrículo
en su parte posterior, se reporto una lesión tumora infratentorial probablemente intraaxial glial o extraaxial de origen
mesenquimatoso. La RM reporto imagen hiperdensa en relación con el parénquima cerebral en la región pancerebelosa que infiltraba
el tallo cerebral en el puente, mesencéfalo y lamina cuadrigeminal, que protruia por la hendidura tentorial encefálica y borra las foliar
cerrebelosas. Por su efecto compresivo sobre acueducto de Silvio, además de ocacionar la dilatación ventricular supratentorial con
hidrocefalia obstructiva.

PREGUNTA
Cual es el diagnostico mas probable?

RESPUESTA
a.- Astrocitoma congénito.
b.- Meduloblastoma congénito.
c.- Ependimoma congénito.
d.- Craneofaringeoma congénito.

PAROTIDITIS. CIENCIAS BASICAS: La parotiditis (paperas) es la inflamación de las glándulas salivares sobre todo de las parótidas. Es una
enfermedad generalizada, viral, aguda y altamente contagiosa y que generalmente ocurre en la infancia. Se transmite por contacto
directo e ingresa al organismo por vía respiratoria y usualmente tarda en aparecer de dos a tres semanas. Los casos de paperas en
nuestro medio han disminuido dramáticamente con la introducción de la vacuna contra las paperas. SALUD PUBLICA: Es una
enfermedad endemo-epidémica, con períodos de aparición que oscilan entre 2 y 6 años. Se estima que en ausencia de inmunización, el
85% de los adultos ha sufrido la enfermedad; una tercera parte de ellos sin síntomas evidentes (la mayoría antes de los dos años de

CURSO ENARM CMN SIGLO XXI TEL: 36246001 Pharmed Solutions Institute PÁGINA 419
MANUAL DE TRABAJO DEL CURSO ENARM CMN SIGLO XXI
vida). Se presenta en invierno y primavera. La afectación ha disminuido en los países que incluyeron la vacuna en forma rutinaria en sus
planes de inmunización. PATOGENIA: El agente es un paramyxovirus (emparentado con los de influenza y parainfluenza), el virios esta
formado por RNA, el virus penetra por una via respiratoria. Tiene como reservorio al hombre y el modo de transmisión es persona a
persona (diseminación de gotitas y saliva del infectado). La incubación dura de 12 a 25 días (media de 18) y el período de contagio va
desde 6-7días antes y hasta 9 días después del edema glandular (la máxima contagiosidad, ocurre 48 horas antes de las manifestaciones
clínicas de la enfermedad). Los expuestos no inmunes, deben ser considerados infecciosos desde el 12° al 25° día de la exposición. La
inmunidad suele ser permanente. DIAGNOSTICO: Sialoadenitis, la glándula que se afecta casi en forma constante es la parótida, puede
comenzar bruscamente con tumefacción y dolor en la región parotídea, la piel suprayacente aparece tensa, pero no roja y sensible a la
presión, consecutivamente el lóbulo de la oreja es empujado hacia arriba y fuera. Ocurre enrojecimiento y tumefacción del conducto de
Stenon. Se caracteriza por fiebre, edema y dolor de una o más glándulas salivales (habitualmente las parótidas), dificultad para masticar
a veces para hablar, cefalea. Por lo general es leve y autolimitada. Puede ser uni o bilateral (70%), alcanza su máxima expresión al tercer
día remite lentamente en un plazo de tres a siete días. Si bien el compromiso generalmente es parotídeo, con cierta frecuencia las
glándulas submaxilares también están afectadas, con mucho menos frecuencia ocurre lo mismo con las glándulas sublinguales.
Orquiepididimitis se presenta en 20-30% de los varones pospuverales, es la manifestación extrasalival más frecuente, es unilateral y se
instaura al ceder la tumefacción parotídea, con un nuevo ascenso febril COMPLICACIONES: Meningoencefalitis (inflamación de las
membranas que cubren el cerebro y la médula espinal). Es la más frecuente dentro de la infancia. 1 por cada 400 casos, y más a
hombres que a mujeres, en proporción de 1 a 4. Evoluciona en la mayoría de los casos hacia la curación espontánea en un período de
cuatro días, sin dejar, secuelas. Orquitis (inflamación del testículo). Muy poco frecuente en la infancia, en la edad mayor se observa en
un 30% de los casos. Comienza generalmente siete días después de la parotiditis, con fiebre, calofríos, dolor de cabeza y dolor en los
testículos y en la parte baja del abdomen. En un porcentaje bajo es bilateral y en tercio evoluciona hacia la atrofia testicular. Sordera.
Se afecta más severamente la audición de tonos altos observando la recuperación en pocas semanas. Es de baja incidencia: Oforitis,
pancreatitis - inflamación del páncreas. El virus puede aislarse en saliva desde una semana antes hasta 4-5 días después del inicio de la
parotiditis. Se le aísla en cultivos de tejidos humanos o de primates y en huevos embrionados. Hoy en día se prefiere la técnica de
ELISA. La VSG permanece normal excepto en caso de orquitis o pancreatitis. TRATAMIENTO: limitado generalmente a la administración
de medicamentos para aliviar el dolor y la ingestión abundante de líquidos. Algunas veces el reposo en cama es necesario durante los
primeros días. Los niños no deben asistir a la escuela hasta que los síntomas hayan desaparecido, es necesario recomendar líquidos
orales, y descartar los ácidos, limón, naranja etc. La orquitis se tratara con medidas locales y reposo en cama. El tratamiento de la
meningoencefalitis es puramente sintomático con analgésicos. PROFILAXIS: La globulina inmunitaria específica antiparotiditis no
previene la parotiditis y ya no se fabrica. Se dispone de una vacuna elaborada con virus vivos atenuados de la parotiditis que se ha
mostrado muy eficaz (95%) en la profilaxis de la enfermedad. Debe administrarse durante el segundo año de vida o bien antes de la
pubertad.

CASO CLINICO
Paciente varón de 4 años y medio, remitido por su pediatra para estudio por haber presentado, en los últimos 9 meses, cuatro
episodios de tumefacción parotídea derecha, de 3-4 días de evolución cada uno, que cedían con tratamiento antiinflamatorio y
antibiótico. Se asociaba sensación dolorosa de la zona, febrícula y, en una ocasión, supuración mucopurulenta por el conducto de
Stenon. No destacaban antecedentes personales de interés y el calendario vacunal (incluido parotiditis) estaba correcto. La exploración
física en el momento de la consulta (intercrisis) era normal.

PREGUNTA
Cuales son los estudios mas adecuados para buscar un diagnostico especifico.

RESPUESTA
a.- BH, QS, EGO.
b.- Radiografia de cráneo y cara.
c.- Serología para VEB, citomegalovirus, adenovirus y VIH.
d.- Factor reumatoide y anticuerpos antinucleares (ANA, anti-Ro y anti-La).

PREGUNTA
Cual de las siguientes complicaciones es la más frecuente.

RESPUESTA
a.- Meningoencefalitis.
b.- Orquitis/orquiepididimitis.
c.- Pancreatitis.
d.- Miocarditis.

HEPATITIS. CIENCIAS BASICAS: Indica proceso inflamatorio del hígado caracterizado por hepatomegalia, anorexia, molestáis
abdominales fundamentalmente gástricas, función hepática anormal, heces despigmentadas y orina oscura. Estas alteraciones pueden
ser ocasionadas por infecciones producidas por bacterias, virus o parásitos o bien por ingestión excesiva de alcohol o fármacos,
trastornos de tipi autoinmunitario o toxico. La mayor parte de las hepatitis son de origen viral. La hepatitis crónica se define como una
lesión inflamatoria del hígado que puede progresar a lesión mas grave y a cirrosis, las causas más frecuentes son: virales, autoinmunes,
toxicas y criptogenicas. En general se habla de cronicidad con una evolución de más de 6 meses, con excepción del caso de las hepatitis
autoinmunes, en las cuales un diagnóstico más temprano, permitirá detectar el padecimiento antes de que se desarrolle cirrosis. Se
sospecha una hepatitis crónica ante una sospecha de recaída de hepatitis aguda, persistencia de hepatitis aguda por más de 3 meses,
datos clínicos de enfermedad hepática crónica y en pacientes asintomáticos de alto riesgo, con historia de transfusiones múltiples,

CURSO ENARM CMN SIGLO XXI TEL: 36246001 Pharmed Solutions Institute PÁGINA 420
MANUAL DE TRABAJO DEL CURSO ENARM CMN SIGLO XXI
como son pacientes con leucemias en remisión, hemofilia, hemoglobinopatías, IRC en programa de diálisis, y neoplasias en los que
existe el riesgo de hepatitis post- transfusional. SALUD PUBLICA: En México que 95% de las hepatitis en niños menores de 215 años de
edad es producida por el virus de hepatitis A. Cada año se registran aproximadamente 1,4 millones de casos de hepatitis A en todo el
mundo. La hepatitis A se asocia a falta de agua salubre y a un saneamiento deficiente. Las epidemias se pueden propagar de manera
explosiva y causar pérdidas económicas considerables. Las mejoras del saneamiento y la vacuna contra la hepatitis A son las medidas
más eficaces para combatir la enfermedad. HEPATITIS POR VIRUS A: La sección hepática más frecuente, pertenece al grupo de los
picornavirus de los enterovirus, y su huésped natural es el ser humano, el genoma está constituido por RNA. La hepatitis A se presenta
en niños entre los 3-12 años de edad, con promedio en la edad preescolar. La vía de transmisión más común es la del tubo digestivo a
través de alimentos o bebidas que se contaminan con matera fecal que contienen el virus. La contaminación por vía aérea no se ha
confirmado. La transmisión también se da en los drogadictos que comparten agujas y jeringas para uso intravenoso. El mayor riesgo de
diseminación del virus es en heces, ocurre 15 días posteriores al inicio de la enfermedad. El estado de portador no existe. La inmunidad
con la infección es permanente. No se ha informado hasta la fecha evolución a la cronicidad. Clínica; Los niños menores de 2 años de
edad, cursan sin ictericia en 90% de los casos. Antes de la aparición de la ictérica se presentan los siguientes síntomas, febrícula,
anorexia, vomito, debilidad, cefalea, dolor difuso en abdomen y esplenomegalia. En la fase ictérica cuya duración puede ser de 8 días a
4 semanas, se hace aparente coluria, hipocolia, decaimiento y anorexia más acentuadas, con presencia de tinte amarillento en
conjuntivas y piel. Laboratorio: Aumento de la bilirrubinas a expensas de la bilirrubina directa, las transaminasas, se encuentran
elevadas, la colinesterasa y la colestasa están disminuidas, las pruebas de floculación de proteínas como es la turbidez al timol se
encuentra elevada, ocurre aumento de la gammaglobulina con disminución de albumina. Se encuentran alteradas la concentración de
inmunoglobulinas los tiempos de protrombina y la concentración de glucosa. Las pruebas serológicas específicas como es la detección
de anticuerpos contra el virus de la hepatitis A por radioinmunoanálisis, sugieren infección reciente o convaleciente. Tratamiento: Las
medidas son de tipo sintomático, ya que no existe medicamentos específicos para el virus de hepatitis A. Dieta; implica restricción de
proteínas, de grasas y aumento en los hidratos de carbono. Reposo; mínima actividad para mantener un flujo circulatorio esplacnico y
hepático adecuado. Aislamiento y protección a contactos; precauciones con el manejo de excretas durante el periodo ictérico y 2
semanas después. Aplicación de gammaglobulina estándar al 16% a todo contacto a razón de 0.2 a 0.12 ml/kg por vía IM en dosis única
como inmunización pasiva. Inmunización: Debe aplicarse la vacuna contra hepatitis A en presentación pediátrica, dos dosis con
intervalos de 1 mes con refuerzo a los 12 meses, la vacuna disponible en México precede de la cepa HM75 inactivada. Son 360 unidades
en 0.5ml y se aplica vía IM en niños mayores de 1 año de edad. VIRUS DE HEPATITIS D: Se trata de un virus RNA defectuoso, formado
por RNA del agenteᵹ, cubierto por antígeno de superficie del virus de la hepatitis B, sin este virus no resulta infectante. Se transmite por
vía parenteral o vía percutánea, en mucosas, transfusión sanguínea, contacto sexual y etapa perinatal. La diseminación es por
portadores y afecta más al adolescente. Su periodo de incubación es de 15-200 días. El 75% del diagnóstico inicial se hace en la fase
cirrótica y por lo tanto tiende a la fase de cronicidad. En laboratorio a parte de la BH, transaminasas, bilirrubinas y EGO, debe hacerse
cuantificación de anticuerpos antivirus de la hepatitis ᵹ (anti-VHD) en donde se encuentran titulaciones mayores de 1:100, las
titulaciones de IgM se encuentran altas en fase temprana, además se cuantifican anticuerpos del virus de la hepatitis B (Ags-VHB).
Tratamiento: Se ha utilizado el interferón sin obtener resultados satisfactorios. No existe vacuna alguna, sin embargo algunos autores
sugieren usar la vacuna contra el virus de la hepatitis B.

CASO CLINICO
RN pretérmino de 36,6 semanas de EG, peso 2.555 g, por depresión neonatal y riesgo infeccioso. APGAR 5/9. La madre presentó, 24 h
antes del parto, un cuadro febril acompañado de dolor abdominal. Por la sospecha clínica de corioamnionitis recibió tratamiento
antibiótico y se indujo el parto, que finalizó en cesárea con anestesia general por no progresión.

PREGUNTA
Cual es la conducta a seguir mas adecuada.

RESPUESTA
a.- Egresar con vigilancia estrecha.
b.- Hemocultivo e inicio de antibiótico.
c.- BH, QS, EGO.
d.- Analisis de LCR.

PREGUNTA
En las siguientes 48 h se deteriora su estado general, con acidosis metabólica, anemia, plaquetopenia (recuento mínimo 10.000/mcl) y
hemorragia digestiva baja, por lo cual se modifica el tratamiento antibiótico, para cubrir una posible enterocolitis necrozante, y se
suspende la alimentación oral. En las 24 h posteriores desarrolla coagulopatía y fallo hepático; presenta petequias generalizadas y
sangrado importante por puntos de canalización venosa umbilical y periférica. Cual es la conducta a seguir.

RESPUESTA
a.- Transfusión de hemoderivados.
b.- Administración de vitamina K (1 mg/kg/día) durante 72 h.
c.- Pruebas de función hepática.
d.- Administracion de albumina.

PREGUNTA
Se obtienen los siguientes resultados donde alcanza concentraciones máximas de transaminasas GOT 957 U/L, GPT 237 U/L y GGT 144
U/L, FA 93 U/L, LDH 5031 U/L, bilirrubina directa: 1,66 mg/dl, con disminución de las proteínas totales (3,75 g/dl) y albúmina (2,19 g/dl).

CURSO ENARM CMN SIGLO XXI TEL: 36246001 Pharmed Solutions Institute PÁGINA 421
MANUAL DE TRABAJO DEL CURSO ENARM CMN SIGLO XXI
Por la sospecha clínica de viremia, y ante la posibilidad de infección herpética, se inicia tratamiento con aciclovir. Cual de los siguientes
patologias es el mas frecuente evaluar para el diagnostico diferencial.

RESPUESTA
a.- Hepatitis neonatal.
b.- Infeccion por VEB, CMV.
c.- Enterovirus y adenovirus.
d.- Toxoplasma, rubéola y sífilis.

INFLUENZA. CIENCIAS BASICAS: Es una infección viral aguda de las vías respiratorias, altamente contagiosa, que puede afectar la
mucosa nasal, la faríngea, bronquios y en ocasiones hasta los alvéolos pulmonares. Los síntomas son parecidos a los del catarro común
o resfriado; sin embargo, son más graves y su inicio es generalmente abrupto. La gripe es causada por el virus de la influenza (virus de
influenza A, B y C), el catarro común es causado por múltiples virus respiratorios (rinovirus, coronavirus, adenovirus, virus
parainfluenza). El cuadro clínico de la influenza puede variar, observándose desde un cuadro febril agudo leve hasta una infección
pulmonar y/ocomplicaciones graves. SALUD PUBLICA: Puede tener un comportamiento endémico, epidémico o de una pandemia. Las
epidemias de influenza son responsables de 36 000 a 50 000 muertes por año en países como EUA en los últimos años, afecta a todos
los grupos etarios, pero principalmente a niños menores de dos años y adultos mayores de 65 años. PATOGENIA: Los virus asociados a
la influenza humana son miembros de la familia Orthomyxoviridae, tienen una morfología helicoidal y su genoma está constituido por
una cadena segmentada de ARN, los virus influenza A y B causan infecciones respiratorias en humanos y animales, el virus A es causa de
pandemias. El virus de influenza C afecta en forma moderada a los niños y en forma ocasional a los adultos. Estos virus se distinguen
por variaciones antigénicas en dos proteínas estructurales (la nucleoproteína y la proteína de la matriz). Los virus de la influenza A se
clasifican en subtipos de acuerdo a dos antígenos de superficie: hemaglutinina (H) y neuraminidasa (N). La hemaglutinina es
considerada el antígeno mayor para la cual está dirigida la producción de anticuerpos neutralizantes y cuya función es la adhesión del
virus mediante residuos de ácido siálico en la superficie del epitelio respiratorio humano. La expresión de la neuraminidasa es menos
abundante en la superficie viral y su papel es facilitar la liberación de viriones de células infectadas del hospedero. Para la clasificación
de este virus se utiliza el siguiente orden en la nomenclatura: el tipo, el lugar en donde fue aislado, el año de aislamiento, el número de
identificación del laboratorio y, en los aislamientos del virus de la influenza A, el subtipo H-N (por ejemplo: A/Moscow/10/99 [H3N2]).
Estudios de epidemiología molecular de los virus de la influenza han demostrado que los subtipos H1N1, H1N2, y H3N2 son los que han
circulado en los últimos años. Las aves constituyen un reservorio potencial de intercambio genético para los virus de la influenza, lo que
representa un riesgo latente de pandemias. Los virus de la influenza A también infectan a pollos, cerdos, caballos y ocasionalmente a
mamíferos marinos. El periodo de incubación es de 1 a 4 días (promedio de dos días). Los adultos pueden ser infectantes un día antes
de que los síntomas inicien hasta aproximadamente 3-5 días después. Los niños pueden ser infectantes por 10 días o más. Las personas
con estados de inmunosupresión pueden excretar los virus por períodos más prolongados. El virus de la influenza A se caracteriza por
causar enfermedad moderada a grave. El virus B de la influenza causa cuadros clínicos menos graves que el tipo A y tradicionalmente
produce infección y enfermedad durante la infancia. La capacidad del virus de la influenza A y B de sufrir cambios antigénicos graduales
en sus dos antígenos de superficie, la hemaglutinina y la neuraminidasa, complica la vacunación contra esta enfermedad. El tipo de
variaciones antigénicas menores o deslizamientos antigénicos (antigenic drift) resulta de la acumulación de mutaciones puntuales de
los genes que transcriben para estas proteínas. DIAGNOSTICO: La influenza se caracteriza por el inicio agudo de síntomas y signos
respiratorios que incluyen: fiebre (elevada de inicio abrupto), tos seca, coriza, cefalea, odinofagia, ardor faríngeo, mialgias, artralgias, y
ataque importante al estado general (postración H3N2). Estos síntomas ocurren en 50 a 70% de las infecciones por influenza, tanto
estacional como por el virus de influenza humana H1N1 2009. Menos comunes son: fotofobia, dolor abdominal, náusea, vómito y
diarrea. La duración de la enfermedad sin complicaciones habitualmente es de una semana, aunque la tos y debilidad pueden persistir
por más de 14 días. En particular, en los pacientes con infección por el virus A H1N1, se presentaron durante la epidemia cuadros
graves de neumonía intersticial y progresión a pulmón de choque. Para la infección por el virus influenza A humana H1N1 2009, se
establecieron definiciones epidemiológicas para la confirmación de los casos: los confirmados son aquellos que tienen síntomas y
signos arriba mencionados y una prueba confirmatoria positiva que consiste en reacción de polimerasa en cadena en transcripción
reversa (RT-PCR) en tiempo real o cultivo viral. Un caso probable es una persona sintomática que es positiva para influenza A por
prueba rápida, pero negativa para H1 y H3 por RT-PCR. Una persona sospechosa es aquella que no cumple con la definición de probable
o confirmado, no tiene prueba negativa para el virus nuevo, y: Es un individuo previamente sano, menor de 65 años de edad,
hospitalizado por infección respiratoria baja. Vive en un lugar o estado donde no hay casos confirmados pero viajó recientemente a
donde existe uno o más casos probables o confirmados. Tuvo contacto en los siete días previos con un caso confirmado o probable.
Laboratorio: Idealmente, la muestra de exudado faríngeo, nasofaríngeo, lavado nasal, aspirado bronquial o traqueal debe tomarse en
los primeros tres días del inicio de los síntomas. El cultivo viral es el estándar de oro, además permite que el virus sea tipificado y
caracterizado antigénicamente. Los medios de cultivo utilizados son huevos embrionados de gallina o el cultivo de riñón canino de
Madin-Darby, el riñón de chimpancé y otros. También se puede realizar el diagnóstico por determinaciones serológicas al mostrar un
incremento de cuatro veces en la titulación de anticuerpos contra influenza en una segunda muestra. El suero en la fase de
convalecencia debe obtenerse entre los días 10 a 21 del inicio del cuadro. Las técnicas serológicas más frecuentemente empleadas son
las de neutralización y la inhibición por hemaglutinación. COMPLICACIONES: Neumonía primaria por influenza o neumonía bacteriana
secundaria. La etiología más frecuente de la neumonía bacteriana es: Streptococcus pneumoniae en 48% de los casos, Staphylococcus
aureus en 19%, y Haemophilus influenzae no tipificable en 11%. Otras complicaciones del tracto respiratorio incluyen: sinusitis
bacteriana, bronquitis, traqueobronquitis y otitis media. En niños se ha asociado a otras manifestaciones clínicas como son:
convulsiones febriles, encefalopatía por el virus o encefalopatía asociada a la utilización de salicilatos (síndrome de Reye), miositis,
rabdomiólisis, miocarditis y pericarditis. En pocos casos se informa de complicaciones en sistema nervioso central como mielitis
transversa y polirradiculoneuritis. TRATAMIENTO: Permanecer en casa, en reposo para mejorar los síntomas y evitar contagio a otras
personas. En casos leves a mderados; ofrecer líquidos abundantes, aumentar el consumo de frutas con mayor contenido de vitamina A
y C. No se recomienda suspender la lactancia materna si la madre enferma. Deben evitarse los lugares concurridos, así como cambios

CURSO ENARM CMN SIGLO XXI TEL: 36246001 Pharmed Solutions Institute PÁGINA 422
MANUAL DE TRABAJO DEL CURSO ENARM CMN SIGLO XXI
bruscos de temperatura, tabaquismo y exposición a contaminantes dentro de la casa. El control de la fiebre por medios físicos, evitando
en los niños el uso de salicilatos (por la asociación con el síndrome de Reye). Cuando amerite, se podrán administrar paracetamol o
AINES con efecto antipirético (ibuprofeno). No se recomienda utilizar antibióticos profilácticos. Es importante hacer hincapié en los
datos de alarma que sugieren el agravamiento y necesidad de manejo hospitalario. En la edad pediátrica los síntomas incluyen: fiebre
persistente, tos productiva con expectoración, dificultad para respirar, rechazo al alimento, hipotonía o convulsiones. Medicamentos
con utilidad clínica al reducir la duración de los síntomas cuando son empleados en las primeras 48 horas del inicio de la enfermedad.
También pueden ser efectivos en la quimioprofilaxis. La amantadina y la rimantadina inhiben la replicación de los virus de influenza A,
pero no los de influenza B, se administran por vía oral y se utilizan para el tratamiento o quimioprofilaxis, disminuyen la excreción viral
y reducen en promedio un día la duración de la enfermedad. El tratamiento se recomienda por cinco días y cuando se utilizan para
quimioprofilaxis, son efectivos en 70-90%. Se recomienda el uso profiláctico de la amantadina y rimantadina para contactos o
convivientes. Los inhibidores de la neuraminidasa (zanamivir, oseltamivir) tienen actividad contra influenza A y B, bloqueando el sitio
activo de la neuraminidasa, disminuyen la excreción del virus y reducen la duración de los síntomas de la influenza por 36 horas. En
ensayos clínicos se ha encontrado que el oseltamivir disminuye las complicaciones de las vías respiratorias bajas (neumonía y
bronquitis), disminuye uso de antibióticos y el riesgo de hospitalización. PREVENCIÓN: La vacunación anual de personas en grupos de
alto riesgo de desarrollar complicaciones y sus contactos, representa la principal estrategia. Las vacunas de mayor uso son producidas
de virus crecidos en huevos embrionados inactivados con formaldehido o propiolactona, pueden producirse de virus completos
tratados con detergentes o de antígenos de superficie (hemaglutinina y neuraminidasa) purificados. Son más recomendadas las que
contienen antígenos purificados, ya que dan menos reacciones alérgicas. Habitualmente contienen los tres tipos virales recomendados
por la OMS. Diversos estudios han demostrado una eficacia de 80% en niños y de 77% en adultos. Las vacunas contra la influenza son
recomendadas en Mexico para: a) Vacunación a personas que pueden transmitir influenza a personas en grupos que están propensos a
desarrollar complicaciones: 1) Vacunación a personal de salud (personal médico, enfermería, paramédico). 2) Personal que labora en
asilos de ancianos y casas de estancia. 3) Personas que viven en contacto intradomiciliario con personas que tienen alto riesgo de
desarrollar complicaciones (niños que viven con un adulto con asma). b) Vacunación de mujeres con embarazo de alto riesgo:
específicamente aquellas en el segundo y tercer trimestre del embarazo por tener el mayor riesgo de complicaciones. c) Vacunación de
personas mayores de 65 años de edad. Este grupo presenta el mayor riesgo de hospitalización y muerte asociado a complicaciones por
influenza. d) Vacunación de personas de entre 50 a 64 años. En este grupo se concentra la mayor pre-valencia de personas con
condiciones de alto riesgo (enfermedades crónico-degenerativas). e) Vacunación de pacientes de cualquier edad con padecimientos
crónicos. Estos padecimientos incluyen: enfermedades pulmonares (asma o enfermedad pulmonar obstructiva crónica) o
enfermedades cardiovasculares (insuficiencia cardiaca). Así como aquellos con enfermedades metabólicas crónicas (diabetes mellitus,
insuficiencia renal) y en pacientes inmunodeprimidos (VIH positivos o trasplantados). f) Vacunación de niños de 6 a 36 meses de edad.
Se ha demostrado que en esta población existe un riesgo aumentado de hospitalización por complicaciones asociadas a la influenza.
Aunque el riesgo en niños menores de seis meses es aún mayor, la vacuna trivalente no está aprobada para su uso en este grupo. g)
Viajeros. Se recomienda en aquellos viajeros que no fueron vacunados en el año precedente si planean viajar a los trópicos, o al
hemisferio sur durante los meses de abril a septiembre. Medidas generales: Las medidas no farmacológicas que se recomiendan para
ayudar a prevenir la propagación de enfermedades respiratorias como la influenza son: Cubrirse la nariz y la boca con un pañuelo
desechable al toser o estornudar. Tirar el pañuelo desechable a la basura después de usarlo. Lavarse las manos frecuentemente con
agua y jabón, especialmente después de toser o estornudar. Puede utilizarse alcohol-gel. Evitar tocarse los ojos, la nariz o la boca. Evitar
el contacto cercano con personas enfermas. Al enfermarse, debe permanecer en casa por siete días a partir del comienzo de los
síntomas o hasta que hayan pasado 24 horas desde que desaparecieron. Esto disminuye la posibilidad de infectar a otras personas y
propagar más el virus. Se deben seguir las recomendaciones de la Secretaría de Salud con relación al cierre de escuelas, evitar
frecuentar sitios con multitudes y tomar medidas de distanciamiento social.

CASO CLINICO
Paciente de 17 años con fiebre, tos y dolor de garganta, cefalea, rinorrea y dolor abdominal, por lo que iniciaron tratamiento
sintomático para rinofaringitis con respuesta no favorable por lo que regresa a los 3 dias con empeoramiento de los síntomas.

PREGUNTA
Cuales son los síntomas o signos cardinales para considerar caso sospechoso de influenza.

RESPUESTA
a.- Fiebre con tos o dolor de garganta.
b.- Cefalea, rinorrea y coriza.
c.- Artralgias, mialgias y postración.
d.- Dolor torácico, dolor abdominal y congestión nasal.

PREGUNTA
Cuales son las carateristicas que presentan mayor complicaciones.

RESPUESTA
a.- Edad >60 años y <2 años.
b.- Enfermedad crónica o debilitante.
c.- Embarazo y primeros 6 meses postparto
d.- Pacientes que acudan a una segunda consulta por deterioro o sin mejoría clínica.

PREGUNTA

CURSO ENARM CMN SIGLO XXI TEL: 36246001 Pharmed Solutions Institute PÁGINA 423
MANUAL DE TRABAJO DEL CURSO ENARM CMN SIGLO XXI
Se ingresa ingresa anteriomente referido por fiebre de 40 grados, dolor abdominal, dolor toraxico con dificultad respiratoria, durante su
estancia hospitalaria presenta vómito y diarrea persistentes, TA 100/70, ¿Cuál es su conducta a seguir.

RESPUESTA
a.- Unidad médica hospitalaria con aislamiento estándar, precauciones de gotas y de contacto.
b.- Aislamiento domiciliario estricto, con listado de datos de alarma, con indicaciones para reevaluar al día diguiente si fuera necesario.
c.- Aislamiento domiciliario estricto, con listado de datos de alarma.
d.- Compensación de enfermedad crónica, manejo de complicaciones y tratamiento antiviral (oseltamivir).

PEGUNTA
Cual es el esquema terapeutico antiviral mas apropiado.

RESPUESTA
a.- 60 mg cada 12 hrs por 5 días.
b.- 45 mg cada 12 hrs por 5 días.
c.- 75 mg cada 12 hrs por 5 días.
d.- 150 mg cada 12 hrs por 5 días

CASO CLINICO
Niño de 10 años que consultó al servicio de urgencias después de presentar crisis convulsiva tonicoclónica generalizada (TCG) de 3 min
de duración, después de 4 días de fiebre, tos, inapetencia general y fatiga; en el hospital cursó febril, con debilidad, desorientado; unos
minutos después presentó otra crisis TCG de 3 min, se controla las crisis con mejoría 12 horas después presenta las siguientes
manifestaciones Inflitrados pulmonares en más de dos cuadrantes, índice de oxigenación PaO2/FiO2 menor de 250 y distensibilidad
pulmonar disminuida.

PREGUNTA
Cual es la conducta a seguir.

RESPUESTA
a.- 60 mg cada 12 hrs por 5 días.
b.- Ceftriaxona 50-100 mg/kg/día IV c/24 hrs por 7-10 días.
c.- FiO2: el necesario para mantener PaO2 arriba de 60 mm Hg.
d.- Presión menor de 35 cm H2O y meseta menor de 30 cm H2O.

MENINGITIS. CIENCIAS BASICAS: Es un proceso inflamatorio agudo del SNC causado por microorganismos que afectan a las
leptomeninges (aracnoides y piamadre). El sistema nervioso puede infectarse por diferentes microorganismos, incluyendo bacterias,
virus, hongos, protozoos y helmintos. La presentación clínica de estas infecciones puede ser aguda, subaguda o crónica dependiendo de
la etiología, la virulencia del microorganismo y la localización del proceso infeccioso. SALUD PUBLICA: Un 80% ocurre en la infancia,
especialmente en niños menores de 10 años. En la última década, con la introducción de nuevas vacunas frente a los gérmenes
causales más frecuentes y con el desarrollo de antibióticos más potentes y con buena penetración hematoencefálica, ha disminuido la
incidencia y ha mejorado el pronóstico de la infección, pero las secuelas y la mortalidad no han sufrido grandes cambios. MENINGITIS
VIRAL (95%): Los virus que se detectan con más frecuencia en meningitis aséptica son los enterovirus (EV), seguidos de virus herpes
simple (VHS) y varicela zoster (VZV). Entre los muchos tipos de EV causantes de brotes de meningitis, destacan Echovirus 30, 13, 6, 11 y
9, Coxsackie B5 y Coxsackie A9. Los EV son también la causa principal de meningitis y sepsis neonatal. PATOGENIA: El más general sería
la colonización del SNC a través de diseminación hematógena del virus durante la primoinfección vírica. En el caso de los alfa
herpesvirus un segundo mecanismo, cuyo ejemplo más típico sería la encefalitis herpética, consistiría en la invasión del SNC tras
reactivación de la infección latente desde los ganglios nerviosos regionales a través de las fibras nerviosas. DIAGNOSTICO: Las
meningitis de carácter vírico suelen tener un curso más insidioso, con escasa afectación del estado general, fiebre de intensidad
generalmente moderada (en ocasiones ausente) y cefalea que puede ser intensa. La punción lumbar puede generar un alivio transitorio
de los síntomas. En el caso de la meningoencefalitis vírica, se describe una triada clásica: fiebre, cefalea y alteración conciencia. El LCR
aunque muetsra pleocitosis generalmente no rebasa de 200-500 celulas y de predomino de mononucleares. Los estuidos serológicos o
de estudio de virus esclarecen el diagnostico. MENINGITIS BACTERIANA (5%): Se caracteriza por la presencia de signos y síntomas
meníngeos y encefálicos con grados variables. La morbilidad en Mexico es de 50% de los casos, se presenta en niños de 3 meses a 3
años de edad y su mortalidad varia de 5-15 %, las secuelas son retraso psicomotor leve hasta retraso mental grave, sordera,
convulsiones e hidrocefalia, varían de acuerdo a edad, germen causal, oportunidad y eficacia del tratamiento. Agentes causales en RN a
3 meses son baterías gramm negativas (E. coli y Klebsiella), estreptococo beta hemolítico del grupo B, Listeria monocytogenes y
meningococos. De 3 meses a 5 años son: Haemophilus influenzae del tipo B, Streptococo pneumoniae, Neisseria meningitides,
neumococos y meningococos. PATOGENIA: La mayor parte de los casos inician con una bacteriemia a partir de focos infecciosos,
respiratorios, gastroinetstinales y urinarios, también ocurre diseminación directa de un foco parameníngeo por ejemplo: pansinusitis u
otitis recurrente donde los gérmenes involucrados son diferentes de acuerdo al proceso infeccioso primario, también suele ocurrir por
invasión directa como en el caso de fracturas de cráneo, disrrafías de la línea media, procedimientos neuroquirurgicos, colocación de
catéteres. Unas vez que ocurre la bacteriemia existe un proceso inflamatorio del endotelio de los vasos cerebrales con aumento del
tamaño de los poros o puentes intercelulares, lo que permite el paso de moléculas de gran tamaño (proteínas y bacterias) al intersticio
del parénquima cerebral (rotura de la barrera hematoencefalica) y se produce un edema cerebral vasógeno que repercute en la entrega
de oxígeno a las neuronas, así como edema cerebral de tipo citotóxico, irritando las meninges. La inflamación y el daño celular neuronal
causan metabolismo anaeróbico con producción de lactato, consumo de glucosa y elevación de proteínas en el LCR. DIAGNOSTICO: La

CURSO ENARM CMN SIGLO XXI TEL: 36246001 Pharmed Solutions Institute PÁGINA 424
MANUAL DE TRABAJO DEL CURSO ENARM CMN SIGLO XXI
sintomatología varía según la edad del paciente en RN: inespecífica, sospechar en RN con sepsis, fiebre, hipotermia, irritabilidad, rigidez
de nuca, rechazo al alimento, vómito, diarrea y convulsiones (40%). Lactante: sospechar cuando hay infección respiratoria o
gastrointestinal de varios días de evolución con fiebre persistente, vomito, irritabilidad, convulsiones, fontanela anterior tensa y
abombada (es un signos tardio presente en 20%), rechazo a la vía oral o bien rigidez de nuca, hiperreflexia, signos de Kerning (dolor de
espalda con la extensión pasiva de la rodilla estando los muslos flexionados) y Brudzinsky (flexión espontánea de los miembros
inferiores al flexionar pasivamente el cuello), de Babinski y cefalea. Los isgnos de focalización neurológica hemiparesia, cuadriparesia y
paralisis facial pueden ocurrir temprana o tardíamente en 15% de los casoso por trombosis venosa o arterial, secundaria e inflamación.
Laboratorio, BH reporta leucocitosis con predominio de neutrófilos y bandas, así como Hb baja. Debe tomarse cultivo de orina,
electrolitos séricos para corregir el balance hídrico. QS, para verificar la cuantificación de glucosa en sangre, pruebas de coagulación,
cuantificación de plaquetas. Punción lumbar para citología de LCR con raquimanometro para registrar la presión de apertura del LCR
(normal 70-150mm). Se hacen pruebas de diagnóstico rápido como coaglutinación en látex que permite un buen porcentaje de
sensibilidad u especificidad existiendo reactivos para detectar H. influenzae tipo B, Streptococo pneumoniae, N. meningitidis, E. coli. Se
debe tomar hemocultivo e iniciar de inmediato el tratamiento antimicrobiano. La TAC está indicada en : 1. Pacientes con
neuroinfección en quienes se sospecha alguna complicación como higroma subdural , absceso etc. 2. Pacientes con persistencia de
alteraciones neurológicas a pesar de un tratamiento integral adecuado. 3. En quienes presentan datos de focalización, hipertensión
intracraneana o infección por enterobacterias. TRATAMIENTO: Varia según resultado de cultivo de LCR y hemocultivo, sin embargo se
sugiere iniciar tratamiento de acuerdo a la edad. RN: ampicilina 150-200mg/kg/día + gentamicina 7.5mg/kg/día ó amikacina
22mg/kg/día ó cefotaxima 150-250mg/kg/día. Niños mayores de 3 meses: ceftriaxona 100mg/kg/día, cefotaxima 200mgs/kg/día,
cloranfenicol 75-100mg/kg/día. Preescolares y escolares: penicilina 250-300 000/kg/día + cloranfenicol 75-100mg/kg/día. MENINGITIS
FIMICA: Puede representar alrededor del 1% de todas las formas y es más
frecuente en población de países PARAMETRO NORMAL M. BACTERIANA M. VIRICA M. TUBERCULOSA
subdesarrollados, niños y Presión de apertura 70-150mm Alta N/alta Alta
Aspecto de LCR Turbio o purulento Claro opalescente
pacientes infectados por el VIH. La
Glucosa (mg/dl) 60-80% de la <66% de la sanguínea Normal <40mg/dl
localización meníngea puede glucemia (hipoglucorraquia)
producirse por vía hematógena durante Cels/mm3 <10 (MMN) >1000 (PMN) <300 <1000
la primoinfección o la reactivación, o Proteínas (mg/dl) <45 Aumentadas ++ N o aumentada+ Aumentada +++
bien por ruptura al espacio (100-500mg/dl)
subaracnoideo de un foco parameníngeo ya existente. En cualquier caso se produce una inflamación granulomatosa de las meninges
basales y pequeños focos tuberculosos (focos de Rich) en las meninges, el cerebro o la médula espinal. . Los síntomas más frecuentes
son fiebre y cefalea de curso subagudo o crónico, vómitos, decaimiento, rechazo de las tomas, añadiéndose posteriormente
disminuciones del nivel de conciencia y/o alteraciones de la conducta, otros síntomas y signos de hipertensión intracraneal, afectación
de los pares craneales (III,VI,VII), hidrocefalia, y diferentes formas de alteraciones neurológicas focales. El cuadro clínico, por lo general
es de instalación lenta (2-3semanas) y por lo reguilar debe existir un estudio de Combe positivo. MENINGITIS MICOTICA: Son raras
evolucionan lentamente, se presentan a menudo en pacientes inmunodeprimidos o con desnutrición crónica. El estudio citoquímico de
LCR es indistinguible del de una meningitis fimica. Posponer punción lumbar: Inestabilidad hemodinámica. Insuficiencia respiratoria.
Diátesis hemorrágica (<100.000 plaquetas). Hipertensión intracraneal confirmada. Infección superficial en el lugar de la punción.

CASO CLINICO
Una adolescente de 14 años previamente sana que acudió en estado comatoso. Tres días antes había comenzado con tos, fiebre,
rinorrea y cefalea ocasional, siendo diagnosticada de catarro y tratada con antitérmicos y antihistamínicos. Unas horas antes del
ingreso presentó varios episodios de pérdida de fuerza de los 4 miembros, cefalea intensa y fiebre de 38 °C. En el momento previo al
ingreso la madre la había encontrado apoyada en la pared realizando movimientos incoordinados de las 4 extremidades, sin respuesta
a estímulos verbales, mirada pérdida y después caída al suelo, pérdida de conciencia y relajación de esfínteres. Temperatura, 37 °C;
presión arterial, 130/70 mmHg; frecuencia cardíaca, 110 lat./min; frecuencia respiratoria, 19 resp./min; saturación de oxígeno, 100 %;
coloración normal de piel y mucosas; desconectada del medio; Glasgow de 8; hipotonía generalizada; reflejos osteotendinosos
normales; pares craneales normales; pupilas midriáticas pero reactivas, y signos meníngeos negativos. El resto de la exploración física
fue normal. Líquido cefalorraquídeo: proteínas, 53 mg/dl; glucosa, 60 mg/dl; leucocitos, 5/ ml.

PREGUNTA
Cual es el agente etiológico mas frecuente considerando la edad y el cuadro clínico.

RESPUESTA
a.- Streptoccoco del grupo B, E. coli ó listeria.
b.- Haemophilus influenzae, Straptococo pneumoniae ó E. coli.
c.- Streptococcus pneumoniae ó neisseria meningitidis.
d.- Haemophilus influenzae, neisseria meningitidis ó streptococcus pneumoniae.

CASO CLINICO
Lactante de 8 meses. En la última semana presentaba cuadro febril de hasta 39°C en el contexto de una gastroenteritis aguda. Tras
presentar respiración superficial y bradicardia, precisa intubación y conexión a ventilación mecánica. Se practica TAC craneal en el que
destaca hidrocefalia tetraventricular por lo que se decide colocación de derivación ventricular externa con salida de líquido turbio con
glucosa ligeramente disminuida (0,35g/l) y proteínas elevadas (0,57g/l). La citología muestra un perfil bacteriano más por la fórmula
que por el número total de células (240 células nucleadas con 80% de segmentados y 20% de linfocitos). Destaca la emisión de
abundante orina transparente en una niña con deshidratación leve (3%) hasta ese momento y cuadro diarreico de una semana de
evolución, que se intensifica a la vuelta de quirófano llegando a presentar poliuria de 30cc/kg/h. Destacan hipernatremia de 153mEq/l,

CURSO ENARM CMN SIGLO XXI TEL: 36246001 Pharmed Solutions Institute PÁGINA 425
MANUAL DE TRABAJO DEL CURSO ENARM CMN SIGLO XXI
hiponatriuria de 11,9mEq/l, densidad urinaria de 1003, osmolaridad urinaria de 110mosmol/kg, osmolaridad plasmática de
318mosmol/kg y poliuria con acentuación de la deshidratación a pesar de reposición horaria de la diuresis.

PREGUNTA
Cual es la complicación mas probable.

RESPUESTA
a.- Falla renal.
b.- Sindrome de secresion inapropiada.
c.- Diabetes insípida central.
d.- Necrosis hipofisiaria.

OTITIS. CIENCIAS BASICAS: La otitis media aguda (OMA), es la infección supurada del oído medio, que tienen un inicio súbito y de corta
duración; asimismo, denota inflamación de la cubierta mucoperióstica del oído medio. La membrana timpánica inflamada se presenta
opacificada, protruyente o con ambas características. Según su tiempo de evolución la enfermedad puede subdividirse en a) aguda,
cuando el proceso dura no más de 3 semanas; b) subaguda, cuando la infección perdura de 3 semanas a 3 meses, y c) crónica, cuando la
enfermedad se prolonga por más de 3 meses. SALUD PUBLICA: La OMA es un problema mundial que afecta a uno de cada 4 niños
menores de 10 años y es la infección bacteriana más frecuente en niños menores de 5 años. De 25-40% de las visitas pediátricas en
menores de 5 años son por OMA. Hay estudios que indican que 80% de los preescolares menores de 3años, han tenido cuando menos
un episodio de OMA. Hay mayor incidencia durante otoño e invierno., los niños que son llevados a guarderías parecen ser
especialmente susceptibles a padecer OMA, y los expuestos a humo de tabaco. CLASIFICACION: 1. Miringitis: cuando se trata de una
inflamación de la capa externa de la membrana timpánica que puede ocurrir sola o asociada a una inflamación del conducto auditivo
externo. 2. Otitis media aguda supurada: infección aguda del oído con exudado y de corta duración. 3. Otitis media secretoria:
presencia de líquido en oído medio detrás de una membrana timpánica integra sin signos agudos o síntomas. 4. Otitis media crónica
supurada: presencia de descarga crónica del oído medio a través de una perforación de la membrana, a veces hay perforación sin
descarga. 5. Otitis media recurrente: presencia de 3 episodios de otitis media aguda en 6 meses, o 4 en un año, o 2 cuadros diferentes
con un mes entre ellos. PATOGENIA: Entre las funciones del oído se incluyen la ventilación y el equilibrio de las presiones atmosféricas,
así como la protección ante el sonido y el drenaje de secreciones a la nasofaringe. Cuando estos mecanismos se alteran, contribuyen a
la proliferación bacteriana, lo que desencadena la infección. La obstrucción tubaria también inhibe el drenaje del líquido del oído medio
a la nasofaringe, lo que contribuye al desarrollo de la infección. La falla de estos mecanismos puede deberse a inflamación, infecciones
frecuentes, problemas alérgicos o neoplásicos. Aunado a ello, la trompa de Eustaquio de los niños es más corta, más horizontal y tienen
menor soporte cartilaginoso. En 2/3 partes de los pacientes con OMA pueden aislarse patógenos bacterianos del líquido del oído. En
30-50% de los casos se encuentra Streptococo pneumoniae. El Haemophilus influenzae causa entre 20-27% de los casos de OMA.
Moraxella se aisla Moraxella catarhallis se isla en 7-23% de los casos. Los niños hospitalizados desarrollan gérmenes del medio
ambiente hospitalario. Las infecciones virales representan el 41% de los casos de OMA. Los virus syncitial respiratorio (74%),
parainfluenza (52%) e influenzae (42%). Algunas patologías como anormalidades craneofaciales y el reflujo gastroesofágico se
relacionan con la presencia de otitis media. El uso de chupón es un factor de riesgo para otitis media recurrente. DIAGNOSTICO: clínica;
se presentan como un cuadro de coriza, rinitis, fiebre, tos, irritabilidad y anorexia. El síntoma más fidedigno es la otalgia. Los niños muy
pequeños no verbalizan el dolor pero se jalan las orejas o rehúsan a deglutir y lloran constantemente. Debe sospecharse cuando el
dolor despierta a los niños por la noche. Exploración del oído medio, mediante el uso de un otoneumatoscopio (sensibilidad de 95% y
especificidad de 80%), debe realizarse de manera que el conducto auditivo externo se encuentre libre para visualizar correctamente la
membrana timpánica y valora su movilidad, los cambios de coloración y vascularización, así como la presencia de niveles hidroaereos
por detrás de la misma, debe realizarse son suavidad para no lastimar. La OMA se manifiesta clínicamente con retracción, eritema,
disminución del reflejo luminoso e hipomotilidad de la membrana timpánica; asimismo, se presenta una disminución de la audición en
rangos de baja frecuencia. La membrana timpánica, particularmente la pars flácida, se encuentra hiperemica, con la vascularidad
visible, y las pars tensa, abombada y turbia. En un estado avanzado de la enfermedad puede haber ruptura de la membrana timpánica,
habitualmente en la pars tensa, con salida de material francamente purulento, sanguinolento o seroso; la perforación habitualmente es
pequeña, lo que la distingue de etiologías más raras (necrosante, TB, recurrencia de otitis), y una vez que la pus drena, los síntomas
disminuyen de manera importante. De manera ideal se debe realizar timpanometria para identificar diferencias en la presión dentro y
fuera del oído y confirmar la presencia de líquido; asimismo la reflectometria acústica es un método no invasivo para identificar
derrame de oído medio mediante el reflejo del sonido. TRATAMIENTO: Es importante hacer notar que existen pocas herramientas que
ayuden a la pediatría para formular una estrategia adecuada de tratamiento; sin embargo, en recientes revisiones se ha observado que
el uso temprano de la terapia antibiótica mejora el curso clínico de la OMA, disminuyendo la sintomatología y la incidencia de
complicaciones. La mayoría de los expertos continua recomendando amoxicilina (primera elección) para iniciar tratamiento empírico de
todos los episodios de otitis media a dosis de 80-90mg/kg de peso día durante 5-10 días, la acción de este puede llegar a ser limitada
(20-30% son resistentes). La selección debe basarse en la concentración de los mismos en el sitio de infección, en caso de resistencia,
enfermedad grave y H. influenzae y M catarrhalis, se recomienda amoxicilina/clavulanato( 90mg/kg/dia con 6.4 mg/kg/día dividido en 2
dosis). El manejo conservador sugiere 10 días de antibioticoterapia, aunque hay resultados aceptables con 5-7 días de tratamiento. Si
un paciente no mejora con amoxicilina después de 72hrs de manejo se debe cambiar de antibiótico. El TMP/SFX tienen una elevada
resistencia para Streoptococus pneumoniae 30-44.6%, no se recomienda su uso. En pacientes alérgicos a betalactamicos, las
alternativas por su adecuada concentración en ido medio son macrolidos y clindamicina. Todos los niños menores de 6 meses con
diagnóstico de OMA deben recibir tratamiento para evitar complicaciones. No se recomienda el uso de gotas oticas, con o sin
antibiótico, gotas nasales, mucolíticos, antihistamínicos, pues no aportan beneficios al tratamiento de la OMA. La administración de
paracetamol o ibuprofeno es necesaria, para el manejo de la fiebre y el dolor. Tratamiento quirúrgico: Miringotomia; cuando hay
otalgia severa o complicaciones intratemporales o intracraneales, optimiza la regulación de las presiones en el oído medio.
COMPLICACIONES: Pérdida auditiva (la mas común), perforación de la membrana timpánica, mastoiditis aguda, laberintitis, parálisis

CURSO ENARM CMN SIGLO XXI TEL: 36246001 Pharmed Solutions Institute PÁGINA 426
MANUAL DE TRABAJO DEL CURSO ENARM CMN SIGLO XXI
facial, colesteatoma (acumulación de tejido epitelial de crecimiento rápido que va destruyendo el hueso temporal y su contenido).
PREVENCION: La aplicación de vacuna conjugada antineumococica, se asocia a una reducción de la colonización nasofaríngea por
Streptococo pneumoniae. La eficacia estimada de la vacuna heptavalente conjugada para evitar otitis media aguda es de 6%, para otitis
media recurrente 9%, hasta un 57% de las OMA causadas por neumococo y 90% de las otitis causadas por los serotipos de neumococo
incluidos en la vacuna hepatavalente. Se recomienda a los 2, 4 y 6 meses de edad y refuerzo entre los 13 y 15 meses.

CASO CLINICO
Paciente varón de 6 años de edad, sin antecedentes médicos de interés, con vacunación correcta, incluida vacuna antineumocócica.
Cuadro catarral de 5 días de evolución, con otalgia izquierda de 6 horas de evolución, fiebre de hasta 39,9°C axilar y parestesias en la
hemicara derecha, que posteriormente se extienden en menor intensidad a la mano y la pierna derechas. En la exploración clínica
mostraba rigidez de nuca, con signo de Brudzinsky positivo. El resto de la exploración neurológica era normal. En la otoscopia se veían
ambos tímpanos hiperémicos, no abombados, y no mostraba despegamiento del pabellón auricular.

PREGUNTA
Cual es la conducta diagnostica mas adecuada.
a.- Biometria hemática.
b.- Factores inflamatorios.
c.- TAC craneal.
d.- Rx de cráneo.

CASO CLINICO
Mujer de 2 años de edad, sin antecedentes patológicos, con vacunación correcta, incluida vacuna antineumocócica, que consulta por
cuadro de 4 días de evolución de fiebre (38,8°C axilar máxima), odinofagia y tortícolis, con desviación cefálica hacia abajo y la izquierda.
Tratada desde hacía 24h con ibuprofeno vía oral VO y calor local, sin presentar mejoría. En la exploración presentaba desviación cefálica
hacia abajo y hacia la izquierda, con intensa contractura del esternocleidomastoideo izquierdo, sin otros hallazgos patológicos. La
exploración neurológica era normal. En la otoscopia se veían ambos tímpanos hiperémicos, no abombados, y no había despegamiento
del pabellón auricular. Se realizó un hemograma, con leve leucocitosis (11.800 cel/μL) sin desviación izquierda, y PCR de 67mg/l; el
resto de la analítica era normal. Se realizó un hemocultivo, negativo. Dada la persistencia del tortícolis, se realizó una TC craneal, en la
que se visualizó ocupación de caja timpánica, aditus, antro mastoideo y celdas mastoideas, por lo que se catalogó de otomastoiditis
izquierda.

PREGUNTA
Cual es la conducta a seguir.

RESPUESTA
a.- Clindamicina y vancomicina
b.- Ceftriaxona mas gatifloxacina.
c.- Dicloxacilina y amikacina.
d.- Ampicilina y gentamicina.

SINUSITIS. CIENCIAS BASICAS: Es la infección de uno o varios senos paranasales (etmoidal, maxilar, frontales, esfenoidal) causada por
la obstrucción del ostium de drenaje. Secundario, entre otros factores, a la desviación septal, adenoiditis o pólipos, con producción de
secreción mucopurulenta e inflamación de la mucosa (habitualmente implica siempre un grado de afectación de la mucosa
nasal=rinosinusitis) y sintomatología de dolor dependiendo del seno afectado. SALUD PUBLICA: La sinusitis es infrecuente en los
menores de un año, quizá por el menor desarrollo anatómico de los senos paranasales en este grupo etáreo. No se ha guardado
relación con predominio por raza o sexo. La remisión espontánea puede ser hasta de 70%. CLASIFICACIÓN: 1. Sinusitis aguda; dura
menos de cuatro semanas, los episodios se resuelven con tratamiento médico, sin daño residual en la mucosa. 2. Sinusitis sub-aguda;
duran de cuatro semanas a tres meses. 3. Sinusitis crónica; tiene una duración de más de tres meses, con alteraciones estructurales en
la mucosa sinusal ó 6 episodios de sinusitis aguda al año, con alteraciones en la TAC de senos paranasales. 4. Sinusitis aguda recurrente;
cuando el paciente presenta más de 4 episodios de sinusitis aguda en un período de seis meses y existe adecuada respuesta al
tratamiento médico, sin daño residual de la mucosa. PATOGENIA: Los siguientes son los principales factores de riesgos que conllevan a
que se presente sinusitis en la infancia: Las infecciones respiratorias de las vías aéreas superiores, inmadurez inmunológica e inmuno-
deficiencias, hipertrofia adenoidea y la adenoiditis, asistencia a la sala cuna o jardín infantil, la exposición al humo de los cigarrillos,
contaminación ambiental y la alergia, cuerpos extraños, pólipos y tumores nasales. El papel de la infección bacteriana como causa
primaria de sinusitis crónica es controvertido, siendo más consecuencia de factores no infecciosos. Las infecciones víricas predisponen
al desarrollo de la sinusitis bacteriana (80% de los casos), mientras que el componente alérgico es significativo en el 20% restante. La
sinusitis bacteriana casi siempre surge como complicación de una de las anteriores. La microbiología de la sinusitis bacteriana aguda
(SBA) es semejante a la de la otitis media aguda, los gérmenes que con más frecuencia causan tanto la sinusitis aguda como crónica,
independiente de la edad, en orden de importancia son: Virus: Adenovirus, influenza, parain-fluenza, Estreptococo pneumoniae (30%),
Haemophilus influenzae (20%), Moraxella catarrhalis (20%), Branhamella catarralis, Streptococo beta hemolítico, Stafilococo aureus,
Gérmenes anaerobios. La sinusitis aparece posterior a la rinofaringitis aguda o crónica, presentando antecedentes de inflamación viral
en vías aéreas superiores o inferiores. Si persiste por más de diez días predispone a la inflamación de la mucosa de los senos
paranasales. DIAGNOSTICO: Clínica; Existen dos situaciones clínicas que nos hacen sospechar una SBA: 1. Síntomas catarrales leves
(rinorrea, tos diurna) que no ha empezado a mejorar tras 10 días (criterio de Persistencia): es la presentación más habitual y debe
diferenciarse de los catarros encadenados. 2. Concurrencia de fiebre elevada (≥ 39 ºC) y rinorrea mantenidas 3 días (criterio de
Gravedad) con afectación del estado general. Síntomas específicos: Se ve más en niños mayores, dolor facial, cefalea, dolor

CURSO ENARM CMN SIGLO XXI TEL: 36246001 Pharmed Solutions Institute PÁGINA 427
MANUAL DE TRABAJO DEL CURSO ENARM CMN SIGLO XXI
periorbitario y anosmia entre otros (presentes en 1/3 de los casos). Síntomas inespecíficos: Secreción nasal y obstrucción nasal uni o
bilateral. Fiebre. Halitosis. Tos. Identificación en el niño menor: Los síntomas son inespecíficos y se suelen manifestar como irritabilidad.
Se identifica con fiebre alta, aspecto tóxico. Puede presentar celulitis periorbitaria o aumento de volumen ocular. La rinorrea puede ser:
Hialina. Mucosa. Purulenta. Sanguinolenta. Es de anotar que las características de las secreciones no se relacionan con el agente
etiológico. En estos casos de diferenciación es mas importante el tiempo de evolución y el estado general del paciente. RINORREA
HIALINA: Se ve en los cuadros agudos, por edema y secundario a enfermedades sistémicas, principalmente rinofaringitis aguda, rinitis
alérgica. Hay inflamación de la mucosa con aumento de la permeabilidad de los capilares, que produce un exudado seroso en la cavidad
sinusal. Se presenta edema del estroma con infiltración celular, linfocitico y células plasmáticas. RINORREA MUCOSA: Por aumento de
las glándulas productoras de moco, la secreción se estanca creando un medio de cultivo excelente para el desarrollo de gérmenes, en el
estroma aparecen histiocitos y células productoras de moco. La cavidad nasosinusal con la secreción, va absorbiendo el aire del seno,
creando presión negativa que conlleva a mayores alteraciones histológicas. RINORREA PURULENTA: La coloración podrá variar de
amarillo, verdoso, marrón, mal oliente y de gran viscosidad, adherente a la mucosa. Hay infiltrado de polimorfonucleares y de otras
células plasmáticas, histiocitos y fibroblastos. Hay obliteración glandular, vascular y neural. RINORREA SANGUINOLENTA: Debido a la
ruptura vascular se presenta salida de sangre; aparecen granulomas de colesterol, que actúa como factor irritativo tisular. La patología
se encuentra en un nivel más profundo y hay compromiso de los vasos sanguíneos. La presencia de rinorrea prolongada de más de 10
días permite predecir alteraciones radiológicas asociadas a una infección bacteriana (90% en edades entre 2 y 6 años, y 70% en
mayores de 6 años). La presencia de un catarro común sin sinusitis puede provocar alteraciones radiológicas similares, que permanecen
hasta dos semanas después de la mejoría clínica. Además no distingue entre una sinusitis bacteriana, viral, o de otras causas. Las
pruebas de imagen deben reservarse a casos de fracaso terapéutico o empeoramiento de síntomas, y no están recomendadas en
menores de 6 años. El tratamiento empírico estaría pues justificado sin realizar ninguna prueba previa. Los cultivos de secreción nasal o
exudado faríngeo no tienen correlación con los sinusales. La punción y aspiración directa del seno es el patrón oro diagnóstico, pero
sólo está indicada en contados casos hospitalarios (enfermedad grave con aspecto tóxico, inmunodeficiencias y complicaciones
supurativas intracraneales). Criterios diagnósticos, los cuales a su vez pueden ser mayores (Dolor, presión facial. Congestión nasal.
Obstrucción nasal. Rinorrea anterior o posterior. Hiposmia o anosmia. Cefalea) o menores (Halitosis. Fiebre. Dolor dental. Fatiga. Tos.
Otalgia). TAC es una importante ayuda imagenológica para documentar la existencia del compromiso sinusoidal, y tiene unas
indicaciones muy precisas: Persistencia de la sintomatología luego de manejo apropiado por 10-15 dias. Cronicidad de la sinusitis.
Complicaciones con celulitis facial o periorbitaria. Coexistencia con asma, datos de hipertrofia severa adenoidea ó rinitis o rinorreas
persistentes. Sospecha de sinusitis micótica. COMPLICACIONES: Celulitis periorbitaria y orbitaria, la trombosis del seno cavernoso, el
absceso subperióstico, el absceso cerebral, y el empiema epidural y subdural. TRATAMIENTO: El uso de antibióticos inicialmente no
está indicado en la mayoría de los casos si no presenta criterios de gravedad o persistencia. Como tratamiento sintomático analgésico y
antipirético son efectivos paracetamol (10-15 mg/kg/dia) e ibuprofeno (4-6mg/kg/ dosis cada 6-8hrs). Suelen ser necesarios pocos días.
El uso de descon- gestionantes nasales sigue siendo motivo de controversia, pero se conceptúa que inicialmente mejora el drenaje de
secreciones provocando alivio sintomático, siendo su uso no mayor de cinco días, los de aplicación tópica se encuentra Oximetazolina
al 0.025- 0.05% en dosis de 2-3 gotas o aspersiones en fosas nasales cada 12 horas. Corticoides intranasales: Se aconsejan cuando
existen estados de recurrencia o cronicidad y coexiste rinitis alérgica sobre todo si son persistentes. En algunos estudios se destaca el
efecto antiinflamatorio local. Se encuentran entre otros: Furoato de Mometasona, Fluticasona, Budesonida durante tres meses. Los
corticoides orales no son usados excepto en casos de enfermedades asociadas como poliposis o crisis asmática. Antihistamínicos
Podrían dificultar el drenaje de secreciones. Si existe una base alérgica se recomienda su uso al final del manejo antibiótico, en casos de
estados bacterianos. Se encuentran: Loratadina: Presentación: 5mg/5ml a dosis de 0.2 mg/kg/día cada 12 horas Desloratadina:
Presentación: 2.5 mg/ 5ml: 1-5 años de edad: 1,25mg una dosis dia. 6 A 12 Años: 2,5 mg/día: 5 ml/d Cetirizina: 0.5 mg/kg/día cada 12
horas Levocetirizina: Presentación: sol. Oral. 0.5 mg/ ml. Gotas: 5mg/ml. Su dosis es de 0.125 mg/kg. Cuando existen procesos
infecciosos bacterianos los antibióticos son terapéutica central dentro del tratamiento a realizar. El antibiótico de elección es la
Amoxicilina. Se administra a una dosis de 45 mg/ kg/día, puede aumentarse a 80-90mg/kg/dia. El fracaso terapéutico por resistencia
antibiótica se debe en el caso del neumococo (40%) por cepas con susceptibilidad disminuida, mientras que para Haemophilus
influenzae (20%) y Moraxella catharralis (80%) son por producción de betalactamasas. Por todo lo cual si no existe mejoría con el
tratamiento inicial se utilizará de segunda línea es la Amoxicilina/clavulanato. La dosis es 40-90 mg/kg/día. No obstante, puede ser
usado como primera elección en caso de riesgo alto de resistencia (edad menor de 2 años, toma reciente de antibiótico o zonas de alta
prevalencia de neumococo resistente). Otras alternativas de antibióticoterapia son: Cefuroxime a una dosis de 30mg/kg/día y el
Cefprozil a 30 mg/ kg/día. Los días de duración del manejo antibiótico varían de acuerdo a los autores. Se suele recomendar que sea
entre 5-14 días. Son indicaciones de derivación hospitalaria urgente la afectación severa del estado general (aspecto séptico, cefalea
facial intensa), la sospecha de complicaciones (craneales, endocraneales y oculoorbitarias, salvo la celulitis preseptal que puede
tratarse inicialmente en Primaria) o tumor (síntomas unilaterales persistentes como epistaxis, obstrucción y rinorrea, siendo la
deformidad facial un síntoma tardío) y la existencia de un entorno familiar de riesgo que no garantice cuidados generales,
cumplimiento terapéutico y vigilancia eficaz. En rinorrea persistente unilateral siempre se descartará la presencia de un cuerpo extraño
nasal.

CASO CLINICO
Varón de 9 años y 8 meses, sin antecedentes de interés, remitido al servicio de Urgencias de Pediatría, desde su centro de salud por
cefalea y proptosis. Inicio del cuadro 4 días antes, como fiebre, vómitos, cefalea y congestión nasal; diagnosticado en su centro de salud
de sinusitis y en tratamiento antibiótico con amoxicilina/clavulánico desde hace 48 horas, sin mejoría de los síntomas. Refiere cefalea
intensa fronto-temporal derecha y edema en párpado derecho que ha evolucionado hacia proptosis en las últimas horas. A la
exploración física, se evidencia ligera proptosis del ojo derecho, con leve tumefacción palpebral, así como una obstrucción de la fosa
nasal del mismo lado. En la exploración neurológica solamente destaca una leve dificultad para la mirada conjugada

PREGUNTA
Cual es la conducta a seguir.

CURSO ENARM CMN SIGLO XXI TEL: 36246001 Pharmed Solutions Institute PÁGINA 428
MANUAL DE TRABAJO DEL CURSO ENARM CMN SIGLO XXI

RESPUESTA
a.- Envio a pediatría.
b.- Realizar radiografia.
c.- Cambio de antibiticos.
d.- Agregar anti-inflamatorio esteroideo.

PREGUNTA
Cual de los siguientes criterios no america envio urgente a segundo nivel.

RESPUESTA
a.- Edema, eritema o dolor facial.
b.- Alteraciones visuales.
c.- Datos meníngeos.
d.- Falta de mejoría y fiebre elevada.

PREGUNTA
Cuál de las siguientes condiciones requiere un envio ordinario a segundo nivel.

RESPUESTA
a.- Factores predisponentes.
b.- Infección oportunistas.
c.- Compromiso inmunológico.
d.- Refractario al tratamiento.

PREGUNTA
Cuál no forma parte del protocolo de envio a segundo nivel.

RESPUESTA
a.- Biometria hemática.
b.- Eosinófilo en moco nasal.
c.- Radiografia de senos paranasales.
d.- Cultivo de moco nasal.

CASO CLINICO
Niña de 10 años que presenta alteraciones del habla y de la marcha, adormecimiento, un vómito y fiebre. En tratamiento con
amoxicilina-ácido clavulánico por amigdalitis diagnosticada hace 4 días. En la exploración se observan signos meníngeos, y en las
exploraciones complementarias un líquido cefalorraquídeo (LCR) con 2.840 leucocitos/μ l (60 % polimorfonucleares); hemograma con
21.300 leucocitos/μ l (77 % granulocitos) y velocidad de sedimentación globular (VSG), 35 mm/h. Se diagnostica de meningitis
bacteriana y se inicia tratamiento con penicilina intravenosa.

PREGUNTA
Cual es la conducta farmacológica a seguir.

RESPUESTA
a.- Cefotaxima, vancomicina y metronidazol.
b.- Vancomicina, metronidazol y ampicilina.
c.- Ampicilina, metronidazol y amikacina.
d.- Gentamicina, dicloxacilina y metronidazol.

FARINGITIS, FARINGOAMIGDALITIS (FA). CIENCIAS BASICAS: La faringitis es un proceso inflamatorio causado por diferentes
microorganismos de observación frecuente en la edad pediátrica, que involucra las membranas adyacentes de la faringe en forma
aguda o crónica; en el 50 a 60 % de los casos puede identificarse el agente causal. En la mayoría, la etiología es viral. Debería reservarse
el término “faringitis” en sentido estricto a la infección primaria viral o
bacteriana, circunscrita a la faringe, y no a la hiperemia faríngea que se
produce en el transcurso de un cuadro catarral vírico de vías altas
(rinofaringitis aguda). Es recomendable unificar y describir simplemente
como faringitis aguda una patología que recibe distintos nombres
(amigdalitis, tonsilitis, FA, etc) y que no es más que la misma enfermedad.
SALUD PUBLICA: Debemos tener en cuenta el alto costo en salud que esta
patología conlleva y el alta tasa en el uso de antibióticos, muchas veces
innecesarios. Afecta fundamentalmente a niños en edad escolar, 5-10 años.
Es más prevalente en climas fríos o templados y en los periodos de invierno y primavera. La transmisión es por contacto estrecho
persona-persona a través de las secreciones. El 65-80% de las FA agudas tienen una etiología viral, y ocurre como parte de un catarro
de la vía aérea superior que afecta, por lo general, a niños menores de 3 años. PATOGENIA: En la mayoría de los casos, el contagio se
produce por contacto directo a través de la inoculación de gotas o transmisión física de secreciones respiratorias infectadas. Los

CURSO ENARM CMN SIGLO XXI TEL: 36246001 Pharmed Solutions Institute PÁGINA 429
MANUAL DE TRABAJO DEL CURSO ENARM CMN SIGLO XXI
posibles agentes causales de faringitis son múltiples y los podemos diferenciar en: A) VIRALES: Inespecífica suele tener un inicio
gradual, con fiebre moderada, faringodinia, tos irritativa de intensidad variable, poca afección del estado general, hiperemia variable, el
cuadro suele resolverse en 3-6 días. A veces puede ser sugestivo de infección especifica: los rino, corona, adeno y virus sincicial
respiratorio (VSR) son los más frecuentemente asociados a cuadros de resfrío común, con malestar general, cefaleas, mialgias y
conjuntivitis; los virus coxsackie y echovirus son los causales de la herpangina, caracterizada por la presencia de pequeñas vesículas (1-2
mm) sobre el paladar blando, la úvula y los pilares amigdalinos anteriores, y acompañada de un síndrome febril con odinofagia intensa,
por lo general, en niños de corta edad. Puede asociarse a lesiones vesiculares con localización en manos y pies del tipo del denominado
síndrome mano-pie-boca. La infección por Herpes simple tipos 1-2 presenta lesiones vesiculosos o ulcerosas en paladar, configurando
una gingivoestomatitis, adenopatía regional dolorosa y fiebre. En los pacientes con trastornos inmunológicos se puede observar como
enfermedad mucocutánea crónica. El VEB, causante de la mononucleosis infecciosa, cursa en la mitad de los casos con una faringitis
exudativa acompañada de fiebre, decaimiento, cefalea, poliadenopatías no dolorosas y esplenomegalia que nos hace presumir el
diagnóstico. La faringitis puede ser un elemento característico de una primoinfección por el VIH; suele estar acompañada de
linfadenopatías, a veces de ulceraciones en las mucosas y síntomas generales como fiebre mialgias, artralgias e hipersomnia. B)
BACTERIANAS: La presencia de rinorrea, tos, ronquera, diarrea, conjuntivitis o la edad inferior a 3 años hacen poco probable la etiología
bacteriana. La difteria es una enfermedad infrecuente en esta época, dada la efectividad de la vacuna existente y la elevada cobertura
lograda en la población. De todas maneras, hay que pensar en esta etiología ante la aparición de una faringitis membranosa de color
blanquecino claro a oscuro firmemente adherida a la mucosa amigdalina y faríngea, acompañada de un cuadro tóxico-infeccioso en un
paciente con un esquema incompleto de vacunación. La faringitis aguda por Neisseria gonorrhoae ha aumentado en años recientes; en
la mayoría de los casos, la infección es asintomática y afecta a la población sexualmente activa. Cuando se aisla este germen en
menores de edad, hay que pensar en la posibilidad de abuso sexual. En adolescentes, debemos investigar otras enfermedades de
transmisión sexual. Las cepas de estreptococos de los grupos C y G tienen un comportamiento similar a los del grupo A y se asocian
fundamentalmente a brotes epidémicos de origen alimentario común (los huevos hervidos fríos están reconocidos como un vehículo
importante). En la faringitis por anaerobios, el compromiso por flora mixta (anaerobios y espiroquetas) es poco frecuente y suele
producir un exudado purulento con aliento fétido. En la forma invasiva periamigdalina puede haber compromiso vascular
(tromboflebitis séptica de la vena yugular); afecta con mayor frecuencia a los adolescentes o adultos jóvenes. El Arcanobacterium
haemolyticum produce una faringitis exudativa con erupción escarlatiniforme pruriginosa en extremidades, cuello y espalda; no
compromete palmas, plantas, cara, abdomen o nalgas y afecta con mayor frecuencia a los adolescentes, es sensible a eritromicina.
Dentro de las infecciones bacterianas, la más frecuente (15-25%) corresponde al ESTREPTOCOCO B HEMOLÍTICO DEL GRUPO A
(EBHGA) o Streptococcus pyogenes: Se lo puede aislar durante todo el año aunque presenta 2 brotes anuales en otoño y primavera. La
faringitis es de aparición aguda; afecta mayormente a los niños entre los 5 a 15 años, cursa con fiebre elevada (>39°C), odinofagia
(intensa), dolor abdominal (por adenitis mesentérica) con náuseas y vómitos, adenopatías cervicales y un exudado que cubre la faringe
posterior y el área amigdalina con edema de úvula, a veces pronunciado, exantema escarlatiniforme o ambiente epidemiológico
positivo. La infección con cepas capaces de producir toxina eritrogénica provoca erupción eritematosa característica de la piel
denominada escarlatina. Esta misma toxina en huéspedes comprometidos por otras infecciones, como varicela, puede producir el grave
cuadro del shock tóxico estreptocócico, con un riesgo de mortalidad del 50 %. Se puede presentar frecuentemente impétigo, celulitis.
Menos frecuentes sonneumonia, osteomielitis, meningitis, fascitis necrotizante, las complicaciones supurativas: adenoflemón, adenitis
supurada, absceso retrofaringeo, mastoiditis, mediastinitis. Las complicaciones no supurativas: fiebre reumática (FR), el riesgo de
desarrollar durante una epidemia de EBHA en pacientes no tratados es aproximadamente del 3%, la artritis posestreptocóccica
(pacientes con artritis y evidencia de infección estreptocóccica que no cumplen con los criterios para FR) y la glomerulonefritis aguda
posestreptocóccica (GNA) está más relacionada a cepas nefritógenas, asociadas con mayor frecuencia a infecciones de piel. La
transmisión de la enfermedad: la diseminación del germen a los convivientes con el caso índice no tratado oscila entre el 25 al 35%.
DIAGNÓSTICO: En la mayoría de los casos no es posible realizar un diagnóstico etiológico solamente sobre la base de datos clínicos, el
examen físico de las estructuras faríngeas debe ser minucioso, buscando elementos clinicos que puedan ayudar al diagnóstico. El
cultivo de fauces en placas de agar sangre es el paso diagnóstico definitivo (gold estándar), debe tomarse la muestra de faringe y
amígdalas y, si no puede sembrarse inmediatamente, puede quedar el hisopo seco en un tubo estéril a temperatura ambiente por 24
horas hasta su siembra en los medios correspondientes. La sensibilidad de los cultivos oscila entre el 73 al 100%. Es conveniente
examinar los cultivos hasta las 48 horas posteriores a su siembra para poder determinar su verdadera negatividad. Actualmente, los
métodos rápidos de detección de antígeno estreptocócico son un adelanto en el diagnóstico de faringitis. Las pruebas que emplean el
sistema de aglutinación de látex son muy específicas (95-97%) y moderadamente sensibles (75 al 90%) en comparación con el cultivo de
fauces, y se requiere de una técnica relativamente sencilla, de bajo costo y de información rápida. Las pruebas de inmunoensayo
enzimático de fase sólida, inmunoensayo óptico y sondas de DNA dan resultados similares aunque requieren de un laboratorio
especializado y son más costosas. Dado su buen rendimiento, las pruebas de detección de antígenos nos permiten, ante un resultado
positivo, establecer el diagnóstico de faringitis estreptocócica e iniciar el tratamiento. La faringitis estreptocócica es rara antes de los 3
años y excepcional antes del año. Sin embargo hay que recordar que niños que asisten a guardería pueden presentar a partir de los 18
meses, especialmente en brotes epidémicos. El empleo de los métodos rápidos es también de utilidad para evitar la presión ejercida
con frecuencia en la indicación de antibióticos para facilitar el regreso de los niños a sus actividades habituales. Se ha demostrado que
el uso de los métodos rápidos en áreas de emergencia ha logrado una reducción significativa en la prescripción de antibióticos en
faringitis del 70% y en un 93% en aquéllos con cultivos negativos. A veces es necesaria la realización de otros análisis para confirmar el
diagnóstico etiológico: cultivos especiales cuando se piensa en gonococo (medios de Thayer-Martin), difteria (medio de Loeffler),
tinciones especiales cuando se piensa en espiroquetas o fuso-bacterias, realización de hemocultivos ante cuadros severos de sepsis,
pruebas serológicas específicas ante la sospecha de EBV, CMV, VIH, búsqueda de antígenos, PCR, cuerpos de inclusión o en cultivo de
células. TRATAMIENTO: El tratamiento debe estar relacionado con el agente etiológico aislado. FA viral: Sintomático con paracetamol o
naproxen de 3 a 5dias. Ante la enfermedad herpética en el paciente inmunosuprimido, se puede realizar un tratamiento por vía oral
con aciclovir a200 mg, en 5 tomas por día, por 7 días. No es recomendable el tratamiento de un primer episodio agudo con aciclovir en
un huésped normal. El mismo planteo se puede aplicar para la Influenza A con el uso de Amantadina 6 mg/kg /día por vía oral, o
inhibidores de la neuraminidasa. Faringitis por EBHA: Antibiótico de elección sigue siendo la penicilina V (fenoximetilpenicilina potásica)

CURSO ENARM CMN SIGLO XXI TEL: 36246001 Pharmed Solutions Institute PÁGINA 430
MANUAL DE TRABAJO DEL CURSO ENARM CMN SIGLO XXI
VO por 10 días <12 años: 250mg c/12 hrs, >12 años: 500mg c/12 hrs. Peniclina G benzatina IM profunda <12 años 600000U, >12 años
1200000 U (hasta 27 kg de peso se indican 600.000 U; más de 27 kg se indican 1.200.000 u. Es muy importante que su aplicación sea
realizada por personal idóneo y en niños mayores de 2 años). Amoxicilina VO por 10 días 40-50mg/kg/día c/12 o 24 hrs (máximo
500mgs c/12hrs). Ninguno erradica el 100% de los casos de faringitis por EBHA. No deberemos indicar antibióticos sin confirmar el
diagnóstico. Si por alguna razón se inicia el tratamiento hasta tener confirmación de los cultivos, debemos suspenderlo si fueran
negativos. No hay ninguna evidencia de que una terapia precoz disminuya la recurrencia. Es importante también lograr una adherencia
a cumplir el tratamiento instituido. En pacientes alérgicos a penicilina, el estolato de eritromicina a una dosis de 40 mg/kg/día, en 3 ó 4
dosis diarias, cefadroxilo 30mg/kg/día c/12 hrs por 10 días. Azitromicina 10mg/kg/dia por 5 dias p 20 mg/kg/día por 3 días (máximo
500mgs dosis). Resistencia a macrolidos clindamicina 20mg/kg/día c/12 hrs por 10 días. En la actualidad, sólo azitromicina, cefadroxilo,
cefixima y cefdinir está aprobada por la FDA como tratamiento una vez al día para faringitis por Streptococus en niños. Los episodios
causados por Streptococcus beta hemolítico de los grupos C y G requieren del mismo tratamiento antibiótico. Los relacionados con el
Arcanobacterium haemolyticum se pueden tratar con penicilina o macrólidos. Con respecto a este grupo de pacientes portadores de
EBHA en contacto íntimo con convivientes con FR o aquéllos que padecen esta enfermedad y son difíciles de erradicar, se pueden
plantear esquemas alternativos de antibióticos, aunque se sabe que este grupo, por características propias, tiene menor riesgo de
desencadenar la enfermedad. En estado portador asintomático no existe riesgo de complicaciones (FR o GNA) y no se requiere
tratamiento. No hay urgencia en iniciar un tratamiento antibiótico ante un caso de faringitis con sospecha de ser por EBHGA, dado
puede demorarse hasta 9 días de comenzado los síntomas, para evitar la complicación no supurativa del Streptococcus pyogenes. En las
faringitis por anaerobios, la droga de elección es la penicilina y puede ser administrada por vía endovenosa u oral de acuerdo a las
circunstancias y necesidades del paciente; otros esquemas pueden ser propuestos, como clindamicina, ampicilina-sulbactam. Puede ser
necesario que paciente requiera un tratamiento quirúrgico combinado, por ejemplo, de un absceso periamigdalino. La difteria y el
gonococo tienen tratamiento específico. En la primera deberá indicarse antitoxina diftérica y el antibiótico correspondiente. El
tratamiento quirúrgico (amigdalectomía) es una indicación excepcional y se puede plantear ante la obstrucción de la vía aérea, absceso
periamigdalino (por su alta recurrencia 20%) y discutido en FR y amigdalitis crónica. De realizarse extirpación de la amígdala, es
importante su envío para completar el estudio anatomopatológico a fin de poder descartar otras etiologías. Es importante diferenciar el
agente etiológico ante la posibilidad de realizar un tratamiento específico para evitar así las complicaciones supurativas y las no
supurativas (fiebre reumática y glomerulonefritis).

CASO CLINICO
Paciente femenino de 24 meses de edad el cual inicia súbitamente padecimiento caracterizado por irritabilidad, disminución en la
ingesta de alimento, dificultad para dormir, leve cianosis peribucal principalmente con el llanto, a la exploración física se observa
estridor con respiración superficial y dificultad respiratoria, presentando vomito durante la revisión.

PREGUNTA
Cuál es la conducta a seguir.

RESPUESTA
a.- Indica antibiótico de amplio espectro y desinflamatorio.
b.- Indica manejo ambulatorio y signos de alarma.
c.- Suministra primera dosis y envía a segundo nivel.
d.- Sumistra oxigeno, antibióticos y antipirético.

PREGUNTA
El paciente presenta leve mejoría sin embargo durante las siguientes 24 horas disminuye volumen urinario mas vomito recurrente, se
maneja restitución de liquidos sin respuesta adecuada, cual es la manifestación mas importante para envio a segundo nivel.

RESPUESTA
a.- Oliguria.
b.- Vomito.
c.- Examantema.
d.- Fiebre persistente.

CASO CLINICO
Se trata de masculino de 7 años de edad el cual cuenta con antecedentes de cuadros repetidos infecciones de vías superiores altas, el
cual inicia con dolor faríngeo de inicio súbito, con fiebre no cuantificada refiere la madre y cefalea, a la exploración física se observa mal
estado general, exudado amigdalino e hiperemia.

PREGUNTA
Cual es la conducta a seguir.

RESPUESTA
a.- Medidas generales y antipirético.
b.- Realizar exudado faríngeo.
c.- Determinación de antiestreptolisinas.
d.- Penicilina benzatinica 1200,000 UI dosis única.

PREGUNTA

CURSO ENARM CMN SIGLO XXI TEL: 36246001 Pharmed Solutions Institute PÁGINA 431
MANUAL DE TRABAJO DEL CURSO ENARM CMN SIGLO XXI
Cual de los criterios no es por Centor para el diagnostico de faringitis estreptocócica.

RESPUESTA
a.- Ausencia de tos.
b.- Linfadenitis.
c.- Exudado amigdalino.
d.- Inicio súbito.

CASO CLINICO
Niña de 5 años acude al servicio por presentar fiebre (38,6 °C), odinofagia, trismus leve, rechazo alimentario, dolor y leve tumefacción
latero-cervical izquierda y cierta dificultad a la movilización cervical. No presentaba signos de disnea. El cuadro empezó cuatro días
antes. Se conoce alérgica a penicilina, portador de tetraologia de fallot, acompañado de retrazo mental.

PREGUNTA
Cual es la conducta a seguir.

RESPUESTA
a.- Amoxicilina- clavulanico.
b.- Trimetropima + sulfametoxazol.
c.- Clindamicina.
d.- Cefepime.

EPIGLOTITIS. CIENCIAS BASICAS: También llamada supraglotitis es una celulitis bacteriana de la supraglotis, que puede obstruir
completamente la vía aérea. Se reconoce como una entidad rápidamente progresiva, que pone en peligro la vida y que afecta
predominantemente a niños. SALUD PUBLICA: Clásicamente se presenta en niños de 2-6 años, sin embargo se puede presentar a
cualquier edad, Incluso en neonatos. Existe una mayor incidencia en invierno y primavera. PATOGENIA: El Haemophilus influenzae tipo
B es el microorganismo identificado en el 90% de los casos. Esta bacteria puede ser parte de la flora normal de la nasofaringe o puede
ser adquirido mediante transmisión respiratoria. Otros patógenos aislados son el estreptococo B hemolítico de los grupos A y B,
estafilococos, neumococos, Klebsiella, Haemophilus parainfluenzae, Pseudomonas y Candida. Cuando el edema aumenta como
resultado del infiltrado inflamatorio difuso, la vía aérea se obstruye parcialmente, pero el moco puede obstruirla completamente. Niño
grave con aspecto toxico. La progresión es rápida en término de horas. DIAGNOSTICO: Clínica; los 3 signos más comunes de
supraglotitis son fiebre, dificultad respiratoria e irritabilidad. El paciente muestra estridor inspiratorio, retracciones, sialorrea, además
de odinofagia y disfagia. Generalmente no cursan con disfonía, ni tos. El estridor se presenta cuando la obstrucción de la vía aérea es
casi completa. Asumen una posición apoyados hacia delante y como olfateando (“posición en trípode”). La aspiración de secreciones en
la vía aérea ya comprometida puede desencadenar laringoespasmo. El diagnostico de epiglotitis se hace mediante observación directa
de la supraglotis, en un ambiente controlado, generalmente en quirófano. Laboratorio, se puede tomar BH, en la que se observa
leucositosis severa con neutrofilia. La utilización de radiografías es controversial. Aunque no se cuestiona que la supraglotis, puede ser
diagnosticada en una placa lateral de cuello (signo del pulgar), la pregunta es si realmente es un estudio necesario y sobre todo seguro.
TRATAMIENTO: Si se sospecha de una supraglotitis, el primer paso es evitar la estimulación del niño. Una vez que se establece el
diagnostico hay que asegurar la vía aérea. Por muchos años se realizaba una traqueotomía, en cuanto se diagnosticaba al paciente.
Durante los últimos 20 años la intubación endotraqueal ha sido el método estándar de manejar la supraglotitis aguda. Se recomiendan
la intubación de rutina en todos los pacientes con epiglotitis; en el momento de la intubación se deben obtener muestras de la epiglotis
para cultivo y tomar hemocultivos. Se debe iniciar antibiótico intravenoso; se maneja con ampicilina y cloranfenicol. Se recomienda
como 2 línea de elección el ceftriaxone (100 mg/kg/día), cefotaxime (100 mg/kg/día) o ampicilina-sulbactam (200 mg/kg/día); la
duración del tratamiento es usualmente de 10 a 14 días, complementado con antibióticos orales. El tratamiento con corticoesteroide es
copntroversial. Con el tratamiento adecuado el edema de la supraglotis cede dentro de las 48-72 hrs, por lo que se puede extubar al
paciente casi siempre al tercer día. COMPLICACIONES: La neumonía la más frecuente y, en menor porcentaje, meningitis, adenitis
cervical, pericarditis, artritis séptica y otitis media. Pueden presentarse complicaciones secundarias a la hipoxia, como daño en el SNC,
la liberación de la obstrucción súbita de la vía aérea puede producir edema pulmonar.

CASO CLINICO
Niña de 4 años, previamente sana, con fiebre y síntomas catarrales de 24 horas de evolución. Los padres la encuentran con quejido
respiratorio, a la exploración clínica revela tiros intercostales, cianosis peribucal, respiración superficial, sibilancias audibles a distancia y
estado de alerta disminuido, llanto débil pero disfonico.

PREGUNTA
Cual es la gravedad de la obstrucción de la via aérea.

RESPUESTA
a.- Leve.
b.- Moderada.
c.- Grave.
d.- Severa.

PREGUNTA
Considerando el cuadro clínico cual es la patologia mas frecuente para establecer un diagnostico diferencial.

CURSO ENARM CMN SIGLO XXI TEL: 36246001 Pharmed Solutions Institute PÁGINA 432
MANUAL DE TRABAJO DEL CURSO ENARM CMN SIGLO XXI

RESPUESTA
a.- Cuerpo extraño.
b.- Difteria laríngea.
c.- Epiglotitis post-vacunacion.
d.- Traqueitis bacteriana.

PREGUNTA
Cuál es la conducta a seguir.

RESPUESTA
a.- Iniciar tratamiento farmacológico.
b.- Administración de oxigeno.
c.- Traslado a hospital.
d.- Administracion de prednisona.

LARINGOTRAQUEOBRONQUITIS (CRUP). CIENCIAS BASICAS: Es un síndrome respiratorio clínico agudo, de inicio súbito, de
presentación en edad pediátrica. Caracterizado por la tríada estridor laríngeo inspiratorio, tos ronca o perruna (traqueal), y disfonía.
Hay signos de dificultad respiratoria secundario a la obstrucción laríngea o traqueal; término usado casi exclusivamente para referirse a
la laringotraqueítis de origen viral, frecuentemente precedida de un episodio de coriza, rinorrea clara y leve aumento de la
temperatura. Se considera una enfermedad que se autolimita, sin embargo puede evolucionar a dificultad respiratoria grave. Algunos
niños con historia de atopia, tienen un cuadro no precedido de síntomas virales que se le ha llamado CRUP espasmódico. Ambos son
tratados similarmente siempre y cuando los signos y síntomas sean los mismos. SALUD PUBLICA: Afecta a niños entre 3 meses y 6 años
de edad, con un pico a los 2 años. Más frecuente en sexo masculino. Predomina en otoño e invierno. Con una incidencia anual de 18
por 1.000 niños menores de 6 años de edad. PATOGENIA: Los agentes causales más frecuentes son el virus parainfluenza (65%) tipo 1
(50%), 2 y 3; con menor frecuencia; adenovirus, sincitial respiratorio, influenza A y B, sarampión y excepcionalmente; Mycoplasma,
enterovirus, parotiditis, rinovirus y difteria. El padecimiento inicia en la nasofaringe, se extiende por el epitelio respiratorio de la laringe
y la tráquea. El virus se adhiere directamente a la mucosa, mediante los macrófagos o los linfocitos, provocando degranulación con
liberación de mediadores inflamatorios preformados o de neo formación, que ocasionan inflamación difusa con eritema y edema que
en las cuerdas vocales, disminuye la movilidad y ocasiona disfonía y participa en el estridor; en la tráquea provoca tos ronca. En la
región subglótica que es la porción más estrecha de las vías respiratorias superiores en el niño, con esqueleto cartilaginoso, el edema
provoca disminución del calibre, con dificultad para el flujo aéreo, que se manifiesta por estridor inspiratorio. Si la enfermedad
progresa se forman exudados fibrinosos y pseudomembranas que aumentan la obstrucción. La extensión hacia los bronquios o
alveolos, ocasiona laringotraqueobronquitis, o laringotraqueobronconeumonitis respectivamente, frecuentemente son provocados por
complicación bacteriana de la enfemedad viral inicial. DIAGNOSTICO: Se recomienda realizar el diagnostico de CRUP basados en las
manifestaciones de la triada clínica: disfonía, estridor laríngeo inspiratorio y tos traqueal; aunados a los siguientes antecedentes, inicio
súbito, fase prodrómica de 12-48hrs previas con rinorrea, fiebre y tos no traqueal. Suelen presentarse signos progresivops de dificultad
respiratoria. Puede encontrase disminución del murmullo vesicular por la obstrucción y datos variables de dificultad respiratoria como
agitación por hipoxia, aleteo nasal, tiros supraesternales e intercostales, más severos cianosis, tiros supraclaviculares, inquieto y
ansioso. Generalmente no se requieren estudios de laboratorio; BH leucocitos con elevación moderada con predominio de linfocitos.
Se pueden solicitar cultivos de virus y pruebas serológicas para identificar el virus causal. La radiografía de tórax muestra como dato
característico, estrechamiento de la columna aérea a nivel subglotico y el signo de punta de lápiz o en torre de capilla. CLASIFICACION:
Forbes la cataloga en 4 etapas de acuerdo a gravedad y progresión: Etapa 1; fiebre, disfonía, tos traqueal y estridor laríngeo
inspiratorio al explorar al paciente. Etapa 2; Estridor respiratorio continuo y signos de insuficiencia respiratoria. Etapa 3;
manifestaciones clínicas de hipoxia e hipercapnia: inquietud, ansiedad, palidez, diaforesis y taquipnea. Etapa 4; cianosis intermitente,
cianosis permanente y paro respiratorio. Con el objetivo de iniciar rápidamente el tratamiento, es conveniente calificar la gravedad de
la enfermedad. Con la escala del cuadro adjunto. También se puede utilizar la siguiente valoración: LEVE: tos traqueal intermitente,
estridor solo cuando se agita, no en reposo, taquipnea menor de 40/min y taquicardia menor 100/min. No hay inquietud, el niño esta
hidratado y su estado mental es normal. MODERADA: Estridor audible en reposo, que aumenta cuando se agita, tos traqueal constante,
aumento del trabajo respiratorio con tiros, frecuencia cardiaca >100/min y respiratoria >40/min, pero menor de 50. Puede mostrase
exigente, pero se encuentra alerta y es confortado por sus padres. SEVERA: Se agrega al cuadro clínico anterior hipoxia con cianosis,
gran inquietud y alteraciones de la conciencia con empeoramiento de las manifestaciones de insuficiencia respiratoria. TRATAMIENTO:
En 90% de los casos es ambulatorio, menos de 10% se hospitaliza, apenas 1% llega a requerir apoyo ventilatorio. 1. Medidas generales:
reposo ambiente tranquilo, control de la temperatura, hidratación adecuada. 2. Vía aérea permeable: la manera tradicional efectiva es
humedecer el ambiente. El ambiente húmedo es tan efectivo como la nebulización fría, este último es más seguro evita quemaduras
por el agua caliente. La nebulización fría, humedece la secreciones, produce vasoconstricción que ayuda a desinflamar la mucosa,
también disminuye la viscosidad de las secreciones mucosas de tráquea y bronquios. Se recomienda agregar oxígeno al 30-40% para
prevenir o tratar la hipoxia. 3. Epinefrina (adrenalina): Se administra por nebulización simple, ha hecho que en la actualidad la
traqueotomía no sea necesaria. Se cree que su mecanismo de acción, es la estimulación de los receptores α adrenérgicos con la
subsecuente vasoconstricción de los capilares arteriales, lo que da lugar a reabsorción de líquido – en lugar de fuga-, a partir del espacio
intersticial, con la disminución consecuente del edema laríngeo y de secreciones mucosas de tráquea y bronquios. Debe vigilarse la
aparición de taquicardia e hipotensión. Deberá administrase en un servicio de urgencias solo a niños severamente enfermos (>5); se
emplea al 2.25% en 3 ml de solución salina normal durante 20 min, 0.25 ml en niños con peso de <20Kg, 0.5ml si peso 21-39Kg y 0.75ml
si el peso es >40Kg. 4. Corticoesteroide: La dexametasona es el más aceptado y empleado, tiene acción prolongada (V ½ de 36-72 hrs).
Su efecto antiinflamtorio disminuye el edema de la mucosa laríngea entre 2-6 hrs después de su administración. La dosis terapeutioca
varia de 0.15-0.6 mg/kg IM en una sola dosis. O.6mg/kg es la más efectiva, sin pasar de 10 mg como dosis total. 5. Intubacion

CURSO ENARM CMN SIGLO XXI TEL: 36246001 Pharmed Solutions Institute PÁGINA 433
MANUAL DE TRABAJO DEL CURSO ENARM CMN SIGLO XXI
endotraqueal: si no hay respuesta a las medidas anteriores, y se observa obstrucción progresiva de la via aérea, con signos de fatiga en
el niño por trabajo respiratorio laborioso que sugiera hipoxia o hipercapnia y alteración de la conciencia. Los antibióticos no están
indicados, solo están reservados en aquellos pacientes que se sospeche una sobreinfección bacteriana, una laringotraqueitis bacteriana
o una laringotraqueobronconeumonitis, y se debe cubrir principalmente Staphylococcus aureus, Streptococcus pyogenes,
Streptococcus pneumoniae, y Haemophilus

CASO CLINICO
Ingresa mascilino de 4 años de edad el cual inicia padecimiento súbito con estridor laríngeo inspiratorio, refiere la madre que hace dos
días inicio con tos, rinorrea y fiebre no cuantificada, motivo por el cual acudió a consulta en centro de salud donde fue indicado
tratamiento con analgésico, medidas generales y datos de alarma, agrega que una hora antes del ingreso se incrementaron los
síntomas con dificultad para hablar, inquietud, llanto con disfonía y tos seca.

PREGUNTA
Cuantos criterios clínicos clásicos presenta el paciente para LTA.

RESPUESTA
a.- 1
b.- 2
c.- 3
d.- 4

PREGUNTA
Cual es la conducta a seguir para establecer la diagnostico clinico.

RESPUESTA
a.- Biometria hematica.
b.- Radiografia de cuello.
c.- Administrar corticoides.
d.- Administrar adrenalina.

PREGUNTA
Cual es la conducta terapéutica a seguir.
a.- Administracion de dexametasona .60mg/Kg DU.
b.- Budesonida inhalada.
C.- Fluquitasona.
d.- L-epinefrina nebulizada 1:1000

CASO CLINICO
Se trata de femenino de 22 meses de edad la cual es llevada a urgencias por su madre debido a que la observa que no responde a
estimulos, el único antecedentes es la presencia de tos, fiebre y rinorrea que fue tratada con fármacos sintomáticos, a la exploración
física se observa paciente letárgica, hipotónica, con dificultad respiratoria, estridor laríngeo, palidez distal y cianosis central, FC 69, FR
42.

PREGUNTA
Considerando la sintomatologia cual es el estado de la LTA según los criterios de Westley.

RESPUESTA
a.- Leve.
b.- Moderada.
c.- Severa.
d.- Amenaza la vida.

PREGUNTA
Cual es la conducta a seguir.

RESPUESTA
a.- Iniciar RCP.
b.- Intubacion endotraqueal.
c.- Ventilacion positiva intermitente.
d.- L-epinefrina SC.

CURSO ENARM CMN SIGLO XXI TEL: 36246001 Pharmed Solutions Institute PÁGINA 434
MANUAL DE TRABAJO DEL CURSO ENARM CMN SIGLO XXI

BRONQUITIS. CIENCIAS BASICAS: La bronquitis aguda,


desde un punto de vista fisiopatológico, se refiere a la
inflamación aguda de la mucosa bronquial. Clínicamente, no
está muy bien definida, pero se trata de una infección
respiratoria de etiología viral que no suele durar más de dos
semanas. La mayoría de los autores están de acuerdo en
que la tos, con o sin expectoración, es el síntoma más
comúnmente observado. Es una enfermedad adquirida en
la comunidad autolimitada. SALUD PUBLICA: Afecta a niños
en edad pediátrica, más frecuente en menores de 2 años.
No hay estadísticas exactas. Ocupa la séptima causa de
muerte infantil en el apartado de infecciones respiratorias
junto con la bronquiolitis. PATOGENIA: Los virus
respiratorios: influenza, parainfluenza, virus respiratorio
sincitial son los responsables de la mayoría de los casos de
bronquitis aguda. Otros gérmenes aislados ocasionalmente
son: Bordetella pertusis, parapertusis, Mycoplasma
pneumoniae y Chlamidia pneumoniae. Las bronquitis
ocasionadas por el virus de influenza A, son graves y
frecuentemente producen pandemias. El virus entra por la
nariz o por la boca, se replica en los acúmulos linfáticos del
anillo de Waldeyer; puede provocar viremia y
posteriormente se extiende más abajo llegando a los bronquios, en la mucosa bronquial se adhieren directamente a ella o bien son
transportados por macrófagos, a los cuales pueden destruir o no; estos últimos liberan mediadores inflamatorios ya elaborados y
almacenados en sus granulos o de una nueva elaboración, los que provocan daño a la mucosa, edema, inflamación con disminución de
la motilidad ciliar, aumento en la secreción mucosa y disminución de la luz bronquial, con resistencia al flujo del aire, mayor cuanto
menor es el niño, ya que hay relación directa entre la edad del paciente y el calibre de la luz bronquial. Dependiendo del germen, de su
virulencia, del tamaño del inoculo y de las condiciones del hospedero, pueden ocasionar necrosis de las células superficiales de la
mucosa de las vías respiratorias. El periodo de incubación va de 2-14 días según agente etiológico. DIAGNOSTICO: Eminentemente es
clínico; Estornudos, rinorrea hialina, dolor, ardor faríngeo y fiebre de moderada a severa. La tos es inicialmente seca, ronca, por la
irritación de la mucosa faríngea y traqueal y posteriormente se vuelve húmeda, productiva; puede ser hemetizante y frecuentemente
se acompaña de dolor retroesternal. Al examen físico el paciente está inquieto, febril, se observa la mucosa nasal edematosa y
enrojecida, la faringe hiperemica con moco hialino. A la auscultación se escuchan estertores bronquiales diseminados en ambos
campos pulmonares, es frecuente encontrar roncus y sibilantes. El esputo o las flemas pueden ser transparentes, de color blanco,
amarillo o verdoso; estos cambios de coloración no están relacionados con la sobreinfección bacteriana, sino con la peroxidasa liberada
por los leucocitos presentes por el moco. Por lo tanto, el moco verde no es motivo para instaurar tratamiento antibiótico. La
enfermedad en los niños puede presentarse en tres fases clínicas: 1. Pródromos; 2-3 días, fiebre, estornudos, rinorrea serosa, tos seca
disfonica. 2. Estado; 4-6 días, fiebre, mal estado general y tos productiva. 3. Recuperación; persistencia de la tos que se vuelve
paulatinamente seca y aislada, puede durar hasta 2 semanas. En caso de complicación bacteriana, la enfermedad se prolonga y se
agregan mal estado general, tos seca molesta con dolor retroesternal y dificultad para respirar. Si el germen es Mycoplasma
pneumoniae con frecuencia se presenta neumonía y complicaciones como derrame pleural, absceso pulmonar e insuficiencia
respiratoria grave. Los exámenes de laboratorio solo con fines de estudio o enseñanza, o en caso de complicaciones: BH; leucocitosis
leve con linfocitosis o neutrofilia. La radiografía de tórax es normal o bien muestra engrosamiento de la trama broncovascular y la
imagen específica en caso de complicación. Se puede solicitar cultivo para virus a partir de secreciones faríngeas, también se pueden
realizar pruebas serológicas. Las pruebas de inmunofluorescencia en secreciones nasofaríngeas son altamente sensibles y son útiles en
el diagnóstico rápido. La PCR se ha convertido en el estándar de oro para diagnóstico de infección pop B. pertussis. TRATAMIENTO: Es
sintomático, medidas generales tendientes a mantener buena hidratación, temperatura en límites normales y si la tos es frecuente y
molesta, en primera instancia ambiente húmedo con nebulizador ultrasónico tibio con termostato. Si no se mejora la tos agregar
mucolíticos como el ambroxol que se ha referenciado mejora la motilidad ciliar y facilita la recuperación de la función de la mucosa así
como la eliminación de las secreciones. En casos específicos y graves se recomienda el empleo de antivirales: por ejemplo amantadina
en infecciones por virus de la influenza o ribavirina si esta ocasionado por el virus sincitial respiratorio. Si se sospecha o confirma
infección bacteriana concomitantemente o complicando el cuadro inicial, debe emplearse el antibiótico de elección para el germen de
que se trate, si se sospecha tos ferina (azitromicina), infección por Mycoplasma pneumoniae (un macrólido). Los broncodilatadores
inhalados no están indicados de manera rutinaria, sólo en aquellos casos en los que se asocie broncoespasmo. Sin embargo, hay niños
con hiperreactividad bronquial que presentan un mínimo e incluso no apreciable broncoespasmo a la exploración, pero que responden
de manera espectacular al broncodilatador inhalado, con resolución de la tos.

CASO CLINICO
Masculino de 3 años de edad con síndrome de Down y cardiopatía congénita, es traído a urgencias por cuadro catarral de 7 días de
evolución con tratamiento sintomático el cual mejoro parcialmente pero fue lentamente reagudizado, durante las dos semanas previas
cambio en dos ocaciones de tratamiento, existe el antecedente que sus dos hermanos presentaban cuadro catarral. A las 12 h del
ingreso presenta empeoramiento progresivo de la dificultad respiratoria con aparición de fiebre, secreciones mucopurulentas y tos
continua, taquicardico y taquipnea.

PREGUNTA

CURSO ENARM CMN SIGLO XXI TEL: 36246001 Pharmed Solutions Institute PÁGINA 435
MANUAL DE TRABAJO DEL CURSO ENARM CMN SIGLO XXI
Cual es el agente etiológico mas probable de este caso.

RESPUESTA
a.- Influenza A, influenza B, parainfluenza.
b.- Virus sincitial respiratorio.
c.- Coronavirus, adenovirus y rinovirus.
d.- Bordetella pertusis, Mycoplasma pneumoniae y C pneumoniae.

BRONQUIOLITIS. CIENCIAS BASICAS: Es una inflamación de los bronquiolos, habitualmente de etiología viral, aguda y frecuentemente
contagiosa. Debido a la inflamación de los bronquiolos, se manifiesta por tos, sibilancias, disnea y taquipnea. Por lo general
autolimitada. SALUD PUBLICA: Se presenta en menores de 2 años de edad, el 80% ocurre en menores de 1 año (pico entre 6-8 meses).
Predomina en el sexo masculino. Es más frecuente en invierno, pero puede presentarse a lo largo del año. PATOGENIA: El germen
patógeno más frecuente encontrado en caso de bronquiolitis es el virus sinsitial respiratorio (VSR), hasta en 60% de los casos. El resto
es ocasionado por parainfluenza, adenovirus, rinovirus y el virus de la influenza. También puede haber coexistencia entre VSR con
Chlamydia trachomatis Y Mycoplasma pneumoniae, así como virus diferentes a los ya señalados, los que en conjunto constituyen entre
5-10%. El VSR se introduce al organismo principalmente por contacto directo (se extienden hasta 2 m) con secreciones nasales de
enfermos; quien elimina virus hasta por 10dias después del inicio del padecimiento. En gotas grandes el virus puede sobrevivir y
mantener contaminantes las superficies en las que se encuentre, su periodo de incubación es de 4-6 días. Afecta el sistema respiratorio
por colonización y replicación en la mucosa de los bronquios. En los niños pequeños los bronquiolos son el sitio primeramente
afectado, mientras que en niños mayores y adultos, se afectan las vías respiratorias superiores. Hay necrosis de las células ciliadas con
proliferación de células no ciliadas como linfocitos, células plasmáticas y macrófagos en las áreas peribronquiales; se liberan
mediadores inflamatorios, lo que ocasiona edema, inflamación y congestión de la submucosa con taponamiento de los bronquiolos por
moco y detritus celulares; que en conjunto ocasionan estrechamiento de las vías respiratorias periféricas. La respuesta pulmonar a
estos cambios, es un aumento en la capacidad residual funcional, aumento en la resistencia al paso de aire y aumento en los
cortocircuitos pulmonares. Todos estos factores aumentan el trabajo respiratorio dando lugar a alteraciones en el recambio de gases
secundario a obstrucción y atelectasia de las vías aéreas. También puede presentarse hipoxia y retención de CO 2, secundarios a la
alteración de la relación ventilación/flujo, junto con hipovetilación. DIAGNOSTICO: Es clínico, se presentan con antecedente de 1-5 días
con rinorrea profusa habitualmente hialina y congestión nasal, en algunos tos y fiebre leve a moderada. Al 4-5 día se agrega dificultad
para respirar con tos húmeda, taquicardia y sibilancias. El niño se muestra irritable y letárgico; en los muy pequeños se puede presentar
apnea, la enfermedad progresa con insuficiencia respiratoria grave (cianosis, actividad de músculos accesorios de la respiración, disnea
y taquipnea, más grave quejido, aleteo y tiro supraesternal) y ameritan intubación. En cuadros severos y con varios días de evolución
puede haber deshidratación. También hay signos de hiperinflación (sobre distensión pulmonar), manifestados por aumento del
diámetro anteroposterior del tórax y crecimiento de hígado y bazo que se palpan abajo del borde costal. A la auscultación se pueden oír
sibilancias inspiratorias, disminución de la entrada de aire y estertores inspiratorios. Datos de gravedad por Shaw, para predecir
evolución y tratamiento: 1. Aspecto toxico. 2. SaO2 <95%. 3. Edad gestacional menos de 34semanas. 4. Frecuencia respiratoria >70/min.
5. Atelectasia en la radiografía de tórax. 6. Edad menor de 3 meses. Por otro lado se puede hacer la determinación de PaCO2,
habitualmente se encuentra entre 30-35mm Hg. Valores entre 45-55mmHg son peligrosos e indican falla respiratoria inminente. El
criterio de oro para el diagnóstico es el cultivo nasofaríngeo positivo para VSR. Existen pruebas rápidas que incluyen ELISA y tinción
directa de anticuerpos fluorescentes, son más rápidas y tienen especificidad y sensibilidad cercana al 90%. La serología para
anticuerpos del virus tiene limitaciones. La determinación de leucocitos y electrolitos solo debe hacerse si hay deshidratación o se
sospecha sepsis. La gasometría se indica para valorar severidad de compromiso respiratorio. La radiografía de tórax no es necesaria,
puede ser útil en los siguientes casos: insuficiencia respiratoria leve a moderada, sibilancias recurrentes, aspecto toxico. La imagen
puede mostrar horizontalización variable de las costillas, con aumento del espacio intercostal, diafragmas abatidos, hiperinflación
(hiperclaridad), atelectasia, engrosamiento peribronquial e infiltrados intersticiales difusos. Aprox 10% de las placas son normales y
puede no haber correlación con la severidad de la enfermedad. TRATAMIENTO: Primero valoración de la gravedad. Las medidas más
importantes son la hidratación (si hay deshidratación o hay bajo nivel de eliminación urinaria) y oxigenoterapia por puntas nasales,
mascarilla o casco cefálico, debe medirse con frecuencia la saturación de O2 con oximetro de pulso para adecuar la cantidad de O2 a
administrar. La aplicación de nebulización de epinefrina racemica (mezcla de isómeros d y L) es aceptada, mejora la saturación de O2 y
los datos clínicos de insuficiencia respiratoria, aunque en muchos países no está disponible por lo que se usa la adrenalina (L-
epinefrina), con los mismos resultados, solo se usa en niños severamente enfermos y en quienes es inminente la intubación traqueal. La
dosis recomendad es epinefrina racemica al 2.25% en 3 ml de sol. Salina normal durante 20 min. En niños con peso de <20Kg, 0.5ml si
peso 21-39Kg y 0.75ml si el peso es >40Kg (0.5ml de epinefrina racemica equivalen a 5ml de L-epinefrina). El empleo de esteroides y β-
agonistas en estos pequeños no es aconsejable a luz de múltiples reportes. El único antiviral acepado para este padecimiento es la
ribavirina, el cual tienen actividad virostatica, se recomienda en niños muy pequeños, inmunodeprimidos, con malformaciones
cardiacas y los severamente enfermos, solo se emplea en inhalación y el personal de salud debe protegerse de aspirar el medicamento
ya que se han descrito crisis de espasmo bronquial y malformaciones congénitas. Hay reportes de empleo de factor surfactante,
oxigenación por membrana extracorpórea (OMEC) y vitamina A.

CASO CLINICO
Se trata de masculino de 6 meses de edad con peso adecuado, con antecentes, con diagnóstico de reflujo gastroesofágico, en
tratamiento con Domperidona y Omeprazol. Vacunas completas. Antecedentes familiares: Padre con antecedente de broncoespasmo y
atopía. Comenzó 5 días previos al ingreso con tos y dificultad respiratoria, por lo cual se evaluó en la guardia de otro hospital,
indicándose tratamiento ambulatorio con broncodilatadores cada 4 hs. y Prednisona durante 5 días. El paciente evolucionó sin mejoría
clínica, intensificándose la dificultad respiratoria durante las últimas 24 hs. y se agregó regular actitud alimentaria. No presentó fiebre.
Consultó en la guardia, donde se lo encontró taquipneico, (fr) 70 por minuto (pm), con (fc) de 145 pm, Sat 91% aire ambiente (aa), tiraje
subcostal e intercostal, regular entrada de aire bilateral y sibilancias audibles.

CURSO ENARM CMN SIGLO XXI TEL: 36246001 Pharmed Solutions Institute PÁGINA 436
MANUAL DE TRABAJO DEL CURSO ENARM CMN SIGLO XXI
PREGUNTA
Cual es la conducta a seguir inmediata a seguir es la menos adecuada?

RESPUESTA
a.- Nebulizaciones con broncodilatadores
b.- Hidrocortisona a 10 mg/kg.
c.- Monitorizacion de saturación periférica.
d.- Evaluación de gases arteriales.

BRONCONEUMONIA. CIENCIAS BASICAS: El término es introducido en 1837 por Seiffert, a través de la idea de infiltrados pulmonares
de origen broncogénico o bronquiologénico. Se ha relacionado con neumonía multifocal (o lobulillar), ahora frecuentemente llamada
de focos múltiples. Infección de la vía respiratoria producida por un microorganismo (bacteria o virus). Es una infección de inicio
violento y repentino que produce inflamación en pulmones y bronquios, la cual genera trastornos respiratorios que si no son tratados
oportunamente pueden conducir a la muerte. Es ocasionada por virus o bacterias y afecta particularmente a niños, personas de edad
avanzada y a pacientes con daño en su sistema inmunológico (inmunosuprimidos). SALUD PUBLICA: Los bebés que tienen entre 3-6
meses de edad son quienes resultan más comúnmente afectados, si bien esta enfermedad puede aparecer hasta los dos años. Se
calcula que aproximadamente uno de cada tres bebés tendrá bronconeumonía en algún momento durante su primer año de vida. Es
más frecuente durante los meses de invierno. PATOGENIA: La bronconeumonía es una lesión secundaria que aparece generalmente
como complicación de una enfermedad (gripe, resfriados mal cuidados, bronquitis, bronquiolitis o tos ferina). A diferencia de la
neumonía, no posee fases evolutivas y el exudado no contiene fibrina o tiene muy poca. La bronconeumonía es causada por uno de
varios tipos de virus. En tres cuartos de los casos, es provocada por el virus sincitial respiratorio, por contacto directo con la tos de un
enfermo. Tumores pulmonares que evitan la ventilación adecuada del aire a los pulmones, los cuales favorecen el establecimiento de
bacterias. La característica dominante de la bronconeumonía es la consolidación parcheada del pulmón. DIAGNOSTICO: Clínica; Cuadro
febril sin otro síntoma, o bien como dolor de tórax, o abdomen. Tos, decaimiento, inapetencia o rechazo al alimento, cuadro de resfrío
que empeora progresivamente, insuficiencia respiratoria severa, taquipnea, hundimiento de costillas al respirar, aleteo nasal, quejido,
retracción xifoidea. Anatomía patológica: Las lesiones bronconeumónicas consisten en focos de condensación pequeños, a veces,
confluentes, gris rojizos, secos, finamente granulosos. Frecuentemente, sin embargo, son poco notorios macroscópicamente y se
manifiestan como zonas ligeramente levantadas, hiperémicas, que se descubren mejor por palpación que por inspección. Estos focos
pueden pasar inadvertidos macroscópicamente, no así en el examen del pulmón fijado previamente. Los focos bronconeumónicos se
encuentran frecuentemente en las regiones dorso-basales y laterales de los lóbulos inferiores. En el centro del pulmón los focos son
mayores que en la periferia, donde tienden a ser más densos. A menudo alcanzan la pleura, donde se desarrolla entonces una pleuritis
fibrinosa o purulenta. BRONCONEUMONÍA ESTAFILOCÓCICA: Se observa en el 31% de las autopsias. Corresponde al 5% de las
neumonías bacterianas. Mortalidad cercana al 20%. El tipo de reacción inflamatoria es la inflamación necrotizante y abscedante. Es una
afección secundaria a piodermitis, furunculosis, endocarditis, osteomielitis y otras. Los abscesos se forman a partir de embolias sépticas
en arteriolas y capilares. El empiema y el pioneumótorax son acompañantes frecuentes. La neumonía aerógena, primaria o secundaria
a bronquitis viral, se observa como una neumonía con infiltrados purulentos mal delimitados, confluentes con tendencia a la necrosis.
El pronóstico de esta forma es muy malo, especialmente la forma primaria en lactantes. BRONCONEUMONÍA ESTREPTOCÓCICA: Se
caracteriza por un exudado hemorrágico y flegmonoso, pobre en fibrina. En la forma hematógena, se constituyen infiltrados maculares
(bronconeumónicos) y simultáneamente se desarrollan flegmones pleurales, septales y perilobulillares. La forma aerógena es más
frecuente y corresponde en verdad a una bronconeumonía purulenta confluente. La pleuritis purulenta y el empiema son también
frecuentes. BRONCONEUMONIA POR ASPIRACION: También predominan en lóbulos inferiores y más frecuentemente al lado derecho.
Los cuerpos extraños pueden producir una estasis de secreciones bronquiales, heridas de la pared con necrosis y neumonía purulenta
consecutiva, a menudo gangrenosa. La circunstancia más frecuente es la aspiración de vómitos con contenido gástrico, que produce
una inflamación necrotizante, bronquial y alveolar, por la acción corrosiva del ácido. En la agonía se puede producir aspiración de
contenido gástrico y digestión pulmonar post-mortem (neumomalacia ácida). Otra situación frecuente es la aspiración de líquido
amniótico. El feto respira in útero e inhala líquido amniótico normalmente. En la asfixia neonatal aparece meconio en el líquido
amniótico y en el pulmón se observan partículas de vérnix caseoso o de meconio y escamas córneas. Como es un fenómeno normal, la
distinción suele ser difícil y, al final, la diferencia es sólo cuantitativa. Cuando la aspiración es masiva, puede observarse una
bronconeumonía leucocitaria reactiva.

CASO CLINICO
Paciente de 17 dias de vida extrauterina que presenta fiebre, irrabilidad, y ampollas en el cuerpo. EF febril, hidratado, lloroso e
intranquilo, ligeramente taquipneico con exantema de distribución centrípeta en estadios evolutivos de vesículas y pustulas distribuido
en cara, cuello y tronco. Orofaringe congestiva y disminución de murmullo vesicular bilateral en campos pulmonares. Rx de torax:
presencia de infiltrado bronconeumonico bilateral a predominio de hilios y campos inferiores. Laboratorios leucos 11 mil, acidosis
metabolica compensada. VSG 1ah: 14 mm y PCR 0.03 mg/l.

PREGUNTA
Cual es la conducta terapéutica mas adecuada a seguir en el caso?

RESPUESTA
a.- Oxigeno, liquidos y aines.
b.- Oxigeno, liquidos y electrolitos, vigilancia.
c.- Oxigeno, antibióticos, liquidos y aines.
d.- Oxigeno, antibióticos, liquidos y corticoides.

CURSO ENARM CMN SIGLO XXI TEL: 36246001 Pharmed Solutions Institute PÁGINA 437
MANUAL DE TRABAJO DEL CURSO ENARM CMN SIGLO XXI
NEUMONIAS. CIENCIAS BASICAS: Se define como la inflamación y condensación del parénquima pulmonar causada por un agente
infeccioso o por factores no infecciosos como la aspiración de ácido gástrico, cuerpos extraños e hidrocarburos; así como las reacciones
de hipersensibilidad y las neumonitis inducidas por fármacos o radiaciones. El la neumonía existe reemplazo del contenido aéreo de los
alveolos y conductos alveolares por células y exudado inflamatorio, que se manifiesta por la presencia de síntomas y signos de infección
aguda y la presencia de imágenes radiológicas que indican ocupación alveolar. SALUD PUBLICA: En 2002 en México se reportaron 189
806 casos de neumonía. La mayoría de los casos se reportaron en niños de 1-5 años. Su incidencia en este grupo de edad es muy
elevada, y se reportan 10-40 casos: 1000 niños/año. PATOGENIA: Durante los 3 primeros años de vida las neumonías están causadas
por virus, especialmente por el sinsitial respiratorio (VSR) y los virus de la gripe, también virus influenza A y parainfluenza tipo 1. A
partir de los 3 años pueden aislarse Mycoplasma pneumoniae y Chlamydia pneumoniae. Aunque los agentes bacterianos se asocian
menos frecuentemente con neumonías, se ha evidenciado que causan infecciones más graves. Patógenos comunes: Streptococo
pneumoniae, Mycoplasma pneumoniae, Chlamydia pneumoniae, Coxiella burnetii, virus respiratoria. Patógenos “no comunes”:
Legionella pneumophila, Haemophilus influenzae, enterobacterias, flora saprofita bucal, Moraxalla catarrhalis. El evento que con mayor
frecuencia altera los mecanismos de defensa de la vía aérea es una infección viral que altera las propiedades de las secreciones, inhibe
la fagocitosis, modifica la flora bacteriana y puede temporalmente romper de manera temporal al epitelio de la vía aérea. Es común
que se presente una infección viral unos días previos al desarrollo de neumonía bacteriana; sin embargo, su presencia no es
indispensable para que las bacterias induzcan enfermedad. Los microorganismos que causan neumonía se adquieren en la inmensa
mayoría de los casos, por vía respiratoria y alcanzan las vías respiratorias bajas por trayecto descendente desde las vías respiratorias
altas. De manera inicial, el edema reactivo favorece la proliferación de los microrganismos y colabora en su diseminación a porciones
adyacentes al pulmón. Uno más lóbulos generalmente se involucran, en niños pequeños predomina un patrón más difuso que sigue la
distribución bronquial y se caracteriza por muchas áreas de consolidación alrededor de las vías respiratorias pequeñas, el daño
permanente es raro. Los pacientes con inmunodeficiencias primarias o secundarias tienen un riesgo mayor de infección, recurrencias y
complicaciones. DIAGNOSTICO: Clínica; las manifestaciones son secundarias a la respuesta inflamatoria sistémica y local a la infección;
por consiguiente, son de dos tipos; generales como fiebre, malestar general escalofríos y cefalea y respiratorios como tos, disnea,
taquipnea y anomalías en la auscultación torácica. La intensidad de los síntomas así como la gravedad de la enfermedad varían según el
agente etiológico, ya que mientras unos dan lugar a una consolidación pulmonar localizada, otros provocan una inflamación más difusa.
También depende de la edad del paciente, especialmente en niños pequeños. La radiografía de tórax es útil para corroborar el
diagnóstico y descartar complicaciones. En aproximadamente el 20% de las neumonías hay derrame pleural, que en una minoría de
niños evolucionara a empiema. Las neumonías atípicas suelen ocasionar un infiltrado heterogéneo y poco denso, con aspecto de vidrio
despulido, que tiende a estar situado cerca del hilio, sobre todo en los lóbulos inferiores; a menudo las imágenes de ocupación alveolar
afectan varios lóbulos, en ambos pulmones; así el patrón radiológico mas frecuente es el de un infiltrado parahiliar peribronquial
unilateral o bilateral. Aunque se ve pocas veces es muy característico de infección por M.pneumoniae la presencia de unas imágenes
reticulonodulillares localizadas en un solo lóbulo inferior. Los hemocultivos tienen pobre sensibilidad en las neumonías (10-30%). La
detección de antígenos bacterianos en sangre y orina tienen utilidad diagnostica variada. Los estudios serológicos, útiles en
epidemiologia, tienen escasa utilidad clínica. Las pruebas rápidas de detección de antígenos bacterianos en secreciones nasofaríngeas
mediante inmunofluorescencia directa o ELISA resultan muy útiles para la identificación de virus respiratorios, pero tienen
disponibilidad limitada y altos costos. INFECCIONES VIRALES; típicamente rinorrea, estornudos y tos. Temperatura ligeramente elevada,
podemos encontrar taquipnea, tiros intercostales, aleteo nasal y el uso de músculos accesorios, las infecciones graves se acompañan de
cianosis y dificultad respiratoria de moderada a grave especialmente en lactantes. A la auscultación de tórax se puede evidenciar
estertores y sibilancias. Las neumonías víricas también tienden a presentar un patrón de infiltrado parahiliar peribronquial, más o
menos difuso a veces acompañado de atelectasias; puede haber imágenes micronodulares difusas y sobredistensión pulmonar,
también son posibles otras imágenes, como el aumento de densidad localizada, segmentario o lobular. Es difícil identificar el agente
etiológico. INFECCION POR STREPTOCOCO PNEUMONIAE: Historia clásica de escalofríos de inicio súbito, seguidos por fiebre alta, tos y
dolor de tórax. Los lactantes presentan varios días de congestión nasal, inquietud e hiporexia de manera inicial. De manera súbita
presentan fiebre, astenia y aprensión, el paciente se ve enfermo, a la auscultación de tórax puede proporcionar pocos signos, pero
puede evidenciarse un área de consolidación por percusión, así como incremento de las vibraciones torácicas y estertores. Los niños
más grandes tienen escalofríos y fiebre después de una infección respiratoria alta. También tienen ataque al estado general, taquipnea,
tos seca y en casos graves alteraciones de la conciencia, a la auscultación, buscar lo mismo que en los lactantes y además buscar
3
derrame por percusión. NEUMONIA POR NEUMOCOCO: La BH, muestra formula blanca incrementada (15,000-40,000/mm ), con
predominio de PMN. La gasometría muestra hipoxia con hipercapnia. El neumococo puede ser aislado de las secreciones nasofaríngeas
en la mayoría de los pacientes, sin embargo, debe tenerse en cuanta que 10-15% de la población sana es portadora. El aislamiento de la
bacteria en sangre o en líquido pleural es diagnóstico. En las radiografías se evidencia consolidación aun antes de la detección de la
neumonía. INFECCION POR H. INFLUENZAE TIPO B: Inicio más insidiosos que aquellos que están infectados por neumococo y el curso
de la enfermedad se prolonga por varias semanas, a la exploración: taquipnea, dificultad para respirar, matidez a la o percusión,
estertores alveolares y bronquiales. Los niños pequeños pueden presentar derrame pleural. El diagnostico se establece por la historia
clínica, los síntomas y los hallazgos de la exploración. Pueden encontrarse imágenes segmentarias, compromiso lobular simple o
múltiple, derrame pleural o neumatoceles. Diagnóstico de certeza se establece con aislamiento del organismo de la sangre, líquido
pleural o aspirado pulmonar. COMPLICACIONES: Empiema y bacteriemia para la mayoría de los agentes etiológicos, artritis séptica en
casos de infección por Hib y Streptococo del grupo A, pericarditis, meningitis, osteomielitis, abscesos de tejidos blandos,
pioneumotórax y neumatoceles en las infecciones por S. aureus y celulitis, meningitis y artritis supurada en los casos de infección por
Hib. TRATAMIENTO ESPECIFICO: Usualmente en caso de neumonías por virus se requieren de medidas de soporte mínimas como la
administración de líquidos IV, oxígeno y en los casos graves ventilación mecánica, los únicos agentes específicos de que se dispone son
la amantadina y la ribavirina en aerosol, las cuales son eficaces contra influenza A y ribavirina para Virus sisitial respiratorio, se
recomienda solo en niños con enfermedad grave, que tienen cardiopatía congénita, displasia broncopulmonar, enfermedad pulmonar
crónica o que reciben tratamiento inmunosupresor. La neumonías causadas por neumococo son tratables con penicilina IV, el fármaco
de elección es la penicilina G (100000U/Kg/24 hrs). Debe considerarse el empleo de una cefalosporina de tercera generación
(cefotaxima 150mg/kg/24h, o ceftriaxona 75 mg/kg/24h), en el caso de que el S. peneumoniae sea resistente a la penicilina pero

CURSO ENARM CMN SIGLO XXI TEL: 36246001 Pharmed Solutions Institute PÁGINA 438
MANUAL DE TRABAJO DEL CURSO ENARM CMN SIGLO XXI
sensible a cefalosporinas. La vancomicina (40mg/kg/24h) se debería emplear si el aislamiento es resistente a las anteriores.
Aproximadamente el 20-30% de los neumococos son resistentes a los macrólidos, En los casos de neumonía por Streptococo del grupo
A deben ser tratados con penicilina G (100 000 U/kg/24h). Se emplea penicilina parenteral de manera inicial y debe completarse un
tratamiento de hasta 2-3 semanas con penicilina por la via bucal. Si se presenta empiema debe realizarse una toracocentesis con
propósitos diagnósticos y para evacuar líquido. Para Staphylococus aureus dicloxacilina 100-200mg/kg/24h). En los casos de derrame
pleural aunque sea mínimo o de pioneumotórax, se recomienda la instalación de un drenaje tóracico. Para H. influenzae de tipo b debe
incluir ceftriaxona (75mg/kg/24h) o cefotaxima 150mg/kg/24h). Los derrames y la pioartrosis requieren de drenaje. TRATAMIENTO
EMPIRICO: Se hace en función de la sospecha del agente etiológico. En niños de 2-3 años con cuadro clínico y radiológico de neumonía
y con buen estado general se puede tratar por VO con dosis altas de amoxicilina (80-90mg/kg/día) cada 8 h, asociada o no a ac.
Clavulanico. Si el estado general está afectado o vomita es preferible ingresarlo y tratarlo por vía IV con cefotaxima (100-
150mg/kg/día), ceftriaxona (50-100mg/kg/día) o amoxicilina/ ac. Clavulanico (100mg kg/día), durante mínimo 48-72 hrs si hay buena
evolución seguir con amoxicilina oral por 7-10 días. En latantes menores de 3 meses la mejor opción terapéutica es la cefotaxima sola o
asociada a ampicilina si se considera que listeria es una posible causa, siempre por via IV. En el niño mayor de 3 años con una neumonía
típica, si ingresa por afectación del estado general, compromiso repiratorio o intolerancia digestiva, se tratara del mismo modo que si
tuviera menos de 3 años, pero con un especial consideración al empleo de penicilina G (100 000mg/kg/24h) IV si el cuadro clínico no es
grave. Los macrólidos son el tratamiento de elección en los niños con neumonia por Mycoplasma pneumoniae y Chlamydia
pneumoniae, VO 7-10 dias o 3 dias si se utiliza azitromicina.

CASO CLINICO
Un niño de 5 años de edad se presentó con fiebre y tos no productiva durante 3 días. El examen físico reveló múltiples nódulos
linfáticos cervicales. Fue tratado en su centro de salud durante 7 dias con medidas generales, regresando a su escuela sin embargo a los
5 dias después inicia con tos productiva, dificultad respiratoria, cansancio, malestar generalizado, aleteo nasal y fiebre de 39.2 grados.
Examen torácico se apresiaron estertores bronquiales y broncoalveolares de predominio en la zona inferior. La saturación de oxígeno
de 92% en aire ambiente. La radiografía de tórax (RXT) al ingreso mostró consolidación del lóbulo inferior derecho. Leucositos de
13,500. Refiere el familiar que el esquema de vacunación se encuentra al corriente. Vive en zona rural y es el segundo hijo de 5.

PREGUNTA
Cual de los siguientes estudios solicita para tomar una decisión diagnostica.

RESPUESTA
a.- Proteina C reactiva.
b.- Biometria hemática.
c.- Cultivo de secresiones.
d.- Hemocultivo.

PREGUNTA
Cual es el criterio para tomar la radiografia de torax como factor desicivo para tratamiento.

RESPUESTA
a.- Paciente ambulatorio.
b.- Sospecha de neumonía.
c.- Fiebre alta con sitomas respiratorios.
d.- Presencia de estertores sin taquipnea.

PREGUNTA
Considerando el cuadro clínico cual es el agente causal más probable de este caso.

RESPUESTA
a.- Estreptococcus pneumoniae.
b.- Mycoplasma pneumoniae.
c.- Haemophilus influenza.
d.- Clamydia pneumoniae.

PREGUNTA
Cual es la conducta terapeutica mas apropiada.

RESPUESTA
a.- Amoxicilina 80-90 mg/kg/dia.
b.- Azitromicina 10 mg/kg dia.
c.- Eritromicina 50mg/Kg/dia.
d.- Penicilina procainica 400,000 UI cada 24 hrs IM.

PREGUNTA
Cual de los siguientes criterios es mas importante para el envio a segundo nivel.

RESPUESTA

CURSO ENARM CMN SIGLO XXI TEL: 36246001 Pharmed Solutions Institute PÁGINA 439
MANUAL DE TRABAJO DEL CURSO ENARM CMN SIGLO XXI
a.- Presencia de apneas.
b.- Dificultad respiratoria moderada.
c.- Falta de seguridad en casa.
d.- Deshidratacion.

RINITIS ALERGICA (RA). CIENCIAS BASICAS: Es una enfermedad crónica definida como una hipersensibilidad de la mucosa nasal a
sustancias extrañas mediada por IgE. Los sujetos que presentan RA requieren de exponerse a bajas dosis de alérgenos durante varios
años para desarrollar los síntomas. SALUD PUBLICA: Tienen una prevalencia entre 15-20%. La rinitis estacional raramente se presenta
en niños menores de 5 años y la mayoría de los pacientes con la enfermedad presentan los síntomas antes de los 20 años. En la Ciudad
de México, 14% de los niños cursan con rinitis alérgica. Hasta 78% de los pacientes con asma tienen rinitis alérgica. PATOGENIA:
Factores de riesgo; ablactación temprana, deprivación de alimentación al seno materno, contaminación ambiental, tabaquismo
materno durante el 1 año de vida, antecedente de alergia en padres, historia familiar de rinitis alérgica, higiene excesiva y niveles
elevados de IgE a los 9 meses y a los 6 años, pacientes asmáticos. Existen aeroalergenos más frecuentemente responsables de
ocasionar rinitis intermitente (pólenes de árboles, gramíneas y malezas, como así también esporas de hongos del exterior), pero la
persistencia de estos antígenos varía según las estaciones del año y la
región geográfica analizada, pudiendo un alérgeno estacional generar una
rinitis persistente. Los principales antígenos involucrados en la rinitis
persistente son los ácaros del polvo de habitación. La liberación de
mediadores químicos (histamina, leuicotrienos entre otros), originados en
la activación del mastocito, sensibilizado con IgE especifica de alérgeno, es
la responsable de los síntomas del paciente. La enfermedad alérgica
presenta dos etapas: a) sensibilización (predisposición genética), y b) la
presencia de síntomas (interacción huésped-medio ambiente). En la
primera etapa, debe existir un huésped predispuesto a inducir una
respuesta inmunitaria a los alérgenos (Th2), susceptibilidad dependiente
de que se genere un arreglo genético para producir IgE, capaz de
sensibilizar a la célula mastocitaria. En la segunda etapa, el contacto de
esta célula sensibilizada con el alérgeno produce la activación celular que
desencadena el proceso inflamatorio en dos fases: inmediata (dependiente
de mediadores químicos e IL) a los pocos minutos del contacto y tardía,
dependiente del infiltrado celular (eosinófilos, neutrófilos, mastocitos).
Esta activación del sistema por interacciones vasculares y neurogénicas genera los síntomas. Los principales mediadores químicos con
un papel central en el desencadenamiento de la sintomatología son: Histamina: es el principal mediador en la fase inmediata de la
reacción alérgica posterior a la provocación antigénica. Se almacena en los gránulos del basófilo y del mastocito. La histamina actúa
sobre los receptores H1 de varias células y causa los principales síntomas de rinitis. Leucotrienos: son formados de novo desde el ac.
araquidónico por vía de la lipooxigenasa, liberados principalmente por el mastocito en la fase temprana y por eosinófilos y neutrófilos
en la fase tardía. Los leucotrienos producen bloqueo e incremento de la secreción, pero no estornudos. Citoquinas: son liberadas por
los linfocitos T durante la reacción de fase tardía y por el mastocito;
resultan importantes para mantener la inflamación crónica.
CLASIFICACION: La OMS recomienda utilizar la establecida por ARIA
(Allergic Rhinitis and Its Impact on Asthma), de acuerdo a duración de los
síntomas: intermitente (<4 días a la semana y <4 semanas al año) o
persistente (>4 días a la semana y >4 semanas al año). Rinitis intermitente
comprende aproximadamente al 20% de los casos de rinitis alérgica y la
rinitis persistente afecta al 80% restante. DIAGNOSTICO: Para un adecuado
diagnóstico es importante realizar una cuidadosa historia clínica, que
permitirá caracterizar la sintomatología para clasificar la rinitis. En el
interrogatorio se debe poner énfasis en el análisis de los síntomas: 1.
Obstrucción nasal: respiración bucal crónica, falta de aire, hiposmia-
anosmia, voz nasal, babeo, ronquido, disfunción tubariataponamiento
otico. 2. Rinorrea: ruidos nasales, tos faríngea, nausea, dilatación
abdominal por aerofagia. 3. Prurito palatino y nasofaríngeo: frotamiento,
cloqueo, gárgaras. 4. Estornudos. 5. Otros: epistaxis, fatiga, irritabilidad,
prurito ocular, hiremia conjuntival, edema palpebral. Examen físico: La observación directa permite detectar signos característicos,
denominados: facies alérgica, cianosis infraorbitaria (ojeras), pliegue palpebral supernumerario (signo de Denie Morgan), pliegue
transversal en la nariz (producido por el prurito y la limpieza constante). Esta facies puede estar asociada a signos de respiración bucal,
como consecuencia de la obstrucción nasal. También son frecuentes el edema y la hiperemia conjuntival. En el diagnóstico, es
indispensable el examen de la cavidad nasal por rinoscopia anterior; se debe realizar con especial atención a la estructura ósea del
septum nasal, en busca de desviaciones o deformidades que impidan el normal flujo de aire. La mucosa puede presentarse inflamada,
pálida, gris-azulada, con secreción cristalina. Los cornetes edematizados pueden obstruir, de manera total o parcial, las fosas nasales.
Se analizarán las principales pruebas que permiten establecer un diagnóstico etiopatogénico adecuado: Recuento de eosinófilos en
sangre periférica. Citología nasal. IgE total y específica. Pruebas cutáneas. Estudios complementarios especiales como: Rinofibroscopia,
el mejor modo de examinar el interior de las fosas nasales es con la ayuda de un espéculo nasal y luz frontal o con otoscopio. Radiología
convencional: La frecuente asociación entre la rinitis y las alteraciones estructurales de la vía aérea superior, asociadas u originadas en
el proceso inflamatorio, pueden necesitar del estudio por imágenes. TRATAMIENTO: No farmacológico: Educación, medidas de control
ambiental. Evitar alérgenos. Farmacológico: ver cuadros anexos. En los cuadros intermitentes con grado de gravedad leve, los fármacos

CURSO ENARM CMN SIGLO XXI TEL: 36246001 Pharmed Solutions Institute PÁGINA 440
MANUAL DE TRABAJO DEL CURSO ENARM CMN SIGLO XXI
de elección son: antihistamínicos orales tópicos, solos o asociados a descongestivos. En las rinitis intermitentes de intensidad
moderada-grave y en las persistentes leves, los fármacos de elección son: corticoides inhalatorios, antihistamínicos orales y tópicos, o
descongestivos y cromonas.

CASO CLINICO
Niño de 7 años de edad con antecedentes de resfriados frecuentes y atopias, se le diagnosticó asma a los 5 años. No hay exposición al
humo asi como hacinamiento, sin embargo la casa esta alfombrada, conservando que en invierno y verano se agudizan sus cuadros
caracterizados por escurrimiento nasal de secresion hialina, estornudos repetidos de predominio matutino, a la exploración física se
observar rasgado nasal, e hiperemia conjuntival y alineación nasal.

PREGUNTA
Cual de los siguientes estudios solicitaria en el paciente para establecer su diagnostico.

RESPUESTA
a.- Citologia nasal.
b.- Endoscopia nasal.
c.- Biopsia nasal.
d.- Pruebas cutáneas.

PREGUNTA
Cual de las siguientes opciones terapéuticas indica.

RESPUESTA
a.- Motelucask
b.- Budesonina.
c.- Loratadina.
d.- Cromoglicato.

CURSO ENARM CMN SIGLO XXI TEL: 36246001 Pharmed Solutions Institute PÁGINA 441
MANUAL DE TRABAJO DEL CURSO ENARM CMN SIGLO XXI
ASMA. CIENCIAS BASICAS: Es una enfermedad crónica inflamatoria de las vías respiratorias (participan células cebadas, eosinófilos,
linfocitos T, neutrófilos y células epiteliales), que se caracteriza por obstrucción reversible o parcialmente reversible de éstas, además
de hiperrespuesta a diferentes estímulos y se caracteriza clínicamente por: tos, disnea y sibilancias. Es particularmente importante la
reversibilidad de la obstrucción bronquial, completa ó parcial, ya sea en forma espontánea o en respuesta a tratamiento. SALUD
PUBLICA: Primer causa de ausentismo escolar, tiene un gran impacto en la calidad de vida del que la sufre, que produce importantes
alteraciones en economía y la dinámica familiar. Las únicas encuestas realizadas en México de l948 a l991 que aparecen en la literatura
reportan una prevalencia de 1.2 a 12.5%. La atopía; término que define a algunas enfermedades con niveles elevados de IgE asociados
a una predisposición genética definida, constituye el factor epidemiológico más consistente para el desarrollo de asma en la infancia. El
asma es una causa rara de muerte infantil en niños y adolescentes. PATOGENIA: Factores de riesgo: 1. Alérgenos (ácaros, mascotas,
hongos, cucarachas), la exposición temprana a ellos tiene una alta correlación con asma. 2. Virus desencadenan 90%. 3. Tabaquismo de
los padres es otro factor que se ha comprobado en meta-análisis que incrementa el riesgo de padecer asma. 3. Contaminación, ozono,
monóxido de carbono, bióxido de nitrógeno, partículas suspendidas. 4. Sustancias químicas. 5. Ejercicio, su importancia radica en que
nos indica indirectamente la presencia de inflamación bronquial a pesar de que el niño no tenga síntomas en reposo. 6. Frío. 7.
Emociones. 8. Medicamentos, como AINES. La remodelación de la pared de la vía respiratoria explica la importancia de la inflamación
crónica recurrente en el asma. Los factores desencadenantes ó los alérgenos inhalados inducen la activación de las células cebadas y
macrófagos, con la consecuente liberación de varios mediadores pro-inflamatorios, incluyendo leucotrienos, factores quimiotácticos y
citocinas. Los antígenos procesados y presentados por los macrófagos a los linfocitos Th0, bajo la influencia un patrón adecuado de
citocinas, estimula la diferenciación a un patrón de citocinas Th2 lo que a su vez estimulan la liberación de mayores cantidades de IL- 4
e IL- 5, los cuales a su vez causan la síntesis de IgE por los linfocitos B y eosinofilia, respectivamente. Las citocinas derivadas de
macrófagos tales como IL -1, TNF- alfa, INF- gama, activan las células endoteliales, aumentando la expresión de las moléculas de
adhesión tal como ICAM 1 y VCAM 1. Más aun la IL - 4 derivada de células T selectivamente aumenta la expresión de VCAM 1. Esto
permite la salida de leucocitos de la vasculatura a la mucosa de las vías aéreas. Las consecuencias de este infiltrado de células
inflamatorias incluyen la esfacelación del epitelio respiratorio, anormalidades en el control autonómico del tono de las vías aéreas,
cambios en la función mucociliar y aumento de las respuestas bronquiales. Esta reacción inflamatoria autoperpetuable de esas células
efectoras (eosinófilos y células cebadas) es capaz de sintetizar citocinas que posteriormente promueven un incremento del proceso
inflamatorio. En resumen: Contracción muscular bronquial (broncoespasmo o broncoconstricción), edema e infiltración celular de la
mucosa bronquial (inflamación) e incremento de la secreción bronquial. DIAGNOSTICO: El diagnóstico se basa en la presencia de signos
objetivos de obstrucción bronquial, básicamente por medio de la exploración física (signos de dificultad respiratoria, sibilancias,
espiración prolongada, hipoventilación, opresión torácica, etc.). Estos signos obstructivos deben de ser recurrentes, esto es,
presentarse en forma de exacerbaciones episódicas (crisis), aunque en grados más severos los síntomas obstructivos pueden ser
persistentes, y aun así, presentar episodios de agravamiento. Otra característica básica del diagnóstico es que la obstrucción bronquial
es reversible, o al menos parcialmente reversible, a veces en forma espontánea o en base a tratamientos con broncodilatadores y/ó
anti-inflamatorios. Además de lo anterior, en la gran mayoría de los casos se puede documentar en la historia clínica el fenómeno de
hiper-reactividad bronquial, esto es, el inicio ó la exacerbación de signos y síntomas de reacción bronquial (tos, secreción bronquial,
sibilancias, ó disnea) a una diversidad de estímulos físicos, químicos ó emocionales (ejercicio, olores penetrantes, humos, cambios de
temperatura ó humedad ambiental, etc.). Funcional: FEM (Flujo Espiratorio Máximo) Este valor proporciona una medición simple y
cuantitativa de la obstrucción de las vías aéreas. Se realiza con un flujómetro portátil. El monitoreo de FEM es una herramienta clínica
de gran valor en el consultorio, en el hospital y hogar del paciente ya que permite valorar: La respuesta al tratamiento durante una
crisis aguda, respuesta al tratamiento crónico, detectar el deterioro asintomático de la función respiratoria, antes de que se vuelva más
grave. Identificar factores desencadenantes como por ejemplo el ejercicio. Espirometría: prueba de broncodilatador. FEV-1 (Volumen
Espiratorio Forzado en el primer segundo): Es la fracción de volumen que se expulsa en el primer segundo del esfuerzo respiratorio
máximo, y normalmente representa aproximadamente el 80% de la CVF. Una disminución >20% de este volumen, es indicativa de un
proceso obstructivo. Todos los pacientes con diagnóstico presuntivo de asma deben ser valorados con una espirometría, al menos al
iniciar su tratamiento y, posteriormente, controles de seguimiento dependiendo de cada caso. Laboratorio: BH con eosinofilia, niveles
IgE, pruebas cutáneas, citología nasal, coproparasitoscópicos. Tele de tórax en busca de complicaciones, radiología de senos
paranasales, gasometría, pruebas de función pulmonar. CLASIFICACION DE ASMA: 1. ASMA INTERMITENTE: Síntomas menos de una
vez a la semana. Exacerbaciones breves. Síntomas nocturnos no más de 2 veces al mes. FEV1 >80%, variabilidad del FEV1 <20%. 2.
ASMA LEVE PERSISTENTE: Síntomas >1 vez a la semana pero <1 vez al día. Exacerbación afecta la actividad o sueño. Síntomas nocturnos
>2 veces al mes. FEV1/FEM >80 %, Exacerbaciones NO Uno o más/ año Una vez/sem
variabilidad de FEV1 20-30%. 3. ASMA
MODERADA PERSISTENTE: Síntomas Diarios. Exacerbaciones que afectan la actividad o sueño. Síntomas nocturnos >1 vez a la semana.
Uso diario de agonistas B2 Inhalados de acción rapida. FEV1 o PEF de 60-80%, variabilidad del FEV1 >30%. 4. ASMA SEVERA
PERSISTENTE: Síntomas Diarios. Exacerbaciones frecuentes. Síntomas nocturnos frecuentes. Limitación de actividad física. FEV1 <60%,
variabilidad FEV1 >30%. CLASIFICACION DE LAS CRISIS ASMATICAS: Leve: limita actividades fuertes (ejercicio), habla sin problemas, no
lo despierta. Flujometria: PEF > 80%, FC <100 lpm, sibilancias: moderadas. Moderada: limita actividades habituales, habla con dificultad.
Flujometría: FEM 60-80% FC 100-120 lpm, sibilancias: intensas. Severa: incapacidad de realizar actividades, dificultad respiratoria en
reposo, cianosis, habla palabras. Flujometría: < 60% FC >120 lpm, sibilancias: intensas. Paro inminente: conciencia: confundido, tiros:
respiración paradojia, bradicardia, sibilancias: silencio. TRATAMIENTO: Medidas Generales: Control ambiental, evitar exposición a
desencadenantes, vacuna anti influenza, evitar AINEs, reducir o eliminar la inflamación. Las metas en el tratamiento del asma son: a.-
Control adecuado de los síntomas. b.- Prevenir las exacerbaciones. c.- Mantener FR dentro de la normalidad. d.- Mantener actividad
normal. e.- Evitar efectos adversos de medicamentos. f.- Prevenir el asma fatal. Los medicamentos utilizados son: ESTEROIDES
INHALADOS: Mejoran la función pulmonar, disminuyen la hiperreactividad bronquial, los síntomas y las exacerbaciones, mejoran la
calidad de vida. Las dosis recomendadas son: Beclometasona: niños 100-800 ug/día (leve-moderada:200, grave: > 500 dosis máxima:
1000). Budesonida: niños 100-400 ug/día (leve moderada:200, grave: > 400, dosis máximas: 800). Fluticasona: niños: 100-200 ug/día
(leve-moderada: 100, grave: > 200, dosis máximas: 500). Ciclesonida : Dosis 100 a 1200 mcg; Vía de administración : Inhalado.
Triamcinolona: Dosis 400 a 2000 mcg/día vía de administración: Inhalado Reacciones secundarias: Candidiasis oral, disfonía, tos

CURSO ENARM CMN SIGLO XXI TEL: 36246001 Pharmed Solutions Institute PÁGINA 442
MANUAL DE TRABAJO DEL CURSO ENARM CMN SIGLO XXI
(irritación de la vía aérea), prevención: uso de espaciadores. ESTEROIDES SISTEMICOS: Esta terapia se utiliza para el control de los
pacientes con asma severa persistente. Han demostrado disminución de signos patológicos de inflamación y mejoría de la
hiperreactividad bronquial. Prednisona: 0.5-1 mg/kg/día dosis de reducción. Deflazacort : 5-10 mg/kg/día dosis de reducción.
Metilprednisolona: Dosis 4-6 mg/kg cada 6-8 hrs. Hidrocortisona: Dosis 4-6 mg/ cada 6-8 hrs. Efectos secundarios: Osteoporosis, DM,
glaucoma, obesidad, estrías cutáneas, supresión del eje hipotalámico, metilxantinas: broncodilatador cuyo mecanismo de acción está
relacionado con la inhibición de la fosfodiesterasa. Infusión: 5 mg/kg/dosis IV 0.7-0.9 mg/kg/hr. Reacciones secundarias: Nauseas y
vómito, cefalea, taquicardia, arritmias. B2 AGONISTAS: Relajan la musculatura lisa bronquial, mejoran el aclaramiento mucociliar,
disminuyen la permeabilidad vascular y la liberación de mediadores inflamatorios. Los más utilizados son: Salbutamol: 200mcg PRN.
Fenoterol: 50mcg PRN. Salmeterol: 25mcg cada 12 hrs. Terbutalina: 500mcg PRN. B2 Agonistas y Esteroides: Disminución de los
síntomas y de crisis de asma nocturna, mejora la función pulmonar, reduce el número de exacerbaciones. MODIFICADOR DE
LEUCOTRIENOS: Cistenil leucotrieno, Montelukast, Pranlukast, Zafirlukast. ANTICOLINÉRGICOS: Bromuro de Ipratropio y Bromuro de
oxitropio, mecanismo de acción: Broncodilatador. Medicamentos controladores: Esteroides inhalados, agonistas de larga acción, Anti
leucotrienos, Xantinas. Medicamentos de rescate: agonistas de corta acción, anticolinérgicos. Crisis asmáticas: Salbutamol + bromuro
de ipratropio, 2-4 disparos c/20min x hora. LEVE: 2-4 disparos cada 3-4 hrs, MODERADA: 6-10 disparos por 1 ó 2 hrs, SEVERA: 10
disparos o más. ESTADO ASMATICO: Forma letal del asma, caracterizado por ataques cada vez peor que no mejora con tratamiento
apropiado, que culmina con la insuficiencia de la válvula pulmonar: empleo de músculos accesorios de la respiración, pulso paradójico,
cianosis. Criterios hospitalizar. Terapia intensiva: Ataques frecuentes y repetidos, última vez asma grave que culmino en hospitalización.
Consumo diario y excesivo de broncodilatadores y corticoesteroides. Empleo de músculos accesorios de la respiración. Pulso paradójico
que rebasa los 18 mmHg en adolescentes y 10 mmHg en niños. Cambios en la conciencia, cianosis. Neumotórax, neumomediastino.
FEV1 o PEFR (velocidad de flujo espiratorio máximo) menor de 20% de la cifra calculada. PaO2 < 60mmHg. PaCO2 > 40mmHg en
presencia de disnea y sibilancias. Acidosis metabólica. Anormalidades electroencefalográficas.

CASO CLINICO
Niña de 7 años que acude a urgencias por cuadro de tos, dificultad respiratoria de dos semanas de evolución y pico febril 38°C. con
antecedentes patológicos de obesidad, roncadora habitual y respiración bucal. adenoidectomizada hace 2 años por sospecha clínica de
SAHS sin control posterior. Peso 56kg (p>97), FC 135 lpm (p>95), TA 109/70, SatO2 82% con FiO2 21%. Facies anímica, implantación
baja del pelo, estrabismo y obesidad mórbida. Aceptable estado general, normohidratada, palidez cutánea y retracciones subcostales
moderadas. Auscultación cardiaca normal. Auscultación respiratoria con hipoventilación generalizada moderada, sibilantes espiratorios
finos y roncus dispersos.

PREGUNTA
Cuál es la conducta farmacológica mas adecuada.

RESPUESTA
a.- Salbutamol, bromuro de ipatropio inhalados y corticoides endovenosos.
b.- Salbutamol, corticoide, loratadina.
c.- Salbutamol, oxigeno, ambroxol y prednisona.
d.- Ambroxol, prednisona, Bromuro de ipatropio y oxigeno.

PREGUNTA
El paciente empeoro su cuadro clínico, con mayor dificultad respiratoria. Se decide enviar a segundo nivel. Al ingreso se realizó BH con
discreta leucocitosis, bioquímica normal y gasometría capilar: pH 7,31, PC02 52,3mmHg, PO254,7mmHg, HC0326mmol/l. Precisa FiO2
inicial de 1 para mantener SatO2>92%. Presenta evolución lenta y respuesta parcial al tratamiento. Tras los primeros días de ingreso
persisten necesidades de oxigenoterapia elevadas (Fi 2≈0,5) durante el sueño. En controles de gasometría destaca hipercapnia
nocturna (PCO2 56,3mmHg). La evolución tórpida obliga a nuevo planteamiento diagnóstico y realización de pruebas complementarias.

PREGUNTA
Cuales fueron los criterios más importantes para el envio a segundo nivel.

RESPUESTA
a.- Duda diagnostica.
b.- Asma complicada.
c.- Asma no controlada con tratamiento.
d.- Antecedetes de exacerbaciones.

CASO CLINICO
Se trata de un adolescente de 13 años y 90kg de peso con antecedentes de asma sin tratamiento de fondo. Acude por dificultad
respiratoria de varias horas de evolución sin que hubiera recibido broncodilatadores previamente. Se constata una crisis asmática grave
con taquipnea y retracción costal marcadas, y se inicia tratamiento con nebulizaciones de salbutamol (15mg en la primera hora) y
bromuro de ipratropio (500 mg/h) asociados a metilprednisolona por vía intravenosa (60mg). Transcurrida 1 h de tratamiento, se
realiza gasometría venosa que muestra acidosis metabólica con hiperlactacidemia.

PREGUNTA
Cuál es la conducta a seguir mas adecuada?

RESPUESTA

CURSO ENARM CMN SIGLO XXI TEL: 36246001 Pharmed Solutions Institute PÁGINA 443
MANUAL DE TRABAJO DEL CURSO ENARM CMN SIGLO XXI
a.- Mantener el tratamiento.
b.- Enviar a segundo nivel.
c.- Ingreso a la UCI.
d.- Realizar rx de torax.

PREGUNTA
Al ingreso hospitalario se mantiene tratamiento broncodilatador (salbutamol y bromuro de ipratropio) horario. Tras 4h de ingreso, se
mantiene el trabajo respiratorio, la taquipnea y la taquicardia, con saturación de oxígeno del 92% y con fracción inspiratoria de oxígeno
(FiO 2 ) del 35%. La radiografía de tórax presenta hiperinsuflación sin condensación neumónica, atelectasia o escape aéreo. La
gasometría muestra empeoramiento de la acidosis láctica. Se ingresa al paciente en cuidados intensivos para el inicio de la ventilación
no invasiva tipo presión positiva con dos niveles de presión con los parámetros iniciales (presión positiva inspiratoria de la vía aérea de
12; presión positiva espiratoria de la vía aérea de 6; FiO 2 del 40%), se mantiene el tratamiento broncodilatador y se expande la
volemia. Cuál es pronostico esperado.

RESPUESTA
a.- Mejoría ventilatoria.
b.- Descenso significativo de la FR.
c.- Normotenso, caliente y bien perfundido.
d.- Incremento de acido láctico.

FIBROSIS QUISTICA (FQ). CIENCIAS BASICAS: Es la enfermedad genética más letal, de carácter recesivo, multisistémico y progresivo,
afecta de preferencia a poblaciones caucásicas. La enfermedad se caracteriza por el espesamiento del mucus producido por las
glándulas exocrinas induciendo compromiso sino-pulmonar con daño pulmonar progresivo, insuficiencia pancreática y por lo tanto
síndrome de mala-absorción, con consecuente desnutrición, esterilidad masculina por atrofia de los conductos deferentes y elevación
de electrolitos en el sudor. El íleo meconial siempre debe obligar a descartar FQ. Dependiendo de las mutaciones involucradas existe
una gran diversidad de formas clínicas. SALUD PUBLICA: Su incidencia varía de 1 por cada 3,000 a 1 por cada 8,000 nacidos vivos.
PATOGENIA: El gen de la FQ se encuentra localizado en el brazo largo del cromosoma 7, codifica una proteína de 1480 aminoácidos que
se ha llamada Proteína Transportadora de Transmembrana (CFTR), esta se localiza en el polo apical de las células epiteliales. Es una
glicoproteína cuya función es actuar como canal de cloro, está constituida por dos regiones transmembránicas (hidrofóbicas) separadas
por una región de unión al ATP. La primera mutación encontrada fue la ∆F508, localizada en el dominio N D1, presente en alrededor
del 75% de la población caucásica. El resultado de todas las mutaciones detectadas que alteran la función de la CFTR es el mismo: la
imposibilidad de transportar cloruro. Cualquiera que sea la mutación en el gen CFTR, cada paciente muestra las siguientes
anormalidades en distintos grados: A) Concentración anormal de los iones en las secreciones de las glándulas serosas, manifestada por
aumento en la concentración de cloro y sodio en el sudor. B) Incremento en la viscosidad de las secreciones de las glándulas secretoras
de moco, asociado con obstrucción y perdida secundaria de la función glandular. C) Aumento en la susceptibilidad a la colonización
endobronquial crónica por grupos específicos de bacterias (Staphylococcus aureus, H. influenzae, Pseudomona aeruginosa,
Burkholderia cepacia). Se han descrito 6 clases de mutaciones: las I a III son las más comunes y generalemnrte se relacionan con
insuficiencia pancreática. En Mexico se han identificado 46 diferentes mutaciones que afectan a 77% de los cromosomas de la fibrosis
quística. DIAGNOSTICO: La mayoría de los casos de FQ se manifiesta por la triada clásica: a) enfermedad pulmonar obstructiva
progresiva crónica con infección agregada, b) insuficiencia pancreática exocrina, c) elevación en las concentraciones de Cl y Na en el
sudor. Los RN afectados rara vez muestran síntomas respiratorios, aunque los menores de 6 meses de edad pueden experimentar
taquipnea, sibilancias, incremento del trabajo respiratorio, sobresdistension del tórax y atelectasias. En 10-20% de los pacientes, el íleo
meconial puede ser la primera manifestación de la enfermedad. Se produce por la implantación de meconio deshidratado en el íleo
terminal, con un cuadro de obstrucción intestinal. Puede sospecharse antes del parto por ecografía u ocurrir al nacimiento con
distensión abdominal progresiva y vómitos biliosos y falta o retardo en la eliminación de meconio en las primeras 24-48hrs de vida. La
radiografía de abdomen suele revelar asas intestinales dilatadas con áreas de aire mezclado con meconio deshidratado. En la mayoría
de los casos se encuentran una o más expresiones clínicas de la enfermedad, que se confirma con la prueba de sudor. Casi todos los
pacientes exhiben enfermedad sinusopulmonar crónica y el 85-90% tiene insuficiencia pancreática exocrina. Es necesario hace notar
que debe considerarse como FQ clásica, aquella con niveles de test del sudor altos, compromiso pulmonar severo, progresivo, de
instalación temprana, rinosinusitis, síndrome de malaabsorción e infertilidad masculina. Se ha denominado como FQ no clásica o
enfermedad vinculada a FQ aquella con al menos una mutación del gen de FQ, lo que le confiere función parcial a la proteína CFTR,
generalmente no tienen síndrome de maladigestión por estar preservada la función exocrina y por lo tanto mejor estado nutricional,
con niveles de test del sudor discretamente altos o normales, enfermedad pulmonar de instalación tardía y de menor severidad,
rinosinusitis y azoospermia, con clara mejor sobrevida. DIAGNOSTICO PREIMPLANTACIONAL: determinar características genéticas del
embrión a partir de una sola célula obtenida mediante biopsia embrionaria, sin prejuicio para la viabilidad del mismo. Por consiguiente,
se requiere la obtención de embriones mediante fecundación in vitro. DIAGNOSTICO PRENATAL: Se analiza el ADN de células de
vellosidades corionicas o líquido amniótico. Se realiza si los padres son portadores o si existe un hermano con fibrosis quística. ESTUDIO
NEONATAL: Se basa en el hecho de que las concentraciones séricas de tripsina de los enfermos con insuficiencia pancreática pueden
ser incluso 8 veces mayores a lo normal, se analizan: tripsina, tripsinogeno o complejo tripsina 1 anti-tripsina. El primer estudio se
realiza entre el 1-5 días de vida; si es positivo se repite entre la segunda y octava semanas si las concentraciones se mantienen
elevadas, se hace la prueba de Gibson y Cooke y estudio genético. Test del sudor: la iontoforesis de pilocarpina por el método de
Gibson y Cooke, continua siendo el gold standard que permite medir los valores de sodio y cloro en el sudor; en el túbulo de la glándula
sudorípara está bloqueado el reingreso de cloro a la célula, por lo cual tampoco lo hace el sodio, teniéndose un sudor con mayor
cantidad de estos electrolitos. Se considera los siguientes valores: Positivo: >60 meq/lt, Limítrofe: 40 a 59 meq/lt, Negativo: <40 meq/lt.
I.- Test secretina pancreozimina: gold standard para medir función pancreática. ANATOMIA PATOLOGICA: Los hallazgos macroscópicos
son limitados y solo sugestivos de fibrosis quística. Los hallazgos histopatológicos son variables e incipientes en RN con afección

CURSO ENARM CMN SIGLO XXI TEL: 36246001 Pharmed Solutions Institute PÁGINA 444
MANUAL DE TRABAJO DEL CURSO ENARM CMN SIGLO XXI
principal a glándulas mucosas, y se vuelven más evidentes al avanzar la enfermedad los más representativos: A. Páncreas: 93% de los
casos tienen cambos histopatológicos. Antes de las 40 semanas puede verse normal, el volumen acinar va disminuyendo y puede ser
25% menor a los cinco meses de vida. B. Tubo gastrointestinal: en RN, el íleo meconial se ha asociado como hallazgo patológico inicial
de fibrosis quística (15 a 20%). La atresia intestinal (íleon, yeyuno, o ambos) afecta a 15-25% de los casos de fibrosis quística. La
peritonitis meconial ocurre en 33 a 50% de los pacientes con fibrosis quística debido a la perforación intestinal intrauterina. C. Hígado y
vías biliares: 60% de los pacientes sufren alteraciones clínicas y morfológicas, como cirrosis biliar focal, manifestada por proliferación de
conductos biliares dilatados con material eosinofílico intraluminal y áreas de fibrosis irregular portal e infiltración de linfocitos. D. Vías
respiratorias: las manifestaciones morfológicas principales son: hiperplasia de glándulas submucosas bronquiales con moco espeso
intraluminal, bronquiectasias, atelectasias, neumonías, obstrucción bronquial por moco y células inflamatorias que expanden las vías
aéreas y que se extienden al parénquima pulmonar. TRATAMIENTO: Son cinco los pilares básicos del tratamiento de esta enfermedad:
1) tratar la infección y la inflamación, y reparar el aclaramiento mucociliar, 2) mantener un buen estado de nutrición, 3) tratar la
insuficiencia del páncreas exocrino, 4) iniciar fisioterapia respiratoria y 5) detectar y tratar de manera oportuna enfermedades
concomitantes (diabetes, hepatopatía y osteopenia). El germen que más frecuentemente (60%) se aísla en el esputo de los enfermos de
fibrosis quística es P. aeruginosa. Existe gran interés en los antibióticos aerosolizados. Los beneficios potenciales de administrar
antibióticos vía aerosol incluyen: el depósito directo en el sitio endobronquial de la infección, la disminución de la toxicidad, mejor
relación costo-beneficio y mejor calidad de vida. En un estudio basado en evidencias de la Asociación Americana de Fibrosis Quística se
recomienda, para tratar las manifestaciones pulmonares, administrar alfa dornasa inhalada (Pulmozymes®), que es una
desoxirribonucleasa recombinante que actúa como mucolíticos, degradando el ADN. El gasto de energía incrementado en el paciente
con fibrosis quística se debe a la insuficiencia pancreática, la malabsorción de nutrimentos y la inflamación. La adecuada alimentación
favorece la síntesis proteica. Últimamente se ha autorizado la administración de Kalydeco (ivacaftor), el cual funciona como un
potenciador, ya que facilita que el canal transporte más cloro. Staphylococcus aureus meticilina sensible: Cloxacilina: 100 a 200
mg/Kg/día (d), c/6h por 21 días, oral o endovenoso, según la situación del paciente, o Flucloxacilina: 50 a 75 mg/Kg/d, c/8h por 21 días,
oral. Staphylococcus aureus meticilina resistente: Vancomicina: 50 mg/Kg/d, c/6h por 21 días, IV, o Linezolid: 150 a 300 mg c/12h por
21 días, oral o IV. Está indicado en caso de alergia a Vancomicina, no disponibilidad de vías ev o tratamiento ambulatorio. Haemophilus
influenzae ß-lactamasa negativo: Amoxicilina: 90 mg/Kg/d, c/12h por 14 días, oral. Haemophilus influenzae ß-lactamasa positivo:
Amoxicilina-ácido clavulánico: 90 mg/Kg/d, c/12h por 14 días, oral, o Cefotaxima: 100 mg/Kg/d, c/6h por 14 días, IV. Pseudomonas
aeruginosa: Ceftazidima: 200 a 250 mg/Kg/d, c/6h por 14 a 21 días, IV más Amikacina: 20 a 30 mg/Kg/d ev, o Tobramicina: 10 mg/Kg/d
IV. Alternativa de segunda línea: Ciprofloxacina: 30 mg/Kg/d, c/12h oral (máximo 300 mg c/8h IV en infecciones severas) por 14 a 21
días.

CASO CLINICO
Se presenta el caso de un paciente de tres años de edad, raza blanca y sexo masculino; presenta, tos húmeda, vómitos flemosos,
antecedentes patológicos personales de fibrosis quística y perinatales de parto eutócico a término, con peso al nacer de 6,8 libras, y
apgar 7/9, por aspiración de líquido meconial, no íctero, y caída del cordón umbilical a los tres días. Lactancia materna hasta los tres
meses y lactancia artificial actual con leche evaporada. Vacunación actualizada y medio-ambiente social favorable. Presentó
antecedentes patológicos personales de broncoaspiración de meconio al nacer y un ingreso al mes y medio por atelectasia, además de
un segundo ingreso a los siete meses de nacido por polipnea y no ganancia de peso, a pesar de alimentarse bien, así como deposiciones
pastosas con grasa seg n refirió la mamá, “suda mucho y el sudor es salado”.

PREGUNTA
Cual es la conducta a seguir para establecer el diagnostico secundario que presenta el paciente?

RESPUESTA
a.- TC abdominal.
b.- USG abdominal.
c.- IRM abdominal.
d.- LAPE.

CURSO ENARM CMN SIGLO XXI TEL: 36246001 Pharmed Solutions Institute PÁGINA 445
MANUAL DE TRABAJO DEL CURSO ENARM CMN SIGLO XXI
INFECCION DE VIAS URINARIAS (CISTITIS, PIELONEFRITIS). CIENCIAS
BASICAS: El término infección de vías urinarias (IVUs) se aplica a una
amplia variedad de trastornos que afectan el aparato urinario, indica la
presencia de bacteriuria significativa con o sin sintomatología general
de infección y/o sintomatología uretrovesical: desde infecciones
asintomáticas, hasta aquellas que ponen en peligro la vida del enfermo
como la pielonefritis. SALUD PUBLICA: La IVU tiene mayor gravedad en
los niños <1 año de edad con frecuencia aprox 1%. Después de esta
edad la prevalencia aproximada de 3% en niños y de 5 a 8% en niñas.
La tasa reportada de recurrencia es de 12-30%, con mayor probabilidad
en menores de seis meses, en caso de reflujo vesicoureteral (RVU)
grave y en aquellos con gamagrafía renal anormal al momento de la
primera infección. Entre un 8 y 40% de los menores de seis años con
IVU tienen RVU; otras anormalidades comunes incluyen hidronefrosis,
uropatía obstructiva y doble sistema colector. De un 10 a 65% de los de
menores de dos años presentarán cicatrices renales. Estas últimas se asocian con el desarrollo de hipertensión y enfermedad renal
terminal. Se ha encontrado que entre 10 y 25 % de los enfermos con insuficiencia renal crónica, tienen como causa pielonefritis crónica.
CLASIFICACION: 1.- BACTERIURIA ASINTOMÁTICA: Pacientes que incidentalmente presentan bacteriuria sin los síntomas clásico de
IVU, para confirmar el diagnóstico se requieren 2 cultivos, que muestren el mismo microorganismo con recuento de 10 UFC o más por
milímetro de orina. 2.- INFECCIONES URINARIAS BAJAS: Infecciones que no afectan por el momento el parénquima renal, constituyen
el grupo de mayor frecuencia y requieren de un tratamiento adecuado que cure las molestias del paciente y evite su diseminación a las
estructuras renales. Se subdividen en: Uretritis Aguda, Cistitis Aguda, Prostatitis aguda, la cistitis aguda es la de mayor importancia en el
paciente pediátrico, esta es la afección de la vejiga que se manifiesta por disuria, urgencia miccional, dolor supra púbico, incontinencia
y orina fétida. La cistitis no produce fiebre, ni daños renales, la orina puede ser turbia incluso con hematuria. 3.- INFECCIONES
URINARIAS ALTAS: Corresponden a la pelvis renal, cálices, parénquima renal. Su incidencia se incrementa en pacientes con factores
predisponentes como malformaciones obstructivas, RVU, estenosis de cuello vesical, y del meato urinario, etc. En otros casos es
causada por una diseminación hematógena del germen. Se subdividen
en: Pielonefritis aguda; representa el tipo más grave de IVU, no solo
produce más morbilidad, sino también mayor potencial para causar
daño irreversible. En los niños de mayor edad con pielonefritis aguda
se manifiesta fiebre, dolor e hipersensibilidad en el flanco y se
relaciona con piuria y urocultivos positivos. Los resultados de
laboratorio casi siempre revelan leucocitosis, VSG aumentada. Los RN y
lactantes menores son los grupos de edad con mayor riesgo de
formación de tejido cicatrizal renal posterior a pielonefritis.
Pielonefritis crónica; se caracteriza por alguna o todas las siguientes
manifestaciones dolor abdominal o en flanco, fiebre, mal estado
general, nauseas, vomito, ictericias en los RN, y en ocasiones diarrea.
PATOGENIA: Aproximadamente 95% de las IVUs son causadas por
enterobacterias. La E. coli es responsable del 80 al 90%. El porcentaje
restante puede ser ocasionado por Staphylococcus saprophyticus,
Enterococcus sp., Klebsiella sp., Enterobacter sp., Pseudomonas sp. y Proteus sp. De las 150 cepas de E. coli, diez de ellas son
responsables de la mayoría de las IVU. Esta situación está relacionada con la presencia de factores bacterianos virulentos, como la alfa
hemolisina (proteína citolítica que lesiona la membrana celular), sideróforos (proteína quelante de hierro que prolonga la vida de la
bacteria), y polisacáridos capsulares (que disminuyen la activación del complemento). La presencia de fimbrias en las bacterias, que
favorecen su adherencia al urotelio, también es un factor de virulencia importante: El 91% de las cepas de E. coli que producen
pielonefritis tienen fimbrias. Las bacterias pueden acceder al tracto urinario a través de cuatro vías: 1. Vía ascendente, desde la uretra y
la vejiga a los riñones, la más común. 2. Hematógena: En pacientes inmunocomprometidos ó en neonatos. 3. Linfática, desde el recto,
colon y linfáticos periuterinos, difícil de comprobar. 4. Directa, a través de fístulas rectovaginales ó a cualquier parte del tracto urinario.
Los factores del huésped que están implicados en la presencia de bacteriuria se pueden resumir en: 1. Edad: Los neonatos tienen mayor
predisposición a IVU por la inmadurez de su sistema inmunológico. Además existe una elevada colonización periuretral en el primer año
de vida. 2. Colonización fecal, periuretral y prepucial: El uso indiscriminado de antibióticos de cualquier tipo favorece la proliferación de
cepas virulentas y multirresistentes. 3. Género: factores anatómicos inherentes a la mayor accesibilidad de la vejiga a los gérmenes en
las niñas que en los niños por tener la uretra más corta. 4. Genéticos: Con mayor frecuencia los niños con IVU recurrentes tienen en su

CURSO ENARM CMN SIGLO XXI TEL: 36246001 Pharmed Solutions Institute PÁGINA 446
MANUAL DE TRABAJO DEL CURSO ENARM CMN SIGLO XXI
epitelio urinario receptores, glucolípidos antígenos del grupo
sanguíneo P, que facilitan la adhesión de las fimbrias o pili de E. coli. 5.
Anormalidades genitourinarias: Estas alteraciones deben ser
identificadas tempranamente, ya que si no son corregidas a tiempo
pueden llevar a secuelas como cicatrices renales, pérdida de
parénquima renal, hipertensión arterial e insuficiencia renal crónica. Es
por esta razón que se recomienda el estudio imagenológico en todos
los pacientes que se presenten con IVU. DIAGNOSTICO: Clínica; Los
síntomas y signos más comunes en menores de cinco años por los que acuden a urgencias con el primer episodio de IVU son: fiebre
80%, irritabilidad 52%, anorexia 49%, malestar 44%, vómito 42%, diarrea 21%. Los síntomas menos comunes (en menos de 20%):
disuria, orina fétida, dolor abdominal, frecuencia y hematuria. La presencia de fiebre >38ºC, bacteriuria y dolor lumbar sugiere
pielonefritis, mientras que la presencia de síntomas urinarios como disuria asociada a bacteriuria, pero no a síntomas sistémicos,
sugiere cistitis o IVU baja. En niños de dos a 12 años de edad, en su
primer episodio de IVU, los síntomas más frecuentes fueron disuria y
urgencia en el 82%, dolor abdominal 35%, enuresis 45%, fiebre 26%,
hematuria 20% y balanitis 20%. El ABC del diagnóstico de la infección
urinaria se basa en: EGO, para reconocer la presencia de estearasa
leucocitaria, reducción de nitratos a nitritos cuenta de células
inflamatorias (más de 10 células) y presencia de bacterias. Esta prueba
tiene una sensibilidad de 75 a 90% y una especificidad de 70 a 82%.
Cultivo de orina. La limitante de este estudio es disponer de una
muestra adecuada para el proceso. Si la orina se obtiene de una bolsa
colectora, la sensibilidad y especificidad son muy bajas ya que el 80%
de las muestras se hallaron contaminadas. Si la orina se obtiene por
catéter, la sensibilidad y especificidad son superiores a 70%; por punción suprapúbica la presencia de cualquier número de colonias
bacterianas permite asegurar el diagnóstico El número de unidades formadoras de colonias (UFC) necesarias para establecer el
diagnóstico de IVU está en función del tipo de muestra que se obtiene, ver cuadro 2 anexo. Imagenología. El estudio estándar es la
urografía excretora con histograma miccional, aunque el ultrasonido lo ha ido desplazando gradualmente. Este último es una prueba de
escrutinio para descartar malformaciones mayores. Sin embargo, la sensibilidad y especificidad para el diagnóstico de RVU son bajas en
ambas. El histograma miccional debe realizarse cuando el niño este afebril o el cultivo urinario sea negativo. Los estudios con
radioisótopos pueden ser de utilidad para evaluar la función e integridad renal en pacientes con pielonefritis. INFECCIONES
RECURRENTES: La frecuencia de recurrencias en el primer año de vida es menor de 20% en niños y de menos de 30% en niñas; en niños
mayores de un año, es superior a 30%. En niñas que han tenido más de dos episodios previos, la recurrencia puede llegar a 75%.
TRATAMIENTO: Neonato. La infección urinaria a esta edad es un problema de extrema gravedad, con alto riesgo de sepsis,
complicaciones, secuelas y muerte. Su tratamiento debe realizarse en el hospital. La probabilidad de malformaciones de las vías
urinarias es muy alta. El tratamiento antomicrobiano para sepsis neonatal es el indicado. La asociación de un betalactámico más un
aminoglucósido permite una cobertura de más del 90% de los microorganismos involucrados ver cuadro 3 y 4. El tratamiento debe ser
de 10 a 14 días. Cuando se pueda prescindir de la vía endovenosa, se puede recurrir a la intramuscular o a la vía oral cuando sea
posible. Cualquiera de las combinaciones es igualmente eficaz y segura; la diferencia es principalmente el costo. En neonatos con
hiperbilirrubinemia, no se recomienda la ceftriaxona. Los lactanetes menores de 3 meses con IVU deben ser tratados con los mismos
criterios. Otras edades. El tratamiento se debe guiar de acuerdo al sitio de la infección: INFECCIONES URINARIAS BAJAS:
CISTOURETRITIS; El tratamiento de preferencia debe ser por vía oral. Pueden utiliozarse amoxicilina, TMP/SFX y nitrofurantoina, ver
cuadro 5. La diferencia en la respuesta clínica a la amoxicilina o a la amoxicilina/clavulanato no es significativa. Los perfiles de
sensibilidad de E. coli a TMP/SMX han mostrado un incremento progresivo en la resistencia. La duración del tratamiento por vía oral
debe ser de 5 a 7 días 2. Los tratamientos menores a 4 días fallan con mayor frecuencia. Tratamientos más prolongados (mayor de 7
días) no tienen ventaja e incrementan el riesgo de efectos adversos. INFECCIONES DE VÍAS URINARIAS ALTAS (PIELONEFRITIS): Menos
de 20% de los pacientes requieren hospitalizaciónción, ya que son de mayor edad, toleran la vía oral y tienen menos manifestaciones
sistémicas. El tratamiento debe iniciarse por vía parenteral por tres a cinco días; debe continuar por vía oral, hasta completar diez a 14
días. El cambio de vía parenteral a oral se basa en la desaparición de la fiebre y la mejoría del estado general. Los esquemas de
tratamiento incluyen monoterapia con cefalosporinas de tercera generación, aminoglucósidos, fluoroquinolonas o la combinación de
un beta lactámico y un aminoglucósido. La elevada frecuencia de resistencia de las enterobacterias a la amplicilina, TMP/SMX y a las
fluoroquinolonas debe tomarse en cuenta en cada región geográfica, antes de considerarlos como opciones de tratamiento para
pielonefritis, ver cuadro 6.

CASO CLINICO
Paciente de 14 años masculino, sin antecedentes personales, ingresa por síndrome febril de 48 hs. de evolución, acompañado de
intolerancia oral y dolor abdominal localizado en hipocondrio derecho y epigastrio, de tipo continuo, con mala respuesta al tratamiento
sintomático. El paciente ingresa en regular estado general, impresiona enfermo, febril (39.5Cº), hemodinamicamente estable, buena
suficiencia cardiorrespiratorio, normohidratado y taquicardico. Mecánica ventilatoria conservada, buena entrada bilateral de aire, sin
ruidos agregados. Ruidos netos, silencios libres, bien perfundido, pulsos periféricos presentes y simétricos con relleno ungueal
conservado. El abdomen era blando depresible y doloroso a la palpación profunda en epigastrio e hipocondrio derecho, con puño
percusión derecha positiva, sin defensa ni contractura muscular. Ruidos hidroaereos presentes. Resto del examen físico sin
particularidades. Refiere diuresis y catarsis conservada. Laboratorio de ingreso: Leucocitosis moderada a predominio neutrofilico
Glóbulos blancos: 18.600 (N: 87/ L: 5) Glóbulos rojos: 4970000 Plaquetas: 162000, Función hepática normal ( TGO: 62 TGP:23), Función
renal normal (Urea: 42 Creatinina: 0.9) Glicemia: 140. VES: 25 mm / hs. Orina completa con sedimento: densidad 1030, pH 6, proteínas
(++++), hemoglobina (+++), células (+), regular cantidad de leucocitos y piocitos. Abundantes hematíes.

CURSO ENARM CMN SIGLO XXI TEL: 36246001 Pharmed Solutions Institute PÁGINA 447
MANUAL DE TRABAJO DEL CURSO ENARM CMN SIGLO XXI

PREGUNTA
Cual es el agente etiológico mas probable?

RESPUESTA
a.- Klebsiella.
b.- Escherichia Coli.
c.- Proteus.
d.- Peptuscoccus.

GLOMERULONEFRITIS. CIENCIAS BASICAS: Es un proceso inflamatorio del glomérulo enfocado desde el punto de vista histológico. La
glomerulonefritis aguda más frecuente es la glomerulonefritis aguda posinfecciosa. CLASIFICACION: Glomerulonefritis Primarias:
Nefropatía por IgA o IgM, Glomerulonefritis proliferativa mesangial (GNPM), Glomerulonefritis membranosa (GNM), Glomerulonefritis
extracapilar (GNEC). Secundarias: Posinfecciosa por bacterias; glomerulonefritis posestreptocóccica (GNPE), Posbacteremia
estafilocócica, neumcócica, sífilis. Virales (hepatitis B, influenza A y B, etc). Parasitarias (malaria, toxoplasmosisi, etc). Enfermedades
Multisistémicas: LES, Purpura de Henoch Scholein, Vasculitis necrosante, Síndrome hemolítico urémico/purpura trombocitopenica
trombotica (SHU/PPT), Granuloma de Wegener. PATOGENIA: Se debe a un proceso inflamatorio del glomérulo casi siempre de origen
inmunológico, este proceso inflamatorio se presenta en el mayor de los casos por la formación de complejos inmunes, cuyos antígenos
pueden ser exógenos como virus, parásitos, bacterias o sus productos, drogas y alimentos. OP bien pueden ser endógenos como la
propia membrana basal glomerular, productos celulares, núcleos, DNA, RNA, mitocondrias, lisosomas como sucede en LES y otras
enfermedades autoinmunes. Siendo los anticuerpos las inmunoglobulinas IgG, IgM, IgA. Constituyéndose a si el complejo inmune tanto
intra como extraglomerular, este complejo tienen la facultad de activar el sistema del complemento, tanto por la vía clásica como por la
vía alterna, una vez activado el sistema induce el proceso inflamatorio por diferentes mecanismos, uno es por ataque directo a la
membrana basal por un sistema de proteínas formadas por C5-C9, otro es por activación del sistema de coagulación y finalmente por la
inducción de la proliferación de células intrínsecas (endoteliales, mesangiales, epiteliales y extrínsecas del glomérulo como son: el
sistema macrófago), monocitos y PMN, los cuales producen factores inflamatorios denominados de daño glomerular como IL-1,
leucotrienos, tromboxano A2, PG E2, radicales de oxígeno, TNF. Todos estos elementos en su conjunto inducen alteraciones en la
permeabilidad de la membrana basal glomerular y disminución de la tasa de filtración glomerular que clínicamente se manifiesta como
hematuria, proteinuria y falla renal en grado variable, se manifiesta con elevación de los azoados en sangre y una disminución en la
depuración de creatinina, estas alteraciones provocan una serie de ajustes renales en los que hay desequilibrio glomerulotubular, en
donde existe una disminución de Na a nivel del túbulo proximal, pero un incremento en la reabsorción absoluta de Na y agua a nivel
distal lo que lleva a un balance positivo de estos elementos, estas dos situaciones son responsable de un incremento de volumen en el
espacio intravascular (disminuye la renina plasmática y aldosterona), aumentando la presión hidrostática intracapilar que aunado a una
disminución de la presión coloidosmotica, por la proteinuria, inducen a un reajuste de las fuerzas de Starling y de esta manera en la
clínica se presenta el edema, aumento de gasto cardiaco (aumento de precarga) y la hipertensión arterial. TRATAMIENTO: Disminuir la
sobrecarga hídrica a través de diuréticos de asa del tipo furosemida dosis 3-5mg/kg/dosis, vigilando frecuentemente el gasto urinario y
balance de liquidos, si no hay resultado se debe emplear procedimiento dialítico. Para coadyuvar al tratamiento de la hipertensión se
podrá usar vasodilatadores sanguíneos del tipo de prazosina 1mg c/8-12hrs o hidralazina a 3mg/kg/día dividido en 3 dosis y
ocasionalmente IECAS. El edema y la hipertensión arterial inician su descenso los primeros días del tratamiento y el síndrome nefrítico
desaparece en términos de 5-10 días, aunque los datos pueden existir hasta 4 semanas, de tal manera de que si la hematuria
macroscópica y la hipertensión arterial persisten después de este periodo o bien existe un síndrome nefrótico asociado y sobre todo
incremento de la creatinina sérica, son indicaciones precisas de biopsia renal. El tratamiento de la glomerulonefritis crónica secundaria
será controlando la enfermedad primaria que le dio origen, por ejemplo dando inmunomodulación en LES, evitando las reacciones
anafilácticas en purpura vascular. O dando quimioterapia cuando son secundarias a neoplasias. Para las glomerulopatias crónicas
primarias; inmunomodulación, anticoagulación, plasmaferesis, citotóxicos, ninguno a demostrado resultado satisfactorio a largo plazo,
la mayoría evoluciona a insuficiencia renal crónica, por lo que la conducta será cuidar la reserva renal y evitar la progresión rápida de la
enfermedad renal terminal. PRONOSTICO: Bueno si se controlan las complicaciones en la fase aguda. La mortalidad es menor de 0.5% y
prácticamente se recuperan en forma espontánea. La recuperación total puede durar hasta 2 años, de ahí que si la hematuria
microscópica dura más de este periodo es indicación de biopsia renal. Si la lesión de los glomérulos fue muy grave el pronóstico es
sombrío y esto dependerá del porcentaje de la formación de “medias lunas”, de modo que si estas son más de 70% de los glomérulos
observados, el paciente llegara a enfermedad renal terminal. GLOMERULONEFRITIS AGUDA POSESTREPTOCÓCICA (GNPE): Es la
infecciosa más frecuente. Se presenta con mayor frecuencia en niños, generalmente entre 5 y 10 años de edad, con ligero predominio
en el sexo masculino. En 85% de los casos se encuentra antecedente de infección faríngea, 7 a 21 días antes de la aparición de las
manifestaciones clínicas de nefropatía; en otros países, la infección previa que predomina es piodermitis. La infección estreptocócica se
asocia a fiebre reumática o glomerulonefritis pero no ambas. El etreptococo nefritogénico, presenta productos proteicos de la pared
bacteriana como son la proteína M y T, de las cepas 1, 2, 4, 12, 18, 25, 49, 55, 57, y 60 que se asocian a nefritis. El daño
fundamentalmente se debe a la formación de complejos inmunes donde el antígeno es un producto celular especifico (proteína M), de
la bacteria o bien un componente propio del organismo que ha sido modificado por la infección y que ha sido reconocido como no
propio, cualquiera que sea este mecanismo este antígeno se une a anticuerpos como IgG, IgM o IgA formando complejos inmunes que
activan el sistema del complemento provocando daño tisular. Posterior a la infección estreptocócica, por lo general de las vías
respiratorias o de la piel, existe un periodo asintomático que va de 15-21 días para posteriormente presentarse el síndrome nefrítico.
Se presenta con mayor frecuencia en la etapa escolar y predomina en varones 2:1. Cuando la infección es en la piel la probabilidad de
presentar nefritis es 5 veces mayor. Laboratorio: EGO; hematuria, proteinuria, eritrocitos dismórficos, leucocituria, cilindros hemáticos
y hialinos. BH; anemia normocItica normocromica, leucocitosis, aumento VSG. Alteraciones bioquímicas; aumento de Cr sérica,
hiponatremia dilucional, hiperpotasemia, acidosis metabilica, disminución de depuración de Cr. Bacteriología; cultivos positivos para
estreptococo del grupo A en piel y nasofaringe. LESION GLOMERULAR MINIMA: Los glomérulos son normales a la microscopia de luz, la

CURSO ENARM CMN SIGLO XXI TEL: 36246001 Pharmed Solutions Institute PÁGINA 448
MANUAL DE TRABAJO DEL CURSO ENARM CMN SIGLO XXI
inmunofluorescencia es negativa y en la microscopia electrónica hay fusión de los podocitos. Clínicamente se expresa como síndrome
nefrótico puro corticosensible. GLOMERULONEFRITIS PROLIFERATIVA: En esta lesión encontramos proliferación endocapilar que
significa un aumento de las células dentro del penacho vascular: células mesangiales, endoteliales y podocitos (proliferación
centrifuga), a su vez esta lesión puede ser focal, cuando se encuentra en menos de la mitad de los glomérulos observados, o bien difusa
cuando la lesión esta en más de la mitad de dichos glomérulos, las manifestaciones clínicas son graves: casi siempre síndrome nefrótico
y síndrome nefrítico con fallo renal. Hay que señalar que las lesiones difusas se pueden acompañar de lesiones extracapilares, es decir
proliferación de estructuras fuera del penacho vascular, o células epiteliales de la capsula de Bowman (proliferación centrípeta),
formando lo que se conoce como “medias lunas”, es importante reconocerlas ya que todas evolucionan a fibrosis y esclerosan el
glomérulo en donde se encuentran, condicionando falla renal irreversible, si hay más de 70% de afección, puede llevar a enfermedad
renal terminal en un promedio de 3 meses. GLOMERULONEFRITIS MEMBRANOPROLIFERATIVA (GNMP): Existe una proliferación
importante de células mesangiales, así mismo hay un engrosamiento y desdoblamiento de la membrana basal glomerular, dando un
aspecto de doble contorno o una imagen de “doble riel”. Clínica; se expresa como síndrome nefrótico y nefrítico con complemento en
sangre bajo, se presenta con mayor frecuencia en adolescentes y predomina en el género femenino. O bien presentarse como
hematuria, proteinuria o hipertensión en forma iaslada. Esta es una de las glomerulopatias crónicas más frecuentes y que ocupa uno de
los primeros lugares como causa de insuficiencia renal crónica. GLOMERULONEFRITIS MEMBRANOSA (GNM): Poco frecuente en
pediatría, casi siempre es secundaria a enfermedades autoinmunes como LES o enfermedades granulomatosas. En las características
histológicas hay un engrosamiento importante de la membrana basal glomerular, la cual protruye a través de los podocitos dando una
imagen de “espinas” que el capilar asemeja “una rueda dentada”. No hay proliferación de células mesangiales. La manifestación clínica
es un síndrome nefrótico impuro corticoresistente, asociado a las características de la enfermedad principal, aunque hay GNM
idiopática.

CASO CLINICO
Paciente de 15 años con historia de 1 semana de decaimiento progresivo, palidez, náuseas con vómitos ocasionales y cefalea vespertina
intensa. Dos días previos al ingreso se agregó edema periorbitario, mayor decaimiento y se constató bajo volumen urinario. Al
momento de la admisión al hospital, la paciente presentaba discreto edema facial y de extremidades inferiores, palidez de piel y
mucosas, presión arterial de 150/105 y frecuencia cardíaca de 82 x'. Entre los exámenes de ingreso destacaban hematocrito 19,9%,
leucocitos 9500 x mm3 (53% neutrófilos), plaquetas 150.000 x mm3, velocidad de eritrosedimentación 62 mm/h, protrombina 86%,
TTPK 11,7'', nitrógeno ureico 106 mg/dl, creatininemia 10,1 mg/dl, uricemia 6,9 mg/dl, calcio 7,4 mg/dl, fósforo 6,7 mg/dl, sodio 136,6
mEq/l, potasio 6,6 mEq/l, cloro 107 mEq/l. Examen de orina: proteinuria 2 g/l, hematíes incontables por campo, cilindros hialinos,
céreos y gruesos con inclusiones granulosas numerosos en la preparación.

PREGUNTA
Cual es la conducta terapéutica inmediata mas apropiada para el caso?

RESPUESTA
a.- Diurético, antihipertensivo, hemodiálisis aguda y biopsia renal.
b.- Diurético, antihipertensivo, hemodiálisis aguda y metilprednisolona en bolos de 1 g al día.
c.- Diurético, antihipertensivo, hemodiálisis aguda, metilprednisolona y biopsia.
d.- Diurético, antihipertensivo, metilprednisolona en bolos de 1 g al día.

SINDROME NEFRITICO. CIENCIAS BASICAS: Es la expresión clínica de las glomerulonefritis sobre todo de las agudas, es decir, de un
proceso inflamatorio renal con asiento exclusivo o predominante en el glomérulo. En el desarrollo de dicho proceso, intervienen
fundamentalmente los mecanismos de la respuesta inmune, se caracteriza por edema, hematuria, oliguria e hipertensión arterial, en la
mayoría de los casos hay hipocomplementemia. Las glomerulopatías agudas no se manifiestan necesariamente con el síndrome
nefrítico. En efecto, pueden también presentarse como síndrome nefrótico, síndrome urémico, hematuria recurrente y, más aún,
pueden cursar en forma totalmente asintomática. El síndrome nefrítico es una asociación de hallazgos clínicos y la expresión
<<glomerulonefritis>> indica una lesión anatómica. Debe enfatizarse que estos términos no representan ninguna definición
etiopatogénica de la enfermedad. PATOGENIA: El síndrome nefrítico puede presentarse en ausencia de etiología infecciosa; las causas
más frecuentes son lupus eritematoso sistémico, púrpura de Henoch-Schönlein, glomerulonefritis por IgA, crioglobulinemia esencial,
síndrome de Guillain-Barré y nefritis pos-radiación. La incidencia de GLOMERULONEFRITIS AGUDA POSESTREPTOCÓCICA (descrita en el
tema de glomerulopatias), ha disminuido en varios países desarrollados, pero aún puede considerarse como la causa más frecuente de
síndrome nefrítico agudo a cualquier edad. Prácticamente en todos los casos existen evidencias de patogenia inmunológica por
formación de complejos inmunes, tales como disminución de los niveles de complemento hemolítico en suero, detección glomerular de
inmunoglobulinas y complemento de aspecto granular y periodo de “latencia” entre la infección y las manifestaciones clínicas de lesión
renal. La activación del sistema de complemento es inducida por la formación de complejos antígeno-anticuerpo. DIAGNOSTICO:
Clínica; edema, es uno de los signos cardinales, habitualmente se localiza en cara, es matutino, blando, pálido (fascies nefrítica), en
niños pequeños suele encontrarse anasarca y en niños mayores o adolescentes es pedio o pretibial. Hipertensión arterial: relevante
porque en niños suele ser asintomática, pero esta es la complicación más grave de la enfermedad ya que puede evolucionar a crisis
hipertensiva que aunado al edema agudo pulmonar, son las causas de muerte en la fase aguda. Hematuria: es un hallazgo universal en
el síndrome, es macroscópica hasta 60%, esta es obscura, indolora, total, sin coágulos. En el sedimento urinario hay cilindros hemáticos
y eritrocitos dismorficos. Proteinuria: si hay suele ser <40mg/h/m2 SC, encontrándose en un 30-60% de los casos, es una proteinuria no
selectiva. Oliguria: la presencia de esta o de anuria es de mal pronóstico, es el signo más inconstante, cuando se presenta la
glomerulonefritis es muy grave, sobre todo cuando es secundaria a vasculitis, enfermedades autoinmunes, con lesiones de necrosis
cortical y obliteración de los capilares glomerulares. TRATAMIENTO: Debe estar enfocado a las complicaciones sobre todo de la
hipertensión arterial y sus consecuencias como la encefalopatía hipertensiva y la insuficiencia cardiaca, es decir, disminuir la sobrecarga
hídrica a través de diuréticos de asa del tipo furosemida dosis 3-5mg/kg/dosis, vigilando frecuentemente el gasto urinario y balance de

CURSO ENARM CMN SIGLO XXI TEL: 36246001 Pharmed Solutions Institute PÁGINA 449
MANUAL DE TRABAJO DEL CURSO ENARM CMN SIGLO XXI
líquidos, si no hay resultado se debe emplear procedimiento dialítico. Para coadyuvar al tratamiento de la hipertensión se podrá usar
vasodilatadores sanguíneos del tipo de prazosina 1mg c/8-12hrs o hidralazina a 3mg/kg/día dividido en 3 dosis y ocasionalmente IECAS.
El edema y la hipertensión arterial inician su descenso los primeros días del tratamiento y el síndrome nefrítico desaparece en términos
de 5-10 días, aunque los datos pueden persistir hasta 4 semanas, de tal manera de que si la hematuria macroscópica y la hipertensión
arterial persisten después de este periodo o bien existe un síndrome nefrótico asociado y sobre todo incremento de la creatinina sérica,
son indicaciones precisas de biopsia renal. Los casos en los que se demuestra infección estreptocócica, deben recibir tratamiento de
erradicación a base de penicilina. El esquema terapéutico aconsejado es la aplicación intramuscular de 800,000 unidades diarias de
penicilina procaínica durante cuatro días, seguidas de una dosis de 1´200,000 U de penicilina se puede emplear eritromicina oral por 10
días a dosis de 30 mg/kg/día. La erradicación del estreptococo no influye sobre la lesión renal y sólo tiene interés epidemiológico para
evitar la diseminación del germen. COMPLICACIONES: Sobrecarga hidrosalina, la insuficiencia cardiaca, encefalopatía hipertensiva,
uremia e hipercaliemia infecciones que desempeñan un papel etiológico en el desarrollo de la glomerulonefritis e infecciones
sobreañadidas, insuficiencia renal irreversible.

CASO CLINICO
Varón 3 años, acude por: coluria, edema palpebral mañana del ingreso, abdomen globuloso y edemas de miembros pelvicos, ganancia
de peso, refiere la madre un cuadro catarral hace una semana con Tª máx 37.6ºC, Peso: 19.4 kg Talla: 99 cm Sc: 0.74 m2, Fc: 94 Fr: 20
Tª: 36.5ºC TA: 140/100 (p>95), Edemas pretibiales que dejan fóvea con ligero edema testicular. Hemo: Hemat: 3,68, Hb: 8.8, Hto: 26.2
%, VCM: 71, HCM: 24.4 pg Leucos: 19.680 (S 44%, L 46%, M7%), Plaq: 400.000. Bioq: Gluc: 94, Urea: 69, Creat: 0.68, PT: 6.07, Na: 131, K
5.36, PCR 17.7, Ca: 8.67, P: 5.13. Coag: Tp: 10.9, IQ: 100, Aptt: 26.7, Fibrinógeno: 420. Gasometría: pH: 7.42, pCO2: 32.5, HCO3: 21.2,
EB: - 3.2. Sedimento orina: Densidad 1020, pH 6.0, Prot >1000. 200-250 hemat/campo (10% dismórficos), 4-6 leucos/campo, escasos
cilindros hialino-granulosos.

PREGUNTA
Cual de los siguientes factores es la mas adecuada para identificar la evolución del padecimiento de base?

RESPUESTA
a.- Diuresis
b.- Densidad de la orina.
c.- Fiebre.
d.- Tension arterial.

SINDROME NEFROTICO (SN). CIENCIAS BASICAS: Es la asociación de proteinuria masiva, hipoalbuminemia y edema y a esta triada se
asocia habitualmente hiperlipidemia, lipiduria y alteración de la coagulación y ocasionalmente hay hipertensión arterial, hematuria o
falla renal. El síndrome nefrótico es una de las expresiones clínicas de las glomerulopatias, es decir no hay SN en las tubulopatias o en
las uropatías. SALUD PUBLICA: En EUA la incidencia en niños menores de 16 años es de 2 por cada 100,000 niños. El 74% de los niños
(2-7 años de edad) presenta síndrome nefrótico con lesión glomerular mínima. Predomina en varones 2:1. CLASIFICACION: SN primario
o idiopático: no se conoce una causa específica y SN secundario: LES, amiloidosis, purpura de Henoch-Schonlein, vasculitis necrosante,
artritis reumatoide, DM, hipotiroidismo, enf. De Alport, síndrome nefrótico congénito, enf. De Fabri, neoplasias, infecciones bacterianas
(glomerulonefritis posestreptocócica, endocarditis bacteriana, nefritis por shunt, sífilis), virales, hepatitis, drogas. PATOGENIA: La
proteinuria masiva (>40mg/hora/m2 SC): se puede presentar por un daño anatómico o estructural de la barrera glomerular (endotelio
vascular del capilar glomerular, membrana basal glomerular y podocito) entre la sangre y el espacio urinario, lo que se conoce como
proteinuria no selectiva, es decir que en la orina se encuentra albumina, transferrinas y globulinas, habitualmente el daño se hace por
complejos inmunes vía la activación del complemento y bien por anticuerpos contra la membrana basal. Pero también puede haber un
daño fisiológico, o sea, que se pierda la capa electronegativa de esta barrera y esto sucede en las etapas tempranas de la nefropatía
diabética, hipertensión arterial y algunas alteraciones inmunológicas, la proteinuria de esta característica se le conoce como proteinuria
selectiva, es decir, albuminuria. El edema: es la acumulación del líquido en espacio intersticial, por alteración en el equilibrio de las
fuerzas de Starling. Hay tres mecanismos que contribuyen a su formación: la hipoalbuminemia que contribuye a disminuir la presión
oncótica del plasma, pero al mismo tiempo debe haber incremento en la presión hidrostática, esto hace que el paciente tengan balance
positivo de Na y H2O, además si el paciente tiene una lesión glomerular compleja, disminuye la tasa de filtración glomerular que
contribuye a la retención de líquidos. Hiperlipidemia: se conoce que las lipoproteínas de densidad baja y muy baja deben metabolizarse
en partículas más pequeñas, para ingresar a los tejidos, este paso metabólico es realizado por una enzima lipoproteinlipasa, la cual es
activada por un cofactor conocido como ApoII, esta es una pequeña proteína que se pierde en la orina de los pacientes son síndrome
nefrótico, por otro lado a nivel hepático no se capta las lipoproteínas de baja densidad y del mismo modo que aumenta la síntesis de
albumina, aumenta la de colesterol. Trastorno de la coagulación: El tromoboembolismo es una complicación grave del síndrome
nefrótico, es raro en niños, as común en adolescentes. DIAGNOSTICO: Si se sospecha SN, lo primero es confirmar la proteinuria, de
manera cuantitativa a través de proteinuria horaria con colección de orina de 24 hrs, es masiva cuando el resultado es >40mg/h/m2 de
superficie corporal, por lo difícil de la colección y la no indicación de colocación de sonda vesical para la colección, se ha planteado la
realización de índices urinarios proteinuria/creatininuria >2.5 =proteinuria masiva. En sangre encontramos hipoproteinemia <5g/dl,
hipoalbuminemia <2.5g/dl, hipercolesterolemia, hipertrigliceridemia (elevación de lipoproteínas de baja y muy baja densidad). BH,
aumento de Hb, de Hto y tendencia a la plaquetosis. Las inmunoglobulinas se encuentran disminuidas. TRATAMIENTO: Enfocado en
tratamiento de complicaciones extrarrenales y de las glomerulopatías. Para edema: reposo relativo, restringir de dieta la ingesta de Na
y agua, solo se administrara la cantidad de líquidos por perdidas insensibles y egresos, se restringen las proteínas o se da una dieta
normoproteica. El uso de diuréticos solos o en combinación con bloqueadores de la aldosterona o con el uso de albumina. Furosemida
1-3mg/kg por dosis c/8h. Espironolactona 1-3mg/kg/dia. El uso de albumina humana pobre en sal tiene indicaciones precisas las cuales
son: edema refractario al uso de diuréticos, si la ascitis o anasarca intervienen con la mecánica respiratoria o si el edema representa
una deformidad corporal importante. La dosis 0.5-1g/kg/dosis, si hay elevación de creatinina sérica no se debe aplicar este

CURSO ENARM CMN SIGLO XXI TEL: 36246001 Pharmed Solutions Institute PÁGINA 450
MANUAL DE TRABAJO DEL CURSO ENARM CMN SIGLO XXI
medicamento. El uso de statinas y los anticoagulantes no están indicados en pediatría. Los antibióticos serán de acuerdo a infecciones
especificas (de preferencia no aplicar IM). SINDROME NEFROTICO CON LESION GLOMERULAR MINIMA (LGM): Representa la situación
más frecuente en pediatría, la edad de presentación e después de 1 año y antes de los 7 años de edad, casi siempre presentan un
síndrome nefrótico puro, es decir, sin hematuria, sin hipertensión arterial (que son componentes del síndrome nefrítico), y la
característica más importante es que estos pacientes responden muy bien al uso de esteroides (corticosensible), se debe a una
alteración del equilibrio de linfocitos CD4 y CD8 con predominio de estos últimos los cuales liberan IL-2, la cual neutraliza la
electronegatividad de la membrana basal glomerular con la proteinuria subsecuente. Tratamiento: prednisona a 60mg/m2 de superficie
corporal, dosis única por la mañana por 4 semanas, posteriormente 40mg/m2/SC días alternos por 4 semanas.

CASO CLINICO
Se trata de preescolar que es traido a la consulta debido a la aparición brusca de 48 horas de edema facial, asi como de miembros
pélvicos edematosos, con escasa micción urinaria. El único antecedente de importancia es un proceso infeccioso en garganta que
refiere la madre, tratato con medidas caseras, los laboratorios de relevancia fueron los siguientes: Hipoalbuminemia (< 2,5 g/dl).
Proteinuria 42 mg/m2/hora y dislipidemia. A la EF se observa decaimiento, anorexia, vómitos y dolor abdominal. TA nornal.

PREGUNTA
Cual de los siguientes factores es la mas adecuada para identificar la evolución del padecimiento de base?

RESPUESTA
a.- Diuresis
b.- Densidad de la orina.
c.- Fiebre.
d.- Tension arterial

GASTROENTERITIS (GEA). CIENCIAS BASICAS: La GEA suele considerarse como “una inflamación de la mucosa gástrica e intestinal
producida por un agente infeccioso o toxico, que se traduce clínicamente en un cuadro de diarrea de instauración rápida”,
generalmente es infecciosa y autolimitada. GEA aguda: Disminución de la consistencia de las heces (blandas o líquidas) y/o un
incremento en la frecuencia de evacuación (más de 3 en 24 horas) con o sin fiebre o vómitos, de una duración habitualmente menor a 7
días y nunca superior a 14 días. SALUD PUBLICA: Son la segunda causa de muerte infantil en el mundo, los más vulnerables son los
menores de 5 años. PATOGENIA: Etiología; 1. Virus (rotavirus, virus de Norwalk, otros). 2. Bacterias (S. aureus, Bacilus cereus,
Clostridium perfirngens, difficile y botulinum, Salmonella, Shigella, Campylobacter, listerya monocytogenes, Vibrio cholerae, Yersinia,
Eschericgia coli). 3. Parasitos (Entamoeba hystolitica, Giardia lamblia, Isospora belli). 4. Malaabsorcion (intolerancia a la lactosa,
enfermedad celiaca, fibrosis quística, intolerancia a proteínas de la leche, y/o alimentos específicos). 5. Farmacos (antobioticos,
laxantes, quimioterapia, antiácidos, diuréticos). 6. Toxinas (hongos o setas, crústaceos, pescados o mariscos, conservadores de
alimentos, metales pesados). 6. Otros (mala técnica alimenaria, condimentos, alimentación inadecuada para edad, radioterapia). En
Mexico, los agentes que ocasionan hasta 70% de los cuadros diarreicos son: rotavirus, especies de Shigella, E. coli enterotoxigénica, E.
coli enteropatógena y Campylobacter. La infección se adquiere por la vía oral, a partir de un enfermo, de un portador asintomático, o
de un reservorio animal; con transmisión de forma directa, a través de alimentos contaminados o de vectores. Puede aparecer como un
caso esporádico o en brotes, con mayor frecuencia durante el verano. Los cuadros esporádicos son debidos a cualquiera de los agentes
citados anteriormente, pero los brotes suelen ser producidos por Salmonella o por toxinas estafilocócicas preformadas. Existen tres
mecanismos de producción de las manifestaciones clínicas: .1. Síntesis de toxinas: Que alteran los procesos de manejo hidroelectrolítico
celular a través del AMPc, inhibiendo la absorción de los iones sodio y cloro, y estimulando la secreción de cloro y bicarbonato.2.
Invasión directa de la mucosa intestinal: Destruyen el borde en cepillo y las células adyacentes, provocando inflamación local y
ulceración.3. Mecanismo mixto o no preciso: Por posible adherencia directa o secreción aumentada de moco. Actúan así agentes como
Giardia lamblia y Escherichia coli enteropatógena. DIAGNOSTICO: El objetivo de la valoración clínica bes la distinción entre los casos
leves, benignos y autolimitados. Debe considerarse según el tipo de diarrea, perdidas hídricas alcanzan entre 10-200ml/kg; que supone
el 20% del peso corporal que puede constituir una situación de urgencia. Los factores a identificar para una valoración correcta son: 1.
Gravedad de la enfermedad. 2. Duración de la diarrea. 3. Contexto epidemiológico, en el que aparece el caso. 4. Situación del huésped
en cuanto a inmunidad y defensas. En primer lugar se debe delimitar lo que se considera como diarrea: de acuerdo con los criterios
establecidos, evacuaciones de 200g, casi siempre líquidos y aumento de la frecuencia. En una primera evaluación serán elegidos como
potencialmente graves a los que presenten alguno de los siguientes signos: deshidratación importante con sensación de sed o
reducción de diuresis; diarrea sanguinolenta; dolor abdominal o fiebre elevada. En la exploración física se consideran como datos que
confirman la sospecha de severidad establecida en la anamnesis, los siguientes; obnubilación o disminución del nivel de consciencia,
sequedad de mucosas, hipotensión ortostatica, en la exploración del abdomen, la presencia de resistencia, dolorimiento o signos de
peritonitis. Se consideran criterios de hospitalización; diarrea hemorrágica, signos de toxicidad sistémica (obnubilación), fiebre alta,
deshidratación severa, distensión abdominal. Por otra parte debe considerarse que algunas infecciones por Salmonella, Shigella,
Yersinia y E. coli enterohemorrágica, pueden complicarse con afectación de otros órganos o sistemas: síndrome hemolítico urémico,
PTT, Síndrome de Reiter, pericarditis, glomerulonefritis. En cuanto a la duración de la diarrea, la persistencia de la sintomatología
durante 4-5 días es un rasgo de severidad, por asociarse a microorganismos más virulentos. Pertenecen a grupos de alto riesgo los
menores de 5 años, por la situación inmunitaria y defensas del huésped, para cuya atención y protección se han elaborado normas y
programas en diversas partes del mundo, sobre todo en los países en vías de desarrollo como México. SIGNOS DE PELIGRO: No puede
beber o amamantarse, vomita todo, tienen convulsiones, esta letárgico o inconsciente, muestra ausencia o disminución de los ruidos
intestinales. TRATAMIENTO: Factores de mal pronóstico: 1. Menor de 2 meses. 2. Desnutrición moderada a grave. 3. Presencia de
inmunodeficiencia. 4. Muerte de un menor de 5 años en la familia. 6. Madre analfabeta o menor de 17 años. 7. Dificultad para el
traslado al médico si se agrava el niño. La enfermedad es casi siempre autolimitada y la mortalidad se relaciona con las complicaciones,
en las que la deshidratación es la causa de muerte en el 70% de los casos. El adecuado manejo de la deshidratación con VSO evita 90%

CURSO ENARM CMN SIGLO XXI TEL: 36246001 Pharmed Solutions Institute PÁGINA 451
MANUAL DE TRABAJO DEL CURSO ENARM CMN SIGLO XXI
de estas muertes, por eso las acciones contra la diarrea incluyen este recurso, impulsando su uso en los casos. Par el tratamiento es
necesario evaluar el estado de hidratación del paciente, por lo que se deberá considerar las siguientes definiciones operacionales. CASO
SIN DESHIDRATACION: Es aquel que presentan menos de cuatro evacuaciones liquidas en 24h, ausencia de vómito, sin signos clínicos de
deshidratación. CASO CON DESHIDRATACION: Es aquel que presenta dos o más de las manifestaciones clínicas siguientes: Inquieto o
irritable, ojos hundidos, llanto sin lágrimas, boca y lengua seca, saliva espesa, respiración rápida, sed amentada, bebe con avidez,
elasticidad de la piel, > a 2 seg., pulso rápido, llenado capilar de 3-5seg., fontanela anterior hundida (lactantes). CASO CON CHOQUE
HIPOVOLEMICO: Es aquel que presenta dos o más de las manifestaciones siguientes: inconsciente o hipotónico, no puede beber, pulso
débil o ausente, llenado capilar mayor de 5 seg. De acuerdo con la valoración, el manejo de los pacientes que presentan algún grado de
deshidratación se basa en 3 planes. PLAN A: Para pacientes con enfermedad diarreica sin deshidratación con atención en el hogar.
Continuar con alimentación habitual, aumentar la ingestión de líquidos de uso cotidiano en el hogar, Sueros de VSO, en los menores de
1 año, ofrecer media taza (75ml) y en lo mayores de un año una taza (150ml) y administrarlo a cucharadas o mediante sorbos pequeños
después de cada evacuación. Capacitar a la madre para reconocer los signos de deshidratación y datos de alarma por enfermedades
diarreicas (sed intensa, poca ingestión de líquidos y alimentos, numerosas heces liquidas, fiebre, vómito y sangre en las evacuaciones)
con el propósito de que acuda nuevamente a solicitar atención medica en forma oportuna. PLAN B: Pasar pacientes con diarrea y
deshidratación con atención en la unidad de salud; administra VSO 100ml/kg, en dosis fraccionada cada 30min durante 4 hrs por vía
oral. Si el paciente presenta vómito, esperar 10 minutos e intentar otra vez la hidratación oral más lentamente. Al mejorar el estado de
hidratación, pasar a plan A. En caso contrario repetir el plan B en 4 hrs, de no existir mejoría pasar al plan C. PLAN C: Pacientes con
choque hipovolémico por deshidratación. Iniciar inmediatamente administración de líquidos por vía IV, con solución de Hartmann, si no
se encuentra disponible, use sol. Salina al 0.9%, de acuerdo con el siguiente esquema: Primera hora 50ml/kg, segunda hora 25ml/kg,
tercera hora 25ml/kg. Evaluar al paciente continuamente. Si no mejora aumentar la velocidad de infusión. Cuando pueda beber
(usualmente 2-3hrs), administrar sobres de VSO, a dosis de 25ml/kg/h, mientras sigue líquidos IV. Al completar la dosis IV, evaluar al
paciente para seleccionar plan A o B, y retirar venoclisis, o repetir plan C. Si selecciona el plan A, observar durante 2 hrs para asegurarse
de que el responsable del paciente pueda mantenerlo hidratado con sobres de VSO, además alimentarlo en su domicilio.
ANTIMICRONIANOS: No son útiles en el tratamiento de las enfermedades diarreicas en 90% de los casos, el manejo inadecuado de
antimicrobianos puede propiciar que la enfermedad se prolongue y ocasione resistencia bacteriana. Los antimicrobianos debe
prescribirse en pacientes con diarrea por: especias de Shigella, Vibrio cholerae, trofozoitos de Entamoeba histolytia o Giardia lamblia.
Cuando el paciente presenta fiebre, se deben utilizar medios físicos para su control, hidratar al paciente, mantenerlo con ropa ligera y
de ser necesario, darle un baño con agua fría. Puede emplearse un antipirético como acetaminofén a dosi de 30-40kg/dia en niños de 2
meses a 4 años de edad, repartidos cada 4-6 hrs, sin pasar de 5 en 24 hrs. COMPLICACIONES: Choque hipovolémico, desequilibrio
hidroelectrolítico, acido-base, ileo paralitico, diarrea persistente, insuficiencia renal, neumatosis intestinal, infarto intestinal,
perforación intestinal, peritonitis. PREVENCION: Vigilancia de la cloración del agua, procedimientos de desinfección (potabilizcion,
ebullición, cloración, yodación), promoción de la lactancia materna (exclusiva los primeros 4-6 meses de edad), mejorara las prácticas
de ablactación (a partir del 4to mes), promoción de la higiene en el hogar (lavado de manos, eliminación adecuada de excretas, manejo
adecuado de heces en niños), vacunación contra el sarampión, complemento con vitamina A.

CASO CLINICO
Un niño de 4 años de edad, que inicia hace 2 dias con dolor difuso abdominal con 5 evacuaciones semiliquidad, niega fiebre, nauseas o
vomito. A la exploración física se observa alerta y reactivo, signos vitales dentro de rango, incremento de movimientos intestinales y se
ausculta periltaltismo incrementado, las mucosas orales se observan hidratadas, su llanto contiene lágrimas, se encuentra irritable,
como antecedente se encuentra en tratamiento para desnutrición y la familia es de escasos recursos pero sus inmunizaciones se
encuentran al corriente.

PREGUNTA
Cual es la conducta a seguir mas adecuada?

RESPUESTA
a.- Solucion de rehidratación oral.
b.- Antibioticos profiláctico.
c.- Envio a casa con SRO.
d.- Proporcionar datos de alarma.

CASO CLINICO
Femenino de 3 años de edad la cual inicia 3 dias previos a su ingreso, la madre refiere diarrea con moco y sangre, asi como fiebre alta
no cuantificada, esto se acompaño de rinorrea y dificultad para consiliar el sueño, a la exploración física se observa, decaída con
aspecto deteriorado y respuesta alterada irritable, signos vitales taquicardico y taquipnea, se observa disminución de la turgencia de la
piel, se ingresa a urgencias para tratamiento.

PREGUNTA
Cual es el volumen recomendado de SRO para este caso.

RESPUESTA
a.- SRO 30 ml/kg durante 4 hrs.
b.- SRO 50 ml/kg la 1era hora luego 30 ml/Kg.
c.- SRO mayor a 50 ml/kg durante 4 horas.
d.- Plan de hidratacion intravenoso.

CURSO ENARM CMN SIGLO XXI TEL: 36246001 Pharmed Solutions Institute PÁGINA 452
MANUAL DE TRABAJO DEL CURSO ENARM CMN SIGLO XXI
CASO CLINICO
Un niño de 9 años de edad fue hospitalizado debido a dolor agudo abdominal y vómitos. Fue diagnosticado inicialmente con
apendicitis. El día 3 después de la admisión, aunque estaba totalmente alerta y orientado, su examen se caracterizó por
empeoramiento presentando disfonía, boca seca, ojos secos, estreñimiento, abdomen timpánico, retención urinaria, taquicardia,
taquipnea y disnea.

PREGUNTA
Cual es la conducta a seguir.

RESPUESTA
a.- SRO 30 ml/kg durante 4 hrs.
b.- SRO 50 ml/kg la 1era hora luego 30 ml/Kg.
c.- SRO mayor a 50 ml/kg durante 4 horas.
d.- Plan de hidratacion intravenoso.

PARASITOSIS. CIENCIAS BASICAS: Los parásitos


intestinales afectan con más frecuencia a los niños
que a los adultos. Los viajes, la inmigración y la
adopción internacional han hecho que su incidencia
aumente en los últimos años. Aunque los síntomas
clínicos con frecuencia son inespecíficos, algunos
datos de la anamnesis pueden orientarnos en la
predicción del microorganismo causante.
DIAGNOSTICO: En general es preciso recoger tres
muestras de heces en días diferentes para tener una
mayor seguridad en la detección de los parásitos.
Áscaris lumbricoides (nematodo): Al ingerir los
huevos infectivos, las larvas inician un ciclo en el
organismo (circulación, pulmones, tráquea, faringe,
intestino). Los gusanos adultos de A. lumbricoides
viven en el intestino delgado. Diagnóstico. Examen
parasitológico de las heces donde se visualizan
huevos del parásito. Clínica, durante la migración
pulmonar: neumonitis generalmente asintomática en
ocasiones síndrome de Löffler (infiltrados
pulmonares transitorios, fiebre, disnea, tos,
sibilancias y eosinofilia). Fase intestinal:
generalmente asintomática. Síntomas dispépticos, diarrea. Raramente, obstrucción intestinal y vólvulo. Posibilidad de colecistitis,
colangitis, absceso hepático, apendicitis, diverticulitis o pancreatitis por migración del parásito. Tratamiento. Mebendazol (100 mg/12
h/3 días o 500 mg a dosis única). Alternativas: pamoato de pyrantel, 10 mg/kg (máximo 1 g), dosis única; albendazol, 400 mg, dosis
única (niños entre 2-5 años: 200 mg); piperazina, 75 mg/kg (máximo 3,5 g en adultos o 2,5 g en niños entre 2-12 años). La ivermectina
también es eficaz. (200 μg/ g dosis nica). Enterobius vermicularis (oxiuro, nematodo): El vermes adulto vive en el ciego y en las zonas
adyacentes. La hembra migra hacia la zona anal para hacer la puesta. Diagnóstico. El examen parasitológico de heces sólo es positivo en
un 10-15% de los casos en los que se identifican los huevos del parasito o gusano adultos. Mejores resultados tiene el examen de cintas
adherentes transparentes después de su aplicación en los márgenes del ano. En caso de exámenes repetidamente negativos y alta
sospecha de la infección, realizar tacto rectal para la obtención de muestra fecal. Clínica: Asintomático, prurito anal y perineal de
predominio nocturno. En casos excepcionales, colitis, dolor abdominal y granulomas peritoneales. Puede provocar vulvovaginitis en
mujeres jóvenes. Epidemiologia y profilaxis: Cosmopolita, transmisión mediante ropa y sabanas contaminadas. Después de ser
ingeridos, los huevos de E. vermicularis liberan las larvas en el duodeno y migran hacia el íleo. Medidas higiénicas básicas: lavarse las
manos después de la defecación. Se recomienda dar el tratamiento a toda la familia para interrumpir la transmisión. Strongyloides
stercoralis (nematodo): Los gusanos adultos de S. stercoralis viven en el intestino delgado. Tiene capacidad de autoinfestación, base de
la larga persistencia en el organismo infectado (décadas). Diagnóstico. Examen parasitológico de las heces. Cultivo para estrongiloides.
Produce hipereosinofilia periférica (puede faltar en personas inmunodeprimidas). Demostración de larvas en el aspirado duodenal
obtenido mediante fibrogastroscopia. Clinica durante la fase pulmonar, las larvas migratorias, pueden ocasionar infiltrados
eosinonofílicos y tos (síndrome de Löeffler). Manifestaciones gastrointestinales: dispepsia, dolor abdominal, diarrea intermitente.
Puede producir un síndrome de malabsorción. Capaz de autoinfestaciones. En personas inmunodeficientes puede producir un
Sindrome de hiperinfestación de pronóstico grave y elevada mortalidad. Puede producir manifestaciones cutáneas: larva currens. Las
personas se infectan cuando las larvas filariformes del nematodo penetran a través de la piel, en contacto con el suelo infectado.
Despues de un ciclo dentro del organismo humano (piel, circulación, pulmones, tráquea, faringe), los gusanos adultos viven en el
intestino. Entamoeba hystolitica (protozoo): Es un patógeno, con capacidad invasiva y de diseminación extraintestinal. En el intestino
afecta al intestino grueso. Diagnóstico. Examen de parásitos en heces (visualización de trofozoitos y/o quistes); Detección de antígeno
en heces. Coprocultivo en medios especiales. Serología. Examen anatomopatológico de biopsia intestinal. Difícilmente se visualiza E.
histolytica en el pus de los abscesos hepáticos, el cual tiene una coloración amarronada (pasta de anchoas). Pruebas de imagen
(ecografía, TC). La amibiasis puede manifestarse como cuadro diarreico; molestias abdominales inespecíficas; disentería amebiana;
colitis fulminante con desarrollo de megacolon tóxico; ameboma. Puede afectar a zonas extraintestinales, siendo el absceso hepático
amebiano la localización más frecuente que puede extenderse a zonas contiguas (pleura, pulmón, pericardio, etc.); otras afectaciones

CURSO ENARM CMN SIGLO XXI TEL: 36246001 Pharmed Solutions Institute PÁGINA 453
MANUAL DE TRABAJO DEL CURSO ENARM CMN SIGLO XXI
son mucho menos frecuentes (cerebrales, cutáneas,
genitales). Hay personas asintomáticas. Indicaciones
quirúrgicas. En casos de megacolon tóxico y
perforación. Punción aspirativa transcutánea de los
abscesos amebianos, sólo indicado en aquellos
abscesos de gran tamaño con riesgo de rotura hacia
otras estructuras adyacentes. Giardia lamblia
(protozoo): Produce colonización del intestino delgado,
con adherencia del parásito a la mucosa intestinal.
Diagnóstico. Identificación del parásito en las heces.
Detección de antígeno en heces (ELISA). Clínica.
Asintomática. Dispepsia, diarrea crónica intermitente.
Síndrome de malabsorción sobre todo en pacientes
inmunodeprimidos (sida, déficit de IgA). Taenia
saginata (cestodo): Reside en el intestino delgado del
huésped. Diagnóstico. Identificación de las proglótides
en las heces de los pacientes. No puede diferenciarse
de la T. solium excepto con el recuento de las ramas
uterinas de las proglótides grávidas. Clínica. Asintomática en la mayoría de los casos; síntomas dispépticos. Anorexia, urticaria, prurito,
cefalea, convulsiones. De forma inusual pueden producirse complicaciones por la migración de proglótides a lugares como apéndice,
conducto biliar o pancreático. Epidemiología. Cosmopolita. Transmisión oral a partir del consumo de carne cruda (o poco cocida) de
ganado vacuno contaminada. Taenia solium (cestodo): Vive en el intestino delgado del huésped. Clinica y diagnostico igual que T.
saginata, La cisticercosis, una enfermedad que puede afectar al SNC y también al globo ocular, es una de las complicaciones de la T.
solium. Trichuris trichiura (nematodo): Los parásitos adultos de T. trichiura viven en el colon y el recto, adheridos a la mucosa
intestinal. Criterios diagnósticos. Examen parasitológico de las heces, donde se visualizan los huevos del parásito. Clínica. Puede ser una
infección asintomática, provocar molestias abdominales inespecíficas; diarreas, colitis y prolapso rectal en niños. En los casos de
hiperinfestación pueden provocar disentería. Trichinella spiralis (nematodo): La triquinosis es una enfermedad que se produce por la
ingestión de quistes de triquina en carne contaminada. A partir de ella se liberan larvas que penetran en la mucosa intestinal, y se
desarrollan hasta el estadio adulto (30-40h). A los 5 dias comienza la fase larviposicion, donde las larvas son liberadas en la mucosa y
atreves de los linfáticos acceden a la circulación general.

CASO CLINICO
Niño de 11 meses de edad con antecedentes de 1 día de movimientos lentos. Evacuaciones sin sangre, asociados con 10-15 episodios
de vómito y diarrea en 4 ocaciones. Se observa letárgico y con disminución de uresis. Fue hidratado y se le dio vía ceftriaxona y
metronidazol. El niño tuvo un episodio de vómitos donde encontro un gusano. En el examen, parecía aburrido, con deshidratación
moderada. Tenía un pulso de 137, una frecuencia respiratoria de 28 por minuto, y una temperatura de 37 ° C. Su estatura era superior
al percentil 95, su peso estaba en percentil 5.

PREGUNTA
Cual es el estado nutricional.

RESPUESTA
a.- Normal
b.- Desnitrición leve
c.- Desnutrición moderada
d.- Desnutrición severa

PREGUNTA
El paciente mejora su estado hidrioelectrolitico con SRO, cuál es manejo farmacologico.

RESPUESTA
a.- Albendazol.
b.- Mebendazol.
c.- Nitaxozanida.
d.- Metronidazol

PREGUNTA
Considerando el cuadro clínico cual de los siguientes helmitos intestinales es mas probable.

RESPUESTA
a.- Enterobius vermiculares.
b.- Ascaris lumbricoides.
c.- Trichuris trichiura.
d.- Ancylostoma duodenale.

PREGUNTA

CURSO ENARM CMN SIGLO XXI TEL: 36246001 Pharmed Solutions Institute PÁGINA 454
MANUAL DE TRABAJO DEL CURSO ENARM CMN SIGLO XXI
Considerando la presencia de parasitos en tubo digestivo alto se incrementa el síndrome de loeffler, cual no es una manifestación que
lo orienta.

RESPUESTA
a.- Prurito nasal.
b.- Parasitos en heces negativo.
c.- Rx de torax con infiltrados intersticiales.
d.- Rectificación nasal.

CASO CLINICO
Una niña de 4 años de edad con dolor abdominal y distensión desde hace 6 días, según su madre. Ella también tenía antecedentes de
vómitos desde 3 días. El dolor de inicio gradual, cólicos, seguido por distensión abdominal y aumento en la intensidad del dolor y los
vómitos. El vómito era de color verdoso y contenía partículas de alimentos no digeridos. El niño fue vacunado parcialmente. En la
exploración física estaba mal alimentado con palidez, temperatura oral fue de 37,7 ° C, la presión arterial era de 98/60 mmHg, pulso
regular, con una frecuencia de 110 latidos por minuto, frecuencia respiratoria de 34 por minuto. El examen abdominal reveló un
abdomen distendido de todos los cuadrantes. En la auscultación, ruidos intestinales hiperdinámico eran audibles. Examen rectal reveló
heces con sangre y moco asi como varios gusanos vivos.

PREGUNTA
Considerando las manifestaciones, cual es el agente etiológico más probable asociado.

RESPUESTA
a.- Entamoeba histolytica.
b.- Giardia Intestinalis.
c.- Enterobius vermicularis.
d.- Dientamoeba fragilis.

PREGUNTA
Considerando el agente etiológico que considero mas probable en el caso anterior, cual es la conducta terapéutica a seguir.

RESPUESTA.
a.- Iodoquinol 30-40 mg/kg/día vo (máx 2 g) en 3 dosis x 20 día, albendazol 15 mg/kg/día vo x 5-7 días (max 400 mg).
b.- Metronidazol 5 mg/kg vo tid (máx 750 mg/día) x 5 días, nitazoxanida 200 mg x 3 días.
c.- Tetraciclina 40 mg/kg/día (máx 2 g) vo en 4 dosis x 10 días, metronidazol 20-40 mg/kg/día vo en 3 dosis x 10 días.
d.- Albendazol 15 mg/kg/día vo x 5-7 días (max 400 mg), metronidazol 5 mg/kg vo tid (máx 750 mg/día) x 5 días.

CASO CLINICO
A 16 meses de edad de sexo masculino presentó a la sala de urgencias después de tener dificultad para respirar y cianosis peribucal en
casa. Su enfermedad comenzó tres semanas antes, con diarrea sin fiebre, varios miembros de la familia también estaban enfermos.
Aunque la diarrea del paciente paro, continuó hipoactivo y prefiere gatear a caminar. Cuatro días antes, desarrolló fiebre y en el día de
la admisión sus padres notaron repentina dificultad para respirar y color azulado alrededor de los labios. Su temperatura era de 40,5 °
C, frecuencia cardíaca de 180 latidos / minuto, la respiración 62/minuto.

PREGUNTA
Cual es su conducta más importante a seguir.

RESPUESTA
a.- Iniciar amoxicilina más albendazol y paracetamol.
b.- Manejo de la fiebre por medios físicos, observación y signos de alarma.
c.- Envio a segundo nivel, previo manejo de temperatura.
d.- Manejo de fiebre por medios físicos más paracetamol, amoxicilina, albendazol y laboratorios.

PREGUNTA
El paciente presenta los siguientes datos 6 horas después: saturación de oxígeno del 96%, disminución del murmullo vesicular en la
base del pulmón izquierdo. Células blancas de 12,000, con neutrófilos segmentados 58%, 13%, linfocitos, monocitos 15% 6%, 2% y
eosinófilos. El hematocrito fue del 37,5%, plaquetas 48,000, velocidad de sedimentación globular de 33 mm / hora y la proteína C
reactiva 57,4 mg / dL.

PREGUNTA
Considerando los resultados de laboratorios y gabinete cual de las siguientes patologías es más probable.

RESPUESTA
a.- Parasitosis y neumonía en paciente con factores de riesgo.
b.- Neumonia adquirida en la comunidad.
c.- Neumonia sin relación a la parasitosis previa.
d.- Procesos independientes en paciente con inmunocompromiso.

CURSO ENARM CMN SIGLO XXI TEL: 36246001 Pharmed Solutions Institute PÁGINA 455
MANUAL DE TRABAJO DEL CURSO ENARM CMN SIGLO XXI

CASO CLINICO
Niña de 9 años de edad que presenta edema facial de predominio matutino de 3 meses de evolución, sin lesiones de urticaria ni otra
sintomatología acompañante. En la anamnesis realizada no se encontró relación con alimentos, fármacos ni otros posibles agentes
desencadenantes, ni tampoco antecedentes personales ni familiares de interés alergológico. En la exploración física realizada destacaba
un ligero sobrepeso y un edema en surcos nasogenianos.

PREGUNTA
Cual es la conducta a seguir mas adecuada.

RESPUESTA
a.- Iniciar antihistamínico, previa administración de corticoide oral.
b.- Solicita BH, QS, EGO.
c.- Solicita pruebas cutaneas, anticuerpos y perfil tiroideo.
d.- Envia paciente a tercer nivel.

PREGUNTA
El paciente presento una leve mejoría, sus estudios presentaron: análisis de orina y de sangre (hemograma, bioquímica general,
hormonas tiroideas, estudio de complemento, IgA, IgG e IgM) fueron normales. Los anticuerpos antinucleares y antitiroideos fueron
negativos. La cifra de IgE total fue de 9 U/ml. Las pruebas cutáneas para hipersensibilidad inmediata con una batería estándar de
neumoalérgenos habituales de la zona, fueron negativas.

PREGUNTA
Cual es la conducta a seguir.

RESPUESTA
a.- Coproparacitoscopico.
b.- Indica antihistaminico.
c.- Indica metronidazol
d.- Indica albendazol.

DESHIDRATACION, LIQUIDOS Y ELECTROLITOS. CIENCIAS BASICAS: Para fines de terminología médica, deshidratación se define como
el estado clínico consecutivo a la perdida de líquidos y solutos en el cuerpo humano. Sin embargo, es posible encontrar depleción
corporal de agua sin pérdida de solutos, de causas diversas, sin denominarse deshidratación. SALUD PUBLICA: La primera causa de
deshidratación en el mundo es la diarrea aguda con mil millones de episodios anuales y más de 2.5 millones de muertes secundarias a
deshidratación. En México la tasa de mortalidad en 2005 fue de 21.6 por 100 000 habitantes en menores de 5 años. PATOGENIA: las
principales causas de deshidratación, están determinadas por dos mecanismos: 1) Incremento en las pérdidas (Intestinales; vómitos,
diarrea, sondas, fistulas intestinales. Extraintestinales; quemaduras, uso de diuréticos, diuresis, osmótica, poliuria, fiebre). 2) Falta de
aporte (por vía oral, por vías parenterales). Otras causas de deshidratación son cetoacidosis diabética, diabetes insípida, estrés
postquirúrgico y privación de agua. La distribución de líquido en el cuerpo está determinada por la edad. En el recién nacido el líquido
corporal total es de 70-75%, pero va disminuyendo conforme avanza la edad hasta ser de 60% en el adulto. El líquido corporal total está
distribuido a su vez en los espacios intracelular y extracelular; este último está conformado por el espacio intersticial y el espacio
intravascular. La pérdida de líquidos produce diferente déficit en los compartimentos de los espacios extracelular e intracelular. En la
deshidratación aguda (menos de 2 días), la perdida de líquidos en su mayoría es a expensas del líquido extracelular (75%); mientras que
en la deshidratación prolongada, la perdida de líquidos es aproximadamente la misma en ambos espacios. La osmolaridad plasmática
está dada por los diferentes solutos, de acuerdo con la siguiente fórmula: Osmolaridad = 2(Na+ mEq/L) + (NUS en mg/dl)/2.8 + (glucosa
en mg/dL)/18. La osmolaridad del plasma se modificara dependiendo de la causa de la deshidratación y del mecanismo de ésta; por
ejemplo, en pacientes con deshidratación por diarrea aguda, puede haber mayor pérdida de líquido que de solutos (gastroenteritis por
rotavirus) o mayor pérdida de solutos que de líquido (cólera); con base en lo anterior se produce deshidratación isosmolar, hiposmolar
o hiperosmolar. DIAGNOSTICO: El diagnostico de deshidratación se establece mediante un minucioso interrogatorio y examen físico.
Sin embargo, el médico o personal de
salud de primer contacto debe estar
entrenado para detectar datos clave.
Durante la valoración de un paciente
deshidratado se debe poner especial
énfasis en tres aspectos: 1) Establecer
el grado o severidad de la
deshidratación. 2) Determinar el tipo
de deshidratación, así como sus
complicaciones concomitantes. 3)
Planear la forma de rehidratación. Una
revisión sistematizada demostró que
los tres datos clínicos más importantes
para identificar deshidratación son:
llenado capilar prolongado, turgencia
de la piel y patrón respiratorio.

CURSO ENARM CMN SIGLO XXI TEL: 36246001 Pharmed Solutions Institute PÁGINA 456
MANUAL DE TRABAJO DEL CURSO ENARM CMN SIGLO XXI
Determinación del tipo de deshidratación. El tipo de deshidratación está dado principalmente por la cuantificación sérica de Na+: 1)
Deshidratación isotónica (isonatrémica), con Na+ sérico entre 130 y 150 mEq/L. 2) Deshidratación hipertónica (hipernatrémica), con Na+
sérico mayor a 150 mEq/L. 3) Deshidratación hipotónica (hiponatrémica), con Na+ sérico menor a 130 mEq/L. El tipo de deshidratación
dependerá entonces de la pérdida que
predomine: si la depleción de líquido es
menor que la de solutos, se presentará
una deshidratación hipertónica; y si es
mayor la pérdida de solutos, será
hipotónica. En términos de frecuencia,
la isotónica representa 80% del total de
las deshidrataciones, la hipotónica 15%
y la hipertónica 5%. Las manifestaciones
de la hiponatremia se correlaciona con
la velocidad de descenso del Na sérico:
si el decremento sucede en un tiempo
menor a 12h, se denomina
hiponatremia aguda, la cual mostrara
datos clínicos más tempranamente que
la desarrollada en un tiempo mayor. Las
manifestaciones graves son las del SNC:
irritabilidad, vómitos, nausea,
fasciculaciones, cefalea, e incluso crisis
convulsivas o coma con cifras menores a 120 mEq/L. Otros elementos a considerar en sujetos con deshidratación son el estado acido-
base y la hipokalemia. Estos pacientes pueden pre4sentar acidosis metabólica (pH <7.35 con HCO3 <29mEq/l), debido al incremento en
las perdidas de bicarbonato; la disminución en la capacidad del riñón para eliminar hidrogeniones y la ganancia de ácido láctico por
hipoperfusion que se presenta en la deshidratación moderada y severa. Las manifestaciones clínicas de la acidosis metabólica son
polipnea (como mecanismo de compensación), depresión miocárdica, arritmias, dilatación arteriolar, entre otras. La hipokalemia
(K<3mEq/l), es producida por 2 mecanismos, las perdidas incrementadas a través de la heces y la excreción renal aumentada para
mantener la homeostasis con Na. Las manifestaciones clínicas pueden ir de un paciente totalmente asintomático, hasta sujetos con
íleo, rabdomiolisis o arritmias. Laboratorio: El examen de orina muestra densidad específica, generalmente mayor a 1.020, debido al
mecanismo homeostático de resorción de líquidos. En caso de registrar densidad específica menor a 1.020, se deberá descartar alguna
nefropatía intrínseca, por la incapacidad de realizar esos mecanismos. La determinación de electrolitos séricos (Na+, K+, Cl-), tiene
especial importancia para determinar el tipo de deshidratación hipo o hipernatrémica. El potasio sérico puede modificarse a su vez por
estados de alcalosis o acidosis; si el pH desciende 0.1 unidades por debajo de 7.35, el K+ sérico incrementa 0.5 mEq/L y viceversa, siel
pH aumenta 0.1 unidades, por arriba de 7.45,el K+ sérico desciende 0.5 mEq/L. La determinación sérica de cloro ayuda generalmente a
calcular la brecha aniónica (anion gap), la cualpermite descartar otras causas de acidemia, como acidemias orgánicas, o cetoacidosis
diabética. Se calcula mediante la siguiente fórmula: Na+ - (Cl- + HCO3-). Tiene como valor normal entre 8 y 12 mEq/L. TRATAMIENTO:
Como ya se mencionó, la diarrea aguda es la principal causa de deshidratación en nuestro país. Par esta causa la OMS específicamente
recomienda un plan a seguir con base en los datos clínicos que presente el niño. El plan A en pacientes sin deshidratación o
deshidratación leve, el plan B en casos de deshidratación moderada y el plan C en enfermos con choque. PLAN A: Continuar con
alimentación habitual, aumentar la ingestión de líquidos de uso cotidiano en el hogar, VSO, en los menores de 1 año, ofrecer media
taza (75ml) y en lo mayores de un año una taza (150ml) y administrarlo a cucharadas o mediante sorbos pequeños después de cada
evacuación. Capacitar a la madre para reconocer los signos de deshidratación y datos de alarma por enfermedades diarreicas (sed
intensa, poca ingestión de líquidos y alimentos, numerosas heces liquidas, fiebre, vómito y sangre en las evacuaciones) con el propósito
de que acuda nuevamente a solicitar atención medica en forma oportuna. Esta formula consta de Na 90mEq/l, K 20mEq/l, HCO3
30mEq/l, Cl 80mEq/l, glucosa 111mEq/l. la OMS actualmente recomienda el uso global de esta fórmula con sodio 75 mEq/L,
osmolaridad 245 mOsm/L y glucosa 75 mEq/L. PLAN B: Pasar pacientes con diarrea y deshidratación con atención en la unidad de salud;
VSO 100ml/kg, en dosis fraccionada cada 30min durante 4h VO. Si el paciente presenta vómito, esperar 10 minutos e intentar otra vez
la hidratación oral más lentamente se puede intentar infusión por sonda nasogástrica a razón de 20-30 mL/kg/h. Al mejorar el estado
de hidratación, pasar a plan A. En caso contrario repetir el plan B en 4h, de no existir mejoría pasar al plan C. PLAN C: Iniciar
inmediatamente administración de líquidos por vía IV, con solución de Hartmann, si no se encuentra disponible, use sol. Salina al 0.9%,
de acuerdo con el siguiente esquema: Primera hora 50ml/kg, segunda hora 25ml/kg, tercera hora 25ml/kg. Evaluar al paciente
continuamente. Si no mejora aumentar la velocidad de infusión. Cuando pueda beber (usualmente 2-3h), administrar sobres de VSO, a
dosis de 25ml/kg/h, mientras sigue líquidos IV. Al completar la dosis IV, evaluar al paciente para seleccionar plan A o B, y retirar
venoclisis, o repetir plan C. Si selecciona el plan A, observar durante h para asegurarse de que el responsable del paciente pueda
mantenerlo hidratado con sobres de VSO, además alimentarlo en su domicilio. La OMS recomienda la administración de solución de
Ringer-lactato o sol. Fisiológica en dosis de 20 mL/kg de peso hasta que el pulso, la perfusión y el estado de conciencia regresen a la
normalidad. En sujetos en quienes está contraindicada la vía oral o tienen otras causas de deshidratación se sugiere un plan de
tratamiento que se divide en cinco fases: La primera fase de la rehidratación se debe llevar a cabo en los primeros 30 a 60 minutos
(NaCl 0.9% 20 mL/kg en bolo). Su objetivo es la restitución rápida de la depleción de volumen circulante, en pacientes con choque
hipovolémico. En estos pacientes se inicia la reposición de líquidos con bolos de 20 mL/kg de cristaloides. Para el manejo de choque
hipovolémico en niños y RN, existen las siguientes recomendaciones clínicas. La primera elección para la resucitación inicial es la
solución salina; si se requieren grandes cantidades de líquidos, es posible utilizar coloides sintéticos por su mayor duración en la
circulación y por último el volumen inicial debe de ser de 20 mL/kg; dependiendo de la respuesta clínica, se deberá valorar el número
de dosis necesarias y, en pacientes con problemas cardiacos o recién nacidos, se recomiendan bolos de 10 mL/kg. La segunda fase se
lleva a cabo dentro 1-9 hrs de iniciada la hidratación del paciente. Su finalidad es la restitución parcial del déficit del espacio

CURSO ENARM CMN SIGLO XXI TEL: 36246001 Pharmed Solutions Institute PÁGINA 457
MANUAL DE TRABAJO DEL CURSO ENARM CMN SIGLO XXI
extracelular y del estado ácido–base. El plan terapéutico en esta fase es administrar 1/3 de líquidos de mantenimiento diarios y ½ del
déficit de líquidos. La recrea fase se lleva a cabo a las 9hrs a 24h de haber iniciado la hidratación. Su objetivo es la restitución de
volúmenes del los compartimentos EEC y EIC y del estado acido-base. Administrar 2/3 de los líquidos de mantenimiento diario y 1/3 del
déficit de líquidos. La cuarta fase se lleva a cabo dentro de las 25 a 48 horas de iniciada la hidratación. Su finalidad es la corrección total
de electrolitos, estado ácido–base y volumen de líquidos, dar líquidos de mantenimiento y reponer las pérdidas actuales, así como
iniciar la vía oral si las condiciones lo permiten. La quinta fase se lleva a cabo, durante los 2-14 dias de haber iniciado la hidratación. Su
objetivo es restaurar el déficit calórico y proteico. Esto se logra con una adecuada dieta por vía oral. CORRECCIÓN DE TRASTORNOS
ELECTROLÍTICOS Y ÁCIDO-BASE CONCOMITANTES: En pacientes con hiponatremia (Na <13mEq/l), se debe agregar a las soluciones de
mantenimiento y a las del déficit la cantidad necesaria de sodio para llevarla a niveles séricos normales.41 La corrección no debe ser
más rápida que 1 mEq por hora o 12 mEq/L en 24 horas, debido al riesgo de producir cambios osmolares en el sistema nervioso central.
La corrección se hace mediante la siguiente fórmula (Na+ ideal - Na+ medido) x peso en kg x 0.6= mEq a administrar en 24 horas. A este
resultado se suman los requerimientos normales. En pacientes con hiponatremia sintomática y Na sérico <120mE/l, se debe
incrementar el valor de Na+ por arriba de esta cifra, en un periodo de una hora, con solución salina al 3% de 5 a 10 mL/kg; esto elevará
el Na+ sérico de 4 a 8 mEq/L. En pacientes con hipernatremia (Na sérico >145mEq/l), se debe corregir el déficit de líquidos en un
periodo mayor, llegando hacer hasta 48h, debido a que el descenso rápido de Na también puede causar cambios osmolares en el SNC,
provocando edema cerebral y mielinolisis pontina. La meta es descender el Na sérico un máximo de 15mEq/dia (1mEq/h). La
hipokalemia se debe tratar solo con incrementos del aporte diario en pacientes asintomáticos, aumentando la concentración de potasio
en soluciones a 60-80mEq/l; pero en pacientes que presentan sintomatología, como arritmias, debilidad muscular importante o
dificultad respiratoria atribuible a esta, deben ser estrechamente monitorizados y administrar KCL a razón de 0.1-0,2 mEq/Kg/h, y la
hiopocalemia con alteraciones ritmo cardiaco que pongan en peligro la vida, la infusión de K+ debe ser de 0.3 mEq/kg/h, siempre
mediante monitorización de electrocardiografía estrecha y la administración del potasio IV mediante bomba de infusión. La acidosis
metabólica se trata desde el inicio con la restitución de volumen; sin embargo, la administración parenteral de HCO3, esta indicada en
pacientes con pH menor a 7.10 o HCO3- menor de 10 mEq/L, y se calcula por medio de la siguiente fórmula: HCO3 (mEq/L) = Déficit de
base x peso(kg) x 0.3, debiéndose administrar al paciente de un cuarto a la mitad de la cantidad resultante de HCO3. Un cuarto de la
dosis equivale aroximadamente a 1mEq/kg (peso) de HCO3.

CASO CLINICO
Paciente masculino de 4 años de edad con diagnostico de gastroenteritis el fue tratado de forma ambulatoria, a las 6 horas acude a
urgenciar debido a que continuo con vomito en 8 ocaciones y 5 deposiciones liquidas, a la exploración física se observa ojos hundidos,
llanto sin lagrimas, somnoliento, hiporreactivo.

PREGUNTA
Cual de los siguientes signos es esta mas relacionado con el choque descompensado.

RESPUESTA
a.- Taquicardia.
b.- Taquipnea.
c.- Tiempo de llenado capilar prolongado.
d.- Hipotensión.

CURSO ENARM CMN SIGLO XXI TEL: 36246001 Pharmed Solutions Institute PÁGINA 458
MANUAL DE TRABAJO DEL CURSO ENARM CMN SIGLO XXI
QUEMADURAS. CIENCIAS BASICAS: Son lesiones traumáticas primariamente de la piel pero que pueden afectar a otros tejidos, por
acción de diversos agentes físicos, químicos, biológicos, con alteraciones, de enrojecimiento a destrucción total. Suelen acompañarse
de una variedad de complicaciones que cuando son graves o inesperadas pueden ocasionar el fallecimiento del paciente. SALUD
PUBLICA: Las quemaduras constituyen un problema médico, psicológico, económico y social que involucra al médico, psicólogo,
económico y social que involucra al médico, al paciente, al familiar del paciente y a la sociedad en sí. En México se calcula se atienden
en hospitalización 10 000 pacientes. Más de 66% de los pacientes quemados recibidos en unidades especiales, son niños. Pacientes
tienen el 77% de mortalidad cuando hay combinación de quemadura de vía aérea con quemadura cutánea. PATOGENIA: El mecanismo
de lesión más común es la escaldadura 72.7%, seguida del fuego directo 18.7%, por contacto 10.5%, eléctricas 8%. Otros: por ignición,
por inhalación, quemaduras químicas, por deflagración, por frio, por radiación. La quemadura se debe a la transferencia de energía de
una fuente de calor al organismo; no es homogénea en las regiones corporales afectadas ya que se produce más lesión en el área de
mayor exposición. En la parte central donde la transferencia de energía es máxima, ocurre una necrosis cutánea irreversible, la zona de
coagulación. Alrededor de ella se dispone una zona que se caracteriza por una intensa reacción inflamatoria, la zona de estasis; esta ara
puede evolucionar a la recuperación pero si se presenta disecación o infección puede evolucionar hacia una completa destrucción en
cuyo caso incrementa el área de afección. En la periferia se encuentra la zona de hiperemia en donde el daño celular es mínimo y la
recuperación espontanea es muy rápida. CLASIFICACION: Por su profundidad; 1er GRADO (epidérmica o eritematosa); Involucra
epidermis, la piel luce eritematosa, de color rojo o rosa brillante, seca, es progresivamente pruriginosa y presenta hiperalgesia. Sana
espontáneamente sin dejar cicatriz. Ejemplo: quemadura solar. 2do. GRADO SUPERFICIAL (de espesor parcial o flictenular); Involucra
epidermis y dermis papilar, se manifiesta por un color rojizo, es suave, húmeda y existen flictenas, Cicatriz normalmente en un lapso de
10-14 días. 2do. GRADO PROFUNDA; Involucra epidermis,
dermis pailar y en profundidad variable la dermis reticular, pero
dejan intactas las faneras epidérmicas profundas. Color rojo
moteado o blanquecinas, no existen flictenas y son
hipoalgesicas. La cicatrización ocurre en 21-28 días, siendo
frecuente la cicatrización hipertrófica, requieren injertos. 3er.
GRADO (espesor total); Son de color blanquecino, amarillo
pálido, momificación, marrón o negro (carbonización). El tejido
tiene una apariencia acartonada y suelen observarse vasos
trombosados. Injerto de piel >1 cm. Efectos sistémicos si es extensa, puede haber amputaciones. Curan > 42 días. La cicatrización
ocurre principalmente por contracción y es deformante. Clasificación por su gravedad LEVE; Quemaduras de 2do grado superficial o
profunda o ambas, de menos de 10% de SC. Quemadura de 3er grado de menos de 2 % de SC. Sin afección de áreas especiales.
MODERADAS; 2do grado superficial o profundas o ambas entre 10-20% de SC. De 3er
grado entre 2-10% de SC. Sin afección de áreas especiales. SEVERAS; 2do grado
superficial o profundo >20% de SC. De 3er grado >10%. Afección de áreas especiales
(cara, ojos, cuello, manos, pies, genitales, articulaciones y pliegues). En menores de 1
año. Inhalación, corriente eléctrica. Presencia de quemaduras más fracturas o traumas
graves, enfermedades asociadas. DIAGNOSTICO: Valorar, gravedad, tipo y extensión de
la quemadura esta última con las Tablas del porcentaje de los segmentos corporales
según edad de "LURD y BROWDER" o más conocida como REGLA DE LOS "9". Ésta y otras
formas de medición significa no solo conocerlas, sino práctica en su uso, por lo que una
forma sencilla de saber la extensión de una quemadura es utilizar la regla de la palma de
la mano, la cual representa aprox., el 1% de SC. Datos clínicos que indican quemadura de
vías aéreas, inhalación o ambas; cianosis, expectoración carbonácea, depósitos
bucofaríngeo de carbón, ampollas y cambios inflamatorios en lengua y paladar, cambios
de voz, ronquera, tos persistente, quemadura de cara, cejas y delas vibrisas, historia de
confinamiento cuando ocurrió el fuego, estado de alerta alterado. El choque
hipovolémico puede desarrollarse rápidamente en los pacientes pediátricos. Un niño con
10Kg de peso, con una quemadura de 20% de extensión, tienen perdidas por evaporación cercanas a los 475ml, equivalentes al 60% de
su volumen circulante. CRITERIOS DE HOSPITALIZACION: 1) Extensión de la quemadura en una área mayor del 10% (<5 años con
superficies mayores a un 5%) de la SCT. 2) Quemadura de cara, cuello, área glúteogenital y eventualmente manos en quemaduras
palmo digitales intermedias o profundas. 3) Quemadura eléctrica de alto voltaje o de bajo voltaje. 4) Quemadura circular de
extremidades, tórax o cuello. 5) Quemadura por ácidos o álcalis. 6) Rescate desde un espacio cerrado con ambiente invadido por humo
(Sospecha de Quemadura Respiratoria). 7) Traumatismo mecánico importante asociado. 8) Enfermedad metabólica o sistémica
asociada. 9) Sospecha de maltrato infantil. 10) Marginalidad o ruralidad extrema. 11) Caso social (analfabetismo o escasa escolaridad de
los padres o personas a cargo del niño, recursos económicos escasos, etc.). TRATAMIENTO: Inmediatamente evolución de vías aéreas y
ventilación (edema masivo, obstrucción ventilatoria, lesión por inhalación), estado hemodinámico, cuantificar tamaño y profundidad de
quemadura, agente etiológico, tiempo y lesiones asociadas, recordar mientras más pequeño el paciente es más susceptible. La
evaluación tradicional del déficit neurológico no debe olvidarse. Traslado del paciente. Tratamiento: Pacientes no intubados requieren
terapia respiratoria agresiva con broncodilatadores y humidificadores con mezclas de Heliox y O2, ventilación mecánica, cámara
hiperbárica. Corticoesteroides no reducen el edema de la vía aérea. Elevar la cabeza 30 grados en la resucitación inicial disminuye el
edema de la vía aérea. Reposición hídrica: <2 años: >10% SCQ. >2 años: >15% SCQ. Calcular pérdidas insensibles: (25 + %SCTQ) X (SCT
2 2
m ). SCTm = Peso x 4 + 7/peso + 70. Fórmula de Parkland y Baxter, se utiliza en las primeras 24h, con Hartmann 24 después coloides. 4
ML x KG x % SCT Quemada. La mitad del déficit total en las primeras 8h sol Hartmann. La segunda mitad en las 16h restantes. Calcular
volumen circulante 60cc x kg. Paciente de 8 kg que tiene una quemadura de 25% de su cuerpo y se decide utilizar Parkland modificado
para restituir líquidos. 4ml x kg x % SCQ, 4 x8 x 25 = 800 ml, pasar 400ml para 8h y 400ml para 16h. Formula de Galveston: Primer día:
50% en 8h y 50% en 16h = 500ml de sol. Hartmann/kg/SCQ + 200ml de sol. Glucosada al 5%/kg/SCQ. El segundo día: 3750 ml/kg/ SCQ
Hartmann + 1500 mil/kg/ SCQ glucosada 5% en 24h. Vigilar continuamente y mantener adecuada PVC (8 - 10), mantener TAM >

CURSO ENARM CMN SIGLO XXI TEL: 36246001 Pharmed Solutions Institute PÁGINA 459
MANUAL DE TRABAJO DEL CURSO ENARM CMN SIGLO XXI
60mmHg, gasto urinario 0.5 - 1.0ml/kg/h, evitar administración excesiva de líquidos, uresis (2ml/kg/h), vigilar hipotermia. Manejo de
dolor con narcóticos, analgésicos, sedantes: Morfina 10 mg IV c/3-4 h. Nalbufina 10 mg IV c /3-4 h. NO administrar antibióticos
empíricos. Cuidado de quemaduras: Sulfadiacina de plata (Silvadene), cubrirlas con sábanas estériles, curaciones quirúrgicas (qx
plástica). La infección en pacientes quemados es la mayor causa de morbilidad, en algunos centros especializados, la sepsis es causante
de 50-60% de las muertes. Se sabe que durante las primeras 24h posteriores a la quemadura predominan gérmenes gramm+ (S.
aureus). Después del final de la primera semana se inicia colonización por gramm- (P. aeruginosa. La infección por anaerobios se ha a
atribuido principalmente a Bacteroides. Se manejara antibiótico acorde a germen implicado. El diagnóstico definitivo de infección de la
lesión se basa fundamentalmente en el estudio histopatológico por medio del cultivo biopsia.

CASO CLINICO
Ingresa masculino de 12 años de edad el cual fue traido por ambulancia ya que fue sobreviviente de incendio en su casa, refieren los
paramédicos que fue encontrado con somnolencia, ya que al parecer estuvo expuesto al humo por tiempo desconocido, a la
exploración física se encuentra conciente alerta, orientado levemente ansioso, observa quemadura leves y superficiales de cara y
manos, se observa abundante pelo quemado, presenta tos seca moderada.

PREGUTA
Cual es la conducta a seguir.

RESPUESTA
a.- Ingresar a vigilancia y colocar oxigeno 3 Lt por minuto.
b.- Enviar a unidad especializada en quemadura.
c.- Realizar tele de torax.
d.- Manejo ambulatorio.

CASO CLINICO
Femenino de 9 años con parálisis cerebral infantil quien ingresa por quemaduras de 9% de superficie corporal. Madre de 30 años,
analfabeta. Padre campesino, analfabeta. 7 hermanos sanos. Medio socioeconómico bajo. Producto de Gesta V, sin control prenatal,
parto domiciliario. Retraso grave en el desarrollo psicomotor. Ignora inmunizaciones. Hace 5 días presenta quemadura por escaldadura,
afectando glúteo, pierna y muslo izquierdos. Manejada por médico particular con arnica y al no mejorar açude a esta unidad.
Exploración física: Peso: 15 kg Talla: 104 cm Temp: 37°C Escolar femenino con PCI, bien hidratada, sin compromiso cardiopulmonar ni
abdominal. Lesiones por quemaduras de 2° grado superficiales y profundas que afectan glúteo izquierdo y cara posterior de muslo y
pierna, con tejido de granulación y áreas necróticas. No hay exudado purulento.

PREGUNTA
Cual es la complicación mas probable en este caso?

RESPUESTA
a.- Infección de la herida.
b.- Infección sistémica.
c.- Falla organica multiple.
d.- Coagulación intavascular diseminada

CASO CLINICO
Masculino de 4 años de edad ingresa a urgencias por presencia de quemadura eléctrica en la boca, refiere la madre del paciente que el
niño se encontraba jugando y súbitamente comenzó a llorar, señalando la boca, agrega que solo estaba un cable de luz que parecía
mordido y levemente quemado, a la exploración se observa a paciente alerta, con facies algicas, llanto intenso, sus contantes vitales se
encuentran dentro de parámetros normales, la mucosa oral se encuentra lesión tipo quemadura que incluye comisura labial que
incluye piel peribucal.

PREGUNTA
Cual es su conducta a seguir.

RESPUESTA
a.- Lavar la herida y colocación de gasas vaselinadas.
b.- Debridar lesión e ingreso a observación.
c.- Envio inmediato a tercer nivel.
d.- Manejo conservador, con control del dolor.

CURSO ENARM CMN SIGLO XXI TEL: 36246001 Pharmed Solutions Institute PÁGINA 460
MANUAL DE TRABAJO DEL CURSO ENARM CMN SIGLO XXI
PICADURAS DE ARAÑAS. CIENCIAS BASICAS: El envenenamiento por picadura de araña, es un problema importante de salud pública.
Se han reportado tasas de más de 200,000 accidentes por año debido a 3,000-5,000 por picadura de araña. Los accidentes por
artrópodos venenosos en México como las arañas de los géneros Loxosceles y Latrodectus. En todo el mundo existen cerca de 30,000
especies o subespecies. Las arañas poseen en el
abdomen glándulas en donde se produce una
sustancia proteica con la que forman sus nidos y
redes, es decir las “telas de araña”. PICADURA
POR CAPULINA: Latrodectus mactans. El
lactrodectismo es un problema frecuente, la
araña capulina o viuda negra es frecuente
encontrarla aun en zonas cosmopolitas como el
DF. A pesar de ser un animal tímido, cuando es
molestado muerde una o más veces. A diferencia
del alacrán, la mordedura no duele en el
momento; sin embargo los síntomas se inician
30-60min después y se inician en el sitio de la
lesión, hipopigmentación e inflamación en el
sitio de la picadura y luego se diseminan, es
característico el dolor articular, temblor y
fasciculaciones prácticamente de todos los
músculos, calambres abdominales, diaforesis
profusa, angustia y la fascies lactrodectísmica y
trismos. El veneno desestabiliza los canales iónicos con liberación de catecolaminas en las terminaciones adrenérgicas además
depleción de acetilcolina en nervios motores (despolarización prolongada de los receptores postsinapticos). La ponzoña de esta araña
es una proteasa alfa-latrotoxina, que tiene actividad neurotóxica y una potencia 15 veces mayor a la del veneno de la serpiente
cascabel. Tratamiento: Colocar un torniquete para evitar la diseminación de la toxina, traslado a clínica más cercana. Analgésicos.
Neostigmina o prostigmina 0.5 a 1 mg c/8-12 h por vía parenteral. Por vía oral 15 mg c/8h 2-3 días. Este fármaco inhibe la acción de la
colinesterasa en los tejidos y la sangre, e impide la destrucción de la acetilcolina. Cortisona y corticotropina, que ayudan a controlar los
síntomas Generales. Suero aracmyn faboterapia, el 90% de los pacientes responde a una sola dosis; el 10% restante, responde a 2 ó 3
frascos. Dosis 1 ampolleta IV lenta y los resultados son extraordinarios, los síntomas como dolor, calambres, angustia y miedo, mejoran
en los niños dentro de los primeros 30 min, y en los adultos solo un poco más. Las dosis deben ser individuales y aplicar nuevas dosis
solo previa valoración y persistencia de la sintomatología. PICADURA POR ARAÑA VIOLINISTA: Loxosceles, El loxocelismo es el cuadro
que se presenta cuando una persona es picado por la araña llamada araña violinista o araña del rincón. De color marrón, con una
mancha más plano y tres pares de ojos. 100 especies en todo el mundo. México: L. boneti mide de 9-25 mm, es de color café. Habita
lugares obscuros, poco ventilados, depósitos de madera. Se esconde en cuadros, muebles. Patogenia: El veneno, producido en poca
cantidad, tiene una acción dermonecrotizante, hemolítica, vasculítica y coagulante. Acompañando el cuadro cutáneo puede existir
compromiso sistémico que se manifiesta por hemólisis. Diagnóstico: Loxoscelismo cutáneo, es la forma de presentación más frecuente,
oscila en 84-97% según distintos autores. La picadura suele producir dolor de poca intensidad inicialmente y, muchas veces, el paciente
no sabe precisar cuándo ocurrió, pero también se ha descripto un dolor urente. La lesión inicial se caracteriza por edema, eritema y, a
veces, dolor urente. Evoluciona en 24-36 h a la característica “placa marmórea o livedoide” (áreas intercaladas de palidez y equimosis)
de bordes irregulares, bien definidos, circundada por eritema y edema indurado, dolorosa a la palpación. Más tarde pueden aparecer
vesículas y flictenas en zonas declive. Las lesiones presentan un patrón de “necrosis hemorrágica gravitacional”, posiblemente
relacionada con la acción conjunta de las esfingomielinasas y la hialuronidasa. Como síntomas generales del loxoscelismo cutáneo
pueden presentarse náuseas, vómitos y fiebre. Entre el 5º y 7º día la lesión cutánea se circunscribe con la formación de una escara
negra, que comienza a desprenderse a partir de la segunda semana. Ésta deja una úlcera con fondo de granulación que tarda varios
meses en cicatrizar y puede requerir cirugía reparadora. La forma edematosa es una variante clínica del loxoscelismo por picadura en
una zona de tejidos laxos, habitualmente la cara. Se caracteriza por eritema y edema sin evolucionar a la necrosis. Las lesiones
cutáneas, una vez establecidas, son muy difíciles de tratar, aun con un tratamiento precoz y adecuado, y pocas veces es posible detener
su evolución. Loxoscelismo cutáneo-visceral: Se considera un cuadro más grave que el anterior caracterizado por hemólisis
intravascular y coagulación intravascular diseminada (CID) asociadas a la lesión cutánea anteriormente descrita. Tratamiento:
Inespecífico. Incluye tratamiento de sostén, asepsia de la lesión, profilaxis antitetánica en caso de requerirla y analgésicos. Dapsona: 1-2
mg/kg/día una dosis, evita necrosis. Vigilar: leucopenia, IR, metaheoglobinemia. Aplicar: a) 4 mg de fosfato de dexametasona por vía
subcutánea en el sitio de la picadura. b) 40 a 80 mg de acetato de metil-prednisolona por vía intramuscular. c) Repetir el tratamiento
local con 21-fosfato de dexametasona, cuatro horas después. d) Si continúan apareciendo vesículas, repetir la dosis de metil-
prednisolona a las 24 horas. e) Si ocurre hemólisis intravascular: 50 mg de prednisolona por vía intravenosa, seguidos de 25 mg cada 6
horas, hasta que desaparezca la hemólisis. f) En caso necesario, previa valoración general, 5,000 U de heparina cada 12 horas. g)
Administrar Aracmyn Plus® (Faboterapia). PICADURA DE ALACRAN: Dentro de la clase Arachnida, hay artrópodos que llaman
especialmente la atención del hombre, ya que desde tiempos inmemoriales le han causado lesiones traumáticas, intoxicaciones,
invalidez, enfermedades severas y muerte.1 La picadura de alacrán está considerada como una urgencia médica por la naturaleza que
representa el ataque, que es la inoculación de veneno que provoca la alarma en la persona afectada, así como a sus familiares, más aún
si la agresión fue a un menor. SALUD PUBLICA: México es el país de mayor diversidad de escorpiones en el mundo, con siete familias, 90
géneros y 200 especies. Son siete las especies peligrosas; se encuentran en la zona occidental y central del país, los estados de Nayarit,
Colima, Jalisco, Durango, Michoacán, Puebla, Guanajuato, Oaxaca son los más importantes. El IMSS reporta en su semanario
epidemiológico 60 000 casos por año. PATOGENIA: Los Suffusus, Ticumanus, Limpidus, Noxius, Sculpturatus, Nayaritus, entre otros son
los responsables de las intoxicaciones severas en nuestro medio y las costas del pacifico son las más afectadas. PATOGENIA: El veneno
de los alacranes está compuesto principalmente por polipéptidos, enzimas proteolíticas, proteínas de bajo peso molecular, serotonina y

CURSO ENARM CMN SIGLO XXI TEL: 36246001 Pharmed Solutions Institute PÁGINA 461
MANUAL DE TRABAJO DEL CURSO ENARM CMN SIGLO XXI
aminoácidos de acción neurotóxica. Es inoculado directamente a la víctima por vía subcutánea y se disemina por la vía hematógena; es
eliminado por la orina y por la secreción biliar. CLASIFICACION: Por su estado clínico se distinguen 4 grados de envenenamiento: I. Dolor
local, parestesias en la zona de la picadura. II. Se agrega parestesias en áreas remotas a la zona del piquete. III. Disfunción somática o de
pares craneales. Inquietud, estremecimiento involuntario, espasmos musculares. Midriasis, visión borrosa, diplopía, ceguera temporal,
movimientos incoordinados, sialorrea, dificultad para deglutir, fasciculaciones de la lengua, espasmo laríngeo, habla farfullante. IV.
Disfunción somática musculo esquelética y de pares craneales. DIAGNOSTICO: Signos vitales: taquicardia, taquipnea, hipertensión
arterial, hipertermia. Laboratorio: no hay exámenes específicos. Tratamiento general: Compresas frías en tanto recibe tratamiento
específico. Analgésico, antihistamínicos en caso necesario sintomáticos por la hipertensión y ansiolíticos. Específico: faboterapia con
alacramyn, dosis inicial una ampolleta diluida en 5 ml de solución inyectable aplicación lenta en la vena, los resultados se miden a
través de mejoría de los síntomas antes de aplicar una segunda dosis.

CASO CLINICO
Una lactante de 17 meses de edad, presentó irritabilidad aguda, llanto, dolor en el pie izquierdo. La madre de la paciente le quitó los
zapatos y se encontró una viuda negra muerta en el zapato izquierdo del niño. Se desarrollo rápidamente inflamación del pie izquierdo.
El EF mostro presión arterial 145/103 mm Hg y taquicardia (frecuencia cardíaca: 160-180 latidos por minuto). El examen reveló edema y
eritema en el pie izquierdo y los párpados, y ninguna lesión diana clásica vista en el sitio. Se observaba con dolor significativo. La
glucosa sérica fue elevada a 186 mg / dl, y su recuento de glóbulos blancos se aumentó a 17 000/mm3.

PREGUNTA
Cual de las siguientes especies es la más probable que se encuentre involucrada en el caso.

RESPUESTA
a.- Loxosceles.
b.- Latrodectus.
c.- Phoneutria.
d.- Tityus serrulatus.

PREGUNTA
Cual es el pronóstico de la aracnismos presuntivo del caso?

RESPUESTA
a.- Dolor agudo, opresión precordial, taquicardia, dificultad respiratoria y “sensación de muerte inminente”.
b.- Alteraciones electrocardiográficas, hipertensión y edema de pulmón.
c.- Sensación punzante, dolor urente o quemante y edema local.
d.- Pápula, que puede evolucionar a una placa eritematosa o placa marmórea.

PREGUNTA
El paciente fue tratado con diazepam, morfina y difenhidramina. Se traslado a la unidad de cuidados intensivos pediátricos ~ 5 horas
después de la mordedura de la araña, se quedó irritado y con dolor, con un firme, pero no rígido, abdomen y notable eritema del pie
izquierdo. Cuales medidas son más utiles previo al traslado a una unidad de segundo nivel.

RESPUESTA
a.- Retirar todos los elementos que puedan comprimir y agravar el edema en la zona afectada.
b.- En las lesiones localizadas en los miembros mantener posición de Trendelemburg.
c.- Realizar antisepsia local, evaluar la necesidad de profilaxis antitetánica y utilizar analgésicos.
d.- Uso de corticoides y antibióticos.

PREGUNTA
Cual de las siguientes patologías es el más probable para el diagnoticos diferencial?

RESPUESTA
a.- Picadura de escorpión.
b.- Picadura de Latrodectus.
c.- Picadura de himenóptero.
d.- Picadura de miriápodo.

CASO CLINICO
Un niño de 7 años de edad se presentó con fiebre, convulsiones y dificultad respiratoria de inicio súbito sin causa aparente. Presentaba
GCS 12, disnea, taquipnea y taquicardia. Había aleteo nasal y estertores crepitantes bilateralmente a la auscultación.

PREGUNTA
Cual es la conducta a seguir.

RESPUESTA
a.- Vigilar por dos a cuatro horas en ayuno estricto y con monitoreo de signos vitales cada 20 minutos.
b.- Iniciar tratamiento faboterápico polivalente inmediatamente.

CURSO ENARM CMN SIGLO XXI TEL: 36246001 Pharmed Solutions Institute PÁGINA 462
MANUAL DE TRABAJO DEL CURSO ENARM CMN SIGLO XXI
c.- Si no mejora referir al paciente independientemente de la edad.
d.- Administrar analagesico IV, antibiótico y gluconato de calcio.

PREGUNTA
El paciente evoluciono desfavorablemente y fue canalizado a segundo nivele donde se reportan los siguiente laboratorios 35.000
leucocitos / mm3 con predominio de leucocitos polimorfos nucleares, hemoglobina: 9 g / L y plaquetas: 150.000 / mm3. pH 7,23, pO2
40 mmHg, pCO2 40 mmHg, BE-13, HCO3 15 mmol / L, y un nivel de lactato de 10 mmol / L. Sus enzimas cardíacas fueron anormales.
CPKMB fue de 103 ng / ml y cTnI fue de 6 ng / mL. Un ECG del paciente mostró elevación del ST. Rayos X infiltrados bilaterales
sugestivos de edema pulmonar, considerando la evidencias cual es la conducta a seguir.

RESPUESTA
a.- Un frasco de faboterápico antialacrán IV.
b.- Dos frascos, valorar repetir la dosis a los 40 minutos, de acuerdo con la evolución.
c.- Tres frascos, valorar repetir la dosis, de acuerdo con la evolución.
d.- Cuatro frascos, valorar repetir la dosis a los 40 minutos de acuerdo a la evolución.

MORDEDURAS. CIENCIAS BASICAS: MORDEDURA POR SERPIENTES VENENOSAS: Se denomina accidente ofídico al cuadro tóxico
desencadenado por la inoculación de veneno a través de la mordedura de serpientes que poseen esta propiedad. Las serpientes
venenosas, poseen una cabeza triangular con foseta real (órgano termosensible que detecta radiaciones de calor producidas por sus
presas) ubicado en el hocico entre la nariz y ambos ojos, y pupilas que se verticalizan con la luz. Cuerpo cubierto por plaquetas
escamadas que al tacto le imprimen rugosidad, con dibujos de distintas configuraciones y de colores poco llamativos. La distribución de
los sitios anatómicos de las mordeduras de serpientes fue la siguiente: 72% en pies y tobillos, 14% en muslos, 13% en manos, 1% en
cabeza. SALUD PUBLICA: Aproximadamente se considera 15% de las 3 mil especies de serpientes se consideran peligrosas en el mundo.
En México del total de las serpientes, el 81.36% son inofensivas y el 18.64% son venenosas. En México hay dos especies consideradas
como venenosas: Viperidae: accidentes ofídicos 90%. Elapidae. El grupo etáreo con mayor riesgo de afección es entre los 15-44 años de
edad (48.75%). Las especies más frecuentemente asociadas a mordedura son: Cascabel (44.9%), Nauyaca (42.8%), Corales (4%), otras
especies (3.6%), se desconoce (5.1%). PATOGENIA: Mecanismos de acción de los venenos, las toxinas de los principales grupos de
serpientes se clasifican: Hemotoxinas (tipificados por la serpientes cascabel sudafribana (Crotalus terrificus), neurotoxinas (como los de
la serpiente de coral (Elapidae - Micrurus), miotoxinas, nefrotoxinas, necrotoxinas. Locales; destrucción intensa de tejido, dolor intenso
eritema, edema que comprometen la extremidad afectada. DIAGNOSTICO: Los signos y síntomas locales: Huellas de la mordedura,
dolor, edema, incapacidad funcional, equimosis, sangrado, vesículas, necrosis local y parestesias regionales. Síntomas sistémicos:
náuseas, vómito, taquicardia, dolor abdominal y torácico, somnolencia, pérdida de la conciencia, hematuria, hipotensión y estado de
choque. Hematológicos: lisis del eritrocito, Hb, Hto, plaquetas, TP, TPT, tiempo de sangrado, hematuria, melena, hematemesis,
epistaxis, hemoptisis. Renales: glomerulonefritis, endarteriris proliferativa progresiva, necrosis cortical, insuficiencia renal aguda.
Neuromuscular: hormigueo, salivación, parestesias, debilidad, reflejos anormales, depresión del SNC (disnea, parálisis bulbar (4-7h),
parálisis difusa (1-2h). Los signos y síntomas evolucionan
rápidamente y pueden ocasionar paro respiratorio.
Pulmonar, edema pulmonar. El diagnóstico definitivo de la
intoxicación por veneno de serpiente requiere la
identificación de las características morfológicas de la
serpiente venenosa y la correlación de las manifestaciones
clínicas de envenenamiento. Los signos de
envenenamiento pueden variar dependiendo la cantidad
de veneno inoculado, tiempo de evolución, región afectada
y número de mordidas. El 20% de las mordidas se reportan
como frías o secas, es decir, no existe envenenamiento. Es
recomendable siempre tomar cultivos de la lesión por el
riesgo de infecciones agregadas, incluyendo un cultivo para
clostridium. Además se deben monitorizar: BH, cuenta de
reticulocitos, EGO, Na, K, Cl, Ca, urea, Cr, gasometría
arterial, Tiempo de sangrado, TP, TPT, fibrinógeno,
plaquetas. CLASIFICACION: Las manifestaciones clínicas y el
grado de envenenamiento por mordedura de serpiente
venenosa se clasifican dependiendo del nivel del edema y
de los siguientes datos agregados, de acuerdo a la
clasificación de Christopher-Roding para la familia Vipiridae y para la familia Micrurus. Ver cuadro 1 y 2. TRATAMIENTO: Traslado
inmediato a clínica más cercana. Todos los niños se internan mínimo 24 h. Agua y jabón, vendaje seco y estéril. No antibióticos
profilácticos. Elevación de la extremidad para disminuir edema e inmovilizarla. Si se considera peligro de provocar reacción alérgica
administrar difenidramina 1 mg/kg. No se recomienda aplicar succión, ya que no ha demostrado remover el veneno. Actualmente, está
contraindicado el uso de torniquetes, ya que complica la circulación linfática del miembro afectado, condicionando una mayor
morbilidad. Si se dispone de bandas de presión se pueden utilizar en el miembro afectado a una presión de 50-70mmHg, inmovilizando
el miembro y aplicando vendaje para disminuir la perfusión del veneno. Evaluar desde el principio el perímetro del miembro afectado.
Esta acción debe registrarse cada 15-20min, para evaluar progresión, ya que hay riesgo de desarrollar síndrome compartamental. Se
debe estabilizar al paciente y administrar analgésico para el dolor. En pacientes mordidos por serpiente Vipiridae se recomienda el uso
de opioides en lugar de AINES, debido al riesgo asociado de desarrollar coagulopatia o trombocitopenia. Tratamiento específico: la
administración de faboterapicos ha demostrado una disminución del tiempo de hospitalización y recuperación. Se recomienda que sea

CURSO ENARM CMN SIGLO XXI TEL: 36246001 Pharmed Solutions Institute PÁGINA 463
MANUAL DE TRABAJO DEL CURSO ENARM CMN SIGLO XXI
de acuerdo al grado de envenenamiento, según la clasificación de Christopher-Rodning, para niños y adultos, y grado de
envenenamiento para la familia Micrurus. Personas hipersensibles al antiveneno que presenten nauseas, vomito, choque anafiláctico
administrar: Adrenalina 1x 1000 aplicado 0.5 ml cada 15 minutos según requiera. Oxígeno, esteroides. Gluconato de calcio: No.
COMPLICACIONES EN NIÑOS MAS FRECUENTES: Necrosis tisular, coagulación intravascular diseminada, síndrome compartamental.
Administración de faboterápico en base a grado de envenenamiento de acuerdo a clasificación de Christopher y Rodning para niños y adultos (vipiridae)
GRADO TRATAMIENTO
0 No requiere faboterápico
I Aplicar 4 frascos, vía IV. Durante la primera hora se puede diluir en 100-250ml de sol. Salina 0.9%. Observar 12-24h. Si aún no se ha controlado. Aplicar 4
frascos diluidos en 100-250ml de sol. Salina 0.9%. Pasar en infusión continua durante 1 hora. Valorar estado de paciente
II Aplicar 5 frascos vía IV, diluidos en 100-250ml de sol. Salina 0.9% durante la primera hora. Observar 12-24h. Si no hay control, aplicar 10 frascos via IV
diluidos en 100-250ml de sol. Salina 0.9% durante 1 h, cada 4-6 hrs, hasta completar tratamiento de 18 h.
III Aplicar 6-8 frascos vía IV diluidos en 100-250ml de sol. Salina 0.9% vía IV, durante la primera hora. Observar 12-24h. Si no hay control, aplicar 6-8 frascos
IV, diluidos en 100-250ml de sol. Salina durante una hora, cada 4-6h, hasta completar tratamiento de 18 h.
IV Aplicar 25 frascos vía IV, diluido en 100-250ml de sol. Salina 0.9% durante la primera hora. Observar 12-24h. Si no hay control, aplicar 6-8 frascos via IV
diluidos en 100-250ml de sol. Salina 0.9% durante una hora, cada 4-6 h, hasta completar tratamiento de 18 h.
No olvidar considerar la cantidad de líquidos administrados junto con los faboterapicos en la evaluación de balance hídrico total.
Administración de faboterápico en base al grado de envenenamiento por mordedura de serpiente Micrurus en niños y adultos
GRADO TRATAMIENTO
LEVE Aplicar 2 frascos vía IV, diluidos en 100mlde sol salina en la primera hora. Si no hay control, continuar con 2 frascos la siguiente hora. Si hay control
mantener en observación de urgencias por 12-24h
MODERADO Aplicar 5 frascos via IV, diluidos en 100ml de sol. Salina durante la primera hora. Si no hay control, continuar con 2 frascos la siguiente hora. Si hay
control observar 12-24h.
SEVERO Apliacr 8 frascos via IV, diluidos en 100ml de sol. Salina durante la primera hora. Si no hay control, continuar con 2 frascos en la siguiente hora. Si hay
control observación en uregencias por 12-24h

CASO CLINICO
Niña de 2 años que acudió a urgencias por mordedura de serpiente en el maléolo tibial derecho, ocurrida hacía 90min. Según la familia,
la serpiente, de color grisáceo, medía aproximadamente 30cm. En el momento del ingreso presentaba edema en el dorso de pie con
extensión hasta el tobillo, calor local y pulso pedio y tibial posterior presentes, con dolor a la palpación y movilización. Se apreciaban
dos pequeñas incisiones puntiformes separadas 1cm entre sí en maléolo tibial.

PREGUNTA
Cual es la conducta a seguir.

RESPUESTA
a.- Vigilar posible síndrome compartimental
b.- Corticoide, analgésico y antibiótico.
c.- Vigilancia, vendaje compresivo y administración de suero antiviperino.
d.- Corticoide, analgésico, gluconato calcio, suero anticrotalico.

MORDEDURAS DE PERRO (RABIA). CIENCIAS BASICAS: La rabia es una zoonosis de los mamíferos causada por el Lyssavirus (virus de la
rabia), y se transmite al hombre principalmente por la saliva de animales infectados, a partir de una mordedura, rasguño o una
lamedura sobre mucosa o piel, con solución de continuidad. SALUD PUBLICA: El virus está distribuido en todo el mundo. Los perros
constituyen el 54% de los trasmisores. Animales silvestres (42%). Murciélago (4%). Es un problema de salud pública por cuanto conlleva
una mortalidad de 100%. La rabia se presenta en dos modalidades, considerándose la rabia "urbana" cuando el reservorio
predominante es el perro, y "silvestre", si los reservorios son mamíferos, especialmente los quirópteros (murciélagos hematófagos).
PATOGENIA: Etiología, el virus pertenece a la familia Rhabdoviridae. Género Lyssavirus. La proteína G es el principal componente
antigénico y específico de grupo. Peplómeros es el otro antígeno viral importante, localizado en la membrana y origina la formación Ab.
La cápside viral está formada por lipoproteínas. Ingresa al organismo por herida o mucosas expuestas a secreciones que contienen virus
vivo. Una primera replicación en el sitio de la herida en las células musculares. Infecta los nervios sensoriales y motores con una
diseminación centripeta hacia nervios periféricos a través de axones y células de Schwann. A una razón de unos 3mm/hr hasta alcanzar
el SNC. Posteriormente la segunda replicación se realiza en el sistema límbico, una nueva replicación en la neocorteza. La diseminación
a través de nervios periféricos hacia otros órganos o tejidos. La respuesta inmunitaria es insuficiente para prevenir la enfermedad, ya
que por su localización hace poco accesible a la acción de los mecanismos de defensa. Periodo de incubación: 95% es inferior al año. (1
-3 meses). DIAGNOSTICO: En el periodo prodrómico que dura 1-2 días (fase melancólica), hay fiebre, cefalea, anorexia, fatiga, náusea,
vómito. Alteraciones psíquicas: ansiedad, depresión, irritabilidad, agitación, nerviosismo, aprensión, temor. En el periodo de excitación
que dura de 1-3 días: hiperactividad, excitación, desorientación, hiperreflexia, alucinaciones, convulsiones, salivación y espasmos
laringeos y faríngeos. En el periodo paralitico que es una etapa breve 1 día: disminución de los fenómenos de excitación, parálisis de
extremidades y pares craneales, alteración del SNA (lagrimeo, salivación), parálisis muscular generalizada.(ascendente). Anatomía
patológica: La lesión patognomónica de la rabia son los corpúsculos de Negri. Son inclusiones citoplasmáticas eosinófilicas presentes en
el asta de Ammon, corteza y tronco encefálico, células de Purkinje. Diagnóstico definitivo: Aislamiento del virus: Saliva, LCR, tejido
(cerebro). Demostración serológica. Demostración del antígeno vírico. TRATAMIENTO: Vacunas antirrábicas: obtenidas por cultivo de
células diploides (HDCV), células VERO o fibroblastos de embrión de pollo (PCEC). Aplicación, por vía intramuscular, de 3 dosis los días 0,
7 y 21 o 28, en la región deltoidea. La primera dosis se cuenta como día 0. Cada dosis es de: en casos de vacuna HDCV, 1 ml, en el caso
de vacuna VERO, 0.5 ml, en el caso de vacuna PCEC, 1 ml. Esquema de vacunación antirrábica en EXPOSICION LEVE: Una dosis de
vacuna, en los días 0, 3, 7, 14 y 28 (30), por vía intramuscular en la región deltoidea en adultos, y en niños pequeños también puede
aplicarse en la cara anterolateral externa del muslo. En exposición grave: inmunoglobulina y la vacuna antirrábica humana, de
preferencia el día cero. La aplicación de inmunoglobulina antirrábica humana, debe hacerse lo más cercano al día de la agresión, de no
ser así no importa el intervalo transcurrido, administrándose en niños y adultos como sigue: Inyectar inmunoglobulina antirrábica

CURSO ENARM CMN SIGLO XXI TEL: 36246001 Pharmed Solutions Institute PÁGINA 464
MANUAL DE TRABAJO DEL CURSO ENARM CMN SIGLO XXI
humana, a razón de 20 U.I/ kg, como dosis única. Infiltrar alrededor de la herida, si ésta lo permite por su localización y extensión la
mitad de la dosis total que requiere el paciente. Aplicar el resto por vía intramuscular. PRONÓSTICO: Es muy grave. Iniciando la
sintomatología su letalidad es el 100%.

CASO CLINICO
Femenino de 15 años de edad que acude a la consulta por haber sufrido hace cuatro horas una mordedura del perro de su hermana.
Entre los antecedentes personales de la paciente destaca que está correctamente vacunada del tétanos (última dosis hace seis años).
En cuanto a los antecedentes del animal, no ha presentado ninguna enfermedad relevante y está bien vacunado. En la anamnesis la
paciente únicamente se queja de dolor en la zona de la mordedura. A la exploración se observan dos heridas poco profundas de unos 3
mm de diámetro en la cara dorsal del antebrazo derecho. El sangrado por las heridas es escaso. No se observan alteraciones en ninguno
de los siguientes datos: sensibilidad y movilidad del antebrazo y de los dedos de la mano; pulsos, coloración y temperatura del
antebrazo.

PREGUNTA
Tras realizar la cura local de la herida, cual es la conducta a seguir mas adecuada?

RESPUESTA
a.- Profilaxis con antibióticos tópicos.
b.- Profilaxis antibiótica sistemica.
c.- Profilaxis antitetánica únicamente.
d.- Profilaxis inmunoglobulina antirrábica.

INTOXICACIONES. CIENCIAS BASICAS: Los agentes que más afectan a los niños son: medicamentos, sustancias de uso en el hogar
(productos de limpieza, quitagrasa y plaguicidas), hongos y plantas (tienen que ver con costumbres de cada población, medicina
tradicional). Los grupos más afectados son los lactantes y adolescentes. Los AINES y la cabamazepina desplazaron al AAS y a los
barbitúricos como agentes comunes de intoxicaciones de niños. PATOGENIA: El mecanismo de acción es accidental y por intento
suicida. Factores predisponentes son: espacios reducidos, abundancia de tóxicos en casa, mal almacenados y al alcance, muy
importante es la ausencia física o emocional de ambos padres y todo esto se traduce como descuido y/o falta de atención para los
niños y adolescentes. TRATAMIENTO: Para el abordaje del niño intoxicado, se toma en cuenta como en cualquier toro padecimiento, el
estado de gravedad (ATLS, ACLS), antes que el toxico responsable y posteriormente se busca evitar mayor absorción
(descontaminación) y al final revertir los efectos del toxico. Primero Estabilizar las constantes vitales: Aire, ventilación, circulación,
deterioro neurológico, eutermia, mantener vigilancia estrecha de posibles alteraciones metabólicas (hipoglucemia, desequilibrio acido-
base, hipoxemia, flujo urinario). Evitar mayor absorción: 1. Vía aérea: retirar al paciente de la zona contaminada y aplicar oxígeno a
100%. 2. Vía cutánea: lavado generoso y cuidadoso, con agua y jabón, el que hace lavado con protección mínimo guantes y cubre bocas.
3. Vía digestiva: la vía de entrada más común. En la ingestión de cáusticos o derivados del petróleo, no usar medidas heroicas, el daño
se establece antes de tener opción de “neutralizar”, la acción del álcali o acido; amen de la reacción exotérmica que se presenta,
incrementa el daño. Situación semejante en el caso de hidrocarburos, sería preferible la neumonitis química que el riesgo de
broncoaspiración con sus consecuencias. Después de 4h es menos probable encontrar el toxico en estómago y entonces el
procedimiento debe cambiar. Vaciamiento gástrico (primeras 4 h de ingestión). Emesis: si se provoca lo ideal sería disponer de jarabe
de ipecacuana en todos los hogares y administra 10ml en lactantes y 15mlen escolares; pero si se tiene jabón de tocador y de acuerdo a
la edad, lo más rápido posible dar 1-2 cucharadas de ralladura del jabón + 1-2 vasos de agua, el vómito se presenta en 10-20 min
(recordar que no debe hacerse en ingestión de cáusticos, ni hidrocarburos). Lavado gástrico: para mayores de 6 meses, recambios de
50-100ml, hasta obtener liquido claro, antídoto local: en ocasiones al inicio o al final agregar 1g/kg/ de carbón activado. Gastrodialisis:
así se denomina, al uso repetido de carbón activado+ catártico (carbón activado 0.5g/kg + Manitol al 20%, 2ml/kg) administrado por
sonda nasogástrica c/6h, está contraindicado en ingestión de cáusticos, hidrocarburos, íleo paralitico, coma, sangrado digestivo.
Catárticos: acelerar el tránsito intestinal es un procedimiento que ocasionalmente se emplea ya que es sustituido por la gastrodiálisis.
Por ultimo revertir los efectos del toxico (antídotos): Azul de metileno (1mg/kg dosis diluir en 50ml sol. Salina 0.9%, pasar en 1 h), Ácido
ascórbico (1g dosis c/8h, diluir en 50-100ml para 1 h), Atropina (0.5mg dosis), Fisostigmina (0.02-0.06mg/kg dosis, diluir en 10ml de sol.
Inyectable, pasar muy lento), Naloxona (10µ/kg/infusión continua), Flumazenil (0.1mg/dosis IV directo, se puede repetir cada 5min),
Pralidozina (30/50 mg/kg diluir en 10ml sol. Glucosada 5%, pasar en 1 h) Atropina+ pralidozina (combinación usada en intoxicación por
organofosforados), N-acetilcisteína (inicial 140mg/kg-sostén 70mg/kg cada 4h por sonda nasogástrica). Quelantes: D-penicilamina
(50mg/kg dia VO repartir en 3 dosis por 10 dias cada 10 dias. INTOXICACION POR AINES: Tienen en común ser inhibidores de la síntesis
de prostaglandinas por su acción sobre la enzima ciclooxigenasa. Se emplean como analgésicos, antiinflamatorios, existe gran
disponibilidad por lo que son comunes en las intoxicaciones. El directamente involucrado es el naproxeno. Comparten en mayor o
menor grado efectos tóxicos y afectan aparatos como digestivo (nausea, vomito, dolor abdominal y sangrado), hematológico (discrasias
sanguíneas, anemias, leucopenia, agranulocitosis), cardiológico (congestión por retención de líquidos), neurológico (debilidad,
confusión, somnolencia, insomnio, visión borrosa, convulsiones), renal (retención hídrica, de sodio, potasio, falla renal), ginecológicas
(disfunción uterina, sangrado). Tratamiento: El manejo inicial antes mencionado. Administrar soluciones: 1500ml/m2 para mantener
hidratación, corregir desequilibrio H-E sol. Salina 0.9% (30ml/kg/dosis). Descontaminación vaciamiento gástrico, para evitar absorción,
dosis repetidas de carbón activado más catártico manitol al 20%. INTOXICACION POR CAUSTICOS: Especialmente lo álcalis como el
hidróxido de Na (sosa caustica, lejía), es el toxico más peligroso que pueda existir en el hogar. Es conocido que el tiempo que tarda en
producir el daño es de 10 seg y que produce necrosis con licuefacción de las proteínas, afectación vascular del tejido quemado que
finalmente ocasiona isquemia con daño celular extenso, así la lesión es en profundidad con riesgo de perforación. Cuando la lesión
involucra todas las paredes del esófago, la cicatrización produce estenosis con disfunción esofágica que en el mejor de los casos la
obstrucción se “resuelve” con trasposición de colon pero nunca volverá a tener función normal. Tratamiento: esteroides, antibiótico,
estudio radiológicos, endoscopia. INTOXICACION POR ORGANOSFOSFORADOS: Grupo de plaguicidas de empleo frecuente en los

CURSO ENARM CMN SIGLO XXI TEL: 36246001 Pharmed Solutions Institute PÁGINA 465
MANUAL DE TRABAJO DEL CURSO ENARM CMN SIGLO XXI
hogares, se encuentran en varios insecticidas de uso corriente. Actúan bloqueando en forma irreversible la enzima acetilcolinesterasa
que es la encargada de catalizar la acetilcolina, romper la molécula en acetato y colina para su reutilización por las terminaciones
nerviosas (axones). La presencia de cantidades importantes de acetilcolina en las terminaciones nerviosas colinérgicas hacen que se
presenten 3 síndromes; síndrome muscarinico (broncorrea, sialorrea, epifora, despeñe rectal y vesical), síndrome nicotínico (temblores,
fasciculaciones, calambres abdominales, debilidad), síndrome neurológico (crisis convulsivas, miosis, arreflexia, como). INTOXICACION
POR PARACETAMOL: El paracetamol o acetaminofén es un derivado del paraaminofenol, tienen efecto antipirético y analgésico, pero
sin apenas efecto aniinflamatorio. La absorción digestiva (oral, rectal), es rápida consiguiéndose efectos terapéuticos y efecto clínico
entre 30minuts y 2 h, después de una dosis terapeutica de 10-15mg/kg cada 4h (dosis máxima 90mg/kg/dia). Normalmente el 90% del
paracetamol es conjugado en el hígado a glocoronidos o sulfatos, que son eliminados en la orina, y cerca de 2% es excretado en la orina
sin cambios. Aproximadamente el 3-8% es metabolizado en hígado por el complejo enzimático P450, por procesos de oxidación, esta
ruta metabólica crea un metabolito reactivo toxico, N-acetil, paranezoquinona imina (NAPQI), el cual es rápidamente ligado al glutatión
y desintoxicado, si este se acumula se adhiere a las membranas celulares de los hepatocitos generando la muerte celular y la
consecuencia necrosis hepática, también puede generar falla renal aguda. La dosis requerida para producir toxicidad varía según la
función de la citocromo P450, se sugiere que con dosis mayor a 150-200mg/kg en niños, es potencialmente toxico agudo. La toxicidad
crónica se presenta si se ingiere más de 4 gr de paracetamol al día, luego de 2-8 días. Tratamiento: Iniciar con ABC, lavado gástrico, se
recomienda administrar en cada irrigación la cantidad de 15ml/kg en los niños. El uso de carbón activado, reduce una media de 52% la
curva de concentración de paracetamol, siempre que se administre dentro de la primera hora post-ingesta. Niños 0.5g/kg de peso
corporal diluidos en 100ml de agua/sulfato de sodio. Menores de 12 años: 250mg/kg de peso corporal disuelto en 200ml de agua
(catártico). El uso de catartatico está indicado en caso de dosis múltiples de carbón activado. Antídoto por excelencia es la NAC (N
acetilcisteina), cuyo efecto reside, sobre todo, en la posibilidad de regeneración del glutatión, su indicación debe ser lo más precoz
posible. Existen varias pautas de tratamiento con NAC. Vía oral: la dosis recomendada es una carga inicial de 140mg/kg seguida de 17
dosis de 70mg/kg cada 4 hrs. Vía intravenosa: dosis inicial de 140mg/kg en una hora. Cuatro horas después se inician 12 dosis de
mantenimiento de 70 mg/kg en una hora cada 4 hrs.

CASO CLINICO
Se trata de una niña de once años que presentaba: boca seca, confusión, habla incoherente, incapacidad para reconocer a los
miembros de la familia, que también se presenta vómito incontrolable, trastornos visuales, auditivos y alucinaciones visuales. El
examen clínico reveló alteración de la conciencia, coma la escala de Glasgow (GCS) se evaluó a 13/15, las pupilas eran iguales y
reactivas, presentó polipnea a 26 ciclos por minuto, pero estaba afebril y hemodinámicamente estable, con ictericia mucocutánea,
reflejos tendinosos eran agudos y difusos. Recuento de sangre de rutina completo, pruebas de función renal y hepática reveló citolisis
hepática.

PREGUNTA
Cual es la conducta a seguir.

RESPUESTA
a.- Realizar lavado gástrico.
b.- Administrar diazepam 5 mg.
c.- Administrar anticolinergico.
d.- Administrar fisostigmina.

CASO CLINICO
Niño de 8 años de edad, en urgencias 2 horas después de la ingestión de una cantidad desconocida de un líquido. Los familiares refieren
que encontraron un frasco abierto, el padre se dedica al campo, el paciente estaba asintomático, y los resultados del examen físico
fueron normales.

PREGUNTA
Cual es la conducta a seguir.

RESPUESTA
a.- Administrar carbón activado.
b.- Mantener en vigilancia.
c.- Enviar a casa y citar al dia siguiente.
d.- Enviar a segundo nivel.

PREGUNTA
El paciente recibió 2 dosis de carbón activado y observación durante la noche. Fue dado de alta con un cita. En casa, el paciente
presentó fatiga generalizada, ingesta oral nula, dolor abdominal, hematemesis, y disminución de la diuresis. Sus signos vitales eran
taquicardia leve, y un examen físico reveló sequedad de las membranas mucosas y el eritema faríngeo moderado que estaban ausentes
durante la evaluación inicial.

PREGUNTA
Cual es las siguientes substancias es la más probable que se encuentre involucrada.

RESPUESTA
a.- Organofosforados.

CURSO ENARM CMN SIGLO XXI TEL: 36246001 Pharmed Solutions Institute PÁGINA 466
MANUAL DE TRABAJO DEL CURSO ENARM CMN SIGLO XXI
b.- Carbamatos.
c.- Organoclorados.
d.- Paraquat.

TRAUMATISMO CRANEOENCEFALICO (TCE). CIENCIAS BASICAS: Lesiones del cráneo y su contenido, provocadas por el contacto
violento de un agente físico contra la cabeza o por el choque de la misma contra una superficie más o menos dura, o por lo
movimientos de aceleración y desaceleración súbitos que en forma directa o que por sus complicaciones, puede llegar a producir la
muerte, o dejar como secuelas grados variables de invalidez. Son frecuentes en pediatría debido a las características psicomotoras del
niño: inquietud, inexperiencia e interés de exploración, la desproporción de la cabeza con el cuerpo y su menor talla en relación con el
adulto son condiciones que lo hacen más vulnerable a accidentes. SALUD PUBLICA: Primera causa de hospitalización en más de 30% de
los casos, implica lesiones de cráneo y su contenido. Son más frecuentes en el sexo masculino por personalidad y características de
conducta. La etiología más frecuente en niños menores de dos años son las caídas (traumatismo leves desde la cama, de una mesa o al
iniciar la deambulación. En niños menores de un año con TCE grave debe sospecharse maltrato. En niños mayores e 2 años las causas
habituales son accidentes de trafico, bicicleta o deporte. PATOGENIA: El niño tienen alta frecuencia de sufrir daño neurológico debido a
que los huesos del cráneo son más delgados y tienen menor protección, hay menor mielinizacion de las fibras nerviosas y el cerebro en
desarrollo en repuesta al trauma ya que puede desencadenar mayor edema e hipertensión intracraneana. La ventaja que el lactante
tiene es que la presencia de fontanelas y la ausencia de cierre de las suturas permite un poco de mayor elasticidad (compliance)
craneal, que pudiera favorecer en algún momento la presencia de hidrocefalia si el incremento del contenido intracraneal es paulatino
y lento. Al tener contacto traumático con la cabeza, se inicia una serie de eventos que van a determinar la extensión de la lesión, así
como su severidad y secuelas. Primarias: son el resultado directo de la energía física transmitida durante el impacto y cuya gravedad
depende del tipo y la cantidad de está traduciéndose como conmoción, contusión o laceración cerebral. Las fuerzas producidas por
aceleración lineal tienden a causar lesión focal, es decir, fracturas, contusión o laceración. La aceleración por rotación establece en el
cerebro fuerzas de arrancamiento que alteran en forma difusa la función traduciéndose como conmoción, lesión axonal y edema.
Secundarias: se originan a partir de los procesos que incrementan los efectos de lesión primaria, son resultado de hipotensión
sistémica, hipercapnia e hipoxia, así como las alteraciones metabólicas producidas por el traumatismo que dan mayor lugar a isquemia
y edema cerebral. La primera es hipoxia que se presenta inmediatamente del golpe y que puede aumentar por la obstrucción de vías
aéreas o por fractura, luxación del cuello o tórax. DIAGNOSTICO: Existen diferentes signos y síntomas de acuerdo al grado de lesión:
cefalea (provocada por cambios en el flujo cerebral), vomito (por hipertensión intracraneana), letargia, somnolencia, delirio, estupor o
coma (por cambios a nivel de flujo cerebral), triada de Cushing (bradicardia, irregularidades en la respiración e hipertensión arterial
sistólica por alteración de perfusión cerebral), alteraciones de la memoria (alteración celular, isquemia e hipoxia), cambios de la
personalidad (alteración a nivel de membrana celular), papiledema (edema cerebral, hipertensión intracraneana), alteraciones de pares
craneales (hipoxia, isquemia compresión en el sitio de fractura III, midriasis del lado afectado), alteración en escala de coma de Glasgow
(edema cerebral, cambios metabólicos que aumentan deterioro neurológico), convulsiones o focalización (por proceso inflamatorio,
efecto de masa (hemorragia, hematoma) o metabólica), anisocoria y alteración pupilar (edema cerebral). Dependiendo de daño
cerebral y área afectada se puede encontrar descerebración o descorticacion, proporcional al grado de daño celular e isquemia que
haya existido. Los movimientos oculares (oculoencefálicos y oculovestibulares), evalúan la región media cerebral en el área del sistema
reticular ascendente y la respuesta motora usando 5 niveles internacionales se usan: normal 5, debilidad moderada 4, debilidad severa
3, trazas de movimiento 2, flacidez 1. La exploración externa de la cabeza nos puede dar datos de fractura importante de la base del
cráneo la anterior: equimosis periorbitaria signo de mapache. Medio: rinorraquia, otorragia o otorraquia. Posterior: Signo de Battle,
equimosis retroauricular. La exploración de pares craneales indican un proceso expansivo intracraneal, asimetría de la respuesta
motora de los miembros, indica distorsión de los pedúnculos cerebrales o bien compresión sobre el tallo cerebral. Este examen se
puede realizar en menos de 5 min y permite clasificar al paciente pediátrico según el grado de trauma. El valor de la radiografía de
cráneo enel traumatismo craneal está siendo cada vez más cuestionado, de forma que no se recomienda su uso en la mayoría de
situaciones de trauma craneal si la TAC está disponible. CLASIFICACION: Trauma leve o GRADO I: Escala de coma de Glasgow 15-13
puntos, es aquel paciente que posterior al trauma no presenta ninguna manifestación clínica, o bien, tiene una perdida transitoria de la
memoria que durara algunos minutos u horas pudiendo estar asociado a pérdida del estado de conciencia en forma también transitoria
(segundos a minutos) y no existe por lo general alteración en la escala de coma de Glasgow, se ha asociado a síndrome pos conmoción
donde el paciente presenta episodios de cefalea, irritabilidad y falta de concentración, en niños más grandes se puede asociar agresión,
ansiedad, baja atención y cambio de conducta, se presenta meses posteriores al trauma y es autolimitado. Se ha descrito una triada en
trauma leve: somnolencia, irritabilidad y vomito; que ocurre en 48-72 h posteriores al trauma se cree que es debido a leve torsión del
tallo cerebral. Trauma moderado o GRADO II: Escala de coma de Glasgow de 12-9puntos, perdida del estado de alerta menor a 5 min,
tendencia a la somnolencia sin déficit neurológico. Trauma severo o GRADO III: Escala de coma de Glasgow 8-3 puntos. Perdida del
estado de alerta mayor a 5 min, incapacidad para obedecer órdenes, confusión mental, lenguaje incoherente e inapropiado, anisocoria
o lenta respuesta pupilar; la respuesta motora puede variar a la localización del dolor o posturas anormales. Muerte encefálica o
GRADO IV. Cabe mencionar que hay ciertas condiciones en un TCE leve que puede transformarlo en moderado o severo por lo que el
examen debe ser cuidadoso y si existe deterioro rápido de la escala de coma de Glasgow manifestando un conjunto de síndrome de
deterioro rostro-caudal. TRATAMIENTO: A la llegada a urgencias de un TCE, se debe realizar una valoración rápida y ordenada de la
situación del paciente (ABCD). Si existe alteración de la conciencia o se presume, por la historia o la exploración, que el TCE puede ser
moderado o grave. Asegurar la permeabilidad de la vía aérea. Administrar oxígeno. Monitorizar al paciente (FC, FR, TA y SAT O2).
Canalizar una vía venosa periférica. Ante la presencia de inestabilidad respiratoria y/o hemodinámica, se procederá con las maniobras
de reanimación cardiopulmonar (RCP). En los casos con alteración de con ciencia importante (Glasgow < 9) será necesario asegurar la
vía aérea mediante la intubación orotraqueal. Objetivos del tratamiento: PIC <20mmHg , PAM (normal para edad), PPC > 50mmHg,
Sat O2 > 95% con PCO2 35‐40mmHg. CRIERIOS DE HOSPITALIZACION DE TCE: Alteración de los signos vitales:
Convulsiones postraumáticas. Funciones mentales alteradas. Perdida de la conciencia prolongada. Déficit de memoria persistente.
Signos neurológicos focales. Fractura craneal deprimida. Fractura craneal basilar. Edema de piel cabelluda amplio. Cefalalgia severa

CURSO ENARM CMN SIGLO XXI TEL: 36246001 Pharmed Solutions Institute PÁGINA 467
MANUAL DE TRABAJO DEL CURSO ENARM CMN SIGLO XXI
persistente, especialmente con rigidez de nuca. Vómitos persistentes. Fiebre inexplicable. Anormalidades neuroradiologicas que
sugieran abuso infantil.

CASO CLINICO
Paciente escolar, masculino, blanco, de 11 años de edad, con antecedentes de salud, que mientras jugaba en su poblado natal, fue
atropellado por un vehículo, recibiendo traumatismos múltiples a predominio craneal. Llegó al servicio de urgencias manejado por el
sistema integrado de urgencias médicas, en estado de coma. Examen físico al ingreso: Coma moderado, pupilas isocoricas y reactivas,
hemiparesia izquierda, herida contusa frontal medial y hacia la izquierda, con salida al exterior de tejido cerebral y líquido
cefalorraquídeo (LCR). Exámenes complementarios al ingreso: Hemoglobina: 130 g/l, tiempo de coagulación: 7 minutos, tiempo de
sangramiento: 2 minutos. Tomografía axial computarizada (TAC) al ingreso: Se aprecia fractura del hueso frontal hacia la izquierda con
fragmentos que penetran el tejido cerebral, focos de contusión hemorrágica diseminados por regiones frontales polares de ambos
hemisferios cerebrales.

PREGUNTA
Cual es el grado de TCE que presenta el caso?

RESPUESTA
a.- Grado I.
b.- Grado II.
c.- Grado III.
d.- Grado IV.

CEFALEAS. CIENCIAS BASICAS: Es el concepto genérico para considerar el dolor de la cabeza y es el síntoma más común del hombre
civilizado, puede ser de la misma intensidad si su origen es benigno o maligno dependiendo de la tolerancia de cada individuo, por
tanto la cefalea representa una incapacidad relativa de la persona para manejar las incertidumbres de la vida, un síntoma que refleja
más una alteración subyacente del pensamiento o de la conducta, que una enfermedad del sistema nervioso y aunque es considerada
frecuentemente una manifestación frecuentemente asociada a otros signos y síntomas que puede originarse frecuentemente asociada
a otros aparatos y sistemas. Las cefaleas más frecuentes son las de origen orgánico. SALUD PUBLICA: La cefalea ocurre entre 26-45% de
los pacientes en alguna etapa de la vida. La migraña tiene mayor frecuencia en escolares con una relación de 2:1 en niños comparado
con niñas. CLASIFICACION: De origen orgánico: relacionada con procesos infecciosos o inflamatorios, que afectan vías aéreas
superiores, cráneo, TCE. De origen neurológico: migraña y cefalea vascular, por neuroinfecciones, hipertensión intracraneana de
etiología neoplásica en menores de 6 años, y para el grupo de escolares y adolescentes, la migraña, cefalea tensional y epilepsia.
Clasificación International Headache Society 2004: 1. Migraña 2. Cefalea tensional 3. Cefalea en racimos y otras cefaleas trigeminales 4.
Otras cefaleas primarias 5. Cefalea atribuible a TCE 6. Cefalea atribuible a problemas vasculares 7. Cefalea asociada a trastorno IC de
origen no vascular 8. Cefalea atribuible al abuso o depravación de sustancias 9.Cefalea atribuible a procesos infecciosos 10. Cefalea
atribuible a trastornos metabólicos 11. Cefalea o dolor facial asociado a alteraciones del cráneo, cuello, ojos, oídos, nariz, senos,
dientes, boca u otras estructuras faciales o craneales 12. Cefalea atribuible a patología psiquiátrica 13. Neuralgias craneales 14. Cefaleas
no clasificables. De acuerdo a evolución: agudas, subagudas y crónicas que por mecanismo directos e indirectos genera modificaciones
en la homeostasis de manera que el síntoma remite una vez resuelta la patología que la condiciono. Clasificación Migraña; Migraña sin
aura 60-85%. Migraña con aura 15-30%. Migraña Basilar. Migraña hemipléjica familiar. Síndromes periódicos infantiles. PATOGENIA: El
dolor de cabeza se origina por estimulación de las estructuras intracraneales y extracraneales sensibles a dolor que son: piel, tej.,
subcutáneo, musculo, arterias y periostio del cráneo. Estructuras del ojo, oído, dientes y cavidad nasal. Senos venosos intracraneales.
Regiones de la duramadre en la base del cráneo, arterias durales interiores de la duramadre. Nervios craneales trigémino,
glosofaríngeo, vago. Los 3 primeros nervios cervicales. La estimulación de una o de varias de estas estructuras sensibles a dolor
producen cefalea a través de uno o más de los siguientes mecanismos: irritación directa por inflamación, compresión, tracción,
desplazamiento, dilatación y distensión de las arterias. Teoría vascular: es la teoría más antigua, producida por la vasodilatación de
vasos cerebrales, que provocaría una activación de fibras nociceptivas de las arterias intracraneales. Se consideraba que la
vasoconstricción posterior era la responsable de los síntomas que ocurren durante el aura. Teoría neural: “depresión propagada” como
posible explicación para el aura. Se trata de la existencia de unas ondas de excitación que se propagaban a los largo de la corteza
cerebral a razón de 3mm por segundo. Teoría neurovascular: surge ya que ninguna de las dos teorías anteriores logra explicar todos los
fenómenos de la crisis de migraña. En esta teoría el desencadenante inicial sería neuronal: hipometabolismo-hipoperfusión del
hemisferio ipsilateral al dolor. Posteriormente, se produciría la liberación de algunos neuropéptidos vasoactivos, como la sustancia P.
Estas sustancias liberadas pueden dar lugar a la inflamación neurógena, induciendo vasodilatación y posterior extravasación de
proteínas plasmáticas. Papel de la proteína C-fos: ésta es una fosfoproteína que regula la expresión de diversos genes neuronales. La
serotonina (5- hidroxitriptamina) se ha considerado como el mediador principal en esta cascada de eventos. Se ha comprobado una
marcada depleción de la concentración de serotonina en plaquetas, lo que sugiere que ésta fuera la causa de la migraña. Las teorías
vascular y neuronal han contribuido al conocimiento de la enfermedad, pero son incompletas. Proponen que la migraña es el resultado
de interacciones entre el cerebro y el sistema circulatorio sobre un terreno genéticamente predispuesto. DIAGNOSTICO: Historia
clínica dirigida, exploración física integral. Hay que recordar que un dolor de cabeza es variable en su intensidad, según la tolerancia al
dolor que tiene cada persona, por eso no es posible conocerlo con precisión y mucho menos cuantificarlo. La diversidad de sus causas
se resume en cuadro anexo. Auras más frecuentes: Escotoma binocular (77%). Distorsiones de los objetos (16%). Escotoma monocular
(7%). Manchas, balones, colores, arco iris, síndrome de Alicia en el país de las maravillas.
TIPOS DE CEFALEA MANIFESTACIONES SINTOMAS ACOMPAÑANTES
MIGRAÑA Unilateral o bilateral, de inicio hemicraneal. Pulsátil, incremento gradual. El ejercicio lo Nauseas, vomito, fotofobia, escotomos
incrementa. Duracion de mas de 60 min, menos de 1 semana. Disminuye en la oscuridad y el
sueño. Predomina en sexo femenino
CEFALEA TENSIONAL Ambos sexos. Bifrontal u occipital pulsatil. Dolor opresivo o pulsátil, duración de dias o Depresión, ansiedad, estrés

CURSO ENARM CMN SIGLO XXI TEL: 36246001 Pharmed Solutions Institute PÁGINA 468
MANUAL DE TRABAJO DEL CURSO ENARM CMN SIGLO XXI
semanas, intermitente, asociado a fatiga y lo desencadena el estres
TUMOR CEREBRAL Unilateral o bilateral, sin predominio de sexo. Pulsatil, la despierta el dolor. Frecuencia una vez Papiledema, vomito, crisis epiléptica, paresias
en la vida, aparición subita
NEUROINFECCIONES Cefalea universal u occipital. Sin predominio de edad o sexo, puede despertarlo, irritabilidad o Ataque al estado general, crisis epiléptica, signos de
fiebre irritación meníngea.
CEFALEA AGUDA: Se trata de un dolor de cabeza único de duración corta en términos generales, considerada menor de 2 h, y aunque
puede ser el inicio del complejo sintomático de múltiples causas como infección de vías respiratorias, neuroinfecciones, ametropías,
gastroinetsinales, neoplasias del SNC, toxicomanías, HTA. CEFALEA RECURRENTE: Ocurre en forma periódica o recurrente con factores
específicos desencadenantes y características particulares, pero sin olvidarse de la patología casi siempre relacionada con la localización
en la cabeza, dentro de las causas: migraña, cefalea tensional, en racimos, postraumática, hipertensión intracraneana crónica, epilepsia,
trastorno del sueño. CEFALEA CRONICA: Generalmente cuando está relacionada con evolución los problemas serios sistémicos bien
originados en el SN, se requiere manejo integral e investigar en forma prioritaria la causa y establecer un tratamiento específico, no
solo sintomático como en los casos anteriores. En el caso de que se torne crónica pero no evolucione puede deberse al uso crónico de
fármaco, que están indicados como desencadenantes de respuesta de rebote, causas: hipertensión intracraneana, neoplasia del SNC,
epilepsia, migraña, cefalea tensional, sinusitis, disfunción de articulación temporopmandibular. TRATAMIENTO: Medidas generales;
basada en A) Controlando, deben evitarse los alimentos o circunstancias que FARMACO INDICACIONES
desencadena el dolor de cabeza. B) Deteniendo, automedicación nunca será AAS Todos los tipos de cefalea
conveniente, debe evitarse la ingestión de fármacos para disminuir un problema Paracetamol (: 10-20 mg/kg)
asociado que prolonga los dolores de cabezas como es la cefalea de rebote. C) Ketorolaco
Ibuprofeno Profiláctico
Previniendo, si se sigue una vida sana, evitando factores desencadenantes de
Naproxen Migrana
dolor de cabeza, no ingerir fármacos no indicados. Algunas medidas generales Ketoprofeno Cefalea tensional
han permitido disminuir la manifestación como: 1. Aplicación de frio en forma Clonixinato de lisina Migraña, profiláctico
de compresas de hielo, paño húmedo, alternado cada 10 min por 30 min. 2. En Aceclofenaco Todas las cefaleas
ocasiones los dolores de cabeza se incrementan con la luz, por lo que es Clordiacepoxido Cefalea postraumática
Tramadol ó neoplasias
recomendable usar lentes oscuros, cerrar las cortinas o persianas. 3. En caso de
Propanolol Migraña
que el dolor de cabeza se acompañe de vómito, para evitar deshidratación, Metoprolol Prfilactico
consumir abundantes líquidos. El tratamiento médico tiene por propósito en Imipramina Migraña
control del síntoma, pero también la causa que lo genera, de manera que Clorimipramina Profilactico
pueden considerarse para el primer grupo a) analgésicos no narcóticos, b) Fluoxetina Cefalea tensional
Cinarizina Migraña
analgésicos narcóticos, c) AINES, d) antidepresivos tricíclicos. Para los casos de
Flunarizina Profiláctico
migraña se divide el tratamiento farmacológico en dos fases: Abortiva; para la Ac. Valproico Migraña
etapa aguda donde la utilidad de los triptanos sea comprobado y en menores de Fenitoina Profilactico
6 años de edad se recomienda el uso de zolmitriptano y solo en mayores de 6 Topiramato Asociasion con epilepsia
años se puede usar sumatriptan, eletriptan. Profilactico: se recomiendan fármacos para espaciar los cuadros agudos como pueden ser
los AINES del tipo ibuprofeno, meloxicam y en su caso bloqueadores de Cox durante periodos máximos de 6-8 semanas, pero aunado a
medidas higienicodieteticas.

CASO CLINICO
Niña de 13 años de edad presenta cefalea intensa tras un proceso vírico inespecífico cuatro semanas antes. Lo describía como una
pesadez continua que la incapacitaba para hacer vida normal, cuya intensidad aumentaba en el transcurso del día y se atenuaba al
colocarse en decúbito. Durante el día los síntomas fluctuaban, y mejoraban tras la administración de analgésicos. Dormía mal y no tenía
ánimos para emprender las actividades cotidianas, de manera que que faltaba al colegio y si intentaba acudir, invariablemente tenía
que ser recogida por su familia a media mañana. Estaba tomando analgésicos (ibuprofeno) hacía cuatro semanas, tres veces al día sin
cambios en la sintomatología.

PREGUNTA
Cual es la conducta a seguir mas adecuada.

RESPUESTA
a.- Cambiar de medicamento a narproxen 250 cada 12 hrs.
b.- Agregar ergotamia y cafeína profiláctica.
c.- Enviar a evaluación neuropediatrica.
d.- Retirar AINES y medidas generales para disminuir estimulos.

EPILEPSIAS. CIENCIAS BASICAS: Desorden del cerebro caracterizado por una predisposición perdurable para genera crisis epilépticas
con las consecuencias neurobiológicas, cognitivas, psicológicas y sociales de esta condición. De acuerdo a la OMS, la epilepsia es la
presentación crónica, recurrente de fenómenos paroxísticos por descargas eléctricas anormales en el cerebro (crisis epilépticas) que
tiene manifestaciones clínicas variadas y causas diversas. Crisis convulsiva; representa un evento de inicio brusco, generalmente
autolimitado, caracterizado por una actividad muscular excesiva, pudiendo ser clónica (pérdida de la conciencia con contracciones
clónicas rítmicas de las cuatro extremidades. Posictal con confusión), tónica (contracción muscular sostenida, duran menos de dos
minutos y tienen un periodo posictal de confusión) o mioclonica (contracciones súbitas y muy breves de grupos musculares, a veces
generalizadas, de tal forma que pueden hacer caer al sujeto. El EEG con polipuntas generalizadas). Las crisis epilépticas; son la
manifestación clínica de una descarga anormal de una población neuronal, generalmente pequeña, localizadas ya sea en la corteza
cerebral o bien en la profundidad del parénquima cerebral, de forma repetida y crónica, con correspondencia electroencefalografica.
SALUD PUBLICA: México su prevalencia oscila entre 1.8 a 2 % lo que representa más de un millón de pacientes. El 76% de los
epilépticos inician su padecimiento antes de la adolescencia; por lo que la Organización Mundial de la salud (OMS) reconoce a la
epilepsia como un problema de salud pública. Las epilepsias afectan aproximadamente 40-50 millones de personas alrededor del

CURSO ENARM CMN SIGLO XXI TEL: 36246001 Pharmed Solutions Institute PÁGINA 469
MANUAL DE TRABAJO DEL CURSO ENARM CMN SIGLO XXI
mundo. (1-2% de la población mundial). Al menos 5 millones la padecen en Latinoamérica y 3 millones no reciben tratamiento.
PATOGENIA: Etiología diversa. Causas; accidente cerebrovascular, TCE, neuro-infección, alteraciones metabólicas, efecto de alcohol o
drogas, deprivación de alcohol, asfixia. Hipótesis para crisis generalizadas; Hipótesis Neuroanatómica: (Gibbs y colab.) Aquí se invoca
una alteración cortical generalizada sumado a un dismetabolismo en los neurotransmisores especialmente los inhibidores, como el
GABA. Hipótesis micromorfologica: perdida neuronal, esclerosis hipocampal y/o cortical sumado a trastornos corticales ocasionados
por alteración de la migración neuronal. Esta migración neuronal anormal afectaría la conducción trasmembrana de los iones Ca+ y Na.
Anomalías funcionales: constituidas por disminución del consumo de glucosa y del flujo circulatorio parenquimatoso; posteriormente
comunicaron disminución del número de receptores benzodiazepinicos. Para las crisis focales: Cicatriz cerebral: las distintas lesiones
ocasionan una cicatriz glial que actuaría como foco irritativo, que inestabiliza eléctricamente las membranas celulares. Este mecanismo
excitador seria mediado por los ácidos glutámico y aspártico. Disminución de los mecanismos inhibidores gabaergicos: debido
fundamentalmente a pérdida de neuronas gabaergicas en las distintas lesiones. TIPOS: Las GENERALIZADAS, representan los síntomas
derivados de la activación de grupos de neuronas extendidas en ambos hemisferios cerebrales; se clasifican como convulsivas:
míoclónicas, tónicas, clónicas y tónico-clónicas y las no convulsivas: Ausencias y atónicas. Las PARCIALES (cuando los síntomas iniciales
representan la activación de un grupo de neuronas circunscritas a parte de un hemisferio cerebral) y pueden tener síntomas: motores o
sensoriales localizados a parte de un hemicuerpo, autonómicos o psíquicos (ilusiones, alucinaciones, pensamiento forzado, etc).
DIAGNOSTICO: El diagnóstico de las crisis epilépticas y de los síndromes epilépticos es de carácter clínico, examen neurológico,
condiciones médicas relacionadas: Retraso del desarrollo, cefalea tensional y/o migraña, trastornos psiquiátricos, trastornos de sueño.
La confirmación de los mismos se efectúa a través de los métodos complementarios de diagnóstico. Electroencefalograma. Es la prueba
de elección para demostrar el carácter epiléptico de un paroxismo y es insustituible para definir muchos síndromes epilépticos. Los
estudios de EEG con privación parcial de sueño, foto estimulación y/u otras activaciones son útiles para discriminar actividad
epileptiforme no visible en el EEG convencional. Neuroimagenes. La TAC y RM son las técnicas de elección para detectar lesiones
estructurales del SNC, siendo la segunda más sensible y específica, especialmente para el estudio de la esclerosis temporal. CRISIS
CONVULSIVAS GENERALIZADAS: Esta forma es la más frecuente y se la conoce y denomina habitualmente como “Crisis de grand mal”,
básicamente se trata de movimientos tónico-clónicos generalizados, de presentación abrupta, con alteración aguda de la conciencia,
que duran pocos minutos y que generalmente cursan con tres fases definidas: Fase tónica: Perdida de conocimiento brusca con caídas e
hipertonía muscular generalizada. Fase clónica: Movimientos alternativos de flexo-extensión, con sacudidas rítmicas a nivel cefálico y
los cuatros miembros simultáneamente Es habitual que se acompañe de mordedura de lengua y labios e incontinencia urinaria. Fase
poscrítica: Recuperación paulatina de la conciencia, con amnesia de lo ocurrido, cefalea, dolores musculares difusos; al cabo de una
hora el paciente se halla habitualmente recuperado. ESTADO EPILEPTICO: El termino se utiliza para describir cualquier tipo de crisis
continuas lo suficientemente prolongadas que pueden producir daño neuronal. La liga Internacional contra la Epilepsia define al estado
Epiléptico como “una crisis que no muestra datos de recuperación de lo que duraría una crisis habitual, o crisis recurrentes sin
recuperación de alerta durante el periodo interictal, o recuperación de la función basal normal del sistema nervioso”. Desde el punto de
vista operativo se acepta una duración mayor de 5 minutos como suficiente para iniciar tratamiento. CRISIS FEBRIL: Se puede
identificar cuando un niño de 6 meses a 6 años de edad tienen como única causa de las crisis febriles un trastorno genético que le hace
susceptible de manifestar crisis convulsivas exclusivamente cuando se tiene hipertermia, mientras que las crisis desencadenadas por
fiebre, el paciente frecuentemente tiene una causa bien establecida de epilepsia, incluso ha tenido crisis sin fiebre y la hipertermia es
solo otro factor precipitante de las crisis. Los criterios para identificar las crisis febriles “simples” incluyen: inicio entre los 3 meses y los
5 años de edad, no tienen antecedente familiar de epilepsia, no tienen patología neurológica definida ni datos de neuroinfeccion, no
haber presentado una crisis previa en estado afebril, tienen una duración menor de 5 min y no presentan déficit neurológico postictal.
Una crisis febril compleja es aquella que el inicio es parcial, su duración es mayor de 15 min, con un periodo posictal prolongado, con
más de 1 crisis en 24h o más de una por episodio febril, y a la exploración física muestra algún déficit neurológico, en menores de 6
meses o en mayores de 5 años. Tratamiento: Manejo inicial ABC, controlar fiebre con medios físicos y/o paracetamol 10-
15mg/kg/dosis, realizar historia clínica, examen físico y exploración neurológica. En caso de crisis febriles complejas agregar Valproato
de Magnesio a dosis crecientes, hasta llegar a 30mg/kg/día. CRISIS DE AUSENCIA: a. Típica: Desconexión del medio por pocos segundos
durante los cuales el paciente se ve con la mirada perdida. Puede presentarse la ausencia simple o acompañarse de fenómenos clónicos
leves (parpadeo), automatismos (deglutir, chuparse los labios), fenómenos atónicos (caída de la cabeza), fenómenos tónicos
(contracción de los músculos del tronco), fenómenos autonómicos (palidez, rubicundez, piloerección) No hay periodo posictal y el
paciente recupera bruscamente el estado de alerta total. El EEG muestra descargas generalizadas de complejos punta onda lenta de 3
ciclos por segundo. b. Atípicas: Desconexión del medio un poco más prolongada que la típica, frecuentemente acompañada de
fenómenos tónicos y recuperación lenta hasta la alerta total. El EEG muestra complejos de punta o polipunta onda lenta de 2 – 2.5
ciclos por segundo. TRATAMIENTO ANTIEPILEPTICO: Crisis primariamente generalizadas convulsivas: Fenitoina (impregnación de
18mg/kg/dosis lentamente y vigilando la función cardiovascular y mantener con dosis de 5-7mg/kg/día, dividido en 2-3 dosis),
Fenobarbital (impregnación 20mg/kg/dosis y mantener en dosis 3-5mg/kg/día dividido en 1-2 dosis) o Valproato de Magnesio en dosis
de 15mg/kg/dosis, seguido de 25-50mg/kg/día. Crisis parciales simples, complejas o secundariamente generalizadas: Se prefiere utilizar
Valproato de Magnesio o Carbamacepina a dosis de 15-20mg/kg/día divido en 2-3 tomas al día. No convulsivas. Ausencias Típicas y
atípicas, primera elección valproato de magnesio (30-60mg/kg/día), Etosuximida. 20-30 mg/kilo/día, fraccionada c / 8 h. Se debe evitar
la administración de benzodiacepinas cuando el evento convulsivo ha terminado, ante la imposibilidad de evaluar el estado mental y
modificar la exploración neurológica. En el caso de una crisis febril o afebril prolongada se podrá administrar diacepam IV a dosis de
0.3mg/kg/dosis (máximo 10 mg total) o a dosis de 0.5mg/kg/dosis y evaluar el tratarlo como estado epiléptico. Síndrome de West +
Esclerosis tuberosa; vigabatrina. Síndrome de Lennox-Gastaut; topiramato, valproato de magnesio. Epilepsia de ausencias infantiles y
juveniles; valproato de magnesio (30-60mg/kg/día), lamotrigina. Epilepsia mioclónica juvenil; valproato de mg. Crisis convulsiva fase
ictal: Medicamentos primera línea: benzodiacepinas Lorazepan, diazepam (60-80%). CRITERIOS DE HOSPITALIZACION: < 1 año,
Glasgow < 15, datos de hipertensión endocraneana, meningismo, crisis dura más de 15 minutos, recurrencia de las crisis convulsivas 12
horas. Lamotrigina: primera línea, atónicas, tónicas, ausencias, tónico clónicas generalizadas, crisis parciales. Segunda línea en
mioclonicas. Topiramato: primera línea, crisis tónico clónicas generalizadas, mioclónicas, parciales con generalización. Segunda línea,
ausencias, tónicas, atónicas. Vigabatrina: crisis parciales con eficacia de 50%

CURSO ENARM CMN SIGLO XXI TEL: 36246001 Pharmed Solutions Institute PÁGINA 470
MANUAL DE TRABAJO DEL CURSO ENARM CMN SIGLO XXI
CASO CLINICO
Varón de 15 años, ingresó tras sufrir un episodio de parestesias en el hemicuerpo izquierdo, junto con desviación de la comisura bucal
hacia la izquierda, seguido de cefalea frontal intensa y pulsátil, náuseas y vómitos. Antecedentes: producto de un embarazo y parto
normales con un desarrollo psicomotor y del lenguaje adecuados, con trastorno del aprendizaje. No presentó antecedentes familiares
de interés excepto que su abuelo (materno) sufría de migrañas. A los 4 años, tras un traumatismo craneoencefálico leve, sufrió un
cuadro de inestabilidad y somnolencia de varias horas de duración. A los 5 años presentó en vigilia, de forma súbita, una hemiparesia
aguda izquierda asociada a disartria y confusión, que cedió en 10 minutos. El examen físico y neurológico completo evidenció, una leve
dismetría del miembro superior izquierdo.

PREGUNTA
Cuál de los siguientes diagnosticos es el más probable.

RESPUESTA
a.- Crisis parciales simples.
b.- Crisis parciales complejas.
c.- Crisis generalizadas convulsivas.
d.- Crisis generalizadas no convulsivas.

PREGUNTA
Cual es el auxiliar diagnostico mas adecuado para idenficar la causa.

RESULTADO
a.- MVEEG.
b.- EEG.
c.- IRM.
d.- TAC.

PREGUNTA
Cual es la conducta terapéutica mas adecuada para el caso.

RESPUESTA
a.- Carbamazepina 10 a 20 mg/kg.
b.- Lamotrigina 200 a 400 mg/día administrados en dos dosis.
c.- Valproato de magnesio 60 mg/kg/día.
d.- Topiramato 100 mg/dia.

ENURESIS. CIENCIAS BASICAS: La enuresis pertenece al grupo de trastornos de la eliminación siendo definido por el DSM-IVTR como el
repetido vaciado de orina en la ropa o en la cama, tanto si el vaciamiento es involuntario como intencionado. Esta conducta debe
ocurrir dos veces por semana durante 3 meses por lo menos o que genere un malestar clínico significativo o altere el funcionamiento
social o académico del niño. La enuresis es un problema evolutivo de la micción. La definición más aceptada por la comunidad científica
establece que la enuresis consiste en la “emisión involuntaria y persistente de orina durante el día o la noche, después de una edad en
la que el niño ya debería haber aprendido a controlar la micción (5 años) y no existen indicios de patología. Se manifiesta si el niño no
ha aprendido a evacuar voluntariamente la orina en los lugares adecuados, es decir, si la micción no se realiza bajo control del
individuo. SALUD PUBLICA: El 15% aproximadamente de la población infantil mayor de 5--6 años tiene enuresis primaria. La incidencia
familiar es alta. Un 44% tienen un padre que fue enurético en la niñez y si ambos padres lo fueron el niño tiene hasta un 77% de
probabilidades de tener enuresis. TIPOS: Enuresis primaria: el niño no ha logrado control suficiente y puede encontrarse en cualquier
estadio del aprendizaje del control voluntario de la micción. Enuresis secundaria: se trata de casos en los que después de un amplio
periodo de control (más de 6 meses) el niño recae, en ocasiones, asociado a un accidente, a periodos de hospitalización largos, a
nacimiento de hermanos o a pérdidas familiares. La enuresis funcional (no organica) es el tipo más común (más del 90%) y es aquella en
la que los niños no presentan ningún otro problema bien sea de origen congénito orgánico, infeccioso o traumático. El niño no ha
alcanzado todavía control sobre la micción pero este problema no puede ser achacable a ninguna otra causa. Su desarrollo general
(motor y verbal) es normal y aparentemente sólo muestra dificultades en el aprendizaje del control de la micción. En los otros casos se
habla de enuresis orgánica bien sea de tipo congénito o traumática. La distinción entre enuresis diurna y nocturna es obvia, sin
embargo, su tratamiento plantea problemas diferentes. PATOGENIA: Decimos que es un problema evolutivo porque todos nacemos
con el reflejo de micción, de forma que cuando la vejiga está llena se relajan los esfínteres y se libera la orina, pero en el curso del
desarrollo aprendemos a controlar el reflejo y a orinar en el lugar apropiado. El momento a partir del cual controlamos la micción es
muy variable y por tanto el diagnóstico de enuresis se establece arbitrariamente a partir de los cinco años. Teorías: Una de ellas es, que
el sueño especialmente profundo de los niños que mojan la cama impide que las contracciones de los músculos de la vejiga despierten
al niño antes de iniciar la micción. Sin embargo, los estudios electroencefalográficos, los escáneres y tomografías cerebrales no han
podido constatar diferencias que pudieran justificar la enuresis de algunos casos aunque algunos resultados de investigación apuntan a
que los enuréticos presentan un umbral para despertar más elevado que los no enuréticos. También asociado al sueño se ha señalado
la posibilidad de que las contracciones vesicales fueran demasiado débiles para provocar el despertar, al mismo tiempo que se ha
propuesto que durante el sueño profundo el nivel de contracción de los esfínteres era deficiente. Se ha sugerido también que los casos
de enuresis presentan una capacidad funcional de vejiga disminuida y, por tanto, no pueden aguantar toda la noche sin evacuar la
orina. La teoría comúnmente mantenida respecto a la enuresis desde una perspectiva psicológica es que ésta es debida a un déficit de
aprendizaje. El control de la micción diurno y, especialmente, nocturno es una habilidad compleja que comporta muchos pasos y

CURSO ENARM CMN SIGLO XXI TEL: 36246001 Pharmed Solutions Institute PÁGINA 471
MANUAL DE TRABAJO DEL CURSO ENARM CMN SIGLO XXI
requiere un correcto entrenamiento, lo que no quiere decir que haya que ocuparse activamente de que el niño aprenda sino que hay
que permitir que se den las condiciones para el aprendizaje. DIAGNOSTICO: Por dentición, el niño enurético debe tener examen físico
normal de vías excretorias, tanto en sus aspectos anatómicos como funcionales. Para hacer el diagnóstico en niños con enuresis es
necesario tener en cuenta historia clínica detallada, examen físico y estudios paraclínicos. Es indispensable que se investigue como fue
el entrenamiento del esfínter vesical y anal. Es necesario evaluar la situación familiar, identificando la funcionalidad, eventos
relevantes, situación escolar, dinámica con hermanos y compañeros de escuela. Estudios básicos: urocultivo, EGO, química sanguínea,
ultrasonido abdominal (vías urinarias). TRATAMIENTO: El primer paso es la educación y apoyo a los padres, puntualizando en la
paciencia. Siempre ha que descartar causas orgánicas, de presentarse pasan a primer término. Las alternativas terapéuticas son varias
de las que se pueden emplear las siguientes: modificación conductual, entrenamiento de la capacidad vesical, distribución de la ingesta
de líquidos (aumentar consumo de líquidos durante el día; 40% en la mañana, 40% por la tarde y 20% al anochecer)y tratamiento
farmacológico. En lo que respecta al tratamiento motivacional, es importante incluir la participación del niño con el propósito de
hacerlo responsable y a su vez darles un reconocimiento de las noches secas (llevar calendario de días “secos” y días que “mojo la
cama”). Farmacológico: imipramina, inhibe la recaptura sináptica de noradrenalina y serotonina. Posee actividad anticolinérgica débil y
acción antiespasmódica directa sobre musculo liso vesical, alterando la inervación sináptica. También se ha demostrado que disminuye
el tiempo de sueño MOR. Dosis para niños de 5-8 años es de 25mg, para niños mayores de 50mg o a dosis de 0.9-1.5mg/kg/día. Se
administra en una sola dosis por la noche cuando la dosis es mayor de 50mgs, debe fraccionarse en dos tomas. Durante tratamiento
evaluar frecuencia cardiaca y presión arterial, así como realizar ECG, para detectar eventuales anomalías de la conducción. Otro
medicamento que se puede utilizar es la desmopresina.

CASO CLINICO
Se trata de una paciente de 12 años de edad, quien cursa séptimo grado y vive con su madre y sus dos hermanos (un niño de 10 años y
una niña de 5 años), La madre refiere que desde hace 3 años presenta enuresis y enconpresis en cualquier momento y en cualquier
situación, refiere la madre además aislamiento social, tendencia al mutismo, llanto fácil y somnolencia diurnia, refiere además
hiporexia, irritabiliadd, ideas de muerte, minusvalía y desesperanza

PREGUNTA
Cual es la conducta mas adecuada a seguir?

RESPUESTA
a.- Iniciar con imipramina.
b.- Enviar al psicólogo.
c.- Enviar al psiquiatra.
d.- Ingreso psiquiátrico obligado.

ENCOPRESIS. CIENCIAS BASICAS: Es difícil establecer un acuerdo unánime sobre este concepto. Es un trastorno de la eliminación.
Consiste en la emisión fecal, voluntaria o no (incapacidad de controlar el esfínter anal), de forma regular, sobre lugares socialmente
inapropiados, generalmente la ropa. SALUD PUBLICA: El control esfinteriano se adquiere progresivamente con la edad. En culturas
occidentales son encopreticos alrededor de 5%. A los 16 años no hay prácticamente ninguno, ya que la encopresis declina con la edad
en un porcentaje muy elevado (28%). El riesgo relativo de los niños respecto de las niñas es de 3-6 veces superior a partir de los 4 años
de edad. La encopresis secundaria iguala o supera ligeramente a la primaria. Los episodios de encopresis son más frecuentes durante el
día que por la noche. La encopresis retentiva es la más frecuente. CLASIFICACION: Según su etiología: FUNCIONAL O NO ORGANICA (no
hay causas orgánicas conocidas) y ORGÁNICA (puede ser provocada por trastornos gastrointestinales que cursan con diarrea,
intolerancia a la lactosa, insuficiencia pancreática, CUCI, lesiones posquirúrgicas, disrafismo espinal oculto, en estos casos la
incontinencia parece estar provocado por la fatiga de los músculos del suelo pélvico y del esfínter externo, después de que el niño los
haya contraído de forma continuada para preservar la continencia). Según la aparición del problema: PRIMARIA O CONTINUA (ausencia
de control desde que nació, mejor pronóstico, son niños desatendidos, regresivos, desinhibidos emocionalmente sin sentimiento de
vergüenza) y SECUNDARIA O DISCONTINUA (presencia de control por un periodo de continencia de al menos un año, coincide con
sucesos estresantes, son niños sobreprotegidos, inhibidos emocionalmente, con rasgos obsesivos, sentimientos de vergüenza). Según
su fisiopatología: RETENTIVA (con estreñimiento y rebosamiento, se caracteriza por un ciclo de varios días de retención, una expulsión
dolorosa, otro periodo de retención que ocasiona distensión y desensibilización de la pared rectal con ello se pierde la sensación de la
necesidad de defecar, aparece un megacolon psicógeno, las heces se acumulan y finalmente se defeca por rebosamiento) y NO
RETENTIVA (sin estreñimiento, el problema puede ser consecuencia de entrenamiento inadecuado, reacción fisiológica ante el estrés
ambiental o una forma de evidenciar conductas de oposición ante las normas establecidas). PATOGENIA: La defecación es el eslabón
final de todo el proceso digestivo. Empieza con la ingestión de alimentos y continúa por el tracto digestivo, y acaba en el intestino
grueso que se divide en: ciego, colon y recto. Estos últimos son los responsables de la defecación y la continencia. Al menos dos
factores contribuyen a la continencia, la distribución anatómica de la parte terminal del intestino grueso (ángulo anorectal) y la
contracción tónica del musculo puborectal y del esfínter externo. Dichas condiciones anatómicas intervienen para evitar el flujo fecal
cuando la presión intraabdominal se eleva súbitamente, como sucede con los estornudos, la risa o cambios posturales bruscos. Cuando
cierta cantidad de heces entra en el recto y se llena, se origina un reflejo que inhibe la contracción tónica del esfínter interno, que se
relaja, la presión del canal anal disminuye y las heces pueden descender. Ante la distensión rectal, el sujeto experimenta sensación de
plenitud que se acompaña del deseo de defecar. Hay que recordar que este reflejo no se produce en los sujetos con enfermedad de
Hirschsprug, debido a la ausencia congénita de células nerviosas en algunos segmentos del colon, lo que provoca estreñimiento y
obstrucción intestinal. La contracción (relajación) del esfínter externo ante la distensión del recto es una respuesta voluntaria que se
aprende durante el segundo año de vida, ligado al desarrollo neuromuscular y al entrenamiento del niño en hábitos de eliminación. Por
lo tanto, la mayoría de los individuos con incontinencia fecal muestran anomalías en algunos de estos mecanismos: 1. Capacidad para
percibir la distensión rectal (sensibilidad anorectal). 2. Capacidad del recto para almacenar heces (acomodación y adaptación al bolo

CURSO ENARM CMN SIGLO XXI TEL: 36246001 Pharmed Solutions Institute PÁGINA 472
MANUAL DE TRABAJO DEL CURSO ENARM CMN SIGLO XXI
fecal): función de depósito. 3. Habilidad de contracción del esfínter externo. 4. Motivación para ejecutar las respuestas adecuadas. En
los bebes la defecación es un proceso automático (relajación de los esfínteres de forma refleja cuando el recto está lleno).
DIAGNOSTICO: Puesto que la encopresis es un trastorno psicofisiológico y multicausado, conllevara la realización de una exploración
médica y psicológica. El principal objetivo es descartar la existencia de cuadros orgánicos que puedan ser responsables de la
incontinencia. En general el protocolo es: Historia clínica. Exploración física (abdominal, perianal, tacto anorectal). Análisis de sangre
específicos. Urocultivo. Estudios radiológicos. Manometría anorrectal. Biopsia rectal. El objetivo de la evaluación conductual reside en:
1. Averiguar si se han adquirido buenos hábitos de higiene y defecación. 2. Determinar factores ambientales, sociales y personales que
pueden estar interfiriendo. 3. Establecer relaciones funcionales entre estos. TRATAMIENTO: Consta de 2 fases: Fase inicial de limpieza
fecal o desimpactación (enemas). Fase de mantenimiento (laxantes). Modificaciones dietéticas (fibra). Ingestión de líquidos. El
protocolo de Levine, incluía una combinación de tratamiento médico y conductual. Respuesta no significativa a fármacos (imipramina).
Tratamiento conductual: Identificar las conductas objeto de modificación fecal. Enseñar conductas requisito para la defecación.
Instaurar hábitos rutinarios. Técnicas empleadas: reforzamiento positivo, reforzamiento negativo, castigo, psicoterapia, entrenamiento
en hábitos de defecación.

CASO CLINICO
Niño varón de cuatro años que presenta problemas relacionados con el control de esfínteres. La madre consulta porque el niño
muestra retención voluntaria de heces y orina y rechazo total a sentarse en el retrete. Refiere que el niño solicita y exige a los padres
que le pongan el pañal, y solo entonces lleva a cabo las deposiciones.

PREGUNTA
Cual es la conducta mas apropiada a seguir?

RESPUESTA
a.- Psicoterapia conductual.
b.- Psicoterapia gestáltica.
c.- Psicoterapia cognitiva.
d.- Psicoterapia psicoanalítica.

TRASTORNO POR DÉFICIT DE ATENCIÓN E HIPERACTIVIDAD (TDAH). CIENCIAS BASICAS: Es el trastorno psiquiátrico más frecuente en
la infancia; es un proceso crónico con una alta comorbilidad que va a influir en el funcionamiento del individuo en la edad adulta. El
TDAH según el DSM-IV (o trastorno hipercinético según el CIE-10) se define como un determinado grado de déficit de atención y/o
hiperactividad-impulsividad que resulta desadaptativo e incoherente en relación con el nivel de desarrollo del niño y está presente
antes de los 7 años de edad. Las manifestaciones clínicas deben persistir durante más de 6 meses. El cuadro debe ser más severo que lo
observado en otros niños de la
misma edad, el mismo nivel de Criterios diagnósticos del TDAH y diferencias entre DSM-IV y CIE-10 (negrita CIE-10)
desarrollo e inteligencia. Debe Déficit de atención:
1. A menudo no presta atención suficiente a los detalles o incurre en errores por descuido en las tareas escolares,
estar presente en varios en el trabajo o en otras actividades.
ambientes como familia, 2. A menudo tiene dificultades para mantener (no mantiene) la atención en tareas o en actividades lúdicas
escuela, amigos. Debe producir 3. A menudo parece no escuchar cuando se le habla directamente (lo que se le dice)
4. A menudo no sigue instrucciones y no finaliza tareas escolares, encargos u obligaciones en el lugar de trabajo
serios problemas en la vida 5. A menudo tiene dificultad (presenta alteración) para organizar tareas y actividades
diaria. SALUD PUBLICA: TDAH 6. A menudo evita (o muestra una fuerte aversión), le disgustan tareas que requieren un esfuerzo mental
es un importante problema de sostenido
7. A menudo extravía objetos necesarios para tareas o actividades
salud pública debido a varias
8. A menudo se distrae fácilmente por estímulos irrelevantes
razones: 1. Su alta prevalencia 9. A menudo es descuidado en las actividades diarias
(Utilizando los criterios del Hiperactividad
DSM-IV la prevalencia se sitúa 1. A menudo mueve en exceso manos o pies, o se remueve en su asiento
2. A menudo abandona su asiento en la clase o en otras situaciones en que se espera permanezca sentado
entre el 3-7%. Con la CIE-10 la 3. A menudo corre o salta excesivamente en situaciones en que es inapropiado hacerlo
prevalencia es del 1,5% y los 4. A menudo tiene dificultades para jugar o dedicarse tranquilamente a actividades de ocio
cuadros más severos) 2. El 5. A menudo “está en marcha” o suele actuar como si “tuviera un motor” (Exhibe un patrón persistente de
actividad motora excesiva que no se modifica sustancialmente por el contexto o exigencias sociales)
inicio en etapas precoces de la
Impulsividad
infancia. 3. A ser un proceso 1. A menudo precipita respuestas antes de haber sido completadas las preguntas
incapacitante y crónico. 4. A la 2. A menudo tiene dificultades para guardar su turno (A menudo no espera en la cola o no guarda su turno en
afectación de las diferentes juegos o situaciones de grupo)
3. A menudo interrumpe o se inmiscuye en las actividades de otros (p.ej., se entromete en conversaciones o
esferas comportamentales juegos)
(familiar, escolar y social), y 5. 4. A menudo habla en exceso (sin una respuesta apropiada a las limitaciones sociales)
A su alta comorbilidad, por lo
que siempre deben investigarse otras patologías asociadas que conlleven un gran riesgo para el futuro del niño, como son trastornos
del aprendizaje, problemas de conducta o trastornos emocionales, entre otros. La relación varón/mujer varía según los estudios desde
6/1 a 3/1. CLASIFICACION: El DSM-IV, describe 3 subtipos: 1. Subtipo con predominio inatento (inatención). Entre 20-30% de los casos.
2. Subtipo con predominio hiperactivo-impulsivo del 10-15%. 3. Subtipo combinado (cuando están presentes los 3 tipos de síntomas).
Del 50-75% de los casos. Según la CIE-10; es necesaria la existencia simultánea de los 3 tipos de síntomas: déficit de atención,
hiperactividad e impulsividad, constituyendo el cuadro de “alteración de la actividad y la atención”. Reconoce además una categoría
separada, “el trastorno hiperquinético de la conducta”. PATOGENIA: Se trata de un trastorno multifactorial con una base
neurobiológica y predisposición genética que interactúa con factores ambientales. Las hipótesis sobre la etiología abarcan diversas

CURSO ENARM CMN SIGLO XXI TEL: 36246001 Pharmed Solutions Institute PÁGINA 473
MANUAL DE TRABAJO DEL CURSO ENARM CMN SIGLO XXI
áreas: genética conductual (mayor prevalencia en pacientes con familiares con TDAH y otros trastornos psiquiátricos. En gemelos) y
molecular (factores genéticos, en relación con mutaciones de varios genes que codifican para los transportadores y receptores de la
dopamina y el gen transportador de la noradrenalina, fallo en el desarrollo de los circuitos cerebrales, en que se apoya la inhibición y el
autocontrol), factores biológicos adquiridos (en periodo prenatal, perinatal y postnatal como exposición intrauterina al alcohol, nicotina
y determinados fármacos [benzodiazepnias, anticonvulsivantes], prematuridad, bajo peso al nacer, encefalitis, traumatismos, hipoxia,
hipogluecemia), la neuroanatomía (implicación del córtex prefrontal y ganglios basales [áreas encargadas de regular la atención], se
sugieren alteraciones en las redes corticales cerebrales frontales y frontoestriadas. Sistema atencional anterior: lóbulo frontal. Sistema
atencional posterior: lóbulo parietal y cerebelo), neuroquímicos (la NA y DA, son los neurotransmisores de mayor relevancia en la
fisiopatología y tratamiento de TDAH, ambos implicados en la función atencional y la DA también en la regulación motora), bioquímica
cerebral, neurofisiología (alteraciones de la actividad cerebral como: reducción del metabolismo/flujo sanguíneo en el lóbulo frontal,
córtex, parietal, stratium, cerebelo), neuropsicología y el entorno psicosocial (severidad y expresión de los síntomas puede verse
afectado a través de la interacción gen-ambiente. Factores como: inestabilidad familiar, problemas con amigos, trastornos psiquiátricos
en los padres, paternidad y crianza inadecuada, relaciones negativas padres-hijos, niños que viven en instituciones con ruptura de
vínculos, adopciones y bajo nivel socioeconómico). DIAGNOSTICO: Los tres síntomas esenciales del TDAH son: 1. El déficit de atención
(falta de perseverancia en la atención o atención dispersa), 2. La hiperactividad (excesivo movimiento) y 3. La impulsividad (dificultad
en el control de impulsos). El déficit de atención guarda una mayor relación con las dificultades académicas y el logro de metas,
mientras que la hiperactividad e impulsividad están más relacionadas con las relaciones sociales y los resultados psiquiátricos. Las
manifestaciones clínicas varían en grado e intensidad según la edad del paciente en el momento del diagnóstico. El DSM-IV requiere
que estén presentes al menos 6 de los 9 síntomas tanto para el subtipo con predominio del déficit de atención, como para el subtipo
hiperactivo-impulsivo. El TDAH de tipo combinado requiere una combinación de ambos tipos de síntomas de déficit de atención y de
hiperactividad impulsividad (6+6). Para la CIE-10 se requiere 6/9 síntomas de déficit de atención además de al menos 3/5 síntomas de
hiperactividad y 1/4 síntomas de impulsividad. La impulsividad constituye un síntoma importante en el DSM-IV pero no tanto en el CIE-
10. Existen una serie de criterios diagnósticos adicionales que se requieren para poder hablar de trastorno y no solo de síntomas
transitorios: Edad de inicio. Algunos síntomas deben haber estado presentes antes de los 6-7 años. Duración. Los criterios sintomáticos
deben haber persistido al menos durante los últimos 6 meses- Ubicuidad. Algún grado de disfunción debida a los síntomas debe haber
estado presente en 2 situaciones o más (escuela, trabajo, casa). Disfunción. Los síntomas deben ser causa de una disfunción
significativa (social, académica o laboral). Discrepancia. Los síntomas son excesivos en comparación con otros niños de la misma edad,
desarrollo y nivel de inteligencia. Exclusión. Los síntomas no se explican mejor por la presencia de otro trastorno mental como
ansiedad, depresión o esquizofrenia, entre otros. Además, es imprescindible obtener registros de síntomas mediante cuestionarios para
evaluar la gravedad o intensidad de los síntomas y su presencia en varios ambientes. Cuestionarios como el “Cuestionario de Evaluación
del TDAH” son muy útiles para obtener de forma rápida los síntomas que están presentes, tanto desde el punto de vista de los padres
como de los profesores. TRATAMIENTO: Un plan de tratamiento individualizado y multidisciplinar para un niño con TDAH y su familia
casi siempre debe incluir 3 aspectos fundamentales: 1. Entrenamiento a los padres: Psicoedución sobre el TDAH. Entrenamiento en
técnicas de manejo conductual del niño. 2. Intervención a niveles académico y escolar: En el colegio. En casa. 3. Medicación específica
para el TDAH. El uso de medicación debe ser una parte del plan de tratamiento inicial en la mayoría de los niños en edad escolar y
adolescentes con TDAH. El reciente estudio de tratamiento multimodal del TDAH, del Instituto Nacional de Salud Mental, ha
demostrado que un tratamiento farmacológico cuidadoso y estandarizado se asocia con una reducción de síntomas significativamente
mayor, en la mayoría de los niños, que un tratamiento de intervención psicosocial con diferentes tipos de psicoterapia. Pero el grupo de
terapia combinada con psicoterapia y medicación obtuvo mejores resultados que el grupo de terapia con medicación sola en el
porcentaje de niños en remisión. Tratamiento farmacológico, los más utilizados y de primera elección son los psicoestimulantes, una
segunda alternativa los antidepresivos tricíclicos si existe comorbilidad con problemas afectivos y emocionales y podría utilizarse los
inhibidores selectivos de la recaptura de serotonina, en algunos casos en donde la impulsividad es muy evidente. Uso de medicaciones
estimulantes en el tratamiento del TDAH: El metilfenidato (dosis de 0.1mg/mg, para fines practicos se inicia con dosis bajas de 2.5mg,
es el único estimulante indicado para el tratamiento TDAH, mejoran la atención, el comportamiento en clase y el desarrollo académico
a corto plazo, mejora la interacción con compañeros y disminuye el comportamiento impulsivo. Se presenta en comprimidos de 5,10 y
20 mg. Existen opciones farmacológicas diversas útiles en el manejo del TDAH solo o con comorbilidad, como los medicamentos no
estimulantes (atomoxetina, antidepresivos tricíclicos, agonistas alfa adrenérgicos y modafinilo).

CASO CLINICO
Niña de 10 años, remetida a psicólogo a petición del colegio por fracaso escolar. Se trata de una niña con buena capacidad intelectual
que aunque no le gusta el estudio, se aplica trabajando en los deberes escolares, nunca ha mostrado comportamiento hiperactivo ni
han existido dificultades a la hora de obedecer las normas ni en casa ni en el colegio; la única queja de sus padres es “lo distraída que
es” (no se concentra, le mandas a hacer un recado y se entretiene en cien mil cosas sin llegar a hacerlo, desorganizada, pierde las
cosas,etc) la considera una niña feliz, aunque comenta que sus dificultades escolares hace que a veces se entristezca.

PREGUNTA
Cual es el diagnostico mas probable?

RESPUESTA
a.- TDAH.
b.- TDA.
c.- TDAH-R
d.- Trastorno del control de impulsos.

CURSO ENARM CMN SIGLO XXI TEL: 36246001 Pharmed Solutions Institute PÁGINA 474
MANUAL DE TRABAJO DEL CURSO ENARM CMN SIGLO XXI

CIRUGIA

1) ESOFAGITIS, ACALASIA, REFLUJO ESOFAGICO Y ERGE.


2) DISPLASIAS DE ESOFAGO, ESOFAGO DE BARRETT, CANCER DE ESOFAGO.
3) VARICES ESOFAGICAS, RUPTURA ESOFAGICA Y HERNIA HIATAL.
4) GASTRITIS, ULCERA GASTRICA Y ULCERA DUODENAL, CANCER GASTRICO.
5) COLANGITIS, COLECISTITIS, COLEDOCOLITIASIS
6) CANCER DE PANCREAS, COLANGIOCARCINOMA,
7) PANCREATITIS AGUDA Y CRONICA.
8) ISQUEMIA MESENTERICA, INFARTO MESENTERICO, DIVERTICULITIS.
9) APENDICITIS, POLIPOS, COLITIS, CROHN, CUCI.
10) HEMORROIDES, FISTULA ANAL, CANCER DE COLON
11) HERNIAS DIAFRAGMATICAS, HIATAL, DE PARED E INGUINAL,
12) ORQUIEPIDIDIMITIS, TORSION TESTICULAR, CANCER TESTICULAR.
13) HIPERTROFIA PROSTATICA BENIGNA, CANCER DE PROSTATA
14) TCE, TRAUMA FACIAL Y TRAUMA RAQUIMEDULAR.
15) TRAUMA TORACICO, TRAUMA ABDOMINAL Y PELVICO
16) PIE DIABETICO Y COMPLICACIONES QUIRURGICAS DE LA DM.
17) RETINOPATIA DIABETICA E HIPERTENSIVA.
18) URGENCIAS OFTALMOLOGICAS, PERFORACION, GLAUCOMA. (CIRUGIA)
19) TRAUMA OTICO RUPTURA, PERFORACION, TRAUMA FACIAL.

CURSO ENARM CMN SIGLO XXI TEL: 36246001 Pharmed Solutions Institute PÁGINA 475
MANUAL DE TRABAJO DEL CURSO ENARM CMN SIGLO XXI
ESOFAGITIS. CIENCIAS BASICAS: Es un término general para cualquier inflamación, irritación o hinchazón del esófago, el tubo que va
desde la parte posterior de la boca hasta el estómago. Causas: Disminución de la eficacia de los mecanismos antirreflujo esofágico, en
particular del tono del Esfínter Esofágico Inferior (EEI), su tono normal es de 10-25mmHg. Presencia de una Hernia hiatal por
deslizamiento. Eliminación inadecuada o lenta del material refluido. Reducción de la capacidad de reparación de la mucosa esofágica
por exposición prolongada a jugo gástrico. Clínica: Acidez y ardor (pirosis), que el enfermo localiza en el epigastrio y en la región
retroesternal. Otros síntomas de reflujo gastroesofágico, regurgitaciones ácidas o vómitos, crónicos, dolor retroesternal, disfagia.
Factores incrementan el riesgo de esofagitis: Consumo de alcohol, de cigarrillos, cirugía o radiación en el pecho (por ejemplo, el
tratamiento para el cáncer pulmonar), tomar ciertos medicamentos sin mucha agua. Clínica: Disfagia, pirosis, regurgitación,
hematemesis. La etiología más frecuente es la esofagitis péptica, secundaria a reflujo gastroesofágico no tratado. Otras causas son:
ESOFAGITIS INFECCIOSA (candida, herpes): Afectación de la mucosa esofágica por distintos agentes fúngicos, víricos o bacterianos. Se
debe sospechar en sujetos con factores favorecedores y disfagia. Esofagitis por Candida: La infección esofágica más frecuente.
Favorecida en sujetos malnutridos e infectados por el virus de la inmunodeficiencia humana. ESOFAGITIS MEDICAMENTOSA:
alendronato, tetraciclina, doxiciclina, ibandronato, risedronato y vitamina C. ESOFAGITIS CÁUSTICA: Injuria de la mucosa esofágica
secundaria a la exposición de la misma a agentes cáusticos (ácidos o álcalis). El daño sobre el esófago y el estómago por la ingesta de
cáusticos se asocia a una alta morbimortalidad. Las causas de la ingesta pueden ser accidentales, frecuentemente en niños, o
voluntarias con fines suicidas, generalmente en adultos. Detección temprana de probable perforación (examen clínico + Rx tórax y
abdomen). Interrogatorio al paciente o familiar. Debemos interrogar acerca de: tipo de cáustico Ingerido (álcali o ácido), cantidad
ingerida e intencionalidad. No usar sonda nasogástrica ni lavado. No usar agentes neutralizantes. No inducir el vómito. ESOFAGITIS
EOSINOFÍLICA: es una entidad que se caracteriza por la infiltración tisular muy significativa, en este caso al nivel del esófago, de
eosinófilos. DIAGNÓSTICO: clínico por endoscopia. Clasificación de Los Angeles para esofagitis: Grado A: Una o más rupturas de la
mucosa confinadas a los pliegues mucosos, cada uno < 5 mm. Grado B: Por lo menos una ruptura de la mucosa > 5 mm, confinada a los
pliegues mucosos pero sin continuidad en los bordes de dos de los pliegues. Grado C: Por lo menos una ruptura de la mucosa entre los
límites de dos pliegues de la mucosa, pero sin ser circuferencial. Grado D: Ruptura circunferencial de la mucosa. TRATAMIENTO:
Específico del tipo de Esofagitis que se trate, se utilizan medicamentos como Ranitidina, Famotidina, Omeprazol entre otros.

CASO CLINICO
Paciente de sexo masculino de 75 años con antecedentes de hipertensión arterial que ingresó a la Unidad de Cuidados Intensivos con el
diagnóstico de shock. Presentó vómitos alimentarios y pirosis durante las veinticuatro horas previas al ingreso. La familia refirió un
episodio de fiebre y escalofríos. Se colocó una sonda nasogástrica observándose al aspirar un contenido en borra de café. No refería
hematemesis o melena. El paciente fue reanimado con infusión de cristaloides y requirió drogas vasopresoras durante las primeras 24 h
(noradrenalina 0,1 mcg/kg/min). A su ingreso se realizaron hemocultivos y urocultlvo, ambos fueron positivos a las 48 h para
Escheríchia coli multisensible. Se inició tratamiento empírico con ceftriaxona 1 gr cada 12 h que luego se cambió según sensibilidad y
cumplió tratamiento antibiótico por 14 días. La analítica demostró leucocitosis y anemia con hematocrito de 30%.

PREGUNTA
Cual de las siguientes medida no es apropiada en este momento?

RESPUESTA
a.- Inhibidores de la bomba de protones por vía endovenosa.
b.- Sucralfato por vía oral.
c.- Metronidazol 500 mg/cada 8 h por siete días.
d.- Prednisona via oral.

PREGUNTA
La presentación clínica más común de esta patología es?

RESPUESTA
a.- Sangrado digestivo alto.
b.- Hematemesis.
c.- Vómitos en borra de café
d.- Dolor epigástrico.

PREGUNTA
Cual de los siguientes diagnosticos diferenciales es menos frecuente?

RESPUESTA
a.- Necrosis local secundaria a infecciones.
b.- Hematoma intramural del esófago.
c.- Lesion por cáusticos.
d.- Síndrome pilórico con reflujo severo.

CURSO ENARM CMN SIGLO XXI TEL: 36246001 Pharmed Solutions Institute PÁGINA 476
MANUAL DE TRABAJO DEL CURSO ENARM CMN SIGLO XXI
ACALASIA. CIENCIAS BASICAS: Es un trastorno primario de la motilidad esofágica caracterizado por la ausencia de peristálsis esofágica y
por la relajación incompleta del EEI, afecta los dos tercios inferiores. La etiología principal se debe a la perdida selectiva de las
motoneuronas inhibitorias, liberadores de péptido intestinal vasoactivo y óxido nítrico del plexo mientérico. La pérdida de estas
neuronas ocasiona el incremento en la presión basal, la relajación incompleta del EEI y la desaparición de la latencia y naturaleza
peristáltica de la contracción del cuerpo esofágico. Estas alteraciones conllevan a una disminución de la propulsión esofágica y un
incremento en la resistencia del EEI, son las responsables de la dificultad de tránsito y de la disminución del aclaramiento esofágico y
son la base de los síntomas clínicos y signos radiológicos, endoscópicos y manométricos de la enfermedad. La acalasia se clasifica en
primaria o idiopática que comprende el mayor número de casos, y en secundaria (neoplasias, infiltraciones de esófago, estómago,
linfomas, enfermedad de Chagas SALUD PÚBLICA: La prevalencia de la enfermedad es de aproximadamente 10 casos por 100.000 hab.
Su incidencia se ha mantenido bastante estable durante los últimos 50 años en aproximadamente 0,5 casos por 100.000 hab por año.
Puede manifestarse a cualquier edad, sin predilección racial y con igual frecuencia en hombres y mujeres, pero la mayoría de los casos
es diagnosticada generalmente entre los 25 y 60 años de vida. PATOGENIA: La región principal afectada es el plexo esofágico mientérico
(Auerbach) que incluye zonas de respuesta inflamatoria en su mayor parte linfocitos T citotóxicos CD3 y CD8 positivos, un número
variable de eosinófilos y los mastocitos, la pérdida de células ganglionares y cierto grado de fibrosis. En casos avanzados puede haber
afección de las neuronas colinérgicas, incluso a nivel central. DIAGNOSTICO: Clínico; El síntoma inicial y más común es la disfagia (90%),
que al ser de origen motor y no mecánico, es tanto a solidos como a líquidos, suele empeorar con las comidas rápidas y la tensión.
Emocional. Puede haber dolor y también regurgitación (75%) la cual se debe a la retención de alimentos no digeridos en la porción
inferior del esófago, esta situación con el tiempo puede desencadenar esofagitis y pirosis, pero no es lo más común. Hay pérdida de
peso a lo largo de meses o años, cuando esta es acelerada se debe considerar la posibilidad de acalasia secundaria a una neoplasia
maligna. Radiografía de tórax se puede llegar a observar ausencia de cámara gástrica y un nivel hidroaéreo en mediastino. El trago de
bario muestra dilatación esofágica distal que termina en punta de lápiz, y en casos avanzados un esófago sigmoideo. La prueba con la
sensibilidad más alta en la diagnostico de la Acalasia es La manometría esofágica y el único método posible para el diagnóstico de la
enfermedad en sus fases iniciales, en las que todavía no se han producido los cambios morfológicos detectables por métodos
radiológicos o endoscópicos. Todas las sospechas diagnósticas de Acalasia deben confirmarse mediante manometría. La especificidad
diagnóstica de la ME no es del 100%, es importante destacar que el patrón manométrico de la Acalasia es indistinguible del ocasionado
por la obstrucción mecánica del EEI –casi siempre por un tumor– que origina un cuadro denominado pseudoacalasia‖, que corresponde
casi al 5% de los diagnósticos manométricos de Acalasia y que debe ser particularmente sospechado en pacientes ancianos o con una
clínica rápidamente progresiva. La endoscopia es útil para excluir las lesiones orgánicas, en particular aquellas que son causas
secundarias de Acalasia, como el carcinoma del cardias gástrico, entre otros. Durante el procedimiento se observa a menudo dilatación
y atonía del cuerpo esofágico, con tortuosidades en casos avanzados y que el lumen esofágico está lleno de saliva y restos de comida, y
la región de la unión esófago gástrico está cerrada. TRATAMIENTO: No hay tratamiento específico. Se recomienda dieta blanda.
Fármacos: anticolinérgicos resultan poco efectivos, nitritos o antagonistas de calcio antes de los alimentos, pero no son muy efectivos.
El sildenafil regula la función motora del EEI. La toxina botulínica tiene eficacia de 60% a los seis meses, se utiliza más bien como una
medida temporal, antes de la cirugía. Por lo general a los pacientes menores de 50 años, se les propone el quirúrgico de primera
instancia, de no haber éxito se pueden hacer dilataciones. A los mayores de dicha edad se les recomienda dilataciones neumáticas o la
aplicación de toxina botulínica de manera periódica, si no hay respuesta se pierde proponer tratamiento quirúrgico. Después del ERGE,
la acalasia es el trastorno funcional más frecuente del esófago que amerita intervención quirúrgica. El alivio puede lograse mediante
una rotura instrumental no controlada del musculo del esfínter o con una miotomía quirúrgica controlada, esta conlleva un bajo índice
de morbilidad y logra mejores resultados a largo plazo. La miotomía se puede acompañar de funduplicatura parcial para disminuir los
eventos de reflujo posquirúrgico. En la práctica la mayoría de los pacientes se somete a dilatación. Un riesgo inherente de la dilatación
neumática es la rotura esofágica. La miotomía modificada de Heller se practica en todas las capas musculares: se extiende en sentido
distal sobre el estómago 1-2 cm por debajo de la unión y se prolonga 4-5cm en sentido proximal sobre el esófago. La esofagectomía es
el mejor tratamiento para los pacientes con disfagia y enfermedad benigna prolongada cuya función esofágica se destruyó por
enfermedad o múltiples procedimientos quirúrgicos.

CASO CLINICO
Masculino de 56 años, fumador, diagnosticado con acalasia hace 6 años, sin tratamiento por decisión del paciente, refiere que hace 3
meses presenta disfagia a liquidos y solidos además perdida de peso, se realiza endoscopia donde se apresia abundante liquido, con
paredes dilatadas, mucosa con aspecto “desquebrajado”, friable. La biopsia reporta positivo para neoplasia.

PREGUNTA
Cual de las siguientes tipos de neoplasia es mas frecuente observar?

RESPUESTA
a.- Leiomioma.
b.- Tumor estroma gastrointestinal.
c.- Carcinomas de células escamosas.
d.- Carcinomas de células pequeñas.

CURSO ENARM CMN SIGLO XXI TEL: 36246001 Pharmed Solutions Institute PÁGINA 477
MANUAL DE TRABAJO DEL CURSO ENARM CMN SIGLO XXI
ENFERMEDAD POR REFLUJO GASTROESOFAGICO (ERGE) Y REFLUJO GASTROESSOFAGICO (RGE). CIENCIAS BASICAS: El RGE se refiere
al paso de contenido gástrico hacia el esófago puede ser fisiológico (máximo 5 episodios al día, fenómeno diurno, de corta duración, no
evoca síntomas, no daña la mucosa esofágica) o patológico. Se habla de ERGE cuando el reflujo ocasiona síntomas o complicaciones. La
enfermedad por reflujo se manifiesta de diferentes formas: ERGE no erosiva (70%), ERGE erosiva (25%), ERGE asociada a esófago de
Barret (4%). La regurgitación se define como el paso del contenido gástrico hacia el esófago y/o hacia la boca “sin esfuerzo alguno “es
decir, no hay contracción del diafragma; es el regreso involuntario hacia la boca, de comida o secreciones previamente deglutidas.
SALUD PUBLICA: El reflujo gastroesofágico ocasiona cerca de 75% de los padecimientos esofágicos. El ERGE se presenta en 3-4% de la
población general, con aumento en su prevalencia en embarazadas y adultos mayores PATOGENIA: Los tres mecanismos más
importantes que condicionan la incompetencia de la barrera anti-reflujo son: 1) Las relajaciones transitorias del EEI, 2) la hipotensión
del EEI, y 3) la alteración anatómica de la unión condicionada por la presencia de hernia hiatal. Otros factores que interactúan; la
ansiedad, la depresión o el estrés, de tal manera que estímulos mínimos en el esófago desencadenan dolor. La ingesta de alimentos
irritantes y grasas también pueden intensificar la percepción del ácido. La obesidad es un factor de riesgo para la ERGE. El reflujo ácido
estimula el nervio vago y esto es la causa de la broncoconstricción; así como el reflujo de pepsina es la causante de la enfermedad
ulcerosa péptica. El tabaquismo incrementa los síntomas de RGE. Aunque no hay estudios concluyentes, se ha demostrado que la
ingesta en exceso de cítricos, bebidas carbonatadas, café, cafeína, chocolate, cebolla, especies, comidas ricas en grasas, menta,
yerbabuena, alimentos salados y la fibra de algunos tipos de pan, pueden estar relacionados con el reflujo. Pueden aumentar los
síntomas en la ingesta de medicamentos no esteroides y AINES. DIAGNOSTICO: Clínico; Los síndromes esofágicos de la ERGE se
caracterizan por la presencia de síntomas típicos pirosis (más característico y frecuente) y regurgitaciones, e incluso dolor torácico o
asociados a lesiones esofágicas como: erosiones, estenosis, Barrett y adenocarcinoma. Las manifestaciones extra-esofágicas más
comunes son tos (38%), dolor torácico (30%), disfonía (22%), asma (5%), escurrimiento nasal posterior (5%). Al parecer la presencia de
síntomas típicos está relacionada con reflujo ácido, mientras que el reflujo no ácido (bilis, aire, etc.) se relaciona con la aparición de
síntomas atípicos. El RGE que afecta severamente a la laringe puede contribuir a la aparición de cáncer en la laringe. Al parecer el 46%
de todas las apneas están relacionadas con episodios de reflujo. Se ha observado que la pobre calidad de sueño está relacionada a un
gran número de eventos de reflujo mediante pH-metría. Dentro de las herramienta diagnosticas se cuenta con cuestionarios de
síntomas que otorgan una puntuación, con base en lo cual se determina si el paciente tienen o no ERGE. Ensayo terapéutico con
inhibidores de bomba de protones (IBP) en pacientes no complicados, con una dosis estándar por 1-2 semanas (omeprazol 40mgs
c/12hrs, pantoprazol 80 mg c/12hrs). La desaparición de los síntomas con el tratamiento y la recaída al suspenderlo confirma el
diagnóstico. En caso de pacientes con síntomas extraesofágicos se recomienda extender el tiempo de administración hasta tres meses.
La sensibilidad y especificidad varían entre 75 y 92% y 55 al 90% respectivamente. La pH-metría es la prueba más sensible y específica y
está indicada en ERGE, que no respondió a tratamiento, en ERGE posquirúrgico. Los estudios radiográficos son útiles para visualizar
defectos anatómicos como: Presencia de hernia hiatal, anillos, estrecheces, pero no es útil para valorar el reflujo ni la esofagitis. El
esofagograma es útil para valorar el tránsito esofágico, en casos de disfagia posoperatoria o cuando se sospecha neoplasia, debe
realizarse en el segundo o tercer nivel de atención en pacientes posquirúrgicos de hernia hiatal, en los cuales hay disfagia.
TRATAMIENTO: No recostarse justo después de las comidas, evitar alimentos que relajen el EEI, evitar ropa ajustada (mejoría 20%). Los
antiácidos controlan síntomas leves de pirosis y regurgitación, pero no curan la esofagitis. Los inhibidores de la secreción acida; son la
piedra angular, los antagonistas H2 (ranitidina), logran curación de esofagitis en 50%. Los IBP logran curación de 80% a 4 sem y cerca de
90% a 8 sem. El tratamiento de reflujo alcalino es a base de sucralfato, colestiramina o hidróxido de aluminio. La cirugía antirreflujo
está indicada en pacientes con esofagitis, manifestaciones extraesofágicas de difícil control. Antes de la cirugía se debe realizar una
manometría y, además se sugiere corroborar el diagnostico por pH-metría en 24 hrs. El procedimiento antireflujo más frecuente es la
funduplicación de Nissen que solo usa el fondo gástrico para envolver el esófago, la funduplicación se calibra con un dilatador 60
francés y la longitud de la misma se limita a 1-2cm. Cuando la motilidad esofágica no es normal y la fuerza propulsiva no es suficiente
para vencer la obstrucción de una funduplicatura completa, está indicada una funduplicación parcial. En pacientes con esófago corto
por estrechamiento, esófago de Barret o una hernia hiatal grande se realiza una gastroplastia de Collis como procedimiento para
prolongación esofágica.

CASO CLINICO
Se trata de paciente femenino de 24 años de edad, acude a consulta por opsomenorrea y oligomenorrea, agrega que no tiene vida
sexual activa desde hace 2 años, refiere además que últimamente se siente fatigada y ha presentado mareos ocacionales, niega uso de
sustancia, agrega que ocacionalmente toma ranitidina, finalmente refiere regurgitación y pirosis. A la exploración física se observa IMC
de 17, pelo quebradizo, palidez generalizada, llenado capilar de 3 segundos, los exámenes de laboratorio reportaron hemoglobina 10.

PREGUNTA
Cual de los siguientes diagnosticos es la menos frecuente en el caso clínico?

RESPUESTA
a.- Gastritis erosiva.
b.- Esofagitis.
c.- Enfermedad por reflujo gastroesofagico.
d.- Anorexia nerviosa.

PREGUNTA
Luego de tratamiento con omeprazol 20 mg/día, presentando leve mejoría sin embargo no se presento mejoría de la hemoglobina y el
hematocrito, a la exploración se observa halitosis y hemorragia suconjuntival, refiere que ha incrementado su ansiedad por presiones
laborales ya que su trabajo requiere mantenerse en talla 5, considerando la evolución del paciente cual es la patologia asociada que
esta modificando la respuesta?

CURSO ENARM CMN SIGLO XXI TEL: 36246001 Pharmed Solutions Institute PÁGINA 478
MANUAL DE TRABAJO DEL CURSO ENARM CMN SIGLO XXI
RESPUESTA
a.- Anorexia purgativa.
b.- Esofagitis erosiva.
c.- Esofago de barret.
d.- Ulcera gástrica.

PREGUNTA
La paciente regresa a consulta de seguimiento 3 meses despues, su tratamiento instalado fue de fluoxetina 20 mg/24 hrs, asi como
complejo b y hierro oral, los laboratorios reportaron hemoglobina 11 y hematocrito de 38, se realizo endoscopia donde se reporta una
lesión mucosa mayor de 5 mm, sin continuidad entre la parte más prominente de 2 pliegues mucosos, que grado presenta la paciente
de la Clasificación de Los Ángeles de la esofagitis.

RESPUESTA
a.- Grado A.
b.- Grado B.
c.- Grado C.
d.- Grado D.

PREGUNTA
Se indico cambios en el estilo de vida, tratamiento farmacológico por 6 meses, a la exploración se observo IMC de 23, se realizo una
endoscopia de control con el siguiente reporte, unión escamoso-columnar se encuentra desplazada proximalmente de color rojo de la
mucosa columnar el reporte histológico reporto existencia de neutrófilos y eosinófilos que infiltraban la mucosa y la lámina propia,
considerando las condiciones actuales, cual es la conducta a seguir.

RESPUESTA
a.- Vigilancia estrecha endoscopia anual.
b.- Funduplicatura.
c.- Fototerapia endoscópica.
d.- Esofagectomia.

CASO CLINICO
Masculino de 52 años de edad que acude a consulta debido a que presenta sensación de quemazón retrosternal que se irradia de
manera ascendente a cuello y espalda, que ha identificado se presenta entre media hora y una hora después de la alimentación,
acteriormente se presentaba solo cuando comia grasas e irritantes, pero actualmente con casi cualquier alimento y solo se resuelve
tomando antiácidos OTC. Antecedente de alcoholismo, tabaquismo positivo, tos crónica y osteoartitis controlada con diclofenaco. A la
exploración física se observa perdida del esmalte dental y halitosis.

PREGUNTA
Cuál es la conducta a seguir este caso?

RESPUESTA
a.- Omeprazol 20 mg cada 12 hrs.
b.- Misoprostol 400 µg cada 12 hrs.
c.- Sales de bismutos 10 mg cada 24.
d.- Malgadrato 5 ml cada 8 hrs.

CASO CLINICO
Varón de 46 años, sin alergias conocidas, fumador de 12 cigarrillos al día y bebedor de 40 g de alcohol diarios. Antecedentes personales
de apendectomía a los 14 años. Hace cuatro años fue medicado con ansiolíticos por haber padecido varias crisis de ansiedad. Desde
hace 18 meses aqueja sensación de quemazón en región epigástrica que se irradia hacia la parte central de tórax y, ocasionalmente,
hasta el cuello. Las molestias aumentan después de la ingesta así como en decúbito. Mejoran sólo un poco, y temporalmente, con la
toma de antiácidos. No refiere tener regurgitación, disfagia, odinofagia ni dolor torácico. Tampoco padece síntomas como náuseas o
vómitos. Su estado general es bueno sin haber presentado astenia, anorexia ni adelgazamiento. Medicado por su médico de cabecera
con ranitidina 300 mg por la noche, ha tenido una mejoría parcial que refiere como 3 en una escala de 0 a 10. A la exploración física
encontramos un paciente consciente y orientado. Coloración normal de piel y mucosas. No se objetivan adenomegalias ni lesiones
dérmicas. Auscultación cardiaca y pulmonar sin alteraciones. Abdomen blando y depresible; no se palpan ni se percuten masas ni
visceromegalias. Exploración neurológica sin alteraciones.

PREGUNTA
Ante la clínica observada en este paciente cual es el diagnostico mas probable?

RESPUESTA
a.- Dispepsia funcional.
b.- Enfermedad por reflujo gastroesofágico.
c.- Esofagitis por cándida.
a.- Acalasia.

CURSO ENARM CMN SIGLO XXI TEL: 36246001 Pharmed Solutions Institute PÁGINA 479
MANUAL DE TRABAJO DEL CURSO ENARM CMN SIGLO XXI
PREGUNTA
Cuál sería la siguiente exploración a realizar para confirmar el diagnóstico clínico?

RESPUESTA
a.- pH-metría esofágica de 24 horas.
b.- Esofagograma.
c.- Prueba de tratamiento empírico con omeprazol.
d.- Manometría esofágica.

PREGUNTA
El paciente continuo con manifestaciones clínicas, considerando este punto cual es la conducta diagnostica más adecuada a seguir para
establecer finalmente un diagnostico?

RESPUESTA
a.- pH-metría esofágica de 24 horas.
b.- Endoscopia y, si ésta fuese negativa, pH-metría esofágica de 24 horas.
c.- Endoscopia y, si ésta fuese positiva, pH-metría esofágica de 24 horas.
d.- Manometría esofágica.

PREGUNTA
La pH-metría esofágica fue informada como reflujo gastroesofágico ácido cuantitativamente normal con un índice bajo de correlación
sintomática, cual es el diagnostico considerando los datos observado?

RESPUESTA
a.- Dispepsia funcional.
b.- Somatización.
c.- Ansiedad.
d.- Pirosis funcional

PREGUNTA
¿Considerando la dirección que ha tomado el caso actual, cuál cree que sería el tratamiento más adecuado para este paciente?

a.- Alcalinos después de las comidas.


b.- IBP a dosis habituales.
c.- IBP a dosis superiores (al menos doble).
d.- Antagonista de los receptores H2

CURSO ENARM CMN SIGLO XXI TEL: 36246001 Pharmed Solutions Institute PÁGINA 480
MANUAL DE TRABAJO DEL CURSO ENARM CMN SIGLO XXI
ESOFAGO DE BARRET (EB). CIENCIAS BASICAS: Es la transformación metaplásica del epitelio esofágico, en el estudio patológico sin
importar la extensión. La marca distintiva de la metaplasia intestinal es la presencia de células caliciformes intestinales. Recodar
que RGE provoca un proceso inflamatorio de la mucosa esofágica, llamada Esofagitis por Reflujo. Esta última, predispone a
complicaciones o secuelas crónicas como úlceras, estenosis ó displasia, las que son generalmente irreversibles y que en algún caso,
como sucede en el EB, son lesiones consideradas como precancerosas o de mayor riesgo de cáncer. SALUD PUBLICA: El EB se presenta
en hasta 10% de los pacientes con RGE crónico y éste se presenta con una frecuencia estimada de entre 20 y 40% de la población. Es
más frecuente en el sexo masculino y en caucásicos, y la edad media al diagnóstico es de 60 años aproximadamente. Se ha descrito
también asociación familiar. La estimación del riesgo de adenocarcinoma es variable, va de 1 en 50 a 1 en 400 pacientes-año. Cerca de
la tercera parte de los pacientes con EB presenta neoplasia maligna. PATOGENIA: Normalmente toda lesión del epitelio epidermoide
(escamoso) cura mediante regeneración de células epidermoides. En el EB, por causas aún desconocidas, el epitelio epidermoide
lesionado es reemplazado por epitelio columnar ó cilíndrico pseudoestratificado. Inicialmente se postuló un origen congénito, pero
actualmente se acepta el EB como una condición adquirida, en la que una lesión intensa y sostenida sobre el epitelio del esófago distal,
generalmente debida a RGE, provoca el reemplazo del epitelio plano pluriestratificado normal por un epitelio columnar metaplásico,
más resistente al ácido. El origen de este nuevo epitelio serían células totipotenciales ubicadas en la capa basal o germinativa del
epitelio esofágico. Estudios de pHmetría de 24 hrs muestran que los pacientes con EB tienen mayor intensidad y frecuencia de
exposición al reflujo ácido que el resto de los pacientes con RGE, lo que pudiera relacionarse con presiones del esfínter esofágico
inferior más bajas y peristalsis esofágica menos activa. DIAGNOSTICO: No existe una clínica del esófago de Barrett, los síntomas que
presentan estos pacientes son los propios del reflujo gastroesofagico, o de sus complicaciones: Pirosis, regurgitaciones, epigastralgias,
dolor torácico retro-esternal que se irradia al cuello y brazos, asfixia nocturna, tos. La realidad de su diagnóstico depende
esencialmente de la endoscopia y de la biopsia, con histología. Biopsia se hace a nivel distal de la unión escamosa-columnar, y ella se
practica en el área Inferior a la línea z, que es la (unión escamo-columnar), El endoscopista reconoce el borde próximo de los pliegues
gástricos aun con la existencia de una Hernia Hiatal, es la marca endoscópica más segura para identificar la unión mucosa escamosa
columnar, además el llamado el epitelio intestinal, también debemos considerarlo como esófago de Barrett , por este motivo la nueva
definición de esófago de Barrett es: La presencia de mucosa columnar, con metaplasia intestinal en el esófago inferior, incluida la
mucosa que tapiza el esfínter esofágico inferior. La importancia de este cambio de definición que la metaplasia de Barrett, se ha
convertido en un criterio histológico más que endoscopio, ya que solo sería posible identificar la metaplasia intestinal, mediante el
estudio histológico, con esta nueva definición se puede distinguir dos tipos de esófago de Barrett: 1. El esófago de Barrett clásico de 3 ó
más cm. 2. El esófago de Barrett corto ó incluso ultracorto menos de 3cm. La mucosa escamosa, tiene una coloración blanquecina ó
rosa pálido en cambio la mucosa columnar y tiene una coloración rojiza. Seguimiento endoscópico-biopsia: En relación al riesgo de
malignización y a los factores de riesgo de desarrollar cáncer, se propone diferenciar tres grupos de pacientes con EB, para un
seguimiento diferenciado, de manera que la relación costo-beneficio sea adecuada; 1. Paciente de bajo riesgo: que incluiría aquellos
cuyo epitelio metaplásico sea de tipo cardial o fúndico y los marcadores tumorales sean negativos. En estos casos es suficiente con un
control endoscópico-biópsico cada 5 años, hasta los 70 años de edad en los que se suprime el seguimiento. 2. Pacientes de riesgo
medio: comprende a los fumadores y bebedores importantes, a los que tienen una extensión de la metaplasia igual o mayor de 5 cm, y
a aquellos cuyo epitelio metaplásico sea de tipo intestinal especializado. En este grupo, el control endoscópico-biópsico debe ser anual,
suprimiéndolo también a los 70 años de edad. 3. Pacientes de alto riesgo: son los que tienen displasia en el estudio histológico de las
biopsias obtenidas durante la primera exploración o alguno de los marcadores tumorales, distintos de la displasia, positivos. En este
caso debemos diferenciar dos subgrupos: a) Pacientes con displasia leve o algunos de los marcadores tumorales positivos; se hará
control endoscópico-biópsico cada 6 meses hasta que en dos exploraciones consecutivas no encontremos displasia ni otros marcadores
tumorales positivos, con lo cual se pasaría a control anual como en el grupo anterior. b) Pacientes con displasia severa, en los que se
repite la endoscopía-biopsia inmediatamente, por la posibilidad de coexistencia de cáncer. Y debe plantearse seriamente el tratamiento
quirúrgico u otras opciones terapéuticas si hay contraindicación a la cirugía. TRATAMIENTO: La esofagitis por reflujo es el primer
eslabón en la progresión de la enfermedad hacia el EB. Esta progresión sólo puede evitarse haciendo que el material refluido pierda su
agresividad o que no se produzca el reflujo. Medidas higieno-dietéticas comunes a todos los pacientes y el empleo de una medicación
(IBP, por tiempo prolongado y altas dosis) que ha mostrado ser muy eficaz. En teoría, la cirugía antirreflujo, si consigue evitarlo sería el
mejor método de tratamiento de la esofagitis y de profilaxis del EB. Sin embargo, la cirugía sólo se reserva para los pocos casos en los
que fracasa el tratamiento médico o en circunstancias individuales especiales. Hasta ahora, ni el tratamiento médico ni el quirúrgico del
RGE han demostrado en forma consistente una regresión significativa del epitelio metaplásico o disminución del riesgo de cáncer
esofágico, ni la terapia médica ni la quirúrgica evitan la necesidad del seguimiento endoscópico-biópsico para detectar precózmente
displasia o cáncer de la mucosa metaplásica. Tratamiento quirúrgico: Funduplicatura de Nissen; que consiste básicamente en el cierre
del hiato esofágico, la sección de vasos cortos para movilizar el fondo gástrico y la colocación de puntos que toman el fondo gástrico
anterior, la pared muscular el esófago y el fondo gástrico posterior. Las complicaciones de esta técnica son la disfagia, el desgarro de la
sutura con pérdida de la válvula, y la imposibilidad de eructar por hipercontinencia de la válvula. Para prevenir estas complicaciones
existen variantes de la técnica de Nissen. No son pocos los autores que proponen la Esofagectomía como único tratamiento con
posibilidades de curación, por supuesto asociada a alta morbi-mortalidad operatoria. Para pacientes con Barrett largo y con larga
expectativa de vida, buen desempeño funcional y capacidad física y psíquica de tolerar la cirugía, debe considerarse la extirpación
quirúrgica mediante la esofagectomía o la técnica de Merendino, como primera elección, en centros con experiencia y que acrediten
sus resultados de aceptablemente baja morbimortalidad. Para los casos que no deseen cirugía, o para aquellos con pobre reserva
funcional, las posibilidades son dos: tratarlos en un protocolo de ablación endoscópica o ser sometidos a vigilancia endoscópica-
histológica. La primera debe considerarse aún en etapa de investigación pues no se ha demostrado fehacientemente su rol terapéutico
en el largo plazo, no obstante sus innegables ventajas de buena tasa de eliminación del epitelio alterado en el corto y mediano plazo,
manteniendo inalterada la continuidad anatómica del tubo digestivo superior. La segunda ciertamente es la conducta menos agresiva y
probablemente de mayor atractivo para los pacientes, pero de más riesgo de progresión a cáncer y obliga en todo caso a varias
endoscopías cada año. Actualmente es considerada una opción aceptable para pacientes de menor expectativa de vida, con pobre
reserva funcional o de alto riesgo quirúrgico, que sean debidamente informados y cumpliéndose las condiciones de control
protocolizado y sistematizado.

CURSO ENARM CMN SIGLO XXI TEL: 36246001 Pharmed Solutions Institute PÁGINA 481
MANUAL DE TRABAJO DEL CURSO ENARM CMN SIGLO XXI
CASO CLINICO
Un hombre de 67 años de edad, acude por disfagia a alimentos sólidos y líquidos, tos de predominio nocturno, cuenta con antecedente
de ERGE con tratamiento irregular, agrega que ha perdido 10 kilos en los últimos tres meses, niega sangrado sus constantes vitales se
encuentran dentro de parámetros normales.

PREGUNTA
Cual es su conducta a seguir?

RESPUESTA
a.- Realizar pH-metria.
b.- Realizar manometría.
c.- Realizar serie gastroesofágica.
d.- Realizar endoscopia.

PREGUNTA
Se realizaron diversos estudios dentro de los cuales se encontra una endoscopia la cual mostró una lesión plana, de color rojizo en la
mitad de esófago. Esta lesión no se tiño con solución de Lugol. Cual es su impresión diagnostico en este momento?

RESPUESTA
a.- Adenocarcinoma.
b.- Esofagitis por reflujo.
c.- Acalasia.
d.- Esofago de barrett

PREGUNTA
Considerando su diagnostico, cual es el tratamiento mas apropiado para el caso?

RESPUESTA
a.- Dilatacion neumática.
b.- Funduplicatura de nissen.
c.- Esofagectomia.
d.- Terapia fotodinámica.

PREGUNTA
Luego del tratamiento indicado, 3 meses después el paciente presenta dolor severo, disfagia a liquidos con perdida de 5 kilogramos,
cual es la conducta a seguir?

RESPUESTA
a.- Dilatacion neumática.
b.- Funduplicatura de nissen.
c.- Esofagectomia.
d.- Terapia fotodinámica.

CASO CLINICO
Luego de realizarse una endoscopia una paciente de 41 años presenta incremento de los cuadros de reflujo gastroesofagico de
predominio nocturno, la endoscopia reporto lesiones displasicas en el tercio distal del esófago y mucosa gástrica despulida, la paciente
agrega disminución de peso corporal debido a que come poco por la sensación ardorosa a la deglución, existen antecedentes de cáncer
en la familia.

PREGUNTA
Cuál es el tratamiento que presenta mejor pronóstico.

RESPUESTA
a.- Fototerapia.
b.- Cirugía.
c.- Radioterapia.
d.- Quimioterapia.

PREGUNTA
Cual es la conducta si hay displasia leve?

RESPUESTA
a.- Endoscopia y biopsia.
b.- Endoscopia y biopsia confirmada.
c.- Endoscopia y biopsia seriada.
d.- Endoscopia sin biopsia.

CURSO ENARM CMN SIGLO XXI TEL: 36246001 Pharmed Solutions Institute PÁGINA 482
MANUAL DE TRABAJO DEL CURSO ENARM CMN SIGLO XXI
CASO CLINICO
Se trata de masculino de 41 años de edad el cual acude a consulta debido a que presenta pirosis, disfagia y regurgitación, tratado con
omeprazol por un año a dosis adecuada.

PREGUNTA
Complicaciones más frecuente que esperaría en este caso

a.- Ulcera de barett.


b.- Estenosis de barett
c.- Adherencia
d.- Cáncer de esofago

PREGUNTA
En paciente con esofagitis y esófago de barett, cual será la conducta a seguir mas adecuada?

RESPUESTA
a.- Reparación quirurgia y funduplicatura.
b.- Repetir y corroborar con biopsia.
c.- Corroborar con dos patólogos.
d.- Ablacion con fototerapia.

PREGUNTA
Con cual tipo de barret anatomica esta mas frecuente relacionado con presentacion de Ca?

RESPUESTA
a.- Barett corto.
b.- Barett largo.
c.- Barett bajo.
d.- Barett alto.

PREGUNTA
Cual es el objetivo del tratamiento medico del esófago con metaplasia columnal?

RESPUESTA
a.- Regresión de la metaplasia columnar.
b.- Retrasar la progresión.
c.- Disminuir la displasia columnar.
d.- Evitar el adeno carcinoma.

PREGUNTA
Cual es el tratamiento quirúrgico cuando la biopsia reporta metaplasia columnar?

RESPUESTA
a.- Fototerapia.
b.- Funduplicatura.
c.- Esofagectomia.
d.- Ablación.

CURSO ENARM CMN SIGLO XXI TEL: 36246001 Pharmed Solutions Institute PÁGINA 483
MANUAL DE TRABAJO DEL CURSO ENARM CMN SIGLO XXI
CANCER DE ESOFAGO. CIENCIAS BASICAS: Se origina en la capa más interna (la mucosa) y crece hacia fuera (a través de la submucosa y
la capa muscular). Debido a que dos tipos de células pueden cubrir el esófago, existen dos tipos principales: el carcinoma de células
escamosas y el adenocarcinoma. El esófago está normalmente cubierto con células escamosas (carcinoma de células escamosas), el
cual puede ocurrir en cualquier lugar a lo largo del esófago. En cierto tiempo, el carcinoma de células escamosas fue el tipo más común
de cáncer de esófago en los Estados Unidos. En la actualidad, estos cánceres constituyen menos de la mitad de los cánceres esofágicos
en este país. Los cánceres que se originan de células glandulares se llaman adenocarcinomas. Este tipo de célula no es normalmente
parte del revestimiento interno del esófago. Antes de que se pueda desarrollar un adenocarcinoma, las células glandulares tienen que
reemplazar un área de las células escamosas, como en el caso del esófago de Barrett. Esto ocurre principalmente en la parte inferior del
esófago, donde ocurren la mayoría de los adenocarcinomas. Los cánceres que se originan en el área donde el esófago se une al
estómago (la unión GE), lo que incluye aproximadamente las primeras 2 pulgadas del estómago (llamada cardias), suelen comportarse
como cánceres de esófago de manera que son agrupados como cánceres esofágicos. Existen tumores malignos esofágicos no epiteliales
(no carcinomas), de presentación infrecuente, entre los que podemos destacar el leiomiosarcoma (derivado de la musculatura del
esófago). Clasificados como tumores benignos esofágicos se encuentran, entre otros, el papiloma escamoso, el adenoma esofágico, y
los pólipos. SALUD PUBLICA: El cáncer de esófago es el cuarto tumor más frecuente del aparato digestivo (por detrás del cáncer
colorrectal, gástrico y hepático) y se sitúa entre los diez cánceres más frecuentes en el mundo. Su incidencia es mayor en areas de alta
frecuencia Asia y África Central y del Sur (más de 100 casos por 100.000 habitantes/año). Es más frecuente en el hombre que en la
mujer, pudiendo oscilar la relación entre 3 y 10 hombres por cada mujer. La edad habitual de presentación es entre los 55 y los 70 años,
siendo infrecuentes los casos en personas por debajo de los 40 años. En los últimos años la incidencia del adenocarcinoma del tercio
distal del esófago y de la unión GE se ha incrementado de forma paralela a la ERGE, especialmente en personas con alto índice de masa
corporal. PATOGENIA: Los factores de riesgo asociados al desarrollo de carcinoma escamoso de esófago son: tabaco, alcohol, extracción
social baja, dieta: déficit nutritivos, hipovitaminosis, ingesta de alimentos con alto contenido de nitrosaminas, acalasia, esófago de
Barrett, lesiones por cáusticos, Síndrome de Plummer-Vinson, neoplasias de cabeza y cuello, cáncer de mama cuyo tratamiento haya
comprendido la radioterapia, ingesta de bebidas calientes. Los factores de riesgo para adenocarcinoma de esófago son: RGE, obesidad,
infección por H. pylori, sexo masculino, estrato social bajo, tabaco, alimentos con alto contenido de nitrosaminas. El adenocarcinoma
de esófago distal y de la unión gastroesofágica surge típicamente en un epitelio con metaplasia, circunstancia conocida como esófago
de Barret. Esta condición premaligna se caracteriza por la presencia de un epitelio columnar que abarca a tres o más centímetros del
esófago tubular distal, ya sea en presencia o en ausencia de hernia de hiato. Aproximadamente el 60% de los casos de adenocarcinoma
de esófago distal o de la unión gastroesofágica presentan evidencia de esófago de Barret. DIAGNOSTICO: En las etapas iniciales de la
enfermedad, el cáncer de esófago habitualmente es asintomático. Los síntomas más frecuentemente asociados al cáncer de esófago
son: Disfagia (primero, solidos luego a líquidos), pérdida de peso, dolor retroesternal, síntomas respiratorios (tos e infecciones
respiratorias), otros síntomas: sangrado, disfonía (ronquera) e hipo, hemorragia intensa o parálisis de cuerdas vocales. Es importante
resaltar que estos síntomas también pueden estar causados por otras enfermedades del esófago, o por otras causas menos serias. El
diagnostico se hace con endoscopia y toma de biopsia, biopsias tanto de la mucosa del esófago como de ganglios linfáticos regionales
sospechosa. TAC de tórax y abdomen: permite conocer, basándose en su tamaño, la extensión local del tumor, la posible afectación de
ganglios cercanos o a distancia, y la extensión a otros órganos (pulmón, hígado.). La estadificación debe incluir TAC de tórax y abdomen
y en candidatos a tratamiento quirúrgico se debe agregar esofagograma y ultrasonido transendoscopico, para definir con mayor
precisión T y N. Si se tiene disponible la tomografía por emisión de positrones (PET), se puede utilizar con el fin de detectar metástasis
a distancia ocultas. Clasificación simplificada: Estadio 0: también denominado carcinoma in situ. El cáncer se localiza sólo en la parte
más superficial de la mucosa esofágica, sin sobrepasarla. Estadio I: Tumores bien diferenciados, sin infiltración ganglionar locorregional
por metástasis, que alcanzan como máximo la capa adventicia. Estadio II: El tumor no alcanza la capa adventicia, asociando un máximo
de dos ganglios linfáticos locorregionales infiltrados por metástasis; ó alcanza sin sobrepasar la capa adventicia sin presentar ganglios
linfáticos con metástasis. Estadio III: El tumor afecta a la capa adventicia, asociando un máximo de 2 ganglios linfáticos locorregionales
infiltrados por metástasis; ó presenta afectación ganglionar con más de 2 ganglios infiltrados. Estadio IV: El tumor se ha extendido a
distancia, a otros órganos del cuerpo. TRATAMIENTO: Existen tres opciones fundamentales de tratamiento para el cáncer de esófago:
Cirugía: Su objetivo es extirpar el tumor con márgenes libres, es decir, sin dejar enfermedad residual. Se considera indicada en estadios
iniciales de la enfermedad (resecable). Es el tratamiento más frecuentemente utilizado para el cáncer de esófago. El tipo de cirugía
varía en función del tamaño, localización y extensión de la enfermedad. La esofagectomía es la extirpación quirúrgica de parte o la
totalidad del esófago, conectando la parte sana restante al estómago, para así permitir la alimentación del paciente. Es factible la
utilización de parte del intestino para realizar la conexión. En la esofagectomía se incluye la extirpación de los ganglios linfáticos
cercanos al esófago, para su estudio al microscopio, con el fin de conocer el grado de extensión de la enfermedad. En ocasiones puede
ser necesaria la realización de una gastrostomía de alimentación, ante imposibilidad para la ingesta de alimentos (condicionada por una
estrechez infranqueable del esófago). Esta técnica puede realizarse vía endoscópica o de manera quirúrgica. Radioterapia: se puede
emplear como tratamiento único ó formando parte de una estrategia multidisciplinar (combinada con la cirugía y/o la quimioterapia).
Se utiliza con intención curativa, en estadios iniciales de la enfermedad, ó con intención paliativa, en estadios avanzados, para aliviar
síntomas tales como el dolor y la disfagia. Radioterapia externa: consiste en la utilización de una máquina fuera del cuerpo para enviar
la radiación al área concreta que se desea tratar. Radioterapia interna: en ésta, una sustancia radioactiva se coloca (mediante un
sistema sellado) muy cerca o dentro del tumor, permitiendo así alcanzar más dosis de radiación con menores efectos secundarios sobre
los tejidos sanos. En el cáncer de esófago generalmente se utiliza para el tratamiento de la disfagia. Quimioterapia: Consiste en la
introducción de un fármaco al torrente sanguíneo (quimioterapia sistémica) para eliminar las células cancerosas de todo el cuerpo
(dentro y fuera del esófago). La quimioterapia se puede emplear como tratamiento único o formando parte de una estrategia
multidisciplinar (combinada con la cirugía y/o la radioterapia). Su utilización de forma aislada, previa a la cirugía, se considera un
tratamiento en investigación. Otros tratamientos con posible indicación en el cáncer de esófago son la terapia láser (destrucción del
tumor con láser), y la colocación de prótesis esofágicas (stent), consistente en la implantación endoscópica de un tubo expandible. Esta
última técnica se utiliza para resolver la estrechez esofágica provocada por el tumor y permitir así la adecuada alimentación del
paciente. Tratamiento. Estadios Localizados (I,II,III), el tratamiento fundamental es la cirugía. En la mayoría de pacientes con cáncer de
esófago estadio I la cirugía exclusiva es el tratamiento estándar, salvo en casos con diagnóstico en etapas muy tempranas de la

CURSO ENARM CMN SIGLO XXI TEL: 36246001 Pharmed Solutions Institute PÁGINA 484
MANUAL DE TRABAJO DEL CURSO ENARM CMN SIGLO XXI
enfermedad en los que se puede plantear la resección endoscópica. No obstante, cabe destacar que el diagnóstico de la enfermedad en
etapas tan precoces es un hecho infrecuente. En los casos más habituales de diagnóstico de la enfermedad, en estadios II y III, ante los
no óptimos resultados de tratamiento quirúrgico aislado, ha existido un interés creciente por utilizar un tratamiento multidisciplinar. En
muchas ocasiones se necesita de la combinación de la utilización de quimioterapia y/o radioterapia, con el objetivo de mejorar los
resultados obtenidos solamente con la cirugía. La mayoría de estos pacientes son tratados inicialmente con quimioterapia o con la
combinación de la quimioterapia y radioterapia previamente al tratamiento quirúrgico. En ocasiones es posible realizar una
reevaluación precoz al tratamiento con quimioterapia para así seleccionar a los pacientes que más se benefician de esta estrategia de
tratamiento previo a la cirugía. Por estadios avanzados de cáncer de esófago entendemos el estadio IV. El objetivo del tratamiento es la
paliación, tratando igualmente de prolongar la supervivencia del paciente. El pilar es la quimioterapia, pudiendo realizarse dentro de
ensayos clínicos. Con el objeto de mejorar la sintomatología del paciente (dolor, imposibilidad para la ingesta de alimentos...) puede
estar indicado el uso de radioterapia externa, radioterapia interna, terapia láser, prótesis esofágicas, y cirugía paliativa.

CASO CLINICO
Mujer de 51 años, con antecedente de tabaquismo y asma. Consultó por disfagia, náuseas, vómitos y baja de peso. La endoscopia
mostró una lesión polipoidea, mamelonada y ulcerada en el tercio medio del esófago que comprometía casi todo el lumen esofágico. El
estudio anatomopatológico de la lesión mostró un tumor sólido, infiltrante, con características morfológicas e inmunohistoquímicas
compatibles con un melanoma maligno. La tomografía computada de tórax mostró una lesión tumoral, con crecimiento intraluminal, en
el tercio distal del esófago. La lesión comprometía el cardias y comprimía la aurícula izquierda. No hubo evidencia de compromiso de
órganos adyacentes ni metástasis. La tomografía computada de abdomen y pelvis fue normal.

PREGUNTA
Cual es el pronostico del caso?

RESPUESTA
a.- 6 a 12 meses de vida.
b.- 12 a 18 meses de vida.
c.- 18 a 24 meses de vida.
d.- 24 a 30 meses de vida.

CASO CLINICO
Hombre de 59 años, sin antecedentes mórbidos. Consultó por disfagia asociada a baja de peso. La endoscopia digestiva alta demostró
una lesión polipoidea de 5 cm, pigmentada, en el tercio distal del esófago. El estudio anatomopatológico de la biopsia endoscópica
diagnosticó un tumor maligno indiferenciado. El examen físico y los exámenes de laboratorio fueron normales.

PREGUNTA
Cual es el tipo mas probable de neoplasia que presente el caso?

RESPUESTA
a.- Leiomioma.
b.- Escamoso mucoide.
c.- Adenocarcinoma.
d.- Melanoma.

CASO CLINICO
Hombre 58 años con disfagia. Comienza hace 2 meses con disfagia progresiva sin remisiones, primero para sólidos y luego para
semisólidos. Actualmente ingiere líquidos y papillas blandas. Sensación de detención del bolo alimenticio en región esternal baja. Desde
hace meses sialorrea sobretodo nocturna. En algunas oportunidades ha tenido regurgitaciones. No presentó dolor torácico de ningún
tipo. Adelgazamiento de 10kg en los últimos meses. No astenia, ni anorexia. AP: Fumador intenso de 2 cajillas/día. Bronquítico crónico,
etilista y enolista moderado. Se aporta una fibrogastroscopia que informa lesión vegetante e infiltrante, en el esófago, cuya biposia
informó carcinoma epidermoide.

PREGUNTA
Radiología del tórax frente y perfil, valorando elementos de remodelación torácica (horizontalización costal, aplanamiento
diafragmático) y rarefacción pulmonar, a su vez descartará la presencia de metástasis pulmonares y/u óseas poco frecuentes siendo
este estudio poco sensible para su detección. Cual es el tratamiento mas adecuado para el caso?

RESPUESTA
a.- Quimioterapia.
b.- Radioterapia.
c.- Reseccion quirúrgica.
d.- Protesis expansible

CURSO ENARM CMN SIGLO XXI TEL: 36246001 Pharmed Solutions Institute PÁGINA 485
MANUAL DE TRABAJO DEL CURSO ENARM CMN SIGLO XXI
VARICES ESOFAGICAS. CIENCIAS BASICAS: Son vasos colaterales porto sistémicos, es decir, canales vasculares que unen la circulación
venosa porta y la sistémica. Se forman como consecuencia de la hipertensión portal (una complicación progresiva de la cirrosis),
preferencialmente en la submucosa del esófago inferior. La ruptura y el sangrado de las várices esofágicas son complicaciones mayores
de la hipertensión portal (HTP) y se acompañan de una alta tasa de mortalidad. SALUD PUBLICA: El sangrado varicoso representa 10–
30% de todos los casos de sangrado gastrointestinal alto. Es una complicación frecuente de hipertensión portal 30-50%. Elevada
mortalidad hospitalaria 20-50%. Riesgo elevado de resangrado, 30% en primera semana y 50% durante el primer año. 15-20% de
pacientes cirróticos que sufren una hemorragia por varices esofagogastricas fallecen en las 6 sem siguientes. PATOGENIA: La cirrosis, el
estadio terminal de la hepatopatía crónica, es la causa más común de HTP. La presión venosa portal (P) es el producto de la resistencia
vascular (R) y el flujo sanguíneo (Q) en el lecho de la vena porta (Las varices esofágicas se rompen cuando la tensión ejercida sobre su
Las VE se rompen cuando la tensión ejercida sobre su pared es excesiva y sobrepasa un valor umbral pared es excesiva y sobrepasa un
valor umbral). En la cirrosis hay un aumento tanto de la resistencia vascular intrahepática como del flujo porta. La HTP lleva a la
formación de colaterales porto-sistémicas. Sin embargo, debido a su mayor resistencia y mayor flujo aferente venoso porta, estas
colaterales no logran disminuir la hipertensión. La mejor manera de evaluar la hipertensión portal (indirectamente) es utilizando la
medición de la presión venosa hepática enclavada (PVHC). Para que se formen las várices se necesita una diferencia de presión entre la
circulación portal y sistémica (gradiente de la presión venosa, GPVH) de 10–12 mmHg (aunque no es suficiente). La GPVH normal es 3–5
mmHg. Cuando la HTP es clínicamente significativa, hay formación de varices esofágicas con una presión >10mmHg, y hemorragia de
varices esofágicas >12mmHg. Correlación entre la presencia de várices y severidad de la hepatopatía: Pacientes Child–Pugh A: 40%
tienen várices. Pacientes Child–Pugh C: 85% tienen várices. Algunos
pacientes pueden presentar várices y hemorragia en las primeras
etapas de la enfermedad, aún en ausencia de cirrosis. Los pacientes
con hepatitis C y fibrosis en puente: 16% tienen várices esofágicas. La
presencia de várices gastroesofágicas se correlaciona con la gravedad
de la enfermedad hepática. La severidad de la cirrosis puede
clasificarse utilizando el sistema de clasificación de Child–Pugh.
DIAGNOSTICO: La esofagogastroduodenoscopía es el patrón oro para
el diagnóstico de várices esofágicas. Estándar de oro para diagnóstico:
Várices esofágicas Soehendra: GRADO I: Repleción leve, diámetro < 2
mm, apenas se eleva del esófago relajado, se acentúan en posición
cabeza abajo. GRADO II: Repleción moderada, curso serpenteante,
diámetro de 3-4 mm, restringidas a la mitad inferior del esófago.
GRADO III: Repleción completa, tensa, con diámetro > 4 mm, paredes
finas, fenómeno de várice sobre várice, pasan al fondo gástrico.
GRADO IV: Repleción completa, tensa, ocupan todo el esófago, a menudo combinadas con várices gástricas o duodenales. Si no se
dispone de patrón oro, otros posibles pasos diagnósticos serían la ecografía con Doppler de la circulación sanguínea (no la ecografía
endoscópica). Si bien es una mala segunda opción, ciertamente puede demostrar la presencia de várices. Entre otras alternativa, se
puede utilizar la radiografía baritada (con ingestión de bario) del esófago y estómago, y angiografía y manometría de la vena porta. Es
importante valorar la ubicación (esófago u estómago) y el tamaño de las várices, los signos de sangrado inminente, que puede ser un
primer sangrado agudo, o recurrente, y (si corresponde) debe considerarse la causa y la gravedad de la hepatopatía. TRATAMIENTO:
Sangrado por varices esofágicas: Actuaciones iniciales, catéteres venosos: 1-2 vías periféricas gruesas. Reserva de concentrados de
hematíes. Analítica con pruebas de coagulación. Reposición de la volemia. En hemorragia grave, vía central y sonda vesical. Reposición
cuidadosa de la volemia (Hcto 21-27%). Prevención de las infecciones: Norfloxacina VO 400mg cada 12hs (7 días). Ceftriaxona IV 1g día
(7 días) (Child C). Tratamiento de la encefalopatía (Lactulosa). Monitoreo de la función renal (40 ml/hora). Intubación endotraqueal
(encefalopatía). Terapia farmacológica: Vasoconstrictores esplácnicos; vasopresina, somatostatina, β-bloqueantes no cardioselectivos
(propanolo, nanodol). Terlipresina Primera opción; derivado sintético de la Vasopresina pero con menos efectos secundarios,
administración en bolos, único fármaco que ha demostrado disminución de la mortalidad. Somatostatina, segunda opción es eficaz para
detener la hemorragia (por lo menos transitoriamente) en hasta 80% de los pacientes. La somatostatina puede ser superior a su
análogo octreotida. A pesar de emplearse en dosis adecuadas, alrededor de 30% de los pacientes no responden a β-bloqueantes con
una reducción en el gradiente de presión venosa hepática (GPVH). Además, los β-bloqueantes pueden provocar efectos colaterales
tales como fatiga e impotencia, que podrían disminuir la adherencia al tratamiento (especialmente con los varones jóvenes), o pueden
estar contraindicados los β-bloqueantes por otras razones. Venodilatadores: Nitratos solos no están recomendados. El 5-MNI reduce la
presión porta, pero su uso en los pacientes cirróticos está limitado por sus efectos vasodilatadores. La terapia de combinación lleva a un
efecto sinérgico en reducir la presión porta. Se ha demostrado que la combinación de 5-MNI con β-bloqueantes no selectivos tiene
efectos aditivos al reducir la presión porta y es particularmente eficaz en pacientes que no responden a la terapia inicial con β-
bloqueante solo. Sin embargo, estos efectos beneficiosos pueden verse contrarrestados por los posibles efectos deletéreos sobre la
función renal y la mortalidad a largo plazo, especialmente en los pacientes mayores de 50 años. Por lo tanto no se recomienda el uso de
rutina del tratamiento combinado. Terapias locales: Escleroterapia o ligadura endoscópica de las várices (EVL). No tiene efecto sobre el
flujo o la resistencia porta. Terapia de derivación. Quirúrgica o radiológica (derivación portosistémica intrahepática transyugular, TIPS).
Reduce la presión porta. La escleroterapia endoscópica y la ligadura varicosa logran interrumpir el sangrado en hasta 90% de los
pacientes. La ligadura endoscópica con banda es una escleroterapia eficaz, pero se acompaña de menos efectos colaterales. Sin
embargo esta técnica puede ser más difícil de aplicar que la escleroterapia en los pacientes con sangrado activo severo. La derivación
TIPS es una buena alternativa cuando fracasan el tratamiento endoscópico y la farmacoterapia. El uso de taponamiento con balón está
disminuyendo ya que existe un alto riesgo de resangrado después de desinflar el balón y hay un riesgo de complicaciones mayores. Sin
embargo, el taponamiento con balón logra detener la hemorragia por lo menos temporalmente en la mayoría de los casos, y puede ser
utilizado en regiones del mundo donde no se disponga fácilmente de EGD y TIPS. Puede ayudar a estabilizar al paciente para ganar
tiempo y acceder a EGD y/o TIPS más adelante. La hemorragia aguda de las várices a menudo se acompaña de infección bacteriana

CURSO ENARM CMN SIGLO XXI TEL: 36246001 Pharmed Solutions Institute PÁGINA 486
MANUAL DE TRABAJO DEL CURSO ENARM CMN SIGLO XXI
debido a translocación intestinal y trastornos de la motilidad. La antibióticoterapia profiláctica ha demostrado aumentar la tasa de
sobrevida.

CASO CLINICO
Masculino de 43 años de edad que consulta por distención abdominal y episodios de sincope. El paciente tiene antecedentes de cirrosis
hepática de causa alcohólica (estadio C de Child-Pugh), hipertensión portal con multiples episodios de sangrado por varices esofágicas y
síndrome ascítico-edematoso. El paciente continuaba con el consumo de alcohol sin cumplir la medicación de espironolactona y
propanolol. Al Ingreso se encontraba vigil y orientado con TA de 85/50 mmHg y FC 100 lpm. FR 18.

PREGUNTA
Cual es la conducta a seguir en este momento?

RESPUESTA
a.- Reposición de liquidos y hemoderivados.
b.- Realizar TAC para buscar sangrado.
c.- Conducta expectante y medidas antiamonio.
d.- Preparar para laparatomia exploratoria.

PREGUNTA
Se realizan estudios y medidas de rutina para la patología de base, asi como búsqueda de hemorragia, por TAC se observa sangrado
peritoneal, y se prepara al paciente para cirugía, cual de las siguientes causas de hemoperitoneo es mas frecuente?

RESPUESTA
a.- Rotura de várices esofágicas.
b.- Rotura de canales linfáticos
c.- Carcinoma hepatocelular
d.- Metástasis hepáticas

PREGUNTA
Cual es la mortalidad del paciente en caso de recurrir a TIPS considerando particularmente su estadio C de Child-Pugh?

RESPUESTA
a.- 100 %.
b.- 90 %.
c.- 80 %.
d.- 70 %.

PREGUNTA
Cual de la siguientes medidas es mas apropiada para reducir el riesgo de sangrado por varices esofágicas en el caso presentado?

RESPUESTA
a.- Espironolactona.
b.- Propanolol.
c.- Propanolo e isosorbide.
d.- Espironolactona y propanolol.

CASO CLINICO
Paciente varón de 58 años de edad fumador de 40 cigarrillos al día desde los 15 años, bebedor de unos 70 gramos etanol/ día. Presenta
de forma súbita 3 vómitos muy abundantes de color rojo oscuro con restos semejantes a posos de café. Exploración física: TA: 86/60 y
frecuencia cardiaca 140 lpm. Consciente, orientado con intensa palidez cutáneo mucosa, sudoración y frialdad cutánea. Eupneico.
Desnutrido. Telangiectasias malares e hipertrofia parotídea. AP: mvc, AC ritmico a 140 lpm sin soplos ni extratonos. Abdomen: RHA
conservados no se palpan masas hepatoesplenomegalia de tres traveses de dedo, circulación colateral periumbilical, no signos de
irritación peritoneal. Extremidades: pulsos periféricos presentes, simétricos y débiles, eritema palmar. Neurológico normal sin flapping
ni signos de encefalopatía. Hemograma: hematocrito 28%, hemoglobina 10.3 g/dl, VCM 104 fl, leucocitos 10.3 mil/mcL, plaquetas 100
mil/mcL. Perfil hepático: GPT 60 UI/L, GOT 93 UI/L, fosfatasa alcalina 183 UI/L, gamma GT 200 UI/L. Estudio de coagulación: normal.
Endoscopia digestiva: varices esofágicas con signos de sangrado reciente, resto hemáticos en estómago con mucosa gástrica normal,
duodeno hasta tercera porción normal.

PREGUNTA
Cual es la conducta a seguir mas adecuada?

RESPUESTA
a.- Realizar escleroterapia.
b.- Restablecimiento de liquidos.
c.- Aminas vasoactivas.
d.- Colocacion de sonda de balones.

CURSO ENARM CMN SIGLO XXI TEL: 36246001 Pharmed Solutions Institute PÁGINA 487
MANUAL DE TRABAJO DEL CURSO ENARM CMN SIGLO XXI
CASO CLINICO
Acude a urgencias paciente de 61 años de edad la cual refiere dolor ardoroso en la región media del tórax de predominio anterior, el
dolor ha incrementado en las últimas semanas, refiere vomito en tres ocasiones con trazas de sangre. Tiene antecedentes de diabetes y
osteoartritis bajo tratamiento, en últimas fechas han incrementado el dolor de sus manos por lo que aumento su dosis de
medicamente.

PREGUNTA
Cuál es el manejo farmacológico más adecuado.

RESPUESTA
a.- Ranitidina.
b.- Omeprazol.
c.- Sales de Bismuto.
d.- Hidroxido de aluminio y magnesio.

CASO CLINICO
Se trata de paciente masculino de 45 años de edad, alcoholismo desde hace 20 años el cual refiere vomito con sangre en 3 ocasiones
posterior a ingestión de alcohol durante 5 días, a la exploración física se observa diaforético, ansioso y desorientado, durante su
estancia presenta dos ocasiones más vomito con trazas de sangre, sus constantes vitales son TA 110/70 mmHg, FC 87, FR 19. 150,000
Plaquetas

PREGUNTA
¿Cuál es la conducta más adecuada para controlar el sangrado en este caso?.

RESPUESTA
a.- Colocación de sonda de balones.
b.- Escleroterapia.
c.- Vitamina K.
d.- Inhibidores de Bomba H+

CURSO ENARM CMN SIGLO XXI TEL: 36246001 Pharmed Solutions Institute PÁGINA 488
MANUAL DE TRABAJO DEL CURSO ENARM CMN SIGLO XXI
RUPTURA ESOFAGICA. CIENCIAS BASICAS: Patología relativamente rara/grave, que sin tratamiento puede progresar a mediastinitis y
shock séptico. Causas; traumática, iatrogenia, cuerpos extraños, agentes corrosivos, trauma torácico, espontanea, Síndrome de
Mallory-Weiss, hematoma intramural, Síndrome de Boerhaave, neoplasias, infecciosa. DIAGNOSTICO: Representan un difícil reto
diagnóstico, originando frecuentes retrasos en su tratamiento. Manifestaciones clínicas clásicas son: a) Vómitos. b) Dolor retroesternal.
c) Enfisema mediastínico (Tríada de Macler). Signos clínicos sutiles e inespecíficos; hipotensión, sepsis, fiebre. SINDROME DE
MALLORY-WEISS; Este síndrome se caracteriza por hemorragia digestiva superior después del vomito repetido o esfuerzos fuertes y
prolongados para vomitar o toser. Aproximadamente 4 casos por cada 100.000 personas. Más a los hombres que a las mujeres y puede
aparecer a cualquier edad. 5% de las hemorragias de la parte alta del tracto gastrointestinal. Suele ocurrir en varones con historia de
consumo de alcohol y se presenta clínicamente como vómitos repetidos seguidos de hematemesis. Desórdenes alimenticios y en
algunas evidencias se demuestran la presencia de hernia de hiato como una condición pre-disponente. Puede estar asociado a la
ingesta repentina de salicilatos. El mecanismo es similar a la perforación esofágica espontanea, o sea el aumento súbito en la presión
intrabdomional contra la glotis cerrada en un paciente con hernia hiatal. Los desgarros de Mallory-Weiss se caracterizan por
hemorragia arterial que puede ser masiva. Puede aparecer con cierta frecuencia en el punto de unión entre el esófago y el
estómago. Para establecer el diagnóstico es preciso mantener un alto índice de sospecha en un paciente que presenta hemorragia
digestiva alta después de vómito y arqueo prolongado. La endoscopia confirma el diagnostico al identificar una o más fisuras
longitudinales en la mucosa del estómago herniado. En casi todos los pacientes la hemorragia cede en forma espontánea (24-48hrs)
con tratamiento médico. Cicatrización en aproximadamente 10 días. Solo en ocasiones se requiere la operación para detener la pérdida
de sangre. El procedimiento consiste en laparotomía y gastrostomía alta con sutura del desgarro lineal. SÍNDROME DE BOERHAAVE:
Consiste en una rotura transmural completa del esófago, generalmente en el lado izquierdo a unos 2-3 cms por encima de la unión
gastroesofágica, con salida de la sangre hacia el mediastino. Afecta con más frecuencia a hombres (60-78%) y se relaciona con vómitos
violentos y consumo de alcohol. Clínicamente se presenta con un cuadro dramático de dolor, vómitos, neumotórax, enfisema
subcutáneo y shock. No suele haber hematemesis (a diferencia del Mallory-Weiss). Un signo radiológico clásico es la “V de Naclerio”.
Enfisema mediastínico lineal en ángulo costovertebral izq (entre aorta inferior y diafragma). A menudo existe derrame pleural de
predominio izquierdo.

CASO CLINICO
Se trata de masculino de 34 años de edad el cual se encuentra en urgencias traído por ambulancia ya que en su trabajo presento
vomito con sangre fresca y abundante en más de 4 ocasiones, a la exploración física se encuentra diaforético, pálido, con pulsos
disminuidos, taquicardico e hipotenso, se estabiliza, su hematocrito es de 41 % y plaquetas de 45,000.

PREGUNTA
Cuál es el tratamiento adecuado para mejorar el pronóstico del paciente.

RESPUESTA
a.- Endoscopia y escleroterapia.
b.- Sonda de balones.
c.- Transfusión sanguínea.
d.- Desmopresina y Inhibidor de H+.

CASO CLINICO
Se encuentra en urgencias por sangrado de tubo digestivo alto, refiere familiar que encontró al paciente inconsciente en el baño,
refiere que también presento dos evacuaciones fétidas abundantes mas vomito con sangre fresca, agrega que es alcohólico desde hace
25 años, no ha recibido tratamiento previo hasta la fecha.

PREGUNTA
Cuál es la complicación asociada que presenta peor pronóstico.

RESPUESTA
a.- Trastorno renal asociado.
b.- Trastorno plaquetario secundario.
c.- Hipertensión portal.
d.- Estado de choque.

CURSO ENARM CMN SIGLO XXI TEL: 36246001 Pharmed Solutions Institute PÁGINA 489
MANUAL DE TRABAJO DEL CURSO ENARM CMN SIGLO XXI
HERNIA HIATAL. CIENCICAS BASICAS: Es el prolapso del estómago proximal hacia el tórax a través del hiato esofágico del diafragma. La
hernia hiatal es una condición esencialmente adquirida que no sólo es la más frecuente de las hernias diafragmáticas, sino una de las
anormalidades más frecuentes que afectan el TGI superior. El hiato esofágico del diafragma se encuentra localizado a la izquierda de la
línea media a nivel de la décima vértebra torácica; es un anillo musculotendinoso compuesto por fibras de los pilares derecho e
izquierdo del diafragma, que provienen de la cara anterior y discos intervertebrales de las cuatro primeras vértebras lumbares, rodean
la aorta y el esófago y se insertan en el centro tendinoso del diafragma. Aunque existen variaciones anatómicas, se ha encontrado que
en más del 80% de los casos, el hiato esofágico se forma principalmente de fibras del pilar derecho del diafragma. CLASIFICACION: Tipo
I o hernia hiatal por deslizamiento (axial) donde se presenta desplazamiento superior de la unión esófago-gástrica hacia el mediastino
posterior. Se denomina hernia “por deslizamiento” ya que tiene un saco de peritoneo parietal parcial, cuya pared posterior está
formada por el estómago. Tipo II o hernia paraesofágica caracterizado por desplazamiento superior del fondo gástrico, anterior y lateral
al esófago, con la unión esófago-gástrica localizada en su posición intraabdominal normal. Tipo III o mixta donde hay desplazamiento
superior tanto de la unión esófago -gástrica como del fondo gástrico. El 85-90% de las hernias hiatales son de tipo I, en tanto que las
hernias paraesofágicas puras son encontradas muy infrecuentemente. SALUD PUBLICA: La incidencia de hernia hiatal se estima en 5
por 1000 en la población general, aunque una verdadera incidencia es difícil de determinar debido a que un gran número de pacientes
son asintomáticos. La edad de presentación más frecuente es entre 4ª a 6ª década de la vida y no existe diferencia en cuanto al sexo,
aunque las hernias paraesofágicas son más frecuentes en mujeres. DIAGNOSTICO: Cuando se hacen manifiestas, la sintomatología está
dada básicamente por pirosis, regurgitación y dolor retroesternal, síntomas clásicos del RGE que es la manifestación más significante
en los pacientes con hernia hiatal. Puede presentarse disfagia generalmente asociada a esofagitis, estenosis péptica, anillo de Schatzki
(pliegue mucoso en el esófago distal, dentro de los 3 mm proximal a la unión escamocolumnar, que siempre está asociada con hernia
hiatal y constituye una causa de disfagia en estos pacientes) o por la compresión de los pilares diafragmática en la porción herniada del
estómago. Las hernias paraesofágicas en general son asintomáticas, incluso cuando alcanzan grandes tamaños. Grandes herniaciones
pueden producir dolor retroesternal o disnea por disminución de la reserva respiratoria. Se puede presentar disfagia secundaria a la
compresión esofágica por la hernia o a la rotación de la UEG dentro del saco herniario. Síntomas de RGE se presentan también en un
alto porcentaje. En pacientes con ERGE documentado, se ha encontrado además una relación directa entre el tamaño de la hernia
hiatal y el grado de disfunción del EEI, la duración de los episodios de reflujo, el aclaramiento ácido y por consiguiente con la severidad
de la esofagitis. La función como esfínter del diafragma está dada principalmente durante la inspiración y situaciones de estrés
dinámico, como durante la deglución y en incrementos súbitos de la presión intraabdominal (tos), de tal forma que los episodios de
reflujo relacionados a incremento de la presión intraabdominal son más probables que ocurran en pacientes con hernia hiatal.
Sangrado oculto o moderado puede encontrarse hasta en un tercio de los pacientes con hernia hiatal por deslizamiento sintomática,
siendo el sangrado masivo poco frecuente. El volvulus gástrico puede ser organoaxial cuando el estómago rota a lo largo de su eje
longitudinal o mesoentero axial cuando rota sobre el eje que une la curvatura menor con la mayor. La laxitud o ausencia de los
ligamentos de fijación gástrica son la causa primaria del volvulus gástrico. Al incrementarse la rotación del estómago se produce
compromiso del flujo sanguíneo y del retorno venoso del estómago herniado, llevando a necrosis, perforación, sepsis e incluso la
muerte. Radiografía de tórax se puede evidenciar una masa dependiente de tejidos blandos en el mediastino posterior, con nivel
hidroaéreo en el caso de grandes hernias. Los estudios con medio de contraste baritados son más exactos si se logra definir la relación
del hiato esofágico del diafragma con la UEG. La presencia de una indentación por encima del diafragma, lo que usualmente se
considera como el sitio de transición entre el cardias gástrico y el vestíbulo esofágico, implica la existencia de una hernia hiatal. En el
caso de hernias paraesofágicas, los estudios con medio de contraste muestran la porción del fondo gástrico situado por encima del
diafragma con la UEG ubicada en posición normal a nivel del diafragma; adicionalmente puede demostrarse la presencia de un volvulus
gástrico. Endoscopia: Normalmente la unión de la mucosa escamocolumnar (línea Z), que corresponde aproximadamente a la
localización de la UEG, se encuentra a menos de 2 cm por encima del hiato diafragmático, de tal forma que una distancia mayor entre
estas estructuras es consistente con la presencia de una hernia hiatal. La posición del hiato diafragmático puede hacerse más evidente
durante una inspiración profunda. Otros estudios como la manometría y pHmetría, no son útiles para el diagnóstico de hernia hiatal,
pero son de utilidad en el estudio del RGE asociado. La manometría es fundamental para determinar la localización, tamaño y presión
del EEI, como también para descartar trastornos de la motilidad esofágica previa al tratamiento quirúrgico. TRATAMIENTO:
Modificación en el estilo de vida, uso de drogas supresoras de la secreción ácida y agentes proquinéticos. El manejo quirúrgico de la
hernia hiatal sintomática tiene varias indicaciones, y/o por manifestaciones directas de la hernia hiatal, como: 1. hernia hiatal
encarcelada con disfagia, 2. dolor torácico asociado a hernia hiatal gigante, 3. pacientes con severa deficiencia de hierro secundaria a
las erosiones o ulceraciones en la hernia hiatal, 4. hernia paraesofágica. Existen varias opciones quirúrgicas de funduplicatura total o
parcial, que pueden ser realizadas por vía transabdominal (funduplicatura de Nissen, Hill, Toupet) o transtorácica; o por
manifestaciones directas de la hernia hiatal, como: 1. hernia hiatal encarcelada con obstrucción sintomática y disfagia, 2. dolor torácico
asociado a hernia hiatal gigante, 3. pacientes con severa deficiencia de hierro secundaria a las erosiones o ulceraciones en la hernia
hiatal, 4. hernia paraesofágica. Existen varias opciones quirúrgicas de funduplicatura total o parcial, que pueden ser realizadas por vía
transabdominal (fundoplicatura de Nissen, Hill, Toupet) o transtorácica (funduplicatura de Nissen o Belsey - Mark IV) ya sea en forma
abierta o por cirugía mínimamente invasiva, pero que en general cumplen los mismos principios quirúrgicos que son: reducción de la
hernia hiatal, cierre del hiato esofágico, restablecer la función del EEI, reposicionar el esófago intraabdominal y crear un mecanismo de
válvula antirreflujo. En aquellos casos de acortamiento esofágico se debe realizar una gastroplastia de Collis para alargar el esófago y
posteriormente la funduplicatura total o parcial, sin tensión sobre el esófago. El curso impredecible de las hernias paraesofágicas, ha
llevado a considerar a la mayoría de los cirujanos, que todas deben ser corregidas quirúrgicamente una vez hayan sido diagnosticadas,
aun en ausencia de sintomatología e independiente de su tamaño, dado su potencial de complicaciones incluyendo el volvulus, la
estrangulación y perforación gástrica que obligarían a intervenciones de urgencia que conllevan mayor mortalidad

CASO CLINICO
Acude paciente a consulta externa de cirugía para valoración preoperatoria, la paciente tiene 41 años de edad y refiere que desde hace
4 años ha presentado reflujo nocturno ha recibido tratamiento con ranitidina, omeprazol y cisaprida, hace 6 meses recibió tratamiento
por presencia de H. pilory sin embargo ahora presenta tos irritativa.

CURSO ENARM CMN SIGLO XXI TEL: 36246001 Pharmed Solutions Institute PÁGINA 490
MANUAL DE TRABAJO DEL CURSO ENARM CMN SIGLO XXI

PREGUNTA
Cuál es el criterio más importante para considerar un procedimiento quirúrgico definitivo.

RESPUESTA
a.- Cambios displasicos.
b.- Reflujo gástrico.
c.- Falla de tratamiento.
d.- Estado de salud.

PREGUNTA
Cual es la complicación mas frecuente de este padecimiento?

RESPUESTA
a.- Esofagitis.
b.- Esofago de barett.
c.- Cancer esofágico.
d.- Estrangulación esofágica.

PREGUNTA
Se realizo endoscopia la cual no reporto datos de esofagitis, sin embargo se obervaron carateristicas compatibles con hernia tipo I, la
biopsia confirmo la presencia de H, pilory, cual es el tratamiento que presenta mejor pronostico para el caso?

RESPUESTA
a.- Omeprazol, metronidazol y amoxicilina.
b.- Omeprazol, amoxicilina y claritromicina.
c.- Nessen, omeprazol, amoxicilina y claritromicina.
d.- Porfimero sodico, nessen, omeprazol, amoxicilina y claritromicina.

PREGUNTA
El paciente regresa a consulta de seguimiento un año después, refiere que se sintió muy bien durante los tres meses subsecuentes a la
terapéutica, posteriormente se presento pirosis, regurgitación y disfagia, actualmente refiere que duerme con tres almohadas, ha
perdido peso ya que no puede comer sin presentar síntomas, refiere además dolor epigástrico que se irradia a la espalda, asi como
fatiga y astenia, cual es el diagnostico mas probable a la exploración se apresia leve tinte ictérico, ganglio de irish y virchow?

RESPUESTA
a.- Cancer pulmonar.
b.- Cancer esofágico.
c.- Cancer gástrico.
d.- Cancer hepático.

CASO CLINICO
Paciente varón de 44 años que consulta por cuadro de pirosis, ardores y dolor epigástrico irradiado a zona retroesternal de dos años de
evolución. Fue tratado con omeprazol, metoclopramida sin resultados adecuados, En estudio endoscópico se observa hernia hiatal sin
signos de esofagitis, que se confirma en tránsito esofagogastricoduodenal donde además se observa reflujo gastroesofágico
espontáneo. En la manometría esofágica se aprecia disminución del tono del esfínter esofágico inferior. Se realiza radiografía de tórax
donde se observa la existencia de masa de densidad grasa que ocupa todo el ángulo cardiofrénico derecho, hallazgos que se confirman
en TAC toracoabdominal.

PREGUNTA
Considerando el cuadro clínico asi como los estudios de gabinete cual es el tipo de hernia mas probable?

RESPUESTA
a.- Hernia hiatal tipo I
b.- Hernia hiatal tipo II.
c.- Hernia hiatal y de morgagni.
d.- Hernia hiatal y de bochdalek

PREGUNTA
Cual de las siguientes manifestaciones es mas frecuente encontrar para pensar en esta entidad nosológica?

RESPUESTA
a.- Dolor abdominal.
b.- Oclusión intestinal.
c.- Dificultad respiratoria.
d.- Regurgitacion gastrointestinal.

CURSO ENARM CMN SIGLO XXI TEL: 36246001 Pharmed Solutions Institute PÁGINA 491
MANUAL DE TRABAJO DEL CURSO ENARM CMN SIGLO XXI

PREGUNTA
Se ha reportado esta asociación en menos del 3 % de casos de hernias hiatales, cual de los siguientes órganos es el menos frecuente
que se comprometa?

RESPUESTA
a.- Jejuno.
b.- Colon.
c.- Higado.
d.- Epiplón.

PREGUNTA
De los siguientes paraclinicos es mas habitual para su diagnostico?

RESPUESTA
a.- TAC.
b.- Radiografia de torax.
c.- Endocopia.
d.- Estudio de transito esófago-gastro-duodenal.

PREGUNTA
Considerando el caso clínico asi como el diagnostico confirmado, porque esta indicado el tratamiento quirugico definitivo y a la
brevedad.

RESPUESTA
a.- Riesgo a malignización.
b.- Riesgo de estangulación.
c.- Riesgo de broncoaspiración.
d.- Riesgo a la invaginación.

CURSO ENARM CMN SIGLO XXI TEL: 36246001 Pharmed Solutions Institute PÁGINA 492
MANUAL DE TRABAJO DEL CURSO ENARM CMN SIGLO XXI
GASTRITIS. CIENCIAS BASICAS: Es una enfermedad inflamatoria aguda o crónica de la mucosa gástrica producida por factores
exógenos o endógenos que produce síntomas dispépticos atribuibles a la enfermedad y cuya existencia se sospecha clínicamente, se
observa endoscópicamente y se requiere confirmación histológica. La gastritis es etiológicamente multifactorial, de los que el más
común es la infección por Helicobacter Pylori. Factores exógenos; Helicobacter pylori, AINES, imitantes gástricos, drogas, alcohol,
tabaco. Factores endógenos; acido gástrico, pepsina, bilis, jugo pancreático, uremia, inmunes SALUD PUBLICA: Es una entidad de
elevada morbilidad a nivel mundial. PATOGENIA: El daño de la mucosa gástrica depende del tiempo de permanencia del factor o
factores lesionates, jugando un rol importante la capacidad que tiene la mucosa gástrica a través de la denominada barrear gástrica
para resistir a estos factores o a los efectos deletéreos de sus propias
secreciones. La barrea gástrica está constituida por componentes pre
epiteliales (barrera de moco, bicarbonato, fosfolípidos), epiteliales
(capacidad de restitución de epitelio, gradiente eléctrico que previenen la
acidificación celular, los transportadores acidobásicos que transportan el
bicarbonato hacia el moco, prostaglandinas, y oxido nítrico) y sub epiteliales
(flujo sanguíneo que descarga nutrientes y bicarbonato en el epitelio y
adherencia y extravasación de los leucocitos, que inducen lesión histica y
quedan suprimidos por la prostaglandinas endógenas). El trastorno de uno u
más de los componentes defensivos, originan lesión de la mucosa
permitiendo la acción del ácido, proteasas y ácidos biliares en mayor o
menor grado y que pueden llegar hasta la lámina propia, sitio en el que
producen lesión vascular, estimulan las terminaciones nerviosas y activan la
descarga de histamina y otros mediadores. Las causas más comunes de
lesión son por H. pylori, lesiones por estrés y AINES. DIAGNOSTICO: Clínicas;
Las gastritis pueden ser totalmente asintomáticas y en caso de existir
síntomas estos no son propios, sino atribuibles a ella, como es la presencia
de ardor, o molestias postprandiales en epigastrio, llenura precoz, RGE,
nausea, distensión abdominal, pirosis, síntomas que también pueden estar
presentes en dispepsia no ulcerosa, úlceras o neoplasias gástricas o
duodenales y aún en el colon irritable. Además pueden manifestarse con
hemorragias crónicas o agudas que podrían llegar a ser masivas con hematemesis y melena. Hallazgos endoscópicos: Los signos
endoscópicos asociados a esta entidad incluyen edema, eritema, mucosa hemorrágica, punteados hemorrágicos, friabilidad, exudados,
erosiones, nodularidad, pliegues hiperplásicos, presencia de signos de atrofia de la mucosa dada por visualización de vasos submucosos
con aplanamiento o pérdida de los pliegues acompañados o no de placas blanquecinas que corresponden a áreas de metaplasma
intestinal. Estos signos endoscópicos pueden localizarse topográficamente a nivel del antro, cuerpo o en todo el estómago,
denominándose gastritis antrales, gastritis corporal o pangastritis respectivamente. Hallazgos histológicos: No se debe abusar del
diagnóstico de gastritis, por lo que se requiere realizar la biopsia para confirmación histológica, establecer la presencia o ausencia de
Helicobacter pylori o de otras formas de gastritis específicas. Exámenes de laboratorio: Las pruebas de laboratorio pueden usarse para
determinar algunas causas de gastritis, como en el caso del Helicobacter pylori a través de métodos invasivos como la endoscopía y
biopsias para el estudio histológico, realizar la técnica de la ureasa rápida, el cultivo y o el empleo de métodos no invasivos como la
serológica para Ig G, la detección de antígeno en las deposición, y la prueba del aliento del C13 o C14 espirado con sensibilidades /
especificidades de o más de 90/90 % a excepción de la serológica 80/90% y el cultivo 50/100%. La gastritis aguda se caracteriza por un
infiltrado inflamatorio que es predominantemente neutrofilico y es usualmente transitorio en su naturaleza, puede acompañarse de
hemorragia de la mucosa, erosiones y si las lesiones son muy severas asociarse a sangrado. La gastritis crónica se caracteriza por un
infiltrado con linfocitos, células plasmáticas o ambas, si además presentan PMN toma la denominación de gastritis crónica activa.
TRATAMIENTO: Dieta sin sustancias irritantes (café, tabaco, alcohol, ají ) así como también drogas que contrarresten la agresión de la
barrera gástrica indicando ya sea antiácidos orales, citoprotectores de la mucosa gástrica (sucralfato, bismuto, misoprostol),
antagonistas de receptores H2, Inhibidores de la bomba de protones, a los que se puede añadir gastrocinéticos (metoclopramida,
domperidona, cisaprida, mosaprida, cinitaprida) si existe evidencias de trastornos de motilidad gastroesofágica o gastroduodenal. Los
inhibidores de bomba de protones son más efectivos que los bloqueadores H2 en el manejo de gastritis IHQ abierta totalmente el
manejo debe de ser hospitalario. Omeprazol o pantoprazol 80 mg intravenosos en bolo seguidos de infusión continua de 8 mg/h por 72
horas). El mantener el PH por arriba de 4 ayuda a restituir la mucosa gástrica. El omeprazol es el fármaco de primera elección para el
tratamiento de la Gastritis Aguda. Los esquemas de primera línea pueden ser triples o cuádruples (cuando se agrega una sal de
bismuto). El basado en las tasas de éxito alcanzadas en prácticamente todas las regiones del mundo se recomienda el uso de 1 g de
amoxicilina dos veces al día, 500 mg de claritromicina dos veces al día y dosis doble de un IBP durante 14 días. El esquema de segunda
línea recomendado para la erradicación de Helicobacter pylori es levofloxacino 500 mgs cada 24 hrs, amoxicilina 1 gr cada 12 hrs,
inhibidor de bomba de protones dosis estándar cada 12 hrs por 10 a 14 días o moxifloxacino 400mg al día, amoxacilina 1 gr cada 12 hrs
e inhibidor de bomba de protones cada 12 hrs. En el caso de alergia a la amoxicilina, se puede emplear como alternativa tetraciclina
(500 mg cuatro veces/día) o metronidazol (250 mg cuatro veces/día). Las sales de bismuto, en forma de subsalicilato o subcitrato,
deben administrarse en cuatro tomas al día y en dosis promedio de 525 mg. Para erradicar Helicobacter pylori hay que confirmar la
erradicación de la bacteria. H.pylori: La infección por Helicobacter pylori se calcula que afecta la mitad de la población mundial, con
mayor prevalencia en países en desarrollo, llegando a cifras hasta del 90%, asociada a niveles socio económicos bajos, adquiriéndose a
edades más tempranas en comparación con los países desarrollados, Todos los sujetos que presentan Helicobacter Pylori desarrollan
gastritis, un 15 a 20% presentan ulcera – péptica y menos del 1% adenocarcinoma gástrico existiendo variaciones regionales. La
infección por estas bacterias se inicia en el antro gástrico, ubicándose a lo largo de la superficie epitelial, se extiende hacia el cuerpo
llevando en los casos crónicos de larga evolución a atrofia, metaplasia intestinal que pueden terminar en displasia y finalmente
adenocarcinoma.

CURSO ENARM CMN SIGLO XXI TEL: 36246001 Pharmed Solutions Institute PÁGINA 493
MANUAL DE TRABAJO DEL CURSO ENARM CMN SIGLO XXI
CASO CLINICO
Se trata de paciente femenino de 21 años de edad la cual acude a servicio médico de la escuela refiriendo presentar nauseas con
vomito con mínimo contenido gástrico, niega sangrados, agrega gases y eructos ocasionales, se alimenta dos veces al día y come lo que
puede al salir de la escuela rumbo a su trabajo.

PREGUNTA
Cuál de las siguientes medidas mejora el pronóstico.

RESPUESTA
a.- Tratamiento para H. pilory.
b.- Cambio de hábitos dietéticos.
c.- Endoscopia con biopsia.
d.- Doble esquema antiácido.

CASO CLINICO
Se ingresa a paciente femenino de 51 años de edad la cual acude a urgencias por presencia de dolor epigástrico, vomito y nauseas,
además agrega disminución del apetito, agrega que desde hace 3 meses ha recibido tratamiento por anemia previamente diagnostica
para lo cual indicaron hierro.

PREGUNTA
Cuál es el método diagnostico más adecuado.

RESPUESTA
a.- Serie gastroesofagica.
b.- Endoscopia.
c.- La clínica es suficiente.
d.- Biometria hemática.

CASO CLINICO
Se trata de masculino de 31 años de edad el cual acude a consulta por cefalea tensional, al interrogatorio refiere que presenta gastritis
recurrente y ha sido sometido a diversos tratamientos farmacológicos para este problema pero han sido incompletos, fue sometido a
endoscopia donde se diagnostico gastritis crónica con presencia de H. pilory, existen antecedente en la familia de cáncer gástrico por lo
que el paciente se encuentra más preocupado.

PREGUNTA
Considerando el cuadro clínico y el diagnostico endoscópico cual es la conducta farmacológica más adecuada.

RESPUESTA
a.- Metronidazol, claritromicina y omeprazol.
b.- Claritromicina, amoxicilina y omeprazol.
c.- Metronidazol, sales de bismuto y ranitidina.
d.- Amoxicilina, metronidazol y pantoprazol.

CURSO ENARM CMN SIGLO XXI TEL: 36246001 Pharmed Solutions Institute PÁGINA 494
MANUAL DE TRABAJO DEL CURSO ENARM CMN SIGLO XXI
ULCERA GASTRICA Y DUODENAL. CIENCIAS BASICAS: La ulceras duodenales casi siempre están a 1-2cm del píloro. A menudo se
acompañan de hipersecreción de ácido. Las ulceras gástricas se dividen en varios tipos: Tipo I; la más frecuente, en la parte proximal del
antro o del cuerpo. Tipo II; secundaria a ulcera duodenal con estenosis pilórica. Tipo III; ulcera prepilórica o del canal pre pilórico, sus
causas son similares a las de la ulcera duodenal. SALUD PUBLICA: La mayor incidencia de la ulcera gástrica es entre los 50-65 años de
edad. La mayor parte de casos de ulcera duodenal se presenta en la cuarta década de la vida. PATOGENIA: 3 causas básicas:
hipersecreción de ácido, colonización por H. pylori y AINES. H.pylori se encuentra en 95% de los casos de enfermedad ulcerosa
duodenal y en 80% de enfermedad por ulcera gástrica. Sin embargo resulta difícil probar la relación causa-efecto. Los AINES suprimen la
síntesis de prostaglandinas y debilitan la barrear mucosa. Entre 10-30% de los consumidores crónicos de estos fármacos presenta
ulcera péptica. Patogenia de ulcera duodenal: En general los pacientes secretan más acido por aumento en la cantidad de células
parietales, principales o factores tróficos (gastrina). H. pylori incrementa la liberación de gastrina y produce duodenitis. Algunos
pacientes también tienen trastornos de la motilidad con vaciamiento gástrico rápido de líquidos. Lo que expone al duodeno a una
cantidad de ácido mayor de la normal. La
anormalidad fisiológica más frecuente es la
disminución de la secreción de bicarbonato.
Patogenia de ulcera gástrica: El defecto básico
generalizado radica en la defensa de la mucosa
gástrica contra el ácido y la pepsina. El gasto acido
es normal o apenas detectable, en contraste con la
ulcera duodenal. La anormalidad básica suele ser
el reflujo del contenido duodenal. Es posible que
exista disfunción pilórica, también secundaria al
tabaquismo. El reflujo de ácidos biliares, lisolecitina
y secreciones pancreáticas ejerce un efecto nocivo
y dañan la mucosa. El ASA tiene un efecto similar.
DIAGNOSTICO: Clínica; dolor es lo más frecuente,
es epigástrico agudo o cólico. El dolor de la ulcera
duodenal se presenta muchas horas después de
una comida, cuando el bulbo duodenal esta vacío;
se alivia con alimentos y sustancias alcalinas. Por el contrario el dolor de la ulcera gástrica, se intensifica con la ingesta de alimentos. El
dolor suele ser crónico y recurrente. Otras manifestaciones son nausea, pérdida de peso y ligera sensibilidad. Por lo general el
diagnóstico se establece por endoscopia de tubo digestivo superior o una serie esofagogastroduodenal. La endoscopia es necesaria en
todos los casos de ulcera gástrica, debido al riesgo de cáncer, aquí se obtienen el material para la detección de H. pylori.
TRATAMIENTO: En pacientes jóvenes se basa solo en los síntomas, sin estudios. En ancianos es necesario realizar endoscopia por riesgo
de tumor maligno. Con una serie radiológica de tubo digestivo superior de doble contraste se detecta 90% de las ulceras gástricas y
duodenales. Medico; 1) Neutralizar el ácido gástrico. 2) Inhibir la secreción. 3) Proteger la mucosa gástrica de la lesión. Los antiácidos
neutralizan el ácido del estómago. Los antagonistas de los recetores H2 bloquean dichos receptores en las células parietales. Los
fármacos más potentes son los bencimidazoles sustituidos que bloquean la H+/K+ ATP asa (IBP) Y que funcionan incluso en pacientes con
gastrinoma. La mucosa gástrica puede protegerse con análogos de las prostaglandinas E2 (misoprostol) que incrementa el flujo
sanguíneo de la mucosa y la producción de bicarbonato y moco. El sucralfato se une a las proteínas de los cráteres ulcerosos, por lo
que favorece la cicatrización. El bismuto cubre la proteína expuesta y tiene actividad contra H. pylori. Quirúrgico: Ulcera duodenal;
vagotomía ultra selectiva COMPLICACIONES: Perforación (5-10%), Hemorragia y obstrucción (<5%). Entre 15-20% de las ulceras
pépticas producen hemorragia macroscópica (gástrica o duodenal). Los pacientes refieren melena o hematemesis (gástrica).
SINDROME DE ZOLLINGER-ELLISON: Se identifica en 0.1-1% de todos los pacientes con enfermedad ulcerosa péptica y cerca de 20% de
estos padece también síndrome de neoplasia endocrina múltiple tipo I. Los síntomas incluyen hipergastrinemia y ulceración péptica
grave secundaria a un gastrinoma. Es probable que las ulceras sean múltiples y más distales. El gasto basal de acudo es muy alto, el
nivel de gastrina puede ser superior a 1000pg/ml. Es posible inducir niveles equívocos de gastrina con la administración IV de calcio o
secretina. Los gastrinomas son neoplasias de enrome malignidad (>90%), pero de crecimiento lento; 50% de los tumores tiene
metástasis ganglionares para el momento del diagnóstico.

CASO CLINICO
Acude paciente femenino de 41 años de edad para revisión de tratamiento previo por anemia perniciosa, los resultados de laboratorio
no fueron traídos por la paciente, sin embargo la endoscopia afirmo el diagnostico, refiere que ha presentado dolor en el cuadrante
superior derecho, se realizo un USG de vías biliares con presencia de lito en vesícula biliar y resto de vías normales.

PREGUNTA
Cuál es el diagnostico diferencial para iniciar tratamiento correcto.

RESPUESTA
a.- Serie gastroesofagica.
b.- Frotis de sangre periférica.
c.- Tomografía abdominal.
d.- Biopsia gástrica.

CURSO ENARM CMN SIGLO XXI TEL: 36246001 Pharmed Solutions Institute PÁGINA 495
MANUAL DE TRABAJO DEL CURSO ENARM CMN SIGLO XXI
CANCER GASTRICO. CIENCIAS BASICAS: Es un tipo de crecimiento tisular maligno producido por la proliferación contigua de células
anormales con capacidad de invasión y destrucción de otros tejidos y órganos, en particular el esófago y el intestino delgado. En las
formas metastasicas las células tumorales pueden infiltrar los vasos linfáticos de los tejidos, diseminarse a los ganglios linfáticos y,
sobrepasando esta barrera, penetrar en la circulación sanguínea y diseminarse a cualquier órgano del cuerpo. Factores de riesgo:
Ambientales; exceso de consumo de sal, nitritos, nitrosaminas, aminas aromáticas, bajo consumo de frutas y verduras, infección por H.
pylori, VEB (ca de cardias), trabajadores del metal mineros caucho, polvo asbesto. Genéticos; historia familiar, grupo sanguíneo A.
Otros; gastrectomía subtotal, anemia perniciosa, pólipos, Enf de Menetrier (perdedora de proteínas). Factores de riesgo definitivos y
de vigilancia sugerida: Displasia de Alto Grado (75-100% evoluciona a cáncer), poliposis Adenomatosa Familiar, adenomas, esófago de
Barrett. Definitivos: Metaplasia Intestinal (80% se asocian a cáncer), Gastritis Crónica Atrófica, infección por Helicobacter pylori
carcinógeno tipo I, cáncer Colorrectal Hereditario sin poliposis. Probables: post–Gastrectomía, vagotomia Píloroplastia. Condiciones
premalignas: Mentrier y ulcera gástrica. SALUD PUBLICA: El cáncer gástrico se reporta como la segunda causa de muerte a nivel
mundial con aproximadamente 600,000 muertes al año. El adenocarcinoma representa 95%. 3er lugar en incidencia, mayores de 50
años (69.7%). 5to lugar en incidencia, mayores de 55 años (55.2%). Clases socioeconómicas bajas. Más varones 2/1. El riesgo de
infección por H.pylori a lo largo de toda la vida en países desarrollados es del 40
a 60%. • Países en desarrollo puede alcanzar hasta el 90%. PATOGENIA:
Secuencia de carcinogénesis gástrica: Infección por H. pylorigastritis
crónicaatrofia gástricametaplasia intestinaldisplasia bajo/alto
gradocáncer gástrico (adencarcinoma) precoz/avanzadometastasis.
ADENOCARCINOMA: Edad media: 65-74 a. En hombres a edades más jóvenes.
Mayor tasa de mortalidad en hombres. Cáncer gástrico precoz/superficial: No
invade más allá de la submucosa, respetando la capa muscular, con
independencia del compromiso linfático, operado tiene buen - pronóstico.
Adenocarcinoma tipo intestinal: Más frecuente (antro, curvatura menor), de
crecimiento expansivo, prevalente en poblaciones de alto riesgo, hombres de
mayor edad (55años), relación H:M 2:1, largo proceso de pre cáncer, rara
diseminación, mejor pronóstico. Adenocarcinoma tipo difuso: metástasis
tempranas, se presenta por igual en toda la población, más frecuente en
poblaciones jóvenes (48años), relación H:M 1:1 relacionada con factores
genéticos, Infiltra toda pared, células en anillo de sello, diseminación. Carcinoma
gástrico precoz: Confinado a mucosa y submucosa independiente de la presencia o
ausencia de metástasis ganglionares. Carcinoma avanzado: Infiltra la pared
muscular como mínimo, usualmente asociado con extensión distante, escasa
posibilidad de curación. Metástasis al momento de diagnóstico en el 50% se
encuentran en estadios III o IV. Metástasis más frecuentes: Hígado (40%) por vía
hematógena Superficie peritoneal Nódulos locoregionales o distantes. Aspecto
Macroscópico de adenocarcinoma: Ulcerativo 75%, polipoideo 10%, infiltrante
10%, superficial 5%. ENFERMEDAD POR EXTENSION: Nódulo periumbilical (nódulo de la hermana maría josé). Ovarios (tumor de
krukennberg). Ganglio supraclavicular izquierdo (Virchow). Nódulo axilar izquierdo (nódulo irlandés). DIAGNOSTICO: Clínica; Cáncer
que no penetra la muscular propia, asintomático 80% de los casos. Enfermedad avanzada al momento del diagnóstico: pérdida de peso
62%, dolor abdominal persistente 52%, náusea y vómito, anorexia, disfagia, melena, saciedad temprana, sangrado. Tumores en antro,
saciedad temprana y vómito. Tumores en cardias, disfagia, emesis fecal o alimentos no digeridos en heces, fístula gastrocólica por
invasión a la pared del colon. Histología: Adenocarcinoma: 90% de las neoplasias gástricas malignas. Linfoma gástrico 10% de estos 95%
linfoma no Hodgkin. Tumor Carcinoide gástrico 0.2%. GIST Tumores estromales GI, 50-60% ocurren en el estómago. Leiomiosarcomas.
Metástasis: Mama y melanoma. Laboratorio y gabinete: Rx esófago gastroduodenal. Endoscopia y biopsia. Rx de tórax. Ecografia de
abdomen. Eco endoscopía. TAC de abdomen con y sin contraste. Laparoscopia diagnostica estadificación. RNM grado de invasión
extraserosa y compromiso linfático. PET . Gen e – cadherin en recidiva especificidad del 75%. Gen p53 (mutado) cromosoma 17
alterado en 35%. FSA Antígeno fetal sulfoglicoproteina en el 96 %. CEA y ca 19,9, Láctico-deshidrogenasa, Alfafetoproteína.
TRATAMIENTO: Operables: Tumores móviles sin extensión mtts o carcinomatosis. Inoperables: mtts pulmonares con insuficiencia
respiratoria, carcinomatosis peritoneal, ascitis, insuficiencia ponderal y/o metabólica no corregibles. Irresecables: carcinomatosis no
diagnosticada en preoperatorio, fijación por adherencias tumorales a otras vísceras, hígado y colon no son contraindicación absoluta.
Cirugía: El tratamiento laparoscópico del cáncer gástrico temprano con linfadenectomia, es factible de realizar, con resultados similares
a las técnicas abiertas. Laparoscopia diagnostica para estadificación. Los procedimientos quirúrgicos: A) Gastrectomía total 27%. B)
Gastrectomía Subtotal 35,7%. C) Gastroenteroanastomosis 12%. D) Lap. Exploradora 18,3%. E) Gastrect. Polar Superior 7%.
Radioterapia: Tumores irresecables, con resección incompleta, negativa a la cirugia y / o qt, dolor, hemorragia. Quimioterapia: Estadio II
o más: 3 ciclos de epirrubicina, cisplatino, 5-FU, pre y postoperatorio. Otros esquemas: 5FU-Doxorubicina –Cisplatino o Etopósido-5FU-
Leucovorina.

CASO CLINICO
Se trata de paciente masculino de 41 años de edad, originario de Hermosillo, inicia padecimiento hace 6 meses, caracterizado por
disminución de peso de forma continua, agrega fatiga, adinamia, sensación de plenitud temprana, dolor en cuadrante superior
izquierdo, a la exploración se observa palidez generalizada, deshidratación moderada, disneico con tos seca, niega tabaquismo ó
alcoholismo, existen antecedentes de tuberculosis y cáncer en la familia, resto sin datos que agregar. En el laboratorio se reporto
sangre oculta en heces y anemia por deficiencia de hierro.

PREGUNTA
Cuál es el tratamiento más adecuado para el padecimiento.

CURSO ENARM CMN SIGLO XXI TEL: 36246001 Pharmed Solutions Institute PÁGINA 496
MANUAL DE TRABAJO DEL CURSO ENARM CMN SIGLO XXI

RESPUESTA
a.- Resección quirúrgica.
b.- Radioterapia.
c.- Quimioterapia.
d.- Resección quirúrgica y quimioterapia.

PREGUNTA
Cuál es el signo o método diagnostico más certero para este padecimiento.

RESPUESTA
a.- Sangre en heces.
b.- Ganglio de Virchow.
c.- Serie gastrointestinal.
d.- Marcadores tumorales.

CASO CLINICO
Paciente 66 años de edad, sexo masculino, con antecedentes mórbidos de hipertensión arterial y diabetes mellitus (DM) tipo 2 en
tratamiento. Ingresa por cuadro clínico de 4 meses de evolución caracterizado por astenia, adinamia, ardor epigástrico, pirosis y
regurgitación, asociado a lesiones dérmicas, de aparición súbita en ambos pies, no pruriginosas. El examen físico reveló un paciente en
buen estado general, en el que destacaba únicamente presencia de lesiones queratócicas, seborreicas e hiperpigmentadas en ambos
pies lo cual asociado a historia clínica sugería signo de Leser-Trélat.

PREGUNTA
Cual es la conducta a seguir mas adecuada.

RESPUESTA
a.- Realizar endoscopia.
b.- Realizar TAC.
c.- Realizar IRM.
d.- Realizar Biopsia de lesión dérmica.

PREGUNTA
Posterior a diversos estudios incluyendo biopsia de lesión dérmica se diagnostico adenocarcinoma gástrico, el cual se encontraba en
condiciones de tratamiento quirúrgico, cual es el área mas frecuente donde encontrar estas lesiones en caso de observarse?

RESPUESTA
a.- Pies o manos.
b.- Torax.
c.- Cabeza.
d.- Abdomen.

CURSO ENARM CMN SIGLO XXI TEL: 36246001 Pharmed Solutions Institute PÁGINA 497
MANUAL DE TRABAJO DEL CURSO ENARM CMN SIGLO XXI
COLANGITIS, COLECISTITIS, COLEDOCOLITIASIS:
VESICULA BILIAR, LITOGENESIS: La vesícula biliar es un órgano piriforme de aproximadamente 7 - 10 cms. de largo por 3 cms. de
ancho, con una capacidad habitual de 30-35cc, pudiendo contener volúmenes muy superiores en condiciones patológicas. Se le
reconocen tres partes principales: fondo, cuerpo y cuello. Desde éste emerge el conducto cístico, mediante el cual la vesícula se une al
conducto hepático común, para dar origen al conducto colédoco. La longitud de éste es de aproximadamente 7,5 cms., variable según
el punto de desembocadura del conducto cístico. Alcanza un diámetro normal de 5,5 mm, el cual es mucho mayor en caso de patología
obstructiva. Se reconocen 4 porciones bien definidas: 1. Porción Supraduodenal: Desciende en el ligamento hepatoduodenal frente al
hiato de Winslow. Se sitúa por delante y a la derecha de la vena porta. La arteria hepática y su rama gastroduodenal se sitúan a su
izquierda. 2. Porción Retroduodenal: Se relaciona íntimamente con la primera porción del duodeno, ubicándose lateralmente respecto
a la vena porta y frente a la cava. 3. Porción Pancreática: Se extiende desde el borde inferior de la primera porción del duodeno hasta
un punto en la pared posteromedial de la segunda porción del duodeno. 4. Porción intramural duodenal: Corre en sentido oblicuo hacia
abajo y lateralmente dentro de la pared del duodeno en una extensión de más o menos 2 cms. El colédoco suele unirse al conducto
pancreático justo al interior de la pared duodenal en el 89% de los vasos. Formación de cálculos biliares: Existen dos tipos de cálculos,
de acuerdo a su composición bioquímica: cálculos puros y cálculos mixtos. Los cálculos puros son de origen exclusivamente metabólico,
y están compuestos de colesterol o bilirrubina. Los cálculos mixtos presentan en su etiopatogenia una causa inflamatoria, y están
formados por una mezcla de colesterol, sales de calcio y pigmentos biliares, lo que se deposita sobre una base de naturaleza orgánica
formada por células epiteliales, material proteico y bacterias. El cálculo de bilirrubinato de calcio (mixto) se forma a partir de un núcleo,
de preferencia en los conductos biliares. Corresponden al 82% de los cálculos. El aumento de bilirrubina libre en la bilis (lo normal es
20%) determinará el fenómeno de litogénesis primaria que corresponde a la formación de bilirrubinato de calcio. Los cálculos de
colesterol se producen por una alteración del equilibrio bioquímico de los componentes de la bilis: sales biliares, fosfolípidos (lecitina),
colesterol y ácidos biliares. Para que el colesterol no precipite, la bilis debe estar bajo la forma de micelas constituidas por un centro de
colesterol y una cubierta de sales biliares y fosfolípidos. En esta forma micelar el colesterol es hidrosoluble. Las alteraciones tanto en la
calidad como en cantidad de las sales biliares y la lecitina van a determinar la precipitación y formación posterior de cálculos. A esta
bilis alterada en su composición la denominaremos Bilis litogénica (Bilis formadora de cálculos). El aumento de la excreción de
colesterol se ha asociado a una mayor incidencia de litiasis biliar. Por otro lado la ectasia biliar, la infección biliar y factores metabólicos
también se relaciona con formación de colelitiasis. Los estrógenos aumentan el índice de saturación de colesterol, favoreciendo
colelitiasis. También habría factores genéticos relacionados con litiasis biliar. Se ha observado mayor frecuencia de litiasis biliar en:
diabéticos, obesos, embarazadas y mujeres que usan anticonceptivos de tipo androgénico.

COLANGITIS. CIENCIAS BASICAS: La colangitis aguda es la infección severa de la bilis dentro de los conductos biliares intra y extra
hepáticos. Está producida por la existencia de bacterias en la bilis, en una situación de obstrucción biliar, como consecuencia de la
obstrucción del flujo biliar, se produce un aumento en la presión intraductal que favorece el paso de gérmenes a la circulación portal y
linfática, produciendo episodios de bacteriemia con septicemia o sin ella. Su etiología es debido a enfermedades concomitantes
(colangitis primarias: coledocolitiasis (70%) parasitosis, estenosis, quistes de colédoco, enfermedad de Caroli, colangitis esclerosante y
tumores) o por actuaciones directas o indirectas, diagnósticas o terapéuticas, sobre el hígado o la vía biliar (colangitis secundaria). La
colangitis será supurada o no supurada si la bilis en la vía biliar está contaminada o es purulenta; los gérmenes más frecuentes
involucrados n son E. Coli, Kleibsella, pseudomona estreptococo fecalis, enterobacter, bacteroides y clostridium. DIAGNÓSTICO: Clínico:
Se basa en la asociación de signos y síntomas de infección con los propios de una obstrucción biliar. Dolor abdominal, en el cuadrante
superior derecho ó en el centro del abdomen superior, puede ser intermitente, agudo, tipo cólico o sordo, puede irradiarse a la espalda
o debajo del omóplato derecho, escalofríos, fiebre generalmente alta, con escalofríos e ictericia (triada de Charcot). Cuando se añade
confusión mental y shock se denomina Pentada de Reynolds, con menos frecuencia y que se asocia con una colangitis supurada grave.
Puede presentar heces color arcilla, orina oscura así como náuseas y vómitos. Laboratorio y gabinete hallazgos de obstrucción biliar con
aumento variable de las bilirrubinas más de 2md/dl (80%) y enzimas de colestasis (FA, TGO, TGP). Habitual encontrar leucocitosis con
desviación a la izquierda (80%) siendo por lo general las cifras más altas en torno a 20.000 mm3, reflejo de las formas más severas,
hemocultivos positivos durante los escalofríos o picos febriles. La ecografía es la técnica de elección para detectar la existencia de
obstrucción biliar por su elevada eficacia y versatilidad, pudiéndose detectar también complicaciones en la colangitis como absceso
hepático. La colangio-resonancia magnética con la sensibilidad de 90% y una especificidad 100%.La colangiopancreatografía retrógrada
endoscópica (CEPRE) y la colangiografía transhepática percutánea. TRATAMIENTO: Piperacilina 4grs i.v./8h. Tazobactam 4.5grs i.v./8h.
Ciprofloxacino 200mgsi.v./12h. Ceftriaxona 1gr i.v/24h. Metronidazol 500mgs i.v/8h. Analgésicos, antiinflamatorios y antipiréticos.
Medidas higiénicas y dietéticas: El consumo de alimentos será limitado durante su hospitalización. Se recomienda dieta baja en grasas.
Durante su internamiento dependerá de la situación clínica y evolución el inicio de la dieta, puede requerir de nutrición parenteral en
los casos graves por sepsis. Bajar de peso, desintoxicación del hígado y colón. Evitar estrés, ejercicio regular ayuda a tonificar la
vesícula. Mantener una dieta adecuada, realizar ejercicio, evitar alimentos condimentados y grasosos. Indicaciones de hospitalización:
Tener fiebre, ictericia, vómitos recurrentes o se presenta dolor después del diagnóstico o tratamiento, se incluye disnea, diaforesis y
náuseas. Deterioro progresivo y mal estado general. Criterios de evaluación de severidad leve (Grado I), colangitis que responde al
tratamiento médico inicia. Moderado (Grado II) colangitis aguda que no responde al tratamiento médico inicial y no se acompaña de
disfunción orgánica múltiple. Severa (Grado III) colangitis aguda que se asocia con la aparición de disfunción de al menos uno de los
siguientes órganos / sistemas: cardiovascular, hipotensión que requiere de aminas, alteraciones de la consciencia, relación PaO2/Fio2
<300,Cr sérica > 2 mg/dl, TP-RIN>1.5, Plaquetas < 100.000/ul.

CASO CLINICO
Paciente de 59 años sin antecedentes personales de interés que acudió a urgencias por dolor epigástrico y febrícula. En la exploración
se observo ictericia mucocutanea y ocupación del hemiabdomen superior derecho. La analítica demostró: bilirrubina total, 5,3 mg/dl;
transaminasa glutámico pirúvica (GPT), 311 U/l; transaminasa glutámico oxalacética (GOT), 166 U/l, y fosfatasa alcalina (FA), 176 U/l; el
resto fue normal. Se realizó una ecografía y una tomografía computarizada (TC) abdominal que confirmaron la existencia de 2 grandes
quistes hidatídicos en el lóbulo hepático derecho (de 13 y 9 cm) y otro quiste hidatídico calcificado en el lóbulo hepático izquierdo de 5

CURSO ENARM CMN SIGLO XXI TEL: 36246001 Pharmed Solutions Institute PÁGINA 498
MANUAL DE TRABAJO DEL CURSO ENARM CMN SIGLO XXI
cm. Las ramas biliares principales derecha e izquierda y el colédoco en toda su longitud estaban ocupadas por un material con múltiples
ecos lineales. La serología para hidatidosis fue positiva (tuvo contacto con perros y ganado hace años). Se inició tratamiento con
albendazol antes de la cirugía y se realizó una colangiopancreatografía retrograda endoscópica (CPRE), que confirmó la dilatación y la
ocupación de la vía biliar con salida espontánea de material hidatídico.

PREGUNTA
Cuál es los siguientes patologías no es diagnostico diferencial.

RESPUESTA
a.- Hepatitis aguda.
b.- Nefrolitiasis.
c.- Obstrucción intestinal.
d.- Ulcera duodenal.

COLECISTITIS. CIENCIAS BASICAS: Es la inflamación de la vesícula biliar ocasionada principalmente por cálculos (litos) y con menor
frecuencia por barro (lodo) biliar, en raras ocasiones ninguna de estas condiciones está presente. Colelitiasis: presencia de litos en la
vesicula biliar y con menor frecuencia de lodo biliar. SALUD PUBLICA: Es una de las principales causas de consulta en el servicio de
urgencias y en la consulta externa de cirugía general. La colecistectomía electiva es la intervención quirúrgica más frecuente en los
centros hospitalarios. Se presenta en el 5-20% de los pacientes con colelitiasis. La colecistitis aguda se presenta con más frecuencia en
mayores de 40 años, más frecuente en sexo femenino 2:1. PATOGENIA: Los factores que pueden condicionar el desarrollo de
Colecistitis y Colelitiasis: Edad: más frecuente a partir de los 40 años, cerca del 20% de los adultos a partir de esta edad y del 30% en los
mayores de 70 años, sexo femenino, embarazo, sobre todo para el desarrollo de cálculos de colesterol, normalmente son formas
asintomáticas de litiasis biliar, anticonceptivos orales y terapia hormonal sustitutiva con estrógenos, en este caso en mujeres menores
de 40 años y las que reciben una dosis mayor de 50 microgramos de estrógenos. Otros fármacos como los fibratos y la ceftriaxona,
obesidad, antecedentes familiares de litiasis biliar, pérdida rápida de peso, nutrición parenteral, DM, cirrosis hepática, enfermedades
del íleon, enfermedad de Crohn, enfermedades hepáticas y metabólicas. CLASIFICACION: La colecistitis aguda se clasifica en tres
grados, de acuerdo a la afectación de otros órganos, medidos por pruebas de laboratorio: Grado I - leve, Grado II - moderada, Grado III
– grave. DIAGNOSTICO: Clínico: de colecistitis y colelitiasis son: Murphy positivo, dolor y resistencia en cuadrante superior derecho,
nausea, vomito. Manifestaciones agudas; vesícula palpable, fiebre >39°, escalofríos., inestabilidad hemodinámica. La perforación con
peritonitis generalizada se sospecha cuando; existen signos de irritación peritoneal difusa, distensión abdominal, taquicardia,
taquipnea, acidosis metabólica, hipotensión, choque. No hay pruebas de laboratorio específicas para colecistitis aguda, pero pueden
ser de utilidad: biometría hemática: se puede encontrar leucocitosis. Proteína C Reactiva la cual puede encontrarse elevada y es de
utilidad para confirmar el proceso inflamatorio. Ante la sospecha de colecistitis o colelitiasis aguda el ultrasonido es la prueba no
invasiva de primera elección 98% de sensibilidad, los hallazgos que reporta son: engrosamiento de la pared vesicular mayor de 5 mm.
Liquido perivesicular, signo de Murphy ultrasonográfico positivo, alargamiento vesicular 8 cm. Axial y 4 cm. Diametral, lito encarcelado,
imagen de doble riel, sombra acústica, ecos intramurales. Para identificar la gravedad de la Colecistitis y Colelitiasis se solicitará:
Bilirrubinas, BUN, creatinina, tiempo de protrombina, la amilasa sérica es de utilidad para identificar complicaciones como
coledocolitiasis. TRATAMIENTO: para la disolución de los cálculos biliares en pacientes en situaciones especilaes (pacientes con alto
riesgo quirúrgico y aqueloos que se rehúsan a la cirugía), el tratamiento de elección es con acidos biliares como: Acido ursodesoxicolico,
ácido quenodeoxicolico, por un periodo de 1-2 años. Depende del nivel de gravedad, se prefiere una colecistectomía temprana y de
selección adecuada de la técnica con la que se realizará la cirugía. La litiasis vesicular asintomática no es indicación de colecistectomía
excepto que la pared vesicular este engrosada (4mm o más) o calcificada o que los cálculos sean mayores a 3 cm de diámetro. La
colecistectomía puede indicarse ocasionalmente en pacientes con dolor vesicular sin litiasis (colecistitis alitiásica) o porque la vesícula
no funcione adecuadamente (discinesia biliar). En estos casos el diagnostico se realiza mediante una colecistografía oral o un
gamagrama de vesícula. La colecistectomía laparoscópica temprana es el tratamiento de elección en la mayoría de los pacientes,
porque: la recuperación es rápida, requiere menor estancia hospitalaria, reduce costos de tratamiento, reincorporación rápida al
trabajo. Se considera temprana cuando se realiza de 1 a 3 días después del ataque inicial y tardío si se realiza de 6 A 15 días después del
ataque. Factores que dificultan la colecistectomía por laparoscopia son: cirugía abdominal previa, presencia o antecedentes de ictericia.
La combinación de extracción de litos por endoscopia durante una colangio-pancreatografía retrograda endoscópica (CPRE) y la
colecistectomía por laparoscopia, es de utilidad en el tratamiento de pacientes con colecisto y coledocolitiasis. El intervalo entre estos
dos procedimientos es de pocos días (ambos procedimientos se realizarán durante la misma estancia hospitalaria). En la
Colecistectomia abierta no complicada los días de estancia hospitalaria posquirúrgica son de 2 a 3 días. Y el tiempo promedio de
recuperación es de 21 días. En la colecistectomia laparoscópica no complicada dos días de estancia hospitalaria es suficiente. MANEJO
PERIOPERATORIO: Corrección ó control de los factores de riesgo, estudios de laboratorio e imagen solicitados en forma particular a
cada caso, en mayores de 45 años electrocardiograma, telerradiografía de tórax y valoración cardiovascular preoperatoria, el uso
profiláctico de antibióticos se selecciona en base al tipo de herida quirúrgica. En todos los pacientes se debe llevar a cabo prevención
tromboembólica. La administración de AINEs se recomienda a pacientes con cólico biliar, sin embargo no es útil para la remisión de la
enfermedad. Es recomendable el uso de diclofenaco a razón de 75 mgr. IM para el manejo de los cólicos biliares. El manejo del dolor es
pacientes con colecistitis aguda incluye narcóticos como meperidina, en presencia de dolor intenso. El tratamiento de antibiótico se
indicará dependiendo de la gravedad de la colecistitis: para colecistitis leve- un antibiótico, para colecistitis moderada y grave se
recomienda el uso de dos antibióticos, cuando se sospecha o se detecta la presencia de anaerobios agregar metronidazol. Litotripsia
extracorpórea: Indicada en pacientes con litiasis única, no calcificada, con diámetros de 20-30mm. Contraindicada en pancreatitis,
alteraciones de la coagulación, quistes o aneurismas en el trayecto de las ondas de choque. El ácido ursodeoxicolico se indica como
coadyuvante cuando se realiza litotripsia, ya que de esta forma se logra mayor efectividad en la destrucción de litos. No aprobado por
FDA como tratamiento definitivo para litiasis vesicular. COMPLICACIONES: de la colecistectomía laparoscópica generalmente son
debidas a instrumental inapropiado o caduco, y también por la falta de pericia del cirujano como son: lesión del conducto biliar, lesión

CURSO ENARM CMN SIGLO XXI TEL: 36246001 Pharmed Solutions Institute PÁGINA 499
MANUAL DE TRABAJO DEL CURSO ENARM CMN SIGLO XXI
del intestino, lesión hepática. En la cirugía abierta y laparoscópica además puede haber como complicación: las infecciones, íleo,
hemorragia intraperitoneal, atelectasia, trombosis de venas profundas, infección de vías urinarias.

CASO CLINICO
Varón de 32 años, que acude a Urgencias por dolor en hipocondrio derecho de 72h de evolución tipo cólico y vómitos tras la ingesta
que no calman con analgesia convencional y antieméticos. Siendo dado de alta a las 48h. Sin antecedentes médicos. No Sd
constitucional. Ictericia cutánea y mucosa evidente sin prurito asociado. No fiebre. Hemodinámicamente estable. T.ª 36,8°C. Abdomen
blando, depresible, doloroso en hipocondrio derecho, sin masas ni megalias. No peritonismo. Ictericia mucocutánea. La analítica
muestra 10,9 10E3/μl leucocitos y discreta eosinofilia (0,7 10E3/μl). ilirrubina total de 7,9mg/dl, con bilirrubina conjugada de 5,2mg/dl
y aumento de fosfatasa alcalina de 853 y ALT de 268. Lipasa de 346.

PREGUNTA
Cual es la conducta a seguir mas adecuada?

RESPUESTA
a.- Conducta conservadora.
b.- Preparar para colecistectomía.
c.- Realizar USG abdominal.
d.- Realizar TAC abdominal.

CASO CLINICO
Mujer de 17 años que acude a Urgencias por un cuadro de abdomen agudo. Como antecedentes, destacaba un ingreso un año antes
para estudio por un síndrome febril y cefalea. Fue diagnosticada de una infección por citomegalovirus y por virus Coxsackie B6, desde
entonces permaneciendo asintomática y sin tratamiento. La paciente consultó por dolor abdominal de 6 horas de evolución, iniciado en
epigastrio y posterior migración a fosa ilíaca derecha, intenso, de características continuas, con vómitos, sin fiebre ni otros síntomas. A
la exploración presentaba febrícula, defensa y signos de irritación peritoneal en hemiabdomen derecho. Analítica: leucocitos 16.330/μl,
neutrófilos 76,7%, hematocrito 39%, ALT 59 UI/l, GGT 19 UI/l, amilasa 354 UI/l, PCR 0,7mg/dl

PREGUNTA
Cual es la conducta inmediata mas adecuada?

RESPUESTA
a.- Realizar USG pélvico.
b.- Realizar prueba de embarazo.
c.- Realizar RX de abdomen.
d.- Mantener medidas generales.

PREGUNTA
Posterior a la medida realizada se realizar RX de abdomen siendo negativa, y USG con liquido libre en cavidad, cual es la conducta a
seguir?

RESPUESTA
a.- Mantener conducta expectante.
b.- Realizar TAC.
c.- Preparar para LAPE.
d.- Lavado peritoneal

PREGUNTA
Cual es su impresión diagnostica mas probable en este momento?

RESPUESTA
a.- Colecistitis.
b.- Apendicitis.
c.- Embarazo ectópico.
d.- Ulcera perforada.

CASO CLINICO
Se trata de femenino de 85 años, ingresa por dolor epigástrico de 12 horas de evolución de inicio brusco y continuo, antecedentes de
hipertensión arterial y bronquitis crónica sin tratamiento con corticoides, EF no signos de sepsis, abdomen blando depresivo, doloroso
en la región superior, no datos de irritación peritoneal, Hto 41, leucos 11,900 glucosa 128, urea 23, CK 78, DHL 385, resto normal. USG
abdominal mostro microlitiasis con vesicula biliar sin dilatación.

PREGUNTA
Cual es la conducta a seguir mas adecuada?

RESPUESTA

CURSO ENARM CMN SIGLO XXI TEL: 36246001 Pharmed Solutions Institute PÁGINA 500
MANUAL DE TRABAJO DEL CURSO ENARM CMN SIGLO XXI
a.- Realizar endoscopia.
b.- Conducta expectante.
c.- Liquidos, analgésico y antibióticos.
d.- Colecistectomia laparoscópica.

COLEDOCOLITIASIS. CIENCIAS BASICAS: El factor común presente en la gran mayoría de las enfermedades de la vía biliar es la
Colelitiasis. La complicación más frecuente, la Coledocolitiasis, se ha reportado con cifras hasta de un 20%. Se define coledocolitiasis
como la presencia de cálculos biliares en el conducto Colédoco y/o en conducto hepático común. SALUD PUBLICA: Según datos
internacionales los cálculos biliares se encuentran en el 12% de los hombres y el 24% de las mujeres. La prevalencia aumenta con la
edad. Más de un 10% de pacientes portadores de colelitiasis presentan coledocolitiasis. La asociación entre colecistitis crónica litiásica y
coledocolitiasis es de aprox un 15%. La asociación entre colecistitis aguda y coledocolitiasis puede alcanzar hasta un 25%. PATOGENIA:
La mayoría de los cálculos coledocianos se originan de la vesícula biliar, de hecho su forma y composición son similares a la de los
cálculos vesiculares creciendo en el colédoco por aposición de colesterol; simultáneamente se produce una dilatación gradual de la vía
biliar que con los años puede llegar a un diámetro de 2 cm o más. Con menor frecuencia los cálculos coledocianos se originan en la
misma vía biliar, ello se observa en casos de estenosis del hepático común o el colédoco en los que se desarrollan cálculos mixtos o de
bilirrubinato de calcio. Este fenómeno desaparece si se corrige la estrechez (dilatación endoscópica) o se deriva la vía biliar dilatada al
duodeno o yeyuno. En un paciente colecistectomizado puede detectarse coledocolitiasis en el postoperatorio alejado, lo cual se debe a
que durante la operación no se sospechó ni se diagnosticó una coledocolitiasis concomitante. En este caso hablamos de Coledocolitiasis
residual cerrada. Cuando se detecta la coledocolitiasis en el postoperatorio de un paciente sometido a una colecistectomía y además
coledocostomía y por lo tanto, portando una sonda T situada en el colédoco, hablamos de Coledocolitiasis residual abierta. Cuando han
pasado varios años después de realizada la colecistectomía en un paciente y se detecta coledocolitiasis, hablamos de coledocolitiasis
cerrada de Neoformación. DIAGNOSTICO: Puede producir obstrucción del colédoco con impedimento parcial o total del paso normal de
bilis desde hígado al duodeno, dando lugar al denominado Síndrome de Ictericia Obstructiva. La mayoría de estos pacientes tienen
antecedentes biliares, ya sea de cólicos biliares en reacción a ingestión de alimentos grasos, colelitiasis asociada o demostrada
antecedentes de colecistectomía. Es característico, la presencia de dolor abdominal de tipo cólico ubicado en epigastrio o hipocondrio
derecho, que precede a la aparición de coluria e ictericia de piel y escleras. Posteriormente, puede aparecer hipocolia. Se asocia a
estado nauseoso y vómitos rebeldes. Puede prolongarse durante varias horas, y si se alivia con antiespasmódicos tiende a desaparecer
precozmente. La ictericia es fluctante, debido al mecanismo valvular que determina la obstrucción de la vía biliar por él o los cálculos
flotantes que se desplazan y eventualmente se impactan y se desimpactan sucesivamente dentro de ella. Se le puede confundir con un
cólico ureteral derecho; es útil tener presente en este caso que el paciente se encuentra muy inquieto, mientras que si el dolor es de
origen biliar, tiende a permanecer postrado en la cama. La coluria precede a la ictericia y el paciente ictérico orienta a etiología
obstructiva extrahepática, ya que esta se produce debido a que se elimina bilirrubina directa o conjugada a través de la orina. El
enclavamiento de un cálculo en la Ampolla de Vater da origen a una ictericia prolongada, que en los enfermos ancianos puede
complicarse con una enfermedad tubular aguda. A estos síntomas y signos se le agrega prurito, por depósitos de sales biliares en la piel.
En los pacientes seniles, la coledocolitiasis suele ser causa de anorexia y pérdida de peso. Conviene insistir en que muchos enfermos
con cálculos en el colédoco no tienen ningún síntoma que revele su presencia. Por ello es tan importante, durante la colecistectomía
electiva o de urgencia, la cuidadosa exploración radiológica de la vía biliar. Los métodos de diagnóstico por imagen de la coledocolitiasis
pueden ser preoperatorios e intraoperatorios. Dentro de los preoperatorios se encuentran: Ecografía (eco), Colangioresonancia (C-
RMN), Tomografía computarizada (TC), Colangiopancreatografía retrógrada endoscópica (CPRE), Ecoendoscopía (ECO-E), Colangiografía
endovenosa (CIV) y Colangiografía transparietohepática (CTPH). Los intraoperatorios: Colangiografía intraoperatoria (CIO) y ecografía
por laparoscopía (ECO-L). TRATAMIENTO: Si el paciente presenta coledocolitiasis asociada a colelitiasis y la presencia de cálculos
coledocianos ha sido establecida previamente a la intervención quirúrgica, el procedimiento más aceptado hoy día es intentar la
extracción de los cálculos vía endoscópica (CPER asociada a Papilotomía y extracción endoscópica de cálculos), para luego proceder con
una colecistectomía laparoscópica. Si el diagnóstico de coledocolitiasis ha sido establecido durante la realización de una colecistectomía
(mediante colangiografía intraoperatoria), se puede programar una extracción diferida de cálculos coledocianos por vía endoscópica. En
algunos centros se realiza ambos procedimientos en un solo tiempo en forma intraoperatoria. Si no se cuenta con CPER o el paciente
tiene contraindicación para cirugía laparoscópica, se puede resolver ambas situaciones con una cirugía clásica, realizando una
coledocostomía intraoperatoria para extraer los cálculos. Esta alternativa requiere dejar en la vía biliar un drenaje (sonda T) para evitar
una complicación post operatoria (biliperitoneo).

CASO CLINICO
Mujer de 56 años, sin antecedentes de interés, que ingresa por cuadro febril que se acompaña de dolor en hipocondrio derecho y
vómitos repetidos. Se diagnostica de colecistitis aguda y por ecografía se comprueba colelitiasis. Se trata con pauta antibiótica y la
paciente mejora, quedando afebril y asintomática. Una nueva ecografía de control demuestra la persistencia de la colelitiasis, por lo
que se decide tratamiento quirúrgico, realizándose colecistectomía. La intervención quirúrgica transcurre sin problemas y el
postoperatorio inmediato es correcto. La evolución en planta transcurre sin problemas, pero a la mañana del 5º día del postoperatorio
la paciente presente de manera súbita un cuadro clínico de disnea, taquipnea y taquicardia, apareciendo pocas horas después cianosis.
EF: Consciente y orientada, colaboradora, pero inquieta y con importante dificultad respiratoria. Cianosis ligera de piel y mucosas.
Auscultación pulmonar y cardiaca sin hallazgos patológicos. Abdomen blando y depresible, con apósitos de laparotomía limpios y
drenajes ya retirados. Extremidades sin edemas.

PREGUNTA
Cual es la complicación más probable en este caso?

RESPUESTA
a.- Nemonia.

CURSO ENARM CMN SIGLO XXI TEL: 36246001 Pharmed Solutions Institute PÁGINA 501
MANUAL DE TRABAJO DEL CURSO ENARM CMN SIGLO XXI
b.- Edema agudo pulmonar
c.- TEP.
d.- SDRA.

CANCER DE PANCREAS. CIENCIAS BASICAS: El carcinoma pancreático es el tumor periampular más común; las manifestaciones son
similares para todos. La gran mayoría de los tumores malignos son carcinomas ductales, adenocarcinomas. Es un tumor de gran
agresividad biológica. SALUD PUBLICA: Ocupa cuarto lugar como causa de muerte en Estados Unidos. Edad promedio de inicio es de 60
años, afecta más sexo masculino (2:1). Menos de 20% de los carcinomas es resecable al momento del diagnóstico. Tras cirugía
“curativa”: <20% sobreviven al año y <5% a los 5 años. Segundo tumor digestivo más
CLASIFICACION CANCER DE PANCREAS
frecuente. El T. digestivo con peor pronóstico, es el más letal y el fallecimiento se produce Tumores sólidos de páncreas exocrino:
en 95% de los casos. Supervivencia media del no tratado: 4-6 meses. ADENOCARCINOMA: adenocarcinoma (90%), carcinoma acinar y
Surge de los conductos en 90% de los casos y de los acinos en 10%. A menudo la mayor pancreatoblastoma
parte del tumor es estroma fibroso con una zona de pancreatitis. Cerca del 75 % se origina Tumores quísticos del páncreas exocrino:
neoplasia mucinosa quística, neoplasia mucinosa
en la cabeza o el proceso uncinado del páncreas y produce obstrucción biliar que hace papilar intraductal (TMPI*), neoplasia serosa
posible el diagnóstico más temprano, 15% en el cuerpo y 10% en la cola del páncreas. El quística, neoplasia sólida seudopapilar.
tumor puede invadir la vena porta o los órganos adyacentes o producir metástasis al hígado Tumores del páncreas endócrino (de los islotes):
o al peritoneo. Los ganglios linfáticos son positivos en 90% de los pacientes. A veces los insulinomas, gastrinomas, glucagonomas,
carcinomas ampular y duodenal se diagnostican como pancreáticos; que es posibles que somatotatinomas, VIPomas, PPomas, carcinoides
Mesenquimáticos y hematopoyéticos:
sean pequeños al momento de detectarlos. DIAGNOSTICO: Clínica; cerca de 75% con liposarcoma, leiomiosarcoma, fibrosarcoma,
carcinoma de la cabeza pancreática acude por ictérica obstructiva (82%), pérdida de peso linfoma, Swwanoma
(92%) y dolor abdominal (72%). El dolor es sordo, “como adolorimiento”, medioepigástrico Secundarios: metástasis de carcinoma de mama,
y a menudo se irradia a espalda, de intensidad severa y predominio nocturno, empeora con pulmón, renal y melanoma.
las comidas y con el decúbito supino. El dolor en espalda sugiere invasión retroperitoneal. Son frecuentes la anorexia, la fatiga y el
prurito. En la exploración se encuentra ictericia, hígado palpable (50-70%) y vesícula palpable (30%, si no es sensible y hay ictericia, el
diagnóstico es de cáncer pancreático: Signos de Courvoisier). Puede aparecer tromboflebitis migratoria recurrente (signo de
Trousseau). Esplenomegalia e hipertensión porta. El 20% de los pacientes padece diabetes de inicio reciente. En caso de carcinoma
ampular el dolor es menos frecuente, a menudo cólico, y la ictericia es intermitente. Los tumores de cuerpo y cola pancreáticos
producen síntomas en una fase más tardía de la enfermedad, por lo que la etapa es más avanzada al momento del diagnóstico. Las
metástasis al ganglio supraclavicular izquierdo (ganglio de Virchow), umbilical (ganglio de la hermana María José) y del piso pélvico
(repisa de Blumer), indican enfermedad incurable. Metástasis: Frecuentes en hígado, seguido de los ganglios linfáticos regionales,
peritoneo y pulmones. En las pruebas de laboratorio hay aumento en el nivel de bilirrubinas, en el de fosfatasa alcalina y solo un
aumento ligero en transaminasas. El CA19-9, con marcador tumor sérico con sensibilidad de 80% y especificidad de 90%, a veces ayuda.
Muchos pacientes se valoran primero con ultrasonido abdominal, pero la TAC espiral es más precisa. En algunos casos resulta útil la
CEPRE, sobre todo cuando no se encuentra masa alguna en la TAC. La aspiración con aguja fina está indicada cuando hay evidencia
topográfica de irresecabilidad de manera que el diagnostico tisular puede comprobarse sin una operación (la aspiración negativa, no
descrata la enfermedad). TRATAMIENTO: Se hará en función de poder o no extirparlos quirúrgicamente: Resecable: 10-20%. Avanzado
localmente (no resecable): 40%. Metastásico: 40%. Es necesario corregir pronto el estado nutricional, la anemia y estado de volumen,
además de valorar la función renal. La pancreatoduodenectomía (operación de Wipple) representa la única esperanza de curación para
los tumores de la cabeza pancreática y es más útil en caso de carcinoma localizado, ya sea ampular, duodenal o del colédoco distal. Los
pacientes con tumores irresecables obtienen cierto beneficio de la colecistoduodenostomía o coledocoduodenostomía y
gastroyeyunostomia paliativas. La inyección transoperatoria en el plexo celiaco alivia el dolor. El paciente con alto riesgo obtienen
beneficios con la CEPRE o el drenaje transhepático. La radioterapia combinada con 5-fluoracilo tiene cierto valor como coadyuvante y
paliativo. La Gemcitabina es el medicamento de quimioterapia más comúnmente utilizado para tratar el cáncer de páncreas.
Seguimiento de los pacientes: Cada 3 meses los primeros 2 años. Cada 4 meses en el 3er año. Cada 6 meses en el 4to y 5to años.
NEOPLASIAS QUÍSTICAS: El 5% de todos los tumores pancreáticos, estos tumores se originan en las células de los conductos, tienen
revestimiento epitelial. Las neoplasias serosas (microquisticas), son benignas como el CISTADENOMA SEROSO compuesto por múltiples
quistes pequeños de hasta 2 cm de diámetro con líquido claro, más en mujeres, alrededor de los 65 años, en general asintomático
localización más frecuente en el cuerpo y cola del páncreas. EL CISTADENOMA MUCINOSO es premaligno, lesiones multiloculares y
papilares >5cm, más frecuente en mujeres y puede evolucionar a CISTADENOCARCINOMA 1% de los cánceres de páncreas, son
tumores grandes de 20- 30cm, deben extirparse, ya que presentan buena supervivencia a los 5 años, son tumores de crecimiento lento,
tienen mejor pronóstico y deben tratarse en forma agresiva. A menudo se manifiestan por molestia vaga, menos de 10% presentan
ictericia con anorexia y pérdida de peso. Todas las lesiones macroquisticas deben extirparse. NEOPLASIA INTRADUCTAL PAPILAR
MUCINOSA: Proliferación de células mucinosas en forma de papilas a nivel ductal con hiperproducción de moco. Más en hombres, 68
años. Síntomas sugestivos de pancreatitis crónica. En la cabeza y proceso uncinado, aunque puede afectar difusamente la glándula
pancreática. 30-60% son malignas en el momento de su diagnóstico. Asociada a otros tumores malignos en un 23-36% de los casos.
NEOPLASIAS ENDOCRINAS: Se originan en las células de los islotes pancreáticos, representan aproximadamente el 15% de las
neoplasias pancreáticas, casi todas son malignas. Segregan hormonas, dependiendo de las células de las que proceden son muy raros.
Más frecuente el gastrinoma, que además representa el 10% de todos los tumores endocrinos gastropancreáticos, el segundo en
frecuencia es el insulinoma. INSULINOMA: Tumor pancreático endocrino funcionante más frecuente 50-60%, proviene de células β
pancreáticas, la mayoría en la cabeza del páncreas. 80% benignos. Habitualmente se presenta en la 5ta y 6ta década de la vida, más en
mujeres que en hombres (2:1). La hiperinsulinemia produce hipoglucemia grave y origina convulsiones, depresión y coma. Los síntomas
desaparecen poco después de administrar glucosa. Los criterios diagnostico clásicos, la triada de Wipple, incluyen hipoglucemia en
ayuno (<50mg/dl) durante los ataques, cambios en el SNC y síntomas de hipoglucemia desencadenado por el ayuno, así como reversión
de los cambios con administración de glucosa. La ubicación del tumor se facilita con la angiografía, toma de muestras venosas
selectivas, imágenes por TAC o centellografía con octreótido. El tratamiento es quirúrgico (Pancreatectomía distal, dejando 20-30% del
páncreas), excepto en enfermedad metastásica avanzada. GASTRINOMA: La triada original de Zollinger-Ellison incluye ulceras pépticas

CURSO ENARM CMN SIGLO XXI TEL: 36246001 Pharmed Solutions Institute PÁGINA 502
MANUAL DE TRABAJO DEL CURSO ENARM CMN SIGLO XXI
fulminantes de localización atípica, hipersecreción gástrica extrema y un tumor de células no beta de islote pancreático. El gastrinoma
puede comenzar como una enfermedad sencilla y terminar con complicaciones graves como perforación, obstrucción, hemorragia e
intratabilidad. La enfermedad no responde a tx., medico, ni quirúrgico. Es posible que los pacientes presentes diarrea de gasto alto con
esteatorrea. Dx., hay hipergastrinemia en ayuno (>200pg/ml). Su potencial maligno es >90%. El crecimiento es lento y la metástasis se
produce en etapa tardía. Con frecuencia las lesiones se encuentran en el triángulo del gastrinoma: 1) unión de los conductos cístico y
colédoco, 2) unión de la segunda y tercera porción del duodeno, 3) unión del cuello y el cuerpo pancreáticos. VIPomas: Tumor
productor de péptido intestinal vasoactivo, procedentes de las células D1. casiona diarrea, “cólera pancreático” y un síndrome
compuesto por diarrea acuosa, hipopotasemia y aclorhidria gástrica. Cerca de 50% corresponde a malignos, más común en hombres,
promedio 45 años, de gran tamaño. Provoca el Sx de Werner-Morrison o cólera pancreática: Diarrea hipersecretora + rubicundez (20%)
+ hipokalemia + aclorhidria. 50% de los pacientes con hiperglucemia (glucogenólisis hepática inducida por el VIP y la hipopotasemia).
Diagnóstico: Concentraciones séricas de VIP >200 pg/ml. TAC y USG. Tratamiento: Enucleaciones, pancreatectomías distales
laparoscópicas. GLUCAGONOMAS: Procedentes de células A, más frecuentes en mujeres, entre 50-70 años de edad, más en cuerpo y
cola. Causan lesiones cutáneas (eritema necrolítico migratorio, muy pruriginoso), diabetes, glositis, pérdida de peso depresión y
trombosis venosa. Las tres cuartas partes son malignos. El mejor tratamiento es la resección. Sx de las 4 D´s (Diabetes, Depresión,
Dermatitis, Deep vein trombosis). Diagnóstico: Glucagón sérico>500pg/ml. También puede detectarse hipoalbuminemia,
hipoaminoacidemia, anemia e hipocolesterolemia. TAC. El mejor tratamiento es la resección, Pancreatectomía distal con preservación
esplénica o pancreatoduodenectomía. SOMATOSTINOMAS: Extremadamente raros, en páncreas proximal, 60% ámpula (en células D).
Se manifiestan por diabetes, diarrea, esteatorrea, aclorhidria, calculos biliares, malabsorción, y dolor abdominal. Por lo general se
tratan con estreptozocina, dacarbacina y doxirrubicin. Diagnóstico: concentraciones séricas de somatostatina >100 pg/dl. TAC, RNM.
Tratamiento resección del tumor. PPoma: Es productor de polipéptido pancreático, es el tumor no funcionante más frecuente. No se
asocian con síndromes clínicos de hiperfunción hormonal. Entre la 5ta-6ta década de la vida. En el 50% se ubican en la cabeza, proceso
uncinado y cuello del páncreas.

CANCER DE PANCREAS.
Se trata de paciente masculino de 59 años de edad con antecedentes de DM 2 y tabaquismo positivo, el cual refiere diarrea crónica de
más de 6 meses de evolución, disminución de peso del 15 %, ha disminuido su ingesta de alimento ya que tiene sensación de plenitud y
vomito en varias ocasiones, se observa ictericia y piel verdosa, refiere heces grasosas y flatulencias, fue diagnosticado previamente con
síndrome de Peutz - Jeghers.

PREGUNTA
Cuál es la manifestación clínica más importante para distinguir cáncer páncreas de cabeza vs cuerpo y cola.

RESPUESTA
a.- Dolor de espalda.
b.- Coluria y acolia.
c.- Constipación.
d.- Dispepsia.

CASO CLINICO
Acude paciente de 61 años de edad el cual presenta pérdida de peso y sensación de plenitud abdominal, cuenta con antecedentes de
consumo de alcohol y tabaco de forma social sin llegar a la embriaguez, aproximadamente cada semana, cuenta con antecedentes de
cáncer en la familia sin especificación, además refiere que hace 20 años sufre accidente automovilístico con transfusión sanguínea,
hace 5 años fue diagnosticado con hepatitis C, actualmente presenta distención abdominal, sobrecarga de volumen y ascitis,
confirmando cirrosis desde entonces, se encuentra con tratamiento con espironolactona mas furosemida, se realiza radiografia
abdominal donde se aprecia lesión en lóbulo derecho de 5 cm aproximadamente, se indica TAC donde se confirma lesión de 4,6 cm en
lóbulo derecho cercana a pediculos portales principales, se solicita fetoproteina alfa con 384 ng/ml de resultado, se programa biopsia
percutánea.

PREGUNTA
Considerando los antecedentes y el cuadro clínico actual, cual es el diagnostico más probable.

RESPUESTA
a.- Adenocarcinoma hepático.
b.- Carcinoma hepatocelular.
c.- Carcinoma metastasico.
d.- Colangiocarcinoma.

COLANGIOCARCINOMA. CIENCIAS BASICAS: Es un tumor maligno originado en el epitelio de los conductos biliares intra o
extrahepáticos, se puede ubicar a cualquier nivel de la via biliar, desde los canalículos hasta la ampolla de Vater. Cerca de dos terceras
partes de las lesiones, se localizan en los conductos proximales, a menudo en la confluencia de los conducto hepático derecho e
izquierdo) La mayoría de los colangiocarcinomas son del tipo histológico adenocarcinoma ductal, sin embargo, suelen verse otros como
los adenocarcinomas papilar, mucinoso, mucoepidermoide y el cistoadenocarcinoma. SALUD PUBLICA: Es poco frecuente, representa
menos del 2% de todos los tumores malignos de diagnóstico reciente. La incidencia en Estados Unidos es baja, de 1.0/100.000 por año y
es un poco más alta en Israel y Japón. Se presenta especialmente después de la sexta década de la vida y es ligeramente más frecuente
en hombres que en mujeres. CLASIFICACION: Anatómicamente distinguimos el colangiocarcinoma intrahepático (20% a 25%), el
perihiliar (50% a 60%), el extrahepático distal (20% a 25%) y el multifocal (5%). El extrahepático también se clasifica en el del tercio

CURSO ENARM CMN SIGLO XXI TEL: 36246001 Pharmed Solutions Institute PÁGINA 503
MANUAL DE TRABAJO DEL CURSO ENARM CMN SIGLO XXI
superior de la vía biliar (hepático común, confluencia hiliar o hepáticos derecho e izquierdo), el del tercio medio (colédoco hasta un
plano dado por el borde superior del duodeno) y el del tercio inferior (desde el plano del borde superior del duodeno hasta la ampolla
de Vater). Se denomina TUMOR DE KLATSKIN al colangiocarcinoma situado en la confluencia hiliar. Según su patrón de crecimiento,
puede ser exofitico, polipoideo e infiltrativo. El tumor de Klatskin es de tipo infiltrativo con invasión maligna periductal. El polipoideo
tiene crecimiento endoluminal. PATOGENIA: La etiología no está bien definida, hay algunos procesos patológicos predisponentes, entre
ellos la colangitis esclerosante primaria, la litiasis intrahepática, la enfermedad de Caroli y malformaciones como la atresia de vía biliar y
los quistes de colédoco. También existe asociación con tóxicos cancerígenos de origen industrial como digoxinas, nitrosaminas y
asbesto . Además, se le asocia a infestación crónica de la vía biliar con parásitos endémicos del sudeste asiático como clonorchís
sinensis y opisthorchis viverrin. La infección crónica con salmonella typhi sería otro factor predisponente. Es dudoso que la colelitiasis y
la colecistectomía previa predispongan al desarrollo de colangiocarcinoma. Las alteraciones moleculares también se han asociado al
colangiocarcinoma, como la inactivación de oncosupresores (p53, pl6, bel-2) y la mutación de oncogenes (K-ras, c-myc, c.erbB-2). Sin
embargo, no son hallazgos específicos. DIAGNOSTICO: Clínica; dolor en hipocondrio derecho, ictericia y baja de peso. El signo más
importante es la ictericia progresiva, presente en alrededor de 90%, muchas veces precedida de prurito. También hay dolor no muy
intenso en epigastrio e hipocondrio derecho. El signo de Courvosier-Terrier puede estar presente en los colangiocarcinomas ubicados
en el tercio inferior del colédoco. En algunos pacientes asintomáticos, las fosfatasas alcalinas están aumentadas de 1 a 5 veces y las
transaminasas de 1 a 2 veces. Los marcadores tumorales CA 19-9, CA-125 y CEA están aumentados en el colangiocarcinoma en 85%,
40% a 50% y 30%, respectivamente. No obstante, estos marcadores tumorales también se elevan en otras patologías malignas y en
lesiones hepáticas graves. Un estudio comparativo entre CA 19-9 y CEA realizado por autores chinos, demuestra que el CA 19-9 es más
útil para el diagnóstico del colangiocarcinoma. Actualmente el diagnóstico se ha facilitado por la disponibilidad de variados
procedimientos imagenológicos y endoscópicos. La colangiografía, localiza el sitio exacto de la obstrucción. El hígado y los ganglios
linfáticos son los sitios más comunes de metástasis. TRATAMIENTO: Lo ideal es la resección quirúrgica completa del tumor con
márgenes histológicos negativos es la única terapia curativa. Desafortunadamente, la mayoría de las veces el tumor es irresecable. Para
esos casos hay posibilidad de paliación mediante la colocación de endoprótesis por vía endoscópica o percutánea. Los pacientes con
colangiocarcinoma hiliar en un bajo porcentaje alcanzan a ser tratados con cirugía potencialmente curativa, fundamentalmente por tres
motivos: por diagnóstico tardío, por infiltración neoplásica de la vasculatura del hilio hepático, terapia de elección la resección de la vía
biliar con o sin resección hepática más linfadenectomía y anastomosis biliodigestiva. La mortalidad operatoria de las diferentes series
de cirugía resectiva es de alrededor de 10%. El trasplante hepático, últimamente ha surgido como otra opción válida para tratar
pacientes con colangiocarcinoma hiliar y periférico considerados irresecables pero no diseminados. La curación de los tumores de la
parte distal del coledoco mejoro con la linfadenectomia radical y la pancreatoduadenectomía (Procedimiento de Wipple).

CASO CLINICO
Se trata de paciente femenino de 56 años de edad la cual acude a consulta por malestar generalizado, fatiga y coloración amarillenta, la
paciente es originaria del distrito federal, fumadora durante 15 años así como consumo de alcohol tipo social cada fin de semana,
cuenta con antecedente de intolerancia a la glucosa con IMC de 27 la cual se encuentra con tratamiento con dieta, ejercicio y
metformida con resultados leves, el padecimiento actual inicia hace una semana caracterizado por deposiciones con perdida de
consistencia y coloración disminuida, palidas, agrega prurito generalizado, coloración amarillenta de piel y escleras, a la exploración
física se encuentra apática, afebril, con signos vitales dentro de rango normal, se palpa hepatomegalia dudosa, se ingresa a observación
donde se realiza estudios de laboratorio con AST 119, ALT 158, Fosfatasa Alcalina de 476 UI/L, bilirrubina total de 23 mg/dl, y bilirrubina
directa de 20 mg/dl, se indica ultrasonografia no se observa vesicula biliar, se observa dilatación de conductos biliares intrahepaticos,
pero no se muestra dilatación del colédoco, resto de estructuras sin datos por agregar.

PREGUNTA
Cuál es su impresión diagnostica más probable con los estudios realizados.

RESPUESTA
a.- Cáncer pancreático.
b.- Cáncer de vesicula.
c.- Cáncer hepático.
d.- Colangiocarcinoma.

CASO CLINICO
Se trata de paciente femenino de 47 años de edad, inicia padecimiento hace 2 años caracterizado por dolor en cuadrante superior
derecho, que se presentaba posterior a la ingesta de alimentos, ha sido tratada con analgesicos y antiespamodicos diversos sin
embargo desde hace 6 meses ya no han sido controlados e incluso se encuentra con un estado nauseoso continuo, tolera poco
alimento, a la exploración se observa IMC 17, se observa ictericia importante.

PREGUNTA
Cuál es la conducta a seguir más adecuada.

RESPUESTA
a.- Endoscopia.
b.- Serie gastroesofagica.
c.- USG.
d.- Radiografía simple de abdomen.

CURSO ENARM CMN SIGLO XXI TEL: 36246001 Pharmed Solutions Institute PÁGINA 504
MANUAL DE TRABAJO DEL CURSO ENARM CMN SIGLO XXI
CASO CLINICO
Se trata de paciente femenino de 21 años de edad la cual presenta malformación cística del colédoco la cual se identifico por USG
cuando la paciente refirió dolor en cuadrante superior derecho.

PREGUNTA
Se decide realizar resección quirúrgica, cual es el motivo más importante para la decisión.

RESPUESTA
a.- Degeneración maligna.
b.- Colangitis recurrente.
c.- Cirrosis biliar.
d.- Coledocolitiasis.

CURSO ENARM CMN SIGLO XXI TEL: 36246001 Pharmed Solutions Institute PÁGINA 505
MANUAL DE TRABAJO DEL CURSO ENARM CMN SIGLO XXI
PANCREATITIS AGUDA Y CRONICA:
PANCREATITIS AGUDA (PA). CIENCIAS BASICAS: Enfermedad sistémica caracterizada por un proceso inflamatorio agudo no bacteriano
que resulta de la liberación de las enzimas pancreáticas dentro de la propia glándula, que puede comprometer por continuidad otros
tejidos y órganos vecinos. Usual curso benigno. PAL: mínima disfunción multiorgánica, evolución sin complicaciones. PAG: presencia de
falla orgánica o sistémica y/o presencia de complicaciones locales. SALUD PUBLICA: 20% desarrolla enfermedad severa. 20% de
mortalidad. 4-6ta. Década de la vida. Cerca de 40% de los
casos se debe a cálculos biliares, otro 40% se debe a
alcoholismo. PATOGENIA: El páncreas normalmente secreta
una gran cantidad de enzimas, entre ellas la tripsina, la
quimiotripsina, las amilasas, lipasas, elastasas,
carboxipeptidasa A y B, etc. La gran mayoría de ellas son
enzimas líticas que normalmente se secretan en forma de
precursores inactivos, los cuales junto con inhibidores
enzimáticos presentes en el jugo pancreático, protegen al
páncreas de su autodigestión. Los eventos iniciales
patogénicos de la Pancreatitis Aguda no están del todo
aclarados. Existen evidencias de que la presencia de uno o
más de los siguientes factores, aumento de la presión
intraductal en el sistema excretor pancreático (litiasis biliar,
tumores, páncreas divisum o anular), reflujo de bilis al
Wirsung, hipertrigliceridemia aguda o preexistente, fármacos (aziatropina, clortiazida, estrógenos, furosemida, sulfamidas,
tetraciclinas), infecciones (hepatitis, parotiditis, CMV, candida, áscaris), daño isquémico, post-CEPRE, disrupción de algún conducto
excretor (trauma), etc., desencadena una cascada inflamatoria que se asocia a la activación enzimática intraglandular, con las
consecuencias de daño microvascular, trombosis, necrosis tisular, saponificación del tejido graso, liberación de radicales libres y
eventualmente gatillamiento de una respuesta inflamatoria sistémica. PA Y LITIASIS BILIAR: Es más frecuente la PA en pacientes con
cálculos vesiculares pequeños, coledocolitiasis, conducto cístico ancho, conducto bilio pancreático distal común, se inicia por la
obstrucción del conducto pancreático por un cálculo en la ampolla de Vater, aunque sea una obstrucción transitoria, con aumento de la
presión intraductal, reflujo biliar a los conductos pancreáticos y eventual contaminación bacteriana por este mecanismo. Las sales
biliares desconjugadas y la lisolecitina son toxicas para el páncreas. PA Y ALCOHOL: La ingesta alcohólica excesiva de tipo inveterada es
un factor asociado a crisis de PA a repetición, que si bien el primer episodio puede ser de gravedad, las crisis posteriores son
clínicamente menos importantes y que llevan al cuadro de la pancreatitis crónica CRITERIOS DE RANSON PARA PA
recurrente. En las teorías se ha postulado: (1) Un aumento de la secreción gástrica que Presentes al momento del ingreso:
conduce a un aumento exagerado de la secreción pancreática; (2) Inflamación duodenal y Edad >55años
Cuenta leucocitaria >16 000/µl
periampular con obstrucción de los conductos bilio-pancreáticos; (3) La hiperlipidemia
Glucosa sanguínea >200mg/dl
aguda inducida por el alcohol. En la mayoría de los casos la PA se produce 24 a 48 horas Deshidrogenasa láctica sérica >350 UI/L
después de la ingesta, siendo poco frecuente el encontrar niveles altos de alcoholemia AST >250UI/dl
simultáneamente al evento clínico de la PA. 4. P.A. e hipertrigliceridemia: DIAGNOSTICO: Se desarrollan durante las primeras 48 hrs
Clínica; produce dolor intenso epigástrico (estiramiento de la capsula pancreática) Descenso del hematocrito >10%
Aumento >8mg/dl en BUN
transfictivo, que se irradia a la espalda, se alivia al sentarse y a menudo se acompaña de Calcio sérico menos de 8mg/dl
nausea intensa, puede haber relación con ingesta de alcohol o grasas. También se PO2 arterial >60mmHg
observa sensibilidad abdominal superior y defensa muscular. Alrededor de 90% de los Deficiencia de base >4mEq/l
pacientes tiene fiebre, leucocitosis y taquicardia. El íleo es usual y también es posible que Secuestro estimado de líquido >600ml
ocurra choque por secuestro de líquido y depresión miocárdica. La ictérica se presenta en 0-2 = 0.9% de mortalidad; 3-4 = 16% ;
5-6 = 40% ; >6 = 91 de mortalidad
20-30% de los pacientes, por obstrucción o compresión del colédoco. En ocasiones hay
espasmo carpopedal por hipocalcemia. Alrededor de 1% de los pacientes tiene sangre retroperitoneal alrededor de la cicatriz umbilical
(Signo de Cullen) o en los flancos (Signo de Grey- Turner). Pancreatitis + Fiebre persistente + Resp. Inflamatoria sistémicapensar en
absceso o infección. Laboratorio y gabinete: pueden presentar hiperamilasemia (3 veces lo normal, aumento 2-5 días), manifestación
muy inespecífica (elevada en colangitis, colecistitis, ulcera péptica perforada, salpingitis, IRC, etc). La determinación sérica de lipasa es
más específica (aumenta a partir de las 72 h, regresa lentamente, mejor prueba diagnóstica). La depuración urinaria de amilasa puede
ser útil. El nivel de calcio disminuye a veces (<7.5mg/dl, mal pronóstico, secuestro de calcio en las áreas de necrosis grasa).
Hiperglucemia (aumento transitorio, liberación de glucagón aumentada, glucemia > 200 mg/dL signo de mal pronóstico = necrosis
extensa). PCR: su elevación va en relación con el mal pronóstico. La radiografía puede mostrar un asa centinela con aire intestinal.
Paciente con diagnóstico de PA: hacer ecografía abdominal (litiasis biliar) en las primeras 24-48 horas. La imagen por TAC ayuda a
predecir la gravedad y a establecer el diagnostico de las complicaciones. Índice de severidad por TAC, CRITERIOS DE BALTHAZAR: A
Páncreas normal (0 puntos). B Aumento de tamaño focal o difuso del páncreas (1 punto). C Inflamación del páncreas y/o grasa
peri/pancreática (2 puntos). D Colección pancreática única (3 puntos). E dos o más colecciones peripancreáticas y/o gas
retroperitoneal (4 puntos). Porciento de necrosis: 0% =0 puntos, 30% = 2 puntos, 30-50% = 4 puntos, >50% =6 puntos. Suma de puntos
en TAC = índice de severidad. 0-3 bajo, 4-6 medio, 7-10 alto. TRATAMIENTO: La reposición de las perdidas hidroelectrolíticas es parte
crucial de la atención a estos pacientes, así como la vigilancia de volumen vascular, las valoraciones repetidas de hematocrito y
electrolitos y el reposo intestinal. El uso rutinario de antibióticos en pancreatitis no complicada no se justifica. La colecistectomía está
indicada en la pancreatitis biliar. El lavado peritoneal puede disminuir las complicaciones cardiopulmonares, pero no reduce la
mortalidad. La intervención quirúrgica es necesaria en casos de cálculos impactados, desbridación de necrosis y drenaje de abscesos.
COMPLICACIONES: La complicación más frecuente es el pseudoquiste (colección liquida), que suele presentarse después de 2-3
semanas, puede ser único o múltiple, tx., drenaje quirúrgico, el 50% se resuelven de manera espontánea a las 6 semanas. Necrosis
pancreática temprana y tardía (Dx: punción por aspiración, tratamiento: desbridamiento quirúrgico). Absceso pancreático (4-6
semanas, Dx: punción aspiración, tratamiento: drenaje percutáneo). Infección: principal causa de morbimortalidad, aparece de 2-3

CURSO ENARM CMN SIGLO XXI TEL: 36246001 Pharmed Solutions Institute PÁGINA 506
MANUAL DE TRABAJO DEL CURSO ENARM CMN SIGLO XXI
semanas después, deben usarse antibióticos que penetren en páncreas, primera elección imipenem, también cefalosporinas de tercera
generación y quinolonas, mínimo 14 días, o hasta que no haya complicaciones sistémicas. Las complicaciones sistémicas incluyen SDRA,
insuficiencia renal y depresión miocárdica.

CASO CLINICO
Ingresó paciente de sexo masculino de 68 años de edad, con antecedentes de Diabetes Mellitus en tratamiento con hipoglicemiantes
orales, colecistectomía y alcoholismo en remisión desde hacía 15 años. Presentaba dolor abdominal, inicialmente en epigastrio e
hipocondrio izquierdo de 20 h de evolución, el cual había ido progresando en intensidad y se extendía ya al resto del abdomen. La
aparición del dolor fue brusca y se asoció a sudoración fría y sensación de desvanecimiento, sin pérdida de conciencia. No se acompañó
de vómito ni diarrea. En el examen físico del ingreso destacaba un paciente consciente, algo pálido, afebril, con taquicardia leve, una
presión arterial de 110/60 mm de Hg sin apremio respiratorio y adecuada perfusión periférica. No había estigmas de daño hepático
crónico. En el examen pulmonar había disminución del murmullo pulmonar en ambas bases, con predominio izquierdo. En el examen
abdominal se evidenciaba cierto abombamiento, la pared era depresible pero la palpación provocaba intenso dolor con signo de
Blumberg positivo en hemiabdomen superior. Los ruidos hidroaéreos estaban ausentes. No se palpaban masas pulsátiles.

PREGUNTA
Considerando el cuadro clínico cual es su impresión diagnostica inicial?

RESPUESTA
a.- Ulcera perforada.
b.- Pancreatitis aguda.
c.- Sindrome hepatorrenal.
d.- Ruptura de varices esofágicas.

PREGUNTA
Se obtuvieron los siguientes resultados de laboratorio posteriomente, hemoglobina 9 g/L, recuento leucocitario de 8000/µL,
protrombina 55%, bilirrubina 1,4 mg/dL, albúmina 2,3 mg/dL; amilasemia, lipasemia y proteína C reactiva en rangos normales. Los
estudios radiológicos de tórax mostraron atelectasia en ambas bases pulmonares, sin neumoperitoneo y la radiografía de abdomen
simple mostró aire en el colon, escasa cantidad en intestino delgado y asas no dilatadas, considerando los datos observados que
puntaje presenta de la clasificación de Child-Pugh.

RESPUESTA
a.- Menos de 5 puntos.
b.- De 5 a 7 puntos.
c.- De 7 a 10 puntos.
d.- Mas de 10 puntos.

PREGUNTA
El paciente se manejó con aportes de cristaloides y evolucionó con disminución del dolor y sin compromiso hemodinámico. En los
exámenes de control al segundo día de hospitalización se observó disminución de la hemoglobina a 5 g/L, protrombina 54% y recuento
plaquetario de 59.000/µL. DHL 450 U/l, AST 140, Glucosa de 250 mgs/dl. Cuantos criterios de Ramson presenta?.

RESPUESTA
a.- 2 criterios.
b.- 3 criterios.
c.- 4 criterios.
d.- 5 criterios.

PREGUNTA
Cual de los siguientes diagnosticos diferenciales es el menos probable en este caso clínico?

RESPUESTA
a.- Hemoperitoneo espontáneo en cirrosis.
b.- Rotura de várices o canales linfáticos.
c.- Rotura de carcinoma hepatocelular
d.- Rotura de aneurisma abdominal.

CURSO ENARM CMN SIGLO XXI TEL: 36246001 Pharmed Solutions Institute PÁGINA 507
MANUAL DE TRABAJO DEL CURSO ENARM CMN SIGLO XXI
PANCREATITIS CRONICA. CIENCIAS BASICAS: La definición es vaga, pero por lo general incluye cambios secundarios a los episodios
repetidos de pancreatitis aguda. El páncreas se vuelve, pequeño, indurado y nodular, con acinos e islotes rodeados por tejido fibroso.
Se observa estrechamiento y dilación de los conductos; la calcificación es frecuente,
los cambios estructurales son irreversibles y progresivos. SALUD PUBLICA:
Incidencia oscila entre 5-10 casos nuevos anuales por 100,000 hab y prevalencia de DAÑO POR
30-40 por 100,000 hab. Algunos estudios elevan estas cifras hasta 5% de la ALCOHOL
población general. CLASIFICACION: TIGAR-O; Toxica-metabólica que es la más
frecuente porque incluye el alcohol (70-90%), también la hipercalcemia, y algunos
mencionan la falla renal crónica. Idiopática alcanza de 10-30% se han agrupado en
una forma juvenil que se presenta en las primeras décadas de la vida y una forma
en 4ta-5ta década. Genética, se han descrito algunas mutaciones, en el tripsinogeno
catiónico, CFTR, SPINK1. Autoinmune es una entidad poco frecuente con hallazgos
clínicos y paraclínicos definidos e histológicamente su característica es el extenso
infiltrado linfoplasmocitario. Recurrente, en el que se incluyen pacientes que
posiblemente debido a múltiples episodios de pancreatitis aguda pueden conducir a
pancreatitis crónica, sin embargo existe controversia. Obstructiva, plantea el daño
primario sobre los conductos pancreáticos, posiblemente por daño inmunológico
sobre el epitelio, en forma similar a los que se observa en la colangitis esclerosante
primaria. Páncreas divisum, deficiencia del esfínter de Oddi, traumatismos.
PATOGENIA: La células estrelladas tienen un papel en la fibrosis pancreática, en el páncreas se localizan en la porción exocrina
(espacios periacinar, perivascular), en condiciones normales, por sus propiedades contráctiles ayudan a regular, las presiones en estos
compartimentos, además pueden contribuir a mantener la matriz extracelular y tienen capacidades limitadas de migración y
proliferación. De acuerdo a la teoría conocida como necrosis-fibrosis, se plantea que la necrosis y la inflamación pancreática son
procesos que activan a las células estrelladas, por factores denominados autocrinos por medio de moléculas proinflamatorias que
además aumentan la secreción de colágeno y la expresión de TGF-B1 y endotelina, que estimulan la migración, la contracción y la
liberación de sustancias proinflamatorias, perpetuando sus efectos aun cuando el estímulo inicial haya terminado, favoreciendo el
desarrollo progresivo de fibrosis, el daño repetido y persistente en la pancreatitis crónica lleva a una activación continua de células
estrelladas que finalmente conduce a fibrosis de la glándula y perdida de sus funciones exocrinas. Existe una evidencia clara sobre la
capacidad del alcohol para inducir lesión pancreática directa. Los alcohólicos ingieren dietas deficientes en numerosos micronutrientes,
especialmente selenio y zinc, lo que podría ser origen de mecanismos celulares de lesión pancreática inducidos por la acción de
radicales libres de oxígeno. Además también produce fenómenos de hipertensión intraductal, disminución del flujo sanguíneo
pancreático, toxicidad directa sobre la célula acinar, cambios en la síntesis proteica, incremento de la respuesta inflamatoria y
estimulación de la fibrogenesis pancreática. DIAGNOSTICO: Clínica; Los síntomas comprenden dolor epigástrico o en espalada continuo
o intermitente, anorexia y maldigestión manifestada por pérdida de peso, esteatorrea. La diabetes puede desarrollarse a lo largo de la
enfermedad y se caracteriza por destrucción de células productoras de insulina y productoras de glucagón; la coexistencia de este daño
aumenta el riesgo de hipoglicemia, por la deficiencia concomitante de síntesis de glucagón. Durante los ataques agudos hay vomito. A
menudo se forman pseudoquistes. También son frecuentes las alteraciones de la personalidad. Según su etiología pueden ocurrir otras
manifestaciones como fenómenos de autoinmunidad u hepatopatía. Laboratorios y gabinete: la presencia de cambios morfológicos en
pruebas de imagen y/o en la demostración de insuficiencia pancreática. La CPRE y las imágenes con TAC, RMN ayudan y es probable
que muestren dilatación ductal, cálculos y estrechamientos. Los niveles séricos de amilasa y lipasa tienen poco valor. La calcificación
pancreática es patognomónica. Histológicamente se requiere la presencia de fibrosis y atrofia acinar que se acompaña de un
componente variable de infiltrado inflamatorio crónico (se dispone pocas veces de estudio histológico). TRATAMIENTO: Suprimir el
consumo de alcohol. Analgésicos en forma escalonada: AINES, opiáceos débiles (dextropropoxifeno, tramadol), combinaciones:
dextropropoxifeno + dipirona (Klosidol®); dextropropropoxifeno + ibuprofeno (Supragesic®). Si no hay respuesta: agregar amitriptilina.
Tratamiento de la maldigestión: Reemplazo enzimático: 25.000 a 50.000 U de lipasa en cada comida. IBP (Omeprazol). La
pancreatoyeyunostomía (procedimiento de Puestow), es conveniente en casos de pancreatitis alcohólica crónica, dolor crónico y
alternancia de dilatación con estrechamientos ductales (cadena de lagos). Se obtienen buenos resultados con la
pancreatoduodenectomía cuando la pancreatitis crónica se acompaña de estrechamiento del conducto en la cabeza del páncreas. Si el
conducto es pequeño, se obtienen cierto beneficio con la pancreatectomía de 95%, pero la mortalidad es alta. La abstinencia de alcohol
resulta primordial. El dolor crónico es difícil de tratar, si los síntomas persisten o recurren se puede hacer un bloqueo celiaco.
COMPLICACIONES: Pseudoquistes complicados; la estenosis biliar, duodeno o colon; la trombosis venosa (esplénica, mesentérica-
portal); los pseudoaneurismas; la hemorragia intraquistica, intrabdominal o digestiva; las fistulas internas o externas; las ascitis y el
cáncer de páncreas.

CASO CLINICO
Varón de 66 años, con antecedentes de obesidad, HTA, diabetes mellitus tipo 2, ex fumador, síndrome de apnea obstructiva del sueño
con espirometría con patrón mixto, herniorrafia inguinal, hiperplasia benigna de próstata, HDA por úlcera prepilórica y pancreatitis
litiásica leve. Acude por dolor abdominal intenso, de 2 h de evolución, de inicio brusco en hemiabdomen superior, que no cede con
analgésicos y se acompaña de vómitos. Mantiene buenas constantes vitales (presión arterial, 130/80 mmHg; frecuencia cardíaca, 96
lat/min; frecuencia respiratoria 20/min, y temperatura, 36°C). En la analítica destacan 18.000 leucocitos; amilasa, 5.900; AST, 169; ALT,
105, y BT, 2,5g/dl; PCR, 0,8. En la ecografía abdominal se visualiza una vesícula distendida con barro biliar sin dilatación de la vía biliar.
No se observa bien el páncreas, pero se ve aumentado de tamaño e hipoecoico. Tras 12 h, empeora súbitamente su estado clínico, con
agitación, sudor, taquipnea, livideces y agudización del dolor abdominal (presión arterial, 90/60 mmHg; frecuencia cardíaca, 160
lat/min). En la analítica presenta 15.500 leucocitos; glucosa, 332 mg/dl; creatinina, 1,7 mg/dl; PCR, 6,8 mg/dl. El abdomen está
distendido sin peritonismo. Su situación clínica se agrava con hipotensión, mala perfusión periférica, bradicardia extrema y asistolia que

CURSO ENARM CMN SIGLO XXI TEL: 36246001 Pharmed Solutions Institute PÁGINA 508
MANUAL DE TRABAJO DEL CURSO ENARM CMN SIGLO XXI
precisa RCP. Hay deterioro analítico: leucocitos, 4.470; creatinina, 2,5mg/dl; pH 7,23, y bicarbonato, 10,9mmol/l, y precisa
noradrenalina a altas dosis (30) y FiO2 al 100%.

PREGUNTA
Cuál es su impresión diagnostica más probable con los estudios realizados.

RESPUESTA
a.- Necrosis pancreática grave con acidosis metabolica.
b.- Pancreatitis crónica agudizada de origen alcohólica
c.- Colangitis aguda bacteriana secundaria.
d.- Colecistitis crónica litiasica con obstrucción pancreática.

CASO CLINICO
Varón de 79 años; como antecedentes personales de hipertensión arterial, fibrilación auricular y varios episodios de hemorragia
digestiva por angiodisplasias yeyunales tratadas con gas argón. Ingresa por nuevo episodio de hemorragia digestiva de probable origen
yeyunal. Los hallazgos fueron la identificación en yeyuno proximal de 3 angiodisplasias de tamaño milimétrico que se electrocoagularon
con argón, también un pequeño pólipo de 3 mm que se extirpó con pinzas. El día siguiente del procedimiento, el paciente comenzó con
dolor abdominal intenso y náuseas. La exploración abdominal reflejaba un abdomen globuloso y distendido con dolor y defensa en
epigastrio y flanco izquierdo. En la analítica urgente presentaba 16.400 leucocitos con 88% neutrófilos, una amilasemia de 410 y lipasa
148. Se realiza TAC abdominal urgente que describe hallazgos compatibles con pancreatitis.

PREGUNTA
Cuál es factor de severidad más grave para este caso.

RESPUESTA
a.- Edad mayor a 50 años.
b.- Índice de masa corporal mayor a 30.
c.- Presencia de derrame pleural.
d.- Valores de amilasa y lipasa.

CURSO ENARM CMN SIGLO XXI TEL: 36246001 Pharmed Solutions Institute PÁGINA 509
MANUAL DE TRABAJO DEL CURSO ENARM CMN SIGLO XXI
ISQUEMIA MESENTERICA AGUDA (IMA). CIENCIAS BASICAS: Entidad clínica multietiológica, producida por la interrupción brusca del
aporte sanguíneo a un determinado segmento intestinal, lesionándolo al principio de forma reversible pero que si se mantiene el
tiempo suficiente, hace que deje de ser viable y evolucione hacia la necrosis completa de su pared. SALUD PUBLICA: Supone un tercio
de las isquemias intestinales. Proceso con elevada mortalidad (60-80%) y en el cual es vital la sospecha clínica temprana, ya que la
supervivencia depende directamente de la rapidez con que se aplique el tratamiento. PATOGENIA: Oclusión Arterial (embolia arterial,
trombosis arterial): La arteria más frecuentemente implicada es la mesentérica superior (AMS), el tronco celiaco y la mesentérica
inferior (AMI) también pueden ocluirse, pero gracias a la circulación colateral, no suelen producir lesión isquémica aguda, a menos que
se afecte a la vez la AMS. Oclusión Venosa (trombosis venosa mesentérica): La imposibilidad de retorno venoso produce edema y
aumento de presión venosa que cuando se iguala a la arterial
produce isquemia e infarto hemorrágico. Isquemia no oclusiva
(IMNO): el bajo gasto, la hipotensión o la vasoconstricción local
mantenidos pueden impedir el aporte sanguíneo mínimo necesario
para la viabilidad intestinal. CLASIFICACION: 1. EMBOLIA ARTERIAL:
es la causa más frecuente de IMA (50%) Su origen en un 90% de los
casos es una cardiopatía embolígena y puede existir historia de
embolias previas a otros niveles. La zona más frecuente de
embolización es la AMS, distalmente a la salida de la A. Cólica Media,
afectando al colon derecho y área íleo-cecal. La isquemia que se
produce es muy grave, por la brusquedad con que se instaura y por
la no existencia de colaterales. 2. TROMBOSIS ARTERIAL: supone un
25% de las IMA. Son pacientes con antecedentes vasculares (es
característica la historia previa de Isquemia Mesentérica Crónica)
cuyas arterias tienen disminuido el flujo y han desarrollado
circulación colateral. La trombosis de la AMS suele producirse en su
origen o en sus 3 primeros centímetros y la mortalidad es muy alta,
porque a pesar de que la circulación colateral es capaz de mantener cierto flujo, el territorio afectado es muy extenso3. TROMBOSIS
VENOSA MESENTÉRICA: ha de afectarse de forma muy severa el eje porto-esplénico-mesentérico e incluso en estos casos es
excepcional que una trombosis venosa produzca un infarto intestinal agudo. Antecedente habitual de hipercoagulación o TVP. 4.
Insuficiencia Mesentérica No Oclusiva (IMNO): produce el 20% de las IMA. Son pacientes con el flujo esplácnico de base disminuido por
la arteroesclerosis generalizada o el consumo de fármacos vasoactivos como la digital y que ante una disminución de perfusión
generalizada, desencadenada por un proceso agudo, no pueden mantener el aporte sanguíneo necesario en el territorio intestinal. Los
precipitantes de la IMNO van desde el sock cardiogénico hasta una deshidratación que desencadene hipovolemia. Excepcionalmente el
consumo de tóxicos con actividad simpática como la cocaína o los ergotamínicos, pueden producir vasoconstricción visceral aislada y
desencadenar IMNO. DIAGNOSTICO: Sospecha clínica: es la base del diagnóstico y se establece ante un paciente con perfil de riesgo
definido, que a primera vista impresiona de gravedad y presenta dolor abdominal severo, acompañado de forma más o menos
frecuente de distensión abdominal, nauseas, vómitos, diarrea y/o rectorragia. Llama la atención la normalidad de la exploración física,
sin signos de irritación peritoneal, en contraste con la gran intensidad del dolor. 1. Embolia Arterial: el dolor es de comienzo brusco y
localización periumbilical. 2. Trombosis Arterial: inicio gradual del dolor, incluso de días de evolución, con aumento progresivo de
intensidad y localización difusa. 3. Trombosis Venosa Mesentérica: dolor variable según la extensión de la trombosis, habitualmente
difuso de instauración progresiva e intensidad más leve que en las de causa arterial.4. IMNO: el dolor abdominal es difuso y de rápida
instauración, aunque en ocasiones, por la patología desencadenante son pacientes graves que se encuentran en cuidados intensivos y
sedoanalgesiados en los que el dolor no es valorable. Se sospecha cuando existe episodio reciente de bajo gasto e hipotensión
mantenida y aparece fiebre y leucocitosis, distensión abdominal o sangrado digestivo. Si se produce la necrosis completa de la pared
intestinal aparecen defensa abdominal y signos de irritación peritoneal con hiperperistaltismo primero y luego silencio abdominal total.
Son pacientes inquietos, sudorosos y que no se dejan explorar por la gran intensidad del dolor. Aparecen también signos de deterioro
hemodinámico: taquicardia, hipotensión, oligoanuria, etc. Laboratorio y gabinete: Radiología simple: es normal inicialmente. Cuando
aparece la necrosis podemos ver edema de pared, distensión de asas, y gas intramural (infarto intestinal) o libre en peritoneo
(perforación). BH de inicio discreta leucocitosis. Posteriormente aparece: leucocitosis por encima de 20000 y con gran desviación
izquierda. Acidosis metabólica severa y/o coagulopatía que en este contexto de dolor abdominal sugieren isquemia o sepsis grave.
Hiperamilasemia y elevación de CPK, LDH, GOT y GPT que reflejan la presencia de importante destrucción celular. Puede aparecer
hemoconcentración por presencia de importante edema y tercer espacio abdominal o anemia en los casos que cursan con importante
sangrado digestivo. La arteriografía es método diagnóstico típico, pero es una prueba menos disponible que el TC o el ECO-doppler, que
han aumentado mucho su sensibilidad además de ser más útiles como primera prueba diagnóstica, porque ayudan orientar el
diagnóstico diferencial. TC Helicoidal con contraste IV: muy útil para el diagnóstico diferencial y es muy sensible para detectar signos
indirectos de infarto intestinal: neumatosis intestinal, dilatación de asas, edema de pared. Trombosis Venosa Mesentérica: hoy día es el
Gold standart. Se aprecia un retraso del paso de contraste al sistema venoso, una pared engrosada y la falta de opacificación de la
porta. Oclusión embólica o trombótica Arterial: arteriografía sigue siendo superior al TC. TRATAMIENTO: Inicial: Infusión vigorosa de
cristaloides sonda de aspiración gástrica, analgesia y antibióticos intravenosos (Cefotaxima1 ó 2gr + Metronidazol 500mg cada 8h o
Piperacilina-tazobactam 4gr / 6h). T. definitivo: si existe infarto intestinal establecido el tratamiento siempre es quirúrgico, para
resección del segmento necrosado, con la máxima urgencia posible. 1. Oclusión embólica o trombótica Arterial: tratamiento quirúrgico
para repermeabilizar los segmentos aun viables mediante embolectomía o By-pass 2. Trombosis Venosa Mesentérica: si no hay infarto
intestinal establecido se realiza tratamiento conservador anticoagulando con heparina. 3. IMNO: lo fundamental es la corrección de los
factores desencadenantes. Se puede realizar infusión de vasodilatador (papaverina) intrarterial, durante la arteriografía. Hay autores
que recomiendan el empleo de IECA por la relación del eje renina-angiotensina en la fisiopatología del cuadro.

CURSO ENARM CMN SIGLO XXI TEL: 36246001 Pharmed Solutions Institute PÁGINA 510
MANUAL DE TRABAJO DEL CURSO ENARM CMN SIGLO XXI
ISQUEMIA MESENTERICA CRONICA (IMC). CIENCIAS BASICAS: También conocida como “angina intestinal” se produce cuando existe
una desproporción entre las demandas de oxígeno, la cual ocurre fundamentalmente durante la digestión, y el flujo sanguíneo
proporcionado por el sistema vascular. Generalmente se produce en personas con marcada ateroesclerosis y representa menos del 5%
de los casos de isquemia intestinal. Clínica; dolor abdominal que aparece precozmente tras la ingesta y cede en el plazo de 2 hrs. La
intensidad es mayor tras la ingesta de comidas copiosas con alta proporción en grasa. Los síntomas suelen aumentar con la
consecuente pérdida de peso, descrita hasta en el 80% de los casos debido a la aversión de la comida por miedo al dolor. En algunos
casos puede producirse una IMA sobre una IMC, debido a la formación de trombos sobre placas ateroesclerosas. El diagnostico se
establece por el cuadro clínico compatible, en pacientes con coexistencia de otras enfermedades vasculares (cardiaca, cerebral o
periférica). La demostración arteriografica de obstrucción de al menos dos de los vasos esplácnicos y la exclusión de otras patologías
que cursen con síntomas similares. La demostración por sí sola no constituye una prueba inequívoca de IMC, puesto que estas lesiones
se pueden encontrar en sujetos asintomáticos. La rentabilidad de las pruebas bioquímicas reside en la identificación de factores de
riesgo para desarrollo de IMC, como son la dislipidemias o la DM, así como la identificación de datos que apoyen la malabsorción
intestinal que pueden presentar estos pacientes. El tratamiento incluye construcción quirúrgica y ATP (angioplastia transluminal
percutánea) con o sin la colocación de un stent. La elección va a depender de la experiencia de cada centro, de la edad del paciente,
comorbilidad asociada y del número y severidad de la oclusión vascular. Revascularización quirúrgica: como la reimplantación de la
AMS en la aorta abdominal, la endarterectomía mesentérica y el bypass.

CASO CLINICO
Mujer de 83 años con antecedentes de cardiopatía hipertensiva, diabetes mellitus tipo 2, hipertensión arterial, flutter auricular y
episodio de pancreatitis aguda de origen biliar. Acude a urgencias por episodio de dolor abdominal localizado en epigastrio que se
irradia hacia ambos hipocondrios, acompañado de náuseas, vómitos y distensión abdominal. La analítica muestra leucocitosis (17,68 x
10-3/ul) con predominio de neutrófilos (92,8% N), así como un aumento marcado de LDH (1082 U/l). La gasometría venosa, los
parámetros bioquímicos de perfil hepático y biliar, así como los niveles de amilasa, se encuentran dentro de los límites normales. Se
realiza una RX de abdomen donde se aprecia dilatación de asas de delgado, con presencia de gas distal, sugiriendo suboclusión
intestinal y una ecografía abdominal que mostraba quistes simples renales, sin evidencia de lesiones ni líquido libre intraperitoneal.

PREGUNTA
Cuál es la patología más probable con los datos clínicos.

RESPUESTA
a.- Isquemia mesentérica aguda.
b.- Isquemia mesentérica crónica.
c.- Colitis isquémica.
d.- Oclusión intestinal recurrente.

CASO CLINICO
Mujer de 61 años diagnosticada de obesidad mórbida e hipertensión arterial, que acude a urgencias por dolor periumbilical de 2 días de
evolución. En la ecografía abdominal se observa escaso líquido libre y conglomerado de asas intestinales sin herniación. 24 horas
después la paciente presenta un cuadro de fallo multiorgánico. En la tomografía computarizada de abdomen con contraste intravenoso,
se observa neumatosis intestinal de la totalidad de asas del intestino delgado con eventración abdominal anterior incarcerada y
presencia de gas en venas mesentéricas, perihepáticas, periesplénicas y ramas distales intrahepáticas. En la cirugía se evidencia
isquemia mesentérica masiva y colon transverso incarcerado.

PREGUNTA
Cuál es el antecedente más probable con los datos clínicos y el cuadro clínico actual.

RESPUESTA
a.- Colitis isquémica.
b.- Isquemia focal segmentaria.
c.- Trombosis venosa mesentérica.
d.- Isquemia mesenterica crónica.

CASO CLINICO
Paciente mujer de 55 años, con antecedentes de tabaquismo crónico, hipertensión arterial moderada y dislipidemia en tratamiento,
cuyo cuadro comenzó en el año 2012, caracterizado por dolor abdominal tipo cólico postprandial precoz, acompañado de diarrea
frecuente, alrededor de 10 episodios diarios y vómitos ocasionales, además de baja de peso de alrededor de 15 kilos, siendo su peso
habitual de 55 kg, llegando a un índice de masa corporal de 17,3. Estudiada en el contexto de diarrea crónica y búsqueda de
enfermedad inflamatoria intestinal, se sometió a endoscopia alta y baja, además de enteroscopia, tránsito intestinal y estudio de mala
absorción, lo que sólo concluyó gastritis y colitis inespecífica, no respondiendo a tratamiento habitual, esto es, régimen alimenticio,
antiespasmódicos y medicamentos antiulcerosos.

PREGUNTA
Cual es la conducta a seguir mas adecuada?.

RESPUESTA
a.- Eco-dopler

CURSO ENARM CMN SIGLO XXI TEL: 36246001 Pharmed Solutions Institute PÁGINA 511
MANUAL DE TRABAJO DEL CURSO ENARM CMN SIGLO XXI
b.- Angio-TAC.
c.- Angio- IRM.
d.- Angiografia.

INFARTO MESENTERICO. CIENCIAS BASICAS: El infarto intestinal, isquemia intestinal o necrosis intestinal es la muerte del tejido
intestinal debido a una interrupción del suministro de sangre, de una o más de las arterias mesentéricas, principales arterias que
suministran sangre al intestino delgado. Suele venir precedida de sintomatología de angina intestinal, por insuficiencia vascular del
mesenterio, que degenera en isquemia. Existen varias causas posibles del infarto intestinal: En una hernia, si la irrigación del intestino
queda ocluida, esto puede llevar a isquemia intestinal. El intestino también puede quedar atrapado en tejido cicatricial de una cirugía
previa. Una embolia que bloquee los vasos principales que irrigan el intestino, así como la trombosis arterial, pueden impedir el
suministro adecuado, generalmente a partir de una enfermedad ateroesclerótica (acumulación de colesterol). Recientemente, se ha
sugerido que este bloqueo podría ser en parte causado por un remodelado vascular anómalo. Una trombosis de las venas que evacuan
la sangre del intestino, que se pueden obstruir con coágulos de sangre. Ésta es una afección más frecuente en personas con
insuficiencia hepática, cáncer o trastornos de la coagulación. Una hipotensión, ya que la presión arterial muy baja en pacientes con
obstrucción previa de las arterias también puede ocasionar isquemia intestinal. Clínica; dolor abdominal de inicio súbito, diarrea,
fiebre, vómitos, leucocitosis, leve acidosis. El tratamiento generalmente requiere cirugía, conocida como enterectomía, en la que se
extirpa la porción necrosada y se unen los cabos seccionados. Según la región implicada, se denomina colostomía o ileostomía. Si es
posible, se corrige la obstrucción de las arterias que irrigan el intestino. El tratamiento también puede incluir medicamentos para
prevenir los coágulos, disolver los coágulos ya existentes, o la dilatación de los vasos sanguíneos. Si la angiografía se realiza para
diagnosticar el problema, es posible eliminar al mismo tiempo un coágulo de sangre o abrir una pequeña arteria con la angioplastia. La
angioplastia consiste en utilizar un globo en la punta de un catéter para comprimir la arteria y la extensión de los depósitos de grasa,
haciendo un camino más amplio para que la sangre fluya sin problemas.

CASO CLINICO
Paciente varón de 27 años de edad, con cuadro clínico de 10 horas de evolución, caracterizado por dolor abdominal de comienzo en
epigastrio, sostenido, con migración a FID sin defensa ni peritonismo. Se acompaña de naúseas, sin vómitos ni fiebre con apetito
conservado. Rx de abdomen: nivel hidroaéreo en flanco y fosa ilíaca derecha. Laboratorio: > GB 21700 (89%N) resto: normal, se
diagnostica apendisitis, se realiza apendisectomia, egresando dos días después sin datos anormales. El paciente re-ingresa, con cuadro
clínico de 48 hs de evolución caracterizado por síndrome febril, que cede parcialmente con la ingesta de antipiréticos (Ibuprofeno VO).
Examen físico: > TA 120/70, Fc 133x´, Fr 16x´, T 39,1ºC, > Leve dolor abdominal, mayor sobre zona operada, signos de flogosis y
supuración serohemática en cicatriz de Mc Burney, mantiene tránsito intestinal, sin dificultad al orinar. > Laboratorio: GB 21100 (92%N)
resto normal.

PREGUNTA
Cual es su impresión diagnostica?

RESPUESTA
a.- Sepsis abdominal.
b.- Absceso abdominal.
c.- Trombosis mensenterica.
d.- Diverticulitis aguda.

PREGUNTA
Cual es la conducta a seguir mas adecuada para establecer el diagnostico?

RESPUESTA
a.- LAPE.
b.- Angio-TAC.
c.- Eco-dopler.
d.- USG abdominal.

PREGUNTA
En esta patología en su forma aguda, cual de los siguientes síntomas es el menos frecuente?

RESPUESTA
a.- Dolor abdominal
b.- Anorexia.
c.- Diarrea.
d.- Náuseas y vómitos.

CURSO ENARM CMN SIGLO XXI TEL: 36246001 Pharmed Solutions Institute PÁGINA 512
MANUAL DE TRABAJO DEL CURSO ENARM CMN SIGLO XXI
DIVERTICULITIS. CIENCIAS BASICAS: Enfermedad diverticular: Enfermedad de colon asociado a cambios en el hábito intestinal, dolor y
distensión abdominal. Diverticulosis: Paciente con divertículos en ausencia de cualquier síntoma. Diverticulítis: Complicación
inflamatoria de la enfermedad diverticular. Factores de riesgo: AINES, edad, inmunocompromiso, tabaquismo y alcoholismo, bajo
consumo de fibra, genética, Síndrome de Williams, Enf. Poliquística Renal, Síndrome de Ehlers- Danlos. SALUD PUBLICA: Aproximado de
50-65% en >60 años. 10-15% diverticulosis desarrollaran Diverticulítis.
PATOGENIA: Anatomía patológica: Sitio de “debilidad” en la pared
colónica. Disminución en la colágena tipo 1, incremento en colágena
tipo 3, incremento en la presión intraluminal 90mmHg, hay una
respuesta exagerada sistema simpático, motilidad y electrofisiología,
hay aumento en los movimientos de segmentación y retropulsión.
Histopatología: Hipertrofia e hiperplasia de los segmentos afectados,
debilidad y adelgazamiento en el sitio de protución o salida del divertículo. Teoría de Ryan Tipo 1: anormalidad muscular clásica por
presión aumentada, confinada al colon izquierdo, dolor inflamación y complicaciones inflamatorias. Tipo 2: sin anormalidades
musculares, tejido conectivo afectado, colon derecho, sangrado es el síntoma más común. Teoría de Mann: incremento en la presión
intraluminal secundaria a falta de relajación de la unión recto-sigmoidea, hipertrofia muscular. Localización: Mayo Clinic Sigmoides
29%, sigmoides y otros 68%, otros 3%. CLASIFICACION: de Hinchey modificada: Estadio I; I a Flemón, I b Diverticulítis con absceso
pericólico o mesentérico. Estadio II; Absceso pélvico tabicado, II a Absceso distal capaz de drenarse percutáneamente, II b Absceso
complejo asociado con fístula. Estadio III: Peritonitis purulenta generalizada. Estadio IV: Peritonitis fecal. Estadios clínicos:
Diverticulosis: Presencia de divertículos “asintomático”, incidentalomas, no requiere tratamiento ni seguimiento. Enfermedad
Diverticular no complicada: Presencia de sintomatología asociada al hábito intestinal, distensión abdominal, meteorismo, se confunde
con SII. Enfermedad Diverticular complicada: Sangrado, colon derecho, hematoquezia (vinoso), manejo médico y resolución en 80%,
probabilidad de resangrado del 20-30%, indicación quirúrgica hasta el segundo evento. Fístulas colo-vesical es la más frecuente, más
frecuente en hombres 3:1, complicación menos frecuente de la ED. Diverticulítis no complicada: Pacientes sin afectación sistema por
datos inflamatorios Hinchey I y II, respuesta al tratamiento médico hasta 90%, repetición del cuadro en 25%, indicación quirúrgica
electiva en el 1er cuadro con patología sistémica asociada. Diverticulitis complicada: Hinchey II y III, patología que involucra absceso
pélvico o peritonitis, afectación sistémica, tratamiento quirúrgico, indicado siempre la programación electiva si mejora con tratamiento
conservador. DIAGNOSTICO: Colon por enema sensibilidad del 99%, especificidad del 96%. Colonoscopia sensibilidad del 88%,
especificidad del 74%. TAC sensibilidad del 95%, especificidad del 86%. Colonoscopia virtual. TRATAMIENTO: Tratamiento médico
dependiendo de la severidad, identificar complicaciones, valoración del estado hidroelecrolítico, valoración del estado acido-base,
valorar tratamiento quirúrgico. Medico: Ayuno, soluciones, antibióticos (ciprofloxacino, metronidazol, analgésicos, ketorolaco,
paracetamol, tramadol). Quirúrgico: Indicado en falla al tratamiento médico, sepsis, alteraciones en el estado hemodinámico.
Procedimiento 3 tiempos (three times procedure= Colostomía de transverso y drenaje, resección subsecuente, cierre de colostomía).
Procedimiento de Miculicz (exteriorización y resección con colostomía y fístula mucosa. Restitución del tránsito en segundo tiempo).
Procedimiento de Hartman (Resección de sigmoides. Bolsa de Hartmann o muñón rectal. Restitución del tránsito en segundo tiempo).
Procedimiento en 1 solo tiempo (resección del foco séptico y restitución del tránsito en un solo tiempo quirúrgico). Lavado de absceso.
Drenaje percutáneo. COMPLICACIONES: Sangrado 4-17%, perforación 32.4%, absceso 10.9%, Fístula 5-33% (colovesical, colocutánea,
colovaginal), estenosis 13.4%

CASO CLINICO
Mujer de 96 años que consultó por dolor abdominal de 4 días de evolución localizado en hipogastrio asociado a febrícula y aumento del
perímetro abdominal. A la exploración física se palpaba una masa localizada en hipogastrio de consistencia dura y dolorosa. En la
analítica destacaba 19.400 leucocitos (78,5% PMN) y PCR: 28,2. La radiografía de abdomen mostró una imagen aérea redondeada y la
TAC abdominal informaba de gran burbuja aérea con paso de contraste desde el colon y con pequeños abscesos en la pared. Se
instauró tratamiento médico (antibióticos intravenosos y reposo intestinal) luego egreso por mejoría a los 15 días.

PREGUNTA
Cuál es la indicación posterior a su egreso con mayor evidencia para reducir la recurrencia del presente evento.

RESPUESTA
a.- La fibra de vegetales y frutas.
b.- Abundante ingesta de agua.
c.- Evitar granos y nueces.
d.- Fibra proveniente de cereales.

CASO CLINICO
Se presenta el caso de un varón de 31 años, con antecedentes de LAPE por múltiples heridas de arma blanca, acudió al servicio de
urgencias por un cuadro de dolor abdominal súbito en la región periumbilical y signos de oclusión intestinal. A la exploración física
destacaba un abdomen distendido; en la radiografía de abdomen se observaba la presencia de niveles hidroaéreos y distensión de las
asas intestinales, sin que se apreciara neumoperitoneo, y en la analítica destacaba leucocitosis de 22,000, con neutrofilia.

PREGUNTA
Cuál es la patología más probable con los datos clínicos.

RESPUESTA
a.- Oclusión intestinal mecanica
b.- Oclusión intestinal idiopatica

CURSO ENARM CMN SIGLO XXI TEL: 36246001 Pharmed Solutions Institute PÁGINA 513
MANUAL DE TRABAJO DEL CURSO ENARM CMN SIGLO XXI
c.- Invaginación intestinal
d.- Enfermedad diverticular complicada

CASO CLINICO
Un hombre de 45 años, hasta ese momento sano, consulta por dolor intenso en la fosa ilíaca izquierda, que comenzó 35 horas antes. Ya
antes había notado malestar leve y periódico en esta región pero no solicito tratamiento médico. También presentaba náuseas,
anorexia y vómitos, que los relacionaba con las comidas. En el examen físico estaba febril (38,5ºC) y taquicárdico (110/min). Dolor a la
palpación en la fosa ilíaca izquierda, sin signos peritoneales.

PREGUNTA
Las manifestaciones clínicas de la diverticulitis colónica aguda varían con la extensión de la enfermedad. Cual de las siguientes
observaciones es menos frecuente?

a.- Constipación importante.


b.- Dolor abdominal.
c.- Sensación de plenitud perirrectal.
d.- La prueba del guayaco en las heces es positiva.

PREGUNTA
Cual de los siguiente diagnosticos diferenciales es el menos frecuente?

RESPUESTA
a.- Apendicits aguda
b.- Enfermedad intestinal inflamatoria
c.- Enfermedad de Crohn
d.- Colitis infecciosa

PREGUNTA
Cual es el método diagnostico mas adecuado para el caso descrito?

RESPUESTA
a.- Colonoscopia.
b.- Transito intestinal.
c.- TAC abdominal.
d.- USG abdominal.

PREGUNTA
Considerando el caso clínico, cual es la conducta a seguir mas adecuada?

RESPUESTA
a.- Ingreso, ayuno, hidratación y antibióticos.
b.- Preparar al paciente para cirugía laparoscópica.
c.- Preparar al paciente para cirugía abierta.
d.- Tratamiento ambulatorio.

PREGUNTA
Cual es la cantidad de fibra que deberá consumir posteriormente el paciente para evitar recaidas?

RESPUESTA
a.- 10 gr/dia.
b.- 20 gr/dia.
c.- 30 gr/dia.
d.- 40 gr/dia.

CURSO ENARM CMN SIGLO XXI TEL: 36246001 Pharmed Solutions Institute PÁGINA 514
MANUAL DE TRABAJO DEL CURSO ENARM CMN SIGLO XXI
ENFERMEDAD INTESTINAL INFLAMATORIA: La enfermedad intestinal inflamatoria, incluye colitis ulcerosa y enfermedad de Crohn,
ambas se atribuyen a un defecto en la regulación inmunológica del tubo digestivo que origina una reacción inmunológica
descontrolada a diferentes antígenos. CUCI: La colitis ulcerativa casi siempre se limita al intestino grueso, afecta la mucosa (colitis
mucosa), y es continua desde el recto hacia arriba, y por lo general afecta solo el colon y parte más distal del íleon. Las fistulas son
raras. Una forma más ligera de colitis ulcerativa puede afectar la porción distal del intestino grueso, el recto o el recto y el colon
sigmoide. Se conoce como proctitis ulcerativa o proctosigmoiditis ulcerativa. 90% de estos pacientes responden al tratamiento médico
y nunca experimentan pancolitis. CROHN: Enfermedad de Crohn puede dañar cualquier parte del tubo digestivo, desde la boca hasta el
ano y suele comprometer todo el grosor de la pared abdominal, la mayoría de los pacientes presenta compromiso del intestino delgado
a menudo con una distribución discontinua (segmentos “saltados”). Los estrechamientos son usuales, la enfermedad perianal (fistulas,
abscesos, fisuras), son comunes y tienen un potencial maligno mayor que la CUCI.DIAGNOSTICO: Los cuadros clínicos se sobreponen y
en 15% de los pacientes la colitis es indeterminada. Por lo general los síntomas se deben a la inflamación intestinal e incluyen diarrea,
hemorragia rectal, tenesmo, dolor abdominal, fiebre y pérdida de peso. Es posible que se afecten otros órganos como el sistema
musculoesquelético (espondilitis, artritis), la piel (eritema nodoso, pioderma gangrenosa), el ojo (iritis), sistema hematopoyético,
riñones, vías biliares. TRATAMIENTO: Es similar para ambos; la sulfazalacina, base de tratamiento anterior, ya casi se sustituyó por el
ácido 5-aminosalicilico (5-ASA), el cual tiene muchos menos efectos colaterales y es mejor tolerado por los pacientes, las etapas agudas
se controlan con esteroides, casi siempre prednisona en dosis 20-80mg/día, con reducción rápida de dosis. Los pacientes más graves
deben hospitalizarse e iniciar soluciones intravenosas, reposo intestinal y esteroides intravenosos. La operación casi siempre colectomía
abdominal, se realiza si no se observa respuesta al tratamiento médico en unos cuantos días. La función de los inmunomoduladores
como la 6-mercaptopurina (6MP) y aziatropina se incrementa conforme su eficacia y seguridad se establecen cada vez más. Su
aplicación principal es el tratamiento de mantenimiento y como ahorradores de esteroides en los pacientes con síntomas persistentes
mientras reciben solo preparaciones de 5-ASA. Tratamiento quirúrgico: En CUCI, ya que la inflamación se limita al intestino grueso, la
resección del órgano es curativa, a diferencia de la enfermedad de Crohn, que no puede erradicarse por medios quirúrgicos y tienen un
alto índice de recurrencia después de la resección, la cirugía es la proctocolectomía reconstitutiva (se extirpa colon y parte superior de
recto, se quita la mucosa del recto restante y se construye un reservorio ileal (saco), con el íleon terminal; luego se une con el
reservorio a nivel de la línea dentada. En enfermedad de Crohn la cirugía está reservada para complicaciones que no responden a
tratamiento conservador, casi siempre implica resección del segmento intestinal, afectado.

CASO CLINICO
Paciente masculino de 33 años que ingresa con el idx de brucelosis crónica con tratamiento empírico ciprofloxacino durante 4
ocasiones por periodos de 2 semanas con recidiva de la misma sintomatologia (fiebre intermitente sin predominio de horario, dolor
abdominal en region del epigastrio y rara vez con irradiacion a marco colonico, nauseas, vomito al inicio siempre del padecimiento y
hematoquezia sin diarrea. EF: abdomen blando depresible no doloroso en el momento, no se identifica masa megalias. no hay signos
de irritación peritoneal. Peristaltismo positivo de caracteristicas normales. Se realiza colonoscopia ciego, el cual es de características
macroscópicas normales, se observa valvula ileocecal, se canula la misma y se exploran 15cms del ileon terminal el cual es normal, el
colon ascendente y transverso conservan su morfología y patrón vascular, el colon descendente y rectosigmoides en algunas áreas
conservan su patron vascular y en otral esta perdido, con imagen tubular. Se toma biopsia de de colon ascendente y transverso, así
como de colon descendente y recto de áreas afectadas. Se diagnostica CUCI.

PREGUNTA
Cual de las siguientes manifestaciones son más características de CUCI?

RESPUESTA
a.- Estenosis, perdida de la vascularidad, ulceras, seudopolipos y granularidad.
b.- Estenosis, perdida de la vascularidad, ulceras, polipos y granularidad.
c.- Estenosis, perdida de la vascularidad, ulceras, seudopolipos y agranularidad.
d.- Estenosis, aumento de la vascularidad, ulceras, seudopolipos y granularidad

PREGUNTA
En la escala de Baron, en que estado se encuentra el caso clínico?

RESPUESTA
a.- 0.
b.- 1.
c.- 2.
d.- 3.

PREGUNTA
Cual de las siguientes afecciones extra-intestinales es la menos frecuente en el CUCI?.

RESPUESTA
a.- Artritis.
b.- Axial Espodilitis Anquilosante.
c.- Estomatitis.
d.- Glomerunefritis.

CURSO ENARM CMN SIGLO XXI TEL: 36246001 Pharmed Solutions Institute PÁGINA 515
MANUAL DE TRABAJO DEL CURSO ENARM CMN SIGLO XXI
CANCER DE COLO RECTAL (CCR). CIENCIAS BASICAS: El CCR es un tumor maligno que se origina dentro de las paredes del intestino
grueso, incluye los segmentos; ciego, colon ascendente, colon transverso, colon descendente, sigmoides, recto. El CCR no incluye los
tumores en los tejidos del ano o del intestino delgado. Los adenomas son tumores benignos epiteliales que se consideran lesiones
precancerosas. Los adenomas pueden tener diferentes grados de displasia o diferentes características histológicas: tubular, túbulo-
vellosas, asociados con el aumento de potencial malignidad. Factores de riesgo: Incluye factores ambientales y genéticos.
Aproximadamente el 75% se presentan en pacientes sin factores de riesgo conocidos. Hay dos síndromes genéticos que predisponen al
CCR: poliposis adenomatosa familiar (PAF), con riesgo de 60-90% y cáncer colo rectal hereditario no asociado a poliposis (CCHNP),
considerar en familiares de primer grado de pacientes con diagnóstico de CCR, pero representan menos del 10% de todos los CCR.
Otros factores son CUCI, obesidad, sedentarismo, exceso de alcohol, tabaquismo dieta alta en grasas y/o baja en frutas y vegetales o
fibra. SALUD PUBLICA: El CCR es el cáncer más común del tracto gastrointestinal, la tercera causa más común de cáncer y la cuarta
causa de muerte por cáncer en el mundo. El CCR se presenta con mayor frecuencia a partir de los 50 años de edad. Más de 95% de CCR
son adenocarcinomas. SINDROME DE LYNCH: Síndrome de carácter hereditario, autosómico dominante y se caracteriza por el
desarrollo precoz (antes de 50años de edad) de CCR, de predominio en colon derecho, y una elevada tendencia a presentar lesiones
sincrónicas o metacrónicas, así como neoplasias en otros órganos (endometrio, estomago, páncreas, sistema urinario, ovario, vías
biliares). El diagnostico se establece a partir de la historia familiar según los criterios de Amsterdam. Esta entidad se caracteriza por la
existencia de múltiples mutaciones somáticas en fragmentos repetitivos de ADN. DIAGNOSTICO: Clínica; dependen de la localización y
extensión del tumor, no suele dar síntomas hasta etapas avanzadas. Las neoplasias en el hemicolon derecho producen hemorragia
oculta que el paciente no detecta y es probable que ocasionen síntomas de anemia ferropriva crónica. Este hallazgo en un varón adulto
o mujer postmenopáusica, es indicación para colonoscopia. Los tumores en el hemicolon izquierdo pueden manifestarse con
hemorragia visible (rectorragia), cambio en los hábitos intestinales y dolor abdominal cólico secundario a obstrucción parcial. También
es probable que las primeras manifestaciones del cáncer del intestino grueso sean, metástasis hepática, ascitis y metástasis
pulmonares. No es infrecuente que se detecte una masa en la exploración en el cuadrante inferior derecho del abdomen. Por ultimo
una complicación poco frecuente del cáncer de colon pero que empeora el pronóstico es la perforación intestinal. El cáncer de recto
puede comportarse como un síndrome anorrectal, con urgencia rectal, tenesmo y diarrea con moco y sangre. Cunado su extensión
rebasa la pared rectal, el paciente puede aquejar síntomas urinarios atribuibles a invasión vesical como hematuria y polaquiuria. Si se
ha establecido una fistula rectovesical, hay neumaturia e infecciones urinarias recidivantes. Además de los síntomas locales, CCR puede
causar síntomas generales, como astenia, anorexia o pérdida de peso. Puede aparecer clínica secundaria a la presencia de metástasis a
distancia, como ictericia, trastornos neurológicos, dolores óseos, disnea, ascitis. Ante la sospecha de CCR es imperativo practicar una
colonoscopia con la que además de observar el tumor, se podrán tomar biopsias para su diagnóstico histológico y descartar la
presencia de lesiones sincrónicas. En caso de tumores estenosantes es recomendable completar la exploración con una colonografía
por TAC o RNM. Por elevado porcentaje de presentaciones diseminadas, las exploraciones complementarias incluyen radiografías de
torax, USG o TAC abdominal, gammagrafía óseas, cistoscopia, ultrasonografía endoscópica y RM pélvica. La determinación de antígeno
carcinoembrionario, tienen una reducida sensibilidad y especificidad para CCR. Sin embargo, posee valor pronóstico y su monitorización
es útil para seguimiento del paciente tras cirugía. TRATAMIENTO: La cirugía es usualmente el primer tratamiento o el tratamiento
primario para los cánceres de colon que no se han propagado a partes distantes. También se puede usar la terapia adyuvante
(adicional). En la mayoría de los casos, la terapia adyuvante se administra por aproximadamente 6 meses. Etapa 0: Puesto que estos
cánceres no han crecido más allá del revestimiento interno del colon, todo lo que se necesita es cirugía para extirpar el cáncer. Esto se
puede lograr en la mayoría de los casos con una polipectomía (extirpación del pólipo) o escisión local a través de un colonoscopio. Si el
tumor es demasiado grande para poder extirparlo mediante escisión local, es posible que ocasionalmente sea necesario realizar una
resección del colon (colectomía). Etapa I: Ha crecido a través de varias capas del colon, pero no se han propagado fuera de la pared del
colon en sí, ni ganglios. La colectomía parcial (cirugía para extirpar la sección del colon que contiene cáncer y los ganglios linfáticos
cercanos) es el tratamiento convencional. Usted no necesita terapia adicional. Etapa II: Ha crecido a través de la pared del colon y se
puede extender hacia el tejido vecino, no a ganglios linfáticos. Puede que la cirugía (colectomía) sea el único tratamiento que se
necesita. Se puede recomendar quimioterapia adyuvante si hay riesgos de recurrencia como: es de alto grado, ha crecido hacia los
órganos cercanos, el cirujano no extirpó al menos 12 ganglios linfáticos, se encontró cáncer en o cerca del margen (borde) de la
muestra quirúrgica, lo que significa que algo de cáncer pudo haber quedado, ha bloqueado (obstruido) el colon, causó una perforación
(orificio) en la pared del colon. Las principales opciones de quimioterapia para esta etapa incluyen 5-FU y leucovorín (solo) o
capecitabina, aunque también se pueden usar otras combinaciones. Se puede administrar radioterapia al área de su abdomen donde
está creciendo el cáncer. Etapa III: Se ha propagado a los ganglios linfáticos cercanos, pero todavía no se ha extendido a otras partes del
cuerpo. Cirugía (colectomía parcial) seguida de quimioterapia adyuvante. El régimen FOLFOX (5-FU, leucovorín, y oxaliplatino) o el
régimen CapeOx (capecitabina y oxaliplatino) se usan con más frecuencia, aunque algunos pacientes pueden recibir 5-FU con
leucovorín o capecitabina sola según la edad y sus necesidades de salud. La radioterapia y/o la quimioterapia pueden ser opciones para
las personas que no están lo suficientemente saludables como para someterse a cirugía. Etapa IV: Se propagó desde el colon hasta
órganos y tejidos distantes. Con más frecuencia, el cáncer de colon se propaga al hígado, aunque también se puede propagar a otros
lugares, tal como a los pulmones, el peritoneo (la membrana que recubre la cavidad abdominal) o a los ganglios linfáticos distantes. Es
poco probable que la cirugía cure estos cánceres. Si las metástasis no se pueden extirpar quirúrgicamente debido a que son muy
grandes o hay demasiadas, se puede tratar primero quimioterapia para reducir el tamaño de los tumores con el fin de permitir la
cirugía. Entonces, se puede administrar nuevamente quimioterapia después de la cirugía. Otra opción sería destruir los tumores en el
hígado con criocirugía, ablación por radiofrecuencia, u otros métodos no quirúrgicos. Si el cáncer se ha propagado demasiado como
para tratar de curarlo con cirugía, puede que en algunos casos aún sea necesario realizar operaciones, tales como una colectomía o una
colostomía de derivación (cortar el colon sobre el nivel del cáncer y se adhiere el extremo a una abertura en la piel del abdomen para
permitir que los desechos salgan del cuerpo). Esto puede aliviar o prevenir el bloqueo del colon, lo que puede evitar ciertos problemas.
Algunas veces, se puede evitar tal cirugía al insertar un “stent” en el colon durante la colonoscopia con el fin de mantenerlo abierto. La
mayoría de los pacientes con cáncer en etapa IV recibirá quimioterapia y/o terapias dirigidas para controlar el cáncer. Los regímenes
más comúnmente usados incluyen: FOLFOX: leucovorín, 5-FU, y oxaliplatino (Eloxatin). FOLFIRI: leucovorín, 5-FU, e irinotecán
(Camptosar). CapeOX: capecitabina (Xeloda) y oxaliplatino. Cualquiera de las combinaciones anteriores más bevacizumab (Avastin) o

CURSO ENARM CMN SIGLO XXI TEL: 36246001 Pharmed Solutions Institute PÁGINA 516
MANUAL DE TRABAJO DEL CURSO ENARM CMN SIGLO XXI
cetuximab (Eribitux) (pero no ambos).PRONOSTICO: Se relaciona con la etapa del tumor. Se prefiere el sistema de Dukes por su
sencillez: Dukes A: tumor limitado a submucosa. Dukes B1: tumor que se extiende a la capa muscular. Dukes B2: tumor que se extiende
más allá de la serosa intestinal. Dukes C1: compromiso de ganglios linfáticos; el tumor no se extiende fuera de la pared intestinal. Dukes
C2; ganglios linfáticos positivos; el tumor rebasa pared intestinal. Dukes D: metástasis a distancia. La sobrevida no corregida a los 5 años
para etapas A= 85%, B= 65%, C = 45%, los valores corregidos son 100, 78 y 54% respectivamente

CASO CLINICO
Se trata de paciente masculino de 71 años de edad, refiere que desde hace 6 meses presenta reducción de peso, astenia, adinamia, a la
exploración física se palpa masa no dolorosa en fosa iliaca derecha, con presencia de hematoquesia, se reporto biometría hemática con
hematocrito 31 %, y hemoglobina de 10, el frotis mostro eritrociticos microcitico normocromicos, resto normal.

PREGUNTA
Cuál es el factor de riesgo de mayor importancia para el desarrollo de cáncer de cólon

RESPUESTA
a.- Consumo de alcohol.
b.- Dieta baja en fibra y alta en carnes.
c.- Tabaquismo positivo.
d.- Colitis o Diverticulitis previa.

CASO CLINICO
Acude al servicio de urgencias un paciente masculino de 69 años de edad el cual refiere malestar generalizado y fiebre desde hace 3
semanas de evolución, el paciente es originario de la ciudad de mexico, administrador retirado hace 4 años, tiene el antecedente de
tabaquismo desde la edad de 20 años a la fecha en promedio 20 cigarrillos al día, además hipertensión arterial sistémica controlada con
IECAS mas diurético tiazidicos, a la exploración física se observa con palidez moderada, asi como índice de masa corporal de 21, la
región cardiaca se ausculta murmullo sistólico con predominio mitral, los estudios de laboratorio se detecta hemoglobina de 11 g/dl,
34% de hematocrito, leucocitos y eritrocitos en orina, se realizaron cultivos siendo positivo únicamente el hemocultivo para
streptococcus bovis, debido a los resultado se envía a ecocardiograma, sin tener actualmente los resultados, pero se sospecha
vegetaciones.

PREGUNTA
Cual es la conducta a seguir más adecuada en el paciente para identificar la causa inicial.

RESPUESTA
a.- Realizar una biopsia renal.
b.- Esperar resultado de Ecocardiograma.
c.- Valoracion toxicológica.
d.- Realizar colonoscopia.

CURSO ENARM CMN SIGLO XXI TEL: 36246001 Pharmed Solutions Institute PÁGINA 517
MANUAL DE TRABAJO DEL CURSO ENARM CMN SIGLO XXI
FISURA ANORECTOPERINEAL. CIENCIAS BASICAS: Comunicación anómala entre dos epitelios diferentes. La fístula es el conducto de
paredes fibrosas infectadas, que comunica una cripta anal con la piel o con la luz del recto; el orificio localizado en la cripta se denomina
primario o interno y el cutáneo mucoso o secundario. El orificio primario es donde habitualmente se origina el proceso. Etiología:
Origen criptoglandular en 95%. Secuela de un absceso. Debut en procesos menores. Trauma. Infecciosas (TB, gonorrea). Enfermedades
de intestine irritable (Crohn, CUCI). Radiación. SALUD PUBLICA: Es una afección muy frecuente en la práctica colóproctológica más
común en el sexo masculino y en edades que fluctúan entre 30 a 50 años CLASIFICACION: A)
Interesfinteriana, B) Transesfintérica, C) Supraesfintérica, D) Extraesfintérica. DIAGNOSTICO:
Clínico; Orificio secundario, dolor, secreción, sangrado, absceso. Realizar un adecuado
interrogatorio del cuadro clínico y los antecedentes. secreción purulenta persistente y escasa , que
procede de una elevación cutánea del tejido inflamatorio ( orificio secundario), a veces dolorosa al
tacto, que puede ser única o múltiple, situado en cualquiera de los dos cuadrantes perianales , con
mayor frecuencia en los posteriores a distancias variables del ano , algunas tan lejanas como 12 a
13 cm y aún más . Aproximadamente en la mitad de los casos se encuentra un aumento en el
volumen e hiperpigmentación de la piel en la región que rodea el orificio fistuloso, que traducen la
presencia de un absceso que no fue totalmente vaciado. En conjunto el absceso y la fístula siguen una de las siguientes evoluciones
Exploración Física: Orificio Secundario, tacto rectal. Gabinete: La fistulografía con el propósito de diagnóstico y conocer el trayecto de la
fístula si el material de contraste logra pasar con facilidad los trayectos fistulosos, lo podrá hacer el estilete durante la operación, si por
el contrario es introducido a presión puede romper la pared de la fístula y causar error diagnóstico. El ultrasonido endoanal de 360°es
útil para anatomía de fístulas complejas. CRITERIOS DIAGNÓSTICOS: Historia de absceso perianal. Drenaje anal o perianal de secreción
purulenta o intermitente. Presencia de orificio fistuloso secundario con trayecto fistuloso dirigido hacia la cripta de origen.
TRATAMIENTO: Tratar el absceso y la fístula en la misma operación. Fistulectomía o fistulotomía. La fistulectomia reseca la totalidad
del trayecto fistuloso por lo que es más radical y disminuye la recidiva mayor riesgo de incontinencia; la fistulotomía solo una porción
del trayecto. La incontinencia esfinteriana por fistulectomía es posible en fístulas muy altas que requieren división del anillo rectoanal.
COMPLICACIONES: Fase aguda (abscesos) sepsis graves y muerte, destrucción del esfínter e incontinencia anal (5-10%), deformidad
anal (Keyhole) (3-5%), recidiva (3-5%), cambios neoplásicos malignos en fístulas de larga evolución, complicación post- quirúrgica:
incontinencia anal en cirujanos de poca experiencia.

CASO CLINICO
Se trata de paciente femenino de 31 años de edad la cual acude a revisión posterior a parto hace 45 días, al interrogatorio la paciente
refiere que presenta dolor al evacuar, con salida de sangre fresca leve, niega sensación de pensantes, ni descarga rectal espontánea,
niega ardor o prurito, a la exploración no se observan hemorroides externas.

PREGUNTA
Cuál es la patología más probable en esta paciente.

RESPUESTA
a.- Lo más frecuente es la fisura anal.
b.- Desgarro secundario a parto.
c.- Las hemorroides internas por múltiples embarazos.
d.- Un absceso anal por infección de episiotomía.

CURSO ENARM CMN SIGLO XXI TEL: 36246001 Pharmed Solutions Institute PÁGINA 518
MANUAL DE TRABAJO DEL CURSO ENARM CMN SIGLO XXI
APENDICITIS. CIENCIAS BASICAS: Es la inflamación del apéndice vermiforme, que inicia con obstrucción de la luz apendicular; es un
padecimiento grave, con importantes complicaciones que pueden llevar a la muerte, en particular cuando se retrasan el diagnóstico y la
terapéutica oportuna. La apendicitis es la patología quirúrgica actual más frecuente. SALUD PUBLICA: Edad promedio 31.3 años,
mediana 22 años, error en diagnostico 15.3 %. El 40 % 10-29 años. 260 000 apendicectomías. PATOGENIA: Etiología: Obstrucción de la
luz, **Hiperplasia de tejido linfoide. (55%), fecalito 40 %, parásitos intestinales, tumores, semillas, bacterias aerobias (E.coli, P.
aeruginosa, Klebsiella, Streptococcus), anerobios (Bacteroides fragilis, fusobacterium, peptostreptococcus). Al obstruirse la luz y se
forma un asa cerrada en la porción distal del apéndice que se distiende rápidamente al aumentar la secreción mucosa. Se eleva la
presión hasta 50 o 60 cmH2O y la distensión estimula las terminaciones nerviosas, con aparición de dolor sordo y difuso; esto
incrementa la peristálsis y aparece el cólico característico. El ensanchamiento de la luz apendicular sigue en aumento por la
multiplicación bacteriana, que produce gas, pus, o necrosis. Se excede la presión de las venas y éstas se colapsan e impiden el retorno
de la sangre; en tanto, la circulación continúa por las arterias y provoca congestión hemática, edema y estasis. La distensión progresiva
del apéndice emite reflejos por vía nerviosa y ocasiona náusea, vómito y aumento del dolor. El proceso inflamatorio involucra pronto a
la serosa apendicular y al peritoneo parietal, con lo cual el dolor se traslada a la fosa iliaca derecha; la mucosa apendicular es invadida
por bacterias y éstas se extienden a capas más profundas; todo ello desencadena necrosis y absorción de sustancias provenientes de
tejidos muertos y de toxinas bacterianas, lo que produce fiebre, taquicardia y leucocitosis. Al continuar la distensión apendicular se
ocluye la circulación de arteriolas, se agrava la necrosis, las paredes no resisten y finalmente el apéndice se perfora. El organismo
reacciona rodeando la zona con asas de intestino y epiplón, para limitar la diseminación a un área restringida, pero esto origina absceso
apendicular; entonces se presenta el íleo paralítico para focalizar el proceso. Sin embargo, otras veces el proceso infeccioso no se
localiza y con la rotura apendicular se diseminan diversos líquidos en la cavidad abdominal, ocasionando peritonitis generalizada.
CLASIFICACION: A) Apendicitis aguda: 1. Apendicitis aguda no perforada (Fase hiperémica, edematosa, necrótica o gangrenada). 2.
Apendicitis aguda perforada (Fase purulenta, de absceso, con peritonitis local, con peritonitis generalizada). 3. Apendicitis aguda
reactiva. 4. Apendicitis aguda complicada. B) Apendicitis crónica. DIAGNOSTICO: Clínica: El síntoma más común es el dolor (tipo,
localización, migración, variaciones anatómicas), la anorexia, que acompaña a la apendicitis en 50 a 90 % de las veces; también son
frecuentes náuseas, vómito, diarrea (10 % de los casos), estreñimiento, fiebre. Casi todos los pacientes sufren al principio anorexia y
después dolor abdominal y vómito. Los hallazgos de la exploración física son decisivos, especialmente los datos abdominales. En el
cuadrante inferior derecho se presentan hiperestesia e hiperbaralgesia, dolor intenso bien definido en el punto de McBurney, rigidez
muscular y dolor a la descompresión. Otros datos menos frecuentes son la positividad a las maniobras del psoas (dolor del cuadrante
inferior derecho a la extensión del muslo en decúbito lateral izquierdo) y de Rovsing (la palpación del cuadrante inferior izquierdo suele
producir dolor en el cuadrante inferior derecho). En el apéndice ubicado en la pelvis los signos abdominales son escasos; es más
probable detectar una masa palpable al tacto rectal o vaginal. La apendicitis en el paciente pediátrico es más grave por la alta incidencia
de perforación y por la peritonitis que se desarrolla al demorar el diagnóstico; además, la enfermedad progresa con mayor rapidez. La
apendicitis del anciano también es grave, estos pacientes manifiestan pocos síntomas ya que el dolor es menos intenso y los leucocitos
casi no se incrementan; además, las enfermedades intercurrentes deterioran al paciente, riesgo de perforación 49 %, mortalidad 21 %.
La apendicitis es común durante el embarazo (uno de cada 2 200). Se relaciona con dolor en posición más cefálica que la normal, sin
dejar de ser compatible con la migración del ciego, a partir del cuadrante inferior derecho a la posición subcostal, conforme evoluciona
el embarazo. La apendicitis perforada durante la gestación se acompaña de mayor riesgo para la madre y feto a causa de
complicaciones sépticas. Laboratorio: El dato más constante es la leucocitosis (entre 10 000 y 16 000 mm3), acompañada de un
predominio porcentual de los neutrófilos y presencia de formas en banda; ésta es una reacción inespecífica del proceso infeccioso y no
es exclusiva de la apendicitis. Cifras mayores a 18 000 se observan en el absceso apendicular, la peritonitis generalizada o en el absceso
hepático. El EGO es normal excepto en el apéndice retrocecal, que involucra al uréter o a la vejiga; esto puede hacer pensar al clínico en
una infección de vías urinarias. Las placas simples de abdomen en posición de pie y en decúbito, muestran una o dos asas distendidas
por gas, ubicadas en el cuadrante inferior derecho del abdomen; puede apreciarse el fecalito y también el borramiento de la sombra
del músculo psoas derecho, la ausencia de gas en colon y una zona de opacidad en el cuadrante inferior derecho, imagen en vidrio
despulido. Ultrasonido: Sensibilidad 85 – 90 %, Especificidad 92 – 96 %, útil en embrazo y pacientes ginecológicas. TAC: Sensibilidad 92
– 97 %. Especificidad 90-98 %, observamos masas, abscesos, signo de punta de flecha. TRATAMIENTO: Prequirúrgico; El periodo de
preparación debe ser menor de cuatro horas. Ayuno, reparar deficiencia de líquidos y electrolitos y trastornos ácido-base. Iniciar
antibióticos: 1. Ampicilina, 2. Aminoglucósido, 3. Metronidazol o clindamicina, 4. Ampicilina-sulbactam o cefoxitina. Analgésico no
opiáceo. Antipiréticos. Catéter central, sonda vesical, SNG (paciente critico). La cirugía laparoscópica es útil tanto para la exploración
diagnóstica como para el tratamiento; sigue los mismos principios de la técnica quirúrgica tradicional y su empleo es cada vez más
frecuente. COMPLICACIONES: Perforación, peritonitis focal, peritonitis difusa, pileflebitis. En primer día posquirúrgico: Hemorragia.
Evisceración por mala técnica. Íleo adinámico. Tardía infertilidad (33%).

CASO CLINICO
Masculino de 21 años de edad. Consultó en la madrugada, tras más o menos 16 h de evolución de un dolor abdominal vago que habría
comenzado tras el desayuno, ubicado en la región periumbilical, era de baja intensidad, se mantuvo varias horas para luego localizarse
en fosa ilíaca derecha. No hubo compromiso del estado general, pudo ingerir comida liviana, no tuvo vómitos pero sí algo de náuseas.
Por la tarde consultó médico quien sospechó una apendicitis aguda. Con diagnóstico de apendicitis aguda se hospitalizó y al ingreso se
observo dolor espontáneo y a la palpación en fosa ilíaca derecha con signo de Blumberg. Se coloco en observación y se realizaron
laboratorio pero el paciente refería ya no tener dolor abdominal y al ser examinado no se presento dolor ni resistencia abdominal.

PREGUNTA
Debido a la presentación y evolución del cuadro clínico cual es la conducta a seguir.

RESPUESTA
a.- Explorar posible patología urinaria.
b.- Egresarlo y evaluar en 24 hrs.

CURSO ENARM CMN SIGLO XXI TEL: 36246001 Pharmed Solutions Institute PÁGINA 519
MANUAL DE TRABAJO DEL CURSO ENARM CMN SIGLO XXI
c.- Realizar laparatomia exploratoria.
d.- Realizar apendisectomia laparoscópica.

CASO CLINICO
Masculino de 17 años de edad, que inició con síntomas de dolor abdominal difuso, de gran intensidad que él atribuía a una trasgresión
alimentaría. Se mantuvo en reposo y dieta, no refería vómitos ni diarrea; 24 h más tarde el dolor aún persistía pero de menor
intensidad y se localizaba en abdomen bajo. La descripción y conclusión fueron compatibles con apendicitis aguda, demostrando
diámetros apendiculares de 11 y 8 mm, sensible, no deformable con aumento de ecogenicidad de la grasa adyacente. Al ingreso el
paciente refería poco dolor espontáneo y a la palpación, con un Blumberg positivo. La temperatura axilar era de 37,2°C y el recuento de
leucocitos de 6.700.

PREGUNTA
Cuál es síntoma más relevante en el diagnostico de apendicitis.

RESPUESTA
a.- Dolor típico.
b.- Resistencia muscular.
c.- Leucocitosis con predominio neutrofilos.
d.- Nausea y vomito.

CASO CLINICO
Mujer de 17 años que acude a Urgencias por un cuadro de abdomen agudo. Como antecedentes, destacaba un ingreso un año antes
para estudio por un síndrome febril y cefalea. Fue diagnosticada de una infección por citomegalovirus, desde entonces permaneciendo
asintomática y sin tratamiento. La paciente consultó por dolor abdominal de 6 horas de evolución, iniciado en epigastrio y posterior
migración a fosa ilíaca derecha, intenso, de características continuas, con vómitos, sin fiebre ni otros síntomas. A la exploración
presentaba febrícula, defensa y signos de irritación peritoneal en hemiabdomen derecho. Analítica: leucocitos 16.330/μl, neutrófilos
76,7%, hematocrito 39%, ALT 59 UI/l, GGT 19 UI/l, amilasa 354 UI/l, PCR 0,7mg/dl. Test de embarazo: negativo. Radiología simple:
anodina. No signos de oclusión. Ecografía abdominal: vesícula sin alteraciones. Vía biliar de calibre normal. Pequeña cantidad de líquido
libre en pelvis.

PREGUNTA
Cuál es el método diagnóstico más certero para este padecimiento.

RESPUESTA
a.- Ultrasonido.
b.- Tomografía.
c.- Exploración física.
d.- Rx de pie y decúbito de abdomen.

POLIPOS. CIENCIAS BASICAS: Son protuberancias en la mucosa intestinal, protruyen de la pared hacia la luz intestinal. El patrón
proliferativo alterado de la mucosa del colon que hace que se convierta en pólipo y, posteriormente en carcinoma, deriva de la
activación de un oncogén por mutación de la perdida de genes que suprimen la génesis tumoral. Existen 3 tipos de histológicos:
hamartomas (25%), pólipos hiperplasicos y adenomas. Estos últimos son neoplasias verdaderas con la mayor importancia clínica.
HAMARTOMAS: Es un crecimiento con proliferación excesiva de un tipo de tejido sin cambios neoplásicos verdaderos. SINDROME DE
PEUTZ-JEGHERS: Síndrome autosómico dominante que se manifiesta por pigmentación de zonas mucocutaneas y pólipos
hamartomosos en el intestino delgado y grueso compuestos de cantidades excesivas de muscular de la mucosa. Hay descripción de
progresión de los pólipos a neoplasia maligna, pero no es frecuente. Se observa mayor incidencia de tumores malignos en otros
órganos, como la mama y el ovario. Los síntomas incluyen hemorragia y obstrucción intestinal secundaria a invaginación. POLIPOS
JUVENILES: Por lo general ocurren en niños, pero también se observan en adultos. Son hamartomas compuestos de glándulas dilatadas
y lámina propia anormal. Es posible que ocurra hemorragia secundaria a la autoamputación o invaginación. POLIPOSIS FAMILIAR
JUVENIL: Síndrome autosómico domínate con múltiples pólipos juveniles en todo el colon. Pueden manifestarse por hemorragia u
obstrucción y los pacientes tienen mayor riesgo de presentar tumores malignos en el tubo digestivo. El tratamiento es la colectomía
total. HIPERPLASICOS: Son muy frecuentes y casi siempre miden menos de 5mm; el examen histológico, muestra ausencia de
maduración e hiperplasia sin displasia nuclear. No se consideran premalignos. ADENOMATOSOS: Son los más importantes a causa de
su frecuencia y potencial de malignidad. Algunos se convierten en cáncer, proceso que tarda entre 5-15 años. Tan solo el 5% riesgo de
malignizar. Desde el punto de vista morfológico estos pólipos se describen como pediculados (con un tallo) o sésiles (planos). El
examen histológico, los clasifica como tubulares (87%), vellosos (5%) o tubulovellosos (8%) mixtos conforme el patrón dominante.
Mientras mayor sea el pólipo y mayor componente velloso posea, mas potencial maligno tiene. Casi todos los pólipos benignos son
asintomáticos. En ocasiones los grandes pólipos pediculados, se manifiestan con hemorragia o invaginación. Como regla general, los
pólipos adenomatosos deben extirparse por su potencial maligno. La mayor parte puede extirparse con una trampa por colonoscopia,
casi siempre en una pieza. Los pólipos grandes o sésiles, pueden requerir polipectomía por partes en una o más sesiones. En algunos
casos los pólipos grandes o sésiles de pacientes con riesgo quirúrgico bajo y esperanza de vida larga no pueden extirparse por completo
o en forma segura con una trampa colonoscópica y es necesaria la colectomía. Los pólipos malignos son aquellos en los que los cambios
neoplásicos son profundos, hasta la muscular de la mucosa. En estos casos es probable que la polipectomía aun sea un tratamiento
suficiente si se cubren los siguientes criterios: 1. El pólipo es pediculado. 2. El tallo no está afectado y los márgenes de resección estén
libres. No hay invasión vascular, linfática o neural y la lesión está bien diferenciada. DIAGNOSTICO: La colonoscopia, es la técnica de

CURSO ENARM CMN SIGLO XXI TEL: 36246001 Pharmed Solutions Institute PÁGINA 520
MANUAL DE TRABAJO DEL CURSO ENARM CMN SIGLO XXI
elección para el diagnóstico de pólipos colorectales, es el patrón de oro, además permite la exerecis o la realización de una biopsia, en
los que la exceresis no es posible. Tan importante es la colonoscopia como el estudio histológico del pólipo que permite clasificarlo y
decidir el tratamiento y seguimiento que se debe recomendar. TRATAMIENTO: Todos los pólipos de colon deben tratarse mediante
polipectomía endoscópica completa, ya que su aspecto macroscópico los hace indistinguibles, deben ser recuperados y proceder a
análisis histológico. La polipectomía endoscópica de los adenomas colorectales se ha demostrado que disminuye la incidencia de
cáncer colorectal y es el tratamiento definitivo cuando se efectúa una resección completa. La perforación y el sangrado son as dos
complicaciones más frecuentes descritas.

CASO CLINICO
Paciente femenino de 18 años de edad, con antecedente de hiperpigmentación de retina de ojo derecho y dermatitis atópica. Acude
por sangrado de tubo digestivo bajo, comenzando 3 meses antes de la consulta, el sangrado es rojo, escaso que acompaña de
deposiciones normales diarias, sin dolor defecatorio, ni pujo o tenesmo, agrega que antes del sangrado, presentaba dolor abdominal en
zona epigástrica de forma esporádica, que calma espontáneamente, finalmente refiere dolor articular desde hace 4 meses, sin fiebre ni
causa aparente. El EF es normal, se observa tumoración en zona retroauricular derecha, redondeada de 1 cm de diámetro de
consistencia firme, superficie lisa e indolora, además de tumoraciones en cara interna de ambos pies, sin dolor, redondeada de .5 cm
de diámetro consistente y firme, lisa e indoloro, inspección perineal y anorectal normal.

PREGUNTA
Cual es la conducta a seguir mas adecuada?

RESPUESTA
a.- Transito gastro intestinal.
b.- Panendoscopia.
c.- Polipectomia endoscópica.
d.- Biopsia de pólipo.

COLITIS. CIENCIAS BASICAS: Síndrome gastrointestinal que incluye dolor o molestias abdominales asociado al hábito intestinal en un
periodo de 3 meses. SALUD PUBLICA: 10-20% población general. 20-35% en población económicamente activa. PATOGENIA:
Multifactorial; Sobrecrecimiento bacteriano en intestino delgado, variación en niveles de serotonina en la mucosa intestinal,
incremento en la Colágena Tipo 5. DIAGNOSTICO: Clínico; Dolor abdominal (postprandial, preevacuatorio, asociado al estado
emocional), distención abdominal (postprandial, asociado al estado emocional, evolutivo durante el día). Cambios en el hábito
intestinal; calibre, frecuencia, morfología, consistencia. Síntomas extraintestinales; cefalea, lumbalgia, fatiga, mialgias, urgencias
urinaria, psiquiátricos. Síntomas de alarma; >50 años, pérdida de peso, AHF de Cáncer, fiebre, sangrado, lesiones dérmicas. CRITERIOS
DE ROMA III: Dolor o molestia abdominal al menos 3 días al mes en los últimos 3 meses asociado a los siguientes síntomas. Mejoría con
la defecación. Inicio asociado a cambio en la frecuencia y forma (habito intestinal). “Criterios revisados en los últimos 3 meses e inicio
de la sintomatología en los últimos 6 meses”. TRATAMIENTO: Tratamiento médico: Dieta, fibra, actividad física (ejercicio en grupo).
Diarrea; loperamida, rifaximina, colestiramina, amitriptilina, fibra. Estreñimiento; cinitaprida, mebeverina, tegaserod, dimeticona,
simeticona, fibra. Dolor abdominal; antidepresivos tricíclicos: imipramina, amitriptilina. Probióticos. COLITIS PESEUDOMEMBRANOSA:
Este padecimiento se observa en pacientes que reciben antibióticos de amplio-espectro (clindamicina, penicilinas, semisintéticas,
cefalosporinas).Factores de riesgo: Diabetes Mellitus, VIH, oncológicos, trasplantados, postquirúrgico inmediato. Salud pública: 7% de
adultos son portadores asintomáticos. 13-21% adultos asintomáticos hospitalizados. Hasta 40% de los neonatos <6 meses. Más común
en mujeres. Mortalidad de 2-4%. Pacientes >70 años con +5 días con AB riesgo de 34%. PATOGENIA: Agente causal más frecuente en
diarrea nosocomial (C. difficile) asociada a antibióticos. La alteración de la flora colónica normal permite el crecimiento excesivo de C.
difficile, un anaerobio que produce una exotoxina, con efecto nocivo sobre la mucosa del colon. El uso de antibióticos, induce
esporulación y crecimiento bacteriano logarítmico, la toxina A y B producen daño citológico y lesión epitelial, generando
paseudomembranas. Este síndrome puede presentarse hasta 6 semanas después del tratamiento antibiótico. DIAGNOSTICO: El
espectro clínico varía desde diarrea 93-98% (acuosa, moco, sangre) ligera autolimitada, fiebre 65-70%, dolor abdominal 85-90% (cólico,
abdomen agudo) hasta inflamación transmural intensa, colon toxico y perforación. La leucocitosis no guarda proporción con los datos
clínicos. El diagnostico se establece mediante el análisis fecal en busca de la toxina y por endoscopia, que demuestra las
paseudomemebranas amarillentas típicas, más en recto y sigmoides. TAC: Engrosamiento de la pared colónica, pliegues internos a la
luz, signo de “T-bone Stea ”. CLASIFICACI N: Tipo 1 (summit lesion); Alteración focal del epitelio superficial interglandular, infiltración
de células de reacción en la lámina propia, fibrina subepitelial. Tipo 2 (volcano lesion); Pérdida del patrón glandular, abundante
infiltración de la lámina propia, aparición de psuedomembranas superficiales. TRATAMIENTO: Ayuno. Suspender Antibióticos.
Corrección de balance hídrico y electrolítico. Metronidazol (500mg c/6hrs por 10-14 días) por vía oral o IV (750mg c/8hrs por 10-14
días), primera elección. La vancomicina (125mg c/6hrs por 10-14 dias) es una alternativa, pero debe evitarse siempre que sea factible a
causa del riesgo del surgimiento de enterococos resistentes a este antibiótico. Indicadas cuando no se tenga disponible, alergia o
sensibilidad al Metronidazol o Vancomicina: Bacitracina, teicoplanina, colestiramina, colestipol. Los pacientes con colon toxico o
perforación se someten a resección. El índice de recurrencia después del final del tratamiento se acerca al 20%.

CASO CLINICO
Mujer de 66 años de edad, alérgica a la penicilina. Ingresa para cirugía electiva de divertículos colónicos. Se le practicó sigmoidectomía
asistida por laparoscopia. Como profilaxis antibiótica se utilizó clindamicina y gentamicina. Durante el postoperatorio presenta un
cuadro de abdominalgia, febrícula y diarrea. En los análisis aparece leucocitosis y desviación a la izquierda. Se practica tomografía
computarizada, con resultado normal. Se decide reoperar ante la progresión del cuadro, y no se halla dehiscencia de sutura ni isquemia.
La evolución es tórpida, y la paciente fallece. Los cultivos de heces fueron positivos para la toxina. La anatomía patológica mostró los
hallazgos típicos de colitis seudomembranosa.

CURSO ENARM CMN SIGLO XXI TEL: 36246001 Pharmed Solutions Institute PÁGINA 521
MANUAL DE TRABAJO DEL CURSO ENARM CMN SIGLO XXI

PREGUNTA
El agente etiológico de la colitis pseudomembranoso es:

RESPUESTA
a.- C. difficile.
b.- E. coli.
c.- Enteroco fecalis.
d.- Enterobacter.

CASO CLINICO
Varón de 64 años ingresó por diarrea infecciosa. Destacan también como antecedentes varias infecciones respiratorias en los 2 meses
previos, por los que había recibido tratamiento con cefepime, amikacina y amoxicilina-clavulánico, y portador de una sonda de
nutrición enteral. Al ingreso presentaba 10-15 deposiciones diarreicas/día, acuosas, acompañadas de dolor abdominal de tipo cólico y
febrícula. En la analítica al ingreso sólo destacaba leucocitosis de 13,000 y anemia leve. Se inició tratamiento con metronidazol i.v.
500mg/8 h ante la sospecha de infección por C. difficile, tras toma de muestras para toxina. Se realizó rectoscopia al segundo día de
ingreso que confirma colitis seudomembranosa. Al tercer día del ingreso, el paciente presenta deterioro clínico importante, con
disminución del número de deposiciones y aumento del dolor abdominal, que se hizo continuo, difuso, acompañado de distensión
abdominal e hipotensión. En la analítica destaca leucocitosis de 23,000, insuficiencia renal, con creatinina de 190 mg/dl y urea de 12,6
mg/dl, y lactato sérico de 4 mmol/l. Una TC de abdomen muestra engrosamiento y edema difuso de todo el colon, más evidente en
sigma y colon descendente.

PREGUNTA
El tratamiento médico ideal para colitis pseudomembranosa es:

RESPUESTA
a.- Amikacina
b.- Ciprofloxacino
c.- Penicilina
d.- Vancomicina

HEMORROIDES. CIENCIAS BASICAS: Las hemorroides son dilataciones de los plexos hemorroidarios superior e inferior. Están
localizadas en los últimos centímetros del recto, en el conducto anal y en el recto. Forman parte de la anatomía normal de la región y
cuando sufren alteraciones y producen síntomas se establece la enfermedad. Factores de riesgo: Multifactorial, estreñimiento, dieta,
embarazo, edad, herencia?. SALUD PÚBLICA: En México, 5% de la población general presenta síntomas relacionados con las
hemorroides. Son raras antes de los 20 años de edad; su frecuencia aumenta con la edad y es posible que el 50% de los adultos de 50
años tenga o haya sufrido sintomatología hemorroidaria. Incidencia de 4 a 36%. Incidencia máxima de 30-60 años. Sin diferencia racial.
Más en mujeres pero mayor severidad en hombres. PATOGENIA: Estreñimiento, al implicarse mayor esfuerzo al evacuar, se produce
congestión de los “cojinetes” hemorroidales. Al persistir el pujo constante se distienden los soportes de dichos cojinetes y va
provocando el prolapso hemorroidario por debajo de la línea anorectal hacia fuera del conducto anal. Alteraciones morfológicas del
Traitz. Elongación de las estructuras hemorroidales. Cambio del tejido elástico. Hipertonía del EAI. Cambio irreversible en los vasos.
CLASIFICACIÓN: Anatómica 1. Internas (cubiertas por mucosas): I-IV, grado I; son cuando éstas se exteriorizan hasta el conducto anal y
sangran. Grado II; son aquellas que sangran y se prolapsan al momento de la defecación a nivel de ano pero se reducen
espontáneamente. Grado III; son aquellas que sangran y se prolapsan más allá del ano al momento de la defecación y requieren
reducción manual. Grado IV; son las que sangran y se prolapsan en forma permanente y son irreductibles. 2. Externas (cubiertas por
piel): vascular, pliegue. 3. Mixtas. De acuerdo a su situación: Complicadas; edema, trombosis, sangrado, absceso. No complicadas;
sangrado 89% (Rutilante, ordeña, no doloroso, postevacuatorio, abundante en el papel), prolapso 36% (Pujo, asociado a la esfuerzo,
reducción), prurito 27% (Posterior a la evacuación, aseo deficiente, aseo excesivo), dolor 11% (solo con complicaciones). DIAGNOSTICO:
Clínico; Hemorroides externas, síntomas y signos principales dolor, tumoración y prurito anal. Hemorroides internas,
fundamentalmente son dos: hemorragia y prolapso hemorroidario, la hemorragia: es el signo más frecuente y más constante. La sangre
por lo regular es roja, rutilante, expulsada con la metería fecal, en forma de estrías en las heces fecales o en gotas al final de la
evacuación. Inspección; Identificar hemorroides externas o prolapso de internas. Palpación; Pliegues externos, tejido hemorroidal
redundante. Anoscopía, Identificación y visión directa de los paquetes. TRATAMIENTO: Médico: Incremento en el consumo de agua,
consumo de Fibra (30 gr), flebotónicos: Hidrosmina (sies) 200mg cada 8 horas 5 días, Diosmina (daflon) 450mg cada 8 horas 5 días,
Hisperidina (eletec) 50mg cada 8 horas 5 días. Ligadura con banda elástica. Escleroterapia: Esta consiste en la inyección de pequeñas
cantidades de hidroxipolietoxidodecanol al 3% aplicado en la submucosa y extravascular por arriba del paquete vascular externo.
Crioterapia: Se utiliza óxido nitroso que congela a una temperatura de menos 60° C y menos 80° C o con nitrógeno líquido que congela
a una temperatura de menos 180° C. El principio es la destrucción del tejido por medio de la coagulación. Electrocoagulación: Se basa
en un rayo calórico que evapora el agua intracelular y coagula las proteínas. Se recomienda su aplicación de 1.5 segundos en número de
3 a 4 en la base del tejido hemorroidario interno. Tratamiento Quirúrgico: Milligan- Morgan. Tipo Parks (semiabierta). Ferguson
(cerrada). PPH (Procedure for Prolapse and Hemhorroids). El tratamiento quirúrgico está indicado en todos los pacientes en los que se
ha fracasado el tratamiento médico, en aquellos pacientes que tienen sintomatología de hemorroides externas, en hemorroides
internas grado III y IV y en las complicaciones de la enfermedad hemorroidaria. COMPLICACIONES: Trombosis hemorroidaria única
externa; Es una complicación frecuente de causa desconocida. En forma clásica el paciente refiere dolor de forma brusca, acompañado
de tumoración perianal. El tratamiento puede ser médico o si el dolor es muy intenso, está indicada la trombectomía bajo anestesia
local. Trombosis hemorroidaria múltiple o masiva; Consiste en la trombosis de dos o más paquetes hemorroidarios, acompañada de

CURSO ENARM CMN SIGLO XXI TEL: 36246001 Pharmed Solutions Institute PÁGINA 522
MANUAL DE TRABAJO DEL CURSO ENARM CMN SIGLO XXI
edema y dolor intenso. Dependiendo del tiempo de evolución y su sintomatología, puede ser tributario o de tratamiento médico o
quirúrgico. Hemorroides internas estranguladas. El paciente refiere dolor anal intenso de corta duración, acompañado de múltiples
hemorroides que no se reducen con las maniobras habituales. Si continua su desarrollo se convierte en hemorroides gangrenosas. El
tratamiento siempre debe ser quirúrgico y de urgencia

CASO CLINICO
Se trata de paciente masculino de 46 años de edad el cual refiere dolor en la regio rectal, presenta manchado sanguinolento secundario
a la evacuación, actualmente se encuentra estabilizado y se prepara para realizar procedimiento quirúrgico electivo.

PREGUNTA
Considerando la preparación del paciente, cual es la indicación más apropiada.

RESPUESTA
a.- Indica aumento de fibra a la dieta.
b.- Prepara con polietilenglicol por via oral.
c.- Se requiere enema baritado.
d.- Indica lactulosa por via oral.

HERNIAS DE PARED ABDOMINAL. CIENCIAS BASICAS: Una hernia es la protrusión de una víscera a través de una abertura en la pared
(natural o adquirido) de la cavidad que la contiene, las características importantes de una hernia son el orificio defecto en la capa
aponeurótica más interna del abdomen) y el saco herniarios (protrusión del peritoneo). El cuello del saco herniario corresponde al
orificio. Sitios de herniación: son la ingle (inguinal), la cicatriz umbilical (umbilical), la línea alba, la línea semilunar de Spiegel, el
diafragma y las incisiones quirúrgicas (postincisional). SALUD PUBLICA: Incidencia 10% población general. 90% de hernias
inguinales/incisionales. CLASIFICACION: La hernia es externa si el saco protruye completo a través de la pared abdominal; es interno si
el saco está dentro de la cavidad visceral. Una hernia es reducible cuando la víscera herniada puede regresar al abdomen; es
irreducible cuando esta resulta imposible. Una hernia estrangulada es aquella en la que la vascularidad de la víscera que protruye se
compromete hay sufrimiento del contenido (isquemia), necrosis y puede llegar a sepsis (translocación bacteriana). El estrangulamiento
ocurre en hernias con orificios pequeños y sacos grandes. Una hernia incarcerada es una que no puede reducirse pero que no está
estrangulada. En una hernia de Richter el contenido del saco consiste en solo un lado de la pared del intestino (siempre es
antimesentérica). PATOGENIA: La herniación puede abarcar la grasa preperitoneal, órganos retroperitoneales y un saco herniado de
peritoneo con estructuras intraperitoneales (por ejemplo, epiplón u órganos). Son poco comunes las hernias clínicamente significativas
que carecen de saco peritoneal. Las hernias pueden complicarse cuando la inclusión de una víscera forma una pared del saco herniario.
Ello comprende un órgano parcialmente retroperitoneal y recibe el nombre de hernia por deslizamiento, de las cuales las más comunes
afectan al colon. Etiología: multifactorial causas enfermedad de tejido conectivo, tabaquismo, EPOC, actividad física, edad avanzada.
Por debilitamiento de la fascia transversalis. Aumento de la presión intraabdominalen: Embarazo, ascitis, obesidad, obstrucción
intestinal, esfuerzo defecatorio, prostatismo, ejercicio (Pesas). DIAGNOSTICO: Clínica; la historia natural de las hernias es un
crecimiento lento hasta llegar a ser irreducibles y desfigurantes, con riesgo de estrangulaciones. Las molestias que producen siempre
son peores al final del día y se alivian por la noche cuando el paciente se acuesta y la hernia se reduce. El dolor inguinal sin una hernia
demostrable, casi nunca indica el inicio de una hernia. La mayor parte tiene un inicio insidioso, pero algunas se desencadenan por
esfuerzo muscular intenso. Por lo general un saco herniario con su contenido crece y transmite un impulso palpable cuando el paciente
hace esfuerzos o tose. El paciente debe ponerse de pie en la exploración, porque suele resultar imposible palpar una hernia inguinal
reducida con el paciente en posición supina. Las hernias que no son detectables mediante exploración física pueden demostrarse
mediante ultrasonido o TAC. El estrangulamiento produce dolor intenso en la hernia seguido muy pronto por hipersensibilidad,
obstrucción intestinal y manifestaciones de sepsis. La reducción de una hernia estrangulada está contraindicada cuando hay sepsis o si
se cree que el contenido del saco presenta gangrena. TRATAMIENTO: Casi todas las hernias deben repararse a menos de que las
condiciones locales o sistémicas del paciente impidan un resultado favorable. La posible excepción es una hernia con cuello amplio y
saco poco profundo, en cuyo caso se espera que crezca despacio. Los bragueros son útiles en el tratamiento de hernias pequeñas
cuando la operación esta contraindicada, pero están contraindicados en pacientes con hernias femorales. Reparación laparoscópica:
Actualmente indicada en prácticamente cualquier tipo de hernia de pared, hernias incisionales extensas, hernias incarceradas con o sin
crisis de oclusión. Contraindicaciones absolutas: Eventraciones supergigantes con obesidad asociada y ausencia de elementos
musculoaponeuróticos, fístulas enterocutáneas o foco séptico intraabdominal, cirrosis hepática con ascitis libre, hernias con
estrangulación intestinal en fase temprana. HERNIAS INGUINALES: La ingle es una de las áreas débiles por naturaleza de la pared
abdominal y es el sitio más frecuente de herniación. Los varones tienen una probabilidad de 25 veces mayor de padecer una hernia
inguinal. Las hernias que surgen por arriba del pliegue abdominocrural son inguinales, las que emergen por debajo de ese pliegue son
femorales. Las hernias femorales ocurren ocasionalmente en las mujeres, pero no con tanta frecuencia como las inguinales; en los
varones son raras. Las hernias femorales suelen presentarse como una masa irreducible en la base medial del triángulo femoral de
Scarpa. La incidencia aceptada es de 3-4 %. La estrangulación se produce en 1.3-3.0% de las hernias inguinales. Las hernias inguinales
se dividen en directas e indirectas. Indirecta: Un saco herniario indirecto es un proceso vaginal con dilatación persistente, su saco pasa
en sentido oblicuo o en forma indirecta hasta llegar al escroto, son dos veces más frecuentes en los varones Directa: Los sacos se
originan en el piso del trayecto inguinal (triángulo de Hesselbach), protruye en formas directa y quedan contenidos por la aponeurosis
del musculo oblicuo externo. Es frecuente que la vejiga sea un componente deslizante de un saco herniario directo. Casi todas las
hernias estranguladas son inguinales indirectas, pero las femorales tienen el mayor índice de estrangulación. Diagnóstico: Se
diagnostica mediante la exploración física del canal inguinal y región crural. Aumento de la región inguinal. Aumenta con el esfuerzo,
disminuye con el reposo. Se asocia a dolor en la región inguinal, distinguir si es reductible o irreductible. Las hernias inguinales pueden
ser congénitas o adquiridas. Todas las inguinales indirectas son congénitas. El objetivo de la hernioplastia inguinal es prevenir la
protrusión peritoneal a través del orificio miopectíneo. La integridad se recupera de dos formas fundamentales: 1) cierre aponeurótico

CURSO ENARM CMN SIGLO XXI TEL: 36246001 Pharmed Solutions Institute PÁGINA 523
MANUAL DE TRABAJO DEL CURSO ENARM CMN SIGLO XXI
del orificio miopectíneo hasta la magnitud necesaria y 2) reposición de la fascia transversal defectuosa con unja prótesis sintética
grande. A veces se combinan ambos métodos. Las hernias se reparan por vía anterior a través de una incisión inguinal o posterior por
una incisión abdominal. El abordaje anterior es la incisión más popular para la hernioplastia inguinal. Las reparaciones posteriores se
denominan hernioplastias properitoneales. Las protecciones sintéticas de malla tienen una función importante en el tratamiento de las
hernias inguinales. Las prótesis de malla se usan como parche o tapón del orificio miopectíneo para reforzar una reparación clásica y
para reponer la fascia transversal. HERNIA UMBILICAL: Son más frecuentes en mujeres. Constituyen el 2.4% de los casos de Hernias de
pared. 10% en adultos con antecedente de hernia en la infancia. La obesidad y los embarazos repetidos son los precursores usuales.
Suele observarse estrangulación de colon y epiplón. Las hernias de este tipo que tienen un defecto parietal pequeño solo se cierran con
sutura de polipropileno; aquellas con grandes defectos parietales se corrigen con una prótesis. HERNIA DE SPIEGEL: Son hernias
ventrales que ocurren a lo largo de la porción subumbilical de la línea semilunar de Spiegel y a través de la fascia del mismo nombre.
Estas hernias son raras (1%) y a menos que sean grandes, su diagnóstico se dificulta porque son interparietales y están contenidas por
la aponeurosis del musculo oblicuo externo. Son más frecuentes en el área entre la cicatriz umbilical y la línea que conecta las espinas
ilíacas anterosuperiores, y el área por debajo de la línea arqueada y por arriba de los vasos epigástricos inferiores. 21% incareceradas.
HERNIA EPIGASTRICA: Es una protrusión de grasa properitoneal y peritoneo a través de las fibras cruzadas de la vaina del recto en la
línea media (alba), entre el apéndice xifoides y la cicatriz umbilical. Las hernias epigástricas a menudo son irreducibles, siempre tienen
defectos aponeuróticos pequeños, pueden ser múltiples. HERNIAS INCISIONALES: Son problemas quirúrgicos graves. La obesidad y la
infección son las causas principales, de este trastorno, otras dehiscencia de la herida, hematoma o seroma, técnica deficiente en el
cierre de la herida, desnutrición. El peso del panículo separa la incisión quirúrgica y la infección obstaculiza la cicatrización de la herida.
Una hernia incisional grande produce movimiento abdominal respiratorio paradójico similar al tórax inestable. La función diafragmática
se vuelve ineficiente. La mayor parte de las hernias incisionales pequeñas se trata con cierre simple de defecto aponeurótico. Las
hernias con defectos aponeuróticos mayores de 10 cm tienen índices de recurrencia hasta de 50%. Por tanto muchas de las hernias
incisionales y todas las recurrentes requieren una prótesis para conseguir una reparación exitosa. Se prefiere la hernioplastia de Stoppa
(se implanta una prótesis muy grande de mersilene en la profundidad de los músculos de la pared abdominal sobre la vaina posterior
del recto o el peritoneo). La infección es una complicación grave que se presenta en un 10% de los pacientes. La infección temprana se
trata mediante exposición rápida y completa de la prótesis. Con tratamiento antimicrobiano intenso local y sistémico puede esperarse
la integración completa de la prótesis. La reintegración de la prótesis infectada casi nunca se logra cuando hay infecciones tardías y
debe retirarse la porción secuestrada de la prótesis. HERNIA DE AMYAND: Aquella que contiene el apéndice cecal inflamado a través
de un defecto herniario inguinal. HERNIA PARAESTOMAL: Estas interfieren con las irrigaciones de colostomía y la adhesión de los
artículos para el cuidado de la estoma. Las hernias paracolostomales son más frecuentes que las hernias paraileostomales y la
ocurrencia de ambas es más probable cuando el estoma emerge por la línea semilunar en lugar de la vaina del recto. Por lo general las
hernias parastomales son laterales a la ostomía. Se prefiere el cambio de sitio de la estoma sobre la reparación local. La reparación local
falla con frecuencia porque los m sculos “cinturón” laterales a la ostomía carecen de aponeurosis suficiente, por ello se prefiere
implantar un trozo grande de mersilene con una hendidura para acomodar el estoma.

CASO CLINICO
Paciente de 21 años de edad que acude a consulta por presencia de dolor en abdomen señalando la zona umbilical, a la exploración
física se observa abdomen con masa depresible levemente dolorosa, que protruye espontáneamente al pujar y al incorporarse.

PREGUNTA
Cuál de las siguientes medidas no está indicado en este paciente.

RESPUESTA
a.- Confirmar con laboratorio y gabinete con ECG.
b.- Realización de estudios preoperatorios.
c.- Envió a segundo nivel y seguimiento posoperatorio.
d.- Confirmar hernia umbilical no complicada.

ORQUIEPIDIDIMITIS. CIENCIAS BASICAS: La epididimitis, orquitis y orquiepididimitis se definen como la inflación del epidídimo,
testículo y del testículo/epidídimo respectivamente. Hablaremos de orquiepididimitis aguda si el proceso dura menos de 6 semanas y
crónica si es mayor. La causa más frecuente en pre-púberes son las bacterias coliformes que infectan la vía urinaria, en hombres
sexualmente activos predominantemente se encuentra infección por Chlamidya trachomatis y N. gonorrehae, en hombres mayores con
menor actividad sexual. Y con antecedentes de patología obstructiva urinaria o instrumentación reciente presentan infección por
patógenos urinarios como E. coli y paseudomonas. En homosexuales suele estar causada por coliformes y haemophilus influenzae.
Otras causas son tuberculosis, brucelosis, secundaria a traumatismos y medicamentos (amiodarona, acumulación de
desetilamiodarona) y con menor frecuencia idiopática, antecedente de enfermedad de Behcet (vasculitis multiorgánica idiopática, con
ulceras genitales, ulceras aftosas recurrentes y uveítis). SALUD PUBLICA: Constituye la causa más común (80-90%) de escroto agudo en
pacientes mayores de 18 años. El 56% de los hombres >60años que presentan epididimitis se asocia con obstrucción de tracto urinario.
50% de pacientes con antecedentes de instrumentación, uretral, catéter permanente o intermitente pueden presentar epididimitis
infecciosa y de estos el 80% es de origen bacteriano (E. coli). PATOGENIA: El principal mecanismo patogénico, lo constituye el reflujo de
orina infectada de forma ascendente (retrograda) hacia los conductos eyaculadores y deferente. La orquitis se produce por
contigüidad. DIAGNOSTICO: Clínica; Dolor escrotal intenso de instauración gradual con signos de inflamación en el hemiescroto
afectado, incluyendo el epidídimo y el conducto deferente. Suele acompañarse de afección del estado general con fiebre (en más de
50% de los casos, >38°) y síntomas urinarios (disuria, secreción uretral). Puede evolucionar a absceso escrotal cuyo tratamiento es el
drenaje. La inflamación comienza en la cola del epidídimo y se extiende hacia el testículo. El cordón espermático esta engrosado y
tumefacto y es frecuente la existencia de hidrocele reactivo. La elevación del testículo produce disminución del dolor (signo de Prehn
positivo), reflejo cremasteriano presente, la trasniluminacion, puede ser positivo por hidrocele reactivo. El diagnóstico es

CURSO ENARM CMN SIGLO XXI TEL: 36246001 Pharmed Solutions Institute PÁGINA 524
MANUAL DE TRABAJO DEL CURSO ENARM CMN SIGLO XXI
fundamentalmente clínico, debemos realizar una BH en el que se encontrara leucocitosis y neutrofilia así como un sedimento de orina
que puede ser patológico con piuria y/o bacteriuria. El urocultivo es negativo en 40-90% de los casos. En pacientes con prácticas
sexuales de riesgo y con flujo uretral se solicita: exudado uretral, tinción gramm si se sospecha gonorrea, prueba de ELISA si se
sospecha VIH. En caso de duda diagnostica podemos recurrir a la ecografía doppler que detectara un flujo testicular conservado o
aumentado y signos sugestivos de orquiepididimitis como acentuación de septos, existencia de focos hiperecogpénicos y epidídimo
aumentado de tamaño. TRATAMIENTO: Instaurar un tratamiento empírico sin esperar resultados de cultivos. En caso de sospecha de
infección de transmisión sexual (antecedente de contacto sexual sospechosos, uretritis, infección diagnosticada en pareja sexual) se
deberá iniciar tratamiento con azitromicna o ceftriaxona (1 g IM o IV) en dosis única junto con doxiciclina 100mgs/12hrs. Como
alternativa tenemos levofloxacino 500mgs c/12 hrs. Ambas pautas deben mantenerse por 10 días. En pacientes sin práctica sexual
trimetropim/sulfametoxaxol por 10-14 días. Debe tratarse a los compañeros sexuales del último mes y evitar el coito sin protección
hasta haber terminado tratamiento. Si sospecha infección bacteriana inespecífica disponemos de varias pautas: ciprofloxacino 500mg/
12 hrs, norfloxacino 400mgs/ 12 hrs o amoxicilina-clavulanico 875mgs/ 8 hrs durante 10-14 días. En casos floridos debemos asociar
aminoglucocidos: tobramicina 100mgs IM o IV c/12 hrs o gentamicina 240mgs IM o IV cada 24 hrs. Además del tratamiento antibiótico,
existe una serie de normas generales que favorecen el alivio de los síntomas; actividad física limitada, suspensorio testicular, aplicación
de frio local. Para el control, de dolor se recomienda el uso de AINEs (naproxen o diclofenaco por 2 semanas) que además favorecen la
disminución del proceso inflamatorio. El edema hemiescrotal puede persistir durante 4-6 semanas después del tratamiento y de forma
residual puede persistir una induración epididimaria indefinida. COMPLICACIONES: Absceso en epidídimo, oligoatenosospermia,
oligoatenosteratospermia, azoospermia, dolor crónico.

CASO CLINICO
Varón de 35 años de edad. Consultó por presentar desde hacía tres meses, un aumento del testiculo derecho y por “manchar” la cama
con líquido “como pus”. En la exploración testicular destacaba un aumento irregular del tamaño escrotal, de consistencia dura, con
ulceración escrotal y secreción de tipo purulento. La analítica sanguínea reflejó una leucocitosis como nico hallazgo a reseñar (la
bioquímica y marcadores tumorales fueron negativos).

PREGUNTA
Cuál de los siguientes antecedentes es más frecuente en este caso.

RESPUESTA
a.- Enfermedad de transmisión sexual recurrente.
b.- Presencia de bacterias anaerobias.
c.- Enfermedad de Behcets.
d.- Varicocele no tratado.

TORSION TESTICULAR. CIENCIAS BASICAS: Consiste en la rotación del cordón espermático sobre sí mismo ocasionando una
disminución del aporte sanguíneo al testículo y al epidídimo. SALUD PUBLICA: Presenta una incidencia de 1/4000 varones menores de
25 años. Es la causa más frecuente de pérdida testicular en el varón joven. Puede aparecer a cualquier edad pero presenta dos picos de
incidencia, uno durante el primer año de vida y otro en la pubertad. PATOGENIA: La etiología es muy variada aunque suele existir una
alteración bilateral de la suspensión que permite que al testículo y cordón espermático rotar sobre sí mismos. La tasa de viabilidad cae
desde un 85-100%, cuando se resuelve en las primeras horas, hasta un 20% a las 12 hrs. En las 4 primeras horas se afectan las células
de la espermatogénesis. Las células de Leydig son más resistentes aproximadamente unas 10 hrs. Existen 2 formas anatomoclinicas:
torsión extravaginal; constituye un 5% de las torsiones testiculares. Es prácticamente exclusiva del neonato. Torsión intravaginal; forma
típica del niño, adolescente y adulto joven, con un pico entre 8-15 años. No siempre es de aparición brusca. Existen formas
incompletas. Es la forma más frecuente y constituye el 95% de los casos. DIAGNOSTICO: Se caracteriza por dolor testicular de inicio
súbito o gradual que puede irradiarse a región inguinal o suprapúbica, acompañado de tumefacción escrotal. Pueden aparecer nauseas,
vómitos o hiperpirexia. El 50% de las causas ocurre durante el sueño. En la exploración física el testículo suele estar elevado hacia el
anillo inguinal superficial y horizontalizado (signos de Governeur), ser doloroso al tacto e incluso estar aumentado de tamaño. La piel
del escroto puede estar edematosa y enrojecida, lo que dificulta la exploración testicular. Al elevar el testículo hacia la sínfisis del pubis,
no disminuye o aumenta el dolor (signo de Prehn negativo) y existe ausencia de reflejo cremasterico (reflejo cutáneo que aparece al
rozar la cara superointerna del muslo, produciendo elevación del testículo homolateral o la contracción de la pared abdominal). La
identificación del epidídimo en posición anterior o lateral y la palpación de un cordón espermático de consistencia blanda y algo
congestivo son indicios muy sugestivos de torsión testicular. Si en la exploración física detectamos un testículo ascendido y
horizontalizado con el epidídimo en posición anterior debemos intentar, tras la administración de sedación, la destorsión manual. En
caso de que exista duda diagnostica está indicada la realización de una ecografía doppler; que constituye el. TRATAMIENTO: La torsión
testicular constituye una autentica urgencia urológica dado que la viabilidad testicular es inversamente proporcional a la duración de la
torsión, para restablecer cuanto antes el flujo vascular al testículo evitando así la necrosis y atrofia posterior. El tratamiento definitivo
es quirúrgico, si bien como medida temporal puede intentarse la destorsion manual. Se trata de una medida rápida, segura y no
invasiva. Tras esta maniobra debe realizarse una ecografía para comprobar la reperfusión testicular. Si esta medida es efectiva, la
oquidopexia (fijación del testículo a la pared escrotal), que constituye el tratamiento de elección de la torsión podrá realizarse de forma
electiva. Se recomienda que la orquidopexia sea bilateral ya que la causa anatómica que predispone a la torsión suele serlo. Algunos
autores recomiendan también la orquidopexia contralateral en caso necesario practicar una orquiectomía, en pacientes con historia
previa de torsiones.

CURSO ENARM CMN SIGLO XXI TEL: 36246001 Pharmed Solutions Institute PÁGINA 525
MANUAL DE TRABAJO DEL CURSO ENARM CMN SIGLO XXI
CASO CLINICO
Masculino de 18 años de edad, presentó con dolor testicular izquierdo pocos días de duración. No hay otros síntomas. El examen físico
reveló una induración en el polo superior del testículo izquierdo. Los marcadores tumorales fueron normales. Ecografía demostró una
lesión intratesticular mal definida, hipoecoica, 13,2 x 12,6 mm. En ecografía Doppler de potencia, una zona hipoecoica apareció
completamente avascular en contraste con el resto del testículo.
PREGUNTA
Cual se las siguientes manifestación considera más relevante para el diagnostico.

RESPUESTA
a.- Signo de Prehn positivo.
b.- Reflejo cremasterico.
c.- Trasluminación positiva.
d.- USG testicular.

CURSO ENARM CMN SIGLO XXI TEL: 36246001 Pharmed Solutions Institute PÁGINA 526
MANUAL DE TRABAJO DEL CURSO ENARM CMN SIGLO XXI
CANCER TESTICULAR:
CIENCIAS BASICAS: El tumor maligno de testículo es una de las neoplasias solidas más común y curable en el hombre entre 15-35 años.
El 95% de los tumores malignos de testículo se origina del epitelio germinal primordial y 5% son del estroma gonadal. Factores de
riesgo: Historia familiar de cáncer en familiares de primer grado, criptorquidea (principal factor), infertilidad, síndrome de Klinefelter,
tumor testicular contralateral, orquitis viral por sarampion. SALUD PUBLICA: Ocupa el 2% de todas las neoplasias. Tiene 3 picos de
incidencia importante en la infancia, edades de 25-40 años y a los 60 años. CLASIFICACION: Los tumores malignos de testículo de origen
germinal se clasifican en semínoma puros 50% (extremadamente sensibles a radioterapia) y no seminoma (responden a quimioterapia
basada en platino) 35% son puros y 15% mixtos. Los no seminomatosos incluyen los subtipos histológicos de senos endodérmicos,
coriocarcinoma, carcinoma embrionario y teratoma maduro e inmaduro. DIAGNOSTICO: Temprano; la presencia de una masa testicular
solida indolora, es patognomónica de tumor testicular y se puede acompañar de dolor local, dolor de espalda y en flanco ipsilateral, tos,
hemoptisis. Se pueden presentar signos y síntomas de enfermedad metastásica. Se debe realizar una exploración detallada del área
genital incluida la transiluminacion escrotal, abdomen en búsqueda de masas palpables, en tórax y abdomen buscando ginecomastia o
ganglios supraclaviculares. El 20% de los casos presentan Hidrocele secundaria. En neoplasia hormonosecretante: ginecomastia,
macrogenitosomia. Laboratorio y gabinete: Se debe efectuar determinación sérica de marcadores tumorales: alfafetoproteina, normal
<15ng/ml (AFP), se eleva en tumores de senos endodermicos y en carcinoma embrionario, nunca se eleva en seminomas puros.
Fracción beta de la hormona gonadotropina corionica Humana (HGC), se eleva principalmente en coriocarcinoma. Deshidrogenasa
láctica (DHL), se eleva en 60% de pacientes con tumores no seminomatosos y en el 80% de los seminomas en etapas avanzadas. Se
debe realizar ultrasonido testicular para confirmar diagnóstico de tumor sólido. Para la etapificación del tumor conformado el
diagnóstico de cáncer se debe realizar: TAC abdominopelvica y Rx de tórax PA y lateral. Confirmación mediante histología. El PET-CT es
útil para evaluar viabilidad de tumor residual posterior a tratamiento con quimioterapia en pacientes con tumores seminomatosos.
TRATAMIENTO: El cáncer testicular es altamente curable, incluso en etapas avanzadas. La orquiectomía inguinal radical y ligadura del
cordón espermático en el anillo inguinal profundo respetando así el drenaje linfático, es el estándar de tratamiento para los tumores
testiculares. SEMINOMA: La etapa clínica I tres opciones de manejo: vigilancia, radioterapia, quimioterapia (carboplatino). Etapa IIA:
radioterapia. Etapa IIB: radioterapia o quimioterapia (bleomicina, etoposido, platino=BEP). Etapa IIC y III: se considera enfermedad
avanzada y debe manejarse con quimioterapia (BEP). NO SEMINOMA: Etapa clínica I: de bajo riesgo, con vigilancia, quimioterapia,
linfadenectiomía retroperitonela neuropreservadora. Etapa I: de alto riesgo, quimioterapia (BEP), linfadenectiomia retroperitonela
neuropreservadora, vigilancia si la quimioterapia está contraindicada y no se desea cirugía. Etapa IS: (marcadores séricos sin evidencia
radiológica de tumor) deberá recibir quimioterapia (BEP). Etapa IIA y IIB con marcadores tumorales negativos puede manejarse con
vigilancia, linfadenectiomia retroperitonela neuropreservadora. Etapa IIA y IIB, con marcadores tumorales elevados deberá manejarse
con quimioterapia. Etapa IIC y III, deberá manejarse con quimioterapia. PRONOSTICO: El de mejor pronóstico seminoma. El de peor
pronóstico coriocarcinoma. Seminomas estadios I o IIA operados y en irradiación ulterior o quimioterapia; sobrevida mayor a 5 años.
No seminomatosos 75 a 80% de éxito. Estatus libre de enfermedad: RIESGO NO SEMINOMA SEMINOMA
primeros dos años: determinación de marcadores biológicos, RX de Riesgo Tumor primario testicular o Cualquier sitio primario y
tórax cada 2 meses y TAC cada 3 meses. 3 años: marcadores biológicos, bajo retroperitoneal y sin sin metástasis
(bueno) metástasis pulmonares o pulmonares o viscerales
RX tórax cada 3 meses y TAC cada 6 meses. 4 años: controles viscerales. Cualquier valor de y AFP: normal. hCG
semestrales. 5 años: controles anuales. TUMORES GERMINALES: los siguientes: AFP cualquier valor. LDH
CORIOCARCINOMA: Afortunadamente infrecuentes (<2%). Presenta <1,000ng/ml. hCG <5,000iu/l. cualquier valor.
elementos cito y sincitiotrofoblasticos, extremadamente infrecuentes. LDH <1.5 por arriba del límite
normal
Elevada tasa metastasica. Se presenta en forma de lesión pequeña y
Riesgo Tumor primario testicular o Cualquier sitio primario y
con centro necrohemorragico. Su diseminación es rápida y por vía interme retroperitoneal y sin con metástasis
hematógena habitualmente. Presenta casi 100% de metástasis al dio metástasis pulmonares o pulmonares o viscerales
momento del diagnóstico. TERATOCARCINOMA Y TERATOMAS: viscerales, con AFP 1,000- y AFP: normal. Hcg
Derivan de una o más hojas embrionarias, contiene restos de las tres 10,000ng/ml. hCG 5,000- cualquier valor. LDH
50,000iu/l. LDH1.5-10 por cualquier valor.
capas embrionarias (ecto, meso y endodermo). La forma inmadura arriba de limite normal.
contiene elementos celulares primitivos indiferenciados. Alta Riesgo Tumor primario en mediastino No existen pacientes
resistencia a los tratamientos radio y quimioterapicos. Metastasis en alto y sin metástasis pulmonares o clasificados con riesgo
15% al momento del diagnóstico. CARCINOMA EMBRIONARIO: Forma viscerales con AFP >10,000 alto
ng/ml. hCG >50,000iu/l. LDH
el 25% de los tumores germinales. Existe en 2 formas; La del adulto (Ca.
>19 por arriba de limite
Embrionario propiamente dicho). La infantil (tumor de Yolk, del saco normal
vitelino o del seno endodermico), más frecuente tumor testicular en
niños. Predominan las formas mixtas y necrohemorragicas. Elevada tasa metastásica (60%), ganglios regionales. El tumor el saco
vitelino es el considerado más agresivo de la línea germinal. TUMORES NO GERMINALES (Estromales). En general derivan del conducto
de Wolf, son por regla benignos. Se presentan en los tejidos de soporte y en los productores hormonales de los testículos. TUMOR DE
CELULAS DE LEYDIG: Suponen el 1-3 % del total de todos los tumores testiculares. Es el más común de los tumores testiculares de
células no germinales. Su origen deriva de las células intersticiales del testículo, células productoras de androgeneos. La bilateralidad en
los tumores testiculares es excepcional, solo ocurre en el 0,7% sincrónicamente, y con carácter metacrónico (hasta en los 5 años
posteriores al diagnóstico del contralateral) en el 1,5%. Sin embargo en los tumores de Leydig y Sertoli puede elevarse al 5-10%. El
tumor de Leydig en el adulto provoca manifestaciones endocrinas en el 20-40% de los casos. Estos tumores son hormonoproductores
por lo que pueden dar manifestaciones endocrinas, y cuando las dan serán distintas dependiendo de la edad de los pacientes: Adulto:
ginecomastia, disfunción eréctil y disminución de la libido, oligozoospermia y atrofia del otro testículo. Niños: A virilización precoz con
desarrollo de caracteres sexuales secundarios: aumento del tamaño de los genitales externos, presencia de vello púbico, cambios de la
tonalidad de la voz. etc.; solo en el 10% de éstos se acompaña de ginecomastia. Ninguna hormona va a ser secretada en niveles
suficientes para que pueda ser útil como marcador tumoral.

CURSO ENARM CMN SIGLO XXI TEL: 36246001 Pharmed Solutions Institute PÁGINA 527
MANUAL DE TRABAJO DEL CURSO ENARM CMN SIGLO XXI
CASO CLINICO
Masculino, de 35 años, con antecedentes de agenesia testicular derecha, que asistió a consulta externa por un aumento del volumen de
la fosa ilíaca derecha (FID). En el examen físico se constató un tumor a este nivel, interpretándose como plastrón apendicular, por lo
que se le administró antibioticoterapia. Posteriormente se le realizó ultrasonido abdominal, visualizándose en el hipogastrio y la FID, en
correspondencia con el aumento de volumen, una imagen tumoral heterogénea vascularizada (al aplicarle Doppler color), de
13×8×13cm, que recordaba la ecoestructura de un testículo aumentado de tamaño. Se exploró, además, el testículo izquierdo, que
tenía ecoestructura y tamaño normales.

PREGUNTA
Cuál es el factor de riesgo más importante para el cáncer testicular.

RESPUESTA
a.- Paciente con Klinefelter.
b.- Infertilidad.
c.- Criptorquidia no reparada en la pubertad.
d.- Familiar con cáncer testicular.

HIPERTROFIA PROSTATICA BENIGNA (HBP). CIENCIAS BASICAS: En la clínica puede significar: 1) detección microscópica de la
hiperplasia, es decir la proliferación del estroma y el epitelio; b) crecimiento de la próstata detectado por el examen rectal digital o por
ultrasonido y c) un grupo de síntomas asociados con la hiperplasia prostática y definidos con el término “síntomas de tracto urinario
inferior” (STUI). Las diferentes definiciones surgen por que el tamaño de la próstata no siempre correlaciona con los síntomas (solo 30-
50% de los hombres con HBP detectada por tacto rectal o ultrasonido, presentan síntomas), así el termino HBP implica uno o más de los
hallazgos ya mencionados. La HBP es considerada en la actualidad una enfermedad progresiva con origen hormonal, en donde la
dihidrotestosterona (DHT), producto de la acción de la enzima 5-añfa-reductasa tipo 2 sobre la testosterona, es responsable. SALUD
PUBLICA: La prevalencia de HBP aumenta en forma lineal con la edad, en todos los grupos étnicos. En general afecta a los hombres
mayores de 45 años y la presentación de los síntomas suele darse a los 60-65 años de edad. Su prevalencia es de >50% y 90% a los 60 y
85 años respectivamente. La mortalidad y la frecuencia de complicaciones serias asociada con la HBP bajas, reportándose en 1-3%
incluyendo estas complicaciones la retención urinaria aguda. CLASIFICACION: Desde el punto de vista histológico se pueden distinguir
al menos los siguientes cinco tipos de hiperplasia prostática benigna: Estromal. Fibromuscular. Muscular. Fibroadenomatosa.
Fibromioadenomatosa. PATOGENIA: Los STUI asociados a HBP, tienen origen tanto anatómico como neuro-hormonal, ya que pueden
ser producto de la obstrucción mecánica al flujo urinario secundaria al crecimiento prostático y/o por el aumento de tono y presión del
musculo liso en la uretra, estroma y cuello de la vejiga, mediado por alfa-1-adrenoreceptores. En los hombres entre los 50 y 70 años, la
masa hiperplástica e hipertrófica de la próstata está formada en un 14% por nódulos y en un 86% es difusa, que ocurre principalmente
en la zona de transición. En las primeras fases de la hiperplasia predomina ampliamente el componente estromal de la zona de
transición, donde actuarían al menos tres factores con acción inductora mesenquimatosa embrionario-símil: Factor básico de
crecimiento fibroblástico (bFGF), factor de crecimiento transformador tipo B1 (TGF-B1), factor de crecimiento transformador tipo B2
(TGF-B2). Los tres factores, perfectamente identificados, actúan sinérgicamente llevando el estroma a un estado mesenquimático.
Además bFGF es mitogénico, lo que significa crecimiento glandular; éste es regulado por TGF-B2. Es de conocimiento muy antiguo que
para que haya hiperplasia prostática benigna es indispensable la presencia del testículo, lo que afirma la participación de los
andrógenos, ya sea en forma directa, permisiva o activadora, como es la acción del KGF (keratinocitic growth factor), que es el primer
factor probado como estimulante del crecimiento epitelial prostático. Además se necesita de tiempo, es decir, envejecimiento. El
fenómeno histológico de hiperplasia prostática benigna es propia del hombre que envejece. DIAGNOSTICO: Los síntomas de HBP
pueden ser secundarios a: 1. Obstrucción; disminución en el grosor y fuerza del chorro urinario, pujo al iniciar la micción, goteo
terminal, sensación de vaciado incompleto o retención aguda de orina. 2. Disminución de la capacidad de almacenamiento en la vejiga;
nicturia, urgencia e incontinencia urinaria de urgencia y por rebosamiento. No se ha encontrado una relación directa entre la severidad
de los síntomas y el tamaño de la próstata; algunos hombres presentan síntomas severos de obstrucción urinaria con un crecimiento
prostático mínimo, mientras que otros presentan síntomas mínimos pero con próstatas grandes. El International Prostate Symptom
Score (IPSS) permite evaluar los síntomas de manera objetiva y reproducible de acuerdo a su severidad en leves, moderados o graves.
Para realizar el diagnostico clínico de HBP, elabore la historia clínica y un examen físico que incluya: 1.Exploracion de abdomen para
detectar vejiga palpable. 2. Tacto rectal para evaluar las características de la próstata: tamaño, forma, simetría, textura y consistencia.
Utilice IPSS y evalué la afección de la calidad de vida del paciente con sospecha inicial de HBP, para mejorar la detección y catalogar los
STUI de acuerdo a severidad. En pacientes con STUI y sospecha de HBP, solicite un EGO para descartar la presencia de infección urinaria
y/o hematuria. Glicemia: con el objetivo de excluir DM. El ultrasonido vesical y prostático con medición de orina residual puede ayudar
a determinar el tamaño prostático y el volumen urinario residual, lo que ayuda a predecir los beneficios de un tratamiento médico o
guiar al tratamiento quirúrgico. También puede ser útil examinar riñones y vejiga para descartar litiasis en caso de infección de vías
urinarias repetidas y tumores en caso de hematuria persistente, así como la repercusión de la obstrucción prostática sobre el tracto
urinario superior. Hay una relación directa de los niveles de APE y el volumen de la próstata, sin embargo los pacientes con HBP no
tienen mayor riesgo de cáncer de próstata. Los valores altos de APE, se relacionan fuertemente con la probabilidad de tener cáncer de
próstata. Una próstata con un tamaño >30ml, un flujo urinario débil y APE >1.4ng/ml, son factores de riesgo para progresión de HBP
con un riesgo mayor de presentar retención aguda de orina (RAO) y necesidad de cirugía, por la tanto deben de recibir tratamiento. La
endoscopia permite evaluar muy bien el tamaño prostático, el cuello vesical, las repercusiones de la uropatía obstructiva sobre el
detrusor y orificios ureterales. Si bien se puede hacer en forma ambulatoria y bajo anestesia local, hay que ser cauto en su indicación,
pues en pacientes muy obstruidos y muy sintomáticos existe el riesgo de provocar retención completa por edema y dolor, además de
infección. En muchos pacientes se practica la endoscopia como una primera parte del procedimiento terapéutico endoscópico bajo
anestesia. TRATAMIENTO: La terapia de observación y vigilancia, es segura para la mayoría de los pacientes con síntomas leves a
moderados de HBP. Es también un tratamiento adecuado en los casos de pacientes con síntomas moderados a severos pero, que no

CURSO ENARM CMN SIGLO XXI TEL: 36246001 Pharmed Solutions Institute PÁGINA 528
MANUAL DE TRABAJO DEL CURSO ENARM CMN SIGLO XXI
han desarrollado complicaciones secundarias a la obstrucción del flujo urinario (insuficiencia renal, retención urinaria aguda o
infecciones recurrentes). Los pacientes con HBP, son síntomas leves a moderados o moderados a severos pero sin complicaciones y que
no presenten impacto en su calidad de vida, pueden ser manejados con vigilancia estrecha que incluya valoración anual con TR, APE y
US vesical y prostático con medición de orina residual. La cafeína y alcohol tienen un efecto diurético e irritante que aumenta la
frecuencia miccional, la urgencia y la nicturia. La reducción en la ingesta de líquidos reduce la frecuencia miccional en periodos
convenientes. Para mejorara los síntomas, disminuya la ingesta de líquidos libres por la tarde-noche y de irritantes como la cafeína y el
alcohol en cualquier presentación. Las ventajas del tratamiento farmacológico incluyen: conveniencia y el evitar la morbilidad potencial
asociada a la cirugía. La desventaja es que es de forma indefinida. Los alfabloqueadores tamsulosina, terazosina y doxazosina tienen
una eficacia similar sobre el alivio o mejora de los STUI (relajan el musculo liso en la próstata y cuello de la vejiga), son bien tolerados y
su eficacia se mantiene después de 6-12 meses de tratamiento. No reduce el tamaño de la próstata ni alteran la progresión de HBP.
Actualmente existen alfabloqueadores con mejor tolerancia por su uroselectividad sin disminuir su eficacia. Tal es el caso de alfuzosina
y silodosina, estos son más efectivos que los inhibidores de la 5-alfa reductasa, para mejorar los síntomas en el primer año de
tratamiento y su efecto se manifiesta desde el primer mes de inicio de tratamiento. Los alfabloqueadores presentan efectos adversos
como hipotensión ortostatica, cansancio, mareos, problemas eyaculatorios y congestión nasal, se recomiendan en todos los pacientes
candidatos a tratamiento médico. Las dosis recomendadas son: alfazulozina 10mgs diarios, tamsulosina 0.4mg diarios, terazosina 2-5mg
diarios, doxazosina 2-4 mgs diarios. Los I-5ARs inhiben la enzima alfa reductasa, lo que da una disminución de las concentraciones de
dihidrotestosterona a nivel prostático, que resulta en una reducción del tamaño de la próstata, una mejoría de los síntomas, menor
frecuencia de RAO y de intervenciones quirúrgicas, se recomiendan en pacientes con STUI y crecimiento prostático >40cc. Finasteride
5mg diarios. En la actualidad se considera a la RTUP, como el tratamiento más efectivo para la HBP. Los pacientes que son sometidos a
tratamientos quirúrgicos transuretrales mínimamente invasivos (TUMT, HIFU, HoLEP, PVP), tienen una mejoría menor en el puntaje del
IPSS y de la tasa de flujo urinario máximo, que los pacientes a los que se les realiza RTUP. La RTUP no reseca todo el tejido prostático
susceptible a cáncer, por lo tanto deben evaluarse anualmente con TR y APE. La RTUP tiene los siguientes riesgos: mortalidad >0.25%,
intoxicación hídrica 2%, estenosis de uretra y contractura de cuello vesical 3.8%, eyaculación retrograda 65-70%, incontinencia urinaria
de esfuerzo, disfunción eréctil. La prostactectomia abierta está indicada en pacientes con próstatas de >80cc y en aquellos con
complicaciones asociadas, como litiasis vesical o que ameriten resección diverticular.

CASO CLINICO
Se trata de paciente masculino de 62 años de edad el cual acude a consulta de control ya que padece de diabetes mellitus desde hace
15 años, actualmente con depuración de creatinina menor a 60 ml/ min. refiere que desde hace 7 días ha presentado ardor al orinar así
como dificultad, además de tardar en terminar de miccionar, al interrogatorio refiere que desde hace 6 meses presenta disminución del
chorro de la orina así como goteo, niega hematuria usted considera que actualmente cursa con IVU la cual controla y cita para valorar la
próstata, 15 dias porterior al tratamiento por IVU evalua nuevamente al paciente observando al tacto rectal crecimiento no indurado,
levemente doloroso, solicita antígeno prostático reportado en 5.0 ng/ml. Urocultivo negativo, EGO negativo.

PREGUNTA
Cuál es la conducta a seguir para establecer el diagnostico.

RESPUESTA
a.- USG renal.
b.- USG vesical.
c.- USG prostatico.
d.- Enviar a segundo nivel.

CANCER DE PROSTATA. CIENCIAS BASICAS: Es el crecimiento anormal de las células de la glándula prostática, que además tienen la
capacidad, de invadir otros órganos. Es un tumor que nace del epitelio acinar y/o ductal de la próstata que puede variar
considerablemente en su diferenciación glandular, anaplasia y comportamiento. Es una de las neoplasias malignas más frecuente en los
hombres y se incrementa conforme aumenta la edad. En la próstata fundamentalmente se distinguen 3 zonas: la zona Central cruzada
por los conductos eyaculadores que supone un 25% de la glándula, la zona Transicional que rodea a la uretra posterior con un 5% del
volumen glandular y la zona Periférica que ocupa un 70% del volumen glandular. Más de 95% de todos los canceres de próstata son
adenocarcinomas y desde el punto de vista anatomopatologico, el 97% de los tumores se originan en la periferia. La histología del
cáncer de próstata es muy importante, se ha visto que un 95% son adenocarcinomas acinares originados en la porción glandular de la
próstata y solo un 4% es transicional, escamoso o endometroide, estos derivan de los conductos y por lo tanto son
hormonodependientes. El 1% son sarcomas. SALUD PUBLICA: El 87.6% de las defunciones se registraron en la población mayor de 65
años. El cáncer de próstata se ubicó en segundo lugar en el grupo de edad postproductiva, con una tasa de 71.7 por 100 mil habitantes.
Existe diferencia notable en la incidencia del cáncer entre el mundo oriental y occidental, atribuibles entre otros factores a la ingesta
de grasas en la dieta. Un ejemplo lo constituyen los chinos, que de una incidencia de 0.8 por 100 000 hab en su vida nativa, pasan a
tener tasas de 18.6 cuando se integran a la civilización occidental. La raza negra en EE.UU tiene la mayor tasa de incidencia con 100.2
por 100 000 hab. PATOGENIA: En la zona Periférica se desarrollan el 68% de los cánceres, en la zona Central un 8% y en la zona
Transicional un 24%. La zona Periférica es accesible al tacto rectal mientras que los tumores que se desarrollan en las zonas Central y
Transicional se valoran mejor con la ecografía. Se han postulado factores genéticos, las razas negras y los escandinavos tienen una alta
incidencia, mientras que los japoneses tienen la más baja. También existiría una predisposición familiar, pariente en primer grado de
caso índice tiene un riesgo 10% mayor de tener un cáncer prostático. Otros factores implicados son la alimentación y el ambiente. Se ha
visto que el aceite de soya es un protector ya que se transforma en un estrógeno débil; que los habitantes de zonas rurales tienen una
mayor incidencia atribuida a los insecticidas agrícolas; las prostatitis vírales por citomegalovirus o virus herpes a repetición también
aumentarían las posibilidades de desarrollar un cáncer de próstata. Un factor clave es el factor hormonal, se relaciona claramente con
la presencia de testículos funcionales; los eunucos no tienen cáncer de próstata. La presencia de receptores esteroides en las células

CURSO ENARM CMN SIGLO XXI TEL: 36246001 Pharmed Solutions Institute PÁGINA 529
MANUAL DE TRABAJO DEL CURSO ENARM CMN SIGLO XXI
tumorales y la respuesta positiva a la supresión de los andrógenos así como los altos niveles de Dihidrotestosterona en las células
cancerosas, apoyan la hipótesis hormonal. DIAGNOSTICO: En general los cánceres de próstata son silenciosos, crecen lentamente,
invaden primero la cápsula prostática, luego ganglios pelvianos (obturatrices e iliacos) y dan metástasis preferentemente al hueso. Sólo
dan sintomatología obstructiva. Las lesiones en el hueso son osteoblásticas (se ve más blanco y algodonoso el hueso en la radiografía) y
los sitios más frecuentes de metástasis son: Columna sacra, crestas ilíacas, columna lumbar, columna dorsal, cráneo, etc. En hombres
con mayor riesgo, con antecedentes familiares directos de cáncer de próstata o de mama, se debe iniciar un abordaje de detección a
los 40 años. El diagnóstico se establece a través de estudios clínicos y de gabinete: tacto rectal, ultrasonografía, TAC, RMN, y
marcadores séricos del tumor; entre estos, sin duda el más importante es el antígeno prostático específico (APE) el cual se produce
únicamente en el citoplasma de células prostáticas benignas y malignas. Su nivel sérico se correlaciona con el volumen de ambos
tejidos, maligno y benigno. El APE, se determina como normal <4ng/ml, intermedio 4.1-10ng/ml y altamente sospechoso arriba de
10ng/ml. Estos dos últimos requieren de biopsia, ya que las dos terceras partes de cáncer prostático se ubican en personas con
resultados de 10 y más ng/ml. El APE ha demostrado ser útil para supervisar la eficacia del tratamiento y para controlar la recaída
después de que el tratamiento ha terminado. Los niveles de la fosfatasa acida prostática (FAP) en la sangre pueden elevarse en
pacientes que tienen ciertas enfermedades benignas de la próstata o cuyo cáncer está en la fase temprana. En hombres entre 50 y 55
años se da una forma poco frecuente de cáncer prostático con Gleason 8 a 10, de crecimiento rápido y aneploides, en general se
desarrollan en dos o tres meses. Es importante que en el informe que se recibe del patólogo se establezca el grado de Gleason, que es
una clasificación basada en la diferenciación celular y la relación estroma-glándula del cáncer (no mideanaplasia), para esto se le asigna
un puntaje de 1 a 5 a la zona más "mala" del cáncer y otro a la más "buena" y se suman. La escala de Gleason va del 2 al 10, siendo el 2
el cáncer más benigno. En general los cánceres con Gleason del 2 al 4 tienen buen pronóstico, del 5 al 7 son de pronóstico intermedio y
del 8 al 10, son los de peor pronóstico. El Gleason es importante porque determina el pronóstico y evolución. La biopsia de próstata se
considera como "el estándar de oro" pero generalmente se realiza luego de constatar anormalidades en el tacto rectal o elevación del
APE, por lo que la sensibilidad del método está sobrestimada. La Sociedad Americana de Cáncer recomienda a todos los varones
mayores de 50 años se sometan a tacto rectal (TR) y detección de APE. En los casoso de antecedentes familiares, el control se deberá
iniciar a los 40 años. Si el TR y el APE son normales se recomienda control anual. Si el TR es normal y el APE está entre 4.1 y 10 ng/ml se
realizará Ecografía Transrectal (ETR) para determinar la densidad del APE y detectar nódulos con el fin de realizar biopsia prostática
dirigida o aleatoria. Si el TR es normal y el APE mayor de 10 se realizará biopsia dirigida o aleatoria mediante ETR. Si el TR es
sospechoso se realizará biopsia de dicha zona, independientemente del APE. TRATAMIENTO: El tratamiento depende del TNM y de la
edad del paciente. Las personas con cáncer T1-T2 tempranos, localizados, que no han invadido la cápsula, deberán ser sujetos a
prostatectomía radical, resecando la próstata y cuello vesical, ganglios obturatrices, ganglios ilíacos y vesículas seminales. La sobre vida
de estos pacientes, en general es buena; es decir el 90% sobrevive a los 15 años, con tratamiento precoz. Las complicaciones de la
prostatectomía radical son la impotencia que se produce en casi el 100% de los tratados y la incontinencia de orina se ve en un 30%
aproximadamente. Si se diagnostica cáncer TNM IV, con metastasisi a distancia solo se realiza tratamiento paliativo con supresión del
estimulo hormonal, de estos pacientes con o sin tratamiento 10% se muere antes de año y otro 10% vive mas de 10 años. El promedio
de sobrevida es dos a tres años. No es un cáncer tan maligno responde a la supresión de testosterona. Para obtener un bloqueo
hormonal completo se realiza: Orquiectomía. Fármacos bloqueadores androgénicos. Para el bloqueo androgénico se pueden utilizar
análogos de LHRH como el Leuprolide y Decapeptil que tienen el problema de su alto costo. También se puede utilizar flutamida. Se ha
ensayado la administración de estrógenos como el dietilestilbestriol en dosis de 2.5 mg al día, es menos costoso, actúa bloqueando los
receptores periféricos de testosterona. En general con el tratamiento paliativo se obtiene una mejoría de la sintomatología general y
también de la uropatía obstructiva. No se realiza quimioterapia. Radioterapia solo local con implantes en el cáncer TNM I o II. También
se utiliza como tratamiento local de las metástasis de columna, pero sólo es paliativo. Tratamiento del tumor incidental: En los
pacientes con expectativa de vida menor de 10 años y con un tumor focal A1, se les aplicará conducta conservadora efectuando
controles de APE, TR y EcoTR cada 6 meses. En los pacientes con expectativa mayor de 10 años o tumores focales A2 se realizará cirugía
radical o radioterapia local. Tratamiento del tumor localizado, el tratamiento de elección es la prostatectomia radical, siempre que el
estado general sea bueno y la esperanza de vida sea superior a 10 años. En los casos con mal estado general, edad avanzada o negativa
al tratamiento quirúrgico puede ser útil la radioterapia radical o bien el bloqueo anbdrogenico. Tratamiento del tumor con extensión
extraprostática: Los pacientes con afectación capsular no son buenos candidatos para la cirugía radical pues más del 50% van a tener
afectación ganglionar y la exéresis puede ser incompleta. El tratamiento con bloqueo hormonal completo logra disminuir el volumen
tumoral y facilita la cirugía, pero pocos pacientes logran disminuir el estadio tumoral y la supervivencia no se modifica. El tratamniento
adecuado dependerá de la presencia de síntomas obstructivos para instaurara el bloqueo androgénico completo o la desobstrucción
mediante RTU. Tratamiento del tumor diseminado: El tratamiento es mediante la supresión hormonal, bien sea por castración
quirúrgica o bien con análogos de la LHRH. Si se quiere actuar sobre los andrógenos suprarrenales se asociará un antiandrógeno al
tratamiento, efectuándose un bloqueo completo. Prostatectomia radical: En esta cirugía se extirpan completamente la próstata,
vesículas seminales y se restituye la vía urinaria mediante anastomosis vesicouretral. Se asocia a linfoadenopatía de ganglios iliacos
internos (principal indicación: cáncer prostático localizado T2a y T2b), complicaciones: mortalidad (0,2%), incontinencia urinaria (2 –
7%), disfunción eréctil (100%), estenosis uretrovesical (1-3%), linfocele (1-3%). La disfunción eréctil que puede ocurrir después de esta
operación puede ser evitada al respetar los haces neurovasculares localizados en hojas de la fasia pélvica lateral y que están muy
vecinos a la glándula prostática. Se considera que no existe enfermedad residual después de la cirugía cuando el APE desciende al cabo
de algunas semanas a cifras indetectables (<0,02 ng/ml). Por el contrario, el APE que se mantiene detectable o el APE en sostenido
ascenso son indicadores de recidiva tumoral o recidiva bioquímica. Radioterapia: Usada como terapia definitiva en alrededor de 1/3 de
los pacientes. Se emplea radioterapia externa y la radioterapia interticial (braquiterapia). Es eficaz con resultados similares a la cirugía
al tratar tumores pequeños (intra prostáticos). Complicaciones: disfunción eréctil (30-45%), toxicidad gastrointestinal (3-5%),
incontinencia urinaria (menos del 5%). Terapia hormonal: Pilar del tratamiento de cáncer prostático metastásico (etapa IV). Los
andrógenos (testosterona) son fundamentales para el crecimiento y funcionamiento de las células prostáticas. Son producidos
mayoritariamente en los testículos (95%) y en la glándula suprarrenal (5%). La forma más simple de emplear la hormoneoterapia es la
castración quirúrgica. Hormonoterapia de 1° línea; orquiectomía, agonistas LHRH (zoladex, lupron), antagonistas LHRH (abarelix),
antiandrógenos (casodex) (bicalutamida), antiandrógenos/inhibidores de 5alfa-reductasa tipo II (flutamida). Hormonoterapia de 2°

CURSO ENARM CMN SIGLO XXI TEL: 36246001 Pharmed Solutions Institute PÁGINA 530
MANUAL DE TRABAJO DEL CURSO ENARM CMN SIGLO XXI
línea: bloquear síntesis adrenal de andrógenos (hodrocortisona, ketoconazol, aminoglutemida), basados en el hecho de que las
glándulas adrenales continúan produciendo andrógenos después de una castración médica o quirúrgica, estrategia conocida como
bloqueo androgénico máximo. Quimioterapia: No hay pruebas que la quimioterapia prolongue la sobrevida, sin embargo, la
administración de prednisona puede aliviar los síntomas en alrededor de 1/3 de los casos. PRONÓSTICO: Cáncer prostático localizado,
muestra supervivencia de 65 – 75 % a los 10 años. Cáncer prostático diseminado; supervivencia del 55% a los 10 años.- Cáncer
prostático con metástasis óseas; supervivencia a los 10 años: 15%. Medio de sobrevida = 30 meses (aunque un 10% vive más de 10
años)

CASO CLINICO
Se trata de masculino de 81 años de edad el cual se encuentra en cuidados paliativos por cáncer de próstata con metástasis, el cual
recibe tratamiento con goserelina, es ingresado a urgencias debido a que presenta en casa estado confusional con tendencia a letargia,
a la exploración física se observa con estado avanzado de emaciación, mucosas orales secas, deshidratación moderada, pobre
respuesta a estímulos dolorosos pero presente, el examen mental es difícil realizar por embotamiento, las constantes vitales son TA
110/50 mmHg, FC de 112 lpm, se observa afebril, debido a su estado actual y antecedentes se considera que el paciente presenta
hipercalcemia, se realizan medidas de estabilización en espera de laboratorios.

PREGUNTA
Cual es la conducta menos apropiada a seguir considerando el estado de hipercalcemia.

RESPUESTA
a.- Solucion salina.
b.- Pamidronato.
c.- Furosemida.
d.- Dexametazona.

CASO CLINICO
Se trata de masculino de 81 años de edad el cual se encuentra en cuidados paliativos por cáncer de próstata con metástasis, el cual
recibe tratamiento con goserelina, es ingresado a urgencias debido a que presenta en casa estado confusional con tendencia a letargia,
a la exploración física se observa con estado avanzado de emaciación, mucosas orales secas, deshidratación moderada, pobre
respuesta a estímulos dolorosos pero presente, el examen mental es difícil realizar por embotamiento, las constantes vitales son TA
110/50 mmHg, FC de 112 lpm, se observa afebril, debido a su estado actual y antecedentes se considera que el paciente presenta
hipercalcemia, se realizan medidas de estabilización en espera de laboratorios.

PREGUNTA
Cual es la conducta menos apropiada a seguir considerando el estado de hipercalcemia.

RESPUESTA
a.- Solucion salina.
b.- Pamidronato.
c.- Furosemida.
d.- Dexametazona.

CURSO ENARM CMN SIGLO XXI TEL: 36246001 Pharmed Solutions Institute PÁGINA 531
MANUAL DE TRABAJO DEL CURSO ENARM CMN SIGLO XXI
TRAUMA CRANEOENCEFALICO (TCE):
CIENCIAS BASICAS: En adultos, es definido como un intercambio brusco de energía mecánica que genera deterioro físico y/o funcional
del contenido craneal. Se consigna como alteración del contenido encefálico el compromiso de conciencia, la amnesia postraumática
y/o un síndrome vertiginoso o mareos persistentes. También debe considerarse como un signo de disfunción del contenido craneal la
aparición de una cefalea holocránea persistente y progresiva que puede o no acompañarse de vómitos. Se distingue de la Contusión de
cráneo, que corresponde a un impacto mecánico sobre la bóveda craneana que no produce alteración del contenido craneano, y que
puede asociarse a dolor local. SIGNOS DE ALARMA, en la evaluación prehospitalaria o la llegada a urgencias: Deterioro progresivo de la
conciencia (disminución de Glasgow), signos de focalidad neurológica, cefalea progresiva, vómitos explosivos recurrentes, agitación
psicomotora, convulsiones, amnesia anterógrada de más de 30 min, cambio en el tamaño de las pupilas, sospecha de herida craneal
penetrante, intoxicación, evidencia clínica o radiológica de fractura de cráneo, sat. O2 <80%, hipotensión. SALUD PUBLICA: Constituye
una de las principales causas de mortalidad e incapacidad en la población menor de 40 años. Cada año fallece 1,000 000 de personas
víctimas de trauma craneal severo. Los accidentes con vehículo automotor son la principal causa de traumatismos. El trauma causa
150,000 muertes en EU y un tercio se acompaña por TCE severo. Entre 2-10% de pacientes con TCE tienen asociada lesión cervical.
PATOGENIA: Se distinguen 2 mecanismos básicos: la colisión o traumatismo directo, en el cual actúan como formas lesivas la energía
cinética y la deformante. Y el traumatismo indirecto en el que no existe contacto previo, cabeza/objeto agresor, siendo las fuerzas
lesivas la aceleración angular pura y la hiperpresión transmitida pura. El impacto mecánico origina la degeneración neuronal mediante
3 mecanismos básicos: 1. Mecanismo lesional primario, son lesiones nerviosas y vasculares producidas inmediatamente por la agresión
biomecánica, las lesiones resultantes de la agresión primaria son: fracturas craneales, contusiones, laceraciones, hematomas
intracerebrales, lesión axonal difusa. 2. Mecanismo lesional secundaria; se destacan el edema cerebral (con el trauma se activa el
edema vasogénico, que ocurre por aumento de la permeabilidad de los capilares, con ruptura local de la barrera hematoencefálica
(BHE), aumento del líquido intersticial, y el edema citotóxico, que ocurre por daño isquémico de la célula nerviosa), isquemia local,
GRAVEDAD DE TCE glucólisis anaerobia, edema de los astrositos, activación de CLASIFICACION DE TCE SEGÚN OMS
Leve 14-15 puntos cascadas enzimáticas y de las endotelinas, por formación y Fracturas de cráneo:
Moderado 9-13 puntos liberación de radicales libres de oxígeno. En los traumatismos Fracturas de la bóveda
Severo Menos de 9 puntos Fracturas de la base
cerrados, el impacto de las fuerzas se produce en las zonas donde
Fracturas de los huesos de la cara
el cráneo se pone en contacto directamente con el encéfalo, pudiendo producir lesiones en la zona Otras y las fracturas inclasificables
de impacto (lesión por golpe) y en la zona diametralmente opuesta (lesión por Múltiples fracturas que afectan el
contragolpe).,también lesiones por cizallamiento, las lesiones resultantes son; hipotensión, cráneo o la cara con otros huesos
hipercapnia, hipoxemia, hipertermia, hipoglucemia, acidosis, hiponatremia, hipertensión Lesión intracraneal
intracraneal, hematoma cerebral tardío, edema cerebral, convulsiones, vasoespasmo. 3. Mecanismo Conmoción
Laceración cerebral y contusión
lesional terciario; engloban una serie de procesos neuroquímicos y fisiopatológicos complejos, Hemorragia subaracnoidea,
concatenados, con posibilidad de retroalimentación positiva, entre sí que inician inmediatamente subdural y extradural
tras el TCE. CLASIFICACION: En la escala de coma de Glasgow se puede obtener una calificación Hemorragias intracraneales
máxima de 15 y un mínimo de 3. Un paciente que presenta cualquiera de los siguientes signos debe postraumáticas inespecíficas
Lesión intracraneal de naturaleza
considerarse que sufre un TCE severo: Anisocoria, déficit motor localizado, fractura abierta del inespecífica
cráneo con exposición de masa encefálica o salida de LCR, deterioro neurológico, fractura deprimida
de la bóveda del cráneo. Otra clasificación es en: BAJO RIESGO; Asintomática, cefaleas, mareos, hematoma, laceración o scalp de cuero
cabelludo, ausencia de criterios de moderado o alto riesgo. Actitud a seguir; Si no presentan otras lesiones asociadas que requieran
ingreso hospitalario u observación se envían a su domicilio siempre que una segunda persona pueda observar la evolución del paciente
en las próximas horas. Debe informarse por escrito sobre signos o síntomas de alarma, ante la presencia de las cuales debe consultar de
nuevo al hospital. MODERADO RIESGO; Historia de perdida transitoria de la conciencia, intoxicación por alcohol y drogas, cefalea
progresiva, vómitos persistentes, amnesia peritraumatica, politraumatismo que impide adecuada valoración clínica del TCE,
traumatismo facial severo, sospecha de niño maltratado, edad menor de 2 años excepto lesión trivial. Actitud a seguir; deben
permanecer en observación al menos 24 hrs, pueden reducirse a 12 hrs la observación si no hay sintomatología neurológica, TAC de
cráneo normal, no existen otras lesiones asociadas que requieran ingreso hospitalario. Si existe sintomatología neurológica evidente, el
periodo de observación debe prolongarse y valorar TAC craneal de control a las 12-24 hrs. ALTO RIESGO; Disminución del nivel de
conciencia actual o progresivo no claramente debido a otras causas (metabólico, epilepsia), signos neurológicos de focalidad,
hundimiento o herida penetrante de cráneo, sospecha de fractura de la base del cráneo (otorrea, hemotímpano, rinorrea, hematoma
en anteojos, hematoma retroauricular), convulsiones postraumáticas, respiración irregular o apnéica. Actitud a seguir Una vez
diagnosticados y estabilizados, deben pasar al área de tratamiento definitivo (quirófano o UCI). Desde el ´punto de vista patológico,
pueden existir tres tipos fundamentales de lesiones cerebrales: 1. Conmoción o concusión cerebral; caracterizada clínicamente por una
breve perdida de conciencia, con un corto periodo de amnesia seguida de una recuperación rápida y total, sin ningún signo neurológico
focal. No hay lesión estructural macroscópica del cerebro, tan solo se producen lesiones por estiramiento de los tractos axonales de la
sustancia blanca, con perdida reversible de su función, responsables de la pérdida de conciencia transitoria. 2. Contusión cerebral;
sobre todo en polos frontales y temporales, por el contacto entre la superficie cerebral y el interior del cráneo, que abarcan desde una
simple magulladura en una pequeña área cortical, hasta lesiones extensas, a menudo hemorrágicas, de gran parte de la superficie
cerebral, con daño en la sustancia blanca y el mesencéfalo, clínicamente alteración del nivel de conciencia, desde confusión, inquietud
y delirio y grados variables de coma, estos pacientes deben ser hospitalizados para observación, dado el desarrollo tardío de edema
cerebral. Realizar TAC, valora necesidad de iniciar tratamiento para HIC. Las contusiones que producen efecto de masa requieren
cirugía urgente. 3. Lesión cerebral difusa; presencia de un coma prolongado de días o semanas, lesión frecuente con alta mortalidad. Su
diagnóstico es presumible cuando en la TAC cerebral no se aprecia una lesión ocupante de espacio en un paciente con coma profundo,
además de la situación de coma suelen presentar posturas de descerebración o decorticación, y frecuentemente presentan signos de
disfunción autonómica. DIAGNOSTICO: Los complementarios más frecuentemente utilizados para decidir el diagnóstico y tratamiento
de este tipo de traumatismo son los siguientes: Radiografía de cráneo: Se deben indicar tres vistas fundamentales: anteroposterior,
lateral y Towne. Las dos primeras nos permiten apreciar lesiones en la bóveda craneal (fracturas, presencia de cuerpos extraños
intracraneales, desplazamientos de la pineal calcificada, diástasis de suturas). La vista de Towne nos permite visualizar la fosa craneal

CURSO ENARM CMN SIGLO XXI TEL: 36246001 Pharmed Solutions Institute PÁGINA 532
MANUAL DE TRABAJO DEL CURSO ENARM CMN SIGLO XXI
posterior, y parcialmente la proyección posterior de los peñascos del hueso temporal.Angiografía carotídea: Es un estudio invasivo, que
requiere de cierta demora en su ejecución, actualmente ha sido desplazada por la tomografía axial computarizada (TAC), aunque en los
centros que no cuentan con este último recurso, mantiene toda su vigencia e importancia. Permite apreciar desplazamientos de los
patrones vasculares normales debido a lesiones intracraneales con efecto de masa (hematomas, higromas, contusiones). También
permite visualizar zonas con vasoespasmo, fístulas vasculares y lesiones vasculares posteriores al trauma.Tomografía axial
computarizada: Constituye en la actualidad el estudio ecográfico más eficaz frente a este grupo de trastornos. Permite la visualización
directa de lesiones intracraneales, su volumen y forma, así como las estructuras que afecta, y la visualización indirecta de otras
lesiones, como el edema cerebral. A continuación, se expone la clasificación de las lesiones encefálicas detectadas por la TAC craneal en
los pacientes con TCE descrita por Marshall, también denominada como clasificación del Traumatic Coma Data Bank (TCDB): Distingue 4
tipos de lesiones difusas y 2 tipos de lesiones focales: Clasificación de los hallazgos de la primera tomografía tras el TCE: A) Lesión difusa
I; sin patología visible. B) Lesión difusa II; cisternas visibles, con desviación de la línea media hasta 5mm y/o sin lesión mayor de 25 ml.
C) Lesión difusa III; cisternas comprimidas o ausentes, con desviación de la línea media hasta 5 mm y/o sin lesión mayor de 25ml. D)
Lesión difusa IV; desviación de la línea media mayor a 5 mm, sin lesión mayor de 25ml. E) Toda lesión evacuada quirúrgicamente. F)
Lesión mayor de 25ml no evacuada quirúrgicamente. TRATAMIENTO: Inicial de las situaciones de amenaza vital: 1. Asegurar la
permeabilidad de la vía aérea con control de la columna cervical. 2. Oxigenación y ventilación adecuada. 3. Control de la hemorragia
externa y mantener la presión arterial. 4. Evaluación del estado neurológico. 5 Investigar otras lesiones traumáticas. Objetivo del
tratamiento en TCE: Posición de la cabeza a 30° sobre el plano horizontal, analgesia eficaz, normotermia, PaO2 >70mmHg,
normocapnia, presión arterial media >90mmHg, euvolemia, Hb >10mg/dl, osmolaridad plasmática >290mOsm, glucemia <200mg/dl,
profilaxis de convulsiones precoces. Para la conmoción cerebral y síndrome postraumático que pueden ser manejados por la guardia de
Cirugía: vigilar cada 4 horas posibles signos de alarma neurológica (alteraciones de conciencia, pupilas y motilidad de extremidades), los
cuales deben ser tratados sintomáticamente (analgésicos y antigravitatorios según necesidad) durante períodos prudenciales de tiempo
en la sala de observaciones, y de ser preciso o requerirse, serán evaluados por la guardia de Neurocirugía a solicitud de la guardia de
Cirugía General. Si el paciente presenta agitación severa, debe ser sedado, de preferencia con Midazolam 0,10- 0,15 mg endovenoso en
bolo inicial, seguido por infusión endovenosa de 0,2- 0,4 mg x Kg/h. Puede utilizarse también morfina 0,10 mg endovenoso de inicio,
seguido por infusión de 1- 2 mg/h. Farmacología de TCE; MANITOL, su efecto rápido sobre la PIC es especialmente útil en situaciones de
urgencia, su acción es rápida y fugaz, obteniéndose el efecto máximo a los 40 minutos de infusión en bolo, en urgencias, redujo con
mayor frecuencia la dilatación pupilar y mejoro la evolución a los 6 meses, tienden a mostrar mayor sobrevida y mejor pronóstico.
Indicaciones: a todo paciente hemodinamicamente estable con signos de herniación cerebral (anisocoria, signos de decorticación-
descerebración) se le debe administrar manitol en bolo rápido, realizando a continuación una TAC craneal urgente. Dosis se comienza
con un bolo de 1-2 g/kg, se puede repetir cada 6 hrs. FUROSEMIDA, su ventaja sobre el manitol es su efecto sobre la osmolaridad es
menos marcado, aunque puede producir trastornos electrolíticos, la desventaja es que es menos efectivo en el tratamiento del edema
cerebral. Indicaciones: lesiones hemorrágicas cerebrales, debería reservarse para pacientes en los que la expansión de volumen
producida por el manitol pudiera resultar perjudicial (cardiopatías, insuficiencia renal). CORTICOIDES, son eficaces para disminuir el
edema cerebral en los procesos tumorales o inflamatorios pero en el edema, debido al traumatismo, no parecen ser útiles. La
administración de pre-hospitalaria de sol.de cloruro de Na al 7.5% para trauma e hipotensión se asocia con un incremento
significativamente mayor en la presión arterial comparado con la infusión de Ringer-lactato. Profilaxis antibiótica, siempre en los
traumatismos abiertos y penetrantes, cuando existan signos clínicos cardiológicos de fractura de la base de cráneo, se emplearan
antibióticos de amplio espectro. Profilaxis de crisis convulsivas; se establecerá precozmente un tratamiento de fondo preventivo de las
mismas siempre que la lesión sea supratentorial con afectación del parénquima cerebral (hundimientos, contusiones, dislaceraciones,
hematomas intraparenquimatosos). Los anticomiciales usados son; fenobarbital (100mg c/8hrs) tanto enteral como parenteral y
fenitoina (100mg c/8 hrs). TCE grave; el exceso de tono simpático en la dinámica vascular cerebral y sus efectos sobre la PÍC debe ser
controlado con una adecuada sedación y analgesia; opiáceos 8fentanilo, morfina), benzodiacepinas, propofol. La intubación debe ir
siempre acompañada de una adecuada sedación, una correcta relajación muscular ya que si esta no se consigue durante la maniobra de
la PIC esta aumentaría, el relajante muscular ideal para estos pacientes es la succinilcolina. COMPLICACIONES: Hematoma extradural o
epidural: colección de sangre entre el cráneo y la duramadre cuya causa más frecuente es la lesión traumática de la arteria meníngea,
sospechar en paciente que ha sufrido TCE, que tras un periodo de 1-24 hrs entra en estado de coma, pudiendo haber dilatación pupilar
del lado lesionado y hemiparesia contralateral. Este cuadro requiere cirugía inmediata. Hematoma subdural: Debidos a roturas de las
venas comunicantes entre corteza cerebral y duramadre, incidencia mayor en pacientes etílicos y ancianos. Si aparecen en las primeras
24 hrs son agudos, entre 24hrs y 2 semanas subagudos y crónicos cuando aparecen más tardíamente. El hematoma subdural agudo
requiere cirugía urgente. Hemorragia subaracnoidea: se acompaña a menudo de hematoma subdural concomitante o de una contusión
cerebral, el diagnostico se realiza mediante TAC y si es normal el diagnostico se realizara mediante la demostración de un LCR
hemorrágico, no requiere tx., quirúrgico urgente. Hematoma intraparenquimatoso: pueden manifestarse como lesiones rápidamente
expansivas o ser asintomáticas, en la mayoría de los casos existe fractura craneal asociada por golpe o contragolpe. FRACTURAS
CRANEALES: Las fracturas de la bóveda craneal no presentan un cuadro clínico propio, sino que dependen de la afectación o no de
estructuras neurológicas. Son diagnosticadas con relativa facilidad mediante la ayuda de los rayos X (en cambio, las fracturas de la base
del cráneo generalmente no son visibles por estos); poseen un cuadro clínico propio de cada una de sus variedades, lo que favorece la
realización del diagnóstico clínicamente: Fosa craneal anterior: Hematoma periorbitario uni o bilateral: en “gafas” o "en espejuelos" o
signo del mapache (si es bilateral, afectando las dos hemifosas anteriores), o del oso Panda (si es unilateral, que afecta sólo una
hemifosa anterior). Rinorragia o rinorraquia (uni o bilateral, en dependencia de si afecta una hemifosa anterior, o a las dos). Es
importante establecer diagnóstico diferencial, al exponer el goteo en una sábana blanca o un papel, suele separarse el componente
hemático del líquido cefalorraquídeo, signo del “salvavidas” o del “huevo frito”. Puede existir anosmia (por lesión del primer par
craneal, generalmente a nivel de la lámina cribosa del etmoides, menos frecuente por afectación del bulbo o del nervio olfatorio),
alteración de la motilidad de la musculatura periocular (pares craneales III-IV-VI), defecto visual por lesión del nervio óptico (II par
craneal), anisocoria periférica, o desigualdad pupilar con midriasis unilateral, “seudo-anisocoria central” por lesión de ramos del III par
o lesión del esfínter constrictor del iris, o por la presencia de hematoma intraorbitario “extracraneal”, que comprima o irrite al ganglio
ciliar en el polo posterior del ojo). En resumen, los signos y síntomas suelen estar más frecuentemente ocasionados por lesión de los

CURSO ENARM CMN SIGLO XXI TEL: 36246001 Pharmed Solutions Institute PÁGINA 533
MANUAL DE TRABAJO DEL CURSO ENARM CMN SIGLO XXI
pares craneales I-II-III-IV-VI. Fosa craneal media: Sensación de "oído ocupado" o "líquido en el oído", hipoacusia, hemotímpano durante
la otoscopia, otorragia u otorraquia, signo de Batlle (hematoma retroauricular, o pre-esternocleidomastoideo generalmente visible
después de transcurridas las primeras 24 h), vértigos intensos por lesión de los conductos semicirculares, paresia o parálisis facial
periférica por lesión del nervio facial en su curso a través del peñasco (más frecuente en las fracturas transversas). En resumen, los
signos y síntomas suelen estar más frecuentemente ocasionados por lesión de los pares craneales VII-VIII. Las fracturas de fosa
posterior no tienen un cuadro clínico bien definido, y pueden ser visualizadas mediante rayos X (preferiblemente en la vista de Towne).

CASO CLINICO

CURSO ENARM CMN SIGLO XXI TEL: 36246001 Pharmed Solutions Institute PÁGINA 534
MANUAL DE TRABAJO DEL CURSO ENARM CMN SIGLO XXI
TRAUMA RAQUIMEDULAR
CIENCIAS BASICAS: Toda lesión traumática que afecta, conjunta o aisladamente, las diferentes estructuras de la columna vertebral en
cualquier de sus niveles. Es toda lesión de la columna vertebral asociada a una lesión neurológica que puede comprometer a la medula
espinal, raíces o cauda equina. Esta lesión neurológica puede ser reversible o irreversible. Factores predisponentes: Espondilosis
cervical, inestabilidad atloaxoidea, malformaciones vertebrales congénitas, osteoporosis, artropatías inflamatorias. Recuerdo
anatómico: 33 vértebras: 7 cervicales, 12 torácicas, 5 lumbares, 5 sacras, 4 coccígeas, 23 discos intervertebrales. Trauma
vertebromedular: Transmisión abrupta de energía sobre la columna vertebral y su contenido. Lesión medular: Alteración temporal o
permanente de la función motora, sensitiva o autonómica. SALUD PÚBLICA: La enfermedad traumática es la 4ª causa de muerte. 20-30
casos/100.000 habitantes/año. Mortalidad global 4%. Los traumatismos ocasionan severas incapacidades y secuelas invalidantes
permanentes. Más 1/3 accidentes de tráfico. Más de 1/3 politrauma: 5-15% daño neurológico, 30% otras lesiones, 5% fracturas ocultas.
Distribución anatómica: 55% cervical, 10% torácica, 15-20% toracolumbar, 15-20% lumbosacra. 5% de los pacientes con TCE, tienen
TRM. 25% de TRM tienen TCE. PATOGENIA: La zonas más vulnerables son la columna cervical baja y la columna toracolumbar, dos
regiones más predispuestas en el sentido que son zonas de mayor movilidad en movimientos de flexoextensión y rotaciones, por lo
tanto son las zonas donde actúan los vectores luego de la aplicación de alguna fuerza que daña las estructuras ya sean óseas o
discoligamentarias de la columna vertebral. Entre las causas más frecuentes está el trauma automovilístico, generalmente asociado al
alcohol. Estos dos factores están presentes en por lo menos 50% de los casos de trauma espinal. Otras causas son los accidentes
industriales, deportivos y el trauma como resultado de la inseguridad: heridas cortopunzantes y heridas por arma de fuego. Los
mecanismos de lesión pueden ser directos o indirectos, los directos son los menos frecuentes ocurren debido a la aplicación de energía
en el mismo punto donde se produce la lesión de columna vertebral. Los mecanismos indirectos, los más frecuentes, producen lesión
no necesariamente en el sitio de aplicación de la energía. Existen 4 mecanismos principales de trauma espinal: 1. Extensión, 2. Flexión,
3. Rotación, 4. Compresión vertical o carga axial. Estos son movimientos tolerados normalmente por la columna y las estructuras
nerviosas espinales siempre y cuando se mantengan dentro de rangos fisiológicos. De lo contrario se presenta disrupción anatómica y
lesión estructural del continente (columna) y contenido (medula-raíces nerviosas). Con frecuencia en una sola lesión espinal intervienen
dos o más de los cuatro mecanismos. El trauma espinal producido por flexión forzad, la posibilidad de lesiona es causa frecuente de
lesión ósea, ligamentaria y de medula y raíces nerviosas. La flexión forzada asociada o no a carga vertical (axial) se presenta por
ejemplo en casos de “clavados” en piscina o ríos pandos. La hiperextensión puede producir lesión espinal, en esta se produce
disminución del espacio intraespinal. En pacientes de edad avanzada en los cuales por cambios degenerativos el espacio interior del
canal espinal ya esta disminuido, la posibilidad de lesión de las estructuras contenidas dentro del canal espinal aumenta. Desde el
punto de vista fisopatologico hay eventos después del trauma espinal que llevan finalmente a la desintegración de la membrana
celular, es decir, a la “peroxidación lipídica de la membrana celular”. Algunos de estos eventos son la entrada de calcio al espacio
intracelular, el aumento de radicales libres, ac. Araquidónico, prostaglandinas y tromboxano, la liberación de endorfinas y aumento de
norepinefrina. La peroxidación lipídica lleva a desctruccion celular irreversible. DIAGNOSTICO: Sospechar traumatismo raquimedular en
todo paciente politraumatizado, inconsciente o intoxicado. El examen neurológico es de gran importancia para determinar tanto en el
paciente consciente como en el inconsciente, si existe compromiso neurológico de medula, de raíces nerviosas o de ambas, como
resultado de trauma espinal; se debe valorar la fuerza, sensibilidad, reflejos, compromiso de esfínteres y deformidad espinal. Las
lesiones medulares pueden ser completas o incompletas. Indicios de trauma de columna: Dolor, hematomas, pérdida del patrón
respiratorio, hipotensión y bradicardia, globo vesical, hipotonía de esfínter anal, priapismo, debilidad, hipoestesia. Se pueden
manifestar como síndrome medular anterior, central, posterior o de hemisección medular. La sección medular completa se caracteriza
por pérdida total de movimientos, sensibilidad, reflejos y control de esfínteres. En su grado más extremo, el trauma espinal se asocia a
sección anatómica o fisiológica de la medula espinal, con cuadro clínico como shock espinal. En el examen se encuentra parálisis
(paraplejia-cuadriplejia), nivel de anestesia, arreflexia, ausencia de función esfinteriana y sock neurogénico producido por desconexión
del sistema simpático (simpatectomía) y que se caracteriza por hipotensión arterial y bradicardia. Hacen parte de la lesión simpática
vasodilatación, perdida de calor por esta razón e hipotermia. Una vez estabilizado el paciente desde el punto de vista hemodinámico y
respiratorio, el primer estudio que se debe realizar en el paciente politraumatizado es la radiografía lateral de columna cervical. Esta
determinara la presencia o no de lesiones traumáticas de columna cervical en 90% de los casos y permitirá establecer si se puede o no
inmovilizar. Las radiografías AP y transoral, pueden complementar, el estudio imagenológico inicial. Opcionalmente en el paciente TCE,
que se tome TAC cerebral, se puede aprovechar este estudio para realizar de forma complementaria cortes de TAC de las 3 primeras
vertebras cervicales. La RMI, es un excelente método diagnóstico para lesiones espinales traumáticas, puede mostrar contusión
medular, edema o sección medular, una zona de hiperintensidad medular generalmente implica la presencia de sangre dentro de la
medula (hematomielia). De igual manera puede demostrar la presencia descompresión de medula y raíces por disco, hematoma
epidural, fragmentos de vertebra. TRATAMIENTO: Se debe seguir un protocolo secuencial: 1. Inmovilización desde el sitio y momento
del accidente. 2. Diagnóstico clínico e imagenológico. 3 intervención farmacológica para disminuir la severidad de la lesión inicial y las
secuelas neurológicas 4. Reducción de luxaciones, descompresión de tejido nervioso si hay fragmentos de hueso, disco, ligamento,
sobre raíces o medula espinal. 5. Fijación espinal en caso de inestabilidad. 6. Rehabilitación del paciente. Las lesiones traumáticas de
columna que produzcan desplazamiento anormal de sus elementos requieren de reducción. La inmovilización y reducción de lesiones
espinales cervicales puede hacerse mediante la colocación de halo craneano que tracciona el cráneo e indirectamente la columna
cervical, alineando las vértebras. En el trauma medular se debe ante todo evitar el daño secundario, el cual puede estar causado por
hipotensión arterial, bradicardia, hipoxemia, hipercarbia, acidosis y aumento de la viscosidad sanguínea. Se han empleado compuestos
para disminuir o anular la peroxidación lipídica. La metilprednisolona (30 mg/kg en bolo, 5.4 mg/kg/h por 23 hrs) es la sustancia más
aceptada hasta el momento ya que tiene efectos inversos, disminuye la entrada de calcio a la célula, aumenta el flujo sanguíneo e
inhibe a la PG 2 alfa y el tromboxano. El estudio NASCIS II demostró que la administración de esta droga lñogra una mejor recuperación
funcional motora y sensitiva. Su administración constituye la base de la intervención farmacológica en los pacientes con trauma espinal.
Conservador: Tracción, inmovilización, rehabilitación. Quirúrgico: Remoción de fluidos o tejidos que comprimen (medula o racices),
fusión ósea, inestabilidad (que requiere estabilización mediante fijación o artrodesis), compresión medular. NEURONA MOTORA
SUPERIOR: Parálisis, hipertonía (espasticidad), hiperrreflexia, babinski. NEURONA MOTORA INFERIOR: Parálisis, hipotonía,
hiporrreflexia, atrofia. SINDROME MEDULAR ANTERIOR: Se caracteriza principalmente por lesión motora asociada a signos

CURSO ENARM CMN SIGLO XXI TEL: 36246001 Pharmed Solutions Institute PÁGINA 535
MANUAL DE TRABAJO DEL CURSO ENARM CMN SIGLO XXI
piramidales, con conservación de diferente magnitud de la sensibilidad del paciente. SINDROME MEDULAR POSTERIOR: Se manifiesta
clínicamente por una mayor lesión de tipo sensitiva y de diferentes grados de conservación de la función motora. SINDROME DE
BROWN-SEQUARD: Caracterizado por la lesión anatómica de la mitad de la medula espinal, se manifiesta clínicamente por pedida de la
función motora ipsilateral y perdida de la función sensitiva contralateral (dolor, termoalgesia). SINDROME DE SECCION MEDULAR
TRANSVERSA: Se caracteriza clínicamente por presentar una pérdida total de la función sensitiva motora y de los reflejos desde el nivel
de la lesión hacia distal (no necesariamente se asocia a una sección medular anatómica). La lesión de las raíces generalmente se
produce por una tracción por los diferentes mecanismos de lesión o por un desplazamiento de las vértebras que produce un
estrechamiento del diámetro de los orificios de conjunción con lo cual se produce una compresión de las raíces en diferentes
magnitudes. La lesión de la cauda equina se puede producir en forma total o parcial en relación con los desplazamientos
anteroposteriores de la columna vertebral lumbar desde el nivel L2 hacia distal; también puede lesionarse la cauda equina por
protrusión hacia posterior y ocupamiento del canal raquídeo de fragmentos óseos o fragmentos discales. SÍNDROME
CENTROMEDULAR DE SCHNEIDER: Paresia flácida en miembros superiores (afectación de 2ª neurona), disminución termoalgésica en
miembros superiores, dolores por desaferentización en miembros superiores, sin afectación de miembros inferiores o menor que en
extremidades superiores. Esta paresia es espástica (afectación 1ª neurona). MEDULAR CENTRAL (SIRINGOMIELIA): bilateral,
sensibilidad termoalgésica segmentaria, disociación termoalgésica. Exploración clínica: C5 flexión de codo (bíceps), C6 extensión
muñeca (ECRB, ECRL), C7 extensión codo (tríceps), C8 flexor dedo medio (FDP), T1 abducción meñique, L2 flexión cadera (psoas iliaco),
L3-L4 extensión rodilla (cuádriceps), L4 dorsiflexión tobillo, L5 dorsiflexión hallux, S1 flexión plantar (hallux, gemelos, sóleo).

CASO CLINICO

TRAUMA FACIAL. CIENCIAS BASICAS: Trauma se define como el daño que sufren los tejidos y órganos por acción de una energía que
puede actuar en forma aguda o crónica. El trauma maxilofaciales aquél que compromete tanto partes blandas como óseas de la región
facial y ocurre en aproximadamente el 30% de los politraumatizados. Su riesgo es la muerte por asfixia, hemorragias y asociación de
lesiones de columna vertebral y SNC. Las secuelas pueden ser estéticas y funcionales. SALUD PUBLICA: La principal causa de este tipo
de pacientes politraumatizados son los accidentes de tráfico, en el 40% de los casos; seguidos por los accidentes domésticos con un
20% de la totalidad; las caídas casuales se presentan en el 14%; los accidentes laborales suponen un 3% y otras causas el8% restante.
Existe un claro predominio por el sexo masculino presentando un porcentaje del 68%, con una edad media de 32 años y un rango de
edad entre 20 y 45 años. PATOGENIA: La energía involucrada en un trauma es directamente proporcional a la mitad de la masa y al
cuadrado de la velocidad (E= 1/2Mx V2). Con una masa constante, al doblar la velocidad, la energía liberada es 4 veces más y cuando se
triplica es 9 veces más. Según la energía los traumatismos se clasifican en: baja energía: determina fracturas más simples, con menor
desplazamiento disyunciones y escaso compromiso de partes blandas. Alta energía: fracturas más complejas, con mayor
desplazamiento, extensión y conminucion, y con gran compromiso de partes blandas. Las fracturas son el resultado de una sobrecarga
mecánica (energía) que una fracción de segundo supera la resistencia ósea y determina disrupción. Esta lesión establece la interrupción
del flujo sanguíneo tanto en el hueso (cabos de fractura), como en los tejidos adyacentes. DIAGNOSTICO: La base del diagnóstico, en
una buena anamnesis y exploración. Es importante preguntar sensaciones disestesicas o anestésicas faciales, características del dolor,
alteraciones subjetivas de la oclusión, alteraciones de la visión, obstrucción nasal, epifora, alteraciones auditivas. Inspección; heridas,
abrasiones, equimosis, orientan a zona de impacto. Identificar asimetrías, falta de fuerza, deformidades. Palpación; de cefálico a caudal,
terminando con una palpación de la mandíbula y de las regiones de las articulaciones temporomandiblares. Examen intraoral:
hematomas, equimosis, cuerpos extraños, ausencias dentarias, escalones óseos y muy importante la relación oclusal. Palpación de
columna cervical, y tener siempre oculoscopio nasal. El examen gold standard del trauma facial grave es el scanner, en especial en los
que se someterán a cirugía. La radiología convencional: 1) Radiografía de Waters: con cabeza hiperextendida, lo que permite
desproyectar la base del cráneo del tercio medio facial. Útil para pirámide nasal, cuerpos cigomáticos, maxilares superiores, arcos
cigomáticos y rebordes infraorbitarios. Generalmente se completa con hueso propio de la nariz. 2) Radiografía de Towne: útil para
región condilea y subcondilea de la mandíbula, como piso de orbita. 3) Radiografía de Hirtz: para arcos cigomáticos. 4) craneo lateral y
PA. 5) placas de mandíbula 6) ortopantomografia. TERCIO SUPERIOR: Se refiere a las fracturas frontales, nasales, naso orbito
etmoidales (NOE), órbita, complejo cigomático maxilar, maxilar y paladar. FRACTURAS NASALES: Son las más frecuentes (40-50%) de
las fracturas faciales. La mayoría incluye el tercio distal de los huesos nasales, con el margen etmoidal intacto, generalmente las
desviaciones del eje nasal obedecen a desviaciones del septum por fracturas de este en forma exclusiva o asociadas a fracturas de los
huesos propios, el tratamiento de reducción puede ser diferido hasta 12 dias. FRACTURAS DEL COMPLEJO NASO-ORBITO-ETMOIDAL:
Se localizan en el tercio medio central, con alteración en la confluencia de los pilares vertical medial y transversal superior, así como de
sus extensiones posteriores a lo largo de la pared medial de la órbita y el piso. Lo que podría corresponder a una fractura conminuta del
pilar vertical maxilar medial, en especial de la fosa lagrimal, donde se inserta el ligamento acantal. TERCIO MEDIO: FRACTURAS
MAXILARES Le Fort I, II, III, NOE, nasales, naso maxilares y aisladas del maxilar. cigoma, arco cigomático, complejo cigomático maxilar y
combinadas o especiales (panfaciales). Rene Le Fort clasifico las fracturas maxilares en tres tipos, según el trayecto de la línea de
Fractura. LE FORT I: (horizontal) puede resultar de una fuerza dirigida de lesiones bajo en el borde alveolar del maxilar en dirección
hacia abajo. La fractura se extiende desde el tabique nasal hasta el piriforme lateral llantas, viaja horizontalmente por encima de los
ápices de los dientes, cruza por debajo de la unión zygomaticomaxillary, y atraviesa la unión pterigomaxilar para interrumpir los platos

CURSO ENARM CMN SIGLO XXI TEL: 36246001 Pharmed Solutions Institute PÁGINA 536
MANUAL DE TRABAJO DEL CURSO ENARM CMN SIGLO XXI
pterigoideos.; LE FORT II: (Fractura piramidal) La línea de fractura se extiende a través de los huesos propios-nasales y el septum hacia
abajo y hacia atrás por la pared medial de la órbita, cruza el –reborde infraorbitario y pasa por el arbotante cigomatico-maxilar. LE FORT
III: (Disyunción craneofacial) Es una verdadera separación de los huesos de la base del cráneo. El trazo de la fractura pasa por la sutura
nasofrontal, por la pared medial de la –orbita hasta la fisura orbitaria superior, de esta a la fisura orbitaria inferior y por la pared-lateral
de la órbita hasta la sutura cigomático-frontal y cigomático-temporal. Hacia atrás-se fracturan las apófisis pterigoides del esfenoides,
normalmente a un nivel superior al –que aparecen en las otras fracturas de Le Fort. Clinica: Hematoma periorbitario bilateral o en
antifaz, epistaxisis, equimosis conjuntival. Aumento de la longitud del tercio medio facial (cara larga, cara de plato). Mala oclusión.
Mordida abierta anterior: contacto prematuro de los molares respecto al segmento anterior. Vigilar rinolicuorrea y otorrea.
Tratamiento: Cobertura antibiótica y analgésica. Vigilancia de la vía aérea, control de la hemorragia y remitir a centro especializado. Si
existe rinolicuorrea no realizar taponamiento nasal y prescribir tratamiento antibiótico. FRACTURAS CIGOMATICO-MAXILARES:
Caracterizado por la presencia de hematoma, equimosis, y depresión de la eminencia malar, asociado o no a limitación de la apertura
oral. FRACTURAS ORBITARIAS: Estas pueden ser de cualquiera de sus cuatro paredes, siendo más frecuente el piso y las paredes lateral
(en fracturas malares) y medial, lo mas importante es efectuar estudio del globo ocular, para descratar heridas que puedan peligrar la
visión. En el segmento posterior de la órbita, se unen el piso con la pared medial, formando lo que se conoce como la zona clave, la cual
se ve frecuentemente fracturada, su diagnóstico es por scanner, y es la causa más frecuente de enoftalmo post traumático, de tal
manera que se debe buscar en forma dirigida. TERCIO INFERIOR: FRACTURAS DE MANDIBULA: Muy frecuentes, el paciente
generalmente presenta impotencia funcional mandibular y/o alteraciones de la oclusión. Es un hueso de consistencia dura que
presenta tres zonas débiles que son el cuello del cóndilo mandibular, la zona canina-agujero mentoniano y el ángulo mandibular debido
a la confluencia de las ramas horizontal y vertical y la presencia de la muela del juicio. Es más frecuente en varones jóvenes entre 20 y
30 años. La zona de fractura más frecuente es el cóndilo mandibular, seguido del ángulo mandibular y la región parasinfiaria. Es
aconsejable la valoración en conjunto con el odontólogo, por posibles fracturas dentoalveolares asociadas. Clinica: Síntomas: dolor,
tumefacción, disconfort, hematoma, deformidad, movilidad anormal, crepitación, salivación, mal olor, mal oclusión y trismus Signos:
mala oclusión, movilidad del foco de fractura, disfunción a la deglución y masticación, crepitación y tumefacción. Tratamiento:
Preservar vía aérea, control de la hemorragia, sutura de heridas, limpieza de la cavidad oral, remitir a centro especializado, cobertura
antibiótica y analgésica. TRATAMIENTO: Debe ser efectuado idealmente una vez que el paciente se haya estabilizado y se haya
realizado el estudio radiológico adecuado, la cirugía facial se puede retrasar hasta 10 días. Indicaciones de traqueostomia: fracturas
panfaciales. Fractura conminuta de la mandíbula. Edema masivo de tejidos blandos faciales (quemadura). Las hemorragias de partes
blandas, se recomienda compresión, ya que los pinzamientos a ciegas pueden producir graves daños a estructuras nerviosas. En el caso
de epistaxis, el taponamiento nasal anterior continua vigente. El taponamiento posterior puede efectuarse con gasas lo más adecuado
o con sonda Foley, en caso de persistir ligadura arterial selectiva. Como norma general el paciente traumatizado facial asocia heridas de
las partes blandas de la cara, heridas que son complejas por su mecanismo de acción, su tratamiento y su repercusión estética
posterior. Las heridas: en primer lugar, deben de ser lavadas profusamente con suero salino; un cuidadoso y exhaustivo examen para
intentar eliminar cuerpos extraños en su totalidad, bajo anestesia local las manchas de los bordes de las heridas tienen que ser
eliminados para evitar la aparición del tatuaje postraumático cepillado enérgico de los bordes de la herida. Posteriormente se procede
a eliminar aquellos fragmentos de tejido necrótico que puedan dificultar una correcta cicatrización, valorar lesiones de tipo nerviosos
por sección de ramas motoras o sensitivas y lesión de conductos lacrimales y salivares y finalmente se realiza hemostasia y sutura de las
mismas. Profilaxis antitetánica y cobertura antibiótica de amplio espectro. Analgésicos. Dada la riqueza vascular presente en el macizo
facial la sutura de las heridas en este territorio puede demorarse de 6 a 24 horas.

CASO CLINICO

CURSO ENARM CMN SIGLO XXI TEL: 36246001 Pharmed Solutions Institute PÁGINA 537
MANUAL DE TRABAJO DEL CURSO ENARM CMN SIGLO XXI
TRAUMA TORACICO CERRADO Y ABIERTO:
CIENCIAS BASICAS: Es cualquier agresión o trauma sobre las paredes del tórax que producirá un daño en las estructuras sólidas y partes
blandas comprendidas en la caja torácica. Los traumatismos torácicos pueden ser cerrados (contusos) o abiertos (penetrantes).
Traumatismo abierto: se denomina a lesión que rompe la integridad del tejido (atraviesa pleura parietal). Traumatismo cerrado: resulta
por aplicación de energía que provoca lesión sobre los tejidos sin dañar su integridad. SALUD PUBLICA: En México los traumas cerrados
son mayoritariamente por accidente de tránsito, los penetrantes TRAUMA TORACICO CERRADO TRAUMA TORACICO ABIERTO
son por arma blanca, aunque han aumentado las heridas por armas Asociada a compresión y aceleración- Asociada a heridas por arma blanca
de fuego. 8 de cada 100.000 son letales. Principales causas de desaceleración y arma de fuego
Hay fracturas costales múltiples Puede o no haber fracturas costales
traumatismo torácico asociadas: Accidentes de tránsito (43%),
Puede haber hemo o neumotórax Hemo o neumotórax inmediato.
Suicidios (29%), Homicidios (22%.). CLASIFICACION: Según el tardío (>24hrs del trauma)
estado hemodinámico: Normales o compensados El trauma de vía aérea superior se El trauma de vía aérea cursa con
hemodinámicamente Presión Arterial: 90/60 ↑, Frecuencia manifiesta como estenosis gran escape aéreo
Cardiaca: 110 ↓, diuresis satisfactoria. Anormales o Tratamiento quirúrgico: requerido en Tratamiento quirúrgico: requeridos
descompensados hemodinamicamente Presión Arterial: 90/60 o ↓, menos del 10% de lesionados entre 15-30% de lesionados.
Herida de arma blanca: Punzantes, Acción directa ( golpe o choque
Frecuencia Cardíaca: 120 o ↑, oligoanuria o anuria. Ambos pueden cortantes directo)
ser estables (se mantienen los parámetros con el correr del tiempo) Heridas de proyectil de arma de Acción indirecta: Compresión,
o inestables. Según grado de penetración torácica: Grado 1: No fuego: De baja velocidad (< 750 m/s) alteración de la velocidad,
comprometen pleura. Grado 2: Penetran pleura parietal, penetran De alta velocidad (> 751 m/s) Aceleración brusca, Desaceleración
Misceláneas ( Otros elementos lesivos brusca, Torsión, Deslizamiento,
pleura parietal y parénquima pulmonar. PERFORANTES O
internos o externos) Inmersión
TRANSFIXIANTES (Entran y salen del tórax y comprometen otras
cavidades). Grado 3: Penetran pleura, pulmón, mediastino o abdomen o región cervical (Dos cavidades). Grado 4: Penetran pleura,
pulmón, mediastino y el otro hemitorax o abdomen o región cervical (tres cavidades o regiones) Nos da gravedad en un Trauma
torácico: Impactos de alta energía: caída mayor a 6 metros. Impactos de alta velocidad. Pasajeros despedidos del vehículo. Atropello.
Lesión penetrante de cabeza, cuello, tórax, abdomen o región inguinal. Dos o más fracturas proximales de huesos largos.
Quemaduras mayores al 15% de SC o que afecten cara o vías aéreas. Tórax inestable. TIPOS DE TRAUMA TORÁCICO: Seis lesiones
rápidamente fatales (Se detectan en evaluación primaria): Obstrucción de la vía aérea. Neumotórax Hipertensivo. Neumotórax Abierto.
Taponamiento cardíaco. Hemotórax masivo. Tórax inestable. Seis lesiones potencialmente letales u ocultas (Detectadas en la
evaluación secundaria): Lesiones traqueobonquiales. Ruptura diafragmática. Lesión esofágica. Contusión pulmonar. Ruptura aórtica.
Contusión miocárdica. Lesiones no necesariamente letales: Neumotórax simple, evidente u oculto. Hemotorax simple. Enfisema.
Fracturas costales. Fractura de Esternón o Escápula. 1.-OBSTRUCCIÓN DE LA VÍA AÉREA; se puede producir por cuerpos extraños,
secreciones, sangre, el manejo puede ser invasivo o no invasivo, otra clasificación es “vía aérea quir rgica” y “no quir rgica”. Consisten
en el manejo manual de la vía aérea, intubación oro o nasotraqueal, cricotiroidotomia por punción o quirúrgica y la traqueotomía. 2.
NEUMOTORAX HIPERTENSIVO; Ocurre cuando una lesión permite la entrada de aire al espacio pleural durante la inspiración, sin que
pueda salir durante la espiración. Esto lleva al colapso pulmonar, con desplazamiento mediastinal hacia el lado opuesto, se disminuye el
retorno venoso por angulación de las cavas, compromete el gasto cardíaco y el otro pulmón.El diagnóstico es clínico: taquipnea,
dificultad respiratoria aguda, tiraje supraclavicular e intercostal, hipersonoridad y ausencia de murmullo vesicular, distención de venas
del cuello, desviación de la tráquea cianosis. Tratamiento, descomprimir inmediatamente, con punción catéter teflonado calibre 12 o
14, en 2° espacio intercostal línea medio clavicular, con lo cual lo convierte en normotensivo. Proceder a la colocación de avenamiento
pleural bajo agua en 4 espacio intercostal línea medio clavicular. 3.- NEUMOTÓRAX A TENSIÓN; es el escape de aire hacia la cavidad
pleural, mas común neumotórax espontáneos, el mediastino y la tráquea se desplazan hacia al lado opuesto, comprometiendo la
posibilidad de respuesta ventilatoria por parte del pulmón sano, y afectando el retorno venos. Clínicamente se manifiesta por dificultad
respiratoria, taquicardia, hipotensión, desviación de la tráquea, ausencia unilateral de MV, timpanismo del pulmón afectado,
ingurgitación yugular y cianosis tardía. El tratamiento inicial consiste en insertar una aguja, en 2° EIC en LMC del hemitórax afectado,
siempre por el borde superior de la 3° costilla. El definitivo es la inserción de un tubo de tórax en el 5° EIC anterior a la LAM, siempre
por el reborde costal superior de la 6° costilla, en el hemitórax afectado. 4.-NEUMOTÓRAX ABIERTO; Al producirse una herida en la
pared torácica el aire penetrará preferentemente por la herida al igualarse las presiones intratorácica y ambiental y ofrecer menor
resistencia al paso del aire por la herida. El tratamiento inicial será cubrir la herida con un apósito fijado en tres puntas. El tratamiento
definitivo consiste en la instalación de un catéter intercostal y conectarse a un equipo de succion a 10-25 cmH2O. Si el neumotórax es
estable puede mantenberse bajo vigilancia y se reabsorberá a un ritmo de 1.25% por dia. , distante de la lesión, y el cierre de la herida
será quirúrgico. Complicaciones: Inadecuada conexión drenaje-aspirador. Inadecuada colocación del drenaje. Oclusión bronquial.
(Cuerpo extraño, coágulo, rotura). Roturas traqueobronquiales. Grandes laceraciones pulmonares. Severa disminución de la
distensibilidad pulmonar. 5.- HEMOTÓRAX MASIVO: Es el resultado de la acumulación de sangre en la cavidad pleural, igual o superior
a 1500 ml o mas de 200ml/h durante 4 hrs. La principal causa: es la lesión de vasos hiliares y mediastinicos generalmente por heridas
penetrantes. Clínicamente encontrará un paciente en shock, con colapso de los vasos del cuello por hipovolemia o con ingurgitación de
estos por efecto mecánico de las cavidades. El tratamiento será simultáneamente con reposición de volumen de forma agresiva
(cristaloides, coloides y sangre), y descompresión del hemitórax lesionado con un tubo de tórax único N° 28-32 f. Se hará toracotomía
sí: Deterioro hemodinámico sin otra justificación. Persiste un drenaje superior a 1500 ml en las primeras 12-24 horas. Drenaje superior
a 200 ml/hora en 4 horas. Persistencia de la ocupación torácica (hemotórax coagulado). 6.- TAPONAMIENTO CARDIACO: Producto de
una herida penetrante, en su gran mayoría, pero también puede aparecer por lesiones de los vasos pericardicos o traumatismo
cardíaco en un traumatismo cerrado. Desde el punto de vista clínico se manifiesta por la “tríada de ec ”, que consiste en el hallazgo
de: aumento de la presión venosa central, disminución de la presión arterial, apagamiento de los ruidos cardiacos. Ingurgitación
yugular con la inspiración en un paciente ventilando espontáneamente es signo inequívoco de taponamiento cardiaco (signo de
Kussmaul). En el tratamiento, la pericardiocentesis por vía subxifoidea es de elección en el prehospitalario, para descompresionar el
pericardio, basta extraer 15-20 ml, pero es una medida temporal, actualmente se postula que en él SU y Hospital no deben hacerse
pericardiocentesis a menos que sea para dar tiempo a la preparación del pabellón. 7.- TÓRAX INESTABLE: El tórax se torna inestable
cuando un traumatismo contuso produce fractura en 2 puntos de 4 o más costillas. El paciente es incapaz de generar la presión

CURSO ENARM CMN SIGLO XXI TEL: 36246001 Pharmed Solutions Institute PÁGINA 538
MANUAL DE TRABAJO DEL CURSO ENARM CMN SIGLO XXI
negativa suficiente para manetener la ventilación; la intubación y la ventilación con presión positiva son imperativos. Clínicamente
puede no ser detectado en primera instancia por la hipoventilación reactiva al dolor, y por los movimientos del tórax. El tratamiento se
basa fundamentalmente en una buena ventilación, eventualmente mecánica, oxigenación, y tratamiento para el dolor. Si el paciente no
está en shock la infusión de fluidos debe ser cuidadosa para evitar la sobrehidratación y el consiguiente edema pulmonar. Un adecuado
y controlado balance hídrico. Cuando coexisten fracturas costales múltiples en varias costillas consecutivas se produce una inestabilidad
de la pared con movimiento paradojico y alteración de la mecánica respiratoria, con la consiguiente hipoxia. La gravedad de la lesión es
directamente proporcional al grado de alteración del parénquima pulmonar en combinación con el daño de la pared. 8.- CONTUCION
PULONAR: Lesión del parénquima pulmonar que causa hemorragia y edema localizado, producto de traumas en los que hay rápida
compresión y descompresión del tórax. Si se afecta un área considerable es probable que se necesite asistencia mecánica, pero casi
nunca durante más de 48-72 hrs. Se observa falla respiratoria tardía, lenta, progresiva y sutil, con o sin tórax inestable.9.- CONTUCION
MIOCÁRDICA: Difícil de diagnosticar, se sospecha por alteraciones al ECG (arritmias, extrasístoles mono o bifocales, taquicardia sinusal
inexplicable, FA, bloqueo de rama, o claramente un infarto), Eco cardiografía bidimensional e historia compatible. El tratamiento
tratara la manifestación clínica o la arritmia específica. 10.- RUPTURA DE AORTA: Producto de traumatismos cerrados, por laceración o
arrancamiento de los puntos de fijación de la Aorta. Signos clínicos alertan el diagnóstico: Mediastino ensanchado en Rx de Tx. Fx de
1ª y 2ª costillas. Desviación y elevación del bronquio principal, de la tráquea y el esófago hacia la derecha. Depresión del bronquio
principal izquierdo. Opacidad pleural apical. Tratamiento: quirúrgico (reparación o implante). 11.- RUPTURA DIAFRAGMÁTICA:
Presente con más frecuencia en el lado izquierdo ya que se carece de la protección del hígado. Se sospecha el diagnóstico por la
presencia de intestino, estomago en el hemitórax izquierdo. Los traumatismos penetrantes por arma blanca o de fuego a veces pasan
inadvertidos y solo se detectan años después cuando aparece la hernia diafragmática. 12.- LESIÓN TRAQUEOBRONQUIAL: Laringe
Diagnóstico: ronquera, enfisema subcutáneo y crepitación palpable de fractura. Manejo, en caso de vía aérea obstruida: instalación de
IOT o Traqueotomía. Tráquea: las lesiones penetrantes son más obvias que las provocadas por trauma. Se asocia a lesión de esófago y
grandes vasos. Diagnostico a través de broncoscopía. Bronquios: La lesión de un bronquio mayor es rara y mortal, y ocurren a 2-3 cm.
de la carina. Diagnóstico: hemoptisis y enfisema subcutáneo, y se sospecha en neumotórax a tensión con gran escape de aire y es
confirmado por broncoscopía. “Una vez confirmado el diagnóstico de lesión traqueobronquial el tratamiento es la reparación
quir rgica”. 13.- RUPTURA ESOFÁGICA: Se sospechara cuando exista neumo o hemotórax a izquierda sin fracturas costales, trauma
directo al esternón o epigastrio con dolor y shock no explicado, además puede haber salida de partículas de contenido digestivo por
tubo de tórax. Tratamiento: La conducta será la reparación directa, esofagostomía cervical de escape y yeyunostomía de alimentación.
14. FRACTURAS ESTERNALES: Suelen ser transeversas, se encuentran en el manubrio o cerca de él y son dolorosas. Resulta esesncial
descratar la lesion de estructuras adyacentes, en especial el corazón. INSERCIÓN DE PLEUROTOMÍA; cuidados postoperatorios: El
paciente deberá permanecer en el hospital hasta que el tubo torácico sea retirado. Debe quedar conectado a una trampa de agua
donde el pivote del sello de agua debe quedar sumergido a 2cm, y oscilar cuando esté conectado al paciente. Mientras el tubo está
colocado en el tórax del paciente, los enfermeros verifican con cuidado que no haya escapes de aire, dificultades o problemas al
respirar o necesidad de administrar oxígeno adicional al paciente. El paciente tosa y respire profundo para facilitar que los pulmones se
expandan de nuevo, ayudar con el drenaje y prevenir que los fluidos normales se alojen en los pulmones. ATENCIÓN Y MANEJO: Las
muertes relacionadas con Trauma, en la actualidad siguen una curva bimodal (2 picos de frecuencia diferentes). El primer pico, ocurre
dentro de la primera hora. El segundo pico en las primeras 24 a 48 horas después del trauma. La cirugía de control de daños es en
etapas: PRIMERA ETAPA: Control de hemorragia y fuga aérea. SEGUNDA ETAPA: Resucitación en la UCI, manejo agresivo de la
hipotermia, acidosis, coagulopatia. TERCERA ETAPA: Tratamiento definitivo de las lesiones. REGLA DE ORO (GOLD STANDARD): Efectuar
las mínimas reparaciones definitivas, usando técnicas que sean rápidas y fáciles, para acortar al máximo los tiempos quirúrgicos de este
modo. En base a ATLS: A. Vía aérea y control de columna cervical. Permeabilidad, estabilidad y seguridad de la vía aérea (eventual IOT),
aspiración de secreciones, fijación de columna cervical firme y segura, cricotiroidotomía por aguja o quirúrgica. B. Respiración:
Oxigenación, movimientos respiratorios, ventilación asistida, oclusión de heridas torácicas abiertas, toracocentesis y drenajes torácicos.
C. Circulación y control de hemorragias. Compresión directa de los sitios de hemorragia, evaluación de los pulsos, masaje cardiaco
externo, instalación de dos vías venosas periféricas proximales de grueso calibre para alto flujo, reposición de volumen y uso de
fármacos endovenosos, analgesia y sedación, monitorización cardiaca. D. Déficit neurológicos. E. Exposición corporal y abrigo. En el
caso de focos hemorrágicos en cavidades, es imprescindible su detección y esto se utiliza el FAST (Focused Abdominal Sonography for
Trauma), que permite la evaluación del abdomen y del pericardio, del líquido pleural. La mayoría de los traumatismos torácicos 75-85 %
se solucionan con maniobras salvadoras de vida y o procedimientos quirúrgicos menores: 1) Toracocentesis 2) drenaje pleural 3)
Pericardiocentesis 4) Ventana xifopericárdica 5) Cricotiroidotomía 6) Traqueostomía. Alteraciones que requieren toracotomía urgente:
1. Fuga masiva de aire. Indica rotura de la tráquea o de un bronquio principal. Más de 80% de las lesiones se localizas a menos de 2-5cm
de la cariña. 2. Taponamiento pericárdico en presencia de traumatismo. 3. Perforación esofágica.

CASO CLINICO
Paciente de 43 años, fumador habitual, acude a urgencias por dolor costal, después de una caída de 3 metros de altura
aproximadamente. A su llegada al hospital el paciente presenta PA de 130/70 mmHg, FC de 65lpm y saturación de oxígeno del 98%
basal. En la exploración física destaca la presencia de crepitación ósea a la altura de la parrilla costal derecha baja, la radiografía de
tórax es normal. A las 48h el paciente consulta nuevamente a urgencias por aumento del dolor torácico asociado a dificultad
respiratoria y empeoramiento del estado general. La PA fue de 90/60 mmHg, la FC de 105lpm y la saturación basal del 89%. La
radiografía de tórax demuestra derrame pleural importante y el estudio analítico muestra los siguientes valores: Hto del 26% Hb de 7
g/l, plaquetas de 430.000/mm3. Estudio de coagulación: normal.

PREGUNTA
La primera maniobra en este paciente al recibirlo en urgencias es:

RESPUESTA
a.- Colocación de sello pleural.

CURSO ENARM CMN SIGLO XXI TEL: 36246001 Pharmed Solutions Institute PÁGINA 539
MANUAL DE TRABAJO DEL CURSO ENARM CMN SIGLO XXI
b.- Intubacion orotraqueal.
c.- Iniciar con cristaloides.
d.- Paquete globular.

CASO CLINICO
Varón de 33 años que sufrió un traumatismo torácico severo tras atropello. Precisó ventilación mecánica y fármacos inotrópicos.
Presentaba un soplo de insuficiencia aórtica. Se realizó una tomografía computarizada (TC) torácica, que objetivó neumotórax y
fracturas costales bilaterales, sin datos de disección aórtica. El ecocardiograma transtorácico evidenció insuficiencia aórtica severa con
dilatación ligera de la raíz aórtica. En el ecocardiograma transesofágico (ETE) se observó una imagen filiforme, correspondiente a velo
coronariano derecho roto, que protruía hacia el tracto de salida del ventrículo izquierdo, e insuficiencia aórtica severa. En la pared
anterior de aorta ascendente proximal, había una imagen de seudoaneurisma con rotura de capas íntima y media y se observaban
colgajos medio-intimales protruyendo hacia la luz.

PREGUNTA
La triada de Beck consiste en:

RESPUESTA
a.- Hipotensión, ruidos cardiacos velados y distensión venas del cuello.
b.- Hipertensión, edema facial y ruidos cardiacos velados.
c.- Hipotensión, torax inestable y disfagia.
d.- Ruidos cardiacos velados, distensión venas del cuello y disfonía.

TRAUMA ABDOMINAL ABIERTO Y CERRADO:


CIENCIAS BASICAS: Lesión orgánica producida por la suma de la acción de un agente externo junto a las reacciones locales y generales
que provoca el organismo ante dicha agresión. Todo paciente puede presentar lesiones en múltiples órganos abdominales y, por tanto,
debe ser considerado como un paciente con traumatismo grave, desde el momento del ingreso en la unidad de urgencias. Un tercio de
los pacientes que requieren una exploración abdominal urgente tienen un examen físico inicial anodino, tener en cuenta que puede
tener un comportamiento impredecible y desestabilizarse en el momento más inesperado. Importante conocer el mecanismo lesional
con el fin de anticipar las lesiones esperables. ANATOMÍA: Abdomen anterior: se define como el área localizada entre una línea
superior que cruza por las mamilas, los ligamentos inguinales y la sínfisis del pubis como la línea inferior, y las líneas axilares anteriores
lateralmente. Flanco: Área entre las líneas axilares anteriores y posteriores y desde el cuarto espacio intercostal hasta la cresta ilíaca. El
espesor de la musculatura de la pared abdominal a este nivel, más que las capas aponeuróticas más delgadas de la pared anterior,
actúa como una barrera parcial a las heridas penetrantes, particularmente por arma blanca. Espalda: localizada atrás de las líneas
axilares posteriores, desde la punta de la escápula hasta las crestas ilíacas, el espesor de la espalda y los músculos paravertebrales
actúan como una barrera parcial a las heridas penetrantes. Anatomía interna del abdomen: Cavidad peritoneal: cubierto por la parte
baja de la parrilla costal, el abdomen superior incluye el diafragma, hígado, bazo, estómago y colon transverso. Debido a que el
diafragma en una espiración total se eleva hasta el 4º espacio intercostal, las fracturas de costillas inferiores o heridas penetrantes en la
misma área pueden involucrar estas vísceras abdominales. El abdomen inferior contiene el intestino delgado y el colon ascendente,
descendente y sigmoides. Cavidad pélvica: rodeada por los huesos pélvicos, corresponde a la parte baja del espacio retroperitoneal y
contiene el recto, la vejiga, los vasos ilíacos, y en la mujer los genitales internos. Espacio retroperitoneal: contiene la aorta abdominal,
la vena cava inferior, la mayor parte del duodeno, el páncreas, los riñones, los uréteres, así como segmentos del colon ascendente y
descendente. Las lesiones en las vísceras retroperitoneales son muy difíciles de reconocer porque el área es de difícil acceso al examen
físico y sus lesiones no son detectadas por medio del lavado peritoneal diagnóstico, y difícilmente valoradas por la ecografía, además
de ser de difícil exploración física. SALUD PUBLICA: Traumatismos de los más frecuentes, estimándose en 1 por cada 10 ingresos por
traumatismo en los servicios de urgencias. Las principales causas de muerte en los pacientes con traumatismo abdominal son: 1. Por
lesión de algún vaso principal, como vena cava, aorta, vena porta o alguna de sus ramas, o arterias mesentéricas. Las lesiones
destructivas de órganos macizos, como hígado, bazo o riñón, o sus asociaciones, pueden originar una gran hemorragia interna. 2.
Sepsis: la perforación o rotura de asas intestinales o estómago, supone la diseminación en la cavidad peritoneal de comida apenas
digerida o heces, con el consiguiente peligro de sepsis. Los trastornos de vascularización de un asa intestinal por contusión de la pared
intestinal o de su meso pueden manifestarse tardíamente como necrosis puntiforme parietal y contaminación peritoneal con sepsis
grave. CLASIFICACIÓN: Abiertos (penetrantes y no penetrantes), presentan solución de continuidad en la piel. Cerrados; la piel no tiene
solución de continuidad. PATOGENIA: Las principales causas de abiertos son las heridas por arma blanca (lesiones intrabdominales de
20-30%) y arma de fuego cuya frecuencia es creciente. La principal causa de cerrados son los accidentes de tráfico. Otras causas son los
accidentes de trabajo, accidentes domésticos, accidentes deportivos, siendo estos mucho más frecuentes que los abiertos.
TRAUMATISMO ABDOMINAL ABIERTO: Las heridas por arma blanca y de fuego de baja velocidad (< 600 m/seg) causan daño al tejido
por laceración o corte. Ceden muy poca energía y el daño se localiza en la zona perilesional, afectando habitualmente órganos
adyacentes entre sí, siguiendo la trayectoria de, objeto que penetra. Las heridas por proyectiles de alta velocidad (> 600 m/seg)
transfieren gran energía cinética a las vísceras abdominales, teniendo un efecto adicional de cavitación temporal y además causan
lesiones adicionales en su desviación y fragmentación, por lo que es impredecible las lesiones esperadas. TRAUMATISMO ABDOMINAL
CERRADO: Impacto directo; la energía cinética a los órganos adyacentes a la pared abdominal, puede provocar lesión. Desaceleración:
mientras el cuerpo es detenido bruscamente los órganos intra abdominales animados aún por la energía cinética tienden a continuar
en movimiento produciéndose una sacudida, especialmente acusada a nivel de los puntos de anclaje, vasos y mesenterio que sufren
desgarros parciales o totales. Compresión o aplastamiento: entre dos estructuras rígidas, estas fuerzas deforman los órganos sólidos o
huecos y pueden causar su ruptura o estallido de estos. Este es el mecanismo típico de lesión del duodeno, en un accidente de
automóvil con impacto frontal, donde aquel es comprimido entre el volante y la columna vertebral. VALORACIÓN INICIAL: El objetivo
en evaluación primaria es evidenciar o descartar lesiones de riesgo vital e instaurar las medidas necesarias de soporte vital para

CURSO ENARM CMN SIGLO XXI TEL: 36246001 Pharmed Solutions Institute PÁGINA 540
MANUAL DE TRABAJO DEL CURSO ENARM CMN SIGLO XXI
preservar la vida del paciente. El paciente con traumatismo abdominal debe ser considerado como traumatismo grave o
potencialmente grave y por lo tanto, el manejo de estos pacientes debe seguir las recomendaciones del ABC: A. Asegurar la
permeabilidad de la vía aérea, con control cervical. B. Asegurar una correcta ventilación/oxigenación. Descartar neumotórax a tensión,
abierto, hemotórax masivo. Valorar la necesidad de soporte ventilatorio. Si no es necesario administrar oxígeno a alto flujo con
mascarilla (10-15 l/min). C. Control de la circulación. Detener la hemorragia externa. Identificación y tratamiento del shock.
Identificación de hemorragia interna. Monitorización ECG estable. D. Breve valoración neurológica. E. Desnudar completamente al
paciente, controlando el ambiente y previniendo la hipotermia. DIAGNOSTICO: Valoración clínica del estado de shock: aumento de la
frecuencia del pulso, pulso débil y filiforme, piel pálida, fría y sudorosa, disminución de la presión del pulso, retardo en el relleno
capilar, alteración de la conciencia, taquipnea, hipotensión y oligo anuria., dado que la hemorragia intraabdominal es la causa más
frecuente de shock hipovolémico en estos pacientes. Inicialmente, se asume que el estado de shock es el resultado de la pérdida aguda
de sangre y se la trata con una infusión rápida de volumen: un bolo inicial de 1-2 litros para un adulto de SF al 0’9 % o de solución de
Ringer lactato. La restitución por medio de catéteres intravenosos periféricos de calibre grueso (14G – 16G), dada su mayor rapidez de
canalización. El shock refractario a la infusión rápida de cristaloides sugiere sangrado activo y requiere de una laparotomía urgente.
Primeras medidas: Colocar sonda gástrica: cuyo objetivo es aliviar la dilatación gástrica aguda, descomprimir y reducir el riesgo de
broncoaspiración. Si existen graves fracturas faciales o la sospecha de una fractura de la base del cráneo, la sonda debe introducirse por
la boca para evitar el riesgo del paso del tubo hacia el cerebro a través de la lámina cribiforme. Canalización de dos vías venosas
periféricas con catéter de gran calibre. Sonda vesical para aliviar la retención de orina y descomprimir la vejiga, comprobar la
presencia de diuresis, su flujo horario, descartando previamente la presencia de lesión uretral. Se deben sacar muestras de sangre y
realizar una determinación de BH, tiempos de coagulación, gasometría, amilasa, niveles de alcohol, grupo sanguíneo y pruebas
cruzadas. Evaluación Secundaria: Historia clínica, a partir del propio paciente, de sus familiares y de los profesionales que han llevado a
cabo la atención prehospitalaria. Mecanismo de producción del traumatismo: a) En traumatismos cerrados es importante el tipo de
impacto, daño del vehículo, uso de sistemas de seguridad, el estado de otras víctimas. b) Para las heridas penetrantes, puede ser útil
una descripción del arma y de la cantidad de sangre perdida en el lugar del hecho. Tiempo de evolución desde el trauma hasta la
recepción del paciente. APP: alergias, patologías previas, medicación habitual, cirugía previa, ingesta de drogas. Maniobras realizadas
por los profesionales de la atención prehospitalaria: volumen infundido, vías canalizadas, necesidad de resucitación cardiopulmonar.
Exploración: Inspección; observar el tórax, abdomen, espalda, pelvis, periné, espalada. Hay que observar las huellas en la piel y pared
de los puntos de impacto del agente agresor. Auscultación; confirmar la presencia o ausencia de ruidos intestinales. La presencia de
sangre libre intra peritoneal o contenido gastrointestinal pueden producir un íleo que produce una ausencia de ruidos intestinales. Las
lesiones en estructuras adyacentes, por ejemplo, costillas, columna o pelvis, también pueden producir íleo. Percusión; detectar
matidez (presencia de líquidos) en caso de hemoperitoneo; timpanismo (presencia de aire) si hay dilatación gástrica o desaparición de
la matidez hepática por neumoperitoneo. Palpación; debe repetirse periódicamente, por lo que debe ser realizado por la misma
persona para poder evaluar las diferencias que se originen. Primero, debe dirigirse al plano parietal buscando la presencia de
hematomas, o contusiones musculares. Luego debe investigar la presencia de contractura abdominal refleja, que es un signo fiable de
irritación peritoneal, al igual que el signo del rebote positivo. Finalmente, hay que realizar una palpación más profunda buscando la
presencia de puntos o zonas dolorosa cuya topografía nos oriente a relacionarlas con los posibles órganos lesionados. Evaluación
estabilidad pélvica; La exploración del anillo pelviano debe realizarse mediante una cuidadosa compresión lateral y antero posterior,
siendo dolorosa cuando hay fractura pélvica. Una fractura de pelvis puede ser causa de shock hipovolémico, en ocasiones muy severo.
Produce hematoma perineal y genital a las 24-48 horas del traumatismo y puede acompañarse de hematoma retroperitoneal y
ausencia de hemoperitoneo. Examen del periné y genitales; lesiones externas y la presencia de signos de lesión uretral como sangre en
el meato, hematoma escrotal o desplazamiento hacia arriba de la próstata. La laceración de la vagina puede ocurrir en heridas
penetrantes o por fragmentos óseos de una fractura pélvica. PRUEBAS COMPLEMENTARIAS: Analítica: bioquímica, hemograma,
tiempo de coagulación, pruebas cruzadas, niveles de alcoholemia y análisis de orina. ECG y monitorización de constantes vitales.
Radiografía de abdomen. Radiografía de tórax: es importante para descartar la presencia de hemotórax, neumotórax o fracturas
costales. Radiografía de pelvis. Exámenes complementarios: Lavado peritoneal diagnóstico (LPD); procedimiento invasivo que puede ser
realizado de forma rápida presenta una sensibilidad del 68% y una especificidad del 83%. La indicación principal del LPD son Hallazgos
abdominales equívocos. Exploración física no realizable por traumatismo raquídeo concomitante o alteración de la conciencia
(traumatismo cráneo encefálico o tóxico). Imposibilidad de reevaluación continúa. Hipotensión inexplicable. Pérdida progresiva de
sangre (descenso progresivo del hematocrito). Puede realizarse mediante un método abierto o cerrado. Un LPD negativo no excluye la
presencia de lesiones retroperitoneales o desgarros diafragmáticos. El ultrasonido puede detectar presencia de hemoperitoneo, es un
medio rápido, no invasivo y seguro en el diagnóstico de lesiones intra abdominales (cerrada o penetrante) y puede ser repetido
frecuentemente. Es más sensible que el lavado peritoneal diagnóstico para la determinación de lesiones de vísceras macizas, aunque no
lo es tanto como la TAC. Su indicación es absoluta en casos de embarazo, cicatrices abdominales por cirugías previas y alteración de la
coagulación. La TAC requiere el transporte del paciente a la sala de rayos X, administración oral e intravenosa de contraste. Consume
tiempo y es utilizado únicamente en pacientes hemodinámicamente estables en los que no existe la indicación inmediata de
laparotomía. La TAC proporciona información relativa a la lesión específica de un órgano en particular y también puede diagnosticar
lesiones en el retroperitoneo u órganos pélvicos que son difíciles de evaluar en la exploración física o en el LPD. Laparoscopia la utilidad
de la laparoscopia efectuada bajo anestesia local para identificar lesiones diafragmáticas y cuantificar la cantidad de sangre
intraperitoneal. MANEJO DEL PACIENTE TRAS LA VALORACIÓN INICIAL: 1. Traumatismo Abdominal Cerrado, Hemodinámicamente
inestable: (imposibilidad de mantener una TA sistólica por encima de 90 mm Hg, frecuencia cardiaca inferior a 100 lpm o diuresis de 50
ml/h adulto) Si el paciente presenta signos abdominales patológicos (distensión abdominal, peritonismo, neumoperitoneo en la
radiografía simple), entonces la indicación de laparotomía debe ser inmediata. Si los signos abdominales son dudosos y el paciente
presenta un traumatismo craneoencefálico o espinal severo, alteraciones de la conciencia por toxicidad, traumatismos toraco-
abdominales, debemos llevar a cabo un estudio rápido del abdomen que nos ayude a descartar la presencia de patología abdominal,
fundamentalmente líquido libre. Para ello disponemos de dos pruebas que no son excluyentes: Ecografía abdominal. Lavado
peritoneal diagnóstico. Otras pruebas radiológicas que precisan mayor infraestructura y tiempo, como la TAC, no son posibles en el
paciente inestable. Hemodinámicamente estable. Se deben tener en cuenta las siguientes premisas a la hora de manejar un paciente

CURSO ENARM CMN SIGLO XXI TEL: 36246001 Pharmed Solutions Institute PÁGINA 541
MANUAL DE TRABAJO DEL CURSO ENARM CMN SIGLO XXI
con un traumatismo abdominal cerrado: Las vísceras macizas se lesionan con más frecuencia que las huecas. Traumatismo Abdominal
Abierto; Arma Blanca. - Si el paciente está inestable o presenta signos de irritación peritoneal, debe ser sometido a una laparotomía
urgente. - Si está estable hemodinámicamente y no presenta signos de irritación peritoneal, la primera maniobra que se debe realizar
es la exploración del orificio de entrada del arma, comprobando si la herida es penetrante o no. Arma de Fuego; Puesto que la
trayectoria de una bala es difícil de predecir y dado que el 80-90% de los traumatismos por arma de fuego se asociaran a una o más
lesión visceral, el tratamiento de estos pacientes será quirúrgico, realizándoseles una laparotomía urgente. Recomendaciones basadas
en evidencia, Recomendaciones NIVEL I: Laparotomía exploratoria está indicada para pacientes con LPD (+) TAC está recomendado para
pacientes hemodinámicamente estables con un EF equívoco (lesión neurológica, lesiones extrabdominales). TAC es la modalidad
diagnóstica de elección para manejo no operatorio de lesiones de vísceras sólidas. En pacientes hemodinámicamente estables, LPD y
TAC son modalidades diagnósticas complementarias.

CASO CLINICO
Varón de 28 años, traído a urgencias tras haber tenido una caída en bicicleta y con traumatismo cerrado periumbilical, refiriendo dolor
intenso, se observa distención abdominal, además se observa aire subdiafragmatico en RX, signos vitales TA 100/70, FC 105, FR 28, se
apresia diaforético con palidez de tegumentos.

PREGUNTA
Cual es la conducta a seguir.

RESPUESTA
a.- Lavado peritoneal.
b.- Restitución volumétrica.
c.- Laparatomia exploratoria.
d.- Conducta expectante.

CASO CLINICO
Femenino de 66 años de edad, sana cayó de su propia altura. Su miembro inferior derecho estaba en flexión, rotación externa y
abducción con extensión de la rodilla durante la lesión. La paciente experimenta dolor en la cadera grave y no pudo incorporarse.
Describió la sensación de que su cadera derecha había sido desplazada de su articulación momentáneamente.

PREGUNTA
Cual de las siguientes medidas posoperatorias no es necesaria.

RESPUESTA
a.- Control estricto de liquidos.
b.- Analgesico, antibiótico y enoxaparina.
c.- Realizar laboratorio y vigilar sangrado.
d.- Interconsulta a medicina interna.

RETINOPATIA DIABETICA (RD), RETINOPATIA HIPERTENSIVA:


RETINOPATIA DIABETICA (RD). CIENCIAS BASICAS: Es una microangiopatía, que aparece como complicación crónica de DM se
caracteriza por la presencia de lesiones microvasculares típicas en la retina de una persona con diabetes, microaneuriosmas,
hemorragias, exudados duros, manchas algodonosas, alteraciones microvasculares, arrosariamiento venosos, neovasos y tejido
fibroso. Factores de aparición: Control metabólico pobre HbA1c, embarazo, HAS, nefropatía, alcoholismo es un factores de riesgo,
antigüedad de la DM >5años, dislipidemia, obesidad. SALUD PUBLICA: Principal causa de ceguera legal 20-65 años. Primera causa de
ceguera prevenible. Casi todos los pacientes con DM tipo II tendrán evidencia de RD a los 20 años de evolución y más del 21%
presentan RD al momento del diagnóstico. La RD es la causa de 5% de los casos de ceguera en el mundo. PATOGENIA: Se produce por
una microangiopatia el mecanismo más probable es que la hiperglicemia produce alteraciones del metabolismo intracelular que llevan,
como resultado, a un aumento del Sorbitol. Esto produce el engrosamiento de la membrana basal endotelial y la pérdida de los
Pericitos, los cuales son células que envuelven a los capilares retinales, proporcionándoles soporte y actuando como parte de la Barrera
Hematoretinal. La pérdida de pericitos produciría, a su vez, dos secuencias de eventos paralelas: a) Alteración de la barrera
hematoretinal, filtración al espacio extravascular, edema retinal, exudados lipídicos o céreos formados por lipoproteínas. b) Formación
de microaneurismas por debilidad estructural de la pared de los capilares retinales, activación de la coagulación en los
microaneurismas, trombosis intracapilar, obstrucción y cierre capilar. Lo anterior será responsable de la producción de isquemia retinal,
con el consecuente desarrollo de manchas algodonosas, (que corresponden a infartos de la capa de fibras nerviosas)
neovascularización, hemorragias y, en último término, complicaciones tales como desprendimiento de retina traccional, glaucoma y, en
definitiva, ceguera. El crecimiento de neovasos, tanto a nivel retinal como en el iris, se produciría debido a la liberación por parte de la
retina isquémica de un factor soluble estimulador del crecimiento vascular (Factor de Crecimiento Vascular Endotelial, VEGF) y a su
efecto sinérgico junto a un factor de crecimiento vascular presente en la retina (Factor de Crecimiento de Fibroblastos Básico, bFGF).
CLASIFICACION: En una etapa temprana o Retinopatía Diabética No Proliferativa (RDNP) y una más avanzada o Retinopatía Diabética
Proliferativa (RDP). La RDNP se subdivide a su vez en: 1. Leve (Presencia de al menos un microaneurisma, hemorragia retineana,
exudado duro o exudado blando (mancha algodonosa)), 2. Moderada (Exudados duros, rosarios venosos, manchas algodonosas en
menor grado que en la severa), 3. Severa (Regla 4-2-1, 4 cuadrantes con microaneurismas o hemorragias, 2 cuadrantes con rosarios
venosos, manchas algodonosas, 1 cuadrante con IRMA) y 4. Muy severa (2 criterios). La RDP evoluciona en tres etapas de menor a
mayor severidad: temprana, de alto riesgo y avanzada. Esto está dado por la ubicación y extensión de los neovasos, la presencia o
ausencia de hemorragia vítrea y la presencia o ausencia de desprendimiento de retina con compromiso foveal. La aparición de estos

CURSO ENARM CMN SIGLO XXI TEL: 36246001 Pharmed Solutions Institute PÁGINA 542
MANUAL DE TRABAJO DEL CURSO ENARM CMN SIGLO XXI
neovasos es lo que define a la Retinopatía Diabética Proliferativa. Los neovasos se observan con mayor frecuencia en el nervio óptico o
cercano a las arcadas vasculares, pero se pueden encontrar en cualquier parte del fondo de ojo. La presencia de un desprendimiento de
Retina crónico en un paciente con RDP es un factor de riesgo para la neovascularización iridiana y para el Glaucoma neovascular
secundario, cuyo pronóstico es muy sombrío. El Edema Macular es un evento que puede suceder en cualquier momento de la
progresión de la RD los hallazgos son: minimo (algoi de engrosamiento de retina o exudados duros en el polo posterior de la macula),
moderado (engrosamiento de retina o exudados duros que se acercan a ala macula), severo (engrosamiento de retina o exudados
duros que involucran el centro de la macula). DIAGNOSTICO: Clínica; Microaneurismas (relacionados con zonas de baja perfusión.
Signos: puntos rojos), hemorragias (en forma de flama (arteriolas precapilares), intraretineanas: venas, forma de puntos), edema
macular (fuga capilar extensa), exudados duros (edema crónico localizado, macrófagos con depósitos lipídicos y lipoproteinas PE, por lo
general rodean microaneurismas), exudados algodonosos o blandos (infartos, obstrucción de flujo axoplásmico en la capa de fibras
nerviosas, inflamación). Agudeza visual: Es de importancia saber que para la clasificación de la retinopatía diabética no se toma en
cuenta la agudeza visual, sin embargo, cuando está se ve afectada nos habla principalmente de que la macula está comprometida.
Realizar exploración de fondo de ojo bajo dilatación pupilar por un oftalmólogo en DM tipo I: a partir de los 12 años de edad. A partir
del 5to año de diagnóstico y cada año, cuando el diagnostico sea posterior a la pubertad, debe comenzar cada 3 años. En DM tipo II: al
momento del diagnóstico, realizado por un oftalmólogo y debe repetirse anualmente. La ecografía es una prueba muy útil para los
diabéticos con medios opacos. Esta pruebe debe ser considerada cuando los medios no permiten la observación mediante
oftalmoscopio indirecto. TRATAMIENTO: Educación del Paciente que tome conciencia de su enfermedad, de sus riesgos potenciales, y
que acuda a controles periódicos con su diabetólogo y con su oftalmólogo. Durante esta etapa se debe optimizar el control metabólico
de los pacientes, corregir la hiperlipidemia, lograr un adecuado control de la hipertensión arterial, tratar la nefropatía y hacer controles
oftalmológicos más frecuentes en las mujeres embarazadas, en los casos que corresponda. Fotocoagulación con Láser. Vitrectomía vía
pars plana. Antiangiogénicos. La Panfotocoagulación consiste en hacer aplicaciones de láser térmico sobre la superficie retinal. Estas
quemaduras destruyen la retina en el lugar en que son aplicadas, creando una cicatriz. La racionalidad de este tratamiento se basa en
que, al destruir la retina isquémica, ésta sería incapaz de producir el Factor de Crecimiento Vascular Endotelial, el que sería el
responsable de la formación de los neovasos. La disminución de la producción de este factor soluble lograría la regresión de la
neovascularización existente y la prevención de su desarrollo en el futuro. Este tratamiento no es inocuo. La panfotocoagulación
disminuye en un 50% el riesgo de pérdida visual severa, especialmente en los pacientes con RDP de alto riesgo. El edema macular ya
sea focal o difuso, en Edema Macular Clínicamente Significativo (EMCS) y Edema Macular No Clínicamente Significativo (EMNCS). Esta
definición se basa exclusivamente en aspectos oftalmoscópicos que dicen relación con la cercanía del edema al centro de la fóvea y no
considera la agudeza visual del paciente, es decir, un paciente puede tener visión normal y aun así tener un EMCS, el tratamiento del
EMCS focal con Fotocoagulación Focal disminuye en un 50% el riesgo de pérdida visual moderada en los pacientes tratados versus los
controles, y mejora la agudeza visual en el 16% de los pacientes. El EMCS difuso, hasta hace poco no tenía un tratamiento efectivo.
Actualmente, con el advenimiento de la Triamcinolona intravítrea, como tratamiento de este tipo de edema, se han logrado mejorías
espectaculares en la disminución del edema.

RETINOPATIA HIPERTENSIVA. CIENCIAS BASICAS: La Hipertensión Arterial (HTA), por el hecho de ser una enfermedad sistémica,
compromete órganos como el cerebro, el corazón, el riñón y también el ojo. La retinopatía hipertensiva es la principal manifestación
ocular de la HTA, pero no la única. El compromiso conjuntival, la coroidopatía hipertensiva y la neuropatía hipertensiva son otras
manifestaciones oculares de la HTA. Aunque parece haber una asociación clara entre las alteraciones retinieanas y el aumento de las
cifras tensionales medias, parece que existen otros factores asociados a la retinopatía como arteriosclerosis, edad avanzada,
enfermedad de la arteria carótida. SALUD PUBLICA: La prevalencia en pacientes hipertensos oscila entre 7.8-11%. PATOGENIA: Hay
varios aspectos condicionantes en la retinopatía hipertensiva: 1. La gravedad de la hipertensión, 2. La duración, 3. La rapidez de la
instauración, 4. El estado previo del árbol vascular retiniano. Las manifestaciones oculares de la HTA son producidas por el daño que
esta causa en los vasos arteriales conjuntivales, retinales, coroideos y de la cabeza del nervio óptico. La HTA produce vasoconstricción y
arterioloesclerosis. Vasoconstricción: Los vasos arteriales retinales poseen la capacidad de autoregulación y no están comandados por
el tono simpático. Esta autoregulación permite que en la eventualidad de un aumento de la presión arterial, las arteriolas retinales se
contraigan manteniendo constante el flujo. Por esto en pacientes con HTA Crónica, uno de los hallazgos más tempranos es una
disminución difusa del calibre arteriolar, perdiéndose la relación AV normal de 2:3. La vasoconstricción también puede ser focal. Esta
vasoconstricción se produce por espasmo vascular reversible. En los casos de vasoconstricción focal permanente, la causa podría ser
edema de la pared del vaso o áreas localizadas de fibrosis. Arterioloesclerosis: Cuando observamos el fondo de ojo y vemos los vasos
retinales, lo que en realidad estamos viendo es la columna de sangre en el interior del vaso. La pared arteriolar es, en condiciones
normales, transparente. En la arterioloesclerosis se produce inicialmente un aumento del tejido muscular de la media y un aumento del
tejido elástico de la íntima, para luego ser reemplazada por fibras colágenas, la media, y sufrir engrosamiento hialino, la íntima. Esto
ocasiona, en un primer momento, que el brillo arteriolar que produce la luz incidente del oftalmoscopio en la pared del vaso, pierda su
aspecto normal (delgado y en la mitad de la columna de sangre), tornándose menos brillante, más ancho y más difuso. Este es el signo
más temprano de arterioloesclerosis. En una segunda etapa, al aumentar la arterioloesclerosis, disminuye la visibilidad de la columna
de sangre y el brillo arteriolar se torna café-rojizo. Es lo que los clínicos antiguos llamaban vasos en "Hilo de Cobre". En una tercera
etapa, la pared del vaso pierde por completo su transparencia, perdiéndose la visibilidad de la columna de sangre y adquiriendo el vaso
un aspecto blanquecino. Esto se denominaba vasos en "Hilo de Plata". Aunque clínicamente estos vasos parecieran estar sin flujo. En la
Hipertensión Maligna se produce necrosis fibrinoide de las arteriolas retinales, aun cuando la necrosis fibrinoide es más común en los
vasos coroideos. En estos casos se puede apreciar una estrella macular, que es el aspecto en estrella que adquiere la mácula por la
acumulación de macrófagos llenos de lípidos, dispuestos en forma radial, en la capa plexiforme externa de la retina. DIAGNOSTICO:
Manifestaciones oftalmoscopicas: En la hipertensión crónica se produce esclerosis arteriolar por el efecto mantenido de la presión
sanguínea aumentada sobre las paredes vasculares. Se genera un estrechamiento generalizado o localizado, se ha descrito como
arterias en hilo de cobre, arterias en hilo de plata, aumento de la tortuosidad de las arteriolas esclerosadas. Se produce oclusión de
arteriolas finas y generación de manchas isquémicas retinianas profundas denominadas infiltrados algodonosos. En la hipertensión
aguda puede verse una imagen denominada espasmo arteriolar. Trasudados periarteriolares focales intrarretinianos (TPIF): Son muy

CURSO ENARM CMN SIGLO XXI TEL: 36246001 Pharmed Solutions Institute PÁGINA 543
MANUAL DE TRABAJO DEL CURSO ENARM CMN SIGLO XXI
precoces y específicos de la hipertensión maligna. Tienen forma redondeada u oval y el tamaño es variable: desde puntiformes hasta la
mitad del tamaño del disco óptico. Son de color blanco mate cuando son recientes y luego palidecen hasta resolverse. Se localizan en
las zonas profundas de la retina. Las lesiones se desarrollan totalmente en 2-3 semanas y tras su resolución no quedan secuelas
oftalmoscópicas. Manchas isquémicas retinianas profundas (infiltrados algodonosos o exudados blandos): Son áreas focales,
esponjosas, opacas. Tienen formas muy irregulares, comienzan como una película grisácea que se vuelve como una nube blanca y se
resuelve fragmentándose en manchas menores. Cambios capilares retinianos: microaneurimas y shunts arteriovenosos. Se observan
como vasos retorcidos y colaterales venosos. Cambios retinianos venosos: Signos de Gunn: la vena tiene dos zonas pálidas, iguales a
ambos lados del cruce, con pérdida del reflejo y adelgazamiento de los dos extremos. La vena se ve como partida. Signo de Salus: el
curso venoso sufre un cambio de trayecto. Aspecto de U, si el cruce es perpendicular y de S, si es oblicuo. Signo de Bonnet: junto al
cruce se observa una hemorragia o exudado. Hemorragias retinianas: tardía y poco frecuente, adoptan forma de llama. Edema retinal y
macular: los signos precoces de papiledema son rebosamiento de las venas, perdida de la pulsación venosa, hiperemia de la cabeza del
nervio y borrado de los márgenes del disco. Depósitos lipídicos retinianos (exudados duros): son amarillos, redondeados generalmente
y de bordes bien delimitadosLa aguda: se caracteriza por disminución de la agudeza visual, la que puede ser desde visión borrosa (lo
más frecuente) hasta la ceguera. Característicamente esta disminución de agudeza visual es transitoria. El aspecto más característico
del fondo de ojo es una Vasoconstricción Generalizada, se pueden apreciar Hemorragias Retinales, Manchas Algodonosas, Estrella
Macular, Manchas de Elschnig y Edema de Papila. Con mucha menor frecuencia se aprecian Desprendimientos de Retina Serosos. La
crónica: los hallazgos más característicos en el fondo de ojo son: Vasoconstricción difusa, Aumento del Brillo Arteriolar, Cruces AV
Patológicos, Microaneurismas y Macroaneurismas. En casos más graves pueden apreciarse: Manchas Algodonosas, Edema de Retina,
Hemorragias Retinales, Estrella Macular, Manchas de Elschnig y Edema de Papila. Como podemos ver, en las manifestaciones clínicas de
casos severos se aprecian signos de Coroidopatía Hipertensiva y Neuropatía Hipertensiva, además de los de la Retinopatía Hipertensiva.
COROIDOPATÍA HIPERTENSIVA: La coroides, a diferencia de los vasos retinales, está comandada por el tono simpático. En la HTA la
coroides sufre fenómenos de isquemia, observándose lóbulos coroideos sin perfusión debido a necrosis fibrinoide de los vasos. Estas
zonas de ausencia de perfusión coroidea producen focos de necrosis isquémica en el epitelio pigmentario retinal suprayacente, que se
denominan Manchas de Elschnig. La isquemia coroidea también es responsable de desprendimientos de retina serosos por producir
isquemia en las capas externas de la retina. Las manchas de Elschnig, en su etapa aguda, se aprecian como manchas amarillentas
redondeadas en el epitelio pigmentario retinal. Posteriormente, en su etapa crónica, se aprecian como lesiones redondeadas
hiperpigmentadas en su centro y con un halo de hipopigmentación en su periferia. NEUROPATÍA HIPERTENSIVA: Parece que es una
edematizacion secundaria a isquemia, seguida de perdida de axones y gliosis. La isquemia puede tener dos orígenes: 1) afección de los
vasos coroideos, ya que constituye el principal aporte sanguíneo de la cabeza del nervio. 2) difusión de sustancias vasoconstrictoras a la
cabeza del nervio óptico a partir de la coroides peripapilar, lo que produce vasoconstricción de los vasos dentro de la cabeza del nervio
óptico. CLASIFICACION: Aguda se observa principalmente en pacientes con preeclampsia o eclampsia, pero también es causada por
feocromocitoma o Hipertensión Maligna. Crónica, se observa en pacientes con hipertensión de larga duracion y, a diferencia de
aquellos que padecen la forma aguda, no presentan disminución de la agudeza visual. Clasificación de Keith, Wagener y Barker: Grado I
Hipertensión benigna. Estenosis o esclerosis moderada de las arteriolas retinianas. No síntomas. Buen estado de salud. Grado II
Retinopatía hipertensiva más marcada. Esclerosis moderada o marcada de las arteriolas retinianas. Reflejo luminoso arterial exagerado.
Compresión venosa en los cruces arteriovenosos. Tensión arterial mayor y más mantenida que en el grupo I. Asintomáticos. Buen
estado de salud. Grado III Retinopatía angiospástica moderada. Edema retiniano. Manchas algodonosas y hemorragias. Esclerosis y
lesiones espásticas de las arteriolas retinales. Tensión arterial generalmente alta y mantenida (sintomática). Grado IV Hipertensión
maligna. Todo lo anterior y edema del disco óptico. Las funciones renal y cardiaca pueden estar afectadas. Supervivencia reducida.
Clasificación de Scheie: Grado I Aumento de reflejo luminoso arteriolar con compresión arteriovenosa mínima. Grado II Aumento de los
signos y desvío de las venas en los cruces arteriovenosos. Grado III Arteriolas en hilo de cobre y marcada compresión arteriovenosa.
Grado IV Arteriolas en hilo de plata y cambios importantes en los cruces arteriovenosos. TRATAMIENTO: No existe un tratamiento
oftalmológico específico para la retinopatía hipertensiva hay que dar tratamiento de la enfermedad de base. Tiene más urgencia el
tratamiento de la hipertensión arterial en la Retinopatía Hipertensiva Aguda que en la Crónica, pero en ambas es igual de importante.

CASO CLINICO
Un hombre de 31 años de edad, con antecedentes de diabetes mellitus insulino-dependiente de 12 años de evolución con parcial apego
a tratamiento, con alcoholismo ocacional y tabaquismo un paquete al dia, se presenta para su examen oftálmico. Historia oftalmológica
fue notable por desprendimiento total de retina de su ojo derecho y la fotocoagulación con láser de su ojo izquierdo debido a la
retinopatía diabética y edema macular diabético. Examen de dilatación del fondo del ojo derecho mostró etapa final retinopatía
diabética proliferativa con desprendimiento de retina traccional. El ojo izquierdo tenía la neovascularización extensiva y la proliferación
fibrovascular de la retina secundaria a la retinopatía diabética proliferativa avanzada. Estos nuevos vasos anormales mostraron fuga
extensa durante la angiografía con fluoresceína.

PREGUNTA
Cual de los siguientes factores es de mayor importancia para el desarrollo de la retinopatía diabética.

RESPUESTA
a.- Más de 5 años de evolución de la DM.
b.- Tabaquismo positivo intenso.
c.- Progresión rápida de la enfermedad.
d.- Depuración de creatinina menos de 60 ml/ minuto.

CASO CLINICO
Un hombre de 56 años de edad con diabetes presenta una historia de varios meses de disminución de la visión en el ojo izquierdo. El
examen del ojo izquierdo reveló la neovascularización retiniana extensa superior a la del disco óptico.

CURSO ENARM CMN SIGLO XXI TEL: 36246001 Pharmed Solutions Institute PÁGINA 544
MANUAL DE TRABAJO DEL CURSO ENARM CMN SIGLO XXI
PREGUNTA
Cual de los siguientes metas son factores que no modifican el curso de la enfermedad.

RESPUESTA
a.- Glucosa menor a 130 mg/dl
b.- Hb glucosilada menor a 7.
c.- Presión arterial menor a 130/80 mmHg.
d.- Uso profiláctico de triamcinolona.

CASO CLINICO
Femenino de 42 años de edad, con hipertensión arterial de 10 años de evolución actualmente con disminución en el aclaramiento de
creatinina (<60 ml/min), con un evento de isquemia cerebral transitoria, presentando alteraciones del ritmo cardiaco bajo tratamiento,
su nivel de glucosa fue de 210 mg/dl, Colesterol 324 mg, triglicéridos 450 mg, hemoglobina glucosilada de 9.2, acudió a consulta debido
a que presenta diminución de la agudeza visual, asi como alteraciones en la campimetrial, fue enviada a evaluación oftalmológica con el
siguiente reporte. Estrechamiento y constricción focal, edema retiniano, exudados y hemorragias.

PREGUNTA
Considerando el reporte cual es el estadio de Keith-Wagener.

RESPUESTA
a.- Keith-Wagener 1.
b.- Keith-Wagener 2.
c.- Keith-Wagener 3.
d.- Keith-Wagener 4.

URGENCIAS OFTALMOLOGICAS, PERFORACION, GLAUCOMA:


CIENCIAS BASICAS: Hay que distinguir dos grandes categorías de afecciones, ante un ojo rojo, el análisis semiológico debe pasar por el
examen de tres signos mayores: Dolor, fotofobia y agudeza visual. Normas generales: 1. No utilizar colirios anestésicos, salvo para
facilitar la exploración. 2. No presionar el globo ocular traumatizado. 3. Tras el diagnóstico de perforación ocular, abstenerse de
continuar con la exploración; ocluir ambos ojos con gasa estéril sin instilar colirios ni aplicar pomadas. 4. Para la exploración corneal es
de gran ayuda la instilación de colirio de Fluoresceína al 2% y posterior lavado con suero fisiológico, ya que con la luz azul cobalto
evidenciará lesiones y defectos epiteliales dando una coloración amarillo-verdosa. 5. Para una midriasis exploratoria (con colirio de
Tropicamida) o terapéutica en pacientes de edad y, sobre todo hipermétropes, se debe explorar previamente el signo "del sol
naciente". Este signo consiste en la proyección tangencial desde el lado temporal de un foco luminoso que iluminará la mayor parte del
iris ante una cámara anterior profunda. 6. Utilizar guantes desechables ante procesos traumáticos o infecciosos y, en todo paciente al
que se le vayan a tocar los párpados. 7. No realizar oclusión ocular en conjuntivitis o queratoconjuntivitis, ni en úlceras corneales con
infección local. Hay que señalar cinco procesos oculares que son de asistencia inmediata, ya que el pronóstico en cuanto a la función
visual y/o Integridad del globo ocular, dependerá de la rapidez en el diagnóstico y tratamiento: Oclusión aguda de la arteria central de
la retina. Causticacion ocular por alcalis. Herida perforante del globo ocular. Glaucoma agudo. Endoftalmitis. OJO ROJO NO DOLOROSO:
1. Hiposfagma (Hemorragia subconjuntival): Habitualmente son espontáneas, de tamaño variable y curan sin tratamiento. Si es
secundario a traumatismo ocular, buscar signos de perforación, descartarse siempre crisis de hipertensión arterial. 2. CONJUNTIVITIS:
Bacteriana; mas frecuente bilateral. Existe secreción purulenta o mucopurulenta y lagrimeo, tx., aplicar colirio de Tobramicina cada 3
horas o aureomicina cada 2 horas, dura aprox 2 sem. Si es unilateral, purulenta, con dolor y adenopatía preauricular, debe pensarse en
una conjuntivitis gonocócica y se tratará con colirio de Aureomicina cada 2 horas y Penicilina por vía parenteral. Si es unilateral, con
poca secreción, adenopatía preauricular, presencia de folículos en conjuntiva tarsal, es decir, con características más propias de una
conjuntivitis vírica, debe sospecharse conjuntivitis de inclusión (Chlamydia oculogenitalis). Vírica: unilateral o bilateral en pocos días,
poca secreción, lagrimeo y presencia de folículos en las conjuntivas tarsales. Habitualmente, muestra adenopatías preauriculares y
submaxilares, cursará con fotofobia y dolor (queratoconjuntivitis). Etiologia los adenovirus, no existe tratamiento etiológico eficaz, pero
es aconsejable el colirio de Aureomicina cada 2 horas y la pomada de Aureomicina por la noche. Alérgica: Cursa con secreción
blanquecina fibrinosa (estéril), abundante lagrimeo, presencia de papilas en la conjuntiva tarsal superior y, sobre todo, predomina el
picor como síntoma fundamental y característico de esta afección. Es bilateral y, excepcionalmente, unilateral. Se trata con colirio de
Dexametasona cada 6-8 horas y antihistamínicos. OJO ROJO DOLOROSO: 1. EPISCLERITIS: Inflamación de la episclera, enrojecimiento
violáceo, localizado en un sector de la conjuntiva bulbar, por lo general con un nódulo amarillento en su centro. Realmente, la
dilatación vascular afecta tanto a la episclera como a la conjuntiva. La instilación de colirio de Fenilefrina no modifica esta congestión
profunda, sino que blanquea sólo una hiperemia puramente conjuntival (diagnóstico diferencial). Afecta a sujetos jóvenes de forma
recidivante. Cursa con dolor a los movimientos oculares y, sobre todo, a la presión. Con frecuencia es de origen alérgico. A veces forma
parte de enfermedades granulomatosas y reumáticas. Rara vez es signo de tumor intraocular. Tratamiento: Colirio de Dexametasona
cada 4 horas. Si no es suficiente, se recurrirá a la corticoterapia sistémica. 2. QUERATITIS.- La infección o inflamación de la córnea cursa
con dolor (que será más intenso cuanto mayor sea la denudación de las terminaciones nerviosas), fotofobia, lagrimeo, blefarospasmo,
inyección periquerática, deslustramiento de la córnea y alteración dispar de la agudeza visual. A veces existe hipopion (nivel de pus en
la cámara anterior), que traduce una reacción uveal anterior. Tratamiento: Ciclopléjico: Colirio de Atropina al 1% cada 8 horas.
Antibiótico: Tópico: Cada hora. Gentamicina: 9.1 mg/ml. Ceftazidima: 50 mg/ml. Subconjuntival: Cada 12-24 horas. Gentamicina: 20 mg
(0.5 ml.). 3. IRITIS O IRIDOCICLITIS (UVEITIS ANTERIOR): Se trata de una inflamación del tracto uveal anterior (iris y cuerpo ciliar). Sus
síntomas son: Dolor ocular y periocular (irradiaciones en el territorio del trigémino), lagrimeo, fotofobia y disminución de la agudeza
visual (borrosidad). Signos: Inyección periquerática, precipitados en la cara posterior de la córnea, enturbiamiento del humor acuoso

CURSO ENARM CMN SIGLO XXI TEL: 36246001 Pharmed Solutions Institute PÁGINA 545
MANUAL DE TRABAJO DEL CURSO ENARM CMN SIGLO XXI
(efecto Tyndall), iris deslustrado, pupila oco reactiva a la luz y presencia de sinequias posteriores. La tensión ocular por regla general
está disminuida, medida terapéutica urgente encaminada a mitigar el dolor y evitar o liberar las sinequias posteriores, debemos
administrar un colirio ciclopléjico cada 6-8 horas y un colirio de Dexametasona cada 2 horas. 4. GLAUCOMA AGUDO: Cierre total y
súbito del ángulo camerular, que ocasiona una elevación severa de la presión intraocular. Sólo ocurre en ojos a menudo hipermétropes
y nunca miopes, de córnea pequeña, de cámara anterior estrecha y de ángulo estrecho. Estas condiciones anatómicas y el aumento del
volumen del cristalino a lo largo de la vida, son responsables del bloqueo pupilar. Es más frecuente en mujeres (4: 1) y ocurre,
generalmente, a partir de los 65 años. Pueden desencadenar la crisis las emociones, la anestesia general, los derivados atropínicos, los
simpaticomiméticos (jarabes para la tos), los antidepresivos, etc. La forma típica; unilateral, aunque el ojo congénere presentará las
mismas condiciones de predisposición. Tener dolor ocular intenso, similar a una cefalea retrobulbar, acompañado de náuseas y
vómitos. Visión borrosa y percepción de halos de colores en el inicio. Posteriormente, la visión se reduce a percepción de luz. Al
examen con la lámpara de hendidura: Inyección ciliar y conjuntival (gran congestión). Deslustramiento corneal por el edema. Cámara
anterior muy aplanada, casi inexistente. Pupila en midriasis media, oval, vertical, arreactiva. La tensión ocular está muy elevada, por
encima de 50 mmHg. ("dureza pétrea"). Tratamiento,inhibidores de la producción del humor acuoso y agentes hiperosmóticos. Colirio
de Pilocarpina al 2%+ Colirio de Dexametasona cada 5 minutos durante la primera media hora y cada 10 minutos durante la hora
siguiente para ir espaciando las instilaciones paulatinamente. Acetazolamida, 500 mg. por vía IM. o IV. La vía IV. debe manejarse con
cautela debido al contenido en Lidocaína que lleva el preparado. Manitol al 20%: 250 cc. por vía IV. (Aproximadamente 90 gotas por
minuto). El tratamiento se comenzará al unísono con todos los fármacos y, pasadas 3-4 horas del inicio del mismo, se procederá a
realizar una IRIDOTOMIA PERIFERICA LASER, aunque no se haya resuelto el bloqueo pupilar. Si el edema corneal no permite visualizar
bien el iris, presionaremos en la córnea central repetidas veces, a razón de 30 seg. cada una de ellas, para conseguir entreabrir el
ángulo camerular y facilitar el drenaje del humor acuoso, con lo que aumentará la transparencia corneal. TRAUMATISMOS OCULARES:
LESIONES NO PENETRANTES: ABRASIÓN CORNEAL: La instilación de un colirio anestésico nos facilitará la apertura palpebral. El
tratamiento se hace con colirio de Tobramicina cada 4 horas, colirio ciclopléjico cada 8 horas y oclusión semicompresivo. Si observamos
erosiones superficiales lineales o geográficas en la mitad superior de la córnea, se procede a evertir el párpado superior y retirar el
cuerpo extraño que se aloja en la conjuntiva tarsal. Ante un cuerpo extraño enclavado en la córnea, se procede a su extracción y a la
limpieza de óxido si lo tuviera, con una aguja hipodérmica o IM. con poco bisel, siempre previa anestesia tópica y utilizando la lámpara
de hendidura. QUEMADURAS FÍSICAS: Pueden ser ocasionadas por un cigarrillo. En este caso, la zona corneal afecta se blanquea al
instante. El tratamiento es el mismo que el de la abrasión corneal. También pueden ser causadas por radiaciones ultravioletas
(queratoconjuntivitis actínica): Afectación punteada superficial por estallido de las células epiteliales tras la exposición a los rayos UV.
(arco voltaico, radiación solar, aparatos UV.,etc.). QUEMADURAS QUÍMICAS: Son muy graves, en especial las quemaduras producidas
por álcalis y constituyen una causa frecuente de ceguera. Las quemaduras por ácido ejercen su efecto nocivo durante las primeras
horas, precipitan rápidamente las proteinas tisulares levantando barreras contra su propia penetración. Secundariamente, la
neutralización por las proteinas del tejido circundante, tiende a localizar la lesión en el área de contacto. Los álcalis se combinan con los
lípidos de las membranas celulares, produciendo así la total desestructuración de las células con reblandecimiento del tejido.
CONTUSIONES OCULARES: Directas; Producidas por balonazos, pelotazos, puñetazos, etc. Son lesiones conjuntivales el Hiposfagma (H.
subconjuntival). Una lesión corneal es la Queratitis traumática por afectación parenquimatosa, con edema (deslustramiento) y pliegues
o estrías en la Descemet. Se tratará con colirio de Dexametasona cada 6 horas y colirio ciclopléjico cada 8 horas. Las lesiones del iris y
del ángulo irido-corneal, implican con frecuencia hiphemas. Los Hiphemas consisten en la presencia de sangre fresca en la cámara
anterior. Pueden condicionar aumento de presión intraocular. Si es total y coexiste con hipertensión ocular, puede aparecer una tinción
hemática de la córnea. Esta situación exige la valoración urgente por el oftalmólogo. Es más peligroso el resangrado al tercer- quinto
día. El tratamiento consiste en el reposo en posición de antitrendelemburg, oclusión binocular, colirio ciclopléjico cada 8 horas, colirio
de Dexametasona cada 6 horas e hipotensores oculares si está aumentada la presión intraocular. La prednisona oral (1 mg/Kg/día),
evita el resangrado. En cuanto a las modificaciones pupilares, podemos encontrar irregularidades del contorno por ruptura del esfínter.
Sin embargo, hallaremos más frecuentemente midriasis, nunca máxima, oval-oblicua que no responde a la luz, pero sí a la
convergencia. La miosis es rara. La iritis e iridociclitis, ocurren por ruptura de la barrera hematoacuosa. Aunque mínima, siempre existe
en cualquier contusión. Se tratará con colirio de Dexametasona cada 6 horas y colirio ciclopléjico cada 8 horas. También pueden
producirse modificaciones de la presión intraocular. Se puede encontrar tanto hipertensión como hipotensión, en relación con las
diversas afectaciones. Debe sospecharse subluxación o luxación cristaliniana por la iridodonesis (iris trémulo) y por la variación de
profundidad de la cámara anterior. La luxación del cristalino a cámara anterior exige cirugía urgente. En cuanto a la posible aparición de
catarata, inicialmente puede apreciarse alguna opacidad cristaliniana, aunque tardará habitualmente días en formarse. El edema
retiniano ocasiona una importante disminución de la agudeza visual, sobre todo si afecta a la mácula. Al examen del fondo de ojo, se
aprecia un aspecto blanquecino- lechoso de la retina. Se trata con colirio ciclopléjico cada 8 horas y reposo absoluto con oclusión
binocular. HEMORRAGIAS RETINOVÍTREAS: La afectación de la agudeza visual depende de la extensión y localización de éstas. Se
tratará con colirio ciclopléjico cada 8 horas y reposo absoluto. Si la hemorragia vítrea es total, interesa realizar ecografía para
determinar si coexiste un desprendimiento de retina. Los agujeros y desgarros retinianos se producen más frecuentemente en ojos
miopes. Se tratan con laserterapia o cirugía. Cuando se observa un desprendimiento de retina, existe amputación de campo visual.
Requiere ingreso del paciente, con reposo absoluto, oclusión binocular y aplicación de colirio de Atropina al 1% cada 8 horas. Se
realizará intervención quirúrgica programada. Las rupturas coroideas son concéntricas a la papila. No requieren tratamiento. Sin
embargo, si es posterior, debemos atender a los siguientes signos: Pérdida brusca de la agudeza visual inmediatamente después del
traumatismo, marcada hipotonía ocular, aumento acusado de la profundidad de la cámara anterior con iridodonesis, hemorragia vítrea
masiva y hematoma subconjuntival de aspecto gelatinoso. Requiere cirugía urgente. Indirectas: Cuando el traumatismo es de vecindad
o a distancia (traumatismo craneal, aplastamientos torácicos, explosiones, fractura de huesos largos...), puede condicionar hemorragia
y exudados en el segmento posterior (angiopatía traumática de la retina y embolia grasa de la retina). LESIONES PENETRANTES: En las
heridas palpebrales, debe comprobarse si son transfixivas. No existe duda cuando se observa contenido intraocular prolapsado, pero en
el resto de los casos se deben buscar signos indirectos de perforación. Estos signos son los siguientes: 1. Pérdida de agudeza visual. 2.
Hipotonía ocular. 3. Hundimiento o aplanamiento de la cámara anterior. 4. Quemosis marcada5. Laceración corneal o escleral. Para
comprobar si existe disrupción total, se procederá a instilar una gota de colirio de fluoresceína y, con la luz azul cobalto,

CURSO ENARM CMN SIGLO XXI TEL: 36246001 Pharmed Solutions Institute PÁGINA 546
MANUAL DE TRABAJO DEL CURSO ENARM CMN SIGLO XXI
comprobaremos si la fluoresceína es apartada por la salida del humor acuoso, bien espontáneamente o a la presión en casos de herida
valvulada. Este signo se denomina SEIDEL. 6. Hiposfagma: A veces uncuerpo extraño se impacta en el globo ocular atravesando la
esclerótica y quedando la puerta de entrada enmascarada por una hemorragia subconjuntival. Nos cercioraremos con el signo de
Seidel. Siempre que se asista a una herida penetrante en órbita, se debe recurrir a exploraciones radiológicas: Radiografía simple
frontal y lateral de órbita, así como TAC, que nos descartarán o confirmarán la existencia de cuerpo extraño. El tratamiento se basa en
los siguientes puntos: Siempre exige asistencia especializada. Se evitarán las manipulaciones innecesarias. Abstenerse de medicación
tópica, debiendo estar ambos ojos ocluidos con no compresivo. Administración de analgésicos. Profilaxis antitetánica. Profilaxis
antibiótica por vía IV: Gentamicina, 80 mg. cada 8 horas y Ceftazidima, 1 g. cada 8 horas. Reparación quirúrgica bajo anestesia general

CASO CLINICO
Varón de 83 años con enfermedad de Alzheimer avanzada, diabético insulinodependiente de 35 años de evolución, que acudió al
servicio de urgencias por presentar cuadro de dolor ocular en ojo derecho de 2 días de evolución. Se envio al servicio de oftalmología
que refiere que el segmento anterior presenta lesión ulcerativa corneal de 2,5mm de diámetro con un poro endotelial perforante
central de aproximadamente 1mm, sin proceso infeccioso o inflamatorio concomitante. El paciente no presentaba antecedentes
traumáticos.

PREGUNTA
Cuál es la conducta a seguir en esta paciente.

RESPUESTA
a.- Cubrir el ojo.
b.- Colgajo pediculado, tipo Gundersen.
c.- Autoplastia libre conjuntival.
d.- Queratoplastia penetrante.

CASO CLINICO
Paciente de 42 años que acude a Urgencias por cuadro de disminución de agudeza visual (AV) de un día de evolución. Como
antecedentes personales destaca trastorno depresivo en tratamiento. No presentaba antecedentes oftalmológicos de interés y negaba
la utilización previa de gafas. Su tratamiento consiste en olanzapina 10mg, lorazepam 2mg y venlafaxina 150mg retard desde hacía 12
meses. Una semana antes había comenzado a tomar topiramato para perder peso. Al examen se presenta edema corneal y quemosis
conjuntival, una cámara anterior estrecha y cierre angular evidenciado mediante gonioscopia. La presión intraocular (PIO) es de
55mmHg en AO.

PREGUNTA
Cuál es la conducta a seguir en esta paciente.

RESPUESTA
a.- Maleato de timolol.
b.- Manitol 3 g.
c.- Pilocarpina al 3 %.
d.- Acetazolamida 10 mg.

CASO CLINICO
Mujer de 34 años de edad que inició de forma progresiva durante dos semanas visión doble binocular, ptosis palpebral de ambos
párpados superiores y cefalea. En el examen neurológico, se objetivó una parálisis bilateral del nervio motor ocular común, con
afectación pupilar en el ojo derecho; el resto del examen neurológico fue normal. El estudio mediante resonancia magnética (RM)
mostró la presencia de múltiples focos en la sustancia blanca de hiperseñal en las secuencias basadas en T2, compatibles con lesiones
de origen desmielinizante; una de estas lesiones afectaba a la línea media del tegmento del mesencéfalo, y se extendía anteriormente a
la fosa interpeduncular y posteriormente hasta el acueducto de Silvio. Esta lesión, que presentaba realce en las secuencias T1 tras la
administración intravenosa de gadolinio, era probablemente la responsable de los síntomas, ya que afectaba a la localización de ambos
núcleos del III par craneal.

PREGUNTA
Cuál es la conducta a seguir en esta paciente.

RESPUESTA
a.- Acetato de glatirámero.
b.- Interferón β (1a o 1b).
c.- Azatioprina.
d.- Natalizumab.

CURSO ENARM CMN SIGLO XXI TEL: 36246001 Pharmed Solutions Institute PÁGINA 547
MANUAL DE TRABAJO DEL CURSO ENARM CMN SIGLO XXI

PIE DIABETICO:
CIENCIAS BASICAS: Es una complicación crónica de la DM, con etiología multifactorial, teniendo como factores principales la infección,
isquemia, neuropatía y mal control metabólico. SALUD PUBLICA: La incidencia y prevalencia de la úlcera en gente con DM varía de 2,4 a
2,6 y de 4 a 10%, respectivamente.68 Aproximadamente 15% de todos los pacientes con DM desarrollará una úlcera en el pie o en la
pierna durante el transcurso de su enfermedad. PATOGENIA: Las dos lesiones básicas del PD son la neuropatí a (pie indoloro) y las
angiopapatía (pie sin pulso). Estas son origen de las complicaciones isquémicas e infecciosas. La hiperglucemia crónica es el factor de
riesgo más importante para el desarrollo de la neuropatía. Existen dos teorías etiopatogénicas: una metabólica y otra vascular, aunque
probablemente sea la interacción de ambas lo que la produzca. La polineuropatía simétrica distal de distribución en calcetín es la forma
de neuropatía más frecuente que predispone a la aparición de úlceras en el pie. Inicialmente se alteran las fibras nerviosas pequeñas
(fibras sensitivas) provocando la pérdida de la sensación de dolor y temperatura y, más tarde, las fibras nerviosas grandes,
disminuyendo el umbral de percepción de vibración. La neuropatía motora produce hipotrofia muscular apareciendo dedos en martillo
y en garra que favorecen la protrusión de las cabezas de los metatarsianos. La neuropatía autonómica produce sequedad y fisuras en la
piel por afectación de las fibras simpáticas posganglionares de las glándulas sudoríparas, aumento del flujo sanguíneo por apertura de
comunicaciones arteriovenosas que disminuyen la perfusión de la red
CLASIFICACIÓN DE WAGNER.
capilar y aumentan la temperatura de la piel, lo que da lugar a un 0 Ninguna. Pie de riesgo Callos, hallux valgus,
trastorno postural en la regulación del flujo con una respuesta dedos en garra
inflamatoria anormal frente a la agresión a los tejidos. También hay I Úlceras superficiales Destrucción del espesor total de la piel
edema neuropático y aumento de la presión venosa. La osteoartropatía II Úlcera profunda Penetra piel, grasa y ligamentos sin
neuropática o PIE DE CHARCOT es una de las peores consecuencias de la afectar hueso. Infectada
II Úlcera profunda más (osteomielitis)
DM en el pie. Puede encontrarse en una fase aguda que puede confundir absceso Profunda, secreción, mal olor
con infección o en una fase deformante crónica y progresiva caracterizada IV Gangrena limitada Necrosis de una parte del pie
por la destrucción indolora de huesos y articulaciones. En esta situación, V Gangrena extensa Todo el pie afectado, efectos
con los traumatismos se produce distensión ligamentosa y microfracturas, sistémicos
y con el peso corporal ocurre una destrucción articular progresiva que da lugar a fracturas y subluxaciones que son acelerados por el
aumento de la reabsorción ósea osteoclástica, secundaria a la hiperemia causada por la denervación simpática de la microcirculación.
Por estas razones, el pie debe estar inmovilizado con férulas, muleta o silla de ruedas. La neuropatía, con o sin isquemia asociada, está
implicada en la fisiopatología de la úlcera del PD en el 85% a 90% de los casos. Por tanto, el pie insensible, sea bien perfundido o
isquémico, debe considerarse como de alto riesgo de úlcera. DIAGNOSTICO: Tiene características clínicas frecuentes como úlceras,
gangrenas y amputaciones en las extremidades inferiores, ocasionando en el paciente discapacidad parcial o definitiva. Los pacientes
con este problema de salud deben ingresar a un programa de PD que debe ser permanente, estratégico y multidisciplinario incluyendo
la prevención y el tratamiento del PD. Infección: Es frecuenteencontrar manifestaciones de calor, rubor o eritema, edema, dolor, olor
fétido y secreción purulenta. También puede manifestarse con necrosis de los bordes de la herida, aumento del flujo y tejido
desvitalizado. Es multifactorial: La neuropatía y la consiguiente disminución de la sensibilidad predispone a las úlceras a igual que la
disminución del reflejo vasomotor que lleva a disminución de sudoración y a sequedad de la piel y a la aparición de grietas. La micosis
interdigital produce fisuras. La macroangiopatía lleva a insuficiencia arterial con retardo en la cicatrización y deficiente llegada de
antibióticos. El desarrollo de derivaciones arteriovenosas de capilares nutricios también predispone a lesiones de los tejidos. Traumas.
La infección superficial aguda (úlceras no complicadas, celulitis): adquiridas en la comunidad y sin tratamiento antibacteriano, en su
mayoría son monomicrobianas, Staphylococcus y Streptococcus spp. No siempre es necesario el uso de antibióticos ni de la toma de
cultivos, especialmente si es superficial. Las Infecciones profundas y/o crónicas: son polimicrobianas en más de 50% de los casos,
encontrándose dos a tres gérmenes. Hay que tomar cultivo de tejido profundo. A las cocáceas grampositivas de las superficiales se
agregan bacilos gramnegativos y anaerobios.La lesión de fibras sensoriales lleva a disminución de liberación de histamina, con la
consiguiente disminución de signos inflamatorios y de la respuesta inflamatoria. La respuesta inflamatoria es fundamental en los
procesos de defensa y de la cicatrización. TRATAMIENTO: Compromiso del estado general: Indicar si el manejo es ambulatorio o de
hospitalización. Paciente con herida Wagner 3 o grado mayor por lo general necesita hospitalización. Control metabólico: La
hiperglicemia se controla principalmente con insulina, 30 U/d en promedio, se regula según su demanda y se usa sin o con
antidiabéticos orales, principalmente metformina, 1 700 mg/d. En estos pacientes, para el manejo de la dislipidemia, se ha obtenido
buenos resultados con atorvastatina, 40 mg/d. Si hay
hipertensión, generalmente se controla con losartán
o enalapril solos o asociados con hidroclorotiazida, y
si hay edemas se recomienda furosemida. Los
pacientes obesos logran bajar de peso con
educación, dieta de 1 200 kcal y ejercicios
moderados de al menos 30 min/d. Con estas medidas
también mejoran los problemas metabólicos
descritos anteriormente. Además del cambio de
estilo de vida permanente, es de ayuda temporal el
uso de orlistat, 120 mg, tres veces al día. Cuidados de
la úlcera: Desbridamiento quirúrgico, con bisturí
(Figura 1) y, en algunos casos, debridamientos
enzimático y biológico (larvas). El uso de factores de
crecimiento y de injertos han logrado la cicatrización
en un menor tiempo y se ha evitado las
amputaciones. El tratamiento tópico con
propiedades antibióticas o regenerativas también
son importantes. Tratamiento antibiótico: Primer evento y sin compromiso de la extremidad. Manejo ambulatorio con antibióticos vía

CURSO ENARM CMN SIGLO XXI TEL: 36246001 Pharmed Solutions Institute PÁGINA 548
MANUAL DE TRABAJO DEL CURSO ENARM CMN SIGLO XXI
oral por dos semanas: Cefalosporina primera generación (cefalexina), 500 mg, cada 6 horas, Clindamicina, 300 mg, cada 8 horas,
Dicloxacilina, 500 mg, cada 6 horas, Levofloxacino, 500 a 750 mg, cada 24 horas. Úlcera recidivante, celulitis extensa en miembro
inferior, compromiso articular u óseo. Manejo hospitalario con antibiótico intravenoso. El tiempo varía dependiendo de la condición
clínica del paciente: Ceftazidima, 1 g, EV, cada 8 horas, más clindamicina, 600 mg, EV, cada 8 horas. Ceftriaxona, 1 g, EV, cada 12 horas,
más clindamicina, 600 mg, EV, cada 8 horas. Ciprofloxacino, 400 mg, EV, cada 12 horas, más clindamicina, 600 mg, EV, cada 8 horas.
Infecciones que amenazan la vida, sospecha de meticilinorresistencia o multirresistencia y Pseudomonas Vancomicina, 1 g, EV, cada 12
h, más ceftazidima, 1 g, EV, cada 8 h, más clindamicina, 600 mg, EV, cada 8 h. Vancomicina, 1 g, EV, cada 12 h, más meropenem, 500
mg, EV, cada 8 h, más moxifloxacino, 400 mg, VO, cada 24 h. Osteomielitis: El periodo mínimo de tratamiento antibiótico es tres meses
y puede durar seis meses a una año. Para acortar este tiempo se debe realizar curetaje o extracción de huesos infectados y
desvitalizados. Tratamiento hipolipemiante: Existen varias alternativas pero se prefiere atorvastatina, 20 a 40 mg/comida. Tratamiento
del dolor neuropático: Una de las combinaciones que ha dado buenos resultados es asociar pregabalina con tramadol, titulando la
dosis. Amputación: Es el último recurso para evitar la complicación o muerte del paciente con PD. Toda amputación que se realiza en el
pie se denomina menor y las que se realizan por arriba del tobillo, incluyendo la infracondílea y supracondílea, se denominan mayores.
Es preferible una amputación menor porque con esta el paciente casi siempre puede caminar por sí solo. Cuando es amputación
mayor, el paciente deberá usar en forma permanente una muleta, prótesis o silla de ruedas si quiere desplazarse. Con la amputación
mayor, el paciente entra a una discapacidad que deteriora su salud y calidad de vida por lo que necesita mayor apoyo de su familia y de
la sociedad. Se indica amputación mayor cuando un miembro isquémico no se puede revascularizar o es afectado por osteomielitis
difusa que no responde al tratamiento y amenaza la vida del paciente. Cuando el miembro infectado constituye un foco séptico que no
responde al tratamiento antibiótico y desbridamientos, para salvar la vida del paciente se debe tomar la decisión de una amputación
mayor.

CASO CLINICO
Acude a consulta masculino de 58 años de edad el cual refiere que desde hace una semana inicia con una lesión en tipo ulcera plantar
de 3.5 cm aproximadamente, como antecentes de importancia, se trata de diabético controlado con hipoglucemiantes orales, como
moderado apego a tratamiento, sin embargo debido a su nivel socioeconómico primero trato de manejar su lesión por medio
tradicionales, sin embargo usted no percibe datos sugerentes de proceso por clostridium o agentes anaerobios, la extremida presenta
temperatura normal, sus pulso están presentes y solo se observa hiperbaralgesia.

PREGUNTA
Considerando este cuadro clínico, cual es el manejo de primera intensión para este caso antes de realizar un envío a segundo nivel de
atención.

RESPUESTA
a.- Es un paciente que tendría una rápida mejoría en cámara hiperbarica.
b.- Indica antiagregante plaquetario, pentoxifilina y vendaje de jhonson.
c.- Lo más indicado es debridar el area y antibióticos.
d.- Indica debridacion, pentoxifilina, antibiótico y vendaje.

CURSO ENARM CMN SIGLO XXI TEL: 36246001 Pharmed Solutions Institute PÁGINA 549
MANUAL DE TRABAJO DEL CURSO ENARM CMN SIGLO XXI
INGLES

CASE REPORT

A recently married 29-year-old nulliparous African-American female presents with uncomplicated cystitis. She is otherwise healthy. She
reports that she is currently using barrier birth control methods but plans to discontinue this soon, as she would like to become
pregnant.

QUESTION
Which one of the following supplemental vitamins or minerals would you advise this patient to
begin taking?

a) Thiamine
b) Iodine
c) Folate
d) Calcium

QUESTION
Which one of the following is true regarding testing the rotator cuff for injury?

ANSWER
a) The subscapularis is tested with lateral rotation against resistance
b) The teres minor is tested with medial rotation against resistance
c) The infraspinatus is tested with medial rotation against resistance
d) The supraspinatus is tested with abduction against resistance

CASE REPORT
You have recently begun caring for a 25-year-old white female who has multiple complaints. You have seen her 3 times for walk-in
office visits over the past month. She has shown appreciation
for your work during the encounters, but has been critical of your care when talking to the office
staff. At times she has been kind and charming, and at other times she has been rude and verbally
abusive to your staff. She has a string of multiple relationships in the past, none of which has
lasted very long. During times of intense stress, she has sometimes engaged in self-mutilation.
She frequently changes jobs and living arrangements.

QUESTION
Which one of the following strategies would be most appropriate in the care of this patient

ANSWER
a) Strive to develop a close relationship with the patient.
b) Ignore verbal attacks on staff members.
c) Provide detailed, technical explanations for. any therapies provided.
d) Schedule frequent office visits for follow-up.

In an 11-year-old male with dark brown urine and hand and foot edema.

QUESTION
Which one of the following would be most suggestive of glomerulonephritis?

ANSWER
a) RBC casts in the urine
b) Elevated C3 and C4 complement levels
c) Eosinophils in the urine
d) Positive serum antinuclear antibody levels

A 32-year-old white female at 16 weeks gestation presents to your office with right lower quadrant pain.

QUESTION
Which one of the following imaging studies would be most appropriate for initial evaluation of this patient?

ANSWER
a) MRI of the Abdomen
b) Intravenous pyelography
c) CT of the abdomen
d) Ultrasonography of the abdomen

CURSO ENARM CMN SIGLO XXI TEL: 36246001 Pharmed Solutions Institute PÁGINA 550
MANUAL DE TRABAJO DEL CURSO ENARM CMN SIGLO XXI

QUESTION
Which one of the following is the leading cause of blindness in individuals over age 65?

a) Narrow angle glaucoma


b) Diabetic retinopathy
c) Macular degeneration
d) Ophthalmic artery occlusion

CASE REPORT

A 48-year-old white female complains of anxiety and difficulty concentrating at home and at work. She reports that the symptoms have
increased over the last 2 months because of her daughter’s marital difficulties. She has had similar symptoms along with intermittent
depression since she was a teenager. She admits to a loss of pleasure in work and recreational activities.

QUESTION
Which one of the following is LEAST likely to help her coexistent depressive symptoms?

ANSWER
a) Nortriptyline.
b) Buspirone.
c) Escitalopram
d) Venlafaxine

CASE REPORT
An 82-year-old white female visits your office. She reports a 2-week history of shoulder stiffness that is severe in the morning, and a 3-
day history of pain on the left side of the face while chewing food. She denies dental pain or sensitivity. Her family history is negative
for rheumatologic disorders. Your evaluation reveals a 5-lb weight loss and evidence of mild depression. Her temperature is 38.0° C
(100.4° F), she has no dental decay or carotid bruit, and her left temporal scalp is tender. Her lungs are clear to auscultation, there are
no abnormal heart sounds, and her abdomen is nontender. She has no joint swelling or warmth, but experiences discomfort with
shoulder and hip range of motion. Laboratory analysis reveals a hemoglobin level of 11.0 g/dL (N 11.7–16.1), an erythrocyte
sedimentation rate of 80 mm/hr (N 0–30), a serum glucose level of 120 mg/dL, and a BUN level of 24 mg/dL (N 8–23).

QUESTION
Which one of the following is the greatest immediate risk for this patient?

ANSWER
a) Monocular vision loss
b) Hemiparesis
c) Sudden death
d) Adhesive capsulitis of the shoulder

CASE REPORT
A 73-year-old male with COPD presents to the emergency department with increasing dyspnea.
Examination reveals no sign of jugular venous distention. A chest examination reveals decreased
breath sounds and scattered rhonchi, and the heart sounds are very distant but no gallop or murmur
is noted. There is +1 edema of the lower extremities. Chest radiographs reveal cardiomegaly but
no pleural effusion. The patient’s -type natriuretic peptide level is 850 pg/mL (N <100) and his
serum creatinine level is 0.8 mg/dL (N 0.6–1.5).

QUESTION
Which one of the following would be the most appropriate initial management?

ANSWER
a) Prednisone, 20 mg twice daily for 1 week
b) Furosemide, 40 mg intravenously
c) Intravenous heparin
d) Tiotropium

CASE REPORT
A 28-year-old primigravida is at 20 weeks gestation by dates but her fundal height is consistent with a 26-week gestation. She has had
episodes of vomiting during the pregnancy that were more severe than the physiologic vomiting typically seen in pregnancy. A
sonogram performed at about 5 weeks gestation for vaginal bleeding was normal and showed a single fetus.

CURSO ENARM CMN SIGLO XXI TEL: 36246001 Pharmed Solutions Institute PÁGINA 551
MANUAL DE TRABAJO DEL CURSO ENARM CMN SIGLO XXI
QUESTION
Which one of the following would be most appropriate at this point?

ANSWER
a) Expectant management
b) A repeat sonogram
c) MRI of the pelvis
d) A serum hCG leve

CASE REPORT
A 24-year-old male, new to your practice, presents for a mental health evaluation. The patient has a past history of schizophrenia,
diagnosed several years ago.

QUESTION
Which one of the following, if present, would lead to a reconsideration of this diagnosis?

ANSWER
a) Loose associations
b) Elated mood
c) Social dysfunction
d) Auditory hallucinations

CASE REPORT
A previously healthy 22-year-old female presents for her regular prenatal checkup at 38 weeks gestation. She has a blood pressure of
145/95 mm Hg today and this is unchanged 1 hour later. Her blood pressure was normal before pregnancy. She is otherwise feeling
well. She has moderate edema at the ankles and 3+ reflexes at the knees and ankles. A urinalysis for protein is normal.

QUESTION
Given this presentation, which one of the following is the most likely diagnosis?

ANSWER
a) Hemolysis, elevated liver enzymes, low platelets syndrome
b) Preeclampsia
c) Essential hypertension
d) Gestational hypertension

CASE REPORT
A 72-year-old female with longstanding diabetes mellitus presents to your office. During the review of systems, she complains of
difficulty voiding and frequent “dribbling.” A urinalysis is negative for infection and her post-void residual volume is 250 mL.

QUESTION
Which one of the following is the most li ely cause of this patient’s urinary incontinence?

ANSWER
a) A grade II cystocele
b) Atrophic vaginitis
c) Asymptomatic bacteriuria
d) Autonomic neuropathy

CASE REPORT
A 3-year-old male is brought to your office the day after he was stung by a honeybee. He has developed a significant local reaction, with
redness and swelling around the site of the sting on his forearm, and also has had some swelling of his lips for “a couple of hours.” His
mother removed the stinger and gave him some oral diphenhydramine The local reaction has now almost resolved and he has not had
any hives or respiratory distress.

QUESTION
Which one of the following is true concerning this situation?

ANSWER
a) An antibiotic that covers Streptococcus pyogenes and Streptococcus aureus should be administered
b) If venom immunotherapy is begun, it should be stopped after 2 years of treatment
c) The child is at high risk for a systemic reaction if he is stung again in the future
d) Skin tests are not helpful in confirming the presence of insect sting allergy

CURSO ENARM CMN SIGLO XXI TEL: 36246001 Pharmed Solutions Institute PÁGINA 552
MANUAL DE TRABAJO DEL CURSO ENARM CMN SIGLO XXI
CASE REPORT
A 65-year-old white female develops a burning pain in the left lateral thorax, followed 2 days later by an erythematous vesicular rash.

QUESTION
Of the following, the best treatment is?

ANSWER
a) topical capsaicin (Zostrix)
b) oral corticosteroids
c) topical corticosteroids
d) topical acyclovir (Zovirax)

CLINIC CASE
A patient on peritoneal dialysis has received six (6) units of red blood cells for a bleeding duodenal ulcer. Dialysis was discontinued 3
day ago and an emergency gastric surgery is now necessary. The patient arrests following induction of general anesthesia whit propofol
and succinylcholine.

QUESTION
Which of the following is the likely cause of this cardiac event?

ANSWERS
a.- Hypothermia.
b.- Hypovolemic shock.
c.- Hypocalcemia.
d.- Hyperkalemia.

CLINIC CASE
A 79 year old man with a one pack for day smoking history is incidentally found to have a right lower lung mass with moderate, free-
flowing, right-side pleural effusion. On further questioning he admits to some symptoms of dyspnea and a 20-pound weight loss over 3
months, but denies other symptoms.

QUESTION
What should be done next.

ANSWERS
a.- Mediastinoscopy.
b.- Bronchoscopy.
c.- Thoracentesis.
d.- CT of the chest.

CLINIC CASE
A 21 year old woman with a history of cystic fibrosis is admitted to the ICU for pneumonia. Her CXR shows right lower lung zone dense
consolidations with pleural effusion, in addition to previous seen bronchiectasis. She has no drug allergies, and her previous respiratory
culture results are not available at this time.

QUESTION
What empiric antibiotics should be started.

ANSWERS
a.- Imipenem.
b.- Ceftriaxone and metronidazole.
c.- Cefepime.
d.- Piperacilin and Tazobactam.

CLINIC CASE
A 66 year old man complains of gradual onset of dyspnea. His PFT shows: FEV1/FVC 104, FEV1 73% predicted, and FVC 70% of
predicted, TLC 66% of predicted, and DLCO 52% of predicted. The patient denies any known medical problems. He has smoked a few
cigarettes a day since age 18, and reports that he use to work in a shipyard from age 18 to 58. On exam his vitals are normal, and the
lung exam is significant for some bibasular crackies. No clubbing of the fingernails is seen.

QUESTION
What should be done next to evaluate this patient´s dyspnea.

ANSWERS
a.- Transthoracic echocardiogram.
b.- Bronchoscopy for transbronchial biopsy.

CURSO ENARM CMN SIGLO XXI TEL: 36246001 Pharmed Solutions Institute PÁGINA 553
MANUAL DE TRABAJO DEL CURSO ENARM CMN SIGLO XXI
c.- High resolution CT scan of the chest.
d.- Right heart catheterization.

CLINIC CASE
A 70 year old patient with hypercalcemic crisis has an emergency resection of a large parathyroid adenoma. Immediately post
operation, the patient develops convulsions. Calcium IV is administered with minimal response.

QUESTION
What is the etiology of the convulsions ?

ANSWER
a.- Hypocalcemia.
b.- Hypernatremia
c.- Cerebral ischemia.
d.- Hypomagensemia.

CLINIC CASE
A 45 year old male weighing 80 kg is seen in the ER for two episodes of hematochezia within the last 2 hours. He has pulse of 80 bpm, a
BP of 100/60 mmHg, a respiratory rate of 30, and urine output of 20 ml. he is anxious and confused.

QUESTION
What is his estimated blood loss?

ANSWER
a.- 500 ml
b.- 1000 ml.
c.- 2000 ml.
d.- 3000 ml.

CLINIC CASE
A 60 year old woman, who was previously treated with radiation therapy for uterine cancer at age 50, presents with a small bowel
obstruction.

QUESTION
Intraoperatively, one would expect to see?.

ANSWER
a.- A sharp demarcation at the area of obstruction.
b.- Gray thick-walled small bowel matted together.
c.- Multiple areas of stricture in normal appearing small bowel.
d.- Inter loop abscesses and fistula.

CLINIC CASE
A 75 years old female is evaluated for reflux esophagitis. At endoscopy the mucosa is salmon pink with a definite demarcation between
normal and abnormal esophageal mucosa. Biopsy indicates no dysplasia. Three weeks following a Nissen Fundoplication, the patient
has a severe episode of retching following a heavy meal and has vomiting with blood. Two hours later, he has severe epigastric pain and
a spiking temperature of 102 F.

QUESTION
Which of the following is the problem?.

ANSWER
a.- The Nissen has slipped.
b.- Fundoplication disruption has occurred with perforation.
c.- A perforated ulcer has developed.
d.- Acute pancreatitis.

CLINIC CASE
A 62 year old male presents with dyspepsia. An UGI shows thickening of the rugal folds, at upper endoscopy, no mass or lesion is
identified. The gastric wall is thickened but no ulceration is noted.

QUESTION
Following endoscopic biopsy, this patient is best treated by?

ANSWER
a.- Radiation.

CURSO ENARM CMN SIGLO XXI TEL: 36246001 Pharmed Solutions Institute PÁGINA 554
MANUAL DE TRABAJO DEL CURSO ENARM CMN SIGLO XXI
b.- Chemotherapy.
c.- Chemotherapy and radiation.
d.- Subtotal gastrectomy.

CLINIC CASE
Following an open cholecystectomy and common duct exploration for multiple gallstone in a 75 year old female, the colangiogram
obtained 10 days post operatively reveals an impacted 1.5 cm gallstone at the ampulla. The bilirubin is 5.0.

QUESTION
What is the procedure of choice?

ANSWER
a.- Repeat the cholangiogram in 1 week.
b.- T-tube flush with Capmul 8210.
c.- Open sphincteroplasty.
d.- Choledochoduodenostomy.

CLINIC CASE
A 46 year old male has had intermittent flares of ulcerative pancolitis for 10 years. He has increasing diarrhea, blood per rectum, and
fever. He responds to medical management.

QUESTION
Proctocolectomy is indicated for?

ANSWER
a.- Length of colon involved.
b.- Long periods of persistent disease.
c.- Severe dysplasia on initial biopsy.
d.- Persistent stricture.

CLINIC CASE
A 75 year old woman suffered from sudden onset left eye blindness. No pain was associated with her blindness, and no other
abnormalities were seen in the neurological examination.

QUESTION
What is the most likely diagnosis?.

ANSWER
a.- Small lacunar stroke involving the occipital lobe.
b.- Ischemic lesion involving the lateral geniculate nucleus.
c.- Compressive lesion on the optical chiasm.
d.- Retinal ischemia due to retinal artery occlusion.

CLINIC CASE
A 51 year old male started to have a gradual decline in his intellectual functions associated with depressive symptoms and personality
changes. The patient obtained a score of 23/30 on mini-mental status examination and on physical examination chorea was noted. The
diagnosis of Huntington´s disease was suspected.

QUESTION
What is typical inheritance pattern of Huntington´s disease?

ANSWER
a.- X-linked
b.- Autosomal dominant.
c.- Autosomal recessive.
d.- No familial pattern.

CLINIC CASE
A 36 year old woman was referred to the neurology clinic to be evaluated for her progressing headaches. The headaches were
described as dull, mild to moderate in severity, and located on the left side of the head.
The patient has no family history of migraine headaches. Neurological examinations was mostly normal except for right-sided
hyperreflexia.

QUESTION
What is the most likely diagnosis?

CURSO ENARM CMN SIGLO XXI TEL: 36246001 Pharmed Solutions Institute PÁGINA 555
MANUAL DE TRABAJO DEL CURSO ENARM CMN SIGLO XXI
ANSWER
a.- Left middle cerebral artery aneurysm.
b.- Arterio-Venous malformation.
c.- Vascular brain tumor.
d.- Sagittal sinus thrombosis.

CLINIC CASE
A 25 year old man with HIV presented to the ED with right-side weakness and confusion. An MRI brain showed a few ring enhancing
lesions. The differential diagnosis is narrowed to toxoplasmosis and primary CNS lymphoma.

QUESTION
What is the appropriate next step of management?.

ANSWER
a.- Open brain biopsy.
b.- Stereotectic brain biopsy.
c.- Empiric treatment of toxoplasmosis with reimaging to evaluate the response.
d.- High dosis steroids.

CLINICAL CASE
A 32 year old primigravida presents to her obstetrician for routine follow-up at 34 weeks gestation. She has a sonogram of the
developing baby which also shows that she has developed right sided moderate hydronephrosis, Serum creatinine concentration is
checked and is 0.8 mg/dl. She has no symptoms. Urinalysis normal.

QUESTION
At this point, you should:

ANSWER
a.- Send urine for culture assuming infection.
b.- Consult urology immediately.
c.- Admit to the hospital.
d.- No further intervention.

CLINIC CASE
A 65 year old make with pneumonia and septic shock develops acute kidney injury. He has been running a low blood pressure (80/45
mmHg) on several vasopressor agents. Urine output start to fall and his creatinine begins to rise. His urine sodium is high; he has no
proteinuria.

QUESTION
The most common cause of his kidney failure in this setting is:

ANSWER
a.- Obstruction.
b.- Acute tubular necrosis.
c.- Acute interstitial nephritis.
d.- Goodpasture´s disease.

CLINIC CASE
A 54 yearold woman reports a bothersome cough to 6 months duration, She has never smoked, and denies history of wheezing or
heartburn. She has diabetes and hypertension, and has been on metformin and lisinopril for the past 3 years.

QUESTION
What is the next appropriate step in management.

ANSWER
a.- Discontinue lisonopril.
b.- Start a proton pump inhibitor.
c.- Esophageal pH monitoring.
d.- Barium swallow evaluation.

CLINIC CASE
A 48 year old man presents with hemoptysis. He reports that he has also had bloody nasal discharge over the past month. His CXR
shows bilateral nodules. Laboratory data is notable for creatinine of 4.7 mg/dl, and urine analysis shows 250 RBC/hpf

QUESTION
Which of the following serum studies should be ordered next?

CURSO ENARM CMN SIGLO XXI TEL: 36246001 Pharmed Solutions Institute PÁGINA 556
MANUAL DE TRABAJO DEL CURSO ENARM CMN SIGLO XXI

ANSWER
a.- ANCA, anti-GBM antibodies.
b.- Anti-dis-DNA and anti-cardiolipin antibodies.
c.- ESR, C3, C4 leves
d.- RF, anti-citrulline antibodies.

CLINIC CASE
A 62 year old woman presents for evaluation of dizziness. It occurs often when she´s reaching overhead into her cupboards. She also
notice that her face has been swollen, and she has recently developed a cough. On exam, she is noted to have some fullness in her
neck, but the rest of the physical exam is unremarkable. A CXR shows a sharply circumscribed, dense anterior mediastinal mass that
displaces the trachea to the left.

QUESTION
What is the next appropriate step in management?

ANSWER
a.- Radiation therapy.
b.- Radioactive iodine scanning.
c.- Referral for chemotherapy.
d.- Referral for surgical resection.

CLINIC CASE
A 38 year old woman with a history of asthma has had multiple admissions in the past year for asthma exacerbation. She was treated
during the most recent hospitalization for a community-required pneumonia that was seen on CXR. She comes to you for post-
discharge follow-up. She reports she is compliant with her combination steroid/long-acting beta agonist inhaler, but still needs her
rescue inhaler several times a day, and still has a cough occasionally productive of brown plug. Her blood work on the day of discharge
was only significant for an eosinophilic count of 750/mm3.

QUESTION
Which of the following serum studies should be considered?

ANSWER
a.- Chronic eosinophilic pneumonia.
b.- Allergic brochopulmonary aspergillosis.
c.- Steroid non responsive asthma.
d.- Viral bronchitis.

CLINIC CASE
A 70 year old man who previously worked in stonecutting has had bilateral upper lung zone interstitial markings and small nodules on
his CXR for several years. However, on his most recent CXR, it is noted that some of the nodules in the right upper lobe have coalesced
into a mass, and cavitation is noted.

QUESTION
What is the next appropriate step in management?

ANSWER
a.- Sputum induction to rule out tuberculosis.
b.- Bronchoscopy with transbronchial biopsy to rule out malignancy.
c.- Refer to thoracic surgery for resection.
d.- Start antibiotics for treatment of community acquired pneumonia.

CLINIC CASE
A 60 year old man with severe COPD is found to be difficult to arouse after undergoing an outpatient, elective screening colonoscopy.
The following lab data are available: ABG on room air pH 7.24, PaO2 64, PaCO2 78, Electrolytes: Na 140, HCO3 35, Cl 100.

QUESTION
What is his current acid-base status?

ANSWER
a.- Metabolic acidosis with respiratory compensation.
b.- Acute respiratory acidosis with metabolic compensation.
c.- Acute on chronic respiratory acidosis
d.- Chronic acidosis with metabolic compensation.

CURSO ENARM CMN SIGLO XXI TEL: 36246001 Pharmed Solutions Institute PÁGINA 557
MANUAL DE TRABAJO DEL CURSO ENARM CMN SIGLO XXI
CLINIC CASE
A 50 year old man recently sustained a fall while shoveling snow; Four weeks later he began to experience shortness of breath, right-
sided chest discomfort, and a dry cough. He also feels extreme fatigue and feels less productive at work. The patient has no other
significant past medical history and does not smoke. Other than using the indoor hot tub in his condominium community three times a
week since his fall, there are no changes in his usual activities.

QUESTION
What is the most likely diagnosis?

ANSWER
a.- Sarcoidosis
b.- Aspergillosis
c.- Hypersensitivity pneumonitis due to mycobacterium avium.
d.- Active Infection with mycobacterium avium.

CLINIC CASE
A 48 year old man presents with a week of fever, cough, and malaise. He has no significant past medical history and he takes no
medications. His vital signs are normal but an exam reveals right sided crackies. A CXR shows a right lower lobe infiltrate.

QUESTION
What antibiotic should be started?

ANSWER
a.- Doxycycline.
b.- Trimethoprim/Sulfamethoxazole.
c.- Cephalexine.
d.- Amoxicillin/Clavulanate.

CLINIC CASE
A 46 year old woman with a history of asthma calls you for a refill of her albuterol inhaler. Her asthma has always been well-controlled
on a daily dose of moderate-dose inhaled corticosteroid and she´s long misplaced her albuterol inhaler. However 3 weeks ago she had
an upper respiratory tract infection. All her acute symptoms have resolved but since then she has felt more wheezing, and often coughs
at night. She´s been using her daughter´s albuterol inhaler and finally decides to call you to get her own prescription.

QUESTION
What is the next appropriate step in management?

ANSWER
a.- Prescribe a course of oral fluoroquinolone.
b.- Prescribe albuterol inhaler for as needed rescue use.
c.- A short course of oral corticosteroid therapy.
d.- Double her moderate-dose inhaled corticosteroid therapy.

CLINIC CASE
A 52 year old Hispanic female is seen for consultation in the hospital. She states that she had a complete physical with laboratory
testing 2 months earlier that was completely normal. She feels poorly now for the past few weeks. Her legs have become swollen and
she is having shortness of breath. Laboratory testing reveals a positive ANA 1:360, an elevated ds-DNA low C3 and C4 leves, a 24 hour
urine protein of 2.5 grams with significant RBCs on her urinalysis, and renal failure. An echocardiogram shows a significant pericardial
effusion, but no evidence for tamponade. A kidney biopsy reveals diffuse proliferative lupus nephritis.

QUESTION
Which of the following is the most appropriate therapy at this time?

ANSWER
a.- Cyclophosphamide.
b.- Azothioprine.
c.- Solumedrol.
d.- Plasmaferesis.

CLINIC CASE
A 44 year old Hispanic female presents with inflammatory arthritis of one of her wrists and MCP´s interstitial lung disease, a non
specific rash, and Raynaud´s phenomenon. Lab test reveal and alevated AntiNuclear Antibody 1:640, Rheumatoid Factor of 50 (Normal
< 14), and a normal anti-U1 RNP.

QUESTION
Which of the following is the diagnosis?

CURSO ENARM CMN SIGLO XXI TEL: 36246001 Pharmed Solutions Institute PÁGINA 558
MANUAL DE TRABAJO DEL CURSO ENARM CMN SIGLO XXI

ANSWER
a.- Undifferentiated connective tissue disease.
b.- Mixed connective tissue disease.
c.- Systemic lupus erythematosus.
d.- Rheumatoid arthritis.

CLINIC CASE
A 30 year old patient with sjogren´s syndrome is placed on a cytotoxic agent as well as oral steroids. At 3 months she begins to have
right thigh and knee pain that is increased with walking and standing. Physical examination reveals normal range of motion of her knee,
but mild pain on internal and external rotation of her hip.

QUESTION
What is the most sensitive and appropriate test to diagnose his condition early?

ANSWER
a.- Plain film radiography.
b.- Technetium-99m bone scanning.
c.- Magnetic resonance imaging.
d.- CT scan.

CLINIC CASE
A 32 year old white female with systemic lupus erythematosus is 32 week pregnant and presents to your office with a headache.
Physical examination is remarkable for a blood pressure of 200/110 mmHg. Laboratory studies show a hemoglobin of 8.0 g/dl, platelet
count of 38,000, a low haptoglobin level, and an elevated LDH, AST, and ALT.

QUESTION
What test would be best to determine the cause of her problems?

ANSWER
a.- Complement levels.
b.- Erythrocyte sedimentation rate.
c.- Anti-Smith antibody.
d.- C-reactive protein.

CLINIC CASE
A 24 year old female has diagnosed systemic lupus erythematosus for the past 5 years. She has had episodes of discoid rashes as well as
polyarthritis and mild hematologic abnormalities, but none in the past year.

QUESTION
What pharmacological agent would be most appropriate for her?

ANSWER
a.- Prednisone.
b.- Hydroxychloroquine.
c.- Cyclophosphamide.
d.- Metotrexate.

CLINIC CASE
A 70 year old mal presents with weakness and low grade temperature for a few month. He has no Raynaud´s. He has tightening and
erythema of the skin of his upper arms and legs with prominence along the course of the superficial veins. There is considerable muscle
weakness. Laboratory testing reveals a normal CK, ANA, RF and a significantly elevated CRP and ESR. A CBC reveals a hemoglobin of
11.2 g/dl with a moderate increase in monocytes and eosinophils.

QUESTION
Which of the following is the most likely diagnosis?

ANSWER
a.- Parasitic Involvement.
b.- Occult neoplasm.
c.- Alergic reaction.
d.- Eosinophillic fasciitis.

CURSO ENARM CMN SIGLO XXI TEL: 36246001 Pharmed Solutions Institute PÁGINA 559
MANUAL DE TRABAJO DEL CURSO ENARM CMN SIGLO XXI
CLINIC CASE
A 29 year old male has lower back pain without radiation for the past 3 weeks. He does not recall any specific instigating event. Pain is
severe and worse with sitting as well as standing. His neurological examinations is unremarkable.

QUESTION
Which of the following is the most appropriate action at this time?

ANSWER
a.- Check x-ray of his lower back.
b.- Check an MRI of his lower back.
c.- Start physical therapy and NSAID´s.
d.- An EMG evaluation of his lower extremities.

CLINIC CASE
A 69 year old morbidly obese white female states that she has severe disabling right knee pain. The pain has been ongoing for the past
two weeks and has been keeping her awake at night. She does not recall any preceding trauma and has been using a pillow between
her knees with some benefit. Physical examination reveals crepitus and mild tenderness of the knee with severe pain 4 cm below the
medial joint line.

QUESTION
What is the most appropriate next step?

ANSWER
a.- X-ray of the knee.
b.- MRI of the Knee.
c.- Corticosteoid injection.
d.- Local supportive therapies.

CLINIC CASE
A 76 year old black male with a history of chronic gout on uric acid lowering therapy and severe Class IV renal disease presents with
multiple swollen joints. Aspiration of his knee reveals uric acid crystals intracellularly.

QUESTION
What is the most appropriate therapy for him a this time?

ANSWER
a.- Start indomethacin.
b.- Start prednisone.
c.- Discontinue allopurinol.
d.- Start febuxastat.

CLINIC CASE
A 42 year old male comes in with a history of recurrent episodes of hives over the past 5 day that are intensely pruritic on the arms,
chest, and back. He has a medical history of Diabetes Type II, coronary artery disease and a recent urinary tract infection. His
medications include glucophage, lisinopril, baby aspirin and trimethoprim- sulfamethoxazole. He has applied over the counter Calamine
lotion and has been taking Benadryl without much relief. He denies any new use of soaps or products and has not been out of the
country recently. He is not in contact with any plants or chemicals at work.

QUESTION
What is the mechanism of hypersensitivity in this case?

ANSWER
a.- Overabundance of IgA.
b.- Overabundance of IgG.
c.- Prostanglandin synthesis.
d.- Mas cell degranulation.

CLINIC CASE
A 47 year old white female with diabetes mellitus type II presents to the clinic with an erythematous rash in her axillae and groin of one
month duration. Over the counter cortisone has failed to clear her rash. She denies using any deodorants or creams.

QUESTION
What is the best treatment option for this diagnosis?

ANSWER

CURSO ENARM CMN SIGLO XXI TEL: 36246001 Pharmed Solutions Institute PÁGINA 560
MANUAL DE TRABAJO DEL CURSO ENARM CMN SIGLO XXI
a.- Erythromcin.
b.- Trimethoprim-sulfamethoxazole.
c.- Ketoconazole cream.
d.- Ciprofloxacin.

CLINIC CASE
A 18 year old male comes to your office after returning home from college for summer break. He complains of an intensely pruritic rash
all over his body. He notes that one week ago he went to the student health clinic with complaints of fever, fatigue, sore throat, neck
pain, and generalized body aches. He informs you that he was given an antibiotic but does no remember the name. He denies any other
symptoms today. On exam, you note a generalized erythematous morbilliform rash from the base of the neck to the dorsal aspect of
the feet bilaterally. You note his tongue and palate.

QUESTION
What do you suspect the original diagnosis was in this patient?

ANSWER
a.- Influenza.
b.- Adenovirus.
c.- Mononucleosis.
d.- Bacterial pneumonia.

CLINIC CASE
Entrance to emergency room female patient aged 48, originally from Veracruz, agricultural employee, married 17 years, gravida 5, for 3,
1 abortion, 1 cesarean, hysterectomy for fibroids 38, does not smoke or drink alcoholic beverages, apparently healthy, takes no
medications, dental cavities previously presented but were treated for 7 months however until three months ago that started current
condition characterized by fatigue, dyspnea on moderate exertion, drowning sensation predominantly nocturnal, is entered because it
has a cough with sputum thin pink, mild effort dyspnea, physical examination patient shows serious condition, swollen with central and
peripheral cyanosis, jugular venous distention, breath mediodiastolico III / IV which predominates in the upright position, edema pelvic
limbs with ischemic lesions in fingers 1 and 2 of the right foot, treated and contaminated local remedies, perceives no odor or pus, vital
signs are as follows TA 125/85 mmHg, HR 102, RR 26, temperature 37.5 C. The patient receives a diagnosis of congestive heart failure.

QUESTION
What is the most likely cause for congestive heart failure in this case.

ANSWER
a. - Bacterial endocarditis with valvular insufficiency.
b. - Mitral valve stenosis.
c. - Mixioma handset.
d. - Metastatic melanoma.

CLINIC CASE
Stabilize the patient and imaging studies are performed which identifies an intracavitary tumor of 6 cm, in the studies were CT, MRI and
catheterization, 4 hours after the patient has severe respiratory distress, abundant presence of cough with sputum asalmonelado, so is
intubated for ventilatory support, peripheral cyanosis is observed with loss of distal pulses of lower limbs, absence of bowel sounds
followed by cardiac arrest twice, declaring his death two hours later.

QUESTION
What is the intervention was more likely that culminated in massive embolism in this patient.

ANSWER
a. - No prophylactic anticoagulation.
b. - Cardiac catheterization unnecessary.
c. - The mixioma detachment.
d. - The patient already had signs of stroke.

CURSO ENARM CMN SIGLO XXI TEL: 36246001 Pharmed Solutions Institute PÁGINA 561

También podría gustarte